Está en la página 1de 1228

portada TROPA 19/3/07 19:51 Página 1

FUERZAS ARMADAS
PROFESIONALES
CURSO DE APOYO
A LA PREPARACIÓN
DE LAS PRUEBAS DE ACCESO
A UNA RELACIÓN DE SERVICIOS
DE CARÁCTER PERMANENTE

CIENCIAS DE LA NATURALEZA
1ª parte
Unidades didácticas 1, 2, 3 y 4

DIGEREM

MINISTERIO
DE DEFENSA
FUERZAS ARMADAS SUBDIRECCIîN GENERAL
DE TROPA Y MARINERIA
PROFESIONAL
PROFESIONALES
CURSO DE APOYO
Ó
A LA PREPARACION
DE LAS PRUEBAS DE ACCESO
A UNA RELACIÓN DE SERVICIOS
DE CARÁCTER PERMANENTE

CIENCIAS DE LA NATURALEZA
1ª parte
Unidades didácticas 1, 2, 3 y 4

-Pág.1-
La Ley 8/2006 de Tropa y Marinería, en su artículo 16,1, establece que “la formación
en las Fuerzas Armadas garantizará que los militares profesionales de tropa y
marinería puedan adquirir, actualizar o ampliar sus conocimientos para un mayor
desarrollo personal y profesional”. En cumplimiento de este mandato, el Ministerio
de Defensa edita el presente material didáctico para facilitar a los militares
profesionales de tropa y marinería, alumnos de los cursos de formación
presencial que se imparten a través de la Dirección General de Reclutamiento y
Enseñanza Militar, los apoyos necesarios para preparación de dichos cursos, que
permitirán, siempre que superen las pruebas correspondientes, la obtención de la
titulación de graduado en Educación Secundaria, acreditación para el acceso a
los ciclos formativos de la Formación Profesional de grado medio o de grado
superior, acceso a las Escalas de Suboficiales, Tropa Permanente, Guardia Civil
y Policía Nacional.

CATÁLOGO GENERAL DE PUBLICACIONES


http://www.060.es

Edita:

© Autor y editor
NIPO: 076-10-204-9 NIPO: 076-10-205-4 (edición en línea)
Depósito Legal: M-32363-2009
Diseño y programación: cimapress
Tirada: 1300 ejemplares
Fecha de edición: septiembre, 2010

Prohibida la reproducción total o parcial de esta obra, por cualquier medio sin autorización escrita del editor
CIENCIAS DE LA NATURALEZA
1ª parte

SUMARIO
Unidad didáctica Pág.

1. DIVERSIDAD Y UNIDAD DE ESTRUCTURA 5


DE LA MATERIA

2. LA ENERGÍA 33

3. LOS CAMBIOS QUÍMICOS 61

4. LA TIERRA EN EL UNIVERSO 77

-Pág.3-
U . D . 1 - D IVERSIDAD Y U N I D A D D E E S T R U C T U R A D E L A M AT E R I A

ÍNDICE
OBJETIVOS . . . . . . . . . . . . . . . . . . . . . . . . . . . . . . . . . . . . . . . . . . . . . . . . . . . . . . 2

INTRODUCCIÓN . . . . . . . . . . . . . . . . . . . . . . . . . . . . . . . . . . . . . . . . . . . . . . . . . 3

MAPA CONCEPTUAL . . . . . . . . . . . . . . . . . . . . . . . . . . . . . . . . . . . . . . . . . . . . . 4

DESARROLLO DE CONTENIDOS . . . . . . . . . . . . . . . . . . . . . . . . . . . . . . . . . . 5

1. CARACTERÍSTICAS DE LOS SISTEMAS MATERIALES.


PROPIEDADES MÁS IMPORTANTES. ESTADOS DE AGREGACIÓN.
SISTEMAS HOMOGÉNEOS Y HETEROGÉNEOS . . . . . . . . . . . . . . . . 5

2. MEZCLAS Y SUSTANCIAS PURAS. ELEMENTOS Y COMPUESTOS . . 9

3. DISCONTINUIDAD DE LOS SISTEMAS MATERIALES. TEORÍA


ATÓMICA. NATURALEZA ELÉCTRICA DE LA MATERIA . . . . . . . 10

4. CLASIFICACIÓN DE LOS ELEMENTOS QUÍMICOS. SISTEMA


PERIÓDICO. METALES Y NO METALES. REGULARIDADES DE LOS
ELEMENTOS DEL SISTEMA PERIÓDICO. UNIÓN ENTRE ÁTOMOS . . . 13

5. ELEMENTOS Y COMPUESTOS MÁS ABUNDANTES EN LOS SERES


VIVOS Y EN LA MATERIA INERTE. UTILIZACIÓN DE MATERIALES DE
INTERÉS EN LA VIDA DIARIA. VERSATILIDAD DEL CARBONO
EN LA FORMACIÓN DE COMPUESTOS . . . . . . . . . . . . . . . . . . . . . . . . . . . . 20

RESUMEN . . . . . . . . . . . . . . . . . . . . . . . . . . . . . . . . . . . . . . . . . . . . . . . . . . . . . . . 24

EJERCICIOS DE AUTOCOMPROBACIÓN . . . . . . . . . . . . . . . . . . . . . . . . . . 26

RESPUESTAS A LOS EJERCICIOS . . . . . . . . . . . . . . . . . . . . . . . . . . . . . . . . . 28

-Pág.5-
C I E N C I A S N AT U R A L E S

OBJETIVOS
Al finalizar el estudio de esta Unidad Didáctica, el alumno será capaz de:

• Apreciar el conjunto de propiedades que nos permiten identificar distintas


clases de materia.

• Distinguir las clases de materia que componen los cuerpos.

• Apreciar que los cuerpos que nos rodean se presentan en tres estados físicos.

• Distinguir entre masa, peso y volumen de un cuerpo.

• Definir la densidad de un cuerpo y conocer su unidad en el Sistema


Internacional.

• Distinguir las partículas que componen el átomo.

• Reconocer los átomos como los constituyentes básicos de la materia y las


moléculas como la parte más pequeña de las sustancias.

• Interpretar los datos que nos ofrece el Sistema Periódico sobre los átomos y
moléculas.

• Aprender las normas de formulación de compuestos binarios.

• Reconocer las fórmulas y nombres de sustancias sencillas.

-Pág.6-
U . D . 1 - D IVERSIDAD Y U N I D A D D E E S T R U C T U R A D E L A M AT E R I A

INTRODUCCIÓN
T odos los cuerpos están formados de materia y se distinguen unos de otros
porque están constituidos por distintas clases de ella.
La materia puede encontrarse en los tres estados físicos.
Aunque algunas sustancias se encuentran puras en la naturaleza, la mayoría
de los cuerpos están formados por mezclas. Las sustancias que forman una
mezcla pueden estar en distinto estado físico, por ejemplo, el agua del mar
es líquida pero contiene sólidos disueltos, como las sales y gases como el
oxígeno.
En esta unidad intentaremos responder entre otras a las siguientes preguntas:
¿Cómo se puede presentar la materia?
¿Podemos distinguir unos cuerpos de otros por su materia?
¿Qué propiedades se necesitan para reconocer una clase de materia?
¿Cómo se mide la cantidad de materia de un cuerpo?

-Pág.7-
C I E N C I A S N AT U R A L E S

M A PA C O N C E P T UA L
ELECTRONES CORTEZA
dividido
PROTONES Y NEUTRONES NÚCLEO

IÓNICO, COVALENTE Y METÁLICO ENLACES unidos

LA QUÍMICA

es una ciencia
que estudia Átomos

LA MATERIA está formada por TEORÍA ATÓMICA

tiene propiedades se presenta en se clasifica en HETEROGENEA

CAMBIOS
ESTADOS DE AGREGACIÓN HOMOGENEA
DE ESTADO

GENERALES CARACTERÍSTICAS SÓLIDO MEZCLAS


HETEROGENEAS

LÍQUIDO se producen a las


MASA VOLUMEN MEZCLAS
GAS HOMOGENEAS

ELEMENTOS
su cociente es

COMPUESTOS

DENSIDAD SOLUBILIDAD TEMPERATURAS DE


FUSIÓN Y EBULLICIÓN

-Pág.8-
U . D . 1 - D IVERSIDAD Y U N I D A D D E E S T R U C T U R A D E L A M AT E R I A

1. CARACTERÍSTICAS DE LOS SISTEMAS MATERIALES.


PROPIEDADES MÁS IMPORTANTES. ESTADOS DE
AGREGACIÓN. SISTEMAS HOMOGÉNEOS Y HETEROGÉNEOS

1.1. CARACTERÍSTICAS DE LOS SISTEMAS MATERIALES


Todo lo que nos rodea, ya sea sólido, líquido o gaseoso, está compuesto por materia,
podemos concluir que ofrece una gran diversidad . Definir la materia resulta difícil y para
poder hacerlo se recurre inicialmente a las propiedades generales de la materia. Los objetos
materiales ocupan un volumen y poseen una masa, mientras que los entes inmateriales no.
Podemos así definir materia como todo aquello que ocupa un volumen y posee una masa,
cualquier cosa que ocupe un espacio.

Físicamente se define la materia como la forma de energía que tiene los atributos de
poseer una masa y una extensión en el espacio y en el tiempo.

La Química es la ciencia que estudia las propiedades de la materia y los cambios que se
producen en ella.

1.2. PROPIEDADES MÁS IMPORTANTES DE LA MATERIA


Como ya hemos dicho, toda materia tiene dos propiedades generales: una masa y un
volumen.

Masa: Toda la materia tiene masa, cuyo efecto más apreciable es la fuerza de atracción
que ejerce la Tierra sobre ella. Incluso los gases pesan. Puedes probar a pesar una rueda de
coche o bicicleta antes y después de llenarla de aire o con una pelota. (En el Sistema
Internacional -S.I.- la unidad es el Kilogramo).

Unidades de Masa Equivalencias

- Tonelada (t) 1 t = 1000 Kg


- Kilogramo (Kg) 1 Kg = 1000 g
- Gramo (g) 1 g = 1000 mg
- Miligramo (mg)

TAMadrid
5
-Pág.9-
C I E N C I A S N AT U R A L E S

Volumen: La materia ocupa un lugar en el espacio, incluso en estado gaseoso. Si


aprietas el émbolo de una jeringuilla taponando la salida, puedes reducir mucho el volumen
interior, pero no del todo, puesto que el aire ocupa un volumen. (En el Sistema Internacional
se utilizan como unidad de volumen el metro cúbico y sus submúltiplos y como unidad de
capacidad el litro).

Unidades de Volumen y Capacidad Equivalencias

VOLUMEN
- Hectómetro cúbico (hm3) 1 hm3 = 1.000.000.000 L
- Metro cúbico (m3) 1 m3 = 1.000 L
- Decímetro cúbico (dm3) 1 dm3 = 1 L
- Centímetro cúbico (cm3) 1 cm3 = 0,001 L = 1 mL
CAPACIDAD
- Litro (L)
- Mililitro (mL)

Para medir la masa utilizamos la balanza. La más clásica, de dos brazos, compara el
peso de una masa desconocida que queremos medir con otras masas conocidas (pesas). Hoy
en día se emplean balanzas más sofisticadas electrónicas, que han de calibrarse con pesas
tradicionales.

La medición del volumen difiere según se trate de gases, líquidos y sólidos. Los gases,
como tienden a ocupar el máximo espacio, ocuparán todo el volumen del recipiente que lo
contiene. Los sólidos, pueden medirse geométricamente cuando tienen forma de figura
regular. Si son irregulares, se sumergirá el sólido en un líquido y se medirá el aumento de
volumen. Se calculará por diferencia de volumen entre el total que ocupan y el volumen
inicial que ocupaba el líquido sólo. Para medir el volumen de los líquidos existen
recipientes graduados de laboratorio (matraces, pipetas graduadas y probetas) (cuando se
desea precisión se emplean recipientes aforados leyendo la medida en el enrase a la altura
del menisco que se forma en la parte superior del líquido).

Además de las dos propiedades ya estudiadas existen propiedades características de


cada materia entre las que podemos definir:

Densidad: La definimos como la cantidad de materia (masa) en el volumen que ocupa.

Su formula es:
d = m/V

6 TAMadrid

-Pág.10-
U . D . 1 - D IVERSIDAD Y U N I D A D D E E S T R U C T U R A D E L A M AT E R I A

La densidad por lo tanto nos va a permitir diferenciar entre cuerpos que ocupan el
mismo volumen pero tienen distinta masa. Por ejemplo dos bolas del mismo tamaño pero
una de plástico y la otra de hierro, ocupan el mismo volumen pero una pesa más que la otra
por tener una más masa que la otra por lo que la densidad de la de mayor masa será también
mayor. Las unidades más empleadas son el g/cm3 y el Kg/m3.

Punto de fusión y punto de ebullición: Los cambios de estado de la materia se


producen a temperaturas determinadas (a presión fija) que son características de cada
sustancia. Por ejemplo el agua funde a 0ºC (punto de fusión) y se transforma en vapor a
100ºC (punto de ebullición), estos puntos para el hierro son 1539ºC y 2750ºC.

Solubilidad: Es la capacidad de un cuerpo para disolverse. Cuando las sustancias se


mezclan íntimamente forman lo que se conoce como disoluciones. Por ejemplo la sal en el
agua. La sal es el soluto (está en menor proporción) y el agua el disolvente (está en mayor
proporción) y decimos que la sal es soluble en agua. Cuanto mayor cantidad de soluto
podamos disolver, decimos que tiene mayor solubilidad en ese disolvente. Por el contrario
otras sustancias como la arena no se puede mezclar y decimos que son sustancias
insolubles.

1.3. ESTADOS DE AGREGACIÓN


La materia se puede presentar en tres estados distintos: sólido, líquido y gaseoso
conocidos como estados de agregación. Podemos inicialmente pensar que el hierro es sólido,
el agua líquida y el aire gaseoso de forma inmutable, pero esto no es así y la materia puede
pasar de un estado a otro en función a la temperatura, presentando un estado característico
a cada temperatura. Un ejemplo muy claro lo podemos ver con el agua que a 0ºC es sólida,
a temperatura ambiente es líquida y a 100ºC es gaseosa (vapor). La presión es el otro factor
que influye en el estado de las sustancias, así, el gas butano cuando se encuentra comprimido
a alta presión dentro de una bombona a temperatura ambiente se encuentra en estado líquido,
mientras que al salir al exterior y bajar la presión se transforma en gas.

Para interpretar fenómenos como el del gas butano y otros, los científicos han propuesto
la teoría cinética, que supone que la materia está compuesta por un gran número de
pequeñas partículas (que podemos representar como pequeñas esferas), en continua
agitación, entre las que existen fuerzas de atracción. La agitación de las partículas está
relacionada con la temperatura a que está sometida la materia. Así en el estado sólido las
partículas están muy empaquetadas permitiendo solamente pequeños movimientos de
vibración. Si aumentamos la temperatura los movimientos de vibración son cada vez más
intensos provocando la dilatación, pero si no cambia la temperatura el sólido ocupa un
volumen fijo y mantiene su forma fija. En el estado líquido las vibraciones son lo
suficientemente amplias para que las partículas abandonen su posición y se deslicen una

TAMadrid
7
-Pág.11-
C I E N C I A S N AT U R A L E S

sobre otras, aunque siempre juntas y normalmente en la parte baja del recipiente. Los
líquidos van a tener forma variable (adaptada al recipiente que lo contiene) y volumen fijo
(un litro de agua ocupa el mismo volumen en una jarra, en una botella o repartida en vasos).
En el estado gaseoso, al ir aumentando la temperatura, las partículas se mueven tanto que
comienzan a escapar a las fuerzas de atracción y se mueven tendiendo a ocupar mayor
volumen. En el estado gaseoso se tiende a ocupar el mayor volumen posible y la forma será
la del recipiente que lo contiene.

Entre los tres posibles estados de la materia se pueden pasar de uno a otro, tanto por
aumento de calor como por pérdida. A partir del sólido, al aumentar la temperatura aumenta
la agitación y llega un momento que ha de pasar a líquido (fusión) y si sigue aumentando
la temperatura pasará a vapor (vaporización). Si por el contrario enfriamos un gas,
disminuye la agitación de sus partículas y se convierte en líquido (condensación o
licuación) y si seguimos enfriando pasa a sólido (solidificación). Existen además ciertos
sólidos que pasan directamente a gas (sublimación) y de gas a sólido (sublimación inversa).

1.4. SISTEMAS HOMOGÉNEOS Y HETEROGÉNEOS


Para poder clasificar la materia, tenemos en primer lugar que determinar si ésta tiene las
mismas propiedades y composición en todos los puntos que la componen.
La materia que presenta zonas con propiedades diferentes, se dice que es heterogénea,
recibe el nombre de mezcla heterogénea (el granito, en el que se distinguen los granos
correspondientes al cuarzo, al feldespato y a la mica; la tierra, etc.).
La materia de aspecto homogéneo, con las mismas propiedades en todos sus puntos se
dice que es homogénea (el agua, el azúcar, etc.).

8 TAMadrid

-Pág.12-
U . D . 1 - D IVERSIDAD Y U N I D A D D E E S T R U C T U R A D E L A M AT E R I A

2. MEZCLAS Y SUSTANCIAS PURAS.


ELEMENTOS Y COMPUESTOS
En la materia heterogénea, siempre tendremos varias sustancias mezcladas (mezcla
heterogénea), mientras que en la materia homogénea nos vamos a encontrar bien con una
única sustancia (sustancia pura) o con una mezcla homogénea o disolución de varias
sustancias que podemos separar por métodos físicos. Las disoluciones se pueden presentar
en todos los estados físicos. El aire es una disolución de distintos gases, el oro de primera
ley es una disolución de plata en oro, el acero es una disolución de carbono en hierro.

Sustancia pura es la que no puede descomponerse en sustancias más sencillas por


procesos físicos. Su aspecto es homogéneo y presenta las mismas propiedades en cualquier
punto que se muestree.

Dentro de las sustancias puras, existen distintas clases según su comportamiento químico.
El agua se descompone al pasar corriente eléctrica de cierta intensidad y sufre cambios
químicos (métodos químicos) transformándose en hidrógeno y oxígeno que ya no se pueden
descomponer. Decimos que el oxígeno y el hidrógeno son sustancias sencillas (todos sus
átomos son iguales). Podemos así decir que las sustancias que no se pueden descomponer en
otras más simples por medios químicos se llaman sustancias simples o elementos. Las que si
se pueden descomponer se llaman sustancias compuestas o compuestos.

Las mezclas son materiales claramente heterogéneos, a simple vista se aprecian los
distintos componentes diferenciados por alguna propiedad; por ejemplo: la arena.

Para separar las mezclas se recurre a procesos físicos y químicos, como ya se ha


mencionado. En los procesos físicos las sustancias conservan su naturaleza. Procesos
mecánicos (filtración, magnetismo -con imanes-, decantación -las sustancias más densas se
sitúan abajo y salen antes-, por diferente solubilidad), sirven para separar mezclas
heterogéneas y no exigen cambios de estado.

Disolución es una mezcla homogénea donde hay un disolvente (el más abundante) y un
soluto (sustancia disuelta).

En el caso de las disoluciones es preciso recurrir al cambio de estado para separar, por
ejemplo mediante evaporación eliminamos el agua y reaparece el sólido que estaba disuelto
cristalizado de nuevo.

La turbidez se debe a la presencia de pequeñas partículas sólidas en el seno del agua,


al dejarla reposar, sedimenta. A estas mezclas heterogéneas se le llama suspensión.

TAMadrid
9
-Pág.13-
C I E N C I A S N AT U R A L E S

La leche está compuesta mayoritariamente por agua, el color blanco opaco se debe a
pequeñas partículas de sustancias insolubles en agua (nata) más pequeñas que las de las
suspensiones por lo que se mantienen sin sedimentar. Se dice que la leche es un coloide.

Las emulsiones son coloides compuestos por dos líquidos inmiscibles dispersos en
minúsculas gotas unidas entre sí por sustancias llamadas emulgentes que actúan como
"pegamento". La mahonesa constituye un ejemplo de coloide.

3. DISCONTINUIDAD DE LOS SISTEMAS MATERIALES.


TEORÍA ATÓMICA. NATURALEZA ELÉCTRICA
DE LA MATERIA

3.1. DISCONTINUIDAD DE LOS SISTEMAS MATERIALES


Toda la materia, aunque muy diversa entre sí tiene en común dos propiedades: masa y
volumen. Toda ella tiene un origen común: las estrellas. De su nacimiento y muerte surgen
todos los átomos que forman el universo. En las estrellas, como si se tratase de auténticas
fábricas se han formado los distintos elementos (aproximadamente un centenar) formadores
del sistema solar y de toda la vida que pueda contener. Podemos decir que la materia está
formada por unas partículas muy pequeñas denominadas átomos, por lo que decimos que la
materia es discontinua.

3.2. TEORÍA ATÓMICA


A principios del siglo XIX, John Dalton elaboró la primera teoría atómica, corregida hoy
en día por una serie de hechos experimentales. En ella se dice que las sustancias simples
(elementos) están formadas por átomos iguales. Estos son indivisibles e inmutables y
pueden juntarse con otros diferentes para dar moléculas, que son las porciones más
pequeñas de las sustancias compuestas (compuestos).

A principios del siglo XX, se descubrieron distintas partículas subatómicas, que eran las
mismas en todos los átomos, tan solo se diferenciaban unos de otros por el número de estas
partículas. En todos los casos, se distingue en los átomos tres clases fundamentales de
partículas subatómicas (electrones, con carga negativa y masa muy pequeña, que se

10 TAMadrid

-Pág.14-
U . D . 1 - D IVERSIDAD Y U N I D A D D E E S T R U C T U R A D E L A M AT E R I A

sitúan en el exterior o corteza, los protones, con carga positiva y masa mayor, que se sitúan
en el interior o núcleo y los neutrones, formados por la asociación de un protón y un
electrón, también en el núcleo). El núcleo, se encuentra en el centro y contiene la práctica
totalidad de la masa muy empaquetada y ocupando un espacio muy pequeño del total del
átomo. La corteza contiene los electrones girando alrededor del núcleo sin parar en órbitas
elípticas. Para que la carga del elemento sea neutra, debe contener el mismo número de
electrones que de protones existen en el núcleo.

El átomo más sencillo es el del hidrógeno que contiene un electrón y un protón, pueden
poseer también uno o dos neutrones, denominados isótopos del hidrógeno (deuterio -un
neutrón- y tritio -dos neutrones-). Un isótopo de un determinado elemento tendrá el mismo
número Z (número atómico), pues tiene el mismo número de protones y diferente
A (número másico) pues poseen diferente número de neutrones (ver en el apartado 4,
clasificación de los elementos químicos).

Al ir aumentando el número de protones y electrones, van apareciendo nuevos


elementos, así el átomo de helio tiene dos protones, dos electrones y dos neutrones y así
sucesivamente.

3.2.1. AGRUPACIÓN DE ÁTOMOS: MOLÉCULAS

La agrupación de dos o más átomos se llama molécula.

Si pudiéramos ver la unidad estructural más pequeña del agua, veríamos que contiene
dos átomos de hidrógeno y uno de oxígeno formando una molécula que será la unidad
mínima del agua, de tal forma que si tenemos sed, de nada nos serviría tomar por separado
el hidrógeno y el oxígeno, que tienen propiedades muy diferentes al agua.

TAMadrid
11
-Pág.15-
C I E N C I A S N AT U R A L E S

Podemos definir las moléculas como la parte más pequeña que sigue teniendo la
misma naturaleza que la sustancia a la que pertenece.
Existen algunas sustancias como los gases nobles (como el neón), en que los átomos se
encuentran libres y se les llama sustancias monoatómicas. Pero en la mayoría de las
sustancias están formadas por moléculas propiamente dichas. Algunas sustancias en estado
sólido no forman moléculas de un número fijo de átomos, sino que forman cristales
(estructuras ordenadas) con un número indefinido de átomos unidos entre sí, este es el caso
de la sal común (NaCl, cloruro sódico) y de ciertos metales como el hierro (Fe).

3.2.2. ELEMENTOS Y COMPUESTOS

Las sustancias simples (o elementos) son aquellas que están formadas por átomos
iguales, ya sean sustancias monoatómicas, o formadas por moléculas o cristales (por
ejemplo el oxígeno -O2- o neón -Ne-).
Las sustancias compuestas o compuestos, son aquellas cuyas moléculas o cristales están
formadas por átomos de distintos elementos. Se representan mediante fórmulas químicas que
nos indican los elementos que forman la sustancia y el número de átomos de cada elemento
o la proporción en que se encuentran (por ejemplo la sal común -NaCl - y el agua- H2O-).

3.3. NATURALEZA ELÉCTRICA DE LA MATERIA


Toda la materia está formada en última instancia por átomos y en éstos existen protones
en el núcleo de carga positiva y electrones en la corteza con carga negativa. Son los
electrones más exteriores los que tienen capacidad en ciertas condiciones de "saltar" de un
átomo a otro. Esta circunstancia tiene dos posibilidades, cuando el electrón se pierde, la
carga total ya no es neutra, sino que hay un protón más que electrones y la carga es positiva.
Cuando el electrón se gana, el cómputo total queda con carga negativa porque hay un
electrón más que protones.
Los electrones se pueden mover y abandonar el átomo, y de esta forma se origina la
electricidad. Los protones y neutrones normalmente no se mueven. En los fenómenos
eléctricos, sólo se mueven los electrones.
Los materiales en los que los electrones se pueden mover de átomo en átomo muy
fácilmente se llaman conductores, la mayoría de los metales son buenos conductores. Si por
el contrario se mueven con dificultad, decimos que el material es aislante. Son aislantes la
madera, el vidrio, la lana, los plásticos y la seda entre otros.

La corriente eléctrica será el movimiento de electrones a lo largo de un conductor.

12 TAMadrid

-Pág.16-
U . D . 1 - D IVERSIDAD Y U N I D A D D E E S T R U C T U R A D E L A M AT E R I A

CLASIFICACIÓN DE LOS ELEMENTOS QUÍMICOS.


4. SISTEMA PERIÓDICO. METALES Y NO METALES.
REGULARIDADES DE LOS ELEMENTOS DEL
SISTEMA PERIÓDICO. UNIÓN ENTRE ÁTOMOS.
EL LENGUAJE QUÍMICO: LAS FÓRMULAS

4.1. CLASIFICACIÓN DE LOS ELEMENTOS QUÍMICOS


La palabra elemento, no sólo expresa la idea de sustancia simple, cuando nos referimos
a un elemento químico, hablamos de todos los átomos que tienen una característica común:
su número atómico (Z), es decir, el número de protones que hay en el núcleo, que es el que
identifica al elemento, por ser este un número fijo para cada uno.
Otro dato a conocer de un elemento es su número másico (A) que es la suma del
número de protones y del número de neutrones del núcleo de un átomo.
Los nombres de los elementos derivan en general del latín, y en algunos casos el nombre
y el símbolo son bastante diferentes entre sí.

4.2. SISTEMA PERIÓDICO


En la tabla periódica los elementos se clasifican en orden creciente de su número
atómico (Z). Como en toda clasificación, se busca agrupar aquellos elementos que tengan
propiedades semejantes. El resultado es una red de grupos (columnas) y periodos (filas).
Los elementos de un grupo tienen propiedades parecidas. Estas propiedades varían
gradualmente al avanzar por los periodos. Conocemos átomos que tienen desde uno hasta
más de un centenar de protones en el núcleo; por ello los científicos distinguen más de 100
elementos ordenados en el sistema periódico.
En el sistema periódico siempre se incluyen algunos datos en cada casilla además del
nombre y el símbolo, los más significativos son el número atómico (Z) y la masa atómica
que es el número de protones y neutrones del núcleo.
A
Todo átomo se representa de la forma Z E , donde E es el símbolo del elemento.
Denominamos isótopos a los átomos de un mismo elemento (igual Z, número de protones)
que tiene distinto número de neutrones. Anteriormente ya se vió el ejemplo del hidrógeno
y sus isótopos. Otro ejemplo es el litio que se presenta en la naturaleza en forma de dos
isótopos: el 73 Li en un 92,5% y el 63 Li en un 7,5%. El 73 Li tiene 3 p+, 4 n y 3 e-, mientras
que 6 Li tiene 3 p +, 3 n y 3 e -.
3

TAMadrid
13
-Pág.17-
C I E N C I A S N AT U R A L E S

14 TAMadrid

-Pág.18-
U . D . 1 - D IVERSIDAD Y U N I D A D D E E S T R U C T U R A D E L A M AT E R I A

4.3. METALES Y NO METALES


Los elementos pueden ser clasificados por sus propiedades en dos grandes grupos:
metales y no metales.
La mayoría de los elementos son metales, como el hierro, el cobre, el aluminio, el sodio,
el mercurio, el estaño, etc. Algunos se encuentran libres en la Naturaleza, como la plata o
el oro, otros se encuentran formando compuestos.
Los metales poseen entre otras las siguientes propiedades:
- Son sólidos a temperatura ordinaria (excepto el mercurio que es líquido).
- Tienen brillo metálico y un color especial grisáceo, excepto el oro amarillo y el
cobre rojizo.
- Conducen bien el calor y la electricidad.
Los no metales no poseen brillo, no son buenos conductores del calor ni de la
electricidad y son sólidos o gaseosos a temperatura ambiente (excepto el bromo que es
líquido).
A estos dos grandes grupos hay que añadir los gases nobles y los semimetales. Los gases
nobles también llamados gases inertes, son gases a temperatura ambiente, no se combinan
con otros elementos químicos (corresponden a la columna VIII) y son He, Ne, Ar, Kr, Xe,
Rn.
Los metaloides, son elementos que no tienen definido su carácter metálico o no
metálico y se encuentran en la línea divisoria: germanio, polonio, antimonio, arsénico, etc.

4.4. REGULARIDADES DE LOS ELEMENTOS


DEL SISTEMA PERIÓDICO
Como ya se ha indicado, la ordenación de los elementos químicos en el sistema
periódico permite encontrar características que son comunes a cada columna y a cada fila.
Entre las características más interesantes van a ser las valencias.

METALES VALENCIA NO METALES VALENCIA

Li, Na, K, Ag 1 H 1
Be,Mg,Ca,Sr,Ba, Zn 2 B 3
Al 3 C 2, 4
Fe, Co, Ni 2, 3 Si 4
Cu, Hg 1, 2 N, P, As 3, 5
Au 1, 3 O 2
Pb, Pt, Sn 2, 4 S, Se, Te 2, 4, 6
F 1
Cl, Br, I 1, 3, 5, 7

TAMadrid
15
-Pág.19-
C I E N C I A S N AT U R A L E S

Para explicar el concepto de valencia, vamos a estudiar con más detalle la estructura de
la corteza del átomo. Los electrones de la corteza de un átomo, están moviéndose en capas
alrededor del núcleo. Los electrones de la última capa son los responsables de la valencia
del elemento y, por tanto, de su comportamiento químico. Por eso, a estos electrones se les
conoce como electrones de valencia. En el sistema periódico, el número de la columna
indica el número de electrones que tienen los átomos en su última capa, esto hace que los
elementos situados en la misma columna tengan propiedades similares.

Los gases nobles, que están en la columna VIIIA, tienen por tanto 8 electrones en su
última capa y son los más estables porque no se combinan con ningún otro elemento. Los
demás átomos, al unirse con otros, tienen tendencia a adquirir la estructura de gas noble, es
decir, los 8 electrones en su última capa, para alcanzar su máxima estabilidad. Esto lo
consiguen ganando, perdiendo o compartiendo los electrones de esta capa. De aquí se
deduce el concepto de valencia, que es el número de electrones ganados, perdidos o
compartidos por un átomo.

4.5. UNIÓN ENTRE ÁTOMOS


Para que los átomos formen moléculas es preciso que se unan de la forma más estable
posible, mediante el establecimiento de enlaces.

Enlace es cualquier tipo de interacción que hace que los átomos de un cristal o una
molécula estén unidos entre sí. En los enlaces son los electrones los encargados de su
formación. Estos enlaces pueden ser de tres tipos: iónico, covalente y metálico.

Enlace metálico: los átomos de los metales se organizan en grandes grupos (cristales
metálicos), haciendo un "fondo común" con sus electrones más externos que se mueven con
libertad por todo el cristal. Así se explica que los metales conduzcan la electricidad. Ej.: el
sodio.

16 TAMadrid

-Pág.20-
U . D . 1 - D IVERSIDAD Y U N I D A D D E E S T R U C T U R A D E L A M AT E R I A

Enlace iónico: Tiene lugar entre un elemento metálico y otro no metálico. Los átomos
del elemento metálico ceden electrones y se convierten en iones positivos (cationes). Los
electrones son captados por los átomos del elemento no metálico para transformarse en
iones negativos (aniones). Los iones positivos y negativos se atraen entre sí mediante unas
fuerzas llamadas electrostáticas. Un ejemplo es la sal común (cloruro sódico - NaCl).

Enlace covalente: Dos átomos no metálicos comparten uno o varios pares de


electrones, por lo que permanecen así unidos, formando una especie de nube entre los
elementos enlazados. Pueden así formarse moléculas (por ejemplo el cloro) o grandes
estructuras llamadas cristales covalentes (por ejemplo: El diamante).

En el proceso de formación de los enlaces, siempre entra en juego una cierta cantidad
de energía química que se pone de manifiesto cuando esos enlaces se rompen para formar
otros, transformando la sustancia mediante reacciones químicas, se dice entonces que los
enlaces que unen los átomos de una sustancia almacenan energía química.

TAMadrid
17
-Pág.21-
C I E N C I A S N AT U R A L E S

4.6. EL LENGUAJE QUÍMICO: LAS FÓRMULAS


Las fórmulas de los elementos se escriben utilizando el símbolo de la tabla periódica
excepto las moléculas diatómicas siguientes: H2, N2, O2, F2, Cl2, Br2, I2.

Entre los compuestos, vamos a ver los binarios llamados así porque están formados por
dos elementos. Los más usuales son las combinaciones de los elementos con el oxígeno,
con el hidrógeno y entre un metal y un no metal.

4.6.1. COMBINACIONES CON EL OXÍGENO

Denominamos óxido a los compuestos formados por un elemento y el oxígeno. Se


formulan colocando a la izquierda el símbolo del elemento y a la derecha el del oxígeno
(O). La valencia del elemento (e) se escribe como subíndice del oxígeno, y la del oxígeno
(que es dos), como subíndice del elemento, y después se simplifican si ello es posible.

Por ejemplo, al combinar el hierro con valencia 2 con el oxígeno también con valencia
2 quedaría la fórmula Fe2O2 que podemos simplificar y queda FeO. También se puede
combinar el hierro con valencia 3 con el oxígeno y su fórmula sería Fe2O3. Estos
compuestos se pueden nombrar de dos formas distintas admitidas: la nomenclatura Stock y
la Sistemática.

Fórmula Stock Sistemática

E2Oe Óxido de elemento(1) (prefijo) óxido de


(valencia del elemento) (prefijo) elemento (2)
FeO Óxido de hierro (II) Monóxido de hierro
Cl2O5 Óxido de cloro (V) Pentaóxido de dicloro
SO2 Óxido de azufre (IV) Dióxido de azufre
CuO Óxido de cobre (II) Monóxido de cobre
(1) La valencia del elemento se escribe en números romanos. No es necesaria si sólo tiene una.
(2)Los prefijos indican el número de átomos: mono=1; di=2; tri=3;tetra=4...No es necesario el prefijo mono.

4.6.2. COMBINACIONES CON EL HIDRÓGENO

Las combinaciones de los elementos con el hidrógeno se llaman hidruros excepto las que
realizan con F,Cl, Br, I, S, Se y Te. Se formulan escribiendo a la izquierda el símbolo del elemento
y a la derecha el del hidrógeno. La valencia del elemento se coloca como subíndice del hidrógeno
y la del hidrógeno (siempre 1) como subíndice del elemento, que no es necesario escribir.

18 TAMadrid

-Pág.22-
U . D . 1 - D IVERSIDAD Y U N I D A D D E E S T R U C T U R A D E L A M AT E R I A

Si se combina el calcio, con valencia 2, con el hidrógeno, con valencia 1, el compuesto


formado será CaH2.

Fórmula Stock Sistemática

EHe Hidruro de elemento (1) (prefijo) hidruro de


(valencia del elemento) elemento (2)
CaH2 Hidruro de calcio Dihidruro de calcio
NaH Hidruro de sodio Hidruro de sodio
CH4 Hidruro de carbono (IV) Tetrahidruro de carbono
(1) La valencia del elemento se escribe en números romanos. No es necesaria si sólo tiene una.
(2)Los prefijos indican el número de átomos: mono=1; di=2; tri=3;tetra=4...No es necesario el prefijo mono.

Algunos hidruros se conocen por su nombre común: NH3, amoníaco; CH4, metano.

Los compuestos de F,Cl, Br, I, S, Se y Se se llaman ácidos hidrácidos. Para formularlos


se escribe el símbolo H a la izquierda y a la derecha el del elemento, utilizando siempre la
menor valencia. En los dos sistemas de nomenclatura, se nombran igual, el nombre del
elemento (acabado en -uro) de hidrógeno. Por ejemplo H2S, sulfuro de hidrógeno.

4.6.3. COMBINACIONES ENTRE METAL Y NO METAL

Las combinaciones entre un metal y un no metal se conocen como sales neutras. Se


formulan colocando el símbolo del metal a la izquierda (A) y el del no metal a la derecha
(B). Las valencias se intercambian, como en los casos anteriores. El no metal actúa siempre
con su valencia menor y el metal puede actuar con cualquiera de sus valencias. Por ejemplo,
si se combina el sodio, con valencia 1, con el azufre, con valencia 2, el compuesto formado
será Na2S.

Fórmula Stock Sistemática

AaBb No metal -uro de metal (pref.) no metal -uro


(1) (valencia del metal) de (pref.) metal
Na2S Sulfuro de sodio Sulfuro de disodio
FeCl3 Cloruro de hierro (III) Tricloruro de hierro
CuBr2 Bromuro de cobre (II) Dibromuro de cobre
(1) La valencia del elemento se escribe en números romanos. No es necesaria si sólo tiene una.
(2)Los prefijos indican el número de átomos: mono=1; di=2; tri=3;tetra=4...No es necesario el prefijo mono.

TAMadrid
19
-Pág.23-
C I E N C I A S N AT U R A L E S

5. ELEMENTOS Y COMPUESTOS MÁS ABUNDANTES


EN LOS SERES VIVOS Y EN LA MATERIA INERTE.
UTILIZACIÓN DE MATERIALES DE INTERÉS
EN LAVIDADIARIA. VERSATILIDAD DELCARBONO EN
LA FORMACIÓN DE COMPUESTOS

5.1. ELEMENTOS Y COMPUESTOS MÁS ABUNDANTES


EN LOS SERES VIVOS Y EN LA MATERIA INERTE

5.1.1. LOS ELEMENTOS DE LA MATERIA VIVA

Los seres vivos, animales y plantas están formados por sustancias, que constituyen lo
que denominamos "materia viva". El 99% de la materia viva está formada por 4 elementos:
carbono, hidrógeno, oxígeno y nitrógeno.

El carbono es el elemento más importante, pues sirve de base en la constitución de las


moléculas de la materia viva al unirse con el hidrógeno, oxígeno, etc.

Otros elementos también son indispensables para la vida, como es el hierro, el calcio, el
sodio y el yodo, entre otros, aunque están en una proporción mucho menor.

Los elementos químicos, individualmente, muestran gran simplicidad; sin embargo, el


conjunto de todos ellos esconde el milagro de la vida. Cuando se unen para formar
moléculas que, a su vez, se organizan en otras moléculas mayores y se asocian en
estructuras celulares, manifiestan la propiedad más compleja y maravillosa que es capaz de
exhibir la materia en el universo: la vida.

Resulta asombroso que, de los más de 100 elementos químicos del sistema periódico,
sólo seis de ellos basten para construir casi todas las moléculas necesarias para formar la
totalidad de los seres vivos. Se les denomina bioelementos y son: C, O, N, H, P y S. A partir
de ellos, como si fueran piezas de un juego de construcción, se originan biomoléculas
formadoras de células vivas, que reciben el nombre de Principios Inmediatos y pueden ser
de seis clases diferentes: agua, sustancias minerales, hidratos de carbono, lípidos,
proteínas y ácidos nucleicos.

20 TAMadrid

-Pág.24-
U . D . 1 - D IVERSIDAD Y U N I D A D D E E S T R U C T U R A D E L A M AT E R I A

5.1.2. LOS ELEMENTOS DE LA MATERIA INERTE

El hidrógeno forma parte de numerosos compuestos, entre los que destaca el agua
(H2O), el gran disolvente y el agua oxigenada (H2O2), de utilidad en la vida diaria como
decolorante y desinfectante.

El nitrógeno es la sustancia simple más abundante del aire y entre sus compuestos
destaca el amoníaco (NH3), utilizado como disolvente doméstico y en la fabricación de
ácidos y abonos.

El carbono que no forma parte de los seres vivos, se presenta en la naturaleza como
diamante y grafito. Éste se ha empleado tradicionalmente en la fabricación de minas de
lápices, aunque en la actualidad, su importancia industrial ha aumentado con la fabricación
de fibra de carbono o de componentes mecánicos para maquinaria.

Son compuestos del carbono y de otros elementos la madera, el papel, el azúcar, los
derivados del petróleo, los plásticos, el algodón, los detergentes, el alcohol, algunos
productos farmacéuticos entre otros.

5.1.2.1. ALGUNOS TIPOS DE SUSTANCIAS

Óxidos. Son sustancias formadas por oxígeno y otro elemento, ya sea metálico o no
metálico. Algunos de ellos son:

Dióxido de carbono u óxido carbónico (CO2). Gas que se encuentra en la atmósfera en


pequeñas cantidades. Es un residuo de nuestra actividad eliminado por la respiración. Las
plantas, sin embargo, lo necesitan para realizar la fotosíntesis.

Óxido férrico o trióxido de dihierro (Fe2O3). Sustancia rojiza que se forma al oxidarse
el hierro. Está presente en algunos minerales mena del hierro, como el hematites.

Sales. Son compuestos formados por cristales iónicos. Dentro de la gran variedad de
sales existentes, la más conocida es:

Cloruro sódico o cloruro de sodio (NaCl). Es la sal común que ponemos en las comidas.
Nuestro organismo la necesita y es preciso reponerla, ya que se pierde por la orina y el
sudor.

Ácidos. Algunos se encuentran en los alimentos, pero los más fuertes son muy
peligrosos pues son muy corrosivos.

TAMadrid
21
-Pág.25-
C I E N C I A S N AT U R A L E S

Ácido clorhídrico (HCl). Nuestro estómago tiene este ácido entre sus jugos digestivos.
Cualquier otra parte de nuestro cuerpo sufriría su acción corrosiva.

Ácido sulfúrico (H2SO4). Más corrosivo aún que el clorhídrico y tan usado como el
anterior en el laboratorio. Es el líquido de la batería del coche, por lo que hay que manejarla
con precaución.

Es importante seguir las indicaciones de las etiquetas de los productos para evitar
accidentes.

5.2. UTILIZACIÓN DE MATERIALES DE INTERÉS


EN LA VIDA DIARIA
El sodio forma parte de la sal común (cloruro de sodio NaCl) y de la sosa (hidróxido de
sodio NaOH). La sosa se utiliza en la fabricación de jabones y lejía, en la industria del papel
y en las refinerías de petróleo. El bicarbonato de sodio, se usa en la fabricación, de pastas
dentífrica y en panadería y bollería para esponjar la masa.

De los compuestos del calcio, el carbonato de calcio (CaCO3) que forma las rocas
calizas, es muy abundante en la corteza terrestre, sirve de materia prima para obtener otros
compuestos. El óxido de calcio o cal viva (CaO) se utiliza para la fabricación de cemento,
vidrio y polvos de blanqueado.

Otro compuesto de interés en la construcción es el sulfato de calcio o yeso (CaSO4), que


se endurece mezclándolo con agua. Un tipo de yeso blanco es la escayola, útil también en
la fabricación de moldes.

Los metales tienen múltiples aplicaciones. El estaño (Sn) sirve para fabricar envases
metálicos y latas de conserva y se utiliza en soldaduras. El hierro es utilizado en la
fabricación de aceros.

Con el niquel (Ni) se hacen monedas y recubrimientos metálicos (niquelados); con el


cobre (Cu), cables eléctricos y aleaciones como el latón con el cinc y el bronce con el estaño
(Sn). El aluminio (Al) es muy ligero y por eso se utiliza en la construcción de aviones,
ventanas, como papel de envolver, etc.

22 TAMadrid

-Pág.26-
U . D . 1 - D IVERSIDAD Y U N I D A D D E E S T R U C T U R A D E L A M AT E R I A

5.3. VERSATILIDAD DEL CARBONO EN LA FORMACIÓN


DE COMPUESTOS

El carbono, es la base de la materia orgánica por una característica especial, es capaz de


formar cadenas muy largas.

El carbono posee cuatro electrones en su última capa y es capaz de combinarse


permitiendo que se vayan uniendo átomos de carbono, rellenando los espacios vacíos hasta
completar los cuatro enlaces entre los átomos de carbono, formando dobles o triples enlaces
o sencillos, rellenando los huecos con hidrógeno (hidrocarburos). Pero sobre todo es la base
de la vida.

Existen varias moléculas como el metano (CH4) que es el principal componente del gas
natural, que es el hidrocarburo más sencillo, otro como el butano (C4H10) posee cuatro
carbonos unidos en cadena y con hidrógeno completando para que cada carbono quede
rodeado por cuatro enlaces.

H H H H

H C C C C H

H H H H

BUTANO

TAMadrid
23
-Pág.27-
C I E N C I A S N AT U R A L E S

RESUMEN

Materia es todo aquello que ocupa espacio. Sus propiedades generales son la masa y el
volumen.

La Química es una ciencia que estudia las propiedades de la materia y sus cambios.

La materia puede encontrarse en tres estados de agregación distintos: sólido, líquido y


gas. Puede pasar de uno a otro al aumentar o disminuir la temperatura.

La teoría cinética explica el comportamiento de la materia suponiendo que está formada


por diminutas partículas que se mueven constantemente de una forma u otra, dependiendo
del estado de agregación.

La densidad, los puntos de fusión y ebullición, y la solubilidad son algunas propiedades


características de la materia.

La densidad es la masa que posee un cuerpo en cada unidad de volumen.

Materia homogénea es la que tiene la misma composición y propiedades en todos sus


puntos. De no ser así, se trata de materia heterogénea.

Si la materia homogénea no se puede separar por medios físicos, se denomina sustancia


pura. Cuando se puede separar en varias sustancias se llama disolución.

Las sustancias puras son elementos si no se pueden descomponer en otras más simples
por medios químicos y son compuestos si la descomposición es posible.

Las mezclas heterogéneas pueden separarse mediante filtración, decantación, diferencia


de magnetismo o de solubilidad, procesos mecánicos basados en las diferentes propiedades
de las sustancias.

Las mezclas homogéneas o disoluciones han de separarse por procesos que supongan
cambios de estado como la cristalización o la destilación.

Los procesos físicos no alteran la naturaleza de la sustancia, los químicos sí lo hacen.

La materia está formada por unas partículas muy pequeñas llamadas átomos.

En un átomo se distinguen dos zonas: el núcleo y la corteza. El núcleo está formado por dos
tipos de partículas subatómicas: los protones y los neutrones. En la corteza se sitúan los electrones.

24 TAMadrid

-Pág.28-
U . D . 1 - D IVERSIDAD Y U N I D A D D E E S T R U C T U R A D E L A M AT E R I A

Molécula es la parte más pequeña de una sustancia que sigue teniendo su misma
naturaleza.

Todos los átomos de las sustancias simples son del mismo tipo; sin embargo, en los
compuestos encontramos átomos de diferentes elementos químicos.

Dos átomos del mismo elemento (mismo número de protones), pueden tener distinto
número de neutrones y, por tanto, distinta masa atómica. Se dice que son isótopos.

Cuando un átomo pierde electrones, se transforma en un ión positivo (catión). Si capta


electrones se convierte en un ión negativo (anión).

La tabla periódica clasifica los elementos agrupándolos según la semejanza de sus


propiedades y en orden creciente de su número atómico, que es el número de protones que
hay en el núcleo.

Enlace químico es la unión que existe entre los átomos que constituye una sustancia.
Pueden ser iónico, covalente y metálico.

Las fórmulas químicas representan una sustancia y expresan su composición.

Los elementos que constituyen casi todas las moléculas de los seres vivos son sólo seis:
C, H, O, N, P y S. Se denominan bioelementos y las moléculas que forman biomoléculas o
principios inmediatos.

TAMadrid
25
-Pág.29-
C I E N C I A S N AT U R A L E S

EJERCICIOS DE AUTOCOMPROBACIÓN

1. La ciencia que estudia las propiedades de la materia y los cambios que en ella se realizan es:

A. La Física. B. La Química.
C. La Teoría Cinética. D. La Botánica

2. La Teoría Cinética dice que las partículas que componen la materia se mueven:

A. En cualquiera de los tres estados. B. En los líquidos y gases.


C. Únicamente en los gases. D. Sólo en los líquidos.

3. Un matráz aforado sirve para:

A. La densidad. B. La masa.
C. El volumen. D. El peso.

4. La materia que presenta distintas zonas con propiedades diferentes se denomina:

A. Homogénea. B. Heterogénea.
C. Sustancia pura. D. Nada de lo anterior

5. El proceso en el que se vaporiza un líquido y después de vuelve a condensar se denomina:

A. Cristalización. B. Decantación.
C. Destilación. D. Vaporización

6. En un átomo se distinguen dos zonas:

A. Protón y neutrón. B. Corteza y Núcleo.


C. Molécula y cristal. D. Nada de lo anterior.

7. La porción más pequeña de una sustancia que sigue teniendo su misma naturaleza se denomina:

A. Molécula. B. Átomo.
C. Ión. D. Protón.

26 TAMadrid

-Pág.30-
U . D . 1 - D IVERSIDAD Y U N I D A D D E E S T R U C T U R A D E L A M AT E R I A

8. En la tabla periódica, los elementos que tienen propiedades parecidas se encuentran:

A. En el mismo periodo. B. Sin ningún orden.


C. En el mismo grupo. D. Todo lo anterior.

9. Una sustancia y su composición se expresa mediante:

A. Su fórmula química. B. Su masa atómica.


C. Sus tipos de enlaces. D. Sus elementos.

10. Las moléculas formadoras de materia viva se llaman:

A. Bioelementos. B. Células.
C. Principios inmediatos. D. Principios activos.

TAMadrid
27
-Pág.31-
C I E N C I A S N AT U R A L E S

RESPUESTAS A LOS EJERCICIOS

11. B
12. A
13. C
14. B
15. C
16. B
17. A
18. C
19. A
10. C

28 TAMadrid

-Pág.32-
U. D. 2 - L A E N E R G Í A

ÍNDICE
Pag.
OBJETIVOS. . . . . . . . . . . . . . . . . . . . . . . . . . . . . . . . . . . . . . . . . . . . . . . . . . . . . . . . . . . . . . . . . 2

INTRODUCCIÓN . . . . . . . . . . . . . . . . . . . . . . . . . . . . . . . . . . . . . . . . . . . . . . . . . . . . . . . . . . . 3

MAPA CONCEPTUAL . . . . . . . . . . . . . . . . . . . . . . . . . . . . . . . . . . . . . . . . . . . . . . . . . . . . . . . 4

DESARROLLO DE CONTENIDOS . . . . . . . . . . . . . . . . . . . . . . . . . . . . . . . . . . . . . . . . . . . 5

1. CUALIDADES DE LA ENERGÍA: PRESENCIA EN TODA ACTIVIDAD,


POSIBILIDAD DE SER ALMACENADA, TRANSFERIDA,
TRANSFORMADA Y DEGRADADA . . . . . . . . . . . . . . . . . . . . . . . . . . . . . . . . . . . . . 5

2. CLASES DE ENERGÍA. ENERGÍA CINÉTICA Y POTENCIAL.


ENERGÍA INTERNA. TEMPERATURA Y SU MEDIDA. FENÓMENOS
ASOCIADOS AL CAMBIO DE TEMPERATURA. CAMBIOS DE
ESTADO . . . . . . . . . . . . . . . . . . . . . . . . . . . . . . . . . . . . . . . . . . . . . . . . . . . . . . . . . . . . . . . 7

3. MECANISMOS DE MODIFICACIÓN DE LA ENERGÍA DE LOS


SISTEMAS: TRABAJO Y CALOR . . . . . . . . . . . . . . . . . . . . . . . . . . . . . . . . . . . . . . 12

4. PRINCIPIO DE CONSERVACIÓN DE LA ENERGÍA . . . . . . . . . . . . . . . . . . . . 15

5. MOVIMIENTO ONDULATORIO. LUZ Y SONIDO . . . . . . . . . . . . . . . . . . . . . . 16

6. LA ENERGÍA Y LA SOCIEDAD ACTUAL. UTILIZACIÓN DE


RECURSOS. ENERGÍAS ALTERNATIVAS . . . . . . . . . . . . . . . . . . . . . . . . . . . . . . 21

RESUMEN . . . . . . . . . . . . . . . . . . . . . . . . . . . . . . . . . . . . . . . . . . . . . . . . . . . . . . . . . . . . . . . . . 24

EJERCICIOS DE AUTOCOMPROBACIÓN . . . . . . . . . . . . . . . . . . . . . . . . . . . . . . . . . . 26

RESPUESTAS A LOS EJERCICIOS . . . . . . . . . . . . . . . . . . . . . . . . . . . . . . . . . . . . . . . . . 27

-Pág.33-
C IENCIAS DE LA N AT U R A L E Z A

OBJETIVOS
Al finalizar el estudio de esta Unidad Didáctica, el alumno será capaz de:

• Observar las distintas formas de energía y descubrir algunas de sus


transformaciones.

• Distinguir entre trabajo y esfuerzo y saber calcular el valor de la potencia


desarrollada al realizar el trabajo.

• Calcular el trabajo realizado como medida de la cantidad de energía


transferida.

• Diferenciar entre calor y temperatura y saber que la temperatura mide el


grado de agitación de las partículas de un cuerpo.

• Valorar críticamente el impacto medioambiental que provoca la utilización


de las distintas fuentes de energía renovables y no renovables.

• Identificar la naturaleza de las ondas y su forma de propagación.

• Conocer los fenómenos que experimentan las ondas tanto luminosas como
sonoras.

-Pág.34-
U. D. 2 - L A E N E R G Í A

INTRODUCCIÓN
L a vida no podría existir sin transportar, transformar y almacenar energía. Está
implicada en todo cuanto acontece en el universo.

Al quemar 100 gramos de carbón, se obtiene aproximadamente la misma energía


que:

- Gastamos al correr durante 30 minutos.

- Se necesita para elevar 150 kilogramos a un metro de altura.

- Gastamos durmiendo 6 horas.

- Consume una bombilla en 4 horas.

- Usando 0,001 gramos de uranio en una central nuclear.

Pero:

¿Qué es energía?

¿Cuántas clases de energía existen?

¿Se transforman unas clases de energía en otras?

¿Es todo lo mismo?.

-Pág.35-
C IENCIAS DE LA N AT U R A L E Z A

M A PA C O N C E P T UA L
RENOVABLES
FUENTES DE ENERGÍA
NO RENOVABLES

se extrae de las LA ENERGÍA


FUERZA EN
ACCIÓN

INTERNA Se manifiesta se es
ELÉCTRICA CAPACIDAD
TÉRMICA
NUCLEAR
QUÍMICA CONSERVA DEGRADA para producir
ELECTRO- FORMAS
MAGNÉTICA
CUANDO que se
TRABAJO TIEMPO
realiza en
cierto
SE TRANSFORMA SE TRANSFIERE

En forma de Que se POTENCIA


ENERGÍA ENERGÍA emplea en P=w/t
CINÉTICA POTENCIAL
CALOR ONDAS
FUERZA Unidad:
WATIO
Eª EN ACCIÓN Eª EN ESPERA AUMENTA LA
E. CINÉTICA A través de DESPLAZAMIENTO

W=F . s RADIACIÓN
Ec=1/2 . m . v2 Ep = m . g . h CAMBIOS DE ELECTRO-
ESTADO MAGNÉTICA
EL VACÍO Y
LA MATERIA LA MATERIA LA LUZ
Unidad JULIO FOTONES
ONDAS SONORAS producidas por
CONVECCIÓN TRYECTORIA
VIBRACIONES RECTILINEA
RADIACIÓN se propaga por es experimenta
experimenta REFRACCIÓN
REFLEXIÓN
CONDUCCIÓN
ENERGÍA EN REFLEXIÓN
TRÁNSITO ECO
DISPERSIÓN

SE TRANSFIERE DE UNA AUMENTO DE LA


SUSTANCIA A OTRA TEMPERATURA

-Pág.36-
U. D. 2 - L A E N E R G Í A

1. CUALIDADES DE LA ENERGÍA: PRESENCIA


EN TODA ACTIVIDAD, POSIBILIDAD DE SER
ALMACENADA, TRANSFERIDA, TRANSFORMADA
Y DEGRADADA

El concepto de energía es uno de los más ampliamente utilizado en la ciencia, y nos


referimos a él cuando analizamos cualquier actividad o cambio que se produzca en una
situación determinada. Todos los fenómenos, sean naturales o artificiales, cumplen la Ley
de la Conservación de la Energía, según la cual la energía total no varía.

1.1. CUALIDADES DE LA ENERGÍA


Se puede hacer una aproximación al concepto de energía mediante la descripción de sus
características. Entre las cualidades de la energía destacan las siguientes:

• Es una propiedad de los cuerpos o sistemas materiales que se manifiesta en las


transformaciones. Por ejemplo si estiras de un muelle y sueltas, éste vibra; si
chutas la pelota, ésta se mueve, si calientas agua, sube su temperatura, etc.

• Es una propiedad que puede transmitirse o transferirse de unos sistemas a


otros. En los ejemplos anteriores se transfiere energía de la mano al muelle, de la
pierna al balón, del fuego al agua.

• La transferencia suele ir acompañada de una degradación que va eliminando


posibles nuevas transmisiones. Esto significa, tomando como ejemplo el agua
calentada al fuego, que después de enfriarse no vuelve a absorber de nuevo la
energía que ha comunicado al medio y, por tanto, no se elevará otra vez su
temperatura hasta la que tenía inicialmente cuando se separó la cazuela del fuego.

• La cantidad total de energía se conserva aunque haya perdido la capacidad de


transferirse. Si tomamos como ejemplo un cuerpo que cae bajo la acción de la
gravedad terrestre, y hacemos el supuesto teórico de que dicho cuerpo no pierde
energía ni por rozamiento con el aire ni en el impacto con el suelo (cuerpo elástico),
entonces el citado cuerpo rebotará y ascenderá hasta la altura inicial, y así
permanecerá eternamente.

TAMadrid
5
-Pág.37-
C IENCIAS DE LA N AT U R A L E Z A

Por lo tanto podemos decir que la energía es una cualidad de los sistemas físicos que se
manifiesta en las transformaciones, que se puede transferir de unos sistemas a otros o
cambiar a otro tipo de energía, pero que se conserva incluso en los cambios que suponen su
degradación total o parcial.

1.2. PRESENCIA EN TODA ACTIVIDAD, POSIBILIDAD DE SER


ALMACENADA, TRANSFERIDA, TRANSFORMADA
Y DEGRADADA
A nuestro alrededor siempre están ocurriendo cosas, pasan autobuses o coches, los niños
juegan en el parque, el perro ladra, el corazón late y los radiadores están calientes. Todas
estas acciones precisan energía para que se produzcan y siempre que sucede alguna de estas
acciones, está presente una transformación de cada tipo de energía en otra.

La formación de las estrellas, la erupción de los volcanes, el nacimiento de las


montañas, la lluvia, el viento, la vida...Todo esto ocurre porque hay energía. Sin energía el
universo sería un lugar frío y sin vida.

La energía es la capacidad para ejercer una fuerza y producir una acción o


modificar algo. Al adquirir energía, los cuerpos pueden sufrir transformaciones: se
mueven, se calientan, deforman, iluminan o explotan. Pero hay que precisar y se dice que
cuando se gasta energía o se traslada energía de un cuerpo a otro se realiza un trabajo.

Podemos así decir que la energía es la capacidad de realizar trabajo. Realizar


trabajo significa que se ejerce una fuerza sobre un objeto y este se desplaza desde una
posición a otra.

Pero, ¿De dónde surge la energía?. Su origen, se confunde con el del cosmos. El
Universo nació hace 15.000 millones de años en una gigantesca explosión (Big Bang). Este
estallido marcó el principio del espacio y el tiempo. Entonces se creó toda la energía que
hay en el universo y una parte de esa energía se transformó en materia. Surgieron cuatro
fuerzas que actúa sobre la materia y rigen el mundo físico (fuerza nuclear fuerte, nuclear
débil, electromagnética y gravitatoria. Y desde aquel momento la energía se convirtió en el
"motor" de todo lo que sucede.

Visto de este modo, la materia no es más que una forma de energía: materia y energía
sería las dos caras de una misma moneda: la materia sería energía condensada y la energía
sería materia descondensada. De hecho, se puede utilizar la propia materia como fuente de

6 TAMadrid

-Pág.38-
U. D. 2 - L A E N E R G Í A

energía, pues la descondensación de pequeñas cantidades de materia suministra grandes


cantidades de energía, como ocurre en las estrellas. La luz y toda la energía radiante del Sol
y de las demás estrellas se obtienen a partir de reacciones nucleares que transforman
pequeñas cantidades de hidrógeno gaseoso en enormes cantidades de energía. Materia y
energía son como dos caras de una misma moneda relacionadas mediante la ecuación
propuesta por Einstein que dice que la Energía es igual al producto de la masa por la
velocidad de la luz al cuadrado (E=m.c2).

La energía se manifiesta de diversas formas y recibe distintos nombres según las


acciones y los cambios que provoca o los fenómenos a los que se asocia.

La energía se esconde, por ejemplo, en el agua embalsada de un pantano: cuando se deja


caer desde cierta altura, la energía se manifiesta asociada al movimiento del agua que, al
mover una turbina, genera electricidad. También se esconde en el carbón y el petróleo:
cuando se queman, se libera y se manifiesta como energía térmica o calor. Todas estas
formas que tiene de manifestarse la energía (interna, eléctrica, térmica, electromagnética,
química y nuclear), no son más que manifestaciones de la energía cinética o de la energía
potencial.

CLASES DE ENERGÍA. ENERGÍA CINÉTICA


2. Y POTENCIAL. ENERGÍA INTERNA. TEMPERATURA
Y SU MEDIDA. FENÓMENOS ASOCIADOS AL
CAMBIO DE TEMPERATURA. CAMBIOS DE ESTADO

2.1. CLASES DE ENERGÍA


La energía aparece en fenómenos y en situaciones de muy diversa índole, la hallamos
en una atleta corriendo, o en el agua de un embalse. Si analizamos las energías relacionadas
con diferentes fenómenos y situaciones, podemos decir que están asociadas, bien al
movimiento de un cuerpo, bien a la posición que dicho cuerpo tiene en un lugar del espacio
donde actúa alguna interacción (gravitatoria, electromagnética, etc.).

TAMadrid
7
-Pág.39-
C IENCIAS DE LA N AT U R A L E Z A

2.2. ENERGÍA CINÉTICA


El tenista de origen croata Igor Ivanisevic ha llegado a imprimir a la pelota una
velocidad en su saque de 226 Km/h. Esto significa que ha transferido una gran cantidad de
energía de sus músculos a la pelota.

La energía que se manifiesta como movimiento se denomina energía cinética (Ec).


Obviamente cuanto mayor sea la energía transmitida en el golpe, mayor será la velocidad
adquirida por la pelota. Si con el mismo golpe se diera a una pelota mucho mayor, la
velocidad con que saldría ya no sería igual. Podemos decir que la energía cinética de un
cuerpo depende de su masa y de su velocidad.

La relación matemática entre esta forma de energía y las variables masa y velocidad es
la siguiente:

Energía cinética (Ec)=1/2 m.v2

La unidad de energía en el Sistema Internacional (S.I. es el Julio, aunque existen otras


unidades de energía. Para ello la masa se mide en Kilogramos y la velocidad en m/s
(metros/segundo)

UNIDADES DE ENERGÍA: SU RELACIÓN ES:


Julio (J) 1 J = 0,24 cal
Caloría (cal) 1 cal = 4,18 J
Kilojulio (KJ) 1KW/h = 3.600.000 J
Kilovatio/hora (KW/h) 1 Kcal = 1000 cal
Kilocaloría (Kcal) 1 KJ = 1000 J

2.3. ENERGÍA POTENCIAL


La energía potencial, es un tipo de energía almacenada que está "a la espera" de realizar
una acción; está asociada con la masa de los cuerpos (m) y la posición que ocupan
(elevados, estirados,...) y la poseen por el hecho de encontrarse bajo la influencia de una
interacción (gravitatoria, electrostática, etc). Va a ser el producto de la masa del cuerpo (m)
por la gravedad (g) y por la altura (h).

8 TAMadrid

-Pág.40-
U. D. 2 - L A E N E R G Í A

La fórmula matemática será Ep=m.g.h

Para expresar la energía potencial en julios, la masa se expresará en Kilogramos, la


gravedad como 9,8 m/s2 y la altura en metros.

2.4. ENERGÍA INTERNA


Las partículas que componen las sustancias (átomos, iones o moléculas) se encuentran
en constante agitación en los tres estados (sólido, líquido y gaseoso) mediante movimientos
imperceptibles de vibración, rotación y otros. Cada partícula en movimiento adquiere una
minúscula cantidad de energía cinética, de manera que si sumamos la energía cinética de
todas ellas, obtenemos la energía total contenida dentro de cada sustancia. Esta energía se
llama energía interna.

2.5. TEMPERATURA Y SU MEDIDA


Es frecuente utilizar incorrectamente las palabras calor y temperatura. Cuando decimos
que un objeto "está caliente", no es porque "tenga mucho calor", sino porque tiene mucha
temperatura, es decir mucha energía interna. Si por alguna causa, la agitación interna de las
partículas de un cuerpo aumenta, más rápidos serán los movimientos y más energía cinética
tendrán. El cuerpo adquiere mayor energía interna, está más caliente y, por tanto, su
temperatura aumenta.

La temperatura de una sustancia es una magnitud física, como la masa, el tiempo y el


espacio, que mide el grado de agitación de sus partículas, es decir, la cantidad de energía
interna que posee, y se mide con el termómetro.

2.6. FENÓMENOS ASOCIADOS AL CAMBIO DE TEMPERATURA

2.6.1. EL CALOR: ENERGÍA EN TRÁNSITO

Cuando los científicos hablan de calor, se están refiriendo a un tipo muy particular de
energía, también llamada energía térmica o calorífica. El calor es energía en tránsito, que
sólo se manifiesta cuando se transfiere energía desde un cuerpo caliente (que posee mayor
temperatura) hasta otro más frío (con menor temperatura): La cantidad de energía
transferida es el calor, y se mide con el calorímetro.

TAMadrid
9
-Pág.41-
C IENCIAS DE LA N AT U R A L E Z A

Puesto que el calor es una forma de energía (energía térmica o calorífica), cuando se
suministra calor a un cuerpo se le está suministrando energía, que se puede transformar en
otras formas de energía (interna, cinética, eléctrica, química, etc.) y también puede realizar
trabajo.

El calor es una forma de energía y se mide en julios (J). Existe además otra unidad, la
caloría (c), que se define como la unidad de calor necesaria para elevar un grado centígrado
la temperatura de un gramo de agua. La equivalencia entre ambas unidades es: 1 caloría
(c)= 4,18 julios (J)

2.6.2. EFECTOS DEL CALOR SOBRE LOS CUERPOS

El calor aumenta la energía interna y la temperatura de los cuerpos por lo que los
movimientos de agitación aumentarán siendo la causa de dos tipos de fenómenos: La
dilatación y los cambios de estado.

2.6.2.1. Dilatación y contracción

La dilatación es un proceso de expansión térmica que experimentan los cuerpos cuando


aumenta la temperatura, ocupando más volumen.

La dilatación es la causa de que las obras de ingeniería, los raíles, edificios y puentes
mantengan a tramos ciertas separaciones para evitar que al dilatarse se monten unos sobre
otros. El agua es singular porque al enfriarse y convertirse en hielo se dilata, es decir,
disminuye su densidad por lo que flota sobre el agua líquida.

2.6.2.2. Los termómetros: miden la dilatación

Los termómetros son los instrumentos que utilizamos para medir la temperatura de los
cuerpos, que es una magnitud física que mide la cantidad de energía interna de una
sustancia. El funcionamiento de los termómetros clásicos se basa en la dilatación y
contracción que experimenta un depósito de mercurio o alcohol teñido encerrado en vidrio
y asciende o desciende por un tubo capilar graduado según una escala. Existen varias
basados en distintos puntos de referencia. La escala Celsius o centígrada se basa en situar
el cero en el punto de congelación del agua y el 100 en la de ebullición, cada unidad es 1
grado centígrado (ºC). Otras escalas como la Farenheit se usa en los países anglosajones, en
esta escala la temperatura de congelación es 32ºF y la de ebullición 212ºF.

ºC
___ ºF-32
Relaciones = _____ ºK = ºC + 273
100 180
10 TAMadrid

-Pág.42-
U. D. 2 - L A E N E R G Í A

2.7. CAMBIOS DE ESTADO


Al estudiar los estados de agregación de la materia vimos que la materia puede
encontrarse en tres estados: Sólido, líquido y gaseoso.

Según la teoría cinética, los cambios de estado se producen cuando el calor incrementa
la energía cinética de las partículas de un cuerpo, por lo que sus movimientos de vibración
se hacen más rápidos, se desplazan más lejos y las fuertes uniones que las mantenían en su
posición comienzan a debilitarse.

Lo que caracteriza a los cambios de estado es que la temperatura se mantiene constante


durante el proceso de cambio de un estado al otro. Mientras dura la fusión, por ejemplo, el
calor no se invierte en aumentar la temperatura del sólido, sino en romper las fuertes
uniones que mantienen a sus partículas en posiciones fijas y en desmoronar sus estructuras
hasta que cambia al estado líquido.

2.8. FORMAS DE PROPAGACIÓN DEL CALOR


El calor es una forma de energía en tránsito que siempre fluye desde un cuerpo caliente
a otro más frío. Pero no siempre se propaga de la misma manera, ya que puede viajar de tres
formas distintas: por conducción, por convección y por radiación.

a) Conducción

El fenómeno de transferencia de energía por conducción es el causante de que


cuando dos cuerpos a distinta temperatura se ponen en contacto se produzca una
transferencia de energía en forma de calor del cuerpo que se halla a mayor
temperatura hacia el que posee temperatura inferior.

La rapidez con la que se propaga la energía depende, entre otras causas, de la


naturaleza de las sustancias, ya que la energía se va transmitiendo por choques entre
las moléculas que se ponen en contacto.

Son buenos conductores térmicos los metales y malos conductores los gases y
materiales sólidos como el corcho, el barro, el hielo y determinadas fibras sintéticas
utilizadas en el aislamiento de edificios.

Por conducción, el calor se transmite sin que haya movimiento entre los cuerpos, a
diferencia de la convección.

TAMadrid
11
-Pág.43-
C IENCIAS DE LA N AT U R A L E Z A

b) Radiación

La energía emitida por el Sol puede llegar hasta nosotros a través del espacio vacío.
Este tipo de energía se denomina radiante.

El calor se propaga por radiación cuando se transfiere mediante un tipo de


ondas llamadas radiación infrarroja que no necesita de medios materiales para su
propagación, pues viaja a través del vacío a la velocidad de la luz.

No sólo los cuerpos a elevada temperatura emiten energía por radiación (estrellas,
filamento incandescente de la bombilla, etc.) sino que también lo hacen otros
cuerpos que irradian a temperatura más baja, como por ejemplo la radiación
nocturna de tipo infrarrojo que emite la superficie de la Tierra.

c) Convección

La propagación del calor por convección sólo se da en los fluidos (líquidos y gases)
en movimiento.

Cuando un foco de calor calienta un fluido (una cacerola con agua puesta al fuego
y tapada), éste se expande, aumenta de volumen y se hace menos denso, por lo que
asciende. El fluido frío se desplaza y ocupa su lugar, hasta que se calienta y también
asciende, creándose una circulación en el seno del fluido. Son las corrientes de
convección, que propagan el calor desde las zonas más calientes hasta las más frías.
En la naturaleza se producen una gran cantidad de fenómenos de carácter
convectivo: vientos, corrientes marinas, tormentas, etc. El aire (o cualquier gas o
líquido) se calienta y sube, se va enfriando y baja.

3. MECANISMOS DE MODIFICACIÓN DE LA ENERGÍA


DE LOS SISTEMAS: TRABAJO Y CALOR

La energía se transfiere o se propaga desde la posición que ocupa un cuerpo a otra


posición, o desde un cuerpo a otro en forma de calor (transferido desde la llama al cazo y
al agua), ondas (ondas microondas) o trabajo.

12 TAMadrid

-Pág.44-
U. D. 2 - L A E N E R G Í A

3.1. TRABAJO

Una de las formas de variar la energía de un sistema es mediante la realización de un


trabajo físico. Se realiza trabajo físico cuando se transfiere energía a un cuerpo que cambia
de una posición a otra.

Realizar trabajo significa que la energía transferida se emplea para ejercer una fuerza
sobre un objeto y provocar su desplazamiento en la misma dirección en que se aplica la
fuerza:

Energía transferida = Trabajo realizado = Fuerza ejercida x Distancia del desplazamiento

Puesto que el trabajo es una medida de la cantidad de energía transferida, la unidad de


medida para ambos -trabajo y energía- es la misma, el julio (J).

No se realiza trabajo si la fuerza no logra desplazar el objeto. Es lo que diferencia el


esfuerzo (cuando no hay desplazamiento) del trabajo (cuando si hay desplazamiento). El
trabajo se representa por la letra W (del inglés Work, que significa trabajo) y su fórmula
matemática es:

W=F.s

Donde F es la fuerza ejercida en Newtons y s es la distancia del desplazamiento (en


metros). (La fórmula matemática para calcular la fuerza F (Newtons) = masa (Kg) x
aceleración (m/seg2)

Si un atleta, levanta unas pesas de 100 kg de masa a 2 metros del suelo, realiza un
trabajo que se puede calcular de la siguiente forma:

W (trabajo) = F (fuerza) x s (distancia)

En este caso, la fuerza (F= masa x aceleración) que se ejerce es la del peso
(masa · gravedad) de las pesas y se calcula como:

F = Peso = m . g y s = altura = h

Por lo tanto,

W = m . g . h = 100 Kg . 9,8 m/s2 . 2 m = 1960 J = Ep

Es decir, el atleta ha gastado 1960 julios de energía muscular, que se ha transferido,


mediante el trabajo realizado, a las pesas; éstas, situadas ahora a dos metros del suelo,
tienen 1960 julios más de energía que cuando estaban en el suelo y está almacenada como

TAMadrid
13
-Pág.45-
C IENCIAS DE LA N AT U R A L E Z A

energía potencial. Debido a esto, por mucho que se esfuerce el atleta, si no consigue mover
las pesas del suelo, no se realiza trabajo porque el desplazamiento s = 0 y las pesas no ganan
ni pierden energía.

Si queremos que un automóvil adquiera mayor velocidad, o, lo que es lo mismo, que su


energía cinética se incremente, sus motores deberán desarrollar como mínimo un trabajo
equivalente a ese aumento de energía cinética, y para ello deberá ejercer una fuerza y
producir un desplazamiento del mismo. Si pretendemos que ese mismo automóvil suba por
una pendiente y que, a su vez, su velocidad sea creciente, el trabajo desarrollado por el
motor tendrá que ser todavía mayor, pues deberá ser, como mínimo, la suma del aumento
de las energías cinética y potencial que experimente el automóvil. Además hay una pérdida
calorífica debida al rozamiento, que también precisa de una energía para vencerla.

Por lo general, cuando una forma de energía se transforma en otra o se transfiere a un


cuerpo, se degrada, pierde calidad y puede realizar menos trabajo; sólo una parte es
utilizable y es capaz de realizar trabajo. El resto se pierde en forma de energías degradadas
como el calor o el ruido. Las máquinas sólo utilizan una parte de la energía que reciben, el
resto se transforma en otras formas de energía residual que supone un desperdicio: la
fricción entre los ejes y los engranajes genera calor y ruido, esto es, energía térmica y
sonora, que no se puede convertir en otra cosa.

Una bombilla eléctrica, tan sólo transforma en energía lumínica un 2% de la energía


eléctrica que recibe, el 98% restante se desperdicia en forma de energía térmica (calor).

3.1.1. POTENCIA

Vamos a introducir el concepto de Potencia (P) para medir el ritmo, es decir, el tiempo
en que se realiza un trabajo o se transfiere o se transforma la energía. La ecuación
matemática que las relaciona es:

P = W/t

La unidad de potencia es el vatio (W) y es igual a 1 julio partido por un segundo.

Imagina que subes hoy las escaleras despacio y ayer las subiste corriendo. En ambos
casos tu peso es el mismo y la altura de las escaleras es la misma, por lo que el trabajo que
realizas es el mismo, pero lo que varia es el tiempo que tardas por lo que lo que varía será
la potencia con que las subes.

14 TAMadrid

-Pág.46-
U. D. 2 - L A E N E R G Í A

3.1.2. LAS MÁQUINAS SIMPLES

El desarrollo de la civilización ha estado basado en la búsqueda de nuevos tipos de


energía y en la invención de máquinas y herramientas que facilitan la realización del
trabajo.

Las máquinas simples como la palanca, el plano inclinado, el torno, la polea, etc. son
instrumentos que facilitan la realización de un trabajo, porque amplifican la fuerza
necesaria para conseguirlo.

3.2. CALOR
Si existen fuerzas de fricción entre dos cuerpos, con movimiento relativo, o se produce
un choque, toda o parte de la energía mecánica se transfiere en forma de energía calorífica
o calor. Este no es el único método de transferencia de energía calorífica. En las reacciones
químicas, principalmente en las de combustión, parte de la energía interna de las moléculas
iniciales (combustible y oxígeno) se transfiere en forma de energía calorífica.

El calor también puede ser una energía de transferencia entre dos cuerpos a distinta
temperatura (hasta que se igualan).

4. PRINCIPIO DE CONSERVACIÓN DE LA ENERGÍA

La energía necesaria para que tenga una acción no se crea de la nada, sino que siempre
procede de otra forma de energía que experimenta una transformación, y es durante la
transformación cuando se ponen de manifiesto las distintas formas de energía.

En cualquier transformación siempre se cumple el principio de conservación de la


energía y dice que la energía ni se crea ni se destruye, solamente se transforma. Cuando una
forma de energía se transforma en otra, la cantidad total de energía que se obtiene después
de la transformación es la misma que la que había al principio, es decir, permanece
constante.

TAMadrid
15
-Pág.47-
C IENCIAS DE LA N AT U R A L E Z A

5. MOVIMIENTO ONDULATORIO. LUZ Y SONIDO

Movimiento ondulatorio, es aquel que avanza en forma de ondas. En las ondas


distinguimos las crestas (que son las partes altas) y los valles (que son las partes bajas).

Cresta
Altura de la onda

Longitud de onda

Un segundo
Valle

Frecuencia: 1,5 ondas por segundo

Entre los fenómenos que se propagan en forma de ondas, encontramos la luz y el sonido.
Existen no obstante muchos tipos de ondas (sonoras, luminosas, de radio, sísmicas...), y no
todas se propagan de la misma manera ni a la misma velocidad. Es importante señalar que
en los movimientos ondulatorios no existe transporte de materia. Cuando tiramos una piedra
al agua y se forman ondas, cuando ponemos un barquito de papel, veríamos que éste sube
y baja al tiempo que las ondulaciones, pero no se movería, es decir, no avanzaría. El oleaje
circular que se observa es la serie de crestas y valles de la onda que recorría el agua, es decir
formas que cambian de lugar.

Una onda es, pues, la forma que tiene de propagarse una perturbación; o bien, una de
las formas que tiene de propagarse la energía en un medio, sin que haya desplazamiento de
materia.

5.1. ONDAS SONORAS


Cuando algo empieza a vibrar se produce un sonido. Cuando la vibración se produce en
el aire, éste vibra y se transmite en forma de ondas sonoras. Cuando éstas se alejan de
nosotros se debilitan. El sonido se mueve en el aire a una velocidad de 340 metros por
segundo (la luz viaja a 300000 Km cada segundo). Los sonidos son altos o agudos si la

16 TAMadrid

-Pág.48-
U. D. 2 - L A E N E R G Í A

vibración es rápida, si es lenta, son bajos o graves. El oído humano no oye todos los sonidos
que existen. No oímos, por ejemplo los ultrasonidos (sonidos muy altos) ni los infrasonidos
(sonidos muy bajos). Algunos animales, sin embargo, sí pueden hacerlo.

Los sonidos que nos llegan habitualmente nos llegan por el aire, pero aún se propagan
mejor a través de líquidos y sólidos. Los indios americanos pegaban la oreja al suelo para
saber si venían jinetes a lo lejos.

Las ondas sonoras rebotan o se reflejan en las barreras. El sonido reflejado es aquel que
cambia de dirección al chocar contra una superficie que no lo absorbe. La reverberación o
persistencia del sonido en espacios cerrados después de haber cesado la emisión del sonido.
Es un fenómeno debido a la reflexión reiterada del sonido en superficies que no lo absorben
adecuadamente (coloquialmente se dice que la sala tiene mala acústica, ya que ésta estudia
el comportamiento de las ondas en interiores).

Otro efecto de choque del sonido es el eco (es el sonido reflejado). El tiempo que tarda
el eco en volver depende de la distancia que existe entre el emisor y la superficie contra la
que choca, el sonar es un aparato de navegación para detectar objetos (submarinos, bancos
de peces,...) midiendo el tiempo que tardan el llegar hasta el "blanco" y volver al punto
de partida.

También se pueden producir interferencias cuando las ondas sonoras se entrecruzan o


combinan entre sí.

5.2. LA LUZ
Luz es todo aquello que nos permite ver y existir. Pero sin algo que la genere la luz no
puede existir. La luz puede ser natural (procedente del Sol) y artificial (eléctrica).

El Sol rige nuestro sistema solar. En su interior se producen reacciones nucleares que
desprenden cantidades ingentes de energía y llegan a la Tierra después de recorrer 150
millones de Kilómetros por el vacío (por el espacio no hay aire, no hay materia, no hay nada,
por eso hablamos de vacío).

La energía del Sol es de tipo radiante, es decir, que viaja y se propaga en forma de ondas,
o sea en todas direcciones. Debido a su naturaleza, la energía solar la denominamos
electromagnética. Esto significa que puede reflejarse (cuando se encuentra con una
superficie que no puede atravesar), refractarse (cuando cambia de medio) y propagarse a
través del vacío y la materia a la velocidad de la luz.

TAMadrid
17
-Pág.49-
C IENCIAS DE LA N AT U R A L E Z A

La luz al atravesar el universo es invisible, al chocar contra la atmósfera, choca contra


las partículas de aire, polvo y agua, dispersándose por reflexión y haciéndose visible a
nuestros ojos y le llamamos luz, dándole color al cielo y las nubes. Pero además de la luz
visible, el Sol emite una amplia gama de ondas o radiaciones electromagnéticas que varían
entre ellas por la longitud de onda en que son emitidas, cada radiación tiene una longitud
de onda determinada y así tenemos, de mayor a menor longitud de onda: ondas de radio o
hertzianas, ondas microondas, rayos infrarrojos, luz visible, rayos ultravioleta, rayos X y
rayos gamma que conforman en total el espectro electromagnético. Entre ellas podemos
mencionar las radiaciones ultravioleta (responsables del bronceado) y las radiaciones
infrarrojas (que aunque invisibles, podemos sentir como calor).

La luz, además de su naturaleza de onda, está constituida por diminutas partículas de


energía llamadas fotones (se dice que tiene naturaleza corpuscular). Otras radiaciones
electromagnéticas poseen también esta doble naturaleza onda-partícula. Otra característica
es que siempre se propaga en línea recta.

Según sea el comportamiento de los cuerpos ante la luz, los cuerpos pueden clasificarse
en opacos (como la madera) que no deja pasar la luz; transparentes (como el cristal), dejan
pasar toda la luz y se ve con claridad y translúcidos (como el cristal esmerilado) que dejan
pasar la luz pero a través de ellos no se ve con nitidez.

La luz es una forma de energía que pone en funcionamiento la vida en la Tierra. El


primer escalón para conseguirlo es el proceso de la fotosíntesis por la cual el pigmento
verde de las plantas (clorofila) es capaz de capturar los fotones de la luz y fabricar su propio
alimento a partir de agua, sales minerales, dióxido de carbono y luz solar. Como
consecuencia de este proceso liberan oxígeno como producto de desecho. Así, las plantas
serán siempre el primer escalón de cualquier cadena alimentaria que exista en la naturaleza.

5.3. FENÓMENOS DE REFLEXIÓN Y REFRACCIÓN


La formación de imágenes depende de los cambios que experimenta la luz cuando choca
con una superficie.

Cuando la luz llega a una superficie que no puede atravesar, "rebota" sufriendo un
repentino cambio de dirección que se denomina reflexión.

Todos los cuerpos reflejan una parte de la luz que les llega, si no fuera así no los
veríamos. Las superficies brillantes reflejan más luz que las opacas y las claras más que las

18 TAMadrid

-Pág.50-
U. D. 2 - L A E N E R G Í A

oscuras, ya que estas absorben casi toda la luz que les llega. Un espejo es una superficie
pulida capaz de formar imágenes por reflexión de los rayos luminosos procedentes de los
objetos. Cuando el espejo es convexo, los rayos divergen después de reflejarse mientras que
en un espejo cóncavo, tras la reflexión, los rayos reflejados se unen en un punto (foco)
formando una imagen reducida e invertida.

Ángulo incidente Ángulo reflejado

Normal
El rayo que llega a una superficie
Rayo Rayo plana, o incidente, y el que sale, o
incidente reflejado
reflejado, forman ángulos iguales
respecto a la normal.

Espejo Convexo

Los rayos paralelos que chocan


con un espejo convexo se
dispersan o divergen desde un
punto después de reflejarse, y
forman una imagen virtual
reducida y derecha.

Espejo Cóncavo
Los rayos paralelos que chocan
con un espejo cóncavo, se unen o
convergen en un punto, llamado
foco, después de reflejarse, y
forman una imagen reducida e
invertida, si el objeto está lejos, y
derecha y aumentada, si está cerca.
Foco

El fenómeno de la refracción de la luz se produce cuando ésta pasa de unos medios a


otros que aunque permiten el paso, lo hace a velocidades distintas produciendo
"desviaciones" de la trayectoria inicial cambiando la dirección. A las sustancias que
permiten el paso de la luz se les asigna un número, llamado índice de refracción, que indica

TAMadrid
19
-Pág.51-
C IENCIAS DE LA N AT U R A L E Z A

la capacidad que tiene un medio para refractar la luz, cuanto más refracta la luz una
sustancia, más despacio viaja por ella y, por tanto, el índice de refracción es más alto.

rayo incidente normal (N)

ángulo de
incidencia aire
ángulo de agua
refracción

rayo refractado

rayo incidente N

Se observa que cuando un rayo


aire pasa de un medio más rápido (aire)
agua a otro más lento (agua), se acerca a
la normal.

1 rayo refractado

N
rayo refractado

Cuando un rayo pasa de un medio


aire
más lento (agua) a otro más rápido
agua
(aire), se aleja de la normal.

2 rayo incidente

Si un rayo llega perpendicularmente a la superficie de separación de los dos medios, no


se refracta, es decir, entra sin desviarse.

Aprovechando estos fenómenos, el hombre ha ideado la posibilidad de usar estas lentes


convexas o divergentes y cóncavas o convergentes para captar imágenes (cámaras de
fotografía), amentar las cosas muy pequeñas (lupas y microscopios), acercar visualmente
objetos lejanos (prismáticos), y corregir defectos visuales.

20 TAMadrid

-Pág.52-
-Pág.52-
U. D. 2 - L A E N E R G Í A

5.4. LA DESCOMPOSICIÓN DE LA LUZ BLANCA: LOS COLORES


La luz que nosotros percibimos, parece blanca o incolora, pero en realidad es un
espectro de radiaciones que emiten en varios colores. Al hacer pasar la luz blanca por un
prisma (el arco iris actúa de igual forma), la luz blanca se descompone y podemos ver los
colores del espectro visible (rojo, naranja, amarillo, verde, azul, añil y violeta). La suma de
todos los colores, da el blanco, mientras que la suma de los colores que denominamos
primarios (rojo, azul y amarillo), da el negro.

LA ENERGÍA Y LA SOCIEDAD ACTUAL. UTILIZACIÓN


6. DE RECURSOS. ENERGÍAS ALTERNATIVAS

6.1. LA ENERGÍA Y LA SOCIEDAD ACTUAL


La energía siempre ha supuesto una necesidad para calentarse, para mover las máquinas,
para cocinar..., pero en la actualidad las necesidades energéticas son mayores dado que para
casi todas las actividades que realizamos se precisa la energía (tanto a nivel doméstico como
industrial), para que se produzca el avance de la humanidad, se precisa obtener energía de
nuevas fuentes energéticas.

Se llaman fuentes de energía a los fenómenos de la naturaleza o a las sustancias de las


que se puede extraer energía.

6.2. UTILIZACIÓN DE RECURSOS


Las fuentes energéticas existentes se pueden dividir en dos grupos: Unas que se agotan
con el tiempo, las no renovables, y otras que son inagotables, las renovables.

6.2.1. ENERGÍAS NO RENOVABLES

Las fuentes de energía que la Tierra no renueva de forma inmediata, se llaman fuentes
no renovables. Las principales son los combustibles fósiles y la energía nuclear.

TAMadrid
21
-Pág.53-
C IENCIAS DE LA N AT U R A L E Z A

Las fuentes de energía como el carbón, el gas natural y el petróleo se llaman


combustibles, porque utilizan la energía química almacenada en sus enlaces para generar
calor durante su combustión y realizar trabajo.

El carbón se originó por la transformación de los bosques de eras pasadas, el petróleo y


el gas por la acción de antiguos organismos microscópicos de los océanos. Por eso se llaman
combustibles fósiles. Por otro lado, en las centrales nucleares, el "combustible" son los
átomos de uranio, que no se quema, tan sólo se parten los átomos en un proceso que se
llama fisión nuclear, liberando una enorme cantidad de energía.

Las energías no renovables, son las fuentes de energía más utilizadas, pero presentan dos
grandes inconvenientes: se agotan y contaminan. Al ritmo actual de consumo, se calcula que
queda carbón para cinco siglos y petróleo y gas para cinco décadas, siendo precisos
millones de años para que la Tierra vuelva a crear más combustibles de estos tipos.

La contaminación se produce cuando se obtienen (impacto medioambiental por


contaminación de ríos, destrucción del paisaje, etc.) y cuando se utilizan. El carbón, el gas
y el petróleo, cuando se queman, emiten gases a la atmósfera, entre ellos el dióxido de
carbono, responsable del efecto invernadero (que produce el calentamiento del planeta) y
óxidos de azufre, responsables de la lluvia ácida, que causa la destrucción de los bosques.
Los residuos de las centrales nucleares, llamados cenizas, son aún muy radiactivos, y su
peligro perdura miles de años.

6.2.2. ENERGÍAS RENOVABLES O ALTERNATIVAS

Se basan en el aprovechamiento de fenómenos de la naturaleza. Son fuentes de energía


renovable las que la Tierra posee de forma inagotable. Como evitan el consumo de
combustibles fósiles, reciben también el nombre de fuentes alternativas.

ORIGEN ENERGÍA
Sol Solar
Mareas Maremotriz
Presas Hidráulica
Calor de la Tierra Geotérmica
Viento Eólica
Restos orgánicos Biomasa

22 TAMadrid

-Pág.54-
U. D. 2 - L A E N E R G Í A

Casi todas las fuentes de energía se emplean en generar electricidad, pues es la forma de
energía más fácil de transportar y de reutilizar.

En los ríos se construyen presas, para regular el caudal, permitiendo aprovechar la


energía cinética del agua y transformarla en electricidad. En los mares, se aprovecha la
subida de las mareas para hacer girar turbinas y producir electricidad, de la misma forma
que se utiliza el viento.

La energía solar, se recoge en paneles, que son dispositivos fotovoltaicos donde se


convierte directamente en electricidad o bien se recoge en colectores que permite calentar
agua para uso doméstico. La energía geotérmica, se emplea en países con actividad
volcánica, como Islandia. Se utiliza el calor interno de la Tierra para transformar el agua en
vapor y generar electricidad, incluso se utiliza el agua caliente que emana a la superficie
para los circuitos de calefacción de las casas.

La biomasa es otra fuente de energía que se utiliza cada vez más en los lugares en que
se produce. Son restos orgánicos, por ejemplo, las ramas y hojas que quedan tras talar los
árboles, y el metano producido por una fermentación de los excrementos.

Es importante concienciarnos del ahorro energético, intentando, en la medida de lo


posible la utilización de fuentes de energía renovables.

TAMadrid
23
-Pág.55-
C IENCIAS DE LA N AT U R A L E Z A

RESUMEN

Energía significa fuerza en acciòn y se define como la capacidad para realizar trabajo,
esto es, ejercer una fuerza sobre un objeto y desplazarlo de una posición a otra; ambas
magnitudes, trabajo y energía se miden en julios.

La energía se manifiesta de diversas formas y recibe distintos nombres -interna,


eléctrica, térmica, electromagnética, química, nuclear- según las acciones y los cambios que
provoca o los fenómenos a los que se asocia, aunque todas ellas son manifestaciones de la
energía cinético y la energía potencial.
El principio de la conservación de la energía afirma que ésta no se crea ni se destruye,
solamente se transforma.
La energía se transfiere o se propaga desde la posición que ocupa un cuerpo a otra
posición o desde un cuerpo a otro en forma de calor, ondas o trabajo.
Cuando la energía se transforma o se trasfiere, se conserva la cantidad pero no la
calidad, pues después de la transformación o transferencia se degrada y puede realizar
menos trabajo.
Las máquinas simples como la palanca, el plano inclinado, el torno o la polea son
instrumentos que facilitan la realización de un trabajo pues amplifican la fuerza necesaria
para conseguirlo.
La temperatura de una sustancia es una magnitud física que mide el grado de agitación
o energía cinética de sus partículas. Se mide con el termómetro en grados centígrados (ºC)
o grados Farenheit (ºF).
El calor es energía en tránsito que solo se manifiesta cuando se transfiere, desde un
cuerpo caliente (que posee mayor temperatura) hasta otro más frío (posee menor
temperatura); la cantidad de esta energía se mide en julios o calorias.
La dilatación es un proceso de expansión térmica que experimentan los cuerpos cuando
aumenta la temperatura.
El calor fluye o se propaga desde un cuerpo caliente a otro más frío por conducción,
convección o radiación.
Se llaman fuentes de energía a los fenómenos de la naturaleza o a las sustancias de las
que se puede extraer energía.
Una onda es la forma que tiene de propagarse una perturbación (una forma de
propagación de energía) sin que haya desplazamiento de materia. La luz, la voz y los
seismos se propagan de esta manera.

24 TAMadrid

-Pág.56-
U. D. 2 - L A E N E R G Í A

Las ondas sonoras se producen cuando algo empieza a vibrar y no pueden viajar por el
vacio, necesitan medio material. Las ondas electromagnéticas o energía radiante, pueden
viajar a través del vacío (y tambien de la materia) a la velocidad de la luz.
Cada radiación electromagnética tiene una longitud de onda y una cantidad de energía
característica. El conjunto de todas ellas, ordenadas en función de su longitud de onda (o de
su frecuencia) se llama espectro electromagnético. La luz es un tipo de radiación
electromagnética que pone en marcha la vida en la Tierra gracias a la fotosíntesis.
La luz se propaga en línea recta en forma de rayo luminoso. Cuando cambia de medio
se refracta y cuando rebota y cambia de dirección se refleja.

TAMadrid
25
-Pág.57-
C IENCIAS DE LA N AT U R A L E Z A

EJERCICIOS DE AUTOCOMPROBACIÓN

1. Indica cual de estas expresiones de la energía cinética es la correcta:


1 1
A. E = m.c2 B. Ec = m.v2 C. Ec = – m.v2 D. Ec = – m.v2
2 3

2. Indica cual de estas expresiones de trabajo es la correcta:


A. W = F/s B. W = F+s C. W = F.s D. W = F – s

3. Indica cual de estas expresiones de la energía potencial gravitatoria es la correcta:


1
A. Ep = –.m.g.h. B. Ep = m.g.h2 C. Ep = m.g.h D. Ep = g.h
2

4. Cuando un automóvil se mueve su motor transforma la energía química almacenada en el


combustible en:
A. Energía cinética. B. Energía calorífica.
C. Energía sonora (ruido). D. Energía mecánica.

5. La temperatura de una sustancia es una magnitud física que mide:


A. El calor que se transfiere de un cuerpo más caliente a otro más frío.
B. La energía interna de la sustancia.
C. La energía cinética o agitación de sus partículas.
D. Ninguna de las anteriores

6. Cuando el agua se congela y se convierte en hielo:


A. Se expande y se dilata. B. Se contrae.
C. Se hace menos densa. D. Se hace más densa.

7. El proceso mediante el cual las partículas de un sólido pasan directamente al estado gaseoso se
denomina:
A. Vaporización. B. Licuación. C. Sublimación. D. Ebullición.

8. Las corrientes de convección propagan el calor en los:


A. Sólidos, líquidos y gases. B. Líquidos y gases.
C. Gases. D. Sólidos.

26 TAMadrid

-Pág.58-
U. D. 2 - L A E N E R G Í A

9. Las ondas sonoras se propagan:


A. En el aire. B. En el agua.
C. En el vacío. D. Ninguna de las anteriores.

10. Las sombras se producen cuando la luz incide sobre cuerpos:


A. Opacos. B. Transparentes.
C. Translúcidos. D. Ninguna de las anteriores.

TAMadrid
27
-Pág.59-
C IENCIAS DE LA N AT U R A L E Z A

RESPUESTAS A LOS EJERCICIOS

1. C
2. C
3. C
4. A, B, C
5. B, C
6. A, C
7. C
8. B
9. A, B
10. A

28 TAMadrid

-Pág.60-
U. D. 3 - L O S CAMBIOS QUÍMICOS

ÍNDICE
Pag.

OBJETIVOS . . . . . . . . . . . . . . . . . . . . . . . . . . . . . . . . . . . . . . . . . . . . . . . . . . . . . . . . . . . 2

INTRODUCCIÓN . . . . . . . . . . . . . . . . . . . . . . . . . . . . . . . . . . . . . . . . . . . . . . . . . . . . . . 3

MAPA CONCEPTUAL . . . . . . . . . . . . . . . . . . . . . . . . . . . . . . . . . . . . . . . . . . . . . . . . . . 4

DESARROLLO DE CONTENIDOS . . . . . . . . . . . . . . . . . . . . . . . . . . . . . . . . . . . . . . . 5

1. INTRODUCCIÓN A LAS TRANSFORMACIONES QUÍMICAS.


CONSERVACIÓN DE LA MASA. ECUACIÓN QUÍMICA,
SU SIGNIFICADO . . . . . . . . . . . . . . . . . . . . . . . . . . . . . . . . . . . . . . . . . . . . . . . . . 5

2. INTERCAMBIOS ENERGÉTICOS EN LAS REACCIONES QUÍMICAS.


ALGUNAS REACCIONES DE INTERÉS (REACCIONES ÁCIDO-BASE,
COMBUSTIONES, ETC.) . . . . . . . . . . . . . . . . . . . . . . . . . . . . . . . . . . . . . . . . . . . 7

3. ALGUNOS FACTORES QUE MODIFICAN EL PROCESO DE LAS


REACCIONES QUÍMICAS. ANÁLISIS CUALITATIVO DE LOS MISMOS.
CATALIZADORES . . . . . . . . . . . . . . . . . . . . . . . . . . . . . . . . . . . . . . . . . . . . . . . . . 9

4. IMPORTANCIA DE LAS REACCIONES QUÍMICAS EN RELACIÓN CON


ASPECTOS ENERGÉTICOS, BIOLÓGICOS Y DE FABRICACIÓN DE
MATERIALES. . . . . . . . . . . . . . . . . . . . . . . . . . . . . . . . . . . . . . . . . . . . . . . . . . . . 10

RESUMEN . . . . . . . . . . . . . . . . . . . . . . . . . . . . . . . . . . . . . . . . . . . . . . . . . . . . . . . . . . . 13

EJERCICIOS DE AUTOCOMPROBACIÓN . . . . . . . . . . . . . . . . . . . . . . . . . . . . . . . 14

RESPUESTAS A LOS EJERCICIOS . . . . . . . . . . . . . . . . . . . . . . . . . . . . . . . . . . . . . 16

-Pág.61-
C IENCIAS DE LA N AT U R A L E Z A

OBJETIVOS
Al finalizar esta Unidad Didáctica, el alumno será capaz de:

• Definir ecuación química y distinguir los reactivos de los productos.

• Representar reacciones mediante reacciones químicas.

• Definir y enumerar reacciones de síntesis, de descomposición, de sustitución y de


combustión.

• Explicar la fotosíntesis y la respiración de los seres vivos.

• Conocer la importancia de los procesos químicos industriales para el progreso de


la humanidad, los problemas medioambientales que pueden generar y su
repercusión sobre la vida en nuestro planeta.

-Pág.62-
U. D. 3 - L O S CAMBIOS QUÍMICOS

INTRODUCCIÓN
E n las reacciones químicas, unas sustancias se transforman en otras. Son la base
para, entre otros, de todos los procesos que tienen lugar para mantener la vida:
la nutrición, la respiración, etc.

También intervienen en numerosos procesos industriales que contribuyen a mejorar


nuestras vidas, proporcionarnos energía, etc.

¿Sabemos qué es una reacción química?

¿Son todas las reacciones iguales?

¿Sabemos cuáles son las reacciones básicas para el mantenimiento de la vida en la


Tierra?.

-Pág.63-
C IENCIAS DE LA N AT U R A L E Z A

M A PA C O N C E P T UA L
.

LAS REACCIONES QUÍMICAS se distinguen diferentes tipos

REACCIONES DE
son se representan SÍNTESIS
simbólicamente a través de A+B AB

TRANSFORMACIONES
LAS ECUACIONES REACCIONES DE
DE LA MATERIA
QUÍMICAS DESCOMPOSICIÓN
AB A+B
CATALIZADORES

que producen
que se ajustan con unos
REACCIONES DE
SUSTITUCIÓN
AB+C AC+B
CAMBIOS DE UNA O AB+CD AC+BD
COEFICIENTES
VARIAS SUSTANCIAS
(REACTIVOS)

para que resulte MANTIENEN LA VIDA


en

IGUAL Nº DE ÁTOMOS DE mediante


OTRAS NUEVAS CADA CLASE EN LOS
O PRODUCTOS REACTIVOS Y EN LOS
PRODUCTOS
RESPIRACIÓN FOTOSÍNTESIS

producen CONTAMINACIÓN CONTROL

PROGRESO Y
AVANCES

-Pág.64-
U. D. 3 - L O S CAMBIOS QUÍMICOS

INTRODUCCIÓN A LAS TRANSFORMACIONES


1. QUÍMICAS. ECUACIÓN QUÍMICA, SU SIGNIFICADO.
CONSERVACIÓN DE LA MASA

1.1. INTRODUCCIÓN A LAS TRANSFORMACIONES QUÍMICAS


Las mezclas son fenómenos físicos, en ellas las sustancias que intervienen siguen
conservando sus propiedades. Sin embargo, cuando se produce un fenómeno químico las
sustancias se transforman en otras diferentes. En estas transformaciones se parte de unas
sustancias en el estado inicial (reactivos) y se obtienen otras diferentes en el estado final
(productos).

reacción química
estado inicial estado final

Hay que señalar que la mayoría de las transformaciones químicas no se producen de


forma espontánea, tiene que encontrarse en condiciones favorables, que van a ser distintas
para cada reacción.

1.2. CONSERVACIÓN DE LA MASA


El científico francés Lavoisier enunció en 1789, la Ley de Conservación de la Masa en
las reacciones químicas, que dice: la suma de las masas de los reactivos es igual a la suma
de las masas de los productos obtenidos. Esto ocurre tanto a escala atómica, como a escala
de laboratorio y a escala industrial. Tomando por ejemplo la producción de dióxido de
carbono (CO2) a partir de sus componentes, tendríamos:

C (sólido) + O2 (gas) CO2 (gas)

12 umas + 32 umas = 44 umas

12 gramos + 32 gramos = 44 gramos

12 Kg + 32 Kg = 44 Kg

TAMadrid
5
-Pág.65-
C IENCIAS DE LA N AT U R A L E Z A

Se denomina uma a la unidad de masa atómica, y coincide con el número másico A que
aparece en el sistema periódico.

1.3. ECUACIÓN QUÍMICA, SU SIGNIFICADO


Un método sencillo y cómodo para representar lo que ocurre en una transformación
química consiste en formular las sustancias que intervienen en la reacción. Por ejemplo,
para representar la obtención del agua a partir de las sustancias simples de sus elementos
escribimos:

estado inicial estado final

hidrógeno + oxígeno agua

Según este esquema, una molécula de hidrógeno reaccionaría con otra de oxígeno para
dar una molécula de agua.

Pero si contamos los átomos que forman estas moléculas, observamos que intervienen
dos átomos de oxígeno y se forma una molécula de agua (se precisa sólo un átomo de
oxígeno). Esto no ocurre realmente, pues no se puede perder un átomo de oxígeno. Por ello
se hace necesario ajustar la reacción, de tal forma que el número de átomos de cada clase o
elemento sea el mismo en el estado inicial y en el estado final, porque la materia se conserva
tanto en los fenómenos físicos como en los químicos.

En la reacción anterior basta con introducir un 2 delante del agua para que aparezcan los
dos átomos de oxígeno iniciales, pero ahora ya no tenemos 2 átomos de hidrógeno en el
estado final sino el doble, o sea 4. Para resolverlo basta con poner otro 2 delante del
hidrógeno del estado inicial, con lo que la reacción quedaría:

2 H2 + O2 2 H2O

Con este esquema expresamos que dos moléculas de hidrógeno reaccionan con una de
oxígeno para dar dos moléculas de agua. Se trata de una ecuación química.

Podemos intentarlo igualmente para la síntesis del amoníaco:

N2 + H2 NH3

6 TAMadrid

-Pág.66-
U. D. 3 - L O S CAMBIOS QUÍMICOS

Para ajustar esta reacción se coloca un 2 delante del NH3 y así igualar el número de
átomos de nitrógeno (2), y un 3 delante del H2 para igualar del número de átomos de
hidrógeno (6). La reacción ajustada es:

N2 + 3 H2 2 NH3

2. INTERCAMBIOS ENERGÉTICOS
EN LAS REACCIONES QUÍMICAS.
ALGUNAS REACCIONES DE INTERÉS

2.1. INTERCAMBIOS ENERGÉTICOS EN LAS REACCIONES


QUÍMICAS
Las reacciones químicas pueden ser fundamentalmente de dos tipos: de descomposición
y de síntesis.

En las de descomposición se observa que se puede producir una reacción química a


partir de una sola sustancia. Si colocamos unos trocitos de mármol en un tubo de ensayo y
calentamos fuertemente a la llama de un mechero de laboratorio, se observa que se
desprende un gas (dióxido de carbono CO2) y queda un residuo blanco (cal viva u óxido de
calcio, CaO). Son reacciones de descomposición las de las sales, como el ejemplo visto, La
descomposición de los óxidos por el calor y la del agua al paso de la corriente eléctrica.

Las reacciones de síntesis o de combinación se basan en la obtención de un compuesto


a partir de las sustancias simples que poseen sus mismos elementos, por ejemplo, la del
dióxido de azufre a partir del azufre y el oxígeno.

Si se colocan unos gramos de azufre en polvo en una cápsula de porcelana y, con


precaución, se acerca una cerilla encendida, el azufre arde en presencia del oxígeno del aire
y produce dióxido de azufre (SO2), gas tóxico de olor sofocante.

Según la necesidad de energía para que se produzca la reacción las dividimos en


endotérmicas, cuando precisan calor para que reaccionen y exotérmicas, cuando
desprenden calor.

TAMadrid
7
-Pág.67-
C IENCIAS DE LA N AT U R A L E Z A

2.2. ALGUNAS REACCIONES DE INTERÉS (REACCIONES


ÁCIDO-BASE, COMBUSTIONES, ETC.)

2.2.1. LAS COMBUSTIONES

Cuando arde un trozo de papel, se produce una reacción de combustión y se obtiene


energía térmica, que puede ser aprovechada, y energía luminosa, que se manifiesta en la
llama formada. Para que comience a arder el trozo de papel hay que aplicar una llama, así
las moléculas de las sustancias que constituyen este material entran en contacto con las del
oxígeno y dan lugar al dióxido de carbono y agua. Sin esta ayuda inicial, el papel no se
prende, aunque esté en contacto con el aire.

La combustión de gas metano puede esquematizarse en la siguiente reacción química:

CH4 + 2 O2 CO2 + 2 H2O

Pero no en todas las combustiones se obtienen las mismas sustancias, en algunas sólo se
obtiene dióxido de carbono, como ocurre cuando se quema carbón, e incluso puede que no
se desprenda ni este gas, como ocurre al arder el azufre o el magnesio, en cuyas
combustiones se obtiene, respectivamente, dióxido de azufre (SO2) y óxido de magnesio
(MgO).

Para que se inicie la combustión es necesario comunicar una energía inicial al


combustible para que adquiera la alta temperatura que necesita este proceso, aunque
después la reacción prosiga por sí misma desprendiendo energía.

Son combustibles, que también contienen carbono e hidrógeno como el papel y la


madera, el alcohol, las grasas, aceites, ceras, el gas natural y los derivados del petróleo,
como las gasolinas, el butano, el propano, etc. Otros combustibles, solo contienen carbono,
como el carbón e incluso no lo contienen, como el azufre y otros materiales.

En toda combustión es indispensable el oxígeno.

Cuando no hay suficiente oxígeno para que la reacción de combustión se produzca en


condiciones óptimas, la combustión no es completa, y se desprende un gas venenoso (el
monóxido de carbono, CO). Esto puede suceder al quemarse la gasolina en el motor de un
coche dentro de un garaje, o cuando arde la leña o el carbón en un recinto poco ventilado.
Cuando la combustión es incompleta, se origina también hollín, formado por partículas de
carbón.

8 TAMadrid

-Pág.68-
U. D. 3 - L O S CAMBIOS QUÍMICOS

2.2.2. REACCIONES ÁCIDO-BASE O DE NEUTRALIZACIÓN

La neutralización es una reacción que se produce cuando a una sustancia ácida se le


añade otra básica. Las frutas, el vinagre, el yogur, etc., contienen sustancias ácidas. Son
sustancias básicas, entre otras, el amoníaco y la sosa.

La picadura de abeja contiene una sustancia ácida que se puede neutralizar con
amoníaco. La de la avispa contiene una básica que se alivia con vinagre.

La reacción general de sustitución entre un ácido y una base es:

Ácido + base sal + agua


Reacción de
Neutralización.

Es decir, si por ejemplo, a una disolución de hidróxido de sodio (sosa NaOH) se añade
ácido clorhídrico (HCl), el hidrógeno del ácido es desplazado por el sodio y éste ocupa su
lugar obteniéndose el cloruro de sodio (sal común) y agua.

HCl + NaOH NaCl + H2O

Para medir lo ácido y lo básico de una sustancia se emplea la escala de pH. Está dividida
en 14 unidades y el 7 corresponde a la neutralidad. Hacia abajo están los ácidos (7 a 1) y
hacia arriba las bases (7 a 14).

3. ALGUNOS FACTORES QUE MODIFICAN EL PROCESO


DE LAS REACCIONES QUÍMICAS.ANÁLISIS
CUALITATIVO DE LOS MISMOS. CATALIZADORES

En la industria química, existen unas sustancias llamadas catalizadores, que tienen la


propiedad de acelerar o frenar el desarrollo de una reacción química. En la naturaleza,
dentro de los seres vivos, también existen muchas sustancias que tienen esta propiedad y se
llaman enzimas.

TAMadrid
9
-Pág.69-
C IENCIAS DE LA N AT U R A L E Z A

IMPORTANCIA DE LAS REACCIONES QUÍMICAS


4. EN RELACIÓN CON ASPECTOS ENERGÉTICOS,
BIOLÓGICOS Y DE FABRICACIÓN DE MATERIALES

4.1. REACCIONES QUÍMICAS QUE MANTIENEN LA VIDA


Las reacciones químicas que se producen en los seres vivos se denominan reacciones
bioquímicas. La reacción bioquímica más importante para el mantenimiento de la vida es
la respiración y tiene lugar en todos los seres vivos.

La respiración es un proceso parecido a la combustión, pero no necesita altas


temperaturas para realizarse, ni arde ninguna sustancia poniéndose incandescente y
generando una llama. En general se parte de los azúcares, que contienen carbono e
hidrógeno y tras varias reacciones químicas se produce dióxido de carbono, agua y energía.
Esta energía se utiliza para realizar otras reacciones en las que se forman compuestos
imprescindibles para "construir" las células y los tejidos y, según las necesidades de cada
tipo de organismo, para producir movimiento, regular la temperatura corporal, etc.

Glucosa + O2 CO2 + H2O + energía.

La segunda reacción fundamental para la vida es la fotosíntesis. En ella las plantas y


otros seres autótrofos utilizan el agua, el dióxido de carbono y la luz solar para producir
sustancias que contienen gran cantidad de energía interna y desprendiéndose oxígeno. Estas
sustancias, los azúcares, se utilizan después para fabricar sus células o sus tejidos. Las
plantas además las almacenan en sus semillas y frutos, como reserva de energía para futuras
plantas. Puede decirse que la fotosintesis es la reacción contraria a la respiración.

4.2. LAS REACCIONES QUÍMICAS EN LA INDUSTRIA


En la industria química se fabrican numerosos materiales, utilizados para la
construcción de edificios, automóviles, aviones, etc., para mejorar la producción de
alimentos y nuestras condiciones de vida.

A partir de compuestos de carbono e hidrógeno, como el petróleo y otras materias


primas, se obtienen actualmente plásticos, caucho artificial, detergentes, medicamentos,
tejidos, abonos, insecticidas, pinturas, disolventes y otros materiales que utilizamos en
nuestro provecho.

10 TAMadrid

-Pág.70-
U. D. 3 - L O S CAMBIOS QUÍMICOS

Además del agua, el dióxido de carbono, el oxígeno y la luz solar, los fertilizantes o
abonos, se utilizan para mejorar el crecimiento de las plantas por contener otros elementos
necesarios para ellas, como es el nitrógeno, el fósforo y el potasio, que a veces no está
presente en la tierra y es necesario añadirlo.

Otros compuestos fabricados industrialmente son los insecticidas, que protegen las
cosechas de las plagas de insectos; los fungicidas, utilizados contra los hongos y los
herbicidas, que atacan a las malas hierbas.

4.2.1. LA INDUSTRIA FARMACÉUTICA: LOS MEDICAMENTOS

Los medicamentos, elaborados por las industrias de productos farmacéuticos, responden


a una gran gama de necesidades.

Algunos ayudan a la curación de enfermedades, como los antibióticos, los


anticancerosos, etc. Otros se diseñan con el fin de aportar determinadas sustancias que
en algún momento nuestro cuerpo deja de producir: pueden ser hormonas, como la
insulina para los diabéticos, la hormona del crecimiento, que ayuda a los niños con
problemas de crecimiento o las hormonas sexuales, muy necesarias en determinadas
enfermedades, sobre todo femeninas, y muy utilizadas en determinados tipos de
anticonceptivos, etc.

También se fabrican multitud de anestésicos utilizados durante las operaciones


quirúrgicas, calmantes contra muchos tipos de dolores, complejos vitamínicos, productos
de belleza, etc.

4.2.2. LA CONTAMINACIÓN INDUSTRIAL

La industria química que tanto ha contribuido a mejorar nuestra calidad de vida,


genera durante sus procesos de producción una gran cantidad de sustancias que degradan
el medio ambiente y producen daños en los animales y las plantas, perjudicando también
nuestra salud. Podemos mencionar la contaminación aérea con gases tóxicos como SO2,
NO2, NO, CO, CO2; metales pesados como el mercurio y el plomo y partículas sólidas
como el hollín y las cenizas. Todas estas sustancias son las responsables de la lluvia
ácida que destruye los bosques, el efecto invernadero (que recalienta la atmósfera), el
agujero de la capa de ozono y la producción de CO, gas muy tóxico que como ya se ha
indicado puede llegar a producir la muerte. También se contaminan las aguas (residuos,
aceites, petróleo, marea negra) y la tierra (residuos radiactivos y residuos sólidos como

TAMadrid
11
-Pág.71-
C IENCIAS DE LA N AT U R A L E Z A

metales, plásticos, sales inorgánicas y orgánicas y residuos líquidos como pesticidas,


mercurio, aceites, carburantes...).

Por ello, cada día se deben tomar las medidas necesarias para que esas sustancias sean
controladas y eliminadas antes de verterlas al medio, y por nuestra parte utilizar los recursos
con conciencia intentando reciclar todo aquello que sea posible.

12 TAMadrid

-Pág.72-
U. D. 3 - L O S CAMBIOS QUÍMICOS

RESUMEN

Las reacciones químicas son cambios de la materia por las que unas sustancias se
transforman en otras distintas.

Los reactivos son las sustancias iniciales y los productos, los nuevas sustancias que se
obtienen.

Las reacciones se representan por medio de las ecuaciones químicas, que deben estar
ajustadas, es decir, el número de átomos de cada clase en el estado inicial ha de ser igual al
número de átomos de los productos.

Existen reacciones de síntesis, de descomposición de combustión y de sustitución,


principalmente.

La forma de medir los ácidos y las bases es mediante el pH.

En las reacciones de síntesis se obtiene un compuesto a partir de las sustancias simples


de sus mismos elementos. En las de descomposición ocurre lo contrario.

La respiración es una reacción bioquímica (una combustión) que permite a los seres
vivos obtener la energía necesaria para mantener la vida.

La fotosíntesis es otra reacción bioquímica por la que los seres fotosintéticos


(fundamentalmente las plantas) obtienen alimento.

Las reacciones industriales proporcionan multitud de materiales que ayudan a mejorar


el desarrollo de nuestras actividades cotidianas y la calidad de vida. También producen
sustancias contaminantes nocivas para los seres vivos.

TAMadrid
13
-Pág.73-
C IENCIAS DE LA N AT U R A L E Z A

EJERCICIOS DE AUTOCOMPROBACIÓN

1. Una reacción química es:


A. Un reagrupamiento de átomos de varias sustancias para obtener otras.
B. Una mezcla de sustancias simples y compuestas.
C. Un proceso en que siempre se produce un cambio de estado.
D. Son ciertas A y C.

2. La reacción de combustión del butano con el oxígeno del aire es una reacción:
A. Ácido-Base. C. De combustión.
B. De precipitación. D. Ninguna de las tres.

3. Las sustancias que modifican la velocidad de una reacción sin intervenir como reactivos se
llaman:
A. Exotérmicas. C. Productos.
B. Catalizadores. D. uma.

4. En una combustión con cantidad insuficiente de oxígeno se produce el compuesto tóxico:


A. CO2 C. H2CO3
B. CO D. CO4

5. Cuando se quema un papel, ¿Qué tipo de reacción se produce?.


A. Reducción. C. Precipitación.
B. Combustión. D. Síntesis.

6. Las ecuaciones químicas se ajustan porque:


A. Se ha de cumplir la Ley de la conservación de la masa.
B. Se ponen los subíndices de las fórmulas de los compuestos.
C. Al perderse parte de las moléculas, saber las que nos quedan.
D. Da lo mismo puesto que la reacción ya se conoce.

7. Para que se inicie la combustión es necesario:


A. Comunicar una energía inicial al combustible para que adquiera la alta temperatura que
necesita el proceso.
B. Dejar en reposo el combustible para que se sedimente.
C. Eliminar el oxígeno presente.
D. Todas son ciertas.

14 TAMadrid

-Pág.74-
U. D. 3 - L O S CAMBIOS QUÍMICOS

8. En las reacciones de neutralización:


A. Reaccionan un óxido con un ácido. C. Reaccionan un ácido con una base.
B. Reaccionan una sal con un ácido. D. Reaccionan un óxido con una base.

9. Si el pH de una disolución es 4, se trata de una sustancia:


A. Ácida. C. Neutra.
B. Básica. D. Nada de lo anterior

10. Entre las sustancias contaminantes producidas por cierto tipo de industrias están:
A. Los metales pesados como el plomo y el mercurio.
B. Ciertos gases tóxicos como el CO, NO2 y SO2.
C. Hollín y cenizas.
D. Todas son ciertas.

TAMadrid
15
-Pág.75-
C IENCIAS DE LA N AT U R A L E Z A

RESPUESTAS A LOS EJERCICIOS

1. A
2. C
3. B
4. B
5. B
6. A
7. A
8. C
9. A
10. D

16 TAMadrid

-Pág.76-
U. D. 4 - L A TIERRA EN EL UNIVERSO

ÍNDICE
Pag.

OBJETIVOS . . . . . . . . . . . . . . . . . . . . . . . . . . . . . . . . . . . . . . . . . . . . . . . . . . . . . . . . . . . 2

INTRODUCCIÓN . . . . . . . . . . . . . . . . . . . . . . . . . . . . . . . . . . . . . . . . . . . . . . . . . . . . . . 3

MAPA CONCEPTUAL . . . . . . . . . . . . . . . . . . . . . . . . . . . . . . . . . . . . . . . . . . . . . . . . . . 4

DESARROLLO DE CONTENIDOS . . . . . . . . . . . . . . . . . . . . . . . . . . . . . . . . . . . . . . . 5

1. EL SISTEMA SOLAR. COMPONENTES, TAMAÑOS Y DISTANCIAS . . . 5


1.1. EL SOL . . . . . . . . . . . . . . . . . . . . . . . . . . . . . . . . . . . . . . . . . . . . . . . . . . . . . . 5
1.2. LOS PLANETAS . . . . . . . . . . . . . . . . . . . . . . . . . . . . . . . . . . . . . . . . . . . . . . . 5
1.3. COMETAS . . . . . . . . . . . . . . . . . . . . . . . . . . . . . . . . . . . . . . . . . . . . . . . . . . . 8

2. EL PROBLEMA DE LA POSICIÓN DE LA TIERRA EN EL UNIVERSO.


ALGUNAS EXPLICACIONES HISTÓRICAS . . . . . . . . . . . . . . . . . . . . . . . . . 9

3. LA TIERRA COMO PLANETA. MOVIMIENTO DE LA TIERRA Y LA


LUNA. EXPLICACIONES DE ALGUNOS FENÓMENOS, COMO LAS
ESTACIONES, LAS FASES DE LA LUNA Y LOS ECLIPSES. . . . . . . . . . . 10
3.1. LA ROTACIÓN . . . . . . . . . . . . . . . . . . . . . . . . . . . . . . . . . . . . . . . . . . . . . . . 10
3.2. TRASLACIÓN . . . . . . . . . . . . . . . . . . . . . . . . . . . . . . . . . . . . . . . . . . . . . . . . 11
3.3. LAS FASES DE LA LUNA . . . . . . . . . . . . . . . . . . . . . . . . . . . . . . . . . . . . . . 12
3.4. LOS ECLIPSES . . . . . . . . . . . . . . . . . . . . . . . . . . . . . . . . . . . . . . . . . . . . . . . 12

4. EL UNIVERSO. COMPONENTES, ESCALAS Y MEDIOS DE


OBSERVACIÓN. . . . . . . . . . . . . . . . . . . . . . . . . . . . . . . . . . . . . . . . . . . . . . . . . . 13
4.1. COMPONENTES. . . . . . . . . . . . . . . . . . . . . . . . . . . . . . . . . . . . . . . . . . . . . . 13
4.2. ESCALAS . . . . . . . . . . . . . . . . . . . . . . . . . . . . . . . . . . . . . . . . . . . . . . . . . . . 15
4.3. MEDIOS DE OBSERVACIÓN . . . . . . . . . . . . . . . . . . . . . . . . . . . . . . . . . . . . 16

RESUMEN . . . . . . . . . . . . . . . . . . . . . . . . . . . . . . . . . . . . . . . . . . . . . . . . . . . . . . . . . . . 17

EJERCICIOS DE AUTOCOMPROBACIÓN . . . . . . . . . . . . . . . . . . . . . . . . . . . . . . . 18

RESPUESTAS A LOS EJERCICIOS . . . . . . . . . . . . . . . . . . . . . . . . . . . . . . . . . . . . . 20

-Pág.77-
C IENCIAS DE LA N AT U R A L E Z A

OBJETIVOS
Al finalizar esta Unidad Didáctica, el alumno será capaz de:

• Reconocer la composición del Universo conocido.

• Diferenciar los conceptos de galaxias, sistemas, estrellas, planetas, etc.

• Conocer las unidades de medida de las distancias astronómicas.

• Manejar datos sobre nuestro sistema solar.

• Conocer los movimientos de la Tierra y sus consecuencias.

-Pág.78-
U. D. 4 - L A TIERRA EN EL UNIVERSO

INTRODUCCIÓN
¿C ómo definiríamos el Universo?. Es difícil, puesto que es un todo de
dimensiones inabarcables. Es el espacio, las galaxias, las estrellas, nuestro
sistema solar, la Tierra y todo lo que en ella existe, incluyéndonos a nosotros
mismos.

El tema es apasionante y ha despertado el interés y la curiosidad de científicos y


profanos de todos los tiempos. Como dato curioso basta con saber que a pesar de las
enormes dimensiones y características de los astros que existen, todos ellos están
formados por los mismos elementos químicos que ya conocemos.

En este tema intentaremos responder entre otras a las siguientes preguntas:

- ¿Cómo se originó el Universo?

- ¿Cómo se miden las distancias en el Universo?

- ¿Dónde vivimos nosotros y cuál es nuestro entorno más próximo a nivel


"galáctico"?

- ¿Qué influencia tiene la Luna sobre nosotros?

-Pág.79-
C IENCIAS DE LA N AT U R A L E Z A

M A PA C O N C E P T UA L
.
EL UNIVERSO

originado por: formado por: las distancias regido por: métodos de


se miden en: estudio
BIG BANG GALAXIAS
LEY DE LA -TELESCOPIO.
- AÑOS LUZ
GRAVITACIÓN - RADIOTELESCOPIO.
- UNIDADES
UNIVERSAL - ESPECTROSCOPIO.
ASTRONÓMICAS
- METEORITOS.
Son asociaciones de
- SONDAS, MUESTRA
SOL ESTRELLAS

Alrededor del
cual giran Son el centro de los

SISTEMAS
COMETAS

ASTEROIDES

PLANETAS

el nuestro es el

Formado TEORÍA DE LA
SISTEMA SOLAR Originado por
por NEBULOSA

JÚPITER, SATURNO,
MAYORES
URANO, NEPTUNO
Según su tamaño tienen

MENORES MERCURIO, VENUS,


MARTE, PLUTÓN
ÓRBITAS ELÍPTICAS

LA TIERRA

Tiene movimientos Su satélite Se observan

ECLIPSES
ROTACIÓN TRASLACIÓN LA LUNA
Pueden ser
DIAS (24H) AÑO (365 D) Según le de la Produce
luz del Sol La Luna entre el Sol
DE SOL
y la Tierra
FASES MAREAS
DE LUNA La Tierra entre el Sol
y la Luna

-Pág.80-
U. D. 4 - L A TIERRA EN EL UNIVERSO

EL SISTEMA SOLAR. COMPONENTES, TAMAÑOS Y


1. DISTANCIAS

El Sistema Solar, es la zona de nuestra galaxia (la Vía Láctea) ocupada por el Sol y todos
los cuerpos que se ven atraídos por su fuerza gravitatoria: Los planetas y sus satélites, los
asteroides, los meteoritos, los cometas y el polvo cósmico.

Se extiende a una distancia de cien mil billones de kilómetros del Sol y tan sólo
conocemos una pequeña parte. En la actualidad los astrónomos están de acuerdo en la teoría
de la nebulosa para explicar su origen. Esta teoría supone que el Sol y los planetas se
originaron a partir de una gran masa fría de gas y polvo. Si una nebulosa no gira, se
condensará en una estrella sin planetas, pero si, como es el caso de nuestro sistema, gira, se
forman remolinos que se separan del centro de la nebulosa, formando los planetas.
Pequeños remolinos de gas y polvo quedarían girando alrededor de éstos dando lugar a los
satélites.

1.1. EL SOL
Es una de las cuatrocientos mil millones de estrellas que componen la Vía Láctea,
siendo una de las de menor tamaño.

El Sol es una inmensa bola de hidrógeno en cuyo interior se alcanzan temperaturas de


14 millones de grados centígrados. Con estas condiciones los átomos de hidrógeno chocan
entre sí en violenta explosiones atómicas, liberando al sistema solar enormes cantidades de
energía. La superficie es mucho más fría, tan sólo 5500ºC. En ella se pueden apreciar zonas
más frías: las manchas solares y las fulguraciones o llamaradas que salen del Sol. El Sol es
una estrella joven que no se agotará hasta dentro de 5000 millones de años.

1.2. LOS PLANETAS


Planeta es el nombre que dieron los antiguos griegos a los cuerpos que cambian de
posición en el transcurso de las noches. No poseen luz propia, la luz que emiten es un reflejo
de la luz solar.

Las órbitas de los planetas alrededor del Sol están casi en el mismo plano que la órbita
terrestre, por eso los planetas están en una estrecha franja del firmamento, la

TAMadrid
5
-Pág.81-
C IENCIAS DE LA N AT U R A L E Z A

correspondiente al zodíaco, aunque resulta difícil su visualización pues varían su posición


en sucesivas noches. Distinguimos dos grupos, los menores (Mercurio, Venus, Marte
y Tierra), más pequeños y los mayores, tremendamente grandes (Júpiter, Saturno, Urano
y Neptuno).

1.2.1. MERCURIO

Es el planeta más próximo al Sol. Tarda el mismo tiempo en dar una vuelta alrededor
del Sol y en darla sobre sí mismo (dura prácticamente lo mismo el día que el año). Las
temperaturas superan los 400ºC, mientras que en la cara oculta baja de los -200ºC.

1.2.2. VENUS

Es el más brillante de los cuerpos que podemos contemplar en el firmamento. Su


tamaño es similar a la Tierra, pero su atmósfera contiene dióxido de carbono y ácido
sulfúrico. El efecto invernadero (atrapa el calor del Sol y no lo deja escapar) provocado
por el CO2, hace que en la superficie del planeta se alcancen temperaturas de más de
400ºC.

1.2.3. LA TIERRA

Se le denomina Geo y Planeta azul. Es el único planeta del Sistema Solar que alberga
vida (gracias a la atmósfera rica en nitrógeno, pero con abundancia de O2 y CO2. Es el único
planeta de nuestro Sistema que alberga vida. Por la temperatura media, se permite que el
agua se encuentre en los tres estados. Su radio medio es de 6370 Km y su densidad 5,52
g/cm3. Su satélite o Luna se encuentra a 380.000 Km.

1.2.3.1. LA LUNA

El 20 de julio de 1969, N. Armstrong y E, Aldrin, llegaron a la Luna en una nave


espacial. Desde este primer viaje, se han realizado otros, trayendo muestras de rocas
lunares para su estudio y se ha estudiado el satélite. Sabemos que tiene forma esférica,
con un diámetro de 3475 Km, se encuentra a 380.000 Km de la Tierra y su densidad es
3,36 g/cm3.

6 TAMadrid

-Pág.82-
U. D. 4 - L A TIERRA EN EL UNIVERSO

1.2.4. MARTE

Es el planeta rojo. Su color se debe a la abundancia de minerales de óxido de hierro en


su superficie. Las sondas espaciales enviadas, no han negado ni confirmado la existencia de
vida aunque es un mundo frío y seco, sin agua líquida. Posee dos satélites de pequeño
tamaño que se cree puedan ser asteroides atrapados por su campo gravitatorio.

1.2.5. LOS ASTEROIDES

Entre Marte y Júpiter orbitan decenas de miles de asteroides con diámetros inferiores a
mil kilómetros, se cree que pudieran ser restos de algún planeta que no se formó.

Los planetas que están más allá de Marte se suelen denominar planetas exteriores, son
mucho más grandes que los interiores, su atmósfera es más voluminosa que la parte sólida
del planeta. Son tan fríos que si tuvieran una atmósfera como la terrestre, se volvería
líquida.

1.2.6. JÚPITER

Es el más grande de los planetas del Sistema Solar, su tamaño es 10 veces el de la Tierra
y el que tiene el día más corto, apenas 10 horas. Consiste en una esfera fluida, sin superficie
sólida, formada por hidrógeno y helio. Tiene 16 satélites, el más conocido por su actividad
volcánica es Io.

1.2.7. SATURNO

Muy parecido de tamaño a Júpiter, aunque algo menor. Se le denomina el "Señor de los
Anillos", pues su imagen va siempre asociada a los anillos que lo rodean, como si se tratase
de un satélite que hubiera estallado. Es menos denso que el agua y atrapa en su campo
gravitatorio a 21 satélites.

1.2.8. URANO

Es cuatro veces mayor que la Tierra. Gira tumbado ya que su eje de rotación es casi
paralelo al de su órbita. Está rodeado por 15 satélites.

TAMadrid
7
-Pág.83-
C IENCIAS DE LA N AT U R A L E Z A

1.2.9. NEPTUNO

Menor que Urano, tiene una atmósfera de hidrógeno y helio. Posee 8 satélites. Fue
descubierto mediante cálculos matemáticos a partir de perturbaciones encontradas en la
órbita de Urano en 1846.

1.3. COMETAS
Los cometas son cuerpos celestes menores, rocosos y cubiertos de hielo. Al acercarse
al Sol, éste los calienta y el hielo se vaporiza (se sublima). La energía solar repele e ilumina
el gas formado, originan así las colas que los caracterizan. Las órbitas de los cometas son
más alargadas (más elípticas) que las de los planetas. Los cometas, más tarde o más
temprano acaban chocando con los planetas o con los satélites. Se cree que los cometas son
cuerpos del cinturón de asteroides o de la nube de Oort, que por atracciones gravitacionales
de los planetas son desviados de su órbita hasta quedar atrapados alrededor del Sol.

Podemos aquí definir los meteoritos como cuerpos que vagan por el sistema solar sin
una órbita definida. Su tamaño oscila entre el de un grano de arena (polvo cósmico) a varias
toneladas. Se cree que los cráteres que hay en la Luna son impactos de meteoritos. Los
meteoros, son fragmentos de roca o metal procedente del espacio exterior que al quedar
atrapado por un planeta cae hacia su superficie desintegrándose por completo, al ponerse
incandescentes por el rozamiento, se denominan estrellas fugaces.

8 TAMadrid

-Pág.84-
U. D. 4 - L A TIERRA EN EL UNIVERSO

Algunas características físicas de los componentes de nuestro sistema solar:

Distancia media
Diámetro Período orbital Período de rotación
PLANETA al Sol (millones Masa* (Tierra=1)
ecuatorial (Km) (año del planeta) (día del planeta)
de Km)

Sol 1.392.530 -- -- 25,4 días 332.948,34


Mercurio 4.878 58 88 días 58,6 días 0,06
Venus 12.104 108 225 días 243 días 0,81
Tierra 12.756 150 1 año 1 día 1
Marte 6.794 228 1,9 años 24,6 horas 0,11
Júpiter 142.800 778 11,9 años 9,8 horas 317,89
Saturno 120.000 1.427 29,5 años 10,2 horas 95,14
Urano 52.000 2.870 84,0 años 16-28 horas** 24,52
Neptuno 48.400 4.497 164,8 años 18-20 horas** 17,25

24
*Masa de la Tierra = 5,98. 10 Kg.
**Estos valores no son totalmente seguros.

2. EL PROBLEMA DE LA POSICIÓN DE LA TIERRA EN EL


UNIVERSO.ALGUNAS EXPLICACIONES HISTÓRICAS

Históricamente, dado que la concepción del Universo se basaba en lo que el ser humano
era capaz de percibir se pensaba que la Tierra estaba fija y que el Sol, la Luna y los planetas
giraban alrededor de ella (Teoría geocéntrica). Pero esta concepción fue cambiando
conforme las investigaciones y los medios fueron desarrollándose y así se llegó a la Teoría
Heliocéntrica. Esta concepción de considerar el centro al Sol y todo girando a su alrededor
se conoce como Teoría Heliocéntrica (se denomina así porque al Sol se le conoce con el
Helios).

TAMadrid
9
-Pág.85-
C IENCIAS DE LA N AT U R A L E Z A

LA TIERRA COMO PLANETA. MOVIMIENTO DE LA


3. TIERRA Y LA LUNA. EXPLICACIONES DE ALGUNOS
FENÓMENOS, COMO LAS ESTACIONES, LAS FASES
DE LA LUNA Y LOS ECLIPSES
Como consecuencia de los movimientos de la Tierra (de rotación sobre sí misma y de
traslación alrededor del Sol), se observan una serie de fenómenos.

3.1. LA ROTACIÓN
La Tierra realiza un giro completo alrededor de su eje polar cada 24 horas: es el
movimiento de rotación. El sentido de rotación de la Tierra es de oeste a este (lo sabemos
por que el Sol sale por el este y se pone por el oeste), es el responsable de la alternancia
día/noche.

El movimiento de rotación explica que distintos lugares del mundo tengan distinta hora,
o que en unas zonas sea de día cuando en otras es de noche. Esta división de la Tierra en 24
horas se puede ver en los mapas mundi donde encontramos los husos horarios (es decir los
lugares o países donde la hora es la misma). En países muy grandes como Estados Unidos
o Australia, cada zona del país tiene distinta hora. En Estados Unidos puede haber una
diferencia de 7 horas de costa este a costa oeste. En España, pasa lo mismo con las Islas
Canarias, que al estar situadas un huso horario más al oeste que la península, existe una hora
menos.

10 TAMadrid

-Pág.86-
U. D. 4 - L A TIERRA EN EL UNIVERSO

3.2. TRASLACIÓN
La Tierra no permanece fija mientras rota sobre sí misma. Todos los planetas del sistema
solar, incluida la Tierra, se trasladan en órbitas elípticas alrededor del Sol.

Nuestro planeta realiza una vuelta completa cada 365 días, 5 horas y 49 minutos y se
llama movimiento de traslación. El plano que contiene la órbita se denomina plano de la
eclíptica.

Cuando la Tierra se traslada alrededor del Sol, no lo hace con su eje polar perpendicular,
sino con una inclinación de 23º 27´. Esta inclinación es la responsable de los cambios
estacionales.

De modo inicial, podríamos pensar que como la órbita alrededor del Sol es elíptica, unas
veces estará más cerca y hará más calor y otras más lejos y hará frío, pero esto no es así, ya
que la órbita es en realidad casi circular. Como la Tierra se traslada alrededor del Sol
ligeramente inclinada, los rayos solares caen con distinto ángulo a medida que pasan los
días, por eso las sombras que hacen los objetos son de distinto tamaño. Sin embargo,
importa mucho la inclinación con que nos llegan sus rayos y el número de horas que el Sol
nos ilumina.

El día del año que el Sol deja una sombra más corta se llama solsticio de verano. Es el
día en que está más alto y nos ilumina más horas. Entonces comienza el verano. A partir de
ese día el Sol estará cada vez más bajo, con lo que las sombras también se alargarán, hasta
llegar al solsticio de invierno, que marca el comienzo del invierno y es la noche más larga
del año. A partir de ese momento, el Sol vuelve día a día, a elevarse sobre el horizonte.
Cuando comienza la primavera y el otoño el día tiene la misma duración que la noche: es
el equinoccio de primavera y el de otoño. Podemos generalizar que los hemisferios de la
Tierra reciben una vez los rayos solares más perpendiculares y durante más tiempo
(primavera/verano) y otras más inclinado y durante menos tiempo.(otoño/invierno).

TAMadrid
11
-Pág.87-
C IENCIAS DE LA N AT U R A L E Z A

3.3. LAS FASES DE LA LUNA


La luna gira alrededor de la Tierra dando una vuelta completa cada 28 días, modificando
durante este recorrido el aspecto que ofrece a nuestra vista y los efectos que se producen
sobre las mareas especialmente. El movimiento de rotación de la Luna dura lo mismo que
el de traslación alrededor de la Tierra. Por eso siempre presenta hacia la Tierra la misma
cara y la otra permanece oculta.

La Luna la vemos de distinta forma según la situación del triángulo imaginario que
forman el Sol, la Tierra y la Luna. Así, cuando la luna está entre el Sol y la Tierra, se
produce la Luna nueva, visible de día y cercana al Sol. Durante los 14 días siguientes irá
creciendo hasta alcanzar la forma de una letra D y apartándose del Sol hasta llegar a la fase
de Luna llena, parará entonces por nuestra referencia 12 horas después que el Sol. Los 14
días siguientes decrecerá, hasta formar una letra C, luego empieza otra vez la fase de Luna
nueva. La Luna es muy grande, por lo que la Luna ejerce influencia sobre nuestro Planeta;
esta atracción se percibe de diversas maneras, pero la más llamativa es la que ejerce sobre
el mar, provocando subidas y bajadas periódicas de su nivel, llamadas mareas.

3.4. LOS ECLIPSES


Cuando la Luna está entre el Sol y la Tierra nos oculta la luz del Sol. Es un eclipse de
Sol. Cuando la Tierra está entre el Sol y la Luna, la luz solar no llega a la Luna. Se produce
un eclipse de Luna. Pero los eclipses se producen tan sólo cuando la Luna gira alrededor de
la Tierra en el plano de la Eclíptica, habitualmente lo hace en un plano ligeramente
inclinado de modo que cuando la Luna está entre el Sol y la Tierra pasa por encima o por
debajo del Sol.

12 TAMadrid

-Pág.88-
U. D. 4 - L A TIERRA EN EL UNIVERSO

EL UNIVERSO. COMPONENTES, ESCALAS


4. Y MEDIOS DE OBSERVACIÓN

El Universo nació hace unos 15.000 millones de años de una gigantesca explosión, el
Big Bang. Este estallido cósmico marcó el comienzo del espacio y del tiempo. Se creó toda
la energía del mundo y una parte de esa energía se transformó en materia. Desde el
momento de su creación, el universo no ha dejado de expandirse como si de un enorme
globo que no cesara de inflarse. La pregunta es ¿Hasta cuando?. El vacío de inmensas
regiones del universo puede que no esté tan vacío. Los astrónomos suponen que puede estar
lleno de un tipo de materia, de naturaleza desconocida, conocida como materia oscura. En
caso de que no exista materia oscura, el universo puede seguir su expansión eterna, hasta
convertirse en un enorme cementerio frío y oscuro. En caso de que exista materia oscura,
su enorme masa generaría una atracción gravitatoria tan fuerte que frenaría la expansión y
comenzaría la contracción del universo: el Big Crunch.

La unión entre todos los elementos que componen el universo se produce gracias a la
fuerza de atracción que ejercen unos cuerpos celestes sobre los otros, siguiendo las leyes de
la gravitación universal (se verán en la Unidad dedicada a las fuerzas).

4.1. COMPONENTES
Los principales cuerpos celestes y agrupaciones que se diferenciaron a partir de la gran
explosión, la materia y la energía que existían previamente concentradas son las estrellas,
galaxias y nebulosas.

4.1.1. LAS ESTRELLAS

Las estrellas, también llamadas soles, son astros que generan grandes cantidades de
energía por combustión. Su brillo es producto de su elevada temperatura interna y el color,
de su temperatura externa.

Algunas son solitarias, como nuestro Sol, pero la mayoría tienen estrellas compañeras.
Los sistemas suelen ser dobles, con dos estrellas girando una alrededor de la otra, aunque
también los hay múltiples (constelaciones).

TAMadrid
13
-Pág.89-
C IENCIAS DE LA N AT U R A L E Z A

Las constelaciones más conocidas en nuestra esfera celeste son: Andrómeda,


Casiopea, Pegaso, Osa Mayor, Osa Menor, Cruz del Sur, Orión y las Constelaciones del
Zodíaco.

La mayor parte de las estrellas emiten o irradian luz visible o infrarroja; otras emiten
rayos X y otras ondas de radio. Ciertas estrellas, las supernovas, tienen tanto brillo como la
galaxia completa en la que están contenidas, mientras que otras, los agujeros negros, no son
visibles ni a escasos kilómetros de distancia (aunque acumulan gran cantidad de energía).
Algunas estrellas están rodeadas de planetas. Para clasificarlas se recurre a características
físicas como el color, tamaño, brillo, etc.

El tamaño de las estrellas varía mucho de unas a otras. Hay algunas, por ejemplo, las
llamadas enanas, que son como la Tierra. Otras, las gigantes, tienen un radio de más de 300
millones de kilómetros. No obstante la masa varía poco de unas estrellas a otras.

El color de las estrellas es una característica que varía con la temperatura de su


superficie; desde el color rojo de las frías hasta el azul de las más calientes. Cuando una
estrella envejece, aumenta su diámetro y luminosidad hacia gigante roja, disminuyendo su
temperatura; tras convertirse en enana blanca, al apagarse se convierte en enana negra (sin
luz), cuando la estrella es pequeña. Si la estrella es muy grande en su final se producirá una
gran explosión formándose un agujero negro, con una gran atracción gravitatoria y una
supernova, desprendiéndose una gran cantidad de polvo cósmico.

Las estrellas junto con los planetas, sus satélites, los asteroides y cometas forman un
sistema.

4.1.2. GALAXIAS

Una galaxia se compone de gas, polvo y miles de millones de cuerpos celestes:


Estrellas, planetas, asteroides, etc.

Por su apariencia física, las galaxias se clasifican en elípticas, lenticulares, espirales e


irregulares. La Vía Láctea es una galaxia de tipo espiral. En el universo se calcula que
existen al menos mil millones de galaxias.

Las galaxias a su vez se agrupan en cúmulos. Algunos de ellos están formados por
unas cuantas docenas de galaxias; otros, sin embargo, contienen miles. El cúmulo
llamado Grupo Local contiene, entre otras, dos galaxias de gran tamaño, la Vía Láctea
y la M31.

14 TAMadrid

-Pág.90-
U. D. 4 - L A TIERRA EN EL UNIVERSO

La Vía Láctea, recibe este nombre por el aspecto blanquecino y lechoso que presenta la
franja irregular de apretadas estrellas que surca el firmamento. Es una galaxia espiral, en
uno de cuyos brazos se encuentra nuestro sistema solar, contiene más de 100.000 millones
de estrellas. En el centro de la galaxia existe un potente emisor de rayos X lo que apoya la
hipótesis de la existencia en este lugar de un gran agujero negro.

4.1.3. NEBULOSAS

Las nebulosas son nubes de gas y polvo cósmico iluminadas por cercanas estrellas
calientes o por supernovas.

Una estrella de temperatura media, confiere un aspecto blanquecino a la nebulosa a la


cual ilumina (Ej.: la nebulosa iluminada por las Pléyades). Si está asociada a una estrella
más caliente, presenta un aspecto verdoso (por ejemplo, la nebulosa iluminada por la
constelación de Orión).

Una de las nebulosas más brillantes es la gran nebulosa de Orión, que está a una
distancia de 1.300 años-luz de la Tierra.

4.2. ESCALAS
La medida empleada para medir distancias en el Universo es el Año-luz, que es la
distancia que recorre la luz en un año (la velocidad de la luz es de 300.000 Km/s). Para
hacernos una idea de la magnitud de estas distancias basta señalar que una nave espacial
que viajará a 6000 km/s (más rápida que las actuales) tardaría más de 100 años en llegar al
planeta más alejado de nuestro sistema.

Otra unidad empleada es la Unidad Astronómica (U.A.) que es la distancia entre la


Tierra y el Sol (son 150.000.000 Km).

Dada la inmensidad de estas medidas, para poder comprender el Universo se hacen


modelos a escala, que nos facilitan la comprensión de las enormes distancias y períodos de
tiempo que resultan inaccesibles a nuestra experiencia.

TAMadrid
15
-Pág.91-
C IENCIAS DE LA N AT U R A L E Z A

4.3. MEDIOS DE OBSERVACIÓN


Para llegar a conocer el Universo se emplean distintos métodos cada vez más
sofisticados:

Telescopios. Consiste en la acumulación de lentes, distribuidas de tal forma, que se


consiguen imágenes más grandes de los astros, para que puedan ser examinados
detalladamente, y captar, a la vez la mayor cantidad de luz posible. Van a ser
características importantes del microscopio la luminosidad y el poder resolutivo. La
luminosidad viene determinada por el diámetro de las lentes, por tanto, cuanto
mayor sea éste, mayor será la cantidad de luz que entra en el telescopio. El poder
resolutivo es la capacidad de un instrumento óptico para poder distinguir entre dos
puntos próximos sin que aparezcan mezclados.

Es mejor decir observatorios. Dado que el espectro electromagnético es amplio, y


las estrellas pueden emitir o absorber (los agujeros negros) radiaciones, se emplean
receptores y emisores de tal forma que lo que "vemos" son espectros, también se
observa en el espectro visible al modo más tradicional. Así tenemos
radiotelescopios que reciben y captan radiaciones no luminosas emitidas por los
astros. En 1984 se instaló uno de los radiotelescopios más potente del mundo en la
isla de La Palma en las Canarias. Otro tipo es el espectroscopio, que es un
instrumento que descompone la luz procedente de los astros en los diversos colores
del arco iris, y según la composición del astro, por medio de un prisma óptico y las
plasma sobre una placa fotográfica.

Sondas enviadas al espacio. Provistas de sensores diferentes y sistemas de recogida


de muestras.

Meteoritos caídos.

16 TAMadrid

-Pág.92-
U. D. 4 - L A TIERRA EN EL UNIVERSO

RESUMEN
Los planetas que pertenecen al sistema solar son: Mercurio, Venus, Tierra, Marte,
Júpiter, Saturno, Urano y Neptuno.
La Tierra tiene dos movimientos planetarios: el de rotación, que origina los ciclos día-
noche, y el de traslación alrededor del Sol, responsable del ciclo anual o de las estaciones.
Nuestro planeta tiene un solo satélite, la Luna. Se encuentra a una distancia de 385.000 Km.
Según reciba la luz del Sol, la Luna se ve desde la Tierra iluminada en mayor o menor
área, es decir, se producen las fases de la Luna.
La atracción de la Luna sobre la Tierra produce las mareas.
Los planetas son los astros que giran alrededor de las estrellas. Los satélites giran
alrededor de los planetas. El conjunto de estos tres tipos de astros se llama sistema.
Los cuerpos que componen el Universo, se atraen y se mueven siguiendo las leyes de la
Gravitación Universal.
La Vía Láctea es nuestra Galaxia. Tiene forma de espiral. Contiene más de 100.000
millones de estrellas (algunas poseen planetas) además de nebulosas y polvo cósmico.
El Universo se encuentra en continua expansión, las galaxias que lo componen, más de
100.000 millones, se alejan unas de otras debido a que se originaron hace 15.000 millones
de años, en una gigantesca explosión, el Big Bang. Si existe materia oscura, el Universo
dejará de expandirse para empezar el Big Crunch.
Las distancias en el Universo son tan increíblemente grandes que necesitamos introducir
nuevas unidades de medida, como el año-luz, que es la distancia que recorre la luz en un
año (a 300.000 km/s).
La composición de las estrellas, incluido el Sol es mayoritariamente hidrógeno y helio.
Mediante los radiotelescopios, espectroscopios y telescopios, se estudia el mundo
exterior.

Ejercicio resuelto:
1.- Calcula la distancia en kilómetros de un año luz. Recuerda que un años tiene 365 días; un día 24
horas; una hora tiene 60 minutos y un minuto tiene 60 segundos. Dado que cada segundo la luz
recorre 300000 Km, para saber la distancia que recorre en un año debes calcular los segundos que
tiene un año y después multiplicarlo por 300000. Haz tú los cálculos, puedes usar calculadora.

El resultado que aparece en la calculadora es 9,4608... 12. Esto significa 9,4608 x 1012 . Cada
uno de ese número doce significa un lugar a la derecha de las unidades. El número es:
9.460.800.000.000. ¿Qué unidades son las que estamos midiendo?. Mira la pregunta del ejercicio.
Distancia en kilómetros. Luego la respuesta correcta sería 9.460.800.000.000 Km.

TAMadrid
17
-Pág.93-
C IENCIAS DE LA N AT U R A L E Z A

EJERCICIOS DE AUTOCOMPROBACIÓN
1. La distancia en kilómetros de una semana luz es:
A. 0,18144x1012 Km B. 7,638624x1010 Km
C. 8,4112x1012 Km D. 7,638624x1012 m

2. ¿Cuantas unidades astronómicas tiene un año luz?.


A. 5,12x106 Km B. 90102,85 Km
C. 5,12x106 U.A. D. 63.072 U.A.

3. Los asteroides:
A. Pudieran ser restos de algún planeta que no se formó.
B. Se encuentran situados entre Marte y Júpiter.
C. El diámetro es inferior a 1.000 Km
D. Todas son verdaderas.

4. Los cometas:
A. Vagan por el espacio sin órbita definida.
B. Son cuerpos celestes menores, rocosos y cubiertos de hielo.
C. Los cráteres de la Luna están producidos por impactos de cometas.
D. Todas son verdaderas.

5. Mas allá de Plutón podría haber algún otro planeta por descubrir porque:
A. Se han visto imágenes que así lo indican.
B. El Sol puede atraer cuerpos cuyas órbitas estén a 100.000 billones de kilómetros.
C. Es imposible debido al desajuste gravitacional que supondría.
D. Si pero es pequeño, similar a Plutón, aunque aún se está investigando.

6. La causa de los cambios estacionales a lo largo del año es:


A. La mayor a menor cercanía al Sol durante su órbita.
B. Las corrientes de aire cíclicas que se producen en los dos hemisferios.
C. La inclinación de 23° 27´ del eje polar de la Tierra.
D. Todas son verdaderas.

7. Las galaxias:
A. Son agrupamientos de estrellas situadas en una zona reducida.
B. Se componen de gas, polvo y miles de millones de cuerpos celestes.
C. Existen pocas en el Universo.
D. Todas son verdaderas.

18 TAMadrid

-Pág.94-
U. D. 4 - L A TIERRA EN EL UNIVERSO

8. Las distancias en el Universo se miden en:


A. Kilómetros. B. Año-luz.
C. Miliómetros. D. Todas son ciertas.

9. El movimiento causante de la diferencia horaria entre las distintas zonas de la Tierra se llama:
A. Rotación. B. Traslación.
C. Eclíptica. D. Nada de lo anterior.

10. El sentido de rotación de la Tierra es:


A. De este a oeste. B. No se mueve.
C. De oeste a este. D. De norte a sur.

TAMadrid
19
-Pág.95-
C IENCIAS DE LA N AT U R A L E Z A

RESPUESTAS A LOS EJERCICIOS

1. A
2. D
3. D
4. B
5. B
6. C
7. B
8. B
9. A
10. C

20 TAMadrid

-Pág.96-
portada TROPA 19/3/07 19:51 Página 1

FUERZAS ARMADAS
PROFESIONALES
CURSO DE APOYO
A LA PREPARACIÓN
DE LAS PRUEBAS DE ACCESO
A UNA RELACIÓN DE SERVICIOS
DE CARÁCTER PERMANENTE

CIENCIAS DE LA NATURALEZA
2ª parte
Unidades didácticas 5, 6, 7 y 8

DIGEREM

MINISTERIO
DE DEFENSA
FUERZAS ARMADAS SUBDIRECCIîN GENERAL
DE TROPA Y MARINERIA
PROFESIONAL
PROFESIONALES
CURSO DE APOYO
A LA PREPARACIÓN
DE LAS PRUEBAS DE ACCESO
A UNA RELACIÓN DE SERVICIOS
DE CARÁCTER PERMANENTE

CIENCIAS DE LA NATURALEZA
2ª parte
Unidades didácticas 5, 6, 7 y 8
La Ley 8/2006 de Tropa y Marinería, en su artículo 16,1, establece que “la formación
en las Fuerzas Armadas garantizará que los militares profesionales de tropa y
marinería puedan adquirir, actualizar o ampliar sus conocimientos para un mayor
desarrollo personal y profesional”. En cumplimiento de este mandato, el Ministerio
de Defensa edita el presente material didáctico para facilitar a los militares
profesionales de tropa y marinería, alumnos de los cursos de formación
presencial que se imparten a través de la Dirección General de Reclutamiento y
Enseñanza Militar, los apoyos necesarios para preparación de dichos cursos, que
permitirán, siempre que superen las pruebas correspondientes, la obtención de la
titulación de graduado en Educación Secundaria, acreditación para el acceso a
los ciclos formativos de la Formación Profesional de grado medio o de grado
superior, acceso a las Escalas de Suboficiales, Tropa Permanente, Guardia Civil
y Policía Nacional.

CATÁLOGO GENERAL DE PUBLICACIONES


http://www.060.es

Edita:

© Autor y editor
NIPO: 076-10-204-9 NIPO: 076-10-205-4 (edición en línea)
Depósito Legal: M-32363-2009
Diseño y programación: cimapress
Tirada: 1300 ejemplares
Fecha de edición: septiembre, 2010

Prohibida la reproducción total o parcial de esta obra, por cualquier medio sin autorización escrita del editor
CIENCIAS DE LA NATURALEZA
2ª parte

SUMARIO

Unidad didáctica Pág.

5. LOS MATERIALES TERRESTRES 5

6. LA DIVERSIDAD Y UNIDAD DE LOS SERES VIVOS 41

7. LAS PERSONAS Y LA SALUD 85

8. INTERACCIÓN DE LOS COMPONENTES ABIÓTICOS


Y BIÓTICOS DEL MEDIO NATURAL 149
U . D . 5 - L OS M AT E R I A L E S T E R R E S T R E S

ÍNDICE
OBJETIVOS . . . . . . . . . . . . . . . . . . . . . . . . . . . . . . . . . . . . . . . . . . . . . . . . . . . . . . . . . . . . .2

INTRODUCCIÓN . . . . . . . . . . . . . . . . . . . . . . . . . . . . . . . . . . . . . . . . . . . . . . . . . . . . . . . . . .3

MAPA CONCEPTUAL . . . . . . . . . . . . . . . . . . . . . . . . . . . . . . . . . . . . . . . . . . . . . . . . . . . . . .4

DESARROLLO DE CONTENIDOS

1. LA ATMÓSFERA . . . . . . . . . . . . . . . . . . . . . . . . . . . . . . . . . . . . . . . . . . . . . . . . . . . . . . . . .5

2. EL AIRE . . . . . . . . . . . . . . . . . . . . . . . . . . . . . . . . . . . . . . . . . . . . . . . . . . . . . . . . . . . .10

3. EL AGUA . . . . . . . . . . . . . . . . . . . . . . . . . . . . . . . . . . . . . . . . . . . . . . . . . . . . . . . . . . . .11

4. LAS ROCAS Y MINERALES FUNDAMENTALES QUE COMPONEN


EL RELIEVE ESPAÑOL . . . . . . . . . . . . . . . . . . . . . . . . . . . . . . . . . . . . . . . . . . . . . .15

5. EL SUELO. DESTRUCCIÓN, CUIDADO Y RECUPERACIÓN . . . . . . . . . . . . .27

RESUMEN . . . . . . . . . . . . . . . . . . . . . . . . . . . . . . . . . . . . . . . . . . . . . . . . . . . . . . . . . . . .29

EJERCICIOS DE AUTOCOMPROBACIÓN . . . . . . . . . . . . . . . . . . . . . . . . . . . . . . . .34

RESPUESTAS A LOS EJERCICIOS . . . . . . . . . . . . . . . . . . . . . . . . . . . . . . . . . . . . . . .35

-Pág.5-
C I E N C I A S D E L A N AT U R A L E Z A

OBJETIVOS
Al finalizar el estudio de esta Unidad Didáctica, el alumno será capaz de:

• Conocer los materiales que componen el planeta Tierra.

• Distinguir las magnitudes empleadas para la predicción del tiempo y los


instrumentos para medirlas.

• Identificar las propiedades del agua con los procesos vitales y como agente de
modelación del paisaje y la corteza terrestre.

• Observar la estructura de la Tierra y cómo se forman y destruyen los


continentes.

• Comprender cómo se forman las rocas y las clases que existen.

-Pág.6-
U . D . 5 - L OS M AT E R I A L E S T E R R E S T R E S

INTRODUCCIÓN
L a Tierra muestra una estructura zonal concéntrica o en capas: la corteza, el manto y
el núcleo. Su tamaño es el adecuado para que la fuerza de gravedad le permita
mantener una atmósfera, que resulta ideal e imprescindible para mantener la biosfera. La
existencia de agua en grandes cantidades constituye la hidrosfera, cuya temperatura y
presión son las adecuadas para que ésta aparezca en los tres estados de agregación:
sólido, líquido y gaseoso. Su temperatura media de unos 15°C la convierten en el paraíso
deseado para que hace cuatro mil millones de años surgiera y evolucionara nuestra vida.
En este tema vamos a estudiar cómo es este planeta en la actualidad y cómo se modifica.

-Pág.7-
C I E N C I A S D E L A N AT U R A L E Z A

M A PA C O N C E P T UA L
MATERIALES
TERRESTRES

ROCAS Y
LA ATMÓSFERA EL AIRE EL AGUA
MINERALES

FENÓMENOS
Formada por VIDA HIDROSFERA MINERALES ROCAS
METEOROLÓGICOS

TROPOSFERA - Calidad
- Eléctricos y - Potabilidad. ESTRUCTURA Tipos
luminosos. - Depuración.
ESTRATOSFERA - Eólicos.
- Acuosos.
MESOSFERA - Mixtos (acuosos MAGMÁTICAS
y eólicos) SEDIMENTARIAS
IONOSFERA METAMÓRFICAS

EXOSFERA TIEMPO
ATMOSFÉRICO
Conforma la

PRESIÓN
ATMOSFÉRICA ESTRUCTURA
DE LA TIERRA

MODELO MODELO
DINÁMICO GEOQUÍMICO

TECTÓNICA CALOR POR CORTEZA


DE PLACAS CONVECCIÓN MANTO
NÚCLEO

ZONAS DE SUBDUCCIÓN

DORSALES OCEÁNICAS
PLIEGUES
DEFORMACIONES FALLAS
MOVIMIENTO DE PLACAS
DE LAS ROCAS
DIACLASAS

-Pág.8-
U . D . 5 - L OS M AT E R I A L E S T E R R E S T R E S

1. LA ATMÓSFERA
La capa gaseosa que rodea la Tierra se llama atmósfera. Su composición y grosor han ido
variando a lo largo de los 4600 millones de años del planeta.

Inicialmente, la atmósfera estaba formada por gases, dióxido de carbono, hidrógeno y helio,
que procedían de la nebulosa solar de la que se originó el Sol y su sistema planetario. El
hidrógeno y el helio, los gases más ligeros se escaparon al espacio exterior dejando a la Tierra
durante cientos de millones de años casi en el vacío. En estas condiciones, los meteoritos y
cometas impactaron con facilidad contra la superficie de la Tierra abriendo fisuras y cráteres
originando volcanes con erupciones de una gran potencia. Los gases emitidos en las erupciones
originaron una atmósfera formada probablemente por nitrógeno, dióxido de carbono, vapor de
agua, amoniaco, dióxido de azufre y metano.

La radiación solar ultravioleta desempeñó un papel fundamental: disoció el vapor de agua y


el dióxido de carbono liberando el oxígeno a la atmósfera, no obstante la temperatura seguía
siendo bastante elevada y la presión atmosférica superior a la actual.

Los volcanes siguieron emitiendo grandes cantidades de vapor de agua, dióxido de carbono
y nitrógeno. El vapor de agua que ascendía se enfriaba al mismo tiempo, con lo que se
originaron las gotas de lluvia, pero éstas al caer sobre la superficie aún muy caliente volvían a
evaporarse contribuyendo al enfriamiento de la litosfera.

Hace 2000 millones de años, oxidados los metales superficiales, las moléculas de oxígeno
(O2) comenzaron a quedar libres incorporándose a la atmósfera. Al combinarse entre sí, surgió
otro nuevo gas, el ozono (O3), que impedía el paso de cierto tipo de radiaciones ultravioleta
emitidas por el Sol, asegurando así la vida en las zonas acuáticas tanto como terrestre.

En la actualidad la capa gaseosa tiene un grosor de unos 2500 Km.

1.1. VARIACIÓN DE LA COMPOSICIÓN, DENSIDAD,


TEMPERATURA Y PRESIÓN CON LA ALTURA
La masa de aire que actualmente envuelve la Tierra, aparece dividida en regiones
concéntricas, las cuales no varían en composición, pero sí lo hacen en densidad y temperatura
según la altitud.

5
-Pág.9-
C I E N C I A S D E L A N AT U R A L E Z A

Estas capas son:


– Troposfera. Es la capa más próxima a la superficie de la Tierra. Tiene un espesor medio
de 12 Km, que oscila entre los 7 Km en los polos y los 18 Km en el ecuador. En esta región
tienen lugar los fenómenos meteorológicos: lluvia, viento, etc. Contiene el 80% del total
de la masa de aire atmosférico. La temperatura desciende desde 20°C a -58°C en su límite.
– Estratosfera. Es la capa que sigue a la troposfera y que se extiende hasta los 50 Km de
altitud; se denomina así porque el aire se encuentra dispuesto en capas o estratos. La
temperatura varía desde los -58°C en su inicio hasta los 80°C en su límite. Entre los 25 y
45 Km de altitud se da un fenómeno de singular importancia: la absorción de la radiación
ultravioleta, de carácter mortal para los seres vivos, por parte de la molécula de oxígeno
(O2) para formar ozono (O3) que se localiza en la banda protectora que se denomina
ozonosfera.
La absorción de la radiación solar infrarroja hace que se eleve la temperatura del aire
gradualmente hasta alcanzar 80°C en su límite superior.
– Mesosfera. Se extiende hasta los 90 Km de altitud, y la temperatura desciende
gradualmente hasta -90°C en su límite superior.
– Ionosfera o Termosfera. Va desde los 90 Km hasta los 550 Km. En esta zona se origina
el fenómeno luminoso denominado aurora polar y se reflejan las ondas de radio. Los
meteoritos cuando entran en contacto con esta región se ponen incandescentes, incluso en
algunos casos se hacen visibles, como estrellas fugaces, y en muchas otras situaciones se
desintegran.
– Exosfera. Se extiende desde los 550 Km hasta los 2500 Km, aproximadamente. En esta
zona apenas quedan gases ya que su temperatura es elevadísima. En esta región, al
contener poco aire, permite el movimiento alrededor de la Tierra de los satélites
artificiales.

1.2. EL PAPEL PROTECTOR DE LA ATMÓSFERA


La atmósfera nos protege de las radiaciones ultravioleta, permite el calentamiento del aire y
la Tierra permitiendo con ello la vida, permite igualmente el paso de la luz y la existencia de
oxígeno en la proporción adecuada para que sea posible el fenómeno de la vida.

1.3. FENÓMENOS ATMOSFÉRICOS


La meteorología es la ciencia que estudia los fenómenos meteorológicos.
El viento solar, la humedad, la temperatura y la presión condicionan ciertos fenómenos
meteorológicos.

6
-Pág.10-
U . D . 5 - L OS M AT E R I A L E S T E R R E S T R E S

Los fenómenos meteorológicos los podemos dividir en 4 grupos:


1. Fenómenos eléctricos y luminosos. Podemos mencionar las auroras polares,
originadas por el viento solar que es una corriente de partículas cargadas eléctricamente
procedentes de la superficie del Sol que al encontrarse con los iones de la ionosfera produce ese
fenómeno luminoso que es más frecuente en el hemisferio norte. Los espejismos, son
fenómenos locales que se observan cuando la luz incide sobre la superficie de separación entre
dos capas de aire de diferente temperatura, la superficie más caliente actúa como un gran espejo
reflejando imágenes más o menos deformadas de los objetos que se hallan envueltos en la capa
fría, algunos de ellos situados a gran distancia, incluso a decenas de kilómetros. Las tormentas
eléctricas, son fenómenos que se producen con frecuencia en el atardecer y las noches de
primavera y verano. El desplazamiento de las nubes y el movimiento de las gotas de agua dentro
de ellas producen electricidad estática que al liberarse hacia el suelo en una descarga muy fuerte
es el rayo, el resplandor que produce es el relámpago. El aire que circunda al rayo se calienta
bruscamente y produce un estampido, es el trueno. Como el sonido se propaga a una velocidad
mucho menor que la luz, el relámpago se ve casi instantáneamente mientras que el trueno se oye
después de desaparecer el resplandor.
2. Fenómenos eólicos. El peso por unidad de superficie de la masa total de aire sobre un
punto determinado se denomina presión atmosférica o barométrica. A nivel del mar y a 45° de
latitud, tiene un valor de 1 atmósfera, que equivale a 760 mm de Hg. En las masas de aire cálido,
llamados ciclones o borrascas (B), la presión es más baja debido a la menor densidad del aire
(que al pesar menos ejerce menor presión). En las masas de aire frío, llamadas anticiclones, la
presión es mayor debido al fenómeno contrario. Todo movimiento horizontal de masas de aire
en la superficie terrestre o paralelo a ella se denomina viento, las principales causas de este
movimiento son la fricción o roce entre las masas de aire, el movimiento de rotación de la
Tierra, los accidentes del relieve terrestre y las variaciones de la presión atmosférica (se forman
corrientes de convección del aire que se calienta y sube y mientras que circula por las zonas
altas se enfria y baja, rellenando el espacio “vacío” del aire caliente que ascendió). El des-
plazamiento se realiza siempre desde una zona de alta presión (aire frío, seco y denso) a una
zona de presión más baja (aire cálido, menos denso y húmedo).
3. Fenómenos acuosos. Los de mayor importancia son: las nubes, que son
condensaciones de una masa de aire cálido y húmedo que asciende y se enfría alrededor de las
partículas de polvo y forma las gotas de agua, cuando éstas y los cristales de hielo son
suficientemente grandes, la nube se hace visible. La lluvia, es la caída de gotas de agua de
pequeño tamaño (0,7 a 5 mm de diámetro), se produce al enfriarse una masa de aire cálido que
asciende, se condensa y precipita. La nieve, es la solidificación de las gotas de agua que caen
cuando atraviesan una capa de aire frío (temperatura inferior a 0°C). El granizo, es la preci-
pitación sólida en forma de hielo que acompaña a las tormentas y otros fenómenos atmosféricos
eléctricos.
4. Fenómenos mixtos: acuosos y eólicos. Los cambios que sufre el vapor de agua
atmosférico en un remolino o torbellino de aire producen los llamados meteoros acuoso-eólicos,

7
-Pág.11-
C I E N C I A S D E L A N AT U R A L E Z A

como el huracán y el tornado. El huracán se trata de un remolino con diámetro entre 400 y 500
Km que se caracteriza por un centro de acción (ojo del huracán) o masa de aire caliente y
húmedo de muy baja presión que asciende rápidamente, dejando un vacío a nivel del mar que
es llenado por un viento muy fuerte y rápido. Los huracanes se desplazan en la dirección de los
vientos dominantes en el área intertropical. El tornado es un remolino o torbellino nuboso
constituido por gotitas de agua mezcladas con polvo y diversas partículas, de aproximadamente,
1 km de diámetro, largo y estrecho que a modo de columna va desde una tormenta al suelo. Si
se produce en el mar se denomina tromba; es de efectos más destructivos que el huracán,
pudiéndose presentar en muchos lugares del mundo (incluida España) especialmente en
primavera.

1.4. ALGUNAS VARIABLES QUE CONDICIONAN EL TIEMPO


ATMOSFÉRICO
Debido al movimiento de rotación de la Tierra, se producen variaciones de temperatura
atmosférica a lo largo del día llamadas oscilaciones térmicas diarias. También existe una
variación anual debido al movimiento de traslación y la inclinación del eje de giro.
Una variable que resulta determinante es la presión atmosférica que es la fuerza que ejerce
el aire sobre cada unidad de superficie de la Tierra, se mide en pascales (Pa) que es 1N/m2, no
obstante en meteorología se emplea el milibar, que es la milésima parte de la presión
atmosférica registrada a nivel del mar, y la atmósfera. La equivalencia es 1 atmósfera son 760
mm de Hg (presión atmosférica a nivel del mar) y es igual a 1013 milibares, ésta es la presión
atmosférica normal a nivel del mar. El aire frío es más denso, por lo que pesa más y por lo tanto
ejerce mayor presión mientras que al aire caliente le sucede lo contrario. Conforme ascendemos
del nivel del mar, la presión atmosférica decrece unos 100 milibares por cada Km y en la
superficie terrestre existen áreas de altas presiones y áreas de bajas presiones.
Cuando el aire está caliente, disminuye su presión sobre el suelo. Por tanto en zonas de bajas
presiones el aire asciende y son ocupadas por aire frío de otras zonas. Este fenómeno es el que
da origen a los vientos y a las lluvias, como ya se ha indicado. En las zonas de altas presiones
el aire desciende y se desplaza hacia las de baja presión.

1.5. APARATOS DE MEDIDA


Son varios los parámetros meteorológicos que se miden, existiendo un aparato específico
para cada uno.
Para medir la temperatura se emplea el termómetro. Es muy útil el de máximas y mínimas
que son dos tubos capilares en forma de U que presentan un índice coloreado dentro de las
columnas y marca en una la máxima y en la otra la mínima para el período marcado.

8
-Pág.12-
U . D . 5 - L OS M AT E R I A L E S T E R R E S T R E S

Para medir la presión atmosférica se emplea el barómetro. Cuando el barómetro marca un


valor por debajo de la presión atmosférica normal, hay tiempo de lluvias y el aire es húmedo,
porque a presiones bajas el agua de los mares, lagos y ríos se evapora en gran cantidad pasando
a la atmósfera.
Para medir la humedad atmosférica, es decir, la cantidad de vapor de agua que contiene el
aire, se emplea el higrómetro. Es un factor determinante la temperatura, de modo que al
aumentar la temperatura del aire, aumenta la capacidad de retención de vapor de agua. Se habla
de humedad absoluta como el peso del vapor de agua contenido en un metro cúbico de aire y la
humedad de saturación, es la cantidad de vapor de agua que admite una masa de aire a una
temperatura determinada. La humedad relativa (se expresa en %) es la humedad absoluta
dividida por la humedad de saturación y multiplicada por 100 (es la más utilizada).
La humedad relativa varía entre un 20% en climas desérticos y un 90% en zonas húmedas.
Para medir la humedad relativa se emplea los higrómetros, el más conocido es el
psicrómetro, que está constituido por dos termómetros, uno de ellos con el bulbo humedecido
permanentemente, se basa en el descenso de temperatura producido por la evaporación del agua
(que es más intensa cuanto más seco sea el aire investigado). La diferencia de temperatura
observada entre los dos termómetros indica el grado de humedad del aire (consultando unas
tablas de proporcionalidad).
Para medir la velocidad del viento se emplea el anemómetro y la veleta para medir su
dirección. La cantidad de lluvia caída se mide con el pluviómetro.

1.6. LOS RASGOS MÁS CARACTERÍSTICOS DE LOS MAPAS


DEL TIEMPO
Los mapas meteorológicos se preparan para zonas concretas a partir de los datos recogidos
de los distintos observatorios. Con estos datos se preparan los mapas de isobaras. Éstas son
líneas que unen los puntos de la superficie terrestre con la misma presión, presentándose como
líneas concéntricas. Se suele representar de 4 en 4 milibares, cuanto más próximas estén más
intensidad tendrá un frente. Los vientos no soplan en línea recta desde los anticiclones (altas
presiones) a las borrascas (bajas presiones) sino que sufren una desviación a causa de la rotación
de la Tierra, giran en espiral casi en la dirección de las isobaras. En los anticiclones el viento
sigue la dirección de las agujas del reloj y en las borrascas la contraria. Por eso cuando hay una
borrasca, los vientos vienen del sur y luego del oeste, trayendo lluvia. Si los vientos vienen del
este o del norte, se acerca un anticiclón y tendremos tiempo soleado.
El frente se indica como una línea perpendicular a las isobaras que lleva unos triángulos o
es de color azul, cuando el frente es frío y rojo con semicírculos cuando el frente es cálido.

9
-Pág.13-
C I E N C I A S D E L A N AT U R A L E Z A

2. EL AIRE

2.1. COMPOSICIÓN
El aire es una mezcla de gases: el 79% es nitrógeno, el 20% oxígeno y el 1% restante, vapor
de agua, dióxido de carbono y otros gases.
Podemos igualmente encontrar otros gases y sustancias como consecuencia de la contami-
nación atmosférica.

2.2. PROPIEDADES: PESO, MOVIMIENTO DE SUS PARTÍCULAS,


COMPRESIBILIDAD, CAPACIDAD DE ALTERAR MATERIALES
El aire posee las propiedades que posee la materia, ocupa espacio (volumen) y posee masa;
por lo tanto también posee densidad que disminuye al ascender en la atmósfera.
El aire tiene la capacidad de desplazarse desde unas zonas calientes hasta otras frías, por eso
decimos que el aire tiene la propiedad de la fluidez. En la atmósfera, se traslada de unas
regiones a otras originando el viento.
Otra propiedad del aire es la de permitir que el calor se propague por él. La atmósfera y la
propia Tierra, reciben energía solar por radiación, en forma de calor, se transmite y es absorbida
por el suelo, de forma que la Tierra se transforma en una fuente de calor que se vuelve a emitir
a la atmósfera. Actualmente, la Tierra se calienta unos dos grados cada cincuenta años, a causa
de la contaminación (sobre todo el exceso de CO2), que actúa como pantalla, calentándose
también la atmósfera, no pudiendo escapar el exceso de calor al espacio exterior, esto se conoce
como efecto invernadero.

2.3. IMPORTANCIA PARA LOS SERES VIVOS


Los seres vivos, tanto animales como vegetales, necesitan el aire para poder vivir.
Para nutrirse toman alimentos, que son sustancias que se han de transformar o destruir para
producir energía. Esta energía es la que utiliza el animal o vegetal para poder realizar sus
funciones vitales de crecimiento, desarrollo, reproducción, etc. La destrucción del alimento para
producir energía se realiza mediante la respiración celular. El alimento se quema en presencia
de oxígeno, en un proceso parecido a una combustión y produce energía, dióxido de carbono y
agua. El consumo de oxígeno en la Tierra es muy grande, debido a la enorme cantidad de seres

10
-Pág.14-
U . D . 5 - L OS M AT E R I A L E S T E R R E S T R E S

vivos que hay en nuestro planeta, pero el nivel no desciende gracias a la función clorofílica o
fotosíntesis que realizan las plantas las cuales forman materia orgánica a partir de dióxido de
carbono y agua en presencia de la luz solar.

Por otra parte aunque los soportes de la vida son el medio terrestre y el acuático, algunos
animales emplean el medio aéreo, es decir, vuelan o “se apoyan en el aire” para desplazarse.

Aunque en la actualidad son las aves las dueñas del aire, millones de años antes de que
existieran ya volaban los insectos.

3. EL AGUA
El agua es el agente básico en la formación y modelado de la Tierra, pero evidentemente su
mayor importancia radica en el hecho de ser indispensable para el mantenimiento de la vida. En
cuanto a su composición, se trata de una sustancia pura, cuya molécula está formada por 3
átomos, H2O, y su cualidad más relevante es que disuelve gases, líquidos y sólidos.

Las características de los seres vivos, que diferencian el mundo animal del inerte, están
basadas en las funciones de autoconservación, autorregulación y reproducción, y en todas ellas
el agua representa un papel determinante.

3.1. PROPIEDADES: BUEN DISOLVENTE, GRAN CAPACIDAD


CALORÍFICA, CAPACIDAD DE ALTERAR MATERIALES

La importancia del agua para los seres vivos se basa en una serie de propiedades que hacen
de ella una sustancia única:

– Es un gran disolvente. Muchas de las sustancias fundamentales para la vida son solubles
en el agua. Como las sustancias reaccionan más rápidamente disueltas en un líquido,
todos los procesos vitales se producen más fácilmente en disolución.

– Cambia poco su temperatura. Para subir o bajar un grado la temperatura hace falta darle
o quitarle mucho calor. De esta forma, se regula la temperatura de los seres vivos y se
evitan las temperaturas extremas.

– Cuesta mucho evaporarla. Para convertirla en gas, también es necesario mucha energía,
por lo que sirve de refrigerante cuando hace mucho calor.

11
-Pág.15-
C I E N C I A S D E L A N AT U R A L E Z A

- Al convertirse en hielo flota. Esto hace que, en caso de frío extremo, los mares, ríos y
lagos se hielen sólo en la superficie y puedan mantener las capas inferiores a una tempe-
ratura más moderada, nunca por debajo de 0°C.

3.2. LA HIDROSFERA
La hidrosfera es el conjunto de toda el agua que hay en la corteza terrestre, suponiendo el
70% de la superficie de nuestro planeta.
El agua se puede dividir en aguas continentales y aguas oceánicas. No olvidemos que
además parte del agua se encuentra en la atmósfera en forma de vapor.
Las aguas continentales, son dulces, se encuentran en los continentes, lagos, ríos, glaciares,
tanto a nivel superficial como subterráneo (emergen mediante pozos y manantiales).
Las aguas oceánicas, son aguas saladas de mares y océanos. Ocupan la mayor parte de la
superficie terrestre y teniendo en cuenta que la profundidad media de los océanos es de 4000
metros, la cantidad de agua es inmensa. En el fondo del mar existen también cordilleras,
llanuras, abismos y volcanes.

3.3. EL CICLO DEL AGUA


El agua está en la Tierra, manteniendo un ciclo constante, fundamental para la vida en el
medio terrestre y en los ríos y lagos. El motor que mueve este ciclo, igual que el de los vientos,
es el Sol.
Partiendo del mar, el Sol evapora con su calor el agua, que pasa a estado gaseoso. Al
ascender encuentra corrientes frías de aire, que hacen que se condense (licúe) en pequeñas
gotitas, que son las nubes. Estas gotitas de las nubes pueden juntarse en gotas más grandes, y
caer en forma de lluvia o de nieve, si hace suficiente frío. La lluvia que cae en los continentes
se acumula en los ríos y lagos (escorrentía superficial) o se infiltra a través del suelo y las rocas
(escorrentía subterránea) acumulándose como aguas subterráneas hasta que desemboca en el
mar. La nieve se almacena en las zonas frías hasta que, con el deshielo, baja por los ríos.

3.4. IMPORTANCIA PARA LOS SERES VIVOS


El agua es imprescindible para el mantenimiento del arbolado que ayuda a hacer un clima
habitable.

12
-Pág.16-
U . D . 5 - L OS M AT E R I A L E S T E R R E S T R E S

Los árboles

- Aumentan la humedad y por lo tanto el nivel de precipitaciones.

- Sujeta la tierra con sus raíces, haciendo los suelos fértiles.

- Mediante la fotosíntesis, rebajan los niveles de dióxido de carbono y aumenta el del oxígeno.

Las agresiones hacia las masas arbóreas son cada vez mas abundantes, los incendios
provocados, las talas masivas e indiscriminadas y la lluvia ácida, están haciendo perder gran
superficie arbolada en toda la Tierra, lo que conlleva la pérdida de la capa vegetal, la erosión y
la desertización.

3.5. EL PROBLEMA DEL AGOTAMIENTO DE LOS RECURSOS

3.5.1. CALIDAD
El agua posee una calidad natural (sabor, contenido en sales, etc.) diferente según los
elementos que adquiere en su recorrido hacia los torrentes, ríos o lagos. En este trayecto va
cargándose de multitud de partículas orgánicas e inorgánicas, microorganismos, sales minerales
y otras impurezas generalmente derivadas de la contaminación agrícola (pesticidas, abonos,
excrementos, ...), a esto hay que añadir la contaminación industrial y la contaminación urbana
(detergentes, basuras caseras, ...).

Por todo ello la calidad del agua destinada a nuestro consumo puede variar mucho. La
posibilidad de mejorar dicha calidad implica resolver una serie de problemas, entre los cuales
se encuentran:

– Control de los vertidos industriales y urbanos.

– Renovación permanente de los sistemas de detección y control de los diferentes vertidos.

– Estudios de seguimiento de las variaciones en la calidad del agua de cada cuenca


hidrográfica, con objeto de detectar vertidos incontrolados.

– Fuertes sanciones para las empresas que contaminen clandestinamente.

3.5.2. POTABILIDAD
El proceso al que se somete al agua captada en los ríos, embalses y pozos hasta llegar a la
red de distribución se denomina potabilización. Para ello se realiza primero una precloración
para eliminar los microorganismos, que se depositan en el fondo, posteriormente se somete a

13
-Pág.17-
C I E N C I A S D E L A N AT U R A L E Z A

una fase de floculación, decantación y filtración, para eliminar los materiales finos y las
impurezas. Finalmente se somete a una nueva cloración antes de ser llevada a un depósito para
su distribución.

3.5.3. DEPURACIÓN
El tratamiento de las aguas residuales incluye varias fases, destinadas cada una de ellas, a
eliminar un tipo de contaminante. Al proceso completo se le llama depuración. Un agua residual
ya utilizada adquiere diferentes impurezas físicas, químicas y biológicas que dependen de su
procedencia, de forma que se debe tratar para su reutilización o vertido al río o al mar.

Para depurar las aguas residuales, se somete a una decantación para eliminar por gravedad
los sólidos mayores, después los microorganismos degradan la materia orgánica, eliminando en
un tercer paso los sólidos orgánicos que queden por decantación, posteriormente se produce el
desarenado mediante el paso por filtros de distinto tamaño y finalmente se inyecta cloro para
desinfectar el agua.

3.5.4. EL AGUA, UN RECURSO ESCASO


La cantidad de agua disponible para el ser humano en las zonas desérticas es mucho menor
de la necesaria y en zonas industrializadas se contamina y se hace mal uso de ella.

Aunque el 70% de la superficie del planeta está cubierta por agua, el 97% de la misma es
salada por lo que no puede utilizarse para uso doméstico y riego sin un tratamiento de
desalinización.

El equilibrio atmósfera-hidrosfera es muy sensible a la acción humana. Todo lo que cada uno
de nosotros pueda hacer para ahorrar agua, repercute en la salud de nuestros ríos y mares.

Está demostrado que cuanto mayor es el grado de desarrollo de un país, más crecen las
necesidades de agua de sus habitantes y más debemos esforzarnos en el ahorro.

14
-Pág.18-
U . D . 5 - L OS M AT E R I A L E S T E R R E S T R E S

4. LAS ROCAS Y MINERALES FUNDAMENTALES


QUE COMPONEN EL RELIEVE ESPAÑOL

4.1. LOS MINERALES. LAS ROCAS

4.1.1. LOS MINERALES

Los minerales son sustancias sólidas, inorgánicas, que se originan de forma natural y que
poseen una composición química definida y una estructura interna ordenada: una estructura
cristalina.

Todos los minerales están compuestos por elementos químicos. Algunos de ellos se
presentan en estado puro, son los llamados elementos nativos (oro, plata, azufre), pero en
general se presentan combinados siempre de igual manera, pudiéndose representar dicha
composición mediante una fórmula química, las partículas que integran los minerales están
perfectamente ordenadas, decimos que tienen estructura cristalina que puede ser visible
externamente, con caras planas y ángulos de valores fijos (forman un cristal). La principal
característica de un cristal es que el orden interno de sus partículas hacen que tengan una forma
geométrica, no teniendo porqué ser transparentes. Cuando carecen de ordenación, tienen
estructura amorfa.

Los cristales se pueden clasificar por su forma en siete sistemas cristalinos: cúbico (fluorita),
tetragonal (calcopirita), hexagonal (apatito), romboédrico (turmalina), rómbico (antimonita),
monoclínico (yeso) y triclínico (albita).

Cuando el mineral tiene una composición alta de un metal, decimos que es una mena, en
algunas ocasiones los cristales son de extraordinaria belleza, son las llamadas gemas o piedras
preciosas.

Para estudiar los minerales nos servimos de las propiedades específicas que los diferencian.
Las más características son: color (tanto del mineral como del polvo al hacer una raya), brillo
(metálico, sedoso, cristalino o carece de él), dureza (un mineral es más duro que otro cuando
puede rayarlo, se usa la escala de Mohs y ordena de 1 a 10 a los minerales, siendo el más duro
el diamante y el más blando el talco), exfoliación (indica como se rompe el mineral, en
escamas, en láminas, en cubos, etc.), efervescencia (cuando se trata de carbonatos, producen
burbujeo en contacto con cualquier ácido). Esta última prueba la puedes realizar con un poco
de mármol y unas gotas de vinagre (ácido acético).

15
-Pág.19-
C I E N C I A S D E L A N AT U R A L E Z A

4.1.2. LA TIERRA EN ACCIÓN: RELIEVE Y PAISAJE

4.1.2.1. ESTRUCTURA DE LA TIERRA


Al principio de los tiempos, cuando la Tierra estaba recién formada, los materiales se
encontraban fundidos y se fueron depositando según su densidad, de tal forma que los más
pesados, como el hierro y el níquel, se hundieron hasta el centro de la Tierra para formar el núcleo.
La manera en que se desplazan las ondas sísmicas por el interior de la Tierra, indica que los
materiales rocosos que componen el planeta están distribuidos en capas: corteza, manto y
núcleo. Para explicar el comportamiento de las ondas sísmicas en su propagación por el globo
terráqueo, se han propuesto dos modelos, el modelo geoquímico y el modelo dinámico.
En el modelo geoquímico, la existencia de discontinuidades en el interior de la Tierra señala
zonas de separación entre capas que presentan distinta composición química (corteza, manto y
núcleo), distinta composición mineral (manto externo y manto interno) o estado físico diferente
(núcleo externo y núcleo interno). La corteza es la capa más externa de la Tierra, y se extiende
hasta la discontinuidad de Mohorovicic, puede ser continental (con un grosor de hasta 70 km,
formada por rocas sedimentarias, magmáticas y metamórficas donde abundan el granito y la
andesita) y oceánica (entre 6 y 12 km de espesor, constituida por rocas muy densas como
basaltos y gabros). El manto, discurre entre las discontinuidades de Mohorovicic y la de
Gutemberg, constituido fundamentalmente por peridotitas -silicato ferromagnésico- cuyo
principal mineral es el olivino, la presión es muy alta, por lo que los minerales se encuentran
compactados en lo que se conoce como cambio de fase o transición, de las que hay dos, una a
los 400 km de profundidad y la segunda a 670 km, es la discontinuidad de Repetti y marca la
separación entre los mantos superior e inferior. Este último posee una composición química más
homogénea -óxidos de magnesio y silicio-. El núcleo se extiende desde la discontinuidad de
Gutemberg hasta el centro de la Tierra, la parte exterior es líquida, compuesta por hierro, níquel,
algo de azufre, silicio y oxígeno y separado del núcleo interno por la discontinuidad de Weichrt-
Lehman, esté último es sólido compuesto probablemente por cristales de una aleación de hierro
y níquel.
En el modelo dinámico, la estructura de la Tierra está dividida en sucesivas capas. El calor
interno se debe al calor primordial y a la desintegración de elementos radiactivos. Las elevadas
presiones hacen que los materiales rocosos, aunque se mantienen sólidos, adquieran ciertas
características de fluidos, en concreto son capaces de evacuar el calor interno hacia el exterior
mediante corrientes de convección. Los movimientos de los fluidos del núcleo externo, son la
causa del campo magnético terrestre. La capa más interna se denomina endosfera (constituido
por el núcleo interno, sólido por la enorme presión y núcleo externo “líquido”), las células de
convección del núcleo externo acumulan el calor en la capa D (que se detecta por una pequeña
disminución de la velocidad de las ondas sísmicas en el límite de separación entre el núcleo
externo y el manto inferior), desde donde se propaga, a través de las celdas de convección que
se establecen en la mesosfera y en la astenosfera (en su parte superior tiene lugar la formación

16
-Pág.20-
U . D . 5 - L OS M AT E R I A L E S T E R R E S T R E S

de magmas tan importantes en los fenómenos relacionados con la tectónica de placas: la


aparición de las dorsales oceánicas y las zonas de subducción), hasta la litosfera (existe la
litosfera continental muy gruesa, unos 100 km de espesor y la oceánica, de unos seis km de
espesor), que es la capa más externa, dividida en placas litosféricas que flotan sobre la astenos-
fera y mantienen un equilibrio de flotación que recibe el nombre de isostasia.
Una décima parte del calor acumulado en la capa D escapa de forma errática en forma de
chorro de magma muy caliente que asciende a través del manto y de la astenosfera hasta
alcanzar la litosfera oceánica o continental y originar un punto caliente con intensa actividad
volcánica.
Si las placas sufren erosión sufrirán un empuje hacia arriba, de la misma forma si se
acumulan sedimentos en las cuencas sedimentarias o hielo durante las glaciaciones, lo sufrirán
hacia abajo, restableciéndose el equilibrio. Las placas se encuentran unas junto a otras, a modo
de puzle en continua evolución. Estos bordes, son las zonas más inestables, donde las placas
crecen o desaparecen.
La tectónica de placas estudia cómo y por qué se mueven las placas litosféricas. También
explica en conjunto todos los fenómenos dinámicos que se producen en la Tierra (volcanes,
terremotos, formación de montañas, ...)
En realidad, los cinturones de terremotos y volcanes no son más que la manifestación
geológica asociada a los bordes de las placas: cada placa litosférica está limitada por dorsales
oceánicas, zonas de subducción o fallas de transformación.
El movimiento de las placas se debe al magma caliente que asciende por el manto hasta la
corteza en forma de células de convección y la desgarra. Se forma así una enorme grieta (rift)
a partir de la que se produce una dorsal oceánica que es una cordillera submarina, ampliando la
litosfera oceánica. A través de estas dorsales sale magma a borbotones, que se enfría y da lugar
a rocas basálticas en el fondo de los océanos. A lo largo de millones de años, la litosfera
oceánica va creciendo a ambos lados de la dorsal. Cuando se enfría, se hace más densa y se
hunde de nuevo en el manto, formando una zona de subducción, situadas en los abismos de los
océanos por donde desaparece la litosfera oceánica formando las profundas fosas oceánicas. En
su camino de regreso a las profundidades, los materiales rocosos de la litosfera oceánica
describen un plano inclinado denominado plano de Benioff.
Las fallas de transformación son desgarros del terreno que aparecen en zonas sometidas a
empujes horizontales en sentidos contrarios. Estas fracturas permiten que las placas se deslicen
unas junto a otras en sentidos contrarios, lo que genera actividad sísmica.
Las consecuencias geológicas del movimiento de las placas:
– Expansión de los océanos. Como consecuencia del crecimiento del fondo del océano a
ambos lados de las dorsales, los océanos se hacen cada vez más grandes.

17
-Pág.21-
C I E N C I A S D E L A N AT U R A L E Z A

– La danza de los continentes. Los continentes situados sobre las placas se mueven. Unas
veces se separan y otras colisionan.
– Volcanes y terremotos. En las zonas de subducción, grandes masas de rocas chocan entre
sí y originan terremotos. Las rocas de la placa que se hunde se convierten en magma, que
escapa por las grietas y forma los volcanes.
– Formación de montañas (orogénesis). El empuje de la placa que se hunde aplasta a los
sedimentos acumulados en las zonas de subducción, los pliega y fractura y luego los
levanta hasta formar las grandes cordilleras de montañas.
Los volcanes se forman cuando el magma, procedente del manto asciende hasta la super-
ficie, se enfría y da lugar a erupciones de gases, cenizas y rocas fundidas llamada lava. Los volcanes
de fisura se encuentran a lo largo de las dorsales, en el fondo de los océanos. Sus erupciones
son tranquilas y la lava, cuando se enfria, origina rocas basálticas que constituye la litosfera
oceánica. Pero en la superficie terrestre, las erupciones volcánicas no suelen ser tan tranquilas.
Los volcanes tienen la forma de un gigantesco cono que, en ocasiones, se convierte en una
enorme montaña de fuego. Cuando el magma procedente del manto se encuentra, aproxima-
damente, a un kilómetro de la superficie, se estanca y forma una bolsa llamada cámara magmá-
tica. Los gases que estaban disueltos en el magma dejan de estarlo, aumentan la presión de la
cámara y empujan al magma que, a través de la chimenea, asciende y sale por el cráter.

4.1.2.2. LAS ONDAS SÍSMICAS


En la profundidad de la Tierra se producen fracturas y deslizamientos de enormes masas
rocosas que causan los seismos o terremotos. Las vibraciones originadas se propagan en forma
de ondas sísmicas, que precisan un medio físico para propagarse. A lo largo de sus trayectorias
experimentan determinados cambios en la velocidad y en la dirección de propagación,
relacionados con la naturaleza y las propiedades de las rocas que atraviesan. Las ondas sísmicas
se generan en un foco o hipocentro, localizado a varios kilómetros de profundidad y se captan
mediante unos receptores denominados sismógrafos que se registran en gráficos denominados
sismogramas. Cuando se produce un terremoto se generan tres tipos de ondas sísmicas:
– Ondas P o primarias, son las más veloces. Se propagan a través de todos los medios,
sólidos, líquidos y gaseosos.
– Ondas S o secundarias. No se propagan a través de los medios fluidos (líquidos y gases).
– Ondas L o de superficie. Cuando las ondas P y S alcanzan el epicentro, que es la zona
de la superficie terrestre situada sobre el foco, generan ondas superficiales, llamadas
también L o lentas.
Las ondas P y S, se utilizan para obtener información del interior de la Tierra. Así se ha
puesto de manifiesto la presencia de discontinuidades entre las capas que forman la Tierra.

18
-Pág.22-
U . D . 5 - L OS M AT E R I A L E S T E R R E S T R E S

4.1.3. LAS ROCAS


Las rocas son agregados de minerales, constituidas por un solo mineral (como la caliza,
formada por calcita) o por la unión de varios minerales (como el granito, formado por cuarzo,
feldespato y mica).
Todas las rocas están formadas por los llamados minerales petrogenéticos. Los minerales
que encontramos en la composición de las rocas son los carbonatos (como la calcita y el
aragonito), óxidos como la magnetita y el oligisto, o sulfuros, como la pirita o el cinabrio. Pero
los más abundantes son los silicatos, que constituye el 98% de la composición de la corteza
terrestre.
La unidad fundamental de los silicatos es el grupo SiO 4–4 , en el que el átomo de silicio se
encuentra en el centro de un tetraedro imaginario rodeado por cuatro átomos de oxígeno
situados en los vértices. Cada tetraedro se puede unir con otros tetraedros vecinos, al compartir
átomos de oxígeno o, sin compartirlos, mediante cationes como el Fe2+ o el Mg2+.
Todas las rocas pueden ser de tres orígenes: magmático, sedimentario o metamórfico.

4.1.3.1. ROCAS MAGMÁTICAS


También llamadas ígneas. En el interior de la Tierra, los minerales formadores de rocas se
encuentran en el magma. El magma es un fluido formado fundamentalmente por silicatos y una
cantidad importante de gases disueltos. También puede contener fragmentos de roca sin fundir.
Cuando el magma asciende a la superficie, se enfría y da origen a las rocas magmáticas, que
pueden ser intrusivas (o plutónicas ) o extrusivas (o volcánicas).
En las rocas intrusivas, los cristales que se forman son más grandes que en las extrusivas
porque el enfriamiento es más lento, siendo todos los cristales de tamaño similar. Ejemplos de
rocas magmáticas son: el granito (intrusiva), que lo encontramos a nivel superficial por la
erosión sufrida por las rocas que se encontraban en los niveles superiores; el basalto (extrusiva)
cuyos cristales son muy pequeños porque la lava se enfrió rápidamente en el exterior; la piedra
pómez, es extrusiva, muy poco densa y llena de oquedades, debido a los gases.

4.1.3.2. ROCAS SEDIMENTARIAS


Están formadas por trozos de otras rocas que han sido erosionadas. En el fondo del mar los
sedimentos se acumulan y forman capas sucesivas o estratos. El peso de las capas superiores
aplasta a las inferiores, que se van compactando hasta formar las rocas sedimentarias
(litificación), cuyas partículas quedarán unidas entre sí por cementos naturales (sílice o
carbonato cálcico). El proceso de formación de rocas sedimentarias se denomina diagénesis o
litogénesis. En conjunto este proceso consta de una serie de pasos: compactación (debido al
peso de los sedimentos), cementación (se debe a la percipitación de sustancias disueltas en el

19
-Pág.23-
C I E N C I A S D E L A N AT U R A L E Z A

agua que circula entre ellos) y alteraciones químicas y mineralógicas (por reacciones químicas
entre los fragmentos depositados y las sustancias disueltas en el agua).
Las rocas sedimentarias pueden ser de distintos tipos:
– Detríticas. Son el resultado de la litogénesis de fragmentos de otras rocas. Los principales
son conglomerados (brechas, de cantos angulosos y pudingas, de cantos redondeados),
arcillas y areniscas.
– Rocas de origen físico-químico. Se producen por precipitación de sustancias químicas
disueltas. En este grupo se encuentran loa rocas carbonatadas, como las calizas y las
dolomías, y las rocas evaporitas, como el yeso y la halita.
– Rocas de origen orgánico. Se forman a partir de restos de seres vivos. Dentro de este
grupo también hay calizas como la creta, formada a partir de animales diminutos y plantas
marinas. También pertenecen a este grupo el carbón y el petróleo.
Tanto el carbón como el petróleo, se formaron hace millones de años, por lo que se les
conoce como combustibles fósiles. El carbón se formó por acumulación de restos vegetales en
regiones pantanosas y la posterior actuación de las bacterias sobre ellos. En este proceso se va
incrementando la concentración en carbono, existen varios tipos: turba, lignito, hulla y antracita.
El petróleo, es una mezcla de hidrocarburos que se formó por acumulación de plancton marino
muerto en zonas de poca profundidad, que después fue cubierto por material inorgánico
impermeable y modificado por acción de las bacterias. Si existen estructuras geológicas
(trampas petrolíferas), que frenen al petróleo, se forma un yacimiento petrolífero, en el cual, los
componentes volátiles, se sitúan en la parte superior y se forma el gas natural.

4.1.3.3. ROCAS METAMÓRFICAS


El aumento de temperatura, es debido al contacto con masas igneas calientes, al enterra-
miento o, incluso, al calentamiento generado por el rozamiento de materiales. El aumento de la
presión, es debido al peso de los materiales (presiones litostáticas) o a movimientos de la corteza
(presiones dirigidas).
Ejemplos de estas rocas son las pizarras, de estructura laminar debido a las fuertes presiones
a que se ven sometidas. También el mármol, el gneis y la cuarcita.

4.2. INTRODUCCIÓN AL CICLO PETROGENÉTICO


Como ya se ha visto, aunque existe una gran cantidad de tipos de rocas, tan sólo pueden
tener tres orígenes y su situación va a estar muy relacionado con el tipo de borde que limite las
placas litosféricas.
El magma asciende por las dorsales oceánicas o en los volcanes. Los sedimentos se van
acumulando en las cuencas sedimentarias (sobre las zonas de subducción) y el peso va

20
-Pág.24-
U . D . 5 - L OS M AT E R I A L E S T E R R E S T R E S

compactando y hundiendo estos sedimentos hasta hacerlos desaparecer en las profundas fosas
oceánicas. Las rocas se irán formando y transformando como consecuencia de estos procesos.

4.3. DEFORMACIONES DE LAS ROCAS: PLIEGUES, FALLAS


Y DIACLASAS
El movimiento de las placas litosféricas, generan fuerzas tectónicas (de distensión, cuando
se separan las placas y de compresión, cuando colisionan) que comprimen o rompen las rocas.
Los estratos rocosos se deforman y originan estructuras diferentes a las que tenían. Se producen
pliegues, fallas y diaclasas.
Las diaclasas son grietas y fisuras que se producen en las rocas, se producen cuando los
materiales se rompen por diversas causas (distensión, enfriamiento de los magmas, etc.) sin que
se produzca desplazamiento de los bordes.
Las fallas son fracturas de las rocas duras y rígidas con desplazamiento de los bloques
resultantes. Como consecuencia de su actividad se puede producir un terremoto. Puede ser
normal o directa, cuando un labio se desplaza hacia abajo sobre el plano de falla; se origina
por fuerzas de distensión e inversa, producida por fuerzas de compresión que hacen que uno de
los bloques se deslice hacia arriba sobre el plano de falla.

Labios de falla.- Bloques desplazados a lo largo del plano de falla.


Plano de falla.- Superficie a lo largo de la cual se produce la rotura y el desplazamiento.
Salto de falla.- Desplazamiento entre dos puntos que estaban unidos antes de la rotura. El
resalte que queda en el terreno es el escape de falla.

21
-Pág.25-
C I E N C I A S D E L A N AT U R A L E Z A

Los pliegues son ondulaciones de las rocas estratificadas que se producen cuando las fuerzas
tectónicas deforman los estratos sin llegar a romperlos. Para que una roca dura y frágil se
flexione tiene que encontrarse a gran profundidad, donde se vuelve plástica debido a las altas
presiones y temperaturas y, además, la fuerza de deformación tiene que actuar durante millones
de años para que se modele el pliegue.

Flanco.- Zona lateral del pliegue a ambos lados de la charnela.


Charnela.- Línea que une los puntos de máxima curvatura del estrato.
Núcleo.- Zona más interna del pliegue.
Plano axial.- Superficie imaginaria que divide el pliegue en dos partes. Pasa por todas las
charnelas de los estratos del pliegue.
Eje del pliegue.- Línea de intersección del terreno con el plano axial.
Distinguimos el sinclinal, que es el pliegue con la convexidad hacia abajo. En el núcleo se
sitúan los materiales más modernos y el anticlinal, que es el pliegue con la convexidad hacia
arriba y en su núcleo se sitúan los materiales más antiguos.

22
-Pág.26-
U . D . 5 - L OS M AT E R I A L E S T E R R E S T R E S

4.4. LA METEORIZACIÓN
Es el conjunto de procesos físicos y químicos, regulados por el clima, que descomponen las
rocas. La meteorización física o mecánica, las variaciones de temperatura y la congelación del
agua en las grietas, produce cambios en la estructura de la roca, disgregándolas, pero sin
modificar la composición química, tiene lugar en climas desérticos y en las montañas.
La meteorización química es más frecuente en climas húmedos, templados y ecuatoriales.
Se produce por la capacidad de las sustancias de carácter ácido que lleva disuelta el agua de
lluvia y reacciona con las rocas calizas y las disuelve, produciendo el modelado kárstico.

4.5. MODELADO DEL RELIEVE


Durante millones de años, las montañas se van desgastando lentamente, el paisaje va
cambiando lentamente. Las transformaciones son producidas por el agua, el viento y los seres
vivos. Se les llama agentes geológicos externos. Los procesos geológicos externos son la
erosión, el transporte, la sedimentación y la formación de rocas sedimentarias. La acción de
cada agente va a estar condicionada por el clima, la naturaleza de las rocas y su estructura.

La meteorización fragmenta y desgasta las rocas, dejando el camino preparado para la


erosión. La erosión consiste en el ataque a las rocas por las fuerzas del viento, la lluvia, los ríos
y el hielo. Las rocas se desintegran en pequeños fragmentos y son transportados y sedimentados
lejos de su lugar de origen.

El motor que mueve estas potentes fuerzas erosivas es el Sol, responsable de la formación
de los vientos y de las lluvias. Parte del agua de lluvia se filtra y da lugar a las aguas
subterráneas. La gravedad actúa sobre el agua que no se filtra y hace que corra pendiente abajo,
primero en forma de torrentes y aguas de arroyada (sin cauce fijo) y luego en forma de aguas
encauzadas en los ríos.

Los ríos desgastan montañas y escavan valles y modelan un paisaje de colinas con
pendientes suaves conocido conocido como penillanura. Los valles fluviales tienen forma de V.

Cuando discurren sobre terrenos blandos, como las arcillas originan cárcavas. Si entre los
materiales blandos hay intercalados grandes bloques duros, se forman pirámides de tierra. En los
terrenos duros y solubles, como las calizas, dan lugar a lenares o lapiaces. Pero cuando las aguas
salvajes discurren sobre laderas inestables se producen desprendimientos y deslizamientos.

La erosión es mayor en terrenos desprotegidos de vegetación. Las partículas erosionadas son


transportadas y depositados cuando el río pierde velocidad. Durante las crecidas se depositan y
contribuyen a originar llanuras aluviales y terrazas fluviales. Cuando se depositan en el mar, en
la desembocadura se forman deltas y estuarios.

23
-Pág.27-
C I E N C I A S D E L A N AT U R A L E Z A

En las altas montañas el agua está en forma de nieve que se acumula en el circo del glaciar
y se transforma en hielo, desde aquí desciende, es la lengua del glaciar a una velocidad de unos
2 metros por día y transporta fragmentos de roca denominadas morrenas. Los valles que
originan tienen forma de U. Al descender en altitud, el agua se derrite.
Las costas, son remodeladas permanentemente por el incesante movimiento de las aguas
oceánicas (olas, mareas y corrientes marinas) que actúan en combinación con otros factores: la
naturaleza de las rocas, la posición de los estratos y la presencia de fallas y diaclasas. El oleaje
del mar erosiona con fuerza los acantilados, llegando a desmoronarse, retrocediendo la línea de
mar y formando la plataforma de abrasión. Cuando el mar erosiona una roca dura, lentamente
va dejando un saliente (cabo), mientras que la roca más blanda, retrocede más rápidamente y se
origina un entrante (golfo, cala, ensenada y bahía). En otros casos los fragmentos se depositan
originando playas, barras costeras (La Manga del Mar Menor), tómbolos (enlaza un islote con
la costa), deltas y estuarios.
Al nevar o llover, parte del agua, en rocas permeables y con poros, se infiltra y se convierte
en aguas subterráneas, que al encontrar terrenos impermeables se acumula formando acuíferos.
A veces vuelve a salir en forma de manantial. Cuando el agua cargada de CO2 penetra en
terrenos calizos, penetra, disolviendo y tallando figuras, configurando galerías, cuevas, estalactitas
y estalagmitas. Este modelado se llama kárstico y es el resultado de la meteorización química
que transforma los carbonatos en bicarbonatos (solubles).
Los desiertos, son zonas de bajísima pluviosidad, aquí, el principal agente erosivo es el
viento que origina las dunas.

4.6. PROPIEDADES E IMPORTANCIA ECONÓMICA: YACIMIENTOS


MINERALES
Existen muchos procesos que dan lugar a la formación de minerales de forma natural,
aunque los principales son los siguientes: cristalización, por enfriamiento de un magma
(olivino y ortosa); sublimación inversa, a partir de algunos vapores presentes en el magma, que
se depositan alrededor del cráter de un volcán (azufre); evaporación, a partir de una disolución
acuosa que origina la precipitación de las sales que estaban en disolución (halita y yeso), y
metamorfismo, cuando tienen lugar recombinaciones y recristalizaciones que originan nuevos
minerales (clorita y cianita).
Los yacimientos minerales son acumulaciones naturales de un mineral que permiten su
explotación con un rendimiento económico.
Muchos yacimientos se forman en masas magmáticas, o en sus cercanías, son los
yacimientos magmáticos. Cuando la mayor parte del magma cristaliza, se pueden formar en su
interior acumulaciones de algunos de sus componentes minoritarios, dando lugar a yacimientos
ortomagmáticos (diamante y platino). Al finalizar la consolidación magmática, cuando la

24
-Pág.28-
U . D . 5 - L OS M AT E R I A L E S T E R R E S T R E S

temperatura y la presión disminuye, las sustancias gaseosas se escapan y arrastran elementos


metálicos, que se depositan en las grietas de las rocas y forman los yacimientos neumatolíticos
(casiterita y wolframita). Cuando precipitan los minerales arrastrados por el vapor de agua, se
originan los yacimientos hidrotermales (galena y cinabrio).
Los yacimientos sedimentarios son lugares donde se acumulan minerales que se han
“arrancado” de otras zonas y se han transportado y sedimentado. Los más importantes son los
placeres, depósitos continentales de minerales pesados y piedras preciosas situados en los
fondos de los ríos y sobre las playas, que proporcionan la mayor parte del oro, el platino y el
estaño del mundo. También existen yacimientos evaporíticos de minerales que estaban en
disolución y que cristalizan en los mares y lagos interiores por evaporación (yeso, halita,
silvina). Cuando los depósitos minerales sufren metamorfismo, se habla de yacimientos
metamórficos (grafito y talco).
En España, los principales yacimientos minerales son:
– Yacimientos de hierro. Las zonas ferríferas se localizan en el Sistema Ibérico, en Sierra
Nevada y en la Cordillera Cantábrica, como el de Somorrostro (Bilbao), de origen
hidrotermal.
– Yacimientos de plomo y zinc. Generalmente aparecen asociados por tener el mismo
origen. Estos yacimientos hidrotermales se sitúan alrededor de focos volcánicos que dan
lugar a filones de galena (PbS) y blenda (ZnS), que atraviesan rocas de caliza y arenisca
o las propias rocas volcánicas. Los más importantes son los de Cartagena (Murcia) y La
Carolina (Jaén).
– Yacimientos de cobre. La mena más importante es la calcopirita (CuFeS2), asociada
generalmente con la pirita (FeS2), que se localiza en filones de origen hidrotermal. Las
minas de cobre de Riotinto (Huelva), están formadas por grandes masas de pirita y
calcopirita, situadas en formaciones rocosas de origen volcánico, originadas posiblemente
por una fumarola submarina que depositó el cobre.
– Yacimientos de aluminio. Son de origen sedimentario y su mena principal son los
yacimientos de bauxita, como los del Pirineo leridano o los de Portillo de Luna (León).

4.7. GRANDES UNIDADES LITOLÓGICAS DE ESPAÑA


En España, se distinguen cinco unidades geológicas:
– El macizo hespérico. Es el núcleo más antiguo, formado por materiales paleozoicos que
se encuentran muy erosionados y que forman parte de una antigua cordillera que se formó
en la orogénesis hercínica. Se sitúa en la zona centro-occidental de la península y
comprende el macizo Galaico-portugués, el Sistema Central, los Montes de Toledo y los
Montes de León.

25
-Pág.29-
C I E N C I A S D E L A N AT U R A L E Z A

– Las cordilleras periféricas. Son las cordilleras que circundan el macizo hespérico. Sus
relieves son más jóvenes, ya que se han formado más recientemente: Pirineos, cordillera
Costero-catalana y cordillera Ibérica.
– Las depresiones o fosas. Son hundimientos de la corteza terrestre que al final del
Mesozoico se convirtieron en lagos y que se han rellenado por sedimentos terciarios de
origen continental (Depresiones del Duero, Tajo y Ebro) o de origen marino (depresión
del Guadalquivir).
– Las cordilleras Béticas. Se sitúan en la zona meridional de la península Ibérica y se
forman durante la orogenia alpina.
– Los archipielagos. El archipiélago canario, es de origen volcánico y algunas de sus islas
se han formado en épocas muy recientes de la historia de la Tierra, por lo que en ellas se
pueden observar los rasgos característicos del modelado volcánico. El archipiélago balear
es una prolongación de la plataforma continental.
Por otra parte, la península Ibérica se puede dividir en tres grandes unidades litológicas:
los terrenos silíceos, los terrenos calizos y los terrenos arcillosos.
Los terrenos silíceos, incluyen rocas en cuya composición predomina el cuarzo y
corresponde a los relieves más antiguos (precámbrico y paleozoico) de la Península.
Predominan el granito (roca plutónica) en grandes masas (batolito) que presentan una enorme
cantidad de grietas; predominan en el sistema Central, en los Pirineos, en la zona galaico-
extremeña y en la parte de Zamora y Salamanca. Las pizarras, gneis y cuarcitas, aparecen en
todas las zonas modificadas por la orogenia Hercínica, como los montes de León y de Toledo.
Los terrenos calizos, corresponden a los materiales mesozoicos que afloran en las
cordilleras periféricas del macizo Ibérico. Se distinguen fácilmente por la superposición de
capas y produce efervescencia. Su origen es variado y a menudo contienen fósiles.
Los terrenos arcillosos, coinciden con las depresiones rellenadas con sedimentos terciarios
y cuaternarios, como las del Ebro, el Tajo, el Duero y el Guadalquivir. Es una roca impermeable
que procede de la meteorización de los granitos y otras rocas magmáticas y metamórficas.

4.8. LOS FÓSILES


El estudio de las rocas nos revela a través de los fósiles las formas de vida animal y vegetal
que había en la Tierra, hace muchos millones de años. Los fósiles son restos mineralizados de
seres vivos (animales o vegetales) o las huellas (pisadas, madrigueras y rastros) que han
quedado de su actividad biológica.
Un organismo puede fosilizar cuando su cuerpo es sepultado rápidamente por sedimentos
que le aislan del contacto con la atmósfera y con los microorganismos; así, las partes más

26
-Pág.30-
U . D . 5 - L OS M AT E R I A L E S T E R R E S T R E S

blandas se descomponen, y las más duras, como los dientes, los huesos y las conchas, son las
únicas que se conservan. Otras veces, las partes más duras también desaparecen y se forma un
hueco o molde, que se rellena de minerales y da lugar a un fósil (vaciado) con la misma
apariencia externa o interna que el organismo original. En algunos casos, se conserva el
organismo original, como es el caso de ciertos insectos que quedaron atrapados en el ámbar
(resina fósil).
La Paleontología es la ciencia que estudia los fósiles.

EL SUELO.
5. DESTRUCCIÓN, CUIDADO Y RECUPERACIÓN

5.1. EL SUELO
El suelo es el resultado de un lento proceso de formación en el que intervienen varios
factores: roca madre, seres vivos, clima y pendiente.
La roca madre, en la que se asienta el suelo. Parte de los constituyentes del suelo provienen
de la descomposición y disgregación de la roca madre.
Los seres vivos. Las plantas, las bacterias y las lombrices, entre otros, modifican la
composición del suelo, al incorporar a él la materia orgánica.
El clima de una zona determina la erosión y meteorización física de la roca, además de
favorecer una fauna y una flora determinadas. Prácticamente las zonas climáticas coinciden con
las zonas de suelos.
La pendiente. El suelo se asentará más fácilmente en un terreno llano que en una ladera de
una montaña.

5.1.1. FORMACIÓN DEL SUELO


Los factores anteriormente expuestos, determinarán un determinado suelo, en el que se
encontrarán distintos tipos de sustancias. De la roca provienen las partículas minerales. La
materia orgánica en descomposición, procedente de los seres vivos, se encuentra en la
superficie, mezclada con la tierra y forma el humus, de color negruzco. También forman parte
de los suelos el agua y el aire que llenan los huecos que quedan entre las partículas sólidas.
El aire y el agua suponen gran parte del volumen total del suelo. El aire permite que las
raíces de las plantas, los microorganismos y los animales que viven en él respiren. El agua,
al filtrarse en el suelo, remueve las partículas colocándolas en distintas capas. Observando

27
-Pág.31-
C I E N C I A S D E L A N AT U R A L E Z A

las capas de distintos suelos, podremos apreciar sus semejanzas y diferencias, o sea,
clasificarlos.

Se denominan horizontes a las distintas capas que se aprecian en el corte vertical de un


suelo. Aunque no tienen límites perfectamente definidos, suelen presentar colores y aspectos
diferentes. Podemos encontrar:

– Horizonte O, es una fina capa de hojas en descomposición.


– Horizonte A, proliferan las raíces de las plantas, así como microorganismos, ya que es la
zona más rica en humus y nutrientes.
– Horizonte B, es el subsuelo rico en minerales, que se depositan arrastrados por el agua a
través de los huecos del suelo.
– Horizonte C, es el subsuelo infértil, en el que abundan fragmento de roca.
– Horizonte D, es la roca madre.

5.1.2. CARACTERÍSTICAS DEL SUELO


No sólo los horizontes caracterizan a un suelo. Hay una serie de propiedades que, en
conjunto, determinan en buena medida su fertilidad. Vamos a hablar de dos de ellos, tamaño de
las partículas y acidez del suelo.

Dependiendo del tamaño que tengan las partículas, se denominan gravas (más de 2 mm de
diámetro), arenas (entre 0,02 y 2 mm), limos (entre 0,002 y 0,02 mm) y arcillas (menos de 0,002
mm de diámetro).

Cuanto mayor sea el tamaño de las partículas que forman el suelo, mayor será el tamaño de
los espacios que quedan entre ellas, y pasará el agua con más facilidad, es decir, será más
permeable; en los suelos limosos y arcillosos, el agua se encharca.

Por otra parte, dependiendo de la naturaleza de las partículas, los suelos pueden ser ácidos,
neutros y básicos, la mayoría de las plantas prefieren suelos neutros, aunque existen
adaptaciones a los tres tipos.

5.2. DESTRUCCIÓN, CUIDADO Y RECUPERACIÓN


Desde la prehistoria, el ser humano ha utilizado el suelo para su sustento, dedicándolo tanto
a la agricultura como a la ganadería. Mientras que se trató de actividades de subsistencia, el
suelo no se resintió mucho, pero cada vez más la humanidad pretende explotar el suelo y extraer
en el menor tiempo posible todo cuanto se pueda.

28
-Pág.32-
U . D . 5 - L OS M AT E R I A L E S T E R R E S T R E S

El agotamiento y la pérdida de suelo fértil supone la pérdida de la capa vegetal, o sea, la


desertización. La deforestación, también favorecen la erosión y destrucción del suelo, lo mismo
que la minería a cielo abierto o el pastoreo abusivo.
Los métodos poco racionales, sin rotación de cultivos, con abuso de fertilizantes y
pesticidas, empobrecen el suelo y envenenan las aguas. En las zonas de cultivo creadas
artificialmente donde hay lluvias y vientos fuertes, se produce la pérdida rápida de suelo, dando
lugar a barrancos y tierras yermas. La expansión de las ciudades, deja bajo el asfalto amplias
zonas de suelo fértil. El drenaje de zonas húmedas destruye el ecosistema y altera el equilibrio
hidrológico de las zonas próximas.
Debemos concienciarnos para proteger el suelo, la utilización de cultivos biológicos, el
planteamiento de ganadería sostenida, el respeto a los bosques y las plantaciones que protegen
de la erosión, son estrategias cada vez más empleadas y respetadas.

29
-Pág.33-
C I E N C I A S D E L A N AT U R A L E Z A

RESUMEN

— La capa gaseosa que rodea la Tierra se llama atmósfera. Su composición y grosor han
ido variando a lo largo de los 4600 millones de años del planeta.
— La atmósfera está formada por capas: troposfera, estratosfera, mesosfera, ionosfera o
termosfera y exosfera.
— En la atmósfera se producen fenómenos meteorológicos (fenómenos eléctricos y
luminosos), eólicos, acuosos y mixtos.
— Los mapas meteorológicos representan la presión atmosférica.
— La presión atmosférica, es la fuerza que ejerce el aire por unidad de superficie, se mide
en milibares.
— El aire posee las siguientes propiedades: masa, volumen y densidad y transmite el calor
por radiación.
— El agua, tiene una gran importancia, para la vida y para la formación y modelado de la
Tierra. Posee las siguientes características: buen disolvente, cambia poco su
temperatura, cuesta mucho evaporarla, al convertirse en hielo flota.
— Las aguas se dividen en continentales y oceánicas. Todas ellas están relacionadas con
el ciclo del agua.
— Para poder consumir agua, debe prepararse potabilizándola y depurándola.
— Los minerales son sustancias sólidas, inorgánicas, que se originan de forma natural y
poseen una composición química definida y una estructura interna ordenada
(estructura cristalina).
— Para estudiar los minerales nos servimos de sus propiedades específicas. Color, brillo,
dureza, exfoliación y efervescencia.
— La manera en que se desplazan las ondas sísmicas por el interior de la Tierra, indica
que los materiales rocosos que componen el planeta están distribuidos en capas:
corteza, manto y núcleo. Para explicar el comportamiento de las ondas sísmicas en su
propagación por el globo terráqueo, se han propuesto dos modelos, el modelo
geoquímico y el modelo dinámico.
— En el modelo geoquímico, la existencia de discontinuidades en el interior de la Tierra
señala zonas de separación entre capas que presentan distinta composición química
(corteza, manto y núcleo), distinta composición mineral (manto externo y manto
interno) o estado físico diferente (núcleo externo y núcleo interno).

30
-Pág.34-
U . D . 5 - L OS M AT E R I A L E S T E R R E S T R E S

— En el modelo dinámico (endosfera, capa D, mesosfera, astenosfera y litosfera), las


elevadas presiones hacen que los materiales rocosos, aunque se mantienen sólidos,
adquieran ciertas características de fluidos, en concreto son capaces de evacuar el calor
interno hacia el exterior mediante corrientes de convección.
— Las celdas de convección que se establecen en la mesosfera y en la astenosfera (en su
parte superior tiene lugar la formación de magmas tan importantes en los fenómenos
relacionados con la tectónica de placas: la aparición de las dorsales oceánicas y las
zonas de subducción), hasta la litosfera, que es la capa más externa, dividida en placas
litosféricas que flotan sobre la astenosfera.
— La pequeña cantidad del calor acumulado en la capa D, escapa erráticamente a través
del manto y la astenosfera, y origina puntos calientes de intensa actividad volcánica.
— Las placas se encuentran unas junto a otras, a modo de puzle en continua evolución.
Estos bordes, son las zonas más inestables, donde las placas crecen o desaparecen.
— La tectónica de placas estudia cómo y por qué se mueven las placas litosféricas.
También explica en conjunto todos los fenómenos dinámicos que se producen en la
Tierra (volcanes, terremotos, formación de montañas, ...)
— Los cinturones de terremotos y volcanes no son más que la manifestación geológica
asociada a los bordes de las placas: cada placa litosférica está limitada por dorsales
oceánicas, zonas de subducción o fallas de transformación.
— El movimiento de las placas se debe al magma caliente que asciende por el manto hasta
la corteza en forma de células de convección y la desgarra. Se forma así una enorme
grieta (rift) a partir de la que se produce una dorsal oceánica que es una cordillera
submarina, ampliando la litosfera oceánica.
— Cuando se enfría, se hace más densa y se hunde de nuevo en el manto, formando una
zona de subducción, situadas en los abismos de los océanos por donde desaparece la
litosfera oceánica formando las profundas fosas oceánicas.
— Las consecuencias geológicas del movimiento de las placas son la expansión de los
océanos, la danza de los continentes, los volcanes y terremotos y la formación de
montañas (orogénesis).
— Los terremotos son la consecuencia de las vibraciones que se producen en el interior de
la Tierra cuando se deslizan o se fracturan enormes masas rocosas. Las vibraciones se
desplazan como ondas sísmicas.
— Las ondas sísmicas pueden ser de tres tipos, ondas P, ondas S y ondas L.
— Las rocas son agregados de minerales petrogénéticos, constituidas por un solo mineral
(como la caliza, formada por calcita) o por la unión de varios minerales (como el
granito, formado por cuarzo, feldespato y mica). Los minerales petrogenéticos pueden
ser carbonatos, óxidos, sulfuros, aunque los más abundantes son los silicatos.

31
-Pág.35-
C I E N C I A S D E L A N AT U R A L E Z A

— Las rocas pueden ser de tres tipos, magmáticas, sedimentarias y metamórficas.


— Cuando el magma asciende a la superficie, se enfría y da origen a las rocas magmáticas,
que pueden ser intrusivas (o plutónicas ) o extrusivas (o volcánicas).
— En el fondo del mar los sedimentos se acumulan y forman capas sucesivas o estratos.
El peso de las capas superiores aplasta a las inferiores, que se van compactando hasta
formar las rocas sedimentarias (litificación), cuyas partículas quedarán unidas entre sí
por cementos naturales (sílice o carbonato cálcico). Pueden ser detríticas, de origen
físico-químico y de origen orgánico.
— El carbón y el petróleo, tienen un origen de sedimentos orgánicos y se formaron hace
millones de años.
— Las rocas metamórficas se forman por transformación de otros tipos de rocas, es decir,
cuando las rocas magmáticas, las sedimentarias o incluso las metamórficas se ven
sometidas a grandes presiones o a elevadísimas temperaturas en el interior de la Tierra,
sin que se produzca fusión de la roca.
— El movimiento de las placas litosféricas, generan fuerzas tectónicas (de distensión,
cuando se separan las placas y de compresión, cuando colisionan) que comprimen o
rompen las rocas. Los estratos rocosos se deforman y originan estructuras diferentes a
las que tenían. Se producen pliegues, fallas y diaclasas (grietas).
— La meteorización es el conjunto de procesos físicos y químicos, regulados por el clima,
que descomponen las rocas, puede ser física y química.
— Las transformaciones que modelan el paisaje son producidas por el agua, el viento y los
seres vivos. Se les llama agentes geológicos externos. Los procesos geológicos
externos son la erosión, el transporte, la sedimentación y la formación de rocas
sedimentarias. La acción de cada agente va a estar condicionada por el clima, la
naturaleza de las rocas y su estructura.
— Existen muchos procesos que dan lugar a la formación de minerales de forma natural,
la cristalización, sublimación inversa, evaporación y metamorfismo son los principales.
— Los yacimientos minerales son acumulaciones naturales de un mineral que permiten su
explotación con un rendimiento económico. Pueden ser de origen magmático,
sedimentario y metamórfico.
— En España se distinguen cinco unidades geológicas: el macizo hespérico (zona centro-
occidental de la península); las cordilleras periféricas (circundan el macizo hespérico);
las depresiones (hundimientos de la corteza rellenos de sedimentos); las cordilleras
béticas (en la zona meridional de la península) y los archipiélagos.
— En España se distinguen tres grandes unidades litológicas, los terrenos silíceos, los
terrenos calizos y los terrenos arcillosos.

32
-Pág.36-
U . D . 5 - L OS M AT E R I A L E S T E R R E S T R E S

— Los fósiles son restos mineralizados de seres vivos (animales o vegetales) o las huellas
(pisadas, madrigueras y rastros) que han quedado de su actividad biológica.
— Un organismo puede fosilizar cuando su cuerpo es sepultado rápidamente por
sedimentos que le aíslan del contacto con la atmósfera y con los microorganismos.
— El suelo es el resultado de un lento proceso de formación en el que intervienen varios
factores: roca madre, seres vivos, clima y pendiente.
— Se denominan horizontes a las distintas capas que se aprecian en el corte vertical de un
suelo. Aunque no tienen límites perfectamente definidos, suelen presentar colores y
aspectos diferentes. Podemos encontrar: Horizonte O, es una fina capa de hojas en
descomposición; Horizonte A, proliferan las raíces de las plantas, así como
microorganismos; Horizonte B, es el subsuelo rico en minerales; Horizonte C, es el
subsuelo infértil, en el que abundan fragmento de roca y Horizonte D, es la roca madre.

33
-Pág.37-
C I E N C I A S D E L A N AT U R A L E Z A

EJERCICIOS DE AUTOCOMPROBACIÓN

1. La capa gaseosa que envuelve la Tierra es:

A. La atmósfera B. La hidrosfera

C. La troposfera D. Todas son ciertas

2. La dirección del viento se mide con:

A. El barómetro B. El anemómetro

C. La veleta D. El pluviómetro

3. Cuando se acerca un anticiclón la presión atmosférica:

A. Aumenta B. Disminuye

C. Permanece constante D. Es independiente una cosa de la otra

4. La propagación del calor proveniente del Sol a través de la atmósfera es por:

A. Conducción B. Convección

C. Radiación D. Todas son ciertas

5. La presencia de un bosque en una región:

A. Aumenta el efecto invernadero B. Aumenta el nivel de precipitaciones

C. Consume el oxígeno de la atmósfera D. Todas son falsas

6. El calor interno de la Tierra, procedente del núcleo y del manto, es el responsable de:

A. La formación de los vientos B. El ciclo del agua

C. Los movimientos de las placas litosféricas D. Las precipitaciones

7. El granito es una roca:

A. Magmática o ígnea B. Extrusiva

C. Efusiva o volcánica D. Metamórfica

8. El magma procedente del manto asciende hasta la corteza, fluye al exterior y forma volcanes:

A. En las dorsales oceánicas B. En las zonas de subducción

C. En las regiones calizas D. Todas son verdaderas

34
-Pág.38-
U . D . 5 - L OS M AT E R I A L E S T E R R E S T R E S

9. La unión de varios minerales es:

A. Un fósil B. Una roca

C. Una mena D. Una mezcla

10. Las distintas capas de un suelo se denominan:

A. Tipos de suelos B. Horizontes

C. Permeabilidad D. Ninguna de las anteriores

35
-Pág.39-
C I E N C I A S D E L A N AT U R A L E Z A

RESPUESTA A LOS EJERCICIOS

11. A

12. C

13. A

14. C

15. B

16. C

17. A

18. A

19. B

10. B

36
-Pág.40-
U . D . 6 . - D IVERSIDAD Y UNIDAD DE LOS SERES VIVOS

ÍNDICE
OBJETIVOS ............................................................2

INTRODUCCIÓN . . . . . . . . . . . . . . . . . . . . . . . . . . . . . . . . . . . . . . . . . . . . . . . . . . . . . . . . . 3

MAPA CONCEPTUAL . . . . . . . . . . . . . . . . . . . . . . . . . . . . . . . . . . . . . . . . . . . . . . . . . . . . . 4

DESARROLLO DE CONTENIDOS . . . . . . . . . . . . . . . . . . . . . . . . . . . . . . . . . . . . . . . . . . 5

1. LOS SERES VIVOS Y SU DIVERSIDAD . . . . . . . . . . . . . . . . . . . . . . . . . . . . . . . 5

2. LA CÉLULA COMO UNIDAD DE ESTRUCTURA DE LOS SERES VIVOS.


ORGANIZACIÓN CELULAR . . . . . . . . . . . . . . . . . . . . . . . . . . . . . . . . . . . . . . . 20

3. LAS FUNCIONES DE LOS SERES VIVOS: NUTRICIÓN, RELACIÓN


Y REPRODUCCIÓN. EL SER VIVO COMO SISTEMA . . . . . . . . . . . . . . . . . 29

4. INTRODUCCIÓN AL ESTUDIO DE LA HERENCIA . . . . . . . . . . . . . . . . . . . 35

RESUMEN . . . . . . . . . . . . . . . . . . . . . . . . . . . . . . . . . . . . . . . . . . . . . . . . . . . . . . . . . . . 40

EJERCICIOS DE AUTOCOMPROBACIÓN. . . . . . . . . . . . . . . . . . . . . . . . . . . . . . . . . . 44

RESPUESTAS A LOS EJERCICIOS. . . . . . . . . . . . . . . . . . . . . . . . . . . . . . . . . . . . . . . . . 44

-Pág.41-
C I E N C I A S D E L A N AT U R A L E Z A

OBJETIVOS
Al finalizar el estudio de esta Unidad Didáctica, el alumno será capaz de:

• Nombrar e identificar seres vivos.

• Utilizar modelos que permitan explicar la causa de la unidad y la diversidad de los


seres vivos.

• Conocer la estructura celular y los tipos de celula existentes.

• Distinguir las funciones de los seres vivos: nutrición, relación y reproducción.

• Diferenciar los cromosomas y cómo se transmite la herencia.

• Saber qué es una mutación.

-Pág.42-
U . D . 6 . - D IVERSIDAD Y UNIDAD DE LOS SERES VIVOS

INTRODUCCIÓN
S i observamos nuestro entorno con atención, percibiremos la enorme variedad de seres
vivos que hay en él: compañeros, árboles, pájaros, insectos, etc., todos ellos com-
parten un lugar y una época sobre la Tierra.

Pero si miramos con ojos científicos, veremos mucho más; por ejemplo, sus afinidades
y parentescos desde el punto de vista evolutivo, su estructura básica, la forma en que
podemos clasificarlos y otros muchos aspectos. Vamos a descubrir algunos de ellos.

-Pág.43-
C I E N C I A S D E L A N AT U R A L E Z A

M A PA C O N C E P T UA L
SERES VIVOS

DIVERSIDAD ORIGEN:CALDO UNIDAD


PRIMORDIAL
TEORÍA DE LA
Constituidos
EVOLUCIÓN DE por
DARWIN

Han dado CÉLULAS


lugar a

Por el número Por el tipo


REINOS VIRUS
UNICELULARES PROCARIOTAS

MONERAS
PLURICELULARES EUCARIOTAS

PROTISTAS
FUNCIONES ESPECIALIZACIÓN

FUNGI
NUTRICIÓN TEJIDOS

ANIMALES AUTÓTROFA
ANIMALES

VEGETALES HETERÓTROFA
Epitelial
Conectivo
Muscular
RELACIÓN Nervioso

VEGETALES
HERENCIA REPRODUCCIÓN
EMBRIONARIOS

ASEXUAL Meristemos
A veces sufre Regida por
ADULTOS
SEXUAL
MUTACIONES LEYES DE MENDEL Protectores
Parenquimáticos
Conductores
De sostén
Secretores

-Pág.44-
U . D . 6 . - D IVERSIDAD Y UNIDAD DE LOS SERES VIVOS

1. LOS SERES VIVOS Y SU DIVERSIDAD

1.1. LOS SERES VIVOS Y SU DIVERSIDAD


Aunque en la actualidad existe una enorme variedad de seres vivos, éstos, no siempre han
estado aquí, ni siempre han sido iguales.
El fenómeno de la vida, surgió en el agua de los mares, cuando las sustancias químicas se
fueron acumulando dando lugar a lo que los científicos llamaron caldo primordial, que era un
conjunto de materiales presentes en el agua de los mares primitivos a partir de los cuales se for-
maron los primeros seres vivos.
Todas estas sustancias de la sopa primordial evolucionaron hacia formas más complejas;
probablemente se juntaban unas con otras y originaban compuestos cada vez mayores y más
complejos y de una forma que precisó un enorme aporte de energía surgió la primitiva estruc-
tura viva.
La materia inanimada, surgida por evolución a partir de los componentes de la sopa pri-
mordial, se transformó en célula viva cuando fue capaz de obtener energía (mediante la nutri-
ción) y utilizar esta energía para reproducirse y responder a las variaciones del ambiente.
Las primitivas formas vivas eran organismos unicelulares procariotas, parecidos a las bac-
terias actuales.
La teoría de la evolución desarrollada por Darwin (1809-1882), explica que los seres vivos
que vemos hoy no fueron siempre así, sino que evolucionaron, es decir, experimentaron un pro-
ceso de cambio gradual de sus características, de generación en generación, a partir de un leja-
no antecesor.
Para indagar sobre los mecanismos con que opera le evolución, los científicos fijan su aten-
ción en las siguientes observaciones básicas:
— Variabilidad. Entre los animales y plantas hay variaciones, es decir, existe diversidad
biológica entre individuos por lo que no hay seres iguales.
— Capacidad reproductora. Es necesaria para no desaparecer.
— Herencia. Los seres vivos son semejantes a sus padres porque heredan muchos caracte-
res de los mismos, seleccionándose aquellos caracteres que resultan más beneficiosos,
que tenderán a aumentar frente a los que no tengan esa característica.
— La selección natural. El entorno proporciona alimento, refugio y un lugar para vivir y
reproducirse, pero cambios bruscos en las condiciones del mismo (frío, calor, sequía o
nuevos depredadores) obligan a los seres vivos a adaptarse a nuevas condiciones de

5
-Pág.45-
C I E N C I A S D E L A N AT U R A L E Z A

vida. Los que no lo consiguen se extinguen. La selección es, pues, obra del entorno que,
inestable y cambiante, no cesa de seleccionar o de elegir a los que están mejor adapta-
dos a las características ambientales del momento.

1.2. ALGUNAS RELACIONES ENTRE MORFOLOGÍA, FUNCIÓN


Y MODO DE VIDA
Son muchísimas las adaptaciones morfológicas que se producen en los cinco reinos, aunque
es más evidente en los reinos animal y vegetal.
Con respecto al agua, aparecen adaptaciones muy interesantes. Cuando el agua está en
exceso, inicialmente, no existe el problema, pero cuando hay sequía surge la necesidad de con-
servarla o buscar formas de resistencia hasta que lleguen períodos más óptimos. Los cactus,
aclimatados a vivir en zonas áridas, han transformado sus hojas en espinas para evitar la trans-
piración y han convertido su tallo en su reserva de agua, pues de esta forma presenta la menor
superficie al máximo volumen. Las heces de los animales, son más secas, conforme más seco
es el ambiente donde viven incluso la orina, que llega a ser sólida, como es el caso de la rata
del desierto.

1.3. LOS GRANDES MODELOS DE ORGANIZACIÓN DE ANIMALES


Y VEGETALES
Los seres vivos se dividen en cinco reinos: animales. plantas, fungi (hongos), moneras
(bacterias y algas verde-azuladas) y protistas (protozoos). Aparte, podemos mencionar a
los virus, que se hallan en la misteriosa barrera entre lo vivo y lo no vivo.

6
-Pág.46-
U . D . 6 . - D IVERSIDAD Y UNIDAD DE LOS SERES VIVOS

ESPONGIARIOS

CELENTÉREOS

PLATELMINTOS

NEMATODOS

ANÉLIDOS

MOLUSCOS

CRUSTÁCEOS

VIRUS ARTRÓPODOS ARÁCNIDOS

BACTERIAS

REINO INSECTOS
MONERAS
CIANOBACTERIAS
INVERTEBRADOS
EQUINODERMOS
REINO
PROTOZOOS
PROTISTAS
PECES

REINO
FUNGI HONGOS
SERES ANFIBIOS
VIVOS

REINO REPTILES
ANIMALES
VERTEBRADOS
AVES

ALGAS
MAMÍFEROS

REINO MUSGOS
PLANTAS
HELECHOS GIMNOSPERMAS

MONOCOTILEDÓNEAS

ESPERMAFITAS
ANGIOSPERMAS
DICOTILEDÓNEAS

Esquema de los seres vivos agrupados en los cinco reinos.

7
-Pág.47-
C I E N C I A S D E L A N AT U R A L E Z A

1.3.1. EL REINO DE LAS MONERAS


El reino moneras, con las bacterias y las algas verde-azuladas, fueron las primeras formas
de vida procariota del planeta y siguen aquí, rodeándonos por todas partes. Tienen una enorme
capacidad de adaptación a unas condiciones de vida determinadas. Son los agentes de reciclaje
más importantes de la naturaleza.

1.3.2. EL REINO DE LOS PROTISTAS


El reino protistas, comprende al grupo de los protozoos, que son en su mayoría organismos
unicelulares eucariotas. Se desplazan nadando o arrastrándose y algunos son parásitos. Pueden
ser flagelados (uno sólo) y ciliados (a modo de pelillos alrededor), como el Paramecium. Los
rizópodos son capaces de emitir parte de su citoplasma, produciendo unas prolongaciones lla-
madas pseudópodos, con los que se desplazan y capturan partículas alimenticias. Los esporo-
zoos, no tienen movimiento, se dejan arrastrar. Suelen ser parásitos, como el Plasmodium, que
transmite el paludismo. Todos estos seres viven en medio líquido y realizan su alimentación a
través de la membrana celular.

1.3.3. EL REINO ANIMAL

1.3.3.1. Animales invertebrados


Los animales invertebrados, son aquellos que carecen de esqueleto interno articulado. Esto
no implica que carezcan de alguna sujeción de sus órganos y tejidos, ya que muchos de ellos
han desarrollado conchas y esqueletos externos y otros emplean la presión de su líquido inter-
no para sustentarse.
Los invertebrados surgieron en el mar como consecuencia de la evolución de organismos
unicelulares hacia una estructura más compleja y eficiente. A partir de ese momento, evolucio-
naron en infinidad de formas y colonizaron todos los ambientes de la Tierra. Una de sus líneas
evolutivas dió origen a los primeros vertebrados. Invertebrados son las esponjas, los celenté-
reos, los platelmintos, nemátodos, anélidos, moluscos, artrópodos y equinodermos.
Las esponjas, carecen de sistema nervioso y muscular. Poseen un esqueleto córneo elástico,
se alimentan filtrando el agua por sus múltiples poros y se pueden reproducir sexual o asexual-
mente por gemación, es decir a partir de un pequeño bultito como si fuera una yema, nace un
nuevo individuo.
En los celentéreos, encontramos las anémonas, las medusas y los corales. Casi todos pose-
en en su cuerpo una infinidad de células urticantes para atacar o defenderse. Los corales cons-
truyen caparazones calcáreos que al superponerse, generación tras generación, dan lugar a arre-
cifes marinos e incluso islas. Las medusas y los pólipos, son dos estados de una misma especie

8
-Pág.48-
U . D . 6 . - D IVERSIDAD Y UNIDAD DE LOS SERES VIVOS

de celentéreos, es decir, se alternan las dos formas de modo cíclico, aunque en algunos casos
esto no sucede. La reproducción de las medusas es sexual y la de los pólipos por gemación.

Los platelmintos, nemátodos y anélidos, son gusanos. Los platelmintos son gusanos planos,
pueden reproducirse, bien asexualmente, por bipartición, es decir un individuo se escinde en dos
y sexualmente (cada individuo tiene los dos sexos, son hermafroditas); un ejemplo es la plana-
ria, que nada libremente en charcas y arroyos y la tenia, que es un parásito para el hombre que
se instala en su intestino grueso mediante unos ganchos situados en su parte anterior, absorbe
las sustancias ya digeridas por lo que no precisa aparato digestivo propio. Si se la deja prospe-
rar, puede llegar a alcanzar 8 metros de longitud.

Los nemátodos, son gusanos cilíndricos y su reproducción es sexual (cada individuo tiene
un solo sexo). Un ejemplo de este grupo es la triquina, que es un parásito para el hombre y el
cerdo. Forma quistes en los músculos del cerdo, es decir, se encierra enrollada en una cápsula.
Si una persona la ingiere, el gusano se libera y pasa del intestino a los vasos linfáticos, donde
la hembra pone los huevos para que las larvas se repartan por todos los tejidos donde se enquis-
tan pudiendo llegar a producir la muerte.

Los anélidos, son gusanos cilíndricos, en los que se distinguen anillos. También se distingue
la cabeza con una estructura nerviosa de una cierta complejidad y los órganos sensoriales (como
ojos simples). Además poseen quetas en la piel, que son unos pelillos quitinosos que emplean
para desplazarse. Su reproducción es sexual, pero unas especies son hermafroditas y otras tie-
nen los sexos diferenciados. En general son de vida libre; como ejemplo tenemos la lombriz de
tierra que se alimenta de la materia orgánica en descomposición del suelo, lo airean y lo fertili-
zan.

Pertenecen al grupo de los moluscos, los caracoles (gasterópodos), los pulpos (cefalópodos)
y los berberechos (lamelibranquios), entre otros. La estructura corporal típica de un molusco
presenta tres partes diferenciadas: la cabeza, el pie y el manto, que acoge la masa visceral. La
mayoría de ellos poseen concha, fabricadas por el manto con sustancias minerales, que sirven
de protección. El pie es una masa muscular que posee distintas funciones en cada clase de
moluscos. Otro órgano característico es la rádula, una lengua con dientes que les sirve para ras-
par alimentos. La gran mayoría son acuáticos (marinos) que respiran por branquias, aunque
algunos caracoles y babosas viven en tierra, en zonas húmedas y respiran mediante pulmones.
La reproducción es sexual, con individuos de distinto sexo, excepto los pulmonados, que son
hermafroditas. Los gasterópodos, han adaptado su pie para desplazarse reptando. Los lameli-
branquios, tienen un cuerpo blando protegido por la concha de dos valvas, que puede abrir o
cerrar, filtran el agua para alimentarse y su pie tiene forma de hacha para excavar y enterrarse
en la arena. Los cefalópodos, han transformado el pie en tentáculos alrededor de la cabeza, parte
del pie se ha transformado en el sifón, órgano de propulsión por expulsión de agua; su sistema
nervioso y sus ojos están muy desarrollados; poseen un sistema de camuflaje, por cambios de
color de la piel.

9
-Pág.49-
C I E N C I A S D E L A N AT U R A L E Z A

Los artrópodos, son invertebrados que han desarrollados un esqueleto externo articulado.
Parece que proceden evolutivamente de los anélidos pues su sistema nervioso es muy parecido.
Durante su crecimiento sufren mudas. Su reproducción es sexual, con sexos diferenciados y en
muchos casos sufren metamorfosis. Tienen un alto éxito evolutivo ya que un 75% de los ani-
males son artrópodos. Encontramos cuatro grandes grupos, los crustáceos (cigalas, cangrejos,
etc), arácnidos (arañas, escorpiones y ácaros), insectos y miriapodos.

Los crustáceos, respiran por branquias, viven casi todos en el mar, en su cuerpo se distin-
guen dos parte, el cefalotorax (cabeza y tórax fusionado) y abdomen. Muchos de ellos tiene ojos
compuestos pedunculados, son muy conocidos los decapodos (10 patas) y el primer par de patas
se ha convertido en pinzas (cangrejo de río, cigala, etc.).

Los arácnidos, poseen unos apéndices delante de la boca en forma de uña, los quelíceros,
que en las arañas sirven para inocular veneno. El cuerpo aparece dividido en dos partes, cefa-
lotórax y abdomen. En el cefalotórax tienen 6 pares de apéndices articulados, el primero los
quelíceros, el segundo los pedipalpos, especializados en funciones sensoriales y en sujetar ali-
mentos, el resto son patas ambulatorias. Casi todos son de vida terrestre y respiran por sacos
pulmonares y tráqueas (tubos por donde penetra el aire que se ramifican por todo el cuerpo).
Su reproducción es sexual, son sexos separados y se alimentan a través de un órgano bucal
chupador. Las arañas, en concreto, poseen en el abdomen unas glándulas denominadas hile-
ras, por donde segregan hilo de seda que emplean para construir sus nidos, desplazarse o tejer
sus telas de araña para cazar. Los ácaros, son muy pequeños, siendo muchos de ellos de vida
parásita.

Los insectos, son artrópodos con tres pares de patas, por lo que también se les llama hexá-
podos. Su cuerpo consta de tres partes bien definidas, cabeza, tórax y abdomen. Es un grupo
numerosísimo, con una gran cantidad de formas por su enorme capacidad de adaptación. En la
cabeza suele haber un par de ojos compuestos y dos o tres simples, un par de antenas de fun-
ción táctil u olfativa y la boca, especializada en cada caso en un tipo de alimentación (mastica-
dora, el saltamontes; chupadora, mariposa; picadora-chupadora, el mosquito; lamedora, la
abeja; chupadora-succionadora, la mosca). En el tórax se distinguen tres segmentos, con un par
de patas cada uno y un par de alas en el segundo segmento y otro en el tercero. En el abdomen,
se encuentran la mayoría de sus órganos internos. Las alas presentan muchas variaciones, lle-
gando incluso a perderlas. Existen 7 clases de insectos. Himenópteros (abejas y hormigas),
Coleópteros (escarabajos), Ortópteros (saltamontes), Dictiópteros (mantis y cucarachas), Lepi-
dópteros (mariposas), Dípteros (moscas) y Odonatos (libélulas).

Muchas de las especies de insectos sufren metamorfosis. La metamorfosis es un cambio que


se produce en el desarrollo de ciertos animales, por el que la forma de la larva (llamada oruga
en los insectos) sufre un apreciable cambio para convertirse en adulto, el gusano de la seda es
un ejemplo.

10
-Pág.50-
U . D . 6 . - D IVERSIDAD Y UNIDAD DE LOS SERES VIVOS

Oruga Formación de Crisálida parecida Mariposa


la piel nueva a una hoja
de la crisálida

Hebras
de seda

Metamorfosis de una mariposa.

Existen muchos insectos sociales que viven en comunidades, en nidos, que pueden llegar a
ser enormes. Los individuos en estas comunidades se especializan, adquiriendo morfologías
distintas. Hay una o varias reinas, con la capacidad de reproducción, obreras, que son hembras
estériles y los soldados y machos (para fecundar a la hembra y aislar del frío a la comunidad en
el invierno).
Los equinodermos, son animales marinos que suelen poseer placas calcáreas debajo de la
piel de las que salen pequeñas espinas que les dan aspecto rugoso. Poseen simetria radial, es
decir, el cuerpo consta de varias partes iguales que parten desde su centro en las direcciones de
los hipotéticos radios de una circunferencia. Sus sentidos y su forma de reproducción es muy
simple, pueden hacerlo por bipartición y sexualmente. Lo que realmente caracteriza a los equi-
nodermos es el aparato ambulacral, que es un sistema de tubos por el que circula agua de mar,
estos tubos tienen infinidad de prolongaciones que el animal puede sacar por la cara ventral y
les sirven para el movimiento o como ventosas, para abrir las conchas de los moluscos lameli-
branquios, de los que se alimentan. Existen cinco clases: estrellas de mar, ofiuras, que son estre-
llas más finas, erizos de mar, holoturias o pepinos de mar y lirios de mar, que están fijos.

1.3.3.2. Animales vertebrados


Los animales vertebrados, poseen todos ellos un esqueleto interno con columna vertebral,
un cráneo que alberga su cerebro y un sistema nervioso complejo. Los peces, anfibios, reptiles,
aves y mamíferos pertenecen a este grupo.
Todos los peces, viven en el agua, respiran por branquias, tienen el cuerpo cubierto de esca-
mas y se desplazan nadando, sirviéndose de las aletas. Se reproducen sexualmente, pero el esper-
ma fecunda a los óvulos en el agua, a la salida de los óvulos se le denomina desove. Los peces
toman el oxígeno disuelto en el agua por las branquias que son hileras de finísimas laminillas (es
fácil verlas al levantar los opérculos -las agallas- de cualquier pez). Los peces se alimentan de

11
-Pág.51-
C I E N C I A S D E L A N AT U R A L E Z A

otros más pequeños que ellos y los más pequeños comen plancton (que son algas microscópi-
cas y larvas de animales que flotan en el agua). Existen tres tipos de peces: peces sin mandíbu-
la, que son muy primitivos no teniendo ni aletas ni escamas (la lamprea); peces cartilaginosos,
su esqueleto es cartilaginoso, tienen aletas pares y la caudal o cola asimétrica (el tiburón y la
raya) y peces óseos, que tienen la aleta caudal simétrica (salmón).
Los anfibios son vertebrados de temperatura variable y respiración pulmonar. Tienen la piel
húmeda y viscosa, sin pelos ni escamas, y a través de ella también absorben oxígeno (respira-
ción cutánea). Se reproducen en el agua por fecundación externa, y para adquirir la forma adul-
ta y salir a tierra experimentan metamorfosis (pasan de huevo a renacuajos –respiración por
branquias externas– en varios estadios y a rana –respiración por pulmones–). A este grupo per-
tenecen ranas, sapos y salamandras.
Los reptiles son vertebrados de temperatura variable, piel cubierta de escamas, dientes
uniformes y respiración pulmonar. Ponen huevos amniotas, es decir con una capa amniótica
que protege al embrión dentro del huevo, que depositan en tierra y de ellos salen las crias ya
formadas. Casi todos son carnívoros. Los reptiles, dieron lugar, entre otras formas a los dino-
saurios.
Las aves, son animales vertebrados de sangre caliente, bípedos, ya que las extremidades
anteriores han evolucionado para convertirse en alas. Tienen plumas, respiración pulmonar, pico
córneo sin dientes y se reproducen por huevos. Otras características de algunas aves son la fúr-
cula que es un hueso del pecho en forma de horquilla, el buche (las palomas), que es un ensan-
chamiento del conducto digestivo donde almacenan comida antes de digerirla y molleja, que es
un estómago especial que tritura los alimentos.
Para poder volar, poseen un cuerpo aerodinámico y ligero con alas, plumas, algunos huesos
huecos y un sistema respiratorio muy eficiente, con unos compartimentos, los sacos aéreos. Las
plumas les sirven también de aislante térmico. Las aves comen semillas, frutos, peces, insec-
tos...
Los mamíferos son vertebrados de sangre caliente, se caracterizan por la presencia de glán-
dulas mamarias productoras de leche y tienen el cuerpo cubierto de pelo que les sirve de ais-
lante y protector.
El éxito evolutivo de los mamíferos se debe al mantenimiento de la temperatura constante y
a la lactancia que establece relaciones en el cuidado de las crias más allá del nacimiento, con
relaciones de nutrición y aprendizaje. Estos vínculos entre la madre y su prole son la base del
elevado grado de inteligencia que pueden llegar a desarrollar.
Existen mamíferos acuáticos, con muy poco pelo, como el delfín o la ballena y mamíferos
voladores como el murciélago. Los mamíferos se dividen en tres grupos: monotremas, marsu-
piales y mamíferos placentarios.
Los mamíferos monotremas, ponen huevos. Entre ellos se encuentra el ornitorrinco.

12
-Pág.52-
U . D . 6 . - D IVERSIDAD Y UNIDAD DE LOS SERES VIVOS

Los marsupiales, tienen una bolsa ventral, el marsupio, donde transportan a sus crías, que
nacen precozmente y les dan de mamar. Son herbívoros. Entre ellos se encuentra el canguro.
Los mamíferos placentarios, son los más evolucionados. Sus crías no nacen hasta estar com-
pletamente formadas y se desarrollan dentro del útero materno gracias a la placenta, que, a tra-
vés del cordón umbilical, les proporciona alimento y elimina desechos.
Existen adaptaciones en cada caso según su forma de alimentación y el ecosistema en el que
vivan.
A lo largo de la evolución se han producido, en el aparato circulatorio de los vertebrados, y
en concreto en el corazón, diversas adaptaciones como podemos ver en la imagen adjunta; estas
mejoras permiten un mejor sistema de bombeo:
— Peces: Una aurícula y un ventrículo
— Anfibios: Dos aurículos y un ventrículo
— Reptiles: Dos aurículos y un ventrículo parcialmente dividido
— Aves y Mamíferos: Dos aurículos y dos ventrículos perfectamente divididos

Seno
venoso Pulmones
Branquias Aurícula
derecha Aurícula
Aurícula izquierda
Corazón
Aurícula Aurícula
derecha izquierda
Ventrículo Tejidos Ventrículo Ventrículo
Tejidos

ANFIBIOS
PECES

Pulmones
Pulmones

Aurícula Aurícula
Aurícula Aurícula Aurícula izquierda Arco aórtico
izquierda Aurícula
derecha derecha izquierda derecha Aurícula
derecha
Aurícula
izquierda
Ventrículo Ventrículo
derecho izquierda
Ventrículo Ventrículo Ventrículo Ventrículo Ventrículo
Ventrículo izquierdo derecho izquierda derecho izquierdo
derecho Tejidos
Tejidos
AVES Y
REPTILES
MAMÍFEROS

Esquema de los aparatos circulatorios de los vertebrados.

13
-Pág.53-
C I E N C I A S D E L A N AT U R A L E Z A

1.3.4. EL REINO FUNGI


En el reino fungi, están incluidos los hongos. Los hongos y las plantas se parecen porque
sus células están recubiertas externamente por celulosa; la gran diferencia entre hongos y plan-
tas es que los hongos no generan sus alimentos a partir de la energía solar. Los hongos se nutren
de restos de animales y plantas y producen su descomposición, desempeñando un papel funda-
mental en la naturaleza.

Muchos hongos son unicelulares y solo son visibles al microscopio, como es el caso de las
levaduras, que produce la fermentación del pan, de la cerveza, del vino, ... Existen hongos pará-
sitos, que causan enfermedades (el pie de atleta) y plagas (el cornezuelo del centeno) y otros
que el hombre aprovecha en su beneficio como distintas especies de Penicillium, de unas se
obtiene la penicilina y a partir de ella, distintos antibióticos, otros se emplean en la fabricación
de quesos azules (Roquefort, Cabrales, etc.).

Otros son pluricelulares y se conocen coloquialmente con el nombre de setas. En este caso,
el hongo auténtico, son una especie de pelillos, a modo de raicillas que se encuentran bajo tie-
rra en el bosque, denominado micelio, en el momento apropiado se produce la fecundación que
da lugar a un cuerpo fructífero (la seta) que produce gran cantidad de esporas. En el cuerpo
vegetativo (la seta), se reconocen tres partes, el sombrero, bajo el cual se encuentran unas lami-
nillas donde proliferan las esporas, el pie y alrededor de éste, el anillo y bajo el pie, en la tierra,
a modo de raices, el micelio.

1.3.5. EL REINO VEGETAL

Todo el reino vegetal se caracteriza por realizar la función clorofílica. Para ello, sus células
contienen cloroplastos donde existe una sustancia, la clorofila, que posee la característica de
excitarse con la luz del sol, y generan así una energía que permite que a partir del dióxido de
carbono del aire y el agua que absorbe, generar materia orgánica (glucosa) y como subproduc-
to oxígeno.

El reino vegetal se divide en algas, musgos, líquenes, helechos y espermafitas o plantas


superiores.

Las algas, son plantas que no tienen raíz, tallo, hojas ni flores, pero tienen clorofila y reali-
zan la fotosíntesis. Viven en el agua, tanto dulce como salada. En vez de raíz, tienen un órgano
de fijación, que no penetra en el suelo. Muchas algas poseen unas ampollas llenas de aire para
flotar (ej.: Fucus). Las algas marinas poseen además de la clorofila otros pigmentos y así se cla-
sifican en algas rojas, verdes y pardas

Las algas marinas unicelulares, flotan a la deriva y forman parte del plancton marino.

14
-Pág.54-
U . D . 6 . - D IVERSIDAD Y UNIDAD DE LOS SERES VIVOS

Los musgos (Briófitas), desarrollan su vida entre el agua y la tierra; fueron los primeros
organismos fotosintéticos que colonizaron el medio terrestre, pero apenas tienen raíz y la forma
de transportar el agua hasta el resto de la planta es muy rudimentaria y poco eficaz. Tienen un
ciclo biológico en el que se alternan la reproducción sexual y la reproducción por esporas. Han
de vivir en lugares húmedos para proteger sus células sexuales (gametos) de la desecación, por
lo que su reproducción se realiza en el agua, aunque les basta una simple gota de agua. Poseen
un sistema de defensa por el que cuando las condiciones les son adversas, se aletargan, para
revivir cuando llegan las lluvias.

Los líquenes, son asociaciones de un hongo y un alga. La asociación en la que los dos orga-
nismo se benefician se denomina simbiosis. El alga crece en el interior del hongo y le propor-
ciona azúcares. A la vez, el alga se desarrolla bajo la protección del hongo que le suministra
agua. La simbiosis les ha permitido a los líquenes sobrevivir en medios tan austeros como la
superficie de una piedra, donde por separado sería imposible su subsistencia.

Los helechos (Pteridófitas), al igual que las plantas superiores realizan la fotosíntesis, y tie-
nen raíz, tallo y hojas, pero no tienen semillas, se reproducen por esporas que se sitúan en la
parte posterior de las hojas. La reproducción por esporas tiene dos etapas diferenciadas (algo
parecido a lo que les pasaba a las medusas), las esporas se encuentran en el helecho, que reci-
be el nombre de esporófito. Cuando las esporas maduran, se diseminan. Si germinan, dan lugar
a una pequeña masa de células microscópicas que constituyen la otra generación, el gametófi-
to. Éste genera los gametos masculino y femenino, si el suelo está encharcado, el gameto mas-
culino nada y fecunda al femenino. Entonces, a partir del huevo fecundado o zigoto, se des-
arrolla un nuevo esporófito, que es el helecho tal y como lo vemos.

En las plantas superiores (Espermafitas), cada parte cumple una función específica: la
raíz absorbe agua y minerales; el tallo, transporta y almacena alimentos; la hoja los fabrica; las
flores sirven para la reproducción; los frutos y semillas, para la formación de nuevas plantas.

15
-Pág.55-
C I E N C I A S D E L A N AT U R A L E Z A

TRANSPIRACIÓN EXCRECIÓN
H2O CO2 O2
CO2
O2

CO2
H2O
Glucosa
Glucosa

Mitocondria Cloroplastos

Savia bruta por Célula


ATP los vasos leñosos FOTOSÍNTESIS
Célula
TRANSPORTE
RESPIRACIÓN
Savia elaborada
por los vasos liberianos
CO2 ATP

O2
Glucosa

Célula

H2O RESPIRACIÓN

ABSORCIÓN
Sales
minerales

Esquema de los diferentes procesos de la nutrición vegetal.

La raíz, es la parte subterránea de la planta, la fija sólidamente al suelo, de donde extrae


agua y minerales. La raíz se ramifica para ocupar el mayor terreno posible y asegurarse el espa-
cio suficiente para absorber el agua y los minerales necesarios. A lo largo de la raíz podemos
distinguir cuatro partes: zona de ramificación, zona pilífera, zona de crecimiento y zona termi-
nal. El crecimiento en longitud se produce gracias a la existencia de un meristemo que se
encuentra en la zona de crecimiento o ápice. El extremo de la raíz, la pilorriza, protege las célu-
las del meristemo apical que se multiplican rápidamente. En la zona pilífera aparecen los pelos
absorbentes para aumentar la superficie de absorción y cerca de la superficie, se desarrollan las
raíces secundarias.

16
-Pág.56-
U . D . 6 . - D IVERSIDAD Y UNIDAD DE LOS SERES VIVOS

Existen distintos tipos de raices, axonomorfas (se distingue una raíz principal con ramifica-
ciones como raíces secundarias), fasciculadas (cuando no existe ninguna rama principal), napi-
forme (almacena sustancias de reserva) y tuberosa (con engrosamientos a lo largo de las raices).

El tallo, es la parte de la planta que sostiene las partes aéreas de la planta y a la vez esta-
blece la relación entre la raíz y las hojas. Sólo el extremo superior del tallo tiene capacidad de
crecer para elevarse. En los tallos leñosos, bajo la corteza están los vasos liberianos (el floema),
por ellos desciende la savia elaborada, más en el interior se encuentran los vasos leñosos (xile-
ma) por donde asciende el agua y las sales minerales.

Las hojas, es el lugar de la planta donde tiene lugar la fotosíntesis para fabricar sustancias
orgánicas, son expansiones planas, casi siempre verdes, que nacen del tallo y de sus ramifica-
ciones. En una hoja se distinguen tres partes: limbo (es la parte con forma plana), dividido en
el haz (la parte superior) y la inferior, envés; peciolo o rabito, que une el limbo al tallo y la vaina
o parte final del peciolo, que se ensancha en la zona de conexión con el tallo.

Las hojas de las plantas se disponen para recibir la mayor cantidad posible de luz solar. En
su superficie aparecen unas aberturas, los estomas, que intercambian gases, de la fotosíntesis
y el vapor de agua de la transpiración de la planta (para regular la temperatura interna de la
planta).

Las hojas pueden mantenerse durante todo el año (hoja perenne), protegidas por capas cé-
reas para evitar el ataque de insectos y orugas, heladas, granizo, etc., o renovarlas cada prima-
vera (hoja caduca), en este caso las hojas no están tan protegidas.

En algunas plantas tienen formas especiales, las coníferas tienen forma aciculada, para evi-
tar la pérdida innecesaria de agua; los cáctus, han convertido sus hojas en espinas y así redu-
cen al máximo la transpiración; en ellos, el tronco se encarga de realizar la fotosíntesis.

Las flores, son los órganos reproductores de las plantas espermafitas. En la estructura de la
flor se distinguen:
-Los órganos masculinos de la flor, llamados estambres, que generan diminutos granos
de pólen, en cuyo interior se encuentran los gametos masculinos.
-El órgano femenino de la flor llamado carpelo. Consta de tres estructuras: el ovario, que
protege al óvulo y en cuyo interior se encuentra el gameto femenino; el estilo y el
estigma, en cuya superficie se posan los granos de polen.
-Pétalos (corola) y sépalos (cáliz), que son hojas modificadas dispuestas alrededor de los
estambres y el carpelo. Los pétalos tienen vivos colores.

17
-Pág.57-
C I E N C I A S D E L A N AT U R A L E Z A

Hay plantas que tienen flores de un solo sexo, masculino o femenino, en distinto pié y con
características físicas distintas, como el chopo. Otras poseen flores masculinas y femeninas en
distinta parte de la misma planta, como el maíz. También hay plantas cuyas flores poseen ambos
sexos.

Para producirse la fecundación, el grano de polen se ha de posar sobre el estigma, que emite
un tubo, llamado polínico, que recorre el estilo, alcanza al óvulo, y descarga en su interior el
gameto masculino, que fecunda al gameto femenino.

El zigoto resultante se transforma en embrión o germen de la semilla, que dará lugar a una
nueva planta cuando germine.

Por lo general los flores no son fecundadas por su propio polen, sino con el de otras plantas
de la misma especie. La polinización es el transporte del polen de una flor a otra a través del
aire o con la ayuda de un animal. Para asegurarse su reproducción, cuando son los insectos los
encargados, especialmente las abejas, la planta las atraen con el néctar (es una sustancia azuca-
rada que segrega la planta). Las abejas, al intentar libarlo se van pegando los granos de polen al
abdomen y al ir de flor en flor, los van entrecruzando e intercambiando (polinización cruzada).

La semilla es el óvulo fecundado que contiene el germen o embrión desarrollado a partir del
zigoto y reservas nutritivas; todo ello protegido por cubiertas resistentes. Las reservas nutritivas
se almacenan en unas estructuras llamados cotiledones; uno en las plantas monocotiledoneas y
dos en las dicotiledoneas. Las semillas contienen muy poco agua y están especialmente diseña-
das para resistir largas temporadas sin germinar.

La evolución de las plantas ha encontrado distintos caminos para favorecer la dispersión de


las semillas: semillas aladas (del sicomoro), trasladadas por el viento; semillas flotantes (de los
cocoteros), para viajar y colonizar otras costas; semillas con ganchos, para fijarse en los pelos
de los animales, etc. Sin embargo, el método más perfeccionado es el fruto. El fruto es el ova-
rio, que tras haber sido fecundado, engrosa al llenarse de nutrientes y en su interior contiene las
semillas.

Según como se agrupen los órganos reproductores en las plantas, las podemos dividir en dos
grupos: Gimnospermas y Angiospermas.

Las gimnospermas, son las plantas espermáfitas más antiguas. Sus órganos reproductores
femeninos están al desnudo y tienen forma de piña. La polinización y la dispersión de las semi-
llas se realiza por medio del viento. Las gimnospermas más abundantes son las coníferas (pinos,
abetos, cipreses...). Sus hojas tiene forma de agujas o escamas y están adaptadas a climas extre-
mos. Son de hoja perenne.

Las angiospermas son las plantas espermáfitas más evolucionadas; sus órganos reproducto-
res se agrupan en flores vistosas polinizadas por el viento y por animales, fundamentalmente
insectos.

18
-Pág.58-
U . D . 6 . - D IVERSIDAD Y UNIDAD DE LOS SERES VIVOS

Según sus semillas tengan uno o dos cotiledones se clasifican en monocotiledoneas y dico-
tiledoneas.

-Las monocotiledóneas, suelen ser de hojas alargadas y nervios paralelos. En sus tallos,
los vasos transportadores del agua y los nutrientes, están distribuidos por todo el tallo
sin formar anillos. Ejemplo de monocotiledoneas son los cereales y algunos árboles
como la palmera.

-Las dicotiledóneas tienen nervios en sus hojas en forma de red. Los vasos transporta-
dores de savia tienen anillos concéntricos. Pueden ser arbóreas y herbáceas. El tomate,
la vid, el rosal y casi todos los árboles con flor.

1.4. PRESENCIA DE ANIMALES Y VEGETALES EN LA VIDA


COTIDIANA

Desde la antigüedad, el hombre aprendió a domesticar a los animales para su compañía y su


uso (guardar a los ganados, como medio de transporte, ...) y para su alimentación.

Son muchos los animales que el hombre ha utilizado y ha aprendido a criar en granjas (galli-
nas, vacas, peces, ...).

La contrapartida está en la fauna que se encuentra en peligro. El comercio internacional de


animales exóticos, la caza, la pesca abusiva, más la contaminación, destrucción e invasión de
sus territorios por las poblaciones humanas, son algunas de las causas que provocan la desapa-
rición de la fauna. Animales en peligro en España son el lobo, el oso pardo, el águila imperial,
el lince ibérico, la malvasía (ave acuática) y la foca monje.

Para la flora, la sobreexplotación y la excesiva roturación de terrenos, la tala de bosques, la


recolección excesiva de ciertas especies, los incendios, el drenaje de las zonas húmedas, el cre-
cimiento urbano, ..., son las causas principales de la desertización de los suelos y la extinción
de las especies vegetales.

Debemos avanzar en la protección del medio natural, como única forma de salvaguardar el
equilibrio y mantener la biodiversidad.

19
-Pág.59-
C I E N C I A S D E L A N AT U R A L E Z A

LA CÉLULA COMO UNIDAD DE ESTRUCTURA


2. DE LOS SERES VIVOS. ORGANIZACIÓN CELULAR

2.1. LA CÉLULA COMO UNIDAD DE ESTRUCTURA DE LOS SERES


VIVOS
A partir de 1850, con los avances y descubrimientos se emitió la teoría celular, que se puede
resumir en las siguientes ideas:
-Todos los organismos vivos se componen de una o varias células.
-La célula es la unidad estructural y funcional de los seres vivos, es decir, la estructura
más simple dotada de vida propia y con capacidad para realizar las funciones vitales.
-Todas las células proceden de otras preexistentes.
Teniendo en cuenta la complejidad estructural de las células, podemos clasificarlas en dos
grupos:
-Células procariotas. Son células primitivas, relativamente simples y carentes de un
núcleo definido.
-Células eucariotas. Su estructura es más compleja que la de las anteriores y posee un
núcleo definido.
Existen seres constituidos por una sola célula, son los seres unicelulares, y otros por
muchas, los pluricelulares. Entre los primeros están las bacterias, pero la inmensa mayoría de
los seres vivos son pluricelulares.

2.1.1. CÉLULA EUCARIOTA


Presentan una gran variedad de formas y tamaños. La célula tipo animal o vegetal (se entien-
de que eucariota) consta de:
-Membrana: Estructura exterior que la limita.
-Núcleo: Orgánulo central que dirige las funciones celulares.
-Citoplasma: Porción comprendida entre la membrana y el núcleo.

20
-Pág.60-
U . D . 6 . - D IVERSIDAD Y UNIDAD DE LOS SERES VIVOS

La membrana plasmática, no es tan sólo una cubierta, sino que su estructura continúa por
el interior de la célula. Sus funciones principales son:
-Limita a la célula y la separa del medio o de otras células. Así mismo le da su forma
característica.
-Regula la entrada de nutrientes y la salida de los productos residuales de la célula.
En las células vegetales existe además otra membrana rígida y exterior, formada por celu-
losa y denominada pared celular.
El citoplasma, no es homogéneo, sino que en él se encuentran gran cantidad de corpúscu-
los llamados orgánulos citoplasmáticos, todos ellos inmersos en una sustancia fundamental lla-
mada hialoplasma en las células animales y citosol en las vegetales.
Los principales orgánulos que podemos encontrar habitualmente en el citoplasma son:
-Retículo endoplasmático. Es un conjunto de membranas que limitan cavidades comu-
nicadas entre sí, con el exterior y con el núcleo. Las funciones del retículo endoplas-
mático son:
-Canal de comunicación que permite el transporte de sustancias.
-Esqueleto o armazón de la célula.
-Almacén de sustancias, elaboradas o no por la propia célula.
-Ribosomas. Gránulos pequeñísimos que intervienen en la síntesis (fabricación) de pro-
teínas de la célula. Se pueden encontrar unidos al retículo endoplasmático (entonces se
conoce como retículo endoplasmático rugoso) o libres en el citoplasma.
-Aparato de Golgi. Está formado por dictiosomas, que son apilamientos de sáculos,
constituidos por membranas unitarias, cuyo contenido es una mezcla de glúcidos y pro-
teínas. Su función es la de segregar sustancias.
-Mitocondrias. Suelen ser alargadas y están formadas por dos membranas mitocon-
driales, una externa y otra interna. Ésta última emite unas prolongaciones hacia el inte-
rior de la mitocondria, llamadas crestas mitocondriales. El espacio hueco del interior o
matriz está ocupado por un líquido formado por diversas sustancias. Las mitocondrias
son los centros respiratorios de la célula.

21
-Pág.61-
C I E N C I A S D E L A N AT U R A L E Z A

(a)
Núcleoplasma
Ribosomas
Vacuolas

Nucléolo

Cromatina
Membrana
nuclear
con poros
Retículo
endoplasmático
liso
Retículo
endoplasmático
rugoso
Membrana
plasmática

Hialoplasma

Mitocondria
Núcleo Aparato de Golgi
Centriolos

(b)
Núcleoplasma Membrana
nuclear Pared celular
Mitocondria

Cromatina

Nucléolo
Cloroplasto

Núcleo
Aparato de Golgi

Retículo
endoplasmático
rugoso

Retículo
endoplasmático
Ribosomas
liso
Membrana
Vacuola Hialoplasma plasmática

Estructura de: a) una célula eucariótica animal; b) una célula eucariótica vegetal.

22
-Pág.62-
U . D . 6 . - D IVERSIDAD Y UNIDAD DE LOS SERES VIVOS

-Plastos. Son orgánulos propios de las células vegetales. Aunque pueden ser de diversas
formas, los más frecuentes son esferoidales u ovoides. Contienen sustancias general-
mente coloreadas y los más abundantes son los cloroplastos, llamados así por contener
el pigmento verde de las plantas denominado clorofila. El papel de los cloroplastos es
realizar la función fotosintética.
-Centriolos. Son un par de corpúsculos que estudiados con el microscopio electrónico,
aparecen formados por un par de cilindros perpendiculares entre sí. Son muy impor-
tantes en la división celular. A partir de ellos, también se forman cilios y flagelos, que
son como pelitos que al moverse, permiten el desplazamiento.
-Lisosomas. Son orgánulos formados por una membrana que encierra gran cantidad de
sustancias productos de la digestión celular y los enzimas digestivos.
-Vacuolas. Son bolsitas, separadas por una membrana, que acumulan diversas sustan-
cias. En las células vegetales alcanzan gran desarrollo.
El núcleo, dirige las funciones celulares. Cada célula suele tener un núcleo, aunque algunas
células pueden tener 2 o más. Está formado por los siguientes elementos:
-Membrana nuclear. Constituida por dos capas, igual que la membrana plasmática.
Posee poros a través de los cuales el núcleo intercambia sustancias con el citoplasma.
-Cromatina. Sustancia formada por proteínas y ácidos nucleicos. Se visualizan como
cromosomas durante la división celular.
-Nucleolo. Uno o dos corpúsculos, formados por ácido ribonucleico.
-Plasma nuclear o nucleoplasma. Líquido formado por agua y muy diversas sustan-
cias.
Como quedó establecido en la Teoría Celular, todas las células provienen de otras anterio-
res, por división de éstas. Esta división celular, llamada mitosis, requiere de la división tanto
del núcleo, como del citoplasma y de la membrana.
Cuando el núcleo se va a dividir, adquiere un aspecto distinto: la cromatina se va conden-
sando y se forman unos filamentos independientes, llamados cromosomas. Los cromosomas
tienen gran importancia, ya que los caracteres hereditarios dependen de su constitución y con-
tenido.
Las células eucariotas, difieren mucho entre sí, según sean animales o vegetales. Las prin-
cipales diferencias se pueden visualizar en el siguiente cuadro:

23
-Pág.63-
C I E N C I A S D E L A N AT U R A L E Z A

CÉLULA ANIMAL CÉLULA VEGETAL


Sólo membrana plasmática Membrana Plasmática y pared celular.
Ausencia de plastos Presencia de plastos
Presencia de centriolos Ausencia de centriolos
Pocas o ninguna vacuola Gran número de vacuolas

2.1.2. CÉLULA PROCARIOTA


Además de la organización eucariota, que acabamos de exponer, existe otra organización, la
procariota. Las células procariotas fueron las primeras en aparecer sobre la Tierra, hace unos
3.000 millones de años. En la actualidad las bacterias y las algas verde-azuladas presentan esta
estructura. Las células procariotas, presentan pequeño tamaño, agrupándose a veces en colonias.
Los organismos procariotas, son organizaciones simples que:
-Carecen de núcleo diferenciado, pues su material genético, formado por un filamento
circular, se encuentra inmerso en el citoplasma.
-No tienen orgánulos, excepto los ribosomas, algunos acúmulos de sustancias de reser-
va y hialoplasma.
-La membrana plasmática posee invaginaciones en determinadas zonas, denominadas
mesosomas. En ellos se realiza el proceso de la respiración celular o la fotosíntesis.
-Las células procariotas presentan una pared celular exterior, de diferente composición
que la pared vegetal. Algunas pueden tener una cápsula gelatinosa en el exterior.
-En algunos casos aparecen flagelos que les permite el movimiento de distinta estructu-
ra que los de los eucariotas.

2.2. ORGANIZACIÓN UNICELULAR Y PLURICELULAR


Existen organismos que poseen una sola célula eucariota, se les denomina seres unicelula-
res. Estas células, no son células especializadas con funciones específicas, sino que desarrollan
todas las funciones de los seres vivos: nutrición, relación y reproducción.
Los seres pluricelulares están formados por muchas células. Esta organización, más com-
pleja, exige una especialización celular. Las asociaciones de células que tienen estructura y fun-
ción similares se denominan tejidos. Estos tejidos serán diferentes en plantas y animales. Esta
organización ofrece la ventaja de reparto del trabajo y la especialización, con lo que se aumen-
ta la eficacia y se obtienen mejores resultados. Los tejidos especializados, se asocian en órga-
nos (estómago, pulmones,...). Éstos a su vez, se reúnen para formar aparatos y sistemas

24
-Pág.64-
U . D . 6 . - D IVERSIDAD Y UNIDAD DE LOS SERES VIVOS

(locomotor, circulatorio, digestivo,...). Todos ellos constituyen el organismo vivo, y se encargan


de llevar a cabo las funciones básicas que mantienen la vida: la nutrición, la relación y la repro-
ducción.

2.2.1. TEJIDOS VEGETALES


Sólo en las plantas superiores, dotadas de raíz, tallo y hojas, hay verdaderos tejidos. Entre
ellos se debe distinguir los embrionarios, constituidos por células indiferenciadas, capaces de
dividirse y los adultos, cuyas células se han especializado en una determinada tarea y, en con-
secuencia, han perdido su capacidad de reproducción.

2.2.1.1. Tejidos embrionarios: Meristemos


Intervienen en el crecimiento de la planta, ya que sus células son las únicas que conservan
su capacidad reproductora. Existen meristemos primarios y secundarios. Los primarios se loca-
lizan en los extremos del tallo y de la raíz, así como en las yemas y son los responsables del cre-
cimiento en longitud de la planta. Están formados por pequeñas células redondeadas o polié-
dricas unidas entre sí sin dejar espacios vacíos. Los meristemos secundarios, son los
responsables del crecimiento en grosor, son prismáticas y más alargadas y se sitúan entre otros
tejidos adultos del tallo y la raíz.

2.2.1.2. Tejidos vegetales adultos


Según su función, los tejidos adultos se clasifican en: protectores, parenquimáticos, con-
ductores, de sostén y secretores.
a) Tejidos protectores. Recubren exteriormente a la planta protegiéndola de la desecación
y de las infecciones. Hay dos clases, la epidermis y el suber.
-La epidermis. Se localiza en las hojas y en los tallos jóvenes y herbáceos. Está consti-
tuida por una sola capa de células, sin cloroplastos, unidas entre sí e impermeabiliza-
das por su parte externa. De vez en cuando hay poros, los estomas, que se pueden abrir
y cerrar. Permite el paso de la luz por ser transparente.
-El súber. Se conoce como corcho. Recubre los tallos leñosos y las raíces. Lo forman
varias capas de células muertas, engrosadas e impermeabilizadas. Para permitir el inter-
cambio de gases se abren las lenticelas (aparentan ser desgarrones), donde el tejido se
interrumpe.
b) Tejidos parenquimáticos. Constituyen la mayor parte del vegetal y se ocupan de funcio-
nes muy variadas:
-Clorofílico. En el interior de las hojas y tallos verdes. Son células redondeadas carga-
das de cloroplastos, unidas entre sí y con espacios huecos para facilitar el intercambio
de gases. Realizan la fotosíntesis para toda la planta.

25
-Pág.65-
C I E N C I A S D E L A N AT U R A L E Z A

-De reserva. En el interior de tallos y raíces, semillas y órganos de reserva como bulbos
o tubérculos, etc. Acumulan sustancias producidas por la fotosíntesis y que no se han
consumido.
-Aerífero. Característico de plantas acuáticas, para mantenerse flotando.
-Acuífero. Almacena agua en plantas adaptadas a la sequedad.

c) Tejidos conductores. Forman un sistema de tubos que recorre el vegetal transportando


sustancias. Existen dos tipos, los vasos leñosos y los vasos liberianos.
-Los vasos leñosos, o xilema, transportan la savia bruta (agua y sales minerales) desde
la raíz hasta las hojas. Constituido por células muertas, huecas, cuyas paredes tienen
estructuras circulares que les proporcionan gran rigidez.
-Los vasos liberianos o floema, conducen la savia elaborada (sustancias orgánicas de la
fotosíntesis) desde las hojas al resto. Son células vivas y entre ellas existen unos tabi-
ques perforados, las placas cibosas. Los vasos liberianos se encuentran por fuera de los
vasos leñosos.

d) Tejidos de sostén. Dan consistencia y solidez a la planta. Existen dos tipos de tejido de
sostén: el colénquima y el esclerénquima.
-Colénquima. Células vivas muy engrosadas situadas en hojas y tallos herbáceos.
-Esclerénquima. Células muertas de paredes engrosadas y endurecidas, pueden tener
forma alargada (cáñamo, lino y en general fibras textiles) o redondeada (huesos de fru-
tas).

e) Tejidos secretores. Se encargan de la producción de determinadas sustancias, sean de


desecho o útiles para la planta.

2.2.2. TEJIDOS ANIMALES


Los tejidos animales tienen un grado de especialización mayor que los vegetales, además
entre sus células suele existir una sustancia intercelular solida o líquida más o menos abundan-
te. Otra característica es que en la mayoría, las células poseen la capacidad de dividirse y rege-
nerar a las dañadas.
Los tejidos animales los vamos a clasificar en cuatro grupos: epitelial, conectivo, muscular
y nervioso.
a) Tejido epitelial. Las células se unen sin sustancia intercelular. Existen dos tipos: de
revestimiento y epitelio glandular.
-De revestimiento. Recubre y protege el exterior del cuerpo y el interior de las cavida-
des. Según la forma, tenemos: epitelio plano, tapiza los vasos sanguíneos; epitelio cúbi-
co, en los conductos de salida de muchas glándulas y epitelio prismático, en el interior

26
-Pág.66-
U . D . 6 . - D IVERSIDAD Y UNIDAD DE LOS SERES VIVOS

del tubo digestivo. Además existen adaptaciones a funciones concretas, así, las células
que recubre el interior del aparato respiratorio, poseen cilios para eliminar las partícu-
las extrañas, las que tapizan la pared interna del intestino tienen muchos repliegues
(microvellosidades) con el fin de aumentar su superficie sin que lo haga su volumen,...
Por otra parte, según el número de capas celulares que constituyen el tejido epitelial,
cabe diferenciar el epitelio simple (una sola capa de células que tapizan las cavidades)
del epitelio estratificado (comprende varias capas como es el caso de la epidermis)
-Epitelio glandular. Sus células están especializadas en la producción de determinada
sustancia y se unen entre sí para constituir una glándula.
b) Tejido conectivo. Se ocupa de conectar unos tejidos con otros para formar órganos, aun-
que también pueden actuar como tejidos de sostén y de protección. Básicamente están
compuestos por células y abundante sustancia intercelular rica en fibra. Las fibras pue-
den ser colágenas, (resistentes y duras) y elásticas, que conforman redes muy flexibles.
Entre los tejidos conectivos se han de mencionar el conjuntivo, el cartilaginoso y el
óseo.
- Tejido conjuntivo. Están constituidos por dos tipos de células, unas fijas, con forma de
estrella o alargadas y otras móviles con capacidad para fagocitar. Entre la sustancia
intercelular hay muchas fibras colágenas y elásticas. Existen tres variedades de tejido
conjuntivo. El tejido conjuntivo laxo, con sustancia intercelular semilíquida, se locali-
za bajo la piel y rodeando los vasos sanguíneos, nervios, músculos, huesos y órganos.
El tejido conjuntivo denso, con gran cantidad de fibras colágenas, es muy resistente y
constituye los tendones y ligamentos. Tejido conjuntivo adiposo, caracterizado por la
presencia de células redondeadas encargadas de acumular grasa, debajo de la piel y
rodeando ciertos órganos como el corazón y riñones.
- Tejido cartilaginoso. La sustancia intercelular es gelatinosa, rica en fibras colágenas y
elásticas. Es muy resistente y forma el esqueleto de algunos peces como el tiburón.
- Tejido óseo. La sustancia intercelular es sólida, impregnada de sales de calcio. Esta
matríz se dispone en capas concéntricas, entre las que quedan huecos en los que se
sitúan las células con forma estrellada y comunicadas por vasos sanguíneos a través de
canalillos. Existe el tejido óseo compacto, que se sitúa en la caña de los huesos largos
y el tejido óseo esponjoso, muy poroso, que se sitúa en el extremo de los huesos lar-
gos y el interior de los cortos.
c) Tejido muscular. Constituido por fibras musculares que son alargadas y en su interior
hay muchas fibrillas, gracias a las cuales se contraen y se estiran. Existen tres clases de
tejido muscular: el estriado, el liso y el cardiaco. El estriado, son fibras cilíndricas plu-
rinucleadas, que se organizan a modo de bandas permitiendo la contracción voluntaria.
El tejido muscular liso, está formado por fibras en forma de huso, con un único núcleo,
sin estriaciones, su contracción es involuntaria y se localiza en las paredes de órganos

27
-Pág.67-
C I E N C I A S D E L A N AT U R A L E Z A

tales como la vejiga urinaria, el tubo digestivo y los vasos sanguíneos. El tejido cardía-
co, forma la masas del corazón, sus fibras son estriadas, aunque presentan un único
núcleo central, se contraen de forma rápida e involuntaria.

d) Tejido nervioso. Es un tejido muy especializado en la transmisión de impulsos nervio-


sos. Sus células se denominan neuronas. Constan de un cuerpo celular del que parten
prolongaciones. Hacia un lado, las dendritas, cortas, por donde entra el impulso y es
transmitido al cuerpo celular y el axón (prolongación larga y única por donde sale el
impulso hasta la siguiente neurona). Las neuronas se agrupan entre sí dando lugar a
centros nerviosos (agrupaciones de cuerpos neuronales) y a los nervios (agrupación de
axones).

2.3. PRESENCIA EN LA VIDA COTIDIANA DE VIRUS Y BACTERIAS

Los virus son la forma más simple de vida. Solo se pueden observar con el microscopio
electrónico. Son formas especiales de vida, ya que no realizan todas las funciones vitales. Fun-
damentalmente, son formas capaces de reproducirse, y parasitan a todos los seres vivos.

Los virus están formados por una cubierta de proteínas llamada cápsida, en cuyo interior se
encuentra el ácido nucleico, portador de la información genética. Tienen diversas formas: esfé-
ricos, poliédricos, cilíndricos, etc. Según el ser que parasitan pueden ser: bacteriofagos, si para-
sitan bacterias; virus vegetales, si parasitan plantas y virus animales, si producen enfermedades
a los animales (glosopeda al ganado vacuno, la peste porcina a los cerdos, etc.)

De entre los virus animales cabe destacar aquellos que infectan al hombre y le causan enfer-
medades como el sarampión, la gripe y el terrible SIDA.

Las bacterias, constituyen el grupo más importante de organismos con organización proca-
riota. Tienen una gran variedad y presentan distintos tipos de vida. Unas son autótrofas, es
decir, realizan la función clorofílica, y gracias a la clorofila y a la luz solar, fabrican su propio
alimento. Otras son heterótrofas, es decir, han de conseguir el alimento a expensas de otros
seres vivos. Algunas son patógenas, es decir, causan enfermedades a organismos superiores
(cólera, difteria, meningitis, tifus, tuberculosis...) y otras son saprófitas, que viven sobre la
materia orgánica en descomposición o transforman la leche en derivados (fermentación lácti-
ca) o el vino en vinagre (fermentación acética). Por su forma pueden ser cocos, de forma esfé-
rica, que suelen asociarse formando cadenas (estafilococos), racimos (estreptococos), grupos de
dos (diplococos) ..., bacilos, de forma alargada, vibrios, con forma de coma y espirilos, en
forma de espiral.

28
-Pág.68-
U . D . 6 . - D IVERSIDAD Y UNIDAD DE LOS SERES VIVOS

LAS FUNCIONES DE LOS SERES VIVOS: NUTRICIÓN,


3. RELACIÓN Y REPRODUCCIÓN.
EL SER VIVO COMO SISTEMA

3.1. LAS FUNCIONES DE LOS SERES VIVOS: NUTRICIÓN,


RELACIÓN Y REPRODUCCIÓN
La célula es el organismo vivo más pequeño que existe y como tal, realiza las tres funcio-
nes vitales: nutrición, reproducción y relación. A nivel del organismo completo, estas funciones
se verán en la Unidad 7.

3.1.1. NUTRICIÓN AUTÓTROFA Y HETERÓTROFA


Antes de explicar las diferencias entre estos dos tipos de nutrición es conveniente explicar
cual es la diferencia entre materia orgánica e inorgánica. La materia orgánica está constituida
por carbono, hidrógeno, oxígeno y nitrógeno, mientras que la inorgánica, puede estar compuesta
por cualquier elemento. Además las moléculas orgánicas son más grandes y cuentan con
muchos enlaces entre sus átomos, precisamente, es en estos enlaces donde se almacena la ener-
gía química, que será por tanto mayor en los compuestos orgánicos.
La nutrición celular puede ser de dos tipos:
- Nutrición autótrofa.
- Nutrición heterótrofa.
La nutrición autótrofa la realizan las células que son capaces de elaborar su propio ali-
mento, es decir, de transformar la materia inorgánica en orgánica gracias a la energía solar. Este
proceso se conoce como fotosíntesis y tiene lugar en los cloroplastos de las células eucarióticas
vegetales y en los mesosomas de los procariotas.
6 CO2 + 6 H2O C6H12O6 + 6 O2

La materia orgánica generada es utilizada por las mismas células fotosintéticas con el fin de
incorporarla a sus estructuras y obtener la energía precisa para realizar su actividad vital, pues
la energía solar, como tal, no es aprovechable directamente.
Para liberar la energía química contenida en la materia orgánica, se deben romper los enla-
ces con la ayuda del oxígeno. Este proceso se conoce como respiración celular y sucede en las
mitocondrias y en los mesosomas.
C6H12O6 + 6 O2 6 CO2 + 6 vapor de H2O + calor

29
-Pág.69-
C I E N C I A S D E L A N AT U R A L E Z A

La nutrición heterótrofa es realizada por aquellas células que, al no ser capaces de sintetizar
su propio alimento, han de tomarlo elaborado del exterior. Una vez incorporada, al igual que en
el caso anterior, la materia orgánica se utiliza para crear o reponer estructuras celulares y para
obtener la energía química que contiene mediante el proceso respiratorio.
Algunas células procariotas, como ciertas bacterias, liberan la energía química de la materia
orgánica mediante el proceso de la fermentación. Tal proceso ocurre en ausencia de oxígeno y
en él las moléculas orgánicas no se transforman en inorgánicas, sino en otras orgánicas más sen-
cillas, liberándose menor cantidad de energía.
Además de oxígeno, que entra directamente, las células heterótrofas incorporan moléculas
orgánicas complejas que deben ser previamente preparadas para su posterior uso. La ingestión
del alimento tiene lugar cuando la membrana celular, al percibir su presencia, se invagina y lo
engloba hasta dejarlo en el interior de una vacuola. Si el alimento está disuelto, la membrana se
deforma muy poco (pinocitosis), pero si es sólido, se deforma mucho, hasta el extremo de emi-
tir dos falsos brazos que lo rodean, los pseudópodos (fagocitosis). Una vez ingerida, se ha de
descomponer en fragmentos más pequeños, es decir, se ha de digerir, para lo cual los lisosomas
se aproximan a la vacuola formada, hasta llegar a unir sus membranas. En este momento, las
sustancias químicas que contienen los lisosomas (cuya misión es digestiva), se ponen en con-
tacto con el alimento y lo digieren. Una vez digerido, ya está preparado para atravesar la mem-
brana e incorporarse primero al citoplasma y luego a la mitocondria, a este proceso se le deno-
mina absorción.

3.1.2. FUNCIÓN DE RELACIÓN


Las células tienen la capacidad de responder a una gran cantidad de estímulos o variaciones
del medio, como la presencia de sustancias químicas, la luz, el contacto, la sequedad, etc. Las
respuestas pueden ser muy variadas: formación de cubiertas resistentes, desencadenamiento de
reacciones químicas..., si bien la más habitual es el movimiento, con o sin desplazamiento.
Existen muchos tipos de movimiento celular: ameboide (mediante la emisión de pseudópo-
dos), contráctil (por contracción y relajación celular sin desplazamiento), vibratil (mediante
cilios), intracelular (movimiento circular del citoplasma), ...

3.1.3. FUNCIÓN DE REPRODUCCIÓN


Son un conjunto de procesos mediante los cuales los seres vivos (progenitores) se perpetúan
y dan lugar a nuevos organismos, que son sus descendientes. Hay dos tipos de reproducción,
sexual y asexual.

30
-Pág.70-
U . D . 6 . - D IVERSIDAD Y UNIDAD DE LOS SERES VIVOS

3.1.3.1. Reprodución asexual


La reproducción asexual se caracteriza porque un único individuo puede originar gran can-
tidad de descendientes idénticos entre sí y, a su vez, idénticos al progenitor.
En la reproducción asexual, las células hijas reciben exactamente el mismo material genéti-
co que poseía la célula madre. Esto es posible gracias a un proceso de división nuclear conoci-
do como mitosis, que garantiza que cada descendiente reciba idéntico número de cromosomas.
Entre las formas de reproducción asexual destacan tres:

- Bipartición: la célula madre se divide en dos mitades aproximadamente iguales que, al


separarse, dan lugar a dos células hijas.

- Gemación: El núcleo de la célula madre se divide en dos núcleos hijos y uno de ellos
emigra hacia una protuberancia o yema de la célula inicial. Posteriormente la yema se
separa y origina una célula hija.

- Esporulación. El núcleo de la célula madre se divide en varios núcleos hijos, cada uno
de los cuales se recubre de una porción de citoplasma y de una membrana, constitu-
yendo las esporas. Cuando las condiciones ambientales son favorables, la cubierta de
la célula madre que alojaba a las esporas se rompe y las libera, y cada una de ellas da
lugar a una célula hija. Las esporas son formas de resistencia frente a condiciones des-
favorables puesto que pueden permanecer sin abrir durante mucho tiempo.
Aunque la mitosis es un proceso continuo, se diferencian en ella cuatro fases:
- Profase: Se inicia con la desintegración de la membrana nuclear y la desaparición de
los nucleolos. La cromatina comienza a enrollarse sobre sí misma hasta dar lugar a cro-
mosomas que al final de la etapa, están formados por dos cromátidas. Los centriolos
se duplican, emigrando cada pareja a un polo de la célula, organizándose alrededor de
ellos el aster, cuyas fibras se prolongan de un polo al otro tendiendo una especie de
cuerdas denominadas huso acromático. Al final de la profase, los cromosomas se unen
a las fibras del huso por los centrómeros.

- Metafase. Los cromosomas se colocan en el plano ecuatorial de la célula y cada cen-


trómero se divide en dos, dejando libres a las cromátidas.

- Anafase. Las fibras del huso se encogen, arrastrando con ellas a las cromátidas, que
emigran poco a poco hacia los polos de la célula.

- Telofase. Una vez situadas en los polos, las cromátidas comienzan a desenrollarse y se
van convirtiendo nuevamente en cromatina. Aparecen los nucleolos y la membrana
nuclear: desaparece el áster y el huso acromático.

31
-Pág.71-
C I E N C I A S D E L A N AT U R A L E Z A

Profase Metafase Anafase Telofase


Áster
Las cromátidas La membrana
Centriolos se trasladan nuclear se
hacia los reconstruye
polos
Citoplasma opuestos
de la célula
Membrana Los cromosomas
nuclear se desenrollan
y se transforman
Cromosoma en cromatina
La célula
(en realidad, empieza a
2 cromátidas constreñirse
Huso
Centrómero
acromático Filamentos Desaparece la La célula acaba por
Membrana del huso membrana dividirse en dos
celular nuclear

Mitosis. a) profase; b) metafase; c) anafase; d) telofase.

Al final, se produce un estrangulamiento de la célula, que se divide en dos mitades, iguales


con el mismo número de cromosomas. En este momento, cada cromosoma sólo está constitui-
do por una cromátida; por eso, antes de que la célula vuelva a dividirse nuevamente, es nece-
sario que las cromátidas se dupliquen, proceso que sucede en el periodo denominado interfase
y que se desarrolla entre dos divisiones sucesivas. Durante la interfase también tiene lugar el
crecimiento de la célula, ya que cuando se origina, su tamaño es la mitad del de su progenitora
y además se duplica el material genético o cromosómico. A este proceso en conjunto se le deno-
mina ciclo celular.

3.1.3.2. Reprodución sexual


La reproducción sexual es una mejora evolutiva, para perpetuar la especie con mayor rique-
za genética. Los descendientes heredan características paternas y maternas, pero en distintas
proporciones, por lo que son parecidos pero no idénticos. Hace falta la existencia de dos sexos
diferenciados que produzcan células sexuales diferenciadas (gametos), que en los animales se
llaman espermatozoides (las masculinas) y óvulos (las femeninas) que previamente han sufrido
un proceso para quedar cada célula con la mitad de la carga genética total (meiosis) y así al jun-
tarse dar de nuevo la carga genética total del individuo. En algunos organismos existen sexos
separados (macho y hembra), en otros, llamados hermafroditas, cada individuo puede poseer los
dos sexos a la vez. La reproducción sexual implica dos etapas sucesivas:
- Fecundación. Consiste en todos aquellos procesos destinados a que el gameto mascu-
lino llegue hasta el gameto femenino y se fusionen ambas células para formar una
nueva célula, llamada zigoto, que es un óvulo fecundado.
- Desarrollo embrionario. El zigoto es una célula que se divide cientos de miles de
veces hasta formar un embrión. Cuando el desarrollo embrionario está completo se ori-
gina un nuevo individuo adulto.

32
-Pág.72-
U . D . 6 . - D IVERSIDAD Y UNIDAD DE LOS SERES VIVOS

En algunos animales, el embrión se transforma en larva, que tiene vida libre, como la oruga,
y no se parece en nada al individuo adulto, como la mariposa. La transformación de larva en
adulto se llama metamorfosis.

3.2. EL SER VIVO COMO SISTEMA


La vida es la propiedad esencial de los seres vivos, caracterizada por la existencia de unas
funciones de nutrición, de reproducción y de relación.
Los seres vivos, han experimentado una serie de adaptaciones anatómicas y fisiológicas que
les ha permitido colonizar con éxito distintos ambientes. Estas adaptaciones son muy variadas,
aquí nos vamos a referir tan solo a los animales superiores, para mantener sus constantes vita-
les en equilibrio (temperatura corporal, cantidad de gases, de agua o de sales minerales) dentro
de unos límites independientes del medio externo, dichos mecanismos se conocen como home-
ostasis y están sometidos al control del sistema nervioso y del hormonal, estos mecanismos con-
sisten en el control de entradas de nutrientes, agua, gases y sales minerales y la compensación
con la salida de desechos. También los seres vivos cuentan con unos mecanismos de defensa
con los que hacen frente a las agresiones causadas por microorganismos parásitos y otros agen-
tes patógenos, procedentes también del medio externo.
En todos estos mecanismos interviene la sangre pues sirve de transporte de sustancias meta-
bólicas, sales que regulan la acidez, glóbulos blancos que actúan frente a agresiones de micro-
organismos externos, gases, tanto oxígeno como CO2, ...
1. Control del volumen de agua. El agua es un elemento fundamental para los seres
vivos. La falta de agua provoca alteraciones físicas y mentales. La mayor parte del agua corpo-
ral se situa dentro de las células (62,5% del total).
En los vertebrados superiores, todos los líquidos corporales proceden del plasma sanguíneo
y se filtran a través de las paredes de los capilares, que son muy porosas. El control del volu-
men de sangre se realiza a nivel del hipotálamo, mediante las neuronas osmorreceptoras, que
detectan la concentración de sales. Cuando la concentración de sales es alta se ponen en mar-
cha dos mecanismos para evitar la deshidratación corporal, la sed (a nivel del hipotálamo) y la
secreción de la hormona antidiurética (a nivel del hipotálamo), que impide las pérdidas a través
de la orina. El centro de la sed se estimula igualmente cuando hay sequedad en el boca o hay
una intensa hemorragia. Cuando el volumen de sangre aumenta, las sales se encuentran muy
diluidas, en este caso se inhibe la secreción de las hormonas del centro de la sed y de la secre-
ción de la hormona antidiurética.
2. Regulación de la temperatura corporal. Los vertebrados superiores tenemos la capa-
cidad de regular nuestra temperatura interior, de modo que permanezca dentro de unos determi-
nados márgenes, a pesar de que la temperatura de la piel dependa de la ambiental. La capa grasa
que hay debajo de la piel aisla. El calor interno se produce como resultado de las reacciones

33
-Pág.73-
CIE
CN CN
IE IA
CSI AD
SEDL
EALN
AAT U RU
N AT ARLA
ELZE
AZ A

metabólicas y también hay pérdidas, produciéndose un equilibrio calórico. La regulación de la


temperatura sucede a nivel hipotalámico. Los estímulos llegan por corriente sensitiva, proce-
dente de los corpúsculos táctiles del frío o del calor y de la estimulación por la temperatura de
la sangre cuando está por encima de 37°C o por debajo de 36°C. Las respuestas pueden ser de
refrigeración: vasodilatación cutánea y sudor (es el mecanismo más eficaz y se debe al agua
corporal). A través del sudor se excretan además de agua, otros productos, como sal común
–cloruro sódico– y urea. Esta es la razón de que sirva como mecanismo de excreción y de que
intervenga en el control de volumen sanguíneo. Los mecanismos de calefacción son la vaso-
constricción de los capilares cutáneos, los escalofríos y temblores (que aceleran el metabolismo
muscular) y la elevación del metabolismo del resto de las células del organismo. Además puede
acelerarse mediante la actuación del sistema nervioso simpático.
3. Control de la entrada de alimentos: El hambre y la saciedad. Están regulados por dos
centros específicos situados en el hipotálamo. El centro del hambre se excita por el deseo o pre-
sencia de un alimento, también se excita si la cantidad de glucosa en sangre baja demasiado. Por
el contrario, si la cantidad de glucosa es muy alto se estimula el centro de la saciedad y se inhi-
be el del hambre. Cuando comemos, se distienden las paredes del estómago produciendo seña-
les nerviosas que llegan al hipotálamo e inactivan el centro del hambre.
4. Sistemas de defensa contra los agentes patógenos. Para luchar contra los agentes
patógenos externos, se han desarrollado varios mecanismos:
- Barreras externas. La piel, como la epidermis, si se halla intacta, es una barrera bas-
tante efectiva contra la invasión de las bacterias. El sudor y la secreción de las glán-
dulas sebáceas producen una acidez en la piel que la protegen contra bacterias y hon-
gos. Las mucosas que tapizan las aberturas de nuestro cuerpo (ojos, boca, nariz, ano,
etc.) son mas susceptibles, aunque cuentan con las lágrimas, la saliva o las secreciones
mucosas para protegerse. Por otra parte, el ácido clorhídrico presente en el jugo gás-
trico del estómago, es un potente destructor para los microbios.
- Barreras internas. Cuando el agente patógeno rompe las barreras externas del cuerpo,
existe una segunda barrera constituida por los glóbulos blancos o leucocitos (entre los
que se encuentran los linfocitos). Los leucocitos combaten a los agentes infecciosos de
dos maneras diferentes: por ataque directo (por fagocitosis, originando el pus) y un ata-
que indirecto (es la llamada respuesta inmune producida por los linfocitos que tienen
la capacidad de producir anticuerpos específicos ante los antígenos de los agentes inva-
sores). En la respuesta inmune se forman anticuerpos, que son específicos para cada
agente. Cuando un organismo es capaz de resistir la invasión de un determinado agen-
te, se dice que está inmunizado contra él. Esta inmunidad puede ser innata (la que
posee la especie por selección genética, se nace con ella) y adquirida que puede ser
natural (que se adquiere tras haber pasado una enfermedad -activa- o por la leche
materna -pasiva-) y artificial, mediante las vacunas o por la administración directa de
anticuerpos.

34
-Pág.74-
U . D . 6 . - D IVERSIDAD Y UNIDAD DE LOS SERES VIVOS

3.3. LA CÉLULA COMO UNIDAD FUNCIONAL


DE LOS SERES VIVOS
La célula es el organismo vivo más pequeño que existe y, como tal, realiza las tres funcio-
nes vitales: nutrición, reproducción y relación, ya desarrolladas en el apartado 3.

3.4. LA PERCEPCIÓN DE ESTÍMULOS, LA ELABORACIÓN


Y LA PRODUCCIÓN DE RESPUESTAS
Este apartado se verá cuando realicemos el estudio del sistema nervioso como modo de rela-
ción y coordinación en el ser humano.
No obstante, si podemos indicar que todos los seres vivos tienen receptores para determinar
ciertas modificaciones ambientales que les sean de interés. Así, las plantas detectan la luz y rea-
lizan lo que se llama fototropismo, es decir, orientan sus hojas hacia la luz. Los seres unicelu-
lares detectan ciertas sustancias que les pueden servir de alimento (fagocitosis).
En los peces, existe lo que se denomina línea lateral, que es un cordón de terminaciones ner-
viosas especializadas en la detección de corrientes de agua que le permiten al pez nadar.
En general los seres vivos pueden detectar cambios de temperatura, presión, corrientes de
agua, luz, ondas sonoras, olores, .... La integración de la información y la producción de la res-
puesta van a estar determinadas por la complejidad del sistema nervioso y de las necesidades.
Cuando veamos los sistemas de relación y coordinación en el ser humano (Unidad 7), vere-
mos más en profundidad este tema.

4. INTRODUCCIÓN AL ESTUDIO DE LA HERENCIA

4.1. INTRODUCCIÓN AL ESTUDIO DE LOS CROMOSOMAS


Los cromosomas, situados en el núcleo de la célula portan la información genética. Son unos
filamentos independientes que aparecen cuando el núcleo está en fase de reproducción, duran-
te la mitosis. Llevan dispuestos en orden lineal (uno tras otro), los factores hereditarios o genes.
Todas las células, excepto los gametos, poseen una dotación cromosómica formada por un
determinado número de pares de cromosomas semejantes, llamados cromosomas homólogos.
Estas células, por su dotación, se llaman diploides. Los gametos, por el contrario, tienen la

35
-Pág.75-
C I E N C I A S D E L A N AT U R A L E Z A
C I E N C I A S D E L A N AT U R A L E Z A

mitad de la dotación cromosómica, por ello se dice que son haploides. Las células no repro-
ductoras (somáticas) tienen una dotación 2n (diploides), mientras que las células germinativas
o gametos, tienen, gracias a la meiosis una dotación n (haploide).
El número de cromosomas es característico para las células de cada especie y varía mucho
de unos seres vivos a otros.

4.2. LA TRANSMISIÓN DE LA HERENCIA


El estudio de la transmisión de la herencia, científicamente nació en 1860 de la mano de
Gregor Mendel, que se considera el padre de la genética. Hasta entonces habían sido estudios
de hibridación (cruces de variedades diferentes). Mendel, utilizando variedades de guisantes de
características determinadas las cruzó, y estudió de forma científica el proceso. Su trabajo le
llevó varios años y le llevó a enunciar los principios que regían la herencia de los caracteres,
estos principios se conocen como las Leyes de Mendel.
En todos los experimentos, se inicia el cruzamiento entre individuos de razas puras, es decir,
aquel que al cruzarlo con otro individuo igual que el para un determinado carácter, su descen-
dencia es igual que sus progenitores para ese carácter. El cumplimiento de las Leyes de Mendel
en la mayoría de los casos de herencia estudiados ha dado lugar a que se emplee el término
“mendelismo” para designar este tipo de herencia.

4.2.1. PRIMERA LEY DE MENDEL. LEY DE LA UNIFORMIDAD


Si se cruzan dos razas puras (P), todos los híbridos (F1) son iguales entre sí.
En el caso de las experiencias de Mendel, si se cruzan guisantes con la semilla de color ama-
rillo, raza pura, con guisante de semilla verde, todos los híbridos resultantes son de semillas
amarillas. Esto quiere decir que en lo referente al carácter color, el amarillo domina al verde. Se
dice que el carácter amarillo es dominante respecto al verde que es recesivo. El gen que deter-
mina el color de los guisantes se encuentra en un determinado cromosoma de sus células somá-
ticas. Como estas tienen 2n cromosomas, el gen del carácter color se encontrará en los dos cro-
mosomas homólogos. Ambos genes, que ocupan el mismo lugar en los cromosomas homólogos,
reciben el nombre de alelos.

P1 x P2

Los alelos R R r r

1° generación F1 Rr Rr Rr Rr todas son iguales, de color rosa.

36
-Pág.76-
U . D . 6 . - D IVERSIDAD Y UNIDAD DE LOS SERES VIVOS

Con las flores del Dragón, cuyas flores pueden ser rojas (R) o blancas (r):

Generación paterna RR (rojas) rr (blancas)

P1 x P2

Los alelos R R r r

1° generación F1 Rr Rr Rr Rr todas son iguales, de color rosa.


Como se ve en los dos casos, todo es igual excepto el color de las flores de la primera gene-
ración F1. Decimos que la generación paterna RR y rr son homocigotos, es decir, los dos alelos
son iguales. En la generación filial, todos los descendientes tienen un alelo R y otro r, decimos
que son heterocigotos o híbridos para ese carácter. A la manifestación externa de un carácter se
le llama fenotipo. El fenotipo de la generación F1 será rojo en el segundo ejemplo y rosa en el
primero. El genotipo, es la dotación de alelos que posee el individuo.
En el primer ejemplo, no existe relación dominancia-recesividad, sino que los dos alelos tie-
nen la misma potencia, decimos entonces que son equipotentes, como resultado, en carácter es
intermedio, así, el color de las flores es rosa. A estos casos se le llama de herencia intermedia.

4.2.2. SEGUNDA LEY DE MENDEL. LEY DE LA SEGREGACIÓN

Si se cruzan entre sí los híbridos de la generación F1, se observa que los individuos de la F2
no son todos iguales.
Como el genotipo de la primera generación F1 es Rr, los gametos que generan en la meiosis
pueden tener la dotación R o r, al 50%. Los cigotos formados por la fecundación de gametos R
con R serán RR (homocigotos dominantes) y color rojo. Los cigotos formados por la fecunda-
ción de gametos R con r y r con R, serán la dotación Rr (híbridos o heterocigotos), aunque tam-
bién serán rojos. Pero los originados por la fecundación de gametos r con gametos r, tendrán
una dotación rr (homocigotos recesivos y serán blancos). En el caso de la herencia intermedia,
el resultado será RR (25%), flores rojas, Rr (50%) flores rosas y rr (25%) flores blancas.
Volvemos al ejemplo de las Bocas de Dragón, cruzamos plantas de la generación F1, que son
rojas pero heterocigotos (Rr).

37
-Pág.77-
C I E N C I A S D E L A N AT U R A L E Z A

F1 Rr Rr
Rojo Rojo
Alelos R r R r

F2 RR Rr rR rr

4.2.3. TERCERA LEY DE MENDEL. LEY DE LA RECOMBINACIÓN

Si se cruzan razas que difieren en dos o varios alelos, éstos se heredan unos independiente-
mente de otros.
En las experiencias de Mendel, vamos a ver dos caracteres, el color de la semilla, amarillo
(A) es dominante frente al verde (a) y el carácter liso B, que es dominante frente al rugoso (b).
Si se cruza una generación parenteral P, de dihomocigóticos dominantes (AABB) con una
de homocigóticos recesivos (aabb), los gametos que proporcionan, tendrán una dotación AB y
ab respectivamente. Los individuos de la primera generación filial F1 serán forzosamente AaBb,
es decir, dihíbridos o diheterocigóticos. La segunda generación filial F2, los gametos posibles
formados en la primera generación son AB, Ab, aB y ab.
Para facilitar la disposición de los posibles cruces se hace un cuadro de Punnett, en honor a
su inventor:

AB Ab aB ab
AB AABB AABb AaBB AaBb
Ab AABb AAbb AaBb Aabb
aB AaBB AaBb aaBB aaBb
ab AaBb Aabb aaBb aabb

De 16 casos posibles, nueve guisantes serán amarillos y lisos, tres amarillos y rugosos, tres
verdes y lisos y uno verde y rugoso.

38
-Pág.78-
U . D . 6 . - D IVERSIDAD Y UNIDAD DE LOS SERES VIVOS

4.2.4. RETROCRUZAMIENTO
Puede ocurrir que, conociendo solo el fenotipo, se desconozca el genotipo, porque, un gui-
sante de semilla amarilla, ¿Es homocigótico o heterocigótico?.
Para poder establecer el genotipo de un espécimen se realizan cruces de prueba. Para ello se
cruza el problema con un homocigótico recesivo, a este procedimiento se le denomina retro-
cruzamiento.

4.2.5. DETERMINACIÓN DEL SEXO


En las células somáticas, los cromosomas aparecen de dos en dos, idénticos entre sí, son los
autosomas. Sin embargo, existe un par de cromosomas, los heterocromosomas o cromosomas
sexuales, que son iguales en las hembras (XX) y diferentes en los machos (XY).
El sexo se puede heredar y es función de la dotación cromosómica de los gametos. Se here-
dará como un carácter más, siguiendo las Leyes de Mendel.
Además del sexo, los cromosomas sexuales contienen otros genes, y por tanto, a través de
estos, se heredan caracteres que no tienen nada que ver con la determinación sexual, pero cuya
herencia, puede verse condicionada por el hecho de ser macho o hembra, son los caracteres liga-
dos al sexo.

4.3. INTRODUCCIÓN AL CONCEPTO DE MUTACIÓN


Se denominan mutaciones a los cambios repentinos e imprevisibles que se observan en los
genes.
El descubrimiento de la base bioquímica de la genética, permitió adentrarse con profundi-
dad en el concepto de mutación.
La sustancia química que forma los cromosomas es fundamentalmente ADN (ácido des-
oxiribonucleico). Esta compleja sustancia está formada por larguísimas cadenas de otras unida-
des más sencillas llamadas mononucleótidos, estos solo pueden ser de cuatro tipos, pero el
orden es muy importante. Se nombran con las letras A, G, T y C. Formando larguísimas cade-
nas dobles, las posibilidades son infinitas de ahí la enorme importancia, su enorme variabilidad
y una segunda característica es su capacidad para formar moléculas exactamente idénticas a
ella, contiene toda la información para mantener las funciones vitales en el organismo. Los
cambios accidentales producidos en las cadenas de ADN, son la explicación de la existencia de
las mutaciones.

39
-Pág.79-
C I E N C I A S D E L A N AT U R A L E Z A

RESUMEN
El fenómeno de la vida, surgió en el agua de los mares, cuando las sustancias químicas se
fueron acumulando dando lugar a lo que los científicos llamaron caldo primordial, que era un
conjunto de materiales acumulados en el agua de los mares primitivos a partir de los cuales se
formaron los seres vivos.
Las primitivas formas vivas eran organismos unicelulares procariotas, parecidos a las bac-
terias actuales.
La teoría de la evolución desarrollada por Darwin (1809-1882), explica que los seres vivos
que vemos hoy no fueron siempre así, sino que evolucionaron, es decir, experimentaron un pro-
ceso de cambio gradual de sus características, de generación en generación, a partir de un leja-
no antecesor.
Los seres vivos se dividen en cinco reinos: animales. plantas, fungi (hongos), moneras (bac-
terias y algas verde-azuladas) y protistas (protozoos). Aparte, podemos mencionar a los virus,
que se hallan en la misteriosa barrera entre lo vivo y lo no vivo.
Los virus son la forma más simple de vida. Solo se pueden observar con el microscopio elec-
trónico. Son formas especiales de vida, ya que no realizan todas las funciones vitales. Funda-
mentalmente, son formas capaces de reproducirse, y parasitan a todos los seres vivos.
El reino de las moneras, tienen una enorme capacidad de adaptación al medio, son los agen-
tes de reciclaje más importantes de la naturaleza.
El reino protistas son seres unicelulares que viven en medio líquido y realizan su alimenta-
ción a través de la membrana celular. Algunos suelen ser parásitos.
Los animales invertebrados, son aquellos que carecen de esqueleto interno articulado. Los
invertebrados surgieron en el mar como consecuencia de la evolución de organismos unicelula-
res hacia una estructura más compleja y eficiente. Invertebrados son las esponjas, los celenté-
reos, los platelmintos, nemátodos, anélidos, moluscos, artrópodos (crustáceos, arácnidos, insec-
tos y miriápodos) y equinodermos.
Muchas de las especies de insectos sufren metamorfosis. La metamorfosis es un cambio que
se produce en el desarrollo de ciertos animales, por el que la forma de la larva sufre un apre-
ciable cambio para convertirse en adulto, el gusano de la seda es un ejemplo.
Existen muchos insectos sociales que viven en comunidades, los individuos en estas comu-
nidades se especializan, adquiriendo morfologías distintas.
Los animales vertebrados, poseen todos ellos un esqueleto interno con columna vertebral,
un cráneo que alberga su cerebro y un sistema nervioso complejo. Los peces, anfibios, reptiles,
aves y mamíferos pertenecen a este grupo.
El reino fungi, sus células están recubiertas externamente por celulosa. Los hongos se nutren
de restos de animales y plantas y producen su descomposición, desempeñando un papel funda-
mental en la naturaleza.

40
-Pág.80-
U . D . 6 . - D IVERSIDAD Y UNIDAD DE LOS SERES VIVOS

Los musgos (Briófitas), desarrollan su vida entre el agua y la tierra; fueron los primeros
organismos fotosintéticos que colonizaron el medio terrestre, apenas tienen raíz y la forma de
transportar el agua hasta el resto de la planta es muy rudimentaria y poco eficaz. Tienen un ciclo
biológico en el que se alternan la reproducción sexual y la reproducción por esporas.
Los líquenes, son asociaciones de un hongo y un alga, que viven en simbiosis.
Los helechos (Pteridófitas), realizan la fotosíntesis, y tienen raíz, tallo y hojas, pero no tie-
nen semillas, se reproducen por esporas.
En las plantas superiores (Espermafitas), cada parte cumple una función específica: la raíz
absorbe agua y minerales (pelos absorbentes); el tallo, transporta (floema y xilema) y almace-
na alimentos; la hoja los fabrica (fotosíntesis); las flores sirven para la reproducción; los frutos
y semillas, para la formación de nuevas plantas.
Por lo general los flores no son fecundadas por su propio polen, sino con el de otras plantas
de la misma especie. La polinización es el transporte del polen de una flor a otra a través del
aire o con la ayuda de un animal.
La semilla es el óvulo fecundado que contiene el germen o embrión desarrollado a partir del
zigoto y reservas nutritivas; todo ello protegido por cubiertas resistentes. Las reservas nutritivas
se almacenan en unas estructuras llamados cotiledones.
Según como se agrupen los órganos reproductores en las plantas, las podemos dividir en dos
grupos: Gimnospermas (las más antiguas, los órganos femeninos se encuentran desnudos) y
Angiospermas (los órganos femeninos se encuentran en flores “vistosas” completas) que se
dividen en monocotiledoneas y dicotiledoneas.
Teniendo en cuenta la complejidad estructural de las células, podemos clasificarlas en célu-
las procariotas (son células primitivas, relativamente simples y carentes de un núcleo definido)
y células eucariotas (su estructura es más compleja que la de las anteriores, presenta membra-
na, citoplasma y núcleo definido).
Las células eucariotas pueden ser animales (sin pared, sin plastos, casi sin vacuolas y con
centriolos) y vegetales (pared celular, plastos, muchas vacuolas y ausencia de centriolos).
Los orgánulos citoplasmáticos son retículo endoplásmico, ribosomas, aparato de Golgi,
mitocondrias, plastos, centriolos, lisosomas y vacuolas.
En el núcleo encontramos la membrana nuclear,. la cromatina, el nucleolo, y el plasma
nuclear.
La división de la célula se realiza por la mitosis (profase, anafase, metafase y telofase).
Cuando el núcleo se va a dividir, adquiere un aspecto distinto: la cromatina se va conden-
sando y se forman unos filamentos independientes, llamados cromosomas. Los cromosomas tie-
nen gran importancia, ya que los caracteres hereditarios dependen de su constitución y conteni-
do.

41
-Pág.81-
C I E N C I A S D E L A N AT U R A L E Z A

Los organismos pueden ser unicelulares y pluricelulares (en este caso siempre de células
eucariotas, presentan especialización con tejidos que cumplen distintas funciones).
Los tejidos pueden ser vegetales (embrionarios con los meristemos y adultos, protectores,
parenquimáticos, conductores, de sostén y secretores) y animales (epiteliales, conectivo, mus-
cular y nervioso).
Las funciones fundamentales de los seres vivos son las de relación, nutrición (autótrofa o
heterotrofa) y reproducción (sexual o asexual).
Las células tienen la capacidad de responder a una gran cantidad de estímulos o variaciones
del medio, como la presencia de sustancias químicas, la luz, el contacto, la sequedad, etc. Las
respuestas pueden ser muy variadas: formación de cubiertas resistentes, desencadenamiento de
reacciones químicas..., si bien la más habitual es el movimiento, con o sin desplazamiento.
La nutrición autótrofa la realizan las células que son capaces de elaborar su propio alimen-
to, es decir, de transformar la materia inorgánica en orgánica gracias a la energía solar. Este pro-
ceso se conoce como fotosísntesis y tiene lugar en los cloroplastos de las células eucarióticas
vegetales y en los mesosomas de los procariotas.
La nutrición heterótrofa es realizada por aquellas células que, al no ser capaces de sintetizar
su propio alimento, han de tomarlo elaborado del exterior. Una vez incorporada, al igual que en
el caso anterior, la materia orgánica se utiliza para crear o reponer estructuras celulares y para
obtener la energía química que contiene mediante el proceso respiratorio.
La reproducción asexual se caracteriza porque un único individuo puede originar gran can-
tidad de descendientes idénticos entre sí y, a su vez, idénticos al progenitor, las células hijas
reciben exactamente el mismo material genético que poseía la célula madre. Puede ser por
bipartición, gemación y esporulación.
La reproducción sexual es una mejora evolutiva, para perpetuar la especie con mayor rique-
za genética. Los descendientes heredan características paternas y maternas, pero en distintas
proporciones, por lo que son parecidos pero no idénticos. Hace falta la existencia de dos sexos
diferenciados que produzcan células sexuales diferenciadas (gametos), que en los animales se
llaman espermatozoides (las masculinas) y óvulos (las femeninas) que previamente han sufrido
un proceso para quedar cada célula con la mitad de la carga genética total (meiosis) y así al jun-
tarse dar de nuevo la carga genética total del individuo. En algunos organismos existen sexos
separados (macho y hembra), en otros, llamados hermafroditas, cada individuo puede poseer los
dos sexos a la vez.
Los seres vivos, han experimentado una serie de adaptaciones anatómicas y fisiológicas para
mantener sus constantes vitales en equilibrio (temperatura corporal, cantidad de gases, de agua
o de sales minerales) dentro de unos límites independientes del medio externo, dichos meca-
nismos se conocen como homeostasis y están sometidos al control del sistema nervioso y del
hormonal, estos mecanismos consisten en el control de entradas de nutrientes, agua, gases y
sales minerales y la compensación con la salida de desechos. También los seres vivos cuentan

42
-Pág.82-
U . D . 6 . - D IVERSIDAD Y UNIDAD DE LOS SERES VIVOS

con unos mecanismos de defensa con los que hacen frente a las agresiones causadas por micro-
organismos parásitos y otros agentes patógenos, procedentes también del medio externo.
Los cromosomas, situados en el núcleo de la célula portan la información genética. Son unos
filamentos independientes que aparecen cuando el núcleo está en fase de reproducción durante
la mitosis. Llevan dispuestos en orden lineal (uno tras otro), los factores hereditarios o genes.
Todas las células, excepto los gametos, poseen una dotación cromosómica formada por un
determinado número de pares de cromosomas semejantes, llamados cromosomas homólogos.
Estas células, por su dotación, se llaman diploides. Los gametos, por el contrario, tienen la
mitad de la dotación cromosómica, por ello se dice que son haploides
El número de cromosomas es característico para las células de cada especie y varía mucho
de unos seres vivos a otros.
La transmisión de la herencia está regida por las Leyes de Mendel, la primera dice que si se
cruzan dos razas puras (P), todos los híbridos (F1) son iguales entre sí. La segunda dice que si
se cruzan entre sí los híbridos de la generación F1, se observa que los individuos de la F2 no son
todos iguales. La tercera dice que si se cruzan razas que difieren en dos o varios alelos, éstos se
heredan unos independientemente de otros.
En las células somáticas, los cromosomas aparecen de dos en dos, idénticos entre sí, son los
autosomas. Sin embargo, existe un par de cromosomas, los heterocromosomas o cromosomas
sexuales, que son iguales en las hembras (XX) y diferentes en los machos (XY). El sexo se
puede heredar y es función de la dotación cromosómica de los gametos. Se heredará como un
carácter más, siguiendo las Leyes de Mendel.
Se denominan mutaciones a los cambios repentinos e imprevisibles que se observan en los
genes.

43
-Pág.83-
C I E N C I A S D E L A N AT U R A L E Z A

EJERCICIOS DE AUTOCOMPROBACIÓN
1. Las instrucciones necesarias para el mantenimiento de la vida, almacenadas en el ADN,
reciben el nombre de:
A. Larvas B. Genes C. Núcleos D. Ribosomas
2. Las bacterias son:
A. Células procariotas B. Células eucariotas
C. Son protozoos D. Son semejantes a los virus
3. La transformación de larva en adulto, durante el desarrollo embrionario de ciertos anima-
les, recibe el nombre de:
A. Metamorfosis B. Ciclo biológico
C. Reproducción sexual D. Ciclo biomorfológico
4. Los descendientes de la reproducción asexual son:
A. Idénticos B. Parecidos pero no idénticos
C. Hermafroditas D. Todas son falsas
5. El SIDA es una enfermedad causada por:
A. Una bacteria B. Un virus C. Un protozoo D. Un protista
6. La tenia es un gusano parásito que pertenece al tipo de:
A. Los platelmintos B. Los nemátodos C. Los anélidos D. Los ofiuros
7. Los equinodermos se mueven por el fondo marino mediante:
A. Propulsión B. Los pies ambulacrales
C. Unas patas articuladas D. Mediante pseudópodos
8. Todas las aves se caracterizan por tener:
A. Fúrcula B. Respiración pulmonar
C. Escamas D. Glándulas mamarias
9. Las plantas que tienen las flores femeninas al descubierto son las:
A. Angiospermas B. Gimnospermas C. Espermafitas D. Pteridofitas
10. Los poros que tienen las hojas para el intercambio de gases son los:
A. Floema B. Savia C. Xilema D. Estomas

RESPUESTAS A LOS EJERCICIOS


11. B 12. A 13. A 14. A 15. B

16. A 17. B 18. B 19. B 10. D

44
-Pág.84-
U . D . 7 . - L AS PERSONAS Y LA SALUD

ÍNDICE
OBJETIVOS . . . . . . . . . . . . . . . . . . . . . . . . . . . . . . . . . . . . . . . . . . . . . . . . . . . . . . . . . . . . .2

INTRODUCCIÓN . . . . . . . . . . . . . . . . . . . . . . . . . . . . . . . . . . . . . . . . . . . . . . . . . . . . . . . . .3

ESQUEMA DE CONTENIDOS . . . . . . . . . . . . . . . . . . . . . . . . . . . . . . . . . . . . . . . . . . . . . .4

DESARROLLO DE CONTENIDOS . . . . . . . . . . . . . . . . . . . . . . . . . . . . . . . . . . . . . . . . . .5

1. LA SALUD Y LA ENFERMEDAD . . . . . . . . . . . . . . . . . . . . . . . . . . . . . . . . . . . . . .5

2. LA NUTRICIÓN HUMANA . . . . . . . . . . . . . . . . . . . . . . . . . . . . . . . . . . . . . . . . . .10

3. LA REPRODUCCIÓN HUMANA . . . . . . . . . . . . . . . . . . . . . . . . . . . . . . . . . . . . . .30

4. INTRODUCCIÓN A LA RELACIÓN Y COORDINACIÓN HUMANAS.


LA SALUD MENTAL. UTILIZACIÓN DEL SISTEMA SANITARIO.
CONSUMO DE MEDICAMENTOS, SU EFICACIA Y SUS RIESGOS . . . . . .39

RESUMEN . . . . . . . . . . . . . . . . . . . . . . . . . . . . . . . . . . . . . . . . . . . . . . . . . . . . . . . . . . . .57

EJERCICIOS DE AUTOCOMPROBACIÓN . . . . . . . . . . . . . . . . . . . . . . . . . . . . . . . . . .62

RESPUESTAS A LOS EJERCICIOS . . . . . . . . . . . . . . . . . . . . . . . . . . . . . . . . . . . . . . . . .64

-Pág.85-
C I E N C I A S D E L A N AT U R A L E Z A

OBJETIVOS
Al finalizar el estudio de esta Unidad Didáctica, el alumno será capaz de:

• Definir los conceptos de salud y enfermedad.

• Identificar los principales tipos de enfermedades.

• Interpretar las principales normas de prevención de las enfermedades infecciosas.

• Explicar los mecanismos de defensa de nuestro organismo frente a las enfermedades.

• Valorar los efectos de un estilo de vida saludable sobre la salud.

• Diferenciar entre alimentación y nutrición.

• Conocer los distintos sistemas y aparatos que en el organismo realizan el proceso de la


nutrición.

• Clasificar los distintos alimentos según su composición.

• Diferenciar los nutrientes que contienen los alimentos: glúcidos, lípidos, proteínas,
sales minerales, vitaminas y agua.

• Distinguir los buenos hábitos nutricionales y sus consecuencias para la salud.

• Conocer ciertas enfermedades de los sistemas y aparatos implicados en los fenómenos


de la nutrición y saber como prevenirlas.

• Establecer el funcionamiento de los sistemas de relación y coordinación, sistema ner-


vioso y endocrino.

• Identificar los tipos de drogas y los efectos que pueden tener en las personas.

• Asimilar las funciones de reproducción.

-Pág.86-
U . D . 7 . - L AS PERSONAS Y LA SALUD

INTRODUCCIÓN
E l concepto abstracto de salud ha tenido un significado cambiante a lo largo de la
historia de acuerdo con el estado de los conocimientos científicos y la interpreta-
ción y tratamiento de las enfermedades, pero en la mayoría de los casos, la salud se ha
asociado a la ausencia de enfermedad. Hoy en día definimos la salud como el completo
bienestar físico, mental y social, y no meramente la ausencia de enfermedad, abarcando
todas las funciones del ser vivo: nutrición, relación y coordinación.

-Pág.87-
C I E N C I A S D E L A N AT U R A L E Z A

M A PA C O N C E P T UA L
EJERCICIO FÍSICO

DROGAS, SALUD BUCO-DENTAL


TABACO Y SALUD HÁBITOS
ALCOHOL SALUDABLES
PREVENCIÓN DE
ACCIDENTES

PREVENCIÓN DE
FÍSICA MENTAL SOCIAL ENFERMEDADES

INFECCIOSAS NO
FUNCIONES VITALES INFECCIOSAS

VACUNACIÓN
NUTRICIÓN REPRODUCCIÓN RELACIÓN Y SISTEMA
COORDINACIÓN ENDOCRINO

A. GENITAL A. GENITAL SISTEMA


ALIMENTACIÓN MASCULINO NERVIOSO HORMONAL
FEMENINO

ESPERMATOZOIDES ÓVULOS RECEPTORES - VISTA


DIGESTIÓN
- OÍDO
CÉLULAS SEN- - OLFATO
Realizada por NUTRIENTES FECUNDACIÓN
SORIALES - GUSTO
- Hidratos de - SENTIDOS
A. DIGESTIVO carbono. CIGOTO CENTROS DIFUSOS
- Lípidos. NERVIOSOS
A. RESPIRATORIO - Proteínas. EMBARAZO
S.N.CENTRAL NERVIOS
- Vitaminas.
A. EXCRETOR - Minerales. NUEVO SER
- Agua.
A. CIRCULATORIO SISTEMA EFECTORES
LOCOMOTOR

HUESOS GLÁNDULA HORMONAS

MÚSCULOS

-Pág.88-
U . D . 7 . - L AS PERSONAS Y LA SALUD

1. LA SALUD Y LA ENFERMEDAD

1.1. LA SALUD Y LA ENFERMEDAD


Según la OMS, la salud es “el perfecto estado de bienestar físico, mental y social, y no sólo
la ausencia de enfermedad o dolencia”.
Cuando se rompe el equilibrio entre los tres componentes de la salud (físico, mental y
social), por causas internas al individuo o procedentes del exterior, aparece una alteración en el
organismo. Esa alteración es la enfermedad, que se manifiesta en forma de síntomas.
Los síntomas los podemos definir como la manifestación de una alteración orgánica o fun-
cional por parte del organismo, que pueden ser objetivos o signos (el explorador los puede
observar) y subjetivos (percibidos tal sólo por el enfermo). Denominamos síndrome al conjun-
to de síntomas y signos que definen una enfermedad.
La investigación de la salud física de una persona se realiza determinando sus parámetros
fisiológicos más importantes, conocidos como constantes vitales.
Entre las constantes vitales tenemos:
1) La temperatura corporal. De control hipotalámico. Los valores normales oscilan alrede-
dor de 37°C. El termómetro clínico se usa en la axila, boca y recto, presentando ligeras
diferencias de temperatura.
2) El peso corporal. Nos indica el grado de normalidad nutricional de nuestro organismo.
Se debe determinar en ayunas, descalzo y con la menor ropa posible.
3) Pulso. Es el resultado de las variaciones de presión que se producen durante el ciclo car-
díaco, las cuales provocan la dilatación y contracción de las paredes de las arterias.
Detecta la frecuencia cardíaca.
4) Presión arterial. Es la presión sanguínea en el interior de las arterias principales, donde
alcanzan sus valores máximos. Esta presión varía, por lo que se puede establecer una pre-
sión máxima (o sistólica) y una presión mínima (diastólica). Se determina con un esfin-
gomanómetro en la arteria humeral (en el brazo). La presión normal para personas entre
15 y 45 años es 80/120 mm de Hg.
5) Capacidad vital. Es el volumen máximo de aire que puede intercambiarse en una sola
ventilación pulmonar (inspiración y espiración). Se determina con el espirómetro.
6) Hemograma. Es el conjunto de determinaciones analíticas sobre las células sanguíneas
(eritrocitos y leucocitos) y plaquetas.

5
-Pág.89-
C I E N C I A S D E L A N AT U R A L E Z A

7) Composición química del plasma. La determinación cuantitativa de los componentes del


plasma nos indica el grado de eficacia de los diversos procesos metabólicos.
8) Composición química de la orina. Determina la eficacia o estado de salud de los riñones.
Podemos definir las enfermedades como una alteración orgánica o funcional que afecta
negativamente al estado de bienestar de una persona. Las enfermedades se pueden clasificar
atendiendo a varios criterios:
– La edad de aparición: Enfermedades infantiles (sarampión), juveniles (parotiditis), del
adulto (úlcera gástrica), de la vejez (artrosis).
– La localización: Enfermedades localizadas (en un aparato o sistema -enfermedades sis-
témicas-) o enfermedades generales (afectan a todo el organismo, como las infecciones).
– La duración en el tiempo (patocronia): enfermedades agudas (aparecen de forma brus-
ca y transcurren pocos días hasta su resolución, como es el caso del sarampión) y enfer-
medades crónicas, con un inicio lento y se prolongan durante meses o años (la cirrosis
hepática).
– Según su patogenia (mecanismo de actuación del agente agresor sobre el organismo):
- Enfermedades inflamatorias, se produce una inflamación debido a un organismo vivo
(infecciosa) o un agente tóxico o físico (no infecciosa).
- Por modificaciones celulares. Pueden ser atróficas (disminuye el volumen celular),
hipertróficas (aumenta el volumen celular) y tumorales o neoplásicas (alteraciones
morfológicas).
- Enfermedades constitucionales (debido a factores hereditarios que predisponen a una
determinada patología).
- Enfermedades alérgicas.
- Enfermedades metabólicas. Por modificaciones que pueden afectar al metabolismo de
los glúcidos, de los lípidos o de las proteínas.
- Enfermedades degenerativas. Por cambios químicos a nivel celular, que dan lugar a
una alteración y disfunción en los tejidos.
– Según su etiología (factores causales de la enfermedad). Debido a:
- Herencia. Se transmiten características físicas y psíquicas, tanto en el momento de la
concepción (enfermedad hereditaria), como durante el embarazo (enfermedad congé-
nita).
- Agentes físicos. Calor, frío, radiaciones, traumatismos, ...
- Agentes químico-tóxicos. Medicamentos, gases, ácidos, ...

6
-Pág.90-
U . D . 7 . - L AS PERSONAS Y LA SALUD

- Agentes biológicos. Distinguimos entre enfermedades infecciosas (causadas por bac-


terias, virus u hongos) y enfermedades parasitarias o infestaciones (causadas por ani-
males parásitos, como los piojos).
- Alimentación. Problemas en la ingestión tanto en cantidad como en calidad de ali-
mentos.
- Factores psíquicos. Hay enfermedades orgánicas que se producen a consecuencia de
factores psíquicos y enfermedades orgánicas que producen modificaciones psicológi-
cas.
- Civilización. Son lo que se denomina enfermedades de la civilización. Accidentes de
tráfico, stress, depresiones...
Todas las enfermedades en su evolución siguen las siguientes fases:
· Período de incubación o fase de latencia. El agente agresor actúa sin provocar manifesta-
ciones objetivas ni subjetivas.
· Período prodrómico. Aparecen manifestaciones sintomáticas inespecíficas (malestar, alte-
raciones del sueño...).
· Período de invasión. Se manifiestan todos los síntomas y signos de la enfermedad.
· Período de estado. Se alcanza el punto máximo de desarrollo de la enfermedad.
· Terminación de la enfermedad. Hacia una curación completa, hacia una curación incom-
pleta o hacia la muerte.

Las enfermedades se manifiestan mediante unos signos o alteraciones de los parámetros


fisiológicos normales que se denominan síntomas, algunos son subjetivos (dolor, ...) y otros
objetivos medibles (fiebre, ...) que permiten el diagnóstico.

En el caso de las infecciones, que es la penetración y desarrollo de microorganismos pató-


genos en el interior de un individuo sano. Para evitar estas enfermedades, el organismo dispo-
ne de unos mecanismos de defensa: La piel, las mucosas y el sistema inmunitario.

El sistema inmunitario, permite el reconocimiento y eliminación de cualquier microorga-


nismo, célula cancerosa o tejido extraño implantado en el organismo. Este sistema actúa
mediante células especializadas de la sangre: los glóbulos blancos o leucocitos.

Existen distintos tipos de glóbulos blancos, pero solo algunos presentan actividad inmunita-
ria: los neutrófilos, los monocitos y diversos tipos de linfocitos (T y B).

Los neutrófilos y monocitos fagocitan las células extrañas introducidas en nuestro organis-
mo; los linfocitos T, destruyen directamente a algunos microorganismos y un tipo especial de
ellos (linfocitos T4) detectan las sustancias extrañas (antígenos) que presentan las células inva-

7
-Pág.91-
C I E N C I A S D E L A N AT U R A L E Z A

soras y activan la producción de linfocitos T y B; los linfocitos B, producen anticuerpos, que


neutralizan a los antígenos y destruyen las células invasoras que los contienen.
Si el sistema inmunitario funciona correctamente, cada vez que se introduce un antígeno en
nuestro organismo se producen anticuerpos específicos que lo neutralizan. La reacción antíge-
no-anticuerpo es la base de la inmunidad.

1.2. CRECIMIENTO Y DESARROLLO


La noción de desarrollo supone un proceso de cambio unitario y continuo, es un crecimien-
to pautado cuya forma tiene por lo común un carácter progresivo, y cuyo sentido es el de un
aumento de la calidad biosocial de la conducta.
Dentro del desarrollo existen distintos procesos evolutivos, como el desarrollo genital y el
del sistema nervioso entre otros, que evolucionan como partes de un conjunto, con distintas
tasas de crecimiento, aunque existe una evolución conjunta e integrada.
La especie humana es la que posee el período de desarrollo más largo, ya que una tercera
parte de su ciclo vital es de carácter progresivo y está abierto a enriquecimientos comporta-
mentales de importancia. En cambio, a partir de la juventud, la conducta va perdiendo elastici-
dad en muchos aspectos, con lo que las posibilidades de desarrollo se van reduciendo.
En el desarrollo influyen múltiples factores, la mayoría de ellos ambientales, de forma que
los niveles de desarrollo alcanzados por los niños de 10 años en un país altamente industriali-
zado son, por lo general, superiores a los alcanzados por los niños de su misma edad de paises
subdesarrollados. Asimismo, los niños de zonas rurales siguen un ritmo de maduración más
lento que los niños de núcleos urbanos.
La mayoría de los procesos de aprendizaje se desarrollan a partir de las interacciones de la
persona con el medio ambiente. Es muy importante, sin embargo, darse cuenta de que en nin-
guno de los procesos del desarrollo, el ambiente por sí solo crea la conducta. Ciertas potencia-
lidades biológicas del crecimiento individual que se despliegan en un proceso maduracional son
necesarios para que se produzca el desarrollo.
La maduración es el crecimiento resultante de la interacción entre los genes y las condicio-
nes ambientales internas que caracterizan a las especies; por consiguiente, parte del resultado
del ejercicio o del uso de estructuras ya presentes. La maduración, prepara a los niños para
muchas conductas, pero el aprendizaje y los factores ambientales son básicos para su desarro-
llo final.

1.3. IMPORTANCIA DE LA ADQUISICIÓN DE ESTILOS DE VIDA


SALUDABLES
La prevención de las enfermedades, supone el seguir diversas normas. Debemos distinguir
entre la prevención de las enfermedades infecciosas y de las no infecciosas.

8
-Pág.92-
U . D . 7 . - L AS PERSONAS Y LA SALUD

Para prevenir las enfermedades infecciosas, lo primero es evitar el contacto con microorga-
nismos patógenos: correcto aseo personal, precaución en el contacto con las personas enfermas,
correcta manipulación de alimentos, la eliminación controlada de residuos orgánicos y el con-
trol sanitario de los animales transmisores de enfermedades.
La desinfección inmediata de las heridas es otra buena práctica, mediante el uso de antisép-
ticos (alcohol, agua oxigenada, ...).
Otra buena práctica preventiva de las infecciones, es la vacunación, que consiste en intro-
ducir en el organismo sano microbios (con antígenos) muertos o debilitados para que el orga-
nismo produzca los anticuerpos correspondientes.
La reacción antígeno-anticuerpo es parecida a una pequeña enfermedad (reacción de la
vacuna) que, superada por el organismo, lo provee de los anticuerpos correspondientes.
La prevención de las enfermedades no infecciosas supone diversas normas higiénicas, algu-
nas generales y otras más específicas, que determinan un estilo de vida saludable al que debe-
mos tender para conseguir el mayor grado de bienestar físico, mental y social, es decir de salud.
Entre estas indicaciones para nuestra vida diaria podemos mencionar:

1.3.1. EJERCICIO FÍSICO


La práctica habitual de un deporte, o la realización de un ejercicio físico adecuado a las cir-
cunstancias personales, supone una fuente de salud porque evita o retarda la aparición de mucha
enfermedades: arteriosclerosis, enfisema, varices, obesidad, estreñimiento, ...

1.3.2. SALUD BUCO-DENTAL


Aunque las piezas dentarias las cambiamos una vez, las mantenemos desde aproximada-
mente los 6 años, y nos han de durar toda la vida. Es conveniente lavarlos después de las comi-
das y antes de acostarse. Por otra parte conviene reducir el consumo de azúcares que producen
caries y reducir el tabaco que estropea y afea la dentadura.
Es conveniente visitar al dentista al menos una vez al año para revisar la boca y limpiarla en
profundidad. Especialmente indicado para los niños es la fluoración de las encías.

1.3.3. DIETA EQUILIBRADA


Este punto lo desarrollaremos más adelante, en el apartado 2.3.

1.3.4. PLAN DE VIDA EQUILIBRADO


Un horario que distribuya y regularice las horas de sueño (al menos ocho), las comidas, las
horas de trabajo o estudio y la horas de deporte y ocio favorece la higiene del sistema nervioso
y evita la aparición de depresión y estrés.

9
-Pág.93-
C I E N C I A S D E L A N AT U R A L E Z A

1.3.5. EVITAR INTOXICACIONES Y DEPENDENCIAS


El alcoholismo, el tabaquismo y las drogodependencias son causa de múltiples enfermeda-
des físicas (cirrosis hepática, tumores del aparato respiratorio, encefalopatías, gastritis, bron-
quitis crónica, ...) y mentales (psicosis depresiva, agresividad, ansiedad, ...). Además durante el
trabajo y la conducción son causa de importantes accidentes, llegando a producir la muerte o la
inutilidad física a un elevado número de personas.

1.3.6. CONDICIONES DE TRABAJO SALUDABLES


Un horario de trabajo que permita el desarrollo integral de la persona, junto con unas con-
diciones ambientales saludables (postura correcta, bajo nivel sonoro, ausencia de radiaciones y
productos cancerígenos, puede evitar accidentes laborales y enfermedades profesionales muy
diversas.

1.4. PREVENCIÓN DE ACCIDENTES Y ENFERMEDADES


INFECCIOSAS
Es conveniente tanto a nivel laboral, como en nuestros lugares de residencia, establecer ruti-
nas de seguridad y evitar todas aquellas conductas que pueden resultar peligrosas para nosotros
mismos o para cualquier otra persona que conviva con nosotros. Estos cuidados deben extre-
marse con los niños.
Para evitar las enfermedades infecciosas, existe un gran número de medicamentos para
intentar producir la curación. Entre todos los medicamentos podemos destacar los sueros, los
antibióticos y los analgésicos.
Los sueros son concentrados de anticuerpos específicos para una determinada enfermedad
infecciosa. Su finalidad es producir en los individuos ya afectados por la enfermedad la inmunidad.
Los antibióticos, los descubrió Fleming en 1928. Son unas sustancias producidas por ciertas
especies de mohos para luchar contra las bacterias. En la actualidad son sustancias de síntesis,
es decir, fabricadas artificialmente en los laboratorios.

2. LA NUTRICIÓN HUMANA
2.1. LA NUTRICIÓN HUMANA
Como somos seres pluricelulares complejos, la materia que incorporamos desde el exterior
de nuestro organismo debe ser distribuida a cada una de nuestras células, donde tendrán lugar
reacciones químicas, conocidas como metabolismo. Gracias a estas reacciones, las células

10
-Pág.94-
U . D . 7 . - L AS PERSONAS Y LA SALUD

obtienen la energía y la materia imprescindible para su supervivencia, y por lo tanto para el


organismo completo. Así, el proceso de la nutrición consiste en intercambiar materia y energía
con el exterior para su correcto funcionamiento.

La nutrición, por lo tanto es el conjunto de fenómenos involuntarios e inconscientes, por los


que el organismo se asegura el ingreso, absorción y transformación (anabolismo) de los ali-
mentos, para ser utilizados en procesos energéticos y metabólicos, a la vez que son eliminadas
(catabolismo) las sustancias inútiles resultantes. Los alimentos son toda sustancia que introdu-
cida en el organismo, sirve para la “nutrición” de los tejidos o para producir energía.

La nutrición difiere de la alimentación en que ésta última es voluntaria y consciente, es


decir, elegimos los alimentos que queremos comer. La relación entre el proceso alimentario y
nutritivo puede verse en el siguiente esquema.
Desechos
APARATO y agua
ALIMENTOS APARATO EXCRETOR
Y AGUA DIGESTIVO
Vapor
PIEL de agua

SISTEMA CÉLULAS SISTEMA


CIRCULATORIO metabolismo CIRCULATORIO
Calor
OXÍGENO APARATO APARATO
RESPIRATORIO RESPIRATORIO CO2 y vapor
de agua
Esquema del proceso de nutrición en el ser humano.
Como podemos observar, la nutrición es un proceso más complejo que la simple ingestión
de alimentos y en él están implicados los aparatos digestivo, respiratorio, excretor y el sistema
circulatorio.

2.1.1. APARATOS Y SISTEMAS RELACIONADOS CON LA NUTRICIÓN


2.1.1.1. EL APARATO DIGESTIVO
El aparato digestivo es el encargado de desdoblar los alimentos en principios nutritivos más
simples que puedan ser absorbidos y utilizados por nuestro organismo. Está formado por:
– Un largo tubo muscular que comienza en la boca y termina en el ano, dividido en varias
partes, cada una de ellas con una función diferente que son: boca, faringe, esófago, estó-
mago, intestino delgado e intestino grueso.
– Glándulas, situadas fuera del tubo digestivo que segregan jugos digestivos. Son las sali-
vares, hígado y páncreas.

11
-Pág.95-
C I E N C I A S D E L A N AT U R A L E Z A

Sistema digestivo

La cavidad bucal
Está limitada a los lados por las encías, dientes y mejillas. En el suelo está la lengua y en la
parte superior el paladar (que separa la cavidad bucal de la cavidad nasal).
La lengua, dientes y glándulas salivares intervienen en la ingestión y digestión. Los dientes se
encajan sobre las mandíbulas. Son de cuatro tipos: incisivos, caninos, premolares y molares (mue-
las). Los adultos poseen 32 piezas dentarias, distribuídas simétricamente sobre las mandíbulas.
Así, en la parte anterior se sitúan 4 incisivos, que son cortantes, 1 canino a cada lado, para desga-
rrar los alimentos, seguidos de dos premolares y tres molares, con función trituradora. Durante la
infancia se desarrolla la llamada dentadura de leche, que carece de molares y que cuenta, por tanto
con 20 piezas, hacia los 7 años de edad es sustituida por la dentadura definitiva.
La faringe
Es un lugar coincidente de las vías respiratorias y digestivas. Los alimentos tras abandonar
la cavidad bucal se introducen en la cavidad inmediata que es la faringe. En sus paredes latera-
les se abren las trompas de Eustaquio.
El esófago
Es la parte del tubo digestivo comprendida entre faringe y estómago. Desciende entre los
pulmones y atraviesa el diafragma. Sus paredes musculares están muy bien desarrolladas (de
músculo estriado en el tercio superior y de músculo liso en los dos tercios inferiores. En el esó-
fago se originan ondas peristálticas que impulsan el bolo alimenticio hasta el estómago. Existe
una separación (esfínter) entre esófago y estómago llamado cardias que se abre al llegar el bolo
alimenticio.

12
-Pág.96-
U . D . 7 . - L AS PERSONAS Y LA SALUD

El estómago
Es una zona dilatada del tubo digestivo de paredes muy gruesas. Está limitado por dos esfín-
teres, cardiax, que lo separa del esófago y píloro, que lo separa del intestino delgado. En el estó-
mago se realiza la mezcla y digestión de los alimentos con la ayuda del jugo gástrico compuesto
por:
– Enzimas (el más importante es la pepsina, que digiere las proteínas).
– Ácido clorhídrico.
– Moco, que protege las paredes del estómago.
El intestino delgado
Tiene una longitud de 6 metros y está dividido en tres partes: duodeno, yeyuno e íleon.
En el intestino delgado se completa la digestión, para lo que se segrega el jugo intestinal
(rico en enzimas y moco). Además en el duodeno desembocan la bilis (de la vesícula biliar) y
el jugo pancreático (del páncreas). También se van produciendo fenómenos de absorción de los
nutrientes favorecidos por las vellosidades intestinales que son pliegues de la pared para aumen-
tar la superficie de absorción. El avance del quimo (así se llama la “papilla” que ha salido del
estómago) se realiza por movimientos peristálticos hasta llegar al colon.

El intestino grueso
Está formado por varias partes: ciego y apéndice vermiforme, colon ascendente, colon trans-
versal, colon descendente, colon pelviano, recto y ano.
El colon está dispuesto en el abdomen en forma de U invertida, es de paredes gruesas y tiene
gran diámetro. El intestino delgado desemboca de lado quedando un fondo de saco ciego del
que sale una prolongación que es el apéndice vermiforme. La principal función del colon es
transportar los desechos de la digestión hasta el recto y reabsorber agua. El peristaltismo se
mantiene, aunque de forma más lenta.

La pared del tubo digestivo


En general la pared de todo el tubo digestivo consta de tres capas o túnicas:
– Mucosa. La más interna. A nivel de estómago e intestino forma muchos pliegues para
facilitar la absorción.
– Submucosa.
– Muscular. De músculo liso, excepto el tercio superior del esófago que son fibras estriadas.

Por debajo del diafragma, hay que añadir una cuarta capa, el peritoneo, de naturaleza serosa.

Las glándulas salivares


Este término engloba los tres pares de glándulas mayores:
– Parótida. En la mejilla, por delante del conducto auditivo.

13
-Pág.97-
C I E N C I A S D E L A N AT U R A L E Z A

– Submaxilar. Situada por delante del ángulo de la mandíbula.


– Sublingual. Situada en el suelo de la boca.
La secreción mezclada de estas glándulas es la saliva. Diariamente se segregan 1,5 litros. La
digestión de los alimentos empieza en la boca gracias a los enzimas contenidos en la saliva que son:
– Ptialina o amilasa salivar. Desdobla el almidón en maltosa.
– Maltasa. Desdobla la maltosa en glucosa.
Además la saliva realiza otras funciones importantes:
– Lubrifica el bolo alimenticio facilitando su deglución.
– Limpia la mucosa bucal de restos de alimentos.
– Lubrifica y humedece la mucosa bucal y los labios.

El hígado
Es la glándula más voluminosa del organismo, situada debajo del diafragma, en la parte
derecha del abdomen. Interviene en la mayoría de los procesos metabólicos del organismo y
resulta de gran importancia en los procesos de digestión por la secreción de la bilis, que se
forma en toda la glándula hepática y se va colectando hasta llegar a un conducto principal que
desemboca en la vesícula biliar donde se almacena hasta que se necesita.

El páncreas
Es una glándula voluminosa situada en la parte media del abdomen. Su función es doble,
actuando como glándula exocrina produciendo el jugo pancreático, y como glándula endocrina,
produciendo insulina y glucagón. El jugo pancreático se vierte en el duodeno y contiene impor-
tantes enzimas para la digestión.

2.1.1.2. APARATO RESPIRATORIO


Es el encargado de realizar el intercambio de gases entre el medio ambiente y la sangre. Las
diferentes partes que constituyen el aparato respiratorio son las siguientes:
– Fosas nasales: Se abren por su parte anterior a través de las ventanas nasales y por su parte
posterior a través de las coanas.
– Faringe: Conducto donde se entrecruzan el aparato respiratorio y el digestivo.
– Laringe: Porción del tubo donde se une la faringe a la tráquea. Contiene las cuerdas voca-
les. En la parte superior se encuentra la epiglotis que hace de tapadera, impidiendo el paso
de líquidos y sólidos a las vías respiratorias.
– Tráquea: Estructura cartilaginosa abierta en su parte posterior, dividida en su parte baja
en dos bronquios.

14
-Pág.98-
U . D . 7 . - L AS PERSONAS Y LA SALUD

– Bronquios: Penetran en el interior


del pulmón y se dividen sucesiva-
mente en conductos de menor cali-
bre hasta constituir los sacos alveo-
lares que terminan en los alvéolos.
Estos alvéolos están en contacto ínti-
mo con los capilares pulmonares,
realizándose aquí el intercambio
gaseoso.
– Pulmones: Son dos órganos con
forma de semicono situados en su
respectivo hemitórax que descansan
en el diafragma. Están separados por
el mediastino, donde está contenido
el corazón y los grandes vasos
(aorta, cavas y arteria y venas pul-
monares). Los pulmones está dividi-
dos en lóbulos (dos en el izquierdo y
Anatomía del aparato respiratorio tres en el derecho).
Cada pulmón está rodeado por una
membrana serosa llamada pleura, con dos hojas, entre las que existe una cavidad (cavidad pleu-
ral), rellena de una ligera cantidad de líquido. En la tráquea y bronquios, existen células que
segregan moco y células ciliadas, que están en continuo movimiento impidiendo la entrada de
sustancias extrañas.
La respiración
La respiración tiene por objeto procurar una cantidad suficiente de oxígeno a la sangre y una
eliminación de anhídrido carbónico de la misma. Para lo cual se realiza:
– Paso del aire desde el exterior a través de los bronquios hasta los alvéolos pulmonares
(ventilación).
– Paso de sangre por los capilares sanguíneos, que están en estrecho contacto con los alvé-
olos (perfusión).
– Intercambio de gases entre alvéolos y sangre (difusión).
– Distribución de gases a través de la sangre a todos los tejidos periféricos.
Para que la ventilación sea posible, se realizan los movimientos respiratorios:
– Inspiración: Los pulmones se llenan de aire por contracción de los músculos intercostales
y del diafragma, y hace aumentar el volumen pulmonar.

15
-Pág.99-
C I E N C I A S D E L A N AT U R A L E Z A

– Espiración: Se expulsa el aire al exterior. Es un movimiento pasivo por relajación de los


músculos intercostales y del diafragma.
En el ser humano adulto, la frecuencia respiratoria es de 12 a 18 veces por minuto.

2.1.1.3. SISTEMA CIRCULATORIO


Es el encargado de abastecer a todos los tejidos de principios nutritivos y oxígeno, así como
de liberar a dichos tejidos de los productos de desecho. Consta de cuatro partes principales:
– Corazón (actúa como bomba,
impulsando la sangre).
– Arterias (encargadas de distribuir
la sangre).
– Capilares (donde se realizan los
intercambios entre la sangre y los
tejidos).
– Venas (encargadas de devolver la
sangre al corazón).
A.I.: Aurícula izquierda. El corazón
A.C.: Aurícula derecha.
Se encuentra situado entre los dos
V.I.: Ventrículo izquierdo.
V.D.: Ventrículo derecho.
pulmones y descansando sobre el
Esquema de la circulación humana
diafragma. Internamente es un órga-
no muscular hueco, dividido en cua-
tro cavidades, dos aurículas y dos ventrículos, existiendo una tabicación completa entre las aurí-
culas (tabique interauricular) y entre los ventrículos (tabique interventricular). Existe
comunicación entre las aurículas con sus respectivos ventrículos mediante válvulas llamadas
tricúspide (la del lado derecho del corazón) y mitral o bicúspide (la del lado izquierdo).
Las paredes están formadas por tres capas:
– Externa o pericardio. Está formada por dos hojas que deja una cavidad (cavidad pericárdica)
que está rellena de un líquido que lubrifica y permite el libre movimiento del corazón.
– Media o miocardio. Es la más importante y forma casi todo el espesor de la pared cardía-
ca, se denomina músculo cardíaco y está más engrosado en las paredes de las aurículas
que en las de los ventrículos.
– Interna o endocardio. Es una fina membrana que permanece en contacto íntimo con la
sangre y su función es evitar la coagulación de ésta en el corazón, cualquier lesión en esta
capa puede dar lugar a la formación de trombos.
Funcionalmente distinguimos dos partes, el corazón derecho (formado por la aurícula y el
ventrículo derechos) y el corazón izquierdo (formado por la aurícula y el ventrículo izquierdos).
El corazón derecho recoge la sangre procedente de todos los tejidos (es sangre venosa), entra

16
-Pág.100-
U . D . 7 . - L AS PERSONAS Y LA SALUD

a la aurícula derecha procedente de las dos venas cava (superior e inferior), de ahí pasa al ven-
trículo derecho y a través de la válvula semilunar (o sigmoidea pulmonar) que separa al ventrí-
culo derecho de la arteria pulmonar es impulsada por la arteria pulmonar a los pulmones para
su oxigenación (sigue conduciendo sangre venosa procedente de los tejidos). Habiéndose des-
prendido del dióxido de carbono y habiendo tomado el oxígeno, se convierte en sangre arterial
(oxigenada). La sangre vuelve desde los pulmones a la parte izquierda del corazón, a la aurí-
cula por las cuatro venas pulmonares, al ser impulsada pasa al ventrículo izquierdo y de ahí a
la arteria aorta que a través de sus ramas distribuye sangre oxigenada a todos los tejidos. Entre
el ventrículo izquierdo y la arteria aorta se interpone la válvula semilunar o sigmoidea aórtica.

Podemos decir que existen dos circuitos:

– Circulación mayor, que parte del ventrículo izquierdo y a través de la arteria aorta dis-
tribuye sangre oxigenada (arterial) a todos los tejidos a nivel de los capilares donde la
hemoglobina cede el oxígeno y toma el dióxido de carbono (CO2), transformándose en
sangre venosa, que se va colectando a través de las venas, hasta llegar a las venas cava
por donde desemboca de nuevo en el corazón en la aurícula derecha.

Principales arterias del cuerpo humano Principales venas del cuerpo humano

17
-Pág.101-
C I E N C I A S D E L A N AT U R A L E Z A

– Circulación menor, que recorre el camino de la oxigenación a través de los pulmones.


(aurícula derecha/ventrículo derecho/arteria pulmonar/pulmones/venas pulmonares/
aurícula izquierda).
El corazón impulsa la sangre a través de movimientos cíclicos que son de contracción (sís-
tole) y de relajación (diástole). En la contracción auricular, la sangre pasa de las aurículas a los
ventrículos; al llenarse los ventrículos (con las aurículas contraidas) se produce la contracción
de los ventrículos, impulsando la sangre hacia las arterias pulmonar y aorta respectivamente
cada ventrículo. Después se produce la relajación (diástole general) y empieza de nuevo el ciclo.
A su vez, como cualquier otro órgano del cuerpo, el corazón esta irrigado por el sistema cir-
culatorio, a través de las arterias y venas coronarias. Las arterias coronarias se originan a partir
de la arteria aorta, inmediatamente después de salir del ventrículo izquierdo. Las venas corona-
rias, tienen un recorrido similar a las arterias coronarias, pero en sentido inverso y desembocan
a través del seno coronario en la aurícula derecha.
Las arterias
Son los conductos encargados de llevar sangre desde el corazón a los diferentes tejidos. Sus
paredes están formadas por tres capas: una íntima (endotelio) en contacto directo con la sangre;
una media, con fibras musculares lisas que permiten la contracción y dilatación de las paredes
y una externa, formada por tejido conectivo.
Las venas
Son los conductos encargados de recoger la sangre procedente de los distintos tejidos y lle-
varla de nuevo al corazón para su oxigenación. No poseen capacidad de contractilidad como
sucede con las arterias impidiéndose el retorno venoso por unas válvulas existentes en las venas
más importantes en el sentido de avance de la sangre.
Los capilares
Son finísimos conductos a nivel de los tejidos donde se producen los intercambios entre la
sangre oxigenada y la desoxigenada.

2.1.1.4. EL APARATO EXCRETOR


El aparato excretor o urinario es el encargado de librar a la sangre de todos los productos de
desecho, eliminándolos al exterior a través de la orina, formada en los riñones. Consta de dos
riñones, dos uréteres, la vejiga urinaria y la uretra.
Los riñones son un par de órganos con forma de habichuela situados detrás del peritoneo, a
ambos lados de la columna vertebral, en la cara interna de cada riñón hay una cámara en forma
de embudo que es la pelvis renal. En un corte longitudinal distinguimos dos partes:
– La corteza (externa).
– La médula (interna).
La unidad funcional del riñón es la nefrona de la que existen un millón en cada riñón y está
formada por el glomérulo y el sistema tubular. El riñón es uno de los órganos mejor irrigados,

18
-Pág.102-
U . D . 7 . - L AS PERSONAS Y LA SALUD

Anatomía del aparato excretor

ya que recibe no sólo la sangre necesaria para su nutrición, sino que constantemente circula por
él toda la sangre de nuestro organismo.
La orina segregada constantemente por cada riñón es recogida por la pelvis renal, de ahí pasa
a los uréteres para llegar a la vejiga urinaria, donde se acumula hasta ser eliminada por la uretra.
Los riñones forman la orina a partir de la sangre que les llega. En los capilares del glomé-
rulo se filtra el plasma, el producto filtrado es recogido en el espacio capsular, de ahí pasa a tra-
vés de los túbulos renales donde la orina es concentrada. La orina formada en un adulto sano
oscila entre 1 y 1,5 litros al día.
La composición es agua (90-95%), urea (20-33 g), amoníaco (1 g), ácido úrico (0,5-1 g),
cloruros, fosfatos, oxalatos, etc.

2.1.2. ENFERMEDADES DE LOS APARATOS Y SISTEMAS IMPLICADOS


EN LA NUTRICIÓN

2.1.2.1. ENFERMEDADES DEL APARATO DIGESTIVO


Caries dental. Consiste en la destrucción de las envueltas duras del diente, de modo que se
originan orificios que progresan hacia la pulpa dentaria y producen un intenso dolor. La causan
las bacterias que se alimentan de los restos azucarados, retenidos entre los dientes produciendo
un ácido capaz de corroer el esmalte dental. El mejor método de lucha contra la caries es la
higiene mediante el cepillado de los dientes tras las comidas y el empleo de hilo de seda dental.
Gastritis. Es una inflamación de la mucosa gástrica. Se debe a comidas copiosas o muy con-
dimentadas, alcohol, fármacos, gripe, etc. Asociados se observan otros síntomas como son náu-
seas, vómitos, dolor en la región gástrica, a veces fiebre,...

19
-Pág.103-
C I E N C I A S D E L A N AT U R A L E Z A

Ulcus péptico o úlcera gástrica o duodenal. Consiste en una pérdida de sustancia a nivel
de la pared del estómago o del duodeno. Es una enfermedad que evoluciona a brotes y tiene un
curso crónico. Afecta sobre todo a personas nerviosas o en situaciones de estrés.
Apendicitis. Es la inflamación del apéndice vermiforme por causas bacterianas. Comienza
bruscamente con dolor abdominal continuo que aumenta con la presión, al toser o al moverse,
náuseas, vómitos y fiebre no muy elevada. Si no se interviene a tiempo, hay perforación del
apéndice con la consiguiente producción de peritonitis (inflamación del peritoneo).
Hepatitis. Es una inflamación aguda del hígado producida por virus, siendo contagiosa.
Existen cuatro tipo de hepatitis denominadas con las letras A, B, C y E. Las hepatitis pueden
evolucionar hacia una degeneración hepática (cirrosis) y cáncer.
Cirrosis hepática. Es una enfermedad difusa, crónica, progresiva e irreversible del hígado,
con lesiones hepáticas importantes desde el punto de vista anatomo-patológico. Es debido al
alcoholismo, a hepatitis vírica o a infecciones biliares.
Estreñimiento. Se caracteriza por el movimiento lento de las heces a través del colon, lo
que provoca una mayor absorción del agua con el consiguiente endurecimiento de las heces, lo
que dificulta y retarda su expulsión facilitando la formación de fisuras anales y hemorroides. Se
deben a una dieta pobre en fibra que sirve de arrastre y a una falta de ejercicio regular.

2.1.2.2. ENFERMEDADES DEL APARATO RESPIRATORIO


Neumonía. Es una infección de los pulmones originada por microorganismos.
Edema pulmonar. Consiste en la inundación de los pulmones con plasma sanguíneo, lo que
ocasiona su inutilidad por encharcamiento.
Enfisema pulmonar. Es una enfermedad habitual en personas fumadoras. Las sustancias
del tabaco hacen que los alvéolos se lesionen, hinchándose hasta romperse, con lo que se redu-
ce notablemente la superficie de intercambio gaseoso.

2.1.2.3. ENFERMEDADES DEL SISTEMA CIRCULATORIO


Arterioesclerosis. Consiste en la pérdida de elasticidad de las arterias, que se vuelven rígidas,
duras y reducen su luz. Conlleva una disminución en la nutrición de los tejidos y un aumento de la
tensión arterial (no siempre). Se favorece con las dietas ricas en colesterol (grasas animales), dia-
betes, obesidad, sedentarismo y tabaco. Con el tiempo los síntomas que aparecen son somnolencia,
pérdida de memoria, alteraciones del carácter, tendencia a padecer cardiopatía isquémica, etc.
Hipertensión arterial. Consiste en un aumento de presión en las arterias (por encima de
150 mm de Hg de máxima y 90 mm de Hg de mínima, tomada por la mañana, en ayunas y con
el paciente tumbado). Cuando se eleva mucho, puede originar cefaleas matutinas, hemorragias
nasales, palpitaciones, congestión en la cara, sobrecarga cardíaca, etc.
Insuficiencia coronaria o cardiopatía isquémica. Las lesiones en los vasos coronarios ori-

20
-Pág.104-
U . D . 7 . - L AS PERSONAS Y LA SALUD

ginan falta de riego del propio tejido cardíaco, originando enfermedades que se engloban bajo
el término de cardiopatía isquémica. Los factores que predisponen al padecimiento de esta
isquemia son los altos niveles de colesterol en sangre, la obesidad, la hipertensión, el consumo
de tabaco, la diabetes, la tensión emocional, etc. Se manifiesta con los siguientes cuadros:
– Angina de pecho. Es una falta de riego coronario que aparece de forma aguda y es de corta
duración. Se debe a la arterioesclerosis coronaria. Se manifiesta por la aparición súbita de
dolor de tipo opresivo, localizado en la región retroesternal (el “área de la corbata”), que
puede irradiarse bilateralmente alrededor del pecho y a ambos hombros, descendiendo
por los brazos hasta las muñecas (puede ser a un solo brazo que suele ser el izquierdo).
El dolor aparece tras el esfuerzo, dura 2-3 minutos y desaparece con el reposo.
– Infarto de miocardio. Es la forma más grave de la cardiopatía isquémica. Se debe a la arte-
rioesclerosis y obstrucción por trombo o émbolo. Consiste en la necrosis de una parte del
músculo cardíaco por una falta de riego por parte de las coronarias. Se manifiesta por un
dolor igual que en la angina de pecho, pero su duración es mayor y no cede con el reposo.
Tampoco su aparición guarda relación obligada con el esfuerzo como la angina de pecho.

2.1.2.4. ENFERMEDADES DEL APARATO EXCRETOR


Las enfermedades más conocidas del aparato excretor son:
Litiasis renal. Consiste en la presencia de cálculos en el riñón o en las vías urinarias, debi-
do a la precipitación de sustancias que en condiciones normales están disueltas en la orina. Estas
sustancias suelen ser fosfatos, oxalato cálcico y uratos (sales del ácido úrico). El tamaño es
variable, cuando son muy pequeños se habla de arenilla.
Cuando se manifiesta, ocasiona el llamado cólico renal o nefrítico. Se presenta un dolor
intenso localizado en la región lumbar irradiado. La duración de los cólicos es muy variable
desde horas hasta varios días. A veces al finalizar el cólico puede observarse la eliminación de
arenillas o un pequeño cálculo con la orina.
Cistitis. Es la inflamación de la vejiga urinaria. Generalmente es de causa infecciosa.
Insuficiencia renal. Es un estado que aparece cuando el riñón es incapaz de realizar su tra-
bajo o lo realiza de forma incompleta, como consecuencia hay un acúmulo en sangre de dichos
productos.

2.1.3. CLASIFICACIÓN DE LOS ALIMENTOS


Los alimentos se dividen según su función en energéticos (producen energía necesaria para el
trabajo del organismo, son los hidratos de carbono y las grasas), plásticos (intervienen en la
reconstrucción y reparación de las diversas partes del organismo (proteínas y calcio) y regulado-
res (regulan el funcionamiento del metabolismo, son las vitaminas, minerales y el agua).
Los alimentos se pueden clasificar en siete grupos según la composición. En la tabla 1 se
exponen los alimentos, el valor nutritivo que poseen y la función que realizan por grupos.

21
-Pág.105-
C I E N C I A S D E L A N AT U R A L E Z A

Valor nutritivo Alimentos Función


GRUPO 1 Ricos en proteínas. Ricos Leche (de vaca, de Plástica.
Leche y derivados en grasas. Contienen vita- cabra, de oveja,...).
minas A, B1, B2 y D. Derivados de la leche:
Ricos en Ca, Mg y K. queso, yogur, etc.
GRUPO 2 Alto contenido en proteí- Carnes (de ave, de caza, Plástica.
Carnes, pescados nas. Contienen vitaminas etc.). Derivados de la
y huevos del grupo D y sales mine- carne: embutidos. Pes-
rales (Fe y Zn). cados. Huevos.

GRUPO 3 Alto contenido en glúci- Legumbres secas: ju- Energética,


Legumbres, dos y en proteínas. Ricos días, garbanzos, lente- reguladora y
tubérculos en vitamina C (patata). jas, etc. Patatas. Frutos plástica.
y frutos secos desecados. Frutos secos
oleaginosos.
Alto contenido en agua Verduras: lechuga, acel- Reguladora.
GRUPO 4
(75-95%). Ricas en glúci- gas, espinacas, coliflor,
Hortalizas
dos (10%) y pobres en alcachofas, etc. Raíces:
y verduras
proteínas. Bajo contenido zanahorias, nabos...
en sales y alto en fibra Bulbos: cebollas.
vegetal. Ricas en vitami-
nas A y C.
GRUPO 5 Alto contenido en vitami- Frutas: naranjas, limón, Reguladora.
Frutas y derivados nas y agua. Bajo contenido manzana, pera, meloco-
en sales minerales: Ca, Cu, tón, uvas, tomate, etc.
Mg, Fe, etc., y alto conte-
nido en fibra dietética.

GRUPO 6 Ricos en glúcidos. Apro- Trigo (pan). Arroz. Energética.


Cereales, pastas ximadamente 10% de pro- Maíz. Mijo, cebada,
y azúcar teínas. Vitamina B. Mine- avena, centeno.
rales: K, P, etc.
GRUPO 7 Alto contenido en lípidos. Líquidos a temperatura Energética.
Grasas y aceites Ricos en vitamina A. ambiente: aceites (oliva,
girasol, soja, etc.).
Sólidos a temperatura
ambiente: manteca,
sebos, tocino, mante-
quilla, etc.

Tabla 1. Los grupos de alimentos con sus nutrientes, ejemplos y función general.

22
-Pág.106-
U . D . 7 . - L AS PERSONAS Y LA SALUD

2.1.4. SUSTANCIAS NUTRITIVAS

2.1.4.1. HIDRATOS DE CARBONO, GLÚCIDOS O AZÚCARES


Están compuestas por carbono, oxígeno e hidrógeno.
Son atacados por los enzimas digestivos degradándolos hasta glucosa que entra en las célu-
las en unión de la insulina (es una hormona producida en el páncreas). Es esencialmente ener-
gético (1 gramo de glúcidos produce unas 4 kilocalorías). La energía no utilizada se almacena
en depósitos grasos.
Alimentos ricos en hidratos de carbono son las pastas, el arroz, el pan, las mermeladas, el
azúcar, las legumbres, ...
Se precisa que entre un 55-60% de las necesidades de la ingesta diaria esté proporcionada
por los hidratos de carbono.
Existe un grupo importante de azúcares complejos que el organismo no es capaz de digerir,
es decir que no posee los enzimas para degradarlos y atraviesan el tubo digestivo sin ser des-
compuestos, a estos hidratos de carbono se les denomina fibra alimentaria.

2.1.4.2. GRASAS O LÍPIDOS


Están compuestas por carbono, oxígeno e hidrógeno. En algunos casos se asocian fósforo,
nitrógeno, ...
La principal función de las grasas es energética (1 gramo de grasa proporciona unas 9 kilo-
calorías) y plástica (forma parte de las estructuras celulares). Son indispensables para la asimi-
lación de las vitaminas liposolubles.
La mantequilla, los aceites, los embutidos, los pescados azules..., son muy ricos en grasa.
Hay que diferenciar entre las grasas de origen animal, ricas en ácidos grasos saturados y coles-
terol de las grasas de origen vegetal, ricas en ácidos grasos no saturados como es el ácido oléi-
co presente en el aceite de oliva y en general más beneficiosas para la salud.
Entre un 25-30% de las necesidades calóricas diarias se deben cubrir con grasas.

2.1.4.3. PROTEÍNAS
Están compuestas por carbono, hidrógeno, oxígeno y nitrógeno. Están formadas por unida-
des llamadas aminoácidos que los podemos dividir en:
– Esenciales: Es necesario tomarlos de los alimentos porque el organismo no puede sinte-
tizarlos.
– No esenciales: No es necesario ingerirlos porque el organismo es capaz de sintetizarlos.
Su función es fundamentalmente plástica, ya que construye y repara las células desgastadas,
aunque también tiene función energética (1 gramo proporciona aproximadamente 4 kilocalorías).

23
-Pág.107-
C I E N C I A S D E L A N AT U R A L E Z A

Son ricos en proteínas las carnes, los pescados, la leche y los huevos (todas ellas ricas en
todos los aminoácidos esenciales, por lo que decimos que son proteínas de alto valor biológico)
y las legumbres y cereales (ricas en proteínas, aunque no poseen todos los aminoácidos esen-
ciales) y es conveniente tomarlas conjuntamente.
Aproximadamente un 15% de las necesidades diarias deben cubrirse con proteínas (60-80
gramos al día en adultos).

2.1.4.4. VITAMINAS
Son sustancias químicas de composición variable necesarias para la correcta utilización de
los hidratos de carbono, proteínas y lípidos que deben tomarse exógenamente porque el orga-
nismo no es capaz de sintetizarlas. En la tabla 2 se presentan distintas vitaminas con la dosis
necesaria, función y alimentos que la contienen.
Se dividen en dos grupos:
– Liposolubles: Solubles en grasas (Vitaminas K, E, A y D).
– Hidrosolubles: Solubles en agua (Todas las demás: B, C, P, ...).

Vitaminas Fuente Trastornos por Ingesta recomendada Necesidades


alimentaria deficiencia diaria (adultos) de suplemento

A (retinol) Leche, hígado de Ceguera noctur- 0,75-1 mg Niños


pescado, aves. na, alteración de
las mucosas.

B1 (tiamina) Carne de cerdo, Beri-beri. 1,2-1,5 mg Alcohólicos


cereales y legum-
bres.

B2 Hígado, huevos, Alteraciones de 1,5-1,8 mg Niños


(riboflavina) leche y frutos la mucosa y piel.
secos.

B3 (ácido nico- Vísceras, pesca- Pelagra. 13-20 mg Niños y


tínico o vit. PP) dos y cereales. adolescentes

B12 (cianoco- Vísceras, carnes, Anemia perni- 2,5-5 mg Vegetarianos


balamina) pescados y huevos ciosa estrictos

D (calciferol, Arenque, salmón, Raquitismo en 2,5 mg Niños y


colecalciferol) sardina, yema de niños. Osteoma- embarazadas.
huevo y leche lacia en adultos.

Tabla 2. Principales vitaminas que deben ser ingeridas en la dieta.

24
-Pág.108-
U . D . 7 . - L AS PERSONAS Y LA SALUD

2.1.4.5. MINERALES
Los más importantes son el calcio, el hierro y el iodo.
El calcio, es muy importante en el metabolismo óseo y en la coagulación de la sangre. Está
presente en la leche y derivados. Las necesidades diarias son de 600-1400 miligramos, según la
edad y circunstancias de la persona.
El hierro, es fundamental en la formación de la hemoglobina. Está presente en carnes rojas,
huevos, legumbres y frutos secos. Las necesidades diarias están entre 10-12 miligramos.
El iodo, es fundamental para que la glándula tiroides produzca sus hormonas. Está presen-
te en pescados, crustáceos, vegetales, agua potable, fruta, etc. Nuestras necesidades se sitúan en
150 microgramos.

2.1.4.6. AGUA
Es indispensable para que las sustancias nutritivas se disuelvan y sean transportadas hasta
las células. Hay que tomar entre 2-2,5 litros al día.

2.2. LOS HÁBITOS ALIMENTICIOS Y LA RELACIÓN CON LA SALUD


La evolución del hombre va íntimamente ligada a la historia de la alimentación. Ya el hom-
bre paleolítico dependía de la pesca y de la caza para subsistir.
En los países industrializados algunos de los factores que inciden sobre nuestra salud son:
– Mejora del nivel de vida. En el mercado aparecen cada día nuevos y numerosos produc-
tos que en definitiva favorecen la sobrealimentación.
– El progreso en los medios de transporte de mercancías y el gran desarrollo de las técnicas
de conservación facilitan el acceso a una mayor variedad de alimentos durante todo el año.
– La proliferación de alimentos de síntesis y refinado, favoreciendo como consecuencia del
uso regular de las mismas la aparición de ciertas enfermedades: diverticulosis intestinal,
estreñimiento, etc.
– Papel exagerado de la publicidad, desorientando en muchos casos al consumidor, inci-
tando al gasto indiscriminado.
– Limitación de las tradiciones en el comportamiento alimentario.
– Cambios de la forma de vida, con una mayor tendencia al sedentarismo.
Nuestra dieta, como consecuencia de estas características, favorece la aparición de ciertas
alteraciones: obesidad, enfermedades cardiovasculares, diabetes, cuadros carenciales clínicos o
subclínicos de vitamina A, B2, C, D, hierro, calcio, etc.

25
-Pág.109-
C I E N C I A S D E L A N AT U R A L E Z A

Estos problemas se podrían reducir:


– Aumentando el consumo de leche, verduras, legumbres, cereales y féculas (controlando
el incremento de aporte calórico).
– Disminuyendo la ingesta de alimentos ricos en grasas, sobre todo saturadas, consumien-
do preferentemente aceites vegetales.
– Controlar la utilización de sal, azúcar y condimentos.
– Limitar el consumo de proteínas sobre todo las de origen animal.
– Evitar alimentarse con dietas de moda, para seguir los patrones sociales.

2.3. DIETA SALUDABLE Y EQUILIBRADA


Es aquella que contiene las sustancias alimenticias indispensables para cubrir los requeri-
mientos del organismo diariamente (a nivel energético, plástico y regulador) adaptadas a la
edad, actividad física y situaciones especiales (embarazo, enfermedades, ...).
Para conseguir una dieta sana, hay que transmitir unos buenos hábitos de alimentación con-
sistentes en:
– Disminuir el consumo de grasas de origen animal.
– Aumentar el consumo de grasas de origen vegetal.
– Disminuir el consumo de glúcidos.
– Sustituir harinas refinadas por integrales.
– Aumentar el consumo de leche y derivados.
– Aumentar el consumo de frutas y verduras frescas.
– No abusar de fritos.
– Repartir la ingesta diaria de modo que no se concentre todo en una comida.
– Reducir el consumo de alcohol, picantes, gaseosas y café.
– Comer sosegadamente, masticando repetidamente cada bocado.
La dieta que ingerimos debe satisfacer diariamente:

– El metabolismo basal. Representa la energía consumida para cumplir las funciones vege-
tativas. Este gasto suponen unas 1500 kcal (kilocalorías) al día en el adulto.

– Actividad física y trabajo. Clasificada en ligera (oficinista, maestros, ...), mediana (pes-
cadores, agricultores...,) y actividad intensa (mineros, deportistas, tropa, ...). Se consumen
unas 500 Kcal si la actividad es ligera, 800 Kcal en la actividad mediana y 1500-2000 Kcal
en la intensa.

26
-Pág.110-
U . D . 7 . - L AS PERSONAS Y LA SALUD

– Acción dinámico-específica (ADE). La ingestión de los alimentos determina un gasto


energético, debido a la absorción, digestión y asimilación de los nutrientes. (entre un
5-10%).

– Importancia de la dieta equilibrada. Tiene gran importancia en la salud de la


población porque:
- Contribuye al desarrollo físico y al crecimiento.
- Disminuye la mortalidad y la morbilidad.
- Aumenta el rendimiento laboral.
- Favorece el rendimiento intelectual, especialmente en la edad escolar.

En conjunto, la dieta que se ingiere, debe aportar, como valor de referencia unas 40 kcal por
kilogramo de peso corporal y día, lo que suponen unas 3000 Kcal/día en el hombre y unas 2200-
2500 Kcal/día en el caso de la mujer. Adultos y con una actividad moderada en ambos casos.
En la edad escolar, estas necesidades son mayores aumentando hasta 50 kcal por kilogramo de
peso corporal y día.

Es importante utilizar gran variedad de alimentos pues es la mejor manera de proveer al


organismo de los principios inmediatos (proteínas, azúcares y grasas), macronutrientes (sales
minerales que el organismo necesita en cantidades de al menos 100 mg/día y micronutrientes u
oligoelementos (son ciertos minerales necesarios en cantidades muy pequeñas) como hierro,
cobre, flúor, yodo, zinc, etc.

Para elaborar una dieta sana y equilibrada, se debe tener en cuenta la cantidad de proteínas
(15% del total), grasas (30% del total) e hidratos de carbono (el resto hasta el 100%) que se
deben tomar y ajustar la cantidad de alimentos en función de las calorías que se deben tomar
por la edad, situación nutricional y sexo.

2.3.1. TIPOS DE DIETA


En la actualidad existen numerosos tipos de dieta, debiendo diferenciar entre dietas para
individuos sanos y dietas terapéuticas (cuando existe algún problema médico).

2.3.2. DIETAS PARA INDIVIDUOS SANOS


Podemos considerar entre otras:

Dieta mediterránea. Se trata de una dieta muy rica en nutrientes y variada que surge en
la ribera del Mediterráneo, donde por el sabor intenso de los alimentos no se adiciona
demasiada sal. Se basa en el consumo de cereales, legumbres, frutas y hortalizas a las
que se le añaden pequeñas cantidades de carne blanca y pescado y elaboradas con acei-
te de oliva.

27
-Pág.111-
C I E N C I A S D E L A N AT U R A L E Z A

Dieta vegetariana. En ella quedan excluidos los alimentos de origen animal. Existe una
variedad que puede consumir leche y huevos por lo que se le conoce como ovo-lacto-vege-
tariana. Al ser las proteínas vegetales de menor valor biológico que las animales y la
ausencia de vitamina B, pueden ser causa de anemias y alteraciones del sistema nervioso.

2.4. PRINCIPALES ENFERMEDADES DEL METABOLISMO. LA OBESIDAD


Aunque son innumerables las enfermedades metabólicas, nos vamos a centrar en tres, la dia-
betes glucosúrica, la gota úrica y la obesidad.

Diabetes glucosúrica. También conocida como diabetes mellitus o diabetes sacarina, es una
enfermedad del metabolismo de los hidratos de carbono debida a una alteración en la formación
de insulina. A falta de insulina, la glucosa no puede entrar en la célula para ser utilizada por lo
que hay un aumento de glucosa en sangre (hiperglucemia), originando mayor eliminación de
glucosa en orina (glucosuria) que a su vez arrastra una gran cantidad de agua, por lo que el
enfermo orina mucho (poliuria), generando una deshidratación por lo que se tiene mucha sed
(polidipsia). Al no utilizar la glucosa como fuente de energía, el enfermo tiene mucho apetito
(polifagia).

Existe mayor predisposición entre los obesos y las personas que llevan vida sedentaria y su
alimentación es rica en grasas. La herencia juega también un importante papel (un 25% de los
casos tienen antecedentes familiares). Se distinguen dos tipos, la diabetes juvenil, que aparece
antes de los 20 años (son enfermos insulino-dependientes) y la diabetes del adulto (aparece
hacia los 50 años y no es insulino-dependiente).

Gota. La gota es una enfermedad metabólica caracterizada por el metabolismo anormal del
ácido úrico. Es hereditaria, con una serie de factores que actúan como desencadenantes: alcohol,
alimentación rica en carnes y grasas, vida sedentaria y abuso del tabaco. El metabolismo anor-
mal del ácido úrico, provoca hiperuricemia (es decir niveles elevados de ácido úrico en sangre),
ocasionando una precipitación en los tejidos en forma de cristales, principalmente en las arti-
culaciones.

La primera manifestación es el ataque gotoso agudo que se suele presentar de noche en la


articulación del dedo gordo del pie, apareciendo esta zona hinchada, caliente y enrojecida; va
acompañada de cefalea, inapetencia, fiebre (38ºC), etc.

Obesidad. Es un aumento patológico del peso y volumen de un individuo. Se dice que exis-
te obesidad cuando se sobrepasa en más de un 15% el peso que correspondería a la talla y edad.
El peso ideal tradicionalmente se ha considerado que es la talla menos 100 para el hombre y
algo menos para la mujer.

Cuando la obesidad se instaura en la infancia, aumenta el número de adipocitos, mientras


que cuando se desarrolla la obesidad en edad adulta el número de adipocitos es normal, por lo
que será más fácil perder el sobrepeso en este segundo caso.

28
-Pág.112-
U . D . 7 . - L AS PERSONAS Y LA SALUD

Hay múltiples causas: predisposición constitucional, ingesta calórica excesiva, falta de ejer-
cicio físico y problemas endocrinos (por disminución de secreción de la hormona tiroidea,
aumento de secreción de corticoides y disminución de hormonas sexuales).

Los trastornos a los que se ven sometidos estas personas son: hipertensión arterial, arte-
rioesclerosis, disnea, infecciones en aparato respiratorio, gota, varices, diabetes, riesgos en el
embarazo, sobrecarga del corazón, etc., sin olvidar la gran relación existente entre la obesidad
y algunos trastornos psíquicos. Nos encontramos en una sociedad “antigordo” donde los mode-
los imperantes están basados en la delgadez, cayendo en otras enfermedades como la anorexia
(dejar de comer por considerar que se está muy gorda/o) y la bulimia (comer desmesuradamente
provocando después el vómito), ambas con una componente psíquica grande.

Para saber el grado de obesidad, se emplean diversos métodos, entre los que se encuentra la
Fórmula de Lorentz, que al no contemplar la constitución es válida para las personas de com-
plexión media.

Peso “en Kg”=(T cm-100)-[(Talla-150)/4 (en hombres) o 2 (en mujeres)]

2.5. LA CONSERVACIÓN, MANIPULACIÓN Y COMERCIALIZAY Y Y


Y COMERCIALIZACIÓN DE LOS ALIMENTOS

Según la OMS, es “el conjunto de medidas necesarias en la producción, elaboración, alma-


cenamiento y distribución de los productos alimenticios con el fin de obtener un producto en
buen estado, sano, inofensivo y conveniente para el consumo humano”.

Hoy en día se ha impuesto el sistema ARYCPC (análisis de riesgos y control de puntos crí-
ticos) a nivel de la industria, que consiste en ir analizando y estudiando la cadena seguida por
el alimento concreto a estudiar desde el origen hasta el consumidor analizando los posibles ries-
gos y estableciendo medidas para evitarlos.
En cualquier caso hay una serie de criterios a observar en todos los alimentos:
– Los alimentos no deben romper su cadena de conservación durante el transporte del
mismo (Ej.: los congelados no pueden ser transportados a temperatura ambiente).
– Los alimentos deben ser conservados a ser posible en refrigeración.
– No se deben mezclar productos crudos con productos cocinados.
– Los alimentos deben ser manipulados por personal en posesión del carnet de manipulador
de alimentos.
Nuestra salud depende no sólo de las propiedades de los alimentos que consumimos, pues
también influyen en ella las sustancias tóxicas o los microorganismos que pueden haber en

29
-Pág.113-
C I E N C I A S D E L A N AT U R A L E Z A

ellos, que pueden estar contaminados por bacterias, hongos o virus procedentes del agua, el aire,
las personas o los animales. Distinguimos:
– Las intoxicaciones, causadas por toxinas producidas en el alimento contaminado por bac-
terias (Ej.: intoxicación estafilococica, producida por la bacteria Staphylococcus aureus
que afecta al sistema nervioso) o por la ingestión de setas que las contiene y que en algu-
nos casos pueden llegar a la muerte o producir daños irreversibles.
– Las toxiinfecciones, producidas por gérmenes que liberan las toxinas en el alimento des-
pués de morir. Van a producir enfermedad en función de la cantidad ingerida. Los sínto-
mas son diarrea, dolor abdominal, fiebre... La salmonelosis es muy frecuente especial-
mente en los periodos calurosos.
– Las infestaciones, están causadas por un animal parásito que utiliza a otro animal como
vehículo para llegar hasta el ser humano. Entre los parásitos más frecuentes están la tenia
y la triquina que con una inspección veterinaria óptima no debían presentarse.

2.6. LAS PERSONAS COMO CONSUMIDORES


La calidad nutritiva de una dieta es primordial para nuestra salud. Ésta sin embargo, no
depende sólo de los alimentos que consumamos, sino también de las condiciones en que estos
nos lleguen.
Defectos en la manipulación o en la conservación permiten la contaminación por gérmenes y
que estos causen toxiinfecciones que pueden ser graves. También es posible que las propiedades
del alimento manufacturado no se correspondan con las del producto comercializado. Por todo
esto tanto la Legislación española como la comunitaria defienden los derechos de los consumido-
res de alimentos. Si nos encontramos ante un fraude, ante una incorrecta manipulación, etc.,
hemos de recurrir a los organismos oficiales autonómicos o locales o a las asociaciones de consu-
midores de la zona que prestan servicios de tramitación de reclamaciones y defensa jurídica.

3. LA REPRODUCCIÓN HUMANA

3.1. LA REPRODUCCIÓN HUMANA


Los seres humanos se reproducen sexualmente. Este mecanismo consiste en la fusión de dos
células o gametos, uno masculino (espermatozoide) y otro femenino (óvulo). De esta unión se
forma otra célula, el huevo o cigoto, que tras su desarrollo dará lugar a otro individuo.

30
-Pág.114-
U . D . 7 . - L AS PERSONAS Y LA SALUD

Los gametos se originan en los órganos sexuales llamados gónadas. Las gónadas masculi-
nas, que forman espermatozoides, se llaman testículos; y las gónadas femeninas, que forman
óvulos, se llaman ovarios.
En la mayoría de las especies animales existen dos tipos de individuos: las hembras, que
poseen gónadas femeninas, y los machos o varones, que tienen gónadas masculinas.
Los machos tienen sexo masculino y las hembras femenino.
El sexo es la condición orgánica que distingue a la hembra del macho.
Existen diferencias físicas y anatómicas entre el hombre y la mujer (varón y hembra) en
nuestra especie, a esta propiedad se le denomina dimorfismo sexual. Se aprecian por los carac-
teres sexuales secundarios, es decir, aquellas características externas que diferencian al hombre
de la mujer (las mamas, la anchura de cadera, la barba, ...). Estos caracteres secundarios no defi-
nen el sexo, pero son determinados por él. Los caracteres sexuales primarios, que definen los
sexos, son el desarrollo de las gónadas: los testículos en el hombre y los ovarios en la mujer.

3.2. LOS CAMBIOS CORPORALES A LO LARGO DE LA VIDA


El organismo desde que nace va cambiando, creciendo hasta alcanzar la madurez, sufriendo
cambios progresivos y aunque el sexo está determinado desde el nacimiento, es durante la
pubertad (adolescencia) cuando se desarrollan los caracteres secundarios, alcanzándose la
madurez sexual.

3.3. APARATO REPRODUCTOR MASCULINO

3.3.1. ANATOMÍA
Los testículos son dos glándulas externas, de forma ovoidea, que se encuentran fuera de la
cavidad del cuerpo, en un saco llamado escroto.
Cada testículo consta de varios conjuntos radiales de tubos productores de espermatozoides.
Estos tubitos se unen para dar lugar a los conductos espermáticos, que van a parar a otro tubo
muy largo y enrollado, llamado epidídimo, en el cual se almacenan los espermatozoides.
El epidídimo de cada testículo desemboca en un canal deferente, y ambos canales se unen a
la uretra en el mismo punto.
A cada conducto deferente vierte la vesícula seminal antes de penetrar en otra glándula
denominada próstata. Las vesículas seminales y la próstata segregan una sustancia líquida que,
junto con los espermatozoides, constituye el semen, que es el líquido fecundante.

31
-Pág.115-
C I E N C I A S D E L A N AT U R A L E Z A

Aparato reproductor masculino

En el hombre, la uretra se prolonga en el interior de un órgano copulador llamado pene, que


está formado en su interior por un tejido conjuntivo con muchos espacios sanguíneos pequeños,
que al llenarse de sangre se endurece permitiendo ser introducido en la vagina femenina.

3.3.2. PRODUCCIÓN DE ESPERMATOZOIDES


En el interior de los tubos productores de espermatozoides hay unas células, las espermo-
gonias, que se reproducen muy rápidamente, las cuales originan en su transformación los game-
tos masculinos, en un proceso denominado espermatogénesis.
Al principio, los espermatozoides están inmóviles, pero luego, gracias a las contracciones
del epidídimo, salen a la uretra. Al unirse a las sustancias segregadas por las glándulas semina-
les y la próstata, forman el semen. En el seno de este líquido, los espermatozoides se mueven
gracias a sus flagelos.

3.4. APARATO REPRODUCTOR FEMENINO

3.4.1. ANATOMÍA
Los ovarios son dos cuerpos ovales, de color crema, que se encuentran en el interior del
cuerpo de la mujer, debajo de los riñones.
Junto a cada ovario se abre un tubo en forma de embudo, el oviducto o trompa de Falopio,
adonde van a parar los óvulos que salen del ovario. Los oviductos son conductos estrechos y

32
-Pág.116-
U . D . 7 . - L AS PERSONAS Y LA SALUD

Aparato reproductor femenino de frente y de perfil

curvados que desembocan en una cavidad, el útero o matriz. Éste se comunica con el exterior
mediante otro tubo de paredes musculosas llamada vagina. El cuello del útero es un anillo mus-
culoso que lo cierra casi completamente en la zona donde se une con la vagina.

3.4.2. OVULACIÓN. EL CICLO FEMENINO


En el ovario hay gran cantidad de vasos sanguíneos y miles de células, llamadas oocitos, que
pueden madurar y convertirse en óvulos, en el proceso llamado ovogénesis. A lo largo de la vida
de una mujer, sólo unas 500 de estas células se desarrollarán y formarán óvulos maduros.
Los oocitos se encuentran rodeados de otras muchas células y abundantes vasos sanguíneos.
Este conjunto cerrado en forma de bolsita recibe el nombre de folículo (folículo de Graff).
La producción de óvulos se inicia entre los once y dieciséis años de edad, en el período de
vida llamado pubertad.
Cuando el folículo está maduro, se abre y deja salir al ovocito, que sufre ciertas transfor-
maciones y se convierte en óvulo. Este proceso se denomina ovulación.
El óvulo desprendido es recogido por el embudo del oviducto (trompa de Falopio) y se des-
plaza por él lentamente, hasta llegar al útero. Este recorrido puede durar unos tres días.
Los dos ovarios producen un óvulo de forma alternativa cada 28 días aproximadamente, el
cual madura y se desprende hacia el día 14 del ciclo.
El útero, el ovario y la hipófisis tienen una actividad cíclica desde la pubertad de la mujer
hasta el cese de la capacidad de ovular o menopausia.
a) El ciclo del útero se inicia con una hemorragia originada por la destrucción de la mucosa
que reviste interiormente la cavidad uterina, mucosa llamada endometrio. El primer día
del ciclo del útero, es el primer día de la regla o menstruación. Después de la menstrua-
ción se produce la regeneración y espesamiento de las paredes del útero. Este fenómeno
es una preparación para la anidación o implantación del óvulo, en caso de que éste haya

33
-Pág.117-
C I E N C I A S D E L A N AT U R A L E Z A

sido fecundado, como veremos más adelante. En caso negativo, a los 28 días se produci-
rá otra nueva hemorragia menstrual, comenzando de nuevo el ciclo.
b) El ciclo del ovario se caracteriza por la ovulación, que consiste en la maduración de un
folículo en uno de los ovarios. Está regulado por las hormonas hipofisarias (producidas
en la hipófisis) LH y FSH. El folículo se abre dejando libre al óvulo, que se traslada por
el oviducto hacia el útero. Tras el desprendimiento del óvulo, el folículo se transforma en
una pequeña bolita, el cuerpo lúteo, que se convierte en un importante productor hormo-
nal. Sus hormonas son la progesterona y los estrógenos.
c) El ciclo de la hipófisis es también de naturaleza hormonal. Esta pequeña glándula de la
base del cerebro, con la secreción de las hormonas gonadotropinas (LH y FSH), controla
el funcionamiento de los ovarios y especialmente la ovulación.

3.5. FECUNDACIÓN, EMBARAZO Y PARTO


En el acto sexual, copulación o coito, el pene del hombre introduce en la vagina de la mujer
los espermatozoides. Cuando éstos encuentran al óvulo en el oviducto tiene lugar la fecundación.
Los 200 o 300 millones de espermatozoides depositados durante la copulación, se desplazan
por la vagina y el útero hacia el oviducto. Si en éste último tramo encuentran un óvulo, gene-
ralmente sólo uno de ellos se fusiona con él, formando un cigoto o célula huevo.
Si por el contrario, no se produce la fecundación, el óvulo se fragmenta y muere en un plazo
de 24 a 48 horas, mucho antes de llegar a la matriz.

Situación del niño en los momentos previos al parto

34
-Pág.118-
U . D . 7 . - L AS PERSONAS Y LA SALUD

Tras la fecundación, el cigoto se adhiere a la pared del útero, en un proceso denominado ani-
dación, aumentando el tamaño progresivamente. La célula huevo comienza a dividirse sucesi-
vas veces, dando lugar a numerosas células que van poco a poco diferenciándose para formar
diversos órganos y tejidos.
Cuando la célula huevo empieza a desarrollarse recibe el nombre de embrión, a partir del
cual crecen unos tejidos que se introducen en el útero para absorber el alimento de la madre.
Estos tejidos junto con el útero, forman la placenta, que es un órgano mixto entre la madre y el
hijo. La alimentación y el oxígeno los recibe el embrión a través del cordón umbilical.
El feto, o futuro bebé está rodeado por una bolsa de agua, el amnios, que lo protege de posi-
bles daños y le permite moverse con cierta libertad. El amnios se encuentra envuelto en otra
bolsa denominada corión.
Unos días antes del nacimiento, el feto se coloca cabeza abajo, encima del cuello del útero.
Cuando comienza el parto, el útero comienza a contraerse rítmicamente, cada vez las contrac-
ciones son más próximas y fuertes. El cuello del útero se dilata poco a poco hasta permitir el
paso de la cabeza del niño, y las contracciones del útero se refuerzan con las del abdomen.
Se rompe el amnios y el corión y por fin sale el niño a través del cuello del útero y la vagi-
na, enormemente dilatados. Se corta el cordón umbilical y posteriormente la madre expulsa la
placenta como un proceso postnatal. A los pocos días, los restos del cordón que quedaron en el
bebé se secan y se caen; la cicatriz que se produce es el ombligo.
Aproximadamente a las veinticuatro horas del nacimiento, el niño comienza a mamar.

3.6. LA SEXUALIDAD HUMANA COMO COMUNICACIÓN AFECTIVA


Y OPCIÓN PERSONAL. DIFERENTES PAUTAS DE CONDUCTA
La conducta sexual de las personas es algo que forma parte de su intimidad y de su derecho
a la libertad de elección y actuación.
Es una forma de comunicación y acercamiento, de mostrar afecto y cariño, de disfrutar ya
que para nuestra especie, el acto sexual y las caricias, resultan placenteras, llegando al orgasmo.
Sea cual sea la conducta que nos agrade respecto a temas sexuales, cualquier otra, merece
nuestro respeto, aunque no la compartamos o no nos agrade, lo mismo que merecen respeto las
personas que las practican.
La actividad o conducta sexual es el proceso en el que se produce la creación y la resolu-
ción de la tensión sexual. Consta de varias fases:
– Fase de excitación. Se inicia como respuesta a una serie de estímulos tanto físicos como
psíquicos. Existen unas zonas de máxima sensibilidad (zonas erógenas) y desencadenan
la respuesta (congestión vascular, elevación de la tensión arterial, contracción muscular,
enrojecimiento de la piel, salivación, taquicardia, taquipnea).

35
-Pág.119-
C I E N C I A S D E L A N AT U R A L E Z A

– Fase de meseta. Es el mantenimiento de la excitación sexual.


– Fase de orgasmo. Se alcanza el punto máximo de tensión sexual y se libera toda la exci-
tación acumulada (en la mujer contracción del útero y en el hombre eyaculación).
– Fase de resolución. Se vuelve al estado de excitación. En la mujer, si existe un estímulo
se puede volver al orgasmo.

3.7. EL SEXO COMO FACTOR DE DISCRIMINACIÓN


EN LA SOCIEDAD
Hoy en día, siguen existiendo en el mundo sociedades que discriminan a un sexo, por el sim-
ple hecho de pertenecer al mismo, llegándose incluso a situaciones dramáticas. El caso de China
y la discriminación negativa que hace respecto a las niñas es un ejemplo.
Otro caso de discriminación, más cercana es el que se hace hacia la mujer, por el hecho de
serlo en las condiciones laborales (salarios más bajos, imposibilidad de acceso a ciertos pues-
tos, preferencia de elección del sexo masculino por parte de la empresa privada, acoso,...).
Existen otros casos de discriminación practicada hacia personas con conductas sexuales
diferentes. Hacia estas personas, deberíamos permitir la libre elección, por supuesto, siempre
que no hiriera nuestra sensibilidad, ni nos sintiéramos acosados por ellos/ellas. En estos casos
debiera primar la libertad de elección y el respeto a los demás.

3.8. MÉTODOS ANTICONCEPTIVOS Y NUEVAS TÉCNICAS


REPRODUCTIVAS
Mientras que el hombre, a partir de la pubertad, es potencialmente fértil siempre, la mujer
sólo lo es durante algunos días de cada ciclo menstrual entre la pubertad y la menopausia. Cuan-
do una pareja quiere tener relaciones sexuales, evitando la concepción, se recurre a distintos
métodos anticonceptivos, en la mayoría de los casos tendentes a evitar la unión del óvulo con
el espermatozoide.

Estos métodos pueden ser naturales y artificiales.

a) Métodos anticonceptivos naturales:

- Método del ritmo (Ogino-Knauss). Se basa en determinar los periodos infértiles de la


mujer. Los periodos fértiles están comprendidos entre los días 10 y 17 del ciclo. No es
muy seguro.

- Método de la temperatura basal. Se basa en el hecho de que la temperatura del cuer-


po en reposo se eleva de dos a cinco décimas de grado centígrado desde la ovulación
hasta el inicio de la siguiente menstruación.

36
-Pág.120-
U . D . 7 . - L AS PERSONAS Y LA SALUD

- Método de Billings. Se basa en que durante los periodos de fertilidad de la mujer (unos
seis días en el ciclo), el cuello del útero aparece abundantemente humedecido por una
secreción mucosa transparente y elástica, que aumenta la vitalidad de los espermato-
zoides. Así mismo la cristalización del moco cervical o de la saliva sólo es posible
durante el período fértil del ciclo, por lo que utilizando una lupa, se puede determinar
con bastante precisión el momento de fertilidad. La OMS (Organización Mundial de
la Salud) lo considera un buen método de planificación familiar.
b) Métodos anticonceptivos artificiales. Emplean elementos extraños al propio cuerpo.
- Píldoras anticonceptivas. Están elaboradas con distintas hormonas sexuales que alte-
ran el ciclo ovárico femenino impidiendo la ovulación y evitando la posible fecunda-
ción. También afecta a la pared del útero impidiendo la anidación y al cuello del útero,
modificando la mucosidad.
- Preservativos. Son membranas de goma que impiden el paso de los espermatozoides
al interior de la vagina o del útero. Puede ser masculino (no es del todo fiable) o feme-
nino (diafragma, espiral).
- La ligadura de trompas. Es quirúrgico y no tiene vuelta atrás, se aplica a la mujer.
- La vasectomía. Se aplica al hombre y tiene vuelta atrás.

3.9. ENFERMEDADES DE TRANSMISIÓN SEXUAL


Existen enfermedades propias de loa aparatos genitales masculino y femenino.
Entre las del aparato genital femenino, encontramos la amenorrea (falta de menstruación),
dismenorrea (menstruación dolorosa), leucorrea (aumento de secreción fisiológica genital),
endometritis (inflamación del endometrio), salpingitis (inflamaciones de las trompas de Falo-
pio), miomas (tumores benignos de útero) y tumores malignos de útero.
En el aparato genital masculino, como enfermedades encontramos la fimosis (es una estre-
chez del orificio del prepucio, por lo que éste no puede retraerse y dejar al descubierto el glan-
de), criptoquidia (es la falta de descenso de uno o ambos testículos a la bolsa escrotal), adeno-
ma de próstata (aumento de la glándula prostática que al aumentar de tamaño comprime la
uretra) y cáncer de próstata.
Existen asimismo, enfermedades de transmisión sexual. Pueden producirse por:
– Bacterias:
- Sífilis. Contagiosa por vía genital y oral y por transfusión.
- Gonorrea. Se contagia por contacto sexual o por objetos contaminados.

37
-Pág.121-
C I E N C I A S D E L A N AT U R A L E Z A

– Virus.
- Herpes genital. Se produce por contacto sexual. Produce dolor al orinar y durante las
relaciones sexuales.
- Verrugas genitales. Se transmite por contacto genital, lo produce el virus del papiloma.
- SIDA.
– Hongos.
- Candidiasis. Se produce por el desarrollo de Candida albicans, generalmente en la vagi-
na. Es mas frecuente cuando se producen enfermedades que disminuyen las defensas.
– Parásitos. No se transmiten por contacto sexual.
- Pediculosis púbica. Se conocen como “ladillas”. Produce prurito intenso y enrojeci-
miento de la piel del pubis.
- Sarna. Se transmite por contacto cutáneo directo.

3.9.1. PREVENCIÓN DE LAS ENFERMEDADES DE TRANSMISIÓN SEXUAL


Se puede realizar a tres niveles:
– Prevención primaria. Se realiza sobre el sujeto sano a través de educación sanitaria y edu-
cación sexual antes del contacto sexual. Tomando precauciones durante el contacto sexual
(preservativos) y con protección tras el contacto sexual, realizando una micción con rapi-
dez tras el contacto, realizando lavados con jabones de pH ácido y acudiendo al médico
cuando se sospeche el posible contagio.
– Prevención secundaria. Cuando el sujeto está enfermo, mediante diagnóstico y tratamien-
to precoz, aislamiento de los enfermos y encuesta epidemiológica.
– Prevención terciaria. Es la rehabilitación de las secuelas invalidantes.

3.10. HÁBITOS SALUDABLES DE HIGIENE SEXUAL


El Hombre como especie es un ser sexual, con capacidad para relacionarse y establecer
contactos con otras personas. La higiene sexual comprende una serie de actividades que pre-
tenden dotar a la persona de una formación sexual íntegra, que le permita afrontar su faceta
sexual libre de tabúes y prejuicios, lo cual repercutirá positivamente en su salud física y men-
tal. Comprende la educación sexual, las revisiones periódicas, la planificación familiar, la
lucha contra la esterilidad, el conocimiento de los medios anticonceptivos y el consejo genéti-
co (aconseja sobre posibles riesgos de transmisión de alteraciones o enfermedades hereditarias
a la descendencia).

38
-Pág.122-
U . D . 7 . - L AS PERSONAS Y LA SALUD

INTRODUCCIÓN A LA RELACIÓN Y COORDINACIÓN


4. HUMANAS. UTILIZACIÓN DEL SISTEMA SANITARIO.
CONSUMO DE MEDICAMENTOS

4.1. INTRODUCCIÓN A LA RELACIÓN Y COORDINACIÓN


HUMANAS
El sistema nervioso es el responsable de casi todas las funciones de coordinación interna en
el organismo. Las relaciones con el exterior, es decir, la comunicación con el entorno, se reali-
za en el ser humano gracias a la gran especialización de los órganos de los sentidos.

4.1.1. EL SISTEMA NERVIOSO


Está formado principalmente por células especializadas llamadas neuronas. Se encarga de la
mayoría de las funciones de relación. Estas funciones se llevan a cabo mediante la corriente ner-
viosa que transmite la información.
La corriente nerviosa se origina en los receptores de estímulos, se transmite por el sistema
nervioso, a través de las neuronas que se encuentran unidas por sinapsis nerviosas y llega a los
efectores, donde se realiza una reacción de respuesta. Los efectores pueden ser músculos, en
cuyo caso la respuesta es el movimiento, o glándulas, cuya respuesta es la secreción de una sus-
tancia específica.

Transmisión del impulso nervioso

39
-Pág.123-
C I E N C I A S D E L A N AT U R A L E Z A

El impulso nervioso recorre la neurona en un solo sentido. Entra por las dendritas y atraviesa
rápidamente toda la superficie de la neurona, de un extremo al otro en forma de breves impul-
sos eléctricos, hasta que llega al axón. No existe una conexión física entre las neuronas, pues
están separadas por espacios vacíos. El axón de la neurona transmisora libera una sustancia quí-
mica denominada neurotransmisor, desde su terminación nerviosa hacia el espacio llamado
sinapsis, que separa la neurona transmisora de la receptora. Cuando la última célula entra en
contacto con el neurotransmisor, es excitada eléctricamente en toda su superficie, restablecién-
dose así la conexión.
Un estímulo es cualquier situación ambiental capaz de ejercer una influencia sobre el orga-
nismo. Estímulo puede ser la luz, el sonido, ..., todo aquello que no pasa desapercibido.
Los receptores del estímulo son los órganos de los sentidos, formados por células especiali-
zadas, las células sensitivas, capaces de generar corrientes nerviosas.
No siempre se realiza una acción ante la presencia de un estímulo, sino que simplemente se
es consciente de la presencia de éste.
El sistema nervioso humano, comprende un eje cerebroespinal y unos nervios.
– A.- El eje cerebroespinal está formado por el encéfalo y la médula espinal. Ambos for-
man los centros nerviosos, es el sistema nervioso central.
El encéfalo se encuentra en el interior del cráneo que le sirve de protección. Su peso
medio es de 1500 g. Está formado por varios elementos:
- Cerebro. Constituido por dos partes separadas por una fisura interhemisférica, forman-
do dos hemisferios cerebrales. Ocupa las cuatro quintas partes del cerebro. En él reside
la inteligencia y desarrolla las acciones voluntarias.
- Cerebelo. Formado por dos lóbulos laterales, los hemisferios cerebelosos, separados
por un lóbulo estrecho en forma de gusano llamado vermis.
- Bulbo raquídeo. De unos 3 cm de longitud, que pasa por el agujero de la caja craneana
y es donde se inicia la médula espinal.
La médula espinal, es un largo cordón blanco de unos 50 cm, con un diámetro de aproxi-
madamente 1 cm. Está encerrada en la columna vertebral, cuyas vértebras tienen un orificio por
donde pasa la médula.
El encéfalo y la médula espinal están formados por dos tipos de sustancia: sustancia blanca
y sustancia gris. La sustancia blanca, es unas veces interna (en el cerebelo y en los hemisferios
cerebrales) y otras veces externa (en el bulbo raquídeo y en la médula espina). En el corte trans-
versal de la médula espinal, vemos que la sustancia gris, presenta forma de mariposa y se deno-
minan astas a cada “ala”. Hay dos astas posteriores y dos anteriores.
Protegiendo al encéfalo y a la médula tenemos tres membranas llamadas meninges (la
meningitis es la inflamación de dichas membranas, producidas por virus o bacterias).

40
U . D . 7 . - L AS PERSONAS Y LA SALUD

– B.- Los nervios, son cordones filamentosos, formados por neuronas, que desde el encé-
falo y la médula espinal se distribuyen por todo el cuerpo. Hay dos tipos de nervios, los
que parten del encéfalo, llamados nervios craneales (se dirigen especialmente a los órga-
nos de los sentidos), y los que arrancan de la médula espinal, que se denominan nervios
raquídeos (son 31 pares y cada uno se forma por la unión de dos raíces que están situadas
en la prolongación de las astas, mediante divisiones, llegan a todos los puntos de la piel).

4.1.1.1. FISIOLOGÍA DEL SISTEMA NERVIOSO


Los nervios son excitables y conductores. Pueden ser excitados por diversos agentes: gol-
pes, corriente eléctrica, pinchazos, sustancias químicas, etc. El estímulo nervioso es conducido
por el nervio a una velocidad de 20 a 40 metros por segundo.

Lóbulo parietal

Lóbulo
principal

Lóbulo
frontal Lóbulo
occipital

Lóbulo occipital
Lóbulo temporal

Si los nervios conducen la corriente nerviosa desde el punto en el que actúa el estímulo hasta
el cerebro, que es donde se recibe la información, decimos que son nervios sensitivos. Si por el
contrario, los nervios conducen la corriente nerviosa desde los centros nerviosos hasta el efec-
tor, entonces se trata de nervios motores. Si a la vez conducen la corriente en ambos sentidos
decimos que son nervios mixtos.
Los movimientos musculares se realizan por la acción de los nervios motores sobre los mús-
culos. Muchas veces los movimientos musculares son involuntarios e inconscientes. Un hom-
bre dormido, no es consciente de que le molesta una mosca, y sin embargo la ahuyenta; los
movimientos de este tipos se conocen como actos reflejos.
Un estímulo sobre la piel crea una corriente nerviosa hacia los centros nerviosos; en estos
se reconoce y se origina una corriente nerviosa hacia el punto que recibió el estímulo. Si el cen-
tro nervioso es la médula o el bulbo, la reacción se produce de forma automática e involunta-
ria, como si el estímulo se hubiera “reflejado” sobre la médula espinal. De ahí el nombre de
reflejos.

41
-Pág.125-
C I E N C I A S D E L A N AT U R A L E Z A

Muchas de nuestras reacciones son voluntarias, somos conscientes y sensibles a los estímu-
los del medio que nos rodea gracias a los órganos de los sentidos.
Para que se produzca una sensación es necesario que:
– El órgano sensorial haya sido excitado por un agente (luz, contacto, sonido, ...)
– La corriente nerviosa sea conducida por los nervios sensitivos hasta la sustancia gris de
los hemisferios cerebrales.
– La sustancia gris transforme los estímulos en sensaciones.
Los movimientos voluntarios, dependen de la orden que efectúen los hemisferios cerebrales
en los centros motores, que radican en la corteza cerebral de la materia gris. En los hemisferios
cerebrales, en la sustancia gris, no sólo se sitúan los centros motores, que regulan los movi-
mientos voluntarios, sino que también se encuentran en ellos la sede de la sensibilidad cons-
ciente y sobre todo de la inteligencia.
Existen otros actos involuntarios, llamados vegetativos, controlado por el llamado sistema
nervioso vegetativo, que funciona de forma autónoma, es decir, independiente de nuestra volun-
tad. Este sistema emite las órdenes operativas a través del sistema simpático y del parasimpáti-
co. Está controlado por el hipotálamo. Funcionan sobre las vísceras de forma conjunta pero
dando órdenes contrarias, el simpático de gasto energético y el parasimpático de relajación y
ahorro energético.

4.1.1.2. LOS ÓRGANOS DE LOS SENTIDOS


Tradicionalmente se habla de cinco sentidos: vista, oído, olfato, gusto y tacto. No obstante
las sensaciones, no sólo dependen del estímulo, sino también de la memoria y del previo cono-
cimiento.
La vista, capta las sensaciones de luz y color. El órgano específico es el ojo. El ojo actúa
como una cámara, por medio de una lente, que enfoca los objetos y los proyecta de forma inver-
tida sobre la retina; posteriormente, en el cerebro se unirán las sensaciones provenientes de los
dos ojos y formará la imagen definitiva (estereoscópica, es decir, en tres dimensiones).
El ojo consta de una parte esencial, de forma esférica, denominada globo ocular, y de un
conjunto de órganos situados a su alrededor, los órganos anejos, que desempeñan funciones pro-
tectoras (impiden la entrada de objetos extraños las pestañas y párpados; desvían el sudor las
cejas; lubrifican u humedecen las glándulas lacrimales). Además una serie de músculos están
implicados en los movimientos oculares.
El globo ocular, constituye el ojo propiamente dicho. Está recubierto de tres membranas:
esclerótida, coroides y retina. Estas capas dan cobijo en su interior a una lente que se deforma
con facilidad para enfocar las imágenes (el cristalino) y a unas sustancias líquidas que reciben
el nombre de humores (vítreo y acuoso).

42
-Pág.126-
U . D . 7 . - L AS PERSONAS Y LA SALUD

Humor vítreo Coroides Esclerótica

Glándula lacrimal
Retina

Párpado
superior

Cristalino

Pestaña Mancha
amarilla
Pupila

Humor
acuoso
Córnea
Nervio
Iris óptico

Párpado
inferior

Punto
ciego

Estructura del globo ocular

La esclerótica, es la membrana más externa, es de consistencia dura, es blanca salvo la parte


anterior, abombada y transparente llamada córnea. La coroides, es la capa intermedia, con nume-
rosos vasos que irrigan de sangre el ojo, en la parte anterior es sustituida por el iris (da color al
ojo); en el centro del iris se abre la pupila que se dilata cuando hay poca luz y se contrae cuando
hay mucha, mediante músculos específicos. La retina tapiza el globo ocular, contiene las termina-
ciones nerviosas que constituyen los receptores visuales, los conos (más sensibles a la luz) y los
bastones (más sensibles a las diferencias de color), éstos terminan agrupándose para formar el ner-
vio óptico, que transmite la corriente sensitiva hacia el cerebro. El lugar de la retina del cual parte
el nervio óptico se llama punto ciego porque carece de células visuales.
Para que el ojo funcione correctamente, es necesario que mantenga su forma esférica y el
cristalino transparente y con un sistema de enfoque correctos, de esta forma se podrá formar la
imagen invertida en la superficie de la retina. Defectos son la miopía (por excesivo alargamiento
del globo ocular y la imagen se forma antes en la retina), la hipermetropía (por acortamiento del
globo ocular y la imagen se forma detrás de la retina), presbicia (falta de movilidad del crista-
lino por mal funcionamiento de los músculos para el acomodamiento a la visión cercana), cata-
ratas (el cristalino se vuelve opaco) y daltonismo (falta de percepción de algunos colores y con-
fusión del rojo y el verde) que es hereditario.
El oído, es un órgano donde se localiza la audición y el sentido del equilibrio. Posee una
pequeña parte sensitiva y estructuras de captar, concentrar y transmitir los sonidos. Anatómica-
mente, se distinguen tres regiones: oído externo, oído medio y oído interno. El oído externo está

43
-Pág.127-
C I E N C I A S D E L A N AT U R A L E Z A

Oído externo Oído medio Oído interno

Membrana
timpática Canales
semicirculares
Martillo Yunque

Vestíbulo

Nervio
auditivo

Caracol Martillo
Yunque
Conducto Ventana
auditivo externo oral
Músculo
Cavidad del Estribo del estribo
oído medio
Pulpejo
o lóbulo
Trompa de Eustaquio

Tímpano
(cara interna)
Estribo
Mango de Ventana oral
martillo
Anatomía del oído

compuesto por el pabellón auditivo, que gracias a un cartílago mantiene la forma y actúa como
una antena y el conducto auditivo externo, que es un tubo de unos dos centímetros y las pare-
des cubiertas de pelos y glándulas secretoras de cera, que impiden la entrada de partículas per-
judiciales. El oído medio y el interno se encuentran encajados en cavidades óseas del cráneo. El
oído medio comienza en el tímpano, que lo separa del externo, se continúa con una cadena de
huesecillos (martillos, yunque y estribo), también está comunicado con la faringe por la trom-
pa de Eustaquio (su misión es igualar la presión a ambos lados de la membrana timpánica y
airear la cavidad); desde el estribo se conecta con el oído interno por un orificio llamado ven-
tana oval. El oído interno presenta una estructura muy compleja llamada laberinto, dividido en
tres partes: vestíbulo, caracol y canales semicirculares. Todo está bañado en líquido que amor-
tigua el roce con el hueso. En el interior del caracol se alojan los receptores del sonido y en las
otras estructuras, los del equilibrio. El vestíbulo, está tapizado de células sensoriales, sobre las
que descansan pequeñas piedrecillas de carbonato cálcico, los otolitos.
Respecto al equilibrio, se debe señalar que, gracias al oído, es posible detectar tanto la posi-
ción de nuestro cuerpo como los movimientos que realizamos. Las células sensoriales del ves-
tíbulo captan la postura del cuerpo; al cambiar la posición de la cabeza, los otolitos presionan
las células sensoriales y se transmite el estímulo hasta el encéfalo por el nervio auditivo.
El sonido tiene su origen en ondas mecánicas del aire que son concentradas por la oreja y
que al llegar al tímpano, provocan su vibración, transmitiéndose posteriormente ésta a la cade-
na de huesecillos.

44
-Pág.128-
U . D . 7 . - L AS PERSONAS Y LA SALUD

Fosas nasales

El olfato, puede reconocer 10.000 olores distintos. Se localiza en el interior de las fosas
nasales, en la pituitaria amarilla, en ella se encuentran los millones de neuronas que transmiten
la sensación hacia el cerebro por medio del bulbo olfatorio y del nervio olfativo.

El gusto, permite percibir los sabores


mediante los botones gustativos, que se
agrupan en unas estructuras que sobresalen
de la superficie de la lengua: son las papilas
gustativas, apreciables a simple vista. El
gran número de sabores que podemos detec-
tar es siempre mezcla de cuatro fundamen-
tales, dulce, salado, ácido y amargo. La
capacidad de discriminación está localizada
en zonas distintas de la lengua.

Lengua humana
Áreas donde se perciben mejor los sabores básicos

45
-Pág.129-
C I E N C I A S D E L A N AT U R A L E Z A

Distintos tipos de receptores de la piel: 1 del dolor; 2 del tacto; 3 del frío; 4 del calor; 5 de la presión.
Otras estructuras son: A glándula sudorípara; B glándula sebácea; C músculo erector del pelo.

El tacto, es más adecuado hablar de los sentidos somáticos o difusos, que se encuentran dis-
tribuidos por toda la piel. Existen terminaciones nerviosas que nos permiten discriminar tacto,
presión, texturas, temperatura (corpúsculos sensitivos), localización de nuestro propio cuerpo
(propiorreceptores) y dolor.

4.1.1.3. TIPOS DE RESPUESTA: EFECTORES


Una vez elaborada la respuesta al estímulo en el sistema nervioso central, los encargados de
realizarla son los efectores, que pueden ser, como ya se ha indicado los músculos o las glándu-
las, cuyas respuestas son secretoras o motoras, respectivamente.

4.1.1.3.1. EL APARATO LOCOMOTOR


LOS MÚSCULOS
Son efectores de respuesta gracias a la capacidad de sus células de contraerse. Son alarga-
das (se denominan fibras musculares).
Hay músculos (del corazón, tubo digestivo, vasos sanguíneos) que se contraen con
independencia de nuestra voluntad; otros (los músculos esqueléticos) se contraen de
forma voluntaria.
La forma de los músculos esqueléticos es muy variada, hay músculos planos, cortos,
con forma de anillo y alargados. Los músculos se unen a los huesos mediante tendones.

46
-Pág.130-
U . D . 7 . - L AS PERSONAS Y LA SALUD

Cuerpo humano. Musculatura anterior y posterior

Los músculos suelen actuar mediante pares antagónicos, es decir, si uno se contrae, el
otro se relaja, uno hace un movimiento (por ejemplo de levantar la mano) y el otro el con-
trario (bajar la mano).

47
-Pág.131-
C I E N C I A S D E L A N AT U R A L E Z A

LOS HUESOS
Son piezas duras y al conjunto se le denomina esqueleto. Sus funciones son proteger los
órganos más delicados, dar solidez al cuerpo, permitir el movimiento.
Los huesos son duros y frágiles, pero también tienen cierta elasticidad. Están compuestos
por proteínas, fosfatos y carbonatos de calcio. Existen huesos planos (función protectora), cor-
tos (relacionan unos huesos con otros), irregulares (con funciones diversas) y largos (para la
locomoción).

El esqueleto humano y los huesos del cráneo

48
-Pág.132-
U . D . 7 . - L AS PERSONAS Y LA SALUD

En los huesos largos distinguimos los extremos más anchos (epífisis) y la parte central, alar-
gada y cilíndrica (caña o diáfisis).
La unión de dos huesos, es la articulación. Cuando son móviles, los huesos no están en con-
tacto directo, sus superficies se recubren de cartílago y entre ellos hay una cápsula llena del lla-
mado líquido sinovial.

4.1.1.3.2. LAS GLÁNDULAS Y HORMONAS


Las glándulas endocrinas, desempeñan un papel muy importante en el control y la regula-
ción del organismo, actuando conjuntamente con el sistema nervioso. El punto de conexión
entre ambos sistemas se localiza en el hipotálamo.
Las hormonas se transportan por la sangre, sus respuestas son más lentas que las del sistema
nervioso, pero mucho más duraderas. Cuando terminan su actuación se eliminan por la orina.
Estas sustancias realizar distintas funciones, desde controlar el metabolismo celular o la
diferenciación y el crecimiento de las células durante el desarrollo embrionario hasta estimular
a otras glándulas endocrinas para que liberen su secreción.

Principales glándulas endocrinas humanas

Las cantidades hormonales presentes en la sangre se mantienen constantes debido a un


mecanismo de autocontrol que posee nuestro organismo para mantenerlas en el nivel correcto.

49
-Pág.133-
C I E N C I A S D E L A N AT U R A L E Z A

4.2. FACTORES QUE REPERCUTEN EN LA SALUD MENTAL


EN LA SOCIEDAD ACTUAL
Son muchas las agresiones y fuentes de sobreexcitación que existen en la vida moderna que
pueden llegar a causar trastornos de importancia: stress, depresión, fatiga,...

De entre todos los problemas, el que más afecta socialmente es el de la drogadicción tanto
de drogas prohibidas en nuestra sociedad como de aquellas legalmente institucionalizadas
(tabaco y alcohol).

4.3. EL PROBLEMA DEL TABACO, EL ALCOHOL Y LAS DROGAS;


SUS EFECTOS SANITARIOS Y SOCIALES
DROGA, según la OMS (Organización Mundial de la Salud), “droga o fármaco, es toda sus-
tancia que introducida en un organismo vivo, puede modificar una o más funciones de éste”. Es
una definición muy amplia que abarca medicamentos destinados al tratamiento de enfermeda-
des y otras sustancias activas.

FARMACODEPENDENCIA, según la OMS, “es el estado psíquico y físico, causado por la


acción recíproca entre un organismo vivo y un fármaco, droga, que se caracteriza por modifi-
caciones del comportamiento y por otras reacciones, que implican siempre un impulso irrepri-
mible a tomar el fármaco en forma continua o periódica”.

DEPENDENCIA, es el estado interno del individuo mediante el cual crea y mantiene cons-
tantemente un deseo de ingerir esa sustancia. La dependencia puede ser física (implica vincula-
ción bioquímica de la droga en el organismo y su falta crea el síndrome de abstinencia, con
características diferentes según la droga) y psicosocial.

Son varias las causas que derivan en el abuso de consumo de drogas, entre ellas pode-
mos citar:
– Curiosidad por lo misterioso.
– Gusto por lo prohibido.
– Experimentar sensaciones nuevas.
– Sentirse libre.
– Pasotismo.
– Escape de problemas personales.
– Mecanismo de protesta contra el sistema.

50
-Pág.134-
U . D . 7 . - L AS PERSONAS Y LA SALUD

4.3.1. CLASIFICACIÓN DE LAS DROGAS


Se pueden clasificar siguiendo varios criterios:

1. Según su origen, pueden ser naturales (tabaco), sintéticas (anfetaminas) y semisintéticas


(morfina).

2. Según el principal efecto psíquico (estado de humor y nivel de vigilancia).


- Psicolépticos.
- Hipnóticos. Disminuyen la vigilancia (sueño). Ej.: Barbitúricos.
- Neurolépticos. Disminuyen el humor (deprimen). Ej.: Fenotiazinas.
- Tranquilizantes. Disminuyen la vigilancia y el humor. Ej.: Diacepinas.
- Psicoanalépticos.
- Estimulantes. Aumentan la vigilancia (vigilia). Ej.: Cafeína.
- Antidepresivos. Aumentan el humor (euforia). Ej.: Tricíclicos.
- Psicodislépticos. Desvían la vigilancia (captan “realidades” irreales) y el humor.
Ej.: alucinógenos, LSD, mescalina...

3. Según la dependencia que crean.


- Creación de dependencia física y psicosocial (duras). Ej.: Opio, alcohol, barbitúricos...
- Crean dependencia psicosocial (blandas). Ej.: hachís, marihuana, cocaína, LSD...

4. Según la peligrosidad.
- Las drogas más peligrosas: son aquellas que crean dependencia física, con mayor rapi-
dez y poseen mayor toxicidad.
- Las drogas menos peligrosas: son aquellas que crean dependencia psico-social, más
lentamente, poseyendo menor toxicidad.

La OMS establece cuatro grupos:


- GRUPO 1: Opio y derivados (morfina y heroína).
- GRUPO 2: Barbitúricos y alcohol.
- GRUPO 3: Cocaína y anfetaminas.
- GRUPO 4: LSD y cannabis.

5. Según la situación sociológica.


- Institucionalizadas: alcohol y tabaco.
- No institucionalizadas: LSD, cannabis, mescalina, ...

51
-Pág.135-
C I E N C I A S D E L A N AT U R A L E Z A

4.3.2. TRATAMIENTO PREVENTIVO


En la prevención se pueden seguir tres fases:
a) Prevención primaria. Dirigida a conseguir una disminución de la oferta de las drogas:
- Medidas legislativas, asociadas a acciones políticas y policiales (vigilancia de los cam-
pos de cultivo, control aduanero, penalización de los traficantes...).
- Medidas higiénico-sanitarias:
· Información (objetiva e imparcial) sobre las consecuencias del uso y abuso.
· Promoción de hábitos de vida sanos.
· Creación de zonas de recreo y esparcimiento.
· Suprimir la publicidad que favorece el consumo de drogas.
· Potenciar el núcleo familiar e informar para la posible detección de cambios de com-
portamiento.
· Incluir programas de prevención en los centros educativos.
· Cursos de formación a los educadores.
b) Prevención secundaria. Se basa en el diagnóstico y tratamiento precoz de las personas
que presentan drogodependencias.
c) Prevención terciaria. Dirigida a la rehabilitación y reinserción social del drogadicto.

4.3.3. DROGAS INSTITUCIONALIZADAS: TABACO Y ALCOHOL


Es evidente que tanto el tabaco como el alcohol, forman parte del entorno ambiental del
individuo y gozan de un trato de favor por parte de amplios sectores de la sociedad.

4.3.3.1. EL TABACO
Según la OMS, el tabaco crea una “dependencia psíquica fuerte y física leve”.
Los efectos nocivos del tabaco los crean los componentes del humo que son los que sanita-
riamente interesan: nicotina (es la responsable de la adquisición del hábito de fumar); monóxi-
do de carbono (CO, gas tóxico que produce una mala oxigenación del organismo); gases irri-
tantes (producen bronquitis crónica, acompañada de tos irritante); sustancias carcinogénicas
(favorecedoras del desarrollo de cáncer, la más importante es el alquitrán).
El hábito de fumar alcanza en el mundo cotas muy elevadas, hoy en día tanto entre las muje-
res como entre los hombres. La publicidad, la utilización del tabaco como medio de acerca-
miento social, situaciones de estrés (aunque en realidad excita), la falta de información sobre
los riesgos del tabaco, el deseo de imitación al adulto o la introversión entre otros, pueden ser
algunas de las causas que arrastren a las personas a fumar.

52
-Pág.136-
U . D . 7 . - L AS PERSONAS Y LA SALUD

Los problemas derivados del tabaquismo variarán en función de la constitución física, la


edad, el número de cigarrillos...
El tabaquismo, es un hábito que se puede abandonar, siempre y cuando la persona lo quie-
ra dejar realmente, no obstante, hay métodos que ayudan a conseguirlo (farmacoterapia, psico-
terapia, acupuntura...). También, aunque no se elimine el hábito del todo, se pueden intentar
otras alternativas como diminuir el número de cigarros fumados, consumir tabaco bajo en nico-
tina, retirar el cigarrillo de la boca entre chupadas...
Pese a ello, el método más eficaz es la prevención especialmente dirigida a la población
infantil (escuela, prohibición de la publicidad, la venta a menores, prohibición de fumar en cen-
tros educativos, sanitarios y centros públicos, acotando compartimentos aislados para fumado-
res y evitar así los fumadores pasivos).

4.3.3.2. ALCOHOL
Las bebidas alcohólicas, pueden ser de dos tipos:
– Fermentadas. Se obtienen por fermentación de levaduras, así se obtienen la sidra, la cer-
veza, el vino...
– Destiladas. Se obtienen eliminando agua por destilación de las bebidas fermentadas.
Se denomina grado alcohólico de una bebida a la proporción de alcohol que contiene para
un volumen dado de la misma. Las bebidas destiladas tienen un grado alcohólico mucho más
elevado en general que las bebidas fermentadas.
El alcoholismo surge por motivos similares a los expuestos anteriormente sobre otras dro-
gas, aunque más cargado de tópicos debido a la antigüedad y a lo instaurado que está su con-
sumo desde hace ya muchas civilizaciones.
El ingrediente activo de las bebidas alcohólicas es el alcohol etílico. Se absorbe en el tracto
digestivo sin modificar y pasa a la sangre (más rápidamente en ayunas) y a través de ella a todos
los órganos y tejidos. Se metaboliza fundamentalmente en el hígado.
La acción del alcohol depende de muchos de las circunstancias del individuo ( sensibilidad,
fatiga, ayuno, embarazo...), así como de la frecuencia y cantidad ingerida.
Podemos distinguir varias formas clínicas:
– Intoxicación aguda o borrachera. Se desarrolla en cuatro fases, según la alcoholemia, lle-
gando al coma etílico (con grados de alcoholemia entre 4 y 5 g/l de alcohol en sangre).
– Alcoholismo crónico. Es la enfermedad debida al consumo excesivo y prolongado de
bebidas alcohólicas, capaz de producir trastornos físicos, mentales y sociales, que en
muchos casos se hacen crónicos.
Hay que añadir los problemas que a nivel social acarrea el alcoholismo, disgregación fami-
liar, absentismo laboral, bajo rendimiento y accidentes de trabajo, marginación social y acci-
dentes de tráfico por la falta de coordinación motora-visual (es la primera causa de accidentes
de circulación).

53
-Pág.137-
C I E N C I A S D E L A N AT U R A L E Z A

Cuando en el alcohólico, se produce supresión brusca del alcohol, se ocasiona el delirium


tremens con intensa agitación psicomotora, alucinaciones terroríficas, temperatura corporal ele-
vada, deshidratación y alteraciones neurológicas...

Las medidas preventivas del alcoholismo, al igual que las otras drogadicciones pueden ser:

Prevención primaria. Dirigidas para evitar la aparición del alcoholismo:


– Información sobre los perjuicios del alcohol.
– Erradicar los mitos.
– Mejorar el nivel de vida.
– Mejorar la alimentación ya que reduce las consecuencias orgánicas del alcohol.
– Mejorar y potenciar los lugares de recreo y espercimiento.
– Prohibir la venta de bebidas alcohólica en los centros educativos.
– Prohibir la venta de bebidas alcohólicas a menores de 16 años.
– Prohibir la publicidad sobre bebidas alcohólicas.

Prevención secundaria. Intenta el tratamiento precoz del alcohólico, mediante el manteni-


miento de la abstinencia total, la deshabituación y el tratamiento de las alteraciones orgánicas.

Prevención terciaria. Dirigida a la reinserción social del enfermo con la ayuda de psicotera-
pia individual o en grupo.

Aunque el abuso del alcohol es en general peligroso, hoy en día se recomienda el consumo
de cantidades pequeñas de vino tinto por sus cualidades cardiotónicas especialmente en perso-
nas mayores durante las comidas.

4.4. ESTILOS DE VIDA SALUDABLES


Para evitar en la medida de lo posible las agresiones sobre nuestro sistema nervioso y en
general sobre el organismo, es conveniente, llevar una vida lo más saludable posible y tener las
ideas claras sobre los efectos que por la posible “presión del grupo social al que se pertenece”
puede ejercer sobre nosotros, tanto en el consumo de drogas, los juegos de rol, las actividades
que van en contra de nuestra propia forma de pensar, etc.

En general, es conveniente dormir lo suficiente, comer con variedad y sin abusos, llevar un
horario regular en el sueño, las comidas, etc., evitar el consumo de tabaco y alcohol, evitar los
ambientes contaminados (ruido, humos, etc.) y practicar algún tipo de ejercicio físico y muy
importante también desarrollar la capacidad de comunicación con las personas que nos rodean,
evitando lo más posible la agresividad.

54
-Pág.138-
U . D . 7 . - L AS PERSONAS Y LA SALUD

4.5. UTILIZACIÓN DEL SISTEMA SANITARIO


En la actualidad, la salud es un derecho y una necesidad del ser humano por lo que en Espa-
ña, cubre a todos los residentes en lo que es imprescindible para el mantenimiento de una
correcta salud. Todos los trabajadores cotizan una cantidad mensualmente para el manteni-
miento del sistema.

Existe un sistema de salud público que garantiza este derecho, tanto en los servicios médi-
cos, ingresos hospitalarios, necesidad de medicaciones, hasta las ayudas en el caso de incapaci-
dades y hasta pensiones de jubilación.

No obstante, existen compañías privadas que permiten descargar los hospitales públicos,
estableciendo convenios y asistiendo a sus asegurados que atienden igualmente a un gran núme-
ro de personas.

Cuando tenemos cualquier problema de salud, es conveniente acudir al médico, sin automedi-
carnos y en caso de tratarse de un problema grave e inesperado, acudir a los servicios de urgencia.

4.6. CONSUMO DE MEDICAMENTOS, SU EFICACIA Y SUS RIESGOS


Aunque las terapias mediante sustancias químicas y vegetales, ha sido practicada desde la
antigüedad, hoy en día ha aumentado el número de principios activos utilizados, las enferme-
dades tratadas y las formas de producir dichos compuestos.

Es conveniente no perder de vista que los medicamentos son sustancias que ejercen un cier-
to efecto sobre el organismo y que es conveniente no abusar de ellos y usarlos cuando sea nece-
sario tan sólo, pues las vías de eliminación son los riñones y el hígado, y estos órganos sufren
desgaste y deterioro.

Como ya se dijo, históricamente, hay tres grupos de medicamentos que tienen una gran
importancia: los sueros, los antibióticos y los analgésicos.

Los sueros, son concentrados de anticuerpos específicos para una determinada enfermedad
infecciosa. Su finalidad es producir rápidamente una inmunización en individuos ya afectados
por esa enfermedad. Por ser anticuerpos de procedencia exterior, su permanencia es muy limi-
tada. Son sólo curativos. En la actualidad se fabrican a partir de donantes humanos y en el labo-
ratorio de síntesis.

Los antibióticos, como ya se dijo fueron descubiertos en 1928. El primero que se aisló se le
llamó penicilina (por haberse obtenido a partir del hongo Penicillium nutatum). Se ha avanza-
do mucho en la investigación sobre los antibióticos, y en la actualidad hay distintas familias de
compuestos que se elaboran de forma sintética.

55
-Pág.139-
C I E N C I A S D E L A N AT U R A L E Z A

Los analgésicos, actúan aliviando el dolor. Entre los más utilizados está el ácido acetilsalicílico,
sintetizado en 1879, es también un eficaz antipirético y previene el infarto de miocardio, aunque está
contraindicado en las úlceras, hemofilia, lesión renal y durante el último trimestre del embarazo.
Otro producto, que sustituye en parte al anterior es el paracetamol, que es un analgésico no
narcótico, de creciente utilización.
El grupo de analgésicos potentes está constituído por aquellos que presentan propiedades nar-
cóticas como la morfina, la metadona o la codeína, debiendo emplearse bajo control médico.

56
-Pág.140-
U . D . 7 . - L AS PERSONAS Y LA SALUD

RESUMEN

— Según la OMS, la salud es “el perfecto estado de bienestar físico, mental y social, y no
sólo la ausencia de enfermedad o dolencia”. Cuando se rompe el equilibrio entre los tres
componentes de la salud aparece una alteración, es la enfermedad, que se manifiesta en
forma de síntomas.
— Las constantes vitales son la temperatura corporal, el peso corporal, el pulso, la presión
arterial, la capacidad vital, el hemograma, la composición química del plasma y la com-
posición química de la orina.
— El organismo se defiende de las enfermedades infecciosas mediante el sistema inmunita-
rio, que actúa mediante células especializadas, los glóbulos blancos de la sangre.
— Si el sistema inmunitario funciona correctamente, cada vez que se introduce un antíge-
no en nuestro organismo se producen anticuerpos específicos que lo neutralizan. La
reacción antígeno-anticuerpo es la base de la inmunidad.
— La noción de desarrollo supone un proceso de cambio unitario y continuo, es un creci-
miento pautado cuya forma tiene por lo común un carácter progresivo, y cuyo sentido es
el de un aumento de la calidad biosocial de la conducta.
— La mayoría de los procesos de aprendizaje se desarrollan a partir de las interacciones de
la persona con el medio ambiente. Es muy importante, sin embargo, darse cuenta de que
en ninguno de los procesos del desarrollo el ambiente, por sí solo crea la conducta. Cier-
tas potencialidades biológicas del crecimiento individual que se despliegan en un pro-
ceso maduracional son necesarios para que se produzca el desarrollo.
— Para prevenir las enfermedades infecciosas, lo primero es evitar el contacto con micro-
organismos patógenos, la desinfección inmediata de las heridas mediante el uso de anti-
sépticos y la vacunación.
— La prevención de las enfermedades no infecciosas supone diversas normas higiénicas
que determinan un estilo de vida saludable, entre estas indicaciones para nuestra vida
diaria podemos mencionar el practicar ejercicio físico adecuado, una correcta higiene
buco-dental, llevar un plan de vida equilibrado, evitar las intoxicaciones y las depen-
dencias al tabaco, alcohol y drogas y trabajar en condiciones saludables.
— La nutrición es un conjunto de complejos fenómenos metabólicos que proporcionan
materia y energía a todas las células de nuestro organismo.
— En la nutrición están implicados los aparatos digestivo, respiratorio y excretor y el sis-
tema circulatorio.
— El aparato digestivo permite la descomposición mediante enzimas y la asimilación pos-
terior de los nutrientes contenidos en los alimentos, eliminando la parte no utilizable.

57
-Pág.141-
C I E N C I A S D E L A N AT U R A L E Z A

— El aparato respiratorio proporciona oxígeno al organismo eliminando el dióxido de car-


bono resultante del metabolismo celular.
— El aparato excretor elimina mediante la orina ciertos iones, toxinas y ciertos productos
finales del metabolismo celular (como la urea).
— El sistema circulatorio transporta los nutrientes, el oxígeno, el dióxido de carbono y los
productos de desecho.
— Ciertas enfermedades como la caries, la gastritis, la úlcera, la hipertensión, la arterioes-
clerosis y el infarto de miocardio están muy ligados a hábitos alimentarios.
— Los alimentos según los nutrientes que contienen y el origen de los mismos se clasifi-
can en siete grupos: Leche y derivados; Carne, pescados y huevos; Legumbres, tubér-
culos y frutos secos; Hortalizas y verduras; Frutas y derivados; Cereales, pastas y azú-
car y grasas y aceites.
— Los alimentos se dividen según su función en energéticos (contienen hidratos de carbo-
no y grasas), plásticos (reparan los tejidos, son las proteínas y el calcio) y reguladores
(regulan el metabolismo, son las vitaminas, los minerales y el agua).
— Los hidratos de carbono se degradan por los enzimas hasta glucosa que entra en las célu-
las con la ayuda de la insulina. Poseen una función esencialmente energética. Produce 4
Kcal por gramo ingerido. Es necesario que entre un 55-60% de las calorías diarias sean
proporcionados por los glúcidos.
— La fibra alimentaria son azúcares complejos que el organismo no puede digerir facili-
tando por lo tanto el tránsito intestinal.
— Los lípidos cumplen una función esencialmente energética aunque también plástica.
Cada gramo de grasa proporciona unas 9 kilocalorías. Un 25-30% de las necesidades
calóricas diarias deben cubrirse con grasas. Las grasas animales contienen colesterol.
— Las proteínas están compuestas por aminoácidos y tienen carácter eminentemente plás-
tico. Parte de esos aminoácidos no es posible fabricarlos en el organismo, son los ami-
noácidos esenciales. Las proteínas de origen animal poseen todos estos aminoácidos
por lo que se dice que poseen un alto valor biológico. Cada gramo de proteína produ-
ce unas 4 kilocalorías. Se deben cubrir con proteínas un 15% de las necesidades caló-
ricas diarias.
— Las vitaminas son reguladores de los distintos procesos metabólicos. Se dividen en
hidrosolubles (se disuelven en agua como las B, C, P...) y liposolubles (se disuelven en
grasas como A, E, D y K). Se deben ingerir porque el organismo no es capaz de sinteti-
zarlas.
— Los minerales participan también como reguladores del metabolismo y como estructu-
rales (calcio).

58
-Pág.142-
U . D . 7 . - L AS PERSONAS Y LA SALUD

— El agua, aunque no es un alimento, es imprescindible consumir al menos 2 litros al día.


— La mejora del nivel de vida, la mejora del transporte, los procesos tecnológicos de refi-
nado, la desorientación de la publicidad, la limitación de las tradiciones y la mayor ten-
dencia al sedentarismo influyen en nuestros hábitos de alimentación. Estos factores
hacen que la dieta sea excesivamente calórica, con demasiadas proteínas, poco consumo
de verduras, frutas, leche y alimentos ricos en fibra y un excesivo consumo de sal por lo
que se favorece la aparición de obesidad, enfermedades cardiovasculares y diabetes.
— La dieta debe ser equilibrada y adecuada a las necesidades de la persona y lo más natu-
ral posible. Debe cubrir el metabolismo basal, los gastos de actividad física y la acción
dinámico-específica del propio alimento.
— Existen ciertos modelos de dieta como la dieta mediterránea que satisface muy bien
todas las necesidades del individuo sano. Cuando existe alguna enfermedad se debe pau-
tar una dieta terapeútica adecuada al problema del enfermo.
— Ciertas enfermedades como son la diabetes glucosúrica, la gota úrica y la obesidad, tie-
nen un origen metabólico y pueden ser corregidas o evitar el avance mediante dietas
adecuadas.
— En la manipulación de los alimentos se ha de ser muy meticuloso aplicando normas
higiénicas estrictas. A nivel de la industria se aplica el sistema ARYCPC consistente en
controlar y evaluar en cada escalón de la producción los riesgos que existen y aplicar
medidas correctoras para evitarlos.
— Las toxiinfecciones, intoxicaciones e infestaciones son consecuencia de no aplicar las
normas exhaustivamente o ser poco responsable de los productos que se ingieren.
— Dada la importancia que para la salud tiene la alimentación, de debe recurrir a las auto-
ridades locales o regionales para poner en su conocimiento cualquier irregularidad que
se detecte.
— Los seres humanos se reproducen sexualmente. Este mecanismo consiste en la fusión de
dos células o gametos, uno masculino (espermatozoide) y otro femenino (óvulo). De
esta unión se forma otra célula, el huevo o cigoto, que tras su desarrollo dará lugar a otro
individuo.
— Existen diferencias físicas y anatómicas entre el hombre y la mujer (varón y hembra) en
nuestra especie, a esta propiedad se le denomina dimorfismo sexual.
— El aparato sexual masculino consta de dos testículos que producen los esperma-
tozoides y andrógenos (testosterona), regulado a nivel hipofisario; las vesículas semi-
nales y próstata que producen el semen o esperma, expulsado durante la eyaculación
y que sirve de vehículo a los espermatozoides y el pene que es un órgano que al lle-
narse de sangre en sus tejidos aumenta de tamaño y se endurece permitiendo su intro-
ducción en la vagina.

59
-Pág.143-
C I E N C I A S D E L A N AT U R A L E Z A

— El proceso productor de espermatozoides, la espermatogénesis se produce durante toda


la vida.
— El aparato genital femenino consta de una serie de órganos (ovarios, trompas de Falo-
pio, útero, vagina y genitales externos o vulva). Tienen como función acoger al pene
masculino durante el coito, producir hormonas sexuales femeninas, desarrollar el emba-
razo y colaborar en el parto.
— Durante la copulación se depositas más de 200 millones de espermatozoides, de los que
tan sólo uno fecunda y se produce la anidación. Se forma la placenta y el feto se rodea
del amnios y el corión.
— Existen muchos métodos anticonceptivos, entre los naturales están el de Ogino-Knauss,
de la temperatura basal, el de Billings (es el más fiable dentro de los naturales para la
planificación familiar) y entre los artificiales la píldora anticonceptiva, los preservativos,
la ligadura de trompas y la vasectomía.
— Existen enfermedades de transmisión sexual producidas por bacterias (sífilis), virus
(SIDA), hongos y parásitos, es conveniente aplicar las medidas higiénicas necesarias
para evitar el contagio.
— El sistema nervioso está formado principalmente por células especializadas llamadas
neuronas. Se encarga de la mayoría de las funciones de relación. Estas funciones se lle-
van a cabo mediante la corriente nerviosa que transmite la información siempre en un
solo sentido entrando por las dendritas y saliendo por el axón.
— Los receptores del estímulo son los órganos de los sentidos (vista, oído, olfato, gusto y
tacto), formados por células especializadas, las células sensitivas, capaces de generar
corrientes nerviosas.
— El sistema nervioso humano, comprende un eje cerebroespinal (encéfalo y médula espi-
nal) y unos nervios.
— El encéfalo está formado por el cerebro (aquí reside la inteligencia y el control de los
actos voluntarios), cerebelo y bulbo raquídeo.
— Para que se produzca una sensación es necesario que el órgano sensorial haya sido exci-
tado por un agente (luz, contacto, sonido...), la corriente nerviosa sea conducida por los
nervios sensitivos hasta la sustancia gris de los hemisferios cerebrales y la sustancia gris
transforme los estímulos en sensaciones.
— Los movimientos voluntarios, dependen de la orden que efectúen los hemisferios cere-
brales en los centros motores, que radican en la corteza cerebral de la materia gris.
— Existen otros actos involuntarios, llamados vegetativos, controlado por el llamado siste-
ma nervioso vegetativo, que funciona de forma autónoma, es decir, independiente de
nuestra voluntad. Este sistema emite las órdenes operativas a través del sistema simpático

60
-Pág.144-
U . D . 7 . - L AS PERSONAS Y LA SALUD

y del parasimpático. Está controlado por el hipotálamo. Funcionan sobre las vísceras de
forma conjunta pero dando órdenes contrarias, el simpático de gasto energético y el
parasimpático de relajación y ahorro energético.
— Los efectores de la corriente nerviosa son los músculos, ayudados por los huesos que
les sirven de soporte y la secreción de las hormonas, producidas en distintas partes del
cuerpo.
— Droga, según la OMS (Organización Mundial de la Salud), “droga o fármaco, es toda
sustancia que introducida en un organismo vivo, puede modificar una o más funciones
de éste”. Es una definición muy amplia que abarca medicamentos destinados al trata-
miento de enfermedades y otras sustancias activas.
— Dependencia, es el estado interno del individuo mediante el cual crea y mantiene cons-
tantemente un deseo de ingerir esa sustancia. La dependencia puede ser física (implica
vinculación bioquímica de la droga en el organismo y su falta crea el síndrome de abs-
tinencia, con características diferentes según la droga) y psicosocial.
— Existen drogas ilegales y legales (tabaco y alcohol) que producen efectos muy negativos
para la salud, llegando hasta la muerte.
— El sistema sanitario en España es un derecho que asiste a todos los españoles mediante
la cotización de los trabajadores.
— Los medicamentos hay que tomarlos con precaución por el efecto de hábito, dependen-
cia o ineficacia que el abuso puede generar.

61
-Pág.145-
C I E N C I A S D E L A N AT U R A L E Z A

EJERCICIOS DE AUTOCOMPROBACIÓN

1. En el intestino delgado:

A. Se segrega la bilis y el jugo pancreático

B. Se producen movimientos peristálticos

C. Se produce la absorción de los nutrientes ya digeridos

D. Todas son verdaderas

2. Sobre la unión del intestino delgado con el intestino grueso, se puede decir que:

A. Se realiza en el ciego

B. Del ciego sale una prolongación que es el apéndice vermiforme

C. Es una zona delicada porque cuando se inflama se produce la apendicitis

D. Todas son falsas.

3. El páncreas:

A. Produce insulina y glucagón

B. Produce jugo pancreático

C. Produce jugo pancreático, insulina y glucagón

D. Todas son falsas

4. Los pulmones se encuentran rodeados por una membrana serosa llamada:

A. Pericardio

B. Pleura

C. Mucosa pulmonar

D. Exopulmón

5. La respiración nos sirve:

A. Para expulsar el calor producido en el cuerpo

B. Para tomar humedad y evitar la desecación del cuerpo

C. Para tomar oxígeno y eliminar el dióxido de carbono

D. Todas son falsas

62
-Pág.146-
U . D . 7 . - L AS PERSONAS Y LA SALUD

6. En la dieta equilibrada:

A. Hay que distribuir las comidas en seis tomas

B. El 55% aproximadamente de lo ingerido deben ser glúcidos

C. Se aplica para adelgazar únicamente

D. Son ciertas la A y la B

7. La manifestación de una alteración orgánica o funcional por parte del organismo humano lo deno-
minamos:

A. Síntoma

B. Síndrome

C. Signo

D. Todas son falsas

8. La base del sistema inmunitario es:

A. Que nos inyecten sueros cuando sea preciso

B. La vacunación

C. La reacción antígeno-anticuerpo

D. La lactancia materna

9. El dimorfismo sexual es:

A. Una moda actual

B. Una característica de ciertos grupos sociales

C. Las diferencias físicas y anatómicas entre los dos sexos

D. La capacidad reproductora

10. El control del funcionamiento de los ovarios y especialmente de la ovulación se realiza:

A. Es voluntario

B. Se realiza a nivel del encéfalo

C. Se realiza a nivel de las cápsulas suprarrenales

D. A nivel hipofisario

63
-Pág.147-
C I E N C I A S D E L A N AT U R A L E Z A

RESPUESTAS A LOS EJERCICIOS

1. D

2. A

3. C

4. B

5. C

6. B

7. A

8. C

9. C

10. D

64
-Pág.148-
U . D . 8 . - C OMPONENTES A B I Ó T I C O S Y B I Ó T I C O S D E L M E D I O N AT U R A L

ÍNDICE
OBJETIVOS . . . . . . . . . . . . . . . . . . . . . . . . . . . . . . . . . . . . . . . . . . . . . . . . . . . . . . . . . . . . .2

INTRODUCCIÓN . . . . . . . . . . . . . . . . . . . . . . . . . . . . . . . . . . . . . . . . . . . . . . . . . . . . . . . . .3

MAPA CONCEPTUAL . . . . . . . . . . . . . . . . . . . . . . . . . . . . . . . . . . . . . . . . . . . . . . . . . . . . .4

DESARROLLO DE CONTENIDOS . . . . . . . . . . . . . . . . . . . . . . . . . . . . . . . . . . . . . . . . . .5

1. LOS ECOSISTEMAS . . . . . . . . . . . . . . . . . . . . . . . . . . . . . . . . . . . . . . . . . . . . . . . . .5
1.1. DIVERSIDAD DE COMPONENTES EN LOS ECOSISTEMAS . . . . . . . . . . . . . . . .5
1.2. INTERACCIONES ENTRE LOS SERES VIVOS Y LOS FACTORES ABIÓTICOS .7
1.3. PRODUCTORES, CONSUMIDORES Y DESCOMPONEDORES . . . . . . . . . . . . . .8
1.4. LAS ADAPTACIONES . . . . . . . . . . . . . . . . . . . . . . . . . . . . . . . . . . . . . . . . . . . . . . . .11
1.5. RELACIONES TRÓFICAS . . . . . . . . . . . . . . . . . . . . . . . . . . . . . . . . . . . . . . . . . . . .11
1.6. CICLOS DE MATERIA Y FLUJO DE ENERGÍA . . . . . . . . . . . . . . . . . . . . . . . . . .13

2. AUTORREGULACIÓN DEL ECOSISTEMA


EL PROBLEMA DE LAS PLAGAS. LA LUCHA BIOLÓGICA . . . . . . . . . . . .13

3. ALGUNOS ECOSISTEMAS FRECUENTES EN ESPAÑA . . . . . . . . . . . . . . . .14

RESUMEN . . . . . . . . . . . . . . . . . . . . . . . . . . . . . . . . . . . . . . . . . . . . . . . . . . . . . . . . . . . .20

EJERCICIOS DE AUTOCOMPROBACIÓN . . . . . . . . . . . . . . . . . . . . . . . . . . . . . . . . . .23

RESPUESTAS A LOS EJERCICIOS . . . . . . . . . . . . . . . . . . . . . . . . . . . . . . . . . . . . . . . . .24

-Pág.149-
C I E N C I A S D E L A N AT U R A L E Z A

OBJETIVOS
Al finalizar el estudio de esta Unidad Didáctica, el alumno será capaz de:

• Reconocer las características de los sistemas naturales o ecosistemas.

• Distinguir algunas características de los ecosistemas acuáticos y de los biomas


terrestres que forman la biosfera.

• Describir los ecosistemas más frecuentes existentes en España.

-Pág.150-
U . D . 8 . - C OMPONENTES A B I Ó T I C O S Y B I Ó T I C O S D E L M E D I O N AT U R A L

INTRODUCCIÓN
L a naturaleza es una obra maestra, todo en ella está muy bien engranado y sincroni-
zado, como si estuviera pensado para el mejor aprovechamiento. Cada zona es dife-
rente, son páginas del mismo libro con contenidos diferentes. El problema se plantea
cuando nosotros mezclamos los renglones del libro o tachamos aquello que nos parece
y al final, no se parece en nada a lo que había y, lo peor, ha perdido su equilibrio.

Los problemas medioambientales cada vez preocupan más, dada la repercusión a corto,
medio y largo plazo. El intentar mejorar la situación es cosa de todos, cada cual, con su
pequeña aportación del día a día desde el conocimiento y el sentido común.

-Pág.151-
C I E N C I A S D E L A N AT U R A L E Z A

M A PA C O N C E P T UA L
RELACIONES RELACIONES
INTRAESPECÍFICAS INTERESPECÍFICAS
Soledad Competencia
MEDIO NATURAL Colonias Depredación
Familias Simbiosis
Gregarismo Comensalismo
Sociedades Parasitismo
ECOLOGÍA
Tanotocresis
Foresia

CLIMA. COMUNIDAD
SUELO. BIOTIPO CICLO DE
AGUA... MATERIA
BIOCENOSIS

Se representan en ECOSISTEMA CADENAS CICLOS


BIOGEOQUÍMICOS
TRÓFICAS
PIRÁMIDES TIPOS Productores FLUJO
Consumidores DE
Descomponedores ENERGÍA
DEL ECUADOR A LOS POLOS

TERRESTRES ACUÁTICOS EN ESPAÑA

PLUVISELVA PLANCTON MAR MEDITERRÁNEO

PRADE- NECTON MAR CANTÁBRICO


RAS (SABANA Y
ESTEPA)
BENTOS HUMEDALES
DESIERTO
RÍOS
BOSQUE
MEDITERRÁNEO BOSQUE MEDITERRÁNEO

BOSQUE BOSQUE DE RIBERA


CADUCIFOLIO

TAIGA ROBLEDAL

TUNDRA HAYEDO

ANTÁRTIDA BOSQUE DE CONÍFERAS

ÁRTICO
-Pág.152-
U . D . 8 . - C O M P O N E N T E S A B I Ó T I C O S Y B I Ó T I C O S D E L M E D I O N AT U R A L
U . D . 8 . - C O M P O N E N T E S A B I Ó T I C O S Y B I Ó T I C O S D E L M E D I O N AT U R A L

1. LOS ECOSISTEMAS
Hemos visto cómo se desarrollan gradualmente estructuras que, según pasan a un nivel
superior, van aumentando la complejidad de los organismos. A su vez, los organismos indi-
viduales pueden dar lugar a otros niveles superiores de organización. Los individuos de una
misma especie viven juntos en el mismo lugar, constituyen una población. Las distintas
poblaciones de animales y plantas que viven juntas constituyen una comunidad o biocenosis.
En un nivel superior está el ecosistema, definido por la biocenosis y su biotopo (o área que
ocupa) y las relaciones que se establece entre ellos. El conjunto de todos los ecosistemas
forma la biosfera.
El ecosistema es la unión de biotopo y biocenosis y de las interacciones que se producen
entre ellos (y también dentro de ellos).
El biotopo, son las características físicas que definen un espacio, es decir, la composición
del suelo, el relieve, el clima y la concentración de gases y sales (en el medio acuático). Cada
biotopo está habitado por un conjunto de seres vivos, es la biocenosis.
El medio ambiente o entorno de un ser vivo está constituído por un conjunto de elementos
vivos (factores bióticos) y no vivos (factores abióticos), con los cuales establecen un conjunto
de relaciones y redes de interdependencia.
Para que un ecosistema funcione es preciso que exista un aporte de energía, que en la mayo-
ría de los casos es suministrada por el Sol (fijada por los organismos fotosintetizadores).
Un ecosistema está formado por unos determinados componentes, que determinan su estruc-
tura, desarrolla una función, que es el máximo aprovechamiento energético y tiene capacidad
de autorregulación.

1.1. DIVERSIDAD DE COMPONENTES EN LOS ECOSISTEMAS


1.1.1. EL BIOTOPO
Es el conjunto de factores abióticos de un ecosistema. Los organismos se desenvuelven en
dos grandes medios: el terrestre y el acuático y los factores que más influyen sobre ellos son los
climáticos, los del suelo y los del medio acuático.

1.1.2. FACTORES CLIMÁTICOS


El clima influye sobre los seres vivos debido a la temperatura (cambios debidos a la alti-
tud y latitud, las variaciones diarias, la luz solar); la radiación solar (muy importante para la
fotosíntesis); la humedad, muy relacionado con las precipitaciones.

5
-Pág.153-
C I E N C I A S D E L A N AT U R A L E Z A
C I E N C I A S D E L A N AT U R A L E Z A

1.1.3. FACTORES DEL SUELO


El suelo es el resultado de las interacciones de la superficie rocosa de la Tierra con los agen-
tes atmosféricos y los seres vivos, y forma el sustrato sobre el cual se desarrolla la vida en tierra
firme. La estructura y composición del suelo, el pH y la humedad son algunos de los factores que
más influyen.

1.1.4. FACTORES DEL MEDIO ACUÁTICO


La salinidad, la concentración de oxígeno disuelto en el agua, la concentración de sales
(nitratos, fosfatos, sulfatos, etc., y la presión son algunos de los factores que más influyen. La
profundidad influye sobre la luz, la presión y la temperatura y determina la distribución de las
especie, sobre todo de los organismos fotosintéticos.

1.1.5. LOS ORGANISMOS FRENTE A LOS FACTORES DEL MEDIO


Cada especie presenta un margen de tolerancia frente a los factores físico-químicos del
medio, existiendo un óptimo, lejos del cual disminuyen sus posibilidades de vida, convirtién-
dose en un factor limitante. Por ello, los factores ecológicos determinan el hábitat de cada espe-
cie, que es el espacio físico en el que viven o pueden vivir los individuos de dicha especie.

1.1.6. LA BIOCENOSIS
Una comunidad biológica o biocenosis, está formada por una agrupación de poblaciones de
seres vivos que se relacionan entre sí y que habitan en un determinado biotopo con el cual se
interactúan. La abundancia de especies que presenta una biocenosis constituye su diversidad o
biodiversidad, y puede variar mucho de unas biocenosis a otras.
Ningún organismo es independiente de otros seres vivos. Las diferentes poblaciones de
organismos viven juntas formando una comunidad y dentro de ella los individuos se relacio-
nan unos con otros y pueden establecer entre ellos distintas asociaciones. Las relaciones pueden
ser interespecíficas, es decir, entre individuos de distinta especie e intraespecífica, es decir, entre
individuos de la misma especie.
Las relaciones interespecíficas son las que se establecen dentro de una comunidad entre
individuos de especies distintas con efecto beneficioso, perjudicial o neutro (carece de efecto).
Entre éstas podemos mencionar:
– La competencia. Se establecen cuando un mismo lugar es habitado por dos o más
especies con unas necesidades iguales o parecidas, siendo un factor limitante en el
ecosistema.
– La depredación. En esta relación, una especie, el depredador, se come a otra especie,
la presa. Esta relación constituye un mecanismo de regulación de las poblaciones.

6
-Pág.154-
U . D . 8 . - C O M P O N E N T E S A B I Ó T I C O S Y B I Ó T I C O S D E L M E D I O N AT U R A L
U . D . 8 . - C O M P O N E N T E S A B I Ó T I C O S Y B I Ó T I C O S D E L M E D I O N AT U R A L

– La simbiosis. Es la relación que se establece entre dos organismos de diferentes espe-


cies, en la cual ambos resultan beneficiados, también se conoce con el nombre de
mutualismo.
– El comensalismo. En esta relación un organismo se beneficia y el otro ni se beneficia
ni se perjudica.
– El parasitismo. Es una relación donde una parte (el parásito) resulta beneficiada y la
otra (el hospedador) se ve perjudicada. Pueden ser endoparásitos, cuando viven den-
tro del hospedador o ectoparásitos, cuando viven sobre él.
– La tanatocresis. Una especie utiliza elementos de otra especie en beneficio propio sin
fines nutritivos.
– La foresia. Una especie utiliza a otra como medio de transporte.
Las relaciones intraespecíficas, se plantean cuando los individuos viven juntos en un cierto
lugar constituyendo una población. Estas relaciones pueden ser de competencia o de coopera-
ción. Las de competencia entre individuos de una misma especie son mas frecuentes pues tie-
nen que ocupar un mismo nicho ecológico. Entre estas relaciones tenemos:
– La soledad. Viven solos y sólo se relacionan con otros de su misma especie para pro-
crear, marcan los límites de su territorio.
– Las colonias. En organismos marinos, todos proceden de un mismo progenitor, ya que
se reproducen por gemación.
– La familia. Estas pueden ser temporales y permanentes.
– El gregarismo. Viven en grupos, cuando son permanentes, se llaman manadas.
– Las sociedades. Viven juntos y dependen unos de otros (insectos sociales) en socie-
dades jerarquizadas (insectos) con trabajos diferentes y repartidos entre sus miem-
bros.

1.2. INTERACCIONES ENTRE LOS SERES VIVOS


Y LOS FACTORES ABIÓTICOS
Los elementos del ecosistema interaccionan de tal forma que el conjunto formado aprove-
cha con la máxima eficacia la energía. Esta es la función del ecosistema.
Hay distintos niveles de aprovechamiento de los recursos energéticos, estos niveles reciben
el nombre de niveles tróficos.

7
-Pág.155-
C I E N C I A S D E L A N AT U R A L E Z A
C I E N C I A S D E L A N AT U R A L E Z A

1.3. PRODUCTORES, CONSUMIDORES Y DESCOMPONEDORES


Los niveles tróficos en los ecosistemas son:
– Productores: Son los organismos capaces de sintetizar la materia orgánica (fotosinte-
tizadores) a partir de la inorgánica (plantas y fitoplancton).
– Consumidores primarios. Se alimentan de los productores, son los herbívoros.
– Consumidores secundarios: Son los animales carnívoros. Se alimentan de los consu-
midores primarios (depredadores).
– Consumidores terciarios: Son los animales carnívoros que se alimentan de otros con-
sumidores secundarios (superdepredadores) o de animales muertos (carroñeros o
necrófagos).
– Descomponedores: Utilizan los desechos de los demás grupos, excrementos y cadá-
veres, para su alimentación (hongos, animales pequeños y bacterias).
Los vegetales no dependen de los demás seres vivos para su nutrición, puesto que son orga-
nismos autótrofos. Ellos producen materia para todos los demás seres de la comunidad y for-
man el primer nicho trófico, el de los productores. Todos los demás organismos son heterótro-
fos, y obtienen el alimento a partir de los productores, por lo que se les llama consumidores.
Los animales herbívoros se alimentan directamente de los productores y constituyen el nivel
trófico de los consumidores primarios. Los carnívoros que se alimentan de los herbívoros, son
consumidores secundarios y pueden servir de alimento a otros carnívoros que son los consumi-
dores terciarios. Los detritívoros y los carroñeros, son consumidores que se alimentan de restos
de materia orgánica y lo mismo sucede con los descomponedores, hongos y bacterias que degra-
dan la materia orgánica y reciclan así los nutrientes.
Si un determinado recurso es utilizado por distintas especies de la misma biocenosis, entre
ellas se establece una relación de competencia. El recurso puede ser el alimento, el lugar de
nidificación o cualquier otro. Si la competencia entre dos especies es total, se dice que ocupan
el mismo nicho ecológico y, antes o después, una de las dos especies desaparecerá de la bioce-
nosis. Nicho ecológico es la función que desempeña una determinada especie dentro de un eco-
sistema.

1.4. LAS ADAPTACIONES


La adaptación es el resultado de un proceso evolutivo que supone el desarrollo de caracte-
rísticas que permiten a los organismos sobrevivir y reproducirse en un ambiente determinado.
Si las condiciones de ese ambiente cambian y los organismos no son capaces de adaptarse a esos
cambios, mueren y toda su especie puede extinguirse.

8
-Pág.156-
U . D . 8 . - C OMPONENTES A B I Ó T I C O S Y B I Ó T I C O S D E L M E D I O N AT U R A L

Hay veces en que las adaptaciones surgen de la interrelación de los animales de una deter-
minada especie con el ambiente que les rodea, por ejemplo, los linces de climas fríos son más
corpulentos que los de climas cálidos para mantener mejor el calor corporal. Otras veces se
observan adaptaciones en especies distintas provocadas por las relaciones mantenidas entre
estas especies. Por ejemplo, las flores y los polinizadores, tienen estructuras adecuadas para
establecer la relación; a estas adaptaciones se les denomina coevolución.

1.4.1. ADAPTACIONES A LA TEMPERATURA


Los animales que no regulan su temperatura interna se llaman poiquilotermos (de sangre
fría), la temperatura depende de la del medio y no sobreviven demasiado tiempo en climas fríos.
Los lagartos, los anfibios pertenecen a este grupo.
Por el contrario, las aves y mamíferos son homeotermos (de sangre caliente) y son capaces
de regular su temperatura interna y mantenerla estable gracias a mecanismos fisiológicos. Estos
animales además presentan adaptaciones especiales como color y espesor del pelaje, cantidad de
grasa corporal, etc. Dentro de este grupo existen organismos que soportan un estrecho margen de
temperaturas (organismos estenotermos) y deben vivir en climas estables y otros resisten
amplias diferencias de temperaturas (euritermos), habitan en regiones templadas en las que se
dan grandes variaciones estacionales de temperatura.
Existen algunos animales mamíferos que poseen un reloj biológico activado por cambios en
el ambiente (iluminación, temperatura,...) que les hace disminuir su temperatura corporal hasta
situarse un grado por encima de la ambiental y pasar así la estación invernal durmiendo, por dis-
minuir su metabolismo y el gasto energético, a este proceso se le denomina hibernación.
Ciertos animales acuáticos son capaces de resistir las bajas temperaturas añadiendo a su san-
gre sustancias "anticongelantes" naturales producidas por su metabolismo o quedándose en
zonas profundas donde la temperatura es más estable.
Hay aves que evitan el frío emigrando durante las estaciones frías hacia el sur volviendo
cada primavera, son las aves migratorias.

1.4.2. ADAPTACIONES A LA LUZ (DÍA Y NOCHE)


La luz es un elemento que influye de forma diferente en distintos tipos de animales. Para las
plantas es imprescindible para realizar la fotosíntesis. Existen animales diurnos, que realizan su
actividad durante el día y otros nocturnos. Esta característica les permite evitar la competencia
y poder vivir en el mismo hábitat.
En el agua, la intensidad de la luz disminuye conforme aumenta la profundidad, hasta donde
entra la luz, se le denomina zona fótica que alcanza hasta los 100 o 200 metros (en la superfi-
cie se sitúa el fitoplancton, debajo los animales que se alimentan de él y en el límite entre las
dos zonas los animales que se alimentan del plancton y suben de noche a comer) y hacia abajo

9
-Pág.157-
C I E N C I A S D E L A N AT U R A L E Z A

se sitúa la zona afótica, es la noche eterna, de aguas siempre frías, existen adaptaciones para
producir luz, se dice que son bioluminiscentes.
En el medio terrestre, los organismos tienen células sensibles a la luz agrupadas en órganos
denominados ojos; lo animales de vida nocturna, tienen las pupilas grandes, incluso los de vida
subterránea tienen ojos atrofiados, puesto que no los necesitan ni los utilizan.
Las plantas disponen las hojas para exponer al Sol el máximo número de ellas, presentando
fototropismo, es decir se mueven buscando la iluminación (por ejemplo, los girasoles) y cre-
cen siempre buscando la máxima iluminación.

1.4.3. ADAPTACIONES A LA NECESIDAD DE AGUA


Las adaptaciones son muy diversas según las necesidades. En el medio acuático, los orga-
nismos se tienen que adaptarse al agua como medio de vida. Los peces presentan la vejiga nata-
toria que la llenan o vacían de agua para controlar su flotabilidad. Por otro lado tienen que
adaptarse a la viscosidad del medio para lo que adoptan formas hidrodinámicas.
En el medio terrestre los seres vivos se proveen de estructuras para evitar la desecación,
mediante epidermis y tegumentos protectores, almacenando agua en su interior, evitando la
exposición directa al sol, reduciendo la cantidad de agua excretada en la orina por reabsorción
en sus nefronas. Las plantas que viven en climas secos, reducen el número de hojas y engrosan
los tallos (cactus), mientras que en lugares húmedos presentan grandes hojas para transpirar
mejor.
Para adaptarse a la sequía las plantas poseen formas de resistencia (las semillas) y ciertas
bacterias y hongos poseen esporas que les permiten mantenerse como vida latente en espera de
que mejoren las condiciones.

1.4.4. ADAPTACIONES A LA SALINIDAD


El equilibrio e intercambio de sales entre el interior y el exterior del organismo se denomi-
na ósmosis. Todos los organismos presentan unas concentraciones interiores, y varían en gene-
ral de las del exterior y aunque la diferencia sean las sales, lo que realmente entra o sale es agua
y las adaptaciones van encaminadas a mantener el equilibrio interno (nosotros mismos somos
un ejemplo, si comemos un plato de jamón, aceitunas, ..., tenemos sed porque tiene una gran
cantidad de sal y necesitamos ingerir agua para equilibrar la cantidad de sales, posteriormente,
este agua y el exceso de sal se eliminará por la orina).
Los animales los dividimos en poiquilosmóticos, no pueden regular su concentración de
sales interna y la mantiene con valores similares a los del medio y homeosmóticos que sí regu-
lan y pueden vivir en ambientes de salinidad diferente a la de su medio interno.

10
-Pág.158-
U . D . 8 . - C OMPONENTES A B I Ó T I C O S Y B I Ó T I C O S D E L M E D I O N AT U R A L

1.5. RELACIONES TRÓFICAS


Los organismos que forman la biocenosis, establecen relaciones alimentarias entre ellos y
así se producen las interacciones de los niveles tróficos. Estas relaciones alimentarias son el fun-
damento del flujo lineal de la energía que parte del Sol y es captada por los productores median-
te la fotosíntesis. De ellos pasa a los consumidores primarios, a los consumidores secundarios,
y así sucesivamente hasta los descomponedores, en una serie de escalones que constituyen una
cadena trófica. De forma paralela al flujo de energía y en el mismo sentido se produce el flujo
de materia.
En una biocenosis suele haber distintas especies en cada uno de los niveles tróficos y, ade-
más, cada especie de consumidor se puede alimentar de diferentes especies de los niveles infe-
riores, por ello es frecuente que nos encontremos con una compleja trama de cadenas que se
unen y forman redes tróficas.

1.6. CICLOS DE MATERIA Y FLUJO DE ENERGÍA


De todas las relaciones que hay en un ecosistema, predominan aquellas que se establecen
entre los animales, las plantas (o las algas) y los microorganismos para buscar alimento. La vida
de los ecosistemas depende del flujo de energía, que suministra el Sol, que pasa de unos orga-
nismos a otros a través de las cadenas tróficas o alimentarias. Estas relaciones son el funda-
mento del flujo lineal de la energía.
Una cadena trófica, muestra quién come y quién es comido: muestra como fluye la energía
–almacenada en los nutrientes– de unos organismos a otros.
Para que la energía solar resulte útil a los seres vivos, necesita ser transformada. La prime-
ra transformación tiene lugar en las plantas y las algas, que convierten la energía de la luz solar
en energía química (almacenada en los principios inmediatos) mediante la fotosíntesis. El flujo
de materia, se produce de forma paralela y en el mismo sentido que el de energía.
No obstante, la materia utilizada por los seres vivos es un recurso limitado que debe ser reci-
clada, esta función la desempeñan los organismos descomponedores, que transforma la materia
orgánica en inorganica, permitiendo que continue el ciclo de producción de materia orgánica
por parte de los productores primarios gracias al aporte de energía solar.
El flujo de energía es el factor más importante en la organización de los ecosistemas. Se cal-
cula que solamente llega a los seres vivos el 0,1% de toda la energía solar que llega a la Tierra
y sólo entre el 1 y 3% de esa energía se aprovecha para la fotosíntesis. En este punto se inicia
el flujo de energía a través de los niveles tróficos de la biocenosis, que se emplea para desarro-
llar los procesos vitales de los individuos del nivel trófico considerado. Sólo una pequeña parte
es retenida como biomasa y podrá ser utilizada por los niveles siguientes.

11
-Pág.159-
C I E N C I A S D E L A N AT U R A L E Z A

Se denomina biomasa a la cantidad de materia total del conjunto de organismos que se con-
sidera, por ejemplo, un nivel trófico.
Se llama producción bruta a la cantidad total de energía fijada por un determinado nivel
trófico por unidad de tiempo. Si a esa cantidad se le resta la energía que se pierde en los proce-
sos de respiración se obtiene la producción neta.
De la energía almacenada en una planta, es transformada en biomasa por el animal que la
come el 10%, que es el aprovechamiento que se calcula en cada transferencia. Estas relaciones
energéticas condicionan el número de individuos que hay en cada nivel trófico.
Los ecosistemas se representan mediante pirámides ecológicas, donde se representan en
escalones, de abajo hacia arriba desde los productores a los consumidores secundarios. Estas
pirámides pueden representar el flujo de energía y que posee cada nivel trófico; pirámide de
números, donde se representan el número de individuos (suelen parecer pirámides invertidas) y
pirámides de biomasa, donde se representa la biomasa de cada nivel trófico.
En los ecosistemas, como ya se ha indicado, la materia ha de reciclarse, es decir, pasar de
materia orgánica a inorgánica para que así, los productores puedan realizar su función. Como
también existen procesos geológicos implicados, se denominan ciclos biogeoquímicos. Los
más importantes son los del carbono, nitrógeno, fósforo y azufre.
En el ciclo del carbono, tiene una gran importancia el CO2 (dióxido de carbono), una molé-
cula gaseosa y estable que permite el paso del carbono entre la atmósfera, la hidrosfera, la litos-
fera y la biosfera. El CO2 es soluble en agua y puede ser fijado por los organismos fotosintéti-
cos terrestres y acuáticos, que así lo introducen en las cadenas tróficas. Al reaccionar con el
agua, el dióxido de carbono forma bicarbonatos que pueden precipitar como carbonatos y for-
mar rocas calizas.
El nitrógeno (N2) es el gas más abundante en la atmósfera y puede pasar a los seres vivos
por un proceso de fijación biológica o ser transformado en otras formas moleculares por la
acción de las radiaciones solares, de las descargas eléctricas o del ser humano, lo que constitu-
ye un proceso de fijación química. La intervención del hombre en el ciclo consisten sobre todo
en la fabricación de amoníaco (NH3) y abonos y en la liberación de óxidos de nitrógeno (N2Ox)
a la atmósfera por procesos de combustión. En este ciclo, tienen una importancia crucial las bac-
terias, ya que la fijación biológica como la transformación del amoníaco en nitratos es realiza-
da por determinados grupos de bacterias especializadas.
El fósforo, es el factor limitante más importante de los ecosistemas, es eminentemente sedi-
mentario. El fosforo llega a los productores en forma de ión fosfato (PO43-). En los animales se
acumula en el esqueleto y al depositarse, pasa a los sedimentos. La agricultura intensiva emplea
como abono los fosfatos obtenidos de yacimientos minerales y del guano que está formado por
la acumulación de excrementos de aves piscívoras.

12
-Pág.160-
U . D . 8 . - C OMPONENTES A B I Ó T I C O S Y B I Ó T I C O S D E L M E D I O N AT U R A L

El azufre, se encuentra en la litosfera en forma de distintos minerales, principalmente sul-


furos y en combustibles como el carbón. También se incorpora a partir de las erupciones vol-
cánicas. El paso desde el agua y desde el suelo a los seres vivos se realiza en forma de ión sul-
fato (SO42-) que es reducido a la forma H2S para formar algunas biomoléculas y de esta manera
incorporarse a la cadena trófica.

2. AUTORREGULACIÓN DEL ECOSISTEMA.


EL PROBLEMA DE LAS PLAGAS. LUCHA BIOLÓGICA
Un ecosistema tiene la capacidad de autorregularse y mantenerse en equilibrio dinámico, a
lo largo del tiempo, y cuanto mayor sea su complejidad, mayor será su capacidad de autorregu-
lación.
El número de individuos que puede tener una población cualquiera de un ecosistema estará
regulado por una serie de factores, como los físico-químicos, la disponibilidad de espacio, de
alimento, o la presencia de especies depredadoras y competidoras. Si no hubiera factores que
limitaran el crecimiento la población crecería ilimitadamente.
No obstante, los ecosistemas cambian con el tiempo. La secuencia de cambios graduales
experimentados por un ecosistema a lo largo del tiempo recibe el nombre de sucesión ecoló-
gica.
La aparición de un biotopo nuevo, como puede ser una isla volcánica, permite que se insta-
len en ella los primeros seres vivos, son los pioneros, como algunos líquenes. Estos organismos
viven directamente sobre la roca e inician su alteración para formar el suelo. La modificación
del biotopo permite que se instalen nuevas especies que sustituyen a las anteriores en lo que
constituye el comienzo de una sucesión primaria. Una sucesión secundaria es la que se produ-
ce tras la desaparición de una comunidad establecida anteriormente, como sucede tras un incen-
dio o el abandono de un campo de cultivo.
A medida que la sucesión avanza, el número de especies se va haciendo mayor, la comple-
jidad de la biocenosis aumenta y las condiciones del biotopo se van modificando hasta que se
llega a una comunidad en equilibrio con las condiciones ambientales denominada comunidad
climática o comunidad clímax. Un ecosistema maduro, con una comunidad climática instala-
da, tiene la mayor complejidad de todas las que han ido apareciendo a lo largo de la sucesión,
una mayor biomasa y un aprovechamiento energético más eficaz.
La tala masiva y los incendios son las causas más importantes de la desaparición de muchas
de las comunidades clímax en todo el mundo. El uso excesivo de abonos y su arrastre a las
aguas provoca la eutrofización del agua que destruye los ecosistemas acuáticos.

13
-Pág.161-
C I E N C I A S D E L A N AT U R A L E Z A

Hoy en día, uno de los principales problemas a los que nos enfrentamos es la superpobla-
ción. Thomas Malthus afirmaba que en el siglo XXI, no hay contaminación, sino personas.
Hablamos de contaminación porque los 6000 millones de personas que poblamos actualmente
el planeta, liberamos al medio ambiente millones de toneladas de sustancias imposible de reci-
clar por procesos naturales. El agujero de la capa de ozono, la agricultura intensiva para dar de
comer a todos los habitantes que precisa de grandes cantidades de productos químicos, la repo-
blación forestal con especies no autóctonas, las quemas y talas de árboles, el turismo que acce-
de a lugares que servían de refugio que contaminan (ruidos, basuras, etc.) y destruyen el eco-
sistema; las combustiones de carbón y petróleo que produce la lluvia ácida, el efecto
invernadero producido por el acúmulo de dióxido de carbono; el agotamiento de los recursos
por la sobreexplotación que destruye el medio natural y contamina.
Cada vez es más importante que nos concienciemos de no despilfarrar la energía ni los
recursos en general: reciclar las basuras, reutilizar los envases de plástico y papel, ahorrar ener-
gía eléctrica, utilizar productos ecológicos y respetuosos con el medio ambiente, evitar los pro-
ductos desechables y los envases innecesarios y reparar los objetos estropeados.
Las plagas constituyen un problema de la agricultura intensiva, habitualmente se han em-
pleado pesticidas y plaguicidas para luchar contra ellas y, aunque acaban con el problema, gene-
ran otros ya que permanecen en el vegetal y en el suelo afectando en la cadena trófica a todo el
ecosistema. En la actualidad, se plantea el empleo de especies animales muy concretas.

3. ALGUNOS ECOSISTEMAS FRECUENTES EN ESPAÑA


Un ecosistema puede ser algo muy grande o muy pequeño. Tan grande como la biosfera o
tan pequeño como una charca, un roble o un tronco caído.
La biosfera, es el gran ecosistema constituido por todos los seres vivos y una franja de la
propia Tierra que se extiende por la superficie terrestre, la capa más baja de la atmósfera y las
aguas de mares y océanos.
Pero resulta difícil estudiar el ecosistema a escala global. Los organismos no se distribuyen
uniformemente en la biosfera, porque presentan características climáticas y ambientales muy
diferentes y gran variedad de hábitats distintos de unas regiones a otras. Cuanto más pequeño
es un ecosistema, más simple es su estructura y función, y más fácil es su estudio. La biosfera
se suele dividir en dos grandes ecosistemas: acuático (marino y de agua dulce) y terrestre. Éste
a su vez, se divide en otros más pequeños, llamados biomas.
Cada biomas está constituído por una gran comunidad o biocenosis de organismos terrestres
que ocupan una extensa área, cuyas condiciones climáticas y ambientales son muy parecidas.

14
-Pág.162-
U . D . 8 . - C OMPONENTES A B I Ó T I C O S Y B I Ó T I C O S D E L M E D I O N AT U R A L

La latitud, es decir, su distancia al ecuador, es el factor que más determina los biomas terrestres.
Desde la zona ecuatorial hasta los polos a nivel terrestre podemos distinguir los siguientes
ecosistemas:
– La pluviselva. Es la selva ecuatorial y tropical, donde abundan las precipitaciones y
crece una flora exuberante donde se concentran la mayor parte de las especies cono-
cidas.
– Las praderas: Sabana y estepa. Son extensas zonas que presentan algunos árboles y
arbustos dispersos con grandes extensiones de hierba, donde viven grandes rebaños
herbívoros y sus depredadores naturales.
– El desierto. Sus precipitaciones son débiles y su escasa vegetación y fauna que alber-
ga suelen presentar mecanismos de adaptación para retener el agua.
– El bosque mediterráneo. Entre los árboles predominan los pinos, alcornoques y
encinas y entre los arbustos predominan los brezos, madroños, retamas y jaras. Tanto
su flora como su fauna está adaptada a sobrevivir durante los veranos cálidos y secos.
– El bosque caducifolio. Es el bosque de las regiones templadas, formado fundamen-
talmente por árboles de hoja caduca (robles, arces, fresnos, hayas, abedules, alisos, ...)
que proporcionan numerosos hábitats para una gran variedad de animales.
– La taiga. Significa "bosque frío". Es el bosque boreal o bosque de coníferas (cedros,
abetos y piceas) que se extiende al sur de la tundra, donde viven los osos pardos, los
ciervos, los alces, los castores y gran número de aves como el águila real y el urogallo.
– La tundra. Cerca del ártico, la tierra se congela durante el largo y oscuro invierno.
Pero al llegar el verano, el deshielo pone de manifiesto un tapiz verde, formado por
musgos, líquenes y un gran número de flores que alimentan a un enjambre de insec-
tos de todo tipo. Es esta estación hacen su aparición los rebaños de caribús, los alces,
los lobos, los zorros, las liebres, los linces y gran variedad de aves.
– La antártida. Es un continente cubierto de hielo y rodeado de un banco de hielo
marino. Las tierras heladas están casi desiertas, pero las aguas del océano Antártico
rebosan de vida a partir de la primavera: peces, focas, ballenas, pingüinos, etc.
– El ártico. El océano Artico es un inmenso mar de hielo. Cuando parte del hielo se
funde en sus contornos durante el verano se produce una explosión de vida: focas,
morsas, osos polares, ballenas y una gran variedad de peces.
Entre los ecosistemas acuáticos hay que diferenciar entre el medio marino y los de agua
dulce.
Los océanos no forman un medio ambiente uniforme, ni la vida se reparte homogéneamen-
te, pues las características físicas y químicas del biotopo marino experimentan variaciones
considerables de salinidad, temperatura, profundidad, luminosidad, corrientes de agua, etc., de
unos lugares a otros.
La mayor parte de los organismos marinos viven cerca de la costa, sobre la plataforma con-
tinental, hasta donde penetra la luz. Los organismos marinos que viven sobre el fondo se deno-
minan bentónicos (como los pólipos) y los que viven libres por el agua se denominan pelági-
cos. Éstos a su vez pueden ser nadadores activos (como los peces) y constituyen el necton; o

15
-Pág.163-
C I E N C I A S D E L A N AT U R A L E Z A

bien carecer de órganos nadadores, por lo que flotan a la deriva y forman el plancton. Los orga-
nismos del plancton son microscópicos y pueden tener dos orígenes diferentes: una parte pro-
cede de ciertas algas microscópicas (fitoplancton) y otra de las larvas y alevines de algunos ani-
males (zooplancton). Toda la actividad del ecosistema marino se basa en los productores
primarios, constituidos por las algas verdes, rojas y pardas y por el fitoplancton.
Dentro de las aguas dulces, tenemos los ríos que en su recorrido a través de sus tramos alto,
medio y bajo, fertiliza de una manera u otra a casi todos los ecosistemas terrestres y crea nume-
rosos y diversos hábitats que albergan a una gran variedad de plantas, algas, animales y micro-
organismos.
Los remansos, donde el agua no corre, suelen formarse en aquellos lugares en los que aflo-
ran las aguas subterráneas; suelen ser ecosistemas muy débiles y en sus orillas suele crecer exu-
berante vegetación donde viven numerosas aves acuáticas.

En España, los ecosistemas existentes dependen de varios factores:


1. El tipo de suelo: arcilloso, calcáreo o silíceo.
2. El relieve. Las especies que podemos encontrar varían con la altitud.
3. El clima.
Los ecosistemas en España, también se dividen inicialmente en terrestres y acuáticos (mari-
nos y de agua dulce). Los terrestres, vienen definidos sobre todo por la especie vegetal domi-
nante.
El mar Mediterráneo, es un mar cerrado que sólo comunica con el océano Atlántico por 14
Km. de estrecho en Gibraltar, esto unido al bajo aporte de agua de los continentes secos que lo
rodean hace que tenga una concentración salina mayor. En los profundos ambientes marinos,
predominan los animales frente a las plantas por la falta de luz. Entre los 20 y 60 metros de pro-
fundidad se encuentra el ecosistema conocido como coralígeno, que se caracteriza por el pre-
dominio de algas calcificadas. Existen dos tipos, uno de roca litoral, que se genera sobre sus-
trato duro y otro de plataforma, que se genera sobre sustrato blando a partir de restos orgánicos
consolidados por las algas. Ambos presentan básicamente la misma estructura: un estrato arbo-
rescente (Gorgonia y Pajarita), un estrato intermedio (Esponjas, pluma de mar, pepino de mar
negro y ascidia roja) y un estrato incrustado (algas calcareas, coral rojo, anémona, gusano tubí-
cola, quisquilla, langosta y ofiura). Entre los peces que viven en este ecosistema figuran el con-
grio, cabracho, atún, mero, sargo y dorada.
A nivel litoral, encontramos dos zonas, una inframareal, que permanece cubierta siempre por
las algas, habitada por estrellas de mar, cangrejos, erizos, medusas y cangrejos. La vegetación
está compuesta por algas como la lechuga de mar. Es importante la presencia de una planta que
forma verdaderas praderas submarinas donde habitan un gran número de animales es la Posei-
donia, que es endémica de aquí, es decir sólo se encuentra aquí. La otra zona, la zona interma-
real, comprende la zona situada entre la que ocupa la marea baja y la marea alta. Está habitada
por organismos que no pueden estar todo el tiempo bajo el agua ni lejos de ella como gusanos
arenícolas y crustáceos como berberechos, almejas y navajas. Existe una zona supramareal que

16
-Pág.164-
U . D . 8 . - C OMPONENTES A B I Ó T I C O S Y B I Ó T I C O S D E L M E D I O N AT U R A L

reciben la humedad y la brisa marina, es muy común la pulga de arena y en las dunas la arena-
ria y el cardo marino.
Es de señalar así mismo el impacto medioambiental producido por el turismo y la industria
que vierten residuos sólidos, aguas residuales sin depurar y productos químicos de la industria
y la agricultura.
Los humedales, es una zona de la superficie terrestre que está temporal o permanentemen-
te inundada, regulada por factores climáticos y en constante interrelación con los seres vivos
que la habitan. Los mayores humedales de España son el Coto de Doñana, en la desemboca-
dura del Guadalquivir. En invierno se refugian en él multitud de aves procedentes del norte de
Europa, en primavera muchas otras construyen allí sus nidos. Las Tablas de Daimiel es el nom-
bre del humedal manchego situado en la confluencia de dos afluentes del Guadiana con acuífe-
ros subterráneos en una zona caliza con estructura cárstica. El encauzamiento de los ríos para
usos agrícolas, la roturación y la conversión en pastizales de la zona, junto con la extracción
masiva de agua subterránea, han supuesto la destrucción parcial de este enclave.
Los ecosistemas de las zonas húmedas son de gran fragilidad, debido principalmente a la
dependencia del agua y siendo determinante la influencia de la zona colindante. La calidad y
abundancia de agua determinan las características del ecosistema.
Los ríos, presentan condiciones de vida diferentes a lo largo de su recorrido, así, en el tramo
alto, existen algas tapizando el fondo y unos pocos animales se han adaptado a la fuerte corrien-
te. En el tramo medio, se asientan algas y plantas, sobre todo en la orilla; también hay anima-
les que se sitúan a distintos niveles en el curso del río. Es en el tramo bajo donde se desarro-
llan un gran número de seres vivos aunque el fondo está poco iluminado con lo que esta zona
está poco habitada. Entre la vegetación tenemos fitoplancton (algas microscópicas), algas y
plantas como cañizares, carrizos, espadañas, eneas o junqueras y nenúfares. En la fauna encon-
tramos zooplancton (animales microscópicos); invertebrados como caracoles, cangrejos de río,
libélulas, efímeras, zapateros y ditiscos; anfibios como salamandra rabilarga, tritones y rana
común; reptiles como galápagos y culebras de collar y peces que se distribuyen por tramos, así
en el tramo alto hay truchas y salmones, en el tramo medio anguilas, barbos y lucios y en el
tramo bajo carpas, tercas y esturiones.
Los bosques de ribera o galería, flanquea ciertos tramos de los ríos, creando un grado de
humedad alto, suavizando las condiciones climáticas. Cuando el río crece, inunda estas zonas,
dejando sedimentos que aumentan la fertilidad de las tierras. Todo esto determina especies
características como son los sauces, álamos y alisos y en la parte del soto más alejada de la
corriente de agua crecen chopos, fresnos y olmos. Estos bosques, por su estructura se llaman
bosques de galería. La fauna característica está formada por aves como el cormorán, el martín
pescador, la focha común, la polla de agua, el ánade real, la oropéndola y el ruiseñor y mamí-
feros como la nutria (en los tramos altos), el turón y la rata de agua.
El bosque mediterráneo, está adaptado a las condiciones climáticas de inviernos suaves y
húmedos, una irregular distribución de lluvias y veranos calurosos sin lluvias. Esta vegetación

17
-Pág.165-
C I E N C I A S D E L A N AT U R A L E Z A

recibe el nombre de esclerófila, con raíces profundas, corteza gruesa y hojas pequeñas, rígidas
y coriáceas. En el estrato arbóreo destacan las encinas, alcornoques y pinos; en el estrato arbus-
tivo coscojas, madroños, enebros y jaras y en el estrato herbáceo, plantas aromáticas como tomi-
llo, lavanda y romero. Respecto a la fauna se encuentran mamíferos, como erizo, lince, gineta,
meloncillo, gato montés, conejo, venado, gamo, jabalí y zorro; reptiles como culebra bastarda,
lagarto ocelado y víbora y aves como el buitre negro, buitre leonado, águila imperial, águila
culebrera, cárabo, torcaz, gorrión y perdiz.
A partir del bosque mediterráneo, por degradación progresiva, se ha convertido en dehesas
y pastos (en los que hay equilibrio ecológico) o eriales y estepas (de gran inestabilidad).
El robledal, es un bosque templado caducifolio, que se sitúa en la zona norte de la penín-
sula. La vegetación está formada por árboles como robles, abedules, arces, fresnos, tilos, ser-
bales y en algunas zonas castaños y nogales (introducidas por el hombre para su explotación);
arbustos como endrinos, madreselvas, rosales silvestres, espinos y retamas y en el estrato her-
báceo hepáticas, fresales y gran variedad de hongos. La fauna está formada por mamíferos: ardi-
lla, ciervo, corzo, erizo, jabalí, lirón, careto, lobo, oso, tejón, zorro, ratón y topillo; anfibios:
sapo, rana común y rana bermeja; reptiles: lución y lagarto verde y aves: camachuelo, buho,
chochín, zorzal, cárabo, halcón peregrino y urogallo (en peligro de extinción junto al oso).
El hayedo, se sitúa en las montañas del norte de la península y del Sistema Central, por enci-
ma de los bosques de robles. Las hayas, árboles caducifolios, son poco dados a compartir el
suelo con otras especies. Sólo en primavera, crece abundante hierba sobre el suelo.
Los bosques de coníferas, se sitúan en las zonas más altas y frías. Esta zona padece una
importante sequía en invierno debido a que la mayoría de las precipitaciones son en forma de
nieve. Las coníferas presentan hojas en forma de acículas para adaptarse a esta sequía y no per-
der agua. Las especies que aparecen en las montañas españolas son el pino negro, el pino sil-
vestre y los abetos, bien en bosques monoespecíficos, bien en mixtos. En esta región, por enci-
ma del bosque de coníferas, podemos encontrar una zona de matorral de alta montaña formado
por piornos, sabinas y enebros rastreros. La fauna está formada por mamíferos como la marta,
ardilla, armiño y rebeco; anfibios como tritón pirenaico y aves como el águila real y el pito
negro.
El litoral cantábrico, es rocoso, de amplias mareas y fuerte oleaje. El mar Cantábrico posee
un grado de salinidad menor que el mar Mediterráneo y de aguas más frías aunque no tanto
como el océano Atlántico. Posee una gran variedad de especies. Aves como el ostrero, la gavio-
ta, el cormorán y el charrán o golondrina de mar. En la zona supramareal (los seres viven siem-
pre emergidos) líquenes y bígaros. En la zona intermareal, está sometida a oleajes, corrientes y
mareas; los vegetales son algas verdes o rojas y la fauna está compuesta por invertebrados como
la lapa, mejillón, percebe, cangrejo común, quisquilla, navaja, berberecho, almeja, estrella de
mar, erizo de mar, anémona y actinia y entre los vertebrados el baboso y en la zona inframare-
al abundan las algas; los invertebrados como pulpo, centollo, buey, sepia, langosta y bogavan-
te y vertebrados como cabracho, lubina, lenguado, rodaballo, platija, dorada, faneca y congrio.

18
-Pág.166-
U . D . 8 . - C OMPONENTES A B I Ó T I C O S Y B I Ó T I C O S D E L M E D I O N AT U R A L

Mar adentro, en las proximidades del fondo, viven invertebrados como la cigala y vertebrados
como la raya común, rape, maruca, jurel, cazón y besugo y cerca de la superficie viven la sar-
dina, arenque, caballa, bonito, delfín y ballena común.
Además de la península, encontramos los archiélagos Balear y Canario. El archipiélago
Balear es una prolongación de la plataforma continental levantina y tiene unas características
físicas, climáticas y biológicas, en general muy similares a las de Levante.
El archipiélago Canario, tiene una latitud más cercana al ecuador que la península Ibérica,
con un origen volcánico. En ellas se pueden distinguir varias zonas con características muy dife-
rentes. La zona sur del archipiélago canario es desértica con humedad casi nula y gran insola-
ción. Presenta ecosistemas de dunas y pedregosos. La vegetación es gruesa, con espinas y recu-
brimiento céreo para no perder agua. Las especies más representativas son los cardones y las
tabaibas. Un animal propio y endémico es el lagarto gigante de la isla del Hierro. En las zonas
cubiertas por la lava crecen líquenes. En las partes bajas de los barrancos hay palmerales, taja-
rales y saucedas. En riscos y acantilados aparecen los últimos ejemplares silvestres de drago.
En la parte alta de las montañas se encuentra el bosque de pinos canarios, que soportan la mayor
oscilación térmica. En la parte más alta puede haber cedros.
La laurisilva o bosque de la niebla del archipiélago canario, se sitúa en la zona de media
montaña, bajo un clima benigno y húmedo, crecen bosque subtropicales. El estrato arbóreo es
muy denso y está compuesto por laureles y viñátigo; el sotobosque está compuesto por helechos
y musgos, abundan también las lianas como la correhuela del monte y la hiedra, estando situado
a la altura de los bancos de nubes. Algunas aves características son el chau-chau o pinzón vul-
gar, el hornero, el mosquitero común, el mirlo, el reyezuelo, el capirote, el petirrojo, la paloma
rabiche, paloma turqué y el gavilán.

19
-Pág.167-
C I E N C I A S D E L A N AT U R A L E Z A

RESUMEN

El ecosistema es la unión de biotopo y biocenosis y de las interacciones que se producen


entre ellos (y también dentro de ellos) que es el máximo aprovechamiento energético y tiene
capacidad de autorregulación.
El biotopo, son las características físicas que definen un espacio (factores abióticos), es
decir, la composición del suelo, el relieve, el clima y la concentración de gases y sales (en el
medio acuático). Cada biotopo está habitado por un conjunto de seres vivos, es la biocenosis.
La abundancia de especies que presenta una biocenosis constituye su diversidad o biodiversi-
dad, y puede variar mucho de unas biocenosis a otras.
Las poblaciones de organismos de la comunidad de la biocenosis, se establecen relaciones
que pueden ser interespecíficas, es decir, entre individuos de distinta especie (competencia,
depredación, simbiosis, comensalismo, parasitismo, tanatocresis y foresia) e intraespecífica, es
decir, entre individuos de la misma especie (soledad, colonias, familias, gregarismo y socieda-
des).
Para que un ecosistema funcione es preciso que exista un aporte de energía, que en la mayo-
ría de los casos es suministrada por el Sol (fijada por los organismos fotosistetizadores).
Los distintos niveles de aprovechamiento de los recursos energéticos reciben el nombre de
niveles tróficos. Son los productores (organismos fotosintéticos), consumidores primarios (her-
bívoros), consumidores secundarios (carnívoros), consumidores terciarios y descomponedores
(detritívoros y carroñeros).
Nicho ecológico es la función que desempeña una determinada especie dentro de un ecosis-
tema.
La adaptación es el resultado de un proceso evolutivo que supone el desarrollo de caracte-
rísticas que permiten a los organismos sobrevivir y reproducirse en un ambiente determinado.
Si las condiciones de ese ambiente cambian y los organismos no son capaces de adaptarse a esos
cambios, mueren y toda su especie puede extinguirse.
Según la temperatura, los organismos pueden ser poiquilotermos (de sangre fría) y ho-
meotermos (de sangre caliente). Dentro de estos últimos, pueden ser estenotermos, si sopor-
tan un estrecho margen de temperaturas y euritermos, que resisten amplias diferencias de
temperaturas.
Algunos animales mamíferos que poseen un reloj biológico activado por cambios en el
ambiente, que les hace reducir su metabolismo y dormir durante el invierno, es la hibernación.
La migración es otra adaptación que practican muchas aves y vuelan hacia el sur antes del
invierno y retornan en primavera.

20
-Pág.168-
U . D . 8 . - C OMPONENTES A B I Ó T I C O S Y B I Ó T I C O S D E L M E D I O N AT U R A L

La luz es un factor decisivo. Existen animales diurnos y nocturnos que comparten el hábi-
tat. En las plantas es decisiva para realizar la fotosíntesis, presentando así mismo fototropismo
hacia la luz. En los seres acuáticos se distinguen dos zonas, la fótica hasta los 100 o 200 metros
y la afótica donde es noche eterna y se encuentran organismos bioluminiscentes.
El agua es otro factor limitante. Los peces adoptan formas hidrodinámicas para poder
moverse y presentan vejiga natatoria para controlar su flotabilidad. En el medio terrestre, para
evitar la desecación las plantas se protegen con epidermis y tegumentos protectores, engrosan
los tallos, reducen las hojas en zonas secas, presentan formas de resistencia (esporas, semillas).
El equilibrio e intercambio de sales entre el interior y el exterior del organismo se denomi-
na ósmosis. Todos los organismos presentan unas concentraciones interiores, y varían en gene-
ral de las del exterior y aunque la diferencia sean las sales, lo que realmente entra o sale es agua
y las adaptaciones van encaminadas a mantener el equilibrio interno.
Los organismos que forman la biocenosis, establecen relaciones alimentarias entre ellos y
así se producen las interacciones de los niveles tróficos. Estas relaciones alimentarias son el
fundamento del flujo lineal de la energía que parte del Sol y es captada por los productores
mediante la fotosíntesis. De ellos pasa a los consumidores primarios, a los consumidores
secundarios, y así sucesivamente hasta los descomponedores, en una serie de escalones que
constituyen una cadena trófica. De forma paralela al flujo de energía y en el mismo sentido se
produce el flujo de materia. Es muy frecuente que se entremezclen las cadenas formando redes
tróficas.
Del total de energía del Sol, sólo el 0,1% llega hasta los seres vivos y sólo una pequeña parte
es retenida como biomasa y podrá ser utilizada por los niveles siguientes.
Los ecosistemas se representan mediante pirámides ecológicas, pueden representar el flujo
de energía, pirámide de números y pirámides de biomasa.
En los ecosistemas, la materia ha de reciclarse, se denominan ciclos biogeoquímicos. Los
más importantes son los del carbono, nitrógeno, fósforo y azufre.
Un ecosistema tiene la capacidad de autorregularse y mantenerse en equilibrio dinámico, a
lo largo del tiempo, y cuanto mayor sea su complejidad, mayor será su capacidad de autorregu-
lación. No obstante se producen cambios graduales denominados sucesión ecológica.
En un biotopo nuevo, los primeros colonizadores son los pioneros y la comunidad en equi-
librio con el ambiente comunidad clímax.
La biosfera, es el gran ecosistema constituido por todos los seres vivos y una franja de la
propia Tierra que se extiende por la superficie terrestre, la capa más baja de la atmósfera y las
aguas de mares y océanos. La biosfera se suele dividir en dos grandes ecosistemas: acuático
(marino y de agua dulce) y terrestre. Éste a su vez, se divide en otros más pequeños, llamados
biomas.

21
-Pág.169-
C I E N C I A S D E L A N AT U R A L E Z A

Desde la zona ecuatorial hasta los polos a nivel terrestre podemos distinguir los siguientes
ecosistemas: pluviselva, las praderas (sabana y estepa), el desierto, el bosque mediterráneo, el
bosque caducifolio, la taiga, la tundra, la antártida, el ártico.
A nivel marino, la mayor parte de los organismos viven cerca de la costa, sobre la platafor-
ma continental, hasta donde penetra la luz. Viven sobre el fondo (bentónicos), viven libres por
el agua (pelágicos). Éstos a su vez pueden ser nadadores activos (como los peces) y constituyen
el necton; o bien carecer de órganos nadadores, por lo que flotan a la deriva y forman el planc-
ton.
En España, los ecosistemas existentes dependen del tipo de suelo (arcilloso, calcáreo o silí-
ceo), el relieve (las especies que podemos encontrar varían con la altitud) y del clima.
En el mar Mediterráneo, entre los 20 y 60 metros de profundidad se encuentra el ecosiste-
ma coralígeno, que se caracteriza por el predominio de algas calcificadas. A nivel litoral, en la
zona inframareal, se presentan praderas de Poseidonia, que es endémica en esta zona. Es de
señalar el impacto medioambiental producido por el turismo y la industria que vierten residuos
sólidos, aguas residuales sin depurar y productos químicos de la industria y la agricultura.
El humedal es una zona de la superficie terrestre que está temporal o permanentemente inun-
dada, regulada por factores climáticos y en constante interrelación con los seres vivos que la
habitan, siendo de gran fragilidad. Los más importantes son el Coto de Doñana y las Tablas de
Daimiel.
Los ríos presentan condiciones de vida diferentes a lo largo de su recorrido, así, en el tramo
alto, existen algas tapizando el fondo y unos pocos animales se han adaptado a la fuerte corrien-
te. En el tramo medio, se asientan algas y plantas, sobre todo en la orilla; también hay anima-
les que se sitúan a distintos niveles en el curso del río. Es en el tramo bajo donde se desarrollan
un gran número de seres vivos aunque el fondo está poco iluminado con lo que esta zona está
poco habitada.

22
-Pág.170-
U . D . 8 . - C OMPONENTES A B I Ó T I C O S Y B I Ó T I C O S D E L M E D I O N AT U R A L

EJERCICIOS DE AUTOCOMPROBACIÓN

1. La totalidad de los seres vivos que viven en un ecosistema se denomina:

A. Biotopo B. Biocenosis
C. Población D. Biosfera

2. Los animales depredadores pueden ser:

A. Consumidores primarios B. Consumidores secundarios

C. Consumidores cuaternarios D. Detritívoros

3. El lugar donde habita preferentemente cada organismo, es decir, la zona donde se encuentran las
condiciones más favorables para vivir, se denomina:

A. Biotopo B. Hábitat

C. Ecosistema D. Biocenosis

4. En un ecosistema se reciclan:

A. Los nutrientes B. Los bioelementos

C. La energía D. Ninguna de las anteriores

5. Las algas y las plantas son:

A. Productores primarios B. Consumidores primarios

C. Descomponedores D. Todas son ciertas

6. El gran ecosistema constituido por todos los seres vivos y una franja de la propia Tierra se deno-
mina:

A. Biosfera B. Bioma

C. Región bioclimática D. Biosistema

7. El bioma cuya vegetación está formada por musgos, líquenes y una gran variedad de flores, donde
habitan los rebaños de caribús, alces y lobos, se denomina:

A. Taiga B. Tundra

C. Estepa D. Bosque caducifolio

23
-Pág.171-
C I E N C I A S D E L A N AT U R A L E Z A

8. Los organismos marinos que flotan a la deriva porque carecen de órganos nadadores, constituyen:

A. El necton B. El plancton

C. El bentos D. Todas son falsas

9. La Poseidonia, es una planta que está presente en:

A. En el mar Mediterráneo, formando praderas en el fondo

B. En el océano Atlántico, es endémica allí

C. No vive en el agua, sino en las dunas

D. En el mar Mediterráneo, flotando a la deriva

10. La laurisilva, es un bosque formado fundamentalmente por laureles situado en la zona de media
montaña en:

A. Las Islas Baleares B. Las Islas Canarias

C. En Galicia D. En Asturias

RESPUESTAS A LOS EJERCICIOS

11. B 2. B 3. B 4. A 5. A

6. A 7. B 8. B 9. A 10. B

24
-Pág.172-
portada TROPA 19/3/07 19:51 Página 1

FUERZAS ARMADAS
PROFESIONALES
CURSO DE APOYO
A LA PREPARACIÓN
DE LAS PRUEBAS DE ACCESO
A UNA RELACIÓN DE SERVICIOS
DE CARÁCTER PERMANENTE

CIENCIAS NATURALES
3ª parte
Unidades didácticas 9,10 y 11

DIGEREM

MINISTERIO
DE DEFENSA
FUERZAS ARMADAS SUBDIRECCIîN GENERAL
DE TROPA Y MARINERIA
PROFESIONAL
PROFESIONALES
CURSO DE APOYO
A LA PREPARACIÓN
DE LAS PRUEBAS DE ACCESO
A UNA RELACIÓN DE SERVICIOS
DE CARÁCTER PERMANENTE

CIENCIAS NATURALES
3ª parte
Unidades didácticas 9,10 y 11

-Pág.1-
La Ley 8/2006 de Tropa y Marinería, en su artículo 16,1, establece que “la formación
en las Fuerzas Armadas garantizará que los militares profesionales de tropa y
marinería puedan adquirir, actualizar o ampliar sus conocimientos para un mayor
desarrollo personal y profesional”. En cumplimiento de este mandato, el Ministerio
de Defensa edita el presente material didáctico para facilitar a los militares
profesionales de tropa y marinería, alumnos de los cursos de formación
presencial que se imparten a través de la Dirección General de Reclutamiento y
Enseñanza Militar, los apoyos necesarios para preparación de dichos cursos, que
permitirán, siempre que superen las pruebas correspondientes, la obtención de la
titulación de graduado en Educación Secundaria, acreditación para el acceso a
los ciclos formativos de la Formación Profesional de grado medio o de grado
superior, acceso a las Escalas de Suboficiales, Tropa Permanente, Guardia Civil
y Policía Nacional.

CATÁLOGO GENERAL DE PUBLICACIONES


http://www.060.es

Edita:

© Autor y editor
NIPO: 076-10-204-9 NIPO: 076-10-205-4 (edición en línea)
Depósito Legal: M-32363-2009
Diseño y programación: cimapress
Tirada: 1300 ejemplares
Fecha de edición: septiembre, 2010

Prohibida la reproducción total o parcial de esta obra, por cualquier medio sin autorización escrita del editor
CIENCIAS NATURALES
3ª parte

SUMARIO

Unidad didáctica Pág.

LOS CAMBIOS EN EL MEDIO NATURAL, LOS SERES HUMANOS. 5


9.
PRINCIPALES AGENTES DEL CAMBIO

10. LAS FUERZAS Y LOS MOVIMIENTOS 33

11. ELECTRICIDAD Y MAGNETISMO 53

-Pág 3-
U.D. 9.- LOS CAMBIOS EN EL MEDIO N AT U R A L

ÍNDICE
OBJETIVOS . . . . . . . . . . . . . . . . . . . . . . . . . . . . . . . . . . . . . . . . . . . . . . . . . . . . . . . . . . . 2

INTRODUCCIÓN. . . . . . . . . . . . . . . . . . . . . . . . . . . . . . . . . . . . . . . . . . . . . . . . . . . . . . . . 3

MAPA CONCEPTUAL. . . . . . . . . . . . . . . . . . . . . . . . . . . . . . . . . . . . . . . . . . . . . . . . . . . . 4

DESARROLLO DE CONTENIDOS . . . . . . . . . . . . . . . . . . . . . . . . . . . . . . . . . . . . . . . . . 5

1.CAMBIOS NATURALES EN LOS ECOSISTEMAS. . . . . . . . . . . . . . . . . . . . . . . . . 5


1.1.CAMBIOS EN LAS POBLACIONES. . . . . . . . . . . . . . . . . . . . . . . . . . . . . . . . . . . . . . . . . . . 5
1.2. CAMBIOS EN LAS ROCAS DEBIDOS A PROCESOS GEOLÓGICOS EXTERNOS. . . . 6
1.3. LA FORMACIÓN DE ROCAS SEDIMENTARIAS. . . . . . . . . . . . . . . . . . . . . . . . . . . . 6
1.4. LA DINÁMICA INTERNA DE LA TIERRA. . . . . . . . . . . . . . . . . . . . . . . . . . . . . . . . . 7
1.5. ALTERACIONES EN LA DISPOSICIÓN ORIGINAL DE LAS ROCAS. . . . . . . . . . . . 7
1.6. LA CONFIGURACIÓN EN PLACAS DE LA SUPERFICIE TERRESTRE. . . . . . . . . . 7

2. CAMBIOS EN LOS ECOSISTEMAS PRODUCIDOS POR LA ACCIÓN


HUMANA. . . . . . . . . . . . . . . . . . . . . . . . . . . . . . . . . . . . . . . . . . . .. . . . . . . . . . . . . . . . 10
2.1. LAS ACTIVIDADES HUMANAS . . . . . . . . . . . . . . . . . . . . . . . . . . . . . . . . . . . . . . .10
2.2. ACCIONES DE CONSERVACIÓN Y RECUPERACIÓN DEL MEDIO NATURAL . . . .13
2.2.1. TRATAMIENTO DE RESIDUOS . . . . . . . . . . . . . . . . . . . . . . . . . . . . . . . . . . 13
2.2.2. LA CONSERVACIÓN DE LA NATURALEZA . . . . . . . . . . . . . . . . . . . . . . . . 14

3. LA TIERRA, UN PLANETA EN CONTINUO CAMBIO . . . . . . . . . . . . . . . . . . . . 15


3.1. CAMBIOS A LARGO PLAZO . . . . . . . . . . . . . . . . . . . . . . . . . . . . . . . . . . . . . . . . . 15
3.2. LOS FÓSILES COMO INDICADORES . . . . . . . . . . . . . . . . . . . . . . . . . . . . . . . . . . . 17
3.3. ALGUNAS EXPLICACIONES AL PROBLEMA DE LOS CAMBIOS . . . . . . . . . . . . 18
3.3.1. LAS ADAPTACIONES Y EL MEDIO AMBIENTE . . . . . . . . . . . . . . . . . . . . .19
3.4. FIJISMO Y EVOLUCIONISMO. . . . . . . . . . . . . . . . . . . . . . . . . . . . . . . . . . . . . . . . .19
3.5. ALGUNAS RELACIONES ENTRE GENÉTICA Y EVOLUCIÓN . . . . . . . . . . . . . . . .21

RESUMEN . . . . . . . . . . . . . . . . . . . . . . . . . . . . . . . . . . . . . . . . . . . . . . . . . . . . . . . . . . . . . . . . 23

EJERCICIOS DE AUTOCOMPROBACIÓN . . . . . . . . . . . . . . . . . . . . . . . . . . . . . . . . . . .26

RESPUESTAS A LOS EJERCICIOS . . . . . . . . . . . . . . . . . . . . . . . . . . . . . . . . . . . . . . . . . . 28

-Pág.5-
C I E N C I A S D E L A N AT U R A L E Z A

O B J E T I VO S
Al finalizar el estudio de esta Unidad Didáctica, el alumno será capaz de:

• Describir el continuo cambio que sufre todo lo que nos rodea en nuestro
planeta.

• Analizar y comprender las distintas teorías que intentan explicar los procesos
evolutivos.

• Expresar correctamente la necesidad de la defensa del medio natural como


patrimonio necesario para preservar nuestro mundo.

-Pág.6-
U.D. 9.- LOS CAMBIOS EN EL MEDIO N AT U R A L

INTRODUCCIÓN
E l interés que despiertan los dinosaurios o el dicho de que el hombre procede del
mono son cuestiones que rápidamente captan nuestra atención y es que a todos nos
llama la atención la historia del pasado, especialmente si se rodea de un halo de miste-
rio o de una incógnita científica que cada día se desvela un poco más.

-Pág.7-
C I E N C I A S D E L A N AT U R A L E Z A

M A PA C O N C E P T UA L
LOS CAMBIOS
EN EL MEDIO NATURAL

ECOSISTEMAS FÓSILES TIERRA

POBLACIONES ROCAS INTERIOR


DE LA TIERRA

PROCESOS
GEOLÓGICOS
EVOLUCIÓN CORRIENTES
EXTERNOS
DE CONVECCIÓN

EROSIÓN,
TRANSPORTE Y
TEORÍAS SEDIMENTACIÓN TECTÓNICA
DE PLACAS

ROCAS
SEMIMENTARIAS
FIJISTAS EVOLUCIONISTAS

CICLO
DE LAS ROCAS

LAMARCK

DARWIN

MENDEL

-Pág.8-
U.D. 9.- LOS CAMBIOS EN EL MEDIO N AT U R A L

1. CAMBIOS NATURALES EN LOS ECOSISTEMAS


El nivel de mayor complejidad de la materia viva se encuentra en los ecosistemas, donde los
seres vivos, agrupados en poblaciones y comunidades, llevan a cabo el mejor aprovechamiento
de los recursos disponibles.
El mantenimiento de la estructura y de la organización del ecosistema sólo es posible gra-
cias al aporte constante de energía que es captada por los productores y que es necesaria para
conservar el orden interno. Hay que tener en cuenta que, desde el punto de vista termodinámi-
co, todo sistema tiende al desorden y sólo un aporte energético constante desde el exterior puede
retardar esa tendencia.

1.1. CAMBIOS EN LAS POBLACIONES


El número de individuos que puede tener una población cualquiera de un ecosistema estará
regulado por una serie de factores, como los físico-químicos, la disponibilidad de espacio, de
alimento, o la presencia de especies depredadoras y competidoras. Si no hubiera factores que
limitaran el crecimiento, la población aumentaría indefinidamente.
En los ecosistemas, las poblaciones sufren lo que se denomina fluctuaciones cíclicas. Cuan-
do aumenta el número de liebres, es seguido de un aumento del número de linces, al disponer
estos de una mayor cantidad de presas; sin embargo, al aumentar el número de linces se produ-
ce a la larga un descenso de la población de liebres, puesto que el número de liebres cazadas por
los linces es mayor disminuyendo las posibilidades de alimentación. De esta manera las dos
poblaciones se autorregulan.
En cualquier caso, un ecosistema determinado, como puede ser una pradera o un estanque,
no permanece siempre igual, sino que nuevas poblaciones pueden llegar a ella y sustituir a las
existentes. La secuencia de cambios graduales experimentados por un ecosistema a lo largo del
tiempo recibe el nombre de sucesión ecológica.
Los primeros seres vivos que se instalan en un nuevo biotopo se denominan especies pio-
neras. Conforme se modifica el biotopo, permite que se instalen nuevas especies que sustitu-
yen a las anteriores en lo que se considera una sucesión primaria. Una sucesión secundaria,
es la que se produce tras la desaparición de una comunidad establecida anteriormente, como
sucede tras un incendio o tras el abandono de un campo de cultivo.
A medida que la sucesión avanza, el número de especies se va haciendo mayor, la comple-
jidad de la biocenosis aumenta y las condiciones del biotopo se van modificando, hasta que se
llega a una comunidad que está en equilibrio con las condiciones ambientales, es la comunidad
clímax o comunidad climácica. La tala masiva y los incendios son las causas más importantes
de la desaparición de muchas de las comunidades clímax en todo el mundo.

5
-Pág.9-
C I E N C I A S D E L A N AT U R A L E Z A

1.2. CAMBIOS EN LAS ROCAS DEBIDOS A PROCESOS


GEOLÓGICOS EXTERNOS
Las rocas al entrar en contacto con la atmósfera sufren una destrucción, llamada meteoriza-
ción. Puede estar producida por procesos físicos (calor y frío) y en otros casos puede variar la
composición por reacciones químicas producidas entre la roca y los componentes de la atmósfe-
ra (oxígeno, agua y dióxido de carbono). La meteorización actúa lenta, pero implacablemente
sobre toda la superficie terrestre y prepara el camino para la erosión. El tipo de clima y el tipo de
roca y su estructura van a determinar los procesos de meteorización que se vayan a producir.
La meteorización física se produce por la variación de temperatura y la congelación del agua en
las grietas, lo que produce cambios en la estructura de la roca, disgregándola, pero sin modificar su
composición química. Tiene lugar en climas cálidos, especialmente desérticos y en las montañas.
La meteorización química se produce sobre todo en climas húmedos y ecuatoriales. Las
rocas calizas al ser atacadas por el agua de lluvia, cargada de CO2, disuelve la roca y produce
el modelado kárstico.
La erosión, el transporte, la sedimentación y la formación de rocas sedimentarias constitu-
yen los procesos geológicos externos responsables del modelado del relieve. Los agentes geo-
lógicos externos son el agua, el viento y los seres vivos.
Una vez que la atmósfera ha meteorizado las rocas, los agentes geológicos externos desgastan,
rompen y separan fragmentos rocosos del terreno (erosión), que son transportados desde el lugar
donde se han originado a otros puntos de la corteza terrestre. Cuando la energía del agente que
transporta disminuye o cesa, los materiales se acumulan (sedimentación) en las zonas bajas de los
continentes y, sobre todo, de los océanos (cuencas de sedimentación), donde forman las capas de
sedimentos que, después de millones de años, se transformarán en rocas sedimentarias.
La energía que permite la actuación de los agentes geológicos externos procede del Sol y
actúa junto con la fuerza de la gravedad, que hace descender todos los materiales hacia las zonas
bajas de la corteza terrestre. La intensidad de la radiación solar es mínima en los polos y máxi-
ma en el ecuador, esto produce un desequilibrio térmico en el planeta que se compensa median-
te el flujo de energía, en forma de calor desde los lugares cálidos a los fríos. Esta enorme
cantidad de energía pone en movimiento el agua de la Tierra, manteniendo un ciclo constante
llamado ciclo del agua, que es el origen de casi todos los agentes geológicos externos.

1.3. LA FORMACIÓN DE ROCAS SEDIMENTARIAS


Cualquier tipo de roca que exista sobre la superficie terrestre puede verse sometida a proce-
sos de meteorización y erosión. Tras ello se originan fragmentos de diversos tamaños que son
transportados por el agua, el hielo y el viento.
Del mismo modo, cuando el agua y el viento pierden fuerza, los materiales que transportan
se depositan y originan los sedimentos. Cada una de las capas en que se depositan los sedi-
mentos constituye un estrato.

6
-Pág.10-
U.D. 9.- LOS CAMBIOS EN EL MEDIO N AT U R A L

Además de partículas sólidas, el agua transporta otras sustancias en disolución que también
pueden sedimentarse, al igual que pueden formar sedimentos los restos de seres vivos tanto ani-
males como vegetales.
La transformación de sedimentos en rocas sedimentarias se llama diagénesis o litogénesis
y sigue una serie de pasos:
– Compactación: Consiste en la reducción del espacio existente entre los materiales, cau-
sada por el peso de los sedimentos superiores.
– Cementación: Consiste en la unión de los fragmentos de roca gracias a la precipitación
de las sustancias disueltas en el agua que circula entre ellos.
– Alteraciones químicas y mineralógicas. Se producen por reacciones químicas entre las
sustancias disueltas y los fragmentos originales depositados.

1.4. LA DINÁMICA INTERNA DE LA TIERRA


En la Unidad Didáctica 5 hemos estudiado la estructura interna de la Tierra, los modelos
Geoquímico y Dinámico y como los movimientos de convección son la causa fundamental de
los procesos geológicos de la formación de montañas, movimiento de continentes, aparición de
volcanes y terremotos, etc. Y esos movimientos de convección tienen un único origen: el calor
interno de la Tierra.

1.5. ALTERACIONES EN LA DISPOSICIÓN ORIGINAL


DE LAS ROCAS
Este apartado ya ha sido estudiado en la Unidad Didáctica 5. Tanto en lo referente al estu-
dio de pliegues y fallas, como del ciclo de las rocas.

1.6. LA CONFIGURACIÓN EN PLACAS DE LA SUPERFICIE


TERRESTRE
En la superficie de la Tierra no existe ningún lugar que sea inmóvil. La litosfera, formada
por la corteza terrestre y los primeros kilómetros del manto superior, se encuentra fragmentada
en gigantescas piezas rígidas de unos 100 kilómetros de espesor, llamadas placas litosféricas.
Las placas se mueven y empujan entre sí sobre la astenosfera, capa del manto superior fundi-
da, plástica y menos rígida, en la que flotan, manteniendose en equilibrio isostático. Cuando la
erosión elimina materiales de los continentes, la litosfera se eleva para restablecer el equilibrio;
por el contrario, cuando en una zona se acumulan materiales (por ejemplo sedimentos en una
cuenca sedimentaria), la litosfera se hunde parcialmente.

7
-Pág.11-
C I E N C I A S D E L A N AT U R A L E Z A

La tectónica de placas estudia cómo y por qué se mueven las placas litosféricas. También
explica en conjunto todos los fenómenos dinámicos que se producen en la Tierra (volcanes,
terremotos, formación de montañas, etc.).
Aunque todas las placas litosféricas encajan entre sí como las piezas de un rompecabezas,
sus tamaños son muy diferentes. Algunas como la Euroasiática, son muy grandes y abarcan casi
por completo océanos y continentes. Otras, como la del Caribe, son verdaderamente pequeñas.
La mayoría de las placas son en parte oceánicas y en parte continentales.
Las placas litosféricas pueden en algunas ocasiones acercarse a sus vecinas, en otras alejar-
se o, simplemente, acompañarlas en un incansable movimiento. En su incesante actividad pro-
ducen tres tipos de bordes: bordes constructivos o dorsales oceánicas, donde se produce
nueva litosfera; bordes destructivos o zonas de subducción, donde se destruyen las placas y
bordes pasivos, situados en las fallas de transformación, donde las placas se deslizan lentamen-
te, hasta miles de kilómetros, una junto a otra pero en sentidos opuestos.
Las placas litosféricas no se impulsan por sí mismas. Este movimiento se produce porque el
interior del planeta está caliente. El calor interno de la Tierra crea una lenta y continua circula-
ción de los materiales en el manto, son las corrientes de convección. El magma más caliente y
menos denso asciende por las dorsales y se solidifica dando lugar a la litosfera oceánica. Con-
tinúa por debajo de las placas alejándose de la dorsal y, después de enfriarse y hacerse más
densa, desciende, reintegrándose de nuevo al manto en las zonas de subducción. En su descen-
so el magma arrastra la placa oceánica que se hunde y destruye lentamente.
Como consecuencia del continuo movimiento del manto superior, la litosfera es arrastrada
y obligada a moverse. Los continentes, como grandes y viejos pasajeros sobre las placas, se des-
plazan para colisionar unas veces y separarse otras (deriva continental). Pero no sólo los conti-
nentes se mueven, también cambia el aspecto de los océanos, pues el crecimiento del fondo
oceánico a ambos lados de la dorsal hace que los océanos sean cada vez más grandes (expan-
sión oceánica). En las zonas donde chocan dos placas litosféricas, los sedimentos se pliegan y
fracturan, luego se elevan y surgen grandes cordilleras montañosas (orogénesis). Durante años,
la energía de choque se va acumulando y, de forma inesperada, se libera: se ha producido un
terremoto que hace vibrar la tierra. La corteza agrietada deja escapar violentamente gases y
magma del interior hasta la superficie: así se forma un volcán.
Las cordilleras se sitúan en las zonas de colisión de continentes o sobre las zonas de sub-
ducción en los bordes continentales. Forman parte de las regiones de la Tierra con gran activi-
dad sísmica y se disponen en forma de cinturones o cadenas llamadas cinturones orogénicos.
Cuando dos continentes colisionan entre sí, la energía cinética se transforma en grandes
fuerzas llamadas fuerzas orogénicas. Durante millones de años, estas fuerzas oprimen más y
más a los bordes de las placas, que terminan por plegarse y elevarse. La nueva cordillera mon-
tañosa dibuja una cicatriz que señala el lugar donde las dos placas continentales se han solda-
do entre sí. El Himalaya y todas las grandes cordilleras euroasiáticas se han formado de esta
manera.

8
-Pág.12-
U.D. 9.- LOS CAMBIOS EN EL MEDIO N AT U R A L

Pero en la costa oeste de América las montañas, como los Andes, surgieron en zonas donde
una placa oceánica se hundió bajo una placa continental. En la fosa oceánica, situada en la zona
de subducción, se van acumulando los sedimentos procedentes de la erosión del continente. La
presión debida al hundimiento de la placa es tan fuerte que empuja y comprime las capas de
sedimentos (estratos), que se pliegan y fracturan. Al cabo de millones de años emergen y for-
man una cordillera.

Acumulación de sedimentos Plegamiento y elevación Cordillera


de los estratos
Rocas Erosión Capa sedimentaria
metamórficas

Rocas Transporte Capa


Zona sedimentarias metamórfica
de subducción Corteza
Prisma
continental Sedimentos
de acreción Volcán
Zona
de subduccion
Sedimentos Volcán
Corteza Corteza
Oceanica Oceanica
Manto Litosfera Manto
Superior Oceanica Superior
Magma
Astenosfera Litosfera Astenosfera
continental Magma
Foco Sísmico

Manto superior Plutón granítico

• A medida que se hunde en la astenosfera, la litosfera oceáni- • Al cabo de millones de años, los estratos rocosos plegados
ca empuja y comprime lateralmente los estratos rocosos, de emergen y forman una cordillera u orógeno donde antes exis-
forma que se pliegan lentamente, e incluso se fracturan y se tía una fosa y dan lugar a una zona con fuerte actividad volcá-
acumulan al pie del continente, formando un prisma de acre- nica y sísmica. Debido a su origen marino, las capas más
ción, que es el embrión de las futuras montañas. Se trata de un externas de las montañas suelen estar formadas por rocas
conjunto de pequeñas montañas submarinas que en ocasiones sedimentarias, las zonas intermedias por rocas metamórficas y
emerge, y cuyas cumbres originan un archipiélago, como en el las regiones más profundas por rocas plutónicas.
caso de las Islas Barbados.

Océano China
India

Prisma Cordillera
de acreción del Himalaya

China

India

En el pasado, el fragmento
que hoy es la India colisionó
con el continente Euroasiático.

9
-Pág.13-
C I E N C I A S D E L A N AT U R A L E Z A

CAMBIOS EN LOS ECOSISTEMAS PRODUCIDOS


2. POR LA ACCIÓN HUMANA
Un ecosistema es frágil cuando una pequeña modificación de sus componentes bióticos o
abióticos provoca la pérdida de su estructura y de su función. Si, por el contrario, es capaz de
permanecer estable en las condiciones más variadas, diremos que el ecosistema es robusto.
La especie humana es una especie más que ocupa su lugar dentro de la estructura del eco-
sistema, por tanto, utiliza los recursos que obtiene del entorno y le devuelve los productos de
desecho.
Los recursos naturales son aquellas materias que la especie humana obtiene de su entorno
y que son imprescindibles para su subsistencia y bienestar.
No todos los recursos naturales pertenecen a la misma categoría. Algunos, como la madera,
se pueden regenerar a una velocidad parecida a la del consumo; estos son los recursos renova-
bles. Otros, como el petróleo, sólo pueden regenerarse a un ritmo muy lento en la escala huma-
na, son los recursos no renovables.
A pesar de que la especie humana durante los tiempos prehistóricos explotó el ecosistema
de modo destructivo, cazando, pescando y recolectando especies silvestres, no se rompió el
equilibrio del ecosistema, puesto que el número de personas era pequeño y además no se utili-
zaban más recursos que los necesarios. El ecosistema contaba con capacidad autorreguladora
suficiente para mantener su estructura.
En el período Neolítico, nacen la agricultura y la ganadería, permitiendo al hombre mani-
pular el ecosistema para su provecho, permitiendo que el número de individuos de la especie
humana aumente, desequilibrando el ecosistema. Este incremento no es uniforme en el planeta,
siendo mayor en las zonas subdesarrolladas, del mismo modo que tampoco lo son la extracción
y utilización de los recursos.

2.1. LAS ACTIVIDADES HUMANAS


La explosión demográfica ha traído consigo una sobreexplotación de todos los recursos que
ha tenido importantes efectos sobre el medio ambiente y sobre sus habitantes. Estas acciones,
frecuentemente negativas, son lo que conocemos como impacto ambiental. Si el ritmo de explo-
tación de los recursos no se adecua al ritmo de los ecosistemas para mantener su equilibrio, las
consecuencias pueden ser nefastas.
a) La agricultura. Tradicionalmente el abonado se realizaba con estiércol, permitiéndose
la recuperación de la tierra con el barbecho y la rotación de los cultivos. En la actuali-
dad se realizan monocultivos en grandes extensiones, utilizando maquinaria pesada y
gran cantidad de fertilizantes y plaguicidas, que al utilizarse en exceso contaminan y se
incorporan a las cadenas tróficas.

10
-Pág.14-
U.D. 9.- LOS CAMBIOS EN EL MEDIO N AT U R A L

b) Ganadería. Tradicionalmente el ganado se alimentaba de la vegetación natural, abonaba


los campos y la paja se utilizaba como cama. La ganadería intensiva, aglutina a gran
número de animales en un espacio reducido generando una gran cantidad de estiércol que
resulta contaminante para las aguas y suelos. Se alimentan de piensos que se elaboran de
restos de animales que pueden transmitir enfermedades como es el caso de las vacas
locas, que han contraído esta enfermedad al alimentarse de restos de ovejas enfermas.
c) La pesca. La utilización de artes de pesca poco selectivas, como la pesca de arrastre o
las redes de deriva, conlleva la captura de productos del mar de todo tipo y tamaño, la
mayoría de los cuales son desechados. Todas las capturas se realizan en un pequeño
número de caladeros que están prácticamente agotados.
d) Explotación de los bosques. Los bosques son los ecosistemas terrestres con mayor bio-
masa y producción. Se utilizan con el fin de obtener madera para la construcción, la cale-
facción y la elaboración de pasta de papel. De ellos se obtiene también resinas y plantas
medicinales. La tala de árboles es producida por fines madereros, la preparación de terre-
nos para cultivo y pastoreo y la construcción de vías de comunicación. Por otra parte, la
repoblación con especies alóctonas favorece la producción de incendios.
e) Minería. La extracción de rocas y recursos minerales provoca grandes alteraciones del
medio ambiente. Los residuos minerales se acumulan en grandes escombreras, que ade-
más de tener un alto impacto paisajístico negativo, supone el aporte de contaminantes al
suelo y a las aguas.
f) Actividades cotidianas. Se genera una gran cantidad de basuras, formadas por residuos
orgánicos e inorgánicos, que contaminan el suelo, el aire y el agua. Los detergentes, los
medios de transporte y las calefacciones también producen contaminación del aire y del
agua.
g) Obras públicas. Los asentamientos urbanos e industriales requieren grandes espacios e
importantes construcciones con el consiguiente impacto ambiental sobre los ecosistemas
destruidos y sobre el paisaje. La generación de energía precisa de grandes centrales tér-
micas y nucleares que emiten gases y radiaciones nocivas para el medio y para la salud
humana. Se construyen grandes presas para generar energía hidroeléctrica y la construc-
ción de enormes vías de comunicación para personas y mercancías son la causa de atro-
pello de gran cantidad de animales.
h) Industria. Existen muchos tipos de industrias que vierten al medio toda clase de sus-
tancias contaminantes como resultado de los residuos propios de sus actividades.
Todas estas actividades generan contaminación atmosférica, del agua y del suelo.
Se entiende por contaminación atmosférica la presencia en la atmósfera de niveles de energía
anómalos, sustancias extrañas o variaciones importantes en las concentraciones de las existentes,
que tienen efectos perjudiciales o causan molestias a las personas, a los animales y a las plantas.

11
-Pág.15-
C I E N C I A S D E L A N AT U R A L E Z A

Los contaminantes pueden clasificarse en:


– Primarios: Son los emitidos directamente a la atmósfera. Entre ellos están las partículas
de polvo, los compuestos del azufre (SO2, SO3, H2S), los óxidos de nitrógeno (NO, NO2
y N2O3), los óxidos de carbono (CO2, CO), los metales pesados (plomo, aluminio, mer-
curio y cadmio), los compuestos halogenados (clorofluorocarbonados o ClFC), etc.
– Secundarios. Son los que proceden de la reacción química de los primarios. Entre ellos
están los ácidos de azufre y de nitrógeno.
Los problemas medioambientales pueden ser atmosféricos, del agua y del suelo.
Entre los problemas medioambientales atmosféricos tenemos la lluvia ácida, el agujero de
la capa de ozono, el efecto invernadero.
La lluvia ácida se produce por las enormes emisiones a la atmósfera de óxidos de azufre y
nitrógeno procedentes de la industria. Estos ácidos se combinan con el agua y formando parte
de las nubes, pueden recorrer grandes distancias precipitando posteriormente en forma de llu-
via, nieve o niebla ácida. En el suelo, hacen que se disuelvan los metales pesados y que dismi-
nuya la concentración de nutrientes con graves consecuencias para los vegetales y los
microorganismos, dañan las hojas de los árboles. El nivel de acidez de los lagos aumenta, lo que
produce la desaparición paulatina de su fauna.
La capa de ozono, es una región de la atmósfera que se localiza en la estratosfera y actúa
como filtro que absorbe parte de las radiaciones ultravioleta procedentes del Sol. Algunas sus-
tancias como los ClFC y los gases producidos por la combustión de los carburantes de los avio-
nes, reaccionan con el ozono, capturan los átomos de oxígeno e impiden así la regeneración del
ozono. Los ClFC se utilizan como refrigerantes para cámaras frigoríficas, en los aerosoles y
para fabricar espumas plásticas. En ambos polos terrestres existen unas zonas en las cuales la
concentración de ozono es muy baja, constituyendo un agujero, el agujero de la capa de ozono.
Algunos gases atmosféricos, como el CO2 y el metano, por su elevada densidad, tienden a
acumularse en la parte baja de la atmósfera y hacen de pantalla para la radiación infrarroja del
Sol. Esto es lo que hace que la temperatura en la superficie de la Tierra sea la óptima para la
vida. En los últimos años la emisión de gases (CO2 procedente de la combustión de carbón,
petróleo y gas natural; el metano procedente de la ganadería intensiva, los ClFC, etc.), contri-
buyen a aumentar el efecto invernadero provocando el calentamiento global del planeta, contri-
buyendo de esta forma al cambio climático.
Todos estos problemas hicieron que en 1997, en la conferencia de Kioto se acordara la dis-
minución de emisiones de CO2 y otros gases para controlar el cambio climático.
El agua puede estar contaminada por contaminantes físicos (incremento de temperatura y
partículas radiactivas), químicos (cloruros, fosfatos, compuestos nitrogenados, metales, pestici-
das, materia orgánica, detergentes, grasas y petróleo) y biológicos (microorganismos). Los pro-
blemas que causan son la eutrofización de las aguas (por el incremento de sustancias para el
crecimiento de flora) y las mareas negras entre otros. Estos problemas pueden afectar a las
aguas superficiales y subterráneas.

12
-Pág.16-
U.D. 9.- LOS CAMBIOS EN EL MEDIO N AT U R A L

La degradación del suelo consiste en la pérdida de sus propiedades, y, por tanto, de su ferti-
lidad, y/o en la pérdida de alguno de sus horizontes. La actividad humana contribuye también
al deterioro de los suelos a través de distintos mecanismos: erosión (roturación de nuevas tie-
rras de labor con destrucción de terrenos arbolados o el uso de pendientes pronunciadas), la con-
taminación (por el uso de fertilizantes, pesticidas, la ganadería intensiva que genera abonos y el
uso de hormonas y antibióticos; y la actividad industrial y minera), la salinización y encharca-
miento (por la ubicación de regadíos cerca del mar o en suelos poco permeables, así como el
aumento y acumulación de sales) y la desertización (producida por la erosión, la degradación
de la cubierta vegetal, la salinización y la contaminación).

2.2. ACCIONES DE CONSERVACIÓN Y RECUPERACIÓN


DEL MEDIO NATURAL
Es incuestionable que la bioesfera tiene unas posibilidades limitadas tanto para generar
recursos como para absorber los residuos producidos por la actividad humana. Por tanto, es
imprescindible compatibilizar el crecimiento económico con el respeto al equilibrio medioam-
biental. Este planteamiento es lo que se conoce como desarrollo sostenible.
En la actualidad, ante la construcción o la acometida de los proyectos, se ha de realizar la
evaluación de impacto ambiental (EIA) que trata de medir los efectos que tienen sobre los
ecosistemas, la utilización de los recursos, la transformación del medio físico y las sustancias
emitidas a él. De este modo, se puede comparar el coste ambiental de un proyecto con sus bene-
ficios en términos de calidad de vida para tomar la decisión más adecuada.
Otra cuestión se plantea en torno a las técnicas agrícolas, apostando cada vez con más fir-
meza por la agricultura ecológica que se basa en tres principios: evitar la utilización de pro-
ductos químicos, respeto a la estructura del suelo y reducir el consumo de energía. Es necesario
coordinar los mayores avances científicos con las técnicas tradicionales.
Respecto a las fuentes energéticas, se plantean fuentes de energía alternativas a las tradi-
cionales, que en la actualidad suponen un 17% del total de la producción en nuestro país.
Entre las fuentes de energía alternativa se encuentran la energía hidráulica (de los saltos de
agua), la energía de la biomasa (por transformación de los residuos orgánicos en bioalcoholes y
bioaceites), energía solar (paneles solares), energía maremotriz (aprovechando el movimiento de las
mareas), energía geotérmica (producida por el interior de la Tierra) y energía eólica (del viento).

2.2.1. TRATAMIENTO DE RESIDUOS


Los productos de desecho que se generan como resultado de las distintas actividades huma-
nas se pueden presentar en estado sólido, líquido o gaseoso y pueden tener distintos grados de
toxicidad. Por esta razón, es preciso que cada uno de ellos reciba el tratamiento más adecuado
a sus características.

13
-Pág.17-
C I E N C I A S D E L A N AT U R A L E Z A

Las basuras domésticas se almacenan en vertederos controlados, alejados de los núcleos de


población, cuyos suelos están constituidos por rocas impermeables, como las arcillas. Las basu-
ras se compactan y tapan; generan metano que se evacúa por tuberías y puede utilizarse como
combustible.
Algunos residuos industriales pueden contener productos tóxicos por lo que deben ser depo-
sitados en vertederos especiales y tratarlos para reducir su grado de toxicidad.
Otra forma de tratamiento consiste en la incineración de las basuras en plantas adecuadas.
La energía generada por estas plantas se puede utilizar para producir energía eléctrica. No obs-
tante, se deben colocar filtros para evitar la emisión de cenizas y humos.
Otra forma de evitar la acumulación de basuras es el reciclaje, que consiste en rescatar cosas
que aún puedan utilizarse. Para ello, hay que clasificar los componentes mediante plantas espe-
cializadas o la recogida selectiva.
Respecto a las aguas, se deben depurar las aguas residuales para poderlas devolver al medio
natural. Entre estas técnicas están las blandas o naturales (lagunaje, mediante balsas artificia-
les donde la materia orgánica sufre fermentación por las bacterias que contiene y el filtro verde
que consiste en depositar aguas residuales de núcleos urbanos pequeños sobre un terreno cubier-
to por una plantación forestal) y las técnicas duras o tecnológicas entre las que está la depura-
ción convencional.
En la depuración convencional las aguas se someten inicialmente a un tratamiento previo para
eliminar los cuerpos de gran tamaño, consta de tres etapas, desbaste (mediante sistemas de rejas
que retienen los cuerpos grandes), desarenado y desengrasado. Posteriormente se aplica un trata-
miento primario de decantación que separa por gravedad las partículas suspendidas. Después se
aplica un tratamiento secundario para eliminar la materia orgánica biodegradable (con microorga-
nismos). El tratamiento terciario consiste en añadir al agua sustancias que eliminen las sustancias
y los microorganismos patógenos que aún queden y, finalmente, sale el agua depurada. Todos los
fangos que quedan se someten a procesos de espesamiento, digestión con bacterias, secado y eli-
minación; se produce gas metano que se debe eliminar y que se puede utilizar como combustible.

2.2.2. LA CONSERVACIÓN DE LA NATURALEZA


La preocupación social que ha provocado el deterioro del entorno como consecuencia de las
prácticas socioeconómicas, ha obligado a las instituciones nacionales e internacionales a elaborar
leyes que regulen la gestión de los recursos naturales y protejan el medio ambiente. Desde esta
perspectiva se han creado en nuestro país diferentes figuras de protección de espacios naturales:
– Parques Naturales. Son áreas naturales poco modificadas por la acción del hombre que
poseen un gran valor ecológico, estético, educativo o científico. Dentro de estos están los
Parques Nacionales que son las figuras de máxima protección, consideradas de gran
interés por contener algunos de los ecosistemas españoles más representativos.

14
-Pág.18-
U.D. 9.- LOS CAMBIOS EN EL MEDIO N AT U R A L

– Reservas Naturales y Parques Regionales. Se crean con la finalidad de proteger eco-


sistemas, comunidades o elementos biológicos que, por su fragilidad, importancia o sin-
gularidad merecen una atención especial.
– Otras figuras de protección, como Parajes Naturales, Refugios y Reservas Nacionales
de caza, Paisajes Protegidos, Reservas Integrales, etc., son espacios o elementos natu-
rales protegidos por su belleza, singularidad o por poseer valores científicos, culturales o
paisajísticos.
En 1992 se constituyó la Red Española de Reservas de la Biosfera. Estas áreas deben con-
servar la biodiversidad, promover actividades de investigación y formación en educación
ambiental y establecer estrategias para el desarrollo de las poblaciones locales. En estas reser-
vas se pueden encontrar tres zonas:
– Zona del núcleo. Es una zona estrictamente protegida que contiene muestras de ecosiste-
mas poco perturbados.
– Zona de amortiguación. Se encuentra en torno a la zona del núcleo y en ella sólo se per-
miten actividades que no alteren el equilibrio del núcleo.
– Zona de transición. Rodea a la zona de amortiguación. En ella se pueden llevar a cabo
otras actividades socioeconómicas que sean compatibles con la conservación de la
reserva.

3. LA TIERRA, UN PLANETA EN CONTINUO CAMBIO


Los casi 50 millones de especies diferentes que pueblan actualmente la Tierra son la punta
del iceberg de todas las especies que han existido desde que surgió la vida y sólo suponen el
0,1% del total de especies; el 99,9% son especies fósiles, evidencias de la vida pasada, que se
han ido extinguiendo a lo largo del tiempo.

3.1. CAMBIOS A LARGO PLAZO


El tiempo transcurrido desde que se formó la Tierra, hace unos 4600 millones de años, hasta
la edad actual se divide en cuatro grandes subunidades llamadas eones: Hádico, Arcaico, Pro-
terozoico y Fanerozoico. Los tres primeros eones se suelen agrupar también con el nombre de
Precámbrico;el eón Fanerozoico es el que presenta la mayor riqueza y diversidad de fósiles,
por lo que se subdivide en tres etapas (Paleozoico, Mesozoico y Cenozoico), cada uno de ellos
divididos a su vez en varios períodos.

15
-Pág.19-
C I E N C I A S D E L A N AT U R A L E Z A

Dentro del Precámbrico, el eón Hádico, es el más desconocido y en él tuvo lugar la forma-
ción de los primeros núcleos continentales y de los primeros océanos, en los cuales transcurrió
la evolución química abiótica precursora de las primeras formas de vida.
Hace 4000 millones de años comenzó el eón Arcaico y la aparición de la vida no se hizo espe-
rar, surgieron los primitivos procariotas representantes del reino moneras: primero las bacterias
anaerobias, más tarde las cianobacterias fotosintéticas y posteriormente las bacterias aerobias. La
larga vida solitaria de las bacterias duró 2000 millones de años, hasta que, a mediados del eón Pro-
terozoico, hicieron su aparición, tras un proceso de evolución simbiótica, las primeras células
eucarióticas, representantes del reino protista (algas y protozoos). La actividad de los organismos
fotosintéticos (cianobacterias y algas) fue la causa del aumento gradual del oxígeno atmosférico.
A finales del Proterozoico y coincidiendo con las elevadas concentraciones de oxígeno
atmosférico, hacen su aparición los primitivos animales, miembros del reino metazoos, que son
invertebrados de cuerpo blando. Las glaciaciones y el cambio climático del final del Precám-
brico provocaron la extinción de la mayor parte de ellos.
El Paleozoico comienza con el período Cámbrico, en el que tuvo lugar la llamada explo-
sión cámbrica, el Big Bang de la vida animal, pues en tan sólo unos pocos millones de años, la
fauna animal sobreviviente del Precámbrico se diversificó súbitamente en los océanos cám-
bricos haciendo su aparición todos los phyla del reino animal excepto los briozoos (esponjas,
cnidarios, anélidos, moluscos, artrópodos, equinodermos y cordados.).
Los mares y océanos del período Ordovícico estuvieron dominados por una gran variedad
de animales invertebrados, además hicieron su aparición los primeros vertebrados (los peces sin
mandíbula) y los primeros hongos.
Durante el Silúrico continuó el desarrollo de los invertebrados, especialmente de los pobla-
dores de los arrecifes y aparecieron dos innovaciones revolucionarias: la boca dentada con man-
díbulas y las aletas natatorias de los peces. Comenzó la conquista del medio terrestre, primero
por las plantas y, más tarde, por los invertebrados.
El Devónico, es un período de gran diversificación de los peces: cartilaginosos, óseos y con
aletas lobuladas. A partir de estos últimos se desarrollaron los peces pulmonados y los prime-
ros anfibios, los cuales comenzaron también la conquista del medio terrestre. Hicieron su apa-
rición los primeros insectos y comenzó el desarrollo de las plantas vasculares (helechos y
equisetos).
El clima cálido y húmedo del Carbonífero generó extensas marismas en las que prolifera-
ron los bosques de helechos, equisetos y gimnospermas primitivas, al mismo tiempo que lo
hacían los anfibios y una gran variedad de invertebrados, especialmente los insectos. Hicieron
su aparición los reptiles que manifestaron otra gran innovación adaptativa para vivir en el medio
terrestre, el huevo amniota (en cuyo interior se desarrollaba el embrión protegido por el líquido
amniótico).
Durante el Pérmico, todos los continentes se encontraban unidos formando el supercontinente
Pangea y ocurrieron drásticos cambios climáticos que provocaron la mayor extinción en masa que

16
-Pág.20-
U.D. 9.- LOS CAMBIOS EN EL MEDIO N AT U R A L

se ha conocido en la historia de la vida: desaparecieron casi el 95% de las formas vivas, hecho que
marcó el fin del Paleozoico. Desaparecieron gran parte de los invertebrados acuáticos y casi todos
los anfibios; los reptiles supervivientes se diversificaron y surgieron nuevos grupos de insectos
gigantes. Desaparecieron los bosques primitivos y surgieron nuevos helechos y gimnospermas.
El Mesozoico comprende los períodos Triásico, Jurásico y Cretácico, caracterizados por un
clima benigno que favoreció la diversificación de los grandes reptiles, entre los que destacan los
dinosaurios y otros grandes saurios que ocupan todos los habitats. Se diversificaron los peces óseos
y cartilaginosos, e hicieron su aparición los primeros mamíferos y aves. Entre los invertebrados des-
tacan los moluscos ammonites y los belemnites. Se desarrollaron las gimnospermas y los helechos
con semilla (angiospermas). El período Cretácico acaba con un nuevo período de extinción en masa
que señala el fin del Mesozoico: no sobrevivió ningún animal terrestre de más de 25 Kg de peso.
El Cenozoico, comprende los períodos Terciario y Cuaternario que, a su vez, se dividen
en distintas épocas. La desaparición súbita de los dinosaurios dejó libres los numerosos hábitats
que éstos ocupaban, lo cual favoreció la extraordinaria y rápida diversificación (radiación adap-
tativa) de las aves y de los pequeños mamíferos (del tamaño de una rata) que sobrevivieron a la
extinción del Cretácico.
Durante el Terciario, se diversificaron las plantas con flor (angiospermas) que desarrolla-
ron distintas adaptaciones encaminadas a sobrevivir en ambientes diferentes. Tuvo lugar un pro-
ceso de coevolución entre las plantas y los aparatos bucales de los insectos encargados de su
polinización. El cambio climático provocado por la orogenia Alpina produjo el predominio de
las praderas y el retroceso de los bosques, lo cual favoreció la aparición de las grandes aves no
voladoras y de los grandes mamíferos herbívoros que pacían en las extensas planicies junto con
sus depredadores naturales. Otros mamíferos se adaptaron a vivir en hábitats diferentes: aéreo
(murciélagos), acuático (cetáceos), arbóreo (simios), terrestre (grandes herbívoros y sus depre-
dadores), subterráneo (topos), etc. En este período hicieron su aparición los primeros homíni-
dos y desaparecieron los nummulites (protozoos con esqueleto calcáreo).
Desde finales del Terciario y durante el Cuaternario se sucedieron varias glaciaciones
separadas por períodos interglaciares. Todos los reinos, moneras, protistas, fungi, metafitas y
metazoos completaron su diversificación hasta formar las especies que viven en la actualidad.
El retroceso de los bosques obligó a que un grupo de primates tuviera que adaptarse a vivir en
las sabanas africanas, y a partir de ellos se inició una nueva andadura evolutiva que condujo a
la aparición de los humanos modernos Homo sapiens.

3.2. LOS FÓSILES COMO INDICADORES


La Paleontología es la ciencia que estudia los fósiles. Gracias a ella podemos conocer la evo-
lución de los seres vivos y las especies que se han extinguido y reconstruir la historia geológi-
ca de la Tierra. Los fósiles nos sirven para calcular la edad de los estratos en los que se
encuentran, los de los estratos superiores son más modernos que los de los estratos inferiores

17
-Pág.21-
C I E N C I A S D E L A N AT U R A L E Z A

(Principio de la sucesión faunística). A los fósiles que se utilizan como relojes geológicos se les
denomina guía y se caracterizan por haber pertenecido a poblaciones muy numerosas (facilidad
de fosilización), haber ocupado amplias regiones y haber vivido durante un período de tiempo
corto (evolucionaron rápidamente).

Columna estratigráfica general de la Tierra a lo largo


de toda su historia. Se obtiene al ordenar las rocas, de más
antiguas a más modernas, que contienen los fósiles guía.

3.3. ALGUNAS EXPLICACIONES AL PROBLEMA DE LOS CAMBIOS


Todos los seres vivos estamos emparentados porque procedemos de las células ancestrales
surgidas a partir de las incesantes interacciones que tuvieron lugar entre las moléculas acumu-
ladas en los océanos primitivos. Por este motivo, presentamos una unidad de composición, pues
estamos constituidos por los mismos tipos de principios inmediatos, organizados para formar
células, que son las unidades de estructura y función que caracterizan a todos los seres vivos,
tanto a los organismos unicelulares como a los pluricelulares.
La unidad básica de clasificación es la especie, que se define como el grupo de individuos
con caracteres comunes que pueden cruzarse entre sí y tener descendencia fértil; los individuos
que viven en un área determinada constituyen una población.

18
-Pág.22-
U.D. 9.- LOS CAMBIOS EN EL MEDIO N AT U R A L

Cuando se cruzan dos individuos de especies distintas que han acumulado tal número de
diferencias génicas, ya no pueden cruzarse, y si lo hacen, la descendencia es estéril. Un ejem-
plo es la mula que es el cruce entre burro y yegua.
A medida que los individuos poseen menos caracteres comunes se les agrupa en unidades de
clasificación cada vez más amplias. Las especies se agrupan en géneros, los géneros en familias,
las familias en órdenes, éstos en clases, éstas en phylum (divisiones para los botánicos) y, por últi-
mo, éstos en reinos. Los cinco reinos existentes ya han sido estudiados en Unidades anteriores.
Linneo (1707-1778) estableció para nombrar las especies el sistema binomial, llamado así
porque a cada organismo conocido se le asignan dos nombres en latín. El primero indica el
género al que pertenece el organismo y el segundo la especie. Por ejemplo, los humanos perte-
necemos a la especie Homo sapiens.

3.3.1. LAS ADAPTACIONES Y EL MEDIO AMBIENTE


Si miramos a nuestro alrededor observamos que los seres vivos exhiben una extraordinaria
diversidad de caracteres anatómicos, como los tipos de extremidades, de mandíbula, etc; fisio-
lógicos, como la respiración por branquias, pulmones, tráqueas, etc; metabólicos como la foto-
síntesis o la respiración celular.
Hay que tener en cuenta que la diversidad está relacionada con el medio ambiente o entor-
no en el que viven los organismos, siempre cambiante y a veces hostil, que está constituido
por el conjunto de factores bióticos (los demás seres vivos) y abióticos (clima, suelo, lumi-
nosidad, etc.).
La interacción entre los seres vivos y su entorno es constante y recíproca. Esto provoca que
los organismos desarrollen gradualmente adaptaciones mediante las cuales resuelven cada vez
mejor los problemas de reproducción y supervivencia que el medio les plantea.

3.4. FIJISMO Y EVOLUCIONISMO


La evolución es la responsable de la exuberante diversidad de la vida, tanto de los organis-
mos actuales, como de los que se encuentran fosilizados y, tal vez –como sugieren los físicos–
no sea más que la consecuencia de los procesos autoorganizados que ocurren en los sistemas
químicos vivos. Sin embargo, las ideas evolucionistas no fueron aceptadas fácilmente por la
comunidad científica hasta tiempos recientes.
El botánico y naturalista francés Jean Baptiste de Monet, caballero de Lamarck (1744-
1829), fue el primer hombre de ciencia que formuló una hipótesis científica para explicar la evo-
lución de los seres vivos. Su teoría se denomina transformismo, pues parte del supuesto de que,
igual que los fósiles más modernos proceden de las sucesivas transformaciones de los más pri-
mitivos, también las especies evolucionan al transformarse gradualmente unas en otras. Esta
hipótesis se basa en dos principios:

19
-Pág.23-
C I E N C I A S D E L A N AT U R A L E Z A

– La función crea el órgano. Como consecuencia de los continuos cambios ambientales,


los organismos se adaptan a las nuevas condiciones mediante la adquisición de determi-
nados hábitos que provocan la transformación de algunas partes de sus cuerpos: se des-
arrollan los órganos que más se usan para llevar a cabo determinadas funciones y se
atrofian los que no se utilizan.
– Los caracteres adquiridos se heredan. El progresivo desarrollo de unos órganos y la
atrofia de otros a lo largo de las sucesivas generaciones es la causa de que surjan nuevas
especies mediante las transformaciones graduales de otras especies preexistentes.
El error del Lamarckismo fue considerar que los descendientes heredan los caracteres
adquiridos por sus progenitores a lo largo de sus vidas (hay que recordar que sólo son hereda-
bles aquellos caracteres surgidos por mutaciones y recombinaciones que afectan a los genes
localizados en los gametos).
Las ideas evolucionistas de Lamarck fueron rebatidas por sus detractores y durante el siglo
XVIII la teoría imperante fue el fijismo, que defendía la invariabilidad de las especies, basán-
dose en los postulados creacionistas. Hubo que esperar hasta el siglo XIX para que se hicieran
públicos los nuevos estudios de los naturalistas ingleses Darwin y Wallace. Ambos viajaron por
los trópicos y estudiaron gran cantidad de fósiles y una enorme variedad de plantas y animales.
En 1859, Darwin publicó su obra El origen de las especies por medio de la selección natu-
ral, en la que presentó un amplio conjunto de pruebas muy bien documentadas que demostra-
ban la evolución de los seres vivos, a la vez que formulaba la teoría de la selección natural
para explicar el mecanismo evolutivo. La teoría de la evolución de Darwin-Wallace se funda-
menta en los siguientes principios:
– Competencia. La capacidad reproductora de los individuos de una población es muy
grande y nacen más descendientes de los necesarios para remplazar a los que mueren; sin
embargo, el tamaño de las poblaciones se mantiene relativamente constante en el tiempo.
Esto se debe a que los recursos disponibles para mantener a la población son limitados,
por lo que los individuos deben competir para obtener alimentos y reproducirse.
– Variabilidad de las poblaciones. Existe una gran variabilidad entre los individuos de una
población, que presentan determinados caracteres heredables (distintas habilidades para
obtener alimentos, para defenderse, para soportar cambios ambientales, etc.). Como con-
secuencia de la competencia, solamente unos cuantos individuos lograrán sobrevivir hasta
alcanzar la madurez y poder aparearse.
– Selección natural. La naturaleza selecciona aquellos individuos con ciertos caracteres
que les permitan adaptarse mejor a determinadas condiciones ambientales, pues les dan
ventajas en la lucha por los recursos alimenticios y por el apareamiento, por lo que esta-
rán mejor nutridos, más sanos, alcanzarán la madurez y podrán tener mayor número de
descendientes. Estos a su vez heredarán los caracteres favorables y los que no consigan
adaptarse a las nuevas condiciones de vida se extinguirán.

20
-Pág.24-
U.D. 9.- LOS CAMBIOS EN EL MEDIO N AT U R A L

– Evolución de las especies. Los individuos supervivientes originan la siguiente generación


y así se transmitirán las variaciones ventajosas, de manera que los cambios se irán acumu-
lando gradualmente en las poblaciones y, con el tiempo, darán lugar a nuevas especies.

3.5. ALGUNAS RELACIONES ENTRE GENÉTICA Y EVOLUCIÓN


A pesar de la contribución de Darwin al pensamiento evolucionista, su teoría todavía no
podía dar una respuesta satisfactoria a los interrogantes de cómo se transmiten los caracteres
hereditarios de generación en generación, o cuál es la causa de la variabilidad de las poblacio-
nes sobre la que actúa la selección natural.
La evolución es un fenómeno de poblaciones; no es el individuo el que evoluciona, sino la
población, ya que es aquí donde se presenta la variabilidad genética que permite actuar a la
selección natural.
Hubo que esperar para que se llevara a cabo la síntesis entre la teoría de la selección natu-
ral darwiniana y la genética mendeliana, lo que dio origen a la genética de poblaciones. Así
nació el Neodarwinismo o teoría sintética de la evolución, que se fundamenta en los siguien-
tes principios:
– La población es el conjunto de individuos de la misma especie que habitan en una zona
determinada y, por tanto, la mezcla sexual del ADN sólo ocurre entre los individuos que
la componen. En cada población pueden existir varios alelos para un mismo gen, aunque
solo existan dos alelos (iguales o distintos) en cada individuo. Se define acervo génico
(pool de genes) como la suma de todos los genes –cada uno con sus distintos alelos- pre-
sentes en todos los individuos de la población.
– La variabilidad génica de la población significa que, aunque la mayoría de los genes son
comunes a todos los individuos, cada uno presenta un genotipo o combinación génica,
ligeramente distinta a la de los demás, de manera que algunas mutaciones o formas aléli-
cas del acervo génico solo existen en determinados individuos. La variabilidad génica de
las poblaciones es un proceso aleatorio causado por las mutaciones (cambios en la infor-
mación génica almacenada en el ADN) y por la recombinación génica, que tiene lugar
durante la reproducción sexual, y que posibilita la aparición de nuevos genotipos. Las
mutaciones son la causa primaria de la variabilidad génica y, por tanto, constituyen la
materia prima sobre la que actúa la evolución. Las mutaciones se producen al azar, pero
para que se transmitan a la descendencia deben ser heredables, es decir, deben afectar a
los genes de los gametos.
– La selección natural es siempre obra del medio ambiente que, inestable y cambiante, no
cesa de elegir a los que están mejor adaptados a las características del momento, es decir,
a los individuos que manifiestan una combinación génica beneficiosa, con mayor valor
adaptativo.

21
-Pág.25-
C I E N C I A S D E L A N AT U R A L E Z A

– La evolución es un proceso de cambio progresivo de las frecuencias con que aparecen los
distintos alelos del acervo génico, de manera que será más frecuente encontrar los alelos
correspondientes a las combinaciones génicas favorables.
El resultado de la selección natural y del proceso evolutivo es que cada población estará cada
vez mejor adaptada a su ambiente como consecuencia del aumento de las frecuencias de las com-
binaciones alélicas ventajosas. Cada éxito evolutivo es un acontecimiento casual e imprescindible.
Entre los agentes ambientales encargados de llevar a cabo la selección natural están la com-
petencia con otros miembros de la misma especie; las relaciones que se establecen entre los
depredadores y sus presas y las relaciones de simbiosis.
Cuando ningún individuo de una población logra adaptarse a los cambios ambientales, el
resultado de la selección natural conduce a la extinción de la especie.
Entre los factores que favorecen la extinción están la localización de la especie en un área
pequeña, la especialización extrema, la destrucción del hábitat y los cambios climáticos.
El proceso evolutivo que conduce a la formación de nuevas especies se denomina especia-
ción. Para que esto suceda es preciso que las poblaciones de las que procedan hayan estado
sometidas a un aislamiento reproductivo prolongado, causado entre otros factores por las barre-
ras geográficas que se establecen cuando la deriva continental separa los continentes. Existen
dos modelos que explican el mecanismo evolutivo:
– El modelo neodarwinista, también llamado gradualista que propone que la nueva espe-
cie surge tras la acumulación lenta, continua y gradual de muchos cambios pequeños y
distintos en cada población, hasta que llega un momento en que la divergencia genética
es tan grande que las poblaciones se convierten en especies diferentes cuando ya no pue-
den reproducirse entre sí.
– El modelo del equilibrio puntuado, plantea que la evolución opera a empujones: gran-
des períodos de éxtasis, sin cambios apreciables en las especies, interrumpidos por súbi-
tos episodios de cambios que originan la extinción de unas y la radiación adaptativa de
otras, ya que, a partir de un ancestro, surgen una gran cantidad de especies diferentes
adaptadas a los nuevos hábitats que quedaron vacíos. Estos cambios pueden estar provo-
cados por mutaciones de los genes reguladores que controlan la expresión de otros genes.

22
-Pág.26-
U.D. 9.- LOS CAMBIOS EN EL MEDIO N AT U R A L

RESUMEN
En los ecosistemas, las poblaciones sufren lo que se denomina fluctuaciones cíclicas que
permiten que las poblaciones se autorregulen.
La secuencia de cambios graduales experimentados por un ecosistema a lo largo del tiempo
recibe el nombre de sucesión ecológica. Las sucesiones pueden ser primarias y secundarias.
Cuando se llega a una comunidad que está en equilibrio con las condiciones ambientales, es
la comunidad clímax o comunidad climácica. La tala masiva y los incendios son las causas más
importantes de la desaparición de muchas de las comunidades clímax en todo el mundo.
Las rocas sufren la meteorización, cuando están en contacto con la atmósfera, que puede
estar producida por procesos físicos y por reacciones químicas. El tipo de clima y el tipo de roca
y su estructura van a determinar los procesos de meteorización que se vayan a producir.
La erosión, el transporte, la sedimentación y la formación de rocas sedimentarias constitu-
yen los procesos geológicos externos responsables del modelado del relieve. Los agentes geo-
lógicos externos son el agua, el viento y los seres vivos.
La transformación de sedimentos en rocas sedimentarias se llama diagénesis o litogénesis y
sigue una serie de pasos: compactación, cementación y alteraciones químicas y mineralógicas.
La causa fundamental de la dinámica interna de la Tierra y los procesos geológicos que oca-
siona es el calor interno de la Tierra que producen movimientos de convección del magma del
manto.
La litosfera se encuentra fragmentada en gigantescas piezas rígidas de unos 100 kilómetros
de espesor, llamadas placas litosféricas. Las placas se mueven y empujan entre sí sobre la aste-
nosfera en la que flotan, manteniendose en equilibrio isostático.
Cuando la erosión elimina materiales de los continentes, la litosfera se eleva para restable-
cer el equilibrio; por el contrario, cuando en una zona se acumulan materiales (por ejemplo sedi-
mentos en una cuenca sedimentaria), la litosfera se hunde parcialmente.
Las placas litosféricas en su incesante actividad producen tres tipos de bordes: bordes cons-
tructivos o dorsales oceánicas, bordes destructivos o zonas de subducción y bordes pasivos,
situados en las fallas de transformación, donde las placas se deslizan lentamente, hasta miles de
kilómetros, una junto a otra pero en sentidos opuestos.
Las corrientes de convección se producen cuando el magma más caliente y menos denso
asciende por las dorsales y se solidifica dando lugar a la litosfera oceánica. Continúa por deba-
jo de las placas alejándose de la dorsal y, después de enfriarse y hacerse más densa, desciende,
reintegrándose de nuevo al manto en las zonas de subducción
En las zonas donde chocan dos placas litosféricas, los sedimentos se pliegan y fracturan,
luego se elevan y surgen grandes cordilleras montañosas (orogénesis). Durante años, la energía
de choque se va acumulando y, de forma inesperada, se libera: se ha producido un terremoto

23
-Pág.27-
C I E N C I A S D E L A N AT U R A L E Z A

que hace vibrar la tierra. La corteza agrietada deja escapar violentamente gases y magma del
interior hasta la superficie: así se forma un volcán.
Los recursos naturales se pueden dividir en renovables (se generan a una velocidad pareci-
da a como se consumen) y no renovables, cuando la velocidad de renovación es mínima.
La explosión demográfica ha traído consigo una sobreexplotación de todos los recursos.
Estas acciones, frecuentemente negativas, son lo que conocemos como impacto ambiental. Si el
ritmo de explotación de los recursos no se adecua al ritmo de los ecosistemas, para mantener su
equilibrio, las consecuencias pueden ser nefastas.
Se entiende por contaminación atmosférica la presencia en la atmósfera de niveles de ener-
gía anómalos, sustancias extrañas o variaciones importantes en las concentraciones de las exis-
tentes, que tienen efectos perjudiciales o causan molestias a las personas, a los animales y a las
plantas.
Entre los problemas medioambientales atmosféricos tenemos la lluvia ácida, el agujero de
la capa de ozono, el efecto invernadero.
El agua puede estar contaminada por contaminantes físicos (incremento de temperatura y
partículas radiactivas), químicos (cloruros, fosfatos, compuestos nitrogenados, metales, pestici-
das, materia orgánica, detergentes, grasas y petróleo) y biológicos (microorganismos).
La degradación del suelo consiste en la pérdida de sus propiedades, y, por tanto, de su ferti-
lidad, y/o en la pérdida de alguno de sus horizontes.
En la actualidad antes de la construcción o la acometida de los proyectos se ha de realizar
la evaluación de impacto ambiental (EIA) que trata de medir los efectos que tienen sobre los
ecosistemas, la utilización de los recursos, la transformación del medio físico y las sustancias
emitidas a él.
La agricultura ecológica y las fuentes de energía alternativa se plantean como alternativas
válidas que lesionan poco el medio ambiente.
Los productos de desecho que se generan como resultado de las distintas actividades huma-
nas se pueden presentar en estado sólido, líquido o gaseoso y pueden tener distintos grados de
toxicidad. Por esta razón, es preciso que cada uno de ellos reciba el tratamiento más adecuado
a sus características.
La creciente preocupación por el medio natural ha hecho que se reserven áreas bien conser-
vadas y se protejan. Las figuras de protección de los espacios naturales en España son los Par-
ques Naturales, los Parques Nacionales, las Reservas Naturales y los Parques Regionales.
El tiempo transcurrido desde que se formó la Tierra, hace unos 4600 millones de años, hasta
la edad actual se divide en cuatro grandes subunidades llamadas eones: Hádico, Arcaico, Pro-
terozoico y Fanerozoico. Los tres primeros eones se suelen agrupar también con el nombre de
Precámbrico.

24
-Pág.28-
U.D. 9.- LOS CAMBIOS EN EL MEDIO N AT U R A L

Los fósiles nos sirven para calcular la edad de los estratos en los que se encuentran, los de
los estratos superiores son más modernos que los de los estratos inferiores (Principio de la suce-
sión faunística).
La unidad básica de clasificación es la especie, que se define como el grupo de individuos
con caracteres comunes que pueden cruzarse entre sí y tener descendencia fértil; los individuos
que viven en un área determinada constituyen una población.
La interacción entre los seres vivos y su entorno es constante y recíproca. Esto provoca que
los organismos desarrollen gradualmente adaptaciones mediante las cuales resuelven cada vez
mejor los problemas de reproducción y supervivencia que el medio les plantea.
La evolución es un proceso de cambio gradual de las características de los seres vivos, los
cuales desarrollan, generación tras generación, nuevas adaptaciones que son la causa de la
diversidad entre las especies procedentes de un antepasado común, y también son responsables
de la aparición de nuevas especies.
Entre las teorías evolucionistas están el transformismo y la teoría de la selección natural que
no terminan de explicar la transmisión de la herencia.
El neodarwinismo explica la evolución basándose en que afecta a una población donde exis-
te una variabilidad genética y la selección natural la ejerce el medio ambiente. El resultado de
la selección natural y del proceso evolutivo es que cada población estará cada vez mejor adap-
tada a su ambiente como consecuencia del aumento de las frecuencias de las combinaciones alé-
licas ventajosas. Cada éxito evolutivo es un acontecimiento casual e imprescindible.
La evolución es un fenómeno de poblaciones; no es el individuo el que evoluciona, sino la
población, ya que es aquí donde se presenta la variabilidad genética que permite actuar a la
selección natural.
El mecanismo evolutivo se explica por los modelos neodarwinista (evolución gradual) y del
equilibrio puntuado (evolución a empujones).

25
-Pág.29-
C I E N C I A S D E L A N AT U R A L E Z A

EJERCICIOS DE AUTOCOMPROBACIÓN

1. La sucesión ecológica:

A. Es la sucesión de individuos de la misma especie que permanecen siempre ocupando el mismo


nicho ecológico.

B. Es la secuencia de cambios graduales experimentados por un ecosistema a lo largo del tiempo.

C. Es la distribución de ecosistemas en el espacio de tal manera que tanto en altitud como en lati-
tud se suceden los ecosistemas.

D. Todas son ciertas.

2. La meteorización:

A. Se puede producir por el agua.

B. Se puede producir por el frío intenso.

C. Se puede producir por la acción del oxígeno.

D. Todas son ciertas.

3. La litogénesis:

A. Es el proceso por el cual se depositan los sedimentos.

B. Es el proceso por el cual los sedimentos se compactan.

C. Es el proceso por el cual se producen alteraciones mineralógicas en los sedimentos.

D. Es el proceso de formación de rocas sedimentarias.

4. Las placas litosféricas:

A. Se mueven y empujan entre sí sobre la astenosfera.

B. Sufren erosión en las zonas de contacto.

C. Permanecen rígidas y sin movimiento.

D. Todas son falsas.

5. La Tectónica de Placas estudia:

A. El movimiento de las placas B. La formación de montañas.


litosféricas.

C. Los volcanes. D. Todas son ciertas.

26
-Pág.30-
U.D. 9.- LOS CAMBIOS EN EL MEDIO N AT U R A L

6. El movimiento de las placas litosféricas:

A. Se produce por el calor interno de la Tierra.

B. Se produce por las corrientes de convección que se crean.

C. Se produce por la continua circulación de los materiales en el manto.

D. Todas son ciertas.

7. La enfermedad de las vacas locas:

A. Se ha producido por alimentar a las vacas con piensos fabricados con restos de ovejas enfermas.

B. Se ha producido al contagiarse del pasto que comían.

C. Se ha producido por las condiciones de estabulación en que viven los animales.

D. Todas son falsas.

8. El peligro del agujero de la capa de ozono se debe a:

A. Deja pasar más radiaciones infrarrojas.

B. Deja pasar más radiaciones ultravioleta.

C. Produce el calentamiento de la atmósfera.

D. Todas son ciertas.

9. Los primeros seres vivos que se formaron fueron:

A. Las bacterias. B. Los virus.

C. Los protozoos. D. Todas son ciertas.

10. El neodarwinismo propone que:

A. La evolución afecta a una población.

B. La selección natural la ejerce el ambiente.

C. La adaptación se produce como consecuencia del aumento de las frecuencias


de las combinaciones alélicas ventajosas.

D. Todas son ciertas.

27
-Pág.31-
C I E N C I A S D E L A N AT U R A L E Z A

RESPUESTAS A LOS EJERCICIOS

1. B

2. D

13. D

14. A

15. D

16. D

17. A

8. B

9. A

10. D

28
-Pág.32-
U.D. 10.- LAS FUERZAS Y LOS MOVIMIENTOS

ÍNDICE
OBJETIVOS . . . . . . . . . . . . . . . . . . . . . . . . . . . . . . . . . . . . . . . . . . . . . . . . . . . . . . . . . . .2

INTRODUCCIÓN . . . . . . . . . . . . . . . . . . . . . . . . . . . . . . . . . . . . . . . . . . . . . . . . . . . . . . .3

MAPA CONCEPTUAL . . . . . . . . . . . . . . . . . . . . . . . . . . . . . . . . . . . . . . . . . . . . . . . . . . .4

DESARROLLO DE CONTENIDOS . . . . . . . . . . . . . . . . . . . . . . . . . . . . . . . . . . . . . . . . .5

1. ESTUDIO CUALITATIVO DEL MOVIMIENTO . . . . . . . . . . . . . . . . . . . . . . . .5


1.1. MOVIMIENTO . . . . . . . . . . . . . . . . . . . . . . . . . . . . . . . . . . . . . . . . . . . . . . .5
1.2. NECESIDAD DE REFERENCIAS . . . . . . . . . . . . . . . . . . . . . . . . . . . . . . . . . .5
1.3. ESTUDIO CUALITATIVO DE CUALQUIER MOVIMIENTO . . . . . . . . . . . . . .5
1.4. TRATAMIENTO CUANTITATIVO DEL MOVIMIENTO RECTILÍNEO UNIFORME .7
1.5. ACELERACIÓN . . . . . . . . . . . . . . . . . . . . . . . . . . . . . . . . . . . . . . . . . . . . . . .9
1.6. MOVIMIENTO CIRCULAR . . . . . . . . . . . . . . . . . . . . . . . . . . . . . . . . . . . . .10

2. LAS FUERZAS. EFECTO SOBRE LOS CUERPOS . . . . . . . . . . . . . . . . . . . . .11


2.1. LAS FUERZAS . . . . . . . . . . . . . . . . . . . . . . . . . . . . . . . . . . . . . . . . . . . . . .11
2.2. EFECTO SOBRE LOS CUERPOS . . . . . . . . . . . . . . . . . . . . . . . . . . . . . . . . .11
2.3. PRINCIPIOS DE LA DINÁMICA . . . . . . . . . . . . . . . . . . . . . . . . . . . . . . . . .12
2.4. CONDICIONES DE EQUILIBRIO . . . . . . . . . . . . . . . . . . . . . . . . . . . . . . . . .13

3. LA GRAVITACIÓN UNIVERSAL . . . . . . . . . . . . . . . . . . . . . . . . . . . . . . . . . . .13


3.1. EL PESO DE LOS CUERPOS. LA SÍNTESIS NEWTONIANA . . . . . . . . . . . . .14

4. FUERZAS DE INTERÉS EN LA VIDA COTIDIANA . . . . . . . . . . . . . . . . . . .14

ANEXO I . . . . . . . . . . . . . . . . . . . . . . . . . . . . . . . . . . . . . . . . . . . . . . . . . . . . . . . . . . . . .15

RESUMEN . . . . . . . . . . . . . . . . . . . . . . . . . . . . . . . . . . . . . . . . . . . . . . . . . . . . . . . . . . . .16

EJERCICIOS DE AUTOCOMPROBACIÓN . . . . . . . . . . . . . . . . . . . . . . . . . . . . . . . . .18

RESPUESTAS A LOS EJERCICIOS . . . . . . . . . . . . . . . . . . . . . . . . . . . . . . . . . . . . . . .20

-Pág.33-
C I E N C I A S D E L A N AT U R A L E Z A

O B J E T I VO S
Al finalizar el estudio de esta Unidad Didáctica, el alumno será capaz de:

• Utilizar correctamente conceptos como posición, trayectoria, distancia recorrida,


velocidad, fuerza, peso, masa o inercia.

• Poder describir distintos tipos de movimiento.

• Reconocer los principios de la Dinámica.

• Elaborar e interpretar gráficas sobre el movimiento.

-Pág.34-
U.D. 10.- LAS FUERZAS Y LOS MOVIMIENTOS

INTRODUCCIÓN
L as fuerzas y sus efectos, fundamentalmente el movimiento, están presentes en todas
nuestras actividades cotidianas. Las manifestaciones de los distintos tipos de fuer-
zas se muestran continuamente a nuestro alrededor y de una u otra forma nos afectan.
Por eso, su estudio y comprensión son tan importantes para el hombre.

La Cinemática y la Dinámica son las dos ramas de la Física que estudian el movimien-
to y las fuerzas. En este tema vamos a estudiar diferentes conceptos sobre estas ramas
para conocerlas un poco más.

-Pág.35-
C I E N C I A S D E L A N AT U R A L E Z A

M A PA C O N C E P T UA L
las
producen estudia la
TRES
MOVIMIENTO FUERZAS DINÁMICA
PRINCIPIOS

INERCIA
cuyas
lo puede
estudia la tener ACELERACIÓN

MAGNITUDES ACCIÓN
REACCIÓN
CINEMÁTICA VELOCIDAD VELOCIDAD
CONSTANTE VARIABLE

MASA FUERZA N
MOVIMIENTO
UNIFORME tiene
cuyas Kg produce

ACELERACIÓN
CAMBIOS EN
MAGNITUDES EL MOVIMIENTO
(aceleraciones)

CONSTANTE VARIABLE
DEFORMACIONES

según su trayectoria NINGÚN EFECTO


APARENTE
VELOCIDAD m/s
MOVIMIENTO RECTILINEO
UNIFORMEMENTE
ACELERACIÓN m/s2 ACELERADO/DECELERADO

POSICIÓN MOVIMIENTO
CIRCULAR
UNIFORMEMENTE
ACELERADO/DECELERADO

depende de

TRAYECTORIA DISTANCIA

-Pág.36-
U.D. 10.- LAS FUERZAS Y LOS MOVIMIENTOS

1. ESTUDIO CUALITATIVO DEL MOVIMIENTO

1.1. MOVIMIENTO
Todo aquello que cambia de posición estará en movimiento y todo aquello que parezca no
cambiar de sitio estará en reposo. La ciencia que estudia todo lo relacionado con los movi-
mientos se llama Cinemática.

1.2. NECESIDAD DE REFERENCIAS


Para poder estudiar el movimiento es necesario un punto fijo que nos sirva de punto de refe-
rencia. Normalmente solemos tomar como punto de referencia algo íntimamente unido al suelo.
Un cuerpo está en movimiento, respecto a un sistema de referencia, cuando cambia su posición
en relación a dicho sistema.

1.3. ESTUDIO CUALITATIVO DE CUALQUIER MOVIMIENTO


Decimos que un objeto se encuentra en movimiento cuando cambia de lugar, de posición,
con el tiempo. Ese movimiento puede ocurrir con mayor o menor rapidez, y para indicar esa
rapidez utilizamos una magnitud llamada velocidad.
La velocidad se calcula dividiendo el espacio recorrido entre el tiempo empleado.

v = e/t

La velocidad de los coches la indica el velocímetro en kilómetros por hora (Km/h). Sin
embargo, la unidad de velocidad del Sistema Internacional, son los metros por segundo (m/s),
es decir, los kilómetros o metros que recorre un móvil en cada unidad de tiempo (hora o se-
gundo).
El convertir unas unidades en otras, es fácil.
Para pasar de kilómetros a metros se multiplica por mil, puesto que un kilómetro tiene mil
metros. Y para pasar de horas a segundos se divide por 3600, puesto que una hora tiene 60 minu-
tos y un minuto sesenta segundos (60×60 = 3600).
Por ejemplo, para pasar 120 Km/h a m/s se haría:
120 × 1000/3600 = 120000/3600 = 33.33 m/s

5
-Pág.37-
C I E N C I A S D E L A N AT U R A L E Z A

Por el contrario, si se quiere pasar de m/s a Km/h habría que dividir por 1000 para pasar de
metros a Kms y multiplicar por 3600 para pasar de segundos a horas puesto que una hora tiene
3600 segundos.
Así para pasar de 7 m/s a que corre una velocista y expresarlo en Km/h se haría:
(7 : 1000) × 3600 = 25,2 Km/h.
Como se puede ver la relación entre las dos unidades es:
1 Km/h = 1000 m/3600 s = 1/3,6 m/s; 1 m/s = 3,6 Km/h
Ejemplos:
1) Vamos a calcular la velocidad de un tren que recorre 260 Km en dos horas.
Como velocidad es el espacio recorrido dividido entre el tiempo empleado, se pondría:
V = e/t = 260 Km/2 horas. Hay que comprobar las unidades y vemos que están correc-
tas. Ahora dividimos 260/2 y da 130 Km/h.
2) Vamos ahora a calcular la velocidad que desarrolla un atleta que recorre 200 metros en
22,5 segundos. Se procede de igual modo que en el ejemplo anterior.
V = e/t = 200 m/22,5 seg = 8,88 m/s = 32 Km/h.

En el movimiento se distingue la trayectoria, es decir, el camino que sigue el móvil, y el


desplazamiento, es decir, la distancia real recorrida. Por ejemplo, cuando un saltador de longi-
tud realiza un salto, se eleva del suelo describiendo una parábola (trayectoria), pero los jueces
miden en el suelo lo que ha saltado (desplazamiento). Dicho con otras palabras, la trayectoria
es el camino recorrido por un cuerpo en su movimiento y el desplazamiento es la distancia entre
el origen y el final del movimiento.
Cuando la trayectoria es una línea recta, decimos que el movimiento es rectilíneo y cuando
es curva, decimos que es un movimiento curvilíneo. Ejemplos de movimientos rectilíneos pue-
den ser la carrera de 100 metros lisos y de movimiento curvilíneo el del saltador de longitud
(parábola), la parábola que describe la pelota de baloncesto para llegar a la canasta, el movi-
miento de la Tierra alrededor del Sol (elíptico), etc. Así, el movimiento curvilíneo puede des-
cribir parábolas, elipses, círculos, etc.

6
-Pág.38-
U.D. 10.- LAS FUERZAS Y LOS MOVIMIENTOS

1.4. TRATAMIENTO CUANTITATIVO DEL MOVIMIENTO REC-


TILÍNEO UNIFORME
El movimiento se puede representar gráficamente. Para ello representamos en el eje de abci-
sas (eje X –horizontal–) el tiempo y en el eje de ordenadas (eje Y –vertical–) el espacio reco-
rrido. Veamos varios ejemplos.
Cada representación corresponde a unos datos que se recogen en una tabla. Para represen-
tarlo hay que ponerlo en las unidades que se correspondan, es decir, si el espacio va en metros,
el tiempo ha de ir en segundos y si el espacio va en kilómetros, el tiempo ha de ir en horas.
Para trazar los ejes, se dividen los ejes X e Y en tramos iguales (lo ideal es utilizar papel
milimetrado) y se llevan los datos de la misma forma que si jugáramos a los barquitos, cada vez
un par de datos.

Representación del movimiento uniforme

300
Kilómetros

200
Km/h
100
0
0 2 4 6
horas

Espacio (Km) Tiempo (horas)


50 1
100 2
150 3
200 4
250 5

La información que podemos obtener de una representación gráfica es, si se trata de un


movimiento uniforme (es una recta) o variado (es una curva). En las representaciones gráficas,
podemos obtener las velocidades (dividiendo cada espacio entre el tiempo) a partir de un punto
concreto llevándolo a los ejes, conocer cómo de rápido se mueve un móvil, observando la pen-
diente, es decir, la inclinación de la recta representada; cuanto mayor sea la pendiente de la
recta, es decir, cuanto más levantada esté, mayor será la velocidad. También al comparar las grá-
ficas podemos saber cuál se mueve más deprisa.

7
-Pág.39-
C I E N C I A S D E L A N AT U R A L E Z A

Movimiento Uniforme

Espacio (metros)
100
80
60
40 velocidad (e/t)
20
0
0 5 10

Tiempo (segundos)

Mayor rapidez.

Tiempo (s) Espacio (m) Velocidad (m/s)


0 0 0
2 24 24/2 = 12
4 48 (48-24)/2 = 12
6 72 (72-48)/2 = 12
8 96 (96-72)/2 = 12

Movimiento Uniforme
Espacio (metros)

100
80
60
40 velocidad (e/t)
20
0
0 5 10

Tiempo (segundos)

Menor rapidez.

Tiempo (s) Espacio (m) Velocidad (m/s)


0 0 0
2 12 12/2 = 6
4 24 (24-12)/2 = 6
6 36 (36-24)/2 = 6
8 48 (48-36)/2 = 6

8
-Pág.40-
U.D. 10.- LAS FUERZAS Y LOS MOVIMIENTOS

Además de las representaciones espacio/tiempo, se pueden hacer representaciones veloci-


dad/tiempo. En el caso del movimiento uniforme (m.u.), al ser la velocidad constante, su repre-
sentación gráfica sería una recta horizontal. Así, si un móvil fuera a 40 m/s durante todo el
recorrido, su representación gráfica sería:

Gráfica velocidad/tiempo
en el movimiento uniforme
Velocidad

60
(m/s)

40
velocidad (m/s)
20
0
0 50 100

Tiempo (s)

Cuando un cuerpo se mueve según una trayectoria rectilínea y manteniendo constante su


velocidad (esto es, recorriendo espacios iguales en tiempos iguales), decimos que se describe
un movimiento rectilíneo uniforme. En estos casos, la velocidad media del movimiento coinci-
de con la velocidad en cualquier instante.

1.5. ACELERACIÓN
En algunos movimientos, la velocidad no siempre es la misma, existen cambios durante el
recorrido. La aceleración es la magnitud que mide la variación de velocidad en cada unidad de
tiempo.

a = ∆v/t

La aceleración se medirá en (m/s)/s; es decir m/s2.


Cuando se frena, se habla de deceleración, es una aceleración en la que la variación de velo-
cidad tiene un signo negativo. Las representaciones gráficas corresponden a figuras parabólicas.

Movimiento uniforme acelerado

60
Espacio (m)

40
velocidad (m/s)
20
0
0 2 4 6

Tiempo (s)

9
-Pág.41-
C I E N C I A S D E L A N AT U R A L E Z A

Se pueden clasificar los movimientos, atendiendo al hecho de que se produzca o no un cam-


bio en la rapidez:
– Movimientos uniformes: son aquellos en que la velocidad no varía.
– Movimientos variados: aquellos en los que la velocidad varía de una forma u otra. Exis-
ten muchos tipos. El más sencillo que es cuando la aceleración es constante, es el movi-
miento uniformemente acelerado.
Para clasificar un movimiento, debemos hacer referencia a ambas clasificaciones. Por ejem-
plo, un cohete sigue un movimiento rectilíneo uniforme, las agujas del reloj, un movimiento cir-
cular uniforme, y un coche al arrancar un movimiento rectilíneo uniformemente acelerado.
Igualmente este mismo coche al frenar seguirá un movimiento rectilíneo uniformemente
decelerado.

1.6. MOVIMIENTO CIRCULAR


Es el movimiento en el que la trayectoria es una circunferencia. Se habla de movimiento cir-
cular uniforme cuando la velocidad de giro es constante y la trayectoria circular.
El móvil va dando vueltas y al terminar la vuelta se encuentra en el mismo sitio que al prin-
cipio.
Para estudiar estos movimientos, se habla de ángulo recorrido, que es el que forman los
radios de la circunferencia correspondientes a las posiciones inicial y final y así se habla de
espacio angular. Por ejemplo, cuando al montarnos en la noria, nos suben arriba, hemos reco-
rrido un ángulo de 180°C (la mitad de la circunferencia) y corresponde a un espacio angular de
media vuelta.
Para medir la rapidez del giro, se habla de velocidad angular y se expresa en revoluciones
por minuto (r.p.m.), es decir, vueltas por minuto. Este término es muy frecuentemente utilizado
en motores, discos y en general en artefactos que giran.
Es fácil pasar de r.p.m. a m/s. Veamos un ejemplo. ¿Qué velocidad llevan dos puntos de un
disco situados a 5 y 10 cm del eje de giro si gira a 45 r.p.m.
Como la longitud de la circunferencia L = 2πR, el punto que está a 5 cm recorre L = 2π5 =
= 31,4 cm en cada vuelta. Como da 45 vueltas en cada minuto, recorre 31,4 × 45 = 1413
cm/min. Es decir 14,13 metros cada 60 segundos. Por lo que su velocidad será 14,13/60 = 0,235
m/s.
Del mismo modo, la velocidad del punto situado a 10 cm será 0,470 m/s.

10
-Pág.42-
U.D. 10.- LAS FUERZAS Y LOS MOVIMIENTOS

2. LAS FUERZAS. EFECTO SOBRE LOS CUERPOS

2.1. LAS FUERZAS


Las causas que provocan cambios en el movimiento de las cosas se denominan fuerzas. La
Dinámica es la rama de la Física, iniciada por Newton, que estudia la causa de los movimien-
tos, así como la Cinemática estudia el efecto de estas fuerzas, es decir, los movimientos.
Las fuerzas se pueden aplicar por contacto (un empujón) o a distancia (el efecto que produ-
ce el imán sobre el hierro).
Las interacciones a distancia se clasifican según su naturaleza en:
– Gravitatoria: es una interacción «atractiva» que se da entre los cuerpos por el hecho de
estar formados por materia. Es la causa de que los planetas giren alrededor del Sol y de
que nosotros nos mantengamos unidos a la Tierra.
– Magnética: puede ser atractiva cuando los polos del imán son de distinto signo y repul-
siva, cuando los polos del imán son del mismo signo.
– Electrostática: También puede ser atractiva y repulsiva. Será atractiva cuando se pro-
duzca entre cargas de distinto signo, y repulsiva cuando sea entre cargas de igual signo.
Al frotar un bolígrafo contra un trapo seco y lo acercamos a unos papelitos, se observa
que son atraídos por el bolígrafo, en contra de la fuerza gravitatoria de la Tierra.
Las interacciones por contacto, son múltiples y de muy diversa índole. Tienen lugar en
diversas circunstancias, cuando empujas una pared, le das una patada a un balón, etc.

2.2. EFECTO SOBRE LOS CUERPOS


Los efectos que producen las fuerzas son: cambios en el movimiento y deformaciones.
a) Cambios en el movimiento
Podríamos decir que la fuerza es la interacción entre dos cuerpos que puede provocar un
cambio en el movimiento de uno de ellos, esto es, que varía la rapidez (acelera o dece-
lera) o que cambia la dirección.
Las fuerzas se suelen representar mediante vectores (flechas), que serán de distinto tama-
ño de modo proporcional a la intensidad y dirección de la fuerza.
b) Deformaciones
Cuando una fuerza no tiene posibilidad de producir movimiento (la mesa no deja pasar
la plastilina hacia abajo), el efecto que produce dicha fuerza es una deformación (si el

11
-Pág.43-
C I E N C I A S D E L A N AT U R A L E Z A

material lo permite). Atendiendo a la resistencia que oponen los cuerpos a ser deforma-
dos, podemos considerarlos rígidos (son los que se deforman muy poco bajo la acción de
la fuerza) y deformables (se deforman con relativa facilidad). Podemos, según esta carac-
terística, dividir los materiales en dos grupos: materiales plásticos, que son los que al dejar
de aplicar la fuerza la deformación permanece y materiales elásticos, que son los que
recuperan su forma original al desaparecer la fuerza (muelle). Muchos cuerpos son elás-
ticos, si la fuerza deformadora no sobrepasa un cierto valor, denominado límite elástico,
pero si la fuerza supera dicho límite, ya no se recupera la forma original. Asimismo hay
un límite de rotura, que es la máxima fuerza que puede soportar un cuerpo sin romperse.
Los dinamómetros son instrumentos utilizados para medir fuerzas, basados en la deforma-
ción elástica. Los más utilizados son los constituidos por un muelle de acero en el que el alar-
gamiento (x) que experimentan es proporcional a la fuerza (F) que se aplica. Esta relación de
proporcionalidad se conoce como Ley de Hooke.
F=K·x donde K es la constante elástica del resorte.
Existen muchos instrumentos basados en este fundamento: básculas, grapadoras, muelles de
puertas, etc.

2.3. PRINCIPIOS DE LA DINÁMICA


Como ya hemos comentado la Dinámica se basa en las investigaciones de Newton, el cual
enunció tres leyes que son:
– Primera Ley de Newton: Principio de Inercia. (Primer principio de la Dinámica)
Si sobre un cuerpo no actúa ninguna fuerza, o la resultante de las fuerzas aplicadas sobre
él es nula, dicho cuerpo permanece en reposo o continúa moviéndose con movimiento
rectilíneo y uniforme.
El rozamiento es una fuerza que aparece cuando hay contacto entre dos cuerpos, que se mue-
ven uno con respecto al otro, y que siempre se opone al movimiento. Cuando vamos en bici-
cleta se mantiene la tendencia al movimiento, pero tiende a pararse debido al rozamiento del
aire y de las ruedas sobre el suelo.
La presencia de la atmósfera hace que todos los movimientos en la Tierra, incluso el vuelo,
sufran la fuerza del rozamiento.
– Segunda Ley de Newton. (Principio Fundamental de la Dinámica)
La fuerza aplicada sobre un cuerpo es directamente proporcional a la aceleración que le
produce.

F=m·a

12
-Pág.44-
U.D. 10.- LAS FUERZAS Y LOS MOVIMIENTOS

Podemos decir que los objetos pesados tienen más inercia, es decir, oponen más resistencia
a cualquier cambio en su movimiento.
La unidad de Fuerza en el S.I. es el Newton (N).
1 N = 1 Kg. 1 m/s2
– Tercera Ley de Newton. (Principio de Acción y Reacción)
Cuando un cuerpo A ejerce una fuerza (acción) sobre otro cuerpo B, el segundo ejerce
sobre el primero otra fuerza igual y de sentido contrario (reacción).
→ →
FA→B = – FB→A

2.4. CONDICIONES DE EQUILIBRIO


Cuando se ejercen dos fuerzas iguales en sentido contrario (echando un pulso, jugando a
tirar de los dos extremos de una cuerda), éstas se contrarrestan, experimentando el mismo efec-
to que si no existiera ninguna.
El principio de acción-reacción (tercer principio de la dinámica), dice que la fuerza que
ejerce un cuerpo sobre otro se llama acción, y produce otra igual, pero de sentido contrario, ejer-
cida por el segundo cuerpo, que se llama reacción.

3. LA GRAVITACIÓN UNIVERSAL
– Ley de la Gravitación Universal
Dos masas se atraen con una fuerza directamente proporcional al producto de sus masas
e inversamente proporcional al cuadrado de la distancia que las separa.
m 1 · m2
F=G
d2
N · m2
siendo G la constante de Gravitación Universal cuyo valor es 6,67 · 10–11
Kg2
Esta ley afecta al Universo entero aunque, la atracción que ejercen unos cuerpos sobre otros
va a depender de la masa que posean y de la distancia a que estén situados.

13
-Pág.45-
C I E N C I A S D E L A N AT U R A L E Z A

3.1. EL PESO DE LOS CUERPOS. LA SÍNTESIS NEWTONIANA


La atracción gravitatoria que la Tierra ejerce sobre un cuerpo de 1 Kg de masa se conoce
como gravedad de la Tierra (gT). En la superficie terrestre, la gT tiene un valor de 9,8 N/Kg. El
valor de la gravedad de la Tierra disminuye a medida que el cuerpo se aleja de su superficie.
La fuerza gravitatoria que ejerce la Tierra sobre un cuerpo se llama peso (P), y depende de la
masa del cuerpo y de la gravedad de nuestro planeta.
El peso se calcula multiplicando la masa del cuerpo expresada en Kilogramos por el valor
de la gravedad (g) expresada en N/Kg.

P=m·g

En la superficie de la Tierra, donde g = 9,8 N/Kg, el peso de un Kilogramo de masa equi-


vale a:
P = m · g = 9,8 N
A esta fuerza se le da el nombre de Kilopondio (Kp), 1 Kp = 9,8 N

4. FUERZAS DE INTERÉS EN LA VIDA COTIDIANA


El origen de todas las fuerzas se encuentra en la estructura más pequeña de la materia, el
átomo.
Todas las fuerzas que hemos visto hasta ahora pueden clasificarse en dos tipos: fuerzas de
atracción debidas a la masa de los cuerpos (gravedad) y fuerzas provocadas por las atracciones
y repulsiones entre las cargas eléctricas (electromagnetismo).
Para encontrar las otras dos fuerzas fundamentales debemos internarnos en el núcleo de los
átomos. Los protones tienen la misma carga, por lo tanto deberían repelerse, sin embargo, per-
manecen unidos por una fuerza llamada fuerza nuclear fuerte, mucho mayor que la electro-
magnética pero que sólo actúa en distancias tan pequeñas como el núcleo de un átomo.
Existe además la que se denomina fuerza nuclear débil, que se debe a la existencia de áto-
mos cuyos núcleos son inestables, que se desintegran debido a la fuerza mencionada.

14
-Pág.46-
U.D. 10.- LAS FUERZAS Y LOS MOVIMIENTOS

ANEXO I
Como anexo a esta Unidad se presentan las dos tablas siguientes con el fin de tener en un
cuadro todas las unidades expresadas en el Sistema Internacional.

MÚLTIPLOS Y SUBMÚLTIPLOS
Prefijo Símbolo Equivalencia.
Tera T 1012 = 1.000.000.000.000
Giga G 109 = 1.000.000.000
Mega M 106 = 1.000.000
Kilo K 103 = 1.000
Hecto h 102 = 100
Deca da 10 = 10
Deci d 10 –1 = 0,1
Centi c 10 –2 = 0,01
Mili m 10 –3 = 0,001
Micro µ 10 –6 = 0,000 001
Nano n 10 –9 = 0,000 000 001
Pico p 10 –12 = 0,000 000 000 001

MAGNITUDES Y UNIDADES DERIVADAS


EN EL SISTEMA INTERNACIONAL (S. I.)
MAGNITUD Nombre de la Unidad Símbolo de la Unidad Expresión S.I.
Superficie Metro cuadrado m2 m2
Volumen Metro cúbico m3 m3
Velocidad Metro por segundo m/s m/s
Aceleración Metro/segundo cuadrado m/s2 m/s2
Fuerza Newton N Kg · m/s2
Presión Pascal Pa N/m2
Energía-Trabajo Julio J Kg · m2/s2
Potencia Watio W Kg · m2/s3
Densidad Kilogramo/metro cúbico Kg/m3 Kg/m3

15
-Pág.47-
C I E N C I A S D E L A N AT U R A L E Z A

RESUMEN
El movimiento es relativo, no podemos saber qué se mueve ni como lo hace si no estable-
cemos un punto de referencia.
La trayectoria es el camino que sigue el móvil. Desplazamiento, sin embargo, es la distan-
cia en línea recta entre el principio y el final del movimiento, por lo que no tiene por qué coin-
cidir.
La velocidad es la relación entre el espacio recorrido y el tiempo empleado.
La aceleración es una magnitud que indica el cambio que sufre la velocidad a lo largo del
tiempo.
Los movimientos, atendiendo a su trayectoria, pueden ser rectilíneos y curvilíneos (circula-
res, elípticos, parabólicos, etc.).
Atendiendo a la velocidad, los movimientos pueden ser uniformes (con velocidad constan-
te y, por tanto, sin aceleración) o variados (con velocidad cambiante y con aceleración).
En el movimiento rectilíneo uniforme, cuanto mayor sea la inclinación (o pendiente) de la
recta en la gráfica espacio-tiempo, mayor será la velocidad.
Las fuerzas son interacciones entre dos cuerpos que pueden producir cambios en el movi-
miento o deformaciones.
Las fuerzas pueden producirse por contacto y a distancia.
La fuerza de rozamiento se produce al desplazarse un cuerpo sobre otro (o a través de él),
siempre en contra del movimiento.
La presencia de la atmósfera hace que todos los movimientos en la Tierra, incluso el vuelo,
sufran la fuerza del rozamiento.
Si sobre un cuerpo no actúa ninguna fuerza o la resultante de las fuerzas aplicadas sobre él
es nula, no cambia ni su dirección ni su velocidad: si está parado, seguirá parado, y si está
moviéndose, lo hará con movimiento rectilíneo uniforme. Es el principio de inercia o primera
ley de Newton.
Cuanto mayor es la masa de un cuerpo, mayor resistencia presenta a cambiar de movimien-
to o estado de reposo en el que está.
Siempre que hay un cambio en el movimiento, se debe a una fuerza.
La fuerza aplicada para cambiar el movimiento de un cuerpo es el producto de su masa por
la aceleración que le produce. Es la segunda ley de Newton.
A toda fuerza siempre se le opone otra igual en sentido contrario (principio de acción-reac-
ción).

16
-Pág.48-
U.D. 10.- LAS FUERZAS Y LOS MOVIMIENTOS

Un cuerpo deformado que, al dejar de actuar la fuerza sobre él, retorna a su forma inicial,
está formado por material elástico. Si se mantiene la deformación es de un material plástico.
La fuerza de la gravedad, es una fuerza de atracción debida a la masa de los cuerpos que sólo
se hace patente cuando las masas son tan grandes como los planetas o los satélites.
El movimiento de los planetas y las estrellas se debe a las atracciones gravitatorias.
Peso es la fuerza con que la Tierra u otro planeta atrae a los cuerpos que hay en su superficie.
Presión es la fuerza que actúa en cada unidad de superficie. Cuanto mayor sea la superficie
sobre la que actúa una fuerza, menor es la presión que ejerce.

17
-Pág.49-
C I E N C I A S D E L A N AT U R A L E Z A

EJERCICIOS DE AUTOCOMPROBACIÓN

1. El punto de referencia sirve para:

A. Medir el tiempo. C. Determinar si algo se mueve.


B. Medir el desplazamiento. D. Medir la aceleración.

2. El camino que sigue un móvil es:

A. La trayectoria. C. La gráfica espacio-tiempo.

B. El desplazamiento. D. El avance.

3. La trayectoria de los planetas alrededor del Sol es:

A. Parabólica. B. Elíptica. C. Circular. D. Rectilínea.

4. El espacio recorrido en cada unidad de tiempo es:

A. El desplazamiento. C. La aceleración.

B. La velocidad. D. La presión.

5. La velocidad es inversamente proporcional:

A. Al tiempo empleado. C. A la trayectoria.

B. Al espacio recorrido. D. Todas son falsas.

6. Las fuerzas pueden producirse:

A. Sólo por contacto. C. Sólo a distancia.

B. Por contacto o a distancia. D. Todas son falsas.

7. Un cambio en el movimiento indica:

A. Que hay una velocidad. C. Que actúa una fuerza.

B. Que hay una masa. D. Que hay un avance.

8. Los materiales que retornan a su forma original cuando deja de actuar una fuerza son:

A. Elásticos. B. Metálicos. C. Plásticos. D. Transformables.

18
-Pág.50-
U.D. 10.- LAS FUERZAS Y LOS MOVIMIENTOS

9. La tendencia que tienen los cuerpos a no cambiar su movimiento se llama:

A. Presión. B. Aceleración. C. Inercia. D. Fuerza.

10. La fuerza de rozamiento:

A. Se opone siempre al movimiento. C. Se produce a distancia.

B. Es contraria a la gravedad. D. Todas son ciertas.

19
-Pág.51-
C I E N C I A S D E L A N AT U R A L E Z A

RESPUESTAS A LOS EJERCICIOS

1. C

2. A

13. B

14. B

15. A

16. B

17. C

8. A

9. C

10. A

20
-Pág.52-
U . D . 11 . - E LECTRICIDAD Y MAGNETISMO

ÍNDICE
OBJETIVOS . . . . . . . . . . . . . . . . . . . . . . . . . . . . . . . . . . . . . . . . . . . . . . . . . . . . . . . . . . . . . . 2

INTRODUCCIÓN . . . . . . . . . . . . . . . . . . . . . . . . . . . . . . . . . . . . . . . . . . . . . . . . . . . . . . . . . . 3

MAPA CONCEPTUAL . . . . . . . . . . . . . . . . . . . . . . . . . . . . . . . . . . . . . . . . . . . . . . . . . . . . . 4

DESARROLLO DE CONTENIDOS . . . . . . . . . . . . . . . . . . . . . . . . . . . . . . . . . . . . . . . . . . 5

1. FENÓMENOS DE ELECTRIZACIÓN . . . . . . . . . . . . . . . . . . . . . . . . . . . . . . . . . . . 5
1.1. LEY DE COULOMB . . . . . . . . . . . . . . . . . . . . . . . . . . . . . . . . . . . . . . . . . . . . . . . . 5
1.2. CARGAS Y FUERZAS ELÉCTRICAS . . . . . . . . . . . . . . . . . . . . . . . . . . . . . . . . . . . 6

2. CORRIENTE ELÉCTRICA . . . . . . . . . . . . . . . . . . . . . . . . . . . . . . . . . . . . . . . . . . . . 7
2.1. DIFERENCIA DE POTENCIAL E INTENSIDAD . . . . . . . . . . . . . . . . . . . . . . . . . . . 7
2.2. TRANSFORMACIONES ENERGÉTICAS EN UN CIRCUITO ELÉCTRICO . . . . . . . 8

3. IMANES. EFECTO DE UNA CORRIENTE ELÉCTRICA SOBRE UNA AGUJA


IMANTADA . . . . . . . . . . . . . . . . . . . . . . . . . . . . . . . . . . . . . . . . . . . . . . . . . . . . . . . . . . 10
3.1. ESTUDIO CUALITATIVO DE LA INDUCCIÓN ELECTROMAGNÉTICA . . . . . . . .10

4. NORMAS DE SEGURIDAD EN LA UTILIZACIÓN DE LA ELECTRICIDAD . . . 11

RESUMEN . . . . . . . . . . . . . . . . . . . . . . . . . . . . . . . . . . . . . . . . . . . . . . . . . . . . . . . . . . . . . . . 13

EJERCICIOS DE AUTOCOMPROBACIÓN . . . . . . . . . . . . . . . . . . . . . . . . . . . . . . . . . . . 14

RESPUESTAS A LOS EJERCICIOS . . . . . . . . . . . . . . . . . . . . . . . . . . . . . . . . . . . . . . . . . 16

-Pág.53-
C I E N C I A S D E L A N AT U R A L E Z A

O B J E T I VO S
Al finalizar el estudio de esta Unidad Didáctica, el alumno será capaz de:

• Describir algunos fenómenos eléctricos y explicar en qué consiste la electricidad.

• Describir y manejar magnitudes relacionadas con la electricidad.

• Relacionar la electricidad y magnetismo.

• Explicar en qué se basa el funcionamiento de motores y generadores.

• Valorar y aplicar las medidas de seguridad.

-Pág.54-
U . D . 11 . - E LECTRICIDAD Y MAGNETISMO

INTRODUCCIÓN
H asta hace unos cien años, durante la noche, el planeta Tierra era un oscuro punto en
el Universo. En 1879, Thomas Alva Edison fabricó la primera bombilla eléctrica, y
en 1882, tres años después, iluminaba las tiendas y casas de una calle de Nueva York.
Desde entonces..., muchas cosas han cambiado.

-Pág.55-
C I E N C I A S D E L A N AT U R A L E Z A

M A PA C O N C E P T UA L

LA ELECTRICIDAD relacionada con el MAGNETISMO

ELECTROMAGNETISMO
originada por

CARGAS ELÉCTRICAS
NEGATIVAS

de la ENERGÍA (JULIOS) J

CORTEZA DE LOS POTENCIA


ÁTOMOS (WATIOS) W

llamadas INTENSIDAD
(AMPERIOS) A

DIFERENCIA DE
ELECTRONES POTENCIAL
(VOLTIOS) V
su movimiento
origina la RESISTENCIA
(OHMIOS)

CORRIENTE ELÉCTRICA cuyas características son

LEY DE OHM
relacionadas por la
V=I·R

-Pág.56-
U . D . 11 . - E LECTRICIDAD Y MAGNETISMO

1. FENÓMENOS DE ELECTRIZACIÓN
Seiscientos años antes de Cristo, el griego Tales de Mileto descubrió que frotando un trozo
de ámbar pulido, éste atraía pequeñas hojas y pajitas. Como el ámbar entonces se llamaba "elek-
tron", a aquel extraño fenómeno lo llamó eléctrico.
Los cuerpos se pueden electrizar por frotamiento (electricidad estática) (al frotar el bolígra-
fo de plástico -Bic- puede atraer pequeños trozos de papel, o el pelo largo, que al peinarlo cuan-
do está seco se electriza y saltan chispas a veces). Estos fenómenos se deben a las cargas
eléctricas, que hacen que cuando son iguales, se repelen y cuando son distintas, se atraen. Pode-
mos decir que las cargas del mismo signo (positivo o negativo) se repelen y las de distinto signo
se atraen. La electricidad estática es la responsable de los rayos y de esas descargas que nota-
mos después de un viaje en coche.

1.1. LEY DE COULOMB


La fuerza de atracción o de repulsión F, entre dos cargas q y q', es directamente proporcional
al producto de las mismas e inversamente proporcional al cuadrado de la distancia, r, entre ellas:

F = K · q · q'/r2

La constante de proporcionalidad K, depende del medio aislante en el que se encuentran las


cargas. Para el vacío, en unidades del SI, vale:

K = 9 · 109 N · m2/C2

En este sistema la unidad de carga es el Culombio (C) y se define como la carga que repele
a otra igual, situada en el vacío a un metro de distancia, con la fuerza de 9 · 109 N.
A la zona de influencia creada por una carga eléctrica, donde se ejerce atracción o repulsión
eléctrica sobre otras cargas, se denomina campo eléctrico.
Lo mismo que la fuerza, la intensidad de campo eléctrico en un punto, es una magnitud vec-
torial que expresa la fuerza ejercida por la carga que crea el campo en ese punto sobre la uni-
dad de carga positiva.
E=F/q

La intensidad de campo, se mide en N/C (Newtons/Culombios).

5
-Pág.57-
C I E N C I A S D E L A N AT U R A L E Z A

1.2. CARGAS Y FUERZAS ELÉCTRICAS


El origen de la electricidad está en el átomo, siendo los electrones los que se mueven en los
fenómenos eléctricos, así, si un átomo neutro pierde electrones, queda cargado positivamente
(catión), al tener más protones que electrones y si gana electrones, queda cargado negativa-
mente (anión), porque en él habrán más electrones que protones. A los cationes y aniones, en
general se les denomina iones.
Los materiales de acuerdo con la facilidad con que los electrones pueden moverse a través
de ellos se dividen en conductores y aislantes. En los conductores los electrones se pueden
mover con mucha facilidad de átomo en átomo. La mayoría de los metales, las disoluciones de
ácidos, bases y sales (electrolitos) son buenos conductores. En los aislantes o dieléctricos, los
electrones se mueven con dificultad como por ejemplo la madera, el vidrio, el papel, la lana, etc.
Por ser la carga del electrón la más pequeña de la naturaleza en estado libre, ésta se toma
como unidad natural de carga o carga elemental, (e-). La del protón es igual pero positiva.
Del hecho de que la carga dependa del defecto o exceso de electrones se deducen sus pro-
piedades:

– La carga está siempre cuantizada, es decir, cualquier carga Q, es múltiplo entero,


n, de la carga elemental, e:

-19
e- = 1,6 . 10 culombios

Q = n · e-

La unidad de carga en el Sistema Internacional es el culombio (C)

1 culombio
= 6,25 . 10 electrones (e-)
8

-19
1,6 . 10 culombios
electrón

– En los procesos de electrización la carga se conserva, ya que los electrones aban-


donan un material frotado para pasar a otro.

6
-Pág.58-
U . D . 11 . - E LECTRICIDAD Y MAGNETISMO

2. CORRIENTE ELÉCTRICA
La electricidad es una corriente de electrones que fluye por un circuito. Para ello debe cum-
plirse que las cargas tengan energía, es decir, que estén sometidas a una diferencia de potencial
o, en otras palabras, a un campo eléctrico y que fluyan por un conductor.

2.1. DIFERENCIA DE POTENCIAL E INTENSIDAD


En la naturaleza, las cargas se desplazan espontáneamente hasta alcanzar el equilibrio elec-
trostático. Benjamín Franklin postulaba que la electricidad era un flujo eléctrico que iba de los
cuerpos que la poseían en exceso (con carga positiva) hasta los que carecían de ella (con carga
negativa). Esto es lo que sucede de forma violenta en las nubes.
Los electrones circulan si entre los extremos del circuito hay una diferencia de potencial, lla-
mada también tensión o voltaje (V). Se mide en voltios y se representa por V.
Si se pretende que no cese la corriente, hay que establecer de forma permanente una dife-
rencia de potencial o tensión. Esto se consigue mediante un generador eléctrico, que es un apa-
rato que transforma cualquier forma de energía en energía eléctrica.
Cada generador posee una característica llamada fuerza electromotríz, ¡, o energía que
puede comunicar a una unidad de carga, y que equivale al trabajo, W, realizado sobre ella a lo
largo del circuito eléctrico.

Como la diferencia de potencial o tensión establecida entre los extremos del circuito, la fuer-
za electromotríz se mide en voltios.
Los tipos de generadores empleados en la actualidad son de tres tipos: químicos (pilas y
baterías cargadas con sustancias químicas que reaccionan produciendo electrones), electromag-
néticos (transforman la energía mecánica en eléctrica como las dinamos o los alternadores) y
fotovoltaicos (transforman la energía luminosa en eléctrica).
Existen dos tipos de corriente, alterna y continua. En la corriente continua, los electrones
salen del polo negativo, recorren el circuito y retornan al polo positivo (las pilas). En la corrien-
te alterna, los polos del generador cambian de negativo a positivo normalmente cincuenta veces
por segundo. Su ventaja frente a la corriente continua es la facilidad de transporte a grandes dis-
tancias.

7
-Pág.59-
C I E N C I A S D E L A N AT U R A L E Z A

2.2. TRANSFORMACIONES ENERGÉTICAS EN UN CIRCUITO


ELÉCTRICO
El generador y el conductor forman los dos elementos esenciales de un circuito eléctrico o
camino recorrido por las cargas. La parte del generador por donde salen los electrones se deno-
mina polo negativo y la parte por donde retornan, polo positivo.
Los circuitos eléctricos se representan con símbolos que simplifican mucho el trabajo:

La corriente eléctrica en un circuito puede ser suministrada por uno o varios generadores. Si
se desea aumentar la tensión eléctrica, los generadores deben conectarse en serie y, si lo que se
quiere es aumentar la potencia sin variar la tensión, deben conectarse en paralelo.
En la conexión en serie, se conectan unas a continuación de otras, uniendo el polo positivo
del primero con el negativo del segundo, el positivo del segundo, con el negativo del tercero,
etc. El voltaje total, es la suma de los voltajes de cada generador.
En la conexión en paralelo, todos los polos positivos van unidos entre sí y a un punto común
y a su vez los negativos a otro punto común. El voltaje total, en este caso, es común a todos los
generadores.

Conexión de tres pilas


iguales en serie
VT = V1 + V2 + V3 = 9V
Conexión de tres pilas
iguales en paralelo
VT = V1 + V2 + V3 = 3V

La intensidad de corriente I, es la carga q que atraviesa la sección recta de un conductor


en la unidad de tiempo t.
I=q/t

8
-Pág.60-
U . D . 11 . - E LECTRICIDAD Y MAGNETISMO

En el SI, la unidad de intensidad de corriente es el amperio (A). La intensidad de un ampe-


rio, transporta la carga de 1 culombio en el tiempo de 1 segundo.
Para medir la intensidad de corriente se utiliza un instrumento de medida llamado ampe-
rímetro, que se instala en serie con los otros elementos del circuito y en cualquier punto del
mismo.
Una de las leyes más importantes de la electricidad fue descubierta por el físico alemán
Georg Simon Ohm en 1827 y dice que la tensión, V, entre dos puntos de un conductor es direc-
tamente proporcional a la intensidad, I. A la constante de proporcionalidad la llamó resisten-
cia, R.

En su honor, la unidad de resistencia, recibió el nombre de ohmio (Ÿ). Un conductor ofre-


ce la resistencia de un ohmio si al aplicarle la tensión de un voltio pasa la intensidad de un
amperio. (11 = 1V/1A).
Todo conductor, por bueno que sea, presenta alguna oposición al paso de la corriente eléc-
trica debido al choque de los electrones con los átomos del metal y entre sí. La resistencia de
un conductor R, es directamente proporcional a su longitud, l, e inversamente proporcional al
área de su sección S.
l
R= ·
S

La constante  , se llama resistividad y es característica del material con que está hecho el
conductor.
El calentamiento que produce la corriente eléctrica sobre el conductor se aprovecha en las
bombillas, mediante un conductor muy fino, el filamento, que alcanza un rojo tan vivo que se
vuelve blanco y emite luz brillante. En las planchas y estufas eléctricas, se utiliza un conductor
más grueso para que pueda emitir mucho calor.
La potencia de un aparato, nos indica la energía que gasta en un cierto tiempo.

P=E/t

La unidad de potencia es el watio (W).


Un watio de potencia es consumir un julio de energía en un segundo (1W = 1J/1s).

9
-Pág.61-
C I E N C I A S D E L A N AT U R A L E Z A

IMANES. EFECTO DE UNA CORRIENTE ELÉCTRICA


3. SOBRE UNAAGUJA IMANTADA
La interacción magnética fue observada en Magnesia (Asia Menor) por primera vez: un
mineral (llamado hoy magnetita), un óxido de hierro (Fe3O4), atraía clavos de hierro y otros
objetos de este metal.
La atracción es mayor en los extremos, que reciben el nombre de polos (norte y sur); la zona
equidistante, es la zona neutra, y en ella las acciones de los polos se neutralizan. Si se aproxi-
man polos del mismo signo, se repelen y si son de distinto signo, se atraen.
El nombre de polos, viene del nombre empleado en la brújula, invento que ya en el siglo
XIII se empleaba para la orientación.
Los imanes crean campos de fuerzas llamados magnéticos. El campo magnético se repre-
sentan mediante líneas de fuerza, que en este caso son cerradas porque no tienen ni origen ni
fin.
Como resultado del movimiento de los electrones, cada átomo se comporta como un imán
elemental o un dipolo magnético atómico. En la mayor parte de las sustancias estos dipolos
están orientados al azar. Sin embargo, en las ferromagnéticas, existen zonas pequeñísimas, los
dominios magnéticos, en los que los dipolos poseen la misma orientación. Cuando las sustan-
cias ferromagnéticas están sometidas a la acción de un campo magnético exterior, los dominios
magnéticos se orientan en el sentido de ese campo y dichas sustancias se comportan como ima-
nes.

3.1. ESTUDIO CUALITATIVO DE LA INDUCCIÓN


ELECTROMAGNÉTICA
Si se conecta con cables los bornes de una pila a una bombilla y a un interruptor, se forma
un circuito eléctrico. Cuando el interruptor está cerrado, la bombilla se enciende. Si se acerca
una brújula mientras que el circuito lo cerramos, se verá como la aguja se desvia. Esto nos indi-
ca que las corrientes eléctricas , al igual que los imanes, producen campos magnéticos.
Si se enrolla un hilo de cobre alrededor de una barra de hierro y hacemos circular la corrien-
te a través del hilo, la barra de hierro se imanta, pero en cuanto la corriente eléctrica deja de cir-
cular, la barra de hierro deja de ser imán. Si en lugar de hierro, utilizamos acero, los efectos
persisten.
Esta propiedad del hierro de convertirse en un imán cuando circula la corriente eléctrica a
su alrededor es el fundamento de los electroimanes. Se utilizan en muchos mecanismos: alar-
mas, timbres, grúas, etc.

10
-Pág.62-
U . D . 11 . - E LECTRICIDAD Y MAGNETISMO

– La grúa electroimán puede subir grandes bloques de metal o, simplemente, puede


separar objetos de hierro de otros objetos. Para soltarlos hay que cortar la electricidad que cir-
cula alrededor del electroimán.

– El timbre eléctrico es un electroimán. Cuando se pulsa, el interruptor hace circular la


corriente alrededor de un núcleo de hierro, se imanta y atrae un bloque de metal que golpea la
campana; el circuito se abre, se interrumpe la electricidad en el electroimán y la pieza de metal
retorna a su posición inicial, vuelve a cerrarse el circuito y comienza de nuevo el ciclo.

En los motores eléctricos se aprovechan también los efectos que se producen entre las
corrientes y los imanes. Explicado de forma sencilla, un motor consiste en una bobina de cobre
colocada entre los polos de un imán por la que circula la corriente. Al circular la corriente por
el conductor, se crea un campo magnético en la bobina; los polos del imán atraen a los polos
contrarios del campo magnético que se crea en la bobina, y la hace girar. En las centrales eléc-
tricas, la electricidad se produce al hacer girar una bobina conductora entre los polos de un
imán.

NORMAS DE SEGURIDAD EN LA UTILIZACIÓN


4. DE LA ELECTRICIDAD
La electricidad es peligrosa (es una forma de energía), se deben respetar las normas básicas
de seguridad. Las viviendas suelen tener unas protecciones, los diferenciales, que cortan la
corriente cuando surge alguna anomalía en los circuitos o aparatos de la casa.
Los fusibles son otro sistema para proteger los aparatos eléctricos. Un fusible no es más que
un conductor muy débil, más débil que cualquier otra parte del circuito. Si por cualquier causa
se produce una sobrecarga de electricidad, lo primero que se quemará será el fusible, interrum-
piéndose la corriente y se salvará de averías el resto del circuito.
Pero cualquier sistema de protección es inútil si no se cumplen las reglas elementales de
seguridad:
– No desenchufar nunca los aparatos eléctricos tirando del cable.
– No utilizar cables que tengan la protección plástica rota o quebradiza.
– Algunos electrodomésticos, como los televisores, aun estando apagados, pueden pro-
vocar descargas eléctricas si manipulamos en su interior.
– No conectar varios aparatos a la misma toma de corriente, el cable puede recalentarse.
– No utilizar los aparatos eléctricos con las manos mojadas ni con los pies descalzos.

11
-Pág.63-
C I E N C I A S D E L A N AT U R A L E Z A

– Todos los electrodomésticos deben tener una toma de tierra que impida que pase elec-
tricidad a la carcasa del aparato, pues así es desviada por un tercer cable al suelo.
– Si se toca un cable electrificado con la palma de la mano, el movimiento reflejo con-
trae los músculos y no se pueden despegar las manos del cable, por eso es una buena
costumbre tocar los interruptores y cables en dudoso estado, no con la yema de los
dedos, sino con el dorso de la mano, pues el movimiento reflejo separará del punto de
descarga eléctrica.
– No tocar los postes de la luz, ni ninguna puerta ni estructura metálica donde se indi-
que que existen altos voltajes, puede ser muy peligroso.

12
-Pág.64-
U . D . 11 . - E LECTRICIDAD Y MAGNETISMO

RESUMEN
– Los electrones ocupan la periferia de los átomos y son los responsables de los fenó-
menos de la electricidad.
– Cuando se acercan cargas de distinto signo se atraen; cuando las cargas son del mismo
signo, se repelen.
– Cada imán tiene dos polos, norte y sur. Cuando se acercan dos imanes con el mismo
polo, se repelen, si se acercan con polos distintos se atraen.
– Si se introduce una barra de hierro en una bobina por la que circula corriente, el hie-
rro se imanta, pero los efectos sólo duran lo que el paso de la corriente.
– La electricidad es una corriente de electrones que circula por un circuito. Los ele-
mentos que dejan pasar bien la corriente eléctrica se llaman conductores; los que no
la dejan pasar son aislantes.
– Si en un circuito la corriente de electrones circula siempre en un mismo sentido, la
corriente es continua. Si cambia de sentido, la corriente es alterna.
– La diferencia de potencial representa la energía que mueve los electrones. Se mide en
voltios.
– La resistencia nos indica la oposición que presentan los materiales al paso de la
corriente. Se mide en ohmios.
– La intensidad de una corriente es la cantidad de carga eléctrica que pasa por una parte
del circuito en la unidad de tiempo. Se mide en amperios.
– En las centrales eléctricas, la electricidad se produce al hacer girar una bobina con-
ductora entre los polos de un imán, mientras que el motor eléctrico se basa en que una
bobina, por la que circula la corriente eléctrica, gira al colocarla entre los polos de un
imán.
– La brújula es un imán con forma de aguja que es atraído por el polo norte del campo
magnético terrestre.
– Para que circule la corriente por un circuito debe estar cerrado.
– Es sumamente peligroso utilizar los aparatos eléctricos con el cuerpo mojado.
– En la instalación de tu casa, por ejemplo un enchufe, no tiene sentido hablar de polos
porque la corriente es alterna.
– El sentido de la corriente continua en un circuito es del polo negativo al positivo.
– La ley de Ohm, nos relaciona las magnitudes eléctricas fundamentales V = I · R

13
-Pág.65-
C I E N C I A S D E L A N AT U R A L E Z A

EJERCICIOS DE AUTOCOMPROBACIÓN

1. La carga que tienen los electrones es:

A. Negativa. B. Positiva.

C. No tienen carga. D. Cambia por efecto del magnetismo.

2. Un átomo que ha ganado un electrón es:

A. Neutro. B. Negativo.

C. Positivo. D. Protón.

3. Los cuerpos se pueden electrizar:

A. Por calor. B. Por fricción.

C. Por atracción. D. Por repulsión.

4. En el mecanismo de un timbre aparece:

A. Un imán. B. Un electroimán con el núcleo de hierro.

C. Un electroimán con el núcleo de acero. D. Un electroimán sin núcleo.

5. La corriente eléctrica se debe a la circulación de:

A. Electrones. B. Protones.

C. Iones. D. Átomos.

6. Un material que no deja pasar la corriente eléctrica es:

A. Un aislante. B. Un fusible.

C. Un interruptor. D. Un conector.

7. En un circuito de corriente continua, la electricidad circula por el cable:

A. Del polo negativo al positivo. B. Del polo positivo al negativo.

C. Cambia de sentido continuamente. D. Ninguna es correcta.

14
-Pág.66-
U . D . 11 . - E LECTRICIDAD Y MAGNETISMO

8. El amperio es la unidad de medida de:

A. La resistencia. B. La intensidad.

C. El voltaje. D. El trabajo.

9. Si la resistencia es constante y aumenta el voltaje, la intensidad:

A. Disminuye. B. Aumenta.

C. Permanece constante. D. Varía según los casos.

10. ¿Cómo se crea corriente en una bobina?:

A. Al girar la bobina entre los polos de un imán.

B. Si se coloca una bobina en el interior de un fusible.

C. Si se coloca un imán en el interior de la bobina.

D. No se crea corriente.

15
-Pág.67-
C I E N C I A S D E L A N AT U R A L E Z A

RESPUESTAS A LOS EJERCICIOS

11. A

12. B

13. B

14. B

15. A

16. A

17. A

18. B

19. B

10. A

16
-Pág.68-
portada TROPA 19/3/07 19:51 Página 1

FUERZAS ARMADAS
PROFESIONALES
CURSO DE APOYO
A LA PREPARACIÓN
DE LAS PRUEBAS DE ACCESO
A UNA RELACIÓN DE SERVICIOS
DE CARÁCTER PERMANENTE

CIENCIAS SOCIALES
1ª parte
Unidades didácticas 1, 2 y 3

DIGEREM

MINISTERIO
DE DEFENSA
FUERZAS ARMADAS SUBDIRECCIîN GENERAL
DE TROPA Y MARINERIA
PROFESIONAL
PROFESIONALES
CURSO DE APOYO
A LA PREPARACIÓN
DE LAS PRUEBAS DE ACCESO
A UNA RELACIÓN DE SERVICIOS
DE CARÁCTER PERMANENTE

CIENCIAS SOCIALES
1ª parte
Unidades didácticas 1, 2 y 3

-Pág.1-
La Ley 8/2006 de Tropa y Marinería, en su artículo 16,1, establece que “la formación
en las Fuerzas Armadas garantizará que los militares profesionales de tropa y
marinería puedan adquirir, actualizar o ampliar sus conocimientos para un mayor
desarrollo personal y profesional”. En cumplimiento de este mandato, el Ministerio
de Defensa edita el presente material didáctico para facilitar a los militares
profesionales de tropa y marinería, alumnos de los cursos de formación
presencial que se imparten a través de la Dirección General de Reclutamiento y
Enseñanza Militar, los apoyos necesarios para preparación de dichos cursos, que
permitirán, siempre que superen las pruebas correspondientes, la obtención de la
titulación de graduado en Educación Secundaria, acreditación para el acceso a
los ciclos formativos de la Formación Profesional de grado medio o de grado
superior, acceso a las Escalas de Suboficiales, Tropa Permanente, Guardia Civil
y Policía Nacional.

CATÁLOGO GENERAL DE PUBLICACIONES


http://www.060.es

Edita:

© Autor y editor
NIPO: 076-10-204-9 NIPO: 076-10-205-4 (edición en línea)
Depósito Legal: M-32363-2009
Diseño y programación: cimapress
Tirada: 1300 ejemplares
Fecha de edición: septiembre, 2010

Prohibida la reproducción total o parcial de esta obra, por cualquier medio sin autorización escrita del editor
CIENCIAS SOCIALES
1ª parte

SUMARIO

Unidad didáctica Pág.

1. MEDIO AMBIENTE Y CONOCIMIENTO GEOGRÁFICO 5

2. LA POBLACIÓN Y EL ESPACIO URBANO 37

3. LA ACTIVIDAD HUMANA Y EL ESPACIO GEOGRÁFICO 73

-Pág.3-
U. D. 1 . - M E D I O A M B I E N T E Y C O N O C I M I E N TO G E O G R Á F I C O

ÍNDICE
Pag.

OBJETIVOS. . . . . . . . . . . . . . . . . . . . . . . . . . . . . . . . . . . . . . . . . . . . . . . . . . . . . . . . . . . . . . . . . 2
INTRODUCCIÓN . . . . . . . . . . . . . . . . . . . . . . . . . . . . . . . . . . . . . . . . . . . . . . . . . . . . . . . . . . . 3
MAPA CONCEPTUAL . . . . . . . . . . . . . . . . . . . . . . . . . . . . . . . . . . . . . . . . . . . . . . . . . . . . . . . 4
DESARROLLO DE CONTENIDOS . . . . . . . . . . . . . . . . . . . . . . . . . . . . . . . . . . . . . . . . . . . 5
1. INICIACIÓN A LOS MÉTODOS GEOGRÁFICOS . . . . . . . . . . . . . . . . . . . . . . 5
1.1. LA TIERRA, SU FORMA Y LA PERCEPCIÓN DE LA MISMA . . . . . . . . . . . . 5
1.2. LA REPRESENTACIÓN DE LA TIERRA. LA CARTOGRAFÍA. . . . . . . . . . . . . 6
1.3. TÉCNICAS Y SISTEMAS DE REPRESENTACIÓN ESPACIAL . . . . . . . . . . . . . 7
2. LAS FUENTES DE INFORMACIÓN GEOGRÁFICA
Y SU UTILIZACIÓN . . . . . . . . . . . . . . . . . . . . . . . . . . . . . . . . . . . . . . . . . . . . . . . . . . 8
3. EL MEDIO AMBIENTE Y SU CONSERVACIÓN . . . . . . . . . . . . . . . . . . . . . . . 10
3.1. EL PLANETA TIERRA: SUS RASGOS FÍSICOS FUNDAMENTALES . . . . . . 10
3.1.1. LA TIERRA COMO ASTRO . . . . . . . . . . . . . . . . . . . . . . . . . . . . . . . . . . . 10
3.1.2. LA ATMÓSFERA . . . . . . . . . . . . . . . . . . . . . . . . . . . . . . . . . . . . . . . . . . . . 12
3.1.3. LA ESTRUCTURA INTERNA DE LA TIERRA . . . . . . . . . . . . . . . . . . . 12
3.1.4. LA FORMACIÓN DEL RELIEVE . . . . . . . . . . . . . . . . . . . . . . . . . . . . . . 13
3.1.5. LA HIDROSFERA . . . . . . . . . . . . . . . . . . . . . . . . . . . . . . . . . . . . . . . . . . . 16
3.1.6. EL CLIMA . . . . . . . . . . . . . . . . . . . . . . . . . . . . . . . . . . . . . . . . . . . . . . . . . 17
3.1.7. LOS TIPOS DE CLIMAS . . . . . . . . . . . . . . . . . . . . . . . . . . . . . . . . . . . . . . 20
3.2. LOS PAISAJES NATURALES . . . . . . . . . . . . . . . . . . . . . . . . . . . . . . . . . . . . . . . 21
3.2.1. PAISAJES CÁLIDOS . . . . . . . . . . . . . . . . . . . . . . . . . . . . . . . . . . . . . . . . . 21
3.2.2. PAISAJES TEMPLADOS . . . . . . . . . . . . . . . . . . . . . . . . . . . . . . . . . . . . . . 22
3.2.3. PAISAJES FRÍOS . . . . . . . . . . . . . . . . . . . . . . . . . . . . . . . . . . . . . . . . . . . . 22
3.3. EL MEDIO Y EL SER HUMANO . . . . . . . . . . . . . . . . . . . . . . . . . . . . . . . . . . . . 23
3.3.1. LA INFLUENCIA DEL MEDIO SOBRE EL HOMBRE . . . . . . . . . . . . . 23
3.3.2. LA INFLUENCIA DEL HOMBRE SOBRE EL MEDIO . . . . . . . . . . . . . 24
A. USO Y CONSUMO DEL AGUA . . . . . . . . . . . . . . . . . . . . . . . . . . . . . 24
B. LA DEGRADACIÓN DEL MEDIO Y EL CAMBIO CLIMÁTICO . . . 25
RESUMEN . . . . . . . . . . . . . . . . . . . . . . . . . . . . . . . . . . . . . . . . . . . . . . . . . . . . . . . . . . . . . . . . . 26
EJERCICIOS DE AUTOCOMPROBACIÓN . . . . . . . . . . . . . . . . . . . . . . . . . . . . . . . . . . 29
RESPUESTAS A LOS EJERCICIOS . . . . . . . . . . . . . . . . . . . . . . . . . . . . . . . . . . . . . . . . . 31

-Pág.5-
C I E N C I A S S O C I A L E S . G E O G R A F Í A E H I S TO R I A

OBJETIVOS
Al finalizar el estudio de ésta Unidad Didáctica, el alumno será capaz de:

• Asimilar el concepto general de Geografía como ciencia.

• Analizar y comprender las técnicas y los métodos de representación y per-


cepción del espacio.

• Alcanzar un nivel básico en la interpretación de mapas y planos y entienda


su concepto y utilidad, familiarizándose con su uso.

• Comprender la realidad de la Tierra como planeta, sus rasgos y característi-


cas astronómicas y su estructura interna, superficial y atmosférica.

• Entender la interacción de los diferentes aspectos que influyen en la confor-


mación física del espacio terrestre: relieve, hidrografía, clima y vegetación.

• Analizar y comprender la relación del hombre con el medio físico, obser-


vando las consecuencias para la vida humana de los factores físicos terres-
tres y la acción del ser humano sobre el medio físico.

• Adquirir conciencia de la trascendencia de la degradación de la naturaleza,


como consecuencia de la acción humana, e interiorizar actitudes positivas
hacia la conservación del medio ambiente

-Pág.6-
U. D. 1 . - M E D I O A M B I E N T E Y C O N O C I M I E N TO G E O G R Á F I C O

INTRODUCCIÓN
G eografía (Geo=Tierra; Graphía=tratado, descripción), es la ciencia que se
ocupa del estudio de la Tierra. Son muchos los aspectos que dicho estudio
abarca, de ahí que se pueda desglosar el mismo en diversas disciplinas, siendo las
principales:

Geografía Física, que incluye a su vez diversas disciplinas dado lo denso de este
aspecto que abarca la Geología, Geomorfología, Climatología, Edafología, etc.
En definitiva se trata del estudio de la Tierra como realidad física que forma parte
del Sistema Solar, tiene una estructura interna, una atmósfera, una corteza con un
relieve dinámico al que se adaptan las aguas oceánicas y continentales y una intrin-
cada red de relaciones entre estos aspectos que genera la formación de diversos espa-
cios naturales con su relieve, su distribución de aguas, su clima y su vegetación.

Geografía Humana. Esta se ocupa del estudio del hombre como conjunto. La pobla-
ción, su distribución espacial, su crecimiento, los movimientos migratorios, su repar-
to geopolítico, etc.

Geografía Económica, que se ocupa del estudio de las interacción del hombre en el
medio, analizando la explotación de los recursos naturales, su transformación y
comercialización.

En este primer tema nos acercamos a la primera de las disciplinas citadas. Veremos
en primer término las formas de representación de la realidad geográfica y las herra-
mientas que se utilizan para el estudio de los diversos aspectos de los que trata la
Geografía. A continuación, estudiaremos la Tierra como planeta, situándolo en el
conjunto del Sistema Solar y analizando su estructura para completar la visión de su
realidad física. Posteriormente, observaremos las relaciones entre el medio físico y
el hombre valorando las consecuencias de la acción del hombre sobre el medio y las
de las fuerzas naturales sobre el desarrollo humano.

-Pág.7-
C I E N C I A S S O C I A L E S . G E O G R A F Í A E H I S TO R I A

M A PA C O N C E P T UA L

MÉTODOS GEOGRÁFICOS

FUENTES DE
INFORMACIÓN GEOGRÁFICA
MEDIO AMBIENTE
Y CONOCIMIENTO
GEOGRÁFICO
CONSERVACIÓN DEL
MEDIO AMBIENTE

INTERACCIÓN: EL HOMBRE
Y EL MEDIO GEOGRÁFICO

-Pág.8-
U. D. 1 . - M E D I O A M B I E N T E Y C O N O C I M I E N TO G E O G R Á F I C O

1. INICIACIÓN A LOS MÉTODOS GEOGRÁFICOS

1.1. LA TIERRA, SU FORMA Y LA PERCEPCIÓN DE LA MISMA


Frente a los escasamente 100.000 años que nuestra especie, el Homo Sapiens Sapiens,
lleva en su superficie, la edad de la Tierra se nos antoja abrumadora: 4.500 millones de
años han transcurrido desde que se inició su formación, a la vez que el sistema del que
el Sol es centro, hasta hoy. El ser humano, desde su original limitación al medio terrestre
superficial, se ha ido aproximando a lo largo de su historia al conocimiento del lugar del
que formaba parte el suelo sobre el que le sustentaban sus pies.

El conocimiento popular, siempre tentado a dar más crédito a lo fantástico que a lo prag-
mático, imaginaba la Tierra como una gran superficie plana situada entre las tinieblas en
medio de un universo creado en función de un total antropocentrismo. Sin embargo, los
sabios griegos (PLATÓN, ARISTÓTELES, TALES DE MILETO) demostraron conceptualmente y
defendieron la esfericidad del planeta e, incluso, llegaron a precisar la longitud del meri-
diano terrestre con un mínimo error (ERATÓSTENES DE CIRENE).

La tradición judeo-cristiana, creacionista y antropocentrista, perpetuó el concepto cos-


mológico geocéntrico (PTOLOMEO) durante la Edad Media llevando al cadalso o al borde del
mismo a cuantos defendieron opciones distintas. Así, las tesis heliocéntricas de GALILEO,
COPÉRNICO, KEPLER o TYCHO BRAHE fueron alcanzando aceptación sólo como consecuen-
cia de los profundos cambios conceptuales de la cultura europea a raíz del Renacimiento.

Precisamente, los descubrimientos geográficos iniciados durante dicha etapa irán apor-
tando nuevos conocimientos sobre la superficie terrestre culminando con la confirmación
material de la esfericidad de la Tierra tras la circunvalación iniciada por MAGALLANES y
completada por JUAN SEBASTIÁN ELCANO (1522).

Los estudios y expediciones posteriores, del siglo XVIII al XX, permitieron precisar la
forma y dimensiones del planeta, culminando con la constatación técnica y visual que los
medios contemporáneos, encabezados por los viajes espaciales, permiten.

Para una mejor aprehensión de la esfericidad del planeta y localización en su superficie,


quedó definido un sistema basado en la sucesión de líneas imaginarias que tienen como
referentes los polos, es decir, los extremos de la esfera que quedan unidos por el eje de rota-
ción, y el Ecuador, el cinturón de mayor longitud que circunda el planeta y es perpendicu-
lar al eje de rotación.

TAMadrid
5
-Pág.9-
C I E N C I A S S O C I A L E S . G E O G R A F Í A E H I S TO R I A

Con tales referentes se definen:

• Los paralelos, círculos paralelos al Ecuador.

• Los meridianos, círculos que pasan por los polos y son perpendiculares al Ecuador.

Dichos círculos imaginarios definen un entramado de líneas perpendiculares que per-


miten localizar cualquier punto de la superficie estableciendo sus coordenadas, es decir, su
posición, medida en grados, en el paralelo y el meridiano que se cruzan en dicho punto. El
mismo queda así definido por:

– La latitud, distancia en grados desde el Ecuador a dicho punto. La medida se esta-


blece, pues, desde el Ecuador, hacia el Norte y hacia el Sur, de modo que una lati-
tud se expresará como una cantidad x comprendida entre 0 y 90o.

– La longitud, distancia en grados hasta el punto desde el meridiano 0: convencio-


nalmente el meridiano que pasa por el observatorio astronómico de Greenwich en el
Reino Unido. La medida se establece en dirección Este u Oeste como una cantidad
x comprendida entre 0 y 180o.

1.2. LA REPRESENTACIÓN DE LA TIERRA. LA CARTOGRAFÍA


Desde la antigüedad, los intereses económicos, estratégicos o de ocio han llevado a la
necesidad de representar el espacio geográfico. La Cartografía es, precisamente, la repre-
sentación gráfica de la Tierra por medio de planos o mapas.

Partiendo de las básicas representaciones intuitivas conservadas en diferentes superfi-


cies, las primeras representaciones de amplios espacios llegarán con los griegos y roma-
nos, algunas más esquemáticas y referenciales que realistas.

En la Edad Media, las necesidades del tráfico marítimo otorgó un especial protagonis-
mo a las cartas de navegación o portulanos realizados por los pilotos, por lo que la defini-
ción de las costas constituía el aspecto fundamental de las representaciones.

A partir del Renacimiento, los descubrimientos geográficos permitieron ampliar la


visión del mundo en su globalidad y la invención de la imprenta permitió multiplicar la
reproducción cartográfica. Esta auténtica revolución se vio acompañada de la aparición de
escuelas geográficas

Posteriormente, los avances científicos, culminando con la fotografía aérea y los satéli-
tes artificiales, han permitido representaciones cada vez más precisas.

6 TAMadrid

-Pág.10-
U. D. 1 . - M E D I O A M B I E N T E Y C O N O C I M I E N TO G E O G R Á F I C O

1.3. TÉCNICAS Y SISTEMAS DE REPRESENTACIÓN ESPACIAL


Los planos y mapas representan la realidad de una superficie de más de 425.000.000
km2 en un pedazo de papel de unos dm2 . Necesariamente, pues, se debe acudir a una serie
de convenciones que podemos agrupar en tres aspectos: la clase de proyección, la escala de
representación, la orientación y los símbolos cartográficos.

La Proyección
Excepción hecha de las representaciones sobre esferas que permiten, obviamente, la
adaptación de la representación a la realidad de la curvada superficie terrestre, cualquier
otra representación, realizada sobre el plano, experimentará necesariamente una distorsión,
mayor cuanto más grande sea la superficie representada. Los diferentes tipos de proyeccio-
nes pretenden aproximar lo más posible la representación sobre el plano a la realidad.
Existen dos tipos de proyecciones, las equivalentes, que respetan el tamaño pero no las
distancias, y las conformes, que mantienen los ángulos y las formas reales pero no los
tamaños.
Por otro lado, en función del procedimiento empleado para realizarla, se pueden distin-
guir las proyecciones planas, cilíndricas y cónicas.

La escala
Es la relación existente entre las distancias reales y las reflejadas en el mapa. Así, si un
centímetro del mapa representa un kilómetro (100.000 cm) de la realidad diremos que la
escala empleada es 1:100.000. La escala aparece expresada en el plano de forma numérica,
tal como acabamos de citarla, o de forma gráfica, mediante una regla impresa en la que se
precisan las distancias equivalentes.

La orientación
En principio, la orientación utilizada en todas las proyecciones y representaciones es la
Norte-Sur (Norte, arriba; Sur, abajo), aunque en la historia haya existido alguna represen-
tación que prefirió la contraria. Sin embargo, el punto de referencia Norte no es exacta-
mente el mismo en todas las proyecciones, existiendo variaciones de, incluso, varios grados.
Además, si tenemos en cuenta la variabilidad del polo norte magnético y la necesaria vin-
culación de la orientación sobre el terreno a medios como la brújula, este aspecto adquiere
una especial trascendencia cuando se trata de establecer rumbos precisos, algo especial-
mente importante en la navegación marítima.

TAMadrid
7
-Pág.11-
C I E N C I A S S O C I A L E S . G E O G R A F Í A E H I S TO R I A

Hoy, pues, podemos definir a la Tierra como un geoide de revolución, es decir, una esfe-
ra irregular achatada en la zona de los polos y con una ligera disimetría entre sus hemisfe-
rios septentrional y meridional, además de con las irregularidades propias de una superficie
dominada por grandes accidentes de relieve.

Los símbolos cartográficos


La representación cartográfica es precisamente eso, representación, es decir, una apro-
ximación figurada a la realidad, fin para el cual se utilizan una serie de símbolos conven-
cionales que se refieren a aspectos tales como los trazos de las líneas, los colores, el tamaño
y tipo de las letras y otros símbolos.
Los trazos de las líneas, con diferentes colores y grosores permite representar curvas de
nivel (líneas que unen puntos de la misma altitud), carreteras, vías férreas, líneas de tendi-
do eléctrico, ríos, costas...
Los colores y sombras permiten representar áreas montañosas, zonas de diversa vegeta-
ción, áreas urbanas, etc.
El tamaño y tipo de las letras diferencia tipos o importancia de topónimos: ríos, cade-
nas montañosas, picos, ciudades, etc.
Por fin, diversos símbolos iconográficos permiten precisar diferentes elementos tales
como monumentos, edificaciones de diferente carácter, tipo de vegetación, etc.

2. LAS FUENTES DE INFORMACIÓN GEOGRÁFICA


Y SU UTILIZACIÓN

El estudio de la Geografía, entendida en su sentido más amplio, no se limita al análisis


de la cartografía, a la que acabamos de referirnos. Ésta permite acercarnos al conocimien-
to de la Tierra tal como es su relieve y los elementos que sobre el mismo se encuentran
(vegetación, núcleos urbanos, carreteras, etc.). La información geográfica tiene otros ele-
mentos o fuentes.

Mapas o planos conceptuales.—Sobre la base de una cartografía simplificada pueden


aportar una más visual información mapas en los que, olvidando el detalle, se prime algún
rasgo específico o incluso se deformen en función de dichos rasgos:

8 TAMadrid

-Pág.12-
U. D. 1 . - M E D I O A M B I E N T E Y C O N O C I M I E N TO G E O G R Á F I C O

Mapas climáticos
– Térmicos (con líneas isotermas o áreas de diferentes colores en función de las dis-
tintas temperaturas).
– De precipitación (con líneas isoyetas o áreas de colores dependiendo de los nive-
les de precipitación).
– De presiones y tiempo climatológico (con líneas isobaras, frentes, etc.).

Mapas políticos
– De Estados, incluyendo o no las circunscripciones administrativas de los mismos,
con el trazo de sus fronteras y la utilización de colores.
– De unidades geopolíticas.
– De resultados electorales, etc.

Mapas demográficos (de densidad, volumen de población, etc.).

Mapas económicos (con áreas de producción, discriminadores de volúmenes de pro-


ducción por áreas o países, con expresión de líneas de flujos comerciales, etc.).

Estadísticas.—Que aportan informaciones fundamentales para el análisis demográfico


y económico

Gráficos.—Representaciones de aspectos demográficos, económicos o incluso físicos:


Climogramas: representación de la sucesión mensual de temperaturas y precipitacio-
nes en un determinado lugar.
Hidrogramas: representación de la evolución mensual del caudal de los ríos.
Pirámides de población: representación de los volúmenes de población por grupos de
edad y sexos.
Gráficos de producción económica.

Por otro lado, para el análisis de los diferentes aspectos que la Geografía comporta,
resultan también útiles descripciones aportadas por fuentes literarias o informaciones grá-
ficas de carácter audiovisual tales como fotografías aéreas, gráficos de radar, fotografías de
infrarrojos, etc.

TAMadrid
9
-Pág.13-
C I E N C I A S S O C I A L E S . G E O G R A F Í A E H I S TO R I A

3. EL MEDIO AMBIENTE Y SU CONSERVACIÓN

3.1. EL PLANETA TIERRA:


SUS RASGOS FÍSICOS FUNDAMENTALES

3.1.1. LA TIERRA COMO ASTRO


La milenaria observación astronómica ha ido haciendo evolucionar nuestro conoci-
miento del planeta que habitamos acercándonos a su realidad. Desde los ancestrales y míti-
cos conceptos de una Tierra plana, hemos alcanzado un conocimiento bastante preciso
aunque no sea definitivo.

La Tierra, lejos de ocupar una posición céntrica en el Universo, es uno más de los ocho
planetas de un sistema que tiene como centro una modesta estrella, el Sol, que gira en los
suburbios de una más de los miles de millones de galaxias, la Vía Láctea, junto con las más
de 200.000 millones de estrellas que la forman.

La Tierra es el tercer planeta del Sistema y está situada a unos 150 millones de Km. del
Sol, entre las órbitas de Venus y Marte. Tiene un diámetro ecuatorial de 12.756 Km., una
densidad de 5,5 g/cm3 y una aceleración de la gravedad de 978,03 cm/s2

Traslación
Diámetro Masa Distancia al Sol Rotación
(días terrestres) Satélites
(Km) (Tierra = 1) (millones de km) (días terrestres)
(años terrestres)

MERCURIO 4.880 0,055 10,58 1,58 88 –


VENUS 12.104 0,815 108,2 243 224 –
TIERRA 12.756 1 149,6 10,1 365,26 1: Luna
MARTE 6.787 0,108 227,9 1,03 687 2: Fobos y Deimos
JÚPITER 142.800 317,9 778,3 0,41 11,86 17: Io, Europa, Calisto, Ganímedes...
SATURNO 120.000 95,14 1.427 0,42 29,46 20: Tetys, Dione, Titán, Japeto...
URANO 51.800 14,6 2.869 0,46 84,01 21: Miranda, Ariel, Umbriel...
NEPTUNO 49.500 17,2 4.496 0,67 164,8 8: Nereida, Tritón
1
La Unión Astronómica Internacional (UAI) decidió en la XXVI asamblea general (el 24-08-06), que
Plutón perdiera su condición de planeta por su pequeño tamaño. De esta forma, se reducen a ocho (8) los
planetas del Sistema Solar.
Los movimientos del planeta en el espacio tienen una enorme trascendencia sobre
aspectos tales como las temperaturas, zonas climáticas y sucesión de las estaciones.

10 TAMadrid

-Pág.14-
U. D. 1 . - M E D I O A M B I E N T E Y C O N O C I M I E N TO G E O G R Á F I C O

La rotación
La Tierra gira sobre sí misma cada 23 horas, 56 minutos y 4 segundos (movimiento de
rotación). Este movimiento define el día, oficialmente dividido en 24 horas. Si se dividen los
360o de la circunferencia terrestre en 24 partes de una hora resultan 24 husos horarios de 15o
cada uno. Estos husos permiten diferenciar la hora de los territorios incluidos en el mismo,
aunque las medidas económicas de algunos gobiernos les llevan a variar en más o en menos
la hora oficial respecto a la que corresponde a su huso (así, todos los países de la Unión
Europea, salvo el Reino Unido y Portugal y algunos territorios de otros países, como Cana-
rias, tienen la misma hora oficial aunque estén situados en tres husos horarios distintos).

La traslación
La Tierra se traslada alrededor del Sol a una velocidad de 29,8 Km/s tardando un total
de 365,242 días en dar una vuelta completa. La órbita que describe es elíptica con una
pequeña excentricidad que la hace casi circular. El plano sobre el que gira alrededor del Sol
es denominado eclíptica. Pero la Tierra no gira sobre un eje perpendicular a dicho plano
sino que éste tiene una inclinación de 23o 27' sobre la perpendicular a la eclíptica.
Precisamente, esa inclinación explica los cambios estacionales ya que al variar la incli-
nación de la superficie terrestre expuesta a los rayos solares, éstos inciden con mayor o
menor oblicuidad variando así el grado de insolación y consiguientemente el nivel de las
temperaturas. Esta circunstancia define la sucesión de solsticios y equinoccios:
El 21 de marzo y el 23 de septiembre los rayos solares inciden perpendicularmente sobre
el Ecuador donde, ese día, la noche dura lo mismo que el día: equinoccio, marcando el ini-
cio, respectivamente, de la primavera y el otoño del hemisferio norte y del otoño y la pri-
mavera del hemisferio sur.
El 21 de junio los rayos solares inciden perpendicularmente sobre el Trópico de Cáncer
(latitud máxima a la que se produce este fenómeno de perpendicularidad) y la esfera solar
se mantiene visible permanentemente en los lugares situados al norte del Círculo Polar
Ártico y permanentemente oculto en los situados al sur del Círculo Polar Antártico. Es el
solsticio de verano del Hemisferio Norte y el solsticio de invierno del Hemisferio Sur.
El 21 de diciembre los rayos solares inciden perpendicularmente sobre el Trópico de
Capricornio produciéndose los fenómenos opuestos a los del solsticio anterior.
Como consecuencia, además de la sucesión de las estaciones, se definen las llamadas
zonas climáticas:
– Zonas polares, extremadamente frías, por encima del Círculo Polar Ártico y por
debajo del Círculo Polar Antártico.

TAMadrid
11
-Pág.15-
C I E N C I A S S O C I A L E S . G E O G R A F Í A E H I S TO R I A

– Zonas medias, templadas y moderadas con estaciones frías y cálidas, entre los Tró-
picos y los Círculos Polares.
– Zonas cálidas, tropicales y ecuatoriales, con clima cálido, entre los dos trópicos.

3.1.2. LA ATMÓSFERA

La atmósfera es la capa gaseosa que envuelve al planeta. Los gases que la forman
constituyen lo que llamamos aire, que en los primeros 80 Km esta compuesta, de forma
esencialmente homogénea, por un 78% de Nitrógeno, un 20,9% de Oxígeno, 1% de diver-
sos gases nobles, 0,03% de CO2 y vapor de agua en proporciones variables. Además, en el
aire se encuentran en suspensión otros elementos sólidos (polvo de diverso origen, polen) y
líquidos.

Desde la superficie podemos diferenciar distintas capas:

Troposfera.—Es la capa en la que se producen los fenómenos meteorológicos. Tiene un


grosor variable (media 12 Km). En ella, con la altura, van disminuyendo la temperatura (a
unos 6,5o C por Km.), presión y densidad (así a 11 Km. la presión desciende a los 175 mm
y la temperatura llega a los -56o C).

Estratosfera.—Se llama así porque los gases se disponen, según su densidad, en capas o
estratos horizontales, dado que no hay en ella corrientes verticales. Es una zona de equili-
brio dinámico (ausencia de corrientes) y térmico (temperatura constante). Carece de vapor
de agua y de CO2, el Oxígeno está muy enrarecido; pero sí hay Hidrógeno y Helio.
Hacia los 30 Km. se encuentra la capa de Ozono, que absorbe gran parte de las radia-
ciones solares ultravioleta que, de otro modo, perjudicarían seriamente a la vida en la
superficie.

Mesosfera.—Entre 50 y 80 Km. En ella la temperatura va disminuyendo paulatinamente.

Ionosfera o Termosfera.—Los gases que en ella se encuentra están ionizados (de ahí su
nombre) debido a las radiaciones ultravioleta y rayos cósmicos que llegan del Sol, lo que
también hace que la temperatura vaya aumentando.

Exosfera.—Por encima de la ionosfera la atmósfera va haciéndose cada vez menos densa


hasta igualarse con el espacio interestelar. Las temperaturas se hacen elevadísimas.

3.1.3. LA ESTRUCTURA INTERNA DE LA TIERRA

Por debajo de la atmósfera, la Tierra se divide en dos partes, una de las cuales, a su vez,
se subdivide en varias:

12 TAMadrid

-Pág.16-
U. D. 1 . - M E D I O A M B I E N T E Y C O N O C I M I E N TO G E O G R Á F I C O

Corteza o Litosfera.—Estrecha capa superficial donde se encuentran los fondos marinos


y los continentes. Su grosor es variable siendo de entre 30-40 Km en las zonas continenta-
les y de unos 10 Km. en los fondos marinos. Sobre esta capa, adaptándose a su morfología,
se encuentran las aguas oceánicas y continentales que en algunos tratados reciben el nom-
bre de hidrosfera.

Endosfera.—Engloba todo el interior de la Tierra y se divide en dos partes básicamente:


Manto. Llega hasta los 2.900 Km pero dentro de ella existe una capa de materiales en
fusión situada a unos 100 Km. que es la astenosfera.
Núcleo. Tiene una parte externa fluida y un interior compacto que se denomina NIFE
(Ni: Niquel, Fe: Hierro, sus dos componentes)

3.1.4. LA FORMACIÓN DEL RELIEVE

A) La tectónica de placas
La litosfera está dividida en ocho placas principales, es decir, fragmentos de la corte-
za que se desplazan sobre la astenosfera como consecuencia de lo que se llama dinámica
de placas o deriva de los continentes (teoría definida en 1912 por el alemán ALFRED WEGE-
NER).

De dichas placas las hay que son totalmente oceánicas, como la de Nazca o la del Pací-
fico, y otras que incluyen zonas oceánicas y zonas continentales (América del Norte, Amé-
rica del Sur, Euroasiática, Africana, Indoaustraliana, Antártica).
Las dorsales son las líneas en las cuales se produce la ruptura y separación de las pla-
cas, son zonas en las que se produce la incorporación a la corteza del magma procedente de
la astenosfera, su estructura incluye una especie de valle central o rift y dos líneas de cres-
tas a ambos lados.
Los movimientos de convección del magma (ascenso de los materiales más calientes y
menos densos, descenso de los menos calientes y más densos) ayudan a la salida de éste a
la superficie por esas zonas de fractura que son las dorsales y colaboran también al despla-
zamiento de la placa.
En el punto opuesto de la dorsal, las placas se ponen en contacto de varias formas:
– Ambas se hunden
– Una se hunde por debajo de la otra, que a su vez se pliega, como sucede entre la placa
de Nazca, que se hunde, y la de América del Sur que monta sobre ella y se pliega for-
mando los Andes. En la zona donde se produce el hundimiento se forman las fosas.
(la más profunda es la de las Marianas con más de 11 kilómetros de profundidad).

TAMadrid
13
-Pág.17-
C I E N C I A S S O C I A L E S . G E O G R A F Í A E H I S TO R I A

– Las dos chocan y se pliegan, como sucede entre la placa Euroasiática y la Indoaus-
traliana, lo que genera la formación del Himalaya.
De esta manera los movimientos de las placas y la formación de nuevas dorsales que
dividan éstas son los responsables de la formación de las grandes unidades del relieve y
debido a ellos se ha producido a lo largo de la historia geológica la sucesiva fragmentación
de las masas continentales pudiéndose concluir que los continentes actuales estuvieron ini-
cialmente unidos en uno, que posteriormente se dividió en dos (Laurasia y Gondwana) y
sucesivamente en otros hasta la situación actual.
Esa historia geológica divide la formación del relieve en cuatro grandes fases orogéni-
cas u orogenias:

Era Hace... millones de años Orogenias


ARCAICA
4.500 a 570 ARCAICA
(Precámbrico)
PRIMARIA CALEDONIANA
570 a 225
(Paleozoico) HERCINIANA
SECUNDARIA
225 a 65
(Mesozoico)
TERCIARIA
65 a 1,8 ALPINA
(Cenozoico)
CUATERNARIA
1,8 a hoy
(Neozoico)

La dinámica de las placas explica también la generación de los terremotos o seísmos y


la formación de los volcanes.
Los terremotos son movimientos vibratorios de la corteza que se producen en las zonas
internas de la Tierra como consecuencia del choque o rozamiento de las placas. Esa vibra-
ción produce una serie de ondas sísmicas.
El punto del interior donde se produce la vibración se llama hipocentro o foco. El epi-
centro, es la proyección vertical del hipocentro sobre la superficie.
Los volcanes son fisuras de la corteza por las cuales el magma sale a la superficie. Las
partes de un volcán son:
– Cámara magmática: zona profunda donde se encuentra el magma.
– Chimenea: grieta que comunica la anterior con el exterior.
– Cono volcánico: acumulación de materiales consecuencia de anteriores erupciones
que forma el monte en cuya cúspide está el cráter.

14 TAMadrid

-Pág.18-
U. D. 1 . - M E D I O A M B I E N T E Y C O N O C I M I E N TO G E O G R Á F I C O

– Cráter: agujero por el que el magma sale al exterior.


– Conos y grietas adventicias: orificios secundarios laterales por los que sale el
magma, gases, etc.
La erupción es el proceso por el cual el magma sale al exterior formando la lava. En la
erupción también son expulsados gases, cenizas, etc.
Los volcanes son también responsables de la formación del relieve pues sus erupciones
van acumulando materiales que generan la formación de montañas e islas.

B) Los agentes modeladores del relieve


Una vez formadas las grandes unidades del relieve como consecuencia de las orogenias,
el modelado de detalle del mismo se debe a los efectos de los procesos geológicos exter-
nos que son:

• Meteorización.—Fractura y alteración de las rocas que puede ser de distintos tipos:


Mecánica.
– Por cambios de presión.
– Por cambios de temperatura que producen contracciones y expansiones.
– Por el efecto de la congelación y deshielo del agua que se introduce en las grietas.
Química. Reacciones químicas entre las rocas y el agua y el aire.
Biológica. Alteración mecánica y química producida por organismos vivos.

• Erosión.—Desgaste de las rocas por acción del agua o del viento.


Agua.
– El agua precipitada en forma de lluvia arrastra materiales y forma torrentes, cár-
cavas, valles, y modela las formas de las montañas
– El agua en forma de hielo en los glaciares.
– Las aguas subterráneas
– El agua del mar sobre las costas.

Viento.
– Por su velocidad
– Por el choque de los materiales que transporta.

Transporte. El agua líquida de ríos y torrentes, el hielo de los glaciares y el viento trans-
portan los materiales resultado de la meteorización.

TAMadrid
15
-Pág.19-
C I E N C I A S S O C I A L E S . G E O G R A F Í A E H I S TO R I A

Sedimentación. Los materiales transportados por el agua líquida y el viento se van depo-
sitando en función de la velocidad de éstos y el peso de aquéllos. Los transportados por el
hielo de glaciares se depositan en las zonas de deshielo y en sus márgenes.

C) Las formas del relieve


Como consecuencia de los procesos citados en los apartados anteriores se forman las
unidades de relieve y unidades geomorfológicas.
Estas últimas son consecuencia directa de los procesos orogénicos y no siempre coinci-
den con las primeras, fruto simultáneamente de las orogenias y de la erosión.
En las áreas continentales, los tipos de relieve se agrupan en:
– Estructuras de bloques, consecuencia de fracturas de la corteza, elevación y hundi-
miento de las estructuras resultantes. Las mesetas y macizos y algunas zonas monta-
ñosas son las formas de relieve derivadas de estas estructuras geomorfológicas (valgan
como ejemplo la Meseta ibérica, el Macizo Galaico y el Sistema Central español).
– Depresiones y penillanuras. Son terrenos llanos consecuencia de hundimiento de
bloques o del arrasamiento por la erosión de relieves antiguos (ejemplo: la Depre-
sión del Ebro).
– Plegamientos, consecuencia de las presiones horizontales de las placas sobre los
materiales sedimentarios que se doblan formando cadenas montañosas.
En los océanos observamos:
– Dorsales, cadenas de doble alineamiento formadas a ambos lados de las fracturas de
la corteza por las que el magma sale a la superficie y va generando la formación de
las placas.
– Llanuras abisales y cuencas oceánicas, zonas llanas a unos 4 km. de profundidad.
– Plataformas continentales, básicamente la parte de la masa continental que se
encuentra sumergida bajo el mar.
– Fosas marinas, profundos surcos generados por la subducción de las placas.

3.1.5. LA HIDROSFERA
Está formada por el conjunto de las aguas del planeta. Podemos agrupar las diferentes
clases o formas de presentación del agua de diversos modos.
Según su estado:
– Líquida: en océanos y aguas continentales superficiales y subterráneas.
– Sólida: en los casquetes polares, glaciares y cadenas nevadas.
– Gaseosa: diluida en la atmósfera como vapor de agua.

16 TAMadrid

-Pág.20-
U. D. 1 . - M E D I O A M B I E N T E Y C O N O C I M I E N TO G E O G R Á F I C O

Según su composición:
– Agua salada: Supone más del 95% del total encontrándose fundamentalmente en los
mares y océanos.
– Agua dulce: Esencialmente las aguas continentales.

Según su reparto:
– Aguas oceánicas.
– Glaciares y nieves perpetuas.
– Casquetes polares.
– Aguas subterráneas.
– Lagos.
– Ríos.
El agua experimenta continuos cambios como consecuencia de los distintos procesos
que constituyen el llamado ciclo del agua. Éste ciclo sigue los siguientes pasos:
Evaporación y traspiración.—Que, por efecto del calor, trasforma el agua líquida en
vapor que se diluye en la atmósfera.
Condensación.—En determinadas condiciones de presión y temperatura, el vapor de
agua se condensa en nubes de diferentes tipos.
Precipitación.—Si el vapor de agua en las nubes llega a su saturación, se produce la pre-
cipitación en forma líquida (gotas de lluvia) o sólida (nieve o granizo).
Circulación superficial.—El agua precipitada puede quedar almacenada en forma de
nieve o discurrir por laderas y vertientes formando ríos y desembocando en lagos o en el mar.
Filtración.—Parte del agua precipitada es absorbida por el suelo pasando a las plantas y
a cauces subterráneos y acuíferos.

3.1.6. EL CLIMA

Clima es el conjunto medio de condiciones meteorológicas de un determinado lugar.


Visto de otro modo, es la media de los tiempos atmosféricos registrados en un lugar a lo
largo de un prolongado periodo de tiempo.

Los factores que influyen en la determinación del clima son:


– La temperatura.
– Las precipitaciones.
– La presión atmosférica. Que determina a su vez la sucesión de ciclones y anticiclo-
nes, los vientos, la humedad, la pluviosidad.

TAMadrid
17
-Pág.21-
C I E N C I A S S O C I A L E S . G E O G R A F Í A E H I S TO R I A

– La latitud.
– La distribución de tierras y mares.
– El relieve.
– Las corrientes marinas.

A) La temperatura
Es la cantidad de calor que existe en un lugar determinado. En la mayor parte de los
países se mide en grados centígrados o Celsius; en EE.UU. se mide en grados Fahrenheit.
[xo F = (1,8 * yo C) + 32].
En realidad podemos hablar de distintas temperaturas: temperatura media, temperatura
máxima, temperatura mínima.
En cada caso podríamos hablar de temperatura diurna, nocturna y diaria, mensual, esta-
cional, anual.
Para analizar el clima de un determinado lugar deberemos fijarnos en las temperaturas
medias, y en la oscilación o amplitud térmica (diferencia entre la temperatura media máxi-
ma y la temperatura media mínima).
Los factores que influyen en la variación de las temperaturas son:
– Latitud.
– Altitud y relieve.
– Proximidad al mar.
– Sucesión de las estaciones (inclinación eje terrestre).
– Sucesión de días y noches (rotación terrestre).
– Corrientes marinas.
La representación gráfica de las temperaturas se realiza por medio de las líneas isotermas.

B) Las precipitaciones
Es el conjunto de agua caída, en forma de lluvia, granizo o nieve, en un lugar determi-
nado, medido en milímetros por centímetro cuadrado.
La insolación favorece la evaporación del agua que queda en la atmósfera en suspen-
sión. Al ponerse en contacto con masas de aire frío o si llega a su límite de saturación, que
depende de la temperatura, se condensa en nubes. Si las pequeñas gotas y/o cristales se
hacen suficientemente grandes, se precipitan.
Tal precipitación se puede producir de distintas formas:
– Convectivas. Por choque de masas de aire por el relieve o litorales
– Ciclonales o frontales.
La representación gráfica de las precipitaciones se realiza por medio de las líneas isoyetas.

18 TAMadrid

-Pág.22-
U. D. 1 . - M E D I O A M B I E N T E Y C O N O C I M I E N TO G E O G R Á F I C O

C) La presión atmosférica
Presión atmosférica, medida en milibares y representada por líneas isobaras, es el peso
que ejerce la atmósfera sobre la superficie, siendo variable según los lugares y las circuns-
tancias de altitud y temperatura.
Dependiendo de la presión, se forman áreas de mayores o menores presiones:
Ciclones o borrascas.—Zonas de bajas presiones en las que las masas de aire tienden a
ascender y giran en el sentido contrario a las agujas del reloj. Se asocian a la sucesión de
frentes del frente polar. Implican tiempo inestable.
Anticiclones.—Zonas de altas presiones en las que las masas de aire tienden a descen-
der y giran en el sentido de las agujas del reloj. Implican tiempo estable.
La combinación de condiciones meteorológicas determina el tiempo que hace en cada
lugar. La observación de dicho tiempo a lo largo de los años permite abstraer una serie de
condiciones comunes a lugares próximos y de esta manera establecer zonas con tiempos
similares, áreas con el mismo clima.

D) Latitud
Como hemos visto, la latitud influye en las temperaturas como consecuencia de la dife-
rente incidencia de los rayos solares. Como consecuencia de esto y de la sucesión estacio-
nal, se definen las sucesivas zonas climáticas.

E) Distribución de tierras y mares


La proximidad al mar suaviza las temperaturas y aumenta la humedad, mientras que el
alejamiento de masas marítimas contribuye a la sequedad de la atmósfera y a la radicaliza-
ción de las temperaturas.

F) Relieve
La distribución de las unidades de relieve influye poderosamente en la circulación
atmosférica de superficie favoreciendo, en determinados casos, las precipitaciones o la
penetración, en ciertas áreas, de masas de aire cálidas o frías, secas o cargadas de humedad.
Además, la altitud influye en la determinación de las temperaturas.

G) Corrientes marinas
Son fruto de la interacción de las aguas marinas superficiales con la atmósfera. Las
corrientes marinas, que, dependiendo de su disposición latitudinal, mueven masas de aire
frías o cálidas, generan efectos diversos en las zonas costeras afectadas y circunstancias
especiales en los regímenes de vientos.

TAMadrid
19
-Pág.23-
C I E N C I A S S O C I A L E S . G E O G R A F Í A E H I S TO R I A

3.1.7. LOS TIPOS DE CLIMAS

A continuación incluimos, en esquema, los rasgos principales de cada clima y su distri-


bución en el planeta.
Climas terrestres

Clima Características Vegetación


POLAR – Bajas temperaturas (mes más cálido: <10o) – Tundra (musgos + líquenes).
– Verano muy corto e invierno riguroso En Asia, además, árboles enanos: abedul
– Escasa precipitación (en forma de nieve)
CONTINENTAL – Lejanía costas – Coníferas.
FRÍO – Bajas temperaturas en invierno Taiga (abetos, pinos, alarces, abedules)
– Gran amplitud térmica
– Precipitaciones convectivas en meses cálidos
CONTINENTAL – Bajas temperaturas en invierno (menos bajas – Coníferas
TEMPLADO que en continental frío) – Caducifolias
– Altas temperaturas en verano (más de 20o) – Praderas herbáceas
– Gran amplitud térmica
– Precipitaciones en verano
TEMPLADO – Factores determinantes: corrientes marinas y – Especies tropicales (América: manglar, pal-
HÚMEDO vientos alisios. mera) (Asia: bambú, magnolia), y especies
– Temperaturas moderadas en invierno templadas (haya, roble, pino, abedul, casta-
– Temperaturas altas en verano ño, abeeto, picea)
– Precipitaciones abundantes en verano (+ ó -
1.000 mm.) por alisios de verano húmedos
(Asia = monzones)
OCEÁNICO – Influencia masa oceánica – Bosque caducifolio. (En América, latitud
– Temperaturas moderadas alta: sequoias, abetos Douglas)
– Amplitud térmica baja
– Precipitaciones abundantes (+ en invierno, y
- en verano)
MEDITERRÁNEO – Temperaturas suaves en invierno – Plantas xerófilas
– Temperaturas altas en verano – Pino
– Precipitaciones escasas (<500 mm.) – Encina
– Precipitaciones en primavera y otoño
– Meses secos (1 a 4)
ESTEPARIO – Temperaturas relativamente altas todo el año – Arbustos xerófilos
– Precipitaciones escasas (<250 mm.) (en Oceanía: spinifex, arbol botella)
– Más de 4 meses secos
DESÉRTICO – Persistente sequía – Sólo oasis
– Gran amplitud térmica diaria
TROPICAL – Temperaturas altas en verano – Selva tropical caducifolia, y transición a
– Temperaturas suaves en invierno sabana (especies herbáceas altas alternando
– Escasa amplitud térmica con especies arbóreas; en África: acacias) y
– Precipitaciones abundantes (>1.000 mm.) luego a estepa
– Estación seca en invierno
ECUATORIAL – Escasa oscilación térmica – Pluvisilva perennifolia. (En Asia: caucho)
– Precipitaciones abundantes, sobre todo en
equinoccios

20 TAMadrid

-Pág.24-
U. D. 1 . - M E D I O A M B I E N T E Y C O N O C I M I E N TO G E O G R Á F I C O

3.2. LOS PAISAJES NATURALES


Como consecuencia de los aspectos físicos que hemos estudiado en el apartado anterior,
la superficie terrestre y, más concretamente, los espacios continentales, se nos presentan
compartimentados en zonas con unas específicas características de morfología, clima, sue-
los y vegetación.

En la definición de dichos espacios, el clima constituye el factor fundamental si bien no


debe perderse de vista las modificaciones que sobre los rasgos básicos pueden determinar
los otros factores. De este modo, y a grandes rasgos, podemos diferenciar distintos paisajes
naturales.

3.2.1. PAISAJES CÁLIDOS

Corresponden básicamente con las áreas en las que la insolación es mayor a lo largo del
año, es decir, la franja comprendida entre los Trópicos a ambos lados del Ecuador, dentro
de los mismos podemos distinguir los paisajes que se corresponden con los climas ecuato-
rial, tropical y desértico. Desde el Ecuador, en dirección norte y en dirección sur, se obser-
va la paulatina transición entre estos paisajes:

– En la zona ecuatorial tenemos una abundante vegetación, la pluvisilva ecuatorial peren-


nifolia, nutrida por una elevada cantidad de precipitaciones repartidas a lo largo de todo
el año, las especies compiten por el espacio y la luz solar estratificándose en cuanto a
las alturas que alcanzan o mostrando grandes hojas. Los suelos son pobres por sí mis-
mos en nutrientes aunque éstos no faltan a las plantas pues toda materia orgánica que
cae se descompone con rapidez y es rápidamente absorbida. El ambiente húmedo y
caluroso propicia este hecho y la abundancia de vida, tanto vegetal como animal.
– A medida que se alcanzan las zonas tropicales, a causa de la sequía, la selva se hace
estacional y, en consecuencia, menos densa, con especies arbóreas diseminadas y
más especies herbáceas y arbustivas. La paulatina degradación, en tránsito hacia el
desierto, de esta vegetación da lugar a la sabana, en la que las especies herbáceas
predominan. Además, la sequía estacional motiva el avance o retroceso de las líne-
as de transición al desierto. Por último, en las áreas tropicales aparecen también
algunos paisajes ecuatoriales asociados a los cauces de caudalosos ríos.
– A continuación, la insolación unida a la falta de humedad, motivada por la influencia
de la Convergencia Intertropical, genera la formación de áreas desérticas en las que
la pobreza del suelo se une a las adversas condiciones de humedad y temperatura para
motivar la escasez o total ausencia de vegetación. Los paisajes desérticos se nos pre-
sentan así como áreas con grandes roquedales (hammada) muy erosionados por los

TAMadrid
21
-Pág.25-
C I E N C I A S S O C I A L E S . G E O G R A F Í A E H I S TO R I A

fuertes cambios de temperatura entre el día y la noche y por la acción abrasiva del vien-
to seco y cargado de polvo (Harmattan); o bien como zonas rocosas muy fragmentadas y
de pequeño tamaño; o como las típicas formaciones arenosas de dunas móviles(erg).
La presencia ocasional de algún afloramiento de agua desde acuíferos subterráneos,
favorece que el desierto aparezca aisladamente salpicado de pequeñas manchas de vege-
tación, los oasis; y las ocasionales precipitaciones torrenciles la formación de cauces
intermitentes sin desembocadura en el mar,los wadis, ouadis o guadis.

3.2.2. PAISAJES TEMPLADOS


En la zona de climas templados, entre el trópico y el círculo polar ártico en cada hemis-
ferio, se encuentran los paisajes templados correspondientes a los climas característicos de
estas franjas que presentan una apreciable variación en función de su situación latitudinal,
en principio, aunque también condicionada a la distribución continental y a la interioridad
de amplias áreas.
Así, en las latitudes más bajas encontramos paisajes como el mediterráneo o el templa-
do húmedo y en las más altas los paisajes oceánicos y los continentales.
– El paisaje mediterráneo se caracteriza por su semiaridez con el predominio de bos-
ques no muy densos de pinos, alcornoques, encinas, rebollos, mezclados con espe-
cies arbustivas resistentes a los meses de sequía.
– El paisaje templado-húmedo, con una acentuada pluviosidad, presenta especies
arbóreas caducifolias además de otras especies específicas y áreas pantanosas.
– El paisaje oceánico, con temperaturas y precipitaciones abundantes, muestra suelos
ricos y vegetación abundante a base de especias arbóreas de hoja caduca.
– El paisaje continental, por su parte, caracterizado fundamentalmente por las radica-
les diferencias de temperatura entre los meses cálidos y los fríos, presenta también
una abundancia arbórea con predominancia de las coníferas degradándose en altitud
y en las áreas más al interior con una paulatina abundancia de especies herbáceas
hasta convertirse en estepa o, incluso, en desierto frío.

3.2.3. PAISAJES FRÍOS


Estos paisajes, caracterizados inicialmente por sus muy bajas temperaturas, sequedad y
muy pobre o nula vegetación, aparecen en áreas distintas y con caracteres específicos:
– En las franjas más extremas de la superficie terrestre, las áreas polares, encontramos
paisajes dominados por el hielo. Que ocupa con kilómetros de espesor la superficie
de tierras emergidas (Antártida, Groenlandia) y avanza en el mar hasta límites mayo-
res o menores en función de las estaciones.

22 TAMadrid

-Pág.26-
U. D. 1 . - M E D I O A M B I E N T E Y C O N O C I M I E N TO G E O G R Á F I C O

– En algunas áreas del interior de Asia y América del Norte, existen amplias zonas
muy alejadas de las influencias marítimas y con una situación latitudinal favorable
a las bajas temperaturas. En estas áreas se forman paisajes desérticos fríos, con esca-
sa vegetación que desaparece a mayor latitud. La nieve domina el paisaje durante
meses, más cuanto mayor sea la latitud.
– La influencia de algunas corrientes marítimas determina, en algún caso, la aparición
del paisaje desértico frío, tal es el caso del desierto de Atacama, en Chile, propicia-
do por la corriente fría de Humboldt.
– Por último, los paisajes fríos aparecen también en zonas montañosas. Aquí es la alti-
tud el factor que determina los escalones de transición entre paisajes cálidos, tem-
plados y fríos.

3.3. EL MEDIO Y EL SER HUMANO


Hasta ahora, hemos estudiado la Tierra desde un punto de vista físico sin tener en cuen-
ta la presencia sobre su superficie de las especies animales, digamos que hemos analizado
aquello que, en principio, constituye el marco, perdurable en la escala de tiempo de la vida
humana, en la que ésta y la de los animales se desarrolla.

Pero la Humanidad y el medio geográfico se interrelacionan constantemente. De hecho,


el ser humano es lo que es precisamente porque el medio físico impulsó los cambios gené-
ticos que condujeron a su aparición y ha seguido influyendo en la evolución de las especies
que han llevado hasta el Homo Sapiens. Después, el Hombre se convirtió también en un fac-
tor alterador del medio natural entrando en un juego de cuyas consecuencias somos cada
vez más conscientes aunque no siempre seamos capaces de controlarlas.

3.3.1. LA INFLUENCIA DEL MEDIO SOBRE EL HOMBRE

Como hemos dicho, el medio físico tuvo un protagonismo decisivo en las generación de
las condiciones ambientales (vegetación, clima, disponibilidad de recursos alimenticios)
que propiciaron los cambios evolutivos que condujeron a la aparición del Homo Sapiens.
Pero, además, el medio influye poderosamente en la vida del ser humano.
Las condiciones de relieve, climatológicas, de vegetación, etc. mediatizan ocasional-
mente los asentamientos humanos si bien los medios técnicos han ido permitiendo la expan-
sión del poblamiento por zonas físicamente muy adversas.
Pero, si las condiciones más o menos estables y predecibles pueden ser controladas y, en
consecuencia, generar las medidas que permitan afrontarlas, tal posibilidad se vuelve poco

TAMadrid
23
-Pág.27-
C I E N C I A S S O C I A L E S . G E O G R A F Í A E H I S TO R I A

menos que imposible ante las muestras que ocasionalmente ofrece la Naturaleza de su poder
cambiante.
Efectivamente, la Tierra se manifiesta especialmente dinámica, tanto por la acción de
sus fuerzas internas como por la de su atmósfera. Esa dinámica, generadora del relieve y de
la climatología, puede producir puntualmente auténticas catástrofes para la Humanidad.
Catástrofes climatológicas.—Huracanes, monzones, tornados... desatan tempestuosas
fuerzas de viento y agua que pueden arrasar poblaciones enteras, arrastrando edificios,
campos de cultivo, vías de comunicación, etc...
Catástrofes telúricas.—Potencialmente tan catastróficas o más que las anteriores, los
terremotos y erupciones volcánicas pueden acabar con vidas e infraestructuras a gran escala.
La ciencia se esfuerza en el estudio de estos fenómenos con el fin de alcanzar elemen-
tos de análisis suficientes como para poder predecirlos y así mitigar sus efectos.

3.3.2. LA INFLUENCIA DEL HOMBRE SOBRE EL MEDIO


Recién surgido como especie, el Hombre era un elemento más del medio natural sobre
el que ejercía una influencia mínima; aprovechaba lo que la naturaleza le ofrecía sin modi-
ficar las condiciones ambientales sustancialmente.
Sin embargo, a raíz de la revolución neolítica, el ser humano comenzó una sistemática
modificación del medio. Primero de una forma modesta, cuidando más de unas especies
animales y vegetales que de otras, y paulatinamente de modo más trascendente hasta llegar
a alterar peligrosamente condiciones que de una forma o de otra han terminado o pueden
terminar volviéndose contra él.
La explotación de los recursos naturales es el factor causante de la degradación del
medio. Naturalmente, dicha explotación resulta imprescindible para el sostenimiento y
desarrollo de la Humanidad, siendo su indiscriminada, abusiva o errónea explotación la que
motiva los desastres pasados, presentes o previsibles.

A) Uso y consumo del agua


El agua es un elemento fundamental para la alimentación, la agricultura, la industria y
el ocio. Pero los recursos hídricos no están repartidos de forma homogénea ni las necesida-
des de las diferentes sociedades son proporcionales a los recursos que poseen.
Aun cuando la cantidad total de agua en la Tierra sea enorme, la que puede aprovechar-
se para el consumo es realmente muy reducida. Sólo el 4,5% del agua es dulce y de ella el
75% se encuentra en forma de hielo, el 21% en acuíferos subterráneos y el resto en lagos y
ríos. Con todo, tampoco toda el agua dulce al alcance es potable.

24 TAMadrid

-Pág.28-
U. D. 1 . - M E D I O A M B I E N T E Y C O N O C I M I E N TO G E O G R Á F I C O

A esto se une el desigual reparto de las aguas continentales como consecuencia de las
condiciones climatológicas.
Todas estas circunstancias vienen a demostrar lo trascendente de una explotación racio-
nal de los recursos y de la necesaria solidaridad para el reparto de los excedentes entre las
zonas con mayor déficit hídrico.
Por otra parte, el uso del agua para las diversas actividades humanas genera las aguas
residuales que contaminan, si no son adecuadamente tratadas, ríos, lagos y mares.

B) La degradación del medio y el cambio climático


Si bien los complejos mecanismos que gobiernan el clima global y determinan su evo-
lución a grandes escalas de tiempo, no nos es conocido completamente, resulta evidente la
influencia que el Hombre ha tenido y tiene en la modificación de determinados aspectos
que influyen en el mismo.
La explotación indiscriminada de los recursos forestales, la tala de bosques para amplia-
ción de las explotaciones agropecuarias o para la obtención de minerales en minas a cielo
abierto ha ido degradando desde hace siglos los paisajes ecuatoriales, tropicales y templa-
dos y favorecido el avance de los paisajes esteparios y desérticos.
Las actividades industriales que desde el siglo XIX han ido expulsando materias conta-
minantes a las aguas y a la atmósfera. La utilización de gases propelentes y combustibles
fósiles, cuyos residuos van llenando la atmósfera (CO2 , aerosoles) o se introducen negati-
vamente en el ciclo del agua (lluvia ácida), han ido afectando al incremento de la tempera-
tura media, al aumento del agujero en la capa de ozono, a la degradación de la vegetación,
etc. Todo ello puede conducir a un aumento del llamado efecto invernadero con el consi-
guiente recalentamiento, disminución de los casquetes polares y aumento del nivel del mar,
algo que, lógicamente, influirá en las condiciones climatológicas, favoreciendo desastres
naturales, acabará con especies animales y vegetales y podrá poner en riesgo la propia via-
bilidad de la especie humana.
Muchas son las especies animales y vegetales que han desaparecido en los últimos dece-
nios como consecuencia de la acción del hombre, muchas las que se encuentran en trance
de extinción, muchas las que subsisten en un precario equilibrio y todas, incluido el cau-
sante, amenazadas. La concienciación y la acción de todos y no sólo de los gobiernos y las
organizaciones internacionales, por otra parte imprescindible, deberá conducir a una más
racional explotación de los recursos, a la búsqueda de fuentes de energía limpias y de inno-
vaciones técnicas que permitan tratar, depurar y reciclar de forma óptima los inevitables
residuos de una civilización que debe buscar como objetivo su perpetuación y la del medio
natural de la nave, llamada Tierra, en la que viaja por el Universo, considerando la máxima
del proverbio quechúa: “La Tierra no es una herencia de nuestros padres, sino un présta-
mo de nuestros hijos”.

TAMadrid
25
-Pág.29-
C I E N C I A S S O C I A L E S . G E O G R A F Í A E H I S TO R I A

RESUMEN
INICIACIÓN A LOS MÉTODOS GEOGRÁFICOS
La Tierra, su forma y la percepción de la misma
Podemos definir a la Tierra como un geoide de revolución, es decir, una esfera irregular.
Coordenadas: Latitud (distancia al Ecuador, paralelos).
Longitud (distancia al meridiano 0, meridianos).

La representación de la Tierra. La cartografía


La Cartografía es la representación gráfica de la Tierra por medio de planos o mapas.

Técnicas y Sistemas de representación espacial


Proyecciones equivalentes y conformes, planas, cilíndricas y cónicas.
La escala: relación entre las distancias reales y las reflejadas en el mapa.
La orientación.
Los símbolos cartográficos (colores, trazos, tramas, tipos de letra).

LAS FUENTES DE INFORMACIÓN GEOGRÁFICA


Y SU UTILIZACIÓN
• Mapas o planos conceptuales. Climáticos, políticos, demográficos, económicos.
• Estadísticas.
• Gráficos. Climogramas, hidrogramas, pirámides de población, gráficos de producción.

EL MEDIO AMBIENTE Y SU CONSERVACIÓN


El Planeta Tierra: sus rasgos físicos fundamentales

La Tierra como astro


La Tierra: tercer planeta del Sistema Solar a unos 150 millones de Km. del Sol,
Diámetro ecuatorial: 12.756 Km.
Densidad: 5,5 g/cm3.
Gravedad: 978,03 cm/s2.
Rotación: 23 horas, 56 minutos y 4 segundos. 24 husos horarios de 15o cada uno.
Traslación: 365,242 días, velocidad de 29,8 Km/s. Órbita elíptica.

26 TAMadrid

-Pág.30-
U. D. 1 . - M E D I O A M B I E N T E Y C O N O C I M I E N TO G E O G R Á F I C O

Inclinación eje: 23o 27' sobre la perpendicular a la eclíptica.


Cambios estacionales: Equinoccios (21 de marzo y 22-23 de septiembre); solsticios (21
de junio, 21 de diciembre).

La atmósfera
La atmósfera es la capa gaseosa que envuelve al planeta (78% Nitrógeno, 20,9% Oxí-
geno). Troposfera, estratosfera, mesosfera, ionosfera, exosfera.

La estructura interna de la Tierra


Corteza o Litosfera.
Endosfera: Manto y Núcleo.

La formación del relieve


A) La tectónica de placas
La litosfera está dividida en ocho placas principales que se desplazan sobre la astenos-
fera (teoría de ALFRED WEGENER).
Cuatro grandes fases orogénicas u orogenias: Arcaica, Caledoniana, Herciniana y Alpina.
Los terremotos: movimientos vibratorios de la corteza por el choque de las placas
Los volcanes: fisuras de la corteza por las cuales el magma sale a la superficie.

B) Los agentes modeladores del relieve


Meteorización. Mecánica, química, biológica.
Erosión. Desgaste de las rocas por acción del agua o del viento.
Transporte.
Sedimentación.

C) Las formas del relieve


En los continentes: Estructuras de bloques, Depresiones y penillanuras, Plegamientos.
En los océanos: Dorsales, Llanuras abisales y cuencas oceánicas, Plataformas continen-
tales, Fosas marinas.

La hidrosfera
Esta formada por el conjunto de las aguas del planeta.
Agua salada supone más del 95% del total.
Ciclo del agua: evaporación, condensación, precipitación, circulación, filtración.

TAMadrid
27
-Pág.31-
C I E N C I A S S O C I A L E S . G E O G R A F Í A E H I S TO R I A

El clima
Factores: La temperatura, las precipitaciones, la presión atmosférica, la latitud, la dis-
tribución de tierras y mares, el relieve, las corrientes marinas.

Los paisajes naturales

Paisajes cálidos
Zona ecuatorial: pluvisilva ecuatorial perennifolia.
Zonas tropicales: selva estacional. Transición a sabana.
Paisajes desérticos: hammada, erg, oasis, wadis.

Paisajes templados
El paisaje mediterráneo: semiaridez.
El paisaje templado-húmedo: especies arbóreas caducifolias.
El paisaje oceánico: vegetación abundante.
El paisaje continental: bosques de coníferas.

Paisajes fríos
Paisajes polares.
Desiertos fríos.

EL MEDIO Y EL SER HUMANO


La influencia del medio sobre el Hombre
Catástrofes climatológicas.
Catástrofes telúricas.

La influencia del hombre sobre el medio


Degradación del medio: Las actividades industriales, gases propelentes y combustibles
fósiles han ido afectando al incremento de la temperatura media, al aumento del agujero en
la capa de ozono, a la degradación de la vegetación, etc.

28 TAMadrid

-Pág.32-
U. D. 1 . - M E D I O A M B I E N T E Y C O N O C I M I E N TO G E O G R Á F I C O

EJERCICIOS DE AUTOCOMPROBACIÓN
11. ¿Cuál de las siguientes teorías está reconocida en la actualidad como la base para explicar la
formación del relieve de los continentes, así como la actividad sísmica y volcánica?
A. Teoría del proceso de contracción B. Teoría de la tectónica de la gravedad
C. Teoría de la tectónica de placas D. Teoría del metamorfismo generador de plega-
d.– mientos

12. ¿Qué es la latitud?


A. Es cada una de las líneas imaginarias que rodean la Tierra pasando por los polos
B. Es la distancia, medida en metros, entre un punto de la superficie y el meridiano 0
C. Es cada una de las líneas imaginarias que rodean la Tierra paralelas al Ecuador
D. Es la distancia, medida en grados, entre un punto de la superficie y el Ecuador

13. Las orogenias que se han sucedido en la historia geológica de la Tierra han sido:
A. Arcaica, Caledoniana, Herciniana y Alpina
B. Meseta Ibérica, Macizo Galaico y Sistema Central
C. Dorsales, fosas y plegamientos
D. Elevación, erosión y sedimentación

14. Señale la afirmación correcta:


A. El periodo de rotación de la Tierra es de 23 días, 56 minutos y 4 segundos
B. La inclinación del eje de la Tierra sobre la pendicular a la eclíptica es de 23 horas y 27
minutos
C. El periodo de traslación de la Tierra alrededor del Sol es de algo más de 365 años
D. La velocidad de traslación de la Tierra es, aproximadamente, de 29.800 m/s

15. ¿Qué día se produce el solsticio de verano en Santiago de Chile?


A. 21 de marzo B. 21 de junio C. 22 de septiembre D. 21 de diciembre

16. ¿Cómo se denomina el trópico del hemisferio sur?


A. Ecuador B. Cáncer C. Capricornio D. Herciniano

17. La atmósfera se compone fundamentalmente de:


A. Oxígeno B. Nitrógeno
C. Hidrógeno D. Dióxido de Carbono

TAMadrid
29
-Pág.33-
C I E N C I A S S O C I A L E S . G E O G R A F Í A E H I S TO R I A

18. La capa inferior de la atmósfera, en la que se producen los fenómenos meteorológicos, es:
A. La estratosfera B. La troposfera C. La astenosfera D. La hidrosfera

19. La parte más interna del núcleo de la Tierra se denomina:


A. Astenosfera B. NiFe C. Troposfera D. Manto

10. Las formaciones arenosas características del desierto sahariano se denominan:


A. Erg B. Reg C. Hammada D. Kalahari

11. El agujero de un volcán por el que el magma sale al exterior se denomina:


A. Cámara magmática B. Cono adventicio
C. Cono volcánico D. Cráter

12. En los mapas climáticos, las líneas que unen los puntos de igual precipitación se denominan:
A. Isobaras B. Isotermas C. Isoyetas D. Isotrópicas

13. La vegetación característica del paisaje continental templado es:


A. Coníferas y caducifolias B. Bosque caducifolio
C. Tundra D. Taiga y masas de coníferas

14. Entre las plantas tropicales características del clima Templado húmedo en Asia se encuentra:
A. El manglar B. Las piceas C. El spinifex D. El bambú

15. El término “harmattan” corresponde a:


A. Un tipo de formación desértica con B. Un tipo de desierto con predominio de la
a.– predominio del roquedo b.– arena
C. Un macizo africano D. Un viento seco característico del desierto

30 TAMadrid

-Pág.34-
U. D. 1 . - M E D I O A M B I E N T E Y C O N O C I M I E N TO G E O G R Á F I C O

RESPUESTAS A LOS EJERCICIOS

11. C
12. D
13. A
14. D
15. D
16. C
17. B
18. B
19. B
10. A
11. D
12. C
13. A
14. D
15. D

TAMadrid
31
-Pág.35-
-Pág.36-
U. D. 2 . - L A P O B L A C I Ó N Y E L E S PA C I O U R B A N O

ÍNDICE
Pag.
OBJETIVOS. . . . . . . . . . . . . . . . . . . . . . . . . . . . . . . . . . . . . . . . . . . . . . . . . . . . . . . . . . . . . . . . . 2

INTRODUCCIÓN . . . . . . . . . . . . . . . . . . . . . . . . . . . . . . . . . . . . . . . . . . . . . . . . . . . . . . . . . . . 3

MAPA CONCEPTUAL . . . . . . . . . . . . . . . . . . . . . . . . . . . . . . . . . . . . . . . . . . . . . . . . . . . . . . . 4

DESARROLLO DE CONTENIDOS . . . . . . . . . . . . . . . . . . . . . . . . . . . . . . . . . . . . . . . . . . . 5

1. LA POBLACIÓN Y LOS RECURSOS . . . . . . . . . . . . . . . . . . . . . . . . . . . . . . . . . . . . . 5


1.1. LA POBLACIÓN HUMANA . . . . . . . . . . . . . . . . . . . . . . . . . . . . . . . . . . . . . . . . . 5

1.2. DISTRIBUCIÓN DE LA POBLACIÓN EN EL ESPACIO . . . . . . . . . . . . . . . . . . 5

1.3. FACTORES DEL REPARTO GEOGRÁFICO DE LA POBLACIÓN . . . . . . . . . . 7

1.4. MOVIMIENTOS NATURALES DE LA POBLACIÓN . . . . . . . . . . . . . . . . . . . . . 8

1.5. MOVIMIENTOS MIGRATORIOS . . . . . . . . . . . . . . . . . . . . . . . . . . . . . . . . . . . . 11

1.6. ANÁLISIS DE LA POBLACIÓN ACTUAL Y PROSPECTIVAS . . . . . . . . . . . . 12

2. EL ESPACIO URBANO . . . . . . . . . . . . . . . . . . . . . . . . . . . . . . . . . . . . . . . . . . . . . . . . . 16
2.1. EVOLUCIÓN DE LAS CIUDADES . . . . . . . . . . . . . . . . . . . . . . . . . . . . . . . . . . . 16

2.2. LA CIUDAD ACTUAL . . . . . . . . . . . . . . . . . . . . . . . . . . . . . . . . . . . . . . . . . . . . . 19

RESUMEN . . . . . . . . . . . . . . . . . . . . . . . . . . . . . . . . . . . . . . . . . . . . . . . . . . . . . . . . . . . . . . . . . 28

EJERCICIOS DE AUTOCOMPROBACIÓN . . . . . . . . . . . . . . . . . . . . . . . . . . . . . . . . . . 33

RESPUESTAS A LOS EJERCICIOS . . . . . . . . . . . . . . . . . . . . . . . . . . . . . . . . . . . . . . . . . 36

-Pág.37-
C I E N C I A S S O C I A L E S , G E O G R A F Í A E H I S TO R I A

OBJETIVOS
Al finalizar el estudio de esta Unidad Didáctica, el alumno será capaz de:

• Familiarizar al alumno con expresiones y conceptos demográficos.

• Comprender una pirámide de población.

• Entender los factores que determinan el reparto de la población en el mundo.

• Diferenciar y comprender los distintos elementos del paisaje urbano, valore


las condiciones de la vida humana en la ciudad y obtenga conclusiones pro-
pias sobre el diseño urbanístico, los espacios y servicios de las ciudades.

-Pág.38-
U. D. 2 . - L A P O B L A C I Ó N Y E L E S PA C I O U R B A N O

INTRODUCCIÓN
D edicada la primera Unidad Temática del programa al estudio de la Geografía
Física, es decir, de las características de nuestro planeta como planeta, su relie-
ve, clima y vegetación, sus medios y paisajes naturales, hemos dedicado algunas
líneas a aproximarnos a la interrelación Medio - Hombre, observando cómo el pri-
mero ha influido y puede influir en el desarrollo de la Humanidad y en qué medida
el segundo modifica el medio, incluso de un modo dramático.

Llega ahora el turno de la Geografía Humana, es decir, del estudio de la población,


su adaptación al espacio y distribución en el mismo, sus movimientos y las caracte-
rísticas de los mismos.

Nos acercaremos así al conocimiento de las dimensiones que ha alcanzado el pobla-


miento humano y su desigual reparto en la superficie terrestre, analizando los facto-
res que influyen en el mismo.

Estudiaremos los diferentes tipos de movimientos de la población, tanto en cuanto a


su crecimiento como a las migraciones.

Finalmente veremos la forma de hábitat humano que predomina en la actualidad, la


ciudad: sus estructuras, organización, protagonismo, ventajas e inconvenientes.

-Pág.39-
C I E N C I A S S O C I A L E S , G E O G R A F Í A E H I S TO R I A

M A PA C O N C E P T UA L
La Población Humana

Distribución de la población en el
espacio

Factores del reparto geográfico de la


LA POBLACIÓN Y LOS población
RECURSOS
Movimientos naturales
de la población

Movimientos migratorios

Análisis de la población actual y pros-


pectivas

Evolución de las ciudades

EL ESPACIO URBANO

La ciudad actual

-Pág.40-
U. D. 2 . - L A P O B L A C I Ó N Y E L E S PA C I O U R B A N O

1. LA POBLACIÓN Y LOS RECURSOS

1.1. LA POBLACIÓN HUMANA

La Humanidad es, en los umbrales del siglo XXI, una comunidad de unos 6.000 millones de
individuos repartidos por todo el mundo.

Cuando nuestro ancestro Australopithecus comenzó a caminar erguido por el bosque tropical
degradado en el que buscaba su sustento, se inició un lento proceso de crecimiento. Las comunida-
des de Homo Erectus, de Neanderthales y de nuestros directos antepasados, Homo antecessor y
los primeros Homo Sapiens Sapiens, se mantenían en un número modesto y con un leve creci-
miento, siempre controlado por los desastres naturales, los efectos de los depredadores, la escasez
de alimentos y la acción de un sinfín de enfermedades que diezmaban a la población infantil y limi-
taban la longevidad.

A partir de la revolución neolítica se produjo una leve aceleración del crecimiento como conse-
cuencia de la mejora de las condiciones de vida: la sedentarización, la producción de los alimentos,
la regulación de los excedentes para asegurar la nutrición en las épocas de escasez, la división del
trabajo, los iniciales conocimientos científicos, etc... Esto permitió aumentar la esperanza de vida si
bien el hombre seguía limitado en su capacidad de crecimiento por las enfermedades, los desastres
naturales y un nuevo factor, las guerras.

El crecimiento siguió siendo lento hasta la Edad Moderna, acelerándose entonces y, sobre todo,
a partir de la Revolución Industrial. Así, en 1800 se alcanzaron, aproximadamente, los 1.000 millo-
nes de habitantes. Desde ese momento, la mejora de las condiciones de vida, de la alimentación, la
prevención y atención médicas, etc... han conducido a un crecimiento exponencial, a pesar de la trá-
gica continuación de los conflictos armados, haciendo que en menos de 200 años la población de
1800 se haya multiplicado por 6.

1.2. DISTRIBUCIÓN DE LA POBLACIÓN EN EL ESPACIO

El reparto de la población mundial no es ni mucho menos homogéneo. Existen determinadas


áreas geográficas en las que existe una enorme concentración de población que contrastan con otras
en las que el poblamiento es nulo o casi nulo.

TAMadrid
5
-Pág.41-
C I E N C I A S S O C I A L E S , G E O G R A F Í A E H I S TO R I A

La densidad de población mide precisamente la concentración poblacional en una determina-


da zona. El dato se expresa en habitantes por kilómetro cuadrado (hab/km2) y se obtiene dividien-
do el total de población de dicha zona entre la superficie de la misma medida en kilómetros
cuadrados.

Trasladado a mapas, coloreando de forma más intensa las zonas con mayor densidad de pobla-
ción, podemos observar cómo las manchas se concentran en determinadas áreas de la geografía
mundial, resultando:

Zonas de gran densidad de población:


- Franja entre el Reino Unido y el Norte de Italia, pasando por Bélgica, Holan-
da y Luxemburgo, Oeste de Alemania y Suiza.
- Noreste de Estados Unidos y Sureste de Canadá, es decir, área de los Grandes
Lagos y costa Atlántica de Norteamérica en la franja de las latitudes 40-45 º
- Subcontinente indio (India, Pakistán, Bangla Desh)
- Este de Asia (China, Japón, Coreas, Vietnam)
- Sureste asiático (Indonesia, Malasia)

Zonas con alta densidad:


- Zona central de México.
- Costa Sureste de Brasil
- Área del Mar del Plata (Argentina, Uruguay)
- Delta del Nilo (Egipto)
- Golfo de Guinea (Nigeria, Ghana, Benin)

Zonas con densidad media:


- Europa
- Franja meridional de la Rusia asiática
- Resto del Sureste asiático (Myanmar, Thailandia, Filipinas)
- Costa Este de Australia y Nueva Zelanda
- Estados Unidos y Centroamérica
- Costas pacíficas del norte de América del Sur (Colombia, Ecuador, Perú)
- África Subsahariana

6 TAMadrid

-Pág.42-
U. D. 2 . - L A P O B L A C I Ó N Y E L E S PA C I O U R B A N O

Zonas de escasa o nula población:


- Latitudes más septentrionales de Norteamérica (Canadá, Alaska) y zona de las
montañas rocosas.
- Cuenca amazónica
- Extremo del cono sur americano (Chile - Argentina)
- Desiertos africanos (Sahara, Namibia, Kalahari)
- Desierto australiano
- Zonas desérticas del Próximo Oriente
- Zonas montañosas asiáticas (Himalaya)
- Zonas desérticas asiáticas (Gobi, Pamir...)

1.3. FACTORES DEL REPARTO GEOGRÁFICO DE LA POBLACIÓN


El reparto poblacional que acabamos de ver se debe a una serie de factores que, en mayor o
menor grado, dependiendo de los casos, han influido en el asentamiento poblacional. Dichos facto-
res los podemos agrupar en factores físicos y factores humanos.

1.3.1. FACTORES FÍSICOS

Son aquellos que dependen de las circunstancias propias del medio natural.
Relieve.- Las zonas montañosas marcan, en principio, un límite al poblamiento huma-
no como consecuencia de la incomunicación, lo accidentado del terreno y las fuertes
pendientes que dificultan las actividades productivas (De ahí, por ejemplo, el despo-
blamiento de la zona de las Montañas Rocosas en Norteamérica)
Costas.- Las costas, por la suavización de las temperaturas y las posibilidades de
comunicación, han favorecido tradicionalmente el asentamiento humano.
Cauces fluviales.- Su aporte hídrico para las actividades agrícolas e industriales
también ha sido un factor de atracción (Nilo, Yang-Tsé, Indo, Ganges)
Clima.- Las condiciones climáticas mediatizan, sin lugar a dudas, la disponibi-
lidad del poblamiento (áreas polares y desérticas frías y cálidas).
Suelos y Recursos del subsuelo.- La aptitud de los suelos para el cultivo y la
presencia de materias primas y otras riquezas en el subsuelo son otro factor de
reclamo para la población.
Todos estos factores se interrelacionan y, ocasionalmente, se sobreponen. Así en
determinadas zonas en las que, por ejemplo, las condiciones climáticas y de riqueza
del suelo no sean idóneas puede, sin embargo, haber concentración puntual de pobla-
ción como consecuencia de la existencia de yacimientos de materias primas.

TAMadrid
7
-Pág.43-
C I E N C I A S S O C I A L E S , G E O G R A F Í A E H I S TO R I A

1.3.2. FACTORES HUMANOS

Son aquellos que dependen de la sociedad humana en general.


Económicos.- La presencia de materias primas, la radicación de industrias, las
facilidades del terreno para las comunicaciones han sido decisivos en el reparto
y movimientos de población.
Históricos.- Aquellas zonas que fueron pobladas desde la antigüedad se han
mantenido, por lo general, con un nivel elevado de poblamiento.
Políticos.- La elección de la capital de los Estados y otras circunscripciones
administrativas favorece la atracción de población para nutrir los servicios inhe-
rentes a la administración de tales entidades. Además, no podemos olvidar las
razones políticas que han motivado traslados masivos de población.

1.4. MOVIMIENTOS NATURALES DE LA POBLACIÓN

Entendemos con tal denominación los cambios que experimenta la población, tanto en cuanto a
su crecimiento como a sus desplazamientos.

1.4.1. NATALIDAD

Es el dato demográfico que nos indica el número de nacimientos que se produce en un periodo
determinado. Lo más habitual es expresar la natalidad en número de nacimientos por cada mil habi-
tantes a lo largo de un año. Su cálculo se obtiene de contabilizar en número de nacimientos produ-
cidos en el año, dividirlo por el número de habitantes y multiplicar la cifra obtenida por 1000.

Por encima del 30‰ se considera que la natalidad es alta.

Otro dato, relacionado con la natalidad y que resulta ilustrativo, es el índice de fecundidad, es
decir, la media de hijos por mujer.

Los factores que influyen en la natalidad son diversos. Las condiciones climáticas y biológicas
no parecen influir tanto como las económicas y de desarrollo cultural. Así, en los países desarrolla-
dos la natalidad es menor que en los subdesarrollados, independientemente de otros factores.

La falta de control sobre la natalidad se ha revelado como el causante principal de la superpo-


blación de áreas en las que las condiciones económicas no son las más apropiadas para el sosteni-
miento de esa población. Esto se ha convertido en un factor decisivo para el desarrollo de los países
del llamado Tercer Mundo.

8 TAMadrid

-Pág.44-
U. D. 2 . - L A P O B L A C I Ó N Y E L E S PA C I O U R B A N O

1.4.2. MORTALIDAD

Es el dato demográfico que nos indica el número de defunciones por cada mil habitantes en
un periodo determinado. Se expresa, al igual que la natalidad, en tantos por mil y se calcula de la
misma forma pero con el dato de número de fallecidos.

Una tasa de mortalidad de más del 15 0/00 es considerada alta. Las tasas más bajas que se regis-
tran hoy rondan y bajan el 10 0/00. El dato es más bajo en algunos países en vías de desarrollo con
poblaciones muy jóvenes y se mantiene o supera el 10 0/00 en países en los que el desarrollo se une
al hecho de tener una población vieja abundante, lo que hace que la tasa se encuentre en el límite de
lo que podríamos llamar inevitable.

Otro dato muy significativo y que desvela el grado de desarrollo de un país, es el de la morta-
lidad infantil. Ésta se calcula dividiendo el número de niños menores de un año muertos por el
número de nacidos y multiplicando esa cifra por 1000. Los países más desarrollados se mueven en
cifras entre el 4 0/00 y el 9 0/00, mientras que algunos países, especialmente en África, alcanzan
cifras alarmantes por encima del 100 0/00.

Los factores económicos y culturales aparecen como los más trascendentes en la determinación
de las causas de las defunciones y de los niveles alcanzados por la mortalidad. Efectivamente, aun-
que determinadas circunstancias naturales influyen poderosamente, es el grado de desarrollo, de
saneamiento de la economía, los hábitos y posibilidades de alimentación, los medios de prevención
y de intervención sanitaria, los que establecen finalmente los datos.

Así, si hablamos de causas de las defunciones, en los países subdesarrollados, las epidemias, las
enfermedades infantiles, la desnutrición, la falta de asistencia sanitaria juegan a favor de altas tasas
de mortalidad. Mientras, en los países desarrollados, las tasas bajas pero cercanas al 10 se explican
por la inevitable muerte de ciudadanos muy mayores tras una buena alimentación y una elevada
atención médica durante su vida. Junto a estos fallecimientos, los producidos entre población joven
o de mediana edad tienen motivos tan diferentes a los de los países desarrollados como los excesos
de alimentación, efectos del tabaco, alcohol, accidentes de tráfico...

1.4.3. CRECIMIENTO VEGETATIVO

Es el dato demográfico que indica el incremento de población producido en un periodo deter-


minado. Se expresa en número de habitantes de incremento por cada 100 preexistentes y se calcula
restando a la tasa de natalidad la de mortalidad. Consecuentemente, el índice de crecimiento vege-
tativo puede ser positivo o negativo, éste señala una disminución de la población.

TAMadrid
9
-Pág.45-
C I E N C I A S S O C I A L E S , G E O G R A F Í A E H I S TO R I A

La relación entre natalidad y mortalidad determina, pues, el crecimiento. De este modo, en los
países en vías de desarrollo se aprecia un crecimiento elevado como consecuencia de unas tasas de
natalidad muy altas y unas tasas de mortalidad contenidas como consecuencia de la intervención de
la asistencia médica y la paulatina mejora de las condiciones de vida. Por su parte, en los países
desarrollados se combinan unas tasas de natalidad muy bajas con tasas de mortalidad también
muy bajas por lo que el crecimiento es bajo o, incluso, negativo.

1.4.4. MODELOS DEMOGRÁFICOS

El análisis de la evolución demográfica de las sociedades ha llevado a concluir la existencia de


unos esquemas de transformación de los comportamientos en función del grado de desarrollo. Los
pasos parten de una modelo con tasas de natalidad y mortalidad altas y finalizan con sociedades
desarrolladas con tasas bajas.
- Modelo demográfico antiguo.- También llamado preindustrial, presenta tasas
de natalidad y mortalidad altas, el crecimiento es, pues, lento y la población muy
joven. Históricamente corresponde a las sociedades europeas anteriores a la
revolución industrial, fase en la que se encuentran los países subdesarrollados.
- Modelo demográfico de transición.- dentro del mismo se producen tres fases:
- En la primera se mantiene una elevada tasa de natalidad pero se inicia
el control de la mortalidad como consecuencia de la disminución de las
causas catastróficas (epidemias, hambrunas) por la mejora de las condi-
ciones de producción de alimentos y la prevención y asistencia sanitaria.
Debido a estas circunstancias, se produce un crecimiento muy alto, es el
inicio de la llamada explosión demográfica. Históricamente, correspon-
de a los momentos iniciales de la Revolución Industrial y es la fase en
la que se encuentran los países en vías de desarrollo.
- En la segunda fase la mortalidad sigue descendiendo y la natalidad ini-
cia un descenso moderado. El crecimiento es muy alto en plena explo-
sión demográfica.
- En la tercera fase, la mortalidad se estabiliza en tasas bajas y la natali-
dad desciende aceleradamente por lo que el crecimiento se ralentiza.
Históricamente, este modelo corresponde a los momentos iniciales de la Revo-
lución Industrial y es la fase en la que se encuentran los países en vías de desa-
rrollo.
- Modelo demográfico moderno.- Las tasas de natalidad y mortalidad se esta-
bilizan en niveles bajos. La compensación entre ambos datos hace que el creci-
miento disminuya o, incluso, se haga negativo. La población se reparte en los
distintos grupos de edad predominando la población vieja.

10 TAMadrid

-Pág.46-
U. D. 2 . - L A P O B L A C I Ó N Y E L E S PA C I O U R B A N O

1.5. MOVIMIENTOS MIGRATORIOS

Los desplazamientos de población de unos lugares a otros con el objeto de modificar definiti-
va o temporalmente su lugar de residencia, se denominan movimientos migratorios. Los términos
emigración e inmigración dependen del punto de observación de referencia; así, si observamos
desde el punto de partida de los desplazamientos diremos que se produce emigración, mientras que
si lo observamos desde el punto de llegada, hablamos de inmigración.

La diferencia entre el número de emigraciones y de inmigraciones constituye el balance migra-


torio que debe sumarse al crecimiento vegetativo para obtener el índice de crecimiento real.

1.5.1. MIGRACIONES INTERIORES

Se trata de los tránsitos de población dentro de un país, los traslados de unas zonas a otras en
función de determinadas circunstancias y condiciones, pueden ser estacionales (por periodos vaca-
cionales, por trabajos temporales...) o permanentes (abandono de las áreas rurales).

1.5.2. MIGRACIONES EXTERIORES

Se trata de los movimientos migratorios entre países.

Las colonizaciones fueron un importante factor de los movimientos migratorios que explican el
poblamiento mayoritario de América y Oceanía.

Los traslados forzosos de población esclava explican la existencia de población de origen afri-
cano en América.

Las condiciones políticas (regímenes totalitarios de distinta índole) y las guerras han motivado
grandes desplazamientos de población.

Las desigualdades económicas son hoy en día el motor principal de estos desplazamientos. Los
países desarrollados son los receptores de población procedente de las áreas subdesarrolladas.

España, que fue durante muchos años un país de emigración (primero hacia América y a partir
de los años 50 del siglo XX hacia Europa), se ha trasformado en un país de inmigración, recibien-
do población procedente de países iberoamericanos y de África.

TAMadrid
11
-Pág.47-
C I E N C I A S S O C I A L E S , G E O G R A F Í A E H I S TO R I A

1.6. ANÁLISIS DE LA POBLACIÓN ACTUAL Y PROSPECTIVAS


1.6.1. DISTRIBUCIÓN POR SEXOS Y EDAD

Por razones biológicas, los nacimientos de niños son ligeramente superiores a los de niñas (105
niños por cada 100 niñas). sin embargo, la relación varones / mujeres no se mantiene igual en las
edades sucesivas, siendo esta relación distinta dependiendo del grado de desarrollo. Así, en los paí-
ses desarrollados, la proporción favorable a los varones va disminuyendo hasta que, a partir de los
30 años se invierte, acrecentándose paulatinamente el predominio de mujeres sobre hombres. Mien-
tras, en los países subdesarrollados, la desproporción hombres - mujeres se acrecienta con el paso
de los años a favor de los varones; la causa es la elevada mortalidad femenina debido a la temprana
maternidad, los numerosos embarazos, la ausencia de una adecuada asistencia y el trabajo agrícola.

La distribución por edad depende de los modelos demográficos, predomina la población joven
(menos de 14 años) en los países con modelos antiguos y la población adulta (15 a 64 años) en los
modelos modernos. En éstos, además, tiene un importante peso la población vieja (más de 65 años)
cuyo volumen es exiguo en los modelos antiguos.

La pirámide de población

Una herramienta muy útil para el análisis de la estructura poblacional de un país es su pirámi-
de de población. Se trata de un gráfico en el que se refleja la distribución de la misma por grupos
de edad y sexos.

Se establecen dos ejes de coordenadas. En los ejes de abscisas se establece la escala de volumen
total o porcentaje de población, a la izquierda los hombres y a la derecha las mujeres. En los ejes de
ordenadas se establece la escala de grupos de edad, normalmente de 5 en cinco años.

Los correspondientes grupos de edad se representan como barras horizontales.

La apariencia del gráfico en una sociedad joven, dentro del modelo demográfico antiguo, se
aproxima, precisamente, a la denominación de pirámide pues las altas tasas de natalidad y de mor-
talidad definen una población de poca edad muy numerosa que va disminuyendo en los siguientes
grupos de edad.

La disminución de la mortalidad propia del modelo de transición se refleja en la aproximación


en cuanto a longitud de las barras representativas y la posterior disminución de la natalidad produ-
ce una fase en la que la pirámide de población parece invertirse, mostrando grupos de edad madu-
ra muy populosos y grupos de poca edad cada vez más reducidos.

12
-Pág.48-
U. D. 2 . - L A P O B L A C I Ó N Y E L E S PA C I O U R B A N O

Finalmente, las sociedades del modelo moderno tienden a equilibrar las barras de los grupos de
edad en valores reducidos.

1.6.2. ACTIVIDAD ECONÓMICA Y DISTRIBUCIÓN EN SECTORES PRODUCTIVOS

Desde el punto de vista económico, la población se divide, inicialmente, en población activa y


población inactiva.

Población activa.- Es la población que, potencialmente, puede trabajar, es decir, los ciudadanos
entre 16 y 64 años que trabajan o buscan trabajo.

Población inactiva.- Son los menores de 16 años, los jubilados y aquellos que por su fortuna o
rentas no necesitan trabajar.

Entre la población activa, dependiendo de las condiciones económicas de cada país, existe un
porcentaje de individuos que no trabaja. Esta población se mide con la tasa de paro (número de
parados por cada 100 individuos de la población activa).

La población activa se mide con la tasa de actividad (número de personas activas por cada 100
habitantes).

Esta población se distribuye en los diferentes sectores económicos:

Sector primario, el de obtención de materias primas y productos energéticos (ganadería, agri-


cultura, pesca, explotación forestal). Es el sector con más población activa en los países subdesa-
rrollados y el que menos mano de obra ocupa en los países desarrollados.

Sector secundario, el de elaboración de productos manufacturados (industria, construcción).


Fue el más numeroso en el proceso de crecimiento de los actuales países desarrollados.

Sector terciario, el de gestión, administración, comercialización, transportes, ocio, defensa,


educación, sanidad... Es el que más mano de obra ocupa en los países desarrollados.

La evolución del reparto de la población en estos sectores es indicativo del proceso de desarro-
llo. Así, en los países subdesarrollados el sector primario es el más numeroso, con un secundario
casi inexistente y un terciario débil. En los países en vías de desarrollo el sector primario se man-
tiene en niveles altos, el secundario crece pero no demasiado y el terciario va engordando paulati-
namente. Por fin, en los países desarrollados, el sector secundario no es demasiado numeroso, el
primario ocupa a un mínimo de población y es el terciario el abrumadoramente mayoritario.

TAMadrid
13
-Pág.49-
C I E N C I A S S O C I A L E S , G E O G R A F Í A E H I S TO R I A

1.6.3. OTROS INDICADORES

Existen otros indicadores del nivel de desarrollo que nos permiten valorar las condiciones de la
población de cada país.

A) Renta per cápita

Es el dato que nos indica la distribución media de la riqueza de un país por habitante. Se obtie-
ne de dividir el Producto Nacional Bruto (PNB), o sea, el montante total de lo producido en el país,
entre los habitantes de ese país. Naturalmente, en absoluto indica que esa distribución sea equitati-
va pues en países con una Renta Per Cápita alta existen grandes bolsas de pobreza o, incluso, los
recursos están en manos de unos pocos privilegiados mientras la mayor parte de la población se
mantiene en niveles de pobreza.

Se considera una renta per cápita baja aquella que está por debajo de los 700 dólares, aproxi-
madamente, y alta la que se sitúa por encima de los 8.625 dólares.

B) Índice de Desarrollo Humano

Se trata de un índice elaborado por la Organización de las Naciones Unidas. Se calcula sobre la
base de tres indicadores:

Esperanza de vida.- La edad media a la que se produce la defunción de los habitantes, será más
elevada cuanto mayor sea el desarrollo pues las condiciones de alimentación y atención sanitaria
serán mejores.

Renta Per Cápita real

Nivel de educación.- En él se tiene en cuenta la tasa de alfabetización y el volumen de pobla-


ción que se encuentra inmerso en los distintos niveles de educación.

El resultado del análisis es un número entre 0 y 1, considerándose, básicamente, la siguiente


escala:

- 0,000 a 0,499.- Desarrollo bajo.


- 0,500 a 0,799.- Desarrollo medio.
- 0,800 a 1,000.- Desarrollo alto.

El IDH puede calcularse para países en general y por grupos en particular, de modo, por ejem-
plo, que se puede analizar el IDH de la población femenina o de las minorías étnicas.

14 TAMadrid

-Pág.50-
U. D. 2 . - L A P O B L A C I Ó N Y E L E S PA C I O U R B A N O

1.6.4. EL FUTURO DE LA POBLACIÓN MUNDIAL

Los análisis que han venido realizando determinados


organismos dependientes de la Organización de las Nacio-
nes Unidas se han revelado sumamente precisos con el paso
del tiempo de modo que sus prospectivas sobre la evolución
de la población mundial en el futuro pueden resultar bastan-
te fiables.

Teniendo en cuenta los distintos elementos y factores


que intervienen en el desarrollo demográfico: economía,
nivel cultural... se observa un afianzamiento del equilibrio
en los países desarrollados que, en consecuencia, manten-
drán un crecimiento casi nulo; mientras que los países en
vías de desarrollo, cada vez más avanzado, han iniciado un
descenso en sus niveles de crecimiento. Por su parte, los países menos desarrollados irán equili-
brando paulatinamente su crecimiento si bien en los próximos años mantendrán un índice de incre-
mento elevado. Se prevé que hacia 2070 la población mundial se estabilice en una cifra ligeramente
por encima de los 12.000 millones de habitantes.

De este modo, la población europea, la que menos crece en la actualidad, verá descender su por-
centaje de participación en la población mundial y serán Asia y, fundamentalmente, África los con-
tinentes que, dado su mayor expansión demográfica en los próximos años, verán crecer su
participación porcentual en el total de la humanidad.

Ahora bien, la gran pregunta tras la constatación de estos datos es ¿Podrá alimentarse toda la
población?¿Dispone la Tierra de recursos suficientes?

Las discrepancias en este punto son las responsables del mayor o menor alarmismo ante las
prospectivas citadas. Parece obvio que el actual sistema y nivel de producción no es suficiente. Qui-
zás hoy en día sea más una cuestión de deficiencias del sistema de distribución de alimentos, mez-
clado con una miope visión comercial de los países desarrollados, lo que hace que existan unos
paradógicos desequilibrios:
- Existencia de problemas económicos y sociales por no saber qué hacer con los
excedentes en algunas áreas desarrolladas (caso, por ejemplo, de los productos
lácteos en Europa).
- Existencia de áreas precariamente alimentadas y que sufren ocasionalmente
dramáticas hambrunas.

TAMadrid
15
-Pág.51-
C I E N C I A S S O C I A L E S , G E O G R A F Í A E H I S TO R I A

Pero estamos hablando de alimentar a una población de más de 12.000 millones de seres, lo que
hace necesario aumentar considerablemente los niveles de producción, de una parte, y, posiblemen-
te, de una intervención eficaz para contener el crecimiento evitando llegar a las cifras previstas. La
inversión en investigación y desarrollo (I+D) debe dirigirse, a parte de otros aspectos, al fomento de
nuevas técnicas que mejoren los rendimientos y lleve las técnicas a las áreas necesitadas, que opti-
mice los aprovechamientos energéticos, de agua, fertilizantes, aplicaciones genéticas, etc... Pero
también cabe procurar limitar el crecimiento fomentando el desarrollo económico y cultural de los
países más desfavorecidos.

2. EL ESPACIO URBANO
2.1. EVOLUCIÓN DE LAS CIUDADES

2.1.1. LAS PRIMERAS CIUDADES

La aparición de las primeras ciudades está asociada a la revolución neolítica. El inicio de la


actividad agraria regular y organizada determinó la sedentarización de los grupos humanos. En esta
situación de hacía necesario edificar viviendas estables y que cubrieran las necesidades familiares.
Estas viviendas debían organizarse en el espacio permitiendo el tránsito entre ellas. Más adelante,
las prácticas tendentes a la mejora de la producción (abonos, sistemas de riego...) permitieron obte-
ner excedentes que podían ser cambiados por otros bienes necesarios. Esto hizo posible dividir el
trabajo de modo que unos ciudadanos se dedicaran a obtener productos alimenticios y otros a fabri-
car otros bienes o comerciar con ellos. La necesidad de organizar las relaciones hizo surgir los pode-
res lo que unido al diferenciado enriquecimiento de los individuos, se tradujo en viviendas de
distintos tamaños y calidades, culminando con los palacios y los templos que se unían al entrama-
do urbano. Por último, las diferencias de desarrollo entre unas ciudades y otras levantó las ambi-
ciones de las más poderosas o incitó a las carentes de recursos lo que condujo a la guerra y ésta a
la necesidad de fortificación para la defensa.

2.1.2. LAS CIUDADES DE LAS PRIMERAS CIVILIZACIONES

En Egipto, Mesopotamia, Palestina, Anatolia, surgen, a partir del IV milenio a.C., las primeras
civilizaciones (civilización deriva de civis: ciudadano). La ciudad es el epicentro de un poder local
que pronto se inscribe en entidades más amplias que constituyen entidades fuertes y expansionistas.

16 TAMadrid

-Pág.52-
-Pág.52-
U. D. 2 . - L A P O B L A C I Ó N Y E L E S PA C I O U R B A N O

En Mesopotamia, ciudades como Ur, Akkad, Assur, Nínive o Babilonia constituyeron centros
de irradiación de poder que forjaron grandes imperios. Poseían fuertes murallas en cuyo interior se
situaban palacios y viviendas, con zonas comerciales. En el centro, se encontraba el templo presi-
diendo la ciudad desde la zigurat piramidal con sus espacios administrativos, comerciales y religio-
sos, sus almacenes, talleres y demás dependencias.

En Egipto, la individualidad de las ciudades, desde el punto de vista político, pronto se vio aho-
gada por el peso del poder del faraón, sustentado en un elaborado entramado religioso que dio esta-
bilidad al Imperio. El interés de los faraones o de los gobernadores hizo crecer especialmente
algunas ciudades, adornadas con la construcción de palacios y templos esplendorosos. Tal es el caso
de Tebas, Memphis, Luxor, Tinis, Heliópolis, o Abú Simbel.

En Anatolia surgieron también importantes fuerzas económico - políticas, especialmente el


Imperio Hitita, con su sólidamente fortificada capital Hattusa.

Por último, en Palestina, en la encrucijada entre las tres áreas precedentes, aparecieron también
ciudades y poderes locales que, la mayor parte de las veces, estaban bajo el control de alguno de los
grandes imperios colindantes. Ciudades importantes fueron Jericó (que pasa por ser la primera ciu-
dad importante de la historia) y Jerusaleén.

2.1.3. LAS POLIS GRIEGAS

En la civilización griega la ciudad tiene un protagonismo total. La mayor parte de la historia


griega está marcada por la individualidad de sus ciudades - Estado, las polis (palabra de la que deri-
va política). Solían tener una plaza, el ágora, centro de la vida pública y se dotaban de templos y
teatros. Atenas, Esparta, Tebas, Corinto, Rodas, son ejemplos de importantes polis griegas clásicas.

2.1.4. LA CIVITAS ROMANA

La civilización romana pasa por ser un paradigma de organización. La ciudad romana constitui-
da de nuevo cuño seguía unas pautas incluso religiosas para su fundación y trazado lo que incluía la
consulta a los augures y arúspices sobre el momento y lugar más adecuado para que, según la tradi-
ción inaugurada trágicamente por RÓMULO y REMO, un arado tirado por bueyes trazara las dos
vías o calles principales perpendiculares, el cardo y el decumanus, y el perímetro que delimitaba el
espacio interior que quedaba consagrado.

En el punto de cruce de las dos calles principales se situaba el foro, centro administrativo, jurí-
dico, religioso y comercial. Alrededor de su plaza central se situaban los templos, los edificios para
administración de justicia y mercado (basílicas) y otros comercios en locales organizados (taber-

TAMadrid
17
-Pág.53-
C I E N C I A S S O C I A L E S , G E O G R A F Í A E H I S TO R I A

nae). Los servicios públicos, en manos de funcionarios elegidos, dotaban a la ciudad de agua por
medio de acueductos y fuentes públicas (a veces, incluso con agua corriente hasta las viviendas),
evacuaban las aguas residuales por medio de alcantarillas (cloacas), completaban la comodidad y la
belleza de la ciudad con el empedrado de las calles, la canalización de las aguas de lluvia, soporta-
les, galerías y monumentos conmemorativos (columnas, estatuas, arcos de triunfo) y facilitaban
el entretenimiento con termas (para el baño, el ejercicio, la lectura y el deporte), anfiteatros (para
los espectáculos sangrientos: luchas de gladiadores y de fieras), circos (para las carreras de carros)
y teatros.

Además de Roma, la urbs, podemos citar, por su proximidad, algunas ciudades hispanas como
Itálica (cerca de Sevilla) o Mérida.

2.1.5. LAS CIUDADES MEDIEVALES

Europa sufrió una importante crisis urbana en los comienzos de la Edad Media, tras las inva-
siones germánicas. En los momentos en los que esta crisis tiene lugar surge un poder con gran
empuje: el Imperio Musulmán que, desde La Meca, se extiende por el norte de África y Asia. Sus
ciudades (Bagdad, Damasco, Alejandría, Córdoba) alcanzan gran esplendor. Están dotadas de
murallas, reforzadas con torres y contrafuertes, tienen como lugar comunitario la mezquita, cuyo
minarete preside toda la ciudad, y se dotan de una ciudad noble, la alcazaba, donde se encuentran
palacios y dependencias administrativas y comerciales con su alcázar fortificado.

Los restos territoriales del imperio romano, en la zona oriental del mediterráneo, mantuvieron
hasta el siglo XV un cada vez más efímero esplendor. Sin embargo, éste no lo perdió nunca la capi-
tal, la antigua Constantinopla, ahora Bizancio y, tras la conquista de los turcos, Estambul.

Por su parte, la Europa cristiana fue poco a poco saliendo de la crisis inicial y desde unas bases
rurales que dieron poder a una hegemónica nobleza, se fueron construyendo nuevas relaciones eco-
nómicas en las que las ciudades fueron cobrando otra vez protagonismo. Los burgos y sus ciuda-
danos (los burgueses) fueron despertando las ciudades, necesariamente fortificadas y cuyas
edificaciones más elegantes, costosas y monumentales eran las catedrales e iglesias. El plano irre-
gular, el hacinamiento en calles estrechas, la insalubridad y los barrios exteriores (arrabales), fruto
del crecimiento de la población de la ciudad, casi siempre por inmigración, eran típicos de estas ciu-
dades, características que tenían en común con las musulmanas y bizantinas. Entre las ciudades cris-
tianas medievales destacaremos París, Aquisgrán, Viena, Toledo...

No podemos cerrar este apartado sin citar el urbanismo que en el tiempo de la Edad Media del
que llamamos Viejo Mundo (Europa, norte de África y Próximo Oriente), se observó en otras lati-
tudes como en América o el Lejano Oriente. Efectivamente, la civilización India, la China o la Japo-

18 TAMadrid

-Pág.54-
U. D. 2 . - L A P O B L A C I Ó N Y E L E S PA C I O U R B A N O

nesa, en Asia y las Culturas americanas de los Toltecas, Mayas, Aztecas, Quechúa... alcanzaron
importantes desarrollos urbanos que dejando a un lado las enormes diferencias estéticas en las for-
mas de los edificios o los elementos decorativos, llegaron a soluciones muy similares a las europe-
as e incluso, en algunos casos, superiores. Citemos aquí algunos ejemplos de ciudades de estos
mundos: Tiahuanaco, Tenochtitlán, Chichén-Itzá, Palenque, Cuzco (en América); Kyoto
(Japón); Pekín, Nanking, Hang-Cheu (China); Lahore, Delhi, Benarés (India).

2.1.6. EL DESARROLLO URBANO TRAS LA EDAD MEDIA

La ciudad, a partir de la Edad Moderna, mantuvo su importancia como centro de comercio y


actividad artesano-industrial. Sin embargo, su crecimiento fue sostenido hasta la Revolución
industrial. A partir del siglo XIX, las ciudades comenzaron a crecer convirtiéndose en núcleos
industriales y comerciales primero y, posteriormente, en centros de decisión y servicios.

El despegue industrial se tradujo en una indiscriminada y nada planificada instalación de facto-


rías, éstas atrajeron a la mano de obra desplazada del campo que acrecentó la demografía de la ciu-
dad. Ésta supera por completo los límites de las antiguas murallas (ya inútiles por los novedosos
usos de la guerra) y se expande inicialmente de modo incontrolado y después en barrios de ensan-
che planificados y diseñados en función de los grupos sociales a los que cada área se destinaba.

El crecimiento territorial de la ciudad eleva el protagonismo de los sistemas de comunica-


ción y transporte en su interior, tanto para el desplazamiento de las personas hacia sus lugares de
trabajo como para el suministro de los bienes de consumo. Además, la conjunción de residuos y
humos producidos por las factorías, insuficientes sistemas de eliminación de aguas fecales, ruidos
y gases de los sistemas de trasporte, llevó, a mediados del siglo XX a una crítica situación de con-
taminación que, al final del siglo, se había mitigado en parte por la decidida actuación de los ya
concienciados poderes públicos.

2.2. LA CIUDAD ACTUAL


El primer problema al que nos enfrentamos en el análisis de Geografía Urbana es la propia defi-
nición de ciudad. En la breve visión histórica que precede hemos visto el surgimiento y evolución
de las ciudades que, en un principio, y, hasta no hace muchos años, eran todas las agrupaciones
humanas en núcleos, pequeños o grandes, que se dotaban de sistemas comunes. La distinción entre
el habitat rural y el urbano era sencilla pues el primero se caracterizaba por viviendas aisladas y el
segundo por concentración de edificios, generalmente rodeados de una muralla. En ambos casos, la
actividad económica se basaba en la agricultura (muchos ciudadanos salían a diario de las murallas

TAMadrid
19
-Pág.55-
C I E N C I A S S O C I A L E S , G E O G R A F Í A E H I S TO R I A

y se dirigían a trabajar a los campos circundantes), si bien en la ciudad había, además, comercian-
tes y artesanos.

Pero hoy en día debemos establecer una distinción diferente entre el medio rural y el de las ciu-
dades (y aquí, intencionadamente, omitimos el término urbano). Efectivamente, existen explotacio-
nes agropecuarias que van acompañadas de habitat disperso, pero también existen núcleos urbanos
pequeños y orientados casi exclusivamente a la actividad agraria.

Estos razonamientos nos llevan a establecer un criterio demográfico y económico para dife-
renciar las ciudades de los pueblos. Así, podremos determinar como características que debe reu-
nir un núcleo urbano al que llamemos ciudad:
- Población numerosa.
- Trazado complejo y organizado de calles, avenidas, plazas, espacios ajardina-
dos...
- Edificación concentrada y de usos variados (vivienda, ocio, administración,
producción...).
- Actividades económicas centradas principalmente en la industria y los servi-
cios (sectores productivos secundario y terciario).

2.2.1. LA FORMA DE LA CIUDAD

En el estudio y análisis de una ciudad nos importa, en primer término (aunque no necesaria-
mente desde el punto de vista de su trascendencia), su fisonomía, su morfología.

Un primer aspecto que debe ser tenido en cuenta es su situación en el territorio circundante, su
posición más o menos estratégica y el tipo de relación con dicho territorio. El hinterland, nombre
con el que se designa al área con la que se relaciona la ciudad, incluye espacios rurales y otros
espacios urbanos más pequeños que de una u otra forma dependen del núcleo principal de modo
directo. El relieve y otras condiciones naturales, así como circunstancias históricas, pueden influir
en la determinación de la amplitud y características de estos espacios.

En segundo lugar, resulta importante el análisis del emplazamiento de la ciudad. El predomi-


nio de los lugares elevados y fácilmente defendibles, dejó paso, con la modificación de las necesi-
dades urbanas, a la búsqueda de espacios llanos, próximos a corrientes fluviales o costeros y con
cómodas comunicaciones. Naturalmente, la pervivencia de la importancia de ciudades nacidas en
momentos en los que imperaban otros condicionantes, hace que los barrios nuevos busquen esos
espacios al lado de los núcleos antiguos; aunque históricamente se hayan dado casos de abandono
de la ciudad alta para fundar un nuevo núcleo en el llano.

20 TAMadrid

-Pág.56-
U. D. 2 . - L A P O B L A C I Ó N Y E L E S PA C I O U R B A N O

Otro aspecto que hay que estudiar es el plano de la ciudad, su trazado. Las circunstancias his-
tóricas son fundamentales en este punto. Aunque, por lo general, las grandes ciudades, debido a su
dilatada historia, suman distintos tipos de planos en su entramado, podemos distinguir una serie de
trazados tipo:

- Irregular.- Es propio de las ciudades viejas en las que el crecimiento de su


caserío tuvo lugar de un modo lento y sin planificación. Las calles son estrechas
y no siguen líneas organizadas y los edificios se amontonan sin orden.

- Ortogonal o hipodámico.- Las calles se cruzan perpendicularmente forman-


do una cuadrícula. HIPODAMO DE MILETO lo ideó en el mundo griego y pre-
dominó en las fundaciones de la época Clásica. Después fue utilizada en la época
de la expansión europea en las colonizaciones y, más tarde, en los ensanches
decimonónicos en las ciudades industriales.

- Lineal.- Un eje o avenida central sirve de médula espinal a un entramado de


vías secundarias. Tanto la avenida como las restantes vías pueden ser rectas o
curvadas.

- Concéntrico.- Combina vías circulares concéntricas con otras radiales que la


comunican.

La fisonomía de la ciudad depende también de los edificios que la forman. La clase de los mis-
mos, su tamaño, diseño y funcionalidad, definen el carácter de la ciudad.

Hoy en día, por fin, la morfología de la ciudad, combinando todos estos factores, depende de la
planificación que los organismos administradores de la misma establezcan. Esta planificación parte
de la determinación de los distintos usos del suelo, clasificando éste de un modo oficial en suelo
para vivienda y dotaciones sociales, zonas verdes, polígonos industriales y terrenos para explotación
agropecuaria.

2.2.2. ESTRUCTURA URBANA

La estructura de las ciudades, la organización de su plano, se articula en distintas áreas funcio-


nales que se se reparten en dos áreas básicas: el centro y la periferia.

El centro incluye básicamente tres tipos de espacios:

- El casco antiguo.- Dependiendo de la ciudad de que se trate, el casco urbano


es un área de semiabandono, con vivienda precaria y ruinosa y escasa actividad
o bien una zona protegida por la actuación municipal tendente a la recuperación
del área como zona de atracción turística por el valor histórico de sus edificios

TAMadrid
21
-Pág.57-
C I E N C I A S S O C I A L E S , G E O G R A F Í A E H I S TO R I A

y en la que las actividades hosteleras aprovechan esa coyuntura en un espacio


ganado para los desplazamientos peatonales.

- El Centro de Negocios.- En algunos casos se yergue en medio del casco anti-


guo u ocupa un núcleo próximo al mismo. En él se concentran los grandes edi-
ficios de las empresas y las entidades financieras. Es el barrio de los negocios
con edificios modernos y funcionales en los que se vuelcan las innovaciones
arquitectónicas y donde el espacio se gana en altura. En algunas ciudades, este
centro de negocios se expande por determinadas vías que salen del centro o,
incluso, se desplazan, a través de vías radiales, al extrarradio.

- Barrios residenciales.- En el centro de la ciudad muchos edificios de vivien-


das han ido siendo sustituidos por los de negocios. Sin embargo, permanecen
barrios residenciales de bloques de viviendas en manzanas cerradas o pequeños
edificios unifamiliares o de un número reducido de viviendas.

En la periferia se sitúan:

- Barrios residenciales.- Se trata de barrios planificados para dar cabida a la


creciente población atraída por la importancia de las actividades económicas de
la ciudad y su crecimiento demográfico y que, por tanto, disfrutan de dotaciones
organizadas. En unos casos se trata de áreas con bloques de viviendas en man-
zanas abiertas en las que el edificio convive con espacios verdes, en otros se trata
de viviendas unifamiliares adosadas o exentas. Estos barrios precisan de siste-
mas de trasnporte eficaces que comuniquen con las zonas donde se ubican los
puestos de trabajo.

- Zonas industriales.- Las industrias que en la primera Revolución Industrial se


ubicaron en el interior del casco urbano han ido desapareciendo y dando cabida
a zonas verdes o residenciales y de dotaciones sociales, de modo que las moder-
nas industrias han sido ordenadas en zonas específicas, los polígonos industria-
les, con dotaciones adecuadas para el suministro, la evacuación de residuos y el
transporte de los productos.

- Centros comerciales.- El moderno concepto del suministro familiar ha hecho


primar la creación de grandes centros comerciales que se ubican en los espacios
periféricos con disponibilidad de suelo.

- Barrios de infraviviendas.- Aunque presentes en casi todas las ciudades, son


típicos de los núcleos más poblados y con mayor actividad económica. La caren-
cia de suelo edificable, los trabajos precarios y los conceptos de vida seminó-
mada de algunas comunidades, favorecen la aparición de áreas en las que las
viviendas se improvisan utilizando materiales de desecho. Los barrios que así
surgen, carecen de dotaciones sociales, sanitarias y de transporte lo que radica-
liza su marginalidad.

22 TAMadrid

-Pág.58-
U. D. 2 . - L A P O B L A C I Ó N Y E L E S PA C I O U R B A N O

Mención aparte merecen algunos espacios específicos, como:


- Las zonas verdes.- Salvo excepciones, los cascos viejos carecen de estos espa-
cios que aparecen ya en las zonas de ensanche decimonónicas y son protagonis-
tas de algunas de las más modernas actuaciones urbanísticas. Su importancia
para el ocio, la estética y, sobre todo para las condiciones atmosféricas de la ciu-
dad ha sido tenida en cuenta por todos los planes urbanísticos contemporáneos.
- Dotaciones deportivas y de ocio.- Generalmente en la periferia.
- Complejos sanitarios.- Los grandes hospitales se combinan con centros asis-
tenciales de barrio que acercan la salud a los habitantes de la ciudad.
- Complejos educativos.- Colegios y Universidades constituyen espacios defi-
nidos que en época reciente han buscado la periferia o incluso el extrarradio de
las ciudades al aumentar la demanda para la que resultaban insuficientes los cen-
tros situados en el interior.
- Instalaciones militares.- Hasta hace pocos años muchas instalaciones milita-
res se encontraban en el interior de los núcleos urbanos. Las necesidades de
comunicación y movilidad, unido a las complicaciones que el movimiento de
determinado material militar puede provocar en el tráfico urbano, han ido obli-
gando a llevar estas instalaciones a zonas periféricas o, incluso, alejadas de la
ciudad.

En el hinterland de las grandes ciudades se encuentran núcleos urbanos secundarios que


constituyen el extrarradio de la gran urbe. Pueden ser las llamadas ciudades - dormitorio, donde
habita población que diariamente se desplaza a sus puestos de trabajo en la urbe, o núcleos indus-
triales secundarios a los que también acude población de los barrios residenciales de la ciudad prin-
cipal.

El crecimiento de la gran ciudad y el simultáneo de los núcleos del extrarradio produce, en algu-
nos casos, el contacto físico de los límites de estos núcleos, creando lo que se denomina conurba-
ción, una gran metrópoli que, aunque dividida en distintas administraciones locales, precisa
soluciones comunes para los mismos problemas. A veces este hecho redunda en la incorporación
administrativa de la ciudad pequeña en la grande. Algunas de esas grandes conurbaciones en el
mundo son México DF, Río de Janeiro, Buenos Aires, París, Londres o New York-New Jersey,
y, de un modo más modesto, el área urbana de Madrid, unida físicamente con municipios como
Alcobendas, San Sebastián de los Reyes, Coslada, San Fernando de Henares, etc... o el de Barcelo-
na, unida con L'Hospitalet, El Prat, Cornellá, Esplugues, Sant Adriá, Badalona, Sta. Coloma...

2.2.3. LOS TRANSPORTES Y LAS REDES DE COMUNICACIÓN

Tras analizar la realidad de las ciudades del mundo de hoy, resulta innegable la importancia de
los transportes. La necesidad de traslados masivos de población de los lugares de residencia a los de

TAMadrid
23
-Pág.59-
C I E N C I A S S O C I A L E S , G E O G R A F Í A E H I S TO R I A

trabajo y ocio y la de transporte de los productos de consumo, genera una demanda considerable de
medios de transporte. Éste se desglosa en dos grandes bloques:
- El transporte particular, en vehículos pequeños: bicicletas, motos y automóviles.
- Los transportes colectivos.

Los primeros, más autónomos para sus usuarios, generan enormes complicaciones para el trá-
fico rodado de una red viaria que no crece al ritmo del parque automovilístico. Los segundos, deter-
minados por los trazados que las empresas o los organismos públicos consideran (no siempre con el
mejor criterio) más adecuados, se ven mediatizados por el particular y por las aglomeraciones y lar-
gas esperas. Estos transportes se dividen en:
- Transportes de superficie: Taxis, autobuses, tranvías...
- Transportes suburbanos: Ferrocarril metropolitano.

Las grandes concentraciones y el crecimiento de sus hinterland amplían las áreas de extrarradio
y conceden protagonismo al Ferrocarril de cercanías o, incluso, a transbordadores fluviales o marí-
timos.

El gran reto de los grandes núcleos urbanos y sus áreas de influencia es la adecuada coordina-
ción de todas las actuaciones tendentes a hacer más eficaz, rápido y seguro el transporte. Ello pre-
cisa de una red moderna y amplia que permita combinar adecuadamente el transporte individual y
el colectivo.

Las grandes autopistas radiales de acceso al interior desde el extrarradio y desde núcleos más
lejanos, debe combinarse con sistemas, bien subterráneos o de circunvalaciones radiales con el fin
de ampliar las opciones de establecimiento de rutas para evitar aglomeraciones. El interior del casco
urbano tiende a quedar limitado al tráfico de servicios y transporte público. Toda la red debe estar
vigilada y controlada por servicios automáticos y humanos de control del tráfico.

El transporte colectivo de superficie resulta más eficaz cuando se reservan vías específicas
para su circulación (carril-bus, vías de tranvía), mas una adecuada dotación de medios, en cuanto a
número, calidad y comodidad, resulta imprescindible para persuadir al usuario potencial de la con-
veniencia de utilizarlo.

El ferrocarril metropolitano se ha mostrado, en todas las ciudades en las que existe, como un
transporte eficaz gracias a su independencia de las condiciones del tráfico. El ferrocarril de cer-
canías permite completar la oferta del anterior conectando con los mismos criterios el extrarradio.

En todos los casos, la dotación de medios modernos y cómodos, la regulación adecuada del trá-
fico y el aumento de la frecuencia de los servicios, resulta imprescindible para hacer auténticamen-
te eficaces sus esfuerzos.

24 TAMadrid

-Pág.60-
U. D. 2 . - L A P O B L A C I Ó N Y E L E S PA C I O U R B A N O

Un aspecto de gran importancia, sobre todo en los países desarrollados, en relación con el trans-
porte, es el de los accidentes de tráfico, una de las principales causas de muerte en estos países.
La conducción temeraria bajo los efectos del alcohol u otras drogas, los excesos de velocidad y
las malas condiciones de algunas carreteras, son las causas principales de estos accidentes.

2.2.4. LOS PROBLEMAS DE LAS CIUDADES CONTEMPORÁNEAS

Todos los aspectos que hemos ido analizando nos permiten valorar los problemas a los que se
enfrentan los ciudadanos y los administradores de las grandes ciudades del mundo.

El abastecimiento.- Es un aspecto de una importancia decisiva. En principio, el interés por sus


propios beneficios de los suministradores hace que los abastecimientos estén en gran medida ase-
gurados. Sin embargo, las complicaciones que supone, los gastos de transporte y distribución aña-
didos, generan un cierto encarecimiento que se compensa con la competencia en la oferta.

La contaminación atmosférica.- Generada por el tráfico rodado, las emisiones de las indus-
trias y de los sistemas de calefacción. Su control es necesario para limitar las enfermedades respi-
ratorias.

El tráfico rodado.- Relacionado con lo anterior, ya ha sido tratado en el apartado precedente.

La eliminación de residuos sólidos.- El volumen de residuos generados es, lógicamente, pro-


porcional a la población. Su eliminación requiere sevicios adecuados de recogida de basuras. La
concienciación de la importancia del reciclaje ha llevado en muchas ciudades a diferenciar selecti-
vamente los residuos, lo cual complica la labor de recogida.

El tratamiento de las aguas residuales.- Imprescindible si se quiere evitar la contaminación de


las aguas de ríos y mares cercanos a las ciudades.

La contaminación acústica.- El tráfico rodado, algunas actividades de ocio, los aeropuertos,


producen un exceso de ruidos que afectan a la salud física y psíquica.

La marginalidad.- La superpoblación y las desigualdades sociales motivan la existencia de


áreas y grupos marginales en los que terminan anidando los problemas de la delincuencia, la dro-
gadicción, el alcoholismo. El desarraigo y la falta de expectativas de parte de la juventud afecta a
estas circunstancias y se complica con la violencia callejera. Una adecuada política educativa, la
concienciación social, la educación y un justo reparto de la riqueza podrán mitigar los efectos de
estos problemas.

TAMadrid
25
-Pág.61-
C I E N C I A S S O C I A L E S , G E O G R A F Í A E H I S TO R I A

2.2.5. LAS GRANDES CIUDADES DEL MUNDO

Los grandes núcleos urbanos se encuentran, sobre todo, en las áreas de mayor densidad de
población que vimos más arriba. Además, existen algunos otros núcleos aislados de dichas áreas.

Por otra parte, hallamos importantes diferencias entre las ciudades del mundo desarrollado
y las de los países en vías de desarrollo. En éstas, los niveles de infravivienda, la marginalidad
y los problemas que hemos analizado en el apartado anterior se acrecientan; mientras que en las pri-
meras, aunque los problemas existen, el nivel de vida es mucho mayor y las dotaciones sociales y
asistenciales son mucho mejores.

Haciendo una selección, necesariamente parcial, de las ciudades del mundo, elijamos un peque-
ño pero significativo ramillete:

Londres es una gran ciudad europea, capital del Reino Unido. El centro histórico, con los edi-
ficios administrativos tradicionales y modernos, es adyacente al centro de negocios, la city, con
grandes rascacielos y edificios vanguardistas. Los barrios residenciales tienen el típico sabor britá-
nico a base de edificios de dos o tres pisos con una o varias viviendas.

New York es posiblemente la ciudad más conocida del mundo. Su corta historia le priva de un
casco viejo: la original New Amsterdam quedó sepultada bajo el centro de negocios de la isla de
Manhattan. Las zonas marginales se encuentran en el área central mientras que las zonas residen-
ciales de lujo están en la periferia.

México sigue, como otras ciudades iberoamericanas, la herencia colonial de una gran plaza
central y planos ortogonales en la parte central. A ello se han añadido, primero, barrios de residen-
cias de lujo y grandes barriadas de clase media, completándose con extensas zonas populares y de
infravivienda.

Lagos y Kinshasha son las más grandes ciudades de África. En ellas predominan las vivien-
das modestas, incluso las chabolas tradicionales, que se amontonan alrededor de un centro de ori-
gen colonial, muy deteriorado, adyacente a un núcleo de negocios poco desarrollado.

Hong-Kong, Singapur o Tokio nos pueden servir de ejemplo como ciudades asiáticas caracte-
rísticas del desarrollismo de algunos países de la zona. El crecimiento económico ha favorecido la
aparición de grandes centros de negocios con altos edificios que compensan la falta de suelo y
barrios de lujo que contrastan con barrios periféricos marginales.

Calcuta, contrasta con las anteriores ciudades asiáticas, aproximándose a las condiciones urba-
nísticas de las ciudades africanas. Aquí la infravivienda predomina y los niveles de pobreza alcan-
zan cotas inconcebibles.

26 TAMadrid

-Pág.62-
U. D. 2 . - L A P O B L A C I Ó N Y E L E S PA C I O U R B A N O

2.2.6. EL URBANISMO EN ESPAÑA

La mayor parte de la población española, más del 75%, vive en ciudades. El conjunto de Espa-
ña nos presenta una estructura característica en la que Madrid ocupa una posición central desde la
que irradian las principales vías de comunicación por carretera y ferrocarril. La capital se relaciona
radialmente con los demás núcleos importantes y se proyecta por vía aérea hacia otros núcleos euro-
peos y mundiales.

Barcelona cumple un papel similar irradiando hacia Cataluña y las regiones limítrofes, además
de ser un nudo de comunicaciones con proyección internacional.

Valencia, Sevilla, Bilbao, Málaga, Zaragoza, cumplen un papel similar con carácter regional
y, de un modo más limitado, otras capitales de provincia.

Por otra parte, se aprecia un mayor grado de urbanización en las áreas litorales que en el inte-
rior lo que se debe a los importantes movimientos migratorios centrífugos que han llevado a la
población rural del interior a buscar mejores perspectivas.

TAMadrid
27
-Pág.63-
C I E N C I A S S O C I A L E S , G E O G R A F Í A E H I S TO R I A

RESUMEN

LA POBLACIÓN Y LOS RECURSOS


La población humana
Población actual: unos 6.000 millones.

El crecimiento fue lento hasta la Edad Moderna, acelerándose sobre todo a partir de la Revolu-
ción Industrial. En 1800 se alcanzaron, aproximadamente, los 1.000 millones.

Desde entonces crecimiento exponencial.

Distribución de la población en el espacio


La densidad de población se expresa en habitantes por kilómetro cuadrado (hab/km2).

Factores del reparto geográfico de la población


Factores físicos
Relieve
Costas
Cauces fluviales
Clima
Suelos y Recursos del subsuelo

Factores humanos
Económicos
Históricos
Políticos

Movimientos naturales de la población


Natalidad

Número de nacimientos por cada mil habitantes a lo largo de un año.

Índice de fecundidad, es decir la media de hijos por mujer.

28 TAMadrid

-Pág.64-
U. D. 2 . - L A P O B L A C I Ó N Y E L E S PA C I O U R B A N O

Mortalidad

Número de defunciones por cada mil habitantes en un año.

Mortalidad infantil. Niños muertos por cada 1000 nacidos.

Crecimiento vegetativo

Número de habitantes de incremento por cada 100 preexistentes

Tasa de natalidad - Tasa de mortalidad.

Modelos demográficos

- Modelo demográfico antiguo.- Natalidad y mortalidad altas.

- Modelo demográfico de transición.- Tres fases:

- En la primera se mantiene una elevada tasa de natalidad pero se inicia el control de la


mortalidad como consecuencia de la disminución de las causas catastróficas (epidemias,
hambrunas).

- En la segunda fase la mortalidad sigue descendiendo y la natalidad inicia un descenso


moderado.

- En la tercera fase, la mortalidad se estabiliza en tasas bajas y la natalidad desciende ace-


leradamente por lo que el crecimiento se ralentiza.

- Modelo demográfico moderno.- Las tasas de natalidad y mortalidad se estabilizan en niveles


bajos.

Movimientos migratorios
Los desplazamientos de población: emigración e inmigración.

Balance migratorio: diferencia entre el número de emigraciones y de inmigraciones.

Crecimiento real de la población: crecimiento vegetativo más balance migratorio.

- Migraciones interiores.- Pueden ser estacionales o permanentes.


- Migraciones exteriores.- Colonizaciones, traslados forzosos, búsqueda de fortuna.

TAMadrid
29
-Pág.65-
C I E N C I A S S O C I A L E S , G E O G R A F Í A E H I S TO R I A

Análisis de la población actual y prospectivas


Distribución por sexos y edad.
Nacimientos de niños son ligeramente superiores a los de niñas (105 - 100)
En países desarrollados, a partir de los 30 años predominio de mujeres.
En países subdesarrollados, la desproporción se acrecienta a favor de los varones.
La pirámide de población: gráfico en el que se refleja la distribución de la misma por grupos
de edad y sexos.

Actividad económica y distribución en sectores productivos


Población activa.- La que, potencialmente, puede trabajar.
Población inactiva.- Los menores, los jubilados y los rentistas.
Tasa de actividad.- Número de personas activas por cada 100 habitantes.

Esta población se distribuye en los diferentes sectores económicos:


Sector primario, el de obtención de materias primas.
Sector secundario, el de elaboración de productos manufacturados.
Sector terciario, el de los servicios.

Otros indicadores

C) Renta per cápita


Producto Nacional Bruto (PNB), entre los habitantes de ese país.

D) Índice de Desarrollo Humano


Esperanza de vida
Renta Per Cápita real
Nivel de educación

El futuro de la población mundial


Se prevé hacia 2070: por encima de los 12.000 millones de habitantes.
La población europea, la que menos crece.Asia y África las que más.

EL ESPACIO URBANO
Evolución de las ciudades
La aparición de las primeras ciudades está asociada a la revolución neolítica.

30 TAMadrid

-Pág.66-
U. D. 2 . - L A P O B L A C I Ó N Y E L E S PA C I O U R B A N O

Hasta la Edad Moderna plano irregular y amuralladas


Desde Revolución Industrial, crecimiento y problemas (contaminación, marginalidad)
La ciudad actual
Características:
- Población numerosa.
- Trazado complejo y organizado de calles, avenidas, plazas, espacios ajardinados...
- Edificación concentrada y de usos variados
- Actividades económicas centradas principalmente en la industria y los servicios
La forma de la ciudad
Trazado.
- Irregular
- Ortogonal o hipodámico
- Lineal
- Concéntrico
Estructura urbana
Centro:
- El casco antiguo
- El Centro de Negocios
- Barrios residenciales
Periferia:
- Barrios residenciales
- Zonas industriales
- Centros comerciales
- Barrios de infraviviendas
Espacios específicos:
- Las zonas verdes
- Dotaciones deportivas y de ocio
- Complejos sanitarios
- Complejos educativos
- Instalaciones militares

TAMadrid
31
-Pág.67-
C I E N C I A S S O C I A L E S , G E O G R A F Í A E H I S TO R I A

Hinterland: espacio de influencia socio-conómica de la ciudad


Ciudades - dormitorio

Conurbación, conjunto de ciudades próximas, unidas por su plano.

Los transportes y las redes de comunicación


- El transporte particular
- Los transportes colectivos.
- Transportes de superficie
- Transportes suburbanos

Los problemas de las ciudades contemporáneas


El abastecimiento
La contaminación atmosférica
El tráfico rodado
La eliminación de residuos sólidos
El tratamiento de las aguas residuales
La contaminación acústica
La marginalidad.

Las grandes ciudades del mundo


Países en vías de desarrollo. grandes niveles de infravivienda y la marginalidad.
Países desarrollados: mayor nivel de vida y dotaciones sociales y asistenciales.

El urbanismo en España
Más del 75 %, vive en ciudades.
Mayor grado de urbanización en las áreas litorales que en el interior.

32 TAMadrid

-Pág.68-
U. D. 2 . - L A P O B L A C I Ó N Y E L E S PA C I O U R B A N O

EJERCICIOS DE COMPROBACIÓN

11. Si tenemos un pueblo con 10.000 habitantes y se han registrado 10 nacimientos en el año, ¿Cuál
es el índice de natalidad?

A. 1 B. 2,8 C. 11,2 D. 10

2. ¿Cuál de los siguientes es uno de los modelos demográficos?

A. Antiguo B. Joven C. Maduro D. Viejo

3. ¿Qué es la densidad de población?

A. El número de habitantes por provincia y kilómetro


B. El número de habitantes por provincia y región kilómetro
C. El número de habitantes por provincias, regiones y países kilómetro
D. El número de habitantes por kilómetro cuadrado

4. Para la representación de una población por edades se utiliza el sistema de pirámides. ¿Cómo sería
la pirámide que se formaría con un grupo muy alto de edad joven y muy bajo de edad adulta?

A. Con forma de triángulo equilátero B. Un cilindro casi perfecto


C. Con forma de triángulo isósceles D. En forma de barril

5. La población activa de cualquier país está dividida en tres sectores: agricultura, industria y ser-
vicios. ¿Cuál de éstos es el que abarca mayor número de personas en los países desarrollados?

A. Agricultura B. Industria C. Servicios D. Los dos últimos

6. La natalidad indica:

A. El número de mujeres en edad fértil


B. Los nacimientos por mil habitantes en un año
C. Los nacimientos menos las defunciones
D. Los nacimientos en relación a la población activa

7. El crecimiento vegetativo se obtiene de:

A. La tasa de natalidad menos la de mortalidad


B. La tasa de paro dividida por la población activa
C. La tasa de natalidad más las migraciones
D. Las defunciones más la tasa de sustitución

TAMadrid
33
-Pág.69-
C I E N C I A S S O C I A L E S , G E O G R A F Í A E H I S TO R I A

8. La proporción de población activa depende, entre otros factores, de:

A. El número de parados
B. El número de ancianos mayores de 80 años
C. El porcentaje de adultos mayores de 30 años
D. La población jubilada

9. Señale la falsa:

A. Nacen más varones que hembras


B. La mortalidad masculina suele ser superior a la femenina
C. El porcentaje de mujeres es inferior que el de hombres
D. Todas las afirmaciones son falsas

10. ¿Qué característica pertenece al tipo primitivo de régimen demográfico?

A. Elevada mortalidad B. Predominio de viejos adultos


C. Natalidad baja D. Esperanza de vida elevada

11. La población activa que se dedica a transformar los productos, fundamentalmente, los que se
dedican a la industria, pertenecen al sector:

A. Primario B. Minero C. De servicios D. secundario

12. El espacio sobre el que una ciudad ejerce influencia socioeconómica se denomina:

A. Periferia B. Extrarradio C. Hinterland D. City

13. El IDH es:

A. Un organismo de las Naciones Unidas para el control de la población.


B. Una enfermedad, causante del aumento de la mortalidad infantil
C. El Institututo de Defensa del Hombre
D. El Índice de Desarrollo Humano

14. Las previsiones de crecimiento de la población mundial llevan a pensar que ésta llegará:

A. A 100.000 millones en 2100


B. A 1.000 millones en 2050
C. A 12.000 millones en 2070
D. A 20.000 millones en 2030

34 TAMadrid

-Pág.70-
U. D. 2 . - L A P O B L A C I Ó N Y E L E S PA C I O U R B A N O

15. ¿Qué es una conurbación?

A. Es el espacio que ocupa una ciudad


B. Es la zona del plano de una ciudad en la que se sitúa su centro de negocios
C. Es el conjunto formado por varias ciudades cuyos planos se juntan
D. Es el conjunto de los medios de transporte que permiten comunicar la ciudad con su peri-
feria y extrarradio.

TAMadrid
35
-Pág.71-
C I E N C I A S S O C I A L E S , G E O G R A F Í A E H I S TO R I A

RESPUESTAS A LOS EJERCICIOS

1. A

2. A

3. D

4. A

5. C

6. B

7. A

8. D

9. C

10. A

11. D

12. C

13. D

14. C

15. C

36 TAMadrid

-Pág.72-
U. D. 3 . - L A A C T I V I D A D H U M A N A Y E L E S PA C I O G E O G R Á F I C O

ÍNDICE
Pag.

OBJETIVOS. . . . . . . . . . . . . . . . . . . . . . . . . . . . . . . . . . . . . . . . . . . . . . . . . . . . . . . . . . . . . . . . . 2
INTRODUCCIÓN . . . . . . . . . . . . . . . . . . . . . . . . . . . . . . . . . . . . . . . . . . . . . . . . . . . . . . . . . . . 3
MAPA CONCEPTUAL . . . . . . . . . . . . . . . . . . . . . . . . . . . . . . . . . . . . . . . . . . . . . . . . . . . . . . . 4

DESARROLLO DE CONTENIDOS . . . . . . . . . . . . . . . . . . . . . . . . . . . . . . . . . . . . . . . . . . . 5

1. LAS ACTIVIDADES AGRARIAS Y EL ESPACIO RURAL . . . . . . . . . . . . . . . 5


1.1. FACTORES. . . . . . . . . . . . . . . . . . . . . . . . . . . . . . . . . . . . . . . . . . . . . . . . . . . . . . . . 5
1.2. PAISAJES RURALES . . . . . . . . . . . . . . . . . . . . . . . . . . . . . . . . . . . . . . . . . . . . . . . 6
1.3. SISTEMAS Y ESPACIOS AGRARIOS . . . . . . . . . . . . . . . . . . . . . . . . . . . . . . . . . . 7
1.4. LA GANADERÍA . . . . . . . . . . . . . . . . . . . . . . . . . . . . . . . . . . . . . . . . . . . . . . . . . . 9
1.5. LA PESCA . . . . . . . . . . . . . . . . . . . . . . . . . . . . . . . . . . . . . . . . . . . . . . . . . . . . . . . . 9
1.6. LA EXPLOTACIÓN FORESTAL . . . . . . . . . . . . . . . . . . . . . . . . . . . . . . . . . . . . . 10

2. ACTIVIDADES Y ESPACIOS INDUSTRIALES . . . . . . . . . . . . . . . . . . . . . . . . . 10


2.1. LA EXPLOTACIÓN DE MATERIAS PRIMAS Y PRODUCTOS ENERGÉTICOS . 10
2.2. LA INDUSTRIA. . . . . . . . . . . . . . . . . . . . . . . . . . . . . . . . . . . . . . . . . . . . . . . . . . . 14

3. LAS ACTIVIDADES TERCIARIAS Y SU DESARROLLO ACTUAL . . . . . 21


3.1. GENERALIDADES . . . . . . . . . . . . . . . . . . . . . . . . . . . . . . . . . . . . . . . . . . . . . . . . 21
3.2. DISTRIBUCIÓN SECTORIAL . . . . . . . . . . . . . . . . . . . . . . . . . . . . . . . . . . . . . . . 22

4. NIVELES DE DESARROLLO ECONÓMICO E INTERCAMBIO


DESIGUAL EN EL MUNDO . . . . . . . . . . . . . . . . . . . . . . . . . . . . . . . . . . . . . . . . . . 27

5. ESPACIO Y PODER POLÍTICO . . . . . . . . . . . . . . . . . . . . . . . . . . . . . . . . . . . . . . . 30


5.1. LA ORGANIZACIÓN POLÍTICA Y ADMINISTRATIVA DEL TERRITORIO . . 30
5.2. GRANDES ÁMBITOS GEOPOLÍTICOS . . . . . . . . . . . . . . . . . . . . . . . . . . . . . . . 31
5.3. EL ESPACIO POLÍTICO-ADMINISTRATIVO DEL TERRITORIO ESPAÑOL . . 36

RESUMEN . . . . . . . . . . . . . . . . . . . . . . . . . . . . . . . . . . . . . . . . . . . . . . . . . . . . . . . . . . . . . . . . . 43
EJERCICIOS DE AUTOCOMPROBACIÓN . . . . . . . . . . . . . . . . . . . . . . . . . . . . . . . . . . 49
RESPUESTAS A LOS EJERCICIOS . . . . . . . . . . . . . . . . . . . . . . . . . . . . . . . . . . . . . . . . . 52

-Pág.73-
C I E N C I A S S O C I A L E S . G E O G R A F Í A E H I S TO R I A

OBJETIVOS
Al finalizar el estudio de ésta Unidad Didáctica, el alumno será capaz de:

• Analizar y comprender la situación actual de la agricultura en el mundo, los


factores que determinan su distribución y su problemática.

• Entender los problemas y perspectivas de la pesca.

• Diferenciar y comprender las diversas actividades industriales, la influencia


que ha tenido y tiene en el desarrollo de la economía, los factores que influ-
yen en su localización y los problemas que genera.

• Valorar la importancia de las actividades del sector terciario o de los servi-


cios, tanto en sí mismas como en cuando a su papel como indicadores del
grado de desarrollo.

• Comprender la organización geopolítica del mundo, el papel integrador de


los distintos ámbitos espaciales, tanto los de carácter regional dentro de los
Estados como los que agrupan a los Estados en espacios supranacionales.

• Asimilar la organización territorial de España y los datos esenciales de las


Comunidades Autónomas.

-Pág.74-
U. D. 3 . - L A A C T I V I D A D H U M A N A Y E L E S PA C I O G E O G R Á F I C O

INTRODUCCIÓN
V imos en la Unidad Temática 1 todo lo concerniente a la realidad física de la
Tierra, su concepto, la aprehensión de dicha realidad por medio de la carto-
grafía, su forma e imbricación espacial, su relieve y su clima; aspectos todos ellos
que, como decíamos, tienen una cierta perdurabilidad en la escala de tiempo de la
vida humana. También adelantábamos algunas observaciones sobre la relación que
se establece entre esa realidad física y el desarrollo humano.

Precisamente, ese desarrollo humano, desde el punto de vista demográfico, nos ha


ocupado en la Unidad Temática 2, en la que hemos estudiado la población, sus movi-
mientos naturales y migratorios, su presente y su futuro y en la que también hemos
analizado el paisaje urbano.

Llega ahora el turno de aquella vertiente de la geografía que estudia el uso y la trans-
formación del medio por el hombre. Efectivamente, la conformación física de la Tie-
rra y su biosfera ofrecen al ser humano una serie de recursos alimenticios, materias
primas y posibilidades que ha ido aprovechando de modo creciente y que ha ido
transformando en su beneficio; un beneficio que a veces ha supuesto un perjuicio
para otras especies o para el propio medio, lo que, a la larga, ha redundado y puede
redundar en serios perjuicios para el causante.

Nos acercamos, pues, a la Geografía Económica. Buscamos una aproximación al


estudio del entramado productivo, analizando la articulación de los sectores econó-
micos (de los que un primer avance hemos tenido al mencionar el reparto de la
población en los mismos) y observando cómo el funcionamiento de los mismos ha
conducido al distanciamiento entre unos países y otros en cuanto a su grado de desa-
rrollo. También nos fijaremos en la tendencia contemporánea de la economía mun-
dial a la globalización, cubriendo primero los escalones de la formación de espacios
económicos de carácter geopolítico regional.

-Pág.75-
C I E N C I A S S O C I A L E S . G E O G R A F Í A E H I S TO R I A

M A PA C O N C E P T UA L

ACTIVIDADES AGRARIAS Y
ESPACIO RURAL

ACTIVIDADES Y ESPACIOS
INDUSTRIALES

ACTIVIDAD HUMANA
Y ESPACIO ACTIVIDADES TERCIARIAS Y
DESARROLLO ACTUAL
GEOGRÁFICO

NIVELES DE DESARROLLO
ECONÓMICO

ESPACIO Y PODER POLÍTICO

-Pág.76-
U. D. 3 . - L A A C T I V I D A D H U M A N A Y E L E S PA C I O G E O G R Á F I C O

LAS ACTIVIDADES AGRARIAS


1. Y EL ESPACIO RURAL

El espacio rural es la parte del territorio que se dedica a la explotación agrícola y gana-
dera. Aunque en muchas ocasiones existe un relación estrecha entre las dos, parece más ade-
cuado estudiar las característica de ambas explotaciones por separado. En ambos casos,
veremos que los paisajes y las circunstancias productivas son el fruto de la interacción de
diversos factores.

1.1. FACTORES
Físicos
Clima.—Factor decisivo por la trascendencia de las condiciones de temperatura y plu-
viosidad para el desarrollo de las especies vegetales.
Relieve.—Las condiciones de fuertes pendientes y dificultades de acceso impiden que
en zonas abruptas se desarrolle la agricultura. La explotación ganadera o la trasforma-
ción del terreno mediante la construcción de bancales, son las soluciones para algunos
de esos espacios.
Suelo.—Las condiciones edáficas resultan fundamentales para el crecimiento de las
plantas. Este aspecto es, en buena medida, consecuencia de los factores anteriores.

Humanos. Son aquéllos que dependen de la actuación humana para modificar las con-
diciones. Entre otras actuaciones, se pueden citar:
Regadíos.—Quizás la más antigua actuación humana para modificar el medio y las con-
diciones de relieve, clima y suelo. Permite solventar el déficit hídrico de algunas zonas
y diversificar las especies cultivadas.
Roturación.—Es la apertura para el cultivo de tierras improductivas, bosques y otras
áreas.
Fertilización.—Desde la ancestral costumbre de utilizar abono orgánico procedente de
excrementos del ganado, hasta las técnicas químicas más modernas.
Mecanización.—Empleo de maquinaria de diverso tipo para facilitar las labores de cul-
tivo, cuidado y cosecha.
Tecnología.—Aplicación de técnicas especiales como los cultivos hidropónicos, los
invernaderos, etc...

TAMadrid
5
-Pág.77-
C I E N C I A S S O C I A L E S . G E O G R A F Í A E H I S TO R I A

Técnicas biológicas.—Aplicación de técnicas biológicas: injertos, selección, modifica-


ciones genéticas...

Estos factores determinan el grado de desarrollo de las explotaciones así como los sis-
temas de cultivo extensivo o intensivo.

En este sentido, conviene precisar éstos y algún otro concepto que deberemos utilizar en
adelante.
El primer concepto es el de rendimiento. Si la producción de una explotación es el total
de lo obtenido en la misma, el rendimiento es el coeficiente resultante de dividir esa pro-
ducción entre la extensión de la explotación. También se puede valorar el rendimiento por
persona empleada, en este caso el divisor para la obtención del coeficiente será el personal
empleado.
Entendemos por explotación intensiva aquella en la que se cultiva de forma continua,
normalmente en terrenos reducidos, con mucha mano de obra. Por tanto, en ellos el rendi-
miento suele ser alto por hectárea pero bajo por persona empleada.

La explotación extensiva es aquella en la que las explotaciones son extensas y se utiliza


poca mano de obra, por tanto, el rendimiento por hectárea es bajo; pero alto por persona
empleada.

1.2. PAISAJES RURALES


La explotación agraria determina la modificación del paisaje. Dependiendo de los fac-
tores citados y los tipos de cultivo observamos diversos tipos de paisajes rurales.
Así, en lo que respecta a la forma de la explotación, ésta puede ser regular (con forma
geométrica: rectangular, circular, poligonal, etc...) o irregular.
En cuanto a la extensión, pueden ser minifundios (pequeñas explotaciones) o latifundios.
Los límites de las parcelas pueden ser abiertos o cerrados (con límites a base de setos,
vallas...).
Por los tipos de cultivo, pueden ser arbóreos, arbustivos o herbáceos; de monocultivo o
policultivos; de secano o regadío.

Por último, el hábitat rural también adquiere una determinada fisonomía, dispersa o
concentrada.

6 TAMadrid

-Pág.78-
U. D. 3 . - L A A C T I V I D A D H U M A N A Y E L E S PA C I O G E O G R Á F I C O

1.3. SISTEMAS Y ESPACIOS AGRARIOS

1.3.1. LA AGRICULTURA SUBDESARROLLADA

La llamada revolución verde ha modificado considerablemente la agricultura de los paí-


ses en vías de desarrollo (empleo de semillas tratadas genéticamente y de maquinaria para
aumentar los rendimientos), sin embargo, perviven en el mundo sistemas de explotación tra-
dicionales.

Agricultura itinerante o de roza.—Es típico de zonas tropicales y ecuatoriales. La explo-


tación tiene carácter tribal. Se aprovechan espacios de selva en los que se talan los árboles
de una pequeña zona de suelos blandos. Las labores agrícolas suelen realizarlas las mujeres
con aperos rudimentarios y para obtener productos de consumo inmediato (maíz, mandio-
ca, batata...). El rápido agotamiento de los suelos obliga a cambiar de lugar cada 2 ó 3 años.
Los rendimientos son muy bajos.

Agricultura sedentaria de subsistencia.—En zonas de África y Asia fundamentalmente.


Se trata de explotaciones orientadas a la subsistencia con huertas en la inmediación de la
aldea y cultivos rotativos de tres hojas o sectores, dos con cultivos y una en barbecho, es
decir, regenerándose por medio de los excrementos del ganado que completa la producción
de alimentos con su carne y su leche.

Agricultura intensiva monzónica.—Se trata de los cultivos de arroz que suponen la base
alimenticia de los países del sudeste asiático. Los importantes aportes de agua de los monzo-
nes permiten el cultivo de este cereal que precisa mucha cantidad de agua. El sistema de explo-
tación y las condiciones culturales y religiosas obligan al empleo de mucha mano de obra.

1.3.2. LA AGRICULTURA DESARROLLADA

En los países desarrollados, sobre todo en Estados Unidos y en la Unión Europea, las
explotaciones agrícolas están orientadas a la productividad y la competitividad. Se trata de
una agricultura en la que destaca la tecnificación, empleo de maquinaria, abonos, aplica-
ciones científicas y la variedad de los cultivos. Todo ello hace que los rendimientos sean
muy elevados tanto por hectárea como por personal empleado.

Se pueden diferenciar las explotaciones intensivas, cercanas a las ciudades y orientadas


a la producción de hortalizas y otros productos de primera necesidad y las explotaciones
extensivas de cereales.

En el caso de la Unión Europea existen algunos problemas que reducen la productivi-


dad, sobre todo los derivados de estructuras tradicionales y población rural envejecida. Esa

TAMadrid
7
-Pág.79-
C I E N C I A S S O C I A L E S . G E O G R A F Í A E H I S TO R I A

falta de productividad es combatida por una Política Agraria Común (PAC), la de la llama-
da Europa Verde, que incluye cupos de producción por países, subvenciones y ayudas, polí-
tica de precios, etc.

Pero también existen problemas de superproducción que conducen a la obtención de


excedentes que desequilibran el mercado (lácteos, carne, azúcar, vino...), motivo por el cual
la PAC atiende, en dicho terreno, a la subvención para el cambio de producción y a la limi-
tación mediante cuotas.

Por lo que respecta a la agricultura española, ha tenido que afrontar el gran reto de la
integración europea. Desde la entrada, en 1986 se han experimentado, entre otras conse-
cuencias:
– Descenso continuo de la población dedicada a la agricultura.
– Disminución de la aportación agraria al Producto Interior Bruto.
– Explotaciones cada vez más competitivas.
– Reorientación de las producciones como consecuencias de las cuotas.

Las limitaciones impuestas a cultivos tradicionales como vid y olivo o las cuotas leche-
ras han generado problemas importantes para numerosas familias que se han visto obliga-
das a modificar sus explotaciones o abandonarlas a cambio de compensaciones.

En contrapartida, los productos de huerta y frutícolas han encontrado mercados amplios


gracias a su competitividad.

1.3.3. LA AGRICULTURA DE PLANTACIÓN

Se trata de explotaciones en las que las empresas realizan grandes inversiones para la
obtención de productos de gran demanda internacional en los que se obtienen amplios bene-
ficios. Se produce sobre todo en zonas tropicales (África, Asia meridional y Sudamérica).

Son grandes explotaciones de monocultivos (caucho, cacao, bananas, café, té...), con
mano de obra barata y abundante, empleo de tecnología y rápida comercialización a los
mercados internacionales.

La intervención de capitales extranjeros, preocupados en obtener rápidos beneficios,


favorece la proliferación de estas explotaciones pero tiene negativas consecuencias por la
falta de planificación que puede generar superproducciones que hundan los mercados. Ade-
más, los beneficios de la explotación escapan del país objeto de la plantación y el agota-
miento de los suelos limita en el tiempo los posibles rendimientos.

8 TAMadrid

-Pág.80-
U. D. 3 . - L A A C T I V I D A D H U M A N A Y E L E S PA C I O G E O G R Á F I C O

1.3.4. LA AGRICULTURA DIRIGIDA

En los países de economía dirigida, la producción agraria se encuentra sometida a la pla-


nificación estatal. Este tipo de explotación se ha visto considerablemente reducida como
consecuencia de los profundos cambios experimentados en la Europa del Este a partir de
los años 90.

Hoy en día, desterrado como sistema de producción de países como Rusia y los demás
de la antigua URSS y de los países del Este de Europa, incluso se ve modificada en sus
parámetros en países como China, Cuba, Vietnam, en los que los sistemas económicos mix-
tos, con apertura a un incipiente mercado libre, van ganando terreno dentro de regímenes
de carácter socialista.

1.4. LA GANADERÍA
El desarrollo experimentado por la ganadería ha seguido pautas similares a las de la
agricultura.

En los países desarrollados la ganadería se presenta bajo dos formas básicas de explo-
tación:

Ganadería intensiva.—Se trata de explotaciones pequeñas en cuanto a su extensión,


muy tecnificadas (selección de razas, tratamientos genéticos, piensos, estabulación), con
rendimientos muy altos. Se orientan a la producción de leche, carne, huevos...

Ganadería extensiva.—Explotaciones en grandes espacios abiertos. El ganado pasta


libremente y es estabulado sólo temporalmente para su marcado y, posteriormente, para su
selección y conducción hacia la función productiva correspondiente, sea el esquilado, la
obtención de leche o la de carne.

En los países subdesarrollados, las formas de explotación son la extensiva y la de sub-


sistencia. Los rendimientos son en cualquier caso bajos.

1.5. LA PESCA
La pesca y la acuicultura completan el panorama de las actividades productivas del sec-
tor primario.

La riqueza de las plataformas continentales viene siendo explotada desde la antigüedad.


Pero las técnicas empleadas y la sobreexplotación han conducido al agotamiento de los
caladeros tradicionales.

TAMadrid
9
-Pág.81-
C I E N C I A S S O C I A L E S . G E O G R A F Í A E H I S TO R I A

El interés económico de estas explotaciones ha tenido su reflejo en la legislación de carác-


ter internacional que ha llevado al establecimiento de las zonas de interés económico y aguas
jurisdiccionales en las 200 millas. Con esta base, los países con caladeros ricos, establecen
políticas de limitación de las capturas y concesión de licencias, con lo que se pretende, por
una parte, controlar el volumen de la producción y así permitir la regeneración de las especies
y, por otra, rentabilizar la riqueza pesquera de las costas explotadas por flotas extranjeras.

Los medios técnicos han llegado también al mundo de la pesca que se vale, para la loca-
lización de los bancos, de radar, sonar, helicópteros o, incluso, satélites. Las técnicas de cap-
tura, por su parte, se han ido actualizando y regulando para permitir una pesca efectiva pero
no destructiva.

Además de las prácticas pesqueras abusivas, el gran enemigo del futuro de la pesca es
la contaminación causada por los vertidos accidentales de productos petroquímicos y nucle-
ares y los incontrolados de industrias y núcleos humanos.

1..6. LA EXPLOTACIÓN FORESTAL


Los espacios forestales pueden ser también una fuente de obtención de recursos. Baste
citar la madera para construcción o pasta de papel, las resinas, etc.

2. ACTIVIDADES Y ESPACIOS INDUSTRIALES

2.1. LA EXPLOTACIÓN DE MATERIAS PRIMAS Y


PRODUCTOS ENERGÉTICOS
El medio natural terrestre ofrece una serie de materias minerales, vegetales y animales
que pueden ser utilizados por el hombre. Ese conjunto de materias podemos dividirlas en:

Materias primas.—Utilizadas por la industria para la elaboración de productos manu-


facturados.

Productos energéticos.—Empleados como fuentes de propulsión de la maquinaria nece-


saria para los procesos productivos.

10 TAMadrid

-Pág.82-
U. D. 3 . - L A A C T I V I D A D H U M A N A Y E L E S PA C I O G E O G R Á F I C O

2.1.1. MATERIAS PRIMAS

Dentro del conjunto de materias primas y en función de su origen distinguimos materias


vegetales, animales y minerales. El estudio de los dos primeros grupos queda incluido en
los apartados anteriores dedicados a la agricultura y ganadería; por tanto, nos centramos
ahora en las materias de origen mineral.

Mientras las materias vegetales y animales, adecuadamente explotadas, pueden ser ina-
gotables, las materias primas minerales no son renovables, algunas desaparecen tras su
empleo y otras pueden, como mucho, ser recicladas.

Estas materias se hallan en el subsuelo y su explotación requiere la perforación del


mismo, sea en minas bajo tierra, minas a cielo abierto o pozos de extracción.

Materias minerales metálicas


Se utilizan para la elaboración de otros productos y la mayoría requieren procesos pre-
vios de refinado, amalgama, etc.
Entre los más importantes minerales con aprovechamiento industrial están:
Hierro.—Es el metal más abundante, usado para aleaciones con las que se fabrican todo
tipo de maquinarias y herramientas, además de materiales de construcción.
Cobre.—El primer mineral que usó la humanidad, pronto aleado con estaño o cinc. Sus
reservas actuales no son muy abundantes. Se dedica sobre todo a la fabricación de com-
ponentes eléctricos.
Estaño.—Al igual que el cobre ha sido explotado durante miles de años y, en conse-
cuencia, las reservas son escasas.
Mercurio.—Un metal muy escaso pero del que no son necesarias grandes cantidades. Se
usa en electrónica y en instrumentos de precisión.
Aluminio.—Requiere una previa elaboración pues se presenta en la mena de bauxita. Su
uso se ha disparado en las últimas décadas por sus aplicaciones en aeronáutica o en la
industria del automóvil.
Oro.—El metal más codiciado históricamente y que, además de sus usos en joyería, tiene
numerosas aplicaciones industriales.
Plata.—El segundo de los llamados metales preciosos, tras el oro. Y que, como él, tiene
múltiples usos.
Otros.—Además de los citados, tienen importancia, zinc, plomo, manganeso, cromo,
magnesio, wolframio, etc.

TAMadrid
11
-Pág.83-
C I E N C I A S S O C I A L E S . G E O G R A F Í A E H I S TO R I A

Materias minerales no metálicas


Entre éstas, están:
– Azufre, fosfatos y potasio, empleados para fabricar abonos, entre otros usos.
– Arcilla, Gravas, Cal, Yeso, Cemento, utilizados en la construcción.
Además, tenemos los minerales fósiles (carbón, petróleo), que, dado su aprovechamien-
to, fundamentalmente energético, los trataremos en el apartado siguiente.

2.1.2. PRODUCTOS ENERGÉTICOS

Se pueden diferenciar dos grupos en el conjunto de los productos utilizados para la


obtención de energía: aquellos cuya naturaleza los hace inagotables y aquellos otros cuya
utilización va reduciendo sus reservas y terminará por consumirlos totalmente. En la actua-
lidad, precisamente son estos últimos los más utilizados, lo que plantea la necesaria revisión
de las fuentes energéticas, empeño en el que se encuentra el futuro de la industria humana
a medio plazo.
Por otra parte, las reservas, por lo general, se encuentran en países poco desarrollados,
siendo precisamente los desarrollados los principales consumidores.

Productos no renovables
Carbón.—Procede de masas de vegetales metamorfizados tras quedar sepultadas y se
presenta en cuatro variedades que, ordenadas de mayor a menor poder calórico, son:
antracita, hulla, lignito y turba.
Aunque su uso se remonta a siglos (China, Inglaterra), no fue hasta la Revolución Indus-
trial cuando su uso se generalizó, sobre todo como combustible para calentar el agua de
las calderas de vapor y para los altos hornos de las industrias siderúrgicas. En la actua-
lidad se emplea en las centrales térmicas para la obtención de energía eléctrica y en la
siderurgia.
Aunque existen aún reservas en países desarrollados, su explotación ha dejado de ser
rentable por los elevados salarios que deben pagarse, por lo que son los países en vías
de desarrollo los principales abastecedores del mercado mundial.
Uno de los grandes inconvenientes del uso del carbón es su poder contaminante por la
liberación de CO2 y azufre durante su combustión.
Petróleo.—Es un hidrocarburo, compuesto complejo de carbono e hidrógeno proceden-
te de la descomposición de materia orgánica en el subsuelo.
Comenzó a utilizarse de forma generalizada como lubricante y combustible generador
de luz a mediados del siglo XIX, si bien su eclosión como materia de combustión

12 TAMadrid

-Pág.84-
U. D. 3 . - L A A C T I V I D A D H U M A N A Y E L E S PA C I O G E O G R Á F I C O

preeminente fue consecuencia de la invención de los motores de combustión a finales


del siglo XIX y de la proliferación de los medios de transporte basados en el mismo, a
partir de los años treinta.
Hoy en día, la dependencia que la economía mundial tiene del petróleo se aprecia en la
gran variedad de productos derivados del mismo (gasolinas y otros combustibles, plás-
ticos, fibras, lubricantes, etc...) y en los efectos macroeconómicos que produce cual-
quier alteración del mercado, ya sea por las decisiones de disminución o aumento de la
producción por parte de la Organización de Países Exportadores de Petróleo (OPEP), o
por los efectos de determinados conflictos bélicos (como los acontecidos en los años 70
por la Guerra árabe-israelí, la de los años 80 por la crisis generada por la revolución en
Irán o la de los 90 por la Guerra del Golfo).
Al igual que el carbón, la combustión del petróleo genera una fuerte contaminación
atmosférica. Además, los ocasionales vertidos de petróleo refinado o sin refinar produ-
cen también efectos contaminantes terrestres.
Gas Natural.—Se presenta normalmente asociado a los yacimientos petrolíferos en la
parte superior de las bolsas subterráneas en las que éste se encuentra. Su uso es relati-
vamente reciente y se orienta a usos domésticos (cocina, calefacción...) y a la obtención
de electricidad. Es más limpio que los otros combustibles.
Energía nuclear.—Comenzó a utilizarse, sobre la base de los estudios sobre radiactividad
realizados, entre otros, por MAX PLANCK, los esposos CURIE o ALBERT EINSTEIN,
a mediados del siglo XX, tras comprobar el gran poder energético generado por la fisión
nuclear de los átomos, puesto en práctica por primera vez en las bombas nucleares.
Inicialmente aparecía como una fuente barata y muy rentable. Sin embargo, las enormes
inversiones para la construcción de las centrales con las necesarias medidas de seguri-
dad disminuyen su rentabilidad y, además, tienen enormes problemas derivados de la
peligrosidad de las radiaciones producidas y de la dificultad de almacenamiento de los
residuos radiactivos generados, sin contar con los potenciales desastres producidos por
accidentes (baste con recordar los trágicos efectos de la explosión de la central de Cher-
nobyl, en Ucrania, en 1986).
Posiblemente, la investigación orientada a la fusión nuclear, que no genera residuos con-
taminantes, pueda lograr en un futuro solucionar los problemas de rentabilidad y segu-
ridad, mas, por el momento, las decisiones de los gobiernos se dirigen a no construir más
centrales y cerrar paulatinamente las existentes.

Productos renovables
Energía hidroeléctrica.—Se trata de aprovechar la fuerza generada por el agua para,
haciéndola pasar por turbinas, producir energía eléctrica. Su uso comenzó a finales del

TAMadrid
13
-Pág.85-
C I E N C I A S S O C I A L E S . G E O G R A F Í A E H I S TO R I A

siglo XIX; pero el impulso de la producción termoeléctrica (carbón, petróleo) y nuclear


la hizo entrar en crisis a mediados del siglo XX. La desacreditación de estas fuentes ha
hecho que se potencie la construcción de embalses si bien procurando evitar el impac-
to medioambiental que producen los grandes embalses.
Energía eólica.—Utilizada desde la antigüedad como fuerza impulsora de barcos y
molinos. La fuerza de determinados vientos locales más o menos constantes es aprove-
chada con la instalación de molinos con potentes turbinas.
Energía solar.—La energía potencial que puede obtenerse de la irradiación solar es enor-
me. Hoy día el aprovechamiento sigue una doble orientación:
– Energía fotovoltaica. Producción de electricidad recogiendo la luz solar por medio
de paneles de silicio.
– Energía térmica. Obtención de electricidad por medio de turbinas movidas por
vapor de agua a presión producido por el recalentamiento mediante placas.
Energía geotérmica.—Se trata de aprovechar las aguas termales para calefacciones y
otros usos domésticos, además de ésta y el vapor para la producción de electricidad
mediante turbinas.
Energía maremotriz.—Obtención de electricidad por la fuerza ejercida por el oleaje
sobre turbinas.
Otras.—Combustión de materia orgánica para producir gases como metano o etano o
para calentar agua y generar electricidad con el vapor, uso de alcoholes, etc.

2..2. LA INDUSTRIA

2.2.1. EVOLUCIÓN DE LA INDUSTRIA

Si entendemos por industria la elaboración de productos manufacturados, el hombre ha


desarrollado una actividad industrial desde la prehistoria, desde el momento en el que tomó
una piedra o un trozo de madera y lo modificó para fabricar una herramienta.

En la antigüedad la producción de textiles, cerámicas, construcción de muebles, barcos,


carros, aperos de trabajo, productos alimenticios, etc... se realizaba de modo artesanal, un
método que implicaba la participación del artesano en todos los pasos de la elaboración.
Esta actividad quedó organizada durante la Edad Media en gremios cerrados con maestros,
oficiales y aprendices que vieron sus sistemas productivos mejorar con nuevas herramien-
tas durante la Edad Moderna que supone, además, el aumento de los mercados.

14 TAMadrid

-Pág.86-
U. D. 3 . - L A A C T I V I D A D H U M A N A Y E L E S PA C I O G E O G R Á F I C O

En el siglo XVIII se produjo la llamada Primera Revolución Industrial, basada funda-


mentalmente en la industria textil y la siderúrgica e impulsada por la aparición de la máqui-
na de vapor y por el uso del carbón como combustible. La liberación de mano de obra del
campo por la mejora de la producción agraria alimentó la industria. Además, la comercia-
lización de los productos se benefició de la aparición de medios de transporte como el ferro-
carril y el barco de vapor. El crecimiento económico propició el auge de entidades
financieras.

A finales del siglo XIX se produjo la Segunda Revolución Industrial, basada en el petró-
leo y la electricidad, sus actividades productivas punteras son los automóviles, las produc-
ciones metalúrgicas y las químicas. Los medios de transporte sufren un importante cambio
con el predominio del automóvil, tren eléctrico, avión, barco de gasóleo. A esto se une la
aparición de medios de comunicación como el teléfono, la radio o la televisión que son usa-
dos como medios de publicidad para estimular el consumo.

En la actualidad se está produciendo una nueva revolución basada en la energía eléctri-


ca, la informática y la automatización de los procesos productivos. Es la época de las tele-
comunicaciones globales. La industria electrónica, biogenética y aerospacial son las
predominantes. La preocupación por el medio ambiente ha determinado la trasformación de
otros sectores industriales y la búsqueda de fuentes de energía limpias. La globalización de
la economía, la masiva incorporación de la mujer al mundo laboral y el predominio del sec-
tor terciario, completan las características del entramado económico actual.

2.2.2. LAS BASES DE LA INDUSTRIA ACTUAL


Los elementos que determinan la industria de hoy en día son:
La mano de obra.—La potencialidad de la mano de obra es un elemento esencial para el
establecimiento de la industria. Sus características y sus niveles de exigencia salarial son
importantísimas a la hora de decidir la instalación de determinadas industrias (Muchas mul-
tinacionales buscan establecer sus factorías en países en vías de desarrollo en los que los
salarios son más bajos).
Los empresarios.—Inversores decididos a poner en marcha las industrias.
Entidades financieras.—Imprescindibles para las inversiones iniciales y para las nece-
sarias modernizaciones.
Tecnología.—La disponibilidad de medios tecnológicos es hoy enorme. En algunos
casos requiere fuertes inversiones pero puede rentabilizar considerablemente la producción.
Mercado.—Hoy día éste rebasa los límites locales y hasta los regionales y nacionales
caminando hacia la globalización.

TAMadrid
15
-Pág.87-
C I E N C I A S S O C I A L E S . G E O G R A F Í A E H I S TO R I A

Transportes.—El desarrollo de los transportes y la creación de completas redes combi-


nando las posibilidades del transporte terrestre, aéreo y marítimo, es una garantía para la
comercialización.
Instalaciones.—La preocupación de los gobiernos, las entidades locales y la iniciativa
privada por el desarrollo industrial deriva en la creación de espacios especialmente prepa-
rados para las instalaciones industriales con las correspondientes infraestructuras (polígo-
nos industriales).
Política.—La política es un factor decisivo para la industria ya que las inversiones de los
empresarios y la orientación de sus previsiones de producción están determinadas por las
expectativas de beneficios y éstos dependen en gran medida de la estabilidad de los siste-
mas políticos.

2.2.3. LOS SECTORES INDUSTRIALES

El estudio de las diversas actividades industriales permite estructurarlas de diversas for-


mas, bien por los productos generados, por las materias primas utilizadas o el número de
empleados. Escogemos una fórmula de clasificación basada esencialmente en los productos:

A) Alimentación, bebidas y tabaco


Junto a tradicionales o modernas pequeñas empresas, existen grandes multinacionales.
Admite un importante nivel de mecanización. Entre otras producciones destacan carne, lác-
teos, alimentación animal, pan, bollería, pastelería y galletas, bebidas refrescantes no alco-
hólicas, alcoholes, tabaco, grasas y aceites, azúcares...
Incluye también aspectos como las conservas, congelados, precocinados, las empresas
de cátering, etc.

B) Textil, confección, piel


La textil fue la industria impulsora de la Primera Revolución Industrial. El sector incluye:
– Elaboración de las fibras naturales (lino, algodón, lana) y artificiales.
– Confección de telas, prendas y accesorios.
– Cuero y calzado.

C) Madera, corcho y muebles


Este sector incluye el aprovechamiento de los recursos forestales para la obtención de
madera para construcción o fabricación de muebles, pasta de papel, etc.

16 TAMadrid

-Pág.88-
U. D. 3 . - L A A C T I V I D A D H U M A N A Y E L E S PA C I O G E O G R Á F I C O

D) Artes Gráficas
Engloba las actividades de composición, maquetado y en general cuantas se relacionan
con los procesos previos a la impresión, ésta y las de manipulado del papel y el cartón en
general. La creciente informatización de los procesos gráficos hace de este sector uno de
los que se enfrentan a mayores retos de modernización y especialización profesional.

E) Sector energético
La producción más importante es la relacionada con derivados petrolíferos tales como
gasolinas, gasóleos, fuel-oil, aceites pesados y livianos, asfalto, etc.
El sector eléctrico ha sido uno de los que ha experimentado un más fuerte crecimiento.
Dentro del mismo se encuentran la producción termoeléctrica, la hidroeléctrica y la nucle-
ar. Detrás se situarían el resto de fuentes energéticas alternativas.

F) Productos químicos
Entre las producciones destacan especialmente los plásticos y caucho. Tras ellos, los
productos farmacéuticos y los abonos.
Siendo uno de los sectores con futuro más prometedor, los restos que se le presentan son:
– Control de la calidad de los productos.
– Control del envasado y etiquetado de productos potencialmente peligrosos.
– Seguridad e higiene de los trabajadores.
– Seguridad de las instalaciones para prevenir accidentes.
– Tratamiento y eliminación de los residuos de los procesos productivos.
– Reducción de la emisión de contaminantes generados en dichos procesos.

G) Fabricación mecánica
Ha sido uno de los sectores que más ha sufrido el proceso de reconversión por la fuerte
crisis que la siderurgia soporta en el mundo. Se trata, además, de un sector muy diversificado
que podemos compartimentar, para su estudio, en los cuatro subsectores en los que se divide.
1. Industrias de construcción y materiales de equipo (maquinaria en general, inclu-
yendo las de electrodomésticos, motores, etc.).
2. Fabricación de material de transporte (producción automovilística, construcción
naval, industria aeronáutica, industria aeroespacial, material ferroviario.
3. Construcciones metálicas incluye la fabricación de toda clase de objetos metálicos
utilizados en la construcción o los servicios, herramientas, etc.
4. Industria pesada ha sido el subsector que ha sufrido más directamente la reconver-
sión industrial. Se dedica a la primera elaboración de los metales.

TAMadrid
17
-Pág.89-
C I E N C I A S S O C I A L E S . G E O G R A F Í A E H I S TO R I A

H) Construcción
Incluye la edificación, tanto de viviendas como de otra clase de edificios, y las obras
públicas.

2.2.4. LOCALIZACIÓN DE LAS INDUSTRIAS

La localización de las factorías ha dependido de diferentes factores. Podemos destacar,


los siguientes:
Mano de obra.—La existencia de abundante mano de obra atrajo a las ciudades a la
industria textil de la Primera Revolución Industrial y ha seguido siendo un factor importante
para la radicación de las instalaciones de otros sectores. Hoy en día, como citábamos más
arriba es un elemento importante en el sentido de que muchas empresas buscan la mano de
obra barata de determinados países.
Materias primas.—Fue el elemento de atracción principal de la industria siderúrgica. Hoy
en día, salvo por las instalaciones tradicionales, no es un factor excesivamente poderoso.
Proximidad a los mercados.—Con ello se busca abaratar los costes de transporte. En esa
línea también cabe valorar las instalaciones en áreas portuarias con el fin de facilitar dicho
transporte.
Polígonos industriales.—Como ya se ha mencionado, son consecuencia de las actuacio-
nes oficiales con la pretensión de preparar las infraestructuras que atraigan a las empresas.
Subvenciones oficiales.—En el mismo sentido del caso anterior, algunas entidades loca-
les, regionales o nacionales ofrecen mejores perspectivas fiscales a las empresas con el fin
de favorecer su instalación.
Localizaciones aisladas.—Algunas empresas por sus condiciones de producción (rui-
dos, tamaño de las instalaciones, manejo de materias peligrosas), se instalan lejos de núcle-
os urbanos.

2.2.5. LOS ESPACIOS INDUSTRIALES Y SU DESIGUAL DISTRIBUCIÓN


EN EL MUNDO

La mayor parte de las concentraciones industriales se encuentran en Japón, Estados Uni-


dos y Europa.
En Europa, los países de la Unión Europea son los más industrializados, especialmente,
el Reino Unido, Alemania, Bélgica, Países Bajos, Francia, Suecia..., a los que hay que añadir
a Suiza, la República Checa o Hungría. Estos dos últimos después de superar la crisis produ-
cida tras los cambios institucionales motivados por la caída de los regímenes comunistas.

18 TAMadrid

-Pág.90-
U. D. 3 . - L A A C T I V I D A D H U M A N A Y E L E S PA C I O G E O G R Á F I C O

En Norteamérica, Estados Unidos destaca en su papel de gran potencia industrial con


las áreas fuertemente industrializadas del Noreste y de la zona de los Grandes Lagos, área
en la que se suma Canadá.

Japón, con una industria poderosa y especialmente competitiva posee zonas de gran
desarrollo industrial como Tokio, Yokohama, Osaka, Kobe...

En Asia, además, hay que destacar el surgimiento de los llamados Nuevos Países Desa-
rrollados (NPD), que han irrumpido con fuerza en los mercados mundiales, aunque en
buena mediad su despegue se deba a las inversiones de multinacionales gracias a una mano
de obra barata. Son los casos de Singapur, Taiwán, Corea...

En África, sólo Sudáfrica cuenta con un importante desarrollo industrial, al que se van
sumando, por razones similares a las citadas en el párrafo anterior, países como Nigeria,
Camerún, Egipto y los países del Magreb.

América del Sur, también está experimentando un cierto despegue industrial que parte
de núcleos asociados a grandes ciudades.

Australia y Nueva Zelanda completan el panorama de los grandes polos de industriali-


zación.

Por otra parte, la distribución industrial en el mundo debe ser observada en función de
los sectores. Así, apreciaremos que las industrias más peligrosas, sucias, contaminantes y
ruidosas tienden a localizarse en los países en vías de desarrollo, mientras que en los paí-
ses desarrollados, con una industria más poderosa y diversificada, tienden a concentrar su
actividad en las industrias de bienes de equipo y de consumo. De este modo los países desa-
rrollados siguen manteniendo una posición preeminente pues compran a precios bajos los
productos de la industria instalada en los países más pobres, cuyos costes son inferiores por
la mano de obra; y venden a precios altos los productos de consumo.

Esta situación determina que la desventaja de los países pobres se mantenga. Además
los beneficios de la instalación de industrias en éstos quedan reducidos porque la inversión
procede de empresas multinacionales que son las que se llevan los beneficios. Esto acre-
cienta la dependencia tecnológica y financiera y aumenta la deuda externa. Éste es el prin-
cipal problema económico de los países pobres ya que deben dedicar buena parte de su
presupuesto a paliar los efectos de la misma por lo que no pueden dedicarlo a invertir en
infraestructuras propias, por lo que tienen que seguir confiando en este terreno en las inver-
siones extranjeras, lo que aumenta el endeudamiento.

No cabe duda que la industrialización genera importantes beneficios económicos,


aumenta el nivel de ingresos de los trabajadores y consiguientemente su nivel de vida,

TAMadrid
19
-Pág.91-
C I E N C I A S S O C I A L E S . G E O G R A F Í A E H I S TO R I A

aumenta la oferta laboral e impulsa la modernización; pero también supone problemas. El


paisaje industrial lleva aparejada la contaminación atmosférica, acústica y del agua y el
suelo y las modificaciones sociales que produce no siempre son positivas.

2.2.6. LA INDUSTRIA EN ESPAÑA

España es un país industrializado. Desde mediados del siglo XIX comenzó el despegue
industrial definiéndose tres polos o áreas de desarrollo principales: Cataluña, con predomi-
nio de las industrias textiles y mecánicas, País Vasco, con industria siderúrgica, mecánica y
naval y Asturias, centrada en la siderurgia por la proximidad de los yacimientos carboníferos.

Después de los beneficios generados por la venta de productos durante la I Guerra Mun-
dial, aprovechando la posición neutral del gobierno, la falta de inversión adecuada de esos
beneficios y los efectos de la Guerra Civil y de una posguerra marcada por el II conflicto
mundial y una política aislacionista y autárquica, generaron una crisis industrial de la que
España comenzó a salir en los años 60 cuando a los núcleos industriales tradicionales se
unió Madrid y otros polos como Vigo, Valencia, Zaragoza, Valladolid... En esta situación
España alcanzo el 9.o puesto como país industrial en el mundo.

La crisis petrolífera de los años 70, unida a la transición política a la democracia pro-
dujeron un estancamiento del que se empezó a salir tras la entrada en la entonces Comuni-
dad Económica Europea y la estabilidad política. La reconversión de los sectores
emprendida y las inversiones estadounidenses y japonesas reorientaron la industria españo-
la de modo que los sectores tradicionales entraron en decadencia a favor de nuevos impul-
sos en otros.

De este modo los sectores punteros en el entramado industrial actual de España son:
– Automovilístico, con instalaciones dependientes básicamente de capitales de multi-
nacionales.
– Químico.
– Calzado y cuero.
– Alimentación.

20 TAMadrid

-Pág.92-
U. D. 3 . - L A A C T I V I D A D H U M A N A Y E L E S PA C I O G E O G R Á F I C O

LAS ACTIVIDADES TERCIARIAS


3. Y SU DESARROLLO ACTUAL

3.1. GENERALIDADES
Se denomina sector terciario al conjunto de actividades económicas que no son directa-
mente productivas de materias primas ni de productos manufacturados.

El surgimiento de este sector en el entramado económico se vincula a la producción


de excedentes en el marco de las sociedades neolíticas consolidadas. Esos excedentes
permitieron que una parte de la población comenzara a dedicarse de un modo exclusivo
a prestar una serie de servicios a la comunidad desligados de las actividades producti-
vas. Los sacerdotes, grupos dirigentes y comerciantes fueron los primeros en formar este
grupo.

En los últimos años el sector terciario se ha convertido en un sector muy importante,


tanto por el volumen de población activa empleada como por su peso en los PIB. Se man-
tiene en un constante crecimiento, directamente relacionado con el descenso de los otros
dos sectores. La estructura económica del mundo parece favorecer esta dinámica, propia en
general de los países desarrollados. La proximidad a las materias primas, el precio de la
mano de obra, los canales de distribución, el crecimiento de las empresas multinacionales,
etc... orienta la distribución de los sectores económicos globales haciendo de los países en
vías de desarrollo la plataforma ideal para el sector primario; a los llamados “nuevos paí-
ses desarrollados”, para el secundario, dejando a los desarrollados el papel de gestores de
la producción y comercialización.

Las actividades englobadas en el sector son muy heterogéneas y algunas resultan de difí-
cil clasificación.

Algunas de sus actividades como el comercio o distribución de materias primas y pro-


ductos manufacturados sirven de complemento a los subsectores industriales, al igual que
las entidades financieras, que juegan un importantísimo papel en el desarrollo de las empre-
sas y sus actividades productivas.

Otras son diversos servicios a la comunidad, precisamente el conjunto mayor y que tam-
bién da nombre al sector.

TAMadrid
21
-Pág.93-
C I E N C I A S S O C I A L E S . G E O G R A F Í A E H I S TO R I A

3.2. DISTRIBUCIÓN SECTORIAL


A) Entidades financieras
Se trata de un sector de gran trascendencia y que engloba gran número de actividades.
Junto a la actividad financiera oficial, personalizada por los bancos centrales, hay un impor-
tante desarrollo financiero privado. Los grandes bancos japoneses, norteamericanos o sui-
zos, además de algunos británicos o franceses han experimentado un proceso de expansión
internacional aparejado a la globalización de la economía.
En la Unión Europea, el reto de la unión monetaria y económica planea en el horizonte
inmediato del sector.
Las sociedades de seguros y las inversoras se añaden a la actividad del sector que tiene
en la Bolsa el reflejo de sus actividades.

B) Comercio y transportes
El comercio y el transporte, actividades íntimamente relacionadas, tienen una impor-
tancia vital en la economía. Una vez obtenida la materia prima, tanto la utilizada para la ela-
boración del producto final, como la que aporta la energía necesaria para los procesos
productivos, alguien vende, alguien compra y alguien transporta esa materia hasta una plan-
ta transformadora, una fábrica o un almacén de distribución. Los productos ya manufactu-
rados son igualmente vendidos, comprados y transportados un número variable de
ocasiones hasta que se produce su consumo. Incluso los residuos, tanto de los procesos pro-
ductivos como del propio consumo de los bienes producidos, deben ser transportados para
su almacenamiento o destrucción. De esta forma los sucesivos intercambios y transportes
incrementan el precio del producto y generan un monto de beneficios para los que inter-
vienen en ellos.
1. El comercio presenta dos vertientes en función del nivel de los intercambios. Exis-
te un comercio, y en relación con él un transporte interior, que distribuye los productos del
país dentro de sus propias fronteras; y un comercio exterior que engloba los intercambios
con otros países, las exportaciones y las importaciones. Precisamente, la diferencia entre
ambas es la balanza comercial.
Al margen de las grandes cuentas del comercio exterior, analizando la actividad comercial
desde un punto de vista más cercano a la propia actividad, podemos diferenciar dos vertientes:
– El comercio mayorista, representado por los almacenistas y distribuidores, compra-
dores y vendedores al por mayor.
– El comercio minorista, el más cercano al consumidor, constituido por los pequeños
comercios tradicionales, en descenso, y las grandes superficies o hipermercados,

22 TAMadrid

-Pág.94-
U. D. 3 . - L A A C T I V I D A D H U M A N A Y E L E S PA C I O G E O G R Á F I C O

que mantienen desde hace años un constante crecimiento consecuente al cambio de


los hábitos de consumo de la población.
En los últimos años se ha registrado un considerable aumento de las empresas de logís-
tica, que integran todas las actividades de recepción de pedidos, gestión y control de exis-
tencias, almacenamiento, distribución, etc...
Los innovadores sistemas de televenta, quizás impersonales mas cómodos y competiti-
vos, están generando una nueva filosofía comercial.
2. El sector de los transportes resulta fundamental para el desarrollo económico, se
encuentran en un continuo proceso de mejora y ampliación.
Una primera distinción nos permite apreciar que dentro del mundo de los transportes,
no tienen, lógicamente, el mismo tratamiento las mercancías que las personas. Los tipos de
transporte más importantes son:
– El transporte por carretera. Distribución de mercancías por medio de camiones y
furgonetas de diverso tonelaje y transporte de personas en autobuses, turismos o
motocicletas.
– El transporte marítimo. De enorme importancia en algunas rutas comerciales rela-
cionadas con productos tan trascendentales para la economía mundial como el petró-
leo. El transporte de personas tiene un amplio desarrollo en países en los que la
configuración de su territorio impele a su utilización por la presencia de islas, y en
el ámbito del turismo.
– El transporte fluvial. Muy desarrollado en países con fisonomía adecuada o en los
que se han realizado trabajos de creación de infraestructura mediante la construcción
de canales (es, por ejemplo, el caso, respectivamente, de Venecia y los Países Bajos).
– El ferrocarril. El primer gran medio de transporte utilizado tras la Primera Revolu-
ción Industrial ha experimentado un proceso continuo de mejora. Han aparecido tre-
nes cada vez más rápidos y eficaces para el transporte de personas (la red de trenes
de alta velocidad en Japón o en Europa siguen esa línea).
– El trasporte aéreo, desde su inicio ha ido experimentando un paulatino crecimiento.
En cualquier caso, su importancia, en comparación con los anteriores medios, es
mínima pues el volumen de transporte de mercancías es exiguo, por su encareci-
miento, siendo casi exclusivo el movimiento de pasajeros. Los aeropuertos más
importantes del mundo (Chicago, Londres, New York, Los Ángeles, París...) regis-
tran enormes volúmenes de tráfico de pasajeros.
Por otra parte, existen determinados medios de transporte exclusivos para mercancías:
– El cable, es el soporte para el trasporte de electricidad y, también para el de infor-
mación y comunicaciones (teléfono, fax, internet...). En este sentido, también el aire

TAMadrid
23
-Pág.95-
C I E N C I A S S O C I A L E S . G E O G R A F Í A E H I S TO R I A

y el espacio pueden ser vías de transporte (telefonía móvil, comunicaciones vía saté-
lite, radio, televisión).
– Las tuberías y canales permiten transportar gas, petróleo o agua.
3. Los servicios al consumidor completan las actividades del subsector. Entre ellos se
encuentran las organizaciones de información y defensa de los usuarios y consumidores, ya
sean privadas o públicas, y los servicios de asistencia técnica y servicios post-venta.

C) Comunicación
Las comunicaciones han adquirido una importancia trascendental en la sociedad con-
temporánea. Tanto la orientada a la transmisión de información como la dedicada al entre-
tenimiento, su desarrollo se ha disparado en sentido exponencial.
La edición de revistas y periódicos, se ha visto favorecida por los nuevos sistemas de
composición. La proliferación de ediciones generalistas o especializadas da buena muestra
del desarrollo de esta parcela de la comunicación.
Nombres significativos en el panorama mundial de la información escrita son The Times
(RU), The New York Times, The Wahington Post (EEUU), Franckfurter Algemeine Zeitung
(Alemania), Corriere de la Sera, La Repubblica (Italia), Pravda (Rusia), Le Monde, L'Hu-
manité (Francia), El País (España)...
Las agencias de noticias (Europa Press, United Press, Reuter, AFP, Associated Press...)
controlan la difusión mundial de información, sirviendo a periódicos, revistas, radio y tele-
visión material gráfico y de texto.
La publicidad es una actividad de gran desarrollo que genera un importante movimien-
to de capitales. Las agencias de publicidad elaboran las campañas o simplemente diseñan
anuncios y, junto a agentes libres, gestionan la contratación de paquetes publicitarios en
periódicos, revistas, radio, televisión, carteles, etc... Vital para la difusión entre los consu-
midores del conocimiento de los productos, todas las empresas dedican una parte impor-
tante de sus presupuestos a la misma.
La fotografía es una actividad complementaria de las anteriores y de otras actividades
con una personalidad específica.
La “industria” cinematográfica, es otra actividad importante que mueve una gran can-
tidad de capitales. Los Estados Unidos son los principales productores, manejando unos
presupuestos incomparables y generando unos beneficios desorbitados.
La radio, ha pasado a jugar un papel muy específico en la comunicación, orientado a la
información inmediata y al entretenimiento musical en circunstancias en las que no compi-
te con el gran “monstruo” de la comunicación de hoy, la televisión.

24 TAMadrid

-Pág.96-
U. D. 3 . - L A A C T I V I D A D H U M A N A Y E L E S PA C I O G E O G R Á F I C O

La televisión se ha convertido en el principal medio de comunicación. Información y


entretenimiento están casi dominados por completo por la llamada “caja tonta” que ha ido
extendiendo desde sus orígenes sus tentáculos, abarcando cada vez un mayor ámbito geo-
gráfico de difusión y un horario total, buscando enganchar al mayor número de potenciales
televidentes y convirtiéndose en un miembro más de la familia.
Las alternativas a la difusión herciana terrestre, de la mano del cable y la difusión por
satélite, además de las innovaciones tecnológicas de la transmisión digital han diversifica-
do el mercado y contribuido a la “construcción” de la llamada “aldea global”.
La comunicación por ordenador se ha convertido en otro de los grandes “monstruos” del
final del milenio. El crecimiento de Internet conduce a la consolidación de la idea de la
“aldea global”.
El teléfono ha experimentado un fuerte crecimiento en los últimos años, con la incor-
poración de la tecnología digital, la apertura del mercado y la telefonía móvil.
El telégrafo, telefax, videotexto, completan los sistemas de comunicación, cada vez más
integrados.
Los espectáculos, entre los que sin duda deberíamos incluir los deportes de masas (fút-
bol, fútbol americano, baloncesto, béisbol... su organización, el personal que interviene en
ellos, etc., generan un importante volumen de movimientos de capital, completando el
panorama de este subsector de la comunicación.
Los países desarrollados son los que muestran un panorama más completo en lo que a
los aspectos citados respecta. La globalización de la economía mundial hace que muchos
lleguen, por el interés de las grandes compañías, a los países en desarrollo antes que otros
indispensables para su crecimiento.

D) Hostelería y turismo
Las condiciones físicas de los paisajes, la tradición histórico-artística, las ofertas de ocio
e, incluso, los paisajes urbanos, son los principales motores del turismo.
En los países turísticos, como es el caso de España, Estados Unidos, Italia, Francia, es
una actividad de especial importancia para la economía por su participación en el PIB y,
sobre todo, por la aportación de divisas. Es un sector generador de puestos de trabajo; pero,
en contrapartida, existe mucho empleo temporal, gran heterogeneidad en las condiciones
laborales y mucha economía sumergida.
La actividad del sector se puede desglosar en distintos subsectores:
– Alojamiento (hoteles, hostales, pensiones, fondas, apartamentos, paradores, alber-
gues juveniles, campamentos).

TAMadrid
25
-Pág.97-
C I E N C I A S S O C I A L E S . G E O G R A F Í A E H I S TO R I A

– Restauración (restaurantes, bares, cafeterías, cátering).


– Agencias de viajes.
– Actividades diversas.

E) Sanidad
La sanidad va incrementando su peso en la economía a la vez que se va modernizando,
introduciendo nuevas tecnologías en la prevención, diagnosis y tratamiento.
Dentro de las actividades del subsector podemos distinguir las siguientes:
– Atención primaria y comunitaria: centros de salud y consultorios.
– Servicios generales: analítica, diagnóstico por la imagen, radioterapia, medicina
nuclear, dietética.
– Atención especializada: (en hospitales y clínicas) consultas, cuidados especiales,
medicina estética y deportiva, urgencias, intervenciones quirúrgicas.
– Productos sanitarios: farmacia, óptica, ortopedia, prótesis dentales.

F) Servicios socioculturales
Reúne un conjunto diverso de actividades relacionadas con distintos servicios comuni-
tarios de carácter cultural, educativo y asistencial entre los que cabe citar:
– Museos, exposiciones, bibliotecas y archivos.
– Tiempo libre, juegos, videojuegos.
– Educación de adultos.
– Residencias geriátricas.
– Guarderías.
– Asistencia social.
– Empleados del hogar.

G) Educación
El sector de la educación, no sólo es uno de los más importantes desde el punto de vista
económico sino uno de los de más trascendencia por la incidencia que tiene en la formación
humana, técnica y científica de los ciudadanos.
La enseñanza es, precisamente, un de los aspectos que, como vimos en la Unidad Temá-
tica anterior, miden el grado de desarrollo humano. En todos los países existen unos nive-
les de enseñanza obligatoria, si bien no en todos resulta posible alcanzar el objetivo de
escolarizar a la población infantil, lo que se debe a condiciones socioeconómicas (hábitat
disperso, empleo de los niños como mano de obra en las explotaciones familiares, etc.) y a
las carencias de infraestructuras.

26 TAMadrid

-Pág.98-
U. D. 3 . - L A A C T I V I D A D H U M A N A Y E L E S PA C I O G E O G R Á F I C O

Para dar una idea de la situación de la educación, nada mejor que comparar la situación
española, más cercana y al alcance del conocimiento de todos. Podemos citar básicamente
dos datos: El índice de alfabetización, que alcanza en España al 98% de la población, supe-
rado por pocos países y que contrasta con el más bajo, que corresponde a Níger (12,8%),
por encima de la media mundial (76,3%), y en la media de los países desarrollados. Y el de
escolarización que alcanza el 87%, por encima de la media de los países desarrollados.

H) Administraciones públicas
Incluye todas las actividades relacionadas con la administración y gestión de los orga-
nismos públicos, es decir, la Administración central del Estado (Ministerios, Delegaciones
del Gobierno), Administraciones regionales y locales. Si bien sus actividades carecen de
una valoración contable de beneficios o pérdidas, son vitales para la gestión orgánica.

I) Fuerzas Armadas
Sector esencial para la defensa y la seguridad de los Estados. Los cambios producidos
en la política mundial han permitido una reducción de los gastos armamentísticos, si bien
todavía algunos países mantienen presupuestos elevados por latentes o abiertos conflictos.
Los millonarios presupuestos de Estados Unidos y otros países desarrollados contrastan con
los de algunos países subdesarrollados, no por el montante total de las inversiones sino por
el porcentaje que suponen sobre el PIB correspondiente, mucho más elevado en estos últi-
mos que en los primeros.

NIVELES DE DESARROLLO ECONÓMICO


4. E INTERCAMBIO DESIGUAL EN EL MUNDO

Como hemos ido desgranando en los apartados anteriores, los niveles de desarrollo en
el mundo no son, ni mucho menos, homogéneos. Sin embargo, los parámetros en los que se
fundamentaba hace unos años esta desigualdad se han matizado y modificado considera-
blemente.

La desigualdad aludida parte de las consecuencias derivadas de la Primera y sucesivas


revoluciones industriales y de las condiciones geopolíticas del mundo y su evolución. Efec-
tivamente, en el siglo XIX, después de iniciada la Primera Revolución Industrial, los países
europeos, aquellos en los que ésta tuvo lugar principalmente y que disponían de estructuras

TAMadrid
27
-Pág.99-
C I E N C I A S S O C I A L E S . G E O G R A F Í A E H I S TO R I A

políticas internas más o menos sólidas (Reino Unido, Francia, en menor medida Alemania,
Italia), disponían de un incipiente imperio colonial a lo largo y ancho del mundo. Dicho
imperio terminó convirtiéndose en la fuente de la que obtener las materias primas de con-
sumo, las de abastecimiento de las industrias y las energéticas a precios especialmente bajos
y, además, en el mercado idóneo para los productos manufacturados.
Las condiciones internas de los territorios colonizados no eran objetivo esencial salvo si
se referían a la aculturación, difusión de la religión, el idioma y las costumbres, lo que
redundaría en hábitos de consumo adecuados a los fines expuestos. Pero no preocupaba
excesivamente que estos países se fueran dotando de técnicos medios y superiores compe-
tentes o de empresarios y planteles dirigentes indígenas.
Pero los imperios coloniales comenzaron a resquebrajarse tras la II Guerra Mundial. La
debilitada Europa no podía mantener un imperio en el que los afanes independentistas pug-
naban, muchas veces con la violencia por bandera, por desprenderse del yugo de la metró-
poli, alejada de sus problemas no sólo por los kilómetros de la distancia física. La
consecuente aparición de nuevos Estados, sobre todo en Asia y África (a veces sobre bases
territoriales impuestas por las potencias coloniales, lo que condujo de forma casi inmedia-
ta a conflictos militares fronterizos), produjo un cambio importante en las situación econó-
mica mundial. Ahora, los países desarrollados tenían que comprar las materias primas a los
nuevos Estados los cuales concentraron inicialmente sus esfuerzos en aprovechar el tirón de
su producción sin preocuparse excesivamente por obtener beneficios adecuados y reinver-
tir en un crecimiento paulatino de su industria. El intercambio de esas materias primas con
productos manufacturados, más caros, pero necesarios, endeudó paulatinamente a los nue-
vos Estados con sus ex metrópolis o con nuevas potencias y no sólo desde el punto de vista
económico sino también desde el político (Neocolonialismo), aspecto esencial en la estra-
tegia mundial de bloques generada por la tensión Este-Oeste posterior a la Guerra.
En este marco se mantuvieron y acrecentaron las diferencias entre los países desarro-
llados y los subdesarrollados. En los primeros, las nuevas tecnologías de la tercera revolu-
ción industrial dispararon el crecimiento del nivel de vida, mientras en los segundos se
producía un estancamiento y endeudamiento galopante.
Ahora, los planteamientos globalizadores de la economía hacen que el mundo sea visto
por las grandes empresas multinacionales como un gran mercado de mano de obra, mate-
rias primas y productos de consumo, de modo que las estrategias de obtención de materias
primas, producción, comercialización y gestión tienen en cuenta la rentabilidad de la reali-
zación de estas actividades en unos marcos geográficos o en otros.
Así, la obtención de materias primas, mediatizada, claro está, por su localización geo-
gráfica, es objeto de inversiones para su compra a bajo coste o su producción mediante la

28 TAMadrid

-Pág.100-
U. D. 3 . - L A A C T I V I D A D H U M A N A Y E L E S PA C I O G E O G R Á F I C O

intervención de capital en explotaciones directas (plantaciones, pozos petrolíferos,


minas...).

La elaboración es objeto también de estudios estratégicos, valorando la rentabilidad en


función de la calidad, preparación y costes de la mano de obra y las posibilidades de comer-
cialización desde las zonas de implantación de las factorías. Este punto ha llevado a muchas
empresas a radicar sus plantas de producción en países pobres, con mano de obra muy bara-
ta. Esto ha generado la afluencia de capital a manos de dicha población lo que ha aumenta-
do sus posibilidades de consumo pero no se ha canalizado hacia inversiones en una industria
propia. Es, pues, un nuevo medio de interdependencia.

En todo caso, algunos países han sabido aprovechar el tirón de las nuevas estrategias
productivas y han logrado invertir adecuadamente los beneficios en empresas propias, han
conseguido aumentar el nivel de vida y, en general, el desarrollo económico, social y cul-
tural. Estos Nuevos Países Desarrollados (NPD) van descollando por encima de los que
todavía mantienen unos niveles más bajos, aunque en su entramado social restan aún enor-
mes bolsas de pobreza y subdesarrollo.

En la actualidad, pues, existen:

– Áreas muy desarrolladas, con unos niveles de renta y de bienestar muy elevadas, los
Países Desarrollados (PD).

– Otros países que se acercan a ese nivel pero mantienen importantes problemas
estructurales (caso de algunos de los países ex socialistas en los que la modificación
del sistema económico está generando desequilibrios), se trata de los Países en Vías
de Desarrollo (PVD).

– Otros, como hemos visto, han alcanzado altos niveles de renta y producción indus-
trial recientemente aunque mantengan también problemas de desequilibrio, los Nue-
vos Países Desarrollados (NPD).

– Por fin, otros países se mantienen en niveles muy bajos de desarrollo económico y
humano y, aunque en una visión benevolente podríamos tildarlos de países en vías
de desarrollo, se trata de Países Subdesarrollados (PS).

TAMadrid
29
-Pág.101-
C I E N C I A S S O C I A L E S . G E O G R A F Í A E H I S TO R I A

5. ESPACIO Y PODER POLÍTICO


5.1. LA ORGANIZACIÓN POLÍTICA Y ADMINISTRATIVA DEL
TERRITORIO
Las primeras formas de organización política surgieron como consecuencia de las nece-
sidades de jerarquizar la toma de decisiones y coordinar las actuaciones en el marco de las
primeras ciudades (no podemos tratar como organización política la presumible existencia
de jefes de tribu o clan en época paleolítica, aunque, en sentido estricto, sí lo sea). Por ello,
la ciudad fue, en las primeras civilizaciones, la base de la organización política, aunque
pronto, en algunas de ellas, surgieron poderes más amplios que sumaban, bajo la hegemo-
nía de una ciudad poderosa, diversas ciudades. En este nuevo marco aparecen las relacio-
nes administrativas entre la capital y los territorios del imperio, casi siempre sobre la base
de la dominación e imposición de tributos.

El Imperio romano, que surgió de un modo similar, sumo de territorios sometidos, ter-
minó siendo un entramado administrativo complejo con diversas fórmulas de relación que.
Además fueron evolucionando con el tiempo. Los inmensos territorios del imperio se sub-
dividían en provincias para su administración por medio de gobernadores (procónsules, pro-
pretores). Desde la época de la tetrarquía se crearon unidades administrativas superiores: las
diócesis y las prefecturas e inferiores: los conventus. Por su parte, las ciudades, como uni-
dades específicas mantenían un estatus propio de diversa índole (ciudades libres, estipen-
diarias, inmunes, colonias...)

En la Edad Media el concepto de organización política cambió adquiriendo un carácter


personal. El poder y la soberanía pasaron a ser propiedad personal del monarca y, en tal
razón, podía ser heredada e, incluso repartida entre los herederos. La Monarquía se con-
vierte (salvo escasas excepciones), en la forma de gobierno. El poder político y económico
no es exclusivo del monarca pues la nobleza y la Iglesia tienen también sus parcelas de
poder, a lo que deben enfrentarse aquellos monarcas celosos de la hegemonía de su autori-
dad. La administración regional o local adquiere diferentes fórmulas, bien como delegación
del poder real o como estructuras dependientes de los otros poderes citados o, incluso, autó-
nomos (caso de las ciudades que mantenían sólo la vinculación como súbditos al rey y tení-
an su propia administración)

En la Edad Moderna se forja un nuevo cambio en el concepto de poder político y


territorial. Aparece el nuevo concepto de Estado (Estado Moderno). El primer rasgo defi-
nitorio de dicho Estado es la pretensión de monopolizar el poder coercitivo en su propio territo-
rio. Es decir, el Estado es el único capaz de ejercer la violencia legítima como instrumento de
dominio, rechazándose de plano la posibilidad de que dicha violencia, tal como ocurría en la
Edad Media, pueda ser ejercida por determinada clase social, estamento o grupo tradicional.

Dicho concepto moderno de Estado hacer referencia a una instancia concreta con dos
características:

30 TAMadrid

-Pág.102-
U. D. 3 . - L A A C T I V I D A D H U M A N A Y E L E S PA C I O G E O G R Á F I C O

- El estado queda concebido como una institución política, impersonal y soberana, con
jurisdicción suprema sobre su territorio, con capacidad en exclusiva para promulgar leyes
que regulen de modo público y obligatorio impuestos, cargos, privilegios y obligaciones.
- El estado se erige en una estructura unitaria de poder que pretende ser legítima y que
permanece a través de los cambios de gobernantes y gobernados concretos. Dicho poder
llegará a ser ejercido a través de una burocracia funcionarial específicamente formada
para administrar los asuntos públicos.
Vemos, a continuación, la evolución histórica del concepto de Estado Moderno:
La primera forma que adoptó el Estado Moderno fue la monarquía absolutista del Anti-
guo Régimen en la que el monarca representa la voluntad soberana y su palabra es la ley.
Sin embargo las revoluciones de carácter liberal que se producen desde el siglo XVI en
adelante generan una nueva mentalidad según la cual todos los miembros de la sociedad,
incluidos los gobernantes y el monarca, han de someterse a la ley emanada de la soberanía
popular, abriéndose paso así el concepto de imperio de la ley.
La tradición liberal, basada en la creencia de que el individuo constituye el núcleo de la
actuación política da paso a una forma más evolucionada de Estado Moderno, el Estado
Constitucional y Democrático, manifestaciones del Estado de Derecho. En este concepto de
Estado, se abre paso la sociedad civil como el ámbito de la sociedad que integra al conjun-
to de instituciones y mecanismos de coordinación social no dependientes del sistema admi-
nistrativo estatal.
La siguiente manifestación del Estado Moderno, en su evolución histórica, es el Estado
Social que propone establecer la igualdad material y económica para todas las personas,
dentro de una tradición política socialista y democrática.

El cambio conceptual que se puso en práctica a partir de 1917 en Rusia y después en


otros países europeos y del mundo, el socialismo conducente al comunismo y la final desa-
parición del Estado, propugnado por la praxis marxista, se desarboló desde los años 90 y
sólo permanece como forma de gobierno, con profundas modificaciones sobre los conteni-
dos teóricos del marxismo original, en países como China, Cuba, Vietnam...
Por otra parte, el siglo XX ha sido testigo de otras modificaciones importantes en el con-
cepto de poder y administración territorial, nos referimos a los procesos de integración entre
países, en unos casos con contenidos y razones puramente económicas, en otros con aspi-
raciones más pretenciosas. Han surgido así organizaciones supranacionales como la Unión
Europea, Mercado Común del Sur (MERCOSUR), Tratado de Libre Comercio, etc.

5.2. GRANDES ÁMBITOS GEOPOLÍTICOS


Pretendemos aquí hacer una breve aproximación a los grandes espacios geopolíticos en los
que se articula el mundo de hoy, siguiendo un criterio de integración efectiva por medio de
determinadas organizaciones supranacionales o el de la proximidad cultural y socioeconómica.

TAMadrid
31
-Pág.103-
C I E N C I A S S O C I A L E S . G E O G R A F Í A E H I S TO R I A

5.2.1. LOS ESPACIOS DESARROLLADOS

A) La Unión Europea
El proceso de integración europea se inició con la creación de la Comunidad Económi-
ca Europea por el Tratado de Roma en 1958. Bélgica, Holanda, Luxemburgo, Alemania,
Francia e Italia fueron los países fundadores, a los que se fueron uniendo Irlanda, Dina-
marca y Reino Unido (1973), Grecia (1982), Portugal y España (1986), Suecia, Finlandia y
Austria (1995). Las sucesivas reformas institucionales, a través del Acta Única, el Tratado
de Maastricht y el Tratado de Amsterdam han convertido la CEE en Unión Europea y la han
dotado de políticas comunes en diversos aspectos económicos y ha aumentado la dimensión
política y social del proceso de integración.
Con la firma del tratado de Niza en 2001, se acordó la reforma de las instituciones y una
redistribución del poder entre sus miembros, necesaria a causa de las ampliaciones previstas,
como la del 1 de mayo de 2004 con la incorporación de los siguientes Estados: República
Checa, República Eslovaca, Eslovenia, Letonia, Estonia, Hungría, Chipre, Malta y Polonia.
La Comisión Europea, el Parlamento y el Consejo de Ministros son las instituciones cla-
ves de la Unión y se encuentran en proceso de revisión, al igual que las restantes, con el fin
de hacerlas operativas ante el reto de la integración de los países del centro y este de Europa.
El proceso de integración ha encontrado un punto culminante, desde 1999, con la unión
económica y monetaria. El alumbramiento del euro como moneda única pretende favorecer
los intercambios y acelerar la unión económica.
El viernes 29 de octubre de 2004, los Jefes de Estado o de Gobierno y los Ministros de
Asuntos Exteriores de los 25 Estados de la Unión Europea, firmaron en Roma el tratado por
el que se establece una Constitución para Europa. Un vez firmado el tratado, los Estados
Miembros tendrían que ratificar la Constitución de conformidad con sus respectivas normas
internas. Se prevé que este proceso dure unos dos años. El 20 de febrero de 2005 se apro-
bó la Constitución Europea en España.
El 13 de abril de 2006 el Parlamento Europeo aprueba la entrada de Rumanía y Bulga-
ria en la UE. La adhesión de estos países, tuvo lugar el 1 de enero de 2007, pasando de 25
a 27 los Estados de la UE.
La Unión Europea es, junto con Estados Unidos y Japón, una de las mayores potencias eco-
nómicas mundiales. Tiene una economía saneada y desarrollada, con una agricultura moderna
aunque parcialmente deficitaria, una industria fuerte y cada vez más equilibrada, centrada en
la fabricación de bienes de equipo y un sector terciario muy amplio y de gran peso en el PIB.
El Índice de Desarrollo Humano es el mayor del mundo y los niveles de vida y bienes-
tar social son considerables.

B) Otros países europeos


Además de los países integrados en la Unión Europea y excepción hecha de los inte-
grados en la Comunidad de Estados Independientes, se encuentran en Europa otros países
entre los que podemos distinguir tres grupos:

32
-Pág.104-
U. D. 3 . - L A A C T I V I D A D H U M A N A Y E L E S PA C I O G E O G R Á F I C O

Países que no se integraron en la Unión Europea por propia voluntad.—Se trata de países
desarrollados y de alto nivel de vida como Noruega o Suiza. En el primer caso, y por dos oca-
siones (1972 y 1994), su población se ha expresado contraria a la integración en sendos refe-
réndumes y, en el segundo, su tradicional neutralidad le han llevado a mantenerse al margen.
Países que están a la espera de su integración.— Turquia.
Otros países.—Se incluyen otros países que, en principio, por diversos condicionantes
económicos o políticos, no aparecen entre los que afrontan el reto de la Unión (Albania,
Croacia, Macedonia...)

C) La Comunidad de Estados Independientes (CEI)


La ruptura de la Unión Soviética en 1991 condujo a la formación de 15 Estados inde-
pendientes de los que 12 terminaron integrándose en una organización de cooperación eco-
nómica que no ha alcanzado el protagonismo que cabría esperar de la situación precedente.
Rusia por su peso demográfico, superficial y de recursos es el puntal principal, junto con
Ucrania, Bielorrusia, Georgia, etc.
La economía dirigida ha dado paso a una transformación hacia la economía de merca-
do lo que, por el momento, ha generado una importante crisis. La gran cantidad de recur-
sos naturales y la potencialidad de su entramado industrial y de servicios terminarán siendo
el motor de su desarrollo.

D) Estados Unidos y Canadá


Estados Unidos pasa por ser el rector de la economía mundial gracias a sus enormes
recursos de materias primas y su desarrollo industrial. Ocupa una posición puntera en cuan-
do a la producción y reservas de determinadas materias primas (petróleo, carbón, gas natu-
ral...) y domina buena parte de los sectores industriales. Su poder financiero, la fortaleza e
internacionalidad del dólar y la proyección de las grandes empresas multinacionales de ori-
gen estadounidense le conceden ese papel preeminente en la economía mundial, papel que
se extiende también al ámbito político.

E) Japón
Junto con Estados Unidos y Europa es la otra gran potencia mundial. Su desarrollo
industrial es comparable al de los otros ámbitos pero no así sus posibilidades agrarias y de
materias primas en las que se muestra claramente deficitario (dentro del sector primario
sólo es importante la pesca).
La industria, basada en grandes consorcios de proyección multinacional (a los que tie-
nen un carácter familiar se les denomina zaibatsus), se orienta sobre todo a la producción
de automóviles, construcción naval y electrónica.

33

-Pág.105-
C I E N C I A S S O C I A L E S . G E O G R A F Í A E H I S TO R I A

F) Australia y Nueva Zelanda


El empuje de estos países les sitúa, tras Japón, como las mayores potencias económicas
del Pacífico. En el caso de éstos, y a diferencia de Japón, destaca la riqueza de materias pri-
mas. Ejercen un enorme influjo en los pequeños países de la zona y su comercio tiene una
proyección importante hacia las grandes potencias mundiales.
G) Los Nuevos Países Desarrollados del Sureste Asiático
Ya hemos visto que la clave del desarrollo en estos países, entre los que se encuentra
Corea del Sur, Singapur, Taiwán y el territorio chino de Hong Kong, ha sido el crecimien-
to de la industria sobre la base de inversiones extranjeras movidas por los bajos costos de la
mano de obra.
H) China
Es el país con mayor población de la Tierra. El sistema comunista de economía dirigida
se ha ido modificando desde los poderes del Estado dando cabida a sistemas mixtos y a
Zonas Económicas Especiales con experimentos de economía de mercado. Grandes recur-
sos, importante industria pesada y un mercado potencial inmenso, conceden a China una
importancia vital en el panorama económico mundial.

5.2.2. PAÍSES SUBDESARROLLADOS


El térmico Tercer Mundo, utilizado también para designar a estos países tuvo su base en
la existencia, durante la época de la Guerra Fría, de dos ámbitos políticos antagónicos fren-
te a los que estos países constituían una realidad aparte, si bien dependiente, a la postre, de
alguno de los dos bloques. En la actualidad son características que definen a estos países:
– Importantes recursos naturales agrícolas, minerales y energéticos.
– Escaso desarrollo de la industria, aunque en algunos países se produce una inci-
piente aparición de polos industriales.
– Dependencia financiera de los países desarrollados.
– Fuerte endeudamiento.
– Bajo nivel de vida.
– Bajo nivel educativo.
– Bajo nivel asistencial.
E) Los países árabes
Tienen entre sí vínculos religiosos y culturales importantes, sin embargo, los intentos de
integración real y efectiva entre ellos no han alcanzado grandes resultados. La Liga Árabe
es la asociación que engloba a todos ellos (Marruecos, Argelia, Libia, Egipto, Siria, Irak,
Arabia....), si bien su papel no deja de ser básicamente testimonial.
La economía de estos países se basa en la explotación de recursos naturales, fundamen-
talmente energéticos. Su dependencia de la producción de petróleo, gas natural o fosfatos,

34
-Pág.106-
U. D. 3 . - L A A C T I V I D A D H U M A N A Y E L E S PA C I O G E O G R Á F I C O

es trascendental, por lo que las inversiones industriales son bajas. Los enormes ingresos
procedentes del petróleo no son adecuadamente repartidos existiendo fuertes desequilibrios
sociales.
La problemática del integrismo islámico y el fanatismo que éste y otros condicionantes
generan en algunos países ha sido un factor de tensión importante en los últimos años a
nivel internacional (Guerra Irak-Irán, Guerra del Golfo...)

F) África subsahariana
Engloba a los países con mayoría de población negra. Aunque se podrían diferenciar
áreas (básicamente el ámbito francófono del Oeste y Centro, de una parte, el anglófono del
Este y el Sur, de otro, y Sudáfrica).
Sudáfrica se ha convertido, sobre todo tras la retroacción protagonizada por Francia, en el
país rector de la economía de la zona. Esto se debe a su potencialidad industrial sustentada
en unos impresionantes recursos mineros y a la capacidad financiera que ha generado. Ade-
más, desde la reforma constitucional de 1994, que rompió definitivamente con el sistema del
apartheid (segregación racial institucional), su proyección mundial ha sido creciente.
El escaso desarrollo económico en todos los órdenes de extensas áreas es característico de
este ámbito, con una agricultura y ganadería de subsistencia, una industria prácticamente ine-
xistente y un mínimo desarrollo de los servicios. La riqueza minera de algunos de ellos o la de
productos de interés económico mundial (café, banana, caucho...) justifican algunas inversio-
nes, aunque África, por lo general, es el continente menos agraciado por tales inversiones.

G) Sureste asiático
Países con enormes recursos naturales, explotados por interés de los circuitos económicos
internacionales, tienen grandes contrastes, con paradojas tan dramáticas como la existencia de
inversiones estatales en producción de armamento nuclear o en desarrollo de satélites artifi-
ciales mientras la mayor parte de la población permanece por debajo de los umbrales de
pobreza, sin recursos, sin asistencia sanitaria, sin vivienda...(es el caso de India o Pakistán).
El ejemplo de sus vecinos, los NPD que veíamos más arriba, es la esperanza de estos
países en los que, además, para aumentar los problemas, existe un crecimiento demográfi-
co poco controlado.

H) Iberoamérica
Aunque existen áreas de desarrollo industrial importante, la mayor parte de los países
iberoamericanos tienen bolsas de pobreza muy importantes. Aquí también la explotación de
recursos agrícolas y minerales es esencial para la economía. Los sistemas de propiedad y
explotación crean fuertes desequilibrios entre los grupos enriquecidos y las masas pobres
que se traducen en una conflictividad sociopolítica importantísima. Es el caso de la zona
meridional de México o el de Colombia, donde todo se complica con los intereses enfren-
tados por el negocio de la droga.

35
-Pág.107-
C I E N C I A S S O C I A L E S . G E O G R A F Í A E H I S TO R I A

5.3. EL ESPACIO POLÍTICO-ADMINISTRATIVO


DEL TERRITORIO ESPAÑOL
De acuerdo con la Constitución española (1978), el Estado se articula en Comunidades
Autónomas, para cuyo establecimiento, la Constitución fija los mecanismos legales que
deben producirse. El proceso de formación y desarrollo del sistema autonómico quedó con-
solidado con la aprobación de los diferentes estatutos de autonomía de las 17 regiones que
se constituyeron y con la celebración de elecciones a los respectivos parlamentos autóno-
mos que concluyeron en 1983. El colofón fue la aprobación de los estatutos de las ciudades
autónomas de Ceuta y Melilla.

Así pues, respetando los límites provinciales de las 50 provincias, han resultado 7
Comunidades uniprovinciales (Principado de Asturias, Cantabria, La Rioja, Navarra, Illes
Balears, Madrid y Murcia) y 10 multiprovinciales (Galicia, País Vasco, Aragón, Cataluña,
Comunidad Valenciana, Castilla y León, Castilla-La Mancha, Extremadura, Canarias y
Andalucía), además de Ceuta y Melilla.

El desarrollo económico de España ha determinado importantes desequilibrios regiona-


les. Así, el País Vasco, Asturias y Cataluña fueron los primeros focos de instalación indus-
trial con la consiguiente atracción de mano de obra. Surgieron después otros núcleos como
Valencia, Madrid, Valladolid, Vigo, Zaragoza. Este hecho, unido a la crisis de la agricultu-
ra, derivó en un paulatino despoblamiento de las zonas rurales del interior y un crecimien-
to de los núcleos citados.

Por otra parte, la crisis industrial derivada de la general internacional de los años 70 y
80 y de las necesarias adaptaciones a las exigencias comunitarias, motivó la desaparición
de algunas industrias con la consiguiente desaparición de instalaciones y pérdida de pues-
tos de trabajo. La falta de preparación de las estructuras industriales y económicas en gene-
ral para estos problemas coyunturales, generó un aumento extraordinario del paro. La
bonanza económica de los 90 y algunos aciertos en la política económica, llevaron a un cre-
cimiento importante del PNB a finales de los 90 y una considerable disminución del paro.
Sin embargo, persiste un porcentaje muy elevado de población activa en paro y con fuertes
desequilibrios regionales que hacen de Andalucía y Extremadura las CC.AA. con mayores
tasas.

Estos desequilibrios económicos son uno de los principales problemas que debe afron-
tar la política de solidaridad regional, mediante un adecuado sistema de financiación de las
CC.AA. y una aportación equitativa de todas al desarrollo general y a la solución de tales
desequilibrios.

Exponemos a continuación los datos más significativos delas CC.AA.

36 TAMadrid

-Pág.108-
U. D. 3 . - L A A C T I V I D A D H U M A N A Y E L E S PA C I O G E O G R Á F I C O

Galicia
Nombre Oficial: Comunidad Autónoma de Galicia.
Estatuto: L.O. 1/81 de 6 de abril de 1981.
Población: 2.731.669.
Extensión: 29.434 Km2.
Densidad: 93 h/Km2.
Límites: N: Cantábrico.
E: Asturias, Castilla y León (León y Zamora).
S: PORTUGAL.
O: Atlántico.
Provincias: La Coruña/A Coruña, Lugo, Orense/Ourense y Pontevedra.
Capital: Santiago de Compostela.

Asturias
Nombre Oficial: Principado de Asturias.
Estatuto: L.O. 7/81 de 30 de diciembre de 1981.
Población: 1.093.937.
Extensión: 10.565 Km2.
Densidad: 104 h/Km2.
Límites: N: Cantábrico.
E: Cantabria.
S: Catilla y León (León).
O: Galicia (Lugo).
Provincias: Asturias.
Capital: Oviedo.

Cantabria
Nombre Oficial: Diputación Regional de Cantabria.
Estatuto: L.O. 8/81 de 30 de diciembre de 1981.
Población: 527.326.
Extensión: 5.289 Km2.
Densidad: 100 h/Km2.
Límites: N: Cantábrico.
E: País Vasco (Vizcaya).
S: Catilla y León (León, Palencia, Burgos).
O: Asturias.
Provincias: Cantabria.
Capital: Santander.

TAMadrid
37
-Pág.109-
C I E N C I A S S O C I A L E S . G E O G R A F Í A E H I S TO R I A

País Vasco
Nombre Oficial: País Vasco/Euskadi.
Estatuto: L.O. 3/79 de 18 de diciembre de 1979.
Población: 2.104.041.
Extensión: 7.261 Km2.
Densidad: 289 h/Km2.
Límites: N: Cantábrico.
E: FRANCIA, Navarra.
S: Castilla y León (Burgos), La Rioja.
O: Cantabria.
(Casos especiales del Condado de Treviño, provincia de Burgos, dentro de
Álava; y Villaverde de Trucios, provincia de Cantabria, dentro de Vizcaya.)
Provincias: Vizcaya/Bizkaia, Guipúzcoa/Gipuzkoa, Álava/Araba.
Capital: Vitoria/Gazteiz.

Navarra
Nombre Oficial: Comunidad Foral de Navarra.
Estatuto: L.O. 13/82 de 10 de agosto de 1982.
Población: 519.277.
Extensión: 10.421 Km2.
Densidad: 50 h/Km2.
Límites: N: FRANCIA.
E: Aragón (Huesca, Zaragoza).
S: La Rioja.
O: País Vasco (Guipúzcoa, Álava).
Provincias: Navarra.
Capital: Pamplona.

La Rioja
Nombre Oficial: Comunidad Autónoma de La Rioja.
Estatuto: L.O. 3/82 de 9 de junio de 1982.
Población: 263.434.
Extensión: 5.034 Km2.
Densidad: 52 h/Km2.
Límites: N: País Vasco (Álava), Navarra.
E: Navarra, Aragón (Zaragoza).
S: Castilla y León (Soria).
O: Castilla y León (Burgos).
Provincias: La Rioja.
Capital: Logroño.

38 TAMadrid

-Pág.110-
U. D. 3 . - L A A C T I V I D A D H U M A N A Y E L E S PA C I O G E O G R Á F I C O

Aragón
Nombre Oficial: Comunidad Autónoma de Aragón.
Estatuto: L.O. 8/82 de 10 de agosto de 1982.
Población: 1.188.817.
Extensión: 47.650 Km2.
Densidad: 25 h/Km2.
Límites: N: FRANCIA.
E: Cataluña (Lleida, Tarragona), Valencia (Castellón, Valencia)
S y O: Castilla-La Mancha (Cuenca, Guadalajara), Castilla y
León (Soria), La Rioja, Navarra.
Provincias: Zaragoza, Huesca y Teruel.
Capital: Zaragoza.

Cataluña
Nombre Oficial: Cataluña (Generalitat de Catalunya).
Estatuto: L.O. 4/79 de 18 de diciembre de 1979.
Población: 6.059.494.
Extensión: 31.930 Km2.
Densidad: 190 h/Km2.
Límites: N: FRANCIA.
E: Mediterráneo.
S: C. Valenciana (Castellón).
O: Aragón.
Provincias: Lleida/Lérida, Girona/Gerona, Barcelona, Tarragona.
Capital: Barcelona.

Castilla y León
Nombre Oficial: Castilla y León (Junta de Castilla y León)
Estatuto: L.O. 4/83 de 25 de febrero de 1983
Población: 2.545.926
Extensión: 94.193 Km2
Densidad: 27 h/Km2
Límites: N: Asturias, Cantabria, P. Vasco, La Rioja
E: Aragón ( Zaragoza )
S: Castilla-La Mancha (Guadalajara, Toledo), Extremadura
(Cáceres), Madrid
O: PORTUGAL, Galicia (Lugo, Orense)
Provincias: León, Zamora, Salamanca, Palencia, Valladolid, Ávila, Burgos,
Segovia, Soria.
Capital: Valladolid.

TAMadrid
39
-Pág.111-
C I E N C I A S S O C I A L E S . G E O G R A F Í A E H I S TO R I A

Castilla-La Mancha
Nombre Oficial: Castilla-La Mancha (Junta de Comunidades de Castilla-La Mancha).
Estatuto: L.O. 9/82 de 10 de agosto 1982.
Población: 1.658.446.
Extensión: 79.230 Km2.
Densidad: 21 h/Km2.
Límites: N: Castilla y León (Soria, Segovia, Ávila), Madrid.
E: Aragón (Zaragoza y Teruel), Valencia (Valencia y Alicante).
S: Andalucía (Granada, Jaén, Córdoba), Murcia.
O: Extremadura (Cáceres y Badajoz).
Provincias: Guadalajara, Toledo, Cuenca, Ciudad Real, Albacete.
Capital: Toledo.

Madrid
Nombre Oficial: Comunidad de Madrid.
Estatuto: L.O. 3/83 de 25 de febrero de 1983.
Población: 5.030.958.
Extensión: 7.995 Km2.
Densidad: 618 h/Km2.
Límites: N y O: Castilla y León (Segovia, Ávila).
E: Castilla-La Mancha (Guadalajara).
S: Castilla-La Mancha (Toledo, Cuenca).
Provincias: Madrid.
Capital: Madrid.

Comunidad Valenciana
Nombre Oficial: Comunidad Valenciana (Generalidad).
Estatuto: L.O. 5/82 de 1 de julio de 1982.
Población: 3.857.234.
Extensión: 23.305 Km2.
Densidad: 166 h/Km2.
Límites: N: Cataluña (Tarragona).
E: Mediterráneo.
S: Murcia.
O: Aragón (Teruel), Castilla-La Mancha (Cuenca, Albacete),
(Caso especial del Rincón de Ademuz, entre Teruel y Cuenca).
Provincias: Castellón, Valencia y Alicante.
Capital: Valencia.

40 TAMadrid

-Pág.112-
U. D. 3 . - L A A C T I V I D A D H U M A N A Y E L E S PA C I O G E O G R Á F I C O

Baleares
Nombre Oficial: Comunidad Autónoma de las Islas Baleares (Illes Balears).
Estatuto: L.O. 2/83 de 25 de febrero de 1983.
Población: 709.138.
Extensión: 5.014 Km2.
Densidad: 141 h/Km2.
Límites: Mar Mediterráneo.
Provincias: Baleares.
Capital: Palma de Mallorca.

Extremadura
Nombre Oficial: Extremadura (Junta de Extremadura).
Estatuto: L.O. 1/83 de 25 de febrero de 1983.
Población: 1.061.852.
Extensión: 41.602 Km2.
Densidad: 26 h/Km2.
Límites: N: Catilla y León (Salamanca y Ávila).
E: Castilla-La Mancha (Toledo, C. Real).
S: Andalucía (Córdoba, Sevilla, Huelva).
O: PORTUGAL.
Provincias: Cáceres y Badajoz.
Capital: Mérida.

Andalucía
Nombre Oficial: Andalucía ( Junta de Andalucía ).
Estatuto: L.O. 6/81 de 30 de diciembre de 1981.
Población: 6.940.522.
Extensión: 87.268 Km2.
Densidad: 80 h/Km2.
Límites: N: Extremadura (Badajoz), Castilla-La Mancha (C. Real, Albacete).
E: Murcia, Mediterráneo.
S: Mediterráneo, GIBRALTAR, Atlántico.
O: PORTUGAL.
Provincias: Huelva, Sevilla, Cádiz, Córdoba, Málaga, Jaén, Granada, Almería.
Capital: Sevilla.

TAMadrid
41
-Pág.113-
C I E N C I A S S O C I A L E S . G E O G R A F Í A E H I S TO R I A

Murcia
Nombre Oficial: Región de Murcia (Consejo de Gobierno).
Estatuto: L.O. 4/82 de 9 de junio de 1982.
Población: 1.045.601.
Extensión: 11.317 Km2.
Densidad: 92 h/Km2.
Límites: N: Castilla-La Mancha (Albacete).
E: Valencia (Alicante).
S: Mediterráneo.
O: Andalucía (Granada, Almería).
Provincias: Murcia.
Capital: Murcia.

Canarias
Nombre Oficial: Canarias.
Estatuto: L.O. 10/82 de 10 de agosto de 1982.
Población: 1.493.784.
Extensión: 7.242 Km2.
Densidad: 205 h/Km2.
Límites: Atlántico.
Provincias: Las Palmas y Santa Cruz de Tenerife.
Capital: Santa Cruz de Tenerife y Las Palmas de Gran Canaria.

Ceuta
Población: 73.208.
Extensión: 19 Km2.
Densidad: 3.853 h/Km2.
Límites: N, E, S: Mediterráneo.
O: MARRUECOS.

Melilla
Población: 63.670.
Extensión: 13,4 Km2.
Densidad: 4.751 h/Km2.
Límites: N, E, S: Mediterráneo.
O: MARRUECOS.
Dependen también de Melilla el Peñón de Alhucemas, el Peñón de Vélez y las Islas Chafarinas.

42 TAMadrid

-Pág.114-
U. D. 3 . - L A A C T I V I D A D H U M A N A Y E L E S PA C I O G E O G R Á F I C O

RESUMEN
LAS ACTIVIDADES AGRARIAS Y EL ESPACIO RURAL
Factores
– Físicos: Clima, relieve, suelo.
– Humanos: Regadíos, roturación, fertilización, mecanización, tecnología, técnicas
biológicas.
Rendimiento de la producción por hectárea, rendimiento por persona empleada
Explotación intensiva: rendimiento alto por hectárea; bajo por persona empleada.
Explotación extensiva: rendimiento bajo por hectárea; alto por persona empleada.
Paisajes rurales
Forma: regular o irregular.
Extensión: minifundios o latifundios.
Límites: abiertos o cerrados.
Tipos de cultivo: arbóreos, arbustivos o herbáceos; monocultivo o policultivos; secano o
regadío.
Hábitat rural: disperso o concentrado.
Sistemas y espacios agrarios
La agricultura subdesarrollada
Revolución verde (empleo de semillas tratadas genéticamente y de maquinaria para
aumentar los rendimientos).
Agricultura itinerante o de roza.—Espacios de selva preparados. Productos de consumo
inmediato. Agotamiento de los suelos. Rendimientos muy bajos.
Agricultura sedentaria de subsistencia.—Huertas centrales y cultivos rotativos de tres
hojas, una en barbecho.
Agricultura intensiva monzónica.—Cultivos de arroz. Mucha mano de obra.
La agricultura desarrollada
Productividad, competitividad, tecnificación, rendimientos muy elevados.
Explotaciones intensivas (hortalizas); explotaciones extensivas de cereales.
Unión Europea: Política Agraria Común (PAC), excedentes => subvención para el cam-
bio de producción y limitación mediante cuotas.
La agricultura de plantación
Grandes inversiones de capitales extranjerosy amplios beneficios para las empresas.
Monocultivos (caucho, cacao, bananas, café, té...), con mano de obra barata y tecnolo-
gía y rápida comercialización.
La agricultura dirigida
Planificación estatal. Queda en China, Cuba, Vietnam

TAMadrid
43
-Pág.115-
C I E N C I A S S O C I A L E S . G E O G R A F Í A E H I S TO R I A

La Ganadería
Ganadería intensiva.—Explotaciones pequeñas, tecnificadas con rendimientos muy
altos(leche, carne, huevos...).
Ganadería extensiva.—Explotaciones en grandes espacios abiertos (lana, carne, piel).
La Pesca
Agotamiento de los caladeros tradicionales.
Zonas de interés económico y aguas jurisdiccionales en las 200 millas.
Limitación de las capturas y concesión de licencias.
Medios técnicos.
Riesgos de la contaminación.
La explotación forestal

ACTIVIDADES Y ESPACIOS INDUSTRIALES


La explotación de materias primas y productos energéticos
Materias Primas
Minerales metálicos.—Hierro, cobre, estaño, mercurio, alumnio, oro, plata y otros.
Minerales no metálicos.—Azufre, fosfatos, potasio, arcillas, cal, yeso, cemento.
Productos energéticos
– Productos no renovables:
– Carbón.—Antracita, hulla, lignito, turba. Centrales térmoeléctricas. Problemas de
contaminación.
– Petróleo.—Dependencia de la economía mundial del petróleo. Combustibles automo-
ción, maquinaria y centrales termoeléctricas. Papel de la OPEP. Fuerte contaminación.
– Gas Natural.—Usos domésticos, centrales termoeléctricas. Es más limpio.
– Energía nuclear.—Fisión nuclear peligrosa y contaminante, desapareciendo. Fusión
en proceso de investigación.
– Productos renovables
– Energía hidroeléctrica.
– Energía eólica.
– Energía solar.—Fotovoltáica y térmica.
– Energía geotérmica.
– Energía maremotriz.
– Otras.—Combustión de materia orgánica, alcoholes...
La Industria
Evolución de la industria
En la antigüedad modo artesanal.

44 TAMadrid

-Pág.116-
U. D. 3 . - L A A C T I V I D A D H U M A N A Y E L E S PA C I O G E O G R Á F I C O

En el siglo XVIII Primera Revolución Industrial: máquina de vapor y carbón; ferroca-


rril, barco de vapor. Industria textil y siderúrgica.
A finales del siglo XIX Segunda Revolución Industrial: petróleo y electricidad; automó-
vil, tren eléctrico, avión, barco gasóleo. Industria del automóvil, metalurgia y química.
En la actualidad nueva revolución: energía eléctrica, informática y automatización;
industria electrónica, biogenética y aerospacial.
Globalización y predominio del sector terciario
Las bases de la industria actual
La mano de obra.
Los empresarios.
Entidades financieras.
Tecnología.
Mercado.
Transportes.
Instalaciones.
Política
Los sectores industriales
A) Alimentación, bebidas y tabaco
B) Textil, confección, piel.
C) Madera, corcho y muebles.
D) Artes Gráficas.
E) Sector energético.
F) Productos químicos.
G) Fabricación mecánica.
1. Industrias de construcción y materiales de equipo
2. Fabricación de material de transporte
3. Construcciones metálicas
4. Industria pesada
H) Construcción.
Localización de las industrias
Factores.—Mano de obra, materias primas, proximidad a los mercados, creación de
polígonos industriales, subvemciones estatales, condiciones de producción.
Los espacios industriales y su desigual distribución en el mundo
La mayor parte de las concentraciones industriales en Japón, Estados Unidos y Europa.
En Asia han surgido los Nuevos Países Desarrollados (NPD).
En África, sólo Sudáfrica.
América del Sur, despegue asociado a grandes ciudades.
Australia y Nueva Zelanda grandes polos de industrialización.

TAMadrid
45
-Pág.117-
C I E N C I A S S O C I A L E S . G E O G R A F Í A E H I S TO R I A

La industria en España
España es un país industrializado.
Áreas originales: Cataluña, País Vasco, Asturias.
Desarrollismo años 60: se unen Madrid, Valencia, Vigo, Zaragoza, Valladolid.
Crisis de los años 70 y 80
Entrada en la Comunidad Económica Europea y Reconversión.
Sectores punteros: Automóviles, químico, calzado y cuero, alimentación.

LAS ACTIVIDADES TERCIARIAS Y SU DESARROLLO ACTUAL


A) Entidades financieras. Bancos, bolsa, seguros.
B) Comercio y transportes.
1. Comercio interior y exterior (=>balanza comercial).
2. Transportes por carretera, marítimo, aéreo, fluvial, ferrocarril; cable, canales.
3. Servicios al consumidor
C) Comunicación. Revistas y periódicos, agencias de noticias, publicidad, fotografía,
cine, radio, televisión, teléfono, espectáculos...
D) Hostelería y turismo.
E) Sanidad.
F) Servicios socioculturales.
G) Educación.
H) Administraciones públicas.
I) Fuerzas Armadas.

NIVELES DE DESARROLLO ECONÓMICO E INTERCAMBIO


DESIGUAL EN EL MUNDO
Imperios coloniales europeos: Fuente de materias primas baratas y mercado de produc-
tos manufacturados
Independencia de nuevos países: PD compran materias primas y vendan productos
industriales: balanza desigual que propicia una nueva dependencia económica y política
(neocolonialismo).
Hoy: planteamientos globalizadores de la economía. Materias primas, producción con
mano de obra barata, comercialización mundial:
– Países Desarrollados (PD).
– Países en Vías de Desarrollo (PVD).
– Nuevos Países Desarrollados (NPD).
– Países Subdesarrollados (PS).

46 TAMadrid

-Pág.118-
U. D. 3 . - L A A C T I V I D A D H U M A N A Y E L E S PA C I O G E O G R Á F I C O

ESPACIO Y PODER POLÍTICO


La organización política y administrativa del territorio

En la Edad Media El poder y la soberanía son propiedad personal del monarca.


En la Edad Moderna absolutismo monárquico.
A partir del siglo XVIII, despotismo ilustrado.
Siglo XIX liberalismo (Independencia EE.UU, Revolución Francesa). Poder y soberanía
del pueblo.
En el siglo XX procesos de integración entre países (Unión Europea).
Grandes ámbitos geopolíticos
Los espacios desarrollados
A) La Unión Europea.
Tratado de Roma, Acta Única, Tratado de Maastricht, T. de Amsterdam y T. de Niza.
Bélgica, Holanda, Luxemburgo, Alemania, Francia e Italia (países fundadores),
Irlanda, Dinamarca y Reino Unido (1973), Grecia (1982), Portugal y España
(1986), Suecia, Finlandia y Austria (1995), República Checa, República Eslovaca,
Eslovenia, Letonia, Lituania, Estonia, Hungría, Chipre, Malta y Polonia (2004).
Comisión Europea, Parlamento, Consejo de Ministros.
Unión económica y monetaria (euro).
Economía saneada y desarrollada.
B) Otros países europeos.
C) La Comunidad de Estados Independientes (CEI).
Rusia y otros 11 Estados de la antigua Unión Soviética.
Problemas por el cambio de economía dirigida a economía de mercado.
D) Estados Unidos y Canadá.
Estados Unidos rector de la economía y la política mundial.
Producción y reservas de materias primas.
Domina buena parte de los sectores industriales.
Poder financiero, y fortaleza del dólar.
Implantación mundial de sus empresas.
E) Japón.
Potencia mundial.
Desarrollo industrial pero deficitario en materias primas.
Proyección mundial de sus multinacionales (Zaibatsus).
F) Australia y Nueva Zelanda. Potencias económicas del Pacífico.
G) Los NPD del Sureste Asiático. Corea del Sur, Singapur, Taiwán y Hong Kong.
H) China. Sistema comunista con Zonas Económicas Especiales.

TAMadrid
47
-Pág.119-
-Pág.119-
C I E N C I A S S O C I A L E S . G E O G R A F Í A E H I S TO R I A

Países subdesarrollados
– Importantes recursos naturales agrícolas, minerales y energéticos.
– Escaso desarrollo de la industria, aunque en algunos países se produce una inci-
piente aparición de polos industriales.
– Dependencia financiera de los países desarrollados.
– Fuerte endeudamiento.
– Bajo nivel de vida.
– Bajo nivel educativo.
– Bajo nivel asistencial.
A) Los países árabes. Recursos naturales energético. Fuertes desequilibrios.
B) África subsahariana.
Sudáfrica país rector de la economía de la zona.
Agricultura y ganadería de subsistencia
Industria prácticamente inexistente y un mínimo desarrollo de los servicios.
C) Sureste Asiático. Recursos naturales, pobreza, explosión demográfica.
D) Iberoamérica. Bolsas de pobreza, fuertes desequilibrios y conflictividad sociopolítica.
El espacio político-administrativo del territorio español
Constitución española (1978): el Estado se articula en Comunidades Autónomas.
50 provincias, 7 Comunidades uniprovinciales (Principado de Asturias, Cantabria, La
Rioja, Navarra, Illes Balears, Madrid y Murcia) y 10 multiprovinciales (Galicia, País
Vasco, Aragón, Cataluña, Comunidad Valenciana, Castilla y León, Castilla-La Mancha,
Extremadura, Canarias y Andalucía), además de Ceuta y Melilla.
Importantes desequilibrios regionales. Solución mediante un adecuado sistema de
financiación de las CC.AA. y una aportación equitativa de todas al desarrollo general.

48 TAMadrid

-Pág.120-
U. D. 3 . - L A A C T I V I D A D H U M A N A Y E L E S PA C I O G E O G R Á F I C O

EJERCICIOS DE AUTOCOMPROBACIÓN

1. Señale cuál de las siguientes relaciones es correcta:

A. Agricultura itinerante-espacios ganados a la selva


B. Agricultura sedentaria de subsistencia-selvas tropicales
C. Agricultura intensiva monzónica-producción de maíz y verduras
D. Revolución verde-Limitación de la producción para proteger las especies

2. La agricultura de plantación se caracteriza, entre otros aspectos, por:

A. Grandes inversiones de los países donde se ubican las plantaciones


B. Se explotan monocultivos de productos con gran demanda internacional
C. La mano de obra empleada es escasa, lo que abarata los costes
D. La pobreza de los medios técnicos empleados se contrarresta con los altos rendimientos

3. Respecto a la pesca, podemos afirmar que:

A. La riqueza de los caladeros tradiconales permite la producción de excedentes


B. El establecimiento de los límites jurisdiccionales en las 2000 millas peremitió a mucho paí-
ses controlar mejor sus recursos
C. Los medios técnicos asociados a las labores de la pesca son cada vez más sofisticados
D. Los países que controlan los caladeros se someten a los dictados de los países capturadores
y conceden monopolios de explotación a algunos países

4. El carbón procede de masas de vegetales metamorfizados tras quedar sepultadas y se presenta


en cuatro variedades. Indique en cuál de las siguientes relaciones se presentan todas ellas y
ordenadas de mayor a menor poder calórico:

A. Grafito, andesita, bauxita y potasa B. Antracita, hulla, lignito y turba


C. Hierro, cobre, lignito y hulla D. Turba, hulla, lignito y andesita

5. ¿Cuál es la organización en la que se coordinan las políticas de producción del petróleo?

A. La Comisión Nacional de Hidrocarburos B. La Organización de las Naciones Unidas

C. La Liga Árabe D. La Organización de Países Exportadores de


d.– Petróleo

6. De las siguientes materias primas energéticas ¿Cuál podemos considerar como más limpia?

A. Carbón B. Petróleo C. Fisión nuclear D. Gas natural

TAMadrid
49
-Pág.121-
C I E N C I A S S O C I A L E S . G E O G R A F Í A E H I S TO R I A

7. En relación con la evolución del sector industrial, señale la relación errónea:


A. Primer Revolución Industrial-Carbón-Textiles, siderurgia-Ferrocarril
B. Segunda Revolución Industrial-Petróleo-Químicas-Automóviles
C. Revolución Industrial actual-Electricidad-Electrónica-Ferrocarril a vapor
D. Edad Media-Producción artesanal-Gremios

8. Las siglas NPD corresponden a:


A. Naciones Poderosas y Desarrolladas B. Núcleo de Países Democráticos
C. Nuevos Países Desarrollados D. Núcleos Polivalentes de Desarrollo

9. El problema más importante que tienen los países subdesarrollados en la actualidad es:
A. Que su mano de obra es muy barata B. Que padecen un endeudamiento externo enorme
C. Que carecen de las necesarias materias D. Que su industria genera mucho paro y unos
c.– primas D. precios altos

10. La integración de España en la Unión Europea se produjo en:


A. 1978 B. 1986 C. 1991 D. 1995

11. Las comunicaciones han adquirido una importancia trascendental en la sociedad contemporá-
nea. En este terreno, uno de los servicios tradicionales es el de la prensa escrita. Dentro de ella
hay importantes periódicos de gran influencia social. En relación con ellos indique cuál es la
relación periódico-país que es correcta:
A. The Times-Estados Unidos B. Franckfurter Allgemeine Zeitung-Rusia
C. Le Monde-Italia D. La Repubblica-Italia

12. Entendemos por Neocolonialismo:


A. Las relaciones existentes entre Estados que pretenden integrar sus economías en organiza-
ciones supranacionales
B. La relación existente entre una metrópoli y su imperio colonial
C. La relación de dependencia económica y política que se genera entre nuevos países y sus anti-
guas metrópolis u otras potencias
D. El impulso conquistador y de incorporación de territorios que denotan algunos países

13. La manifestación más antigua del liberalismo fue:


A. La Revolución Francesa B. La Declaración de Independencia de los Esta-
B. dos Unidos
C. El Despotismo Ilustrado D. La Revolución Rusa

50 TAMadrid

-Pág.122-
U. D. 3 . - L A A C T I V I D A D H U M A N A Y E L E S PA C I O G E O G R Á F I C O

14. Uno de los siguentes tratados no tiene que ver con el proceso de integración europea que ha lle-
vado a la actual Unión Europea. Señálelo.
A. Tratado de Versalles B. Tratado de Roma
C. Tratado de Amsterdam D. Acta Única

15. Una de las siguientes parejas de países entraron en la Unión Europea al mismo tiempo. Señálelo.
A. España y Grecia B. Portugal y Dinamarca
C. Suecia y Austria D. Alemania y Reino Unido

16. Una de las siguientes instituciones no es una de las de la Unión Europea. Indíquela.
A. Tribunal de La Haya B. Consejo de Ministros
C. Parlamento D. Comisión

17. La Unión Económica y Monetaria en la Unión Europea, con el establecimiento del euro como
moneda única, tuvo lugar en:
A. 1991 B. 1993 C. 1995 D. 1999

18. Indique cuál de las siguientes afirmaciones es errónea:


A. En Iberoamérica existe una importante conflictividad sociopolítica como consecuencia de
los fuertes desequilibrios económicos
B. En países como India y Pakistán se producen paradojas como la existencia de enormes bol-
sas de pobreza junto con políticas de inversión estatal en armamento nuclear o satélites arti-
ficiales
C. Tras la reforma constitucional de 1994 se estableció el apartheid en Sudáfrica, momento a
partir del cuál se produjo el ascenso internacional del país
D. En los países árabes existen fuertes desequilibrios económicos a pesar de los enormes ingre-
sos producidos por los productos energéticos

19. Una de las siguientes Comunidades Autónomas españolas es uniprovincial:


A. Extremadura B. Canarias C. Galicia D. Illes Balears

20. ¿Cuál es la Comunidad Autónoma española que tiene más provincias?


A. Castilla-La Mancha B. Andalucía C. Galicia D. Murcia

21. ¿Cuántas Comunidades Autónomas y Ciudades Autónomas hay en España?


A. 17 B. 19 C. 2 D. 10

22. ¿Cuál es la capital de la Comunidad Autónoma de Galicia?


A. La Coruña B. Ferrol C. Vigo D. Santiago de Compostela

TAMadrid
51
-Pág.123-
C I E N C I A S S O C I A L E S . G E O G R A F Í A E H I S TO R I A

RESPUESTAS A LOS EJERCICIOS

11. A
12. B
13. C
14. B
15. D
16. D
17. C
18. C
19. B
10. B
11. D
12. C
13. B
14. A
15. C
16. A
17. D
18. C
19. D
20. B
21. B
22. D

52 TAMadrid

-Pág.124-
portada TROPA 19/3/07 19:51 Página 1

FUERZAS ARMADAS
PROFESIONALES
CURSO DE APOYO
A LA PREPARACIÓN
DE LAS PRUEBAS DE ACCESO
A UNA RELACIÓN DE SERVICIOS
DE CARÁCTER PERMANENTE

CIENCIAS SOCIALES
2ª parte
Unidades didácticas 4, 5 y 6

DIGEREM

MINISTERIO
DE DEFENSA
FUERZAS ARMADAS SUBDIRECCIîN GENERAL
DE TROPA Y MARINERIA
PROFESIONAL
PROFESIONALES
CURSO DE APOYO
A LA PREPARACIÓN
DE LAS PRUEBAS DE ACCESO
A UNA RELACIÓN DE SERVICIOS
DE CARÁCTER PERMANENTE

CIENCIAS SOCIALES
2ª parte
Unidades didácticas 4, 5 y 6
La Ley 8/2006 de Tropa y Marinería, en su artículo 16,1, establece que “la formación
en las Fuerzas Armadas garantizará que los militares profesionales de tropa y
marinería puedan adquirir, actualizar o ampliar sus conocimientos para un mayor
desarrollo personal y profesional”. En cumplimiento de este mandato, el Ministerio
de Defensa edita el presente material didáctico para facilitar a los militares
profesionales de tropa y marinería, alumnos de los cursos de formación
presencial que se imparten a través de la Dirección General de Reclutamiento y
Enseñanza Militar, los apoyos necesarios para preparación de dichos cursos, que
permitirán, siempre que superen las pruebas correspondientes, la obtención de la
titulación de graduado en Educación Secundaria, acreditación para el acceso a
los ciclos formativos de la Formación Profesional de grado medio o de grado
superior, acceso a las Escalas de Suboficiales, Tropa Permanente, Guardia Civil
y Policía Nacional.

CATÁLOGO GENERAL DE PUBLICACIONES


http://www.060.es

Edita:

© Autor y editor
NIPO: 076-10-204-9 NIPO: 076-10-205-4 (edición en línea)
Depósito Legal: M-32363-2009
Diseño y programación: cimapress
Tirada: 1300 ejemplares
Fecha de edición: septiembre, 2010

Prohibida la reproducción total o parcial de esta obra, por cualquier medio sin autorización escrita del editor
CIENCIAS SOCIALES
2ª parte

SUMARIO

Unidad didáctica Pág.

4. SOCIEDADES HISTÓRICAS 5

5. SOCIEDADES Y CAMBIO EN EL TIEMPO 73

6. DIVERSIDAD CULTURAL 121


U . D . 4 . - S OCIEDADES HISTÓRICAS

ÍNDICE
OBJETIVOS . . . . . . . . . . . . . . . . . . . . . . . . . . . . . . . . . . . . . . . . . . . . . . . . . . . . . . . . . . . . . .2

INTRODUCCIÓN . . . . . . . . . . . . . . . . . . . . . . . . . . . . . . . . . . . . . . . . . . . . . . . . . . . . . . . . .3

MAPA CONCEPTUAL . . . . . . . . . . . . . . . . . . . . . . . . . . . . . . . . . . . . . . . . . . . . . . . . . . . . .4

DESARROLLO DE CONTENIDOS

1. INICIACIÓN A LOS MÉTODOS HISTÓRICOS . . . . . . . . . . . . . . . . . . . . . . . . . . . . . . . . . .5

2. SOCIEDADES PREHISTÓRICAS, PRIMERAS CIVILIZACIONES


Y ANTIGÜEDAD CLÁSICA . . . . . . . . . . . . . . . . . . . . . . . . . . . . . . . . . . . . . . . . . . . . . . . . . . .9

3. LAS SOCIEDADES MEDIEVALES . . . . . . . . . . . . . . . . . . . . . . . . . . . . . . . . . . . . . . . . . . . .40

4. LAS SOCIEDADES DE LA ÉPOCA MODERNA . . . . . . . . . . . . . . . . . . . . . . . . . . . . . . . .49

5. SOCIEDADES DE ÁMBITO NO EUROPEO DURANTE LAS EDADES


MEDIA Y MODERNA . . . . . . . . . . . . . . . . . . . . . . . . . . . . . . . . . . . . . . . . . . . . . . . . . . . . . . .61

RESUMEN . . . . . . . . . . . . . . . . . . . . . . . . . . . . . . . . . . . . . . . . . . . . . . . . . . . . . . . . . . . . . .63

EJERCICIOS DE AUTOCOMPROBACIÓN . . . . . . . . . . . . . . . . . . . . . . . . . . . . . . . . . .65

RESPUESTAS A LOS EJERCICIOS . . . . . . . . . . . . . . . . . . . . . . . . . . . . . . . . . . . . . . . . .68

-Pág.5-
CIENCIAS SOCIALES (GEOGRAFÍA E HISTORIA)

OBJETIVOS
Al finalizar el estudio de esta Unidad Didáctica, el alumno será capaz de:

• Comprender los primeros pasos del desarrollo cultural de la Humanidad hasta la


eclosión de las primeras civilizaciones.

• Diferenciar el paso a la etapa que llamamos Neolítico como una auténtica revo-
lución que cambió, en un proceso lento pero de un modo radical, la forma de
vida del ser humano.

• Asimilar las características esenciales de las primeras civilizaciones postpaleo-


líticas, desde el punto de vista político, social, artístico, religioso, etc. y com-
prender la trascendencia que la civilización clásica griega y romana ha tenido en
la formación embrionaria de la cultura occidental, con sus aportaciones filosó-
ficas, artísticas y políticas.

• Distinguir las específicas formulaciones sociales, políticas, económicas y cultu-


rales de la Edad Media y valorar los cambios producidos a partir del Renaci-
miento con las nuevas tesis que preparan la evolución cultural que conduce a la
modernidad por la vía del humanismo y la razón.

• Valorar las culturas extraeuropeas y aproximarse a su conocimiento con algunas


nociones básicas respecto a las mismas

• Conocer la evolución cultural de España de forma paralela y específica a cada


una de las etapas generales contempladas, valorar y discriminar los rasgos
diferenciadores y los aspectos comunes con el desarrollo general de la historia
europea.

-Pág.6-
U . D . 4 . - S OCIEDADES HISTÓRICAS

INTRODUCCIÓN
R esulta especialmente complicado resumir más de 100.000 años de Historia en un
tema que mantenga unos mínimos requisitos didácticos. Mas el programa de la
presente asignatura nos exige concentrar en éste el desarrollo de la Historia desde las
primeras manifestaciones culturales del Hombre hasta el final de la Edad Moderna, es
decir, desde hace más de 100.000 años hasta los prolegómenos de la Revolución fran-
cesa.

Obviamente, algún aspecto de este conjunto de culturas o etapas, debe ser tratado de
modo pasajero y muchos detalles deben ser dejados de lado en beneficio de los más
trascendentes para el logro de los objetivos propuestos.

En todo caso, apostamos aquí por unos planteamientos que pretenden aproximar al
alumno al conocimiento de los rasgos más caracterizadores de cada una de los periodos
que estudiamos, procurando facilitarle la información necesaria para que comprenda lo
mejor posible los aspectos más importantes de la cultura.

-Pág.7-
CIENCIAS SOCIALES (GEOGRAFÍA E HISTORIA)

M A PA C O N C E P T UA L
SOCIEDADES
HISTÓRICAS

INICIACIÓN A LOS
MÉTODOS HISTÓRICOS

SOCIEDADES SOCIEDADES PREHISTÓRICAS


PREHISTÓRICAS, PRIMERAS
CIVILIZACIONES Y
ANTIGÜEDAD CLÁSICA
LA REVOLUCIÓN NEOLÍTICA

PRIMERAS CIVILIZACIONES

ORIGEN Y DESARROLLO
DEL CAPITALISMO

PREHISTORIA Y ANTIGÜEDAD
EN ESPAÑA
SOCIEDADES NO
EUROPEAS DURANTE LAS SOCIEDADES
EDADES MEDIA MEDIEVALES
Y MODERNA INVASIONES BÁRBARAS FIN
DEL IMPRERIO DE OCCIDENTE

SOCIEDAD Y CULTURA SOCIEDADES FEUDALES


ISLÁMICA DURANTE LA EUROPEAS
EDAD MEDIA

AL ANDALUS Y LOS REINOS


CULTURAS PRECOLOMBINAS CRISTIANOS DE LA PENÍNSULA

LAS SOCIEDADES
DE LA ÉPOCA MODERNA
SOCIEDADES DEL ANTIGUO
RÉGIMEN EN EUROPA

HEGEMONÍA Y DECADENCIA
DE LA MONARQUÍA HISPÁNICA

-Pág.8-
U . D . 4 . - S OCIEDADES HISTÓRICAS

1. INICIACIÓN A LOS MÉTODOS HISTÓRICOS


La Historia es una combinación de hechos, protagonizados por individuos y colectividades
anónimos o personificados que se desarrollan a lo largo del tiempo... de todo el tiempo, por
todos los individuos y colectividades y todos los hechos. Efectivamente, es Historia el desem-
barco de Normandía, el lanzamiento de las bombas atómicas sobre Hiroshima y Nagasaki, el
descubrimiento de América o la conquista de la Luna; lo es la figura y los hechos protagoniza-
dos por personajes como Napoleón, Carlos V, Julio César o Alejandro Magno; pero también es
Historia el trabajo cotidiano de un campesino en plena Edad Media o la búsqueda de frutos sil-
vestres para la alimentación del grupo de una mujer cromañón en el Paleolítico Superior. Los gran-
des acontecimientos tejen los tratados de Historia que, en su peor manifestación, se convierten en
retahílas de nombres y fechas; los hechos cotidianos son la auténtica trama y urdimbre del tejido
de la Historia, su lanzadera y su hilo, y entre los cotidianos están también los grandes hechos. La
Historia, entendida no en su sentido absoluto, sino en el sentido de relato de los hechos..., en his-
toria de la Historia, es sólo un pálido reflejo de esa otra Historia de todo y de todos.

Entendida como ciencia, la Historia ha tenido su propia historia, Los textos más antiguos
que podemos calificar como Historia fueron los anales plasmados en piedra o arcilla puestos al
servicio de los intereses de los monarcas mesopotámicos o egipcios. Como para tantos otros
aspectos de la civilización, Grecia aportó una concepción de la Historia como ciencia y los pri-
meros tratados sobre su carácter y su naturaleza y, también como en otros aspectos, surgieron
los primeros historiadores de nombre conocido: Heródoto, Tucídides, Jenofonte. La Historia
adquirió carta de naturaleza propia en Roma; nombres como Suetonio, Polibio, Tito Livio o
Tácito se unen al del gran Julio César. Durante los primeros pasos de la Edad Media, el relato
histórico se tiñe de poesía épica (Chanson de Roland, Cantar del Mío Cid) para dar paso después
a anales en prosa, impregnados de rasgos fantásticos y religiosos muchas veces, que contaron
también con nombres propios entre los que podríamos citar al inglés Godofredo de Monmouth
(s. XIV), a los franceses Joinville (s. XIII) y Philippe de Commynes (s. XIV) o al español
Alfonso X el Sabio. El Renacimiento y el humanismo que caracteriza dicho periodo dieron
empuje a los relatos históricos (Maquiavelo, Guicciardini, Hurtado de Mendoza, Fernández
de Oviedo, Bartolomé de las Casas), que siguieron cobrando importancia en las etapas poste-
riores y, sobre todo, a raíz del triunfo de las ideas ilustradas y del romanticismo decimonónico.
Precisamente, las nuevas ideas que estas corrientes aportarán y las que se desarrollan con pos-
terioridad, pondrán la base de concepciones diversas sobre la ciencia histórica y el análisis y la
interpretación de los hechos históricos. Antes de hacer una breve semblanza de éstas, citemos,
con el riesgo de olvidar alguno, un pequeño ramillete de historiadores modernos y contempo-
ráneos: Gordon Childe, Galbraith, Tocqueville, Raymond Carr, Edward H. Carr, Pierre
Chaunu, Crouzet, Medvedev, Washington Irving, Indro Montanelli, etc., y, entre los espa-
ñoles, Menéndez Pelayo, Caro Baroja, Claudio Sánchez Albornoz, Gregorio Marañón,
Vicens Vives, Miguel Artola, Javier Tusell, Tamames, Tuñón de Lara, etc.

5
-Pág.9-
CIENCIAS SOCIALES (GEOGRAFÍA E HISTORIA)

El historiador realiza su labor siguiendo esencialmente los siguientes pasos:

1. Documentación.- Búsqueda de datos o antecedentes cuyas fuentes pueden ser:

– Testimonios escritos.- Toda clase de documentos cuyos contenidos pueden ser consi-
derados en el análisis de los hechos que se estudian, desde cartas personales, pasando
por registros contables o documentos oficiales.

– Tradición.- Relatos populares, testimonios orales.

– Historiografía.- Textos precedentes que ya han analizado los hechos en etapas ante-
riores.

– Arqueología.- Restos materiales que informan sobre diferentes aspectos de la cultura


y pueden aportar datos esenciales para la interpretación histórica.

2. Análisis.- Proceso de sistematización y estudio de los datos recopilados, realizando una


labor de crítica, tanto de las fuentes en sí mismas (su autenticidad y trascendencia) como
de los hechos que revelan.

3. Planteamiento de las tesis sobre las causas y efectos de los hechos estudiados, tenien-
do en cuenta los distintos factores que pueden influir.

4. Síntesis o exposición de los resultados del estudio.

Los testimonios escritos han sido la base de los relatos históricos. Y es tanto así que, en un
sentido estricto, sólo se llama Historia a la parte del relato global que se basa en tales docu-
mentos escritos, estableciendo dos partes en el Gran Relato: La Prehistoria y la Historia. Sólo
la segunda se nutre de documentos escritos; ambas de los datos aportados por las investigacio-
nes arqueológicas que, obviamente, son las únicas fuentes que nos informan sobre lo aconteci-
do antes de la aparición de los textos. La propia naturaleza de los restos arqueológicos hace que
ilustren el conocimiento de la cultura material, precisamente y sobre todo la que han dejado las
gentes anónimas, no los grandes protagonistas. De esta manera, los primeros capítulos del Gran
Relato, elaborados sobre las fuentes arqueológicas, se refieren más a los hechos cotidianos y,
necesariamente, son menos densos, es decir, incluyen un número de hechos menor por año rela-
tado o, lo que es lo mismo, un ramillete de hechos requiere un relato alusivo a un número con-
siderable de años. A medida que esos capítulos se aproximan más a nuestros días, el conoci-
miento, por restos más numerosos y por los textos, de un volumen de hechos que crece en
progresión geométrica, hacen que aumente el número de ellos por cada periodo de tiempo y que
éstos se hagan cada vez más cortos: Así, podemos dedicar un tema a una etapa prehistórica de
2,5 millones de años, con la misma extensión que la otorgada a los 25 años transcurridos entre
el inicio de la Transición y el final del siglo XX.

6
-Pág.10-
U . D . 4 . - S OCIEDADES HISTÓRICAS

En este mismo sentido, la sistematización del estudio histórico, ha llevado a una comparti-
mentación temporal que determina la organización tanto de los tratados como de la especiali-
zación de los historiadores. Dicha compartimentación que, tradicionalmente, se apoyó en
hechos históricos con cronología precisa, nos resulta válida siempre que comprendamos que los
límites de los periodos no pueden ser considerados por líneas de cambio radical sino como sim-
ples referencias ya que los procesos históricos, por mucho que en ellos descollen aconteci-
mientos puntuales, se producen a lo largo de años. Además, debemos considerar que los cam-
bios de unas etapas a otras no se produjeron del mismo modo en unas zonas de la Tierra que en
otras por lo cual, salvo para las etapas más recientes de la Historia, dicha periodización sólo nos
vale para el llamado Viejo Mundo, la zona de la Tierra situada entre Europa, Próximo Oriente
Asiático y África del Norte; e incluso dentro de esta zona determinadas etapas muestran tam-
bién su correspondiente desfase en función del mayor o menor desarrollo de cada espacio (así,
mientras en Egipto se desarrolla una civilización histórica que construye las pirámides, en Euro-
pa Central se dan los primeros pasos de la Edad de los Metales sin textos escritos). Dicha perio-
dización, en resumen, es la siguiente:

Paleolítico Inferior Hasta 100.000 a.C.

Paleolítico Paleolítico Medio 130.000 a 30.000 a.C.

Paleolítico Superior 30.000 a 10.000 a.C.


PREHISTORIA
Mesolítico 10.000 a 5.000 a.C.

Neolítico 5.000 a 3.000 a.C.

Edad Edad del Cobre (*)

De los Edad del Bronce (*)


Historia
Metales Edad del Hierro (*)

Antigua (*) hasta 476 d.C.

Historia Medieval 476 a 1453


HISTORIA
Historia Moderna 1453 a 1792

Historia Contemporánea 1792 a nuestros días

(*) Etapas que, dependiendo de las zonas geográficas, se solapan entre la Prehistoria
(Edad de los Metales) y la Historia (Historia Antigua)

Por otra parte, en función de los ámbitos personales, conceptuales, cronológicos o geográfi-
cos, los tratados históricos se clasificarán bajo diferentes criterios y así hablaremos de:
— Según el concepto estudiado:
– Historia Económica
– Historia Social
– Historia Política
– Historia Cultural

7
-Pág.11-
CIENCIAS SOCIALES (GEOGRAFÍA E HISTORIA)

– Historia Sagrada
– Historia de la Ciencia
– Historia del Arte, etc…
— Según la cronología:
– Historia General
– Historia Antigua, Medieval, Moderna….
– Anales
– Crónicas
— Según el ámbito geográfico:
– Historia Local
– Historia Regional
– Historia Nacional
– Historia Universal
Finalmente, tenemos que considerar de un modo expreso un aspecto trascendental que tiene
relación con la fase de la elaboración del estudio histórico en la que el autor considera las cau-
sas y consecuencias de los hechos. Si los hechos en sí no tienen, o no deberían tener interpre-
tación parcialista, las causas sí pueden prestarse a diferentes criterios de interpretación. De
hecho, a lo largo de la historia de la Historia se han planteado teorías diversas sobre las causas
o motores que mueven los grandes procesos históricos y que podríamos esquematizar simplifi-
cadamente del siguiente modo:

MOTORES DE LA HISTORIA

LA VOLUNDAD DE DIOS FACTORES GEOGRÁFICOS


Recursos naturales, clima
FACTORES BIOLÓGICOS
Raza RELIGIÓN, IDEOLOGÍA

ECONOMÍA

ECONOMICISMO MATERIALISMO
HISTÓRICO

8
-Pág.12-
U . D . 4 . - S OCIEDADES HISTÓRICAS

2. LAS SOCIEDADES PREHISTÓRICAS, PRIMERAS


CIVILIZACIONES Y ANTIGÜEDAD CLÁSICA

2.1. LAS SOCIEDADES PREHISTÓRICAS CAZADORAS


Y RECOLECTORAS

2.1.1. EL PALEOLÍTICO

2.1.1.1. Introducción
El término Paleolítico (Paleos=antiguo; lithos=piedra), se aplica a la etapa más extensa de
la historia de la Humanidad y de la que, relativamente, disponemos de menos datos. El término
alude a la tipología de las herramientas conservadas y su significado adquiere sentido en opo-
sición al término Neolítico (Neo=nuevo), entendiendo, desde el punto de vista del utillaje al que
nos referimos, esta dualidad antiguo - nuevo, como la dualidad piedra tallada - piedra pulimen-
tada que, desde un punto de vista tecnológico, diferencia, parcialmente, la etapa que nos ocupa
de la siguiente en el devenir de la Humanidad. Sin embargo, el análisis de las sociedades pre-
históricas nos lleva a observar que no es sólo esta diferenciación técnica la que nos permite con-
templar la evolución de dichas sociedades, sino todo un conjunto de rasgos culturales que dife-
rencian el grado de desarrollo de unas y otras. Así, el Paleolítico aparece como un periodo en el
que el ser humano se define como un depredador, no regenerador del medio, que va perfeccio-
nando sus capacidades creativas y sus relaciones sociales, comenzando a ser consciente de su
propia identidad, proyectada en las primeras creencias de carácter trascendente.

2.1.1.2. Evolución cronológica


Los estudios paleogeológicos, paleontológicos, paleoantropológicos y arqueológicos han
ido definiendo una serie de relaciones entre las diferentes etapas geológicas y la evolución cul-
tural. Todos estos aspectos se hallan interrelacionados cronológicamente pudiendo resumir, de
modo muy simplificado, el periodo que nos ocupa en el siguiente cuadro sinóptico:

9
-Pág.13-
CIENCIAS SOCIALES (GEOGRAFÍA E HISTORIA)

(1) (2) (3) CULTURAS ESPECIES GLACIA- GEOLOGÍA


HUMANAS CIONES

1,8 Australopithecus Donau


1,7
1,6 Pleistoceno
1,5 Pebble Cultur
1,4
1,3 o Cultura de Inferior
1,2
1,1 los Cantos Homo Erectus
1
0,8 Trabajados
0,7 Gunz
0,6
0,5
0,4
0,4 400 Paleolítico Mindel Pleistoceno
0,3 300
0,2 200 Inferior Medio
0,1 100 (400-130) Riss
90 Homo
80 Paleolítico Würm
70 Medio Sapiens
60 Pleistoceno
50 Nenderthalensis
40 (130-31) Superior
30
29 Chatelpe- A
28 rronense R
27 Paleo- T
26 lítico Auriñaciense E
25
24 P Homo
23 Supe- A Würm
22 rior L Sapiens
21 Gravetiense E
20 O Sapiens
19 Solutrense L
18 Í
17 T
16 I
15 C
14 O
13
12 Magdaleniense
11
10

( 1 ): Millones de años, de 100.000 en 100.000 años, ( 2 ): Miles de años de 10.000 en 10.000 años.
( 3 ): Miles de años de 1.000 en 1.000 años.

10
-Pág.14-
U . D . 4 . - S OCIEDADES HISTÓRICAS

2.1.1.3. Evolución cronológica


La polémica sobre el origen del Hombre y los pasos evolutivos que han conducido hasta el
hombre actual, sigue plenamente viva. Desde que aparecieran los primeros fósiles del Hombre
de Neanderthal, en momentos en los que comenzaba a gestarse la teoría de la selección natural
y la evolución de las especies de Charles Darwin, son muchas las teorías que se han argüido,
teorías que posteriores descubrimientos se encargaban de desmontar o, en el mejor de los casos,
completar o confirmar.

El principal impedimento para que cualquiera de las teorías se convierta en certeza radica
en lo exiguo de los datos en los que se fundamentan. Unos fragmentos craneales, un trozo de
pelvis... de forma aislada y sin que podamos asegurar que las características morfológicas del
sujeto al que pertenecían repondieran a la media de sus congéneres. En todo caso, los paleoan-
tropólogos han ido definiendo los caracteres morfológicos y antropométricos y deduciendo las
circunstancias asociadas a diferentes especies humanas o de antropomorfos próximos, evoluti-
vamente, al género Homo; pero no han llegado a establecer con seguridad un esquema preciso
sobre los mecanismos que han posibilitado los saltos en los sucesivos escalones evolutivos ni,
por el momento, han podido aseverar con rotundidad cuál fue el salto cualitativo que transfor-
mó a un simio - animal irracional en un simio - animal racional, eso que hemos venido a deno-
minar el eslabón perdido.

Existen diversas teorías explicativas de este proceso. Aquí solamente vamos a recoger una
breve semblanza del proceso, haciéndonos eco de las últimas aportaciones emanadas de los des-
cubrimientos africanos y europeos, especialmente de los realizados en el yacimiento de Ata-
puerca (Burgos) que han permitido la elaboración de nuevas tesis que arrojan luz sobre uno de
los saltos a los que nos referimos, definiendo una nueva especie, el Homo antecessor que ven-
dría a ser el eslabón del que se separarían las especies que darían lugar a la aparición del Hom-
bre de Neanderthal, por una parte, y de nuestra especie, por otra.

Juan Luis Arsuaga e Ignacio Martínez, dos de los investigadores de Atapuerca, a la luz
de los hallazgos en el yacimiento burgalés y reflexionando sobre las diversas teorías existentes
respecto a la cuestión de la evolución del hombre, han rediseñado el esquema evolutivo de la
especie humana que podríamos resumir en el siguiente cuadro:

11
-Pág.15-
-Pág.16-
U . D . 4 . - S OCIEDADES HISTÓRICAS

AUSTRALOPITHECUS Primer descubrimiento: 1925 (Raymond Dart).


Sudáfrica. Edad: Entre 5 y 1 millón de años. Principales investigaciones: Louis
y Mary Leackey.

Características: Escasa estatura (1,30 a 1,50 m.), 700 cc de capacidad cra-


neana, mandíbula prominente, nariz ancha, frente pequeña y huidiza, arcos super-
ciliares, dentición más próxima al hombre que a los monos, posición bípeda. Gru-
pos de 15 individuos aprox., alimentos vegetales y pequeños animales. Utilización
de piedras y otros objetos mínimamente elaborados (Modo 1).

HOMO ERECTUS Primer descubrimiento: 1891. (Eugène Dubois). Tri-


nil, Java. Edad: Entre 1 millón y 100.000 años Principales investigaciones:
Dubois y von Köenigswald (Java), Schoetensack (Heidelberg), Anderson, Black
y WC Pei (China).

Características: Estatura media 1,65 m., capacidad craneana entre 750 y


1.400 cc, huesos craneales robustos, arcos superciliares prominentes, abertura
nasal y dentición de tipo humano. Grupos abiertos, exogamia, colaboración entre
grupos para la caza, utiliza el fuego, perfecciona las técnicas de trabajo de la pie-
dra (Modo 2).

HOMO SAPIENS NEANDERTHALENSIS Primer descubrimiento:


1856. Alemania. Edad: Entre 140.000 y 50/40.000 años. Principales investiga-
ciones: muy extendidas por el mundo, destacan La Chapèlle aux Saints, Le
Moustier (Francia), Gibraltar, Bañolas, Atapuerca (España), Mugharet es-Skhul
(Israel), Solo (Java, Indonesia), Omo (Etiopía), Broken Hill (Sudáfrica).

Características: Esqueleto robusto, gran musculatura, arcos supraciliares,


carencia de mentón, gran capacidad craneana con media superior al Homo
Sapiens Sapiens. Habitat en cuevas, perfeccionamiento del trabajo de la piedra
(Modo 3 o técnica Levallois), indicios de culto a los muertos.

HOMO SAPIENS SAPIENS (Cro-Magnon, Grimaldi, Chancellade...).


Edad: Entre 100.000 años y hoy.

Características: las del hombre actual, con escalones iniciales de rasgos


primitivos. Perfeccionamiento de las técnicas de fabricación de útiles. Manifes-
taciones artísticas.

13
-Pág.17-
CIENCIAS SOCIALES (GEOGRAFÍA E HISTORIA)

2.1.1.4. Cultura Material


La periodización de las diferentes etapas culturales del Paleolítico viene determinada, fun-
damentalmente, por la evolución de los útiles conservados, casi siempre fabricados en piedra,
pero también en hueso y madera. A tenor de dicha evolución se establece una sucesión de eta-
pas que aparecen resumidas en el cuadro sinóptico de más arriba. En líneas generales, se apre-
cia la paulatina especialización y perfeccionamiento de los utensilios. El proceso se inicia con
la sencilla elaboración de útiles mediante leves retoques en cantos rodados (Modo 1, que da
lugar a la llamada Pebble Cultur o Cultura de los cantos trabajados), continúa con el trabajo
sobre núcleos para crear poliútiles, los llamados bifaces o hachas de piedra (Modo 2), sigue con
el trabajo de obtención de lascas mediante percusión sobre núcleos previamente preparados
(Modo 3 o técnica Levallois), para crear útiles específicos para cada una de las actividades coti-
dianas, y finaliza con la técnica de trabajo de las lascas mediante presión y retoque delicado
(Modo 4), a la que se une la fabricación de objetos en otros materiales como asta, hueso, made-
ra, concha o marfil.

2.1.1.5. Formas de vida


Como característica general, el Hombre del Paleolítico es un depredador. El sustento coti-
diano se lo proporcionan sus actividades recolectora de frutos y cazadora, a las que se unen la
pesca y el marisqueo; de modo que no realiza ninguna actividad regeneradora del medio.
Socialmente se reúne en pequeños grupos vinculados por el parentesco, que viven en
poblados con cabañas hechas de productos vegetales o animales (pieles), abrigos rocosos o
cuevas.
Desde el Paleolítico Medio se atestigua una clara manifestación de creencias de tipo tras-
cendente-religioso, relacionada con el mundo funerario y con la caza.

2.1.1.6. El Arte Paleolítico


El Arte Paleolítico es una manifestación específica del Paleolítico Superior. La más que
probable práctica anterior de algún tipo de creación artística sobre materiales perecederos, como
pieles, madera, etc... no ha llegado a nuestros días.

La temática gira de un modo abrumadoramente mayoritario sobre el mundo animal, dada


su trascendencia en estas sociedades básicamente cazadoras.

14
-Pág.18-
U . D . 4 . - S OCIEDADES HISTÓRICAS

Se puede dividir en dos grandes esferas:

A) Arte mueble o mobiliar

Sobre materiales u objetos transportables.

Muy difundido por Europa, Norte de África y Próximo Oriente.

Soportes: tales como armas, utensilios, objetos de adorno, objetos votivos y otros de utili-
dad desconocida.
Materiales: madera, hueso, piedra, asta, arcilla...
Técnicas: pintura, escultura, grabado.

B) Rupestre o Parietal
Sobre las paredes rocosas de abrigos y cuevas.
Gran concentración de yacimientos en Suroeste de Francia y Norte de España. (Altamira, en
Santander, fue el primer descubrimiento importante).
Plantea una gran dificultad la precisión de su cronología por no darse asociado a estratos
datables.

Técnicas: Pintura con colorantes minerales, vegetales y animales, aplicados con los dedos,
con pinceles, con la técnica del tamponado o mediante soplado, dando lugar a manifestaciones
variopintas que van desde simples trazos esquemáticos hasta diseños con policromía y sensa-
ción de volumen.

También se da el grabado, directamente sobre la pared o tras la previa aplicación de una capa
de arcilla.

Temas: Los animales, predominantes, están acompañados ocasionalmente de signos geo-


métricos o esquemáticos y. más escasamente, de representaciones humanas.

C) Interpretación.

Podemos resumir las teorías sobre la significación y motivación de estas manifestaciones en


el siguiente esquema:
— “Arte por el arte”. “Un simple entretenimiento”. “Decoración, gusto por los diseños”.
– Significado Mágico-religioso:
– Mágico para propiciar la caza (Reinach)
– Mágico para propiciar la fertilidad
– Religioso: santuarios de adoración (Breuil)

15
-Pág.19-
CIENCIAS SOCIALES (GEOGRAFÍA E HISTORIA)

2.1.2. LA REVOLUCIÓN NEOLÍTICA

2.1.2.1. Introducción
El final de la última glaciación supone un cambio climático que va a determinar un paulati-
no e importantísimo cambio en la forma de vida del Hombre: la forma de vida depredadora
va a dar paso a una economía productora.

La transición del Paleolítico a estas nuevas formas de vida se produce con un creciente des-
fase cronológico entre unas zonas del mundo y otras. Dicha transición recibe diferentes nom-
bres:

Mesolítico: para aquellas zonas donde surge el Neolítico posteriormente de modo autóno-
mo.

Epipaleolítico: para aquellas que “importan” la economía productora por influencia de las
zonas de invención.

En algunas zonas, como el Próximo Oriente, Egipto, Valle del Indo, China, la aparición del
Neolítico va a suponer el inicio del desarrollo de las primeras civilizaciones.

2.1.2.2. Significado y características


El Neolítico supone una auténtica revolución (Según la definición de Gordon Childe) que
va a alterar la forma de vida del Hombre.

Tras una etapa de transición, una vez finalizadas las etapas glaciares, con poblaciones reco-
lectoras post-paleolíticas, el hombre dará el gran salto en su evolución cultural al pasar, de un
modo paulatino, a producir sus propios alimentos mediante el cuidado especial del ganado y
el de determinadas plantas, dando lugar a sociedades pastoriles y agrícolas. Todo un conjunto
de causas y consecuencias se irán interrelacionando para transformar estas sociedades en uni-
dades sedentarias rumbo a la civilización (como sucederá en Mesopotamia o en Egipto). Estas
transformaciones irán acompañadas de nuevas formas de trabajo de la piedra (pulimenta-
ción), nuevos útiles (azadas, arados, pesas de telar) y nuevos materiales (vegetales y, sobre
todo, cerámica para recipientes).

En las áreas mediterránea y atlántica europeas, el desarrollo de esta nueva etapa viene acom-
pañado de la difusión de formas constructivas que denominamos megalistismo. Incluye cons-
trucciones relacionadas con el mundo funerario (dólmenes, cistas) y otras de significado a
veces dudoso (crómlech, alineamientos), en ocasiones vinculadas con observaciones astronó-
micas, que hay que relacionar con la trascendencia que las mismas tienen en una nueva coyun-
tura económica dependiente de los cambios estacionales y su influencia en las cosechas.

16
-Pág.20-
U . D . 4 . - S OCIEDADES HISTÓRICAS

Las características que definen el Neolítico son:


– Agricultura
– Ganadería
– Sedentarización
– Innovaciones Técnicas:
- Cerámica
- Telar
- Piedra pulimentada
- Microlitos
- Rueda-Carro
- Vela-Barca...
– Manifestaciones culturales-religiosas:
- Megalitismo (dólmenes, menhires, crómlech...)
- Culto a la fertilidad

2.1.3. LAS PRIMERAS CIVILIZACIONES


Tras el despegue de la civilización, de la mano de la Revolución Neolítica, las diferentes
áreas geográficas de la Tierra comenzaron un acelerado proceso de distanciamiento formándo-
se espacios culturalmente aislados. De esta manera, hasta el inicio del proceso de aproximación
cultural iniciado con los descubrimientos geográficos del s. XV, la Tierra se divide en una serie
de zonas con evoluciones culturales diferentes.

2.1.3.1. Asia
Surgen aquí culturas con un importante desarrollo social, político, económico, cultural y
artístico. Destacan las culturas japonesa, china, india, etc...

2.1.3.2. África
El nivel de desarrollo es menor cuanto más al sur, destacando el hecho de que algunos gru-
pos humanos se mantuvieran hasta época reciente en estadios de evolución paralelos al Paleo-
lítico.

2.1.3.3. América
Desde su poblamiento, quedó aislada y, a su vez, se establecieron dentro de su territorio dife-
rentes áreas culturales. Hasta el inicio de la colonización, los puntos culminantes de su des-
arrollo fueron:

• Norte:
Los Mayas, entre los siglos VII y VIII d.C.,
Los Aztecas, que alcanzaron un importante desarrollo en todos los órdenes: urbanístico,
artístico, científico.

17
-Pág.21-
CIENCIAS SOCIALES (GEOGRAFÍA E HISTORIA)

• Sur:
Los Incas, creadores de un importante imperio, gobernado por una oligarquía, en cuya
cúspide está el Inca, alcanzaron un importante desarrollo en la organización de su gran
imperio.

2.1.4. MESOPOTAMIA
En el arco formado por el valle del Éufrates y el Tigris, se va a ir gestando, desde el final
del Paleolítico, la creación de una civilización,basada inicialmente en el proceso neolítico, que
va a dar lugar a la sucesión de una serie de culturas con unos rasgos genéricos más o menos
comunes. La evolución de las culturas mesopotámicas podemos resumirlas en los siguientes
cuadros:

X-IX milenio: INICIO NEOLÍTICO ( Jarmo) (acerámico).


V milenio: INICIO METALURGIA (cobre) ( El Obeid).
+- 3500 a.C. llegada de los sumerios

SUMERIOS

AKKAD

GUTI

SUMERIOS (UR)

AMORREOS (2006)

(Alta Mesop.) (Baja Mesop.)


ASIRIA BABILONIA

SUCESIÓN CRONOLÓGICA DE LAS ETAPAS


POLÍTICO - CULTURALES
1. Sumerios (3500-2347)
2. Imperio Acadio (2347-2230)
3. Neosumerio (2230-2006)
4. Assur (2006-1792)
5. Imperio Babilónico (1792-1450) (reino de Hammurabi)
6. Imperio Asirio (1450- 625) (con fases intermedias)
7. Imperio Neobabilónico (625- 539)
8. Imperio Persa (539- 332)

18
-Pág.22-
U . D . 4 . - S OCIEDADES HISTÓRICAS

RASGOS IDENTIFICADORES

TEOCRACIA:
1º sacerdote -rey (EN, ENSI)
2º rey militar+sacerdote (LUGAL)
idea de intermisión hombres -dios
finalmente en Asiria carácter semidivino del rey.

ORGANIZACIÓN:
1º ciudades -Estado (Sumeria)
Formación de imperios (Akkad: Sargón I). Supremacía de una ciudad frente a otras.
Desde III milenio: Imperio=Estado personificado en el Emperador dueño de un
territorio con ciudades y Estados vasallos.

ADMINISTRACIÓN:
Centralizada
Máxima autoridad rey-emperador con canciller y funcionarios
Centro de poder: TEMPLO (pol., eco., rel.). Con asirios palacio diferenciado.

EJÉRCITO:
Base mantenimiento de los imperios
Más importante desde asirios (carros...)
Táctica, crueldad...

SOCIEDAD: Estamental cerrada

Rey-sacerdote / Emperador
Libres Ricos (funcionarios, sacerdotes, mercaderes.
Pueblo llano (campesinos, artesanos)

Esclavos: indígenas, prisioneros.

SISTEMA JURÍDICO (Código de Hammurabi 1755 a.C.)

ECONOMÍA:
Centro económico: TEMPLO
Base agrícola
Propiedad privada, estatal y del templo
Trabajo organizado. División del trabajo
Comercio floreciente centralizado
Sistema de intercambio premonetario = necesidad de centralizar intercambios

CULTURA:
Trascendencia de la escritura (cuneiforme )
Religión politeísta: Sol, Luna, Guerra, Fertilidad...(Enki, Enlil, An, Utu, Nanna,
Inanna, Ishtar, Shamash, Marduk...)
Arte: rico y variado (Zigurat, Escultura, Glíptica)

19
-Pág.23-
CIENCIAS SOCIALES (GEOGRAFÍA E HISTORIA)

2.1.5. EGIPTO
A lo largo del valle del Nilo se teje desde el final del Neolítico una civilización agrícola que
crece y se desarrolla al ritmo de las crecidas periódicas del río cuyo control moviliza a las gen-
tes de sus orillas convirtiéndose en acicate de su progreso. El Imperio Egipcio se mantuvo, sin
grandes cambios, con pequeños paréntesis de anarquía o de invasiones, aunque los protagonis-
tas de éstas terminaban por adaptarse a la mentalidad y forma de organización egipcias. Esta
evolución podemos resumirla en el siguiente cuadro:

SUCESIÓN CRONOLÓGICA
MESOLÍTICO (8000-5000)
NEOLÍTICO (El-Fayum, Tasiense, Badariense ) (5000-4000)
CALCOLÍTICO – P. PREDINÁSTICO (Nomos) (4000-3100)
IMPERIO ANTIGUO (Menfis) (Din. I-VI) (3110-2665)
Menes “rey del Alto y Bajo Egipto”
Construcción pirámides (Djoser, Keops, Kefrén, Mikerinos)
Consolidación del Estado. Faraón = Horus

PRIMER PERIODO INTERMEDIO (Din.VII-X) (2181- 2050)

Feudalización. Predominio de los nomos.

IMPERIO MEDIO (Tebas) (Din.XI-XII) (2050-1786)


Mentuhotep II reunifica.
Conquistas exteriores (st con Sesostris III)

SEGUNDO PERIODO DE TRANSICIÓN (1786-1567)


(Din.XIII-XVII)
Hyksos. Resurgimiento de Tebas.
IMPERIO NUEVO (Tebas) (Din.XVIII-XXV) (1567- 518)
Amosis I reunifica.
Amenofis IV – Akhenaton (reforma religiosa)
Relaciones con Mitanni.
Enfrentamiento con hititas (Qadesh: Ramsés II-Hattusilis II)
Emperadores libios (machauach) y etíopes (nubios).
662-653 Conquista Asiria (Asaradón).
Psamético independiza.
Rivalidad con Imp. Neobabilónico.

IMPERIO PERSA (Din. XXVII-XXX) (Satrapía) (518-332)


IMPERIO DE ALEJANDRO MAGNO-
DINASTÍA PTOLEMÁICA (332-30)
IMPERIO ROMANO

20
-Pág.24-
U . D . 4 . - S OCIEDADES HISTÓRICAS

RASGOS IDENTIFICADORES

TEOCRACIA:
Emperador (Faraón) = dios

ORGANIZACIÓN:
1º ciudades-Estado (Nomos)
Formación de dos imperios (Alto y Bajo Egipto).
Unificación en un solo Imperio con Menes.

ADMINISTRACIÓN:
Centralizada
Máxima autoridad dios-faraón con visir y funcionarios

EJÉRCITO:
Base mantenimiento del imperio
Ejército bien estructurado, preparado y dotado (utilización de carros desde Hyksos).
Marina poderosa.

SOCIEDAD: Estamental cerrada

Emperador y su familia (muy amplia, con varias esposas y concubinas)


Libres Ricos (funcionarios, sacerdotes, mercaderes, mandos militares.)
Pueblo llano (campesinos, artesanos)

Esclavos

SISTEMA JURÍDICO (Elaborado sistema garantizado por el Estado)

ECONOMÍA:

Base agrícola. Explotación de recursos mineros


Propiedad privada, estatal y de los templos
Trabajo organizado. División del trabajo
Comercio floreciente centralizado
Sistema de intercambio premonetario = necesidad de centralizar intercambios

CULTURA:
Trascendencia de la escritura (sistema jeroglífico)
Religión politeísta muy elaborada con una casta sacerdotal poderosa e influyente.
(Amón-Ra, Isis, Osiris, Horus, Thot, Nut...
Culto funerario complejo al que se dedican enormes recursos. Momificación,
sepulturas.
Arte: rico y variado (Palacios, Templos, Escultura, Pintura, Orfebrería, Glíptica)

21
-Pág.25-
CIENCIAS SOCIALES (GEOGRAFÍA E HISTORIA)

2.2. SOCIEDAD, POLÍTICA, CULTURA Y ARTE EN EL MUNDO


CLÁSICO: GRECIA Y ROMA

2.2.1. LA CIVILIZACIÓN GRIEGA

2.2.1.1. El marco geográfico y la evolución histórica.


El extremo suroriental de Europa, al sur de la Península de los Balcanes, será la cuna en la
que nazca la Civilización griega, a caballo entre la occidentalidad europea y la orientalidad del
Próximo Oriente.
Desde, aproximadamente, el 4000 a.C. varias pulsaciones culturales desembocarán en la
construcción de la cultura clásica. Dichas fases podemos resumirlas en el siguiente esquema

1. NEOLÍTICO

2. CULTURA CICLÁDICA (3000-1100)


Edad del Bronce Inicial.
Ídolos de mármol femeninos estilizados

3. CULTURA MINOICA (1400-1100)


Knossos, rituales con toros, Talasocracia

4. CULTURA MICÉNICA (1400-1100)


Micenas, Tesoro de Atreo.
Época mítica descrita en la Iliada y la Odisea

5. ÉPOCA DEL GEOMÉTRICO (1100- 750)

6. ÉPOCA ARCAICA (750- 480)

7. ÉPOCA CLÁSICA (480- 323)

8. ÉPOCA HELENÍSTICA ( 323- 168/86)

El marco geográfico griego favorece el individualismo característico del griego clásico por
ser un territorio muy compartimentado. De este modo Grecia no constituyó nunca una unidad
política, organizándose en ciudades-Estado independientes. Sí eran conscientes de la unidad
cultural que había entre ellos, que llamaban koiné; pero no llegaron a unirse políticamente. Esta-
blecieron ocasionalmente, y con duración variable, Ligas de ciudades, bien para enfrentarse
contra enemigos exteriores, como hicieron frente a los Persas (Guerras Médicas); bien para
luchar entre ellos mismos (Guerras del Peloponeso, por ejemplo).

22
-Pág.26-
U . D . 4 . - S OCIEDADES HISTÓRICAS

2.2.1.2. La democracia griega

EL MUNDO GRIEGO

Una de las principales aportaciones griegas a la cultura occidental es, sin duda, el con-
cepto de Democracia. Rompiendo con la milenaria conceptuación teocrática de la organización
política, vigente en las civilizaciones egipcia y mesopotámica, los griegos de la época clásica
prefieren organizar el gobierno de su polis con la participación de todos los ciudadanos, a
través de representantes elegidos, en la toma de decisiones. Aunque en realidad la prosperidad
política y material de toda polis griega se basaba en un principio totalmente opuesto a la idea
de democracia: la existencia de una gran cantidad de esclavos que servían de soporte a la
estructura económica. La forma específica de organización variaba de unas ciudades a otras.
Tomando como ejemplo el caso de Atenas podemos esquematizar como sigue la “Constitu-
ción” diseñada por el reformador Clístenes:
La Ekklesía, o Asamblea, estaba integrada por los ciudadanos libres y elegía a los 500 miem-
bros de la Bulé (50 por tribu), especie de Parlamento que compartía la potestad legislativa con
la Ekklesía. La Pritania era el órgano directivo de la Bulé, renovado cada 36 días. El Areópago
se encargaba de la supervisión. Los Arcontes, elegidos por la Ekklesía, se encargaban del poder
ejecutivo y de ellos dependían los estrategos o generales del ejército.

EKKLESÍA

ARCONTES BULÉ

AREÓPAGO PRITANIA

23
-Pág.27-
CIENCIAS SOCIALES (GEOGRAFÍA E HISTORIA)

2.2.1.3. La Cultura y el Arte


Las aportaciones culturales griegas son innumerables. La Filosofía, las Ciencias y las
Letras están repletas de nombres griegos.

La preocupación del hombre griego por el origen del mundo, del hombre y de las cosas, le
hace meditar e investigar sobre la base del racionalismo que caracteriza al espíritu práctico hele-
no. Sócrates, Platón o Aristóteles, son ejemplos con el suficiente peso para entender la impor-
tancia de la Filosofía griega.

Esa meditación e investigación lleva a conceptualizar todo y permite avanzar en diversos


aspectos científicos, tales como las Matemáticas (Pitágoras, Euclides, Arquímedes), Astro-
nomía (Aristarco, Eratóstenes), etc.

La inquietud del espíritu griego fructificó además en numerosas manifestaciones literarias,


de las que podemos destacar las obras dramáticas por su contenido filosófico y existencial y
por su aportación escénica.

El Arte Griego, por otra parte, tiene en la escultura y en la arquitectura sus principales
manifestaciones.

El estilo griego en la arquitectura cuajará en los tres órdenes: dórico, jónico y corintio. La
estética predomina sobre la técnica pues la base estructural está basada en la edificación arqui-
trabada; mas la belleza y proporcionalidad de sus formas elevan la arquitectura griega a la cús-
pide. El Partenón en Atenas o el Templo de Zeus en Olimpia, son ilustrativos ejemplos.

Por su parte, la escultura muestra una clara evolución desde el hieratismo y frontalidad
de la época arcaica (kuroi y korai) hasta el dominio de las formas y el afán por la plasma-
ción del movimiento de la época helenística. Monumentos de esta evolución son, por ejem-
plo, el Discóbolo de Mirón, las obras de Fidias, Policleto, Praxiteles o Lisipo.

2.2.2. LA CIVILIZACIÓN ROMANA

2.2.2.1. El marco geográfico y la evolución histórica


La Península Italiana, centrada en el marco mediterráneo, escenario de flujos y reflujos cul-
turales, será testigo del surgimiento de una de las civilizaciones más importantes de la historia,
forjadora de un imperio inmenso centrado en la cuenca del Mediterráneo, sobre la base de un
espíritu organizador y racionalista.
Tras la mítica fundación de la ciudad (tradición de Rómulo y Remo), el 21 de abril de 753
a.C., Roma pasó por una serie de etapas que podemos resumir en el siguiente cuadro:

24
-Pág.28-
U . D . 4 . - S OCIEDADES HISTÓRICAS

1. MONARQUÍA (750-510 a.C.)


(los famosos siete reyes de Roma)
2. REPÚBLICA (510- 27 a.C.)
Expansión Italiana
Guerras Samnitas
Guerra Latina
Guerra de Tarento
Guerras Púnicas (inicio expansión exterior)
Conquista de Hispania
Guerras Macedónicas
Guerra contra Antioco III
Reformas políticas y Guerras civiles
Dictadura de Sila
Triunviratos
3. IMPERIO
Dinastía Julio-Claudia (27 a.C.-68 d.C.)
Dinastía Flavia (69-96)
Emperadores Adoptivos (96-192)
(Trajano, Adriano y los Antoninos)
Dinastía de los Severos (193-235)
Anarquía militar (235-305)
Tetrarquía (305-395)
División del Imperio (395)
Imperio de Occidente (395-476)
Imperio de Oriente (395-1453)

25
-Pág.29-
CIENCIAS SOCIALES (GEOGRAFÍA E HISTORIA)

La inicial Monarquía sirvió para consolidar la estabilidad de la ciudad, preservar su inde-


pendencia frente a los Etruscos y extender el control sobre la zona del Lacio.
La República fue la forma de gobierno elegida para organizar la ciudad-Estado; pero fue
también testigo de la expansión territorial en la Península Italiana y del inicio de la expan-
sión exterior (Hispania, las Galias, Grecia, etc..). El desbordamiento institucional que suponía
la administración de las tierras incorporadas al dominio romano, entre otros factores, derivará
en la crisis del sistema. Los intentos de estabilización por medio de dictaduras (Julio César),
o triunviratos (Pompeyo, Craso y César; Marco Antonio, Lépido y Octavio) no terminaron
de solucionar el problema, desembocando en la concesión del título de Augusto al primer empe-
rador, Octavio.
Como imperio, Roma consolida su poder en todo el Mediterráneo incluyendo gran parte
de Europa (hasta las Islas Británicas y parte de Alemania, Croacia, los Balcanes y Rumanía), el
Norte de África y el Próximo Oriente.
Se sucederán una serie de dinastías (Julio-Claudios, Flavios, Antoninos, Severos) y una
etapa de anarquía militar, con la sucesión e incluso simultaneidad de diversos emperadores, que
desembocará en una reforma administrativa conocida como la Tetrarquía, que supondrá la
diferenciación de dos partes dentro del Imperio como preludio de la definitiva separación en dos
Imperios distintos articulada por Teodosio en el 395.
De los dos Imperios resultantes, el occidental desaparecerá hacia el 476, mientras que el
oriental o bizantino pervivirá, reduciendo su extensión paulatinamente, hasta 1453.

2.2.2.2. El marco geográfico y la evolución histórica


La capacidad organizativa romana se pone de manifiesto, además de en los aspectos mate-
riales que veremos en el apartado siguiente, en las estructuras sociales y políticas.
La sociedad romana se desglosa en los siguientes niveles básicos:
Patricios.- Hombres libres con linaje, patronos.
Optimates, nobiles.- Altos funcionarios.
Ordo equester.- Caballeros, latifundistas.
Plebeyos.- Hombres libres sin linaje
Libertos.- Esclavos liberados
Esclavos.- Bienes patrimoniales, sin derechos, fuerza de trabajo y base del sis-
tema económico.

26
-Pág.30-
U . D . 4 . - S OCIEDADES HISTÓRICAS

Políticamente, después de la constitución de la República, los legisladores mostraron una


especial preocupación por evitar que el poder cayera en manos de una sola persona, de ahí que
diseñaran un sistema de gobierno en el que la colegialidad era principio esencial para todos los
cargos.

Las siglas S.P.Q.R.: Senatus PopulusQue Romanus (El Senado y el Pueblo Romano),
resume la filosofía política de la República. Junto al Senado, organismo deliberatorio y asesor
y ratificador de acuerdos populares, integrado por los cabezas de linaje y ex cónsules, los comi-
cios populares elegían a los magistrados que gobernaban la ciudad. Estos magistrados esta-
ban sometidos al principio de cursus honorum que impedía la elección en años consecutivos y
obligaba a desempeñar los cargos inferiores antes que los superiores.

MAGISTRADOS Al iniciarse el Imperio se


CÓNSULES
añade la figura del Emperador
SENADO
MAGISTRADOS o Augusto que dispone de un
PRETORES PLEBEYOS Consejo consultivo. Las pro-
CENSORES TRIBUNOS
DE LA PLEBE
vincias exteriores a Roma se
EDILES reparten entre el Emperador y
EDILES PLEBEYOS
CUESTORES el Senado que nombran sus res-
pectivos gobernadores (Lega-
dos y Procónsules, respectiva-
COMICIOS mente).
COMICIOS TRIBUTOS COMICIOS ASAMBLEA
CURIATOS 4 Tribus urbanas, CENTURIATOS SOCIAL DE
17 tribus rurales LA PLEBE
10 Tribus, 30 Curias (luego 31) 5 clases y 193 centurias
Posteriormente, el poder
PUEBLO ROMANO imperial se va haciendo más
personalista convirtiéndose el
Emperador en la autoridad omnímoda que se vale de un Consejo de la Corona, del Ejército y
de la Administración civil de las provincias para mantener el poder.

2.2.2.3. La Cultura y el Arte. El Derecho Romano


Lejos del estereotipo de pueblo inculto y belicoso, Roma supo edificar, sobre su propia tra-
dición y la influencia de los pueblos conquistados e incorporados al Imperio, una Cultura prós-
pera y rica.

Las Letras y las Ciencias romanas tienen como grandes manifestaciones:

Historia: Polibio, Julio César, Tito Livio, Tácito.

Teatro: Plauto, Terencio.

Retórica: Cicerón, Séneca.

Lírica: Ovidio, Horacio, Virgilio.

27
-Pág.31-
CIENCIAS SOCIALES (GEOGRAFÍA E HISTORIA)

Geografía: Estrabón, Ptolomeo.


Medicina: Celso.
Ciencias Naturales: Plinio.
Pero quizás la aportación más grande de Roma a la cultura occidental fue su Derecho.
Obra maestra de Roma, es una clara expresión de sus virtudes de gobierno y organización. Sus
bases son:
– Igualdad de los ciudadanos ante la ley
– Sentido práctico y concreto
– Supremacía de la ley
– Renovación y perfeccionamiento al ritmo de la experiencia
– Respeto a la tradición.
El Arte es fundamentalmente civil y práctico, lo que contrasta con el carácter esencial-
mente religioso del arte de las grandes culturas que precedieron a Roma. Como servicio públi-
co, la arquitectura se emplea en la construcción de edificios de diversa funcionalidad:
Para espectáculos públicos.
– Teatros, para representaciones dramáticas (Pompeya, Mérida)
– Anfiteatros, para espectáculos sangrientos (Itálica, Mérida, Arlès, Nîmes y,
sobre todo, el Coliseo de Roma)
– Circos, para carreras de carros
Para entretenimiento y asuntos públicos.
– Basílicas, para transacciones económicas y administración de justicia.
– Termas, para baños, gimnasio, biblioteca... (Termas de Caracalla en Roma)
De carácter religioso
– Templos, con influencia griega y etrusca.
– Monumentos funerarios.
Obras de ingeniería
– Puentes, como complemento a la red de calzadas (Alcántara)
– Acueductos, para el abastecimiento de agua a las ciudades (Segovia, de los
Milagros en Mérida)
– Cloacas, eliminación de las aguas residuales.

28
-Pág.32-
U . D . 4 . - S OCIEDADES HISTÓRICAS

Conmemorativos

– Arcos (de Constantino y de Tito en Roma, Tarragona, Cáparra y Bará e


España).

– Columnas (de Trajano en Roma)

La escultura se pone al servicio, esencialmente, del retrato y del relieve histórico.

Por último, las artes plásticas menores tuvieron también un gran desarrollo (cerámica,
mosaico, pintura, orfebrería, joyería, etc...).

2.3. ORIGEN Y DESARROLLO DEL CRISTIANISMO


En el marco del Imperio romano se produjo el surgimiento de una nueva doctrina religosa,
basada en el monoteísmo judaico y en las enseñanzas de Jesús de Nazaret y que fue difundida
durante el primer siglo por sus propios apóstoles y discípulos.

La vida y predicación de Joshua bar Yusef (Jesús, hijo de José), se centran en los momentos
en los que Roma, convertida ya en la potencia hegemónica del Mediterráneo, domina Judea que
se encuentra integrada en el Imperio como una provincia más. Jesús nació en una fecha impre-
cisa, probablemente entre el 6 y el 5 a.C. y la llamada vida pública de la que nos hablan los
evangelistas se desarrolla durante el imperio de Tiberio, cuando aún una parte de Israel mante-
nía cierto grado de autonomía con su rey Herodes Antipas. La crucifixión tuvo que tener lugar
el 7 de abril del año 30 d.C. A partir de ese punto comienzan a formarse en Palestina las pri-
meras comunidades cristianas que pronto se van extendiendo por el Próximo Oriente. Un papel
esencial en la expansión de la doctrina, además de por los Apóstoles, va a ser jugado por Saulo
de Tarso (San Pablo) quien inicialmente fue enviado por el Sanedrín a combatir y perseguir a
las comunidades que habían surgido en Damasco. Pablo no sólo va a difundir la nueva fe sino
que podemos considerarle el auténtico diseñador de los principios doctrinales iniciales.

La convulsión que la nueva religión producía en la sociedad politeísta romana fue conside-
rada inicialmente como una amenaza directa al Estado y, consecuentemente, se procedió a la
sistemática persecución de sus seguidores (especialmente sangrientas fueron las emprendidas
por los emperadores Nerón, Domiciano, Trajano, Marco Aurelio, Septimio Severo, Máximo,
Decio, Valeriano y Diocleciano).

Durante el siglo II comienzan las crisis doctrinales que, desde entonces, han ido jalonando
la historia de la Iglesia. Corrientes luego catalogadas como heréticas plantean interpretaciones
alternativas de determinados principios del dogma o de las costumbres, formas de vida y acti-
tudes ante diferentes aspectos. La reunión de sínodos y concilios se convirtió en algo necesario
para atajar las llamadas herejías.

29
-Pág.33-
CIENCIAS SOCIALES (GEOGRAFÍA E HISTORIA)

Sin embargo, el avance del cristianismo fue imparable consiguiendo primero la permisivi-
dad para el ejercicio de su culto, el posterior reconocimiento y, finalmente, tras el Edicto de
Milán del 313, con Constantino como Emperador, la conversión en la religión oficial del
Estado romano. Este hecho está en la base del papel rector que la Iglesia ejercerá a lo largo de
la Edad Media en la sociedad europea.

2.4. ASPECTOS SIGNIFICATIVOS DE LA PREHISTORIA Y LA EDAD


ANTIGUA EN EL TERRITORIO ESPAÑOL ACTUAL

2.4.1. EL PALEOLÍTICO

2.4.1.1. El Paleolítico
A) Los primeros pasos
La aparición del Hombre en la Península Ibérica es una cuestión en proceso de inves-
tigación y revisión. Los descubrimientos de Atapuerca (Burgos) han permitido a JUAN LUIS
ARSUAGA y a IGNACIO MARTÍNEZ aportar una nueva teoría sobre la evolución del
Hombre en Europa, definiendo incluso una nueva especie, el Homo antecessor, que sería la
primera en poblar el continente (la datación de los fósiles encontrados nos remonta a 800.000
años) y el eslabón desde el que se separarían los neandertales y los cromañones, o sea, nues-
tra propia especie.

B) El Paleolítico Inferior (800.000 a 100.000 a.C. aprox.)


Las primeras manifestaciones abundantes y seguras, de cultura material, debemos situarlas
en el Paleolítico Inferior Chelense y Achelense. Sus principales huellas: La acumulación de
restos de bifaces y otros útiles en los cazaderos de Elephans Primigenius de los valles de
Tajo, Jarama, Henares y Manzanares y en Torralba y Ambrona (Soria).

C) El Paleolítico Medio (100.000 a 50.000 a.C. aprox.)


Esta etapa, que desde el punto de vista de la cultura material recibe el nombre de Muste-
riense, está protagonizada por el Homo Sapiens Neanderthalensis u Hombre de Neanderthal
del que en España destacan los hallazgos de Atapuerca, Lago de Bañolas y Gibraltar.

D) El Paleolítico Superior (50.000 a 10.000 a.C. aprox.)


El primer y más denso poblamiento de nuestra especie, Homo Sapiens Sapiens, en la Penín-
sula se produce en la franja cantábrica y los Pirineos. El periodo cultural que nos ocupa se com-
partimenta en diferentes episodios atendiendo a la morfología de los utensilios líticos. De tales
episodios destacan en la Península:

30
-Pág.34-
U . D . 4 . - S OCIEDADES HISTÓRICAS

Auriñaciense: (restos en Camargo -Santander-, Parpalló -Valencia-).


Gravetiense.
Solutrense: cornisa cantábrica (Altamira, El Castillo) y región levantina
(Parpalló).
Magdaleniense: Auge del arte rupestre paleolítico (Altamira).

2.4.1.2. El Neolítico
En España la llegada de la revolución neolítica se producirá de un modo más tardío que en
el Próximo Oriente e irá asociada en algunas zonas a la aparición de la primera metalurgia. El
Neolítico Hispano, que derivará en la primera edad de los metales (Calcolítico, Bronce Inicial),
está asociado a una manifestación característica del Neolítico de la Europa Occidental: el
Megalitismo. En el caso de España, sus expresiones no son otras que enterramientos colecti-
vos de variada morfología, desde las pequeñas cistas, generalmente rectangulares con pare-
des y techo a base de grandes losas de piedra y luego cubiertas por túmulos, hasta los comple-
jos dólmenes de corredor del suroeste, con pasillos de acceso a cámaras circulares, todo ello
también a base de lajas de piedra y cubiertos por amplios túmulos. Mención aparte merecen las
manifestaciones levantinas y baleares, tales como las taulas y las navetas.

2.4.1.3. Edad del Bronce

A) Bronce Inicial
El inicio de la utilización del metal para la fabricación de útiles y armas se solapa en el caso de
España con la continuación de la costumbre de enterrar en forma colectiva en monumentos mega-
líticos, como se pone de manifiesto en las características culturas Almeriense y de Los Millares.
Esta etapa muestra también una cultura específica, a veces también asociada a los enterra-
mientos megalíticos, que tiene en su tipología cerámica su propia definición: Cultura del vaso
campaniforme, que se extiende por España y Europa en diferentes etapas difundiéndose de
unas zonas a otras distintos tipos cerámicos que guardan como notas comunes la tonalidad
negruzca de las pastas, debido a la cocción reductora, y las decoraciones lineales por impresión
o excisión con ocasionales aplicaciones en los espacios exentos de pastas blancas.

B) Bronce Medio
Se desarrolla una cultura que muestra evidentes paralelismos con otras del Mediterráneo,
que es ya una vía de tránsito cultural y no dejará de serlo. La cultura hispana a la que nos refe-
rimos es la Cultura Argárica. Son características sus espadas y puntas de lanza, su orfebrería y
la morfología de sus recipientes cerámicos, además de sus formas de enterramiento por inhu-
mación del cadáver, en posición fetal, en el interior de tinajas, acompañados de ofrendas.

31
-Pág.35-
CIENCIAS SOCIALES (GEOGRAFÍA E HISTORIA)

C) Bronce Final
La última etapa de la Edad del Bronce va a tener como principal manifestación peninsular
una importante cultura que desde el sureste irradiará su influencia a otras zonas, se trata de la
Cultura Tartésica. Otras sociedades en el interior irán recibiendo influencias procedentes de
Europa que traerán consigo la metalurgia del hierro, preparando el paso a otra nueva etapa y
configurando paulatinamente el panorama cultural que se encontrarán los romanos a su llegada
a la Península.

2.4.2. LOS PUEBLOS PRERROMANOS


Sobre la base del reparto cultural del Bronce Final, la Península va recibiendo una serie de
oleadas invasoras protagonizadas por pueblos indoeuropeos: Primero (900-800 a.C.) Cultu-
ra de los campos de urnas y Cultura de los campos de túmulos y más tarde en, posiblemente,
dos oleadas (750-700 a.C y 650-600 a.C.) de Celtas. Tras las mismas se produce la fusión de
elementos indoeuropeos e indígenas y la definición de las distintas áreas culturales. Las
principales aportaciones de los pueblos indoeuropeos serán la metalurgia del hierro y las
lenguas de origen indoeuropeo que se superpondrán a las indígenas.

2.4.2.1. Zona Meridional


A) Tartessos.- Arrancando del Bronce Final, en el valle del Guadalquivir se desarrolla la
Cultura Tartésica que, entre fechas anteriores al 1000 a.C. y el s. V a.C., jugará un papel pro-
tagonista en el mediodía peninsular. Sus rasgos definitorios son:
– La base de su economía era agraria
– Fuerte desarrollo urbano
– Orfebrería muy desarrollada: bronce, al oro y a la plata.
– Minas cupríferas onubenses; estaño de Galicia, Plata de Sierra Morena.
– Su flota controlaba el paso del Estrecho.
– Competencia comercial con fenicios y con sus herederos cartagineses
– Victoria cartaginesa sobre los griegos en Alalía (535 a.C.) = hegemonía cartaginesa en el
Mediterráneo occidental = Tartessos vencido y sus ciudades sometidas a vasallaje.
– Reyes mítico-históricos: GERIÓN, ARGANTHONIOS.

32
-Pág.36-
U . D . 4 . - S OCIEDADES HISTÓRICAS

B) El mediodía hacia el s. V a.C.


Túrdulos, Turdetanos, Mastienos, Bastetanos, entre otros, cuyas características cultura-
les respondían a los siguientes rasgos:
Economía.
– Urbanismo avanzado
– Ganadería y agricultura a gran escala, uso del arado.
– Trabajo de los metales.
– Industrias especiales: salazón de pescados.
Sociedad.
– Estratificación social con grupos bien definidos: esclavos, libres, nobles,
sacerdotes, familia real.
Cultura.
– Desarrollo de las artes plásticas.
– Sistema de escritura propio (ibérico meridional) y zonas de utilización del
alfabeto fenicio y otra variedad llamada libio-fénice.

2.4.2.2. Zona oriental


Toda la franja costera que se extiende desde la actual Francia hasta Murcia y que penetra
hacia el interior con anchura irregular, estaba ocupada por los iberos. Los iberos estaban divi-
didos en diversas tribus, cada una con su propia organización, unas formando por sí mismas
unidades políticas autónomas, otras divididas en diversas ciudades-Estado. Entre dichas tribus
figuran: Ilergetes, Ceretani, Ausetani, Castellani, Indigetes, Lacetani, Layetani, Ilercaones,
Edetani, Lobetani, Contestani... Sus características culturales más significativas eran:
Economía.
– Agricultura basada en vid, olivo y productos de huerta.
– Cultivo de lino y fabricación de tejidos.
– Ganadería.
– Dualidad: Costas: Ciudades grandes comerciales.
Interior: Ciudades pequeñas agrícolas.

33
-Pág.37-
CIENCIAS SOCIALES (GEOGRAFÍA E HISTORIA)

Sociedad.
– Núcleo aristocrático dirigente,
– Unas ciudades sistema monárquico (ciudades con influencia cartaginesa).
– Otras sistema oligárquico-senatorial (con influencia griega y luego romana).
– Trascendencia social de la guerra (armamento: lanza, escudo largo y falcata).

Cultura.
– Sistema de escritura propio que mezclaba signos alfabéticos y silábicos.
– Amplio desarrollo de las artes plásticas, con importantes hitos en la historia de
la escultura peninsular: Dama de Elche, Dama de Baza, Bicha de Balazote, etc.
– La estatuaria y la cerámica muestran una clara vinculación con las formas
estéticas griegas.
– Religión de carácter naturalista. Procesiones y danzas, ofrenda de exvotos.

2.4.2.3. Zona Central y del Oeste


A) Zona Occidental
Zona que habitan los lusitanos y que se extiende por el actual Portugal hasta el Duero y
abarca buena parte de Extremadura. Sus características culturales son:

Economía.
– Desequilibrio tierras fértiles - tierras pobres = “bandolerismo”

Sociedad.
– Jefes elegidos en asambleas tumultuosas
– Armamento: escudo cóncavo al exterior, cascos tejidos con nervios de ani-
males.

Cultura.
– Carecen de un sistema de escritura propio.

34
-Pág.38-
U . D . 4 . - S OCIEDADES HISTÓRICAS

B) Zona Central
Habitan la meseta pueblos muy heterogéneos que los romanos bautizaron como celtiberos.
Entre ellos se citan: Vettones, Vacceos, Carpetanos, Oretanos, Arévacos...
Economía.
– Base agrícola (trigo y cebada), completada con el pastoreo de ovejas
– Forja de armas, herramientas y aperos.
Sociedad.
– Algunas ciudades tenían a su frente régulos o reyezuelos.
– En otras, o incluso en las mismas, alcanzaba gran importancia el Senado
– Asambleas populares para ratificar decisiones y elegir un caudillo por acla-
mación en caso de guerra
– Aparecen dos instituciones especiales:
- La Devotio o Katáspeisis, u obligación del cortejo del caudillo de darse
muerte en caso de fallecimiento del mismo.
- Clientelas militares: las poblaciones débiles se ponen bajo la protec-
ción de otra más fuerte.
Cultura.
– Carecen de un sistema de escritura propio.
– Destaca la manifestación escultórica de los vacceos en los Toros de Guisando.
– También cabría citar la costumbre de ofrendar a los extranjeros visitantes la
“Hospitalidad” mediante las llamadas teseras de hospitalidad.

2.4.2.4. Zona Norte


Habitada por Galaicos, Astures, Cántabros, Vascones, presenta los siguientes rasgos cultu-
rales:
Economía.
– Agricultura muy rudimentaria
– Predominio de la recolección.
Sociedad.
– Sociedad matriarcal: avunculado, covada. Gens matriliniales.

35
-Pág.39-
CIENCIAS SOCIALES (GEOGRAFÍA E HISTORIA)

– Vestían sayos negros largos.


– Armamento: honda, hacha-bipenne, cascos de cuero con adornos en forma de
media luna.
– Utilizaban la táctica de la guerra de emboscadas.
Religión.
– Cierto culto a la Luna.
– Un dios identificable con Marte, al que se ofrecen sacrificios bebiendo sangre.

2.4.3. LAS COLONIZACIONES


Desde aproximadamente el año 1000 a.C. se inicia a lo largo del Mediterráneo una crecien-
te actividad comercial protagonizada por griegos y fenicios. Ambos pueblos harán gala de un
gran espíritu comercial explorando nuevos horizontes y mercados en el occidente. Los griegos
focenses y los fenicios tirios comenzarán la fundación de factorías. Como consecuencia de ello
en los pueblos colonizados se produce la llegada de importantes influencias culturales:

2.4.3.1. Colonización fenicia


Su relación con la población indígena fue esencialmente comercial sin pretender una asi-
milación cultural ni una dominación política. En cualquier caso, la influencia cultural termi-
nó por ser inevitable y afectó a cuestiones como la transmisión del alfabeto, de las formas
cerámicas y plásticas en general y de los cultos religiosos (culto a Astarté).
Los yacimientos arqueológicos peninsulares muestran la especificidad de los asentamientos
como pequeños núcleos situados en islas o promontorios costeros asociados a la correspon-
diente necrópolis (con dualidad incineración-inhumación), con predominio de materiales rela-
cionados con la actividad comercial, como son recipientes cerámicos de almacenamiento y
transporte (ánforas de diversos tamaños). Los principales asentamientos fueron:
Agadir.- Fundada en la actual Cádiz en el 1100 a.C. según la tradición.
Malaka.- En la actual Málaga.
Sexi.- En Almuñécar (Granada).
Abdera.- Posiblemente en Adra (Almería).
Baria.- Localización imprecisa, posiblemente en Villaricos (Almería).

36
-Pág.40-
U . D . 4 . - S OCIEDADES HISTÓRICAS

2.4.3.2. La colonización cartaginesa


Cartago (Qart-Hadashat), fue fundada en 814 a.C. llegando a convertirse en una poten-
cia comercial y militar en competencia inicial con los griegos. Esa competencia terminó con
un enfrentamiento bélico en el que lograron sobre los griegos la victoria en la batalla naval de
Alalía (535 a.C.), momento a partir del cual los cartagineses pasan a ser los dominadores del
mediterráneo occidental.
La Península, y fundamentalmente su parte meridional, será objeto de una política comer-
cial de explotación de recursos, que incluirá el afán por el control político, sobre todo con la
dinastía de los Bárquidas (Amílcar Barca, Asdrúbal, Aníbal). La obtención de plata, espar-
to, trigo, salazones, así como el reclutamiento de mercenarios, eran los beneficios económicos
y militares que justificaban la dominación. Además de las colonias fenicias, cuyo control asu-
mieron los cartagineses tras el cese de la actividad de aquéllos en el occidente mediterráneo,
éstos fundaron otras colonias:
Ebussus (Ibiza), fundada en 654 a.C.
Akra Leuké (Alicante).
Qart-Hadashat (Cartagena), en 226 a.C.

2.4.3.3. La colonización griega


Los primeros pasos de la colonización griega en la Península hay que relacionarlos con los
iniciales viajes de exploración, como el realizado por Kolaios de Samos hacia el 630 a.C.
Rodios y samios (siglo VII a.C.) y focenses (s. VI a.C.) fueron los primeros colonizadores; des-
pués serán los massaliotas (procedentes de la colonia de Massalia -Marsella- , fundada en 600
a.C.) los que controlen el comercio de la zona (último tercio del siglo VI a.C.) y, finalmente, la
colonia de Ampurias se convierte en la dominadora (s. V a.C.). Las principales colonias grie-
gas fueron:
Mainaké.- entre Almuñécar y Vélez Málaga.
Hemeroskopeion.- En Denia o el Peñón de Ifach.
Emporion.- Al lado de la ciudad indígena Kese, será la más importante.
Rhode.- En Rosas.
Kallipolis.- Cerca de Barcelona.
Pyrenne.- En la Costa Brava

37
-Pág.41-
CIENCIAS SOCIALES (GEOGRAFÍA E HISTORIA)

2.4.4. LA ESPAÑA ROMANA


El avance romano en la Península italiana y su proyección marítima motivarán el choque de
intereses entre cartagineses y romanos, una competencia que derivará en tres importantes
enfrentamientos bélicos que concluirán con la desaparición como potencia de Cartago y lleva-
rán, al menos los dos primeros, a incluir a la Península Ibérica en el entramado de aconteci-
mientos y, al final, a la conquista de su suelo por las legiones romanas.
La conquista romana, que durará dos siglos, seguirá ritmos diferentes y las motivacio-
nes de los avances serán distintos. Concluida la conquista militar, continuará el proceso de
aculturación, de romanización, mucho más lento, que llevará a la asimilación del derecho,
de la religión, de las costumbres, de las formas de organización y de la lengua.
Buena parte de la médula hispana se forjará durante los seis siglos de relación y de ser parte
constitutiva del mundo romano.

2.4.4.1. La conquista romana

2.4.4.1.1. Las Guerras Púnicas


El inicio de la conquista romana de Hispania se producirá en el marco de la II Guerra Púnica
en la que el suelo peninsular será uno de los escenarios del teatro de operaciones. La derrota
cartaginesa en la I Guerra se había sellado con el Tratado del Ebro (226 a.C.) en el que, ade-
más de una fuerte cantidad pecuniaria como rescate de guerra y la pérdida de territorios, Carta-
go tuvo que aceptar la división del Mediterráneo occidental en dos zonas de influencia, de una
de las cuales Roma pasaba a ser la rectora. El afán de revancha por parte cartaginesa, unido a
los intereses estratégicos de Roma y su expansionismo naciente, propiciaron que un aconteci-
miento como la toma de Sagunto por Cartago forzara la declaración del segundo conflicto.
Aníbal preparó su estrategia y emprendió su marcha hacia Roma, a través de los Pirineos
y los Alpes. La expedición de Aníbal forzó el cambio de planes de Roma que se planteó el
ataque a Hispania como una forma de cortar la vía de suministros de dicho ejército para, así,
aislarlo: En 218 a.C. las tropas romanas dirigidas por Cneo Cornelio Escipión desem-
barcan en Emporion (Ampurias) e inician la conquista del territorio. Tras la victoria de
Escipión el Africano en Ilipa (Alcalá del Río)(206 a.C.) sobre los generales cartagineses
Magón y Giscón, avanza hasta Gades, (Agadir para los Cartagineses, o sea, Cádiz) que se
entrega sin combate. La flota cartaginesa abandona la Península y la dominación púnica fina-
liza: Roma domina las costas mediterráneas españolas.

38
-Pág.42-
U . D . 4 . - S OCIEDADES HISTÓRICAS

2.4.4.1.2. La conquista tras el fin de la II Guerra Púnica


Los romanos incumplieron su compromiso de abandonar el territorio ibero tras la derrota de
los cartagineses y transforman el territorio controlado en Provincia romana, como habían
hecho con Sicilia. A partir de este momento y durante casi dos siglos se suceden los aconteci-
mientos que posibilitarán la conquista de toda la Península:

La Guerra de Lusitania (155-136 a.C.) (Viriato)

Las Guerras Celtibéricas (153-133 a.C.) (Incluyendo el sitio de Numancia)

Las Guerras Cántabras (29-19 a.C.) (con la participación del ya emperador Augusto)

2.4.4. Organización política


Organización provincial.

A) Al inicio formó una Provincia con el territorio controlado: Hispania.

B) En 197 a.C. se dividió en dos provincias: Hispania Citerior e Hispania Ulterior.

C) En 27 a.C. Augusto divide la Hispania Ulterior en dos provincias, quedando, pues,


tres: Hispania Ulterior Lusitana o Lusitania, Hispania Ulterior Bética e Hispania
Citerior Tarraconensis.

D) Hacia el 212 d.C., Caracalla forma en el noroeste de la Tarraconensis la provincia


de Callaecia.

E) La reforma administrativa de Diocleciano en el 293 d.C. constituía a Hispania


como una de las tres diócesis de la prefectura de las Galias. La Diócesis de His-
pania se dividía en siete provincias: Callaecia, Tarraconensis, Carthaginensis,
Ballearica, Lusitania, Bética y Mauritania Tingitana (norte de África).

F) Constantino dividió la Tarraconensis, nuevamente, diferenciando la Carthaginensis


y la Ballearica, quedando el resto con el anterior nombre de Tarraconensis.

Al frente de cada provincia había un gobernador (proconsul o propraetor) nombrado por el


Senado romano ante el que eran responsables de su gestión. Los cuestores recorrían la Provin-
cia para recaudar impuestos.

39
-Pág.43-
CIENCIAS SOCIALES (GEOGRAFÍA E HISTORIA)

Las ciudades, eran de diverso tipo: estipendiarias (civitates stipendiariae), libres federa-
das (civitates liberae), libres no federadas o libres inmunes. Por otra parte, Roma funda tam-
bién ciudades para colonos romanos (Coloniae). Cada ciudad privilegiada disponía de un
ordo decurionum y de una asamblea popular. El poder ejecutivo recaía en los dos duumviros
y los dos aediles.

2.4.4.3. El arte y la cultura


Hispania terminará siendo una fuente más de la cultura romana. Baste citar a Marcus Anneus
Séneca y Lucius Anneus Séneca, preceptor de Nerón, MarcuS Anneus Lucanus, Pomponio Mela
(geografía latina), Columela, Marcus Valerius Martialis (Marcial) y Marcus Fabius Quintilianus.
Hispania, en definitiva, se convirtió en una parte sustancial del imperio cuyos ciudadanos lo
eran también de Roma. Y, como tal parte, va a verse, al inicio del siglo V, metida de lleno en el
fenómeno.

3. LAS SOCIEDADES MEDIEVALES

3.1. LAS INVASIONES BÁRBARAS. EL FIN DEL IMPERIO


DE OCCIDENTE

3.1.1. PROCESO
Tradicionalmente se habla de la caída del Imperio Romano, situando la fecha de la misma
en el 476 d.C., al ser depuesto por Odoacro el último emperador Rómulo Augústulo, como
consecuencia de las invasiones de los llamados bárbaros. Obviamente, ni la responsabilidad es
sólo de “las invasiones bárbaras”, ni el Imperio cayó en una fecha concreta. Podríamos decir,
con más precisión, aunque la afirmación tampoco sea una completa visión de los hechos, que el
Imperio Romano de Occidente se desmembró y desapareció tras un largo proceso de des-
composición interna y debilitamiento que posibilitó la entrada en su suelo, desde inicios del
siglo V d.C., de una serie de pueblos que terminaron por constituir Estados propios y sentenciar
la estructura de poder de Roma. El Imperio de Oriente, o Imperio Bizantino, se mantuvo e inclu-
so intentó en varias ocasiones reedificar el imperio occidental.
Los pueblos llamados a sustituir con poderes parciales el poder hegemónico de Roma eran
los llamados bárbaros, término con el que los romanos designaban a todo aquel que no hablaba

40
-Pág.44-
U . D . 4 . - S OCIEDADES HISTÓRICAS

latín ni griego. Tales pueblos, fundamentalmente los germánicos, vivían en la Europa Central y
Nororiental. Desde el siglo III a.C. mantienen relación con Roma pues algunos de ellos habitan
las tierras del limes, de la frontera centroeuropea del Imperio. Desde el siglo IV d.C. se autori-
za a algunos de esos pueblos a asentarse en los confines del Imperio en calidad de foederati
entregándoles una anualidad (annonae foederatae) a cambio del compromiso de defender las
fronteras del Imperio frente a otras tribus; después ese estatus de pueblo federado incluirá el
asentamiento dentro del propio territorio romano.
La presión final, a partir del s. V d.C. se deberá a la necesidad de estos pueblos de buscar
nuevas tierras con más recursos alimenticios al haber sufrido sus tierras originarias cambios cli-
máticos y haber aumentado su demografía. De un modo inmediato, la chispa que prende defi-
nitivamente la fase de la caída final de Roma va a ser el avance desde las estepas asiáticas de
los hiong-nu (hunos) de Atila, lo que empujará a unos pueblos contra otros. Jutos, Anglios,
Sajones, Suevos, Vándalos, Alanos, Lombardos, Francos, Alamanes, Burgundios y Godos (Visi-
godos y Ostrogodos), son los más importantes de estos pueblos germánicos que en unas zonas
u otras irán entrando y consolidando su poder.

3.1.2. ORGANIZACIÓN POLÍTICA SOCIAL Y ADMINISTRATIVA


Va surgiendo una relación de tipo socioeconómico que va a ser protagonista de la Alta Edad
Media en Europa y que se conservará, en determinados aspectos, más allá de la misma: el feu-
dalismo. Comienzan a aparecer vínculos personales de dependencia, a través de pactos de
fidelidad, entre el señor y los campesinos, algo que surge, en parte, sobre la base del sistema
del patrocinio romano: el campesino se pone bajo la protección del señor a cambio de una
aportación en especie.
La pirámide social está encabezada por la nobleza. La base la constituyen los siervos y
esclavos, por debajo del gran grupo de clases libres rurales y urbanas.
La Monarquía se caracteriza por su poder absoluto y la elegibilidad de los reyes. Una
asamblea de nobles elige al nuevo rey al morir el anterior. La corrupción del sistema llevará
a algunos reyes a asociar al trono a sus hijos y buscar los apoyos necesarios para que al morir
fuera elegido el hijo como rey.

3.1.3. LA ESPAÑA VISIGODA


En el años 409, Suevos, Vándalos asdingos, Vándalos silingos y Alanos cruzan los Pirine-
os y entran a saco en las ciudades y tierras de Hispania. Los Visigodos, tras ser admitidos como
federados en el Imperio de Oriente y después de diversos avatares, emprenden, con Alarico, un
largo y complicado periplo que les llevará incluso a sitiar y saquear Roma. Tras ello recibie-
ron de Roma la Provincia gala de Aquitania comprometiéndose a defender el territorio de nue-
vas invasiones. Además, su compromiso se extendía a luchar contra los pueblos que habían
invadido Hispania, lo que hicieron hasta el 418 . Vencidos por los Francos en la batalla de

41
-Pág.45-
CIENCIAS SOCIALES (GEOGRAFÍA E HISTORIA)

Vouillé, debieron centrar su dominio en Hispania hasta que, con el rey Teudis (531-548), el
reino visigodo se convierte expresamente en un reino hispano (545), con una provincia gala
(Septimania).
Leovigildo (573-586), afanado en conseguir la centralización del poder, lucha contra los
bizantinos, contra los vascones y contra los suevos, venciendo a su rey Miro e incorporando
Callaecia a su reino en 585, consiguiendo así la unificación territorial de Hispania. Su hijo
Recaredo (586-601) se convierte al catolicismo, y en 589 promueve el III Concilio de Toledo en
el que se adopta la católica como religión oficial del pueblo godo.

3.2. LAS SOCIEDADES FEUDALES EUROPEAS Y EL DESARROLLO


URBANO; EL ROMÁNICO Y EL GÓTICO

3.2.1. EL FEUDALISMO
Tras las invasiones germánicas, Europa se transforma en un mosaico de reinos indepen-
dientes. La mentalidad y personalidad germánicas se unen con el espíritu y organización roma-
nos y se impregnan con los principios doctrinales de la religión cristiana fructificando en un sis-
tema social, político y económico que llamamos feudalismo.
El feudalismo “es un conjunto de instituciones que crean y regulan las obligaciones de obe-
diencia y servicio por parte de un hombre libre llamado vasallo para con otro también libre lla-
mado señor y las obligaciones de protección y mantenimiento del señor respecto al vasallo”
(Ganshof)
La dependencia personal establecida entre señor y vasallo nace de un acto solemne de tipo
sacramental, que se denomina “homenaje”, en el que el señor coge entre sus manos las manos
del vasallo (“inmixtio manum”) reconociendo así éste la superioridad del señor. Ambos reali-
zan alternativamente el juramento y declaración de fidelidad mutua (“sacramentum”) y el señor
entrega al vasallo un símbolo del feudo entregado (“investidura”).
El vasallo asume como compromiso guardar fidelidad al señor, no dañar al señor y ayudar-
le (auxilium) y asistirle con consejos (consilium). Por su parte el señor asume el compromiso de
guardar fidelidad al vasallo, protegerle judicialmente y con las armas y mantenerle.
Estas relaciones personales afectan a los diversos niveles de la sociedad medieval: rey-
nobles, nobles-vasallos. En todo caso el tipo de relaciones fue muy variada. La expresión más
habitual será la que deriva de instituciones ya existentes en la época romana: Commendatio,
cesión de tierras a un señor a cambio de protección, cediendo éste a continuación la tierra para
su cultivo (beneficio); a lo que se unirá el concepto de subordinación germánico basado en el
juramento de fidelidad.

42
-Pág.46-
U . D . 4 . - S OCIEDADES HISTÓRICAS

3.2.2. ORGANIZACIÓN POLÍTICA Y ECONÓMICA


Económicamente el feudo será la unidad básica. Supone un sistema de autarquía, todo lo
necesario se produce en el feudo con lo cual se reduce el comercio considerablemente y, con
ello, los intercambios monetarios.
La monarquía es la forma política, el rey fundamenta su soberanía en su carisma y su poder
económico (basado en las posesiones territoriales), además de los vínculos personales de depen-
dencia que obligan a sus vasallos a guardarle fidelidad. El rey es el que imparte justicia y sus
decisiones son inapelables. Los nobles, vasallos del rey, ayudan a éste, le aconsejan a través de
Asambleas nobiliarias de carácter consultivo y, por su delegación, administran las regiones.

3.2.3. LAS CRUZADAS


Pilar fundamental del mundo medieval europeo es el cristianismo, religión oficial del Impe-
rio cuando es conquistado por los pueblos germánicos y asumida por éstos. La Iglesia alcanza-
rá un extraordinario poder que se mide no sólo en el número de almas que la componen sino
también en poder político y económico, siendo, a través de monasterios y obispos, parte del
engranaje feudal al convertirse en auténticos señores con sus feudos y vasallos. Esta sociedad
medieval cristiana tiene enfrente otra sociedad confesional en expansión, el Imperio musulmán,
enemigo no sólo potencial sino efectivo que en el occidente conquista España y amenaza al
resto de Europa, y en el oriente avasalla al Imperio Bizantino y conquista los Santos Lugares.
Ante esta situación, el espíritu belicista de la sociedad feudal supera las raíces pacifistas ori-
ginales del cristianismo y lleva a la misma a propugnar la reconquista y la lucha contra el cre-
ciente poder musulmán. Esto llevará a la convocatoria, entre 1090 y 1270 de ocho Cruzadas,
expediciones armadas patrocinadas por reyes y nobles bajo el amparo de la Iglesia, que logra-
ron éxitos pasajeros y fueron retos casi mitológicos para muchos caballeros sedientos de aven-
turas. Los intereses económicos comerciales no estuvieron ausentes de tales proyectos, mas, en
realidad, constituyeron fundamentalmente una Guerra Santa al estilo musulmán. Godofredo de
Bouillon patrocinó la primera, posteriormente, entre los organizadores de una de las cruzadas
se contará el legendario Ricardo Corazón de León.
Las consecuencias que para la sociedad medieval tuvieron las Cruzadas fueron importantí-
simas. Supusieron una expansión económica por la multiplicación del comercio, se elevó el
nivel cultural por el contacto con árabes y bizantinos, el Papa alcanzó la cumbre de su poder
temporal y surgieron órdenes militares de caballería que posteriormente influirían en la política
de sus reinos. Además el arte se alimentó con influencias orientales, la literatura se enriqueció
con la afluencia de temas realistas y fantásticos sobre las Cruzadas y el hombre europeo se reen-
contró con la cultura clásica preservada en Bizancio.

43
-Pág.47-
CIENCIAS SOCIALES (GEOGRAFÍA E HISTORIA)

3.2.4. LAS CIUDADES Y EL COMERCIO


Las ciudades ya habían perdido gran parte de su importancia al final del Imperio cuando se
produjo un reflujo de población hacia el campo al disminuir las perspectivas económicas en las
urbes. Durante la Alta Edad Media las ciudades pierden la mayor parte de su población e inclu-
so algunas quedan deshabitadas.

En torno a las viejas ciudades o en lugares nuevos irán surgiendo núcleos habitados por gen-
tes marginadas del entramado socioeconómico feudal y por comerciantes (núcleos llamados
burgos) que se harán también sedes de los talleres artesanales (maestros, oficiales, aprendices)
luego unidos en gremios, la industria manufacturera medieval. Su creciente importancia econó-
mica hará que los reyes les concedan privilegios y leyes especiales (fueros, cartas pueblas...)
que mantendrán su independencia de los señores feudales y generarán también discordias con
ellos.

Las ciudades terminarán por constituirse en un elemento transcendental. Dotadas de mura-


llas y con sus propios ejércitos serán un factor con el que contar en el equilibrio político de las
monarquías medievales.

El comercio y los gremios artesanales fueron los vértices de las ciudades medievales. Ciu-
dades como Venecia, Génova o Barcelona se dedicaban al comercio mediterráneo; Gante o Bru-
jas eran núcleos artesanales con importantes talleres textiles; en Alemania y las costas del Mar
del Norte y del Báltico una serie de ciudades se asociaban para el tráfico comercial en la Liga
hanseática...

3.2.5. LA SOCIEDAD FEUDAL


La sociedad medieval es una sociedad estamental. Cada estamento ocupa una posición en el
conjunto que es inalterable. Cada individuo nace, vive y muere en su estamento. Dicho conjun-
to forma una pirámide que podemos esquematizar del siguiente modo.

Rey
Alta Nobleza Alto Clero
Caballeros e hidalgos
Patriciado urbano, comerciantes
Pequeños propietarios agrícolas y maestros urbanos
Asalariados rurales y urbanos
Siervos personales, rurales y urbanos
Esclavos

44
-Pág.48-
U . D . 4 . - S OCIEDADES HISTÓRICAS

3.2.6. ROMÁNICO Y GÓTICO


El arte románico. Rasgos definitorios.
– Tradición romana mezclada con influencias bizantinas, difundida por la
reforma cluniaciense.
– El monasterio, con templo, manifestaciones fundamentales.
– Carácter: austeridad y recogimiento: contribuyen paramentos macizos.
– Elementos: arcos de medio punto con capiteles decorados. Planta del tem-
plo de cruz latina, crucero, ábsides, girola, capillas absidiales, torres.
bóveda de cañón, sencilla o reforzada con arcos perpiaños o fajones soste-
nidos por contrafuertes o estribos en el exterior. La bóveda de arista, con
arcos cruzados, aparece después. Naves laterales con dos pisos (superior tri-
forio). Cubrición del crucero: cúpula sobre pechinas. Portadas arco medio
punto, arquivoltas, jambas, tímpano, parteluz.
– Obras: Catedrales de Poitiers, Angulème y Clermont (Francia), Durham y
Peterborough (Inglaterra), Spira, Maguncia y Worms (Alemania), Parma,
Ferrara y Pisa (Italia), Santiago de Compostela, San Isidoro de León y Zamo-
ra (España)
El arte gótico. Rasgos definitorios.
– Soluciones constructivas a la pesadez del románico surgen asociadas a la
reforma monacal cisterciense.
– Elementos: bóveda de crucería, elemento esencial del edificio gótico. Arcos
apuntados, arbotantes, botareles, pináculos, gabletes, vidrierías, rosetón con
tracerías.
– El gótico robusto, s. XIII sencillez.
– El gótico gentil, s. XIV, efectos decorativos.
– El gótico flamígero, s. XV, máxima complicación, bóvedas estrelladas, trace-
rías de formas sinuosas que recuerdan llamas (dando nombre al estilo), profu-
sa decoración interior y exterior (frisos, gárgolas, pináculos) y con la apari-
ción, junto al arco apuntado, del carpanel, escarzano y conopial.
– Obras: Catedrales de Nôtre-Dame de París, Chartres, Reims, Estrasburgo y
Amiens (Francia), Colonia y Friburgo (Alemania), Siena, Orvieto, Milán,
Venecia (Italia), Salisbury, York, Canterbury (Inglaterra), León, Burgos, Tole-
do, Barcelona, Sevilla, Palma de Mallorca (España); Palacio de los Dux de
Venecia, Palacio de la Señoría de Florencia, Lonja de Valencia, Casa del Cor-
dón de Burgos, Palau de la Generalitat de Barcelona.

45
-Pág.49-
CIENCIAS SOCIALES (GEOGRAFÍA E HISTORIA)

3.3. AL ANDALUS Y LOS REINOS CRISTIANOS EN LA ESPAÑA


MEDIEVAL

3.3.1. LA ESPAÑA CRISTIANA


Tras la invasión musulmana iniciada en 711, algunos nobles visigodos se refugian en las tie-
rras montañosas del norte. En la zona occidental, la cornisa cantábrica, uno de esos nobles,
D. Pelayo, consigue frenar el avance islámico tras la batalla de Covadonga e inicia la conso-
lidación de un primer núcleo cristiano, el reino Astur-leonés. En la zona oriental, después de
la victoria del rey de los francos Carlos Martel en la batalla de Poitiers, se constituye, bajo los
auspicios de Carlomagno, la llamada Marca Hispánica, en cuyo seno se formarán una serie de
condados catalanes y aragoneses.
Una vez consolidados los primeros núcleos, España queda separada en dos ámbitos: uno
cristiano, dividido en diferentes reinos, y otro musulmán. Sin embargo, debemos romper con el
mito de dos mundos contrapuestos y permanentemente enfrentados en una guerra secular por el
espacio peninsular y con la religión por estandarte; ambas partes de la realidad de la España
medieval mantuvieron unas fluidas relaciones políticas, sociales, económicas y culturales que,
a ráfagas, dejaban de ser amistosas y daban lugar a los grandes avances de la llamada “Recon-
quista”, proceso que protagoniza la historiografía tradicional del medioevo hispano.
Al compás del avance reconquistador, la España cristiana evolucionará políticamente.
En la zona occidental, tras la formación del Reino Astur-leonés, éste se dividirá, inicial-
mente en dos (s. X): León y Castilla. Ambos reinos experimentarán sucesivas uniones y des-
uniones hasta la definitiva unión en 1230. Portugal, en el siglo XI también se separará de León,
en este caso definitivamente. Entre el siglo X y el XI estos reinos consiguen avanzar frente a
los musulmanes hasta alcanzar el Sistema Central. A finales del siglo XI, se produce la pene-
tración en el valle del Tajo y se conquista Toledo (1086). En el siglo XII se va avanzando por
el valle del Guadiana y en el XIII, se inicia la reconquista del valle del Guadalquivir, tras la
batalla de Las Navas de Tolosa, con la conquista de Sevilla (1248) protagonizada por Fer-
nando III el Santo, en quien se unieron en 1230 León y Castilla, y de Murcia, cuya incorpora-
ción a la corona castellana será obra del príncipe Alfonso, hijo del anterior, que reinaría como
Alfonso X el Sabio.
En la zona nororiental Navarra consolida su independencia ya en el siglo IX, pero su avan-
ce territorial se limita a la línea del río Ebro. Políticamente su vínculo con los condados arago-
neses se mantendrá hasta que éstos, como reino de Aragón, se unan con los condados catala-
nes en el siglo XII formando la Corona Aragonesa. Precisamente esta unidad política será la que
protagonice el auténtico avance reconquistador en la zona cuando Jaime I el Conquistador
incorpore Valencia y Mallorca a la Corona en el siglo XIII.

46
-Pág.50-
U . D . 4 . - S OCIEDADES HISTÓRICAS

Tras los avances en Andalucía, Murcia, Valencia y Mallorca, la España musulmana queda
reducida al Reino de Granada. De modo que hasta finales del siglo XV España queda dividi-
da en cuatro reinos cristianos: Portugal, Castilla, Navarra y la Corona Aragonesa; y un
reino musulmán: Granada.
Al final de todo este proceso, España volverá, con los Reyes Católicos, a su unidad cristia-
na aunque con una importante minoría musulmana, los moriscos, y, para siempre, con un tras-
cendentalísimo acervo cultural heredado de la cultura islámica.

3.3.2. LA ESPAÑA MUSULMANA


En el año 711 el Estrecho fue testigo del paso de un grupo de gentes constituido por una
minoría árabe y una mayoría beréber, cuyo objetivo, inicialmente, era el de ayudar a los parti-
darios del hijo del rey Vitiza, Akhila, en su pretensión de ocupar el trono visigodo para el que
había sido elegido por los nobles Rodrigo. La victoria en la Batalla de Guadalete fue, a la larga,
sólo el primer paso de un proceso más largo y complejo, un proceso que supondrá la caída de
la estructura política del reino visigodo y abrirá en la Historia de España un capítulo de ocho
siglos en el que su desarrollo se redactará con dos escrituras: deberemos, a partir de ese momen-
to, hablar de una España musulmana y de una España cristiana, sin que eso suponga concep-
tuarlas como dos Historias totalmente aisladas.
Tras el triunfo de Guadalete y la posterior derrota en la Batalla de Poitiers, el poder musul-
mán se centró en España siguiendo una evolución política que podemos resumir del siguiente
modo:
– Fase de conquista. España es administrada por un jefe militar (valí)
– El Emirato Dependiente (hasta 756)
– El Emirato Independiente (756-912)
– El Califato (912-1031), proclamado por Abd-Al-Rahmán III, con capital en
Córdoba
– Los Reinos de Taifas. Consecuencia de la desintegración del poder califal
– La unificación almorávide (1086-1145), llamados tras la conquista cristiana
de Toledo
– Nuevos Reinos de Taifas (1145-1172)
– Invasión almohade (1172-1212)
– Nuevos Reinos de Taifas. Tras la derrota de los almohades en Las Navas de
Tolosa. Después, sólo quedará el Reino de Granada

47
-Pág.51-
CIENCIAS SOCIALES (GEOGRAFÍA E HISTORIA)

Economía.- Se producen avances considerables en la explotación agrícola, introduciéndose


ampliaciones considerables del regadío y nuevos productos como los agrios, arroz, caña de azú-
car, algodón... y se intensifica el cultivo del olivo, destinando parte de la producción de aceite
al comercio exterior, lino, esparto... Continúa la explotación de minas de oro, plata, cobre, hie-
rro y mercurio, continuando vigentes los sistemas romanos de producción. Los principales pro-
ductos serán los textiles (sedas, brocados), junto con los curtido, cerámica, vidrio, armas, orfe-
brería, papel...

Población y sociedad.- Desde el principio se manifestó la diversidad racial en la composi-


ción de la población de la España musulmana. Sobre la base de la población autóctona, se super-
pusieron los invasores, árabes y beréberes y a ellos se añadieron otras etnias que fueron llegan-
do a la Península de forma sucesiva conformando el entramado social de la España musulmana
en el que se contaban árabes (clase superior y dominante en algunas zonas), beréberes (de arrai-
go rural, dominantes en otras zonas), eslavos o sagaliba, (grupos de diversa procedencia pre-
dominantes en algunos reinos), muladíes (indígenas convertidos al islam que ocupaban niveles
sociales modestos, mozárabes (cristianos que manteniendo su fe, viven en territorio musulmán),
judíos (presentes en diversas zonas).

Cultura.- Es innegable el extraordinario nivel cultural que alcanzó la España musulmana en


determinadas etapas y en determinadas disciplinas científicas y artísticas. Cabe destacar figuras
como las de los filósofos Averroes y Avempace, el poeta, cultivador de muwashahas y zéjel, Ibn
Quzman. Por otro lado la actividad constructiva y decorativa fue impresionante, estando rega-
das las tierras de España de numerosas muestras de ello. Por citar sólo algunos casos mencio-
nemos los de Granada (Generalife, Alcazaba de la Alhambra), la mezquita de Córdoba, la Torre
del Oro o la Giralda en Sevilla.

48
-Pág.52-
U . D . 4 . - S OCIEDADES HISTÓRICAS

4. LAS SOCIEDADES DE LA ÉPOCA MODERNA

4.1. LAS SOCIEDADES DE ANTIGUO RÉGIMEN EN EUROPA

4.1.1. LA CULTURA RENACENTISTA (SIGLO XVI)

4.1.1.1. Crisis político - religiosa


El Renacimiento supone un cambio diametral respecto a la Edad Media. Sus raíces parte del
siglo XIII y su apogeo se produce en los siglos XV y XVI. Supone una nueva concepción del
mundo, el hombre y las cosas que se gesta en la Baja Edad Media y conduce a Europa hacia una
nueva dimensión social, política económica y cultural: Europa entra en la Edad Moderna.
Política y socialmente se ponen las bases que con el tiempo darán paso a una nueva con-
cepción del Estado y del papel del individuo en el mismo. Los avances técnicos y científicos,
los descubrimientos geográficos, los cambios sociales y políticos y el desarrollo económico se
interrelacionan, cambian la visión del Mundo y encaminan la historia hacia la universalidad.

4.1.1.2. Cambio económico


A partir de la reactivación comercial posterior a las Cruzadas se va reafirmando la econo-
mía monetaria. La burguesía alcanza niveles importantes de riqueza y se convierte en el grupo
social rector de la economía y de la administración de las ciudades.
Surgen entidades de ahorro-préstamo y empresas mercantiles que crecen tras los descubri-
mientos geográficos. Se desarrolla el mercantilismo, incluso entrando en el juego las propias
monarquías: derechos de aduana, acuñación de moneda..., y se procura la obtención de mono-
polios y el control político por medio del dominio económico.
La agricultura se transforma, tanto en cuanto a la producción, favoreciéndose la especiali-
zación en función de los intereses comerciales, cuanto a la estructura social, decayendo la rela-
ción feudal en favor de los arrendamientos y del trabajo asalariado.
El nivel económico crece en importancia en cuanto a ser el factor determinante del nivel
social, generándose un sistema mixto estamental-clasista. Aumenta la importancia de la bur-
guesía, surge el proletariado urbano y rural y se generan ocasionales crisis sociales.

49
-Pág.53-
CIENCIAS SOCIALES (GEOGRAFÍA E HISTORIA)

4.1.1.3. La cultura renacentista.


Culturalmente, el Renacimiento supone la vuelta a la antigüedad clásica y la revisión críti-
ca del mundo medieval.El cambio de mentalidad dará lugar a una eclosión de la cultura y a la
aparición de nuevos conceptos:
– Individualismo: Afirmación del hombre como protagonista de la historia capacitado
para transformar el mundo, en contraste con el fatalismo medieval.
– Humanismo: Estudio de las ciencias humanas (lenguas clásicas, filosofía, historia...).
– Neoplatonismo: Frente al aristotelismo escolástico (tendencia al paganismo por afir-
mación del papel del hombre en el mundo).
Los humanistas cultivarán todas las ramas del saber bajo la protección de mecenas. Entre
dichos humanistas podemos citar a Dante Alighieri, Bocaccio, Petrarca, Baltasar de Castiglio-
ne, Erasmo de Rotterdam o Juan Luis Vives. Esos estudios de las diferentes ramas del saber per-
mitirán el avance en el conocimiento del cosmos, del hombre y de la técnica, produciéndose
importantes descubrimientos en terrenos como
– Astronomía: Sistema heliocéntrico, esfericidad y rotación de la Tierra (Copérnico,
Kepler, Galileo).
– Medicina: Fundamentos físico-químicos de la vida (Paracelso), circulación sanguí-
nea (Servet).
– Técnica: Instrumentos de navegación (brújula, sextante); imprenta de tipos móviles
(Güttemberg, 1445).
El ideal de vida personal y social se alteran, tal como queda reflejado en “El Cortesano”, de
Baltasar de Castiglione, y en “El Príncipe” de Nicolás Maquiavelo. El individuo instruido,
culto, dueño de sí mismo, elegante, sereno, se siente independiente y autosuficiente. De este
modo el cambio de conceptos políticos lleva también a la idea de individualidad, frente a la uni-
versalidad medieval, aparece un nacionalismo incipiente con la idea de un Estado fuerte inde-
pendiente de la Iglesia.

4.1.1.4. El Arte del Renacimiento


Todas las ramas artísticas eclosionan bajo el influjo de las nuevas ideas. La Literatura
encuentra manos y mentes tan trascendentes como las de Rabelais, Ronsard, Tomás Moro,
William Shakespeare, Garcilaso de la Vega... La Música da el paso de la música coral polifóni-
ca a la música instrumental (Palestrina).
Por su lado, Arquitectura, Escultura y Pintura ven proliferar sus manifestaciones, que pode-
mos resumir en el siguiente cuadro:

50
-Pág.54-
U . D . 4 . - S OCIEDADES HISTÓRICAS

ARQUITECTURA
– Vuelta al ideal de las formas clásicas.
– Combinación de elementos clásicos para nuevas formas
(columnas, frontones, pilastras, cúpula y bóveda de cañón)
– Armonía numérica entre las partes constructivas
– Evoluciona, a través del manierismo (s. XVI) y mediante la idea de movimiento y
efectos de claroscuro, hacia las formas barrocas.
Bramante, Miguel Ángel, Vignola, Bernini, Paladio, Sansovino, San Pedro del
Vaticano
ESCULTURA
– Redescubrimiento de la figura humana
– Realismo, naturalismo, clasicismo.
– Temas religiosos y paganos con nueva perspectiva.
– Búsqueda de la expresión del movimiento.
Benvenuto Cellini, Juan de Bolonia, Alonso Berruguete. Miguel Ángel (Moisés,
David, Piedad). Ghiberti, Donatello (S. Jorge, David).
PINTURA
– Realismo, sentido del volumen, perspectiva, sensación de espacio interpuesto,
ambientación, colorido.
Italia
Fra Angelico
Piero della Francesca
Botticelli
Leonardo da Vinci (Última Cena, Gioconda, Virgen de las Rocas)
Miguel Ángel (Capilla Sixtina)
Rafael (Disputa del Santísimo Sacramento, Escuela de Atenas)
Giorgione (Concierto Campestre, Madonna de Castelfranco)
Tiziano (Carlos I en Mühlberg, Venus de Urbino)
Tintoretto
Veronés
España
El Greco (El caballero de la mano en el pecho, Entierro del conde de Orgaz)
Pedro Berruguete
Alemania
Alberto Durero,
Lucas Cranach,
Hans Holbein.
Países Bajos
Van Eyck, Van der Weyden, Bosco, Brueghel.

51
-Pág.55-
CIENCIAS SOCIALES (GEOGRAFÍA E HISTORIA)

4.1.1.5. Descubrimientos geográficos


Otro aspecto fundamental de la época del Renacimiento es la ruptura de los límites del esce-
nario europeo con la incorporación al mundo conocido por los europeos de nuevas tierras.
Tras las Cruzadas medievales y los viajes de Marco Polo, llegaron a Europa noticias de
aquellas lejanas tierras y productos orientales (seda, especias...). La conquista turca de Cons-
tantinopla cerró las vías terrestres para el comercio de estos productos. La búsqueda de nuevas
rutas y el afán renacentista por el conocimiento propiciaron el planteamiento de expediciones
que ampliaron los horizontes.
Portugal y España serán los protagonistas de estas primeras expediciones. Portugal busca-
rá la ruta hacia Oriente bordeando África; España lo hará por el Occidente topándose con todo
un continente.
Todo esto servirá de punto de inicio de la proyección exterior de la cultura europea, amplia-
rá el conocimiento del mundo, posibilitará el crecimiento económico (nuevos productos, meta-
les...), abrirá el contacto con nuevas culturas y pondrá las bases del colonialismo.

NOMBRES E HITOS DE LOS DESCUBRIMIENTOS GEOGRÁFICOS


– Enrique el Navegante.- Monarca portugués patrocinador de los primeros viajes
portugueses por la costa africana
– Bartolomé Díaz.- Dobla el Cabo de Buena Esperanza (1487)
– Vasco de Gama.- Abre ruta hacia la India (1498)
– Cristóbal Colón.- Tras el rechazo portugués a su proyecto, logró el patrocinio de
Castilla (Isabel la Católica) y el 12 de octubre de 1492 desembarca en América (Isla de
Guanahaní).
– Álvarez Cabral.- Conquista Brasil (1500)
– Juan de la Cosa.- Primer mapa (1500)
– Núñez de Balboa.- Descubrimiento del Pacífico (1513)
– Hernán Cortés.- Conquista de México (1519-1521)
– Magallanes-Elcano.- Vuelta al Mundo (1519-1522)
– Pizarro.- Conquista de Perú (1533)
– Orellana.- Paso Marañón-Amazonas (1542)

52
-Pág.56-
U . D . 4 . - S OCIEDADES HISTÓRICAS

4.1.2. LA RUPTURA DE LA UNIDAD RELIGIOSA


El humanismo renacentista (afirmación del hombre), la divulgación de la cultura, la tenden-
cia al paganismo y la corrupción de la Iglesia generan en determinados ambientes europeos
rechazo de la tutela ideológica de la Iglesia y del principio de autoridad absoluta del Papa, ade-
más de un dura crítica frente a la riqueza de la Iglesia y la actitud mundana del clero: vida rela-
jada, con prebendas y privilegios, alejados de su papel espiritual.
Estas circunstancias preparan el ambiente favorable a los cambios dentro de la Iglesia.
Martín Lutero, desde Alemania, será quien se encargue de dar el aldabonazo e inicie la llamada
Reforma Protestante.

PRINCIPIOS DOCTRINALES DE LA REFORMA PROTESTANTE


El justo se salvará por la Fe
El Papa no es infalible
La Biblia única fuente de Fe
Todos los cristianos son sacerdotes
Sacramentos válidos: Bautismo y Eucaristía
Negación del culto a la Virgen y los Santos

La reforma se difunde por Alemania, donde los príncipes la utilizan como arma frente al
poder del emperador (Carlos I), por Suiza (Calvino, Zwinglio), Países Bajos, Escocia, Francia.
En Inglaterra Enrique VIII protagoniza una particular reforma en la que la autoridad del Papa
es la principal víctima. Como respuesta, la Iglesia realiza algunas reformas y combate al pro-
testantismo intentando reafirmarse por medio del llamado humanismo cristiano. El Concilio de
Trento, inicialmente intento de evitar la ruptura, concluirá con la reafirmación del cisma al rati-
ficarse la Iglesia Católica en sus principios doctrinales

4.1.3. LA ÉPOCA DEL BARROCO (SIGLO XVII)


El Barroco supone una nueva sensibilidad, sobre todo ante las formas artísticas, que emana
del Renacimiento. Se inicia en Italia y tiene como bases el espíritu de la Contrarreforma y el
absolutismo monárquico. Supone el rompimiento del mundo estable renacentista, por la divi-
sión de la Iglesia y por la ruptura de la idea de un Universo armónico. Ante esta ruptura se pro-
duce una reacción en el ámbito sociopolítico, buscando una mayor reglamentación, aumentan-
do la autoridad real y centralizando la administración, y en el ámbito artístico, reflejando la idea
de ruptura, rompiendo las líneas espaciales y buscando nuevas expresiones y la conquista del
espacio. El arte pasará a jugar un papel propagandístico de la Contrarreforma.

53
-Pág.57-
CIENCIAS SOCIALES (GEOGRAFÍA E HISTORIA)

4.1.3.1. Ideas políticas y filosóficas


En la Filosofía destaca la búsqueda de métodos racionales para el conocimiento del mundo
poniéndose las bases del racionalismo. Sus pilares fueron:

FRANCIS BACON –––> Fundamentos del empirismo.


JOHN LOCKE –––––––> Sensaciones como fuente del conocimiento
RENÉ DESCARTES ––> Racionalismo, método deductivo, dualismo.
BARUCH SPINOZA ––> Monismo panteísta (Unión Dios-Naturaleza)
LEIBNITZ ––––––––––> Teoría de las Mónadas

La teoría del Estado busca el contrapunto de la idea de ruptura antes mencionada: Es nece-
saria la Soberanía absoluta del rey, la centralización administrativa y la minuciosa organiza-
ción del aparato del Estado. En definitiva se formulan los principios que rigen el Absolutismo
monárquico que preside la Edad Moderna. Y entre sus formuladores figuran Tomás Hobbes,
Jacques Bossuet, Melchor Cano o Francisco de Vitoria.

4.1.3.2. La cultura barroca


El del Barroco es el siglo de la reacción Católica ante la Reforma protestante. Se produ-
ce una reactivación del sentimiento religoso paralelo al avance de la ideología racionalista y,
consiguientemente, al alejamiento del misterio religioso y de la fe, que triunfará en los ambien-
tes aristocráticos ilustrados del siglo XVIII. En la Literatura se manifiesta el predominio de las
obras de carácter moralizador, ascético o satírico. En España es el llamado Siglo de Oro de las
Letras. El panorama literario europeo se puede resumir en el siguiente cuadro

España:
Novela picaresca ---> Mateo Alemán, Quevedo.
Poesía ---> Góngora (culteranismo), Quevedo (conceptismo)
Teatro ---> Lope de Vega, Tirso de Molina, Ruiz de Alarcón, Calderón de
la Barca.
Ascética ---> S. Juan de la Cruz, Sta. Teresa de Jesús
Francia:
Prosa ---> Descartes, Corneille, Pascal
Oratoria ---> Bossuet, Fenelon
Poesía ---> La Fontaine, Boileau
Teatro ---> Molière, Racine

54
-Pág.58-
U . D . 4 . - S OCIEDADES HISTÓRICAS

La música experimenta un importante desarrollo, tanto la instrumental como la vocal. Es la


época de Vivaldi, Purcell, Bach, Haendel, Haydn, Mozart.

4.1.3.2. El arte barroco

ARQUITECTURA
– Los edificios y el espacio urbano en el que se enmarcan se interrelacionan, los espacios
urbanos alcanzan protagonismo, se desarrolla el urbanismo. Los edificios adquieren
una nueva personalidad

Características:
– Interacción del edificio con el entorno. (Versalles, Plaza del Vaticano)
– Centralización de las plantas
– Juego de los espacios interiores
– Ruptura de las líneas longitudinales
– Efectos de claroscuro
– Sensación de movimiento
– Recargamiento decorativo

Autores:
Vignola (Il Gesù)
Carlo Maderna (fachada de San Pedro)
Borromini (S. Carlo de las Cuatro Fuentes),
Bernini (Sant'Ivo della Sapienza, Sant'Andrea al Quirinale)
Churriguera (fachada del Ayuntamiento de Salamanca)

ESCULTURA BARROCA
Características:
– Dramatismo.
– Búsqueda de la plasmación del movimiento.
– Búsqueda de la plasmación de los estados de ánimo.
– Efecto religioso propagandístico.
En España se desarrolla la Imaginería polícroma:
– Realismo.
– Unción religiosa.
– Fuerte expresividad.
– Efectos de claroscuro por el dominio de los pliegues en los paños.
Autores: Gregorio Fernández, Martínez Montañés, Alonso Cano, Salzillo

55
-Pág.59-
CIENCIAS SOCIALES (GEOGRAFÍA E HISTORIA)

PINTURA

Características:
– Realismo
– Efectos de perspectiva
– Efectos de luces y sombras: tenebrismo.
– Temas religiosos y mitológicos

Autores:
– Caravaggio (Martirio de S. Pedro, Vocación de S. Mateo, La buenaventura)
– Rubens (El descendimiento, El juicio de Paris, Las tres gracias)-Rembrandt (La ronda
de noche, Sindico de los pañeros)
– Velázquez (Las meninas, El aguador de Sevilla, El triunfo de Baco o Los borrachos,
Retrato de Felipe IV, La fragua de Vulcano, Retrato del Conde-duque de Olivares)
– Ribera (San Andrés, La Inmaculada)
– Zurbarán (serie sobre la Virgen niña, serie sobre cartujos)
– Murillo (Inmaculadas, Visión de S. Antonio de Padua)
– Georges la Tour
– Van Dyck

4.1.4. LA ÉPOCA DEL RACIONALISMO. LA ILUSTRACIÓN (SIGLO XVIII)


El siglo XVIII es, desde el punto de vista filosófico, político, social y económico, la última
consecuencia de los cambios de mentalidad iniciados en el Renacimiento, ampliados en el
Barroco y culminados con la crisis religiosa de la Reforma. La Ilustración moverá las estructu-
ras políticas propiciando el cambio de orientación del absolutismo monárquico barroco
hacia el despotismo ilustrado y preparando las bases ideológicas que favorecerán la aparición
del Estado Liberal. La difusión de la cultura, el enciclopedismo, jugará un papel fundamental
en la construcción de las nuevas ideas, cuyo principio fundamental será la razón.

56
-Pág.60-
U . D . 4 . - S OCIEDADES HISTÓRICAS

4.1.4.1. Ideas filosóficas, políticas y económicas


La Ilustración, como conocemos al movimiento filosófico, tiene sus raíces en el humanismo
renacentista, pasando por el racionalismo barroco (Descartes, Spinoza, Leibnitz) y por el auge
de las ciencias, la investigación y la técnica. La Razón domina la naturaleza y es la fuerza trans-
formadora de la realidad (Voltaire, Kant).
Sus objetivos, métodos y vías podemos esquematizarlos así:

PRINCIPIOS MÉTODOS FACILITAN OBJETIVOS


DIFUSIÓN
RAZÓN La razón Núcleos urbanos Progreso
La crítica Academias Felicidad
La libertad Universidades Paz eterna
La tolerancia religiosa Masonería Fraternidad
La educación racional
- humanista

En principio las ideas ilustradas son utilizadas como justificación del poder absoluto de los
monarcas lo que edifica el llamado Despotismo Ilustrado. Sobre la idea del poder divino como
justificación del Estado y del poder absoluto del monarca, se impone la idea del Estado artifi-
cial, creado por el hombre y entregado, por medio de un contrato revocable, al Soberano. Éste
debe perseguir la felicidad del pueblo (“todo para el pueblo pero sin el pueblo”). El Despotis-
mo Ilustrado se dibujará como sistema basado en la razón, que, consecuentemente, procurará la
racionalización de la administración y la organización del Estado huyendo de los particula-
rismos. Esto conduce a la edificación del Estado sobre la centralización administrativa y una
burocracia modernizada.
Mientras el Despotismo Ilustrado se consolida, las mismas ideas de base que lo consolidan
van evolucionando hacia postulados más avanzados:
Así Montesquieu (“El espíritu de las leyes”), diseña la idea de la Monarquía constitucional,
con la división de poderes:

PODER EJECUTIVO El Rey


PODER LEGISLATIVO El pueblo a través de
representantes
PODER JUDICIAL Jueces independientes

57
-Pág.61-
CIENCIAS SOCIALES (GEOGRAFÍA E HISTORIA)

Rousseau aporta la idea de la Soberanía nacional, según la cual el pueblo es el depositario


del poder, los gobernantes deben ser meros funcionarios suyos.
Desde el punto de vista de la economía, las nuevas ideas conducen hacia el liberalismo. La
economía debe ser un ordenamiento natural no controlado por el Estado. Se debe recono-
cer la propiedad privada y la iniciativa privada con una libre concurrencia en el comercio. Adam
Smith, fijaba como origen de toda riqueza el trabajo, sobre el cual afirmaba la necesidad de una
división racional. La ley de la oferta y la demanda y la libre competencia debía presidir las rela-
ciones económicas.
Por lo que respecta a la sociedad, por derivación necesaria de los principios antes citados,
debía organizarse de acuerdo con la riqueza que cada individuo fuera capaz de lograr, una
estructura social, pues, basada en las clases, niveles abiertos a los que se adscribirían los indi-
viduos en función de sus rentas.

4.1.4.2. Los avances científicos


El racionalismo imperante favorecerá el progreso del conocimiento, la investigación y, con-
secuentemente, los avances técnicos. Las Matemáticas fueron disciplina casi de culto. Las Cien-
cias Naturales (Buffon, Linneo) alcanzaron gran difusión, basándose esencialmente en el afán
de clasificación de especies. Las investigaciones sobre Física (Fahrenheit, Celsius, Franklin)
llevaron a la aparición de innovaciones técnicas (Watt y su máquina de vapor). También hubo
importantes avances en Química (Lavoisier), Astronomía (Laplace), Medicina (vacunas, medi-
das higiénicas)...Todo el saber de la época fue recopilado en la famosa Enciclopedia (L'enci-
clopedie française), obra realizada por Diderot y d'Alambert, fundamentalmente.

4.2. HEGEMONÍA Y DECADENCIA DE LA MONARQUÍA


HISPÁNICA

4.2.1. LA HEGEMONÍA ESPAÑOLA (SIGLOS XVI Y XVII)


Para España el Renacimiento supone el inicio de la etapa más brillante de su historia al con-
vertirse en la primera potencia mundial. La salida de la Edad Media se produce con la unifica-
ción dinástica de los dos principales reinos peninsulares, Castilla y Aragón, gracias al matri-
monio de sus reyes Isabel I de Castilla y Fernando II de Aragón, la conquista de Granada, la
incorporación de Navarra y el inicio de la expansión exterior americana.
Carlos I, nacido en Gante, en los Países Bajos, posesión de su padre Felipe de Habsburgo,
recibirá en su persona la herencia de los reinos de Castilla y Aragón, uniendo definitivamente
ambas coronas. Además, recibe las posesiones americanas, en continuo proceso de ampliación,
y las mediterráneas aragonesas (Cerdeña, Sicilia, Nápoles). De su padre hereda los Países Bajos
y el derecho a presentarse como candidato a Emperador del Sacro Imperio Romano Germánico.

58
-Pág.62-
U . D . 4 . - S OCIEDADES HISTÓRICAS

Precisamente, sus pretensiones imperiales motivaron indirectamente uno de los más significati-
vos conflictos de su reinado: el movimiento de los comuneros en Castilla; y su posterior elec-
ción como Carlos V, amén de ampliar el horizonte de sus posesiones, le hizo protagonista del
más significativo de tales conflictos: el cisma de la Iglesia tras la reforma protestante.
Felipe II, salvo el Imperio, recibió todas las posesiones de su padre Carlos. Hizo valer el
predominio español forzando a Francia a reconocer su hegemonía. Tuvo que hacer frente al
empuje de los turcos, frente a los cuales obtuvo la importante victoria de Lepanto (1571). Se
enfrentó con éxito inicial a la sublevación de los Países Bajos con Guillermo de Orange a la
cabeza, aunque no terminó de controlar nunca del todo la situación a pesar de la intervención
de personajes tan significativos como el duque de Alba, Alejandro Farnesio, Juan de Austria o
Luis de Requesens. Además, Felipe II incorporó a su corona la de Portugal que, desde 1580,
se mantendrá unido a España hasta 1640. Por otro lado la tradicional rivalidad con Francia
tuvo también tintes de guerra de religión al combatir en la Guerra de los Tres Enriques en favor
del católico Enrique de Guisa frente a los hugonotes. Y en el enfrentamiento con Inglaterra,
también con tintes de guerra de religión por la reforma anglicana, tendrá lugar el desastre de la
llamada Armada Invencible.
Con Felipe III, la crisis económica sirvió de anuncio al paulatino declinar del predominio
español. Instauró un sistema de administración en el que el rey dejó las labores de gobierno en
manos de un valido, que asumió todo el poder mientras el rey se dedicaba a la cómoda vida
palaciega. El duque de Lerma y el duque de Uceda fueron sus validos; entre ambos conduje-
ron a la economía española a la bancarrota. Hecho importante de su reinado fue, además, la
expulsión de los moriscos, lo cual aumentó los problemas económicos para el Estado.
Felipe IV continuó con el sistema de validos. El suyo, y personaje trascendental en la his-
toria española, fue el conde-duque de Olivares quien intentó sin pleno éxito revitalizar la eco-
nomía española.
Con Carlos II, aunque ya de lleno en la etapa barroca, finalizará la etapa del predominio
español, sobre todo cuando, muerto sin descendencia, se entable la Guerra de Sucesión que con-
sagrará el descenso a un segundo plano de la política española frente a la inglesa y la francesa.

4.2.2. LA DECADENCIA DEL PREDOMINIO ESPAÑOL.


LOS PRIMEROS BORBONES (SIGLO XVIII)
En España el nuevo orden, desde el absolutismo al despotismo ilustrado, vendrá de la mano
del cambio dinástico. Carlos II muere en 1700 habiendo dejado como heredero testamentario
a Felipe d'Anjou, segundo hijo de Luis XIV de Francia y su hermanastra María Teresa (hija
de Felipe IV y su primera esposa Isabel de Borbón), que reinará como Felipe V.

59
-Pág.63-
CIENCIAS SOCIALES (GEOGRAFÍA E HISTORIA)

Su acceso al trono fue acompañado del inicio de una Guerra de Sucesión. La perspectiva
de una vinculación dinástica entre España y Francia despertó los temores de las demás poten-
cias. Ante la situación se va a formar una Gran Alianza integrada por el Imperio, Inglaterra
y Holanda y partidaria de la candidatura del Archiduque Carlos de Austria. La Alianza decla-
ra la guerra en Septiembre de 1701. Poco después se les une Portugal. También en el interior
se formaron bandos de apoyo a uno u otro de los pretendientes. Tras 10 años de guerra las poten-
cias enfrentadas comienzan a negociar en 1711. Fruto de las negociaciones son los Tratados de
Utrecht-Rastadt, con importantes cesiones por parte de España, que pierde su papel hege-
mónico en beneficio de Francia.
Desde el principio la Corte española se impregna de influencias francesas ante la debilidad
de los Ministros españoles. Estas influencias tendrán como consecuencia inmediata la reorga-
nización del Estado bajo el prisma del Racionalismo centralista lo que supondrá, por medio
de los Decretos de Nueva Planta:
– Anulación de la autonomía municipal.
– Abolición de los fueros de la Corona de Aragón.
– Creación de las Secretarías de Estado y de Despacho.
Cada Secretaría tenía a su frente un Secretario nombrado por el Rey. Los antiguos Consejos
perdieron poder y sólo el Consejo de Castilla se mantuvo como un auténtico Ministerio del
Interior.
Fernando VI (1746-1759), dada la situación del final del reinado anterior, optó por seguir
los consejos del ministro de su padre, Patiño, y buscar el aislamiento y el pacifismo por la vía
de la neutralidad.
El reinado de Carlos III (1759-1788), hermano de Fernando VI, supone para España la
plenitud del Despotismo Ilustrado. Continúa la política reformista iniciada en el reinado ante-
rior. Es la etapa de las grandes reformas impulsadas por el propio monarca y sus ministros
Campomanes y Floridablanca.
La Revolución francesa presidió los momentos iniciales del reinado de Carlos IV (1788-
1808), causando una crisis ideológica entre los ilustrados partidarios de la Revolución y los
opositores. Por otro lado, el sistema de alianzas entra en crisis, lo que afecta esencialmente a
España que se ve sola ante su tradicional enemigo Inglaterra. El rey nombró al favorito de la
reina María Luisa, Manuel Godoy, que pasa por ser el gobernante más representativo del rei-
nado. Ante los hechos en Francia, Godoy favoreció una alianza con Inglaterra. Tras el fraca-
so de la misma (desembarco fallido en Tolon), Godoy se decidió a pactar con la nueva Francia
y después de la Paz de Basilea (1795) firmó el Tratado de San Ildefonso (1796), que llevó a
España, al subir al poder en Francia Napoleón, a la guerra con Inglaterra y al sacrificio de la
armada en Trafalgar. Posteriormente se firmó el Tratado de Fontainebleau en el que se esta-
blecía el reparto de Portugal entre España y Francia y que sirvió de excusa para la invasión
francesa de nuestro territorio.

60
-Pág.64-
U . D . 4 . - S OCIEDADES HISTÓRICAS

La Ilustración en España.- Cabe destacar la creación de las Sociedades Económicas de


Amigos del País impulsadas por Campomanes. La preocupación por los asuntos económi-
cos y el reformismo propio de los ilustrados dio frutos como el Informe de la Ley agraria
de Jovellanos. Hubo reformas de la enseñanza, considerada ya como un servicio público. Se
impulsaron las obras públicas (carreteras, pantanos, canales, planes urbanísticos) y la repobla-
ción (Sierra Morena con colonos suizos y alemanes). Y se prestó especial atención a las insti-
tuciones culturales: Librería Real, base de la Biblioteca Nacional, fundada por Felipe V, Reales
Academias de la Lengua, Medicina, Historia, etc.

5. SOCIEDADES DEL ÁMBITO NO EUROPEO DURANTE


LA EDAD MEDIA Y MODERNA

5.1. SOCIEDAD Y CULTURA ISLÁMICA DURANTE


LA EDAD MEDIA
Conviene dar una visión general de lo que fue el inicio de la expansión islámica y sus prin-
cipios doctrinales. Mahoma (=el glorificado), predica desde 610 la venida del juicio final y la
existencia del Dios único Alá, admitiendo las doctrinas de profetas anteriores a él (Noé,
Abraham, Moisés, Jesús). Los pilares de la fe islámica son:
1.- Profesión de fe: no hay más Dios que Alá y Mahoma es su profeta.
2.- Oración cinco veces al día en dirección a la ciudad santa de La Meca.
3.- Dar limosna.
4.- Ayunar durante los días del mes Ramadán.
5.- Peregrinar a la Meca al menos una vez en la vida.
Las fuentes doctrinales son El Corán, libro sagrado dictado por Mahoma, la sunna, o tradi-
ción de los hechos y dichos del profeta, y la Iyma (acuerdo unánime de los creyentes). La Yihad
o guerra santa fue incluida posteriormente como justificación a la expansión de la fe y vía segu-
ra de alcanzar el paraíso para quienes en ella murieran.

61
-Pág.65-
CIENCIAS SOCIALES (GEOGRAFÍA E HISTORIA)

Mahoma puso las bases doctrinales e inició un proceso cultural de gran trascendencia que
considera como su propio punto de partida la Hégira (huida de Mahoma de La Meca a Medina),
el 15 de junio de 622. Tras la muerte de Mahoma (632), se inicia la expansión territorial fuera
de Arabia por los sucesores de Mahoma, extendiéndose a lo largo del siglo VII por el Próximo
Oriente, Afganistán, e incluso India, y, por el occidente, por el norte de África. Las principales
etapas que se abren desde este momento son:
– Época de los Califas Electivos (Omar, Otmán y Alí) (632-661)
– Dinastía Omeya (661-750)
– Dinastía Abbasida (750-1258). Desde el s. IX se producen sucesivas independencias loca-
les (Al-Andalus, Magreb, Ifrikiya, Egipto…) de modo que el Califa queda como autori-
dad meramente religiosa. Ocasionalmente, grupos integristas intentan localmente recupe-
rar una autoridad política imperial (fatimíes, almorávides, almohades). Los territorios
orientales, además, caen primero en manos de los mongoles de Tamerlán, después se
forma el Imperio persa chiíta y posteriormente los turcos otomanos se hacen con el con-
trol de territorios cada vez más extensos convirtiéndose finalmente en los artífices de la
reconstrucción de un nuevo gran imperio islámico.
– Imperio Turco Otomano. Se va consolidando durante el siglo XIV y alcanza momentos
de esplendor entre 1413 y 1566. Momento en el que se sitúa la toma de Bizancio (1453)
y con ello la caída del Imperio Romano de Oriente o Bizantino. Es la época del avance en
los Balcanes con Solimán II el Magnífico, después del cual se inicia una crisis que tendrá
como punto culminante la derrota de Lepanto (1571)

5.2. LAS CULTURAS PRECOLOMBINAS


Desde 15.000 a.C. se producen sucesivas oleadas migratorias hacia América desde Asia a
través del estrecho de Béring y se va produciendo el avance hacia el sur.
Sobre la base de culturas Neolíticas se inicia el Periodo Arcaico que durará hasta el año
100. A partir de esa fecha se desarrolla el llamado Periodo Clásico en el que, en determinadas
zonas empezarán a formarse grandes ciudades-estado y terminarán por surgir grandes imperios.
Las zonas de mayor avance cultural y civilización son Mesoamérica y la parte central andina.
• Mesoamérica
Se suceden culturas importantes como Olmecas (200-500), Toltecas (Hacia 800),
Mixtecas, Zapotecas.
Aztecas y Mayas serán los pueblos más avanzados con los que se encontrarán los des-
cubridores españoles.
Los Aztecas, tras la fundación de su capital Tenochtitlán (hacia 1325), iniciarán su fase
de apogeo con monarcas absolutos como ITZCOATL y MOCTEZUMA II, quien cayó
derrotado ante HERNÁN CORTÉS.

62
-Pág.66-
U . D . 4 . - S OCIEDADES HISTÓRICAS

Los Mayas conocieron un desarrollo cultural importante (Calendario, cosmogonía...)


desde el Imperio Antiguo (s. IV a VII) al Nuevo Imperio (s. XV). Su capital: Chichén-
Itzá.
• América del Sur
Con el precedente de culturas como Chavin, Tiahuanaco, Recuay, Nazca y Chimú, se
forja el reino Inca de los Quechúa, cuya capital Cuzco fue fundada por Manco Capac.
Desde 1438 se inicia la expansión del Imperio Inca con Pachacutic y continuará con
incas como Tupac Yupanqui y Huayna Capac hasta que Atahualpa fuera apresado y
muerto por Francisco Pizarro en 1532.

RESUMEN
La historia es una combinación de hechos protagonizados por individuos y colectividades,
anónimos o personificados, que se desarrollan a lo largo del tiempo. La Historia, entendida
como ciencia, ha tenido su propia historia. Los textos más antiguos que podemos calificar como
Historia fueron los anales plasmados en piedra o arcilla puestos al servicio de los intereses de
los monarcas mesopotámicos o egipcios. Como para tantos otros aspectos de la civilización,
Grecia aportó una concepción de la Historia como ciencia y, también, como en otros aspectos,
surgieron los primeros historiadores.

El historiador realiza su labor, esencialmente, a través de los siguientes pasos:

A) Documentación: Con búsqueda de datos o antecedentes, cuyas fuentes pueden ser


mediante testimonios escritos, tradición (relatos populares o testimonios orales), histo-
riografía y la arqueología (restos materiales de otras culturas, como los hallados en Ata-
puerca, hoy considerados como “patrimonio de la humanidad”).

B) Análisis: Mediante un proceso de sistematización y estudio de los datos recopilados.

C) Planteamiento de las tesis sobre las causas y efectos de los hechos estudiados.

D) Síntesis o exposición de los resultados del estudio.

Recuerda que:

– El término Paleolítico (Paleos=antiguo; lithos=piedra) se aplica a la etapa más extensa de


la historia de la Humanidad, y alude a la tipología de las herramientas conservadas.

– El término Neolítico (Neo=nuevo), con nuevas formas de trabajo de la piedra, nuevos úti-
les y nuevos materiales, como la cerámica.

63
-Pág.67-
CIENCIAS SOCIALES (GEOGRAFÍA E HISTORIA)

– Las llamadas “primeras civilizaciones» se producen en una serie de zonas con evolucio-
nes culturales diferentes. Podemos citar las más importantes: El Imperio Egipcio (3110-
518 a.C.); Sumerios (3500-2347 a.C.); Imperio Acadio (2347-2230 a.C.); Imperio Babi-
lónico (1792-1450 a.C.); Imperio Asirio (1450-625 a.C); Imperio Persa (539-332 a.C);
Dinastía Ptolemáica (332-30 a.C.); Civilización Griega (3300 a.C.-86 d.C.); Imperio
Romano (750 a.C.-476 d.C.).

– Los llamados pueblos bárbaros, que vivían en la Europa Central y Nororiental desde el
siglo V a.C., con la caída de Roma van entrando en unas zonas y otras y van consolidan-
do su poder (Suevos, Alanos, Vándalos, Francos, Alamanes, Godos, Visigodos, Ostrogo-
dos, etc.).

– Con las sociedades medievales, el campesino se pone bajo la protección del señor a cam-
bio de una aportación en especie. El feudalismo define esta forma de vida de señor-vasa-
llo. La pirámide social está encabezada por la nobleza; después están las clases libres
rurales y urbanas, y, por último, los siervos y esclavos. La monarquía se caracteriza por
su poder absoluto y la elegibilidad de los reyes. Entre los siglos siete y trece se produce
la invasión musulmana dejando profundas raíces en nuestra cultura. España volverá a su
unidad cristiana poco a poco y con la definitiva intervención de los Reyes Católicos.

– La sociedad moderna, entre los siglos XIII y XVI, supone una nueva concepción del
mundo, el hombre y las cosas, que se gesta en la Baja Edad Media y conduce a Europa
hacia una nueva dimensión social, política, económica y cultural (Renacimiento).

– La época del Barroco (siglo XVII) supone una nueva sensibilidad, sobre todo por las for-
mas artísticas, que emanan del Renacimiento.

– La época del Racionalismo y el movimiento de la Ilustración (siglo XVIII) propiciará


el cambio de orientación del absolutismo monárquico barroco hacia el despotismo ilus-
trado, y prepara las bases ideológicas que favorecerán la aparición del Estado Liberal.
Estos cambios vienen de la mano, también, del cambio dinástico con los reinados de los
primeros borbones.

64
-Pág.68-
U . D . 4 . - S OCIEDADES HISTÓRICAS

EJERCICIOS DE AUTOCOMPROBACIÓN

1. ¿Cuál de las siguientes es colonia fenicia?


A. Ibiza B. Carthago Nova C. Ampurias D. Cádiz

2. ¿Cuál de las siguientes es colonia griega?


A. Ibiza B. Carthago Nova C. Ampurias D. Cádiz

3. Indique en cuál de las siguientes respuestas se sigue un orden cronológico.


A. Paleolítico Superior, Musteriense, Paleolítico Inferior
B. Achelense, Paleolítico Medio, Abbevillense, Musteriense
C. Musteriense, Auriñaciense, Solutrense, Magdaleniense
D. Paleolítico Inferior, Solutrense, Auriñaciense, Musteriense

4. El yacimiento en el que se han encontrado los restos del llamado Homo antecessor es el de:
A. Trinil en Java B. Atapuerca C. Neander D. Heidelberg

5. La especie humana que se asocia al Paleolítico Medio es:


A. Australopithecus Afarensis B. Homo Neanderthalensis
C. Homo Erectus D. Homo Sapiens Sapiens (Cro-Magnon)

6. Señale cuál de las siguentes respuesta es correcta.


A. El Neolítico supone un cambio radical en las costumbres del ser humano que deja de ser un
productor y pasa a ser un depredador
B. El final del Paleolítico se produce como consecuencia del cambio climático motivado por el
inicio de la glaciación Würm IV
C. El Neolítico hispano está asociado a una manifestación característica del Neolítico de la Euro-
pa Occidental: El culto al toro, representado en los Toros de Guisando
D. Uno de los elementos definitorios del Neolítico es la aparición de nuevos utensilios entre los
que destaca especialmente la cerámica

7. La cultura argárica se desarrolla:


A. En el sur de la Península durante el Bronce Final
B. En el mediodía peninsular durante el Bronce Medio
C. En el Centro de España durante el Neolítico
D. En el Levante durante el Epipaleolítico

65
-Pág.69-
CIENCIAS SOCIALES (GEOGRAFÍA E HISTORIA)

8. Uno de los siguientes términos, nombres o conceptos NO tienen relación con Tartessos.
A. Control del paso del Estrecho B. Minas de bronce de Huelva
C. Ganadería D. Gerión

9. Una de las siguientes culturas NO corresponde a la Edad del Bronce.


A. Cultura de Los Millares B. Cultura de los Campos de Túmulos
C. Cultura del Argar D. Cultura del Vaso Campaniforme

10. Una de las principales aportaciones de los pueblos indoeuropeos fue:


A. El Megalitismo B. La metalurgia del hierro
C. Los idiomas semíticos D. El uso del arco

11. ¿Cuál de los siguientes pueblos prerromanos podemos considerarlo heredero o descendiente de los
tartesios?
A. Iberos B. Oretanos C. Turdetanos D. Lusitanos

12. Entre el armamento característico de los iberos se encuentra:


A. El hacha bipenne B. La falcata
C. Un escudo convexo D. La espada larga

13. El sistema de escritura utilizado por los iberos podemos definirlo como:
A. Un código lineal con caracteres ideográficos
B. Un sistema alfabético sin signos vocálicos
C. Un sistema cuneiforme de influencia mesopotámica
D. Un sistema mixto alfabético y silábico

14. Entre las manifestaciones más importantes de la estatuaria de los iberos NO se encuentra:
A. La Bicha de Balazote B. La Dama de Elche
C. Los Toros de Guisando D. La Dama de Baza

15. El fenómeno denominado “bandolerismo” aparece en uno de los pueblos prerromanos de la Penín-
sula. ¿Cuál?
A. Iberos B. Lusitanos C. Astures D. Cántabros

66
-Pág.70-
U . D . 4 . - S OCIEDADES HISTÓRICAS

16. Uno de los aspectos característicos de la sociedad de los pueblos prerromanos del norte de España es:
A. Los sacrificios humanos B. La organización matriarcal
C. La poligamia D. La Hospitalidad

17. Una de las siguientes repuestas referida a los magistrados romanos es correcta. ¿Cuál?
A. Los cónsules eran elegidos por los comicios curiatos
B. Estaban sometidos al llamado “cursus internum”
C. Los cuestores se ocupaban de la policía y eran los más importantes
D. Se elegían 12 pretores tribunicios

18. El arte romano tiene un carácter esencialmente:


A. Civil y práctico B. Religioso C. Estético y sencillo D. Lujoso

19. La cultura egipcia se caracteriza, entre otros aspectos por:


A. La existencia de un poder político representado por el rey-sacerdote o Lugal
B. El elaborado culto relacionado con la muerte
C. La base industrial de su economía
D. La sucesión de diferentes culturas o pueblos dominadores del Valle del Nilo

20. La figura que protagonizó de un modo especial la predicación del cristianismo y su diseño dogmá-
tico en los primeros años de su expansión tras la muerte de Cristo fue:
A. Jesús de Nazaret B. San Pedro C. San Pablo D. Santiago el Mayor

21. La escritura característica de la cultura egipcia es la:


A. Latina B. Jeroglífica C. Cuneiforme D. Alfabética

22. Señale la afirmación correcta.


A. Una de las más importantes aportaciones de la cultura griega a la llamada cultura occidental es
el Derecho
B. La base de la organización política, social y económica de la Grecia clásica era la ciudad
C. Entre los filósofos griegos de mayor trascendencia se encuentran Columela y Séneca
D. Los griegos mantuvieron siempre una estrecha vinculación política, formando una entidad unida

67
-Pág.71-
CIENCIAS SOCIALES (GEOGRAFÍA E HISTORIA)

23. Señale en cuál de las siguientes respuestas los acontecimientos citados aparacen en orden cronoló-
gico.
A. Imperio Medio Egipcio, Imperio Acadio, Época helenística, Fundación de Roma
B. Cultura tartésica, I Guerra Púnica, I Guerra Médica, Invasión de los Hyksos
C. Imperio Medio Egipcio, Imperio Neobabilónico, II Guerra Púnica, imperio de Augusto
D. Inicio de la conquista romana de España, fundación de Cádiz, Imperio sumerio, Imperio Asirio

24. La conquista romana de España se inicio con el desembarco de las legiones romanas en:
A. Cádiz B. Ampurias C. Sagunto D. Cartagena

25. Señale cuál de las siguientes relaciones es correcta.


A. Menes - Fundación del Imperio Neobabilónico
B. Hammurabi - Primer faraón nubio
C. Guerra del Peloponeso - Roma contra Cartago
D. Octavio Augusto - Inicio del Imperio Romano

RESPUESTAS A LOS EJERCICIOS

1. D 2. C 3. C 4. B 5. B
6. D 7. B 8. B 9. B 10. B
11. C 12. B 13. D 14. C 15. B
16. B 17. A 18. A 19. B 20. C
21. B 22. B 23. C 24. B 25. D

68
-Pág.72-
U . D . 5 . - S OCIEDAD Y CAMBIO EN EL TIEMPO

ÍNDICE
OBJETIVOS . . . . . . . . . . . . . . . . . . . . . . . . . . . . . . . . . . . . . . . . . . . . . . . . . . . . . . . . . . . . . .2

INTRODUCCIÓN . . . . . . . . . . . . . . . . . . . . . . . . . . . . . . . . . . . . . . . . . . . . . . . . . . . . . . . . .3

MAPA CONCEPTUAL . . . . . . . . . . . . . . . . . . . . . . . . . . . . . . . . . . . . . . . . . . . . . . . . . . . . .4

DESARROLLO DE CONTENIDOS

1. EL TIEMPO HISTÓRICO . . . . . . . . . . . . . . . . . . . . . . . . . . . . . . . . . . . . . . . . . . . .5

2. EVOLUCIÓN Y DESARROLLO EN LA HISTORIA DEL ARTE . . . . . . . . . .20

3. CAMBIO SOCIAL Y REVOLUCIÓN EN LA ÉPOCA CONTEMPORÁNEA .25

RESUMEN . . . . . . . . . . . . . . . . . . . . . . . . . . . . . . . . . . . . . . . . . . . . . . . . . . . . . . . . . . . . .43

EJERCICIOS DE AUTOCOMPROBACIÓN . . . . . . . . . . . . . . . . . . . . . . . . . . . . . . . . . .45

RESPUESTAS A LOS EJERCICIOS . . . . . . . . . . . . . . . . . . . . . . . . . . . . . . . . . . . . . . . . .48

-Pág.73-
CIENCIAS SOCIALES (GEOGRAFÍA E HISTORIA)

OBJETIVOS
Al finalizar el estudio de esta Unidad Didáctica, el alumno será capaz de:

• Comprender lo convencional de la medida del tiempo y las circunstancias que


han llevado a la determinación de esas convenciones y las fórmulas de medida
actuales.

• Valorar y entender algunos de los procesos históricos significativos tales como


la economía, la vida social, las creencias religiosas y las formas de organización
política y social.

• Observar la evolución y desarrollo en la historia del arte mediante el estudio de


10 obras significativas de distintos periodos y estilos.

• Interpretar los cambios producidos en el mundo en los dos últimos siglos, desde
la crisis del Antiguo Régimen y el triunfo inicial del liberalismo hasta nuestros
días

• Distinguir los cambios producidos en los dos últimos siglos en la Historia de


España, desde el surgimiento del liberalismo durante la Guerra de la Indepen-
dencia hasta nuestros días.

-Pág.74-
U . D . 5 . - S OCIEDAD Y CAMBIO EN EL TIEMPO

INTRODUCCIÓN
N os enfrentamos en éste a un tema dual y, a la vez, unívoco. En el anterior, además
de la valoración de las fuentes y métodos históricos, estudiábamos la evolución de
la historia del ser humano desde sus orígenes hasta el siglo XVIII. Ahora, se trata de con-
tinuar el relato con los acontecimientos acaecidos en los dos últimos siglos. Pero, pre-
viamente, y dado que (aunque sólo sea por la perspectiva más próxima desde la que lo
observamos) esos dos siglos han sido tiempo de continuos cambios, analizamos el con-
cepto de tiempo histórico y las convenciones utilizadas en su medida y reflexionamos
sobre diversas cuestiones que han protagonizado de alguna forma la evolución de la
humanidad y que en sí mismas tienen su propia historia de cambios.

-Pág.75-
CIENCIAS SOCIALES (GEOGRAFÍA E HISTORIA)

M A PA C O N C E P T U A L

SOCIEDAD Y
CAMBIO EN EL TIEMPO

EVOLUCIÓN Y CAMBIO SOCIAL Y


EL TIEMPO HISTORICO DESARROLLO EN LA REVOLUCIÓN EN LA
HISTORIA DEL ARTE ÉPOCA CONTEMPORÁNEA

La crisis del Antiguo


Régimen y las revoluciones
liberales

Revolución Industrial,
desarrollo capitalista e
imperialismo

Las grandes
transformaciones y conflictos
del siglo XX

Cambios y
transformaciones en la España
del siglo XX

-Pág.76-
U . D . 5 . - S OCIEDAD Y CAMBIO EN EL TIEMPO

1. EL TIEMPO HISTÓRICO

1.1. UNIDADES Y CONVENCIONES DE LA CRONOLOGÍA

1.1.1. LA MEDIDA DEL TIEMPO


Medir el tiempo ha sido una obsesión del ser humano desde que tomó conciencia de su pro-
pia existencia. El tiempo, esa inaprehensible dimensión en la que nos movemos, marca el desa-
rrollo de la vida de un ser humano considerado individualmente y el desarrollo de las socieda-
des. Y la Historia es Tiempo, es el conjunto de hechos protagonizados por los seres humanos A
LO LARGO DEL TIEMPO, es la memoria del PASADO que tenemos en el PRESENTE, PRE-
SENTE que será historia en el FUTURO.

El ser humano recibe información de lo que le rodea a través de sus sentidos y forma en su
cerebro los conceptos que le permiten concebir el cosmos (entendido como el TODO), sobre la
base de los datos que le aportan. De este modo, la conciencia del paso del tiempo se basó ori-
ginalmente en la observación de los cambios experimentados por el cosmos circundante entre
momentos diferentes. La ausencia de cambios habría hecho casi imposible la definición con-
ceptual del tiempo. Efectivamente, el cosmos que rodea al ser humano cambia continuamente,
el propio ser humano, los objetos y seres vivos experimentan cambios significativos con el paso
del tiempo: los seres vivos nacen, crecen y mueren; los objetos, dependiendo de la materia que
los forma, se alteran, envejecen e, incluso, se desintegran.

Este tipo de cambios, sin otras referencias, no habrían permitido al hombre establecer siste-
mas sencillos de medida del tiempo. Medir requiere pautas y referencias estables. Y esas pau-
tas y referencias también están en el cosmos circundante. Efectivamente, el primer cambio sig-
nificativo que el hombre pudo observar fue la sucesión del día y la noche. El Sol salía siempre
por un lado del horizonte y se ponía por el contrario, el cielo perdía su color azul y se iba hacien-
do negro; pero cuajado de un sinfín de puntos luminosos que parecían formar figuras familia-
res, las constelaciones, y la Luna venía a iluminar algunas de las noches cambiando vistosa-
mente su forma, unas veces redonda por completo, otras como un gajo de naranja, más ancho o
más estrecho. Y, además, resultaba que las constelaciones iban cambiando noche a noche de
posición como si se desplazaran, en el mismo sentido que el Sol, y volvían a estar en la misma
posición después de unas 365 puestas de Sol = AÑO; y la Luna repetía la misma sucesión de
cambios en su forma, volviendo a la misma tras unas 28 puestas de Sol = MES.

5
-Pág.77-
CIENCIAS SOCIALES (GEOGRAFÍA E HISTORIA)

Efectivamente, el Sol, la Luna y las Estrellas, fueron los responsables, con sus movimientos
aparentes, del establecimiento por parte del hombre de sus medidas de tiempo: DÍA, MES,
AÑO. Era relativamente sencillo combinar DIA y AÑO pues pasaban 365 días hasta que vol-
vían las mismas estrellas al mismo punto en la noche; pero si se repetían las observaciones a lo
largo de un periodo muy prolongado de tiempo se podía comprobar que se producía un ligero
desfase, de modo que cada cuatro años había que esperar un día más para ver esas estrellas en
el mismo punto. En cualquier caso, el ajuste era sencillo. Pero no lo era tanto el generado por
la combinación de las fases de la Luna, los días y el año, pues el periodo lunar no era exacta-
mente de 28 días y un año “normal” contenía 13 periodos lunares y sobraba un día. Todos estos
problemas encontraron diferentes soluciones en las distintas culturas, unas adoptaron calenda-
rios solares, otras calendarios lunares.
Por otra parte, la observación del entorno, llevaba a concluir que el año, además de la refe-
rencia de las estrellas, tenía una referencia de tiempo meteorológico pues había una sucesión de
cambiantes condiciones de frío y calor, lluvias y sequía, lo que parecía ir vinculado de alguna
forma al hecho de que el Sol parecía también elevarse bastante más en el cielo en unas etapas,
las de más calor, que en otras en las que no alcanzaba tanta “altura”, las de más frío. De este
modo se definían las estaciones: Primavera, Verano, Otoño, Invierno. El control de estas
sucesiones se convirtió en algo trascendental después de la revolución neolítica. Ya era impor-
tante para el depredador hombre del Paleolítico pues los rebaños de animales objeto de su caza
se movían estacionalmente; pero, después del inicio de la agricultura, pasó a ser vital ya que las
actividades de labranza, sembrado, regadío, en su caso, y recolección, requerían atención a los
momentos más adecuados en función del ciclo de vida de las plantas, estrechamente asociado a
la sucesión de las estaciones.
La paulatina complicación del entramado social y económico que viene de la mano de las
primeras civilizaciones hizo que, para determinadas actividades humanas, la medida “DÍA”,
resultara demasiado grande o demasiado pequeña. Así surgió, por una parte, la compartimenta-
ción del día en Horas, primero como una referencia aproximada y después con una precisión
matemática que incluía, además, su división en minutos y la de éstos en segundos. En el otro
sentido, también se formaron grupos de días inferiores al mes, también relacionados con las
fases lunares pues se dividían éstas en sus “cuartos”, lo que daba como resultado siete días, o
sea, la SEMANA (en latín septimana, de septi = siete).
La combinación de todos estos aspectos no se ha producido, como ya apuntábamos más arri-
ba, del mismo modo en las distintas culturas que se han desarrollado en el mundo y, de hecho,
hoy en día persisten formulaciones distintas de lo que llamamos calendario. El “triunfo” mun-
dial de la llamada civilización occidental y las necesidades socio - económicas de lo que se
llama globalización, han llevado al predominio del sistema o calendario occidental cuyo origen
se encuentra en las culturas del arco mediterráneo y cuya formulación, podemos decir legal,
correspondió a Roma.

6
-Pág.78-
U . D . 5 . - S OCIEDAD Y CAMBIO EN EL TIEMPO

1.1.2. EL CALENDARIO

Efectivamente, se atribuye al rey Numa Pompilio la regularización del calendario roma-


no, basado en un año normal de 355 días, dividido en doce meses de 28 y 29 días, que comen-
zaban con la primavera, es decir, en nuestro Marzo. Los 10 días que completaban los 365 eran
intercalados a lo largo del año por decisión arbitraria del Pontifex Maximus.
Los meses recibieron inicialmente nombres alusivos a los números de su orden, de ahí que
aún hoy los cuatro últimos meses de nuestro calendario conserven nombres referidos a esos
números: SEPTIEMBRE (séptimo), OCTUBRE (octavo), NOVIEMBRE (noveno), DICIEM-
BRE (décimo) (enero y febrero ocupaban, con sus primitivos nombres, los puestos undécimo y
duodécimo). Hasta la época de Julio César, algunos meses fueron cambiando sus nombres reci-
biendo los alusivos a determinadas celebraciones o dioses. Así, el último mes del año recibió el
de Februarius (FEBRERO) por las fiestas Frebrualia, de purificación ante el nuevo año; el pri-
mero se llamó Martius (MARZO), por Marte, dios de la guerra, al que se consagró el mes en el
que se reiniciaban las operaciones militares, tras el invierno; el siguiente se llamó Aprilis
(ABRIL) y estaba consagrado a Venus, los propios romanos discutían el origen del nombre, para
unos se debía a la latinización de Aphrodite, diosa griega identificada con Venus; para otros pro-
cedería del verbo aperire (abrir), por abrirse las flores durante dicho mes. El tercer mes se llamó
Maius (MAYO), en honor a Maia, diosa de la primavera y el crecimiento, hija de Atlas y madre
de Mercurio. Y, finalmente, el cuarto mes se llamó Iunius (JUNIO), por la diosa Juno, esposa
de Júpiter, protectora de los matrimonios y los nacimientos (para algunos debía el nombre a que
era el mes dedicado a los jóvenes= iuniores).
El calendario de Numa Pompilio sirvió a los intereses de Roma hasta el año 47 a.C. En aque-
lla fecha, el dictator Julio César, asesorado por el griego Sosígenes, reformó el calendario
(calendario juliano). La reforma supuso, en primer lugar, añadir de una sola vez 17 días para
corregir el desfase sobre el tiempo real, fijar la duración del año en 365 días y añadir uno a
Februarius cada 4 años, establecer el inicio del año en el mes de Ianuarius (ENERO), nombre
dado en honor de Jano, dios de las puertas y de los comienzos, y modificar el reparto del núme-
ro de días de algunos meses (enero, por ejemplo, pasó de 29 a 31). Este cambio del comienzo
del año no se mantuvo después en algunos de los Estados nacidos de la desmembración del
Imperio romano: por ejemplo, en Francia el año no comenzó en enero hasta 1564, por decisión
de Carlos IX, y en Inglaterra comenzaba el 25 de marzo hasta 1752.
Tras la muerte de Iulius Caesar (Julio César), en 44 a.C., el Senado dio el nombre de Iulius
(JULIO) al mes Quintilis y le sumó un día a los 30 con los que contaba, quitándoselo a Febre-
ro. Después, también el Senado cambió el nombre y número de días del mes Sextilis, dándole
el del primer emperador, Augustus (AGOSTO), y completándole a 31 días, nuevamente en
detrimento de Febrero, razón por la cual éste tiene 28 días. Posteriormente, hubo otros intentos
de cambiar los ya inapropiados nombres de los ahora cuatro últimos meses: Octubre pudo
haberse llamado Tiberio de no ser porque el segundo emperador rehusó el honor que le querían
hacer los senadores preguntándoles “¿Qué harían sus señorías si el imperio tuviera 13 césares?”;

7
-Pág.79-
CIENCIAS SOCIALES (GEOGRAFÍA E HISTORIA)

y Noviembre pudo llamarse Amazona de no haber sido, en este caso, por el rechazo del Sena-
do a que el emperador Cómodo le diera el nombre de su amante.
La sucesión de años de 365 días intercalando uno de 366 cada tres se mantuvo hasta la Bula
del Papa Gregorio XIII que entró en vigor el 5 de octubre de 1582. A partir de ese momento
se va generalizando el nuevo calendario gregoriano, fruto de un concienzudo estudio ante la
evidencia de que, a pesar de la inclusión de años bisiestos en el calendario juliano, se había pro-
ducido, hasta 1582, un desfase de 10 días, como consecuencia de que el año no dura 365 días y
1⁄ exacto, sino algo más de 1⁄ , razón por la cual la bula papal decretó que al día 5 de octubre
4 4
de 1582 le siguiera el 15 de octubre y no el 6. Naturalmente, en plena efervescencia de la lucha
Reforma - Contrarreforma, los países protestantes, los ortodoxos, e incluso, algunos católicos,
no aplicaron hasta mucho más tarde la reforma (Así, por ejemplo, en Rusia no se aplicó hasta
después de la Revolución comunista, en 1918, cuando el desfase era ya de 13 días). Para que no
se repitiera el desfase, la reforma gregoriana determinó que los años de final de siglo (los ter-
minados en 00) no fueran bisiestos a no ser que fueran múltiplos de 400 (razón por la cual el
2000 sí fue bisiesto).
Como ya hemos indicado más arriba, este calendario, predominante en las relaciones socio
- económicas mundiales, no ha sido ni es el único. Mesopotámicos y egipcios, incas, aztecas y
mayas tuvieron el suyo y, hoy, japoneses, chinos, hebreos y musulmanes, por ejemplo, tienen
su propio calendario, usado sobre todo para cuestiones rituales.
No cabe duda que el actual calendario, al que estamos familiarizados, tiene en realidad
muchos defectos, empezando por el gasto que supone tener que imprimir todos los años un
nuevo almanaque, pues al no ser los días del año múltiplos de 7, el 1 de enero de un año no es
el mismo día de la semana que el año anterior y las fiestas se van desplazando de igual mane-
ra. La reforma impuesta por la República francesa en 1793 fue anulada en 1805 y el calendario
mundial propuesto a la ONU, que solventaría esos problemas, no fue aprobado por el Consejo
Económico y Social (ECOSOC) en su pleno de 20 de abril de 1956.

1.1.3. UNIDADES TEMPORALES


Hasta ahora nos hemos referido a periodos de tiempo que no van más allá del año:
– Día
– Semana 7 días
– Mes 28 a 31 días
– Estación 3 meses
Sin embargo, para determinadas cuestiones, resultan útiles medidas de tiempo que sean múl-
tiplos del año. En nuestra cultura, tales múltiplos son, principalmente:

8
-Pág.80-
U . D . 5 . - S OCIEDAD Y CAMBIO EN EL TIEMPO

– Bienio 2 años
– Trienio 3 años
– Quinquenio o Lustro 5 años
– Década 10 años
– Siglo 100 años
– Milenio 1000 años
En otras culturas se han utilizado otros periodos, como, por ejemplo, la Olimpiada grie-
ga (4 años), el katún maya (7200 días) o el baktún, también maya (144.000 días).

1.1.4. LAS ERAS


Por otra parte, independientemente de que el año se estructure de una manera o de otra, la
Historia se enfrenta a una necesidad esencial que es la de referir los acontecimientos a años
ordenados en una sucesión concreta. Este problema era solventado en los primeros textos his-
tóricos refiriendo los hechos a un año dentro de un determinado periodo, normalmente el del
mandato de cada rey o emperador. Así, podía decirse “…en el tercer año del reinado de…”. Esto
requería un control preciso de la sucesión de los años de los sucesivos reinados que, lógica-
mente, no duraban un número de años completos.
En la Roma republicana, por ejemplo, el hecho de que la más alta magistratura, el consu-
lado, fuera ejercida, tras elección, por dos ciudadanos durante un año, permitía referir los
hechos de ese año a los nombres de dichos cónsules; si bien esto obligaba a guardar relación
precisa de la sucesión de parejas para poder referenciar acontecimientos pasados y que fueran
identificables como sucedidos hacía X años.
Cualquiera de estas fórmulas supedita, como hemos visto, las referencias a la memoria de la
sucesión de esos periodos temporales. Resulta obvio que es mucho mejor contar años de un
modo consecutivo y continuo. Pero dicha cuenta requiere un punto de partida, o varios si se
quieren establecer fases en lugar de acumular cifras. Esto nos lleva a la idea de Era, que podía-
mos definir como el conjunto de años que siguen a una fecha de referencia.
En distintas culturas históricas esta preocupación se ha traducido en el establecimiento de
fechas de referencia a partir de las cuales comenzaron a contar los años de Eras respectivas.
Citemos algunas de las más significativas antes de analizar nuestra propia forma de afrontar este
tema, y lo haremos indicando inicialmente los años que hay que sumar o restar a nuestro núme-
ro anual:
+ 660 años = Era japonesa, desde la entronización del primer emperador: Jimmu.
+ 2637 años = Era china.

9
-Pág.81-
CIENCIAS SOCIALES (GEOGRAFÍA E HISTORIA)

- 78 años = Era Saka o india.


+ 3761 años = Era hebrea
+ 753 años = Era romana, desde la fundación de Roma (“ab urbe condita”)
- 622 años = Era musulmana, desde la Hégira.
Pero al igual que el calendario gregoriano se impone en las relaciones internacionales, tam-
bién lo hace la cuenta de años de la civilización occidental, basada, como el mismo calendario,
en cuestiones relacionadas con el cristianismo. Efectivamente, la Era cristiana es la que cuen-
ta los años antes y después de Jesucristo, aunque no fuera así desde el principio.
Así es, la vida y muerte de Jesús de Nazaret se produjo dentro del Imperio Romano, en la
provincia de Judea, durante los primeros años del Imperio, siendo emperadores Augusto, pri-
mero, y Tiberio, después. Los Apóstoles guardaron memoria de sus hechos, que eran para ellos
más importantes en sí mismos que una referencia cronológica exacta. Además, para cuando se
plantearon dejar constancia escrita, habían pasado ya muchos años desde su muerte.
Lo cierto es que los primeros cristianos no guardaban memoria de la fecha exacta de la
muerte y, mucho menos, del nacimiento, sobre cuyos detalles los apóstoles sabían poco o nada,
ya que conocieron a Jesucristo mucho después. Sólo el Evangelista Lucas habla del nacimien-
to con detalle pero no cita una fecha concreta.
Cuando el lento avance del cristianismo en el marco del Imperio llegó a su culminación con
el reconocimiento como religión oficial del Imperio, habían pasado más de tres siglos desde los
acontecimientos que inspiraban el entramado doctrinal del cristianismo. Nadie tenía referencia
exacta del año del nacimiento y, por tanto, seguía siendo válida la Era romana (“ab Urbe
condita”= AUC).
Y así se mantuvo hasta que Dionisius Exiguus se preocupó de localizar temporalmente el
nacimiento de Jesucristo estableciéndolo en el año 753 AUC. Lentamente se fue imponien-
do la nueva forma de contar los años de modo que el paso del año 1000 sólo tuvo sentido para
algunos cristianos. El predominio cristiano y la expansión europea hizo que la Era cristiana ter-
minara siendo la forma más extendida de contar los años y, junto al calendario gregoriano, sea
hoy en día la forma internacional de citar las fechas. De nada vale, al menos por el momento,
que las investigaciones más recientes lleven a la conclusión de que Dionisio el Exiguo falló en
4 ó 6 años sus cuentas, oficialmente, convencionalmente, estamos en el año que dice el alma-
naque colgado de la pared, encima de la mesa o en la pantalla del ordenador (a pesar de todo,
nadie se ha atrevido hasta hoy a dar una fecha segura en una publicación con respaldo cientí-
fico).

1.1.5. CONVENCIONES CRONOLÓGICAS PARA LA HISTORIA


La adopción de la referencia cristiana lleva a la medida del tiempo histórico en un antes y
un después de la “bisagra” del año 1. Así, hablamos de que en el 218 antes de Cristo (a.C.) se

10
-Pág.82-
U . D . 5 . - S OCIEDAD Y CAMBIO EN EL TIEMPO

inició la conquista romana de Hispania o que en el 14 después de Cristo (d.C) o annus dei
(AD, año de Dios) o de la Era Cristiana (EC), murió el emperador Augusto, y escribimos las
correspondientes abreviaturas detrás de los dígitos, aunque a medida que nos acercamos en la
referencia a los tiempos presentes resulta vacuo consignarlas y, así, no es apropiado decir que
el Descubrimiento de América tuvo lugar en el 1492 d.C., decimos “en 1492”.
Resulta obvio que las fechas anteriores al año 1, las fechas a.C., siguen un orden ascen-
dente hacia atrás, es decir, su cifra es mayor cuanto más antiguo es el hecho referenciado,
todo lo contrario que las fechas d.C. que son menores cuanto más antiguo es el hecho. Sin
embargo, no todos los hechos anteriores a la Era cristiana se citan en años a.C., cuando la
antigüedad se mide en varios miles de años se tiende a expresarlo como “hace X años”, natu-
ralmente se trata de referencias con márgenes amplios de error, menos significativo cuanto
mayor sea el número de años. Así, no es coherente decir que el Homo Sapiens Sapiens “sur-
gió en algún lugar de África en el 100.000 a.C.” sino que resulta más lógico expresarlo como
“hace unos 100.000 años”.
La sistematización de los tratados de Historia exige, por otra parte, otro tipo de convencio-
nes que permitan referir los hechos que se narran de un modo sencillo y estructurado. Así, en
ocasiones se citan las referencias cronológicas en siglos o en milenios. La notación de los
siglos se hace con números romanos y la de los milenios con ordinales: expresamos “el siglo
XVIII” (ocasionalmente la “XVIIIª Centuria”), o decimos “el segundo milenio a.C.”.
Además, como ya vimos en el tema anterior, esa misma sistematización de la Historia ha con-
ducido al establecimiento de una periodización en Edades, marcadas por determinados rasgos
culturales definitorios, y que resumíamos en un cuadro general al que remitimos al alumno.

1.2. EL CAMBIO Y LA CONTINUIDAD A TRAVÉS DEL TIEMPO


El análisis de la historia, sobre la línea del tiempo, se ha orientado tradicionalmente a los
grandes hechos, aquellos que protagonizan los gobernantes y tienen que ver con guerras, con-
quistas, revoluciones. Sin embargo, como ya vimos en el tema anterior, esos aspectos no son los
únicos de la historia de modo que los más recientes tratados prestan tanta o más atención a cues-
tiones sociales, económicas y culturales. Y si se pueden abrir apartados dedicados e dichos
temas en los estudios sobre los diferentes periodos en los que se ha dividido tradicionalmente
la Historia, también se puede analizar la evolución en el tiempo de cada uno de esos aspectos
de forma individualizada. Afrontamos ahora, pues, una breve semblanza de la evolución histó-
rica de cuestiones tales como la actividad económica, la vida privada, las creencias religiosas,
etc…

1.2.1. HISTORIA DE LA ECONOMÍA


Si por economía entendemos el conjunto de actividades realizadas por el hombre para la
obtención de los recursos necesarios para la subsistencia y las relaciones que, orientadas a dicho

11
-Pág.83-
CIENCIAS SOCIALES (GEOGRAFÍA E HISTORIA)

objetivo, se establecen entre hombre y grupos humanos, llegaríamos a la conclusión inicial de


que prácticamente todas las actividades pueden ser consideradas de algún modo economía. Así,
lo sería el trabajo orientado a la obtención de alimentos pero podría serlo también el estableci-
miento de formas de matrimonio que llevan implícita una forma de reparto del trabajo, o el esta-
blecimiento de sistemas políticos que amparen y desarrollen determinados tipos de relaciones de
producción, comercio, trabajo. Mas, lejos de pretender realizar una pormenorizada Historia Eco-
nómica y, mucho menos aún, teorizar sobre el papel de la Economía en la Historia, esquemati-
zamos brevemente los grandes pasos evolutivos de la actividad económica:

– La economía depredadora
Durante los primeros pasos de la evolución humana, el hombre obtenía sus recursos direc-
tamente de la naturaleza sin realizar ninguna actividad importante de modificación del medio:
la caza, la pesca, el marisqueo y la recolección de frutos silvestres proporcionaban los alimen-
tos. La transformación de materias primas comenzó tímidamente a entrar en juego para la adap-
tación de pieles y fibras vegetales a la creación de ropas (naturalmente en los ambientes clima-
tológicos en los que éstas eran necesarias) y, sobre todo, para la adaptación de elementos de la
naturaleza a la obtención de herramientas. Efectivamente, para algunos investigadores es preci-
samente la capacidad de fabricación de objetos especializados lo que confiere al ser humano su
condición (y hablamos de fabricación y no de utilización solamente, pues diversos animales son
capaces de utilizar objetos: por ejemplo, los chimpancés utilizan ramitas para hurgar en nidos
de termitas y obtener larvas o usan piedras para abrir frutos de corteza dura). Los útiles en pie-
dra, madera o hueso, cada vez más especializados, debían ser fabricados para ayudar en las
labores de obtención de recursos: cazar (puntas de lanza y flecha, arco), pescar (arpones, aza-
gayas), preparación de la carne (cuchillos), tratado de la piel (raederas, buriles) o en la propia
fabricación de otros objetos (punzones, raspadores…). Un rudimentario comercio pudo poner-
se en juego al entrar en contacto grupos que intercambiaran objetos de adorno o pieles pero sin
que existiera específica e intencionadamente una fabricación de excedentes orientados al
comercio. Y, por último, debió existir una división del trabajo basada en el sexo.

– La revolución neolítica
Las modificaciones en las condiciones climatológicas al finalizar la última glaciación obli-
garon al hombre, en determinados ambientes, a modificar sus métodos de obtención de alimen-
tos. Simplificando el proceso, podríamos decir que hubo primero una transición que pasó por
fases de cuidados especiales a plantas silvestres y control de rebaños de animales a los que, en
lugar de perseguir para cazar, se retenía por medios distintos para asegurar el suministro de carne
y pieles y, ahora también, de leche (pastoreo); siendo esta fase la del inicio de lo que Gordon
Childe llamaba revolución neolítica. Efectivamente, se pasó de una economía depredadora del
medio a una economía de producción mediante la transformación del medio. Resultaba más prác-
tico concentrar las plantas útiles y los rebaños en un lugar determinado y en él realizar las labo-
res de cuidado de plantas y animales sin tener que desplazarse. Este proceso inició una sucesión
de efectos concatenados que desembocaron en la formación de las primeras civilizaciones.

12
-Pág.84-
U . D . 5 . - S OCIEDAD Y CAMBIO EN EL TIEMPO

La sedentarización condujo a la racionalización de la producción, al aprovechamiento de los


recursos de riego y la consiguiente mejora en calidad y cantidad, lo que llevó a la obtención de
excedentes. Éstos resultaron claves para que parte de la población pudiera dedicarse a otras acti-
vidades que no fueran específicamente productivas o a actividades que permitieran una espe-
cialización del trabajo: el agricultor produce trigo; el artesano arados; el agricultor da parte de
su excedente de trigo para obtener un arado; el artesano da un arado para obtener trigo. Así pues,
por este camino, los excedentes se convierten en la clave del inicio del comercio, así como de
otra sucesión de efectos que podemos esquematizar del siguiente modo:

13
-Pág.85-
CIENCIAS SOCIALES (GEOGRAFÍA E HISTORIA)

Básicamente, el desarrollo de la historia humana desde el inicio de estos cambios se ha man-


tenido invariable en sus principios económicos. Sin embargo, tras la revolución neolítica, la evo-
lución de la economía nos ha presentado otras etapas, que seguimos analizando someramente.

– Las economías esclavistas


Las primeras civilizaciones, Mesopotamia, Egipto, Grecia, Roma, sobre la base del sistema
de causas y consecuencias que podemos observar en el cuadro precedente, fundamentaron su
desarrollo económico y cultural en la esclavitud, una fuerza de trabajo barata sometida a un sis-
tema judicial que consideraba a los individuos sometidos a la misma como propiedades mate-
riales. Los avances técnicos, existentes en todo caso, no fueron lo suficientemente importantes
como para modificar las formas de producir.

– El feudalismo
La Edad Media trajo, con el predominio del cristianismo, la desaparición de la esclavitud
entendida bajo el prisma anterior, aunque aparecieron nuevas formas de vincular legalmente,
incluso moralmente, el trabajo de unas personas a los intereses de otras. Así, se crean los vínculos
personales de dependencia sobre los que se fundamenta el feudalismo: el vasallo se encomien-
da al señor dándole su trabajo, bajo una u otra fórmula (parte de la producción, un pago o tri-
buto, su trabajo personal), y el señor le protege. El desmantelamiento de buena parte de las rutas
comerciales como consecuencia de la caída del Imperio romano se fue poco a poco subsanan-
do, reconstruyéndose, a veces bajo impulsos religiosos (Camino de Santiago), y multiplicándo-
se. La artesanía, bajo el esquema de los gremios, ayudó a impulsar la economía y el comercio,
con un sistema monetario paulatinamente más complejo, y los beneficios generados por el
mismo abrieron la modificación del sistema social.

– El mercantilismo
La evolución de la economía medieval, la aparición de una cada vez más fuerte burguesía
artesanal y comercial y la apertura de los mercados a horizontes mucho más amplios tras los
descubrimientos geográficos, se unió e interrelacionó con las transformaciones filosóficas que
el humanismo aporta y condujo al desarrollo de una economía que, sobre bases agropecuarias
y metalúrgicas, hizo del comercio soporte esencial de la riqueza de los nacientes Estados y las
enriquecidas clases mercantiles.

– El capitalismo
Sobre el soporte de la economía mercantilista, amparada por el intervencionismo de las
administraciones absolutistas que caracterizan la etapa anterior, se producirá la modificación de
la filosofía sobre la economía impulsada por las nuevas ideas de la burguesía, inspiradas en las
ilustradas y que llamamos ideas liberales. La iniciativa privada, la libertad de los intercambios,
la ley de la oferta y la demanda, se imponen y favorecen, unidas a los avances científicos acu-
mulados desde el Renacimiento, la Revolución Industrial, que modificará sustancialmente las
formas de producir y las relaciones sociales. Efectivamente, los factores del capitalismo favo-

14
-Pág.86-
U . D . 5 . - S OCIEDAD Y CAMBIO EN EL TIEMPO

recen la concentración de los beneficios de la comercialización de los productos en manos de


los dueños de los medios de producción y vinculan a los individuos que no poseen dichos
medios a los poseedores bajo nuevas formas de relación patrono - empleado. Estas circunstan-
cias supusieron la aparición de capas numerosas de población desheredada y dependiente de su
fuerza de trabajo con unas condiciones de vida paupérrimas que contrastaban con la riqueza de
los grupos poseedores de las factorías o controladores del comercio. Esto sirvió de caldo de cul-
tivo a nuevas ideologías.

– El socialismo.
Entre esas nuevas ideologías se encuentra el análisis marxista (Carlos Marx), que inspiró
movimientos revolucionarios que pretendían acabar con el sistema capitalista a causa de uno de
los aspectos del mismo: el desequilibrio social en el reparto de la riqueza. El triunfo de alguna
de esas revoluciones (Rusia, 1917; China, 1945), inauguró lo que algunos han denominado
“socialismo real”, es decir, la puesta en práctica de las tesis socialistas marxistas. Desde el punto
de vista económico suponía la estatalización de los medios de producción y la dirección insti-
tucional de las explotaciones y el mercado de trabajo y de productos. Pero el fracaso del siste-
ma, tanto por sus propias carencias para la generación de riqueza como por las consecuencias
sociales y políticas, puso fin al “socialismo real” en la mayor parte de los países en los que se
puso en práctica, a partir de los años 90 del siglo XX.

– La economía de hoy. La globalización.


Quizás no se pueda afirmar que la caída de los sistemas socialistas haya supuesto el triunfo
del capitalismo, tal vez porque realmente el sistema económico de hoy no es un sistema capita-
lista en puridad. La intervención de los gobiernos en la dirección de la economía y los acuerdos
internacionales, lo han cambiado. Hoy, las empresas tienen un arraigo mundial, los intercam-
bios comerciales no se ciñen a estrechos mercados nacionales sino que alcanzan una perspecti-
va global, lo cual interrelaciona hasta el extremo los condicionantes económicos y hace que las
economías de todas las regiones se hagan dependientes o, mejor, tiendan a la constitución de un
único espacio económico.

1.2.2. LA VIDA PRIVADA Y LA MARGINACIÓN FEMENINA


Este aspecto se encuentra también, en cierto modo, vinculado a la evolución de las condi-
ciones económicas, aunque tiene perspectivas propias. Biológicamente, el ser humano es un
animal sexuado, se reproduce sexualmente, es decir, mediante la combinación de cromosomas
aportados por cada uno de los dos individuos, masculino y femenino que forman la especie. El
papel que la biología ha asignado a cada uno de los dos individuos es, evidentemente distinto:
el hombre interviene en la reproducción biológica aportando su esperma; la mujer, presta su
óvulo, pero, a continuación, sustenta en su propio cuerpo el desarrollo del embrión hasta que
eclosiona y nace y continúa prestando cuidado y alimento extraído de su propio cuerpo al inde-

15
-Pág.87-
CIE
CNIE
CNIA
CSIA
SSOC
SOIA
CLIA
ESLE
(GSE
(GOE
GORG
ARFÍ
A AF ÍEAHEI S
H TIO
STRO
IAR)I A )

fenso vástago. La dedicación casi permanente de la mujer al cuidado de los hijos (teniendo en
cuenta que el puerperio era sucedido casi sin interrupción por nuevos embarazos), asignaba al
macho de la especie el papel de ayuda mediante la obtención de alimentos para el sostenimien-
to de la hembra, primero, y de los hijos destetados después. La división de las actividades coti-
dianas estaba repartida en función de estas circunstancias.

La trascendencia del papel de la mujer, unido a la similitud de los procesos que propiciaban
la germinación de las plantas cultivadas con el embarazo y el parto, se combinaron para hacer
de la sociedad neolítica inicial una sociedad matriarcal y que la religión se tiñera de aspectos
en los que la mujer, el principio de la fertilidad, etc… jugaban un papel protagonista.

Sin embargo, la evolución de la sociedad en las primeras civilizaciones, otorgó al varón el


papel director de la economía, la sociedad y la política, de modo que la mujer pasó a jugar un
papel secundario o mejor diríamos complementario, vinculado por el propio varón al terreno de
la reproducción, la satisfacción de los placeres sexuales y la realización de las labores cotidia-
nas del hogar. Ocasionalmente, la personalidad de algunas mujeres propiciaba que alcanzaran
lugares importantes en la vida política (emperatrices, reinas, como Cleopatra) aunque casi siem-
pre a la sombra de sus esposos (caso de la egipcia Nefertiti o de Livia, la esposa de Augusto, y
tantas otras).

La religión, otrora basada en la propia mujer, pero diseñada por el varón, se convirtió en una
justificación de la discriminación hacia las hembras de la especie en prácticamente todas las cul-
turas. La Edad Media siguió siendo así y los cambios sociales, políticos y económicos poste-
riores no mejoraron excesivamente la situación.

Pero la revolución demográfica de finales del siglo XIX, motivada por la mejora de las con-
diciones de producción que acompañó a la Revolución Industrial y por los avances de la medi-
cina, que disminuyeron la mortalidad, propició que disminuyera la vinculación de la mujer a
su papel reproductor: podía tener menos hijos con más esperanza de vida, la población podía
aumentar sin que aumentara proporcionalmente el número de nacimientos. Además, el entra-
mado social se ocupaba de determinadas parcelas de la educación infantil de la que quedaban
liberadas las mujeres. Todo esto propició la toma de conciencia de la mujer sobre sus condi-
ciones de vida y trabajo, se fue incorporando al mundo laboral, para empleos específicos y,
cada vez más, para otros antes exclusivos del hombre y comenzó a reclamar el reconocimien-
to de la igualdad de sus derechos con los de los varones. Temas como el de la mayoría de edad,
durante mucho tiempo cifrado en una edad distinta y superior a la del hombre, la dependencia
socio - legal primero del padre y luego del marido, o la marginación de las decisiones políti-
cas ejerciendo el voto (En España, por ejemplo, las mujeres no pudieron votar hasta 1931),
fueron siendo metas logradas con dificultades y esfuerzos. Aun cuando sigue siendo un tema
por resolver en su totalidad, hoy día se han cubierto muchas etapas del camino hacia una autén-
tica igualdad.

16
-Pág.88-
U . D . 5 . - S OCIEDAD Y CAMBIO EN EL TIEMPO

1.2.3. LAS CREENCIAS RELIGIOSAS

Tema especialmente complejo, por las sensibilidades que condiciona, no es nuestro objeti-
vo hacer aquí un tratado sobre la evolución de las creencias religiosas; pero debemos dar una
visión muy genérica de este aspecto.
Podemos afirmar que la religión es el entramado ideológico que sustenta las creencias del
hombre sobre el origen del cosmos que le rodea y la explicación de los fenómenos que en él se
producen, incluyendo el código moral que rige su comportamiento y las ideas sobre trascen-
dencia de la propia existencia humana y su posible perdurabilidad después de la muerte.
La contemplación del cosmos circundante ofrecía al hombre primitivo un sinfín de fenóme-
nos que, partiendo de su propia existencia, resultaban misteriosos. Él mismo sólo era capaz de
modificar levemente algunos objetos lo cual le hacía pensar que alguna mente infinitamente
poderosa debía ser la responsable que la “construcción” de ese cosmos. Las misteriosas condi-
ciones que hacían cambiar el tiempo, que propiciaban la caza o hacían crecer las plantas debían
ser regidas por esa u otras mentes superiores e invisibles de modo que poco a poco se fueron
tejiendo mitologías complejas.
Después de la diosa - madre, que debió presidir las creencias de las primeras culturas neo-
líticas, aparecieron entramados religiosos politeístas con una compleja estructura doctrinal y de
rituales cuya formulación resultaba más factible desde el momento que el sistema económico
permitía que algunos individuos se dedicaran a cuestiones como la realización de las ceremo-
nias y servir así de intermediarios entre los seres humanos y los dioses. Así, todas las civiliza-
ciones agrarias surgidas en el mundo llegarán a soluciones religiosas similares: el mundo, el
universo fue creado por una o varias divinidades de las que surgieron otras divinidades, crea-
doras y vigilantes de parcelas específicas de la vida humana o de la naturaleza, tremendamente
poderosos y sujetos a pasiones como las humanas magnificadas por su condición divina, de
modo que para aplacar su ira y propiciar su acción beneficiosa para el ser humano había que
ofrecer sacrificios y ofrendas.
Frente a esos sistemas politeístas hubo algunos visionarios que, tras profundos procesos de
reflexión, llegaron a soluciones distintas. Unas veces se trató de concluir que sólo existía un
dios, responsable de todo lo que se atribuía a distintos dioses, otras de concluir que no existía
una divinidad concebida con características humanas sino principios abstractos. Esta última
solución, cuya última formulación conduce, por la vía del racionalismo a la negación de la divi-
nidad, el ateísmo, no encontró eco hasta la época de la Ilustración, extendiéndose desde enton-
ces en determinados círculos intelectuales y siendo también uno de los principios de formula-
ción ideológica del marxismo. La primera de las soluciones contó, por ejemplo, con el faraón
Amenofis IV, quien instauró el culto al dios Atón (al que alude el nombre que adoptó desde
entonces = Akhenatón), imponiéndolo a una sociedad politeísta cual la egipcia y con una casta
sacerdotal celosa de los privilegios que mantenía en el sistema anterior, lo que propició su fra-
caso. Y también desde Egipto surgió la figura de Moisés, artífice de la religión monoteísta

17
-Pág.89-
CIENCIAS SOCIALES (GEOGRAFÍA E HISTORIA)

llamada a ser el germen de un sistema religioso trascendental en la conformación de la cultura


occidental.
Moisés, criado en la corte faraónica, adquirió conciencia de su origen hebreo y posiblemen-
te influenciado por la tradición atónica, combinada con las tradiciones judías, diseñó un siste-
ma religioso monoteísta que fue formulándose sobre todo a finales del 2º milenio a.C. Tras el
éxodo y la conquista de Palestina, la religión hebraica se fue consolidando en el marco exclusi-
vo del pueblo judío y estableciendo un conjunto de doctrinas y rituales propios. Y en el marco
de esa religión monoteísta se produjo el nacimiento, predicación y muerte de Yoshua bar Jusef
(Jesús hijo de José). Su historia pasó casi desapercibida para la mayoría y el poder político cuyo
sistema legal lo condenó, el Imperio Romano, no se percató de las consecuencias que sus pré-
dicas y la interpretación que les dieron sus apóstoles podían tener, hasta que tuvo que hacer fren-
te a su avance en las creencias de la sociedad romana. Distanciado de la religión hebraica, el
cristianismo se expandió en el Imperio romano hasta que se convirtió en la religión oficial del
mismo. Las discrepancias doctrinales generaron la aparición de corrientes heréticas que provo-
caron guerras religiosas y cismas. Los concilios pretendían solucionar las discrepancias pero
muchas veces se convertían en fuente de nuevas herejías. Así, cuando comienza la Edad Moderna,
la Iglesia está de hecho dividida en la occidental católica y la oriental ortodoxa y poco después
experimenta un nuevo cisma al triunfar en buena parte de Europa la Reforma protestante, que
modificó muchos de los principios doctrinales y los ritos católicos. Después, la expansión eu-
ropea por el mundo exportó la religión cristiana, fuera católica o protestante, por América y
África principalmente.
Pero antes, en el siglo VII, había aparecido otra religión monoteísta, basada en la hebraica
y en la cristiana y construida por Mahoma. El Islam se expandió por el Próximo Oriente, el
Norte de África y extensas zonas de Asia Central. El enfrentamiento con el mundo cristiano fue
constante durante la Edad Media (las Cruzadas, la Reconquista en España) y continuó después
(guerras euro - turcas). La guerra y la muerte, en nombre de la divinidad correspondiente, se
enseñorearon del mundo y aún hoy continúan justificando acciones sangrientas, irredentismos
políticos e intransigencias de todo tipo.
Así pues, el mundo de hoy, desde el punto de vista religioso nos muestra como predomi-
nantes las siguientes creencias:

18
-Pág.90-
U . D . 5 . - S OCIEDAD Y CAMBIO EN EL TIEMPO

1.2.4. LAS FORMAS DE ORGANIZACIÓN SOCIAL Y POLÍTICA


Estos aspectos han experimentado también una evolución significativa con el paso del tiem-
po. Antes del Neolítico y del surgimiento de las primeras civilizaciones, la sociedad se organi-
zaba sobre la base del clan, grupo constituido como una familia amplia integrada por los des-
cendientes de una misma madre posiblemente, en el que quizás existiera algún tipo de jefatura.
Las primeras civilizaciones contemplarán la aparición de los estamentos o niveles sociales
como consecuencia de la diferenciación de la riqueza de unos y otros, amén de la aparición de
la esclavitud. Por otro lado, también comenzarán a complicarse los entramados de organización
política apareciendo las Ciudades-Estado, en las que la ciudad era el epicentro que ejercía su
influencia sobre un ámbito reducido, y, más tarde, los Imperios centralizadores y totalitarios
basados en la idea de una autoridad de carácter religioso que incluye, a veces, la identificación
del emperador con un dios (Mesopotamia, Egipto).
Grecia pasa por ser la cuna del germen de un sistema político completamente diferente,
basado en la idea de participación de los ciudadanos en la toma de decisiones: la Democracia.
Cierto es que la Democracia griega se sustentaba en algo tan poco democrático como la escla-
vitud, pero, al menos, supuso la apertura de la política a los ciudadanos.
Roma, que comenzó con fórmulas políticas monárquicas, se dotó desde el siglo V al I a.C.
de un régimen muy estructurado que pretendía evitar la concentración del poder en las manos
de un único individuo, estableciendo un complejo sistema de elección de magistrados que ejer-
cían su autoridad de modo colegiado. Pero los inmensos territorios conquistados condujeron a
la instauración de un régimen imperial que cayó en las formulaciones orientales y terminó por
conceder el carácter divino a su cabeza.
La fragmentación del Imperio condujo a la formación de reinos diversos y al surgimiento
ocasional de nuevos imperios o al mantenimiento del oriental o bizantino; pero ahora con el
carácter sacro elevado a los cielos ya que el emperador no era dios; pero era Dios quien otor-
gaba el poder y, por tanto, quien lo ostentaba, lo ejercía sin mediatizaciones, de un modo abso-
luto aunque secundado o “ayudado” por instituciones diversas.
La conceptuación patrimonial de la monarquía medieval dio paso a la idea de soberanía con
lo que el reino, antes transmisible y divisible como una propiedad personal, se transformaba en
una realidad que trascendía al individuo, el monarca ostentaba la soberanía y la ejercía de un
modo absoluto. Estos conceptos nacen emparejados con la idea de Nación y de Estado como la
integración de una tierra y el pueblo que la habita.
Las ideas ilustradas, sobre la base filosófica racionalista, transformarán el absolutismo
monárquico en despotismo ilustrado, pero también servirán de fundamento a la construcción de
las ideas liberales que triunfarán, de la mano de la burguesía, en el paso del siglo XVIII al XIX
(Independencia de los EE.UU., Revolución francesa), haciendo aparecer las democracias bur-
guesas liberales, sustentadas en los conceptos de Soberanía popular y división de poderes.

19
-Pág.91-
CIENCIAS SOCIALES (GEOGRAFÍA E HISTORIA)

Los ensayos frustrados del sistema socialista tendentes, según la praxis marxista, a la elimi-
nación del Estado, pasando por los pasos de revolución, dictadura del proletariado y Estado
comunista, han dado paso a la consolidación de la democracia como el mejor sistema para
amparar las libertades individuales y servir de marco al progreso humano.

EVOLUCIÓN Y DESARROLLO EN LA HISTORIA


2. DEL ARTE
La definición de Arte no resulta, aunque lo parezca, sencilla. En cierto modo es cualquier
manifestación creativa del hombre, lo cual nos llevaría a considerar arte una pintura y también
un martillo. Tampoco nos valdría decir que es Arte sólo la creación realizada sin pensar en una
utilidad práctica material, movida por el afán de manifestar los sentimientos a través de las for-
mas; pues eso excluiría de la clasificación como Arte a construcciones y creaciones de distinto
tipo realizadas con una finalidad práctica inmediata como, por ejemplo, una Iglesia gótica o un
sarcófago egipcio. En cualquier caso, lo que sí es cierto es que el Arte es una manifestación del
espíritu humano, de sus conceptos estéticos y de su mentalidad.
En el estudio sobre el desarrollo de las culturas humanas en el tema anterior ya hemos tra-
tado aspectos de la evolución del Arte a través de la Historia, desde los orígenes, en las cuevas
paleolíticas, hasta el Neoclásico y continuaremos en los apartados siguientes. Remitimos, pues,
al alumno a una revisión de los puntos correspondientes y nos centramos aquí, siguiendo el pro-
grama del curso, en un ramillete de obras que han trascendido de modo especial.
Realmente, el conjunto de la producción artística humana es inmenso por lo que resulta
complicado escoger un número reducido de obras que sirvan de, digamos, emblema, del desa-
rrollo y evolución del arte, más cuando la sensibilidad de cada uno podrá producir la sensación
de que se ha olvidado citar alguna fundamental. Pero corriendo tales riesgos, citemos esa selec-
ción.

2.1. ALTAMIRA
El arte Paleolítico tiene quizás su
monumento más importante en la Cueva
cántabra de Altamira, en Santillana del
Mar. Las representaciones zoomorfas,
sobre todo de bisontes, pero también de
caballos y cérvidos están cargadas de rea-
lismo a la vez que de sencillez. Indepen-
dientemente del valor e interpretación
que se les ha dado, no cabe duda de su
magnificencia, ejerciendo sobre el obser-
vador una atracción muy especial.

Detalle de las pinturas de Altamira

20
-Pág.92-
U . D . 5 . - S OCIEDAD Y CAMBIO EN EL TIEMPO

2.2. PIRÁMIDES DE GIZEH


Más de cuatro mil años no han bastado
para acabar con la más colosal construcción
de la civilización egipcia. Las profundas
creencias religiosas sobre la eternidad de la
vida del alma justifican la puesta al servicio
del mundo funerario del enorme despliegue
de medios técnicos y de esfuerzo humano
desplegados para levantar toneladas de pie-
dra hacia las estrellas. Lo magnífico de los
resultados, sus dimensiones, su proporciona-
lidad y su belleza engrandecen la capacidad
de la tecnología egipcia. Pirámide de Keops
Durante cientos de años, pirámides como las de Keops, Kefrén y Mikerinos, en la llanura de
Gizéh, sirvieron de tumba monumental a los faraones que emprendían el viaje hacia oriente en
la barca solar para franquear los umbrales del más allá, donde su Ka viviría para toda la eterni-
dad, esa eternidad para la que parecen estar construidas las pirámides.

2.3. EL PARTENÓN
La esfera religiosa ha movido las iniciativas artísticas de la humanidad. El partenón es una
de esas iniciativas artísticas, mas con unas características muy peculiares. Fidias fue el encar-
gado de “esculpir” esta construcción, paradigma de la proporcionalidad, belleza y austeridad del
arte griego. Un templo destinado a guardar
la imagen de la diosa Atenea y a ser con-
templado desde fuera, donde se realizaban
las ceremonias. Sus frontones, metopas y
frisos (hoy en el Museo Británico) ilustra-
ban pasajes mitológicos como el nacimien-
to de Atenea o las guerras de lapitas y cen-
tauros. Parafraseando a Napoleón, respecto
a las pirámides, desde sus piedras, veinti-
cinco siglos nos contemplan.

Partenón de Atenas

21
-Pág.93-
CIENCIAS SOCIALES (GEOGRAFÍA E HISTORIA)

2.4. EL COLISEO
El Anfitreato Máximo (s. I d.C.) fue
construido para servir de escenario a uno de
los principales entretenimientos de la pobla-
ción romana: las luchas de gladiadores y
otros espectáculos sangrientos. Además de
sus magníficas proporciones y de las solu-
ciones arquitectónicas que refleja, su carác-
ter público nos sirve de ejemplo del carácter
del arte romano, más público y popular que
ninguno de los desarrollados hasta entonces.
Anfiteatro Máximo o Coliseum

2.5. LA CATEDRAL DE CHARTRES


La arquitectura gótica supuso un cambio
radical u osado de las técnicas de edificación,
manteniendo un material tan pesado como la
piedra como elemento esencial de la cons-
trucción. Los juegos de traslado de cargas
mediante gráciles arcos, permitió ese aire de
verticalidad y elevación que lo caracteriza.
Muchas son las catedrales y otras construc-
ciones góticas. Ésta es famosa por sus escul-
turas y presuntos valores esotéricos.
Catedral de Chartres

2.6. LA BASÍLICA DE SAN PEDRO


El Renacimiento supondrá la vuelta a las
proporciones y estética clásicas pero bajo
nuevas perpectivas. Los trabajos iniciados en
el siglo XV por Bramante en la sede papal, se
fueron completando con la intervención de
artistas renacentistas y barrocos (Rafael,
Miguel Ángel, Bernini), hasta dar lugar a la
mayor catedral del mundo.

Basílica de San Pedro

22
-Pág.94-
U . D . 5 . - S OCIEDAD Y CAMBIO EN EL TIEMPO

2.7. LA CAPILLA SIXTINA


Sin salir de la Basílica de San Pedro,
nos encontramos con la colosal decoración
del techo y paredes de la capilla en la se
realiza la elección de los papas. Miguel
Ángel fue el encargado de hacerlo y las
soluciones a las que llegó no pudieron ser
mejores pues, además de representar de
modo sublime diversos pasajes de la histo-
ria sagrada, se convirtió en paradigma de
los efectos visuales y de la adaptación de
las escenas a los espacios.

La creación de Adán, detalle de los frescos de la Capilla Sixtina

2.8. LAS MENINAS


Diego de Silva y Velázquez, al servicio del rey Felipe IV, creó una de las obras culminantes
de la pintura de todos los tiempos. El tema, la composición, la luz, el color, han atraído a afi-
cionados e investigadores desde su origen. El pintor, ante un lienzo, mira hacia el espectador,
aunque formalmente está mirando a los reyes, que aparecen reflejados en un espejo del fondo,
a los que se supone está retratando (el efecto ya había
sido utilizado por Van Eyck en su “Matrimonio
Arnolfini”). Una escena cortesana (bufones y mucha-
chas de la corte charlan mientras posan los reyes) se
desarrolla alrededor del pintor. La oscuridad de la
sala es rota por la luz que entra por el ventanal que se
adivina a la derecha, generando efectos de contraluz
en las figuras y por la luz que irrumpe por la puerta
abierta al fondo en la que se recorta una figura. Todo
ello contribuye a crear una sensación de atmósfera,
de espacio interpuesto, incorporando incluso el espa-
cio que debería haber delante de la composición
hacia el que se proyecta.
Las Meninas

23
-Pág.95-
CIENCIAS SOCIALES (GEOGRAFÍA E HISTORIA)

2.9. EL PALACIO DE VERSALLES


La monumental construcción del Palacio de Versalles con su espaciosidad y sus efectos de
profundidad enmarcada en unos grandilocuentes jardines, sirvió de eficaz propaganda al abso-
lutismo monárquico representado por Luis XIV.

Versalles

2.10. EL GUERNICA DE PICASSO


El triunfo de la pintura como arte de referencia se produce a partir del siglo XIX. Antes com-
parte protagonismo con arquitectura y escultura, a partir de entonces, éstas pierden protagonis-
mo. La sucesión de estilos, escuelas y corrientes pictóricas a lo largo del siglo XIX y XX nos
llevaría, si quisiéramos detenernos en cada uno de ellos a una prolija enumeración. Así, tendrían
que pasar ante nosotros obras de Goya, Toulouse-Lautrec, Van Gogh, Gauguin, Monet, Cézanne,
Dalí… Si hay que elegir un ejemplo éste debe ser el español Pablo Ruiz Picasso. Y de su pro-
lífica obra, quizás la más famosa, “Guernica”, magnífico ejemplo del arte abstracto.

Guernica de Picasso

24
-Pág.96-
U . D . 5 . - S OCIEDAD Y CAMBIO EN EL TIEMPO

3. CAMBIO SOCIAL Y REVOLUCIÓN EN LA ÉPOCA


CONTEMPORÁNEA

3.1. LA CRISIS DEL ANTIGUO RÉGIMEN Y LAS REVOLUCIONES


LIBERALES BURGUESAS

3.1.1. LA CRISIS DEL ANTIGUO RÉGIMEN

La sociedad europea de finales de siglo XVIII está marcada por los cambios filosóficos,
políticos y económicos que parten de los trascendentales cambios renacentistas, pasan por el
tamiz del racionalismo, derivan en las ideas ilustradas y, finalmente, en las teorías liberales.

La estructura social estamental heredada de la Edad Media convive con la creciente pro-
yección de las clases basadas en la riqueza personal, de modo que se producen paradojas
como aristócratas empobrecidos que conservan privilegios legales y políticos junto a burgue-
ses ricos carentes de los mismos privilegios. Es precisamente entre estos dos grupos, noble-
za y burguesía, entre los que crecen las ideas ilustradas y, sobre todo entre los segundos, las
ideas liberales.

Los conceptos sobre la organización política, económica y social que diseñan la ideología
liberal arraigan en la burguesía y preparan un movimiento expansivo, una sucesión de conspi-
raciones primero y revoluciones después, que irán transformando el mundo.

Además, la economía experimenta un crecimiento y expansión considerables por el desa-


rrollo del comercio y de la industria. La agricultura también aumentó sus rendimientos y pro-
ducción en general al mejorar los sistemas de producción. Y todo ello favoreció, junto con la
mejora de las condiciones sanitarias y de prevención, la explosión demográfica que alcanzó
valores muy elevados.

El declinar del Antiguo Régimen dará paso a una nueva forma de entender la política, la
sociedad y la economía. El absolutismo, la sociedad estamental y el absolutismo económico
serán sustituidos por el liberalismo, la sociedad de clases y el capitalismo.

Las ideas que combatirán y derribarán el Antiguo Régimen tienen su base en las ideas ilus-
tradas del s. XVIII que sirvieron como puntales al sistema (Despotismo Ilustrado) y sobre
ellas se construirán las que agrupamos en el concepto liberalismo, cuyas notas características
son:

25
-Pág.97-
CIENCIAS SOCIALES (GEOGRAFÍA E HISTORIA)

Política:
División de poderes:
(Montesquieu. El Espíritu de las Leyes).

• Legislativo: representantes del pueblo.


• Ejecutivo: el rey y su gobierno.
• Judicial: tribunales independientes.
Soberanía Nacional
(J.J. Rousseau. El Contrato Social).
Derechos del Hombre (Libertad, Igualdad).
Constitución.
Administración:

• Centralización sobre la idea de igualdad.


Sociedad:

• Igualdad, Clases.
• Derechos políticos (voto, gobierno), primero con carácter oligárquico,
después ampliándose.
La puesta en práctica

• Declaración Independencia de los EE.UU. (4 julio 1776), Constitución de 1787.


• Revolución Francesa (1789-1792). Declaración de Derechos del Hombre y del
Ciudadano.
Repercusión en España

• Crisis monárquica tras la invasión francesa que abrirá un periodo de luchas entre
liberalismo y absolutismo hasta 1833.

• Pasos: Cortes de Cádiz, Trienio Liberal.


• En América: abrirá camino a la emancipación.

26
-Pág.98-
U . D . 5 . - S OCIEDAD Y CAMBIO EN EL TIEMPO

3.1.2. LA INDEPENDENCIA DE LOS ESTADOS UNIDOS DE AMÉRICA

Aunque tradicionalmente se habla de la Revolución Francesa como la primera revolución


liberal burguesa que determina la implantación práctica de las ideas liberales, fue la indepen-
dencia de los EE.UU. el primer caso de aplicación directa de dichas ideas para la construcción
del Estado Liberal Moderno.

En las costas orientales de Norteamérica se habían ido formando desde el siglo XVII trece
colonias británicas cuya población crecía lentamente hasta que, a partir de 1700, dicha pobla-
ción se quintuplicó pasando de 250.000 a 1.700.000 habitantes en menos de cincuenta años.
Dado el crecimiento demográfico y económico de las colonias, surgió en ellas una creciente
burguesía que veía sus actividades comerciales mediatizadas por la reglamentación impuesta
por la metrópoli, lo que aumentó el descontento frente a la administración inglesa.

Tras la Guerra de los Siete Años (victoria inglesa sobre Francia), el gobierno inglés some-
tió a sus colonias a unas rígidas normas tributarias para paliar el déficit que la Guerra había
supuesto para el tesoro. Los colonos argumentaron contra tales normas que las mismas no ha-
bían sido votadas por representantes suyos en el Parlamento (ya que carecían de representa-
ción). Esto originó una creciente hostilidad entre colonias y metrópoli, entre las peticiones de
las primeras y la intransigencia de la segunda, hasta que el 4 de julio de 1776, el Segundo Con-
greso Continental, reunido en Filadelfia, aprobó la Declaración de Independencia de las Trece
Colonias, redactada por Thomas Jefferson, declaración que recogía las ideas que habían esta-
do gestándose en la Europa del XVIII.

La consiguiente Guerra adquirió un carácter internacional cuando, tras la batalla de Sarato-


ga, Francia se decidió a intervenir en la contienda con el afán de desquitarse de la derrota en la
Guerra de los Siete Años y arrastró también a España que, junto con Holanda, ayudaron de
modo decisivo a los rebeldes americanos, que lograron la victoria definitiva en la batalla de
Yorktown.

La Constitución de 1787 proclamaba la existencia de una nación americana estructurada en


13 Estados con un gobierno central y consagraba por primera vez en un documento legal de la
máxima jerarquía los principios liberales de organización política, social y económica, estable-
ciendo derechos y deberes de los ciudadanos. Las consecuencias para Europa y el resto de Amé-
rica iban a ser trascendentales y no se iban a hacer esperar.

27
-Pág.99-
CIENCIAS SOCIALES (GEOGRAFÍA E HISTORIA)

3.1.3. LA REVOLUCIÓN FRANCESA


La sociedad francesa de finales del s. XVIII mostraba de modo acentuado los desequilibrios
y contradicciones ya referidas. Existía un abierto enfrentamiento entre la nobleza y la burgue-
sía. Por otro lado, el Estado tenía una administración central desbordada que no era capaz de
imponer los principios de centralización y que mantenía la hacienda pública en crisis. Además,
gran parte de la nobleza y el clero mantenían a ultranza principios y formas propias del feuda-
lismo, procurando preservar sus privilegios. El descontento de la burguesía y de las capas popu-
lares era creciente.

En esta situación fueron convocados los Estados Generales, órgano consultivo integrado
por los llamados tres estados: Nobleza, Clero y el Tercer Estado. Los integrantes de este último,
junto a algunos de los miembros de los otros dos, reunidos en el Juego de Pelota, decidieron
mantenerse unidos, como Asamblea Nacional, hasta haber dado a Francia una Constitución. El
descontento popular por el hambre y los elevados precios prendió definitivamente y el 14 de
julio de 1789 las masas asaltaban la Bastilla, centro penitenciario que simbolizaba para ellas
el absolutismo monárquico. Los hechos se precipitaron.

Inicialmente la burguesía controlaba la situación, procurando una Constitución que pre-


servara el principio de igualdad entre los hombres y diseñara un Estado que reconociera los
derechos del ciudadano y se organizara sobre la base de división de poderes. El rey no estaba
dispuesto a aceptar la situación y las masas populares no se sentían plenamente satisfechas con
el proyecto. Ante ello el control derivó hacia los grupos más exaltados. Así se inició la fase del
terror que protagonizan personajes como Robespierre, Marat y... la guillotina, que se
ocupó también del cuello del monarca, Luis XVI.

La reacción contra los jacobinos o exaltados llevó a sus dirigentes a la guillotina y mode-
ró la revolución, otorgando el gobierno a un Directorio formado por cinco miembros. Pero
para entonces las potencias absolutistas europeas habían establecido un cerco sobre la Revolu-
ción y la guerra se escapaba del control del Directorio; por esa vía accedió al control del poder
el general triunfador Napoleón Bonaparte.

3.1.4. LA EUROPA DE NAPOLEÓN


Napoleón, nacido en Ajaccio (Córcega), realizó su carrera militar en los momentos en los
que Francia se debatía en los albores de la Revolución. El joven militar, de origen pequeño-aris-
tocrático, se mostró solidario con los principios revolucionarios liberales y consiguió sus ascen-
sos a General de Brigada y de División por sus actuaciones contra los ingleses y contra los rea-
listas que amenazaban el proceso revolucionario. Como General de la República, tras sus
victoriosas campañas en Italia, constituyó el ejército que, acompañado por científicos, desem-
barcó en Egipto como parte de su proyecto de lucha contra los ingleses, y que logró, no sólo una
nueva victoria militar, sino una aportación cultural inestimable al abrir a Europa el conocimien-
to de la milenaria cultura egipcia. Desde Egipto, tras recibir noticia de la crisis que atravesaba el

28
-Pág.100-
U . D . 5 . - S OCIEDAD Y CAMBIO EN EL TIEMPO

Directorio, embarcó rumbo a París donde dio el Golpe de Estado de 18 Brumario (9-10 de
noviembre de 1799) que le convirtió en el Primer Cónsul. Su labor fue trascendental:

– Restauró el orden público.


– Ordenó la hacienda pública.
– Reorganizó la enseñanza.
– Creó el Código Civil, fijando las conquistas de la Revolución.
– Restableció la paz exterior.

Todo ello le hizo ganar el apoyo popular. El Senado le nombró Cónsul vitalicio y, final-
mente, en 1804, constituido el Imperio, le nombró Emperador, siendo coronado por el
Papa Pío VII.

Inglaterra, temerosa del poderío francés, declaró la guerra a Napoleón y logró constituir
una Coalición en su contra. Napoleón proyectó la invasión de Inglaterra pero fracasó al ser ani-
quilada la flota francesa y la española por la del almirante Nelson en Trafalgar. Tras ello, orien-
tó su actividad a la conquista de Europa. Logró constituir un ejército numeroso, fuerte, disci-
plinado y eficaz con el que terminó consolidando la hegemonía francesa en Europa a base de
un entramado de estados vasallos, aliados y familiares. El objetivo final de conquistar Inglate-
rra lo intentó mediante el bloqueo comercial que perjudicó a la larga tanto a Inglaterra como al
continente.

El final del Imperio napoleónico comenzará con la sublevación española en 1808, que
conducirá a las primeras derrotas de los ejércitos franceses. Acabando el conflicto en España,
en 1813 la derrota en Rusia anima a otros países a sublevarse (Prusia, Austria). Finalmente,
tras la decisiva batalla de Waterloo, Napoleón terminará desterrado en la Isla de Santa Elena.

Su intento de crear una unidad europea fracasó, quizás fundamentalmente por pretenderlo
por la fuerza, sin embargo, consiguió que las ideas liberales inspiradoras de la Revolución pren-
dieran en muchos pueblos, como sucedió en el caso de España.

3.1.5. REVOLUCIÓN EN ESPAÑA E INDEPENDENCIA DE LA AMÉRICA


ESPAÑOLA

3.1.5.1. La Guerra de la Independencia en España (1808-1814)

En cumplimiento del Tratado de Fontainebleau, España debe permitir el paso de las tro-
pas napoleónicas hacia Portugal. Pero las tropas francesas fueron ocupando las principales pla-
zas españolas a su paso. Conquistada casi toda España, el General Murat entró en Madrid.
Napoleón llamó a Bayona a Carlos IV y Fernando VII y, allí, los monarcas cedieron a las pre-
siones produciéndose una sucesión de abdicaciones que dejaron en manos de Napoleón el trono
de España.

29
-Pág.101-
CIENCIAS SOCIALES (GEOGRAFÍA E HISTORIA)

Mientras en Bayona se sucedían las claudicaciones, en el resto de España se veía con per-
plejidad el desarrollo de los acontecimientos. La pasividad de la nobleza y la sumisión de las
instituciones hizo que la resistencia fuera eminentemente popular. En Madrid, el 2 de mayo de
1808 la salida del resto de la familia real rumbo a Bayona desató las iras del pueblo que inten-
tó impedirlo concentrándose en la Plaza de Oriente. Las tropas francesas integradas por polacos
y mamelucos cargaron contra la población lo que abrió una sucesión de incidentes. El ejército
se mantenía expectante y sólo algunos oficiales se dispusieron a hacer frente a los franceses:
Capitanes Daoíz y Velarde y Teniente Ruiz. Apaciguada la rebelión, el General Murat, orde-
nó una cruel represión con masivas ejecuciones que se prolongaron durante la noche del 2 al 3
de mayo.

Pero la mecha estaba prendida. El levantamiento se fue extendiendo por casi todas las ciu-
dades y pueblos donde se crearon Juntas Locales y Provinciales que a su deseo de preservar
la independencia de la nación unían el afán por lograr profundas reformas políticas bajo el signo
del liberalismo. Se trató de una “guerra total” entre el ejército francés y el pueblo español.

Ante los iniciales éxitos españoles (Bailén) y el apoyo inglés, Napoleón se decide a inter-
venir personalmente, recuperando el terreno perdido y restableciendo el control. Pero Cádiz se
mantiene inexpugnable y allí se refugia la Junta Suprema Central.

Cuando parecía que el triunfo francés era inexcusable, la guerra da un giro al iniciarse la
práctica del tipo de intervenciones conocido como guerrillas. Pequeñas partidas armadas mix-
tas militares-civiles se dedicaban a entorpecer la marcha y dificultar las comunicaciones de las
fuerzas mediante acciones rápidas en lugares y momentos inesperados.

En 1812 comienza la gran ofensiva hispano-inglesa con las victorias en Ciudad Rodrigo y
Badajoz. El 22 de julio las tropas francesas son derrotadas en Arapiles. En 1813, se produce la
última ofensiva hispano-inglesa, victorias de Vitoria (21 de julio) y San Marcial (31 de agos-
to) que ponen fin a la guerra.

3.1.5.2. La revolución política durante la guerra


Tras el levantamiento, se formaron las Juntas Locales y Provinciales, creadas para el gobier-
no local y la organización de la resistencia. En las provincias se formaron hasta 13 Juntas supre-
mas provinciales. Para coordinar a todas las Juntas se forma la Junta Suprema Central.

En 1810 se convocaron las Cortes y la Junta Suprema Central fue sustituida por la
Regencia, compuesta por cinco miembros y convertida en órgano supremo de gobierno hasta
la llegada de Fernando VII. En 1812, las Cortes reunidas en Cádiz, elaboran la Constitución,
aprobada el 19 de marzo de 1812, que proclama la Soberanía Nacional, la separación de pode-
res y las libertades individuales. La Constitución es el primer documento del liberalismo espa-
ñol y su primera aplicación práctica. Sin embargo, el fin de la Guerra de la Independencia y el
regreso de Fernando VII, marcarán el inicio de una ardua sucesión de esfuerzos para hacer triun-
far finalmente la revolución liberal.

30
-Pág.102-
U . D . 5 . - S OCIEDAD Y CAMBIO EN EL TIEMPO

3.1.5.3. La independencia de la América española


La independencia de las colonias españolas de América debe enmarcarse en el proceso de la
crisis del Antiguo Régimen, siendo consecuencia de un proceso previo en el que se enmarcan
el ciclo revolucionario liberal (Emancipación de los EE.UU., Revolución francesa, Revolu-
ción en España durante la Guerra de la Independencia) y la formación de una conciencia crio-
lla emancipadora, consecuencia de las ideas liberales y el descontento con la administración
española.

El movimiento criollo es contrario a la administración centralizada y conservadurista de las


instituciones españolas americanas regidas por minorías que en muchas ocasiones buscan su
propio provecho. La reacción se producirá como consecuencia de estas situaciones y por los
acontecimientos de fines del siglo XVIII y principios del XIX en España.

En 1810 comienza a madurar el ideal emancipador y surgen las primeras tensiones (Argen-
tina, Uruguay, Méjico, Ecuador). Se van formando ejércitos por parte de las Juntas americanas
e inician relaciones con EE.UU. e Inglaterra. Paulatinamente se irán proclamando independen-
cias, publicando constituciones y adoptándose el régimen republicano.

El proceso seguirá esencialmente tres fases:

– Triunfo inicial de los pronunciamientos independentistas (1810-1814) durante el


desarrollo de la Guerra de la Independencia en España.
– Reacción de la metrópoli y pasajera restauración del régimen colonial (1814-1817).
Coincide con el final de la Guerra de la Independencia y se produce tras el envío de
10.000 hombres bajo las órdenes del General Pablo Morillo que consiguió recupe-
rar el control de casi todos los territorios, salvo de la zona de Argentina.
– Derrumbamiento definitivo de la soberanía española (1817-1824) cuya resistencia
se prolongará hasta 1824 (Batalla de Ayacucho) y en algunos lugares hasta 1826.

En el proceso emancipador que dará lugar al surgimiento de nuevas naciones cabe destacar
el papel jugado por los llamados libertadores: Simón Bolívar (Bolivia, Colombia, Venezuela.
Perú), Sucre (Bolivia, Perú), San Martín (Argentina, Chile, Perú), Belgrano (Argentina),
O'Higgins (Chile), José María Morelos (Méjico), etc.

3.1.6. LA CULTURA ROMÁNTICA

El romanticismo surgió en Alemania y se extendió por toda Europa. Frente al orden, equili-
brio y serenidad característicos del neoclasicismo se impone la emoción, el subjetivismo. El
romántico quiere romper con las formas y estilos, en todos los órdenes, de su pasado reciente.
El gusto por lo popular lleva a los románticos a estudiar las manifestaciones del sentir y la tra-
dición del pueblo, su historia y sus costumbres; ello redundará en la construcción de los nacio-
nalismos. La visión del mundo pasa por el propio yo, es tamizada por los sentimientos per-
sonales. Estalla el gusto por lo espiritual, misterioso, intimista
31
-Pág.103-
CIENCIAS SOCIALES (GEOGRAFÍA E HISTORIA)

Por su expresividad espiritual, el estilo arquitectónico preferido fue el gótico. En la escul-


tura, sin embargo, se prefirieron los ideales clásicos. Pero las artes que alcanzaron un mayor
desarrollo y que tuvieron una personalidad marcadamente romántica fueron la pintura, la lite-
ratura y la música. Frente al equilibrio y quietud de las composiciones neoclásicas, el roman-
ticismo impone los contrastes de colorido y la expresión violenta del movimiento, la tensión
en las figuras y los ambientes misteriosos. Baste citar a Géricault, Rousseau y, sobre todo, a
Delacroix y, como no, a Goya, pintor romántico y precursor de la pintura moderna.

Ese mismo talante impregna la literatura en la que se refleja esa afirmación del yo, el sub-
jetivismo, el sentimentalismo, el lirismo, el gusto por la naturaleza y los temas inspirados en el
pasado medieval. Las ideas liberales y nacionalistas están también presentes. Entre las figuras
más destacadas de la literatura romántica podemos citar a Lord Byron, Walter Scott, Shelley
(Inglaterra), Schlegel, Heine (Alemania), Victor Hugo, Musset, Gautier (Francia), Bécquer,
Espronceda, el duque de Rivas, José Zorrilla (España).

Por otro lado, si el final del siglo XVIII había conocido figuras sublimes de la música (Bach,
Heandel, Mozart), la música romántica, vía trascendental de expresión de los sentimientos
característicos de la época, tendrá entre sus destacados compositores a Haydn, Beethoven,
Schubert, Schumann, Mendelssohn, Strauss, Brahms, Wagner, Chopin, Liszt, Dvorak,
Tchaikowsky, Borodin, Mussorgsky, Rossini, Puccini, Verdi, Bizet, Debussy..., es decir, la
flor y nata de la historia universal de la música.

3.2. REVOLUCIÓN INDUSTRIAL, DESARROLLO CAPITALISTA


E IMPERIALISMO
3.2.1. LOS GRANDES AVANCES TECNOLÓGICOS
El rápido crecimiento de la población entre finales del siglo XVIII y el siglo XIX, generó la
necesidad de aumentar proporcionalmente la producción para abastecer de alimentos y otros
bienes a dicha población. Por un lado, se hacía necesario impulsar la producción de materias
primas lo que favoreció la mejora de los sistemas de producción agropecuaria e impulsó la acen-
tuación de la explotación colonial. Por otra parte, la necesidad de mejorar los rendimientos sir-
vió de acicate a las innovaciones técnicas orientadas a la producción textil, metalúrgica y de los
transportes, fundamentalmente. Finalmente, las nuevas fuentes de energía: carbón, electricidad
y petróleo (éste a finales de siglo), constituirán otro pilar sobre el que se edificará la revolución
industrial.

Los grandes inventos que revolucionarán la producción y los transportes serán:


– La máquina de vapor (Watt).
– El ferrocarril (Stephenson).
– El barco de vapor.
– Los altos hornos.

32
-Pág.104-
U . D . 5 . - S OCIEDAD Y CAMBIO EN EL TIEMPO

3.2.2. LA NUEVA POLÍTICA ECONÓMICA. LA EMPRESA


Las nuevas perspectivas exigen una nueva forma de organización de la economía. El traba-
jo se basa en principios distintos a la tradicional producción artesanal, ahora el obrero trabaja
en una factoría en la que participa en la fabricación de una parte del producto final, siguiendo
los principios de la división del trabajo y la producción en cadena.
Las nuevas empresas precisan grandes inversiones para sufragar los gastos en maquinaria,
locales, materias primas, energía, personal, transporte, política de mercado para la venta de sus
productos, etc. Surge así la empresa capitalista.
Aparecerán así las Sociedades Anónimas, en las que el capital es aportado por distintas per-
sonas que adquieren los títulos que certifican su participación: las acciones. Los beneficios de
la empresa se reparten proporcionalmente al número de acciones de cada uno.
Las compras y ventas de las acciones se realizan en la Bolsa que se convierte en una espe-
cie de termómetro de la economía pues los valores de las acciones bajarán o subirán en función
de que sean más apetecidos o menos, lo cual estará en relación con los beneficios que obtenga
la empresa, con su viabilidad y prosperidad.
Los Bancos serán el otro pilar del nuevo sistema económico. Su misión inicialmente es guar-
dar en depósito los ahorros de los particulares. Con ellos pueden negociar préstamos y realizar
inversiones.

3.2.3. EL MOVIMIENTO OBRERO


La sociedad industrial supone el triunfo del sistema clasista sobre el estamental, lo cual
implica movilidad social en función de la riqueza personal y la igualdad ante la ley de todos.
En la práctica, las nuevas estructuras económicas favorecen la aparición de fuertes contras-
tes entre las clases adineradas, propietarios de empresas, grandes capitalistas, y las masas de
obreros desheredados sujetos a un mísero salario, jornadas extenuantes y carentes de amparos
sociales que aseguren su sustento y el de su familia ante adversidades como enfermedades o
accidentes laborales.
Las ciudades crecen de forma desmesurada ante la atracción que supone la presencia de la
industria. Las masas obreras se hacinan en barrios insalubres, en casas pequeñas y mal provistas.
Estas situaciones de desequilibrio despertarán la conciencia social y llevarán a los obreros a
asociarse en sindicatos cuyos objetivos iniciales serán de tipo estrictamente laboral: reducción
de la jornada, mejora de los salarios, descansos, amparo ante enfermedades o accidentes, etc.;
pero pronto sus revindicaciones superarán el marco inmediato de las condiciones del trabajo
para elevarse a las situaciones políticas, empezando por la reclamación del derecho al voto. Así
terminarán apareciendo partidos políticos de militancia obrera que se asociarán internacional-
mente (Asociación Internacional de Trabajadores).

33
-Pág.105-
CIENCIAS SOCIALES (GEOGRAFÍA E HISTORIA)

Al amparo de este movimiento y alimentándolo, surgirán teorías sociales, políticas y eco-


nómicas de diversa índole:

Anarquismo.- Su principal ideólogo fue Bakunin que propugnaba la desaparición del


Estado y de toda forma de gobierno. La sociedad debía organizarse en pequeños grupos o comu-
nas autosuficientes.

Socialismo.- Karl Marx es su principal postulador (Manifiesto comunista, con Engels, y


El Capital). Su ideología incluye:

– Una interpretación materialista de la historia.


– La idea de la lucha de clases como medio para que el proletariado triunfe en sus aspi-
raciones.
– La revolución del proletariado para conquistar el Estado.
– La dictadura del proletariado, como fase posterior a la conquista del Estado y paso
previo para la eliminación de las clases y el establecimiento de la sociedad comu-
nista.

La trascendencia de estas ideas será decisiva en los inicios del s. XX marcando la lucha
social y política durante toda la edad contemporánea.

3.2.4. EL IMPERIALISMO
Uno de los aspectos fundamentales del s. XIX es la expansión del colonialismo. Tras la
emancipación de las colonias de la primera era del colonialismo (EE.UU. y la América españo-
la), se produce una segunda expansión protagonizada esencialmente por el Reino Unido y Fran-
cia y a la que se unirán otros países en busca de espacios donde obtener materias primas.
Las motivaciones de este empuje colonial son económicas, estratégicas y morales (apoyo a
pueblos atrasados...). De esta manera se constituyen inmensos imperios:

– Reino Unido: en África, Canadá, la India, Australia, Pacífico.


– Francia: en África (zona occidental y Madagascar), Indochina, Pacífico.
– Portugal: Angola, Mozambique...
– Bélgica: Zaire
– Holanda: Imperio basado más en pequeñas posesiones que acercarán a los núcleos de
interés comercial que en los grandes espacios.
– España: Sahara, Guinea Ecuatorial. Amén de las colonias que aún permanecían bajo
control español, como Cuba y Filipinas.
– Posteriormente se incorporarían Alemania (Togo, Camerún, Tanganika, Namibia) e
Italia (Libia, Somalia, Eritrea).

Las consecuencias del colonialismo llegarán casi hasta la actualidad, tanto para la metrópo-
li como para los territorios colonizados que irán accediendo a la independencia en el siglo XX.

34
-Pág.106-
U . D . 5 . - S OCIEDAD Y CAMBIO EN EL TIEMPO

3.2.5. CIENCIA Y CULTURA EN EL SIGLO XIX


Durante el siglo XIX la investigación científica creció considerablemente. Así se produje-
ron importantes avances en los estudios bacteriológicos lo que redundó en beneficio de la medi-
cina al descubrirse las causas reales de determinadas enfermedades y la manera de combatirlas
(Koch, sobre la tuberculosis; Pasteur, sobre la rabia). Dentro de los estudios y avances cientí-
ficos destaca la aparición de teorías novedosas como la argumentada por Darwin sobre la evo-
lución de las especies. Y entre los avances técnicos destaca la aparición del telégrafo y el telé-
fono.
La difusión de la cultura tuvo, por otro lado, un importante pilar en la generalización y difu-
sión de los periódicos, para lo cual fueron trascendentales las innovaciones técnicas aplicadas a
la impresión.
El arte del XIX, después de la etapa romántica, ya descrita, reacciona en busca de la reali-
dad, una realidad a veces cruda y dramática. La literatura refleja esa realidad que le rodea:
(Dickens, Balzac, Galdós, Dostoievski, Tolstoi). En sus páginas se retrata la sociedad industrial
con toda su crudeza. La pintura prefiere los temas que reflejan el mundo circundante, alejándo-
se de los paisajes fantásticos y misteriosos típicos del romanticismo. Al final del s.XIX, sin
embargo, conocerá la difusión de un movimiento artístico original: el Impresionismo. Las inno-
vaciones técnicas que aporta le confieren una personalidad especial. El juego de los colores, los
efectos de composición a base de manchas que adquieren forma con la distancia, el estudio de
la luz, definen dicha personalidad. Entre sus principales representantes figuran Monet, Renoir,
Dégas, Toulouse-Lautrec, Cézanne, ...

3.2.6. ASPECTOS SIGNIFICATIVOS DE LA EVOLUCIÓN POLÍTICA DEL SIGLO XIX


Entre los rasgos definitorios de la evolución de los hechos políticos, ya hemos hablado del
papel decisivo que juega el colonialismo y el imperialismo. En su base, y a la vez en su con-
trapunto, se encuentra el nacionalismo, que justifica la forja de los Imperios y motiva la lucha
de los pueblos por su independencia en contra de los mismos.

Así, en Europa, el último cuarto del siglo será testigo de la construcción de dos naciones,
divididas hasta entonces: Alemania (por obra de Otto von Bismarck) e Italia (por obra de
Cavour y de Garibaldi), que pasarán, tras su unificación, a la construcción de sus Imperios colo-
niales. Se consolida también el Imperio unificador del Centro y el este de Europa: el Imperio
Austro-Húngaro.

Pero los conflictos nacionalistas latentes en el seno de Imperios como el Austro-Húngaro, o


el turco; los problemas socio-económicos y las rivalidades entre imperios preparan el panora-
ma que definirá el siglo XX.

En España, la lucha entre liberalismo y absolutismo continuará durante el reinado de


Fernando VII con alternancias en el poder durante el mismo. Con el reinado de su hija Isabel II

35
-Pág.107-
CIENCIAS SOCIALES (GEOGRAFÍA E HISTORIA)

se consolidará el establecimiento de un Estado liberal, si bien la lucha política no acabará, con-


tinuando entre las tesis liberales moderadas y las progresistas. Esta pugna dilapidará el siste-
ma tras continuos pronunciamientos militares y movimientos revolucionarios, hasta la Revolu-
ción de 1868, que abrirá un periodo democrático en el que se ensayarán dos sistemas políticos:
la Monarquía democrática (Amadeo I), y la República Federal. Los males que pervivirán en
la España del s. XX, se dibujan ya en estos años: el problema religioso, la lucha de clases y el
regionalismo. Estos problemas y los de la lucha política violenta se velarán con el sistema de
la Restauración, establecido a partir de 1876 por Cánovas del Castillo, con la vuelta de los bor-
bones (Alfonso XII) y el diseño de un mecanismo político de alternancia pacífica de los par-
tidos institucionales: el conservador y el liberal. El final del siglo supondrá para España un
golpe espiritual de enormes y dramáticas consecuencias: la pérdida de Cuba y Filipinas. El
orgullo nacional quedará decisivamente dañado como refleja la literatura noventaiochista.

3.3. LAS GRANDES TRANSFORMACIONES Y CONFLICTOS


DEL SIGLO XX. REVOLUCIONES, GUERRAS MUNDIALES
Y DESCOLONIZACIÓN

3.3.1. LAS POTENCIAS DEL SIGLO XX. LA PRIMERA GUERRA MUNDIAL.


LA REVOLUCIÓN RUSA
Las potencias que se habían consolidado a finales del s. XIX inician el XX con la fuerza
que les daba su enorme peso económico, respaldado por una industria pujante y unos imperios
coloniales abastecedores de materias primas. Inglaterra, Francia, EE.UU., Imperio Austro-
Húngaro, Imperio Alemán, Italia, Rusia, Imperio Turco, se repartían el mundo y se dispu-
taban los mercados luchando entre sí por el predominio comercial y político. Dicha rivalidad
anunciaba una inevitable guerra que dilucidara los conflictos. Ante ello, las grandes potencias
emprendieron planes armamentistas de gran envergadura.
Finalmente, la guerra estalló en 1914. El escenario fue esencialmente Europa, donde se
enfrentaron dos coaliciones: de un lado los Imperios centrales (Austria-Hungría, Alemania y
Turquía) y de otro la Entente (Francia, Reino Unido, Italia, Rusia, Estados Unidos y Japón).
Hasta 1917, la guerra se mantenía en frentes bastante estables y sin que la balanza se incli-
nara claramente. En ese mismo año estallaron revoluciones y motines en distintos países, pero
especialmente en Rusia donde el movimiento revolucionario alcanzó cotas trascendentales que
llevaron a la larga a un cambio radical de las estructuras sociales, políticas y económicas.
Sobre la base de las ideas marxistas, Vladimir I. Ulianov, Lenin, forjó la ideología que
hizo posible que desde las estepas rusas se alzaran los campesinos contra el atraso y la incultu-
ra en la estaban sumidos, en contra de las previsiones del propio Marx, quien afirmaba que la
revolución se produciría sobre la base del proletariado industrial en el seno de alguna sociedad
industrializada, como Inglaterra.

36
-Pág.108-
U . D . 5 . - S OCIEDAD Y CAMBIO EN EL TIEMPO

Tras una primera fase revolucionaria de carácter moderado (menchevique), que intenta
establecer una sociedad democrática de corte burgués, se pasa a una etapa de exaltación y
radicalismo (bolcheviques). Se establecen los soviets como células de organización popular
que se incautan la propiedad de las industrias, se suprime la propiedad privada de la tierra y se
inicia la dictadura del proletariado.

Muerto Lenin, le sucede en el poder Stalin, que inicia una fase de persecuciones compara-
ble a la etapa del terror de la Revolución francesa, hasta crear un Estado monolítico en el que
él personaliza el poder. Algunos han identificado esta etapa como la del inicio de la dictadu-
ra del partido sobre el proletariado, afirmando que Rusia había pasado de un poder autocrático
zarista a un poder autocrático stalinista.

Paralelamente al desarrollo de la Revolución rusa, continúa la Guerra mundial en la que inter-


viene desde 1917 EE.UU., inclinando decisivamente la balanza en favor de la Entente. La paz
de 1919 impuso duras condiciones a Alemania, que perdió parte de su territorio, y desarticuló el
imperio austriaco y el turco. Las naciones surgidas como fruto de las negociaciones de despa-
cho, no siempre reflejaban la real distribución de etnias y naciones, lo cual está en la base de
conflictos contemporáneos como el de la antigua Yugoslavia, creada ahora con retazos del
Imperio Austro-Húngaro y del turco.

Se abre el periodo de entreguerras, que se inicia con una fase de expansión económica que
durará hasta 1929 cuando una nueva y dura crisis abrirá los caminos hacia un nuevo conflicto.
Las condiciones de paz, la continuación de las tensiones económicas y las ideologías acrisola-
das durante estos años serán las bases de una nueva Guerra.

3.3.2. LAS IDEOLOGÍAS. LA SEGUNDA GUERRA MUNDIAL


Junto a las tradicionales ideologías conservadora y reformista, el siglo XIX había visto el
nacimiento de la ideología revolucionaria socialista y la anarquista. En el siglo XX, a dichas
ideologías se añadirán nuevas corrientes:

El comunismo soviético constituyó, desde que en 1924 alcanzó el poder Stalin, un régimen
totalitario con un líder carismático. El control absoluto del partido único y la casi total desapa-
rición de la propiedad privada, son la base de un Estado que se articula, bajo la férrea mirada
del partido, en sóviets y sobre la acumulación del poder en lugar de su división.

El fascismo surge en Italia, en sus formulaciones pragmáticas, en 1922 con Benito Musso-
lini. La oposición al socialismo y al capitalismo es su primera disposición. El peligro de una
revolución moviliza a sus bases en una especie de “revolución contrarrevolucionaria”. Los
intereses individuales se subordinan a los del Estado (Estado totalitario) y se exalta la figura del
jefe, del guía del pueblo (Duce: conductor). Las pasadas glorias de la nación son recordadas con
nostalgia y se reclama el derecho a reconstruir el Imperio del pasado aun a costa de la paz.

El nazismo alemán tiene en Adolf Hitler a su inspirador. Su ideología aparece diseñada en su


obra Mein Kampf (Mi lucha). Básicamente coincide con los principios fascistas: omnipresencia

37
-Pág.109-
CIENCIAS SOCIALES (GEOGRAFÍA E HISTORIA)

del Estado, exaltación del jefe, imperialismo; mas el nazismo aporta otro pilar ideológico: el
racismo, la aseveración de que la raza aria es superior al resto. Precisamente uno de los aspec-
tos más sobresalientes, si no el mayor, de su plasmación política será la sistemática persecución
de las llamadas razas inferiores y de todos aquellos grupos que se desviaran de la pureza predi-
cada.

Las condiciones impuestas a Alemania tras la I Guerra Mundial, fueron uno de los condi-
cionantes que favorecieron el ascenso de una ideología como la nazi que propugnaba la afir-
mación nacional y la superioridad alemana. Tras el triunfo del partido Nacional-socialista,
Alemania emprendió su rearme y comenzó su estrategia de expansión buscando la recuperación
de los espacios perdidos y la constitución de un Estado fuerte con su espacio vital asegurado.

Las democracias occidentales contemplaban con preocupación, pero con cierta pasividad, el
avance alemán, que iba incorporando Austria, los Sudetes y estableciendo el protectorado sobre
Bohemia y Moravia. Finalmente, en septiembre de 1939, la invasión de Polonia arrastró a
Inglaterra y a Francia a declarar la guerra.

Se formaron, como en la Primera Guerra, dos bandos. El Eje, estaba integrado por Alema-
nia, Italia y Japón, además de otros estados satélites de Alemania como Rumanía, Hungría, Fin-
landia, Estonia, Letonia y Lituania. El bando oponente, el de los Aliados, por Reino Unido,
Francia, Polonia, la Unión Soviética y, posteriormente, EE.UU., junto con otros países de la
Commonwealth británica (Canadá, Australia, Nueva Zelanda), China, Bélgica, Holanda, Brasil,
Yugoslavia, Noruega, Grecia…

Las victorias iniciales correspondieron al Eje, tanto en Europa como en África, por parte
alemana, como en el Este asiático y el Pacífico, por parte japonesa. Pero a partir de 1942 los
aliados consiguieron contener el avance y en 1943 comienzan sus avances en Italia y los
ejércitos alemanes son derrotados en Rusia. Finalmente, en 1944 se produce la gran contrao-
fensiva aliada tras el desembarco de Normandía que lleva en pocos meses a la capitulación
de Berlín.

En el Pacífico, la contraofensiva aliada se inició en 1942. Paulatinamente, Japón se fue


replegando hasta que en agosto de 1945, tras el lanzamiento de dos bombas atómicas (Hiroshi-
ma y Nagasaki), tuvo que capitular.

3.3.3. EL MUNDO DE POSGUERRA

El fin de la guerra en Europa había contemplado el avance de las fuerzas aliadas sobre Ale-
mania desde dos frentes. Desde el occidente las tropas francesas, inglesas, norteamericanas y
de otros países, y por el este, las del Ejército Rojo. Este avance fue acompañado de la exporta-
ción del sistema político stalinista. De este modo, el final de la Guerra condujo a la división
de Europa en dos bloques antagónicos separados por el llamado Telón de Acero, cuya expre-
sión material era el muro de Berlín, que separaba el sector de la capital alemana controlado por
la URSS, de los sectores aliados (francés, británico y norteamericano).

38
-Pág.110-
U . D . 5 . - S OCIEDAD Y CAMBIO EN EL TIEMPO

La proliferación del amenazante armamento nuclear y la tensión permanente entre las


superpotencias surgidas de la posguerra (EE.UU. y la URSS), sumió al mundo en la llamada
Guerra Fría. El mundo se dividió en dos bloques, en un tablero en el que las superpotencias
movían sus fichas para, por medio de movimientos revolucionarios o golpes de Estado, promo-
ver la afiliación a uno u otro bloque de los diferentes países.

3.3.4. LA DESCOLONIZACIÓN
Uno de los aspectos más sobresalientes del siglo XX ha sido la paulatina desaparición de los
imperios coloniales. La desintegración de dichos imperios se produjo por diferentes vías:

– En unos casos la independización de los territorios dominados se produjo de forma


pacífica, con el paulatino acceso al control de la administración de los nacionalistas.

– En otros, la independencia se produce de forma inmediata, casi por imposición de la


antigua metrópoli lo que sorprende a las nuevas naciones sin cuadros dirigentes.

– Y en otros casos, serán movimientos revolucionarios los que, a través de guerras de


liberación, conduzcan a la independencia.

De esta forma, a lo largo del siglo, los territorios de los antiguos imperios fueron accedien-
do a la Independencia:

En Asia, el Imperio británico de la India se independiza y divide en India, Pakistán, Sri


Lanka y Bangla-Desh. En Indochina, tras una guerra de descolonización contra Francia, se for-
maron diversos países (Laos, Camboya, Vietnam).

En África, se constituyeron una serie de Estados islámicos en el norte (Egipto, Libia, Arge-
lia, Túnez, Marruecos, Sudán), y un mosaico de países en el África negra, países cuyas fronte-
ras no coincidían por lo general con criterios raciales o culturales específicos sino que se debían
normalmente a la herencia de los límites territoriales de las colonias.

La postura de las metrópolis ante el proceso fue distinta.

El Reino Unido adoptó una actitud de aceptación de la descolonización propiciando incluso


la constitución de regímenes autónomos en los que los británicos ejercían un papel de tutela
transitoria hasta la definitiva independencia. Esta actitud permitió la constitución de la Com-
monwealth, en la que se integraban los Estados surgidos del Imperio Británico y que tenían
como elementos vinculadores la Corona británica y una lengua y cultura comunes.

Francia intentó la asimilación cultural y política, lo que provocó tensiones que facilitaron los
movimientos emancipadores, aunque algunas de las colonias prefirieron mantenerse unidas a
Francia como Departamentos o Provincias.

39
-Pág.111-
CIENCIAS SOCIALES (GEOGRAFÍA E HISTORIA)

Portugal mantuvo sus colonias como parte integrante del Estado portugués. La guerra de
liberación en Angola sólo tuvo resultados tras la Revolución del 74 en la metrópoli, que facili-
tó la independización.

Tras la descolonización, muchos de los países surgidos mantuvieron vínculos culturales y


económicos con las antiguas metrópolis. Otros terminaron por quedar vinculados a las nuevas
grandes potencias, lo que generó una nueva forma de sumisión neocolonialista.

Los problemas posteriores a la independencia sumieron a los nuevos Estados en una situa-
ción de atraso cultural y económico. La mayoría de los Estados carecían de una industria impor-
tante, apenas tenían recursos de capital y el hambre y el analfabetismo se enseñoreaban de sus
tierras y sus pueblos. Se dibuja así un Tercer Mundo deprimido cuyos problemas aún hoy no
han sido resueltos.

3.4. CAMBIOS Y TRANSFORMACIONES EN LA ESPAÑA


DEL SIGLO XX

Al comienzo del siglo XX, España seguía siendo un país básicamente agrario; la mayoría de
la población activa trabajaba en el sector primario, la empleada en el sector secundario se con-
centraba en la construcción y la industria textil, y las que trabajaban en el sector terciario lo
hacían en el servicio doméstico.

La población española, en el primer tercio del siglo XX comenzó a crecer de forma soste-
nida, y pasó de 18,5 millones en 1.900 a 23,5 millones en 1.930. En ello influyó la reducción
de la mortalidad, y la mejora de la sanidad, la higiene y la alimentación.

La estructura social también sufrió una gran transformación. Con el envejecimiento del
comercio, la industria y la banca, aumentó la clase burguesa, también creció la clase media y el
número de obreros industriales. Las ciudades crecieron notablemente debido al incremento de
la emigración de los campesinos a las ciudades. Esto provocó una gran transformación en ciu-
dades como Barcelona, Madrid, Valencia, Bilbao, etc.

Con la llegada al trono de Alfonso XIII en 1.902 se produjo una continuación de la etapa
anterior. La Constitución de 1.876 continuó vigente y se mantuvo la alternancia entre los con-
servadores y liberales. En los años de cambio de siglo se produjo el relevo de los dos grandes
políticos de la primera etapa de la restauración: Cánovas y Sagasta. A éstos, le sustituyeron
Antonio Maura y José Canalejas. Los primeros cambios los protagonizó Antonio Maura, líder
del partido conservador, intentando acabar con el caciquismo a través de la reforma de la ley
electoral, de la administración local, obteniendo poco éxito. Más tarde, el liberal José Canale-
jas inició una política descentralizadora, que levantó gran polémica.

40
-Pág.112-
U . D . 5 . - S OCIEDAD Y CAMBIO EN EL TIEMPO

Se puede resumir el período de 1.902 a 1.923 como la crisis de la restauración, con la suce-
sión de períodos de crisis de distinto signo que no dieron soluciones válidas a los problemas del
momento: los conflictos sociales, el incremento del paro, la subida de los precios, la sucesión
de derrotas militares en Marruecos, entre los que destacó el desastre de Annual de 1.921. Entre
1.917 y 1.923 hubo trece cambios de gobierno y treinta crisis parciales; la política española
parecía carecer de rumbo.

Ante esta situación, en 1.923 el General Primo de Rivera dio un golpe de Estado, suspen-
diendo la Constitución de 1.876 y disolviendo las Cortes. Este régimen autoritario marcó un
período de estabilidad social hasta 1.927. Después, comenzó a ser criticado y recibió una fuer-
te oposición hasta dimitir en enero de 1.930, sin el apoyo de la monarquía.

Más tarde, el 14 de abril de 1.931 se proclamaba la II República. La vida política de esta


etapa giró en torno a un gran número de partidos y de tendencias políticas. En diciembre de
1.931 se aprobó una nueva Constitución y se eligió a Alcalá Zamora como presidente de la
República, y a Manuel Azaña jefe del gobierno de coalición de republicanos y socialistas. Tras
unos años de inestabilidad y de imposible gobernabilidad, la fuerte tensión entre izquierdas y
derechas se materializó en la revolución de octubre de 1.934.

Tras dos largos años, en 1.936 hubo nuevas elecciones y triunfó el frente popular (coalición
de izquierdas).

La situación empeoró día a día y llegó a su radicalización total. El 17 de julio de 1.936 se


produjo una sublevación militar en Africa, donde el General Franco asumió el mando. De ahí
se extendió a la Península y el golpe de estado dividió a España en dos bandos. Después de tres
años de luchas fratricidas y cruentas la guerra terminó en abril de 1.939, con la victoria por parte
del bando nacionalista.

La España, bajo la dictadura del General Franco, pasó por distintas etapas. La posguerra
estuvo protagonizada por el aislamiento internacional, el hambre y la autarquía económica. Pos-
teriormente, la orientación de la política exterior hacia los EE.UU. terminó con el aislamiento
y posibilitó el inicio del crecimiento económico, dirigido por tecnócratas.

A partir de los años 60 se produce el "boom" de la industrialización y España se convirtió


en una sociedad industrial parecida a la de la Europa occidental. La crisis de los 70 afectó de
forma importante a España; la salud del Jefe de Estado se debilitaba y en el aparato del gobier-
no se oían voces favorables al inicio de reformas de carácter democrático. Muerto Franco el 20
de noviembre de 1.975, fue coronado como Rey de España D. Juan Carlos I (ya que España era
legalmente un reino desde la aprobación de la Ley de Sucesión en la Jefatura del Estado de
1.947).

El 22 de noviembre de 1975 tuvo lugar la proclamación de Juan Carlos I. A partir de aquí se


acelera el proceso de reformas impulsado por el propio monarca y por el Presidente Adolfo Suá-
rez, con la colaboración de las principales fuerzas políticas democráticas. La Ley para la Refor-

41
-Pág.113-
CIENCIAS SOCIALES (GEOGRAFÍA E HISTORIA)

ma Política de 1976, abrió un periodo constituyente, cuyos pasos principales fueron las elec-
ciones de 1977, en las que ganó el partido Unión de Centro Democrático (UCD) creado y lide-
rado por Suárez, y la aprobación de la Constitución de 1978. España queda establecida como
una monarquía parlamentaria con dos cámaras, el Congreso y el Senado. Se establece la divi-
sión de poderes, se reafirman las libertades democráticas y se reconocen una serie de derechos
para todos los españoles, así como la diversidad de pueblos que componían el Estado, permi-
tiendo una nueva organización territorial.

Una vez aprobada la Constitución en 1978, la vida política ha estado marcada por el fun-
cionamiento democrático a pesar de algunas dificultades, entre las cuales cabe destacar la pre-
sencia casi constante de la banda terrorista ETA, que desde sus primeras actuaciones sangrien-
tas en la sociedad española hasta la actualidad ha pretendido, sin éxito, terminar con la
convivencia democrática. Una clara expresión de la normalidad democrática que se instala en
España desde 1978 es el hecho de que gobiernos de distintos partidos políticos se han ido suce-
diendo y han contribuido a consolidar la democracia.

A finales de 1980 Suárez presentó su dimisión a causa de la división interna de su propio


partido. El día 23 de febrero de 1981, durante el debate de investidura de Leopoldo Calvo Sote-
lo, sucesor de Suárez, se produjo un intento de golpe de estado que, gracias a la firme actitud
del rey apelando al acatamiento del orden constitucional, fracasó.

Las elecciones de 1982 supusieron un importante cambio político, llegando al poder el Par-
tido Socialista Obrero Español (PSOE), grupo liderado por Felipe González y repitiendo man-
dato en sucesivas convocatorias electorales, 1986, 1989 y 1993. La política social experimentó
un gran auge, y la política exterior alcanzó una gran proyección que supuso la incorporación
de España a la OTAN, además del ingreso en la Comunidad Económica Europea (CEE) en
1986.

En 1996, de acuerdo con la lógica de alternancia democrática, el Partido Popular (PP) alcan-
zó el gobierno de España con José María Aznar como presidente, repitiendo victoria en 2000.
La política española de estos años continuó la línea de consolidación democrática y de presen-
cia institucional de España en el exterior. En política económica, el logro fundamental fue el
control del déficit público que permitió a España sumarse a los países de la Unión Europea que
adoptaron la moneda única, el euro, en enero de 2002.

Dentro del clima general de normalidad democrática, las elecciones generales de 2004 supu-
sieron un refrendo del espíritu democrático de alternancia política, otorgando al Partido Socia-
lista, bajo la dirección de José Luis Rodríguez Zapatero, el gobierno de la nación. En los días
anteriores a estas elecciones, el 11 de marzo, se produjo el atentado terrorista de más magnitud
en España, con 192 muertos y más de 1.500 heridos. Se atribuye el mismo al terrorismo
islamista.

42
-Pág.114-
U . D . 5 . - S OCIEDAD Y CAMBIO EN EL TIEMPO

RESUMEN
EL TIEMPO HISTÓRICO
La medida del tiempo
El calendario que hoy usamos deriva del calendario romano, regularizado por Numa
Pompilio, reformado por Julio Cesar (calendario juliano) y fijado definitivamente por el Papa
Gregorio XIII (calendario Gregoriano), en 1582.
La Era Cristiana parte del nacimiento de Jesucristo, situado errónemente por Dionisio el
Exiguo en el 753 desde la fundación de Roma. En función de esto, los años se citan como
“antes” o “después de Cristo”.
El cambio y la continuidad a través del tiempo.
La Historia de la Economía ha pasdo por las fases de Economía depredadora, Revolución
Neolítica, Economías esclavistas, Feudalismo, Mercantilismo, Capitalismo y Socialismo, hasta
llegar a una definición globalizadora en la actualidad.
La vida cotidiana y la marginación de la mujer, las creencias religiosas y las fórmulas de
organización política y social, entre otros aspectos, han evolucionado con el paso del tiempo.
EVOLUCIÓN Y DESARROLLO EN LA HISTORIA DEL ARTE
El Arte también ha cambiado con el tiempo. Algunas de las grandes manifestaciones de esa
evolución han sido: Altamira, las Pirámides de Gizéh, el Partenón, el Coliseo, la Catedral de
Chartres, la Basílica de San Pedro, la Capilla Sixtina, la Meninas, el Palacio de Versalles o el
Guernica de Picasso.
CAMBIO SOCIAL Y REVOLUCIÓN EN LA ÉPOCA CONTEMPORÁNEA
La crisis del Antiguo Régimen y las Revoluciones liberales burguesas marcan el inicio de la
llamada Edad Moderna. El absolutismo, la sociedad estamental y el absolutismo económico
serán sustituidos por el liberalismo, la sociedad de clases y el capitalismo.
La independencia de los Estados Unidos de América (Declaración de 1776, Constitución de
1787), será la primera implantación práctica de las ideas liberales.
La Revolución Francesa (1789-1792), la gran revolución liberal, dejó paso a la aparición de
la figura de Napoleón. La gran expansión europea que protagonizó se cerró tras la batalla de
Waterloo y el destierro a Santa Elena.
En España, la revolución liberal comienza en el marco de la Guerra de la Independencia
frente a Francia (1808 - 1814) y fragua en la publicación de la Constitución de 1812. La
extensión de la revolución liberal a las posesiones americanas desembocó en su inde-
pendencia.

43
-Pág.115-
CIENCIAS SOCIALES (GEOGRAFÍA E HISTORIA)

En el siglo XIX se producen grandes avances científicos y tecnológicos que posibilitan la


Revolución Industrial y los cambios en los parámetros de las relaciones sociales y el sistema
económico. Aparecen ideologías basadas en las condiciones de trabajo y las circustancias socie-
conómicas (socialismo, anarquismo).
La necesidad de materias primas para la industria empuja a los países europeos a una cre-
ciente dominación territorial allende el mar construyendo poderosos imperios ultramarinos.
Las contradicciones del sistema y las rivalidades territoriales conducen a la I Guerra Mundial
(1914-1919) y el empuje de la ideología marxista fragua en la Revolución rusa (1917).
Las tensiones ideológicas (marxismo - fascismo - capitalismo) y el irresuelto problema terri-
torial provocan la II Guerra Munial (1939 - 1945), tras la cual la bipolarización del mundo entre
las dos grandes potencias, EE.UU. y la URSS, generá la llamada Guerra Fría.
La descolonización, lejos de solucionar el problema de los países dominados por las poten-
cias europeas, condujo a la aparición de nuevas formas de dependencia política y económica
(neocolonialismo) y al enquistamiento de las nuevas sociedades en el llamado Tercer Mundo.
España, tras la definitiva implantación del Estado Liberal de la mano de la Revolución de
1868 (“La Gloriosa”), conocerá un período de estabilidad política (la época de la Restauración),
que encubre una creciente crisis, aderezada con la pérdida de las últimas colonias (Cuba y Fili-
pinas 1898) que desembocará en una Dictadura (1923 - 1930), un intento de construcción de
una democracia moderna con la II República (1931 - 1936), una Guerra Civil (1936 - 1939) y
una nueva Dictadura (1939 - 1975). Después, la restauración monárquica en la persona de Juan
Carlos I trajo aparejada la reforma política y el establecimiento, de la mano de la Constitución
de 1978, de una democracia plena y, más tarde, la incorporación a la Unión Europea.

44
-Pág.116-
U . D . 5 . - S OCIEDAD Y CAMBIO EN EL TIEMPO

EJERCICIOS DE AUTOCOMPROBACIÓN

1. La regularización del calendario romano primitivo se atribuye a:

A. Rómulo B. Numa Pompilio

C. Julio César D. Augusto

2. Como punto de partida de la llamada Era cristiana fue establecido el año 753 de la fundación de
Roma. ¿Quién calculó y estableció esa fecha?

A. Jesucristo B. Gregorio XIII

C. Dionisio el Exiguo D. San Pablo

3. La primera puesta en práctica de las ideas liberales tuvo lugar como consecuencia de:

A. La Revolución francesa

B. El levantamiento independentista en España tras la invasión de Napoleón

C. La declaración de independencia de los Estados Unidos

D. La instauración de la República de Weimar

4. ¿Cuál de los siguientes personajes se relaciona con la Revolución francesa?

A. Thomas Jefferson B. Jean Jacques Rousseau

C. Robespierre D. Washington

5. Indique la afirmación correcta respecto a Napoleón.

A. Se proclamó emperador tras el Golpe de Estado del 18 Brumario

B. Era Natural de la Isla de Elba

C. Su mayor triunfo fue la batalla de Waterloo

D. Fue coronado emperador por el Papa en 1804

6. ¿Qué general mandaba las tropas francesas en España cuando se produjo la invasión en los prime-
ros momentos de la sublevación española?

A. El General Castaños B. El propio Napoleón

C. El General Murat D. El general Champolion

45
-Pág.117-
CIENCIAS SOCIALES (GEOGRAFÍA E HISTORIA)

7. Entre los llamados Libertadores de la América Española NO se encuentra:

A. Pablo Morillo B. Simón Bolívar

C. Sucre D. San Martín

8. Entre las figuras más destacadas de la literatura romántica NO se encuentra:

A. Shelley B. Lord Byron

C. El duque de Rivas D. Dvorak

9. El Manifiesto Comunista fue obra de:

A. Lenin y Stalin B. Borodin y Bakunin

C. Lenin y Marx D. Marx y Engels

10. ¿Cuál de las siguientes relaciones potencia imperial - colonia es falsa?

A. Reino Unido - Canadá B. Bélgica - Angola

C. Portugal - Mozambique D. Francia - Indochina

11. Indique cuál de las siguientes afirmaciones NO se corresponde con algún aspecto significativo de la
evolución política del siglo XIX.

A. Alemania se unifica gracias a la política de Otto von Bismarck

B. En España, tras la Revolución de 1876 se establece la I República

C. Cavour y Garibaldi consiguieron la unificación de Italia

D. Cánovas del Castillo fue el artífice de la Restauración borbónica en España

12. Forjó la ideología que hizo posible que fuera el campesinado y no el proletariado urbano el artífice
de la primera revolución comunista, en contra de las tesis de Marx. Estamos hablando de:

A. Josip Stalin B. Engels

C. Emmanuel Kant D. Lenin

13. El acontecimiento que arrastró a Francia e Inglaterra a declarar la guerra a Alemania y que, en con-
secuencia, desató la II Guerra Mundial, fue:

A. La incorporación de Austria a Alemania

B. El ataque alemán a Pearl Harbour

C. La invasión alemana de Polonia

D. El lanzamiento de las bombas atómicas sobre Hisroshima y Nagasaki

46
-Pág.118-
U . D . 5 . - S OCIEDAD Y CAMBIO EN EL TIEMPO

14. Entre los países que surgieron como consecuencia de la desintegración del Imperio Británico de la
India NO se encuentra:

A. Pakistán B. Camboya

C. Sri Lanka D. Bangla Desh

15. ¿Cuál de los siguientes grupos de acontecimientos de la historia de España del siglo XX están cita-
dos cronológicamente?

A. Proclamación de la II República, Dictadura del General Primo de Rivera, Coronación de Juan


Carlos I

B. Semana Trágica de Barcelona, Guerra Civil, Ley de Sucesión en la Jefatura del Estado

C. Proclamación de la II República, Guerra de Marruecos, Ley para la Reforma Política

D. Proclamación de Franco como Jefe del Estado, Guerra de Marruecos, Proclamación de la II


República

47
-Pág.119-
CIENCIAS SOCIALES (GEOGRAFÍA E HISTORIA)

RESPUESTAS A LOS EJERCICIOS

1. B

2. C

3. C

4. C

5. D

6. C

7. A

8. D

9. D

10. B

11. B

12. D

13. C

14. B

15. B

48
-Pág.120-
U . D . 6 . - D IVERSIDAD C U LT U R A L

ÍNDICE
OBJETIVOS . . . . . . . . . . . . . . . . . . . . . . . . . . . . . . . . . . . . . . . . . . . . . . . . . . . . . . . . . . . . .2

INTRODUCCIÓN . . . . . . . . . . . . . . . . . . . . . . . . . . . . . . . . . . . . . . . . . . . . . . . . . . . . . . . . .3

MAPA CONCEPTUAL . . . . . . . . . . . . . . . . . . . . . . . . . . . . . . . . . . . . . . . . . . . . . . . . . . . . .4

DESARROLLO DE CONTENIDOS

1. ELEMENTOS DETERMINANTES DE UNA CULTURA . . . . . . . . . . . . . . . . . .5

2. DIVERSIDAD Y RELATIVISMO CULTURAL . . . . . . . . . . . . . . . . . . . . . . . . .12

RESUMEN . . . . . . . . . . . . . . . . . . . . . . . . . . . . . . . . . . . . . . . . . . . . . . . . . . . . . . . . . . . .16

EJERCICIOS DE AUTOCOMPROBACIÓN . . . . . . . . . . . . . . . . . . . . . . . . . . . . . . . . . .18

RESPUESTAS A LOS EJERCICIOS . . . . . . . . . . . . . . . . . . . . . . . . . . . . . . . . . . . . . . . . .20

-Pág.121-
CIENCIAS SOCIALES (GEOGRAFÍA E HISTORIA)

OBJETIVOS
Al finalizar el estudio de esta Unidad Didáctica, el alumno será capaz de:
• Asimilar el concepto de cultura y reflexionar sobre los aspectos que definen su
cultura y los analice a la luz de las distintas corrientes sociológicas.

• Conocer los rasgos identificativos de distintas culturas y los contrastes con los
propios.

-Pág.122-
U . D . 6 . - D IVERSIDAD C U LT U R A L

INTRODUCCIÓN
I nicialmente, enfrentarnos al término cultura puede hacernos caer en la tentación de
identificarlo con el conjunto de manifestaciones estéticas, visuales o acústicas, o con-
ceptuales que emanan del ámbito que llamamos intelectual. Sin embargo, este (o estos)
aspecto (o aspectos), que, obviamente, quedan incluidos en el término cultura, no son los úni-
cos que la constituyen.

Efectivamente, cultura es en realidad TODO lo que conforma una sociedad humana, las
formas de vestir, los gestos que acompañan la expresión oral, la propia expresión oral,
parte a su vez del sistema de símbolos que se ponen en juego para la comunicación, las
costumbres de comportamiento, etc.

Esta es la cultura a cuyo estudio nos acercamos en este tema. Es inevitable que partamos
de la base de NUESTRA CULTURA, es decir, del conjunto de aspectos que definen
nuestra sociedad puesto que, por ser parte de nuestra mentalidad, será más fácil asimilar
sus significados. Mas conviene, en cualquier caso, tras la abstracción de la misma, acer-
carnos al conocimiento de las soluciones a las que han llegado OTRAS CULTURAS, sin
que en todo caso pretendamos hacer de este tema un tratado enciclopédico de sociología
o de antropología cultural.

-Pág.123-
CIENCIAS SOCIALES (GEOGRAFÍA E HISTORIA)

M A PA C O N C E P T UA L

LA CULTURA

LOS CAMBIOS DE LA
ELEMENTOS CULTURA
DETERMINANTES
DE UNA CULTURA CULTURA Y
ADAPTACIÓN AL MEDIO

ESTRUCTURA SOCIAL

LA FAMILIA,
DEFINICIÓN Y
TRASCENDENCIA

DIVERSIDAD Y
RELATIVISMO LAS FUNCIONES DE
CULTURAL LA FAMILIA

MODELOS
FAMILIARES
U . D . 6 . - D IVERSIDAD C U LT U R A L

1. ELEMENTOS DETERMINANTES DE UNA CULTURA

1.1. LA CULTURA
El concepto de cultura, como decíamos en la introducción, es mucho más amplio que el de
ese conjunto de manifestaciones características de la vida intelectual, de esas que nos llevan a
emplear términos como el de culto o inculto. La cultura engloba todo lo que el hombre elabo-
ra como individuo integrado en una sociedad y que forma parte de la realidad misma de esa
sociedad.

Así, es cultura el conjunto de obras literarias, o las manifestaciones pictóricas, o las obras
musicales; pero también lo es el lenguaje oral, el sistema de símbolos de las señales de tráfico,
las herramientas que utiliza un fontanero, la forma de vestirse, los elementos utilizados y la
forma específica de la higiene personal, la manera de saludarse, los tipos de relación personal,
etc.

El hombre es el único ser vivo capaz de dotarse de una cultura. Su comportamiento está
determinado en buena medida por la cultura aprendida; pero como fruto de su reflexión o de
complejos mecanismos sociales puede cambiar algunos de sus aspectos. Sin embargo, los com-
portamientos de los demás animales están condicionados por el instinto, por las respuestas a los
estímulos externos y aunque algunos animales sean capaces de aprender nuevos comporta-
mientos ante nuevos estímulos, ambos están determinados por los propios instintos y quien
enseña esas nuevas asociaciones estímulo - respuesta es el hombre (caso de la educación a
perros y otros animales domésticos).

1.1.1. DEFINICIÓN DE CULTURA


El antropólogo inglés Tylor (1832-1917), definió cultura como “esa compleja totalidad que
comprende conocimientos, creencias, arte, moral, ley, costumbre y toda otra habilidad y hábi-
tos adquiridos por el hombre como miembro de una sociedad”. De un modo mucho más senci-
llo o generalizador, y quizás por ello más discutible, el sociólogo Johnson la define como “un
conjunto de pautas abstractas sobre la vida y la muerte”.

Efectivamente, llegamos a la conclusión de que cultura es todo lo que hace o piensa el


hombre dentro de su grupo social, existiendo una dualidad pues eso que constituye la cultura
es algo abstracto que se manifiesta en hechos materiales, hechos materiales que son también
cultura. Así pues, hay unos aspectos que tienen que ver con el mundo de las ideas y otros que
son manifestaciones materiales, existiendo, obviamente, una relación entre ambas realidades
si bien, dependiendo de las escuelas sociológicas, dichas relaciones son interpretadas de
modos diversos.

5
-Pág.125-
CIENCIAS SOCIALES (GEOGRAFÍA E HISTORIA)

Por otra parte, de la propia definición de cultura y de la observación de la sociedad actual,


llegamos a la conclusión de que resulta difícil concebir la existencia de una única cultura
humana, pues las soluciones ideológicas y materiales de los diferentes grupos humanos que
existen en el mundo se nos muestran como algo especialmente variopinto. Incluso, aun cuan-
do llegáramos a establecer un número reducido de culturas, basándonos en manifestaciones
más o menos generalizadas y comunes a grupos amplios, deberíamos, al menos, llegar a la con-
clusión de que dentro de las grandes culturas existen innumerables subculturas, definidas por
circunstancias espaciales: regiones, campo - ciudad, o sociales: clases, estamentos, grupos pro-
fesionales, etc…

1.1.2. ELEMENTOS O ESTRUCTURA DE LA CULTURA


1.1.2.1. Valores
Los valores son criterios de definición de lo bueno y de lo malo, de lo positivo o negativo
de cada aspecto, cada idea, cada acción. El conjunto de los valores de una cultura constituye su
sistema de valores, y dicho sistema nos permite distinguir unas culturas de otras. El individuo
se encontrará más integrado en la cultura en la que vive en la medida que acepte como suyo el
sistema de valores que la caracteriza.

Los valores comunes se pueden presentar en diferentes niveles, no únicamente en el nivel


que se califica como macrosociológico, el que corresponde al de las naciones o los Estados, sino
también al nivel que se llama microsociológico, es decir, el de las subculturas de las que hablá-
bamos más arriba. Efectivamente, en una sociedad entendida en el sentido de Estado o Nación
podemos observar una serie de valores comunes, sin embargo, cuanto más amplia sea esa socie-
dad más probabilidades tendremos de encontrarnos con grupos de valores que no son comunes
a todos. Así, por ejemplo, los valores religiosos, que pueden ser muy comunes y homogéneos
en determinados grupos y, por tanto, definir con claridad uno de esos valores, no son, en otros
grupos culturales, ni homogéneos ni comunes. Cuanto más reducido y complejo sea el grupo
humano, más coherente, homogéneo y común será su sistema de valores. Cuanto más comple-
jo, más tendremos que tener en cuenta la formación de lo que llamábamos subculturas.

1.1.2.2. Normas
Las normas materializan el sistema de valores en unos contenidos específicos de lo que hay
que hacer y lo que no hay que hacer. Estas normas ordenan actuar de un modo determinado ante
cada circunstancia y el incumplimiento de tales normas lleva aparejada la correspondiente san-
ción social. Estas sanciones, infligidas por los individuos que observan la conducta a quienes
incumplen las normas, pueden ser de infinidad de intensidades dependiendo de la gravedad del
incumplimiento, siempre dentro del correspondiente sistema de valores: puede ser simplemen-
te un comentario crítico o llegar a poner en práctica la violencia física.

6
-Pág.126-
U . D . 6 . - D IVERSIDAD C U LT U R A L

Estas normas y sanciones de las que hablamos no deben confundirse con las normas y san-
ciones jurídicas o legales pues su carácter es informal y son aplicadas por los propios miembros
del grupo en tanto en cuanto pertenecen a él, mientras que las normas jurídicas se establecen de
modo público, normalmente por escrito, y su cumplimiento o incumplimiento es juzgado por
tribunales establecidos expresamente para tal función. En cualquier caso, cierto número de nor-
mas jurídicas emanan de la propia conceptuación de las normas culturales. Para algunos inves-
tigadores, precisamente se produce la codificación de la norma a partir del momento en que deja
de ser unánime su cumplimiento y es necesario establecer una obligación que pueda ser exigi-
da de modo objetivo. Se basan para llegar a esa conclusión en que las sociedades primitivas,
con sistemas de valores altamente coherentes, no necesitan fijar un sistema jurídico pues el pro-
pio sistema de sanción cultural basta para el mantenimiento de su sistema de valores.

1.1.2.3. Usos y costumbres


Aun cuando semánticamente nos resulte difícil diferenciar los términos uso y costumbre,
sociológicamente, en el marco del estudio sobre los valores y la cultura, sí podemos distinguir-
los en función del grado de gravedad de las normas incumplidas y, en consecuencia, de las san-
ciones que el grupo impone. Los usos corresponden a normas cuyo incumplimiento se conside-
ra menos grave, mientras que las costumbres son normas sociales o culturales de mayor
trascendencia y cuyo incumplimiento es sancionado más enérgicamente por el grupo. La mayor
o menor gravedad dependerá de hasta qué nivel considere el grupo que el incumplimiento de la
norma pone en riesgo la cohesión del sistema de valores que lo caracteriza.

1.1.3. IDEAS Y CREENCIAS


Todo grupo social, toda cultura, posee un conjunto de principios que forman parte de su
sabiduría. El grupo “sabe” determinadas “cosas”. Pero de ese conjunto de “cosas” que “sabe”
el grupo podemos diferenciar las que son ideas y las que son creencias.

– Las ideas son aquellos elementos de la sabiduría del grupo cuya realidad práctica es
comprobable empíricamente, es decir, mediante la observación, el análisis, la prueba.

– Las creencias son, en contrapartida, los elementos que, aunque sean aceptados por el
grupo del mismo modo que las ideas, no son comprobables empíricamente por un
observador, no encuentran contrastación posible con hechos probados, aunque el
grupo puede haber adoptado como tales hechos probados algunos que le han permiti-
do conjeturar alguna de esas creencias.

El desarrollo de las sociedades, a medida que en su seno se han ido desarrollando los cono-
cimientos empíricos por la vía de la investigación y la ciencia, las ideas han ido ganando terre-
no a las creencias. En todo caso, el triunfo del empirismo sobre la creencia sin prueba es empre-

7
-Pág.127-
CIENCIAS SOCIALES (GEOGRAFÍA E HISTORIA)

sa que encuentra numerosas dificultades ya que, al formar las creencias parte sustantiva de la
propia definición del grupo social, éste tiende a defender con fuerza sus principios ante cual-
quier elemento que pueda poner en riesgo su coherencia, aunque ésta se encuentre fundamen-
tada en la irrealidad.

1.1.4. SÍMBOLOS
Un símbolo es, básicamente, una “cosa” que representa otra “cosa”. Puede ser un objeto, un
sonido o cualquier otro elemento reconocible por medio de los sentidos que materializa la repre-
sentación de un concepto, una idea o un conjunto de los mismos.

El ser humano es capaz de abstraer conceptos simbólicos otorgando a “cosas” que, en prin-
cipio, carecen de sentido, un contenido de significación coherente y complejo. De este modo,
puede construir un completo código de símbolos que permitan comunicar conceptos. Pero nece-
sariamente, ya que estamos hablando de relaciones en el marco de un grupo, esa codificación
de símbolos debe ser consecuencia de una convención, de un consenso entre los individuos del
grupo, con el fin de que quien recibe, quien observa el símbolo sea capaz de interpretar el men-
saje de quien expresa o construye el símbolo.

El lenguaje oral es un claro ejemplo de sistema de símbolos. Los sonidos que forman las
palabras no poseen en sí mismos significado alguno, son sencillamente el resultado de hacer
pasar el aire desde los pulmones al exterior colocando a su paso de un modo determinado órga-
nos suya funcionalidad es compartida entre la fonación y la alimentación, el olfato o la respira-
ción. Pero esos sonidos, combinados adecuadamente, sirven para intercambiar sentimientos,
opiniones, deseos, órdenes…la comprensión del código de símbolos que llamamos lengua iden-
tifica a los individuos que forman una cultura o una subcultura.

Del mismo modo que el lenguaje oral, los gestos faciales o corporales en general constitu-
yen también un sistema de símbolos, igual que otros sistemas, más o menos complejos.

1.2. LOS CAMBIOS DE LA CULTURA


El grupo social, con su sistema de valores, sus normas, usos, costumbres, ideas, creencias y
símbolos busca su coherencia interna y en la medida en que esos sistemas se mantengan, el
grupo y su cultura perdurará. El cambio cultural, pues, debe venir de la mano de la alteración
del equilibrio del sistema. Esa alteración parte del desequilibrio en el sistema de ideas y creen-
cias y puede deberse a procesos internos o externos interiorizados voluntaria o forzosamente.

8
-Pág.128-
U . D . 6 . - D IVERSIDAD C U LT U R A L

1.2.1. PROCESOS INTERNOS DE CAMBIO


Ya hemos dicho que en la medida que las pruebas empíricas demuestren la falsedad de una
creencia, aumentará el volumen de ideas en detrimento de las creencias. Esa demostración
puede producirse como consecuencia de un descubrimiento o una invención:

– Descubrimiento debemos entenderlo como la incorporación de una idea nueva al


marco de las ideas de una cultura.

– Invención es cualquier utilización novedosa de una idea ya incorporada por el grupo.

(Por ejemplo, podemos decir que la luz eléctrica fue un descubrimiento mientras que la
bombilla incandescente fue una invención por aplicación del descubrimiento de la electricidad)

1.2.2. PROCESOS EXTERNOS DE CAMBIO


En otras circunstancias, la incorporación de nuevas ideas puede venir de la mano de influen-
cias externas, de la difusión o la aculturación:

El término difusión alude a la influencia cultural desde el punto de vista de la cultura que
influye sobre otra. Decimos que la cultura A difunde un descubrimiento o una invención a otra
cultura B.

Por su parte, la expresión aculturación, alude al mismo fenómeno pero desde el punto de
vista de la sociedad que recibe la influencia. La cultura B recibe la aculturación de la cultura A.

La facilidad para que se produzca la relación efectiva entre dos culturas, la difusión-acultu-
ración, dependerá de lo factible que sea la comunicación entre ambas: su proximidad geográfi-
ca, su proximidad cultural, el grupo o nivel de la sociedad por el que intenten penetrar las
influencias y la mayor o menor utilidad que la novedad pueda tener. Además, será más sencillo
que la transmisión afecte a la cultura material que al mundo de las ideas; en el mundo de lo
material, las tecnologías pueden asumirse con más facilidad por ser más útiles.

1.2.3. EL CAMBIO EFECTIVO


El grupo cultural será tanto más estable cuanto más coherente sea su sistema de ideas y
creencias, en consecuencia, cualquier alteración del mismo con la propuesta de nuevas ideas
que menoscaben el conjunto de las creencias hasta ese momento válidas, producirá, inicial-
mente, una reacción de autoprotección que impedirá el cambio cultural inmediato. Habrá una
tensión social que se resolverá con el rechazo a la novedad y, por tanto, la inalterabilidad de
la cultura, o con la asunción de la nueva idea y el cambio cultural consiguiente que podrá ser

9
-Pág.129-
CIENCIAS SOCIALES (GEOGRAFÍA E HISTORIA)

más o menos traumática y modificar principios sustanciales que alteren por completo la cul-
tura. Esa alteración puede llegar a producir, incluso, la disolución de la cultura y su absor-
ción por la cultura difusora (como ha sucedido de hecho en determinados momentos de la
Historia).

1.3. CULTURA Y ADAPTACIÓN AL MEDIO


La relación de los seres humanos con el medio ambiente es un hecho cotidiano desde que el
hombre apareció sobre la faz de la Tierra (De hecho, la propia aparición del Homo Sapiens fue,
posiblemente, la consecuencia de la adaptación de grupos de homínidos precedentes a unas
determinadas condiciones ambientales). Pero la sociedad humana de hoy no mantiene con el
medio ambiente el mismo tipo de relación que en su origen ni, claro está, que el que mantienen
los restantes seres vivos. Quizás el punto culminante de la definición del Hombre, del Homo
Sapiens Sapiens, fue aquel a partir del cual la adaptación al medio no pasaba por la adaptación
evolutiva mediante cambios genéticos sino por la adaptación cultural mediante la incorporación
de nuevas ideas y procedimientos, en definitiva, a partir del momento en que la sociedad se con-
vierte en una sociedad tecnológica en la que los medios materiales son los que se adaptan a las
necesidades (heramientas, abrigo…).

Un principio esencial a la hora de estudiar los seres vivos y su evolución es el que apunta-
ba Charles Drawin de la lucha por la vida o la existencia. Este principio ha estado presente en
el desarrollo de la vida de todos los seres vivos y ha sido alcanzado con mayor o menor éxito
por las distintas especies. En el caso del hombre, esa lucha por la vida ha dejado a los medios
con los que su organismo cuenta en un segundo plano y han elevado la tecnología y la coope-
ración social como procedimientos de supervivencia.

Así, el oso polar ha necesitado millones de años de evolución para dotarse de una capa de
pelo y grasa, además de un mecanismo de reducción al mínimo de sus actividades orgánicas
durante la hibernación, para adaptarse a la vida en las áreas polares. El hombre, manteniendo su
estructura corporal intacta, puede habitar esos espacios vistiéndose con ropas que él mismo
fabrica y puede obtener alimento durante las etapas más frías, en las que el oso “duerme”, por-
que su tecnología le permite conservar la caza o conseguirla).

Además, el principio de solidaridad ha jugado un papel fundamental para mover al hombre,


que vive en un entramado social, cooperar para conseguir la adaptación y la supervivencia.

Por otro lado, la adaptación al medio es algo dinámico, siempre inconcluso, dado que el pro-
pio medio es dinámico, cambiante y existen múltiples factores que el hombre todavía no puede
controlar. El hombre controla buena parte de los factores del medio ambiente e incluso ha sido
capaz de modificarlo tan sustancialmente que ha alterado el equilibrio ambiental de amplias

10
-Pág.130-
U . D . 6 . - D IVERSIDAD C U LT U R A L

zonas y es potencialmente “capaz” de poner en peligro el equilibrio global. Pero, de momento,


no es capaz de controlar los fenómenos atmosféricos ni los movimientos de la corteza terrestre
generadores de terremotos y erupciones.

1.4. ESTRUCTURA SOCIAL


El hombre es un ser social, no vive aislado por naturaleza sino que encuentra su definición
en tanto en cuanto mantiene relaciones sociales, se relaciona con otros individuos con los que
se “asocia”.

La clase de relación que lleva al individuo a asociarse define dos tipos de asociaciones:

– Sociedad simbiótica, aquella en la que los individuos que la forman hacen demandas
no iguales al medio. Tal es el caso de la asociación que llamamos familia, en la que
viven padres e hijos, individuos de diferentes sexos y edades cuyas demandas son dife-
rentes.

– Sociedad comensalística, la que está formada por individuos cuyas demandas al


medio son similares. Tal es el caso de las asociaciones profesionales, los partidos
políticos, los sindicatos, grupos en los que convergen individuos con las mismas
demandas.

Pero el grupo humano entendido como cultura no es una sola de esas asociaciones sino algo
más complejo. Es, en realidad, una agrupación de asociaciones simbióticas y comensalísticas a
la que llamamos comunidad. Esa comunidad está integrada por un conjunto interrelacionado
de asociaciones simbióticas, al que llamamos “grupo corporado” y un conjunto de asociacio-
nes comensalísticas al que llamamos “grupo categórico”.

Los grupos corporados tienen un carácter esencialmente productivo pues la convergencia


de los individuos que la integran se mueve por la idea de hacer o producir algo independiente-
mente de las demandas que cada individuo tiene como suyas.

Los grupos categóricos, por su parte, vienen determinados por la reacción del conjunto de
los individuos para proteger o defender sus intereses. Estos grupos, llamados por la Ecología
Humana “nichos”, asumen distintas funciones cuya diferenciación hace que en los diferentes
estadios de la evolución de una comunidad uno de los “nichos” se convierta en dominante fren-
te a los demás, lo que se produce en el marco de una competencia entre “nichos”.

Por otra parte, en el seno de la comunidad, se establecen otras circunstancias estructurales,


aspectos que determinan una distribución de elementos, factores, situaciones a los que se puede
denominar elementos estructurales y que son: “rol”, “estatus” e “institución”.

11
-Pág.131-
CIENCIAS SOCIALES (GEOGRAFÍA E HISTORIA)

El “rol”, o papel social, es un cometido concreto en el funcionamiento social, regido por


normas que concretan determinados valores sociales. El grupo “sabe” lo que cabe esperar de
cada “rol” y el individuo que asume ese “rol” “sabe” lo que se espera de él. Así, existen “roles”
como el de “maestro”, “madre”, “avalista”, etc…, cada uno de los cuales incorpora un conjun-
to de normas funcionales conocidas por el grupo y por quien “juega ese rol”. El conjunto de nor-
mas constituye el “modelo” de referencia del “rol”.

El “estatus” o “posición” viene determinado por la valoración social de los “roles”.


Efectivamente, los distintos “roles” son mejor o peor valorados por la sociedad y ello sitúa a
cada uno en una escala o clasificación.

La “institución” es un conjunto de “roles” complementarios entre sí que se organizan en


torno a una función trascendente para el grupo social. La familia es, así, una “institución” inte-
grada por el conjunto de los “roles” complementarios “padre”, “madre”, “hijo”, “hermano”,
“esposo”, “esposa”, que convergen para la satisfacción de determinadas funciones de trascen-
dencia social: satisfacción de las necesidades sexuales de los esposos, de las necesidades afec-
tivas de todos, de la “producción” y preparación de nuevos individuos para el grupo social,
etc… También es una “institución” la asociación de los “roles” “profesor” y “alumno”, por
ejemplo, pues son complementarios (no hay profesor sin alumno ni alumno sin profesor) y se
orientan a una función social, la de la transmisión de las “ideas” del grupo humano.

2. DIVERSIDAD Y RELATIVISMO CULTURAL

Definidos los conceptos precedentes, analizado el concepto de cultura, si utilizamos sus


esquemas para analizar otras culturas distintas a la propia concluimos con la constatación de la
existencia de multiformes culturas y subculturas. La observación de dicha realidad, la diversi-
dad y la profundización en las normas y valores característicos de cada una de ellas, nos lleva-
rá a la conclusión de la relatividad de cada uno de esos principios que son tomados como abso-
lutos por la respectiva cultura y, en caso de enfrentamiento, rechazados violentamente por las
contrarias.

El acercamiento a cualquiera de los rasgos, aspectos, ideas, creencias, roles, instituciones,


de una cultura nos permitiría ver esa diversidad y relatividad. En este caso escogemos una de
las “instituciones” emblemáticas de diferentes culturas para observar estas cuestiones: la
familia.

12
-Pág.132-
U . D . 6 . - D IVERSIDAD C U LT U R A L

2.1. LA FAMILIA, DEFINICIÓN Y TRASCENDENCIA


Ya hemos definido la familia como una institución social integrada por los “roles” comple-
mentarios “padre”, “madre”, “hijo”, “hermano”, “esposo”, “esposa”, para la consecución de
determinadas funciones de trascendencia social. Su importancia en el entramado social ha sido
valorada de formas a veces contradictorias pero siempre dándole un valor fundamental. Efecti-
vamente, en algunos análisis socio-políticos, caso del marxismo, se le ha llegado a considerar
como un obstáculo para la consecución de los fines de movimientos orientados al cambio social.
Los intentos de eliminarla, sin embargo, han resultado vanos pues, de un modo u otro, la insti-
tución se ha mantenido en la práctica.

La relación entre los roles que constituyen la familia es muy importante. De hecho, la rela-
ción existente entre “esposo” y “esposa” es tan específica que constituye en sí misma una ins-
titución, el matrimonio, de cuya existencia o mantenimiento depende la propia permanencia de
la familia.

La institución del matrimonio, base de la familia, se fundamenta en la complementariedad


de los roles de los esposos orientada a la satisfacción de sus necesidades sexuales. Todas las cul-
turas han establecido normas para la constitución, desarrollo y disolución del matrimonio, exis-
tiendo un principio normativo universal, presente en todas las culturas, que es el tabú del inces-
to, es decir, la limitación del intercambio entre familiares. Además, en todas las culturas existen
otras normas comunes relacionadas con la limitación impuesta a la satisfacción de las necesi-
dades sexuales. Así, en toda sociedad conocida se prohíben, en relación con el matrimonio, las
relaciones extramatrimoniales y las homosexuales, si bien, dependiendo de la cultura, existen
muy diversos límites de tolerancia.

2.2. LAS FUNCIONES DE LA FAMILIA


En términos generales, las funciones de la familia son:

1. La satisfacción de las necesidades sexuales entre el esposo y la esposa

2. Proporcionar nuevos individuos, los hijos, al grupo social.

3. Mantener biológicamente a los hijos.

4. Educar a los hijos en las normas culturales del grupo.

5. Asumir sus funciones económicas.

– En las sociedades primitivas constituye una unidad de producción.

– En las modernas es una unidad de consumo.

13
-Pág.133-
CIENCIAS SOCIALES (GEOGRAFÍA E HISTORIA)

6. Proporcionar a los hijos una primera posición o estatus.

– En las sociedades primitivas otorga el primero y definitivo.


– En las modernas, un primer paso del que pueden salir los hijos cuando se inde-
pendizan de su familia original.

7. Satisfacer las necesidades de afecto, primero entre los esposos y después entre todos
los miembros de la familia.

Planteadas así sus funciones, la familia se nos muestra como una institución universal, pre-
sente en todas las culturas, si bien tal institución puede manifestarse bajo muy diversas formu-
laciones materiales.

2.3. MODELOS FAMILIARES


Las formulaciones concretas del modelo familiar, varía de unas culturas a otras. Tales for-
mulaciones podemos analizarlas desde distintos puntos de vista.

2.3.1. TIPOS DE FAMILIA POR EL NÚMERO DE SUS INTEGRANTES


Dependiendo de cómo se constituye el núcleo matrimonial de la familia podemos distinguir
los siguientes tipos de familia, presentes en distintas culturas:

A) Familia nuclear.–Integrada por los cónyuges y la prole. Puede ser:


– Monogámica.–En ella los cónyuges son dos, esposo y esposa.
– Poligámica.–En ella hay varios cónyuges, uno de un sexo y el resto del otro.
En función de la combinación de sexos podrán ser:
Poligínica: Un varón y varias mujeres
Poliándrica: Una mujer y varios hombres

B) Familia extensa.–Integrada, independientemente del tipo de relación conyugal,


por familias nucleares de varias generaciones que conviven formado una agrupa-
ción (clan, tribu, gens).

En las sociedades primitivas, aquellas en las que la función económica de producción


adquiere especial relevancia, la familia extensa es la formulación familiar habitual dado que
este tipo de familia proporciona núcleos numerosos en los que los individuos aportan su fuerza
de trabajo. En este tipo de familias, los matrimonios que las originan suelen constituirse por
interés económico, pactos entre grupos, intercambio de bienes, etc… y no por amor. En ellas
la función de satisfacción de las necesidades afectivas es más difusa como consecuencia de la
dispersión de las vinculaciones afectivas.

14
-Pág.134-
U . D . 6 . - D IVERSIDAD C U LT U R A L

Por el contrario, en las sociedades modernas se observa el predominio de la familia nuclear


monogámica, lo que está ligado a la pérdida de importancia de la función productiva, sustitui-
da por la función de consumo, para el cual es más práctico un número reducido de integrantes.
En contrapartida, las relaciones afectivas adquieren un mayor protagonismo, comenzando por
el hecho de que el matrimonio que la origina se establece normalmente por amor, aunque, para-
dójicamente, resulte más estable el matrimonio y, por tanto, la familia extensa, en la que pre-
domina el interés económico.

2.3.2. TIPOS DE FAMILIA POR LA RESIDENCIA


Una vez producido el matrimonio que constituirá la base sobre la que se construirá la fami-
lia, el lugar donde se establezca determinará la siguiente clasificación tipológica:

A) Familia patrilocal.–Aquella que se establece en la casa de la familia de la que es


originario el padre.

B) Familia matrilocal.–Aquella que fija su residencia en la casa de la familia de la


que es originaria la madre.

C) Familia neolocal.–La que se ubica en un lugar independiente de los hogares de los


que son originarios los cónyuges.

Los dos primeros tipos se dan en los casos de familias extensas y el tercero es característi-
co de las familias nucleares.

2.3.3. TIPOS DE FAMILIA POR LA ASCENDENCIA


La computación de la ascendencia se relaciona con la transmisión de la herencia y lleva apa-
rejada la asignación de la titularidad de la autoridad familiar. En este sentido, los tipos de fami-
lia que pueden diferenciarse son:

A) Familia patrilineal.–Aquella en la que la asignación de la ascendencia sigue la


línea de la del padre. En consecuencia, la autoridad corresponde al padre.

B) Familia matrilineal.–La ascendencia sigue la de la madre y la autoridad le corres-


ponde a ella.

C) Familia abuncular.–La ascendencia sigue la línea del tío materno.

15
-Pág.135-
CIENCIAS SOCIALES (GEOGRAFÍA E HISTORIA)

RESUMEN

LA CULTURA
– Existen muy diversas materializaciones de la cultura.
– El sistema de valores de una cultura está regido por normas y sanciones; usos y cos-
tumbres.
– Las ideas son elementos de la sabiduría del grupo cuya realidad es comprobable; las creen-
cias no son comprobables empíricamente.
– Los grupos culturales generan sus propios sistemas de símbolos para comunicarse.

LOS CAMBIOS DE LA CULTURA


– Los procesos internos de cambio se deben al aumento de ideas en detrimento de las
creencias lo que puede producirse como consecuencia de un descubrimiento o una inven-
ción.
– Los procesos externos de cambio se producen por la incorporación de nuevas ideas por
vía de la difusión o la aculturación.

CULTURA Y ADAPTACIÓN AL MEDIO


En el caso del hombre, la lucha por la vida de la que habla Charles Darwin ha dejado a los
medios con los que su organismo cuenta en un segundo plano y ha elevado la tecnología y la
cooperación social como procedimientos de supervivencia.

ESTRUCTURA SOCIAL
– La clase de relación que lleva al individuo a asociarse define dos tipos de asociaciones:
– Sociedad simbiótica, sus individuos tienen distintos objetivos.
– Sociedad comensalística, las demandas de los integrantes son similares.
– La comunidad está integrada por un conjunto de asociaciones simbióticas (“grupo cor-
porado”), y un conjunto de asociaciones comensalísticas (“grupo categórico”).
– En el seno de la comunidad se establecen unos elementos estructurales que son : “rol”
(papel social), “estatus” (posición) e “institución” (roles complementarios organizados).

16
-Pág.136-
U . D . 6 . - D IVERSIDAD C U LT U R A L

LA FAMILIA
Funciones:
– Satisfacción de necesidades sexuales entre esposo y esposa y de afecto en el conjunto.
– Proporcionar nuevos individuos, los hijos, al grupo social; mantenerlos y educarlos.
– Asumir sus funciones económicas.
Tipos:
– Por el número de sus integrantes.- Familia nuclear (Monogámica, Poligámica-
Poligínica o Poliándrica) y Familia extensa (clan, tribu, gens).
– Por la residencia.- Familia patrilocal, matrilocal y neolocal.
– Por la ascendencia.- Patrilineal, matrilineal y abuncular.

17
-Pág.137-
CIENCIAS SOCIALES (GEOGRAFÍA E HISTORIA)

EJERCICIOS DE AUTOCOMPROBACIÓN

1. Señale la afirmación falsa.

A. El hombre es el único ser vivo capaz de dotarse de una cultura

B. Los comportamientos de los animales están condicionados por el instinto

C. Cultura es todo lo que hace o piensa el hombre dentro de su grupo social

D. Dentro de las grandes culturas no pueden existir innumerables subculturas

2. Señale la afirmación correcta.

A. Cuanto más amplio sea el grupo humano, más coherente, homogéneo y común será su sistema
de valores

B. El conjunto de los valores de una cultura constituye su sistema de normas

C. Los usos corresponden a normas cuyo incumplimiento se considera menos grave, mientras que
las costumbres son normas sociales o culturales de mayor trascendencia

D. Las creencias son aquellos elementos de la sabiduría del grupo cuya realidad práctica es com-
probable empíricamente

3. Un grupo social puede construir un completo código de símbolos que permitan comunicar concep-
tos. Pero necesariamente, ya que estamos hablando de relaciones en el marco de un grupo, esa codi-
ficación de símbolos debe ser consecuencia de:

A. una convención

B. una costumbre

C. una analogía

D. un sistema de valores

4. Indique la afirmación errónea.

A. El lenguaje oral es un claro ejemplo de sistema de símbolos

B. En la cultura actual se han ido desarrollando los conocimientos empíricos por la vía de la inves-
tigación y la ciencia, las ideas han ido ganando terreno a las creencias

C. Costumbres son normas sociales o culturales de mayor trascendencia y cuyo incumplimiento es


sancionado más enérgicamente por el grupo

D. Las normas sociales se caracterizan por que son aplicadas siempre por instituciones jurídicas

18
-Pág.138-
U . D . 6 . - D IVERSIDAD C U LT U R A L

5. Señale el concepto falso.

A. Descubrimiento debemos entenderlo como la incorporación de una idea nueva al marco de las
ideas de una cultura

B. Invención es cualquier utilización novedosa de una idea ya incorporada por el grupo

C. El término difusión y la expresión aculturación son opuestos pues el primero indica la asimila-
ción de cultura de unos pueblos por otros y el segundo señala el rechazo de una cultura a las
influencias de otra

D. Será más sencillo que la transmisión de cultura afecte a la cultura material que al mundo de las
ideas

6. ¿Qué es una sociedad simbiótica?

A. Aquella en la que los individuos que la forman hacen demandas no iguales al medio

B. La que está formada por individuos cuyas demandas al medio son similares

C. El grupo humano entendido como cultura

D. Las asociaciones profesionales, los partidos políticos, los sindicatos

7. El “estatus”…

A. Viene determinado por la valoración social de los “roles”

B. Es un conjunto de “roles” complementarios entre sí

C. Es un cometido concreto en el funcionamiento social, regido por normas

D. Es el conjunto de circunstancias ambientales en las que se desarrolla un “rol”

8. Entre las funciones de la familia NO se encuentra:

A. Proporcionar nuevos individuos, los hijos, al grupo social

B. Asumir sus funciones económicas

C. Proporcionar a los hijos una primera posición o estatus

D. Satisfacer las necesidades de sexuales de los hijos

19
-Pág.139-
CIENCIAS SOCIALES (GEOGRAFÍA E HISTORIA)

9. Señale la afirmación errónea respecto a la familia.

A. Aquella en la que el matrimonio está constituido por un hombre y varias mujeres se llama poli-
gámica

B. Aquella en la que el matrimonio está constituido por un hombre y varias mujeres se llama poli-
gínica

C. Aquella en la que el matrimonio está constituido por una mujer y varios hombres se llama
poliándrica

D. Aquella en la que el matrimonio está constituido por una mujer y varios hombres se llama poli-
gínica

10. Una familia “neolocal” es:

A. Aquella que está constituida exclusivamente por un matrimonio

B. Aquella que está formada por un matrimonio y dos hijos, varón y mujer, y cuya ascendencia es
abuncular

C. Aquella que al formarse no reside ni en casa de los ascendientes de la/s esposa/s ni en casa de
los ascendientes de el/los esposo/s

D. Aquella que al formarse reside en casa del tío de la esposa

RESPUESTAS A LOS EJERCICIOS

1. D 2. C 3. A 4. D 5. C

6. A 7. A 8. D 9. D 10. C

20
-Pág.140-
portada TROPA 19/3/07 19:51 Página 1

FUERZAS ARMADAS
PROFESIONALES
CURSO DE APOYO
A LA PREPARACIÓN
DE LAS PRUEBAS DE ACCESO
A UNA RELACIÓN DE SERVICIOS
DE CARÁCTER PERMANENTE

CIENCIAS SOCIALES
3ª parte
Unidades didácticas 7, 8 y 9

DIGEREM

MINISTERIO
DE DEFENSA
FUERZAS ARMADAS SUBDIRECCIîN GENERAL
DE TROPA Y MARINERIA
PROFESIONAL
PROFESIONALES
CURSO DE APOYO
A LA PREPARACIÓN
DE LAS PRUEBAS DE ACCESO
A UNA RELACIÓN DE SERVICIOS
DE CARÁCTER PERMANENTE

CIENCIAS SOCIALES
3ª parte
Unidades didácticas 7, 8 y 9

-Pág.1-
La Ley 8/2006 de Tropa y Marinería, en su artículo 16,1, establece que “la formación
en las Fuerzas Armadas garantizará que los militares profesionales de tropa y
marinería puedan adquirir, actualizar o ampliar sus conocimientos para un mayor
desarrollo personal y profesional”. En cumplimiento de este mandato, el Ministerio
de Defensa edita el presente material didáctico para facilitar a los militares
profesionales de tropa y marinería, alumnos de los cursos de formación
presencial que se imparten a través de la Dirección General de Reclutamiento y
Enseñanza Militar, los apoyos necesarios para preparación de dichos cursos, que
permitirán, siempre que superen las pruebas correspondientes, la obtención de la
titulación de graduado en Educación Secundaria, acreditación para el acceso a
los ciclos formativos de la Formación Profesional de grado medio o de grado
superior, acceso a las Escalas de Suboficiales, Tropa Permanente, Guardia Civil
y Policía Nacional.

CATÁLOGO GENERAL DE PUBLICACIONES


http://www.060.es

Edita:

© Autor y editor
NIPO: 076-10-204-9 NIPO: 076-10-205-4 (edición en línea)
Depósito Legal: M-32363-2009
Diseño y programación: cimapress
Tirada: 1300 ejemplares
Fecha de edición: septiembre, 2010

Prohibida la reproducción total o parcial de esta obra, por cualquier medio sin autorización escrita del editor
CIENCIAS SOCIALES
3ª parte

SUMARIO

Unidad didáctica Pág.

7. ECONOMÍA Y TRABAJO EN EL MUNDO ACTUAL 5

8. PARTICIPACIÓN Y CONFLICTO POLÍTICO EN EL 29


MUNDO ACTUAL

9. ARTE, CULTURA Y SOCIEDAD EN EL MUNDO 85


ACTUAL

-Pág.3-
U.D. 7.- ECONOMÍA Y TRABAJO EN EL MUNDO ACTUAL

ÍNDICE
OBJETIVOS . . . . . . . . . . . . . . . . . . . . . . . . . . . . . . . . . . . . . . . . . . . . . . . . . . . . . . . . . . . . . . 2

INTRODUCCIÓN . . . . . . . . . . . . . . . . . . . . . . . . . . . . . . . . . . . . . . . . . . . . . . . . . . . . . . . . . . 3

MAPA CONCEPTUAL . . . . . . . . . . . . . . . . . . . . . . . . . . . . . . . . . . . . . . . . . . . . . . . . . . . . . 4

DESARROLLO DE CONTENIDOS . . . . . . . . . . . . . . . . . . . . . . . . . . . . . . . . . . . . . . . . . . 5

1. RECURSOS ESCASOS Y NECESIDAD DE ELECCIÓN COMO CUESTIÓN


CENTRAL EN TODA ACTIVIDAD ECONÓMICA . . . . . . . . . . . . . . . . . . . . . . . . 5

2. DIVISIÓN TÉCNICA DEL TRABAJO, INTERDEPENDENCIA ECONÓMICA


E INSTRUMENTOS DE COORDINACIÓN EN LAS ECONOMÍAS
DE MERCADO . . . . . . . . . . . . . . . . . . . . . . . . . . . . . . . . . . . . . . . . . . . . . . . . . . . . . . . 6
2.1. LOS FACTORES DE PRODUCCIÓN EN LA EMPRESA . . . . . . . . . . . . . . . . . . . . . 9
2.2. EL PROCESO PRODUCTIVO AL NIVEL DEL ESTADO . . . . . . . . . . . . . . . . . . . . . 9

3. DIVISIÓN TÉCNICA Y SOCIAL DEL TRABAJO Y ESTRUCTURA SOCIAL.


EL DESARROLLO TECNOLÓGICO Y SUS REPERCUSIONES EN EL MUNDO
DEL TRABAJO. DESIGUALDADES Y CONFLICTOS SOCIALES;
LOS SINDICATOS; DERECHOS Y DEBERES DE LOS TRABAJADORES;
DIVISIÓN Y DISCRIMINACIÓN EN EL TRABAJO . . . . . . . . . . . . . . . . . . . . . . . 11

4. INTERDEPENDENCIA Y DESIGUALDAD ECONÓMICA, TÉCNICA


Y POLÍTICA ENTRE LOS DISTINTOS PAÍSES DEL MUNDO:
DESARROLLOS DESIGUALES Y NEOCOLONIALISMO . . . . . . . . . . . . . . . . . . 14
4.1. ORÍGENES . . . . . . . . . . . . . . . . . . . . . . . . . . . . . . . . . . . . . . . . . . . . . . . . . . . . . . . 15
4.2. LA DESCOLONIZACIÓN . . . . . . . . . . . . . . . . . . . . . . . . . . . . . . . . . . . . . . . . . . . . 16
4.3. EL NEOCOLONIALISMO . . . . . . . . . . . . . . . . . . . . . . . . . . . . . . . . . . . . . . . . . . . . 17

RESUMEN . . . . . . . . . . . . . . . . . . . . . . . . . . . . . . . . . . . . . . . . . . . . . . . . . . . . . . . . . . . . . . . 20

EJERCICIOS DE AUTOCOMPROBACIÓN . . . . . . . . . . . . . . . . . . . . . . . . . . . . . . . . . . .22

RESPUESTAS A LOS EJERCICIOS . . . . . . . . . . . . . . . . . . . . . . . . . . . . . . . . . . . . . . . . . 24

-Pág.5-
CIENCIAS SOCIALES. GEOGRAFÍA E HISTORIA

O B J E T I VO S
Al finalizar el estudio de esta Unidad Didáctica, el alumno será capaz de:

• Comprender los mecanismos generales de la economía y la importancia de los


elementos o factores que intervienen en sus procesos.

• Analizar y valorar el papel del trabajo tanto en cuanto a su participación en el


proceso económico como en cuanto a su trascendencia social; observando los
problemas y retos que plantea.

• Conocer los desequilibrios económicos mundiales y las consecuencias que los


mismos tienen en las relaciones internacionales.

-Pág.6-
U.D. 7.- ECONOMÍA Y TRABAJO EN EL MUNDO ACTUAL

INTRODUCCIÓN
A bordamos aquí los aspectos generales relativos a la economía y el trabajo. Analiza-
remos los rasgos esenciales presentes en las relaciones económicas y veremos
cómo se desarrolla la actividad económica entorno a sus factores constitutivos. Partiremos
de la comprensión de las razones que mueven la actividad económica, analizaremos los
pasos de la misma y veremos el papel de los elementos intervinientes. Haremos especial
mención al mundo del trabajo y las relaciones de carácter social que genera, para acer-
carnos al planteamiento de sus grandes retos actuales. Y, finalmente, daremos una visión
general de las consecuencias a nivel global que tiene la desigualdad económica.

-Pág.7-
CIENCIAS SOCIALES. GEOGRAFÍA E HISTORIA

M A PA C O N C E P T UA L
ECONOMÍA Y TRABAJO
EN EL MUNDO ACTUAL

RECURSOS

ACTIVIDAD ECONÓMICA

BIENES
ECONOMÍA
DE MERCADO
PRODUCCIÓN

EMPRESAS FACTORES EL ESTADO


Y LA
PRODUCCIÓN
FINANCIACIÓN TRABAJO

TIERRA

CAPITAL

EMPRESARIO

CONDICIONES ACTUALES
DIVISIÓN TÉCNICA Y
SOCIAL DEL TRABAJO
DISCRIMINACIÓN

NEOCOLONIALISMO
INTERDEPENDENCIA
Y DESIGUALDAD
EN EL MUNDO NUEVAS ACTITUDES

-Pág.8-
U.D. 7.- ECONOMÍA Y TRABAJO EN EL MUNDO ACTUAL

1. RECURSOS ESCASOS Y NECESIDAD DE ELECCIÓN COMO


CUESTIÓN CENTRAL EN TODAACTIVIDAD ECONÓMICA
La actividad económica es una parte de la conducta humana. La economía considera obje-
tos, bienes materiales y lo hace en tanto en cuanto se relacionan con decisiones que adopta el
individuo. Efectivamente, el hombre se enfrenta a una serie de necesidades físicas y fisiológi-
cas que debe satisfacer para la conservación de su propia vida, de los suyos y, en mayor exten-
sión, de la especie. Si todos los bienes se presentaran en la naturaleza en cantidades suficientes,
dichas necesidades encontrarían satisfacción inmediata. Sin embargo, la realidad nos presenta
unos recursos escasos y, en muchas ocasiones, sin que sirvan para la satisfacción directa de las
necesidades. Ante esta situación el hombre debe elegir un orden, una jerarquía en la que cada
necesidad ocupe un puesto, en la que unas necesidades tengan preferencia sobre otras. Estos, la
existencia de recursos escasos y la necesidad de elección, están en la base de todo el entrama-
do de relaciones económicas.

Así pues, partimos de unas necesidades o fines que el hombre ordena según su importan-
cia. Para satisfacer esas necesidades o fines el hombre utiliza bienes, que pueden ser, por tanto,
objetos materiales o servicios

Los bienes, para que podamos hablar de actividad económica, deben cumplir dos condicio-
nes o características:
– Han de ser escasos.—Efectivamente, los bienes pueden ser libres, es decir, hallarse en
grandes cantidades, superiores incluso a las requeridas para la satisfacción de las necesi-
dades; mas si todos los bienes fueran libres no sería necesaria la actividad económica.
Pero la mayor parte de los bienes son escasos lo que obliga a administrar adecuadamen-
te aquellos de los que se dispone.
– Deben ser susceptibles de usos alternativos, es decir, cada bien debe tener dos o más uti-
lidades, cifrándose en la correcta elección del fin al que se orienta uno de los aspectos de
la actividad económica.

En definitiva, se genera la actividad económica como consecuencia de la existencia de


necesidades para cuya satisfacción no se dispone de bienes suficientes, debiéndose adoptar
decisiones sobre preferencia ante las necesidades y sobre utilidad de los bienes. Esto nos lleva
a la definición de un proceso económico en el que se producen los siguientes pasos:
– Obtención de los bienes básicos.
– Transformación de aquellos que no pueden ser consumidos directamente.
– Distribución de los bienes para su consumo.
– Satisfacción de la necesidad mediante el consumo del bien o del uso de un servicio.

5
-Pág.9-
CIENCIAS SOCIALES. GEOGRAFÍA E HISTORIA

A lo largo de la historia se han sucedido diversas formas de cumplimentar tales pasos. Esta
evolución podemos resumirla en las siguientes fórmulas:
– Familiarmente.—Forma más primitiva en la que la familia era a la vez unidad de pro-
ducción y de consumo.
– Profesionalmente.—A partir del momento en el que resultó posible intercambiar bienes
entre familias, es decir, cuando una de ellas dispone de excedentes y otra carece de dichos
bienes pero no de la necesidad. Desde ese instante se ponen las bases de la primera divi-
sión del trabajo que terminará permitiendo la especialización y profesionalización, de
modo que cada bien sea producido por un profesional específico y luego intercambiado.
– Industrialmente.—Cuando los procedimientos y medios de producción exceden las
posibilidades individuales surge la empresa, con la creación de talleres y fábricas en las
que el trabajo y la gestión de producción y distribución se dividen en una más específica
división del trabajo.

2. DIVISIÓN TÉCNICA DEL TRABAJO, INTERDEPENDENCIA


ECONÓMICA E INSTRUMENTOS DE COORDINACIÓN
EN LAS ECONOMÍAS DE MERCADO
Establecida la fórmula industrial, con la consiguiente división del trabajo que implica, las
necesidades humanas y su satisfacción precisan, además de su producción, el intercambio entre
sujetos, el comercio, el cual exige: mercado y dinero.
La convivencia económico – social, ante esta situación, llega a la cuestión de cómo se diri-
ge el proceso de producción y consumo:
– Qué bienes han de producirse y en qué cantidad.
– Cómo han de producirse, quién debe producirlos y con qué medios.
– A quién se dirigen los bienes producidos y qué necesidades satisfarán.
Estas tres cuestiones son comunes a todos los países y han sido comunes a lo largo del tiem-
po. La diversidad en la materialización de las respuestas a las mismas da lugar a los diferentes
sistemas económicos.
El sistema triunfante tras la caída de los regímenes de economía dirigida ha sido el de eco-
nomía de mercado. En él la satisfacción de las necesidades humanas con bienes escasos se pro-
duce por acuerdos libres o compraventas entre oferentes y demandantes.

6
-Pág.10-
U.D. 7.- ECONOMÍA Y TRABAJO EN EL MUNDO ACTUAL

– El hombre se agrupa en la familia, que es la unidad de consumo.


– Los bienes precisos se solicitan de la empresa, que es la unidad de producción.
– Las familias reciben los bienes y servicios a cambio del pago en dinero.
En el mercado existen dos grupos diferentes de bienes:
– Bienes de consumo, los que contribuyen a la satisfacción de las necesidades de forma
inmediata.
– Bienes o medios de producción, los que no se aplican directamente a satisfacer necesi-
dades sino que sirven para obtener o producir bienes de consumo.
En consecuencia, existen dos mercados internos, el de bienes de consumo y el de medios de
producción.
Las características básicas que definen el sistema de economía de mercado son:
— Papel preponderante de los precios.
— Importancia de la competencia.
— La empresa privada es la que dirige el proceso de producción de bienes.
— Existencia del derecho de producción de bienes.
— Existencia del derecho de propiedad privada.
— Las decisiones a veces parten del Estado, siempre que sea para:
– Producir bienes y servicios que no generen beneficios.
– Asegurar la producción necesaria.
– Asegurar la competencia.
– Procurar la justicia en la distribución de la riqueza.
– Evitar el paro forzoso.
La empresa protagoniza la producción de bienes, es la unidad de producción del sistema.
Para llevar a cabo su cometido la empresa necesita bienes y servicios con cuya ayuda obtendrá
el producto que venderá en el mercado. Así pues la empresa:
— Adquiere recursos:
– Materia prima.
– Maquinaria o medios de producción.
– Servicio de mano de obra.
— Los transforma.
— Los hace asequibles para el consumo.

7
-Pág.11-
CIENCIAS SOCIALES. GEOGRAFÍA E HISTORIA

Además de este aspecto técnico de fabricación de bienes, la empresa contempla otro aspec-
to vital: el financiero. Efectivamente, para poder llevar a cabo el proceso de producción la
empresa precisa dinero: para comprar la materia prima, para adquirir medios de producción,
para pagar la mano de obra y para cubrir los gastos de distribución y el mantenimiento de las
instalaciones. Los recursos financieros pueden provenir de distintas fuentes:
— Capital aportado por un individuo o grupo reducido de individuos. Esta fórmula
coincide con los primeros pasos de la actividad industrial, cuando el volumen de pro-
ducción y las expectativas de mercado no eran muy grandes.
— Establecimiento de una sociedad. Cuando las necesidades de capital sobrepasan las
posibilidades de las fortunas personales surge la asociación de varios individuos para
aportar el capital. Existen distintos tipos de sociedades:
– Sociedad regular colectiva.—Los socios se responsabilizan totalmente de su apor-
tación, de los beneficios y de las posibles deudas que puedan surgir en la gestión.
– Sociedad comanditaria.—En ella un socio responde íntegramente mientras que
los socios comanditarios solo son responsables de las posibles deudas de la empre-
sa en parte, en una parte proporcional a su aportación a la sociedad.
– Sociedad anónima.—En ella cada socio aporta una cantidad mediante la adquisi-
ción de acciones, que pueden comprarse y venderse sin que se altere, en principio,
la sociedad. Los accionistas o socios participan en la toma de decisiones propor-
cionalmente al número de acciones que poseen, decisiones sobre cuestiones gene-
rales de la empresa y, en particular sobre la elección de los gestores de la misma.
Además, los beneficios obtenidos se reparten proporcionalmente al número de
acciones entre los socios, a lo que se denomina reparto de dividendos.
– Sociedad limitada o de responsabilidad limitada.—En ella la participación en la
empresa se asume proporcionalmente también pero en este caso no existe la posi-
bilidad de transmisión de la “participación”. En los demás aspectos funciona como
una sociedad anónima.
El establecimiento de este tipo de sociedades permite la obtención de los llamados recursos
propios; pero también la empresa puede acudir a la obtención de recursos ajenos. Éstos pue-
den provenir de:
– Emisión de obligaciones.—Títulos adquiridos por particulares en los que la empresa
se compromete a devolver la cantidad aportada con un interés determinado.
– Contratación de créditos.—Obtención de recursos mediante la apertura de créditos
con entidades dedicadas a esta actividad financiera.
En estos casos, naturalmente, la empresa debe contar, en sus cálculos económicos, con la
necesidad de que los beneficios brutos sean suficientes para afrontar los gastos de producción y
para satisfacer las aportaciones ajenas.

8
-Pág.12-
U.D. 7.- ECONOMÍA Y TRABAJO EN EL MUNDO ACTUAL

2.1. LOS FACTORES DE PRODUCCIÓN EN LA EMPRESA


El trabajo es el más importante de los factores de producción y lo es por una doble razón:
– Cualitativamente, porque es realizado por el hombre, lo que le otorga una dimensión,
digamos, espiritual que no concurre en otros factores de producción, que son bienes
materiales.
– Cuantitativamente, porque de él proviene la generación de las rentas que llegan a los
individuos.
El trabajo debe ser compensado por la empresa mediante una retribución dineraria o en espe-
cie, que recibe el nombre de salario.
La tierra o naturaleza es el segundo factor. Por tierra o naturaleza entendemos el conjun-
to de bienes económicos utilizados en la producción tal como el hombre los encuentra en el
medio que habita.
El tercero de los factores es el capital, que debemos entender como el conjunto de medios
de producción y de productos intermedios situados entre los factores anteriores, a los que pode-
mos agrupar con la expresión factores originarios, y los bienes de consumo (en realidad, los
medios de producción son bienes de consumo también; pero unos bienes de consumo produci-
dos para producir).
Gracias al capital empleado en la producción ésta puede elevarse. La privación del consu-
mo inmediato para aumentar los medios de producción se denomina ahorro, que se opone a
consumo pero no a gasto, porque la cantidad ahorrada se gasta en aumentar el capital. A este
empleo de lo ahorrado se le denomina capitalización o también inversión. Al igual que al tra-
bajo se le retribuye con el salario, al capital le corresponde un pago que se denomina interés.
El cuarto de los factores es el empresario. Su misión es orientar los recursos productivos,
dirigirlos adecuadamente hacia la obtención de aquellos bienes que se precisan. Su retribución
se denomina beneficio y se producirá si el empresario acierta a llevar a cabo su misión.

2.2. EL PROCESO PRODUCTIVO AL NIVEL DEL ESTADO


Podemos pasar a considerar cómo discurre en la vida económica de un país este proceso de
producción y el consumo de los bienes producidos.
Entendemos por producción de un país el conjunto de procesos de producción realizados por
las distintas empresas del mismo. El valor de lo producido en el país, nos lleva a contemplar,
inicialmente, dos totales:
Producto Interior.—Suma de lo producido en el país por todas las empresas radicadas en
el mismo a lo que habrá que restarle las compras a otras empresas.

9
-Pág.13-
CIENCIAS SOCIALES. GEOGRAFÍA E HISTORIA

Producto Nacional.—El mismo, al que se le deducen las cantidades pagadas al exterior y


se le suman los ingresos obtenidos en el extranjero.
Además, hay que considerar otro término, el de renta nacional que es el total de rentas,
salarios y beneficios y debe ser igual a su Producto Nacional. La renta nacional es la que per-
mite afrontar el consumo y el ahorro.
No debemos olvidar que además de la producción de bienes materiales, el conjunto de la
actividad económica se completa con los servicios, es decir, con el conjunto de actividades que
no generan un producto palpable. Éstas se orientan a cuestiones como:
– Gestión de las actividades de la empresa.
– Distribución de los productos.
– Otros servicios relacionados con el ocio, la salud, la cultura, transporte, etc.
Una vez llegados al punto de la conclusión del proceso de producción, debemos tener en
cuenta que, una vez completado, el ciclo no puede volver a iniciarse sin realizar un proceso de
amortización, es decir, de reparación o mejora del capital fijo o medios de producción. Esta
amortización no puede considerarse consumo sino inversión.
Por otra parte, no podemos cerrar este apartado sin recordar que, en determinadas áreas de
la actividad económica, sobre todo en aquéllas en las que no existe demanda individual o no
existen perspectivas de beneficio y, por tanto, la iniciativa privada es escasa, debe intervenir
el Estado asumiendo el papel de empresario y empresa. Jugando dicho papel, el Estado dispo-
ne, además de recursos similares a las empresas privadas para su capitalización (participación
asociada, créditos, obligaciones), de los recursos de los que se provee mediante los impuestos,
ya sean éstos directos, cobrados sobre la renta percibida, o indirectos, añadidos a los precios
de bienes y servicios.

10
-Pág.14-
U.D. 7.- ECONOMÍA Y TRABAJO EN EL MUNDO ACTUAL

3. DIVISIÓN TÉCNICA Y SOCIAL DEL TRABAJO


Y ESTRUCTURA SOCIAL (EL DESARROLLO TECNOLÓGICO
Y SUS REPERCUSIONES EN EL MUNDO DEL TRABAJO. DESIGUALDADES
Y CONFLICTOS SOCIALES; LOS SINDICATOS; DERECHOS Y DEBERES
DE LOS TRABAJADORES; DIVISIÓN Y DISCRIMINACIÓN EN EL TRABA-
JO)

El crecimiento de las sociedades industriales ha determinado la definición de un organiza-


ción social del trabajo que muestra caracteres muy diferentes a los que caracterizaron la mayor
parte de la Historia. Las principales características de esa modificación en la organización social
del trabajo son:
– Concentración de capitales y creación de grandes empresas multinacionales.—A
medida que la producción exigía mayores inversiones, las pequeñas empresas se vie-
ron desbordadas por no poder atender a los costes que suponía la financiación del
capital. La solución estaba en la fusión de empresas pequeñas en otras mayores o la
creación de grandes empresas. Por esta vía se corre el riesgo de poner en peligro la
competencia por la creación de oligopolios o monopolios.
– Burocratización y racionalización.—Ante la complejidad creciente de la actividad
de producción se hace necesaria una organización racional de todos los procesos de
producción, la planificación, la distribución, etc.
– Mecanización y automatización.—Las innovaciones tecnológicas han permitido eli-
minar de los procesos productivos más rutinarios la mano de obra humana.
Todo esto ha modificado el papel del trabajador y la estructura social. El trabajador, que
en el taller artesanal participaba en todo el proceso de producción, ahora sólo interviene en una
parte del mismo, normalmente de modo rutinario por lo cual no le es preciso conocer los por-
menores del producto final, ni aprender más allá de lo específico de su actividad. Esto deshu-
maniza el trabajo y afecta negativamente a la satisfacción del individuo provocándole
aburrimiento, fatiga, desinterés. La solución a esta situación pasa por una acertada política, por
parte de las empresas, para mejorar las condiciones del trabajo y así satisfacer las aspiracio-
nes de sus trabajadores con el objetivo último de que, produciendo con satisfacción, lo harán
más y mejor, propiciando la generación de mayores beneficios. Esa mejora se puede producir
diversificando las actividades en las que puede intervenir el trabajador, creando condiciones
de relación y comunicación entre los individuos que participan en todo el proceso y abriendo
las puertas a la participación de los trabajadores en la gestión de la empresa. Este último
aspecto se puede traducir en la creación de cooperativas o empresas en las que los propios tra-
bajadores sean socios; en ellas la cogestión o la autogestión permite a los obreros participar de
los beneficios y, por tanto, implicarse directamente en el proceso completo.

11
-Pág.15-
CIENCIAS SOCIALES. GEOGRAFÍA E HISTORIA

En todo caso, las especiales características del mundo económico se traducen en la práctica
en diferentes tipos de desequilibrios y problemas.
En primer término, las estructuras social, económica y educativa se entremezclan para deter-
minar un reparto de las actividades laborales en el que la extracción social y educativa es fac-
tor determinante del nivel salarial y la exigencia de esfuerzo. Ello es lo que conduce a que
hablemos de trabajos más “nobles” y menos “nobles”, aquellos que son apetecibles por sus con-
diciones de realización, horario y salario, y aquellos que son rechazados por exigir más esfuer-
zo y ofrecer menos salario. De este modo quienes ya antes de integrarse en el mercado de
trabajo gozan de una posición social más elevada optan a los trabajos mejores, mientras que los
de clase más humilde no tienen más opción que emplearse en los menos demandados.
Otro importante problema es el del desequilibrio que se produce, en mayor o menor medida,
dependiendo de los países y sus correspondientes políticas económicas, entre la oferta de pues-
tos de trabajo y la demanda de los mismos lo que genera uno de los mayores problemas de las
sociedades industriales contemporáneas: el paro. Importante tanto para las personas afectadas,
que pueden encontrarse sin recursos, como para la economía en general.
Además, la evolución contemporánea de la política de empresa, asociada al acuciante pro-
blema del paro, ha generado la aparición del llamado empleo precario: los contratos tempora-
les y el trabajo sin contrato, situaciones en las que ocasionalmente el trabajador obtiene
importantes beneficios inmediatos, al igual que el empresario, que se beneficia, en el primer
caso, de ayudas públicas por la promoción del empleo y, en el segundo, por la ausencia de obli-
gaciones sociales; pero que pierde derechos a medio y largo plazo.
Esto nos lleva a considerar otros aspectos de especial trascendencia en el mundo del traba-
jo y de la economía: Los derechos de los trabajadores. Éstos, fruto de un largo proceso de
luchas sociales, se pueden resumir en la siguiente lista:
– Derecho al trabajo.
– Derecho a la elección de profesión.
– Derecho a la promoción a través del trabajo.
– Derecho a una remuneración suficiente.
– Derecho a unas condiciones idóneas de seguridad e higiene en el trabajo.
– Derecho a la defensa de sus intereses mediante el asociacionismo y, en su caso la huelga.
– Derecho a un sistema de seguridad social que proteja al trabajador en caso de enfer-
medad o invalidez transitoria o permanente.
La defensa de estos derechos se realiza por medio de los tribunales, ya que los ordena-
mientos jurídicos de casi todos los países los contemplan como derechos constitucionales, Mas
la acción ante los tribunales o, antes de llegar a ellos, ante la empresa o, incluso, el Estado suele
estar amparada por los sindicatos, que juegan un papel esencial en el desarrollo de la vida labo-
ral, canalizando las aspiraciones de los trabajadores y preservando lo que podríamos llamar
vigilancia cotidiana de sus derechos.

12
-Pág.16-
U.D. 7.- ECONOMÍA Y TRABAJO EN EL MUNDO ACTUAL

Uno de los derechos fundamentales de los trabajadores, por extensión del principio que ins-
pira o debe inspirar todos los demás, es el de la igualdad. Se trata de un aspecto que por sus
especiales connotaciones hemos querido dejar para el final de este apartado. De acuerdo con
este principio no debe existir discriminación alguna por ninguna razón. Sin embargo, lo cierto
es que en la sociedad actual existen aún profundas discriminaciones laborales que crecen a
medida que descendemos en la escala del desarrollo. Esa discriminación se manifiesta esen-
cialmente en la división del trabajo, las condiciones del mismo, la remuneración y el respeto a
los restantes derechos. Esta discriminación no es, desde luego, una discriminación institucional
puesto que las leyes la prohíben expresamente, se trata de una discriminación real que bordea
los límites de la legalidad.
Existe, así, una discriminación en razón del origen étnico y social que lleva a que determi-
nados grupos, menos favorecidos, inmigrantes, etc., sean los que ocupan los puestos de trabajo
menos demandados por el resto y en unas condiciones negativas.
Existe también discriminación hacia los discapacitados físicos y psíquicos. Si bien es cier-
to que determinados trabajos no pueden ser desempeñados por ellos, en función del grado y
carácter de su discapacidad, muchas veces son rechazados sin motivos reales y otras empleados
precariamente.
Y por fin existe una discriminación en razón del sexo. Es cierto que la situación laboral y
social de la mujer de hoy en día era sólo parte del sueño de las mujeres que empezaron a luchar
por sus derechos en el siglo XIX, pero aún hoy algunos trabajos se mantienen cerrados a la
intervención femenina y en muchos casos, aun empleadas en los mismos trabajos que hombres,
perciben salarios inferiores y sus derechos laborales son recortados.
No cabe duda que la sensibilización frente a estas circunstancias ha hecho que se limen
muchas de las aristas del problema, pero sigue siendo un reto para las relaciones laborales
futuras.

13
-Pág.17-
CIENCIAS SOCIALES. GEOGRAFÍA E HISTORIA

4. INTERDEPENDENCIA Y DESIGUALDAD ECONÓMICA,


TÉCNICA Y POLÍTICA ENTRE LOS DISTINTOS PAÍSES
DEL MUNDO: DESARROLLOS DESIGUALES
Y NEOCOLONIALISMO
Como ya vimos en el Tema 3, los niveles de desarrollo en el mundo no son, ni mucho menos,
homogéneos. Sin embargo, los parámetros en los que se fundamentaba hace unos años esta des-
igualdad se han matizado y modificado considerablemente. En cualquier caso, todavía hoy
seguimos utilizando el término Tercer Mundo que nos sugiere de forma inmediata ideas de
pobreza, incultura, desnutrición, subdesarrollo…
El término Tercer Mundo fue acuñado en la época de la Guerra Fría para designar a los paí-
ses no incluidos ni en el bloque capitalista ni el socialista y caracterizados por el papel subsi-
diario de sus economías al ser exclusivamente exportadores de materias primas e importadores
de productos manufacturados.
El conjunto de los países que forman ese denominado Tercer Mundo surge como conse-
cuencia del proceso descolonizador, es decir, del paulatino acceso a la independencia de los
territorios que habían formado parte de los imperios coloniales europeos.
Europa inició en el siglo XV un proceso de expansión exterior que fue debido a:
– La necesidad de materias primas (primero especias, después materias primas indus-
triales).
– El afán imperialista, movido por criterios económicos y de filosofía política.
Comenzando así un proceso de conquista y explotación de nuevas tierras, lo cual produjo
diversos efectos, tanto en las áreas colonizadas como en Europa:
Positivos:
— Conocimiento del mundo,
— Extensión de la cultura europea (¿positivo?):
– Religión.
– Lenguas.
– Formas de vida.
Negativos:
— Explotación irracional.
— Destrucción de culturas autóctonas.
— Sistemas esclavistas.

14
-Pág.18-
U.D. 7.- ECONOMÍA Y TRABAJO EN EL MUNDO ACTUAL

En la mayor parte de los casos, la preocupación europea era explotar los recursos natura-
les de modo rentable y, además, la difusión de la religión junto con las formas de vida euro-
peas. Pero no hubo un proceso de creación de infraestructuras económicas, sociales, políticas
y culturales que permitieran a la población indígena asimilar el torrente de novedades y dar el
salto a la civilización sin traumas.
Esto fue especialmente claro en África y Asia donde el proceso descolonizador dejó en
manos de los indígenas unos Estados muchas veces artificiales en sus fronteras, para cuya admi-
nistración no estaban suficientemente preparados.
Esto favoreció la aparición de unos nuevos sistemas de dominio: el neocolonialismo capi-
talista (control financiero y comercial) y la penetración y dependencia económico-política
soviética.
La desaparición de la U.R.S.S. ha cambiado considerablemente la situación de interdepen-
dencias económico-políticas de los países del Tercer Mundo.

4.1. ORÍGENES
El proceso de expansión europea se inició, como se ha dicho, en el siglo XV. Fue protago-
nizado inicialmente por España y Portugal y tuvo en América su principal campo de acción. Al
afán de obtención de metales, en el caso de la América española, hay que unir el de estableci-
miento de rutas seguras para el comercio de especias por parte portuguesa.
En esta fase inicial del colonialismo, el avance territorial, salvo en el caso de España, fue
mínimo y a los países ibéricos se sumaron pronto los británicos, franceses y holandeses que
abrieron a sus intereses diversas partes del mundo.
Sin embargo, el fenómeno que conocemos con el nombre de Imperialismo es algo específi-
co de finales del siglo XIX y comienzos del XX. El mundo quedará dividido en grandes impe-
rios bajo la autoridad de las potencias europeas.
América se desmarca de este proceso, si bien sigue derroteros que terminarán incluyendo a
sus tierras en fenómenos similares, adelantándose en el tiempo. Efectivamente, la América bri-
tánica experimenta el primer proceso de descolonización cuando, tras la Declaración de Inde-
pendencia, Estados Unidos se convierte en una República Independiente. Tras ello se inicia una
sucesión de revoluciones liberales entre las que se encuentran las que motivan la independen-
cia de las colonias españolas. Posteriormente, al comenzar el fenómeno del imperialismo, en
América se produce la eclosión del llamado neocolonialismo, al convertirse los EE.UU. en los
“protectores” que, al amparo de la “doctrina Monroe” (“América para los americanos”), pre-
servarán a las repúblicas del hemisferio de la expansión imperialista europea.
La Europa industrializada, imposibilitada de evitar la competencia entre sus diversos
Estados, se vio impulsada a la expansión porque necesitaba productos alimentarios, materias

15
-Pág.19-
CIENCIAS SOCIALES. GEOGRAFÍA E HISTORIA

primas industriales y nuevas fuentes de energía, además de mercados para sus productos
manufacturados.
Las diversas potencias europeas se lanzaron a la conquista del mundo justificando sus afa-
nes expansionistas con convencidos tratados políticos, que argüían afirmaciones como:
“Los pueblos coloniales deben estar sometidos a una tutela que les capacite para autogo-
bernarse”.
“Es necesario un período de dominación colonial para pasar a un estadio de civilización
superior” (JOHN STUART MILL, filósofo y economista).
“Somos una raza de gobernantes, predestinada por nuestras cualidades, así como por nues-
tras virtudes, a extendernos por el mundo” (JOSEPH CHANBERLAIN, primer ministro británico).

4.2. LA DESCOLONIZACIÓN
El proceso descolonizador se producirá, sobre todo, a partir del final de la II Guerra Mun-
dial y como consecuencia, casi siempre, de la asunción de conciencia nacional de los territo-
rios colonizados. Los movimientos independentistas fueron inicialmente ahogados por el afán
de las potencias colonizadoras de mantener su posición de ventaja para la obtención de mate-
rias primas.
Finalmente, el acceso a la independencia de los diferentes territorios se produjo de diversas
formas:
– En unos casos, se produjo una descolonización pacífica como consecuencia de la alian-
za o el consentimiento de la burguesía autóctona, si es que existía, o con los jefes o auto-
ridades tradicionales. En estos casos, la potencia colonial procuró mantener la
dominación por medios más sutiles (Este fue el caso de la mayor parte de los territorios
africanos).
– En otros casos se aprovecharon las disensiones entre las fuerzas políticas o religiosas
para imponer las condiciones de las potencias colonizadoras (caso de la India).
– En otros, la independencia fue acompañada del afán por modificar drásticamente las
estructuras económicas, sociales y políticas, en estos casos la guerrilla o la guerra abier-
ta fue inevitable.
En cualquier caso, el proceso de descolonización se prolongó en el tiempo. De hecho, pode-
mos considerarlo como un proceso inconcluso pues aún quedan territorios coloniales, entre los
que destacan casos como el de Gibraltar, último territorio colonial en Europa.

16
-Pág.20-
U.D. 7.- ECONOMÍA Y TRABAJO EN EL MUNDO ACTUAL

4.3. EL NEOCOLONIALISMO
La independencia de los territorios coloniales no supuso siempre una auténtica asunción de
la soberanía pues, en la práctica, la vinculación económica con la antigua metrópoli o con la
nueva potencia dominante, limitaba considerablemente la libertad de actuación de los nuevos
gobiernos.
La dependencia económica entrañaba, de hecho, severas restricciones de la soberanía pues:
– La política económica debía tener en cuenta los intereses de la potencia dominante y
no sólo los nacionales.
– La política exterior estaba también supeditada a las directrices de la potencia neocolonial
a la que el país quedaba ligado por diversos vínculos que derivaban en su “satelización”.
La dependencia se materializa en diversos aspectos:
– Predominio de empresas multinacionales en la explotación de los recursos naturales.
– Producción supeditada a las necesidades de materias primas de los países dominan-
tes, lo que conduce a monocultivos o monoexplotaciones mineras quedando, además, los
países subdesarrollados al margen de la determinación de los precios.
– Paulatino desequilibrio en las balanzas comerciales por el crecimiento de los precios de
los productos industriales y el estancamiento de los de las materias primas.
Ante los desequilibrios económicos generados, las potencias promueven ayudas económi-
cas que, en la práctica, tienden a mantener los intereses propios y aumentan el endeudamiento
de los países subdesarrollados.
Durante la época de la Guerra Fría, a las razones de interés económico hay que unir las de
carácter político y estratégico. De modo que muchos países del denominado Tercer Mundo fue-
ron cayendo en las esferas de influencia de EE.UU. o de la U.R.S.S.
El juego de influencias políticas mantuvo a los nuevos países, en la práctica, como reales
dependencias de sus antiguas metrópolis o de las superpotencias. A la sombra de esta situación
se inició el llamado “movimiento de los no alineados” (I Conferencia de los Países no Alinea-
dos en Belgrado, 1961), catalizado entorno a aquellos países que pretendían mantener su des-
vinculación política de cualquiera de los dos bloques. Esta resolución era a veces más bien
ficticia pues, en realidad, la mayoría de los presuntos “no alineados”, dependían en alguna
medida de alguno de ellos.
Por otra parte, en el momento de la independencia, la mayor parte de los países nuevos acep-
taron las instituciones impuestas por la administración colonial o establecieron otras calcadas
de las de la metrópoli. Dichas instituciones no han aportado la estabilidad necesaria, no han con-
tribuido al desarrollo de la democracia política ni han conducido a la explotación racional de las
riquezas. La ineficacia de tales sistemas condujo en muchos países a su sustitución por regí-
menes militares que no fueron mejores plataformas para el desarrollo.

17
-Pág.21-
CIENCIAS SOCIALES. GEOGRAFÍA E HISTORIA

Otro problema que añadir al panorama de la descolonización es el de las fronteras. La inde-


pendencia de los nuevos países estaba determinada casi siempre por la voluntad de las poten-
cias coloniales que mantuvieron las divisiones territoriales fijadas por ellas. De este modo, en
un mismo Estado se incluían etnias o grupos tribales que habían estado tradicionalmente sepa-
rados, cuando no enfrentados; o, en el caso contrario, algunas fronteras partían entre varias
soberanías las tierras ocupadas durante siglos por un mismo grupo. Estos hechos motivaron y
aún hoy provocan importantes problemas, de los que podemos citar:
– Guerra de Biafra (Nigeria),
– Independencia de Eritrea (primero parte de Etiopía)
– Independencia de Bangla Desh (primero parte oriental de Pakistán).
– Guerras entre los países árabes e Israel
– Luchas entre etnias rivales (Liberia, Zaire, Ruanda, Burundi)
La toma de conciencia de los países en vías de desarrollo y su afán por preservar su sobe-
ranía cuajó, en 1973, en un acto de consecuencias mundiales tras la asunción por parte de los
países exportadores de petróleo de medidas tendentes a una fijación de los precios del crudo
más en consonancia con las necesidades de ellos que con los intereses de las potencias. Aquel
acontecimiento fue la base de una crisis económica mundial: por primera vez los “territorios-
fuente cerraban el grifo” para dosificar sus productos y obtener de ellos beneficios propios.
Los radicales cambios producidos a partir de los años 80, con la desaparición de los bloques,
además de los nuevos entramados económicos mundiales, han abierto nuevas perspectivas a
estos países que de la denominación de Tercer Mundo o Países subdesarrollados, han pasado a
la de países en vías de desarrollo (PVD).
En todo caso, la evolución más reciente de la economía y la política está conduciendo de
hecho a una interdependencia creciente, en el camino hacia eso que se ha dado en llamar glo-
balización, y que, al menos en la superficie, parece ir aproximando modelos y comportamien-
tos. Sin embargo, en ese conjunto de países que hemos llamado Tercer Mundo, se siguen
manteniendo estructuras sociales, económicas y políticas débiles que le hacen ser pasto de la
voracidad económica de las grandes empresas multinacionales.
En esta línea, recordando lo que ya aportábamos en el Tema 3, el Tercer Mundo es visto por
las grandes empresas multinacionales como un gran mercado de mano de obra, materias primas
y productos de consumo, de modo que las estrategias de obtención de materias primas, produc-
ción, comercialización y gestión tienen en cuenta la rentabilidad de la realización de estas acti-
vidades en unos marcos geográficos o en otros.
Así, la obtención de materias primas, mediatizada, claro está, por su localización geográ-
fica, es objeto de inversiones para su compra a bajo coste o su producción mediante la inter-
vención de capital en explotaciones directas (plantaciones, pozos petrolíferos, minas...).

18
-Pág.22-
U.D. 7.- ECONOMÍA Y TRABAJO EN EL MUNDO ACTUAL

La elaboración es objeto también de estudios estratégicos, valorando la rentabilidad en fun-


ción de la calidad, preparación y costes de la mano de obra y las posibilidades de comerciali-
zación desde las zonas de implantación de las factorías. Este punto ha llevado a muchas
empresas a radicar sus plantas de producción en países pobres, con mano de obra muy
barata. Esto ha generado la afluencia de capital a manos de dicha población lo que ha aumen-
tado sus posibilidades de consumo pero no se ha canalizado hacia inversiones en una industria
propia. Es, pues, un nuevo medio de interdependencia.
En todo caso, algunos países han sabido aprovechar el tirón de las nuevas estrategias pro-
ductivas y han logrado invertir adecuadamente los beneficios en empresas propias, han conse-
guido aumentar el nivel de vida y, en general, el desarrollo económico, social y cultural. Estos
Nuevos Países Desarrollados (NPD) van descollando por encima de los que todavía mantienen
unos niveles más bajos, aunque en su entramado social restan aún enormes bolsas de pobreza y
subdesarrollo.

19
-Pág.23-
CIENCIAS SOCIALES. GEOGRAFÍA E HISTORIA

RESUMEN
— La actividad económica es una parte de la conducta humana. En ella el hombre se
enfrenta a necesidades físicas y fisiológicas que debe satisfacer eligiendo un orden de
preferencias.
— Para que se produzca la actividad económica los bienes (objetos materiales y servicios)
han de ser escasos y susceptibles de usos alternativos..
— En la actividad económica se obtienen los bienes básicos, se transforman aquellos que
no pueden ser consumidos directamente, se distribuyen y se consumen.
— Este proceso se ha realizado a lo largo de la historia familiarmente, primero, profesio-
nalmente, después y, finalmente, de modo industrial.
— En el sistema industrial de consumación de los procesos económicos, la satisfacción de
las necesidades de todos exige mercados y dinero.
— En la economía de mercado:
– Hay un mercado de oferentes y demandantes
– La familia es la unidad de consumo: demanda bienes y ofrece trabajo
– La empresa es la unidad de producción: demanda trabajo y ofrece bienes
– La empresa compensa a los individuos por su trabajo con remuneración
— En el mercado existen bienes de consumo y bienes de producción
— La empresa es la protagonista de la producción: adquiere recursos, los trasforma y los
hace asequibles al consumo.
— Para completar el proceso la empresa necesita financiación. Ésta se obtiene:
– Con recursos propios:
- Individuo propietario
- Sociedad
- Sociedad regular colectiva
- Sociedad comanditaria
- Sociedad anónima
- Sociedad limitada
– Con recursos ajenos:
- Obligaciones
- Créditos
— Los factores de producción son: El trabajo (el más importante), la naturaleza, el capital
y el empresario.
— El ahorro en el proceso productivo es la privación de consumo inmediato para aumentar
los medios de producción: capitalización o inversión.

20
-Pág.24-
U.D. 7.- ECONOMÍA Y TRABAJO EN EL MUNDO ACTUAL

— La remuneración del trabajo es el salario; la del capital, el interés o renta; la del empre-
sario, el beneficio.
— El Producto Interior es la suma de lo producido en el país.
— El Producto Nacional es el Producto Interior menos las cantidades pagadas al exterior y
más los ingresos obtenidos en el extranjero.
— Renta Nacional es el total de rentas, salarios y beneficios.
— El Estado interviene en el proceso económico, sobre todo, para afrontar las produccio-
nes y servicios cuyo parco beneficio no atrae la inversión privada.
— La financiación del Estado se obtiene mediante los impuestos directos e indirectos.
— En las sociedades industriales modernas la concentración de capitales con la creación de
grandes empresas, la burocratización y racionalización y la mecanización y automatiza-
ción han modificado el papel del trabajador y la estructura social.
— La mejora de las condiciones de trabajo, la diversificación de actividades y la participa-
ción en la toma de decisiones y en los beneficios, permiten superar los perjuicios para el
trabajador.
— El paro y el empleo precario son las lacras de la sociedad industrial, desde el punto de
vista de los trabajadores.
— Los derechos de los trabajadores se defienden en los tribunales y por los sindicatos.
— El reto del mundo del trabajo de hoy está en la eliminación de las desigualdades por
cuestiones raciales, de discapacidad y, especialmente, por razón del sexo.
— El proceso descolonizador, el crecimiento económico desigual y el neocolonialismo son
los grandes causantes de los desequilibrios económicos entre países.
— El neocolonialismo se caracteriza por la dependencia económica y política de unos paí-
ses respecto a otros y se manifiesta por medio de la supeditación de la economía y la
política, el dominio de las empresas multinacionales y el desequilibrio en las balanzas
comerciales.

21
-Pág.25-
CIENCIAS SOCIALES. GEOGRAFÍA E HISTORIA

EJERCICIOS DE AUTOCOMPROBACIÓN

1. Los que contribuyen a la satisfacción de las necesidades de forma inmediata son los:
A. Bienes de consumo. B. Bienes de producción.
C. Demandas de dinero. D. Ofertas de producción.

2. En la economía de mercado la producción de bienes la protagoniza:


A. El Estado. B. El mercado de trabajo.
C. El mercado de los productos. D. La empresa.

3. El Estado puede intervenir en el proceso productivo. En tal caso su financiación procede de:
A. Los trabajadores. B. Los empresarios.
C. Las rentas. D. Los impuestos.

4. La renta nacional es:


A. La diferencia entre consumo y ahorro.
B. La diferencia entre Producto Nacional y Producto Interior.
C. La suma de salarios, beneficios y rentas.
D. Los beneficios menos los salarios.

5. Uno de los factores que puede contribuir a la mejora de la situación de los trabajadores es:
A. La mecanización. B. La burocratización.
C. La concentración de capitales. D. La participación en los beneficios.

6. La defensa de los derechos de los trabajadores se realiza:


A. Por ellos mismos exclusivamente. B. Ante los tribunales y por los sindicatos.
C. Ante los empresarios. D. Ante los partidos políticos.

7. La Iª conferencia de los países No alineados tuvo lugar en:


A. Berlín. B. Dublín. C. Belgrado. D. París.

8. Las siglas NPD corresponden a:


A. Naciones Productoras Dominantes. B. Nuevas Producciones Diversificadas.
C. Naciones Petrolíferas Desarrolladas. D. Nuevos Países Desarrollados.

22
-Pág.26-
U.D. 7.- ECONOMÍA Y TRABAJO EN EL MUNDO ACTUAL

9. El proceso de reparación o mejora del capital fijo o medios de producción se denomina:

A. Capitalización. B. Amortización.

C. Inversión de activos. D. Ahorro.

10. La retribución del empresario se denomina:

A. Renta. B. Beneficio. C. Salario. D. Excedente.

23
-Pág.27-
CIENCIAS SOCIALES. GEOGRAFÍA E HISTORIA

RESPUESTAS A LOS EJERCICIOS

1. A

2. D

3. D

4. C

5. D

6. B

7. C

8. D

9. B

10. B

24
-Pág.28-
U . D . 8 . - P A RT I C I PA C I Ó N Y CONFLICTO POLÍTICO EN EL MUNDO ACTUAL

ÍNDICE
OBJETIVOS . . . . . . . . . . . . . . . . . . . . . . . . . . . . . . . . . . . . . . . . . . . . . . . . . . . . . . . . . . . . . . 2

INTRODUCCIÓN . . . . . . . . . . . . . . . . . . . . . . . . . . . . . . . . . . . . . . . . . . . . . . . . . . . . . . . . . 3

MAPA CONCEPTUAL . . . . . . . . . . . . . . . . . . . . . . . . . . . . . . . . . . . . . . . . . . . . . . . . . . . . . 4

DESARROLLO DE CONTENIDOS . . . . . . . . . . . . . . . . . . . . . . . . . . . . . . . . . . . . . . . . . . 5

1. LOS PRINCIPIOS E INSTITUCIONES DE LOS REGÍMENES DEMOCRÁTI-


COS. (SU PRESENCIA EN LA CONSTITUCIÓN ESPAÑOLA.
LAS INSTITUCIONES CONSTITUCIONALES ESPAÑOLAS) . . . . . . . . . . . . . . 5

2. LA ORGANIZACIÓN TERRITORIAL DE ESPAÑA. LA NACIÓN ESPAÑOLA,


MUNICIPIOS, PROVINCIAS Y COMUNIDADES AUTÓNOMAS. ÓRGANOS
Y COMPETENCIAS . . . . . . . . . . . . . . . . . . . . . . . . . . . . . . . . . . . . . . . . . . . . . . . . . . 24

3. ESPAÑA EN EL MUNDO: LA COMUNIDAD EUROPEA E IBEROAMÉRICA . . . 31

4. TRANSFORMACIONES Y TENSIONES EN LAS RELACIONES


INTERNACIONALES: ORGANISMOS INTERNACIONALES; LA CRISIS
DEL SISTEMA DE BLOQUES Y LA OTAN; EL PROCESO DE UNIDAD
EUROPEA; UNA CRISIS O CONFLICTO SOCIAL RELEVANTE;
PROBLEMAS Y PERSPECTIVAS PARA LA PAZ . . . . . . . . . . . . . . . . . . . . . . . . . 33

5. CAUCES Y RETOS ACTUALES PARA LA PARTICIPACIÓN CIUDADANA:


PARTIDOS Y ORGANIZACIONES SOCIALES; INSTITUCIONES Y CENTROS
DE INFORMACIÓN Y AYUDA CIUDADANA; EL PAPEL DE LOS MEDIOS
DE COMUNICACIÓN DE MASAS. . . . . . . . . . . . . . . . . . . . . . . . . . . . . . . . . . . . . . 51

RESUMEN . . . . . . . . . . . . . . . . . . . . . . . . . . . . . . . . . . . . . . . . . . . . . . . . . . . . . . . . . . . . . . . 53

EJERCICIOS DE AUTOCOMPROBACIÓN . . . . . . . . . . . . . . . . . . . . . . . . . . . . . . . . . . . 55

RESPUESTAS A LOS EJERCICIOS . . . . . . . . . . . . . . . . . . . . . . . . . . . . . . . . . . . . . . . . . 56

-Pág.29-
CIENCIAS SOCIALES. GEOGRAFÍA E HISTORIA

OBJETIVOS
Al finalizar el estudio de esta Unidad Didáctica, el alumno será capaz de:

• Comprender los principios en los que se fundamente la Democracia y analizar


cómo los mismos se materializan en la Constitución española.

• Entender la estructura territorial española en sus distintos niveles administrati-


vos.

• Asimilar los grandes rasgos de las relaciones internacionales de España y cono-


cer el carácter y estructura de los organismos internacionales más importantes.

• Valorar los diferentes canales de participación política y social.

-Pág.30-
U . D . 8 . - P A RT I C I PA C I Ó N Y CONFLICTO POLÍTICO EN EL MUNDO ACTUAL

INTRODUCCIÓN
L a política en el mundo actual es el contenido que afrontamos en este tema. Partien-
do de la definición de los principios e instituciones que definen el sistema demo-
crático, nos acercaremos a la Constitución Española para estudiar la plasmación de tales
principios en el sistema político español. Además, una vez analizada la Constitución
española, nos acercaremos a la plasmación de la misma en la realidad de la organización
territorial española.
A continuación nos acercaremos a las relaciones internacionales. En primer término
veremos cómo se establecen las principales orientaciones de la política exterior españo-
la para, a continuación, analizar con un poco más de detalle las principales organizacio-
nes internacionales en las que España se integra: ONU, FMI, OTAN y, sobre todo, la
Unión Europea.

Finalmente, observaremos los principales cauces de participación ciudadana en la vida


política y social.

-Pág.31-
CIENCIAS SOCIALES. GEOGRAFÍA E HISTORIA

M A PA C O N C E P T U A L

PARTICIPACIÓN Y CONFLICTO POLÍTICO

DEMOCRACIA. PRINCIPIOS E INSTITUCIONES

CONSTITUCIÓN ESPAÑOLA

ORGANIZACIÓN TERRITORIAL ESPAÑOLA

ESPAÑA EN EL MUNDO

ONU

ORGANIZACIONES FMI
INTERNACIONALES
OTAN

Unión Europea

CAUCES Y RETOS
PARA LA
PARTICIPACIÓN
CIUDADANA

-Pág.32-
U . D . 8 . - P A RT I C I PA C I Ó N Y CONFLICTO POLÍTICO EN EL MUNDO ACTUAL

1. LOS PRINCIPIOS E INSTITUCIONES


DE LOS REGÍMENES DEMOCRÁTICOS.
(SU PRESENCIA EN LA CONSTITUCIÓN ESPAÑOLA.
LAS INSTITUCIONES CONSTITUCIONALES ESPAÑOLAS)

1.1. UNA HISTORIA HACIA LA DEMOCRACIA


La Democracia, entendida como el gobierno de los ciudadanos, nació en la Grecia clá-
sica. Sin embargo, lo que los griegos entendían por “ciudadano” convertía su sistema polí-
tico en una especie de “oligarquía ampliada”. Efectivamente, sólo tenían el “estatus” de
ciudadano un número limitado de habitantes de la polis, del que estaban excluidos la masa
de esclavos que servían de soporte a la producción. En cualquier caso, el talante, la menta-
lidad que personajes como Solón, Clístenes o Pericles imbuyeron en la organización social
y política pusieron las bases sobre las que, posteriormente, se edificaron los sistemas demo-
cráticos.

El sistema político romano de la época de la República, como también hemos tenido


ocasión de analizar en estas páginas (Tema 4), se aproximaba al concepto de Democracia,
salvando las obvias distancias. Al menos era más democrático, más participativo, de lo que,
en cualquier caso, fue cualquiera de los sistemas políticos imperantes antes, durante y des-
pués.

Así es, la Historia ha estado dominada, esencialmente, por sistemas de carácter autoritario
en los que la voluntad de un individuo, por sí, o como máximo exponente de un grupo reduci-
do de privilegiados, se ha impuesto a la mayoría. En Egipto o en Mesopotamia, por ejemplo,
vimos (Tema 4) cómo el sistema se definía como Teocracia: había un poder absoluto de un indi-
viduo que era asimilado a la divinidad, ya fuera como intermediario entre Dios y los hombres,
ya como el mismo Dios. Esa vinculación poder divino – poder humano, de una forma u otra ha
estado presente en los grandes sistemas autoritarios. Lo estuvo en la Roma Imperial, que hizo
de sus emperadores dioses, en la China imperial, en Japón o en las culturas americanas preco-
lombinas. Y lo estuvo en los sistemas monárquicos absolutos de la Europa medieval, en los que
la vinculación de la Iglesia a la vida política era casi absoluta.

El Absolutismo y el Despotismo Ilustrado, cuyas propias denominaciones reflejan su


carácter, fueron el marco en el que se produjeron los cambios sobre los que se fueron edifican-
do las ideas que conducirían a la implantación de los sistemas liberales.

5
-Pág.33-
CIENCIAS SOCIALES. GEOGRAFÍA E HISTORIA

El punto de inflexión hay que situarlo a finales del siglo XVIII. Antes, no podemos olvidar
la mención a algún sistema pseudo democrático como, por ejemplo, el de la República de Vene-
cia, durante parte de la Edad Media y la Moderna. Después, no debemos tampoco olvidar la
existencia de regímenes que cercenaron el desarrollo de la Democracia de modo temporal, aun-
que en algunos casos fuera un tiempo especialmente largo o, aun siendo un tiempo corto, una
intensidad especialmente negativa; como, por ejemplo, han sido la sangrienta experiencia del
nazismo, las dictaduras que han salpicado la Historia de Hispanoamérica, los regímenes comu-
nistas o las más recientes experiencias vividas en los Balcanes.

La Declaración de Independencia de los Estados Unidos (1776), su Constitución (1787)


y la Revolución Francesa (1789-1792), fueron los primeros pasos del establecimiento de sis-
temas democráticos liberales. A partir de ese momento, a lo largo y ancho del mundo, sobre
todo en Europa y América, se vivirá una continua lucha entre las tendencias autoritarias y las
democráticas. Así, en España, por ejemplo, durante el siglo XIX se vive un permanente enfren-
tamiento entre el liberalismo y el absolutismo y, luego, establecido el Estado liberal, en el siglo
XX pasará por momentos dramáticos con dos dictaduras y una Guerra Civil entre ambas, hasta
la restauración democrática del último cuarto del siglo.

1.2. PRINCIPIOS E INSTITUCIONES


Esencialmente, los principios sobre los que se fundamenta la Democracia, son los siguientes:

— Establecimiento y respeto, por parte de los poderes públicos, de un conjunto de dere-


chos individuales y colectivos de los ciudadanos, inspirados por el principio de la
igualdad, entre los que se encuentran:
– Derecho a la vida.
– Libertad ideológica y religiosa.
– Libertad de expresión.
– Derechos de reunión, asociación y manifestación.
– Derecho de participación política: elegir y ser elegido.
– Derecho a la seguridad jurídica.
– Derecho a la educación.
– Libertad en las actividades económicas.
– Derecho al trabajo.

— Soberanía popular. La autoridad, la potestad de gobierno corresponde al conjunto de


los ciudadanos, que gobiernan a través de representantes elegidos por ellos.

6
-Pág.34-
U . D . 8 . - P A RT I C I PA C I Ó N Y CONFLICTO POLÍTICO EN EL MUNDO ACTUAL

— Separación de poderes

Existen en el funcionamiento del Estado una serie de aspectos, atribuciones o funcio-


nes que, por su carácter, pueden agruparse, como estableció Montesquieu, en tres
“áreas” o poderes:

– Poder legislativo.—El de elaborar y aprobar las leyes por las que ha de regirse el
funcionamiento del Estado, su sociedad, su economía, etc.

– Poder ejecutivo.—El que agrupa las funciones de adopción de las medidas espe-
cíficas de gobierno y administración.

– Poder judicial.—El que se encarga de velar por el correcto cumplimiento de las


leyes.

Estos poderes deben mantenerse independientes unos de otros, si bien, existen normalmen-
te mecanismos de relación como es el control que ejerce el poder legislativo sobre el ejecutivo
mediante distintos procedimientos, o el que, bajo determinados y estrictos requisitos, puede rea-
lizar el poder judicial sobre los otros dos.

En función de tales principios, se edifican determinadas instituciones encargadas de mate-


rializarlos.

Así, el poder ejecutivo es ejercido por un individuo, elegido directamente por el pueblo o
indirectamente a través de otras instituciones, que designa una serie de colaboradores que se
reparten las distintas parcelas de la administración del Estado. Los nombres que reciben son
diferentes según los países: Presidente, Presidente del Gobierno, Canciller, Primer Ministro…,
para el caso del primero; Ministros, Secretarios, Consejeros… para el de los segundos.

El poder legislativo, por su parte, es ejercido por asambleas integradas por representantes
elegidos por la población. Las fórmulas de elección, la estructura, las atribuciones y el sistema
de funcionamiento de dichas asambleas es muy diverso. Así, hay Estados en los que existe una
sola cámara de representantes, otros en los que existen dos, con diferente composición; los hay
en los que la elección se realiza por un sistema mayoritario, en otros por un sistema proporcio-
nal; en unos casos tienen amplios poderes de limitación de las actuaciones del Gobierno, en
otras éstos son limitados… Por otra parte, los nombres que estas instituciones reciben dependen
de la tradición y carácter de cada Estado: Asamblea Nacional, Parlamento, Congreso…

El poder judicial, en términos generales, es ejercido por jueces independientes. En todo


caso, el sistema de funcionamiento jurídico de cada Estado presenta matices diversos. Así, hay
países en los que la iniciativa de actuación judicial corresponde al juez; otros en los que corres-
ponde a los fiscales. En unos Estados el juez actúa individualmente, en otros juzga un jurado
integrado colegiadamente por varios jueces, en otros existen jurados populares…

7
-Pág.35-
CIENCIAS SOCIALES. GEOGRAFÍA E HISTORIA

Además, en todos los Estados existe, normalmente con carácter unipersonal, una Jefatura
del Estado. Ésta corresponde en algunos países a un Monarca (Rey, Conde, Duque, Príncipe…)
que hereda el cargo y lo ejerce de forma vitalicia; en otros a un Jefe de Estado elegido para un
tiempo limitado, que recibe diferentes nombres (Presidente de la República, Presidente, Gober-
nador General…). Las atribuciones de la Jefatura del Estado son también diversas según los paí-
ses; puede tratarse de un cargo simbólico de alta representación del Estado, al margen de la
contienda política cotidiana, ejercer un poder de moderación entre los demás poderes o puede
asumir amplias prerrogativas.

Por otra parte, en el ámbito de la administración del Estado, existen diversas fórmulas de
organización territorial que pueden implicar sistemas de funcionamiento del Estado muy dife-
rentes. Así, hay países confederales, federales, unitarios con sistemas de autonomía regional o
unitarios. En función del sistema establecido, se perfilarán distintas instituciones, tanto para la
administración de los Estados, Regiones, etc., por sí mismos, como para la representación del
Estado en esos niveles de administración.

También suelen dotarse los Estados democráticos de otras instituciones. Así, puede haber
un órgano, unipersonal o colegiado, independiente de los tres poderes, que se ocupe de velar por
el respeto de los derechos de los ciudadanos (defensor del pueblo). También pueden existir
organismos colegiados, con miembros designados o elegidos por distintos procedimientos, cuya
consulta sea obligada, vinculante o no, en determinados casos para la elaboración o aprobación
de una ley o la toma de alguna decisión (consejos). Igualmente, suelen existir organismos de
control de las actuaciones económicas de los poderes públicos (tribunales de cuentas), órganos
de carácter temporal para la elección de otros cargos, etc.

Todos estos principios y organismos se establecen en un texto legal general que, bajo el
principio de jerarquía de las normas, ocupa el lugar más elevado en dicha jerarquía, situándose,
pues, por encima del resto de las normas (leyes, decretos, reglamentos, órdenes…), que deben
ajustarse a los principios establecidos en dicho texto y no contradecirlo. Nos estamos refirien-
do a la constitución, carta magna, ley fundamental o como en cada Estado sea denominada.

1.3. LA CONSTITUCIÓN ESPAÑOLA


La actual Constitución Española fue aprobada en referéndum por los españoles el 6 de
diciembre de 1978, entrando en vigor, tras la sanción del Rey, el día de su publicación en el
Boletín Oficial del Estado, el 29 de diciembre de 1978. Es el fruto del proceso de transición que
se inició con el ascenso al trono de Juan Carlos I y que tuvo como pasos esenciales:

– Aprobación de la Ley para la Reforma Política (15 de diciembre de 1976)


– Elecciones a Cortes Constituyentes (15 de junio de 1977)

8
-Pág.36-
U . D . 8 . - P A RT I C I PA C I Ó N Y CONFLICTO POLÍTICO EN EL MUNDO ACTUAL

– Elaboración de la Constitución (Comisión, Ponencia, discusión en los Plenos de Con-


greso y Senado, Comisión Mixta Congreso – Senado)
– Aprobación del texto final por el Congreso y el Senado (31 de octubre de 1978)
– Referéndum (6 de diciembre de 1978)
– Publicación y entrada en vigor (29 de diciembre de 1978)

La Constitución consta de:


– Un preámbulo
– 169 artículos, organizados en un Título Preliminar y diez Títulos
– Cuatro disposiciones adicionales
– 9 disposiciones transitorias
– 1 disposición derogatoria
– 1 disposición final

1.3.1. PREÁMBULO
En él se especifican los motivos por los que se aprueba la constitución (establecer la Justi-
cia, la Libertad y la Seguridad y promover el bien común) y los objetivos que se pretende alcan-
zar con su aplicación, destacando la consolidación del Estado de Derecho, la protección de los
Derechos Humanos, la promoción de la cultura y la calidad de vida y la colaboración con los
pueblos de la Tierra.

1.3.2. TÍTULO PRELIMINAR


En él se establecen una serie de principios generales:

– Forma del Estado: Social y democrático de Derecho, bajo el principio de soberanía


nacional y con la forma de Monarquía parlamentaria.
– Unidad nacional, reconociendo la autonomía de las nacionalidades y regiones.
– Lengua oficial: El castellano, reconociendo el uso oficial de las leguas propias de las
Comunidades Autónomas.
– Bandera: Fija su forma y colores y reconoce el uso de las de las Comunidades Autónomas.
– Capital: Madrid.
– Partidos Políticos: Reconoce su papel fundamental en el sistema.
– Sindicatos: Manifiesta su importancia en la defensa de los derechos de los trabajadores.
– Fuerzas Armadas: Fija su misión.
– Garantías Jurídicas: Principios de legalidad, jerarquía de las normas y publicidad de
las mismas.

9
-Pág.37-
CIENCIAS SOCIALES. GEOGRAFÍA E HISTORIA

1.3.3. TÍTULO I
Se establecen en él los derechos y deberes fundamentales de los españoles, que podemos
esquematizar del siguiente modo:

CAPÍTULO I. DE LOS ESPAÑOLES Y LOS EXTRANJEROS


– Nacionalidad.
– Mayoría de Edad.
– Derechos de los españoles y de los extranjeros.

CAPÍTULO II. DERECHOS Y LIBERTADES


– Principio de Igualdad.

SECCIÓN 1.ª DE LOS DERECHOS FUNDAMENTALES Y LAS LIBERTADES PÚBLICAS


– Derecho a la vida.
– Libertad ideológica y religiosa.
– Libertad y seguridad individual (detención, habeas corpus).
– Derecho al honor y a la intimidad.
– Libertad de residencia y movimientos.
– Libertad de expresión e información.
– Derecho de reunión.
– Derecho de asociación.
– Derecho de participación política: elegir y ser elegido.
– Derecho a la tutela jurídica (juez, presunción de inocencia).
– Derechos de los condenados (reinserción social).
– Prohibición de los tribunales de honor.
– Derecho a la educación.
– Derecho de sindicación y huelga.
– Derecho de petición.

SECCIÓN 2.ª DE LOS DERECHOS Y DEBERES DE LOS CIUDADANOS


– Derecho y deber de la Defensa.
– Deber tributario.
– Derecho al matrimonio.
– Derecho a la propiedad privada y a la herencia.

10
-Pág.38-
U . D . 8 . - P A RT I C I PA C I Ó N Y CONFLICTO POLÍTICO EN EL MUNDO ACTUAL

– Derecho de fundación.
– Derecho y deber del trabajo.
– Derecho de establecer colegios profesionales.
– Derecho a la negociación colectiva.
– Libertad de empresa.

CAPÍTULO III. PRINCIPIOS RECTORES DE LA POLÍTICA SOCIAL Y ECONÓMICA


– Protección de la familia.
– Progreso social, formación y condiciones laborales.
– Seguridad social.
– Protección a los trabajadores españoles en el extranjero.
– Derecho a la protección de la salud.
– Promoción de la cultura, ciencia y técnica.
– Protección del medio ambiente.
– Protección del patrimonio histórico, artístico y cultural.
– Derecho a la vivienda.
– Protección a la juventud.
– Protección a los disminuidos.
– Protección de la tercera edad.
– Protección de los derechos de los consumidores.
– Regulación de las organizaciones profesionales.

CAPÍTULO IV. DE LAS GARANTÍAS DE LAS LIBERTADES Y DERECHOS FUNDAMENTALES


– Forma de regulación: Por ley.
– Protección por parte de los tribunales, recurso de amparo ante el Tribunal Constitucional
y establecimiento del Defensor del Pueblo.

CAPÍTULO V. DE LA SUSPENSIÓN DE LOS DERECHOS Y LIBERTADES


– Circunstancias para la suspensión de determinados derechos en la declaración de los
estados de excepción y sitio.
– Establecimiento de responsabilidades penales en caso de suspensión injustificada o abu-
siva.

1.3.4. TÍTULO II. DE LA CORONA


Definida la forma del Estado como Monarquía parlamentaria en el Título Preliminar, en
el Título II se tratan los aspectos constitucionales relativos a la Corona. El monarca, con el títu-
lo de Rey de España, ostenta la Jefatura del Estado cuyos rasgos básicos iniciales son:

11
-Pág.39-
CIENCIAS SOCIALES. GEOGRAFÍA E HISTORIA

– Es el símbolo de la unidad y permanencia del Estado.


– Arbitra y modera el funcionamiento de las instituciones.
– Asume la alta representación del Estado en las relaciones internacionales.
– Su persona es inviolable y no está sujeta a responsabilidad.

La Corona es hereditaria, es decir, se trasmite de padres a hijos. La Constitución cita


expresamente que dicha herencia se establece en los sucesores de Don Juan Carlos I de Borbón.
En relación con este aspecto de la sucesión destacan las siguientes precisiones:
– En el orden de sucesión se da preferencia al varón sobre la mujer, de ahí que el actual
heredero sea el hijo menor del rey y no su hermana Elena, la primogénita.
– Se fija la vía de una Ley Orgánica para solucionar los conflictos que pudieran producir-
se en la sucesión, como consecuencia de abdicaciones y renuncias.
– Se excluye de la sucesión a aquellos posibles herederos que contraigan matrimonio con-
tra la expresa prohibición del Rey y de las Cortes.
– Se designa a las Cortes como responsables de proveer la sucesión en el supuesto de que
se extinguieren todas las líneas sucesorias.
– El heredero recibe el título de Príncipe de Asturias.

Ante la posibilidad de que en el futuro la persona que herede el trono sea menor de edad o
se encuentre inhabilitada para su ostentación, se establece el sistema de Regencia y el nom-
bramiento de un Tutor del Rey, para el primer caso.

Las funciones, esencialmente de carácter formal y representativo, se definen en los artícu-


los 62 y 63, de ellas destacamos las siguientes:
– Sancionar y promulgar las leyes.
– Convocar elecciones.
– Disolver y convocar las Cortes Generales.
– Proponer el candidato a Presidente de Gobierno, nombrarlo y nombrar a sus ministros.
– El mando supremo de las Fuerzas Armadas.
– Acreditar a los embajadores.
– Manifestar el consentimiento del Estado a los Tratados internacionales suscritos.
– Declarar la guerra y concertar la paz.

En todo caso, la iniciativa de todas aquellas funciones que implican actos de decisión no le
corresponde al monarca sino a quienes la Constitución establece y sus actos deben ser refren-
dados por el Presidente del Gobierno o el del Congreso, según los casos, recayendo en ellos la
responsabilidad de los mismos.

12
-Pág.40-
U . D . 8 . - P A RT I C I PA C I Ó N Y CONFLICTO POLÍTICO EN EL MUNDO ACTUAL

1.3.5. TÍTULO III. DE LAS CORTES GENERALES


El poder legislativo es el objeto del Título III. En él se establece que el sistema parlamenta-
rio español está constituido por las Cortes Generales, formadas por dos cámaras:
– Congreso de los Diputados.
– Senado.

Sus funciones son:


– Ejercer su potestad de iniciativa legislativa.
– Elaborar y aprobar las leyes.
– Aprobar los presupuestos generales del Estado.
– Aceptar o no al candidato a Presidente del Gobierno propuesto por el Rey.
– Otorgar confianza al Presidente de Gobierno cuando éste la solicita.
– Decidir sobre la moción de censura presentada contra el Presidente del Gobierno.
– Controlar la acción del Gobierno.
– Aprobar sus propios presupuestos.
– Aprobar sus propios reglamentos de funcionamiento.
– Elegir a sus Presidentes y a los miembros de las Mesas.

Sus miembros son elegidos por el pueblo para representarle. Para ellos se establecen los cri-
terios básicos de incompatibilidad e inelegibilidad, cuya precisión se encomienda a la Ley Elec-
toral.

La composición de cada cámara y el procedimiento de elección de sus miembros queda


fijada en sus aspectos generales del siguiente modo:

Congreso de los Diputados:


– Elección por sufragio universal, libre, igual, directo y secreto.
– Número total entre 300 y 400 Diputados (en la actualidad 350).
– En cada provincia se elige un número de diputados proporcional a su población. En
Ceuta y en Melilla se elige un Diputado en cada una.
– El sistema de asignación de escaños es proporcional al número de votos (En la actuali-
dad la Ley Electoral establece el sistema proporcional d’Hondt para el recuento y asig-
nación).
– Los Diputados son elegidos para un periodo de cuatro años (Legislatura).

13
-Pág.41-
CIENCIAS SOCIALES. GEOGRAFÍA E HISTORIA

Senado:
– Elección por sufragio universal, libre, igual, directo y secreto de 208 senadores reparti-
dos del siguiente modo:

CONCEPTO TOTAL
4 por cada una de las 47 provincias no insulares 188
3 por cada una de las 3 islas mayores (Mallorca, Gran Canaria y Tenerife) 9
1 por cada una de las 7 islas menores o grupo de islas (Ibiza – Formentera,
Menorca, Fuerteventura, Gomera, Hierro, Lanzarote y La Palma) 7
2 por Ceuta y 2 por Melilla 4

— Elección por sufragio indirecto de un número variable de Senadores. Éstos son desig-
nados por las Comunidades Autónomas en función del número de sus habitantes:
– Todas designan 1 (17).
– En cada una, un senador más por cada millón de habitantes.

— Los Senadores son elegidos para cuatro años (Legislatura).

Por otra parte, Senadores y Diputados son inviolables y gozan de inmunidad, de modo que
no pueden ser procesados sin autorización de su Cámara.

Funcionamiento
– Se reúnen anualmente en dos periodos de sesiones, en sesiones públicas.
– Pueden reunirse en sesiones extraordinarias.
– Pueden reunirse Congreso y Senado en Sesión Conjunta.
– Las cámaras funcionan en Pleno o en Comisiones.
– Pueden nombrar Comisiones Legislativas Permanentes para asumir funciones de apro-
bación de las leyes entre periodos de sesiones.
– Pueden nombrar Comisiones de investigación.
– Pueden recibir peticiones individuales y colectivas.
– Cada Cámara designa una Diputación Permanente de 21 miembros.
– Pueden remitir al Gobierno peticiones.

Los acuerdos que haya que adoptar en el Pleno requieren, inicialmente, quórum, es decir,
la presencia de la mitad más uno de los miembros. A continuación, se requiere la mayoría sim-
ple de los votos emitidos, si bien existen otras mayorías establecidas en la propia Constitución
para determinados casos (absoluta, 2/3…).

14
-Pág.42-
U . D . 8 . - P A RT I C I PA C I Ó N Y CONFLICTO POLÍTICO EN EL MUNDO ACTUAL

El Título reserva el Capítulo Segundo a la elaboración de las leyes. Inicialmente, diferen-


cia los siguientes tipos de leyes:
– Leyes orgánicas.—Las que afectan al desarrollo de los derechos y libertades de los ciu-
dadanos, las de los Estatutos de Autonomía, la Ley Electoral y las que prevé expresa-
mente la Constitución. Requieren mayoría absoluta del Congreso para su aprobación.
– Leyes de Bases.—Aquellas en las que se establecen los límites, principios y criterios de
las Delegaciones Legislativas de las Cortes al Gobierno.
– Decretos Legislativos.—Son las disposiciones del Gobierno consecuencia de la Dele-
gación Legislativa.
– Decretos-Leyes.—Son disposiciones legislativas provisionales dictadas por el Gobierno
por una extraordinaria y urgente necesidad. Deben ser convalidados o derogados por el
Congreso antes de transcurridos treinta días desde su promulgación.

La iniciativa legislativa, es decir, la promoción de una norma con rango de ley, correspon-
de o puede corresponder a:
– Gobierno.—Elabora un proyecto de ley que envía al Congreso
– Congreso de los Diputados.—De sus miembros puede partir una proposición de ley,
según lo que establezca su Reglamento
– Senado.—En su caso sucede lo mismo que en el Congreso.

Además:
– Las Asambleas de las Comunidades Autónomas pueden:
- Solicitar al Gobierno que adopte un proyecto de ley
- Elevar al Congreso una proposición de ley, enviando tres representantes para defen-
derlo.
– Puede promoverse una proposición de ley por iniciativa popular, requiriéndose un
mínimo de 500.000 firmas acreditadas.
También se establecen en el Capítulo los pasos que debe dar el proyecto o proposición de
ley hasta su promulgación y los requisitos para su aprobación, que podemos resumir en los
siguientes:
– Aprobación por el Congreso con la mayoría requerida en cada caso.
– Elevación al Senado, que puede:
- Proponer enmiendas, aprobadas por mayoría simple, que deberán ser aceptadas o
rechazadas por el Congreso por mayoría simple.
- Expresar su veto, que debe ser aprobado por mayoría absoluta del Senado. El Con-
greso deberá ratificarse por mayoría absoluta o, si transcurren dos meses sin lograrla,
por mayoría simple.
– Remisión a la sanción real.
– Publicación.

15
-Pág.43-
CIENCIAS SOCIALES. GEOGRAFÍA E HISTORIA

Por otra parte, se establece que en caso de decisiones de especial transcendencia, el Rey, pre-
via propuesta del Presidente del Gobierno, autorizada por el del Congreso, podrá convocar refe-
réndum.

Por último, el Capítulo Tercero se refiere a los Tratados Internacionales, fijando qué tipo
de tratados requerirán el consentimiento de las Cortes, obligando a que éstas sean informadas
de la conclusión de todos. Si tales tratados atribuyen a instituciones u organizaciones interna-
cionales competencias derivadas de la Constitución, las Cortes deberán autorizarlo mediante
una Ley Orgánica.

Los tratados internacionales no podrán contener estipulaciones contrarias a la Constitución


ya que en ese caso su adopción implica la reforma de la Constitución. Una vez celebrados y
autorizados, en su caso, los tratados internacionales se integran en el ordenamiento jurídico.

1.3.6. TÍTULO IV. DEL GOBIERNO Y LA ADMINISTRACIÓN


El Título IV está dedicado al poder ejecutivo, estableciendo los aspectos básicos sobre su
composición, sistema de nombramiento, procedimiento de control por parte del poder legislati-
vo y funciones fundamentales de la Administración.

El Gobierno, es quien ostenta el poder ejecutivo, su composición es la siguiente:


– Presidente del Gobierno, que dirige y coordina las funciones del Gobierno.
– Uno o varios Vicepresidentes.
– Ministros.
– Otros miembros.

La composición específica del Gobierno se establece por Ley, sobre la base de estos cri-
terios esenciales. En la Historia reciente ha habido Gobiernos con un solo Vicepresidente, con
dos y hasta con tres y los Ministros han variado de número y en el reparto de sus atribucio-
nes; así, por ejemplo, ha habido un Ministro de Economía y Hacienda, en unos Gobiernos, y
uno de Economía y otro de Hacienda, en otros; lo mismo ha sucedido con áreas como Edu-
cación, Cultura, Deportes, Ciencia y Tecnología, que se han integrado, en repartos diferentes,
en distintos Ministerios. Los Ministerios más tradicionales y presentes en todos los Gobier-
nos han sido y son:
– Asuntos Exteriores.
– Interior.
– Defensa.
– Obras Públicas o Fomento.
– Economía.
– Hacienda.
– Educación.

16
-Pág.44-
U . D . 8 . - P A RT I C I PA C I Ó N Y CONFLICTO POLÍTICO EN EL MUNDO ACTUAL

Las funciones esenciales del Gobierno son:


– Política interior.
– Política exterior.
– Administración civil.
– Administración militar.
– Potestad reglamentaria.

Los miembros del Gobierno son nombrados por el Rey a propuesta de su Presidente; pero
éste, que también es nombrado por el Rey, debe someterse a un proceso de investidura que se
puede resumir en los siguientes pasos:
1. Tienen lugar elecciones de renovación del Congreso de los Diputados.
2. El Rey realiza consultas con los representantes de los grupos políticos que hayan obte-
nido representación, decidiendo entre ellos un candidato a Presidente del Gobierno.
Aunque la Constitución no orienta sobre el criterio para la designación, suele recaer en
el candidato del partido con mayoría de Diputados, aunque pudiera ser un candidato
con minoría pero con otros apoyos contrastados en las consultas citadas.
3. El candidato propuesto por el Rey expone su programa de Gobierno ante el Congre-
so de los Diputados y solicita la confianza de la cámara.
4. La confianza se entiende otorgada si obtiene mayoría absoluta de votos, es decir, si
votan a favor la mitad más uno de los Diputados (en la actualidad, con 350 Diputados,
son necesarios 176 votos favorables).
5. Si no se alcanza dicha mayoría se repite la votación 48 horas después. Ahora, la con-
fianza se entiende otorgada si se consigue la mayoría simple, es decir, más votos a
favor que en contra.
6. Si no se consigue tampoco dicha mayoría, se repite el proceso desde el punto 2.
7. Si después de dos meses desde la primera votación ningún candidato obtiene la con-
fianza, el Rey disuelve las Cortes y convoca elecciones, iniciándose el proceso desde
el punto 1.

Si el candidato obtiene la confianza, es investido como Presidente del Gobierno y, a conti-


nuación, propone al Rey a sus Ministros y éstos son nombrados.
El cese de Gobierno puede producirse por los siguientes supuestos:
– Celebración de elecciones generales.
– Tras la pérdida de confianza del Congreso.
– Por dimisión de su Presidente.
– Por fallecimiento de su Presidente.

17
-Pág.45-
CIENCIAS SOCIALES. GEOGRAFÍA E HISTORIA

La responsabilidad criminal de los miembros del Gobierno sólo puede ser exigida por el
Tribunal Supremo y, si la acusación fuere por traición o delito contra la seguridad del Estado,
sólo puede ser planteada por iniciativa de 1/4 de los Diputados

El Título establece también los aspectos generales relativos a la Administración Pública,


fijando como principios que deben inspirar su actuación los siguientes:
– Eficacia.
– Jerarquía.
– Descentralización.
– Desconcentración.
– Coordinación.

Finalmente, respecto al Gobierno, la Constitución, en este Título, fija la existencia de dos


órganos o colectivos que tienen relación con él:
– Fuerzas y Cuerpos de Seguridad del Estado, que dependen del Gobierno y tienen
como misión proteger el libre ejercicio de los derechos y libertades y garantizar la segu-
ridad ciudadana.
– Consejo de Estado, órgano consultivo del Gobierno.

1.3.7. TÍTULO V. DE LAS RELACIONES ENTRE EL GOBIERNO


Y LAS CORTES GENERALES
En este Título se fijan los parámetros de relación entre el Gobierno y las Cortes Generales,
que tienen que ver, fundamentalmente, con el control del Legislativo sobre el Ejecutivo. Estas
relaciones podemos esquematizarlas del siguiente modo:

DE LAS CORTES HACIA EL GOBIERNO:


— Solicitar información y ayuda.
— Reclamar su presencia en plenos y Comisiones.
— Interpelaciones y preguntas.
— Moción de censura, que debe cumplir los siguientes requisitos para prosperar:
– Ser propuesta por al menos 1/10 de los Diputados.
– Incluir un candidato alternativo a Presidente del Gobierno.
– Ser votada cinco días después de su presentación.
– La censura se entiende adoptada si obtiene mayoría absoluta.
– Si no obtiene dicha mayoría no se podrá plantear otra durante ese periodo de sesio-
nes.

18
-Pág.46-
U . D . 8 . - P A RT I C I PA C I Ó N Y CONFLICTO POLÍTICO EN EL MUNDO ACTUAL

DEL GOBIERNO HACIA LAS CORTES:


– Sus miembros tienen acceso a las sesiones de los Plenos y las Comisiones
– Pueden hacerse oír en los Plenos y las Comisiones.
– El Presidente del Gobierno puede presentar al Congreso la cuestión de confianza, que
se entenderá otorgada si es apoyada por la mayoría simple de los Diputados.
– El Presidente del Gobierno puede proponer al Rey la disolución de las Cámaras, salvo
si está en trámite una moción de censura.

Además:
– El Gobierno debe dar cuenta al Congreso de los Diputados de la declaración del estado
de alarma, antes de transcurridos 15 días.
– El Gobierno debe obtener la autorización del Congreso de los Diputados para declarar el
estado de excepción.
– El Congreso de los Diputados es quien debe declarar, a propuesta del Gobierno y por
mayoría absoluta, el estado de sitio.

1.3.8. TÍTULO VI. DEL PODER JUDICIAL


El Título VI está dedicado al tercero de los poderes del Estado, el judicial, estableciendo las
instituciones y procedimientos básicos para la impartición de la Justicia. Según el texto, la Jus-
ticia emana del pueblo y se administra, en nombre del Rey, por Jueces y Magistrados. El pro-
pio Título exige la aprobación de una ley orgánica (Publicada como Ley Orgánica 6/1985, de
1 de julio, del Poder Judicial).

Los órganos judiciales a los que se refiere el Título son:

— Juzgados y Tribunales.—Son los encargados de juzgar y de hacer ejecutar lo juzgado.

— Tribunal Supremo.—Órgano jurisdiccional superior, su Presidente es nombrado por el


Rey a propuesta del Consejo General del Poder Judicial

— Consejo General del Poder Judicial.—Órgano de gobierno del poder judicial cuya
composición (renovada cada cinco años) es:
– 1 Presidente: El del Tribunal Supremo.
– 12 Jueces y Magistrados, elegidos según la ley orgánica (6 a propuesta del Congre-
so y 6 a propuesta del Senado).
– 8 abogados y juristas de reconocida competencia y más de 15 años de ejercicio pro-
fesional, elegidos por mayoría de 3/5, 4 de ellos por el Congreso y los otros 4 por
el Senado.

19
-Pág.47-
CIENCIAS SOCIALES. GEOGRAFÍA E HISTORIA

— Ministerio Fiscal, encargado de promover la acción de la Justicia.


— Fiscal General del Estado.
— Policía judicial.

1.3.9. TÍTULO VII. ECONOMÍA Y HACIENDA


El tratamiento de los aspectos relacionados con la Economía y la Hacienda públicas es el
tema del Título VII, en el que, entre otros aspectos, destacan los siguientes:
– Riqueza subordinada al interés general.
– Reconocimiento de la iniciativa pública en la actividad económica.
– Seguridad Social.
– Mandato para promover la participación en la empresa, las sociedades cooperativas y el
acceso de los trabajadores a la propiedad de los medios de producción.
– Posibilidad de que el Estado planifique la actividad económica general.
– Régimen de los bienes de dominio público.
– Sistema tributario.
– Presupuestos Generales del Estado.
– Requisitos para la emisión de deuda pública.

Especial importancia tiene el tema de los Presupuestos Generales del Estado, elaborados
por el Gobierno y elevados a la aprobación del Congreso de los Diputados tres meses antes de
expirar los del año anterior.
También es importante el establecimiento del Tribunal de Cuentas, supremo órgano fisca-
lizador de las cuentas y de la gestión económica del Estado.

1.3.10. TÍTULO VIII. DE LA ORGANIZACIÓN TERRITORIAL DEL ESTADO


Por medio de tres Capítulos, la Constitución establece los puntos esenciales sobre los que se
fundamenta la organización territorial del Estado.

En el Capítulo Primero se contemplan los principios generales:


– El Estado se organiza en municipios, provincias y Comunidades Autónomas.
– Todas las entidades territoriales gozan de autonomía.
– El Estado debe hacer efectivo el principio de solidaridad entre todas las entidades evi-
tando privilegios económicos o sociales.
– Todos los españoles tienen los mismos derechos en cualquier parte del Estado.
– Se debe preservar el derecho a la libre circulación de personas y mercancías.

20
-Pág.48-
U . D . 8 . - P A RT I C I PA C I Ó N Y CONFLICTO POLÍTICO EN EL MUNDO ACTUAL

El Capítulo Segundo se dedica a la Administración Local, estableciendo las siguientes


entidades:
– Municipios.—Gobernados y administrados por los Ayuntamientos, integrados por
Alcaldes y Concejales.
– Provincias.—Agrupación de municipios cuyo gobierno corresponde a las Diputaciones.
– Agrupaciones de municipios.—Se establece la posibilidad de su creación (Mancomu-
nidades, Comarcas…).
– Cabildos o Consejos insulares.—Órganos de gobierno a nivel insular.

El Capítulo Tercero trata de las Comunidades Autónomas. La mayor parte de los precep-
tos establecidos tienen relación con el proceso para el establecimiento de las diferentes Comu-
nidades, fijando los requisitos y trámites, y con los principios básicos que deben contener los
Estatutos de Autonomía, además de cuestiones como las competencias y principios sobre finan-
ciación, equilibrio y compensación entre Comunidades. Los aspectos fundamentales son:

a) Constitución de las Comunidades Autónomas


Se establecen dos cauces.
El primero y más genérico, fijado en el Artículo 143, determina que la iniciativa, que,
si no se cumplen los requisitos en la primera tentativa, no podrá repetirse hasta cinco
años después, corresponde a:
– Todas las Diputaciones afectadas.
– Dos tercios de los Municipios cuyo total de habitantes representen la mayoría del
censo electoral de cada provincia o isla.
La segunda vía aparece en el Artículo 151, que elimina el plazo de los cinco años a que
se refiere el 143 si se cumplen los siguientes requisitos para la iniciativa:
– Acuerdo de todas las Diputaciones.
– Acuerdo de las tres cuartas partes de los Municipios de cada una de las provincias.
– Ratificación de la iniciativa mediante referéndum por el voto de la mayoría absoluta
de los electores de cada provincia.
La mayor parte de las Comunidades Autónomas se constituyeron al amparo del Artículo
143, a excepción de Galicia, País Vasco, Cataluña y Andalucía, que lo hicieron por el
Artículo 151.
b) Se establece, además, que, independientemente de la vía, se debe elaborar un Estatuto
de Autonomía como ley básica de la Comunidad. Tal elaboración corresponde a una
asamblea integrada por los miembros de las Diputaciones provinciales y los Diputados
y Senadores elegidos por las circunscripciones afectadas. Una vez elaborados, los Esta-
tutos deben seguir trámites distintos según la vía de acceso a la autonomía:

21
-Pág.49-
CIENCIAS SOCIALES. GEOGRAFÍA E HISTORIA

Vía del artículo 143: Se tramita como una Ley en las Cortes.
Vía del artículo 151: Tendrá que seguir los trámites que se citan a continuación:
– Remisión a la Comisión Constitucional del Congreso para su análisis, con la partici-
pación de una delegación de la asamblea.
– Someterlo a referéndum de la población de la Comunidad, debiendo ser aceptado por
mayoría en cada una de las provincias afectadas.
– Elevación a las Cortes. Los Plenos de ambas Cámaras darán o no su voto de ratificación.
– Sanción real y publicación.
Los Estatutos deben incluir los siguientes aspectos:
– Denominación de la Comunidad.
– Delimitación de su territorio.
– Denominación, organización y sede de las instituciones autónomas.
– Las competencias asumidas.
Además, para el caso de las Comunidades que accedan al autogobierno por el Art.º 151,
la Constitución establece que el Estatuto fijará:
– Una Asamblea Legislativa.
– Un Consejo de Gobierno.
– Un Presidente elegido por la Asamblea.
– Un Tribunal Superior de Justicia.
c) También se establecen los resortes para el control de la actividad de las Comunidades
Autónomas:
– Tribunal Constitucional.
– Gobierno.
– Jurisdicción contencioso – administrativa.
– Tribunal Supremo.
d) Además, se determina:
– El nombramiento de un Delegado del Gobierno en cada Comunidad.
– La autonomía financiera y los cauces para la financiación de la Comunidad.
– La creación de un Fondo de Compensación interterritorial.

e) Pero uno de los aspectos fundamentales es el de las competencias de las Comunidades


Autónomas. Los Artículos 148 y 149 se dedican a este aspecto. El 148 detalla las com-
petencias que pueden asumir y el 149 aquellas que son exclusivas del Estado, si bien,
en el Artículo 150, se ofrece la posibilidad de que alguno de los aspectos de estas últi-
mas pueda ser transferido a las Comunidades mediante Ley orgánica.

22
-Pág.50-
U . D . 8 . - P A RT I C I PA C I Ó N Y CONFLICTO POLÍTICO EN EL MUNDO ACTUAL

1.3.11. TÍTULO IX. DEL TRIBUNAL CONSTITUCIONAL


En este Título se fija la composición, jurisdicción, actuación y competencia del Tribunal
Constitucional.

Composición: 12 miembros elegidos entre Magistrados y Fiscales, Profesores de Universi-


dad, funcionarios públicos y Abogados, juristas de reconocida competencia y con más de 15
años de ejercicio profesional.

Jurisdicción: Sobre todo el territorio.

Competencia:
— Recurso de inconstitucionalidad contra leyes. Presentado por:
– Presidente del Gobierno.
– Defensor del Pueblo.
– 50 Diputados.
– 50 Senadores.
– Órganos Ejecutivos de las Comunidades Autónomas.
– Asambleas de las Comunidades Autónomas.

— Recurso de amparo por violación de derechos y libertades.


– Personas naturales y jurídicas.
– Defensor del Pueblo.
– Ministerio fiscal.

— Conflictos de competencia entre el Estado y las Comunidades Autónomas.


– Otras materias atribuidas por la Constitución o las leyes.

1.3.12. TÍTULO X. DE LA REFORMA CONSTITUCIONAL


La Constitución, norma de mayor rango en el ordenamiento jurídico, es la única norma que
puede regular el procedimiento para su propia reforma.

La iniciativa de reforma partirá de las mismas instituciones consideradas para la iniciativa


legislativa. Se distinguen dos posibles casos:

Reforma parcial.—El proyecto de reforma deberá ser aprobado:


– Por 3/5 de los Diputados y 3/5 de los Senadores.
– Caso de no haber acuerdo deberá resolverse por una Comisión Mixta.
– Caso de no haber acuerdo tampoco, se votará nuevamente en cada cámara y será apro-
bado si obtiene mayoría absoluta del Senado y 2/3 del Congreso.

23
-Pág.51-
CIENCIAS SOCIALES. GEOGRAFÍA E HISTORIA

– Aprobada la Reforma por las Cortes será sometida a referéndum si así lo solicita 1/10 de
los Diputados o de los Senadores.

Reforma total o que afecte a los Derechos Fundamentales y libertades públicas:


– Aprobación por mayoría de 2/3 de cada Cámara.
– Disolución de las Cortes.
– Elecciones generales.
– Ratificación de la decisión de reforma por las nuevas Cámaras.
– Estudio de un nuevo texto constitucional.
– Aprobación por 2/3 de ambas Cámaras del nuevo texto.
– Referéndum de ratificación.

2. LA ORGANIZACIÓN TERRITORIAL DE ESPAÑA.


(LA NACIÓN ESPAÑOLA, MUNICIPIOS, PROVINCIAS
Y COMUNIDADES AUTÓNOMAS. ÓRGANOS Y COMPETENCIAS)
Establecidos por la Constitución, en su Título VIII, los principios fundamentales que rigen
la organización territorial del Estado y los cauces de acceso al autogobierno de las Comunida-
des Autónomas, una vez completado el proceso de constitución de éstas, la estructura territorial
del Estado, sus Comunidades Autónomas, Provincias y más importantes municipios han que-
dado establecidos bajo el siguiente esquema:

17 Comunidades Autónomas.
2 Ciudades Autónomas (Ceuta y Melilla).
50 Provincias englobadas en las 17 Comunidades:
7 Comunidades de una sola Provincia.
10 Comunidades con dos o más Provincias.
Más de 8.000 Municipios.
Alrededor de 600 Mancomunidades.

A continuación se citan las Comunidades Autónomas, con los datos principales de cada una
de ellas y sus provincias:

(Los datos de población se refieren al censo oficial de 1991).

24
-Pág.52-
U . D . 8 . - P A RT I C I PA C I Ó N Y CONFLICTO POLÍTICO EN EL MUNDO ACTUAL

GALICIA
Nombre Oficial: Comunidad Autónoma de Galicia
Estatuto: L.O. 1/81 de 6 de abril de 1981
Población: 2.731.669
Extensión: 29.434 Km2
Densidad: 93 h/Km2
Límites: N: Cantábrico
E: Asturias, Castilla y León (León y Zamora)
S: PORTUGAL
O: Atlántico
Provincias: La Coruña/A Coruña, Lugo, Orense/Ourense y Pontevedra.
Capital: Santiago de Compostela

ASTURIAS
Nombre Oficial: Principado de Asturias
Estatuto: L.O. 7/81 de 30 de diciembre de 1981
Población: 1.093.937
Extensión: 10.565 Km2
Densidad: 104 h/Km2
Límites: N: Cantábrico
E: Cantabria
S: Castilla y León (León)
O: Galicia (Lugo)
Provincias: Asturias
Capital: Oviedo

CANTABRIA
Nombre Oficial: Diputación Regional de Cantabria
Estatuto: L.O. 8/81 de 30 de diciembre de 1981
Población: 527.326
Extensión: 5.289 Km2
Densidad: 100 h/Km2
Límites: N: Cantábrico
E: País Vasco (Vizcaya)
S: Castilla y León (León, Palencia, Burgos)
O: Asturias
Provincias: Cantabria
Capital: Santander

25
-Pág.53-
CIENCIAS SOCIALES. GEOGRAFÍA E HISTORIA

PAÍS VASCO
Nombre Oficial: País Vasco/Euskadi
Estatuto: L.O. 3/79 de 18 de diciembre de 1979
Población: 2.104.041
Extensión: 7.261 Km2
Densidad: 289 h/Km2
Límites: N: Cantábrico
E: FRANCIA, Navarra
S: Castilla y León (Burgos), La Rioja
O: Cantabria
(Casos especiales del Condado de Treviño, provincia de Burgos, den-
tro de Álava; y Villaverde de Trucios, provincia de Cantabria, dentro
de Vizcaya)
Provincias: Vizcaya/Bizkaia, Guipúzcoa/Gipuzkoa, Álava
Capital: Vitoria/Gazteiz

NAVARRA
Nombre Oficial: Comunidad Foral de Navarra
Estatuto: L.O. 13/82 de 10 de agosto de 1982
Población: 519.277
Extensión: 10.421 Km2
Densidad: 50 h/Km2
Límites: N: FRANCIA
E: Aragón (Huesca, Zaragoza)
S: La Rioja
O: País Vasco (Guipúzcoa, Álava)
Provincias: Navarra
Capital: Pamplona

LA RIOJA
Nombre Oficial: Comunidad Autónoma de La Rioja
Estatuto: L.O. 3/82 de 9 de junio de 1982
Población: 263.434
Extensión: 5.034 Km2
Densidad: 52 h/Km2
Límites: N: País Vasco (Álava), Navarra.
E: Navarra, Aragón (Zaragoza)
S: Castilla y León (Soria)
O: Castilla y León (Burgos)
Provincias: La Rioja
Capital: Logroño

26
-Pág.54-
U . D . 8 . - P A RT I C I PA C I Ó N Y CONFLICTO POLÍTICO EN EL MUNDO ACTUAL

ARAGÓN
Nombre Oficial: Comunidad Autónoma de Aragón
Estatuto: L.O. 8/82 de 10 de agosto de 1982
Población: 1.188.817
Extensión: 47.650 Km2
Densidad: 25 h/Km2
Límites: N: FRANCIA
E: Cataluña (Lleida, Tarragona), Valencia (Castellón, Valencia)
S y O: Castilla-La Mancha (Cuenca, Guadalajara), Castilla y León
(Soria), La Rioja, Navarra.
Provincias: Zaragoza, Huesca y Teruel
Capital: Zaragoza

CATALUÑA
Nombre Oficial: Cataluña (Generalitat de Catalunya)
Estatuto: L.O. 4/79 de 18 de diciembre de 1979
Población: 6.059.494
Extensión: 31.930 Km2
Densidad: 190 h/Km2
Límites: N: FRANCIA
E: Mediterráneo
S: C. Valenciana (Castellón)
O: Aragón
Provincias: Lleida/Lérida, Girona/Gerona, Barcelona, Tarragona
Capital: Barcelona

CASTILLA Y LEÓN
Nombre Oficial: Castilla y León (Junta de Castilla y León)
Estatuto: L.O. 4/83 de 25 de febrero de 1983
Población: 2.545.926
Extensión: 94.193 Km2
Densidad: 27 h/Km2
Límites: N: Asturias, Cantabria, P. Vasco, La Rioja
E: Aragón (Zaragoza)
S: Castilla-La Mancha (Guadalajara, Toledo), Extremadura (Cáce-
res), Madrid
O: PORTUGAL, Galicia (Lugo, Orense)
Provincias: León, Zamora, Salamanca, Palencia, Valladolid, Ávila, Burgos, Sego-
via, Soria.
Capital: Valladolid

27
-Pág.55-
CIENCIAS SOCIALES. GEOGRAFÍA E HISTORIA

CASTILLA-LA MANCHA
Nombre Oficial: Castilla-La Mancha (Junta de Comunidades de Castilla-La Mancha)
Estatuto: L.O. 9/82 de 10 de agosto 1982
Población: 1.658.446
Extensión: 79.230 Km2
Densidad: 21 h/Km2
Límites: N: Castilla y León (Soria, Segovia, Ávila), Madrid
E: Aragón (Zaragoza y Teruel), Valencia (Valencia y Alicante)
S: Andalucía (Granada, Jaén, Córdoba), Murcia
O: Extremadura (Cáceres y Badajoz)
Provincias: Guadalajara, Toledo, Cuenca, Ciudad Real, Albacete
Capital: Toledo

MADRID
Nombre Oficial: Comunidad de Madrid
Estatuto: L.O. 3/83 de 25 de febrero de 1983
Población: 5.030.958
Extensión: 7.995 Km2
Densidad: 618 h/Km2
Límites: N y O: Castilla y León (Segovia, Ávila)
E: Castilla-La Mancha (Guadalajara)
S: Castilla-La Mancha (Toledo, Cuenca)
Provincias: Madrid
Capital: Madrid

COMUNIDAD VALENCIANA
Nombre Oficial: Comunidad Valenciana (Generalidad)
Estatuto: L.O. 5/82 de 1 de julio de 1982
Población: 3.857.234
Extensión: 23.305 Km2
Densidad: 166 h/Km2
Límites: N: Cataluña (Tarragona)
E: Mediterráneo
S: Murcia
O: Aragón (Teruel), Castilla-La Mancha (Cuenca, Albacete),
(Caso especial del Rincón de Ademuz, entre Teruel y Cuenca)
Provincias: Castellón, Valencia y Alicante.
Capital: Valencia

28
-Pág.56-
U . D . 8 . - P A RT I C I PA C I Ó N Y CONFLICTO POLÍTICO EN EL MUNDO ACTUAL

BALEARES
Nombre Oficial: Comunidad Autónoma de las Islas Baleares (Illes Balears)
Estatuto: L.O. 2/83 de 25 de febrero de 1983
Población: 709.138
Extensión: 5.014 Km2
Densidad: 141 h/Km2
Límites: Mar Mediterráneo
Provincias: Baleares
Capital: Palma de Mallorca

EXTREMADURA
Nombre Oficial: Extremadura (Junta de Extremadura)
Estatuto: L.O. 1/83 de 25 de febrero de 1983
Población: 1.061.852
Extensión: 41.602 Km2
Densidad: 26 h/Km2
Límites: N: Castilla y León (Salamanca y Ávila)
E: Castilla-La Mancha (Toledo, C. Real)
S: Andalucía (Córdoba, Sevilla, Huelva)
O: PORTUGAL
Provincias: Cáceres y Badajoz
Capital: Mérida

ANDALUCÍA
Nombre Oficial: Andalucía (Junta de Andalucía)
Estatuto: L.O. 6/81 de 30 de diciembre de 1981
Población: 6.940.522
Extensión: 87.268 Km2
Densidad: 80 h/Km2
Límites: N: Extremadura (Badajoz), Castilla-La Mancha (C. Real, Albacete)
E: Murcia, Mediterráneo
S: Mediterráneo, GIBRALTAR, Atlántico
O: PORTUGAL
Provincias: Huelva, Sevilla, Cádiz, Córdoba, Málaga, Jaén, Granada, Almería
Capital: Sevilla

29
-Pág.57-
CIENCIAS SOCIALES. GEOGRAFÍA E HISTORIA

MURCIA
Nombre Oficial: Región de Murcia (Consejo de Gobierno)
Estatuto: L.O. 4/82 de 9 de junio de 1982
Población: 1.045.601
Extensión: 11.317 Km2
Densidad: 92 h/Km2
Límites: N: Castilla-La Mancha (Albacete)
E: Valencia (Alicante)
S: Mediterráneo
O: Andalucía (Granada, Almería)
Provincias: Murcia
Capital: Murcia

CANARIAS
Nombre Oficial: Canarias
Estatuto: L.O. 10/82 de 10 de agosto de 1982
Población: 1.493.784
Extensión: 7.242 Km2
Densidad: 205 h/Km2
Límites: Atlántico
Provincias: Las Palmas de Gran Canaria y Santa Cruz de Tenerife
Capital: Santa Cruz de Tenerife

CEUTA Y MELILLA
Ambas son ciudades autónomas de soberanía española en el norte de África.

CEUTA
Población: 73.208
Extensión: 19 Km2
Densidad: 3.853 h/Km2
Límites: N, E, S: Mediterráneo
O: MARRUECOS

MELILLA
Población: 63.670
Extensión: 13,4 Km2
Densidad: 4.751 h/Km2
Límites: N, E, S: Mediterráneo
O: MARRUECOS

30
-Pág.58-
U . D . 8 . - P A RT I C I PA C I Ó N Y CONFLICTO POLÍTICO EN EL MUNDO ACTUAL

ESPAÑA EN EL MUNDO: LA COMUNIDAD EUROPEA


3. E IBEROAMÉRICA
El preámbulo de la Constitución española establece como una de las voluntades de España
la de colaborar en el fortalecimiento de unas relaciones pacíficas y de eficaz cooperación entre
todos los pueblos de la Tierra. Como manifestación de esta voluntad, el Estado promueve unas
relaciones de cooperación con todos los países; pero esencialmente con aquellos que tienen una
especial vinculación con España. En este sentido, la política exterior española se ha caracteri-
zado tradicionalmente por sus buenas relaciones, principalmente, con los siguientes ámbitos
geopolíticos:

3.1. EUROPA
La relación española con Europa parte del hecho de su situación geográfica, en primer tér-
mino, y de su vinculación cultural, por otra parte. Efectivamente, España guarda con el resto de
Europa una relación vital que parte de su historia común, tejida a lo largo de siglos con el hilo
de la sangre de mil batallas y decenas de guerras.

Tras la II Guerra Mundial, como veremos en el apartado siguiente, se inició el proceso de


integración europea que ha llevado a la existencia, en la actualidad, de la Unión Europea. Espa-
ña, estuvo interesada desde el principio en integrarse en el proyecto común europeo. Sin embar-
go, cuando se dieron los primeros pasos, el régimen político español no era admitido en el
ámbito europeo de modo que, a pesar de los intentos realizados en ese sentido, hubo que espe-
rar hasta el establecimiento de la Monarquía Parlamentaria y su consolidación con la aproba-
ción de la Constitución, para que la candidatura española fuera admitida por los, entonces, 9
miembros de la, también entonces, Comunidad Económica Europea o Mercado Común. Des-
pués de unas largas negociaciones, la integración, mediante el correspondiente Tratado interna-
cional, quedó ultimada en 1985, entrando en vigor el 1 de enero de 1986, fecha a partir de la
cual España es miembro de pleno derecho de lo que después ha recibido el nombre de Comu-
nidad Europea y Unión Europea.

Los sucesivos tratados y directivas adoptadas por la Unión Europea han ido integrándo-
se en el ordenamiento jurídico español, llegando a motivar, incluso, el único procedimiento
de reforma constitucional puesto en marcha hasta la fecha y que afectó al derecho de sufra-
gio pasivo, al reconocerse el derecho de los ciudadanos europeos residentes en España a pre-
sentarse a las elecciones municipales como candidatos, además de poder votar en las
mismas.

31
-Pág.59-
CIENCIAS SOCIALES. GEOGRAFÍA E HISTORIA

La integración española en Europa ha llevado, además, a la implicación en el proceso de


unión económica y monetaria que ha conducido a la adopción del euro como moneda común,
con la consiguiente desaparición de la peseta. También ha producido la desaparición legal de las
fronteras interiores con los países de la Unión que han adoptado el Acuerdo de Schengen y a la
libre circulación de personas y mercancías entre los mismos.

3.2. IBEROAMÉRICA
La vinculación cultural con Iberoamérica se fundamenta en 300 años de presencia y coloni-
zación española en América. La lengua y la cultura española están presentes en estos países que
llaman a España Madre Patria.

Las buenas relaciones que se mantienen con estos países se materializan en la periódica reu-
nión de Jefes de Estado y de Gobierno en la Cumbre Iberoamericana donde se aproximan pos-
turas y se acuerdan proyectos de colaboración en muy diversos aspectos.

Por otra parte, esas buenas relaciones hacen que España sea el interlocutor más adecuado
para el establecimiento de acuerdos y contactos entre Europa y dichos países.

3.3. OTROS ÁMBITOS


Además de Europa e Iberoamérica, España mantiene tradicionalmente unas buenas relacio-
nes de vecindad y cooperación con el llamado Mundo Árabe, como consecuencia de la vincu-
lación cultural motivada por ocho siglos de presencia musulmana en España y de unas
influencias obvias en muy diversos aspectos, comenzando por el influjo en la lengua.

Últimamente estas relaciones han podido sufrir cierto distanciamiento como consecuencia
de dos acontecimientos de distinta naturaleza: el primero, de interés mundial, está relacionado
con la participación de España en la guerra de Iraq; el segundo, más concreto, está relacionado
con el conflicto puntual que protagonizaron España y Marruecos relativo a los movimientos de
despliegue de la Isla Perejil.

En cualquier caso, las relaciones entre España y Marruecos siguen estando marcadas por la
mutua necesidad de entendimiento.

32
-Pág.60-
U . D . 8 . - P A RT I C I PA C I Ó N Y CONFLICTO POLÍTICO EN EL MUNDO ACTUAL

4. TRANSFORMACIONES Y TENSIONES
EN LAS RELACIONES INTERNACIONALES.
(ORGANISMOS INTERNACIONALES. LA CRISIS DEL SISTEMA
DE BLOQUES Y LA OTAN. EL PROCESO DE UNIDAD EUROPEA.
UNA CRISIS O CONFLICTO SOCIAL RELEVANTE. PROBLEMAS
Y PERSPECTIVAS PARA LA PAZ)

4.1. ORGANISMOS SUPRANACIONALES

4.1.1. LA O.N.U. Y SU ENTORNO

4.1.1.1. Origen

La O.N.U. tuvo como precedente inmediato y fracasado la Sociedad de Naciones, formada


en el periodo de entreguerras y que se mostró inoperante ante los conflictos internacionales,
tales como el expansionismo alemán en Europa. El primer paso para la constitución de la
O.N.U. fue la Declaración de las Naciones Unidas, firmada por 20 países que se hallaban en
guerra contra las potencias del eje (1 de enero de 1942). Posteriormente, la reunión de China,
U.R.S.S., Reino Unido y EE.UU. en Dumbarton Oaks (EE.UU.) estableció las bases del
organismo (agosto-octubre de 1.944).

4.1.1.2. Fundación
La Organización de las Naciones Unidas fue fundada en la Conferencia de San Francisco,
entre el 25 de abril y el 26 de junio de 1945, con la participación de los 51 países fundadores,
que firmaron la Carta de las Naciones Unidas el 26 de junio de 1945. Dicha Carta es un trata-
do multilateral que establece una serie de derechos y deberes de los Estados firmantes y es el
documento constitucional básico de la Organización.

4.1.1.3. Denominación. Sede. Idiomas oficiales


Denominación: Organización de la Naciones Unidas (United Nations).
Sede: Nueva York.
Idiomas: Chino, español, inglés, francés y ruso.

33
-Pág.61-
CIENCIAS SOCIALES. GEOGRAFÍA E HISTORIA

4.1.1.4. Estados miembros


Todos (191) salvo Taiwán y Vaticano.

4.1.1.5. Financiación
Se sostiene con los fondos aportados por los Estados miembros. Los principales contri-
buyentes, según porcentaje, son: Estados Unidos, Japón, Alemania, Rusia, Francia, Reino
Unido, Italia, Canadá, España, Ucrania.

4.1.1.6. Organización
A) La Secretaría General.—El Secretario General es la figura clave de la ONU: es su
jefe ejecutivo y su administrador principal. Su elección para 5 años prorrogables es realizada
por la Asamblea General por mayoría simple entre los candidatos presentados por el Consejo
de Seguridad. Es el depositario del Tratado de la Organización que define sus funciones eje-
cutivas y administrativas:
Funciones ejecutivas.—Puede actuar como intermediario y consejero informal de los
Gobiernos proponiendo la adopción de medidas para garantizar la paz y la seguridad interna-
cionales. Debe presentar anualmente un informe ante la Asamblea.
Funciones administrativas.—Debe garantizar el buen funcionamiento de los diferentes
órganos, comités, reuniones, etc., y la coordinación de sus actividades. También es responsa-
ble de la elaboración de los presupuestos para su posterior aprobación por la Asamblea.
Los Secretarios Generales hasta la fecha han sido:
TRYGVE H. LYE (noruego) (1946-1953)
DAG H. HAMMARSKJÖLD (sueco) (1953-1961)
U THANT (birmano) (1961-1972)
KURT WALDHEIM (austriaco) (1972-1982)
JAVIER PÉREZ DE CUÉLLAR (peruano) (1982-1992)
BOUTROS BOUTROS GHALI (egipcio) (1992-1997)
KOFI ANNAN (Ghana) (1997- )
B) Consejo de Seguridad.—Es el órgano ejecutivo. Está integrado por 15 miembros de
los cuales 5 son miembros permanentes (EE.UU., Rusia, China, Francia y Reino Unido) que
tienen derecho de veto por considerarlos la Carta como grandes potencias.. Los 10 restantes
miembros son elegidos por la Asamblea General para dos años.

El sistema de votación:
– Cada país un voto.
– Decisiones de procedimiento: mayoría necesaria 9 votos.
– Decisiones en otras materias: también 9 pero siempre que incluyan a los cinco
permanentes.

34
-Pág.62-
U . D . 8 . - P A RT I C I PA C I Ó N Y CONFLICTO POLÍTICO EN EL MUNDO ACTUAL

Funciones: “el mantenimiento de la paz y la seguridad internacionales”.


Actuación: A través de Resoluciones que son de obligado cumplimiento para los países
miembros, orienta la solución de conflictos o establece medidas ejecutivas.

Durante la Guerra Fría se mostró bastante inoperante por el freno que suponía el ejercicio
del veto por parte de la URSS. En la actualidad sus decisiones van cobrando mayor peso.

C) Asamblea General.—Órgano plenario y deliberante de las Naciones Unidas.

Para ser miembro, según la Carta, el Estado debe ser “amante de la paz”, aceptar la Carta
fundacional, “poder y desear” cumplir las obligaciones inherentes a ella, y ser aceptado por los
países miembros (España, 1955), tras ser recomendado por el Consejo de Seguridad.

Cada país tiene un voto. Las cuestiones importantes deben ser aprobadas por 2/3 de los pre-
sentes y votantes y las demás por mayoría simple. En la Asamblea se puede deliberar y adoptar
recomendaciones pero no puede legislar de forma vinculante salvo en cuestiones internas de
la organización como la adopción del presupuesto anual.

Funciona en plenos por sesiones anuales regulares, de tres meses de duración y puede ser
convocada de forma extraordinaria por el Secretario General a petición del Consejo de Seguri-
dad o de la mayoría de los Estados miembros de la Asamblea. Cada país tiene 5 delegados.

D) Consejo Económico y Social (ECOSOC).—Depende de la Asamblea General y tiene


por misión la de dirigir recomendaciones a los Estados miembros en materias relativas a su
competencia. Lo integran 54 miembros repartidos continentalmente y elegidos por tercios
anualmente:
– África, 14.
– Europa occidental y otros, 13.
– Asia, 11.
– Latinoamérica, 10.
– Europa oriental, 6.

Se estructura en:

Comisiones funcionales: Comisiones regionales:


– De Derechos Humanos. – Comisión Económica para África (CEPA), sede Etiopía.
– De Estadísticas. – C.E. para América Latina y el Caribe (CEPALC), Chile.
– De Población. – C.E. y Social para Asia Occidental (CEPAO), Jordania.
– De Desarrollo social. – C.E. para Europa (CEPE). Suiza.
– De Status de la mujer. – C.E. y Social para Asia y el Pacífico (CESPAP). Tailandia.
– De Narcóticos.

35
-Pág.63-
CIENCIAS SOCIALES. GEOGRAFÍA E HISTORIA

E) Organismos especializados:
– Agencia Internacional de Energía Atómica (Viena).
– Alto comisionado de las N.U. para los Refugiados (ACNUR) (Ginebra).
– Conferencia de Comercio y Desarrollo de las N.U. (UNCTAD) (Ginebra).
– Consejo Mundial de la Alimentación (Nueva York).
– Fondo de las N.U. para la Infancia (UNICEF) (Nueva York).
– Y otros.

F) Otros Organismos o Agencias especializadas dentro del sistema de la O.N.U.

GATT.—Acuerdo sobre aranceles aduaneros con sede en Suiza. Desde el 1 de enero de


1995 ha sido sustituido por la OMC (Organización Mundial del Comercio)

BM.—Banco Mundial, integrado por el BIRF: Banco Internacional de Reconstrucción y


Fomento y la AIF: Asociación Internacional de Fomento. Su sede es Washington.

CFI.—Corporación Financiera Internacional. Sede en Washington

FAO.—O. de la N.U. para la Agricultura y la Alimentación. Roma.

FMI.—Fondo Monetario Internacional. Washington.

UNESCO.—O. de las N.U. para la Educación, la Ciencia y la Cultura. París.

Otras: O. Mundial de la Salud (Ginebra)


Unión Postal Universal
O. Internacional de Aviación Civil (Montreal)
O.N.U. para el Desarrollo Industrial (Viena)
O. Meteorológica Mundial
O.I. del Trabajo
Unión.I. de Telecomunicaciones
O. Mundial del Turismo (Madrid)

4.1.2. FONDO MONETARIO INTERNACIONAL (F.M.I.)

Se trata de uno de los organismos especializados de las Naciones Unidas cuyo origen hay
que buscar en los años 40. Finalizada la II Guerra Mundial, los países aliados occidentales deci-
dieron poner en marcha una serie de instituciones económicas internacionales que permitieran
la expansión del comercio internacional y el establecimiento de un adecuado sistema de
pagos.

36
-Pág.64-
U . D . 8 . - P A RT I C I PA C I Ó N Y CONFLICTO POLÍTICO EN EL MUNDO ACTUAL

En abril de 1943, todavía en plena Guerra, se plantearon dos proyectos, uno estadouni-
dense y otro británico sobre la creación de una institución monetaria internacional. El presti-
gioso economista británico J.M. KEYNES, defendía la creación de un banco internacional que
concedería créditos a los países miembros utilizando una moneda nueva que se llamaría ban-
cor. El plan norteamericano consistía en la creación de un fondo de 5.000 millones de dóla-
res aportados por cada nación miembro en partes proporcionales para, sobre ese fondo, reclamar
los créditos. En abril de 1944, tras analizar otros dos planes complementarios, uno canadiense
y otro francés, británicos y estadounidenses se ponían de acuerdo en un plan que recogía bási-
camente los postulados norteamericanos. En la Conferencia Monetaria y Financiera de las
Naciones Unidas, celebrada en Bretton Woods (New Hampshire, EE.UU.), en julio de 1944, se
crearon Fondo Monetario Internacional (FMI) y el Banco Internacional para la Recons-
trucción y el Desarrollo (BIRD).
La misión fundamental del FMI consiste en reducir el desequilibrio de las balanzas de
pagos de los países miembros mediante la concesión de créditos procedentes de los recursos
del fondo y por medio de la promoción de la estabilidad del cambio entre monedas.
Un total de 39 países constituyeron finalmente el Fondo que continuamente se fue amplian-
do hasta la actualidad.
La sede del FMI se encuentra en Washington y está dirigido por un Consejo de Goberna-
dores elegido por los países miembros. Los gobernadores adoptan las decisiones por mayoría
simple teniendo cada uno el mismo número de votos base aumentados en uno por cada 100.000
dólares de la cuota correspondiente al país. También existe un Consejo ejecutivo al que con-
ciernen las operaciones normales del fondo.

4.1.3. ORGANIZACIÓN DEL TRATADO DEL ATLÁNTICO NORTE (O.T.A.N.)

4.1.3.1. Origen y evolución


Finalizada la Segunda Guerra Mundial, empezaron a surgir discrepancias entre los países
aliados. Las fundamentales tenían relación con la diferente concepción de la política y la eco-
nomía entre los aliados occidentales (Reino Unido, Francia, Estados Unidos) y la Unión
Soviética. La desmovilización en los países occidentales no fue seguida por la URSS, que llegó
a superar a los efectivos militares de los países aliados en una relación de 1 a 6 a finales de 1946.
Por otro lado las tensiones entre ambas órbitas iban creciendo paulatinamente. Los aconteci-
mientos de Praga de 1948 que concluyeron con la definitiva caída de Checoslovaquia en la
esfera de influencia Soviética, decidieron a los países occidentales a promover una política
defensiva común. Así, el 17 de marzo de 1948, Bélgica, Francia, Luxemburgo, Países Bajos
y Reino Unido, firman el Tratado de Bruselas, germen de la llamada Unión Europea Occi-
dental (UEO). Pero la escasez de recursos de los países firmantes les llevó a solicitar el apoyo
de Estados Unidos y de Canadá y, sobre esa base, se diseñó un sistema de defensa común.

37
-Pág.65-
CIENCIAS SOCIALES. GEOGRAFÍA E HISTORIA

Definitivamente el 4 de abril de 1949, se firma en Washington el Tratado del Atlántico


Norte, suscrito por 12 países:
Estados Unidos Dinamarca Luxemburgo
Canadá Italia Islandia
Francia Bélgica Noruega
Reino Unido Holanda Portugal

Posteriormente se unieron:
Grecia y Turquía (1952)
República Federal Alemana (1955)
España (1982)
República Checa, Polonia y Hungría (1999)
Bulgaria, Eslovaquia, Eslovenia, Estonia, Letonia, Lituania y Rumania (2004)

La sede de la Organización creada al amparo del tratado, se estableció inicialmente en


París, pero la decisión francesa de retirarse de la estructura militar integrada, en 1966, llevó
aparejada la solicitud de que los organismos de la Organización abandonaran suelo francés. De
esta manera la sede se estableció en Bruselas.

4.1.3.2. Principios y propósitos


El tratado consta de 14 artículos precedidos de un preámbulo que hace constar que la Alian-
za nace en el marco de la Carta de las Naciones Unidas. Sus aspectos esenciales son:
– Se establece el objetivo de preservar la paz y la seguridad en el mundo.
– Los estados miembros se comprometen en favor de instituciones libres y de una políti-
ca económica y social tendente a la estabilidad y el bienestar.
– Se comprometen a mantener y desarrollar la capacidad colectiva e individual para
hacer frente a una posible agresión exterior.
– Se prevén consultas conjuntas y cooperación cuando exista una amenaza contra la
integridad territorial, la independencia política o la seguridad de uno de los países
miembros.
– Cualquier ataque armado a un país miembro será considerado como un ataque
contra todos ellos.

4.1.3.3. Estructura de la Alianza


Una serie de organismos conforman la estructura política y militar de la Alianza que se rige
bajo los principios de seguridad común y decisión por consenso, no por mayorías.

38
-Pág.66-
U . D . 8 . - P A RT I C I PA C I Ó N Y CONFLICTO POLÍTICO EN EL MUNDO ACTUAL

A) Consejo Atlántico.—Es el órgano supremo de la Alianza. Su misión fundamental


es el de la Alianza: el mantenimiento de la paz y la seguridad internacionales. Está com-
puesto por los Ministros de Asuntos Exteriores de los países miembros y se reúne dos veces al
año. Además, durante el resto del año se mantiene en Sesión permanente reuniéndose al menos
una vez por semana los Embajadores Permanentes que actúan como portavoces de sus Gobier-
nos respectivos.
B) Comité de Planes de Defensa.—Paralelo al Consejo, dentro de la estructura militar,
funciona con los mismos criterios reuniéndose dos veces por año los Ministros de Defensa y
manteniendo las reuniones de los Embajadores Permanentes.
C) Secretario Internacional.—El Secretario General de la Alianza preside, además, el
Consejo y el Comité de Planes, así como los distintos Comités que se citan en la estructura polí-
tica de la Organización. En la actualidad desempeña el cargo el británico GEORGE ROBERTSON
en sustitución del español JAVIER SOLANA MADARIAGA, su predecesor.
D) Estructura Civil y Militar.—El Consejo Atlántico y el Comité de Planes, además del
Secretariado, forman la estructura superior de la Alianza y se integran en la llamada estructura
Civil. La estructura Militar enlaza con ésta a través del Comité Militar.

COMITÉS DE LA ESTRUCTURA CIVIL:


— Comité Político. Estructurado en varios subgrupos.
— Comité Económico. Cuestiones económicas con implicaciones de orden político o de
defensa.
— Comité de análisis de la Defensa. Asuntos relacionados con programas de defensa.
— Conferencia de Directores Nacionales de Armamento. Coordinación de programas de
Investigación y Desarrollo (I+D).
— Conferencia de Altos Jefes de Logística. Órgano consultivo en cuestiones de logística.
— Comité de Seguridad. Seguridad de la documentación, instalaciones y autoridades de
la Alianza.
— Comité Superior de Planes de Defensa Civil.
— Comité conjunto para Comunicaciones y Electrónica.
— Comité de Infraestructura. Estudio y puesta en práctica de obras de infraestructura de
interés para la Alianza.
— Comité Científico. Promueve y potencia la investigación científica.
— Comité Sobre Retos de la Sociedad Moderna.
— Comité de Defensa Aérea.
— Comité para Mando, Control, Comunicaciones y Proceso Automático de Datos.
— Comité para Coordinación del Espacio Aéreo Europeo.
— Comité de Operaciones del Consejo y Ejercicios.

39
-Pág.67-
CIENCIAS SOCIALES. GEOGRAFÍA E HISTORIA

— Comité de Presupuesto Civil.


— Comité de Presupuesto Militar.
— Grupo de Planes Nucleares.
— Comité Militar. Compuesto por los Jefes de Estado Mayor. Se reúne dos veces por año
y sigue funcionando con carácter permanente por la presencia de Representantes Mili-
tares Permanentes. Sus misiones son:
– Asesorar al Consejo y al Comité de Planes.
– Enlazar los mandos de la Alianza con el Consejo y el CP.

ESTRUCTURA MILITAR: (NUEVA ESTRUCTURA VIGENTE DESDE EL 04 DE ABRIL DE 1999)


Los Mandos Integrados son órganos supranacionales determinados por cuestiones geo-
gráficas y políticas:
Mando Aliado del Atlántico (ACLANT).—Espacio del Atlántico desde el Polo Norte hasta
el trópico de Cáncer y desde las costas norteamericanas y canadienses a las europeas y africa-
nas. Las fuerzas navales que integran el Mando se estructuran en una Flota de Intervención y
una Fuerza de Submarinos. Se articula en tres mandos regionales:
– Oeste, con su Cuartel General en Norfolk (EE.UU.).
– Este, con su Cuartel General en Northwood (Reino Unido).
– Sureste, con su Cuartel General en Lisboa (Portugal).
Mando Aliado en Europa (ACEUR).—Cubre el espacio europeo y el Mediterráneo hasta
Turquía. La sede de su cuartel general es Mons (Bélgica). Se estructura en dos Mandos regio-
nales, cada uno de los cuales se articula en mandos subregionales y mandos de componente
naval y aéreo:

Mando regional Norte, con Cuartel General en Brussum (Países Bajos):


– Mando subregional norte, en Stavanger (Noruega).
– Mando subregional noroeste, en Karup (Dinamarca).
– Mando subregional centro, en Heidelberg (Alemania).
– Mando de componente aéreo, en Ramstein (Alemania).
– Mando de componente naval, en Northwood (Reino Unido).

Mando regional Sur, con Cuartel General en Nápoles (Italia):


– Mando subregional sur, en Verona (Italia).
– Mando subregional suroeste, en Madrid (España).
– Mando subregional sur-centro, en Larissa (Grecia).
– Mando subregional sureste, en Izmir (Turquía).
– Mando de componente aéreo, en Nápoles (Italia).
– Mando de componente naval, en Nápoles (Italia).

40
-Pág.68-
U . D . 8 . - P A RT I C I PA C I Ó N Y CONFLICTO POLÍTICO EN EL MUNDO ACTUAL

4.1.4. LA UNIÓN EUROPEA

4.1.4.1. Origen y evolución de la Unión Europea

A partir de la II Guerra Mundial fueron apareciendo en Europa diversas organizaciones


internacionales. Pero la idea de la unión no es nueva, además de los intentos de unificación por
la fuerza que ya hemos citado, diferentes voces clamaron a lo largo de la historia por la cons-
trucción de una única Europa. Algunas de esas voces fueron:

– PIERRE DUBOIS, jurista francés del siglo XIV que propuso una confederación europea que
sería gobernada por un consejo de “hombres sabios, expertos y honestos”.
– LUIS VIVES, auténtico ciudadano de Europa del siglo XV, que nació en Valencia, estudió
en París y fue profesor en Brujas, Lovaina y Oxford, propuso la creación de un ejército
europeo para combatir a los turcos.
– WILLIAM PENN, inglés que sugirió en el siglo XVII la formación de un parlamento eu-
ropeo.
– PIERRE J. PROUDHON, francés que propuso, en el siglo XIX la formación de una federa-
ción europea.
– ARISTIDE BRIAND, primer ministro francés del periodo de entreguerras defendía la idea
de crear unos Estados Unidos de Europa.

Todas éstas no son más que algunas muestras de la idea de unidad europea que quedaron
sencillamente en el marco de la entelequia. Fue después de la II Guerra Mundial, cuando las
ideas pudieron pasar a acciones.

Acabada la guerra, muchos pensadores y líderes políticos europeos, conscientes de haber


perdido la hegemonía mundial frente a los Estados Unidos y la Unión Soviética, elevaron sus
voces clamando por una ineludible unión. La misma presentaba un dúo de ventajas:

– Políticamente, era la garantía de una paz tan vapuleada en la historia europea.


– Económicamente, la creación de un mercado único fomentaría el crecimiento en un
marco más competitivo y la mejor distribución del trabajo y la riqueza.

Así, en el marco de la O.N.U., surgió la Comisión Económica para Europa (ECE), cuyo
objetivo era convertirse en instrumento de cooperación de todos los países europeos, tanto del
Este como del Oeste. Pero la división en bloques, la “guerra fría”, hizo fracasar el proyecto y
orientar los procesos de cooperación e integración en cada uno de los ámbitos.

41
-Pág.69-
CIENCIAS SOCIALES. GEOGRAFÍA E HISTORIA

El planteamiento de integración occidental se articulaba bajo dos perspectivas, estrategias o


criterios:
– Funcionalista.—Basado en la idea del mantenimiento de los Estados nacionales que
aproximarían sus políticas pero mantendrían plenamente su soberanía e independencia.
– Federalista.—Supondría la creación de una estructura política federal por encima de los
estados-nación.
La primera de las tesis fundamentó la creación de instituciones como la Organización Euro-
pea de Cooperación Económica (OECE) en 1948 y el Consejo de Europa en 1949. En contra-
partida, seis países europeos decidieron ir más lejos y en 1951 firmaron el Tratado de París que
creaba la Comunidad Europea del Carbón y del Acero (CECA) y en 1957 la Comunidad Euro-
pea de la Energía Atómica (EURATOM), y la Comunidad Económica Europea (CEE). Esos seis
países fueron Francia, Alemania, Italia, Bélgica, Holanda y Luxemburgo. Por su lado, otros
siete países prefirieron asociarse comercialmente con menos afanes integradores y fundaron en
1960 la Asociación Europea de Libre Comercio (European Free Trade Association, EFTA), for-
mada por Reino Unido, Austria, Suiza, Noruega, Suecia, Dinamarca y Portugal.
Pero sin duda la primera piedra de la construcción del edificio de la actual Unión Europea
fue la aludida creación de la Comunidad Económica Europea, consecuencia de la firma del Tra-
tado de Roma, el 25 de marzo de 1957, que entró en vigor el 1 de enero de 1958. Los seis paí-
ses fundadores constituyeron así el llamado Mercado Común, un primer paso de integración
basado en la coordinación de las políticas económicas pero con profundas aspiraciones de uni-
dad política y social.
El Reino Unido reorientó su actitud hacia Europa y presentó su candidatura al ingreso en la
CEE, postura que fue secundada por otros países de la EFTA. Como consecuencia, a partir del
1 de enero de 1973 el Reino Unido, la República de Irlanda y Dinamarca se incorporaron a las
instituciones comunitarias. Noruega, se quedó en la antesala como consecuencia del resultado
negativo del referéndum popular.
La Europa de los Nueve fue consolidando sus instituciones y su funcionamiento mientras se
establecían acuerdos bilaterales y tratados preferenciales con otros países y se producían las
negociaciones de adhesión de nuevos candidatos. Fruto de estas últimas fue la incorporación de
Grecia en 1982 y, en 1986, España y Portugal.
El Acta Única Europea fue un nuevo Tratado, firmado en Luxemburgo el 17 de febrero de
1986 que entró en vigor el 1 de julio de 1987, cuyo objetivo prioritario era poner las bases de
un auténtico espacio común sin fronteras para 1992. Cerca de 300 nuevas disposiciones preten-
dían garantizar la libre circulación de mercancías, servicios y capitales.
Se pretende con el Acta Única armonizar las legislaciones nacionales para eliminar los con-
troles técnicos, físicos y fiscales de las fronteras internas de la Comunidad. Para asegurar la
libre circulación de las personas se requiere la adopción de otras medidas complementarias,
identificadas en el llamado “Documento de Palma”, elaborado durante la presidencia española
de la Comunidad de 1989.

42
-Pág.70-
U . D . 8 . - P A RT I C I PA C I Ó N Y CONFLICTO POLÍTICO EN EL MUNDO ACTUAL

Además, el Acta Única mejora el sistema de la toma de decisiones comunitario y concede una
mayor participación del Parlamento Europeo en el proceso. También amplió las competencias a
la política de medio ambiente y de investigación y desarrollo tecnológicos. Incorpora además el
principio de la cohesión económica y social, materializado, entre otros instrumentos, en los lla-
mados “Fondos Estructurales”. Estos fondos pretenden contrarrestar los desequilibrios regiona-
les para fomentar un desarrollo armónico y homogéneo de todos los espacios de Europa.
Por último, el Acta Única recoge por primera vez las bases de una política exterior común.
En febrero de 1992, se firmó, en la ciudad holandesa de Maastricht, el Tratado de la Unión
Europea que entró en vigor el 1 de noviembre de 1993. El Tratado, que no es un punto final,
pues plantea aspectos evolutivos y de revisión futura, gira entorno a tres ejes:
— La Unión Económica y Monetaria, como culminación del gran espacio económico sin
fronteras.
— La Unión Política centrada en:
– Más democracia (mayores poderes al Parlamento Europeo).
– Más eficacia (con una mecánica institucional más ágil).
– Más solidaridad interna (reforzando la cohesión económica y social).
– Más solidaridad exterior (creando una política exterior y de seguridad común que
permitirá una mayor capacidad de iniciativa internacional).
– Más equilibrio.
— Los temas de Interior y Justicia.
Puesto en marcha el Tratado de Maastricht, se produjo la, hasta ahora, última incorporación
de países: Suecia, Austria y Finlandia. Nuevamente Noruega, como en 1972, se quedó en puer-
tas como consecuencia de una nueva negativa de su población manifestada en referéndum.
La Unión Económica y Monetaria ha seguido su proceso, con reajustes temporales. Decisi-
vo fue el acuerdo firmado en Madrid en diciembre de 1996, tras la reunión del Consejo Europeo,
que bautizó a la nueva moneda única con el nombre de “Euro” y fijó los plazos para la sustitu-
ción de las divisas nacionales por ella, estableciendo el 1 de enero de 1999 como la fecha de la
unión monetaria y el 1 de enero del 2002 como la de la entrada en circulación de las monedas y
billetes que desplazarían definitivamente a las actuales monedas el 1 de marzo del 2002. Estos
plazos fueron confirmados por el Consejo Europeo de Bruselas, celebrado en los primeros días
de mayo de 1998, cuando, sobre la base de las recomendaciones hechas en marzo por la Comi-
sión, el Consejo decidió los 11 países que se integrarían en la moneda única. España, Bélgica,
Alemania, Francia, Irlanda, Italia, Luxemburgo, Países Bajos, Austria, Portugal y Finlandia,
pasaron el examen de las condiciones de convergencia fijadas en Maastricht. Aun cumpliendo
dichas condiciones, Reino Unido y Dinamarca, se autoexcluyeron del proceso y Grecia y Sue-
cia, suspendieron. Finalmente, Grecia, una vez superadas las condiciones, completó el número
de 12 países que el 1 de enero de 2002 cambiaron sus respectivas monedas por el “Euro”. Ade-
más, en la trascendental reunión de Bruselas, el Consejo, decidió el nombre del primer Gober-
nador del Banco Central Europeo, el holandés WIN DUISEMBERG.

43
-Pág.71-
CIENCIAS SOCIALES. GEOGRAFÍA E HISTORIA

El 1 de mayo de 1999 entró en vigor el Tratado de Amsterdam, que ha modificado deter-


minados aspectos del Tratado de Maastricht, afectando, entre otros, a los siguientes:
– Quedan reforzados los poderes del Parlamento Europeo, cuya composición se limita a
un máximo de 700 parlamentarios, independientemente del número de países que inte-
gren la Unión en el futuro.
– Se hace hincapié en los temas de empleo y derechos de los ciudadanos.
– Se remueven los últimos obstáculos a la libre circulación de personas.
– Se establecen líneas de actuación para evitar desigualdades y discriminaciones entre los
ciudadanos.
– Se fomenta la enseñanza, la investigación y la cultura.
– Se refuerza la Política Exterior y de Seguridad Común (PESC), estableciendo como fór-
mula de representación de la Unión en el exterior la llamada "troika" comunitaria, inte-
grada por el Presidente del Consejo de Ministros, el Comisario competente y el Alto
Representante para la PESC (actualmente el español Javier Solana Madariaga). Además,
se incluye la posibilidad de participación de la Unión en misiones armadas de ayuda
humanitaria y mantenimiento de la paz. En todo caso, aunque se han agilizado los proce-
dimientos para establecer las líneas de actuación, quedan aún muchos aspectos sin resol-
ver, de modo que aún no se puede hablar de una política exterior unívoca de la Unión.

Con la firma del Tratado de Niza en 2001 se acordó la reforma de las instituciones y una
redistribución del poder entre sus miembros, necesaria a causa de las ampliaciones previstas
como la del 1 de mayo de 2004 con la incorporación de los siguientes Estados: República Checa,
República Eslovaca, Eslovenia, Letonia, Lituania, Estonia, Hungría, Chipre, Malta y Polonia.
Rumanía y Bulgaria están a la espera de su integración para el año 2007, quedando a la
espera Turquía.
El Consejo Europeo reunido en Laeken adoptó, el 15 de diciembre de 2001, la Declaración
sobre el futuro de la Unión Europea, por la que la Unión se compromete a hacerse más demo-
crática, transparente y eficaz, y a abrir la vía hacia una Constitución para dar respuesta a las
expectativas de los ciudadanos europeos.
El viernes 29 de octubre de 2004, los Jefes de Estado o de Gobierno y los Ministros de Asun-
tos Exteriores de los 25 Estados miembros de la Unión Europea firmaron en Roma el Tratado
por el que se establece una Constitución para Europa.
Una vez firmado el Tratado, los Estados miembros tendrán que ratificar la Constitución de
conformidad con sus respectivas normas internas.
El 20 de febrero de 2005 se aprobó la Constitución Europea en España.
El 13 de abril de 2006 el Parlamento Europeo aprueba la entrada de Rumania y Bulgaria en
la UE. La adhesión de estos países tuvo lugar el 1 de enero de 2007, pasando de 25 a 27 países
miembros la UE.

4.1.4.2. Características de la Unión Europea


Resulta esencial considerar que la Unión Europea no es una simple organización de coope-
ración económica. Por una parte es un conjunto de instituciones comunes a los países y por otra
es el conjunto de esos países que ceden parcelas de su capacidad de decisión a las instituciones
comunes o adoptan las mismas de forma colegiada.
44
-Pág.72-
U . D . 8 . - P A RT I C I PA C I Ó N Y CONFLICTO POLÍTICO EN EL MUNDO ACTUAL

LOS GRANDES PASOS HACIA LA UNIÓN


01-01-1951 CECA (Tratado de París)
01-01-1957 EURATOM
25-03-1957 Firma del Tratado de Roma
Entra en vigor: CEE
01-01-1958
Países fundadores: Bélgica, Holanda, Luxemburgo, R.F. Alemana, Italia, Francia
EFTA: Reino Unido, Austria, Suiza, Noruega, Suecia, Dinamarca y Portugal.
01-01-1960
Más tarde también Islandia y Finlandia.
01-01-1973 Primera ampliación: Reino Unido, Irlanda, Dinamarca
01-01-1982 Segunda ampliación: Grecia
01-01-1986 Tercera ampliación: España y Portugal
01-01-1987 Acta Única (Tratado firmado el 17/02/86 en Luxemburgo)
01-11-1993 Unión Europea (Tratado firmado el 07/02/92 en Maastricht)
01-01-1995 Cuarta ampliación: Suecia, Finlandia y Austria
01-01-1999 Unión Económica y Monetaria
01-05-1999 Tratado de Amsterdam (Tratado firmado el 02/10/1997)
27-02-2001 Firma del Tratado de Niza
01-01-2002 El “euro” sustituye a las monedas nacionales de 12 de los 15 miembros
Quinta ampliación: República Checa, República Eslovaca, Eslovenia, Letonia,
01-05-2004
Lituania, Estonia, Hungría, Chipre, Malta y Polonia.
01-01-2007 Sexta ampliación: Rumanía y Bulgaria

Algunos de sus rasgos definitorios son los siguientes:


A) Ciudadanía de la Unión.—En el camino de la creación de una auténtica nacionalidad
europea, el Tratado de la Unión, otorga tres nuevos derechos a los nacionales de los Estados
miembros: Libre circulación, Derecho de residencia y Derecho a votar y ser elegido en el lugar
de residencia en las elecciones municipales y al Parlamento Europeo.
B) Cohesión económica y social.—Frente a criterios basados en un simple comercialis-
mo egoísta, los países integrados en la Unión, pretenden aproximar sus niveles de riqueza en
busca de la cohesión económica y social y para ello se han dotado, desde el Acta Única, entre
otros instrumentos, de los llamados “Fondos Estructurales”:
– FEDER (Fondo Europeo de Desarrollo Regional).
– Fondo Social Europeo.
– FEOGA (Fondo Europeo de Orientación y de Garantía Agrícola).
C) Unión Aduanera, Mercado Único y políticas comunes.—La unión aduanera permite
el libre tránsito de mercancías y productos orientado a la creación de un mercado único en cuyo
marco inscribir las políticas económicas comunes (agricultura, pesca, transportes, comercio).

45
-Pág.73-
CIENCIAS SOCIALES. GEOGRAFÍA E HISTORIA

Además, se desarrollan otras políticas comunes con un contenido más social, tales como las
de cultura, salud pública, protección a los consumidores, educación, medio ambiente, investi-
gación y desarrollo, formación profesional, etc.
D) La Europa Social.—El Tratado de Roma había regulado aspectos sociales relaciona-
dos con la libre circulación de trabajadores y el Acta Única sólo añadió medidas referidas a la
seguridad e higiene en el trabajo. Tras el Tratado de Maastricht se ponen las bases de una autén-
tica política social en cuestiones como la seguridad social, la protección social, etc.
E) La Unión Económica y Monetaria.—La Unión Económica y Monetaria, planteada de
un modo serio desde el Tratado de Maastricht, ha seguido su proceso, con reajustes temporales.
El Consejo Europeo de Bruselas, celebrado en los primeros días de mayo de 1998, decidió los
11 países que se integrarían en la moneda única bautizada con el nombre de “Euro”, tras la reu-
nión del Consejo Europeo en Madrid en diciembre de 1996. España, Bélgica, Alemania, Fran-
cia, Irlanda, Italia, Luxemburgo, Países Bajos, Austria, Portugal y Finlandia, pasaron el examen
de las condiciones de convergencia fijadas en Maastricht. Aun cumpliendo dichas condiciones,
Reino Unido y Dinamarca, se autoexcluyeron del proceso y Grecia y Suecia, suspendieron.
Finalmente ingresó Grecia formando los 12.
F) Política exterior.—El Tratado de la Unión diseñó una Política Exterior y de Seguri-
dad Común (PESC) como uno de los pilares básicos de la Unión. El Consejo Europeo fijará por
consenso las orientaciones generales de tales “acciones comunes” y el Consejo de Ministros,
establecerá por unanimidad los medios, procedimientos, alcance y duración de cada “acción
común”. La representación de la Unión en la PESC será asumida por el país que ostente la Pre-
sidencia del Consejo de Ministros y por un comisario especial (desde 1999 JAVIER SOLANA).
G) Nuevas políticas y acciones.—Se incorporan nuevas políticas y acciones en distintos
terrenos:
– Política cultural.
– Salud pública.
– Protección a los consumidores y usuarios.
– Política educativa. Proyectos como ERASMUS, LINGUA O LEONARDO DA VINCI,
tienden a lograr una integración de los sistemas de enseñanza.
– Política industrial, con el fin de facilitar reajustes y reconversiones, favorecer la inno-
vación tecnológica y contribuir a mejorar la competitividad de las empresas.
– Política de creación de grandes redes transeuropeas de infraestructura del transporte
terrestre, marítimo y aéreo.
H) Reforzamiento de otras políticas comunitarias.—
– Medio Ambiente.
– Investigación y Desarrollo (I+D) vital para mejorar la competitividad de la industria
europea.
– Formación profesional.

46
-Pág.74-
U . D . 8 . - P A RT I C I PA C I Ó N Y CONFLICTO POLÍTICO EN EL MUNDO ACTUAL

4.1.4.3. Instituciones
Las actuales instituciones de la Unión son consecuencia del establecimiento de una serie de
mecanismos de coordinación de la política comunitaria, diseñados ya en el Tratado de Roma y
que han ido siendo estructurados, ampliados y redefinidos en los sucesivos tratados, hasta lle-
gar a la definición actual conforme a los postulados del Tratado de la Unión, una definición que
deberá modificarse de cara a la ampliación hacia el Este.
A) Tribunal de Justicia
Sede: Luxemburgo
Órgano judicial
Lo integran: 27 jueces y 8 abogados generales designados por los Estados miembros.
Le compete: – Garantizar el respeto a la legislación comunitaria y
– Asegurar una interpretación uniforme del derecho comunitario.
B) Comisión
Sede: Bruselas
Órgano ejecutivo
Lo integran: 27 Comisarios, uno por cada país designados por consenso por los Estados
(1 Presidente y 24 Comisarios).
Le compete: – Velar por los intereses de la Unión, asegurar la aplicación, desarrollo y
cumplimiento de los tratados y del conjunto de las normas comunitarias.
– Iniciativa de la política de la Unión, elaborando y proponiendo las normas
comunitarias.
C) Consejo de Ministros
Sede: Bruselas y según Presidencia
Órgano legislativo
Lo integran: Los ministros de los gobiernos de los Estados miembros.
Ejerce el poder legislativo aprobando, modificando o rechazando las propuestas de la
Comisión y Conduce las Relaciones Exteriores de la Unión.
Las decisiones se adoptan por Unanimidad en temas claves, por Mayoría cualificada en el
resto.
D) Presidencia. Desde el Acta Única es ejercida por turno semestral de acuerdo con el
orden alfabético en el idioma original de cada país.
E) Consejo Europeo
Lo integran los Jefes de Estado y/o Gobierno y Ministros de AA.EE. de los Estados miem-
bros más el Presidente de la Comisión y otro miembro de la misma.
Debe reunirse como mínimo una vez cada seis meses, en el país que ostenta la presidencia.
Le compete: – Orientaciones generales de la acción común.
– Impulso al desarrollo de la integración europea.

47
-Pág.75-
CIENCIAS SOCIALES. GEOGRAFÍA E HISTORIA
CIENCIAS SOCIALES. GEOGRAFÍA E HISTORIA

F) Parlamento Europeo
Sede: Estrasburgo
Órgano legislativo consultivo
Lo integran: 785 diputados a partir del 1 de enero de 2007 frente a los 626 que tenía. El
número de diputados que corresponde a cada Estado se fija en los Tratados Comunitarios en fun-
ción del número de Estados que forman parte de la Unión y del máximo de diputados, con un
reparto de escaños por país proporcional a la población:

99 escaños: Alemania 78 escaños: Reino Unido, Francia, Italia


54 escaños: España, Polonia 35 escaños: Rumanía
24 escaños: Bélgica, Grecia, Portugal,
27 escaños: Holanda
República Checa, Hungria
19 escaños: Suecia 18 escaños: Austria, Bulgaria
14 escaños: Finlandia, Dinamarca,
13 escaños: Irlanda, Lituania
Eslovaquia
9 escaños: Letonia 7 escaños: Eslovenia
6 escaños: Luxemburgo, Estonia, Chipre 5 escaños: Malta
Aunque la elección se realiza conforme al sistema electoral decidido por cada país, la for-
mación de los grupos parlamentarios responde a una agrupación ideológica.
Le compete: – Dictámenes consultivos sobre propuestas de la Comisión.
– Participación en la aprobación de los presupuestos.
– Autorización o no de acuerdos internacionales.
– Control político de la Comisión.
G) Banco Central Europeo
Desde el 01/01/1999, al iniciarse la definitiva unión económica y monetaria, se constituyó
el BANCO CENTRAL EUROPEO, en sustitución del anterior INSTITUTO MONETARIO
EUROPEO. En la reunión del Consejo Europeo, celebrada en Bruselas en mayo de 1998, en la
que se decidieron los países que se integrarían en la moneda única, se nombró el primer Gober-
nador del BCE, el holandés WIN DUISEMBERG.
H) Otras instituciones.—Banco Europeo de Inversiones, Tribunal de Cuentas, Comi-
té Económico y Social, Comité de las Regiones, Defensor del Pueblo.
Los recientes acuerdos de la cumbre de Niza (diciembre de 2000) han planteado la reforma
de las instituciones. La materialización de la misma se producirá en los próximos años y deberá
ser profundamente revisada si se pretende lograr una mayor agilidad en la toma de decisiones.

4.1.5. UN CONFLICTO INTERNACIONAL RELEVANTE: ISRAEL


La historia judía hasta 1948, está marcada por su dispersión por Europa y su sistemática per-
secución, que conoció momentos especialmente cruentos. La época del nacionalismo vio sur-
gir, entre los intelectuales y los ideólogos políticos judíos de cualquier adscripción, el llamado
“sionismo” cuyo objetivo era la creación de un Estado judío en Palestina.

48
-Pág.76-
U . D . 8 . - P A RT I C I PA C I Ó N Y CONFLICTO POLÍTICO EN EL MUNDO ACTUAL

Gracias al respaldo británico, tras la I Guerra Mundial, la Sociedad de Naciones otorgó al


Reino Unido el Mandato sobre Palestina y, bajo su administración, comenzaron a establecerse
asentamientos judíos en Palestina. Pero la inmigración judía movilizó a la mayoría árabe que
se sublevó contra el gobierno británico en los años anteriores a la II Guerra Mundial.
La sistemática persecución de que fueron objeto los judíos por la administración nacionalso-
cialista alemana provocó el holocausto, que, finalizada la contienda mundial, propició la sensi-
bilización de la comunidad internacional, que aprobó, en el marco de la recién nacida
Organización de las Naciones Unidas, el Plan de Partición de Palestina. En medio de una autén-
tica guerra, por la sublevación de la población árabe de la zona, el Consejo Provisional del Esta-
do, proclamó el Estado de Israel el 14 de mayo de 1948. Al día siguiente, Egipto, Irak, Jordania,
Siria y Líbano, se unieron a los palestinos en la Iª Guerra árabe-israelí. Israel aumentó su terri-
torio y la mayor parte de los palestinos huyeron hacia los países limítrofes como refugiados.
Los continuos sabotajes palestinos y la prohibición egipcia al paso de barcos de inmigran-
tes y cargueros israelíes por el Canal de Suez, forzaron a Israel en 1957 a una rápida y eficaz
campaña militar, la IIª Guerra árabe-israelí, que le permitió conquistar la franja de Gaza y toda
la Península del Sinaí.
Los deseos de revancha árabe llevaron a la formación de un mando unificado que preparó
la invasión de Israel. Pero el moderno y bien pertrechado ejército judío se adelantó y en una
rápida operación, la Guerra de los Seis Días, invadió Egipto, Siria y Jordania en junio de 1967.
Cisjordania, incluyendo Jerusalem oriental, Gaza, el Sinaí y los Altos del Golán, quedaron bajo
soberanía israelí.
Los actos terroristas palestinos, con la Organización para la Liberación de Palestina
(O.L.P) al frente, marcaron a partir de entonces la resistencia contra Israel. Además, el afán de
desquite de los árabes llevó a Siria y Egipto a atacar por sorpresa al Estado hebreo, aprove-
chando la fiesta del ayuno sagrado judío, el Yom Kippur, nombre con el que se conoce la IVª
Guerra Árabe-Israelí. El conflicto se internacionalizó al implicarse otros países árabes y del
Tercer Mundo, además de la U.R.S.S., en contra de Israel; y Estados Unidos y otros países occi-
dentales, a su favor. La intervención del Secretario de Estado norteamericano HENRY KISSINGER
posibilitó un acuerdo que detuvo la guerra.
Israel y, sobre todo, su economía, salió malparado de la guerra lo que forzó la dimisión
de su carismática primera ministra GOLDA MEIR, sustituida por ISAAC RABIN, y, finalmente,
la derrota de los laboristas y el triunfo del bloque nacionalista y religioso del Likud, con
MENACHEM BEGIN a la cabeza.
La búsqueda de una paz duradera para Palestina se reveló como algo tremendamente com-
plicado por el intrincado juego de intereses y legítimas reivindicaciones de uno y otro lado. Los
acuerdos de Camp David, de 1979, entre Egipto e Israel, apadrinados por el presidente nortea-
mericano JIMMY CARTER, supusieron la definitiva paz entre ambos Estados, pero no el cierre del
conflicto general. La obligada retirada del Sinaí fue compensada con la definitiva anexión de
los Altos del Golán y, tras las presiones de la O.L.P. desde el Líbano, con la invasión del sur de
este país hasta el río Litani.

49
-Pág.77-
CIENCIAS SOCIALES. GEOGRAFÍA E HISTORIA

Los levantamientos populares palestinos, conocidos como la “intifada”, duramente respon-


didos por Israel, marcaron el final de los 80 y el comienzo de los 90 (1989 - 1997) y vuelven a
presentarse con toda su crudeza en el tránsito del siglo XX al XXI.
La conciencia internacional de la necesidad de una paz definitiva llevó a la convocatoria de
la Conferencia de Paz sobre Oriente Próximo que se desarrolló en Madrid en 1991. Ésta sirvió
de plataforma a las posteriores negociaciones que fraguaron en los acuerdos de Oslo entre el
líder palestino YASER ARAFAT y el primer ministro israelí ISAAC RABIN que permitió la creación
de la llamada Autoridad Nacional Palestina, con la cesión de territorios autónomos para los
palestinos en Cisjordania y Gaza. La paz con Jordania, firmada en Washington en 1994 mejo-
ró aún más la situación; pero el proceso de paz se ha visto amenazado por las actuaciones de
grupos radicales, como el fundamentalista musulmán Hezbolá y otros, tanto palestinos como
israelíes, que han impedido su consolidación abriendo la crisis actual.

4.1.6. LA INESTABILIDAD DEL MUNDO ISLÁMICO


El mundo islámico se ha visto convulsionado por los conflictos en Iraq, la expansión del
integrismo islámico y las tensiones entre israelíes y palestinos.

Iraq ha estado en conflicto durante las últimas décadas. Tras una larga guerra contra Irán
(1980-1988), en 1990 invadió y se anexionó Kuwait para controlar su petróleo, iniciando así la
guerra del Golfo. La ONU exigió a Iraq su retirada, y autorizó una intervención militar inter-
nacional en 1991. Las tropas de la ONU, bajo mando norteamericano, iniciaron un intenso bom-
bardeo aéreo y una ofensiva terrestre que liberó Kuwait de las tropas iraquíes.

Iraq, derrotado, fue sometido a un embargo económico, y tuvo que eliminar sus armas de
destrucción masiva. Pero EE.UU afirmó que Iraq violaba los acuerdos y rehacía su arsenal, y
en el año 2003 atacó de nuevo para derrocar a Saddam Hussein.

El terrorismo islámico también hizo mella en España tras los acontecimientos del 11 de
marzo de 2004 atentando en la red ferroviaria de Madrid y ocasionando 192 muertos y 1800
heridos.

En estos años se extendió por el mundo musulmán el integrismo, doctrina que propugna el
cumplimiento estricto de la tradición islámica. Su violencia provocó conflictos en diversos paí-
ses. En Argelia, en 1991, tras unas elecciones, se ilegalizó el Frente Islámico de Salvación (FIS),
lo que provocó una guerra civil entre los integristas y el propio Estado, que afectó a toda la
población. En Afganistán, el régimen integrista de los talibanes fue derrocado en el 2001 tras
una intervención militar de EE.UU, acusándolo de dar cobijo a Osama Bin Laden y a la orga-
nización terrorista Al Qaeda, responsables del atentado del World Trade Center de Nueva York
en el 2001.

50
-Pág.78-
U . D . 8 . - P A RT I C I PA C I Ó N Y CONFLICTO POLÍTICO EN EL MUNDO ACTUAL

CAUCES Y RETOS ACTUALES


5. PARA LA PARTICIPACIÓN CIUDADANA.
(PARTIDOS Y ORGANIZACIONES SOCIALES. INSTITUCIONES
Y CENTROS DE INFORMACIÓN Y AYUDA CIUDADANA)
La participación ciudadana en la vida pública es cada vez más intensa y personalizada,
abierta a muy diversos campos y bajo muy diversas manifestaciones.

5.1. LA PARTICIPACIÓN POLÍTICA


La opinión ciudadana se manifiesta principalmente a través de las elecciones. Para canali-
zar dicha opinión existen los partidos políticos a los que la propia Constitución española otor-
ga ese papel desde su texto.
Aunque ya en Grecia o en la Roma Republicana se pueden apreciar indicios de lo que podríamos llamar
partidos políticos y a que éstos, aun sin ese nombre ni el papel de los actuales, estaban presentes de alguna
forma en sociedades como la del Imperio bizantino y otras, tenemos que esperar hasta las revoluciones libe-
rales para que su existencia tenga realmente sentido político efectivo y para que pongan en práctica su papel
de encauzadores de la opinión de los ciudadanos.
Efectivamente, en Estados Unidos y en la Francia de la Revolución, surgen los primeros par-
tidos políticos modernos. Por lo que respecta a España, desde la puesta en marcha del liberalis-
mo, con la Constitución de Cádiz, se van definiendo partidos que se consolidarán a partir de la
definitiva implantación del Estado liberal en 1833.
El surgimiento de las ideologías de contenido social y socialista a partir de mediados del
siglo XIX determinará la adscripción de los partidos, además de a una idea, a una clase social
diferenciándose partidos burgueses de partidos de trabajadores. El final del siglo XIX y el
comienzo del XX estuvieron presididos por el enfrentamiento entre ambos “lados” que llegó a
fraguar en la aparición de regímenes nuevos como el surgido de la Revolución de 1917 en Rusia.
Posteriormente, surgieron también partidos de corte social y nacionalista que generaron,
junto a las tensiones acumuladas, un cóctel explosivo que desembocó en la II Guerra Mundial.
En la actualidad, los partidos políticos mantienen su importante papel de formadores de la
voluntad popular y canalizadores de sus afanes políticos.
En España, en la actualidad, existen dos grandes partidos de implantación nacional, repre-
sentativos del centro – derecha y del centro – izquierda, respectivamente, el Partido Popular
(PP) y el Partido Socialista Obrero Español (PSOE). Además, existe otra formación confederal
de ámbito estatal, Izquierda Unida (IU), cuyo nivel de representatividad ha descendido consi-
derablemente en los últimos años.

51
-Pág.79-
CIENCIAS SOCIALES. GEOGRAFÍA E HISTORIA

Junto a estos partidos estatales, existe un amplio número de partidos de carácter naciona-
lista o regionalista de los que los más importantes son:
– Convergència i Unió, coalición catalanista de centro – derecha.
– Esquerra Republicana de Catalunya, partido catalanista de izquierdas.
– Partido Nacionalista Vasco, de adscripción democristiana.
– Eusko Alkartasuna, también vasco como el anterior y de centro.
– Batasuna, Euskal Herritarrok, o Herri Batasuna, nacionalista vasco radical de izquierdas.
– Bloque Nacionalista Galego, de izquierda.
– Partido Andalucista, de centro – izquierda.
– Coalición Canaria, de centro.
– Y otros muchos (Chunta Aragonesista, Unidad Alavesa, Unión del Pueblo Navarro, Uni-
dad Valenciana…).

5.2. LA PARTICIPACIÓN SOCIAL


Los temas de carácter social y económico cuentan con diversos cauces de participación. Por
una parte están las organizaciones de carácter laboral, por otra las de tipo económico o econó-
mico – social, las de tipo humanitario, etc.
Entre las de carácter laboral existen básicamente dos ámbitos, el de los sindicatos de traba-
jadores y el de las organizaciones de empresarios. Ambas canalizan las inquietudes y defienden
los derechos de su colectivo, a veces, casi por definición, en platillos opuestos de la balanza. En
España, actualmente, existe una organización empresarial fundamental, la Confederación Espa-
ñola de Organizaciones Empresariales (CEOE). Por lo que respecta a los sindicatos, existen dos
grandes organizaciones de ámbito nacional y confederal: Comisiones Obreras (CC.OO.) y la
Unión General de Trabajadores (UGT), a las que hay que añadir otras menores como Unión Sin-
dical Obrera (USO) y otras de carácter regional o ceñidas a determinadas ramas de la actividad
laboral, tal es el caso del sindicato vasco ELA-STV o del sindicato agrario ASAJA, por citar
sólo un ejemplo de cada tipo.
Entre las de carácter económico, destacan las asociaciones de consumidores y entre las
sociales las asociaciones de vecinos, por ejemplo.
Especial importancia han cobrado las llamadas Organizaciones No Gubernamentales
(ONG), de carácter humanitario, ya sea de atención médica, de ayuda a los refugiados, a niños
del Tercer Mundo, combatientes contra el hambre, la pobreza, la enfermedad, la discriminación,
la injusticia…
En la sociedad actual la preocupación gubernamental y no gubernamental por ofrecer infor-
mación y ayuda se ha plasmado en instituciones públicas y privadas. En este orden de cosas
habría que situar los centros públicos para información y atención de emergencias de todo tipo
o las instituciones de ayuda de carácter privado.

52
-Pág.80-
U . D . 8 . - P A RT I C I PA C I Ó N Y CONFLICTO POLÍTICO EN EL MUNDO ACTUAL

RESUMEN
— La Democracia tiene como principios fundamentales: Los derechos individuales y
colectivos, la soberanía popular y la separación de poderes.
— Cuando hablamos de división de poderes nos referimos al poder legislativo, al ejecuti-
vo y al judicial, ejercidos por instituciones como un gobierno, un parlamento y unos tri-
bunales, respectivamente.
— En los Estados democráticos, los ciudadanos participan en el gobierno mediante la elec-
ción de representantes que integran, normalmente, el poder legislativo que, a su vez,
suele controlar al ejecutivo de diferentes formas.
— Los principios y los órganos que forman el Estado quedan contemplados en la Consti-
tución.
— La Constitución española actual fue aprobada en referéndum el 6 de diciembre de 1978.
A través de un preámbulo, 169 artículos organizados en once Títulos y otras disposi-
ciones, establece los derechos y deberes de los españoles, los principios que rigen el
funcionamiento del Estado y las instituciones que lo forman:
– Título Preliminar.—Principios generales.
– Título I.—Derechos y deberes fundamentales.
– Título II.—La Corona.
– Título III.—Las Cortes Generales.
– Título IV.—El Gobierno y la Administración.
– Título V.—Relaciones entre el Gobierno y las Cortes.
– Título VI.—El poder judicial.
– Título VII.—Economía y Hacienda.
– Título VIII.—Organización territorial del Estado.
– Título IX.—Tribunal Constitucional.
– Título X.—Reforma de la Constitución.

— España, de acuerdo con la Constitución:


– Es una monarquía parlamentaria hereditaria.
– En la que el poder legislativo es ejercido por las Cortes Generales, integradas por el
Congreso de los Diputados y el Senado.
– El poder ejecutivo lo ostenta el Presidente del Gobierno con sus Ministros.
– El Presidente es nombrado por el Rey tras conseguir la investidura del Congreso de
los Diputados.
– El Gobierno es controlado por las Cortes a través de interpelaciones y de la moción
de censura.

53
-Pág.81-
CIENCIAS SOCIALES. GEOGRAFÍA E HISTORIA

– El poder judicial es independiente y está gobernado por el Consejo Superior del


Poder Judicial.
– El Estado se organiza territorialmente, bajo el principio de autonomía, en munici-
pios, provincias, comunidades autónomas y otras entidades.
– El Tribunal Constitucional es el garante de la constitucionalidad de las normas.
– La Constitución puede ser reformada parcial o totalmente.

— España se organiza en 17 Comunidades Autónomas.


— La proyección internacional de España se orienta, fundamentalmente, hacia Europa,
Iberoamérica y el Mundo Árabe.
— La ONU es una institución que engloba a casi todos los países del mundo. Tiene como
órganos principales: La Secretaría General, la Asamblea General y el Consejo de Segu-
ridad.
— El FMI, Fondo Monetario Internacional pretende reducir el desequilibrio de las balan-
zas de pagos de los países y promover la estabilidad del cambio entre monedas.
— La OTAN, nacida en medio de la guerra fría, ha modificado su razón de ser aunque
mantiene sus objetivos de preservar la paz y la seguridad, bajo los principios de las
Naciones Unidas, y la defensa mutua de sus miembros.
— La Unión Europea integra a 25 países europeos y cuenta, entre sus instituciones más
importantes, con el Tribunal de Justicia, la Comisión, el Consejo de Ministros, la Pre-
sidencia, el Consejo Europeo, el Parlamento Europeo y el Banco Central Europeo.
— Los ciudadanos participan en la vida política mediante los partidos políticos y en la
laboral por medio de los sindicatos y organizaciones empresariales.

54
-Pág.82-
U . D . 8 . - P A RT I C I PA C I Ó N Y CONFLICTO POLÍTICO EN EL MUNDO ACTUAL

EJERCICIOS DE AUTOCOMPROBACIÓN

1. Uno de los principios de la Democracia es el de la división de poderes. Aquel de dichos poderes


cuya actividad se centra en adoptar las medidas específicas de administración es el poder...

A. Judicial B. Administrativo C. Legislativo D. Ejecutivo

2. En relación con la Constitución española indique la afirmación correcta.

A. El Presidente del Gobierno nombra a sus Ministros en nombre del Rey

B. El Rey nombra al Presidente del Gobierno después de que haya obtenido la investidura en el
Congreso de los Diputados.

C. El Senado puede vetar el nombramiento del Presidente del Gobierno.

D. Para obtener la investidura, el Presidente del Gobierno necesita, en la primera votación, el


apoyo de la mayoría absoluta de los miembros del Senado.

3. De acuerdo con la Constitución española, el supremo órgano fiscalizador de las cuentas y de la ges-
tión económica del Estado es:

A. El Tribunal Supremo C. El Tribunal de Cuentas

B. El Tribunal Constitucional D. El Consejo de Estado

4. ¿Cuántas Comunidades Autónomas cuentan con más de una provincia?

A. 17 B. 7 C. 10 D. 5

5. ¿En qué año España pasó a ser miembro de pleno derecho de la Comunidad Europea?

A. 1977 B. 1955 C. 1978 D. 1986

6. El órgano plenario y deliberante de la Organización de las Naciones Unidas es:

A. El Consejo de Seguridad C. El Secretariado General

B. La Asamblea General D. El Consejo Económico y Social

7. ¿Cuál de los siguientes países NO pertenece a la OTAN?

A. España B. Islandia C. Suiza D. Hungría

8. El país europeo que tiene un mayor número de escaños en el Parlamento Europeo es:

A. Reino Unido B. Alemania C. Francia D. Italia

55
-Pág.83-
CIENCIAS SOCIALES. GEOGRAFÍA E HISTORIA

9. La participación social de carácter laboral se encauza a través de:

A. Los partidos políticos C. Los sindicatos

B. Las organizaciones de consumidores D. La Administración pública

10. Una de las siguientes Comunidades Autónomas NO accedió al autogobierno por la vía prevista en
el Artículo 151 de la Constitución, Señálela.

A. Andalucía B. Cataluña C. Galicia D. Castilla y León

RESPUESTAS A LOS EJERCICIOS

1. D 2. B 3. C 4. C 5. D

6. B 7. C 8. B 9. C 10. D

56
-Pág.84-
U . D . 9 . - A RT E , C U LT U R A Y S O C I E D A D E N E L M U N D O A C T U A L

ÍNDICE
OBJETIVOS . . . . . . . . . . . . . . . . . . . . . . . . . . . . . . . . . . . . . . . . . . . . . . . . . . . . . . . . . . . . . . 2

INTRODUCCIÓN . . . . . . . . . . . . . . . . . . . . . . . . . . . . . . . . . . . . . . . . . . . . . . . . . . . . . . . . . 3

MAPA CONCEPTUAL . . . . . . . . . . . . . . . . . . . . . . . . . . . . . . . . . . . . . . . . . . . . . . . . . . . . . 4

DESARROLLO DE CONTENIDOS . . . . . . . . . . . . . . . . . . . . . . . . . . . . . . . . . . . . . . . . . . 5

1. LA CRISIS DEL ARTE FIGURATIVO EN PINTURA Y ESCULTURA


A PARTIR DEL IMPRESIONISMO . . . . . . . . . . . . . . . . . . . . . . . . . . . . . . . . . . . . . 5

2. ARQUITECTURA FUNCIONAL Y URBANISMO ACTUAL . . . . . . . . . . . . . . . . 9

3. FORMAS DE EXPRESIÓN Y MANIFESTACIONES ARTÍSTICAS ACTUALES


A TRAVÉS DE NUEVOS LENGUAJES (VISUAL, PLÁSTICO, MUSICAL…) . . . . . 9

4. LOS RETOS DEL DESARROLLO CIENTÍFICO Y TECNOLÓGICO. PAPEL


Y SENTIDO DE LAS HUMANIDADES EN LA ACTUALIDAD . . . . . . . . . . . . . . 13

5. REDES Y MEDIOS DE COMUNICACIÓN E INFORMACIÓN . . . . . . . . . . . . . . 14

6. FOCOS DE TENSIÓN Y NUEVOS VALORES EN LAS SOCIEDADES


POSTINDUSTRIALES . . . . . . . . . . . . . . . . . . . . . . . . . . . . . . . . . . . . . . . . . . . . . . . . 16

RESUMEN . . . . . . . . . . . . . . . . . . . . . . . . . . . . . . . . . . . . . . . . . . . . . . . . . . . . . . . . . . . . . . . 17

EJERCICIOS DE AUTOCOMPROBACIÓN . . . . . . . . . . . . . . . . . . . . . . . . . . . . . . . . . . . 18

RESPUESTAS A LOS EJERCICIOS . . . . . . . . . . . . . . . . . . . . . . . . . . . . . . . . . . . . . . . . . 20

-Pág.85-
CIENCIAS SOCIALES. GEOGRAFÍA E HISTORIA

O B J E T I VO S
Al finalizar el estudio de esta Unidad Didáctica, el alumno será capaz de:

• Comprender la originalidad y variopinta manifestación de las formas artísticas


del siglo XX.

• Valorar las grandes innovaciones tecnológicas y su importancia para el desarro-


llo de la Humanidad.

• Analizar y entender la importancia de las redes de comunicación en el entrama-


do social contemporáneo.

• Estudiar las principales cuestiones sociales novedosas del final del siglo XX.

-Pág.86-
U . D . 9 . - A RT E , C U LT U R A Y S O C I E D A D E N E L M U N D O A C T U A L

INTRODUCCIÓN
N os asomamos en este último tema a algunos aspectos de la cultura contemporánea.
Más específicamente, analizamos cuestiones relacionadas con el arte y la sociedad
del siglo XX.
Analizaremos el desequilibrio producido en el arte pictórico y escultórico entre el figu-
rativismo y la abstracción a favor de esta última tendencia. Veremos después cuáles son
las tendencias más importantes de la arquitectura y, a continuación, nos detendremos en
dos formulaciones artísticas con un desarrollo especial en el siglo XX: la música, con
manifestaciones totalmente innovadoras respecto a la tradición anterior, y el cine, arte
específico del siglo XX.

Los retos que plantea el desarrollo de la ciencia y la tecnología serán objeto de nuestra
aproximación posterior, así como el protagonismo que en el mundo contemporáneo tie-
nen las redes y medios de comunicación e información.

Cerraremos con una semblanza de los novedosos aspectos sociales que han surgido en
los últimos años del siglo.

-Pág.87-
CIENCIAS SOCIALES. GEOGRAFÍA E HISTORIA

M A PA C O N C E P T UA L

ARTE, CULTURA Y SOCIEDAD

Cubismo

Arte abstracto

ARTE FIGURATIVO Surrealismo


VS.
ARTE ABSTRACTO Pop Art

Otras corrientes

Corrientes más recientes

ARQUITECTURA
Y URBANISMO

MANIFESTACIONES Cine
ARTÍSTICAS
ACTUALES Música

DESARROLLO CIENTÍFICO
Y TECNOLÓGICO

MEDIOS DE
COMUNICACIÓN

NUEVOS
VALORES

-Pág.88-
U . D . 9 . - A RT E , C U LT U R A Y S O C I E D A D E N E L M U N D O A C T U A L

LA CRISIS DEL ARTE FIGURATIVO EN PINTURA


1. Y ESCULTURA A PARTIR DEL IMPRESIONISMO
La historia del arte ha contemplado la rivalidad, que ha supuesto una dualidad estética y con-
ceptual, entre el figurativismo y la abstracción.

En primer término, conviene que demos una definición, lo más sencilla posible, de ambos
conceptos y nada mejor que voces autorizadas por su propia experiencia para hacerlo:

– El arte figurativo “no puede consistir más que en la representación de las cosas reales
y existentes” (Gustave Courbert, pintor realista francés del siglo XIX).
– El arte abstracto es el que “no contiene ningún recuerdo, ninguna evocación de la rea-
lidad, independientemente de que la realidad sea o no el punto de partida del artista”
(Michel Seuphor).

Efectivamente, ya en las primeras manifestaciones artísticas paleolíticas, tanto pintadas


como esculpidas o modeladas, nos encontramos con formas reproductoras de la realidad mate-
rial circundante y con otras, cuya interpretación resulta difícil, en las que la geometría y la abs-
tracción son protagonistas.
En las etapas postpaleolíticas continuó esta dualidad, ahora con soportes y aspectos dife-
rentes y continuó incluso en las culturas más radicalmente figurativas, predominantes a partir
de la época clásica. Así es, en Grecia y Roma, la representación de la realidad triunfó frente a
la abstracción y ese triunfo perduró en la cultura europea hasta el siglo XX.
La cultura próxima en la que más evidente es el triunfo de la abstracción es, sin lugar a
dudas, la musulmana. En ella, las razones religiosas, que prohiben la representación de la figu-
ra humana y animal con el objeto de evitar la idolatría, han conducido a ese triunfo en el que
las figuras geométricas, la caligrafía y las formas vegetales se convierten en el motivo decora-
tivo por excelencia.
La crisis del predominio absoluto del arte figurativo en la cultura occidental comienza a
plantearse en el siglo XIX al surgir movimientos en los que las figuras extraídas de la realidad
siguen siendo el objeto de representación pero la técnica y el tema comienzan a huir de la trans-
cripción del exterior para dar paso a la expresión del interior del autor. Así sucede, por ejemplo,
con pinturas de Goya en las que las figuras reales, estilizadas, sirven para manifestar símbolos
o escenas de carácter onírico.
Pero el punto que marca la inflexión definitiva, con la aparición arrolladora de la abstrac-
ción, va a ser el comienzo del siglo XX, con la figura cumbre de este proceso: Kandinsky. Efec-
tivamente, Kandinsky pasa por ser el precursor del arte abstracto. Un día de 1908 el autor de
origen ruso decía que “la objetividad, la descripción de objetos, no era necesaria en” sus “pin-
turas y, en realidad las perjudicaba”.

5
-Pág.89-
CIENCIAS SOCIALES. GEOGRAFÍA E HISTORIA

Kandinsky pensaba que la renovación del arte debía producirse por la victoria del irracio-
nalismo sobre el racionalismo que caracterizaba a la cultura occidental y que el color y la forma,
sin propósitos representativos, podían ser articulados en un lenguaje simbólico, llegando a la
conclusión de que las formas plásticas podían dar forma externa a una necesidad de expresión
interna.

1.1. EVOLUCIÓN DE LA PINTURA Y LA ESCULTURA DURANTE


EL SIGLO XX

1.1.1. EL CUBISMO
El cubismo va a suponer la ruptura del espacio clásico en el arte. El término parece que fue
utilizado por primera vez por el crítico parisino Louis Vauxcelles para calificar despectivamen-
te las pinturas de Picasso y Braque.

La obra considerada como iniciadora del cubismo es la de Las señoritas de Aviñón de Picas-
so (1906-1907). La solución pictórica aportada por Picasso consiste en modelar el volumen
mediante cambios de colores puros y violentos, esquematizando las figuras que se descompo-
nían en formas geométricas superpuestas. Además, la corriente así iniciada suponía también la
ruptura con la perspectiva clásica y conducía a la plasmación de planos distintos de las mismas
figuras, representando en un sólo plano las visiones de la misma desde distinto ángulos.

Otros pintores cubistas son, además de Pablo Ruiz Picasso y Georges Braque, Albert
Gleizes, Juan Gris, Robert Delaunay, Fernand Légerd, Francis Picabia o Marcel Duchamp.

1.1.2. EL ARTE ABSTRACTO


Tras Kandinsky, representante de lo que se dio en llamar expresionismo abstracto, surgió
otra corriente abstracta, con Piet Mondrian y Málevitch como precursores, a la que se denomi-
na racionalismo geométrico.

Otras corrientes abstractas anteriores a la II Guerra Mundial fueron:

– La iniciada en la Bauhaus, centro pedagógico y experimental de las artes fundada por


Walter Gropius en 1919 y basada en la utilización de círculos, rectángulos, triángulos,
segmentos circulares, etc. A ella se sumaron, entre otros, el propio Kandinsky, Klee, Van
Doesburg, Breuer…

– El dadaísmo, iniciado en Zürich y caracterizado por un delirio del absurdo, estuvo


representado por Tristan Tzara, Picabia, Jean Arp, etc.

6
-Pág.90-
U . D . 9 . - A RT E , C U LT U R A Y S O C I E D A D E N E L M U N D O A C T U A L

Después de la II Guerra Mundial surgió una nueva corriente en la línea del expresionismo
abstracto de Kandinsky, llamada también arte informal, que se basaba en la improvisación, en
la velocidad de trazado y en la intuición.

Se definieron dos áreas de desarrollo del expresionismo abstracto:

— En Nueva York, trabajan autores como Mondrian, Marcel Duchamp, Max Ernst, Marc
Chagall e Yves Tanguy, y allí surgirán dos tendencias dentro del expresionismo:
– La Action painting, con autores como Hans Hofman, Jackson Pollock, Willem de
Kooning o Franz Josef Kline.
– La tendencia de los planos cromáticos con Mark Rothko como máximo represen-
tante.

— En Europa, con autores como los franceses Jean Dubauffet y Georges Mathieu, el espa-
ñol Antoni Tàpies o el canadiense Jean-Paul Riopelle, se forman también dos tendencias:

– El tachismo.
– El arte informal, basado en la improvisación, en la velocidad de trazado y en la
intuición.

A partir de los años 60 del siglo XX comenzó a desarrollarse una nueva tendencia del arte
abstracto basada en la experimentación de los procesos ópticos y psicológicos de la percepción.
Sus seguidores se han dedicado a experimentar con secuencias, ritmos y cambios de imágenes
ópticas, creando efectos ópticos tridimensionales en la pintura o utilizando el movimiento real
con cambios de luces y motores en la escultura. Esta nueva corriente se ha denominado arte
cinético y también Op Art. Destacan en ella autores como Joseph Albers, Yaacov Agam, Vic-
tor Vasarely, J.R. Soto o Bridget Riley.

1.1.3. EL SURREALISMO

Surgido sobre conceptos filosóficos y psicológicos enraizados con la obra de Freud, fue teo-
rizado desde 1919 por André Breton que publicó en 1924 su Manifiesto en el que decía que “el
surrealismo es automatismo psíquico puro, mediante el cual nos proponemos expresar, bien sea
verbalmente, bien por escrito o en otras formas, el funcionamiento real del pensamiento en
ausencia de cualquier control ejercido por la razón, más allá de toda preocupación estética y
moral”. Max Ernst, Yves Tanguy, René Magritte y, sobre todo, Paul Klee, Salvador Dalí y Joan
Miró, son los máximos representantes del surrealismo.

7
-Pág.91-
CIENCIAS SOCIALES. GEOGRAFÍA E HISTORIA

1.1.4. EL POP ART

Aparece a finales de los 50 como reacción al cansancio experimentado, sobre todo por el
público norteamericano, respecto al arte abstracto. Esta corriente se vuelca sobre imágenes
publicitarias y se caracteriza por su tono festivo y divertido. Destacan Robert Rauschenberg,
Roy Lichtenstein, Tom Wesselmann, James Rosenquist, Andy Warhol, etc.

1.1.5. OTRAS CORRIENTES DEL SIGLO


Además de las citadas y dejando para el apartado siguiente las más recientes, podemos citar
como otras corrientes artísticas del siglo XX:
– Fauvismo, de corta duración y caracterizado por el empleo de colores irreales y violen-
tos, su principal representante fue Henri Matisse.
– Expresionismo, movimiento artístico caracterizado por el afán de expresar los senti-
mientos del autor más allá de la representación de la realidad. Entre sus primeros repre-
sentantes se encuentra Ernst Ludwig Kirchner. Terminó desembocando en el
expresionismo abstracto.
– Futurismo, de principios de siglo, centrado en la temática contemporánea, las máquinas
y el movimiento.
– Neoplasticismo, iniciada y representada básicamente por Piet Mondrian, se basa en la
utilización de colores primarios y formas planas.

1.1.6. CORRIENTES MÁS RECIENTES


Entre las corrientes artísticas desarrolladas en el último cuarto del siglo XX destacan:
– Pintura matérica, representada por Antoni Tàpies y caracterizada por su penetración
intuitiva en la memoria y el psiquismo humano.
– Arte pobre, que busca un lenguaje emotivo con materiales que pueden ser considerados
tecnológicamente pobres en un mundo tecnológicamente rico.
– Happenings, realmente acontecimientos – espectáculos en los que la acción, con parti-
cipación del espectador son protagonistas.
– Land art o Earth art, que implica la modificación leve o amplia de la estética natural
(pintura en árboles o rocas, modelado del hielo natural, etc.).
– Body art, que utiliza el propio cuerpo humano como elemento artístico.
Finalmente, en los últimos años del siglo han surgido algunas corrientes neofigurativas
como el neoexpresionismo o el hiperrealismo.

8
-Pág.92-
U . D . 9 . - A RT E , C U LT U R A Y S O C I E D A D E N E L M U N D O A C T U A L

ARQUITECTURA FUNCIONAL
2. Y URBANISMO ACTUAL
La Ingeniería y la Arquitectura contemporáneas han evolucionado hacia la funcionalidad.

En la Arquitectura, el hormigón armado y el acero, además del aluminio y el cristal, se han


convertido en los materiales predominantes. Los retos a la gravedad se ponen de manifiesto en
construcciones en las que el estudio de la resistencia de los materiales permite alcanzar alturas
insospechadas o inclinaciones y formas arriesgadas.

Las obras de ingeniería han ido superando grandes retos. Las grandes presas (Assuán), los
canales (Panamá, Suez), los túneles para ferrocarriles y carreteras, los grandes puentes y, recien-
temente el túnel bajo el Canal de la Mancha.

En las ciudades, después del crecimiento experimentado desde el siglo XVIII, se han apli-
cado diversas teorías urbanísticas. De los trazados hipodámicos (calles paralelas y perpendicu-
lares en forma reticular) se ha pasado a los diseños concéntricos, irregulares, etc. En algunas
ciudades se ha redistribuido el espacio urbano, en otras se ha creado, siendo frecuente la estruc-
tura de un centro dedicado a los negocios y un exterior ocupado por viviendas. Hoy día la ciu-
dad se ha convertido en un espacio geográfico específico, en el que impera la circulación rodada
a la que se adaptan las vías de comunicación.

El crecimiento imparable de las ciudades ha posibilitado la aproximación y unión de los


espacios urbanos vecinos formándose las llamadas conurbaciones o megalópolis (Costa Este de
EE.UU, Cuenca del Ruhr, área de Méjico DF, área de Tokio), cuyo crecimiento se prevé conti-
núe en los próximos años hasta formar grandes espacios urbanos.

FORMAS DE EXPRESIÓN Y MANIFESTACIONES


3. ARTÍSTICAS ACTUALES A TRAVÉS DE NUEVOS
LENGUAJES (VISUAL, PLÁSTICO, MUSICAL…)
El arte contemporáneo, además de las manifestaciones tradicionales de la pintura, la escul-
tura y la arquitectura que hemos analizado en los apartados anteriores, se ha desarrollado en
otras muchas parcelas que afectan a las artes decorativas y funcionales (muebles, objetos coti-
dianos, decoración de interiores), a la música y al cine. Detengámonos brevemente en las dos
últimas.

9
-Pág.93-
CIENCIAS SOCIALES. GEOGRAFÍA E HISTORIA

3.1. EL CINE
Desde su aparición en 1895, gracias a la iniciativa de lo hermanos Lumière, el cine ha pasa-
do de ser un simple entretenimiento a una forma de manifestación o expresión artística. Inten-
tando hacer un breve resumen de la historia del cine, dintinguiremos los siguientes apartados:

Etapa del cine mudo


Durante la misma, la expresividad necesaria por la ausencia de diálogos dominaba el carác-
ter de sus manifestaciones. Es la época de la alocada comedia de Mack Sennett, de las tragicó-
micas alegorías de Charles Chaplin (La quimera del oro, Tiempos modernos) o del
grandilocuente dramatismo de D.W. Griffith (El nacimiento de una nación).

Arte e industria
Ya desde estos primeros pasos se produjo la disociación entre dos tendencias creativas. Por
una parte, aquella en la que la motivación principal del cineasta era la creación de una obra de
entretenimiento de la que obtener beneficios económicos, lo que conduce al surgimiento paula-
tino de la gran industria del cine, radicada principalmente en los Estados Unidos. Por otra parte,
sobre todo en Europa, se desarrolla un cine de autor concebido como obra artística que, por ello
mismo, se adscribe con frecuencia a movimientos culturales específicos.

Inicio del Cine sonoro


En 1927, la productora Warner Bros, lanzó la primera película sonora: El cantor de jazz.
Aunque después siguieron realizándose películas mudas, a partir de ese momento el cine expe-
rimentó una completa revolución: el diálogo y los efectos sonoros pasaron a ser un elemento
más a tener en cuenta en la elaboración del producto final. Además, un nuevo género se abría a
las cámaras y los micrófonos cinematográficos: el cine musical.

El crecimiento de la industria
A partir de los años treinta el crecimiento del cine norteamericano condujo a la aparición de
grandes producciones y de estrellas de renombre (Lo que el viento se llevó), alcanzando así el
cine una mayoría de edad a partir de la cual madurar en diversos estilos y géneros.

Mientras este crecimiento tenía lugar en Estados Unidos, en Europa el cine seguía siendo
una manifestación artística de autor, menos grandilocuente y con menos aspiraciones de bene-
ficio. Además, las especiales circunstancias de la historia europea entre los años 30 y la pos-
guerra, hicieron del cine una fórmula más de manifestación ideológica política.

10
-Pág.94-
U . D . 9 . - A RT E , C U LT U R A Y S O C I E D A D E N E L M U N D O A C T U A L

Innovaciones técnicas
Después del sonido, el cine fue desarrollándose ampliando sus posibilidades técnicas con
sucesivos y paulatinos avances. Así, en 1935 se estrenaba la primera película en color, La feria
de las vanidades de Rouben Mamoulian. Posteriormente, en 1953, la productora Twentieth Cen-
tury Fox estrenó La túnica sagrada, primera película en cinemascope. Inmediatamente después
aparecen otras mejoras técnicas tendentes a ampliar el campo visual tomado por la cámara y, en
consecuencia, proyectado (Todd-AO, Panavisión). Apareció también el cine 3D, de éxito limi-
tado por la necesidad de usar gafas especiales. Con posterioridad, el sonido recibió especial
atención tecnológica, aportando sistemas de grabación y reproducción como la estereofonía,
cuadrafonía, dolby, etc. Además, en el proceso de producción se han ido incorporando avances
considerables en los efectos especiales, que aumentan el carácter realmente creativo del cine.

El cine español
El arte cinematográfico arranca en España dos años después de su invención por los herma-
nos Lumière. Efectivamente, en 1897 E. Jimeno rueda la primera película, al estilo de los
Lumière: Salida de la Misa de doce del Pilar de Zaragoza.

Antes de la Guerra Civil el desarrollo del cine es exiguo, el enorme potencial creativo que
se aprecia en lo poco que se rueda choca con la escasez de recursos. Las, aproximadamente,
veinte productoras existentes financian las obras mudas de Ricardo de Baños, Francisco Gómez
Hidalgo y Florián Rey y, a partir de 1929, las sonoras del propio Florián Rey y de Luis Buñuel
y Benito Perojo.

Durante la Dictadura de Franco destaca la obra de exaltación del nacionalismo español lle-
vada a cabo por Luis Sáenz de Heredia, en películas como Raza (1941), con guión del propio
Franco, o por Juan de Orduña, con cintas como Locura de Amor (1948) o Alba de América
(1951). Además, sobresalen las obras de Luis García Berlanga (Bienvenido Mister Marshall,
1952), Juan Antonio Bardem (Muerte de un ciclista), Ladislao Vajda y Edgar Neville. También
surgen, en los últimos años del franquismo, directores llamados a brillar con luz propia en eta-
pas posteriores; tal es el caso de Carlos Saura, Pilar Miró o Víctor Erice.

Los últimos veinticinco años del siglo han contemplado un desarrollo brillante del cine espa-
ñol. Éste se ha debido, no sólo al ingenio de sus autores, sino también a la financiación oficial
y de empresas privadas (sobre todo cadenas de televisión) que han hecho posible su puesta en
celuloide y su exhibición. Gracias a ello, el cine español ha alcanzado el reconocimiento inter-
nacional, certificándolo con la consecución de tres premios Óscar a la mejor película en lengua
extranjera, logrados por señeros directores de la filmografía española contemporánea: José Luis
Garci (Volver a empezar, 1982), Fernando Trueba (Belle Époque, 1992) y Pedro Almodóvar
(Todo sobre mi madre, 1999).

11
-Pág.95-
CIENCIAS SOCIALES. GEOGRAFÍA E HISTORIA

Además de los citados, destacan en dicho periodo nombres como José Luis Cuerda, Manuel
Gutiérrez Aragón, Gonzalo Suárez, Imanol Uribe, Vicente Aranda, Fernando Colomo, Bigas
Luna, Álex de la Iglesia, Juanma Bajo Ulloa, Alejandro Amenábar…

3.2. MÚSICA
La que viene en definirse como música clásica, durante el siglo XX ha experimentado varia-
ciones radicales respecto a la tradicional concepción del ritmo, la cadencia, la tonalidad. La rup-
tura de la tonalidad se había iniciado de algún modo con Wagner, mas encuentra su completa
definición en el siglo XX. De este resultado atonal derivan movimientos como el serialismo.
Además, se introducen otros sistemas como:
– Los efectos vocales secundarios.
– El percusionismo.
– Experiencias electrónicas.
– Música concreta (que utiliza todos los sonidos percibibles por el hombre, aunque no sean
musicales).
– Ruidos manipulados en laboratorio.
– Microtonalismo.
– Aleatoriedad controlada.
En España, los principales autores son Isaac Albéniz (Iberia), Manuel de Falla (Noches en
los jardines de España, El amor brujo), Joaquín Turina (Sinfonía Sevillana), Enrique Granados
(Goyescas), Joaquín Rodrigo (Concierto de Aranjuez), Vicente Arregui, Moreno Torroba,
Ernesto Halffter, José Subirá, Pau Casals, Andrés Segovia…
Sin embargo, el siglo XX se ha caracterizado por el crecimiento y desarrollo de la llamada
música popular o música pop, que ha tenido en el rock and roll su principal manifestación. Sur-
gido en los años 50 en Estados Unidos, se ha convertido, con sus diversas variantes, en la for-
mulación predominante de la música del siglo XX.
Antes de su aparición, las manifestaciones populares de la música habían dado lugar a movi-
mientos como el blues, el gospel y el jazz; manifestaciones del peculiar ritmo de la comunidad
negra. Este ritmo de la minoría negra cuajó finalmente en el rythm and blues que en combina-
ción con el country and western blanco terminarían por dar lugar al rock and roll.
Los primeros constructores del rock and roll fueron figuras inmortales como Elvis Presley o
Chuck Berry. Desde ellos, su crecimiento fue imparable, desbordando las fronteras norteamerica-
nas para alcanzar Europa y producir aquí otras figuras esenciales en la historia de la música pop.
Tal es el caso del grupo musical más famoso e influyente, de origen británico, The Beatles.
Las variantes del rock and roll y otras corrientes, entre las que podemos citar el soul, folk, heavy,
reggae, rock sinfónico, punk, funky, tecno, rapper, han definido el final musical del siglo XX.

12
-Pág.96-
U . D . 9 . - A RT E , C U LT U R A Y S O C I E D A D E N E L M U N D O A C T U A L

4. LOS RETOS DEL DESARROLLO CIENTÍFICO


Y TECNOLÓGICO. PAPEL Y SENTIDO
DE LAS HUMANIDADES EN LA ACTUALIDAD

4.1. LOS AVANCES CIENTÍFICOS


El conocimiento de los principios de la vida y del ser humano han permitido impresionan-
tes avances médicos: El conocimiento de los procesos neurológicos, el mecanismo de la heren-
cia genética, los avances en el conocimiento del genoma humano, el control de las
enfermedades infecciosas, los progresos de la psicología, las técnicas quirúrgicas y de microci-
rugía, los trasplantes de órganos..., han permitido la erradicación de determinadas enfermeda-
des mortales en el pasado, el aumento de la esperanza de vida y el avance en la prevención y
combate contra otras enfermedades. Además, la tecnología, puesta al servicio de la medicina
(aparatos de medición, análisis, control...) ha mejorado las condiciones de ese combate. Hoy en
día enfermedades como el SIDA o el cáncer son los grandes retos para la medicina.

La ciencia también se ha puesto al servicio de la mejora de las producciones agrarias y gana-


deras, gracias a los avances biogenéticos, que permiten la selección de especies, y químicos, que
posibilitan el empleo de piensos y fertilizantes más eficaces.

4.2. EL DESARROLLO TECNOLÓGICO


La tecnología al servicio de las comunicaciones, los transportes, los servicios y el ocio ha
experimentado avances impresionantes.

La radiodifusión (Marconi), creció desde sus primeros pasos a finales del s. XIX, hasta la
explosión de la radiofonía comercial y la más reciente de la radiotelefonía.

La televisión (John L. Baird), progresó hasta la TV en color y camina hacia la Alta Defini-
ción y la TV interactiva. La difusión de las señales dio un paso decisivo con la utilización de
los satélites orbitales que permiten hoy el enlace global del planeta.

Los sistemas modernos de telefonía, fax, videotexto, se unen a la construcción de sistemas


de comunicación interpersonal cada vez más eficaces.

Los ordenadores personales, tras los incipientes pasos de memoria artificial con computa-
doras de gran tamaño y lentitud de proceso, permiten en la actualidad incluso su transporte
manual y llevan a la integración de sistemas electrónicos complejos y a enormes posibilidades
de proceso de datos, generación de gráficos, etc.

13
-Pág.97-
CIENCIAS SOCIALES. GEOGRAFÍA E HISTORIA

Los electrodomésticos facilitan las labores de mantenimiento del hogar y sus comodidades
(lavadoras, frigoríficos, aire acondicionado, microondas, hornos eléctricos, etc.).

Los avances en los ferrocarriles, tendidos férreos electrificados, nuevos sistemas de sus-
pensión y articulación han aumentado la velocidad en la comunicación (trenes de alta velocidad
que alcanzan los 300 km/h).

El transporte terrestre ha completado su desarrollo con vehículos automóviles cada vez


más rápidos y seguros, tanto para el transporte de personas como de mercancías. La evolución
de los motores va por el camino de la reducción de los consumos y la contaminación.

La gran innovación del siglo XX en los transportes es sin duda el avión, que ha evolucio-
nado hacia el aumento de la velocidad, la comodidad y la capacidad.

Los avances tecnológicos y los retos del conocimiento científico posibilitaron el inicio y el
posterior desarrollo de la gran aventura iniciada por el hombre desde mediados del siglo: la con-
quista del espacio. Desde el lanzamiento del primer satélite artificial, el Spútnik I soviético,
en 1957, la carrera protagonizada por los EE.UU. y la URSS, bajo el aliento de la guerra fría,
llevó al hombre a investigar los planetas por medio de sondas automáticas y a salir él mismo al
espacio (Juri Gagarin, primer astronauta). La inicial ventaja soviética en esta carrera presionó
la conciencia y el orgullo norteamericanos y llevó a su presidente John F. Kennedy a plantear
el gran reto: pisar la Luna. Enormes recursos tecnológicos y económicos se pusieron al ser-
vicio de la empresa, que culminó el 20 de julio de 1969 cuando Neil Armstrong puso su pie
sobre la superficie de nuestro satélite. desde entonces hasta hoy los proyectos se han centra-
do en diversos estudios científicos en condiciones de ingravidez y en la puesta a punto de sis-
temas reutilizables (lanzadera espacial norteamericana) y bases orbitales (Skylab, Mir y
recientemente la estación internacional Alfa). El objetivo futuro es la conquista de nuevos pla-
netas.

5. REDES Y MEDIOS DE COMUNICACIÓN


E INFORMACIÓN
Las comunicaciones han adquirido una importancia trascendental en la sociedad contempo-
ránea. Tanto la orientada a la transmisión de información como la dedicada al entretenimiento,
su desarrollo se ha disparado en sentido exponencial.

La edición de revistas y periódicos, se ha visto favorecida por los nuevos sistemas de com-
posición. La proliferación de ediciones generalistas o especializadas da buena muestra del des-
arrollo de esta parcela de la comunicación.

14
-Pág.98-
U . D . 9 . - A RT E , C U LT U R A Y S O C I E D A D E N E L M U N D O A C T U A L

Las agencias de noticias controlan la difusión mundial de información, sirviendo a periódi-


cos, revistas, radio y televisión material gráfico y de texto.

La publicidad es una actividad de gran desarrollo que genera un importante movimiento de


capitales. Las agencias de publicidad elaboran las campañas o simplemente diseñan anuncios y,
junto a agentes libres, gestionan la contratación de paquetes publicitarios en periódicos, revis-
tas, radio, televisión. carteles, etc. Vital para la difusión entre los consumidores del conoci-
miento de los productos, todas las empresas dedican una parte importante de sus presupuestos
a la misma.

La fotografía es una actividad complementaria de las anteriores y de otras actividades con


una personalidad específica.

La radio, ha pasado a jugar un papel muy específico en la comunicación, orientado a la


información inmediata y al entretenimiento musical en circunstancias en las que no compite con
el gran “monstruo” de la comunicación de hoy, la televisión.

La televisión se ha convertido en el principal medio de comunicación. Información y entre-


tenimiento están casi dominados por completo por la llamada “caja tonta” que ha ido exten-
diendo desde sus orígenes sus tentáculos, abarcando cada vez un mayor ámbito geográfico de
difusión y un horario total, buscando enganchar al mayor número de potenciales televidentes y
convirtiéndose en un miembro más de la familia. Además, se ha producido la introducción en
el mercado de la televisión digital, que, sobre la base de la difusión vía satélite traspasa las fron-
teras y limita las posibilidades de intervención del poder público a la legislación sobre la venta
de aparatos conversores de la señal digital.

La comunicación por ordenador se ha convertido en otro de los grandes “monstruos” del


final del milenio. El crecimiento de Internet conduce a la consolidación de la idea de la “aldea
global”.

El teléfono ha experimentado un fuerte crecimiento en los últimos años, con la incorpora-


ción de la tecnología digital, la apertura del mercado y la telefonía móvil.

El telégrafo, telefax, videotexto, completan los sistemas de intercomunicación, cada vez más
integrados.

15
-Pág.99-
CIENCIAS SOCIALES. GEOGRAFÍA E HISTORIA

FOCOS DE TENSIÓN Y NUEVOS VALORES


6. EN LAS SOCIEDADES POSTINDUSTRIALES
La sociedad de hoy aparece como el fruto de los avances tecnológicos y científicos del siglo.
La caída del telón de acero con la desaparición de los sistemas comunistas que imperaron en el
Este europeo, ha sido el gran acontecimiento del final del siglo y ha recompuesto el sistema de
potencias. La URSS se ha desintegrado y Rusia, como heredera de su potencial, se ha sumido
en una profunda crisis institucional y económica. El mapa europeo se ha recompuesto con la
división de Checoslovaquia en dos Estados y la partición de Yugoslavia, una división violenta,
que se ha convertido en una de las principales espinas de la sociedad internacional.

Las Naciones Unidas han visto crecido su papel al desaparecer la rivalidad casi incondicio-
nal entre las dos superpotencias de la posguerra.

A pesar de los buenos augurios que supuso la desaparición de los bloques, el Mundo se
enfrenta a grandes retos y conflictos.

La solución de la crisis consiguiente a la disolución del “imperio” soviético y del conflicto


yugoslavo, junto con la construcción de la Unión Europea, son los principales retos europeos.

La problemática generada por el radicalismo del integrismo islámico es otro factor impor-
tante que puede desequilibrar la estabilidad mundial.

A corto y medio plazo la resolución de estos problemas es fundamental para la consolida-


ción de un mundo próspero y en paz. Pero, sin lugar a dudas, quedan aún otros retos trascen-
dentalísimos: acabar con el hambre y el atraso cultural del Tercer Mundo y controlar el
crecimiento demográfico.

Por otra parte, el control del medio ambiente, la protección de la naturaleza y las especies
animales y la búsqueda de fuentes de energía eficaces y limpias son otros retos inexcusables de
la sociedad humana.

A estos retos se une el del control de las nuevas enfermedades, como el SIDA o las que afec-
tan o pueden afectar a la cadena alimentaria.

Además, la sociedad de hoy se enfrenta a una reordenación de las relaciones interpersona-


les, generada por diversos factores entre los que cuenta la modificación de los roles tradicio-
nalmente adjudicados al hombre y la mujer. La intervención creciente de la mujer en el mundo
laboral, su independencia económica, se une al aumento de las relaciones matrimoniales rotas
y reconstruidas que incorporan nuevos conceptos sobre los vínculos familiares.

16
-Pág.100-
U . D . 9 . - A RT E , C U LT U R A Y S O C I E D A D E N E L M U N D O A C T U A L

RESUMEN
— El Arte del siglo XX, en pintura y escultura se ha caracterizado por el predominio del
arte abstracto sobre el arte figurativo.
— Se considera precursor del arte abstracto a Kandinsky.
— Las corrientes artísticas más importantes en pintura y escultura han sido:
– El cubismo.
– El arte abstracto (racionalismo geométrico, Bauhaus, dadaísmo, action painting,
planos cromáticos, tachismo, arte informal, Op art...).
– El surrealismo.
– El Pop art.
– Otras (fauvismo, expresionismo, futurismo, neoplasticismo).
– Y las más recientes (Pintura matérica, Arte pobre, Happenings, Land Art, Body art,
neoexpresionismo, hiperrealismo...
— La arquitectura y el urbanismo modernos han evolucionado hacia la funcionalidad.
— Entre las formas de expresión artística del siglo XX han destacado el cine y la música.
— El cine dio sus primeros pasos en 1895 gracias a los hermanos Lumière, pasando por
los siguientes hitos:
– Cine mudo, entre la comedia y el drama.
– Inicio del cine sonoro (1927, El cantor de jazz).
– Crecimiento como industria.
– Innovaciones técnicas: Color (1935, La feria de las vanidades), Cinemascope (1953,
La túnica sagrada), Todd-AO, Panavisión, 3D.
— El cine español ha tenido tres momentos principales:
– Antes de la Guerra Civil, de enorme potencial creativo y escasos medios.
– Durante la Dictadura de Franco.
– Los últimos 25 años, con un crecimiento extraordinario y el reconocimiento inter-
nacional.
— La música clásica contemporánea ha contemplado la aparición de movimientos inno-
vadores caracterizados por la ruptura de la tonalidad.
— La música contemporánea ha tenido como protagonista al rock and roll.
— Los avances científicos y el desarrollo tecnológico han modificado las perspectivas del
futuro de la Humanidad que se proyecta al espacio.
— Los medios de comunicación y las innovaciones tecnológicas aplicadas a ellos se han
desarrollado enormemente, avanzando hacia lo que se ha dado en llamar globalización.
— Los grandes retos sociales de la Humanidad son consecuencia de los cambios experi-
mentados.

17
-Pág.101-
CIENCIAS SOCIALES. GEOGRAFÍA E HISTORIA

EJERCICIOS DE AUTOCOMPROBACIÓN

1. ¿A quién se considera precursor del arte abstracto?

A. Picasso B. Kandinsky C. Mondrian D. Modigliani

2. ¿Quién es el autor de la obra que está considerada como iniciadora del cubismo?

A. Picasso B. Kandinsky C. Mondrian D. Modigliani

3. El llamado Op Art se caracteriza por:

A. Volcarse sobre imágenes publicitarias.

B. Los efectos tridimensionales y de movimiento.

C. Utilizar elementos de la naturaleza.

D. La búsqueda de la manifestación del interior del autor.

4. Una de las siguientes corrientes artísticas NO está vinculada al arte abstracto sino al figurativo

A. Neoplasticismo B. Action Painting C. Dadaísmo D. Neoexpresionismo

5. La primera película sonora fue:

A. La feria de las vanidades C. El cantor de jazz

B. Salida de la fábrica Lumière en París D. La túnica sagrada

6. El cine español cuenta con un reducido número de películas galardonadas con el Premio Oscar a la
mejor película en habla no inglesa, entre los autores de las mismas NO se encuentra:

A. José Luis Garci C. Fernando Trueba

B. Pedro Almodóvar D. Vicente Aranda

7. Se considera que el rock and roll es el fruto de la confluencia de dos estilos musicales precedentes.
¿Cuáles?

A. Blues y Funky C. Gospel y Soul

B. Country y Rythm & blues D. Raper y Punk

18
-Pág.102-
U . D . 9 . - A RT E , C U LT U R A Y S O C I E D A D E N E L M U N D O A C T U A L

8. ¿Cuál fue el primer satélite artificial?

A. Apolo I B. Vanguard I C. Viking I D. Sputnik I

9. Entre los autores españoles contemporáneos de música clásica se encuentra el compositor de El


amor brujo, se trata de:

A. Manuel de Falla C. Enrique Granados

B. Isaac Albéniz D. Pau Casals

10. Salvador Dalí figura entre los principales representantes de

A. El Pop Art B. El Dadísmo C. El Fauvismo D. El surrealismo

19
-Pág.103-
CIENCIAS SOCIALES. GEOGRAFÍA E HISTORIA

RESPUESTAS A LOS EJERCICIOS

11. B

12. A

13. B

14. D

15. C

16. D

17. B

18. D

19. A

10. D

20
-Pág.104-
portada TROPA 19/3/07 19:51 Página 1

FUERZAS ARMADAS
PROFESIONALES
CURSO DE APOYO
A LA PREPARACIÓN
DE LAS PRUEBAS DE ACCESO
A UNA RELACIÓN DE SERVICIOS
DE CARÁCTER PERMANENTE

LENGUA CASTELLANA Y LITERATURA


1ª parte
Unidades didácticas 1 y 2

DIGEREM

MINISTERIO
DE DEFENSA
FUERZAS ARMADAS SUBDIRECCIîN GENERAL
DE TROPA Y MARINERIA
PROFESIONAL
PROFESIONALES
CURSO DE APOYO
A LA PREPARACION
Ó
DE LAS PRUEBAS DE ACCESO
A UNA RELACIÓN DE SERVICIOS
DE CARÁCTER PERMANENTE

LENGUA CASTELLANA Y LITERATURA


1ª parte
Unidades didácticas 1 y 2

-Pág.1-
La Ley 8/2006 de Tropa y Marinería, en su artículo 16,1, establece que “la formación
en las Fuerzas Armadas garantizará que los militares profesionales de tropa y
marinería puedan adquirir, actualizar o ampliar sus conocimientos para un mayor
desarrollo personal y profesional”. En cumplimiento de este mandato, el Ministerio
de Defensa edita el presente material didáctico para facilitar a los militares
profesionales de tropa y marinería, alumnos de los cursos de formación
presencial que se imparten a través de la Dirección General de Reclutamiento y
Enseñanza Militar, los apoyos necesarios para preparación de dichos cursos, que
permitirán, siempre que superen las pruebas correspondientes, la obtención de la
titulación de graduado en Educación Secundaria, acreditación para el acceso a
los ciclos formativos de la Formación Profesional de grado medio o de grado
superior, acceso a las Escalas de Suboficiales, Tropa Permanente, Guardia Civil
y Policía Nacional.

CATÁLOGO GENERAL DE PUBLICACIONES


http://www.060.es

Edita:

© Autor y editor
NIPO: 076-10-204-9 NIPO: 076-10-205-4 (edición en línea)
Depósito Legal: M-32363-2009
Diseño y programación: cimapress
Tirada: 1300 ejemplares
Fecha de edición: septiembre, 2010

Prohibida la reproducción total o parcial de esta obra, por cualquier medio sin autorización escrita del editor
LENGUA CASTELLANA Y LITERATURA
1ª parte

SUMARIO

Unidad didáctica Pág.

1. USOS Y FORMAS DE LA COMUNICACIÓN ORAL 5

2. USOS Y FORMAS DE LA COMUNICACIÓN ESCRITA 33

-Pág.3-
U. D. 1 . - U S O S Y F ORMAS DE LA C O M U N I C AC I Ó N O R A L

ÍNDICE
Pag.
OBJETIVOS . . . . . . . . . . . . . . . . . . . . . . . . . . . . . . . . . . . . . . . . . . . . . . . . . . . . . . . . . . . . . . 2

INTRODUCCIÓN . . . . . . . . . . . . . . . . . . . . . . . . . . . . . . . . . . . . . . . . . . . . . . . . . . . . . . . . . 3

MAPA CONCEPTUAL . . . . . . . . . . . . . . . . . . . . . . . . . . . . . . . . . . . . . . . . . . . . . . . . . . . . . 4

DESARROLLO DE CONTENIDOS . . . . . . . . . . . . . . . . . . . . . . . . . . . . . . . . . . . . . . . . . 5

1. LA COMUNICACIÓN ORAL: ELEMENTOS Y FUNCIONES . . . . . . . . . . . . . 5


1.1. NECESIDADES DE COMUNICACIÓN E INTENCIÓN COMUNICATIVA:
ACTOS DE HABLA. . . . . . . . . . . . . . . . . . . . . . . . . . . . . . . . . . . . . . . . . . . . . . . . . 5
1.2. LA SITUACIÓN DE COMUNICACIÓN Y SUS ELEMENTOS . . . . . . . . . . . . . . 6
1.3. LA LENGUA ORAL COMO FUENTE DE INFORMACIÓN,
CONOCIMIENTO, PLACER, PERSUASIÓN, MANIPULACIÓN, … . . . . . . . . . 7
1.4. RASGOS DISTINTIVOS DE LA LENGUA ORAL: FONÉTICOS
PROSÓDICOS, PARALINGÜÍSTICOS, ETC. . . . . . . . . . . . . . . . . . . . . . . . . . . . . 8

2. TIPOS Y FORMAS DE DISCURSO EN LA COMUNICACIÓN ORAL . . . . 12


2.1. ESTRUCTURAS TEXTUALES BÁSICAS: LA NARRACIÓN, LA
DESCRIPCIÓN, LA EXPOSICIÓN, LA ARGUMENTACIÓN, … . . . . . . . . . . . 12
2.2. LA CONVERSACIÓN, EL COLOQUIO, EL DEBATE, LA ENTREVISTA, … . . . 14
2.3. REGISTRO Y USOS SOCIALES DE LA LENGUA ORAL . . . . . . . . . . . . . . . . 15
2.4. FINALIDAD, SITUACIÓN Y CONTEXTO COMUNICATIVO. . . . . . . . . . . . . . 17
2.5. JERGAS Y LENGUAJES PROFESIONALES . . . . . . . . . . . . . . . . . . . . . . . . . . . 18

3. DIVERSIDAD LINGÜÍSTICA Y VARIEDADES DIALECTALES DE LA


LENGUA ORAL . . . . . . . . . . . . . . . . . . . . . . . . . . . . . . . . . . . . . . . . . . . . . . . . . . . . . . . 18
3.1. LA REALIDAD PLURILINGÜE DE ESPAÑA . . . . . . . . . . . . . . . . . . . . . . . . . . 19
3.2. FENÓMENOS DE CONTACTO ENTRE LAS DISTINTAS LENGUAS . . . . . . 23
3.3. USO Y DIFUSIÓN INTERNACIONAL DEL ESPAÑOL Y DE LAS OTRAS
LENGUAS DE ESPAÑA . . . . . . . . . . . . . . . . . . . . . . . . . . . . . . . . . . . . . . . . . . . . 24

RESUMEN . . . . . . . . . . . . . . . . . . . . . . . . . . . . . . . . . . . . . . . . . . . . . . . . . . . . . . . . . . . . . . . 26

EJERCICIOS DE AUTOCOMPROBACIÓN . . . . . . . . . . . . . . . . . . . . . . . . . . . . . . . . 27

RESPUESTAS . . . . . . . . . . . . . . . . . . . . . . . . . . . . . . . . . . . . . . . . . . . . . . . . . . . . . . . . . . . . 28

-Pág.5-
L E N G UA Y L I T E R AT U R A

OBJETIVOS
Al finalizar el estudio de esta Unidad Didáctica, el alumno será capaz de:

• Reconocer los elementos y funciones de la comunicación oral.

• Diferenciar los diversos tipos y formas de discursos en la comunicación


oral.

• Valorar la realidad plurilingüe de España y la importancia del Español en el


mundo.

-Pág.6-
U. D. 1 . - U S O S Y F ORMAS DE LA C O M U N I C AC I Ó N O R A L

INTRODUCCIÓN
C omunicación es cualquier acto social mediante el cual un individuo (emisor)
cifra (produce) un mensaje y lo envía a otro (receptor) y éste lo descifra (com-
prende).

Cada especie animal, incluida la humana, tiene sus propias formas de transmitirse
información (sistemas de signos). Ejemplo:
Los leones mediante rugidos, melena, etc.
Los insectos, por sus colores, olores, movimientos, etc.
Los humanos a través del lenguaje, etc.
Incluso se da la comunicación entre individuos de distintas especies. Ejemplo:
La serpiente cascabel que hace sonar su cola como señal de aviso de peligro a
cualquier intruso que invade su territorio.
El pastor que mediante silbidos, voces y gestos dirige a sus perros para contro-
lar el ganado.
También se debe considerar la comunicación que se puede establecer entre personas
y máquinas. Ejemplo:
El despertador que nos despierta a una hora determinada.
El ordenador que ejecuta una serie de órdenes que le damos al presionar sobre
su teclado.

Las mismas máquinas pueden establecer comunicaciones entre sí. Ejemplo:


El termostato que pone en funcionamiento el motor de un frigorífico hasta con-
seguir la temperatura idónea.
La alarma que al saltar "ordena" que se bloqueen las puertas automáticamen-
te, etc.

En esta unidad nos referiremos única y exclusivamente a la comunicación oral, que


es la modalidad más utilizada universalmente, tanto por los grupos sociales que
conocen y practican la escritura, como por los que, desgraciadamente, la descono-
cen.

-Pág.7-
L E N G UA Y L I T E R AT U R A

M A PA C O N C E P T UA L
LENGUA
CASTELLANA Y
LITERATURA

LA
USOS Y FORMAS DIVERSIDAD
COMUNICACIÓN
DE LA COMUNI- LINGÜÍSTICA
CACIÓN ORAL VARIEDADES
DIALECTALES
DE LA LENGUA
ELEMENTOS ORAL
TIPOS Y FORMAS EN
LA
COMUNICACIÓN ORAL
FUNCIONES

ESTRUCTURAS

REGISTROS Y
USOS SOCIALES USO Y DIFUSIÓN DEL
ESPAÑOL
Y DE LAS OTRAS
LENGUAS DE ESPAÑA

LA REALIDAD PLURI-
LINGÜE DE ESPAÑA

-Pág.8-
U. D. 1 . - U S O S Y F ORMAS DE LA C O M U N I C AC I Ó N O R A L

LA COMUNICACIÓN ORAL:
1. ELEMENTOS Y FUNCIONES

Antes de entrar en los apartados de este bloque, creemos oportuno establecer la dife-
rencia entre LENGUAJE y LENGUA; conceptos que, a veces, aparecen utilizados erróne-
amente como sinónimos.

Denominamos "lenguaje" a la capacidad que tiene el hombre para comunicarse con los
demás a través de signos; esta capacidad de comunicación es común a todos los hombres
con independencia del lugar o grupo social en el que haya nacido cada uno, y si los signos,
que utiliza, son verbales (palabras, oraciones...etc.) se le denomina lenguaje verbal.

Denominamos "lengua" al conjunto de signos verbales (orales y escritos) y reglas que


forman un código que permite comunicarse a los miembros de una comunidad concreta.

El término lengua no está determinado por el número de hablantes; la misma conside-


ración debe tener la lengua que utiliza una comunidad de 100 millones de hablantes que la
lengua utilizada por un grupo formado por un millón de hablantes. En este sentido, nos
referimos por igual cuando mencionamos el español, el chino, el inglés, el bantú, el hoten-
tote, etc.

1.1. NECESIDADES DE COMUNICACIÓN E INTENCIÓN


COMUNICATIVA: ACTOS DE HABLA
Los hombres, como seres sociales, necesitan relacionarse entre sí para comunicarse lo
que piensan, sienten o desean.

Los componentes de un grupo humano comparten la misma lengua, un código común;


pero el uso que cada hablante hace de esa lengua, eligiendo determinadas palabras, con una
entonación concreta, con una construcción sintáctica adecuada a su necesidad comunicati-
va, es un acto individual y concreto que se llama "habla".

La lengua no tiene una realidad física, es inmaterial, sólo existe en nuestra mente, es
abstracta, la vamos aprendiendo progresivamente, nunca llegaremos a conocerla totalmen-
te ya que no es un código completamente cerrado, el vocabulario es cada vez más extenso,
aparecen nuevos giros de construcción..., pero seguirá siendo común para la comunidad.

TAMadrid
5
-Pág.9-
L E N G UA Y L I T E R AT U R A

En cambio, el habla sí tiene una realidad física, es material. Las palabras se oyen y son
el fruto de una actuación personal, individual, irrepetible; si un mismo hablante repitiese la
misma palabra u oración, se trataría de actos de habla diferentes.

Los actos de habla están relacionados con las necesidades potenciales de la comunicación
oral y la intención comunicativa del hablante como respuesta a las mismas. Los actos de habla
son infinitos, son tantos como "actuaciones habladas" pueden realizar los hablantes.

1.2. LA SITUACIÓN DE COMUNICACIÓN Y SUS ELEMENTOS


Hemos afirmado que el hombre es un ser social que necesita relacionarse con los demás
para comunicarse entre sí lo que piensan, sienten o desean. Esto nos permite calificar a la
comunicación de verdadero acto social, en el que como mínimo, han de participar dos per-
sonas; además deben intervenir otros elementos con sus correspondientes funciones, lo que
se conoce como el circuito de la comunicación oral, cuyo esquema podría ser:

CIRCUITO DE LA COMUNICACIÓN

CÓDIGO

EMISOR CANAL MENSAJE CANAL RECEPTOR


(HABLANTE) (OYENTE)
SITUACIÓN

CONTEXTO

EMISOR: El hablante que produce (cifra) el mensaje y lo envía.

RECEPTOR: El oyente que recibe el mensaje y lo descifra (comprende).

CÓDIGO: Conjunto de signos y reglas que deben conocer el emisor y el receptor (en
nuestro caso el código sería el conjunto de fonemas y reglas de la lengua castellana).

CANAL: Vía por la que circula el mensaje. En una conversación la vía es el aire; en una
conversación telefónica, a través de teléfonos fijos, la vía sería el cable telefónico, etc.

MENSAJE: La información que envía el emisor al receptor; lo produce siempre el emisor.

6 TAMadrid

-Pág.10-
U. D. 1 . - U S O S Y F ORMAS DE LA C O M U N I C AC I Ó N O R A L

CONTEXTO LINGÜÍSTICO: Conjunto de palabras que acompañan a un término dado, al


que complementan y aclaran su significado. Ej:
Trae un cubo de agua
Dos al cubo es igual a ocho

La palabra cubo tiene significado distinto si va relacionada con agua o con operaciones
matemáticas de potencias.

CONTEXTO SITUACIONAL: Conjunto de circunstancias extralingüísticas que ayudan a


comprender perfectamente los matices significativos de un mensaje. Ejemplo ¡Vaya
día!, refiriéndose a un día en el que el termómetro marca 40º, tiene matices semánticos
distintos si lo expresa un bañista en la playa (alegría, aprobación), o si lo expresa un
obrero que está asfaltando una carretera (molestia, desaprobación).

En la forma oral dialogada las funciones de emisor o receptor las desempeña quien habla
o escucha, en un momento determinado del diálogo, los dos (o más) interlocutores desem-
peñan ambas funciones.

Ejemplo práctico de comunicación oral:

Imaginemos un oficial que se dirige a la tropa de su unidad en un día de prácticas de


tiro; el circuito de comunicación sería el siguiente:
Emisor: El oficial que da las órdenes.
Receptor: La tropa que recibe las órdenes.
Código: El de la lengua que use.
Canal: El aire.
Mensaje: El contenido de las órdenes.
Contexto lingüístico: Las palabras relacionadas con la acción de tiro.
Contexto situacional: Campo de tiro, armas, etc.

1.3. LA LENGUA ORAL COMO FUENTE DE INFORMACIÓN,


CONOCIMIENTO, PLACER, PERSUASIÓN, MANIPULACIÓN...
La lengua oral es el sistema de comunicación que permite formar mensajes con una
mayor facilidad. Mediante la cadena hablada podemos encontrar respuestas a cualquier
necesidad potencial comunicativa; el lenguaje oral ofrece infinidad de funciones para tal
finalidad; veamos las más frecuentes:

• Informativa, representativa o referencial: Señala hechos objetivos, intenta informar.

TAMadrid
7
-Pág.11-
L E N G UA Y L I T E R AT U R A

Son las oraciones enunciativas: "Ha llegado Juan"


• Expresiva o emotiva: Señala la actividad subjetiva del hablante; el emisor expresa
sus sentimientos, emociones.. etc. Son las oraciones exclamativas: !Qué bien estás!
• Conativa o apelativa: El hablante intenta influir en la actitud del oyente, o bien , lla-
mar su atención. Suelen ser las oraciones exhortativas y los vocativos: " Juan, invi-
ta a tus compañeros."
• Fática o de contacto: Intenta comprobar que el canal de comunicación sigue abier-
to, que la comunicación no se ha interrumpido. Suelen ser expresiones cortas: "Sí...
sí, evidentemente... ya... etc.
• Lúdica: Intenta conseguir divertimento mediante juegos de palabras: chistes, dilo-
gías, refranes...etc.
• Poética o estética: Su finalidad es llamar la atención sobre cómo es la forma de
construcción del mensaje. Aparece, fundamentalmente, en la lengua literaria: " Las
perlas de tu boca".
• Metalingüística: Es la función del lenguaje para explicar sus propias reglas de ela-
boración: "El sustantivo concuerda en género y número con los determinantes que
lo acompañan".
• A veces en un mismo mensaje, pueden aparecer dos o más funciones. Ejemplo:

En la frase: ! Está lloviendo!, aparecen tres funciones:


• Representativa -> Informa
• Expresiva -> Sentimiento de sorpresa
• Conativa -> Sugiere la utilización de paraguas

1.4. RASGOS DISTINTIVOS DE LA LENGUA ORAL: FONÉTICOS,


PROSÓDICOS, PARALINGÜÍSTICOS...
Cuando nacemos, todos tenemos la misma capacidad para comunicarnos con los demás
mediante sonidos, pero cada comunidad ha seleccionado un número determinado de éstos,
los cuales, con infinitas combinaciones, forman los signos lingüísticos orales.

Estos sonidos específicos de cada lengua son los que en abstracto denominamos como
fonemas; el sonido se percibe, es material, tiene realidad física, el fonema es la imagen
mental del sonido, es inmaterial, se encuentra en nuestra mente, no tiene realidad física.

8 TAMadrid

-Pág.12-
U. D. 1 . - U S O S Y F ORMAS DE LA C O M U N I C AC I Ó N O R A L

Las lenguas no tienen el mismo número de fonemas, aunque suelen tener un número
aproximado de los mismos; en concreto, la lengua española (castellano) tiene 24 fonemas:
5 vocálicos y 19 consonánticos, que constituyen el sistema fonológico del español; se trata,
pues, de un sistema cerrado.

Del estudio de los sonidos se encarga la fonética ,y de los fonemas, la fonología; ambas
disciplinas se complementan perfectamente.

Cada fonema tiene sus propios rasgos distintivos que le permiten diferenciarse del resto
de fonemas, a saber:

• Rasgos distintivos de las vocales:

a) Según la abertura de la boca:


Cerradas: /i/, /u/
Semiabiertas: /e/, /o/
Abierta: /a/

b) Según el lugar de articulación:


Anteriores: /e/, /i/
Central o media: /a/
Posteriores: /o/, /u/

En el siguiente cuadro se puede observar más claramente:

i u mínima
e o abertura media
a máxima
localización
posterior
anterior

media

TAMadrid
9
-Pág.13-
L E N G UA Y L I T E R AT U R A

Rasgos distintivos de las consonantes:

a) Por el punto de articulación -lugar donde se juntan los órganos bucales para emitir un
sonido.

• Labiales (2 labios): /p/, /b/, /m/


• Labiodentales (labio inferior y dientes superiores): /f/
• Interdentales (lengua entre dientes superiores e inferiores): /z/
• Dentales (lengua y dientes superiores): /t/, /d/
• Alveolares (lengua y alvéolos superiores): /s/, /l/, /n/, /r/
• Palatales (lengua y paladar): /ch/, /y/, /ñ/, /ll/
• Velares /lengua y velo del paladar): /k/, /g/, /y, /j/

b) Por el modo de articulación - según la abertura que presente el canal, el aire se verá
forzado a abrirlo para emitir el sonido.
• Oclusivas: el canal vocal se encuentra cerrado y el aire debe abrirlo con un peque-
ño impulso: /p/, /k/, /d/, /t/
• Fricativas: el canal se estrecha y el aire sale rozando: /z/, /f/, /s/, etc.
• Africadas: el canal vocal está cerrado como en las oclusivas, pero no se abre de
golpe, sino suavemente, con lo que se produce un rozamiento: /ch/
• Vibrantes: la punta de la lengua toca los alvéolos de los dientes superiores y el aire
la hace vibrar: /r/, /rr/
• Laterales: el aire sale por ambos lados de la lengua: /l/, /ll/
• Nasales: parte del aire sale por la nariz: /m/, /n/ y /ñ/

c) Por la vibración de las cuerdas vocales: Según vibren o no las cuerdas vocales:
• Sordas: no vibran las cuerdas vocales /p/, /t/, /k/, /j/, /f/, /c^/, /z/, /s/
• Sonoras: vibran las cuerdas vocales. Todas las demás consonantes y las vocálicas.

10 TAMadrid

-Pág.14-
U. D. 1 . - U S O S Y F ORMAS DE LA C O M U N I C AC I Ó N O R A L

INTERVENCIÓN MODO DE
ARTICULACIÓN
LUGAR DE DE LAS

Oclusivos

Africados

Fricativos

Vibrantes
Laterales

Nasales
ARTICULACIÓN CUERDAS
VOCALES
Sordos p
Bilabiales
Sonoros b m
Sordos f
Labiodentales
Sonoros
Sordos z
Interdentales
Sonoros
Sordos t
Dentales
Sonoros d
Sordos s
Alveolares
Sonoros l r,/ r̄/ n
Sordos ^
/c/
Palatales
Sonoros y l / l/ ñ
Sordos ˘
k j
Velares
Sonoros g

Los fonemas son las unidades lingüísticas mínimas, que combinadas entre sí forman
unidades lingüísticas superiores (sílabas, palabras, oraciones...), con una pronunciación y
acentuación determinadas, de cuyo estudio y reglas se encarga la prosodia.

En toda palabra hay una sílaba que se pronuncia más fuerte que las demás, es la sílaba
tónica o dominante, las demás son sílabas átonas.

Dependiendo del lugar que ocupe la sílaba tónica, las palabras se clasifican en:
• Agudas u oxítonas, cuando la sílaba tónica es la última: Madrid, París, Camión.
• Graves, llanas o paroxítonas, cuando la sílaba tónica es la penúltima: madre, llana,
cárcel.
• Esdrújulas o proparoxítonas, cuando la sílaba tónica es la antepenúltima: médico,
último, Cáceres.
El acento permite diferenciar los significados de las palabras formadas por los mismos
fonemas: celebré - célebre - celebre. Canto - cantó
La entonación es otro rasgo prosódico que permite distinguir con precisión el significa-
do de una oración:

TAMadrid
11
-Pág.15-
L E N G UA Y L I T E R AT U R A

¿Tienes mucha suerte? Interrogación


¡Tienes mucha suerte! Exclamación
Tienes mucha suerte. Enunciación

Es obvio que el mensaje de estas oraciones es diferente por su entonación: Pregunta, sor-
presa, información.

También debemos tener en cuenta los rasgos paralingüísticos que intervienen en la


comunicación oral, ya que inciden decisivamente en su significado: las vacilaciones al
hablar, los sollozos, los jadeos..., que influyen en las inflexiones de la voz, aportando una
entonación peculiar.

TIPOS Y FORMAS DE DISCURSO


2. EN LA COMUNICACIÓN ORAL
En general, entendemos como discurso el texto oral encaminado a dar información sobre
cualquier aspecto, narrarlo, describirlo, exponerlo, argumentarlo... En sentido más restringi-
do, el discurso suele ser un texto argumentativo en el que se realiza una exposición interesa-
damente razonada para convencer a un auditorio: un sermón, una arenga, un mitin, etc.

En este bloque nos vamos a referir al discurso en su sentido general, si bien pormenori-
zaremos en sus diversos tipos y formas.

2.1. ESTRUCTURAS TEXTUALES BÁSICAS: LA NARRACIÓN,


LA DESCRIPCIÓN, LA EXPOSICIÓN, LA ARGUMENTACIÓN...
Es importante destacar que en este grupo de discursos el sentido de la comunicación es
unívoco, siempre va desde el emisor (hablante) hasta el receptor (auditorio), suele estar pre-
parado previamente, utiliza un vocabulario selecto y la sintaxis es muy cuidada, por lo que
podemos calificarlos de discursos formales.

LA NARRACIÓN.- Es el relato oral de hechos y acciones reales o ficticias que tiene como
finalidad responder a interrogantes como: ¿ Quién fue?, ¿Qué hizo? ¿Qué hace? ¿Qué ocu-
rrió? ¿ Qué ocurre?, etc. En toda narración deben aparecer los siguientes elementos:

• El ambiente: lugar, época...

• Los personajes: principales y secundarios.

12 TAMadrid

-Pág.16-
U. D. 1 . - U S O S Y F ORMAS DE LA C O M U N I C AC I Ó N O R A L

• La acción: gestas, hechos diversos.

En su estructura distinguimos tres partes:

• Introducción o presentación de la acción.


• Nudo o cuerpo central donde se desarrollan los hechos.
• Desenlace o solución, presenta la resolución de los hechos.

El uso del pretérito perfecto simple, el pretérito imperfecto y el presente histórico sue-
len ser frecuentes.

El texto narrativo es típico en el relato de cuentos, noticias, hechos históricos...

LA DESCRIPCIÓN.– Con el discurso descriptivo se intenta exponer cómo son las perso-
nas, animales, las cosas... explicando sus cualidades, características, etc.

En el caso de describir a las personas se denomina Retrato; si describe a la naturaleza,


Paisaje.

En la estructura de la descripción es importante el orden.

En la descripción el uso de los sustantivos y adjetivos es abundante, así como los ver-
bos en presente y pretérito imperfecto.

El texto descriptivo es típico en el lenguaje de la publicidad, en el repertorio de las guías


de museos...

LA EXPOSICIÓN.– Con el discurso expositivo se trata de explicar objetivamente cual-


quier información.

Si la claridad y la coherencia son importantes en cualquier discurso, en el caso de la


exposición lo son especialmente por su carácter didáctico.

Se apoyará en numerosos ejemplos que servirán para que el auditorio pueda seguir
mejor la explicación...El texto expositivo es típico en las conferencias, clases, charlas...

LA ARGUMENTACIÓN.– Es el tipo de discurso donde la intención del hablante es lo más


sobresaliente, ya que a través de él intenta convencer al auditorio.

La estructura del discurso argumentativo consta de:

• Tesis: idea que el emisor expone.

TAMadrid
13
-Pág.17-
L E N G UA Y L I T E R AT U R A

• Razonamientos a favor o en contra según el interés del emisor con respecto a la tesis.

• Solución final, acorde con el planteamiento de la tesis.

Utiliza abundantes adjetivos de carácter valorativo (favorables o desfavorables) según


intenta persuadir al auditorio para que adopte una actitud a favor o en contra de la tesis.

También son característicos los enlaces subordinantes de causa y consecutivos (por con-
siguiente, puesto que, porque, ya que, así que...).

El texto argumentativo es típico en las valoraciones políticas, críticas literarias, debates,


comentarios radiofónicos...

2.2. LA CONVERSACIÓN, EL COLOQUIO, EL DEBATE,


LA ENTREVISTA...
En este tipo de discursos el sentido de la comunicación es biunívoco, ya que todos los
interlocutores que participan pueden ser emisores (hablantes ) y receptores (oyentes).

LA CONVERSACIÓN: Es el diálogo que establecen dos o más personas, suele surgir sin
premeditación previa, aunque a veces puede haber intercambio de información relevante.

Es un acto de comunicación en el que pueden darse dos supuestos.

• Situación de cortesía, en la que los interlocutores participan activamente, intentan-


do mantener la comunicación.

• Situación de distanciamiento, en la que alguno de los interlocutores intenta que el


acto de comunicación sea lo más eficaz y conciso posible.

EL COLOQUIO: Es el diálogo organizado en el que a partir de un tema van exponiéndose


ideas complementarias, suele darse en situaciones donde las relaciones de los interlocutores
son bastante próximas (amistad, familia...), el vocabulario que se utiliza puede ser poco exten-
so y, generalmente, expresa una gran afectividad entre los componentes del grupo.

EL DEBATE: Es un tipo de diálogo organizado, en el que previamente se ha informado a


los interlocutores sobre el tema a discutir, lo dirige un moderador que se encarga de repar-
tir el tiempo de intervención de cada interlocutor y reconducir el debate.

Los integrantes del grupo pueden discutir libremente, respetándose el turno de palabra
y deben intentar llegar a unas conclusiones que satisfagan a la mayoría.

14 TAMadrid

-Pág.18-
U. D. 1 . - U S O S Y F ORMAS DE LA C O M U N I C AC I Ó N O R A L

Es una de las formas de comunicación en las que la argumentación tiene capital impor-
tancia.

LA ENTREVISTA: Es un acto de comunicación muy frecuente en los medios de comuni-


cación hablada.

Los interlocutores: entrevistador y entrevistado, mantienen un diálogo con alguna de


estas finalidades:

• Informativa: el entrevistador intenta conseguir la información que el entrevistado


tiene sobre un tema de interés.

• Psicológica: el entrevistador pretende conocer datos sobre el carácter, personalidad,


gustos... del entrevistado.

Toda entrevista debe transcurrir en un ambiente de educación y cortesía, especialmente


si el entrevistado se ha prestado a la misma sin ningún tipo de contraprestación.

2.3. REGISTRO Y USOS SOCIALES DE LA LENGUA ORAL


Todos los hablantes que utilizamos el español poseemos un mismo ideal de lengua, el
que reconocemos como modelo de buen español, propio en el habla de las personas cultas,
este modelo es lo que denominamos “coiné”.

Todos utilizamos la misma lengua, el español, con la que nos comunicamos, pero entre
los distintos hablantes se dan diversos niveles de habla, motivados por el grado de instruc-
ción, edad, sexo, intereses profesionales, ambiente...

El nivel de habla que cada uno utiliza es el “registro idiomático”; las personas cultas
pueden cambiar de registro idiomático en función de las personas con las que quiere comu-
nicarse.

Los usos sociales de la lengua oral más característicos son:

El habla culta:

Es el habla que usan las personas de elevada cultura: oradores, científicos, literatos...

Sus características son:

• Gran riqueza léxica: tecnicismos, cultismos...

TAMadrid
15
-Pág.19-
L E N G UA Y L I T E R AT U R A

• Sintaxis clara y correcta.


• Usos correctos de los tiempos verbales.
• Enfasis y entonación adecuadas.
• Propiedad y precisión en los términos empleados.
• Ausencia de muletillas...

La coiné, modelo de lengua ideal, no es identificable con una zona geográfica determi-
nada, sino que pertenece a las personas que se expresan con las características citadas, con
independencia de la región o país donde vivan.

El habla vulgar:

Es el habla que usa la lengua sin respetar sus normas.

Sus características son;

• Pobreza léxica.
• Usos incorrectos de los tiempos verbales: haiga por haya, callar por callad...
• Acentuación incorrecta en algunas palabras: telégrama por telegrama.
• Pérdida de algún sonido: llegao por llegado.
• Cambio de consonantes: abujero por agujero.
• Cambio de vocales: Juaquín por Joaquín.
• Usar el determinante delante del nombre propio : La María por María.
• Loísmos, laísmos y leísmos: lo dejé por le dejé, la hablé por le hablé, le tengo por
lo tengo...
• Dequeísmo: “quiero de que vengas” por “quiero que vengas”.
• Y otra serie de incorrecciones que, junto a las descritas, reciben el nombre de vul-
garismos.

El habla coloquial:

Es el habla que se usa en ambientes un tanto relajados: entre amigos, en familia...

Sus características son:

• Empleo de diminutivos, aumentativos y despectivos.


• Uso de construcciones impersonales: Ya está.

16 TAMadrid

-Pág.20-
U. D. 1 . - U S O S Y F ORMAS DE LA C O M U N I C AC I Ó N O R A L

• Imprecisión en el vocabulario: He perdido una cosa


• Entonaciones exageradas: ! Lo que faltaba¡
• Uso de apócopes: bici, tranqui, cole...
y en general, todo tipo de expresiones reforzadas con gestos, muecas, ademanes...

2.4. FINALIDAD, SITUACIÓN Y CONTEXTO COMUNICATIVO


Si tenemos en cuenta las funciones del lenguaje (FINALIDAD), las circunstancias
extralingüísticas (SITUACIÓN) y el contexto lingüístico (CONTEXTO COMUNICATI-
VO), así como las diversas estructuras textuales que hemos ido viendo a lo largo de esta uni-
dad, podemos dar una clasificación de tipos de discursos y de textos que constituirán las
fuentes de la documentación oral.

TIPOS DE DISCURSOS TIPOS DE DISCURSOS


SEGÚN: SEGÚN:

FINALIDAD SITUACIÓN Y ESTRUCTURA


CONTEXTO SECUENCIAL
BÁSICA

FUNCIONES INTERACCIÓN SOCIAL


SEGÚN LA SEGÚN LUGAR Y
INTENCIÓN EL CONTEXTO NARRATIVO
COMUNICATIVO DESCRIPTIVO
EXPOSITIVO
ARGUMENTATIVO
PEDAGÓGICO COLOQUIAL
INFORMATIVO CULTURAL ETC.
CONATIVO POLÍTICO
FÁTICO CIENTÍFICO
LÚDICO LABORAL
POÉTICO JURÍDICO
METALINGÜÍS- ETC.
TICO
ETC.

TAMadrid
17
-Pág.21-
L E N G UA Y L I T E R AT U R A

2.5. JERGAS Y LENGUAJES PROFESIONALES


Muchos hablantes que normalmente emplean la lengua de acuerdo con su nivel de ins-
trucción y cultura, cuando se encuentran en determinadas situaciones: lugar de trabajo, reu-
nión juvenil., emplean modalidades de lengua denominadas jergas.

Distinguiremos tres tipos:

• Jergas de la delincuencia y grupos marginales; Estos grupos de población utilizan


esta modalidad de la lengua con la intención de que los individuos ajenos a sus grupos no
puedan entenderlos; así pueden actuar más libremente: chute (pinchazo de droga), bofia
(policía), caballo (heroína), lechera (coche policial), etc.

• Jergas juveniles. En todas la épocas los jóvenes han intentado diferenciarse de los
adultos por cualquier medio, incluido por el modo de hablar. Actualmente existen bastantes
términos de las jergas juveniles que están en el uso normal de la lengua: viejos (padres),
chungo (malo), papeo (comida)...

• Jergas profesionales: son debidas a la utilización de los tecnicismos propios de


cada ciencia, profesión u oficio. Son hablas completamente normalizadas, con la única sal-
vedad de que utilizan términos específicos de cada profesión; bisturí, enmienda, fusible,
escofina, alternador, goniómetro,etc.

DIVERSIDAD LINGÜÍSTICA Y VARIEDADES


3. DIALECTALES DE LA LENGUA ORAL
Ya sabemos que el lenguaje oral es la capacidad humana para comunicarse mediante
sonidos articulados, pero esa capacidad no se desarrolla por igual en todas las comunidades
que existen en el mundo; cada comunidad tiene su lengua o idioma, que es la que se utili-
za a nivel oficial (leyes, actos públicos, comunicados oficiales, las informaciones de alcan-
ce nacional... Es la que tienen como modelo todos los integrantes de esa comunidad; así
hablamos del español, inglés, francés...

Ahora bien, en esas mismas comunidades pueden darse derivaciones de esa lengua o
idioma, con usos y formas especiales que originan las variedades dialectales de una lengua
o idioma; así hablamos del extremeño, andaluz, murciano...

18 TAMadrid

-Pág.22-
U. D. 1 . - U S O S Y F ORMAS DE LA C O M U N I C AC I Ó N O R A L

Cuando los dialectos evolucionan y son capaces de crear un sistema de comunicación


que sirve como medio de expresión de su cultura, con sus propios reglas, se independiza de
la lengua de la que procede y se convierte en una nueva lengua. Así por ejemplo, el caste-
llano es un dialecto del latín que al evolucionar se convirtió en lengua.

3.1. LA REALIDAD PLURILINGÜE DE ESPAÑA


Si observamos el mapa, podremos ver que si bien todo el país tiene una lengua en
común, el castellano, existen otras lenguas: gallego, vascuence, catalán y valenciano, ade-
más de las variedades dialectales que han ido surgiendo de todas ellas; incluso existen zonas
donde subsisten variedades del latín que no han llegado a consolidarse como lenguas. Esto
nos indica que la realidad lingüística de España es plurilingüe.

Todas las lenguas españolas excepto el vasco, que ya existía con anterioridad a la llega-
da de los romanos en el año 218 a. JC., son procedentes del latín; es decir, son lenguas
romances, románicas o neolatinas.

A la llegada de los romanos a la península Ibérica se hablaban diversas lenguas: vasco,


ibero, fenicio, tarteso..., pero todas salvo el vascuence o euskera , que aún existe, desapare-
cieron y fueron sustituidas por el latín, que era la lengua del Imperio.

TAMadrid
19
-Pág.23-
L E N G UA Y L I T E R AT U R A

En el siglo V, coincidiendo con la caída del imperio romano, diversos pueblos bárbaros:
suevos, vándalos, alanos, visigodos... irrumpen en la península Ibérica, posteriormente, en
el siglo VIII, se da la ocupación de la península por los árabes; todo ello provocó que se per-
diera la unidad idiomática, el latín, y que en las diversas zonas de la península el latín fuera
evolucionando de maneras distintas, surgiendo diversos dialectos, algunos de los cuales:
castellano, gallego, catalán y valenciano, llegaron a convertirse en lenguas. Otros dialectos:
el astur-leonés y el navarro aragonés no han llegado a erigirse en lengua.

Veamos las diversas lenguas y dialectos que configuraron la realidad plurilingüe de


España:

EL CASTELLANO

Surge como una variedad del latín en la zona de Cantabria, y a medida que avanza hacia
el sur la reconquista, va extendiéndose, abriendo cada vez más su frente, en las zonas recon-
quistadas; incluso logra introducirse y afianzarse como lengua en zonas donde ya existen
otras variedades dialectales del latín: León, Rioja, Navarra y Aragón, que supeditan sus pro-
pias hablas a favor del castellano.

Cuando los Reyes Católicos consiguen la unidad nacional, en 1492, Castilla es el reino
más importante de España y su lengua está presente en todos los acontecimientos del país.

Durante los siglos XVI y XVII es la lengua que usan mayoritariamente las personas cul-
tas de la península y consigue alcanzar una enorme importancia en el contexto internacio-
nal.

Es la lengua oficial del Estado Español y en algunas autonomías comparte la cooficia-


lidad con las respectivas lenguas autonómicas.

El nombre, castellano, o español, queda al libre albedrío del hablante; parece que el tér-
mino castellano tiene más sentido cuando se utiliza en España para no desmerecer las otras
lenguas que también son españolas; en cambio, en el plano internacional, el término espa-
ñol resulta más lógico. De hecho, la mayoría de las lenguas reciben el nombre de sus paí-
ses:

Francés, italiano, alemán..., independientemente de que en los mismos haya otras lenguas.

Dialectos del castellano en España:

• Andaluz.
• Murciano.

20 TAMadrid

-Pág.24-
U. D. 1 . - U S O S Y F ORMAS DE LA C O M U N I C AC I Ó N O R A L

• Extremeño.
• Canario.

EL CATALÁN

Resultó de la evolución del latín en la zona del antiguo Principado de Cataluña, desde
donde se extendió a otros lugares:

Tuvo gran importancia hasta los siglos XVI , XVII y XVIII en los que queda en un
segundo plano por el florecimiento de la lengua y literatura del castellano.

A mediados del siglo XIX vuelve a resurgir gracias a un movimiento de reivindicación


de la lengua catalana, que inició la construcción de una normativa idiomática y ortográfica.

La constitución de 1978 reconoció la lengua catalana como lengua cooficial con el cas-
tellano en Cataluña y Baleares.

Dialectos del Catalán:

• Orientales: Rosellonés, Mallorquín, Alguerés.

• Occidentales: Andorrano.

EL GALLEGO

Surgió en el noroeste peninsular como resultado de la evolución del latín.

Tiene gran afinidad con el portugués, ya que en un principio fueron una misma lengua;
el gallego-portugués; al separarse políticamente los territorios de cada comunidad evolu-
cionaron independientemente.

En la Edad Media fue una lengua importante, pero desde el siglo XIV hasta mediados
del siglo XIX por parecidos motivos que la catalana estuvo en un segundo plano, hasta que
gracias al movimiento de reivindicación de la lengua gallega renació con gran decisión.

La constitución de 1978 reconoció la lengua gallega como lengua cooficial con el cas-
tellano en Galicia.

Dialectos del gallego:

A pesar de que se dan diversas variaciones del gallego en distintas zonas, no es posible
establecer una clasificación que permita identificarlos claramente.

TAMadrid
21
-Pág.25-
L E N G UA Y L I T E R AT U R A

EL EUSKERA

El vascuence, vasco o euskera es la más antigua de las lenguas españolas, diferentes


hipótesis apuntan a que se hablaba 1000 años antes de la llegada de los romanos; su origen
sigue sin aclarar, si bien parece que procede de la zona del Cáucaso entre Rusia y Turquía.

Hasta épocas recientes ha sido una lengua utilizada en ámbitos muy restringidos: fami-
lia y zonas rústicas apartadas.

Se habla en el País Vasco, el norte de Navarra y el departamento francés de los Pirine-


os Occidentales.

Igual que el Catalán y el Gallego fue reconocida por la constitución española como len-
gua cooficial con el castellano en el País Vasco.

Dialectos vascos:
• En España : el vizcaíno, el guipuzcoano y el navarro.
• En Francia: el labortano y el suletano.

EL VALENCIANO

Durante muchísimo tiempo se le ha considerado un dialecto del catalán, en la actuali-


dad su Estatuto de autonomía lo reconoce como lengua cooficial junto al castellano en la
comunidad Valenciana.
El Valenciano, si bien tiene claras influencias del catalán, presenta características foné-
ticas, léxicas y morfológicas que el catalán no tiene, además de numerosos términos arábi-
gos y mozárabes que son característicos de la zona.

EL DIALECTO LEONÉS

Es un dialecto del latín, formado por distintas hablas que tienen más rasgos diferencia-
dores entre sí que comunes.
Se extienden por León, Zamora, Salamanca y norte de Extremadura, también en Astu-
rias (el bable) y la parte occidental de Santander.
En la actualidad, el dialecto de Asturias: el bable, está resurgiendo con extraordinaria
vitalidad. En realidad se debe hablar de bables, pues cada valle tiene su propia habla local
(bable).
En estas zonas, el español es la lengua de cultura, y es la que normalmente se utiliza
como medio de comunicación oral.

22 TAMadrid

-Pág.26-
U. D. 1 . - U S O S Y F ORMAS DE LA C O M U N I C AC I Ó N O R A L

EL DIALECTO ARAGONÉS

Es otro dialecto del latín que tampoco llegó a constituirse en lengua.


Se extiende por los valles pirenaicos de Aragón: Ansó, Hecho, Biescas, Sobrarbe...,
donde goza aún de cierta vitalidad.
La lengua modelo de estas zonas es el castellano.

3.2. FENÓMENOS DE CONTACTO ENTRE LAS DISTINTAS


LENGUAS
Debemos distinguir dos casos:
• Que se trate de dos lenguas que conviven en el mismo espacio geográfico: "con-
tacto total".
• Que las lenguas pertenezcan a zonas geográficas distintas y sólo las separe una
línea divisoria política: "contacto parcial".
a) En el primer caso, zonas bilingües, donde los integrantes de dichas comunidades pue-
den utilizar dos lenguas distintas, están en permanente contacto; la utilización de las dos
lenguas, o bien de una u otra, va a depender del dominio que se tenga de cada una de
ellas y de los beneficios (prestigio social, cultural, profesional...) que puede reportar el
uso de alguna de ellas. En este último supuesto suele utilizarse la lengua que se consi-
dera más importante en las situaciones de comunicación más prestigiosas (educación,
administración, radio, tv. ...), quedando relegada la otra lengua a situaciones de comu-
nicación más restringidas (familia, amigos ...). Este fenómeno se denomina "diglosia".
La diglosia es causa frecuente de enfrentamientos sociales; el grupo de hablantes que
utiliza la lengua dominante suele ocupar puestos de mayor relevancia en la sociedad,
lo que, a su vez, contribuye a que esa lengua aumente su importancia y utilización.
b) En el segundo caso, son las lenguas propias de determinadas zonas que limitan entre
sí ; el concepto de límite geográfico no suele corresponderse con el de límite lin-
güístico, esto sólo suele darse cuando hay una barrera natural que separe a ambas
comunidades (una barrera montañosa, un gran río, un mar ...).
En este tipo de contacto parcial pueden darse dos fenómenos:
• Invasión lingüístca: Una franja bilingüe que suele ocupar más espacio en una de las
zonas, la ocupada por la lengua considerada de menor prestigio.
• De préstamos lingüísticos: Trasvase de vocablos de una a otra lengua.

TAMadrid
23
-Pág.27-
L E N G UA Y L I T E R AT U R A

3.3. USO Y DIFUSIÓN INTERNACIONAL DEL ESPAÑOL


Y DE LAS OTRAS LENGUAS DE ESPAÑA
El español es una de las lenguas más importantes del mundo por su número de hablan-
tes, unos 400 millones, de los cuales sólo 40 millones habitan en España.

24 TAMadrid

-Pág.28-
U. D. 1 . - U S O S Y F ORMAS DE LA C O M U N I C AC I Ó N O R A L

El español es la lengua oficial de España y de 18 repúblicas hispanoamericanas: Argen-


tina, Uruguay, Paraguay, Chile, Perú, Bolivia, Ecuador, Colombia, Venezuela, Panamá,
Costa Rica, Nicaragua, Honduras, El Salvador, Guatemala, Méjico, Cuba y República
Dominicana.
Es también lengua oficial en Puerto Rico (estado asociado con los Estados Unidos),
situación que comparte con el Inglés.
Se habla en Nuevo Méjico y otras zonas del Sur de Estados Unidos (California, Arizo-
na, Tejas...).
En Asia, se habla en Filipinas, donde coexiste con el inglés y el tagalo, aunque en fran-
co retroceso (los jóvenes empiezan a desconocerlo).
En África, se habla en:

• Algunas partes de Marruecos, (Tetuán, Larache, Tánger, Ifni.)


• En el antiguo Sáhara Español.
• En Guinea Ecuatorial (antigua Guinea Española).
También debemos destacar las numerosas comunidades de sefardíes, descendientes de
los judíos españoles expulsados en 1492 por los Reyes Católicos, que viven en Israel,
Rumania, Bulgaria, Turquía, Grecia,... etc. Este español sefardí o "sefardita" conserva
características del español del siglo XV.
Además debemos considerar los numerosos hablantes de otras lenguas que han adopta-
do al español como segunda lengua.
En cuanto a las otras lenguas españolas (gallego, vascuence, catalán y valenciano), sus
áreas de influencia son más reducidas, se circunscriben a los lugares mencionados en la
explicación dada de estas lenguas y a los numerosos grupos de emigrantes que coinciden
en diversas partes de Europa y América.

Es digno de destacar la labor difusora del Instituto Cervantes, fundado en 1991, que
cuenta con más de 30 centros, distribuidos por todo el mundo, con programas de aprendi-
zaje del español y las otras lenguas autonómicas para extranjeros.

TAMadrid
25
-Pág.29-
L E N G UA Y L I T E R AT U R A

RESUMEN
Recuerda que:

– Definimos el “Lenguaje” como la capacidad que tiene la persona para comunicarse


con las demás a través de signos (verbales, gestuales, escritos, etc.); y denominamos
“Lengua” al conjunto de signos verbales (orales y escritos) y reglas que forman un
código, que permite comunicarse a los miembros de una comunidad concreta.
– La lengua oral es el sistema de comunicación que permite formar mensajes con una
mayor facilidad. Mediante la cadena hablada podemos encontrar respuestas a cual-
quier necesidad potencial comunicativa.
– En el acto de comunicación intervienen los elementos siguientes:
Emisor A Canal A Mensaje A Receptor
Código
Contexto

– Los sonidos específicos de cada lengua son los denominador fonemas. Nuestra len-
gua tiene 24 fonemas (5 vocálicos y 19 consonánticos).
– El lenguaje oral ofrece infinidad de funciones: Informativa, expresiva o emotiva,
conativa o apelativa, fática o de contacto, lúdica, poética o estética y metalingüística.
– En nuestro país la lengua oficial es el español o castellano. Pero, también, existen
otras lenguas: Gallego, vascuence, catalán y valenciano. Además, conviven con ellas
los dialectos que han ido surgiendo de todas ellas, como: Astur-leonés, navarro-ara-
gonés, andaluz, murciano, extremeño, canario, mallorquín.
– El español es una de las lenguas más importantes del mundo por su número de
hablantes, unos 400 millones.

26 TAMadrid

-Pág.30-
U. D. 1 . - U S O S Y F ORMAS DE LA C O M U N I C AC I Ó N O R A L

EJERCICIOS DE AUTOCOMPROBACIÓN

1. El conjunto de signos verbales (orales y escritos) y reglas que forman un código que permite
comunicarse a los miembros de una comunidad concreta, se denomina:
A. Lenguaje B. Lengua C. Habla D. Mensaje

2. Un padre llama a su hijo: Juan, ven aquí. ¿Qué función del lenguaje oral se produce?
A. Conativa B. Metalingüística C. Poética D. Fática

3. ¿Cuál de las siguientes es una vocal abierta?


A. a B. e C. i D. o

4. ¿Cuál de las siguientes es una vocal cerrada?


A. a B. e C. i D. o

5. Según el lugar de articulación ¿Cuál de las siguientes consonantes es bilabial?


A. p B. t C. d D. z

6. La palabra Madrid es:


A. Aguda B. Grave C. Llana D. Esdrújula

7. La exposición de cómo son las personas, animales, cosas, etc. explicando sus cualidades, carac-
terísticas, etc. se denomina:
A. Narración B. Descripción C. Exposición D. Argumentación

8. La frase “Tiene mucho valor”, es:


A. Exclamativa B. Enunciativa C. Interrogativa D. Subordinada

9. ¿Cuál de las siguientes lenguas no procede del latín?


A. Gallego B. Catalán C. Vasco D. Valenciano

10. El andaluz es un dialecto del:


A. Extremeño B. Castellano C. Murciano D. Canario

TAMadrid
27
-Pág.31-
L E N G UA Y L I T E R AT U R A

RESPUESTAS A LOS EJERCICIOS

11. B

12. A

13. A

14. C

15. A

16. A

17. B

18. B

19. C

10. B

28 TAMadrid

-Pág.32-
U. D. 2 . - U S O S Y F ORMAS DE LA C O M U N I C A C I Ó N E S C R I TA

ÍNDICE
Pag.

OBJETIVOS . . . . . . . . . . . . . . . . . . . . . . . . . . . . . . . . . . . . . . . . . . . . . . . . . . . . . . . . . . . . . . 2

INTRODUCCIÓN . . . . . . . . . . . . . . . . . . . . . . . . . . . . . . . . . . . . . . . . . . . . . . . . . . . . . . . . . 3

MAPA CONCEPTUAL . . . . . . . . . . . . . . . . . . . . . . . . . . . . . . . . . . . . . . . . . . . . . . . . . . . . . 4

DESARROLLO DE CONTENIDOS . . . . . . . . . . . . . . . . . . . . . . . . . . . . . . . . . . . . . . . . . 5

1. LA COMUNICACIÓN ESCRITA: ELEMENTOS Y FUNCIONES . . . . . . . . . 5


1.1. NECESIDADES DE COMUNICACIÓN E INTENCIÓN COMUNICATIVA:
ACTOS DE HABLA. . . . . . . . . . . . . . . . . . . . . . . . . . . . . . . . . . . . . . . . . . . . . . . . . 5
1.2. LA SITUACIÓN DE COMUNICACIÓN Y SUS ELEMENTOS.. . . . . . . . . . . . . . 5
1.3. LA LENGUA ESCRITA COMO FUENTE DE INFORMACIÓN,
CONOCIMIENTO, PLACER, PERSUASIÓN, MANIPULACIÓN, … . . . . . . . . . 7
1.4. CARACTERÍSTICAS GRÁFICAS DE LA LENGUA ESCRITA . . . . . . . . . . . . . 8

2. TIPOS Y FORMAS DE DISCURSO EN LA COMUNICACIÓN ESCRITA . . 10


2.1. ESTRUCTURAS TEXTUALES BÁSICAS: LA NARRACIÓN, LA
DESCRIPCIÓN, LA EXPOSICIÓN, LA ARGUMENTACIÓN, … . . . . . . . . . . . 10
2.2. REGISTRO Y USOS SOCIALES DE LA LENGUA ESCRITA. . . . . . . . . . . . . . 12
2.3. FINALIDAD, SITUACIÓN Y CONTEXTO COMUNICATIVO. . . . . . . . . . . . . . 13
2.4. USOS ESPECÍFICOS DE LA LENGUA ESCRITA . . . . . . . . . . . . . . . . . . . . . . . 13
2.5. LOS LENGUAJES ESPECÍFICOS: CIENTÍFICO, TÉCNICO, JURÍDICO, … . . . 14

3. DIVERSIDAD LINGÜÍSTICA Y VARIEDADES DIALECTALES DE LA


LENGUA ESCRITA . . . . . . . . . . . . . . . . . . . . . . . . . . . . . . . . . . . . . . . . . . . . . . . . . . . 16
3.1. LA REALIDAD PLURILINGÜE DE ESPAÑA . . . . . . . . . . . . . . . . . . . . . . . . . . 16
3.2. FENÓMENOS DE CONTACTO ENTRE LAS DISTINTAS LENGUAS . . . . . . 18
3.3. USO Y DIFUSIÓN INTERNACIONAL DEL ESPAÑOL Y DE LAS OTRAS
LENGUAS DE ESPAÑA . . . . . . . . . . . . . . . . . . . . . . . . . . . . . . . . . . . . . . . . . . . . 18

4. USOS DE LAS FUENTES DE COMUNICACIÓN ESCRITA . . . . . . . . . . . . . 19

RESUMEN . . . . . . . . . . . . . . . . . . . . . . . . . . . . . . . . . . . . . . . . . . . . . . . . . . . . . . . . . . . . . . . 19

EJERCICIOS DE AUTOCOMPROBACIÓN . . . . . . . . . . . . . . . . . . . . . . . . . . . . . . . . 20

RESPUESTAS . . . . . . . . . . . . . . . . . . . . . . . . . . . . . . . . . . . . . . . . . . . . . . . . . . . . . . . . . . . . 22

-Pág.33-
L E N G UA C A S T E L L A N A Y L I T E R AT U R A

OBJETIVOS
Al finalizar el estudio de esta Unidad Didáctica, el alumno será capaz de:

• Reconocer los elementos y funciones de la comunicación escrita.

• Diferenciar los diversos tipos y formas de discursos en la comunicación


escrita.

• Valorar la realidad plurilingüe de España y la importancia del Español en el


mundo.

-Pág.34-
U. D. 2 . - U S O S Y F ORMAS DE LA C O M U N I C A C I Ó N E S C R I TA

INTRODUCCIÓN
L a comunicación escrita es cualquier acto social mediante el cual el emisor
(escritor) cifra (escribe) un mensaje y lo envía al receptor (lector) y éste lo des-
cifra (lee).

Cuando hablamos o escribimos utilizamos signos lingüísticos; en éstos cabe distin-


guir dos planos:

- Plano de la expresión o significante: los sonidos o letras que forman el signo


lingüístico.
- Plano del contenido o significado: lo que representa e imaginamos.

Ejemplo:
Expresión o significante: “Cuadrado”.
Contenido o significado: “Polígono de cuatro lados y ángulos iguales”.

El expresarse de forma oral es de dominio universal, su aprendizaje es espontáneo y


está al alcance de todos; en cambio, el proceso para el aprendizaje de la expresión
escrita resulta más complejo, requiere de un esfuerzo personal mayor y unas posibi-
lidades económicas que no están al alcance de todos los grupos humanos. Esto pro-
picia que, socialmente, goce de mayor prestigio la comunicación escrita, a pesar de
que nunca podrá lograr los infinitos matices expresivos de la comunicación oral.

En la actualidad, los avances tecnológicos (y seguimos hablando de “posibilidades


económicas”) han revolucionado el mundo de las comunicaciones; parece como si
los factores espacio y tiempo hubieran desaparecido. Hoy, varias personas, residen-
tes en lugares distintos y en el mismo momento, pueden establecer una comunica-
ción escrita de forma simultánea.

En esta unidad nos referiremos fundamentalmente a la comunicación escrita, si bien


tendremos presente lo expuesto en la Unidad 1 (comunicación oral), ambas se com-
plementan y forman la Comunicación Verbal.

-Pág.35-
L E N G UA C A S T E L L A N A Y L I T E R AT U R A

M A PA C O N C E P T UA L

LENGUA CASTELLA-
NA Y LITERATURA

LA
COMUNICACIÓN USOS Y FORMAS DE DIVERSIDAD
LA COMUNICACIÓN LINGÜISTICA
ESCRITA VARIEDADES
DIALECTALES
DE LA LENGUA
ESCRITA
ELEMENTOS
TIPOS Y FORMAS EN LA
COMUNICACIÓN
ESCRITA
FUNCIONES

ESTRUCTURAS

REGISTROS Y
USOS SOCIALES USO Y DIFUSIÓN DEL
ESPAÑOL Y DE LAS
OTRAS LENGUAS DE
ESPAÑA

LA REALIDAD PLURI-
LINGÜE DE ESPAÑA

FUENTES DE LA DOCUMENTACIÓN ESCRITA

-Pág.36-
U. D. 2 . - U S O S Y F ORMAS DE LA C O M U N I C A C I Ó N E S C R I TA

LA COMUNICACIÓN ESCRITA:
1. ELEMENTOS Y FUNCIONES

Tanto en la comunicación escrita como en la oral, el objetivo es el mismo: transmitir infor-


mación; pero entre ambas existen diferencias notables, algunas de las cuales creemos necesario
exponerlas:
- Oral: La materia física que utiliza es el signo fónico (sonido), que se percibe cuando se emite y desaparece
inmediatamente.
- Escrita: La materia física que utiliza es el signo gráfico (letra), que perdura relativamente.

- Oral: Los interlocutores pueden alternarse en las funciones de emisor y receptor.


- Escrita: El sentido de la comunicación va siempre desde el emisor hacia el receptor.

- Oral: Es más espontánea, más ambigua....; si el emisor observa que el emisor no ha comprendido bien el men-
saje, puede ampliar, matizar..., incluso ayudarse de gestos, cambiar la entonación, etc.
- Escrita: Requiere de mayor atención, la planificación y elaboración del mensaje es el resultado de una refle-
xión más profunda, donde la precisión semántica y sintaxis eviten cualquier posible ambigüedad, etc.

1.1. NECESIDADES DE COMUNICACIÓN E INTENCIÓN


COMUNICATIVA:ACTOS DE HABLA
En la Unidad 1 afirmábamos que los hombres, como seres sociales, necesitan relacionarse entre
sí para comunicarse lo que piensan, sienten o desean; de tal forma que podemos afirmar que la
comunicación es la base sobre la que se fundamentan las sociedades humanas.
Las sociedades avanzan y crece el número de necesidades de comunicación, y no siempre es
posible satisfacerlas de forma oral, por lo que la escritura surge, intencionadamente, como una solu-
ción a esas potenciales necesidades.
Ejemplos:
- En el antiguo Egipto, la intención de registrar las crecidas del Nilo surgiría como necesidad de conocer y
transmitir esa información a las generaciones siguientes.
- Ante la necesidad de que conste oficialmente cómo ocurrió un accidente de tráfico, surgirá la intención de
redactar un informe escrito (atestado) que explique los detalles y circunstancias del mismo.
- Adquirimos el periódico con la intención de leerlo porque necesitamos recibir información económica, polí-
tica, deportiva, lúdica, etc.

1.2. LA SITUACIÓN DE COMUNICACIÓN Y SUS ELEMENTOS


Aunque la comunicación escrita tiene un sentido unidireccional, sólo va desde el emisor hasta el
receptor, también la debemos calificar de “acto social”, y, de forma análoga al circuito de la comuni-
cación oral, se puede representar mediante un esquema donde aparezcan todos sus elementos.

TAMadrid
5
-Pág.37-
L E N G UA C A S T E L L A N A Y L I T E R AT U R A

Es importante que comparemos la explicación de ambos circuitos para poder apreciar las dife-
rencias significativas que existen entre ellos.
CIRCUITO DE LA COMUNICACIÓN

CÓDIGO

EMISOR CANAL MENSAJE CANAL RECEPTOR


(ESCRITOR) (LECTOR)
SITUACIÓN

CONTEXTO

EMISOR: El escritor (autor) que cifra (escribe) el mensaje y lo envía.


RECEPTOR: El lector (destinatario) que recibe el mensaje y lo descifra (lee).
CÓDIGO: Conjunto de signos y reglas que deben conocer el emisor y el receptor (en nuestro
caso, los relativos a la escritura de la lengua castellana).
CANAL: Medio por el que se transmite el mensaje. En una obra literaria serían las páginas.
MENSAJE: La información que envía el emisor al receptor; lo produce siempre el emisor.
CONTEXTO LINGÜÍSTICO: Conjunto de palabras que acompañan a un término dado, al que
complementan y aclaran su significado. Ejemplos:
- Ayer se quedó blanca (lívida).
- Ayer se quedó sin blanca (dinero).
La inclusión de la preposición “sin” en la segunda oración propicia que los significados de
ambas oraciones sean completamente diferentes.
CONTEXTO SITUACIONAL: Conjunto de circunstancias extralingüísticas que ayudan a com-
prender perfectamente los matices significativos de un mensaje. Normalmente no es comparti-
do entre el escritor y el lector; el escritor escribe su mensaje en un lugar y tiempo determinados
y el destinatario lo recibe en otro lugar y tiempo posterior. No obstante, debemos hablar de con-
texto situacional en la comunicación escrita si consideramos factores como: Conocimiento
mutuo entre escritor y lector, nivel cultural de ambos, etc.
Excepcionalmente, el escritor y el lector pueden compartir el mismo contexto situacional.
Ejemplos:
- Las notas escritas que se pasan los políticos en las sesiones parlamentarias para evitar que la oposición conoz-
ca ciertas informaciones de carácter reservado.

- La notificación escrita que entrega un agente de circulación a un conductor que acaba de cometer una infrac-
ción de tráfico, donde le informa del tipo de falta cometida, cuantía de la sanción impuesta, plazos para hacer-
la efectiva, etc.

6 TAMadrid

-Pág.38-
U. D. 2 . - U S O S Y F ORMAS DE LA C O M U N I C A C I Ó N E S C R I TA

Ejemplo práctico de comunicación escrita:


Imaginemos que leemos un documento del siglo XVIII;
el circuito de comunicación sería el siguiente:
Emisor: El autor del mensaje.
Receptor: Las personas que leemos el mensaje.
Código: El de la lengua en que se haya escrito el mensaje.
Canal: El papel, cuero...etc.
Mensaje: El contenido del mensaje.
Contexto lingüístico: Las palabras relacionadas con el contenido del mensaje.
Contexto situacional: Las posibles nociones o indicios que pudiéramos tener
sobre el tema o su autor y que nos permite imaginar la situación en que se pro-
duce el mensaje.

1.3. LA LENGUA ESCRITA COMO FUENTE DE INFORMACIÓN,


CONOCIMIENTO, PLACER, PERSUASIÓN, MANIPULACIÓN...
La lengua escrita es el sistema de comunicación que permite formar mensajes que, al estar gra-
bados gráficamente sobre un soporte, permanecen a disposición del receptor, éste puede leerlos
cuantas veces desee y reflexionar sobre los mismos.
Mediante la escritura podemos encontrar respuestas a cualquier necesidad comunicativa; el len-
guaje escrito ofrece infinidad de funciones para tal finalidad; veamos las más frecuentes:
- Informativa, representativa o referencial: Su intención es informar objetivamente. Ej.:
En un periódico, la información relativa a la previsión del tiempo para el día siguiente.
- Expresiva o emotiva: El escritor expresa subjetivamente sus sentimientos, emocio-
nes, etc. Suelen ir entre signos de exclamación. Ej.: ¡Qué bien jugó el portero visi-
tante!.
- Conativa o apelativa: El escrito intenta influir en la actitud del lector, o bien , llamar
su atención. Ej.: Imaginemos un cartel publicitario con la siguiente frase: “Pruebe tal
o cual producto y no querrá otros”.
- Lúdica: Intenta conseguir divertimento mediante juegos de palabras: chistes, dilogí-
as, refranes, etc.
- Poética o estética: Su finalidad es centrar la atención del lector en el mensaje. Para
conseguirlo resalta ciertos elementos formales del mensaje, embellece la expresión o
la hace más chocante, intrigante, etc.
Aparece, fundamentalmente, en la literatura. Ej.: Quevedo, para referirse al precioso
cabello rubio de una dama, escribe: “Rizas en ondas ricas del rey Midas”. (Cabello
rubio = oro; recordemos cómo el rey Midas transformaba en oro todo lo que tocaba).

TAMadrid
7
-Pág.39-
L E N G UA C A S T E L L A N A Y L I T E R AT U R A

- Metalingüística: Es la función del lenguaje para explicar su propio código. Ej.: “Lle-
van tilde en la antepenúltima sílaba todas las palabras esdrújulas: cántaro, médico, etc.”
- A veces, en un mismo mensaje, pueden aparecer dos o más funciones.
Imaginemos un anuncio publicitario con el siguiente mensaje: “Para los que aman el
orden: Ordenadores X”. Podemos observar distintas funciones:
- Representativa -> Informa de una nueva marca.
- Conativa -> Invita a que se compre.
- Estética -> Centra la atención del lector en un mensaje atractivo, interesante:
Orden = Seriedad, responsabilidad, etc.

1.4. CARACTERÍSTICAS GRÁFICAS DE LA LENGUA ESCRITA


La ortografía es el conjunto de normas que regulan la escritura de una lengua.

Como en la mayoría de los idiomas, la escritura española representa la lengua hablada


por medio de letras y de otros signos gráficos.

Cada lengua tiene su propio alfabeto: conjunto ordenado de letras que representan grá-
ficamente a sus fonemas; y lo mismo que pueden diferir en el número de fonemas, también
pueden diferir en el número de letras; incluso la tipografía de éstas puede ser diferente.
Árabe, griego, español, etc.

Desde 1994, el alfabeto español está constituido por 27 letras: 5 vocales y 22 conso-
nantes. Es de origen latino, como la lengua. Cada una de las letras se corresponde con un
signo cuando se emplea como mayúscula y con otro cuando se emplea como minúscula.

El alfabeto español es relativamente simple. Generalmente cada sonido está represen-


tado por una sola letra, y cada letra representa un solo sonido, excepto en los casos
siguientes:
a) El sonido /b/ puede escribirse con b (cebra) o con v (velocidad).
b) La letra h no suele representar ningún sonido.
c) La c representa el sonido /k/ y el sonido /e/(caverna, ceder).
d) La g se utiliza para representar los sonidos /g/ y /x/ (galopar, gineta).
e) La x, según su posición, es /s/ (excluir) o /ks/ (exhaustivo).

8 TAMadrid

-Pág.40-
U. D. 2 . - U S O S Y F ORMAS DE LA C O M U N I C A C I Ó N E S C R I TA

TAMadrid
9
-Pág.41-
L E N G UA C A S T E L L A N A Y L I T E R AT U R A

Además, en la escritura, se utilizan otros signos gráficos: punto (.), coma (,), punto y coma (;),
dos puntos (:), puntos suspensivos (...), signos de interrogación (¿?), signos de admiración o excla-
mación (¡!), paréntesis ( ), raya (-), guión (–), comillas (“”)...etc.

2. TIPOS Y FORMAS DE DISCURSO EN LA


COMUNICACIÓN ESCRITA
En cualquier tipo y forma de discurso escrito deberemos tener presentes las siguientes caracte-
rísticas que son propias de la comunicación escrita:
• El sentido de la comunicación siempre es unidireccional: va desde el emisor hasta el recep-
tor.
• El receptor puede ser:
- Individual: destinatario de una carta....
- Colectivo: Lectores de una obra literaria....
• Utiliza términos monosémicos ( un solo significado) para evitar posibles ambigüedades.
• La permanencia relativa del mensaje (dependiente de la estructura física del soporte sobre
el que se haya grabado: metal, papel, cuero, etc.), que le permite ser leído infinidad de veces.
• Etc.

2.1. ESTRUCTURAS TEXTUALES BÁSICAS: LA NARRACIÓN, LA


DESCRIPCIÓN, LA EXPOSICIÓN, LA ARGUMENTACIÓN...
Las estructuras básicas: la narración, la descripción, la exposición y la argumentación son comu-
nes en la lengua oral y escrita, por lo que no consideramos necesario repetir las características de
las mismas que aparecen en el apartado 2.1. de la Unidad 1; en su lugar indicaremos sus usos típi-
cos y presentaremos un texto escrito de cada estructura básica.
LA NARRACIÓN
Se usa frecuentemente en:
- Libros de historia.
- Noticias de prensa escrita.
- Novelas y cuentos.
Ejemplo de texto escrito narrativo:

10 TAMadrid

-Pág.42-
U. D. 2 . - U S O S Y F ORMAS DE LA C O M U N I C A C I Ó N E S C R I TA

"- señor conde - dijo Patronio -, hubo una mujer llamada doña Truhana, más pobre que
rica, que un día iba al mercado llevando sobre su cabeza una olla de miel. Yendo por el
camino, empezó a pensar que vendería aquella olla de miel y compraría con el dinero
una partida de huevos, de los cuales nacerían gallinas, y que luego, con el dinero que
sacara de la venta de las gallinas, compraría ovejas, y así fue comprando con las ganan-
cias hasta que se vio más rica que sus vecinas. Luego pensó que con aquella riqueza
casaría a sus hijos e hijas e iría acompañada por la calle de yernos y nueras, oyendo a
la gente celebrar su buena ventura. Pensando en esto se empezó a reír con tal alegría
que le bullía en el cuerpo, y, al reírse, se dio con la mano un golpe en la frente, con lo
que cayó la olla en tierra y se rompió en pedazos. Cuando vio la olla rota, empezó a
lamentarse como si hubiera perdido lo que pensaba haber logrado si no se rompiera.
Vos, señor conde....................."
Don Juan Manuel: El conde Lucanor (Adaptación).

LA DESCRIPCIÓN
Se usa frecuente en:
- Folletos de publicidad.
- Guías de museos.
- Libros de viajes. Etc.
Ejemplo de texto descriptivo escrito:

" Recórrense a las veces leguas y más legua desiertas, sin divisar apenas más que la
llanura inacabable donde verdea el trigo o amarillea el rastrojo, alguna procesión
monótona y grave de pardas encinas, de verde severo y perenne, que pasan lentamen-
te espaciadas, o de tristes pinos que levantan sus cabezas uniformes"
Miguel de Unamuno: En torno al casticismo.

LA EXPOSICIÓN
Se usa principalmente en :
- Libros de textos.
- Enciclopedias y diccionarios
- Periódicos, revistas, etc.
Ejemplo de texto escrito expositivo:

"En todas las sociedades humanas los individuos se diferencian unos de otros por su
forma de hablar. Algunas de estas diferencias son idiosincrásicas, pero otras están sis-
temáticamente asociadas con determinados grupos sociales. El ejemplo más evidente
lo ofrecen las variaciones vinculadas con el sexo y la edad: hablan de manera diferente
las mujeres y los hombres, los niños y los adultos."
G. Guy: Lenguaje y clase social.

TAMadrid
11
-Pág.43-
L E N G UA C A S T E L L A N A Y L I T E R AT U R A

- LA ARGUMENTACIÓN
Se utiliza especialmente en :
- Textos de filosofía.
- Crítica y artículos periodísticos de opinión.
- Crítica literaria, etc.
Ejemplo de texto escrito argumentativo:

" Al parecer, la Academia de la Lengua ha decidido, aconsejado o sugerido (...) que en lugar de
Whiski se escriba "güisqui". El acuerdo ha dado lugar a muchos artículos en contra, burlones o
sarcásticos, en su mayoría. Esta reacción no deja de ser curiosa, puesto que la novedad respon-
de a la función básica de la ortografía, que es representar con signos adecuados la pronunciación
de las palabras...
Frente al acuerdo de la Academia, podría aducirse que en algunas zonas de España (...)
el nombre de aquella destilación se pronuncia "uisqui". Muchos de los opinantes dicen
que esta palabra resulta ridícula gráficamente. Es posible....."
R. Carnicer: Tradición y evolución en el lenguaje actual.

2.2. REGISTROS Y USOS SOCIALES DE LA LENGUA ESCRITA


La lengua escrita es un reflejo de la lengua oral; en consecuencia, sus registros y usos socia-
les están muy relacionados, y son similares a los expuestos en el apartado 2.3. de la Unidad 1, por
lo que consideramos innecesaria la repetición íntegra de dicho apartado; únicamente añadiremos un
texto escrito representativo de cada nivel.
El nivel culto:
“Es de justicia que las últimas palabras sean de gratitud a los lingüistas que han
colaborado con nosotros y, sobre todo, a las Academias hermanas que, con su
diligente, minucioso y esmerado trabajo de revisión, han enriquecido el texto y
han sancionado esta obra como la Ortografía de la comunidad hispánica”.
R.A.L: Final del prólogo de ortog. de la Lª Esp.
El nivel vulgar:
“No emos llegao ha ver todas las pelicula en todo el mes de avril. La Maria dice
que ya se a artao”
Anónimo.
El nivel coloquial:
“Querida X, me llevé una una gran sorpresa cuando abrí el buzón y encontré una
postal tuya.....”
Anónimo.

12 TAMadrid

-Pág.44-
U. D. 2 . - U S O S Y F ORMAS DE LA C O M U N I C A C I Ó N E S C R I TA

2.3. FINALIDAD, SITUACIÓN Y CONTEXTO COMUNICATIVO


Si tenemos en cuenta las funciones del lenguaje (FINALIDAD), las circunstancias extralingüís-
ticas (SITUACIÓN) y el contexto lingüístico (CONTEXTO COMUNICATIVO), así como las diver-
sas estructuras textuales que hemos ido viendo a lo largo de esta unidad, podemos dar una
clasificación de tipos de discursos y de textos que constituirán las fuentes de la documentación
escrita.

TIPOS DE DISCURSOS TIPOS DE TEXTOS


SEGÚN: SEGÚN:

FINALIDAD SITUACIÓN Y ESTRUCTURA


CONTEXTO SECUENCIAL
BÁSICA

FUNCIONES INTERACCIÓN SOCIAL


SEGÚN LA SEGÚN LUGAR Y
INTENCIÓN EL CONTEXTO
COMUNICATIVO NARRATIVO
DESCRIPTIVO
EXPOSITIVO
PEDAGÓGICO ARGUMENTATIVO
INFORMATIVA CULTURAL ETC.
CONATIVA POLÍTICO
FÁTICA CIENTÍFICO
LÚDICA LABORAL
POÉTICA JURÍDICO
METALINGÜÍS- ETC.
TICA
ETC.

2.4. USOS ESPECÍFICOS DE LA LENGUA ESCRITA:


EL INFORME, LA MEMORIA, EL RESUMEN...

La relativa permanencia e inalterable significación del mensaje escrito posibilita que tenga
determinados usos que sean específicos de la lengua escrita. Algunos de éstos son:

EL INFORME: Es un documento escrito que consiste en una redacción detallada de los datos
recopilados acerca de un asunto o de una persona.

TAMadrid
13
-Pág.45-
L E N G UA C A S T E L L A N A Y L I T E R AT U R A

Características:
- Hace uso sólo de datos “fiables”.
- Información ajustada y precisa.
- Puede llevar conclusiones finales.

LA MEMORIA: Exposición escrita de hechos, datos o motivos referentes a determinado asun-


to o tema de estudio.
Características:
- Extensión determinada.
- Información pormenorizada y justificada.
- Uso de vocablos técnicos y datos numéricos que se consideren necesarios.

EL RESUMEN: Es la exposición escrita abreviada de cualquier tipo de texto.


Características:
- Debe recoger las ideas fundamentales.
- Orden lógico de las mismas.
- Cada idea fundamental, sola o con su breve explicación, debe aparecer en
párrafo diferente, etc.

2.5. LOS LENGUAJES ESPECÍFICOS: CIENTÍFICO, TÉCNICO,


JURÍDICO, ADMINISTRATIVO, HUMANÍSTICO,
PERIODÍSTICO...
Toda comunicación escrita comparte la misma lengua, pero cada ciencia, cada profesión u ofi-
cio...etc., tiene su lenguaje particular; es decir, su propio registro.
Veamos algunos de estos lenguajes específicos:
LENGUAJE CIENTÍFICO-TÉCNICO: Es el lenguaje propio de las ciencias: Física, Química,
Medicina, etc.
Características:
- Universalidad: Procura utilizar términos que faciliten su comprensión interna-
cional; para ello utiliza numerosos términos tomados del griego y del latín,
especialmente.
- Abundante uso de tecnicismos: neurona, neurita, amperímetro, voltímetro, etc.
- Claridad y precisión en las palabras.
- Sencillez sintáctica.

14 TAMadrid

-Pág.46-
U. D. 2 . - U S O S Y F ORMAS DE LA C O M U N I C A C I Ó N E S C R I TA

- Objetividad: Importa la información, no el informador.


- Verificabilidad: Que sea posible la comprobación de lo expuesto.
- Uso de fórmulas, símbolos, datos numéricos, etc.
- Constante formación de neologismos para designar avances y nuevos descu-
brimientos.
- El registro idiomático que utiliza pertenece al nivel culto.

LENGUAJE JURÍDICO-ADMINISTRATIVO: Es el lenguaje utilizado en los textos legales, jurí-


dicos y administrativos.

Características:
- Aparente ambigüedad: se presta a que se pueda interpretar de diferentes formas
por lo que, en ocasiones, se necesita la autoridad de un juez para su interpreta-
ción exacta.
- Léxico técnico y abstracto. Ej: “La audiencia procederá al recurso de casa-
ción”.
- Uso de fórmula solemnes. Ej.: “Todos los que esta carta de poder vieren....”
- Uso de cultismos de origen grecolatino: “habeas corpus”, “fecha ut supra”,
“pro indiviso”, etc.
- Construcción sintáctica compleja, con alusiones constantes a artículos, códi-
gos, etc.
- El registro idiomático que se emplea pertenece al nivel culto.

LENGUAJE HUMANÍSTICO: es el lenguaje propio de las ciencias humanas: Antropología,


Filosofía, Ética, Sociología, etc.

Características:
- Intención didáctica: Pretende enseñar, hacer pensar, etc.
- Vocabulario abstracto y de carácter globalizador: felicidad, humanidad, com-
portamiento, prototipo, etc.
- Abundantes términos de origen grecolatino: hipótesis, arqueología, paleolítico.
Superego, etc.
- Uso de textos expositivos y argumentativos de autoridad reconocida.
- Construcción sintáctica lógica que facilite la comprensión de cualquier razo-
namiento, etc.
- El registro idiomático que se emplea pertenece al nivel culto.

TAMadrid
15
-Pág.47-
L E N G UA C A S T E L L A N A Y L I T E R AT U R A

LENGUAJE PERIODÍSTICO: Es el lenguaje propio de la prensa escrita: periódicos, revistas, etc.

Características:
- En general, utiliza un lenguaje común, estándar (la prensa intenta llegar a todos
los públicos).
- Las noticias aparecen redactadas en forma impersonal, no aparece la primera
persona, salvo en las citas textuales: Ej.: el candidato dijo: “me presento a las
próximas elecciones”.
- Los temas presentados determinarán el grado de formalidad en el tratamiento
de la noticia.
- Utiliza un lenguaje orientativo, persuasivo; en numerosas ocasiones, manipu-
lador: intenta dirigir la opinión de la masa; la pretendida objetividad de la pren-
sa no siempre es real.
- Aunque su registro idiomático pertenece al nivel estándar, en algunas seccio-
nes informativas se hace uso de otros registros: científico, técnico, humanísti-
co, literario, etc.

3. DIVERSIDAD LINGÜÍSTICA Y VARIEDADES


DIALECTALES DE LA LENGUA ESCRITA

Antes de entrar en la exposición de este bloque, debemos hacer una consideración especial refe-
rente a la lengua escrita: “la lengua escrita es más normativa y conservadora que la lengua oral
y, por tanto, menos predispuesta a los cambios, y es el referente principal para que la lengua
oral sea más unificada”.
La diversidad lingüística es más notoria en la forma hablada que en la escrita; podemos obser-
var que personas que se expresan oralmente con formas “peculiares”, cuando lo hacen de forma
escrita suelen respetar las normas lingüísticas del idioma y sus diferencias son prácticamente insig-
nificantes.
En el bloque 3 (apartados 3.1., 3.2. y 3.3.) de la Unidad 1 se expone, de forma extensa, la diver-
sidad lingüística de España y la importancia de sus lenguas en el mundo, por lo que resulta impro-
cedente volver a repetir los mismos contenidos; en su lugar presentaremos textos escritos referentes
a las diversas lenguas y algunos ejemplos del uso del Español en determinadas zonas de América.

3.1. LA REALIDAD PLURILINGÜE DE ESPAÑA


Los textos que vamos a ver son fragmentos pertenecientes a las lenguas que tienen el reconoci-
miento de Lenguas o Idiomas “oficiales”.

16 TAMadrid

-Pág.48-
U. D. 2 . - U S O S Y F ORMAS DE LA C O M U N I C A C I Ó N E S C R I TA

CASTELLANO:
Desnuda está la tierra,
Y el alma aúlla al horizonte pálido
Como loba famélica. ¿Qué buscas,
Poeta, en el ocaso?
(Antonio Machado: Soledades, galerías y otros poemas)
CATALÁN:
La vella i frágil plata esdeve tarda
Parada en la clamor damunt els camps.
(Salvador Espriu: La pell de Brau (la piel de toro))
La vieja y frágil plata se convierte en tarde
Detenida en la claridad sobre los campos.
GALLEGO:
Quero morrer eiqui (cando me chegue
A hora do viaxe que me agarda).
(Celso Emilio Ferreiro: Eiquí será ( Aquí será)).
Quiero morir aquí (cuando me llegue
La hora del viaje que me espera).
EUSKERA:
Hiltzen naizenean egonen da
Nire lauzaren gainean eskibu hau:
(Gabriel Aresti: Harri eta Herri (Piedra y pueblo)).
Cuando yo me muera se podrá leer
La siguiente inscripción encima de mi tumba:
VALENCIANO:
Dorm, dorm, dorm, ma estrelleta
meva filleta dolça.
Dorm mentres que la mareta
canta i ve.la.
(Anónimo: Canción de cuna.)
Duerme, duerme, duerme, mi estrellita
mi hijita dulce.
Duerme mientras que la madre
canta y vela.

TAMadrid
17
-Pág.49-
L E N G UA C A S T E L L A N A Y L I T E R AT U R A

3.2. FENÓMENOS DE CONTACTO ENTRE LAS DISTINTAS


LENGUAS
Los fenómenos de contacto entre las distintas lenguas suelen darse, normalmente, en la moda-
lidad oral; en la modalidad escrita suele utilizarse la lengua que se considera más importante por los
beneficios que pueda reportar: prestigio social, cultural, profesional, etc.

3.3. USO Y DIFUSIÓN INTERNACIONAL DEL ESPAÑOL


Y DE LAS OTRAS LENGUAS DE ESPAÑA
La reciente edición de la Ortografía de la Lengua Española, revisada por la Asociación de las Aca-
demias de la Lengua Española, es muestra inequívoca de la importancia de nuestra lengua en el mundo.
En América, lo mismo que en España, hay algunas diferencias en el uso de la lengua que son
debidas a las particularidades específicas de cada zona, pero que no son realmente significativas en
el contexto general de la lengua.
Algunas de estas diferencias son:
- El voseo: Utiliza “vos” en vez de tú y de ti.
- Utilizar “ustedes” en vez de vosotros.
- Etc.
Veamos algunos textos representativos del Español escrito en América; nos llamarán la aten-
ción algunas expresiones.
“Durante quince días el alazán había buscado en vano la senda por donde su
compañero se escapaba del potrero. El formidable cerco, de capuera -desmon-
tante que ha rebrotado inextricable -, no permitía paso ni aun a la cabeza del
caballo. Evidentemente no era por donde el malacara pasa”
Horacio Quiroga: Cuentos de la Selva.
“Cauteloso me acerqué
A un indio que estaba al lao.
Porque el pampa es desconfiao
Siempre de todo cristiano:
Y vi que tenía en la mano
El rebenque ensangrentao”.
(José Hernández: Martín Fierro.)
“Después, cuando la bailarinita dio el saltito que da siempre cuando se le acaba
la cuerda....”
Gabriel García Márquez: La hojarasca.
“- Lo que se me ocurría es que podías hablar con tu patrón; algo me podrá ofre-
cer. Alguna chamba”.
Carlos Fuentes: Cantar de ciegos.

18 TAMadrid

-Pág.50-
U. D. 2 . - U S O S Y F ORMAS DE LA C O M U N I C A C I Ó N E S C R I TA

USOS DE LAS FUENTES DE DOCUMENTACIÓN


4. ESCRITA
Entendemos como fuente de documentación escrita a cualquier obra que nos permita obtener
información para satisfacer diversas necesidades: curiosidad, conocer, extraer datos para realizar un
trabajo, acumular información sobre un tema o un personaje, etc.
Podemos considerar dos tipos de fuentes de documentación escrita:
DIRECTAS: Las que podemos utilizar para obtener la información que deseamos: manuales,
guías, diccionarios, enciclopedias, biografías, etc.
INDIRECTAS: Las que nos conducen a las fuentes directas y nos orientan sobre las que más nos
pueden convenir para obtener la información que precisamos: boletines, bibliografías, catálogos de
editoriales, guías de archivos y museos, etc.
La consulta de las fuentes de información resulta imprescindible para la realización sistemática
y planificada en la elaboración de cualquier estudio o trabajo.

RESUMEN

• Comunicación escrita: La escritura ha evitado que la “distancia” fuera un obstáculo infran-


queable para la comunicación humana.
• El documento escrito es una fuente de información “fiable”: su mensaje no varía, siempre
tiene la misma interpretación.
• La lengua escrita reporta prestigio social y potencia las posibilidades de promoción humana.
• El Español escrito, especialmente en la literatura, goza de reconocimiento universal.

TAMadrid
19
-Pág.51-
L E N G UA C A S T E L L A N A Y L I T E R AT U R A

EJERCICIOS DE AUTOCOMPROBACIÓN

11. La frase “¡Qué bien jugó el portero visitante!”, pertenece a la función:

A. Expresiva B. Conativa

C. Poética D. Interrogativa

12. ¿Cuántos consonantes componen el alfabeto español, desde 1994?

A. 20 B. 21

C. 22 D. 23

3. ¿Cuál de las siguientes palabras está escrita correctamente?

A. abano B. habano

C. avano D. hábano

4. En la crítica literaria, se emplea particularmente:

A. La descripción B. La exposición

C. La argumentación D. La narración

5. ¿Cuál de las siguientes palabras está escrita correctamente?

A. Deshabituar B. Desavituar

C. Deshavituar D. Desabituar

6. La exposición escrita abreviada de cualquier tipo de texto, se denomina:

A. Informe B. Resumen

C. Argumentación D. Memoria

7. El lenguaje que se caracteriza por utilizar abundantemente los tecnicismos es:

A. Lenguaje científico-técnico B. Lenguaje humanístico

C. Lenguaje periodístico D. Lenguaje vulgar

20 TAMadrid

-Pág.52-
U. D. 2 . - U S O S Y F ORMAS DE LA C O M U N I C A C I Ó N E S C R I TA

8. La lengua más antigua de España es:

A. El Vasco B. El castellano

C. El Catalán D. El Gallego

9. ¿Cuál de las siguientes letras no representa ningún sonido?

A. Z B. V

C. H D. B

10. ¿Cuál de las siguientes palabras está escrita correctamente?

A. escluir B. exhaustivo

C. esperimental D. escepcional

TAMadrid
21
-Pág.53-
L E N G UA C A S T E L L A N A Y L I T E R AT U R A

RESPUESTAS A LOS EJERCICIOS

1. A

2. C

3. B

4. C

5. A

6. B

7. A

8. A

9. C

10. B

22 TAMadrid

-Pág.54-
U. D. 2 . - M E D I D A , NOTAS
E S T I M AC I Ó N Y C Á L C U L O D E M AG N I T U D E S

-Pág.54-
-Pág.55-
U. D. 2 . - M E D I D A , NOTAS
E S T I M AC I Ó N Y C Á L C U L O D E M AG N I T U D E S

-Pág.54-
-Pág.55-
portada TROPA 19/3/07 19:51 Página 1

FUERZAS ARMADAS
PROFESIONALES
CURSO DE APOYO
A LA PREPARACIÓN
DE LAS PRUEBAS DE ACCESO
A UNA RELACIÓN DE SERVICIOS
DE CARÁCTER PERMANENTE

LENGUA CASTELLANA Y LITERATURA


2ª parte
Unidades didácticas 3 y 4

DIGEREM

MINISTERIO
DE DEFENSA
FUERZAS ARMADAS SUBDIRECCIîN GENERAL
DE TROPA Y MARINERIA
PROFESIONAL
PROFESIONALES
CURSO DE APOYO
A LA PREPARACIÓN
DE LAS PRUEBAS DE ACCESO
A UNA RELACIÓN DE SERVICIOS
DE CARÁCTER PERMANENTE

LENGUA CASTELLANA Y LITERATURA


2ª parte
Unidades didácticas 3 y 4
La Ley 8/2006 de Tropa y Marinería, en su artículo 16,1, establece que “la formación
en las Fuerzas Armadas garantizará que los militares profesionales de tropa y
marinería puedan adquirir, actualizar o ampliar sus conocimientos para un mayor
desarrollo personal y profesional”. En cumplimiento de este mandato, el Ministerio
de Defensa edita el presente material didáctico para facilitar a los militares
profesionales de tropa y marinería, alumnos de los cursos de formación
presencial que se imparten a través de la Dirección General de Reclutamiento y
Enseñanza Militar, los apoyos necesarios para preparación de dichos cursos, que
permitirán, siempre que superen las pruebas correspondientes, la obtención de la
titulación de graduado en Educación Secundaria, acreditación para el acceso a
los ciclos formativos de la Formación Profesional de grado medio o de grado
superior, acceso a las Escalas de Suboficiales, Tropa Permanente, Guardia Civil
y Policía Nacional.

CATÁLOGO GENERAL DE PUBLICACIONES


http://www.060.es

Edita:

© Autor y editor
NIPO: 076-10-204-9 NIPO: 076-10-205-4 (edición en línea)
Depósito Legal: M-32363-2009
Diseño y programación: cimapress
Tirada: 1300 ejemplares
Fecha de edición: septiembre, 2010

Prohibida la reproducción total o parcial de esta obra, por cualquier medio sin autorización escrita del editor
LENGUA CASTELLANA Y LITERATURA
2ª parte

SUMARIO

Unidad didáctica Pág.

3. LA LENGUA COMO OBJETO DE CONOCIMIENTO 5

4. SISTEMAS DE COMUNICACIÓN VERBAL Y NO VERBAL 29


U.D. 3.- LA LENGUA COMO OBJETO DE CONOCIMIENTO

ÍNDICE
OBJETIVOS . . . . . . . . . . . . . . . . . . . . . . . . . . . . . . . . . . . . . . . . . . . . . . . . . . . . . . . . . . . . .2

INTRODUCCIÓN . . . . . . . . . . . . . . . . . . . . . . . . . . . . . . . . . . . . . . . . . . . . . . . . . . . . . . . . . .3

MAPA CONCEPTUAL . . . . . . . . . . . . . . . . . . . . . . . . . . . . . . . . . . . . . . . . . . . . . . . . . . . . .4

DESARROLLO DE CONTENIDOS

1. LA LENGUA COMO PRODUCTO Y PROCESO SOCIAL Y CULTURAL


EN CAMBIO PERMANENTE . . . . . . . . . . . . . . . . . . . . . . . . . . . . . . . . . . . . . . . . . .5

2. LA NORMA LINGÜÍSTICA . . . . . . . . . . . . . . . . . . . . . . . . . . . . . . . . . . . . . . . . . . .6

3. EL DISCURSO COMO UNIDAD DE SENTIDO Y UNIDAD FORMAL.


PARTES DEL DISCURSO. RELACIÓN TEXTO-CONTEXTO.
ADECUACIÓN Y COHESIÓN TEXTUAL . . . . . . . . . . . . . . . . . . . . . . . . . . . . . . .7

4. ORACIÓN SIMPLE Y ORACIÓN COMPUESTA. TIPOS DE ORACIONES.


CONSTITUYENTES ORACIONALES . . . . . . . . . . . . . . . . . . . . . . . . . . . . . . . . . .9

5. RELACIONES MORFO-SINTÁCTICAS. CONCORDANCIAS.


CLASES DE PALABRAS . . . . . . . . . . . . . . . . . . . . . . . . . . . . . . . . . . . . . . . . . . . . .12

6. VOCABULARIO. SENTIDO PROPIO Y FIGURADO. POLISEMIA,


HOMONIMIA, SINONIMIA Y ANTONIMIA. FAMILIAS LÉXICAS
Y CAMPOS SEMÁNTICOS. MODISMOS, LOCUCIONES Y FRASES
HECHAS. NEOLOGISMOS, EXTRANJERISMOS, SEXIMOS. . . . . . . . . . . . .15

7. NORMAS ORTOGRÁFICAS. ORTOGRAFÍA DEL DISCURSO. ORTOGRAFÍA


DE LA ORACIÓN. ORTOGRAFÍA DE LA PALABRA . . . . . . . . . . . . . . . . . . . .19

RESUMEN . . . . . . . . . . . . . . . . . . . . . . . . . . . . . . . . . . . . . . . . . . . . . . . . . . . . . . . . . . . .22

EJERCICIOS DE AUTOCOMPROBACIÓN . . . . . . . . . . . . . . . . . . . . . . . . . . . . . . . . . .23

RESPUESTAS A LOS EJERCICIOS . . . . . . . . . . . . . . . . . . . . . . . . . . . . . . . . . . . . . . . . . 24

-Pág.5-
L E N G U A Y L I T E R AT U R A

OBJETIVOS
Al finalizar el estudio de esta Unidad Didáctica, el alumno será capaz de:

• Valorar la norma lingüística.

• Reconocer y diferenciar las oraciones simples y compuestas.

• Distinguir el sentido de los vocablos según el texto-contexto.

• Conocer y aplicar las normas ortográficas de la Lengua Castellana.

-Pág.6-
U.D. 3.- LA LENGUA COMO OBJETO DE CONOCIMIENTO

INTRODUCCIÓN
C omo sabemos, todas las personas nacemos con la capacidad de comunicación
mediante el lenguaje, pero cada individuo lo manifiesta a través de los signos y
reglas que ha aprendido en la comunidad en la que ha nacido o vivido; es decir, de la
“lengua o idioma” de su colectividad.

En el mundo existen más de tres mil quinientas lenguas diferentes, distribuidas de forma
muy desigual en cuanto al número de hablantes; la nuestra: el Castellano, una de las más
importantes, es compartida “oficialmente” por unos cuatrocientos millones de hablantes,
que, además de permitir comunicarnos “sin barreras”, nos enseña a pensar de forma
similar.

Cuanto mejor se conoce una lengua, mayor es la facilidad para pensar, expresarse y com-
prender.

CONSTITUCIÓN ESPAÑOLA
ARTÍCULO 3
1. EL CASTELLANO ES LA LENGUA OFICIAL
DEL ESTADO. TODOS LOS ESPAÑOLES TIE-
NEN EL DEBER DE CONOCERLA Y EL DERE-
CHO DE USARLA.

-Pág.7-
L E N G U A Y L I T E R AT U R A

M A PA C O N C E P T UA L
LENGUA CASTELLANA Y
LITERATURA

LA LENGUA COMO OBJETO


DE CONOCIMIENTO

PRODUCTO Y PROCESO
LA NORMA LINGÜÍSTICA
SOCIAL Y CULTURAL

EL TEXTO

RELACIÓN ADECUACIÓN Y SECUENCIA TEXTUAL


TEXTO-CONTEXTO COHESIÓN TEXTUAL ORACIÓN - PALABRAS

MORFOLOGÍA SEMÁNTICA ORTOGRAFÍA


Y SINTAXIS (VOCABULARIO) (NORMAS)

-Pág.8-
U.D. 3.- LA LENGUA COMO OBJETO DE CONOCIMIENTO

LA LENGUA COMO PRODUCTO Y PROCESO SOCIAL


1. Y CULTURAL EN CAMBIO PERMANENTE
Lengua, cultura y sociedad son tres conceptos tan estrechamente relacionados entre sí, que
resulta difícil definir cualquiera de ellos sin mencionar a los otros dos.
Las sociedades evolucionan, los modos de vida varían de unas épocas a otras, se producen
nuevos descubrimientos, continuamente surgen inventos, las culturas cambian, y las lenguas,
consecuentemente, permanecen “abiertas” a esos cambios: el vocabulario es cada vez más
amplio, aparecen nuevas formas de comunicación, etc.
Las lenguas se pueden estudiar desde dos puntos de vista: “Sincrónico” y “Diacrónico”, a
saber:
– Punto de vista sincrónico: estudio de una lengua en un momento determinado.
– Punto de vista diacrónico: estudio de una lengua a lo largo de su historia: origen, evo-
lución, etc.
En este apartado nos interesa, fundamentalmente, el punto de vista diacrónico, ya que nos
permitirá poder conocer cómo y cuándo se produjo el nacimiento (origen) del castellano y el
proceso formativo (evolución) hasta la configuración actual.
El Castellano, lengua romance, románica o neolatina por ser procedente del latín, surge en
Cantabria (aproximadamente en el siglo X), donde existían varios condados con fuertes casti-
llos que eran dependientes del reino de León; y “El habla de los hombres de los castillos” fue
lo que dio su nombre: “CASTELLANO”.
Ya, en ese siglo, aparecen los primeros textos en lengua castellana; son las glosas silenses
del monasterio de Silos (Burgos) y las glosas emilianenses del monasterio de San Millán de la
Cogolla (Logroño). Son anotaciones en romance de algunos términos latinos, que nos permiten
afirmar que ya se estaba sustituyendo el latín por la nueva lengua: “El Castellano”.
Su evolución la podemos seguir a través de:
– Los primeros textos escritos íntegramente en lengua romance que se conservan son de
finales del siglo XI y siglo XII, se trata de documentos sobre concesiones, contratos de
compraventa y algunos de tipo literario: las jarchas = cancioncillas en mozárabe, la len-
gua que hablaban los cristianos que vivían en las zonas que dominaban los árabes.
– El Cantar de Mío Cid, que presenta características peculiares de la zona soriana y de
la vecina Aragón. (Siglo XII).
– En el siglo XIII: Los Milagros de Nuestra Señora, obra en verso de Gonzalo de Berceo,
y las diversas obras en prosa que se escriben bajo la tutela de Alfonso X el Sabio.

5
-Pág.9-
L E N G U A Y L I T E R AT U R A

– En el siglo XIV: El conde Lucanor, obra en prosa de don Juan Manuel, y en verso, el
Libro de Buen Amor del Arcipreste de Hita, Juan Ruiz.
– En el siglo XV: Aparece la primera “Gramática castellana” de Elio Antonio de Nebrija.
– En el siglo XVI: El castellano adquiere un gran prestigio internacional y empieza a
denominarse con el término de “ESPAÑOL”. Hacia la mitad del siglo se inicia el
denominado Siglo de Oro, que se prolongará hasta casi el último cuarto del siglo
siguiente.
– En el siglo XVII: Culmina el citado Siglo de Oro. Se publica el diccionario “Tesoro
de la lengua castellana o española” de Sebastián de Covarrubias, y Gonzalo Correas
compone: “Vocabulario de refranes y frases proverbiales” y una “Gramática”. En este
siglo queda fijado el sistema fonético-fonológico del castellano.
– En el siglo XVIII: Se funda la Real Academia Española, que publica: el “Diccionario
de Autoridades”, la “Ortografía” y la “Gramática” en distintas épocas de ese siglo.
– En el siglo XIX: Se establece la obligatoriedad del español en todos los niveles de la
enseñanza.
– En el siglo XX: Se fija la Ortografía, recientemente se ha llevado a cabo una revisión
de la misma por todas las Academias del español.
Se ha producido un proceso de unificación lingüística nacional e internacional, gracias a los
medios de comunicación.
Y, acaso, lo más importante: se ha generalizado el sentimiento de que social y culturalmen-
te, la lengua Española es el patrimonio que compartimos todos los hispano-hablantes por igual.

2. LA NORMA LINGÜÍSTICA
La norma lingüística es el conjunto de reglas para hablar y escribir con corrección.
El código de la lengua está a disposición de los hablantes, la norma nos “obliga” a usar
correctamente el código; es decir, el código lingüístico es un conjunto sistematizado de signos
y reglas, mientras que la norma lingüística impone condiciones (excepciones, preferencias, etc.)
para el uso correcto del citado código.
Esta imposición de condiciones normativas son debidas a motivos históricos, costumbres
generalizadas de usos peculiares, expresiones tradicionales, la etimología de las palabras, etc.

6
-Pág.10-
U.D. 3.- LA LENGUA COMO OBJETO DE CONOCIMIENTO

Un texto puede aparecer como correcto para el código e incorrecto para la norma. Ej.: “Ayer
‘andé’ por la calle nueva”; la forma “andé” según la regla general sería correcta, mientras que
la norma nos impone la forma “anduve”.
El código no impide utilizar ciertos vocablos en cualquier situación. Ej.: “Los regüeldos fue-
ron sonoros”; en cambio para la norma, en algunas situaciones, sería una expresión de mal
gusto, y aconsejaría la expresión: “Los eructos fueron sonoros”. Vemos, por tanto, que la norma
también impone criterios selectivos de uso en cuanto al vocabulario.
Estos desvíos excepcionales de la lengua que la norma impone son aceptados plenamente
por los hablantes, y el conocimiento de los mismos resulta imprescindible para el buen uso de
una lengua.

EL DISCURSO COMO UNIDAD DE SENTIDO Y UNIDAD


3. FORMAL. PARTES DEL DISCURSO. RELACIÓN TEXTO-
CONTEXTO. ADECUACIÓN Y COHESIÓN TEXTUAL
En gramática, entendemos el discurso como la unidad lingüística superior a la oración, for-
mada por un conjunto coherente de ellas y que constituyen una elocución hablada o escrita.
La citada elocución se consigue con una precisa elección y correcta disposición de las pala-
bras que determinarán la formación de un texto que exprese un sentido completo.
Tradicionalmente, se ha entendido el discurso como un texto argumentativo oral en el que
se realiza una exposición interesadamente razonada para convencer a un auditorio: un ser-
món, una arenga, un mitin, etc. Pero, también puede darse como un texto escrito: las cartas
que suelen enviar los partidos políticos en campañas electorales a los electores para recabar
sus votos.
Incluso, como se puede ver en algunos manuales de lengua, actualmente se tiende a identi-
ficar discurso y texto, siempre que éste presente una intencionalidad semántica (unidad de sen-
tido) y una estructura mínimamente organizada (unidad formal).

PARTES DEL DISCURSO


En la estructura de cualquier discurso, siempre se podrán distinguir, claramente, las siguien-
tes partes:
INTRODUCCIÓN O PLANTEAMIENTO: en la que se da a conocer el tema que se va a
exponer y las motivaciones que han aconsejado su elección.

7
-Pág.11-
L E N G U A Y L I T E R AT U R A

CUERPO O DESARROLLO: es la parte más extensa del discurso, donde se argumenta


razonadamente desde el enfoque personal del autor; éste intenta formar y reconducir la opinión
del público en consonancia a su tesis.
CONCLUSIÓN: parte final del discurso; suele ser breve; en ella, el autor vuelve a insistir
en las ideas básicas que ha expuesto en la parte fundamental del discurso.

RELACIÓN TEXTO-CONTEXTO
El mensaje que todo texto intenta transmitir va a depender, en muchas ocasiones, del con-
texto en el que se emite.
En las Unidades 1 y 2, explicábamos el “CONTEXTO”, incluso diferenciábamos entre
contexto lingüístico (que pertenecía a la composición del propio texto) y contexto situacional
(conjunto de circunstancias extralingüísticas que ayudaban a interpretar el texto).
A veces, el contexto lingüístico no es suficiente para la comprensión de un texto. Veamos el
siguiente:
“Los del 7 están felices; esperemos que lo estén por mucho tiempo; da gusto verlos así; real-
mente lo merecían por su paciencia infinita”.
El contexto lingüístico no nos permite interpretar , en este caso, perfectamente el mensaje;
necesitamos de algunos elementos extralingüísticos (contexto situacional) para conocer el ver-
dadero significado del texto. Este podría ser múltiple, ya que depende del lugar donde se haya
producido:
– Grupo de vecinos que viven en el nº 7 de una calle.
– Grupo de abonados del tendido nº 7 de una plaza de toros, etc.

ADECUACIÓN Y COHESIÓN TEXTUAL


Adecuación es la elección idónea de los elementos lingüísticos necesarios para construir un
texto y que se acomodan de la forma más conveniente a un determinado contexto para conse-
guir que la interpretación del mensaje sea lo más conveniente posible.
La coherencia semántica (significación global lógica) de un texto se consigue gracias a su
cohesión lineal (relación sintáctico-semántica) que se establece entre los elementos que com-
ponen las oraciones y párrafos que constituyen el mismo.
Las oraciones que forman un texto no son unidades de comunicación aisladas, sino que entre
ellas existen diversas relaciones que permiten que sus significados se complementen y formen
un todo: “el texto”.

8
-Pág.12-
U.D. 3.- LA LENGUA COMO OBJETO DE CONOCIMIENTO

Estas relaciones de cohesión interna se consiguen mediante:


– Repetición de algunos elementos, bien directamente o a través de sinónimos, pronombres,
adverbios, etc. Ej.: “Los chicos estaban en la piscina. Ellos comieron allí”.
– La “deixis”, que consiste en relacionar ciertos elementos del texto con otros referentes al
contexto. Son elementos deícticos los pronombres personales y demostrativos, y los
adverbios de lugar y tiempo. Ej.: “Aquí, en Madrid está Barajas”.
– Los conectores o nexos: conjunciones, locuciones, adverbios, etc., que unen las oraciones
y les proporcionan una secuencia lógica. Ej.: “La niña había llegado a casa. Previamente,
su padre le había preparado...”

ORACIÓN SIMPLE Y COMPUESTA. TIPOS


4. DE ORACIONES. CONSTITUYENTES ORACIONALES
Oración es la unidad lingüística, formada por una palabra o grupo de palabras, que tiene sen-
tido completo.
Cada oración tiene su propia entonación; en la forma oral está determinada por pausas, y en
la forma escrita, por comas.

ORACIÓN SIMPLE
Es la que consta de un solo sujeto y un solo predicado.
Ej.: Juan (Sujeto) llegó ayer de Asturias (Predicado).
¡Ojo! Las oraciones impersonales no tienen Sujeto.
Ej.: Ayer llovió mucho (Predicado).

ORACIÓN COMPUESTA
Es la que en su estructura, aparecen dos o más predicados con sus correspondientes sujetos.
Ej.: Pedro (Sujeto) llegó ayer (Predicado) y su padre (Sujeto) vendrá hoy (Predicado).
Esta oración, a pesar de que tenga dos sujetos y dos predicados, sólo tiene una entonación,
la que corresponde desde su inicio hasta el punto.
Cada grupo, formado por un sujeto y su correspondiente predicado, recibe el nombre de “Pro-
posición” y no posee significado independiente, ya que la unidad de entonación corresponde a
la oración.

9
-Pág.13-
L E N G U A Y L I T E R AT U R A

TIPOS DE ORACIONES

SIMPLES
Las clasificaremos según la actitud del hablante y según la naturaleza del predicado.
A) SEGÚN LA ACTITUD DEL HABLANTE:
– ENUNCIATIVAS: Informan objetivamente. Pueden ser afirmativas o negativas.
Enunciativa afirmativa: El coche es azul.
Enunciativa negativa: El coche no es azul.
– INTERROGATIVAS: Van entre signos de interrogación y se utilizan para preguntar.
Pueden ser parciales o totales.
Interrogativa parcial: ¿Cuánta fiebre tiene el enfermo?- (respuesta única).
Interrogativa total: ¿Viene Hoy Juan?- (respuesta: sí, no, quizás, etc.).
– DUBITATIVAS: Expresan duda, posibilidad o probabilidad.
Quizá esté allí mi padre.
– DESIDERATIVAS: Expresan un deseo.
Ojalá tengamos suerte mañana.
– EXCLAMATIVAS: Van entre signos de exclamación y expresan estados de ánimo: sor-
presa, alegría, enfado, etc.
¡Qué golpe se han dado esos coches!
B) SEGÚN LA NATURALEZA DEL PREDICADO:
– ATRIBUTIVAS (O COPULATIVAS): Cuando el verbo funciona como nexo o cópula
entre el sujeto y el atributo; son los verbos ser, estar y parecer cuando carecen de signifi-
cación léxica. En estas oraciones el predicado es “nominal”.
– Luis es enfermero.
– Luis está enfermo.
– PREDICATIVAS: Cuando el verbo tiene significación léxica. En estas oraciones el pre-
dicado es “verbal”. Pueden ser activas y pasivas, según vaya el verbo en voz activa o en
voz pasiva.
a) ACTIVAS: El sujeto realiza la acción. Se clasifican a su vez en:
– Transitivas: Si tienen complemento directo. Comeremos fruta (C.D.).

10
-Pág.14-
U.D. 3.- LA LENGUA COMO OBJETO DE CONOCIMIENTO

– Intransitivas: Si no tienen complemento directo. Iremos a Lugo.


– Reflexivas: Si el sujeto que realiza la acción recibe el daño o provecho de la
misma. María se lava las manos.
– Recíprocas: Las dos o más personas del sujeto realizan la acción y reciben el
daño o provecho de la misma de forma mutua. Luis y Juan se vieron ayer.
b) PASIVAS: El sujeto no realiza la acción, sino que la recibe (sujeto paciente). Dicha
acción es realizada por el complemento agente que aparece en el predicado. El enfer-
mo (sujeto paciente) fue operado por el médico (complemento agente).
Las oraciones pasivas reflejas se construyen con el pronombre “se” más un verbo en
activa, pero su significado es pasivo. El cristal (sujeto paciente) se ha roto (predicado).

COMPUESTAS
Se clasifican en:
– COORDINADAS: Las formadas por dos o más proposiciones unidas por conjunciones o
locuciones conjuntivas. Cada proposición tiene su propia estructura sintáctica. Juan viene
hoy e Inés llegará mañana.
– SUBORDINADAS: La proposición o proposiciones subordinadas van incrustadas en la
oración principal. Por esa dependencia sintáctica, algunos autores las denominan “com-
plejas”. Iremos (principal) cuando podamos (proposición subordinada).
– YUXTAPUESTAS: Son las oraciones formadas por proposiciones que van unidas sin
nexos, en su lugar aparece una pausa (coma) y su unidad oracional viene dada por la ento-
nación. No lo hagas, piensa un poco.

CONSTITUYENTES ORACIONALES
Denominamos constituyentes oracionales a los sintagmas (palabra o grupo de palabras sin
significado completo) que desempeñan una función en la oración.
En principio, toda oración está constituida por dos sintagmas: Sintagma nominal sujeto
(Sujeto) y Sintagma predicado verbal (Predicado).
SUJETO: Es la persona, animal, cosa, idea, etc., de los cuales se dice algo. Su núcleo (N)
es el nombre o palabra que funciona como tal. En algunas oraciones puede estar omitido. Tene-
mos mucha prisa (se sobrentiende “nosotros”).
PREDICADO: Lo que decimos del sujeto. Su núcleo (N) es el verbo.
Además, en la oración, pueden aparecer otros sintagmas (complementos):
– Sintagma adjetivo: Su función es la de complementar al sustantivo.

11
-Pág.15-
L E N G U A Y L I T E R AT U R A

– Complemento directo (C. D.): El sintagma que complementa a los verbos transiti-
vos. Andrés tiene dos hijos (C. D.).
– Complemento indirecto: El sintagma que designa al beneficiario o perjudicado de la
acción del verbo. Andrés llevó un libro a Luis (C. I.).
– Complemento agente: Indica en la voz pasiva quién realiza la acción del verbo. El
coche fue pintado por su dueño (C. Agente).
– Complementos circunstanciales: Que designan toda la gama posible de circunstan-
cias (tiempo, modo, lugar, etc.).
– Complemento predicativo: Es un complemento que, en las oraciones predicativas,
complementa conjuntamente al sujeto o al C. D. y al verbo. Luisa paseaba pensa-
tiva (C. Pvo.).
– Atributo: Es un complemento que, en las oraciones atributivas, complementa con-
juntamente al sujeto y al vebo. Madrid es una gran ciudad (Atributo).

5. RELACIONES MORFO-SINTÁCTICAS.
CONCORDANCIAS. CLASES DE PALABRAS

RELACIONES MORFOSINTÁCTICAS
Especialmente, nos interesa estudiar las relaciones posibles entre las proposiciones que
constituyen las oraciones compuestas.
RELACIÓN DE COORDINACIÓN: Va a depender del tipo de nexo (conjunción o locu-
ción conjuntiva) que una a las proposiciones, a saber:
Copulativa: Sus significados se suman. Sus nexos: y, e, ni y que.
Luis viene hoy e Inés llegará mañana.
Disyuntivas: Expresan la posibilidad de elegir entre dos opciones. Sus nexos: o, u y o bien.
¿Salimos ya o comemos antes?
Adversativas: Indican oposición entre las dos proposiciones. Sus nexos: mas, sino, no obs-
tante, sin embargo, etc.
Es muy inteligente; sin embargo, no estudia.

12
-Pág.16-
U.D. 3.- LA LENGUA COMO OBJETO DE CONOCIMIENTO

Distributivas: Expresan alternancia.


Sus nexos: bien...bien, ora...ora, ya...ya, unos...otros, etc.
Bien canta, bien ríe.
Explicativas: La segunda proposición explica, amplía o aclara el significado de la primera.
Sus nexos: es decir, esto es, o sea, etc.
Los pájaros vuelan; es decir, tienen alas.

RELACIÓN POR SUBORDINACIÓN: Al ir formando parte la proposición subordinada


de la oración principal, su relación resulta algo más compleja; veamos sus clases:

Sustantivas: La proposición subordinada desempeña la función de un sustantivo en la prin-


cipal, por lo que pueden ser de:
– Sujeto: Que vengas a casa (tu venida - Sujeto) es agradable.
– C. D. : Quiero que me acompañes (tu compañía - C. D.).
– Complemento del nombre: Tengo la esperanza de que volverá (de su regreso - C. N).
– Complemento del adjetivo: Está feliz de verte (de tu presencia - C. Adj.).

Adjetivas: Desempeñan la función propia del adjetivo; suelen estar introducidas por los
pronombres relativos que, el cual, quien, cuyo y sus variantes quienes, los cuales, etc.
Estos pronombres relativos se refieren al nombre que suele ir expresado con anterioridad.
Pueden ser:
– Explicativas: Van entre pausas (comas). El niño, el cual tú conoces, es simpático.
– Especificativas: Sin pausas (sin comas). La niña que vemos es americana.
Adverbiales: Desempeñan la función de un adverbio o complemento circunstancial.
Pueden ser de:
– Modo: Indican la manera en que se realiza la acción principal de la oración. Suelen
ir introducidas por los nexos: Según, según que, como, como si, etc. Lo hice como tú
dijiste.
– Lugar: Indican circunstancia de lugar. Sus nexos: Donde, por, a, de, etc. Déjalo
donde te he dicho.
– Tiempo: Indican circunstancia temporal. Sus nexos: Cuando, mientras, en cuanto
que, etc. Cantaba cuando trabajaba.
– Causales: Indican la causa o motivo por lo que se da la acción principal. Sus nexos:
Porque, pues, ya que, etc. Lo dejé porque me cansaba.

13
-Pág.17-
L E N G U A Y L I T E R AT U R A

– Finales: Indican la finalidad de la principal. Sus nexos: A, para que, con el objeto de
que, etc. Grita fuerte para que te oigan.
– Consecutivas: Indican la consecuencia de la principal. Sus nexos: Tal...que,
tanto...que, por consiguiente, así que, etc. Hemos acabado, por consiguiente, nos
vamos.
– Condicionales: Indican una condición. Sus nexos: Si, como, caso de que, siempre
que, etc. Te llevo al cine si terminas tu trabajo.
– Comparativas: Sirven de término de comparación. Sus nexos: Tan...como,
más...que, menos...que, etc. Es tan grande como era su padre.
– Concesivas: Indican alguna dificultad con respecto a la acción principal. Sus nexos:
Aunque, aun así, a pesar de que, etc. Lo compro, aunque no lo necesite.
RELACIÓN POR YUXTAPOSICIÓN: Las proposiciones yuxtapuestas van unidas por
comas, y su unidad oracional se consigue mediante la entonación.

CONCORDANCIAS
Definición de concordancia: Es la relación sintáctica entre palabras de un mismo sintagma
o frase por la que determinados morfemas de género y número coinciden.
Destacan dos tipos de concordancia:
– El nombre concuerda en género y número con los determinantes y adjetivos. Los
niños pequeños.
– El núcleo del sujeto concuerda con el verbo en número y persona. Ella (3ª Pª del Sin-
gular) viene (3ª Pª del Singular).

CLASES DE PALABRAS
Las palabras que constituyen el castellano forman dos grandes grupos: variables e invaria-
bles.
VARIABLES: Admiten variaciones en sus morfemas flexivos (género, número, persona,
tiempo, etc.). Son:
Sustantivo: Sirve para designar a las personas, animales, cosas, ideas...etc. (Común,
propio, concreto, abstracto...).
Pronombre: Palabra que sustituye al sustantivo. (Personal, demostrativo, indefinido,
relativo, etc.).
Determinantes: Acompañan y concretan al sustantivo. (Artículos, demostrativos, inde-
finidos, posesivos, numerales, etc).

14
-Pág.18-
U.D. 3.- LA LENGUA COMO OBJETO DE CONOCIMIENTO

Adjetivo: Acompaña al nombre para indicar cualidades del mismo. (Positivo, compara-
tivo y superlativo).
Verbo: Indica acción, proceso o estado (Regulares, irregulares, defectivos, etc).

INVARIABLES: Tienen una forma única, no admiten variaciones.


Adverbio: Complementa al verbo, al adjetivo o a otro adverbio. (De lugar, tiempo,
modo, negación, afirmación, etc.).
Preposición: Sirve para unir palabras o grupos de palabras de distinta categoría semán-
tica. (A, ante , bajo, con, contra, etc.)
Conjunción: Sirve para unir palabras o grupos de palabras de la misma categoría
semántica. (Y, o, pero, sin embargo, no obstante, etc.).
Interjección: No forma parte de la oración, ella misma es equivalente de una oración.
(¡Oh!, ¡ Hola!, ¡Ah!, etc.).

6. VOCABULARIO. SENTIDO PROPIO Y FIGURADO.


POLISEMIA, HOMONIMIA, SINONIMIA Y ANTONIMIA.
FAMILIAS LÉXICAS Y CAMPOS SEMÁNTICOS.
MODISMOS, LOCUCIONES Y FRASES HECHAS.
NEOLOGISMOS, EXTRANJERISMOS, SEXISMOS
En sentido general, denominamos “vocabulario” al conjunto de las palabras propias de una
lengua o idioma. Estas palabras suelen estar catalogadas alfabéticamente con sus definiciones
y, a veces, con informaciones complementarias que permiten conocer mejor sus significados.
Estas “ordenaciones” o “catálogos de palabras” son los diccionarios.
El Diccionario de la Lengua Española, publicado por la Real Academia de la Lengua Espa-
ñola, recoge la totalidad de palabras que son admitidas por la norma culta.
Además, existen otros tipos de diccionarios:
– Bilingües: Contienen palabras equivalentes de dos idiomas.
– Ideológicos: Catalogan las palabras en grupos que tienen que ver con alguna idea
(concepto).

– De sinónimos y antónimos: Palabras de igual y opuesto significado.

15
-Pág.19-
L E N G U A Y L I T E R AT U R A

– Enciclopédicos: Incluyen información general sobre temas culturales.

– Etimológicos: Explican la procedencia de las palabras, etc.

Los diccionarios necesitan ser actualizados continuamente, ya que la incorporación de nue-


vos vocablos a la Lengua es continua.

SENTIDO PROPIO Y FIGURADO


El sentido o significado de las palabras puede variar en función del contexto en el que sean
utilizadas, por lo que debemos diferenciar entre:
SENTIDO PROPIO O DENOTACIÓN: Es el significado real y objetivo que tiene cual-
quier palabra aislada (fuera de un contexto determinado) y que es el que nos suele ofrecer el dic-
cionario.
Madre = Hembra de cualquier especie que ha parido.
SENTIDO FIGURADO O CONNOTACIÓN: Es el sentido asociado o imaginario que el
hablante, de forma subjetiva, asocia al significado real de una palabra.
Esa profesora es una madre (connotaciones de bondad, paciencia, cariño, etc.).
Aunque en el uso normal de la lengua se utilizan expresiones con sentido figurado, su uso
está más generalizado en el lenguaje literario: metáforas, metonimias, etc.

POLISEMIA, HOMONIMIA, SINONIMIA Y ANTONIMIA


Son diversos fenómenos de relación semántica entre los planos de la expresión y del conte-
nido en el signo lingüístico.

POLISEMIA: POLI (Varios) SEMIA (SIGNIFICADO). La polisemia se poduce cuando un


significante tiene varios significados con alguna característica semántica común.

“HOJA”: De árbol, de libro, de afeitar, de navaja, etc.


“BANCO”: De finanzas, de peces, de arena, de sentarse, etc.

HOMONIMIA: HOMO (Igual) NIMIA (Nombre). La homonimia se produce cuando dos


o más palabras coinciden en su expresión pero expresan significados completamente diferentes.

“LLAMA”: Animal.
“LLAMA”: Verbo.
“LLAMA”: Fuego.

16
-Pág.20-
U.D. 3.- LA LENGUA COMO OBJETO DE CONOCIMIENTO

SINONIMIA: SINO (Similar) NIMIA (Nombre). La sinonimia se produce cuando palabras


distintas coinciden en sus significados.
“BURRO”, “ASNO”, “POLLINO”, “BORRICO”. (Mismo animal).
“MORIR”, “FALLECER”. (Final de la vida).
ANTONIMIA: ANTO (Opuesto) NIMIA (Nombre). La antonimia se produce entre pala-
bras de significación opuesta.
“BARBUDO”: Con barba.
“IMBERBE”: Sin barba.

FAMILIAS LÉXICAS Y CAMPOS SEMÁNTICOS


Son dos tipos de agrupaciones de palabras por compartir alguna significación.
FAMILIAS LÉXICAS: Están formadas por una palabra primitiva y todas sus derivadas;
comparten el mismo “lexema” (Raíz de la palabra).
“Zapato”.
“Zapatería”.
“Zapatilla”. Zapat = Lexema
“Zapatero”.
“Zapatear”.
CAMPO SEMÁNTICO: Está formado por el conjunto de palabras que comparten algún
“sema” (significado parcial).
“Atletismo”.
“Fútbol”.
“Baloncesto”. Todos son deportes.
“Ciclismo”.
“Judo”.

MODISMOS, LOCUCIONES Y FRASES HECHAS


Son formas, construcciones y giros especiales que explican el “argot” de la lengua.
MODISMOS: Son modos particulares de hablar, propios o privativos de una lengua, que en
muchas ocasiones se apartan de las reglas generales de la gramática.
“Me vino como caído del cielo”.
“Lo creyó a pie juntillas”.

17
-Pág.21-
L E N G U A Y L I T E R AT U R A

LOCUCIONES: Son conjuntos de palabras que funcionan como un solo elemento oracio-
nal, con el que coinciden esencialmente en la semántica. A veces, aportan matices significati-
vos más amplios.
Pueden ser:
– Adverbiales: Equivalente a un adverbio.
“ Vivían a lo grande”.
– Adjetivas: Equivalentes a un adjetivo.
“Era de armas tomar”.
– Conjuntivas: Equivalen a una conjunción.
“Te veré siempre que pueda”.
– Prepositivas: Equivalen a una preposición.
“Miraba por encima de la tapia”.
FRASES HECHAS: Son las que tienen una forma de construcción fija y un significado
especial.
“Año de nieves, año de bienes”.
“Que Dios te lo pague, hermano”.

NEOLOGISMOS, EXTRANJERISMOS, SEXISMOS


Son grupos de palabras que por diversos motivos: necesidad, moda, etc. se incorporan, jus-
tificada o injustificadamente, a una lengua.
NEOLOGISMOS: Son las nuevas palabras que se van incorporando al vocabulario de una
lengua. Éstas pueden ser: Inventadas, derivadas o compuestas de las existentes, o tomadas
(préstamos) de otras lenguas.
“Vídeo”, “Club”, “UNED”, “SIDA”, etc.
EXTRANJERISMOS: Son los neologismos “innecesarios” procedentes de otras lenguas,
especialmente del inglés, que invaden nuestra lengua. También se les puede denominar “barba-
rismos”.
“Boite”, “Boutique”, “Sandwich”, “Parking”, “Ticket”, etc.
SEXISMOS: Son expresiones que manifiestan una clara actitud discriminatoria de las per-
sonas de un sexo hacia las del otro.
“Machista”, “Coronela”, “Marimacho”, etc.

18
-Pág.22-
U.D. 3.- LA LENGUA COMO OBJETO DE CONOCIMIENTO

7. NORMAS ORTOGRÁFICAS. ORTOGRAFÍA


DEL DISCURSO. ORTOGRAFÍA DE LA ORACIÓN.
ORTOGRAFÍA DE LA PALABRA
La ortografía es la parte de la gramática que se ocupa de la escritura correcta de las palabras
y los demás signos que intervienen en la misma. Para ello estable una serie de normas que debe-
mos conocer.
Algunas normas ortográficas referentes al uso de la “tilde” son:
Uso de la tilde: Llevan tilde:
– Todas las palabras agudas acabadas en vocal o en consonante “n” o “s”. Saltó,
compás, camión, ...
– Todas las palabras graves o llanas acabadas en consonante distinta de n o s.
Cárcel, Gómez, fértil, ...
– Todas las palabras esdrújulas y sobreesdrújulas: Lámpara, médico, cuéntaselo, ...
Tilde diacrítica: Las palabras monosílabas no llevan tilde, salvo en los casos que sea
necesario para distinguir dos palabras iguales de significado diferente:
dé (verbo dar) de (preposición)
él (pronombre) el (artículo)
tú (pronombre) tu (determinante posesivo)
sí (afirm, o pronom.) si (conjunción condicional)
más (cantidad) mas (conjunción adversativa)
mí (pronom. pers.) mi (determinante posesivo)
té (bebida) te (pronombre personal)
aún = todavía aun = incluso
quién (interr. o excl.) quien (pronombre relativo)
cuál (interr. o excl.) cual (pronombre relativo)
etc.

19
-Pág.23-
L E N G U A Y L I T E R AT U R A

La tilde en la vocales en hiato:


– Si las vocales son abiertas, siguen la regla general. Amaos, oyó, ...
– Si la vocal tónica es “i” o “u”, deberá llevar tilde para indicar el hiato, aunque
por la regla general no le corresponda. Leído, vahído, transeúnte, ...
La tilde en las palabras exclamativas e interrogativas: Las palabras qué, quién, cuál,
cómo, cuánto, etc. llevan siempre tilde, y no la llevan cuando no se utilizan para pre-
guntar o exclamar.
La tilde en las palabras formadas por composición:
– La primera palabra, si llevaba tilde, la pierde y todo el compuesto se adaptará
a la regla general. Decimosegundo, decimoséptimo, ...
– Si se forma una palabra esdrújula o sobreesdrújula, ésta llevará tilde. Dígamelo.
– Si el compuesto va unido por guión, cada componente conserva su estado
anterior. Físico-químico, hispano-francés, ...
– Los compuestos, formados por adjetivo más la partícula “mente”, no modifi-
can su estado anterior.
La tilde en las mayúsculas: Seguirán las mismas normas que en las minúsculas.
ESPLÉNDIDO, ÁNGEL, SOLÍS, etc.

ORTOGRAFÍA DEL DISCURSO


En el discurso, entendido como un texto de cierta extensión, formado de diversos párrafos,
se deben contemplar las siguientes reglas ortográficas:
– Todos los párrafos que compongan el texto deben empezar por letra mayúscula.
– El final de cada párrafo se indicará con un punto y aparte, excepto el último que será con
un punto final.
– Si un párrafo se divide en apartados con cierta independencia, éstos podrán indicarse con
guiones.
– Si en un párrafo se quiere resaltar alguna palabra, se podrá hacer mediante comillas.
– Si en un párrafo se quiere explicar alguna palabra, se podrá hacer mediante paréntesis.

20
-Pág.24-
U.D. 3.- LA LENGUA COMO OBJETO DE CONOCIMIENTO

ORTOGRAFÍA DE LA ORACIÓN
En la ortografía de la oración, debemos considerar las siguientes reglas generales.
– La oración se inicia con mayúscula y finaliza con punto.
– Los “límites” son pausas que se indican siempre mediante puntos: seguidos, aparte y final.
– Si una oración va entre signos de interrogación o exclamación, los puntos de cierre de
estos signos determinan el final de la oración y cumplen la misma función que el punto y
seguido, punto y aparte o punto final.

ORTOGRAFÍA DE LA PALABRA
Sólo vamos a exponer algunos usos de algunas letras, cuyo conocimiento consideramos
imprescindible para la ortografía de un buen número de palabras. Concretamente, de: la “b”, la
“v”, la “h”, la “g” y la “j”.

La “B”: Se escriben con “B”


– Los verbos acabados en bir, excepto hervir, vivir y servir. Recibir, escribir, prescribir, etc.
– Las formas verbales acabadas en aba, abas, etc. y el pretérito imperfecto de indicativo
del verbo ir: iba, íbamos, etc.
– Las palabras que empiezan por bi, bis, biz y bio. Bisiesto, biografía, bibliografía, etc.
– Las palabras que terminan en bilidad, bundo, bunda, excepto civilidad y movilidad.
Amabilidad, nauseabundo, etc.

La “V”: Se escriben con “V”


– Los adjetivos acabados en ave, eve, avo, evo e ivo. Suave, leve, vivo, activo, etc.
– Después de “d”. Advertir, adverso, etc.
– Las palabras acabadas en viro, vira, voro y vora, excepto víbora. Herbívoro, carnívora, etc.
– Las palabras que empiezan por vice, viz o vi, vicealmirante, vida, etc.

La “H”: Se escriben con “H”


– Las palabras que empiezan por ue. Hueco, Huelva, Huerto, etc.
– Las palabras que empiezan por Hidr, hiper, hipo, hecto, hexa y homo. Hipódromo, hexá-
gono, hidrógeno, etc.
– Diversas interjecciones. ¡Hola!, ¡hurra!, ¡eh!, ¡oh!, etc.

21
-Pág.25-
L E N G U A Y L I T E R AT U R A

La “G”: Se escriben con “G”


– Las palabras que empiezan por geo, gene, geni, gest. Geografía, general, gesticular, etc.
– Las palabras que terminan en gio, gia, gión, gismo. Colegio, logia, región, ecologismo, etc.
– Los verbos terminados en igerar, ger y gir (excepto tejer y crujir). Refrigerar, proteger, etc.
– Las palabras terminadas en gen. Virgen, origen, etc.
– Las palabras terminas en derivados de logos. Psicólogo, filología, podólogo, antropología, etc.
La “J”: Se escriben con “J”
– Las palabras acabadas en aje, eje y jería. Encaje, relojería, etc.
– Las palabras que se derivan de una primitiva que lleve j. Caja, cajera, baja, bajada, etc.
– Las formas verbales, cuyos infinitivos llevan la j. Dejar, dejo, dejé, etc.

RESUMEN

– El uso correcto de una lengua está sujeto a la “norma lingüística”.


– En el discurso podemos apreciar las siguientes partes: Introducción, cuerpo o desarrollo
y final o conclusión.
– Todo texto debe ser coherente y estar adecuado a un contexto.
– Los constituyentes fundamentales de la oración son: sujeto y predicado.
– Las palabras pueden tener un sentido real (objetivo) y un sentido figurado (subjetivo).
– La ortografía se ocupa del uso correcto de la escritura de las palabras y signos que inter-
vienen en la misma.

22
-Pág.26-
U.D. 3.- LA LENGUA COMO OBJETO DE CONOCIMIENTO

EJERCICIOS DE AUTOCOMPROBACIÓN

1. El nacimiento del “castellano” se produce en:

A. Cantabria B. Galicia C. Cataluña D. Castellón

2. La parte más extensa del discurso corresponde a:

A. La introducción B. El desarrollo C. La conclusión D. El nexo

3. La oración impersonal está formada de:

A. Sujeto y predicado B. Sujeto

C. Predicado D. Verbo y nombre

4. La oración: “Luis es piloto”, según la actitud del hablante, es:

A. Dubitativa B. Enunciativa C. Copulativa D. Extrínseca

5. La oración: “La niña tiene una muñeca”, según la naturaleza del predicado, es:

A. Predicativa B. Atributiva C. Enunciativa D. Salutativa

6. La oración: “Toma dinero y compra el periódico”, es:

A. Coordinada B. Subordinada C. Yuxtapuesta D. Concomitante

7. La oración: “Estudio cuando puedo”, es:

A. Coordinada B. Subordinada C. Simple D. Yuxtapuesta

8. Indique si son variables o invariables las siguientes palabras:

A. Periódico B. Lejos C. Buenos D. Allí

9. Palabras homónimas son:

A. Las que tienen el mismo significado B. Las que se pronuncian igual

C. Las que tienen significados opuesto D. Las que tienen el mismo nombre

10. Ponga las letras que faltan en las siguientes palabras:

A. ..isnieto B. ..ajá..amos C. A..ora D. Relo..ero

23
-Pág.27-
L E N G U A Y L I T E R AT U R A

RESPUESTAS A LOS EJERCICIOS

11. A 12. B 13. C 14. B 15. A


16. A 17. B 8. 19. B 10.
A. Variable A. Bisnieto
B. Invariable B. Bajábamos
C. Variable C. Ahora
D. Invariable D. Relojero

24
-Pág.28-
U . D . 4 . - S ISTEMAS DE COMUNICACIÓN VERBAL Y NO VERBAL

ÍNDICE
OBJETIVOS . . . . . . . . . . . . . . . . . . . . . . . . . . . . . . . . . . . . . . . . . . . . . . . . . . . . . . . . . . . . . . 2

INTRODUCCIÓN . . . . . . . . . . . . . . . . . . . . . . . . . . . . . . . . . . . . . . . . . . . . . . . . . . . . . . . . . . 3

MAPA CONCEPTUAL. . . . . . . . . . . . . . . . . . . . . . . . . . . . . . . . . . . . . . . . . . . . . . . . . . . . . . 4

DESARROLLO DE CONTENIDOS

1. CARACTERÍSTICAS E INTERACCIÓN DE LA COMUNICACIÓN VERBAL


Y NO VERBAL. . . . . . . . . . . . . . . . . . . . . . . . . . . . . . . . . . . . . . . . . . . . . . . . . . . . . . 5

2. IMPORTANCIA DE LOS LENGUAJES NO VERBALES EN LA


COMUNICACIÓN HUMANA . . . . . . . . . . . . . . . . . . . . . . . . . . . . . . . . . . . . . . . . . 5

3. LENGUAJES NO VERBALES EN DETERMINADOS GRUPOS SOCIALES. . 7

4. LOS MEDIOS DE COMUNICACIÓN . . . . . . . . . . . . . . . . . . . . . . . . . . . . . . . . . . 9

5. LENGUAJE VERBAL Y LENGUAJE DE LA IMAGEN . . . . . . . . . . . . . . . . . . 13

6. LENGUAJE VERBAL, MUSICAL, GESTUAL Y LENGUAJES


DE LOS OBJETOS Y DE LOS AMBIENTES . . . . . . . . . . . . . . . . . . . . . . . . . . . 16

RESUMEN . . . . . . . . . . . . . . . . . . . . . . . . . . . . . . . . . . . . . . . . . . . . . . . . . . . . . . . . . . . . . 18

EJERCICIOS DE AUTOCOMPROBACIÓN . . . . . . . . . . . . . . . . . . . . . . . . . . . . . . . . . . 19

RESPUESTAS A LOS EJERCICIOS . . . . . . . . . . . . . . . . . . . . . . . . . . . . . . . . . . . . . . . . . 20

-Pág.29-
L E N G U A Y L I T E R AT U R A

OBJETIVOS
Al finalizar el estudio de esta Unidad Didáctica, el alumno será capaz de:

• Diferenciar claramente entre lenguaje verbal y lenguaje no verbal.

• Identificar las diferentes formas de comunicación según los signos y códigos que
se utilicen en la producción de un mensaje.

• Reconocer y valorar las semejanzas y las diferencias entre distintas formas de


comunicación.

• Distinguir los medios de comunicación en sus diversas intenciones: informar,


formar opinión, entretener, etc.

• Interpretar la intención manipuladora de la publicidad.

• Adoptar una actitud crítica ante cualquier mensaje que denote algún tipo de dis-
criminación: ideológica, religiosa, racial, sexual, etc.

-Pág.30-
U . D . 4 . - S ISTEMAS DE COMUNICACIÓN VERBAL Y NO VERBAL

INTRODUCCIÓN
A lo largo de las unidades precedentes nos hemos referido exclusivamente a la comu-
nicación “verbal” (oral y escrita), atendiendo a sus características, peculiaridades y
usos. En esta Unidad nos referiremos a otras formas, “no verbales”, de transmitir infor-
mación, bien complementando a la comunicación verbal, bien por sí solas.

Estos sistemas de “comunicación no verbal” también tienen sus propios circuitos de


comunicación, con elementos y funciones similares a los descritos en la comunicación
verbal, si bien, con algunas singularidades en algunos de ellos que iremos viendo en esta
unidad.

-Pág.31-
L E N G U A Y L I T E R AT U R A

M A PA C O N C E P T UA L

LENGUA CASTELLANA
Y LITERATURA

SISTEMAS DE
COMUNICACIÓN VERBAL
Y NO VERBAL

VERBALES NO VERBA-

ORAL ESCRITO MÍMICO ICÓNICO MUSICAL TÁCTILES,

COMUNICACIÓN HUMANA

MEDIOS DE COMUNICACIÓN OTROS MEDIOS


– PRENSA – IMAGEN: Fotonovela, cómic, cine,
– RADIO vídeo, mapas, planos, etc.
– TELEVISIÓN – TÁCTIL: Caricias, abrazos, estre-
– PUBLICIDAD, etc. char la mano, etc.
– SONOROS: Música, canción,
aplausos, timbres, silbidos, etc.

-Pág.32-
U . D . 4 . - S ISTEMAS DE COMUNICACIÓN VERBAL Y NO VERBAL

CARACTERÍSTICAS E INTERACCIÓN
1. DE LA COMUNICACIÓN VERBAL Y NO VERBAL
En la 1ª Unidad dábamos la siguiente definición de lenguaje: “Capacidad que tiene el hom-
bre para comunicarse con los demás a través de signos, esta capacidad de comunicación es
común a todos los hombres con independencia del lugar o grupo social en el que haya naci-
do cada uno”, y añadíamos: Si los signos que utiliza son verbales (palabras, oraciones, etc.)
se le denomina LENGUAJE VERBAL”.

Lógicamente, podemos deducir que si los signos que utiliza no son verbales (gestos, mími-
ca, sonidos no articulados, música, dibujos, planos, etc.), nos estamos refiriendo a los “LEN-
GUAJES NO VERBALES”.

Fundamentalmente, en nuestras relaciones sociales utilizamos, de forma consciente, el Len-


guaje Verbal: diálogos a través del teléfono, conversaciones, conferencias, cartas, telegramas,
postales, notas informativas, etc.

Pero también recibimos mucha información a través de los lenguajes no verbales: saludos
con la mano, un gesto de aprobación o desaprobación, una sonrisa, una mirada fría, las imáge-
nes de una valla publicitaria, el pitido de un automóvil, el sonido de un timbre, la luz de subi-
da o bajada de un ascensor, la luz verde, amarilla o roja de un semáforo, la música agradable o
desagradable de ciertos locales públicos, etc.

Es posible que algunos de estos sistemas de comunicación (verbales y no verbales) “actú-


en” por sí solos: una carta, un diálogo a través del teléfono, un saludo levantando las manos, el
insistente pitido de un coche que no puede salir porque se lo impide otro, etc., pero en infinidad
de ocasiones, los diversos tipos de lenguaje son INTERACTIVOS; es decir, se complementan;
ej: en una conversación nos ayudamos de gestos diversos, los actores hablan y adoptan postu-
ras determinadas para hacerse comprender mejor, una persona que explica sus alegrías o penas
suelen expresar sentimientos de pena o alegría en su rostro, un anuncio publicitario emplea pala-
bras e imágenes, etc.

IMPORTANCIA DE LOS LENGUAJES NO VERBALES


2. EN LA COMUNICACIÓN HUMANA
No cabe la menor duda de que el sistema más perfecto de comunicación humana es a través
del lenguaje verbal (signos lingüísticos: orales y escritos), ya que con un reducido número de
sonidos y letras podemos elaborar cualquier tipo de mensaje que nos permita expresar lo que
pensamos, sentimos, necesitamos, etc.

5
-Pág.33-
L E N G U A Y L I T E R AT U R A

Los signos lingüísticos son percibidos por dos importantes sentidos: el oído (oral) y la vista
(escrito), pero el hombre puede percibir otros signos o señales “no verbales” a través de cual-
quiera de los sentidos: vista, oído, gusto, olfato y tacto.
Estas señales o signos no verbales las podemos clasificar desde distintos puntos de vista o
criterios, a saber:
A) Según el aparato receptor que los perciba:
– Signos auditivos: Interviene el oído: cualquier tipo de sonido (melodías, ruidos,
etcétera).
– Signos visuales: Interviene la vista: imágenes, colores, figuras, etc.
– Signos táctiles: Interviene el tacto: dar ligeramente con el codo a alguien para que
deje de hablar, tocar con las puntas de los dedos en el hombro de alguien para hacer
notar nuestra presencia, etc.
– Signos gustativos: Interviene el gusto: los sabores de las comidas, bebidas, etc.
– Signos olfativos: Interviene el olfato: olores de comidas, perfumes, otras sustan-
cias, etc.
B) Según la relación que tengan con lo que expresan:
– Iconos: Poseen una relación de semejanza con lo que representan: planos, fotogra-
fías, dibujos, etc.
– Indicios: Tienen una relación de “causa-efecto” con su referente: el color lívido de
una persona por efecto del frío, los quejidos por cualquier tipo de sufrimiento, etc.
– Símbolos: No tienen una relación “justificada” con lo que expresan; la relación
entre el signo y su referente se ha establecido de una forma arbitraria: las banderas
que representan a los países, los guiones y estandartes de la unidades militares, la
cruz roja o la media luna de los cuerpos sanitarios (occidentales y árabes), el color
negro para expresar luto, etc.
Es a partir de principios de este siglo cuando la semiótica o semiología (ciencia que estu-
dia los signos que percibimos en nuestra relación social) empezó a reconocer y valorar la impor-
tancia que tienen los signos no verbales en la comunicación humana. Tradicionalmente, se venía
prestando toda la atención al estudio de los signos lingüísticos, sin apreciar la considerable
información que nos aportan los signos no lingüísticos en nuestras relaciones de comunicación.
El siguiente relato (hecho real) nos permitirá comprender la indudable importancia que tiene
la comunicación no verbal y la atención que el estudio de sus signos merece:
“Aprendí este oficio con un viejo cocinero que regentaba la cocina del hotel “X”, era un
hombre parco en palabras, pero muy expresivo en sus gestos; jamás levantaba la voz, ni aun
cuando algo me salía mal, su semblante serio de desaprobación y la mirada dirigida a las espe-
cias que había utilizado en cantidades desproporcionadas eran suficientes para comprender

6
-Pág.34-
U . D . 4 . - S ISTEMAS DE COMUNICACIÓN VERBAL Y NO VERBAL

mis desaciertos culinarios, ¡Cuántas veces deseé que me hubiera corregido con palabras, no
me hubiera importado que incluso hubiera gritado! ¡Aquella mirada seria e impasible me hacía
sentir una preocupación que no acababa hasta que su semblante se suavizaba y sus ojos se dul-
cificaban!
Procuraba prestar toda mi atención a sus movimientos con las sartenes y las ollas, cómo
troceaba los productos, el tiempo que los tenía al fuego, cómo observaba el color que la comi-
da iba adquiriendo, y cómo a partir de cierto momento la probaba, a veces ponía algo de sal,
volvía a probarla y apagaba el fuego o lo dejaba un poco más.
Un oficial de cocina, que no paraba de hablar, me dijo en cierta ocasión: Chico, has caído
bien al maestro, todos los lunes te pregunta si has descansado bien en el fin de semana.
Hoy lo recuerdo con cariño, y sé que lo que conozco de mi oficio lo aprendí de él”.

LENGUAJES NO VERBALES EN DETERMINADOS


3. GRUPOS SOCIALES
La comunicación no verbal, lo mismo que la comunicación verbal, tiene su circuito de
comunicación: Emisor, receptor, canal, mensaje, situación, contexto y, por supuesto, código.
Este último junto a los signos (no verbales) es lo que no permite diferenciar los distintos tipos
de lenguajes no verbales.
Los diversos signos no verbales y los códigos que regulan su uso son estudiados por la
PARALINGÜÍSTICA, LA CINÉSICA Y LA PROXÉMICA.
— LA PARALINGÜÍSTICA: Estudia todos los posibles elementos “no lingüísticos”
(cualidades de la voz: intensidad, tono, timbre, y cantidad, jadeos, risas, silencios inten-
cionados, etc.) que intervienen durante una emisión lingüística y colaboran a que el
mensaje sea más perfecto.
— LA CINÉSICA: Estudia la expresión corporal: ademanes, gestos, posturas, expresio-
nes faciales, etc.
— LA PROXÉMICA: Estudia la colocación de las personas y de los posibles objetos que
pueden condicionar una situación de comunicación determinada. Ejemplo:
• En un despacho, la mesa marca una pequeña distancia entre quien atiende y las per-
sonas que presentan alguna solicitud, y obliga a que estén “de frente”.
• La disposición de las cajas cobradoras en algunos establecimientos obliga a formar
“fila india” para pagar.
Estos códigos no verbales son aprendidos en cada colectividad, de tal forma que un mismo

7
-Pág.35-
L E N G U A Y L I T E R AT U R A

signo puede tener significaciones distintas según donde se manifieste. Ej.: En algunas culturas
el color blanco simboliza “luto”, mientras que en otras significa lo contrario.
Incluso, dentro de una misma colectividad cultural, los signos no verbales pueden tener sig-
nificados concretos para determinados grupos, obedeciendo a razones como: el sexo, la edad, la
procedencia, la cultura, etc.
Veamos algunos ejemplos del uso de los lenguajes “no verbales” en estos determinados gru-
pos en los apartados siguientes:

SEXO
– El vestuario, perfume, adornos, etc. tienen pautas de uso muy marcadas, dependiendo si
se pertenece al género masculino o femenino.
– Las mujeres suelen interrumpir menos que los hombres en las conversaciones.
– La voz femenina es más aguda; la masculina, más grave.
– ...

EDAD
– Los colores: En las personas adultas suelen ser más discretos; en los jóvenes, más lla-
mativos y provocativos.
– Un mismo color puede expresar mensajes diferentes. Por ejemplo: El color negro en
determinado grupo de jóvenes es señal de identidad (heavy, por ejemplo), mientras que
en las personas adultas simboliza “luto”.
– Las posturas, ademanes, gestos, etc. en los jóvenes suelen ser más descuidados; en los
adultos, más correctos.
– ...

PROCEDENCIA
– Por los diversos matices de la voz podemos intuir el lugar de procedencia.
– El vestuario que se utiliza en determinadas fiestas regionales.
– La naturalidad o extrañeza que algunas personas muestran ante determinados aconteci-
mientos pueden ser indicios que nos insinúen su lugar de origen, etc.

8
-Pág.36-
U . D . 4 . - S II S
STTE
EMMA
ASS D
DEE C
COOM
MUUN
N II C
CAAC
C II Ó
ÓNN V
VEER
RBBA
ALL Y
Y N
NOO V
VEER
RBBA
ALL

CULTURA
– La distancia que se mantiene al hablar: Entre los latinos suele ser más corta que entre los
anglosajones.
– La voz: Entre los latinos suele ser más elevada que entre los anglosajones.
– Las posturas, ademanes, gestos, etc. Entre los occidentales suelen ser muy expresivos, a
veces “exagerados”, mientras que entre los orientales suelen ser más discretos y come-
didos, etc.

4. LOS MEDIOS DE COMUNICACIÓN


Los continuos avances científicos y técnicos han propiciado una total revolución en la
comunicación humana, especialmente en los medios de comunicación, donde se ha conseguido
eliminar o minimizar las barreras de espacio y tiempo.
Los Medios de comunicación de masas, también denominados “Mass-media”, han adqui-
rido tal importancia, que se han convertido en “imprescindibles” en las sociedades actuales.
La prensa, la radio, la televisión, la misma publicidad, etc., contribuyen decisivamente a que
las lenguas sean:
– Más universales.
– Más unificadas.
– Más “patrimonio” de todos.
– Más perdurables.
– Más consideradas.
– ...

Estos medios pretenden conseguir los siguientes objetivos inmediatos:


– Informar sobre lo que acontece diariamente.
– Persuadir y orientar al público.
– Entretener y divertir a las masas.
– Difundir la cultura.
– Establecer actos de comunicación interpersonales (debates, mesas redondas, etc.).
En los medios de comunicación la información va desde el Emisor (estatal o privado) hacia
el Receptor (indefinido), aunque, a veces, en algunos espacios “determinados” se permite la
intervención de los receptores para dar sus opiniones.

9
-Pág.37-
L
LEEN
NGGU
UAA Y
Y L
L II T
TEER
R AT
AT U
URRA
A

Los medios de comunicación obtienen la información que difunden a través de sus propios
periodistas, corresponsales, otros medios de comunicación y muy especialmente de las Agen-
cias de Información. Estas agencias están permanentemente recopilando informaciones de todo
tipo y las sirven inmediatamente a las redacciones de los periódicos, radios, tv, etc., que se las
solicitan.

4.1. LA PRENSA
Es el medio que transmite la información a través de la escritura, el canal que utiliza es el
papel impreso, y dependiendo de su formato, estructura o periodicidad en su publicación reci-
be el nombre de: diarios, semanarios, revistas, etc.
Atendiendo a la distribución y organización de la información en los periódicos tendremos
en consideración lo siguiente:

Distribución: El contenido de las diversas informaciones aparece en bloques o “secciones”;


cada sección se especializa en dar un tipo de información determinada, y pueden ser:
– Local.
– Nacional.
– Internacional.
– Económica.
– Política.
– Deportiva.
– Cartelera.
– ...

Organización y presentación: La información puede aparecer en forma de:


– EL EDITORIAL: Es un artículo de opinión que está relacionado con la línea ideológi-
ca del periódico, aparece sin firma y lo suele escribir el director del periódico o una per-
sona de su confianza.

– LA NOTICIA: Es un artículo que ofrece una información “novedosa”; los medios com-
piten por ser los primeros en darla, el que la consigue la destaca como “Primicia Infor-
mativa”.
Toda noticia debe responder a las siguientes interrogantes: ¿Qué? ¿Cuándo? ¿Quién?,
¿Dónde? ¿Cómo?, etc.

10
-Pág.38-
U . D . 4 . - S II S
STTE
EMMA
ASS D
DEE C
COOM
MUUN
N II C
CAAC
C II Ó
ÓNN V
VEER
RBBA
ALL Y
Y N
NOO V
VEER
RBBA
ALL

Suele tener la siguiente estructura:


Titular: Frase de corta extensión, en letras en negrita y tamaño grande para llamar la
atención del lector. Ejemplo:

NIEVE EN LA SIERRA MADRILEÑA

Entrada o Entradilla: Es un breve resumen de los elementos principales de la noticia,


suele ir en negrita y en un tamaño menor que el titular.
Ejemplo:
“Las bajas temperaturas y la ausencia de viento han propiciado que las últimas
precipitaciones en forma de nieve hayan formado una capa de 40 cms. de nieve en
la sierra del Norte de Madrid”.
Cuerpo o desarrollo de la noticia con todos sus detalles: Suele ir en un tipo de letra que
no destaca. Siguiendo con el ejemplo anterior, sería:
“Gracias a las últimas precipitaciones que están produciéndose en la zona centro y
en....... (se daría cumplida información de la nieve caída, previsiones del tiempo, estado
de los accesos a la zona, servicios para la práctica del esquí, etc.)”.

– LA CRÓNICA: Es un tipo de artículo en el que el periodista trata cualquier tema de


actualidad dándole unas connotaciones especiales: Comentarios personales, referencias
relacionadas con otras noticias parecidas, ampliación deliberada de algún aspecto, etc.
Ejemplo: Cualquier crónica sobre los debates parlamentarios.

– EL REPORTAJE: Es la información amplia, acompañada de fotografías, planos, dibu-


jos, etc., sobre cualquier tema; se diferencia de la noticia en que no necesita ser “nove-
doso”.
Ejemplo: Cualquier artículo documentado e ilustrado con imágenes, referido a las modas
de temporada.

– LA ENTREVISTA: A veces, los periodistas presentan una información en la que apa-


recen el mismo periodista y una persona de interés público en franca conversación. Antes
de realizar la entrevista debe prepararse un cuestionario con una serie de preguntas cla-
ras, precisas y respetuosas para la persona que se haya prestado a ser entrevistada. Por
supuesto que se trata de un diálogo “elaborado” donde se presentan unas preguntas a las
que irá respondiendo la persona que es motivo de la entrevista.
Ejemplo: Cualquier entrevista a un político, actor, cantante, deportista, etc.

11
-Pág.39-
L
LEEN
NGGU
UAA Y
Y L
L II T
TEER
R AT
AT U
URRA
A

4.2. LA RADIO
Es el medio que transmite la información a través del sonido (palabra hablada, música, etcé-
tera).
Sin duda, se trata del medio de comunicación más rápido para transmitir cualquier tipo de
noticias, y desde que apareció el transistor, el más fácil de sintonizar en cualquier lugar, momen-
to o situación.
El lenguaje de la radio se caracteriza por su claridad y capacidad motivadora para que el
oyente pueda imaginar situaciones, personas, paisajes, objetos, etc.
En la actividad radiofónica alternan las informaciones, los diálogos, espacios musicales,
entrevistas, etc.

4.3. LA TELEVISIÓN
Es el medio de comunicación que combina los sonidos y las imágenes.
La TV es un medio bastante rápido, no tiene la inmediatez de la radio debido a que su forma
de transmitir es más compleja: las informaciones suelen ir complementadas con imágenes que
previamente han debido ser grabadas, seleccionadas y montadas. No obstante, las modernas téc-
nicas permiten transmitir numerosos acontecimientos “en directo” desde cualquier punto geo-
gráfico; si bien necesita de toda una preparación anterior: grúas, cámaras, luces, etc.
El lenguaje de la televisión no necesita ser tan explícito como el de la radio, las imágenes
suplen lo que “no se dice”.
En la actividad televisiva alternan las informaciones, diálogos, espacios musicales y cine-
matográficos, reportajes, entrevistas, etc., pero requiere una atención y dedicación para con-
templarla que la radio no precisa.

4.4. LA PUBLICIDAD
Podemos definir la publicidad como el conjunto de medios que son utilizados para divul-
gar o difundir con rapidez y efectividad la novedad de las cosas y de los acontecimientos; es
decir: de los productos comerciales, determinadas actuaciones institucionales (bandos munici-
pales, edictos, etc.), campañas electorales, guías turísticas, etc.
Fundamentalmente, nos vamos a referir a la publicidad de tipo comercial, ya que es la que
mayoritariamente nos vemos obligados a “soportar” a través de los medios de comunicación,
vallas publicitarias, buzones postales caseros, etc.
Ésta, unas veces, pretende introducir en el mercado de consumo un nuevo producto; otras,
intenta que otros productos, ya conocidos, aumenten su número de ventas.

12
-Pág.40-
U . D . 4 . - S ISTEMAS DE COMUNICACIÓN VERBAL Y NO VERBAL

El mensaje publicitario tiene una finalidad “conativa o apelativa”, es decir, intenta que
influya en la decisión del receptor: “que compre y que consuma”. Para ello utiliza infinidad
de recursos, de entre los que destacamos los siguientes:
– Uso de “eslogan” (frase de corta extensión, fácil de memorizar y con significados con-
notativos).
Ejemplo: Hablar de “TAL” es hablar de calidad.
– Uso de imágenes que resalten las cualidades del producto:
Robustez en un automóvil.
Capacidad en una lavadora.
Satisfacción y felicidad al degustar un producto, etc.

– Uso de oraciones enunciativas que destaquen las “excelencias” del producto.


Ejemplo: Producto X, “el mejor del mundo”.

– Uso de oraciones exhortativas ( o imperativas).


Ejemplo: Compre su “TAL” en “X”.

– Uso de algunos recursos literarios:


Hipérboles (exageraciones desmesuradas).
Metáforas (sentidos figurados).
Símiles (comparaciones), etc.

– En general, cualquier tipo de mensaje que haga alusión a:


Distinción social.
Libertad personal.
Modernidad.
Utilidad.
Economía, etc.

5. LENGUAJE VERBAL Y LENGUAJE DE LA IMAGEN


Hemos visto que el lenguaje verbal y no verbal se complementan, pero en otras ocasiones,
entre estos signos se establece una relación de redundancia, como ocurre entre el lenguaje ver-
bal (signos lingüísticos) y el lenguaje de la imagen (signos visuales), es decir: ambos signos
repiten el mismo mensaje para asegurar su emisión o bien para que pueda llegar a un mayor
número de receptores (no todos los receptores saben leer).

13
-Pág.41-
L E N G U A Y L I T E R AT U R A

Ejemplo: En algunos locales, los carteles con la imagen de una cruz en forma de aspa sobre
un perro, y debajo, la frase: “Perros, no”.
Además de la televisión, existen bastantes medios de comunicación en los que la imagen y
la palabra se complementan para conseguir que la emisión de un mensaje sea lo más perfecta
posible. Entre otros, destacaremos los siguientes:

EL CÓMIC
Es un medio de comunicación en el que se combinan las imágenes y los textos escritos para
narrar historias (historietas).

Estas historietas pueden referirse a cualquier tema: humorístico y de ficción (los más fre-
cuentes), históricos, biográficos, etc.

Elementos y características más importantes del cómic:

La Viñeta: Es cada uno de los cuadros que contiene todos los elementos necesarios para pre-
sentar una acción.

El Encuadre: Es la limitación del espacio utilizado para ilustrar la viñeta. Puede ser cuadra-
do, rectángulo, etc.

Planos de las imágenes:


– Plano General: Presenta la situación de forma completa, los personajes aparecen “ente-
ros”, rodeados de los posibles objetos que haya alrededor: muebles, paredes, animales,
paisajes, etc.
– Plano Americano (o tres cuartos): El espacio aparece representado a partir de las rodi-
llas de los personajes.
– Plano Medio: Muestra las figuras a partir de la cintura.
– Primer Plano: Presenta un detalle concreto de cualquier figura humana, animal u
objeto.

Textos:
Aunque existe el cómic mudo (sin palabras), normalmente suelen complementarse la ima-
gen y la palabra. Los textos pueden aparecer de las siguientes formas:
– El Bocadillo (o globo): Las frases están rodeadas por líneas cerradas: curvas, elipses,
cuadrados, etc., con un pequeño apéndice que señala a la persona que habla.
– La Cartela: Información que va dando el emisor (narrador). Suele aparecer en la parte
superior de la viñeta.

14
-Pág.42-
U . D . 4 . - S ISTEMAS DE COMUNICACIÓN VERBAL Y NO VERBAL

– Onomatopeyas (palabras que imitan sonidos: Plaf, boom, riiing, etc.). Pueden aparecer
en cualquier punto de la viñeta.

LA FOTONOVELA
Es otro medio de comunicación que se apoya en la imagen y en la palabra; consiste en una
narración de una historia, generalmente de tipo amoroso, con una estructura similar al cómic,
pero en vez de dibujos, utiliza fotografías de personas y objetos reales.
Los diálogos que se establecen entre los personajes resultan “interesantes” por el indudable
atractivo que presentan las personas fotografiadas: actores de fotonovelas (elegantes, seducto-
res, atractivos, etc).

EL CINE
Conocido también como “El Séptimo Arte”, es un medio de comunicación que combina la
imagen en movimiento, la palabra y cualquier tipo de sonido (música, ruidos, etc.).
En sus inicios era “mudo”; los actores sólo podían expresarse mediante gestos, y en algunas
ocasiones se utilizaban subtítulos con pequeños mensajes.
En el cine se da más importancia a la imagen que a la palabra; de hecho, en la preparación
de una película (o filme), los decorados, vestuario, peluquería, maquillaje, etc., requieren de un
especial cuidado. Por eso, el cine puede suplir gran parte de la información verbal mediante
imágenes, sin que el mensaje resulte empobrecido. Veamos algunos ejemplos:
– El paso de horas mediante un reloj que marca una hora (las 3, por ejemplo) y a conti-
nuación marca otra posterior (las 7, por ejemplo).
– El paso de fechas, mediante la aparición de una fecha en el calendario (16 de abril) y a
continuación otra posterior (3 de julio).
– El paso de años, mediante la sobreimpresión de un año (1950) y a continuación la
sobreimpresión de otro posterior (1953).
– El cambio de estaciones: utilizando imágenes propias de cada estación (nieve, verdor en
la vegetación, sol radiante, caída de las hojas, etc.).
– El cambio de un lugar a otro: utilizando imágenes referenciales, características de esos
lugares. Ejemplo: Alguien que sale de Madrid (aeropuerto de Barajas) y aparece en Sevi-
lla (al lado de la Giralda), no necesita mayor explicación, las imágenes se bastan para
expresar lo ocurrido.
Para el rodaje de una película es necesario confeccionar previamente un “guión”, que cons-
ta de:
– Elementos literarios: Argumento, diálogos, descripción de lugares, etc.
– Elementos técnicos: Relación de cámaras, grúas, luces, decorados, vestuario, maquilla-
je, etc.

15
-Pág.43-
L E N G U A Y L I T E R AT U R A

EL VÍDEO
Es un aparato electrónico que puede grabar y reproducir imágenes y sonidos. Para ello nece-
sita estar conectado a un monitor de TV, de donde puede tomar la información o bien reprodu-
cirla.
El vídeo permite que se reciba “la información” cuando el receptor considere oportuno, y
puede repetir la misma operación cuantas veces lo desee.

EL ORDENADOR
Es otro aparato electrónico que permite realizar múltiples operaciones:
– Matemáticas a gran velocidad.
– Mecanográficas más completas que en la máquina de escribir.
– Acumular gran cantidad de información.
– Relacionar la información suministrada.
– Aplicaciones fotográficas.
– Aplicaciones musicales.
– Aplicaciones en internet.
– Reproducir cualquier tipo de información procesada (por impresora).
Los ordenadores permiten compartir información, colaborar “a distancia” en trabajos, soli-
citar informaciones, consultar archivos, etc. desde donde esté ubicado el ordenador; su gran
éxito se basa en que ha hecho desaparecer las “barreras” de espacio y tiempo.

6. LENGUAJE VERBAL, MUSICAL, GESTUAL


Y LENGUAJE DE LOS OBJETOS Y DE LOS AMBIENTES
A lo largo de la unidad venimos insistiendo en la importancia que tienen para la comunica-
ción humana las informaciones que pueden aportar los lenguajes “no verbales”.

Estos lenguajes pueden ser infinitos, tantos como la capacidad humana sea capaz de crear;
de entre todos, destacaremos los que consideramos los más importantes:

16
-Pág.44-
U . D . 4 . - S ISTEMAS DE COMUNICACIÓN VERBAL Y NO VERBAL

LA MÍMICA
Es la forma de comunicación que consiste en expresar cualquier mensaje a través de la
expresión corporal: movimientos, posturas, gestos, muecas, etc.

El actor de mimo puede maquillarse la cara, vestir de cualquier manera, auxiliarse de efec-
tos luminosos, moverse, etc.; todo, excepto hablar.

LA CANCIÓN
Es la forma de comunicación que se caracteriza fundamentalmente por emitir con la voz
sonidos armoniosos (palabras y otros) que forman melodías (canciones).

El emisor (cantante) puede hacerse acompañar por otros sonidos (música) con el fin de que
la emisión de su mensaje resulte más melódica y atractiva.

LA DRAMATIZACIÓN
Es la forma de comunicación en la que el emisor transmite su mensaje “actuando”, es decir:
representando una actitud que no es la propia del emisor, sino la que conviene al mensaje para
que sea más creíble.
Se puede dramatizar en las siguientes situaciones:
– En el relato de un acontecimiento.
– Al dar una noticia “extraña”.
– Al recitar unos versos.
– Al relatar un cuento.
– Contando chistes.
– Imitando a otras personas.
– El ejemplo más claro lo tenemos en la actuación de unos actores representando cualquier
obra de teatro, etc.

LA RECITACIÓN
Es la forma de comunicación hablada en la que el emisor lee en voz alta o pronuncia un texto
aprendido de memoria, dotándolo de un tono enfático o artístico.
Para ello debe:
– Pronunciar con claridad y nitidez.
– Respetar las pausas (comas, puntos, etc.).
– Utilizar diversos recursos paralíngüísticos (elevación e inflexiones de voz, etc.).

17
-Pág.45-
L E N G U A Y L I T E R AT U R A

LOS RITOS Y LAS COSTUMBRES


En algunas situaciones de comunicación (actos religiosos, actos institucionales, actos festi-
vos tradicionales, etc.) existe toda una serie de signos “verbales” y “no verbales” que vienen
establecidos por norma o costumbre.
Estos signos se dan como elementos necesarios para que se produzca la situación comuni-
cativa esperada, y se repiten de forma invariable, según el orden establecido por la tradición,
cada vez que se celebra esa ceremonia o acto social determinado.

Ejemplos:
– El vestuario característico para algunas ceremonias religiosas.
– Las “frases hechas” que se utilizan en determinados actos institucionales.

RESUMEN
– La comunicación “verbal” y la “no verbal” son interactivas, y sus relaciones pueden ser:
Complementarias y Redundantes.
– Cada colectividad cultural, grupo humano, etc., puede tener lenguajes “no verbales” pro-
pios, incluso formar subgrupos por razones de sexo, edad, procedencia, cultura, etc.
– Los medios de comunicación social: prensa, radio, TV, etc. tienen una gran influencia e
importancia en la comunicación humana.
– La publicidad intenta “manipular” al receptor.
– El mundo de la comunicación está permanentemente “abierto” a los progresos técnicos:
radio, TV, cine, vídeo, ordenador, internet, etc.

18
-Pág.46-
U . D . 4 . - S ISTEMAS DE COMUNICACIÓN VERBAL Y NO VERBAL

EJERCICIOS DE AUTOCOMPROBACIÓN
Responda a las siguientes cuestiones en hoja aparte. Analice sus respuestas y verifíquelas en
la hoja de soluciones que aparece al final de la prueba.
Aproveche para obtener conclusiones de lo que debe repasar, aclarar o consultar.

1. En una nota escrita se utilizan:

A. Signos verbales B. Signos no verbales

C. Signos gestuales D. Signos parafrasísticos

2. Un plano es un signo de tipo:

A. Indicio B. Icono C. Símbolo D. Algoritmo

3. Los silencios “intencionados” en la cadena oral son estudiados por:

A. La Paralingüística B. La Cinésica

C. La Proxémica D. La Fraseología

4. Los gestos son estudiados por:

A. La Paralingüística B. La Cinésica

C. La Proxémica D. La Sintética

5. La voz entre los latinos suele ser:

A. Menos elevada que entre los anglosajones

B. Más elevada que entre los anglosajones

C. Más estridente que entre los chinos

D. Más musical que entre los japoneses

6. El Editorial de un periódico lo firma:

A. El director B. El jefe de una sección

C. Va sin firma D. Un periodista

7. El desarrollo de la noticia aparece en:

A. El Titular B. La Entradilla C. El Cuerpo D. El Lead

19
-Pág.47-
L E N G U A Y L I T E R AT U R A

8. La radio utiliza signos:

A. Visuales B. Táctiles C. Sonoros D. Sintácticos

9. La expresión “El séptimo arte” se refiere a:

A. La radio B. La TV C. El cine D. La escultura

10. El actor de “mimo” puede:

A. Cantar y bailar B. No puede cantar, pero sí puede bailar

C. Recitar y gritar D. Bailar y chillar

RESPUESTAS A LOS EJERCICIOS

1. A 2. B 3. A 4. B 15. B

6. C 7. C 8. C 9. C 10. B

20
-Pág.48-
portada TROPA 19/3/07 19:51 Página 1

FUERZAS ARMADAS
PROFESIONALES
CURSO DE APOYO
A LA PREPARACIÓN
DE LAS PRUEBAS DE ACCESO
A UNA RELACIÓN DE SERVICIOS
DE CARÁCTER PERMANENTE

LENGUA Y LITERATURA
3ª parte
Unidad didáctica 5

DIGEREM

MINISTERIO
DE DEFENSA
FUERZAS ARMADAS SUBDIRECCIîN GENERAL
DE TROPA Y MARINERIA
PROFESIONAL
PROFESIONALES
CURSO DE APOYO
A LA PREPARACIÓN
DE LAS PRUEBAS DE ACCESO
A UNA RELACIÓN DE SERVICIOS
DE CARÁCTER PERMANENTE

LENGUA CASTELLANA Y LITERATURA


3ª parte
Unidad didáctica 5

-Pág.1-
La Ley 8/2006 de Tropa y Marinería, en su artículo 16,1, establece que “la formación
en las Fuerzas Armadas garantizará que los militares profesionales de tropa y
marinería puedan adquirir, actualizar o ampliar sus conocimientos para un mayor
desarrollo personal y profesional”. En cumplimiento de este mandato, el Ministerio
de Defensa edita el presente material didáctico para facilitar a los militares
profesionales de tropa y marinería, alumnos de los cursos de formación
presencial que se imparten a través de la Dirección General de Reclutamiento y
Enseñanza Militar, los apoyos necesarios para preparación de dichos cursos, que
permitirán, siempre que superen las pruebas correspondientes, la obtención de la
titulación de graduado en Educación Secundaria, acreditación para el acceso a
los ciclos formativos de la Formación Profesional de grado medio o de grado
superior, acceso a las Escalas de Suboficiales, Tropa Permanente, Guardia Civil
y Policía Nacional.

CATÁLOGO GENERAL DE PUBLICACIONES


http://www.060.es

Edita:

© Autor y editor
NIPO: 076-10-204-9 NIPO: 076-10-205-4 (edición en línea)
Depósito Legal: M-32363-2009
Diseño y programación: cimapress
Tirada: 1300 ejemplares
Fecha de edición: septiembre, 2010

Prohibida la reproducción total o parcial de esta obra, por cualquier medio sin autorización escrita del editor
LENGUA Y LITERATURA
3ª parte

SUMARIO

Unidad didáctica Pág.

5. LA LITERATURA 5

-Pág.3-
U.D. 5.- LA L I T E R AT U R A

ÍNDICE
OBJETIVOS . . . . . . . . . . . . . . . . . . . . . . . . . . . . . . . . . . . . . . . . . . . . . . . . . . . . . . . . . . . . . .

INTRODUCCIÓN . . . . . . . . . . . . . . . . . . . . . . . . . . . . . . . . . . . . . . . . . . . . . . . . . . . . . . . . . 3

MAPA CONCEPTUAL . . . . . . . . . . . . . . . . . . . . . . . . . . . . . . . . . . . . . . . . . . . . . . . . . . . . . 4

DESARROLLO DE CONTENIDOS . . . . . . . . . . . . . . . . . . . . . . . . . . . . . . . . . . . . . . . . . . 5

1. LA LITERATURA COMO PRODUCTO LINGÜÍSTICO Y ESTÉTICO . . . . . . . 5

2. LA LITERATURA COMO INSTRUMENTO DE TRANSMISIÓN


Y DE CREACIÓN CULTURAL Y COMO EXPRESIÓN HISTÓRICO-SOCIAL . . 5
2.1. PERÍODOS. CARACTERÍSTICAS HISTÓRICAS, SOCIALES Y CULTURALES . . . 6
2.2. OBRAS Y AUTORES DE ESPECIAL RELEVANCIA DE CADA PERÍODO . . . . . . . 10

3. LOS GÉNEROS LITERARIOS . . . . . . . . . . . . . . . . . . . . . . . . . . . . . . . . . . . . . . . . . 32


3.1. LA ÉPICA (ARGUMENTO, NARRADOR, ESPACIO, TIEMPO, ESTRUCTURA,
TEMA, ETC.). OBRAS DESTACADAS DEL GÉNERO ÉPICO. EVOLUCIÓN
DEL GÉNERO ÉPICO . . . . . . . . . . . . . . . . . . . . . . . . . . . . . . . . . . . . . . . . . . . . . . . 32
3.2. LA DRAMÁTICA (TEXTO Y ESPECTÁCULO; ESTRUCTURA, PERSONAJES,
RECURSOS, ETC.). OBRAS DESTACADAS DEL GÉNERO DRAMÁTICO.
EVOLUCIÓN DEL GÉNERO DRAMÁTICO . . . . . . . . . . . . . . . . . . . . . . . . . . . . . . 33
3.3. LA LÍRICA (VERSO Y PROSA; TEMAS, RECURSOS RETÓRICOS).
OBRAS DESTACADAS DEL GÉNERO LÍRICO . . . . . . . . . . . . . . . . . . . . . . . . . . . 34
3.4. EL ENSAYO Y OTROS GÉNEROS LITERARIOS . . . . . . . . . . . . . . . . . . . . . . . . . . 34
3.5. TEMAS Y MITOS LITERARIOS . . . . . . . . . . . . . . . . . . . . . . . . . . . . . . . . . . . . . . . 35

RESUMEN . . . . . . . . . . . . . . . . . . . . . . . . . . . . . . . . . . . . . . . . . . . . . . . . . . . . . . . . . . . . . . . 35

EJERCICIOS DE AUTOCOMPROBACIÓN . . . . . . . . . . . . . . . . . . . . . . . . . . . . . . . . . . . 36

RESPUESTAS A LOS EJERCICIOS . . . . . . . . . . . . . . . . . . . . . . . . . . . . . . . . . . . . . . . . . 36

-Pág.5-
L E N G U A Y L I T E R AT U R A

O B J E T I VO S
Al finalizar el estudio de esta Unidad Didáctica, el alumno será capaz de:

• Conocer y valorar la literatura como producto lingüístico, estético y cultural.

• Reconocer y valorar la literatura como medio de transmisión cultural y como


expresión histórico-social.

• Reconocer y diferenciar los períodos literarios de la literatura española.

• Reconocer y diferenciar los géneros literarios.

• Expresar con estilo literario (personal) sus inquietudes: sentimientos, fantasías, etc.

-Pág.6-
U.D. 5.- LA L I T E R AT U R A

INTRODUCCIÓN
L a literatura es el arte cuyo medio de expresión es la palabra, tanto en la modalidad
oral como en la escrita, y en el que el lenguaje tiene como función primordial la
formación de un texto que sea estético, chocante, atractivo e interesante.

También denominamos literatura al conjunto de obras literarias de un país, de una época,


de un estilo, etc.

En la comunicación literaria intervienen los mismos elementos que vimos en la comuni-


cación oral y escrita, si bien, con algunos matices:

– El emisor es el autor literario; es el creador, el artista que crea la obra.

– El receptor es el público que lee una obra, escucha un poema o ve una representa-
ción teatral; es un destinatario indeterminado.

– El mensaje va siempre desde el emisor hasta el receptor.

– El código literario utiliza los mismos elementos lingüísticos que la lengua común o
estándar, pero sus palabras suelen tener diversos significados connotativos (figurados).

– El contexto situacional no es compartido por el emisor y el receptor; no sólo los sepa-


ra el espacio, sino también el tiempo; a veces, siglos.

Nuestra Literatura goza de un gran prestigio internacional, a ello contribuyen tanto los
autores españoles como los hispanoamericanos.

-Pág.7-
L E N G U A Y L I T E R AT U R A

M A PA C O N C E P T U A L

LENGUA CASTELLANA Y
LITERATURA

LA LITERATURA

COMO INSTRUMENTO DE
COMO PRODUCTO
CREACIÓN CULTURAL Y GÉNEROS
LINGÜÍSTICO Y
EXPRESIÓN HISTÓRICO- LITERARIOS
ESTÉTICO
SOCIAL Y CULTURAL

PERÍODOS Y
OBRAS Y AUTORES
CARACTERÍSTICAS

LA ÉPICA LA DRAMÁTICA LA LÍRICA EL ENSAYO

-Pág.8-
U.D. 5.- LA L I T E R AT U R A

1. LA LITERATURA COMO PRODUCTO LINGÜÍSTICO


Y ESTÉTICO
Un verso, un poema, una novela, una comedia, etc., son producciones lingüísticas con una
intencionalidad comunicativa determinada y una intencionalidad estética o poética; es decir,
intenta conseguir belleza a través de la palabra.
Toda obra literaria es una obra de arte; el autor literario, igual que el pintor, el músico, el
escultor, etc., toma sus materiales: las palabras, y las selecciona, combina y dispone de la forma
que cree más oportuna para crear su obra: la obra literaria.
Estos materiales (las palabras) están a disposición de todos los hablantes, pero sólo algunos,
“los virtuosos”, son capaces de utilizarlos de la forma precisa para conseguir una obra artística:
la obra literaria.
Cada autor literario, como cualquier artista, procura ser original, especialmente en la forma; un
concepto puede ser expresado, definido, etc., de diferentes formas, pero cuando se hace literaria-
mente, los citados conceptos adquieren una significación más amplia, especial, con matices sor-
prendentes para el receptor. De ahí que la forma (expresión) en el lenguaje literario prevalezca sobre
el fondo (contenido), puesto que va a propiciar que el texto tenga la función estética deseada.

2. LA LITERATURA COMO INSTRUMENTO


DE TRANSMISIÓN Y DE CREACIÓN CULTURAL
Y COMO EXPRESIÓN HISTÓRICO-SOCIAL
A través de la literatura, en las sociedades primitivas de forma oral de padres a hijos, y a par-
tir de la aparición de la escritura, de forma oral y escrita, los pueblos han ido transmitiendo sus
costumbres y tradiciones (inquietudes religiosas, ceremonias sociales, cuentos, leyendas, for-
mas de cocinar, tipos de vestimenta, utensilios utilizados, relaciones de amistad con otros pue-
blos vecinos, etc.).
Todo ese cúmulo de acciones y acontecimientos han ido quedando en la memoria colectiva
de los pueblos gracias a la literatura, de tal forma que podemos afirmar que es, sin duda, el prin-
cipal instrumento de transmisión cultural que tiene la humanidad.
Los autores literarios escriben sus obras, bien como:
– Fruto de su imaginación.
– Inspirados en acontecimientos del pasado.
– O como fruto de la motivación por lo que acontece a su alrededor; en este caso actúan como
auténticos notarios de actualidad de la época y lugar en que viven, con lo que están contri-
buyendo permanentemente a que la cultura de una sociedad sea cada vez más extensa.

5
-Pág.9-
L E N G U A Y L I T E R AT U R A

“Los pueblos que tienen un pasado histórico relevante, suelen tener una extraordinaria
riqueza literaria, tal acontece con la literatura española.”

2.1. PERÍODOS. CARACTERÍSTICAS HISTÓRICAS, SOCIALES


Y CULTURALES
Cada literatura aparece en el seno de una sociedad determinada, y se va desarrollando de forma
paralela a su realidad histórico-social. Esta realidad constituye el contexto, que será distinto en
cada momento, pues dependerá de las circunstancias y acontecimientos propios de cada época.
El conjunto de circunstancias, acontecimientos, intereses, evolución de la propia lengua,
etc., de esos momentos propiciará que los temas y formas de escribir sean distintos de unas épo-
cas a otras, por lo que la literatura podemos estudiarla atendiendo a esos “paréntesis tempora-
les” más o menos extensos, que denominaremos períodos.
Estos períodos suelen compartir las mismas características en cualquier tipo de manifesta-
ción cultural: las artes, la historia, la filosofía... y, por supuesto, la literatura.
En la Literatura Española, desde sus inicios hasta la actualidad, distinguimos los siguientes
períodos:
EDAD MEDIA.—Es un largo período que comprende desde finales del siglo X hasta fines
del siglo XV.
Características
Históricas: Todas las referentes al proceso de la Reconquista hasta su final con la toma de
Granada por los Reyes Católicos.
Sociales: La sociedad feudal estaba constituida por tres estamentos sociales, claramente
diferenciados:
– Nobleza: Grandes y pequeños señores (duques, hidalgos, etc.).
– Clero: Eclesiásticos.
– Pueblo llano: Artesanos, campesinos, etc.
A partir del siglo XIV surge una nueva clase social: La burguesía, que llega a
adquirir un gran poder económico, pero no forma parte de la nobleza.
Culturales: La cultura se desarrolla en los monasterios, después (s. XIII) aparecerán las
universidades.
Aparece el estilo ROMÁNICO y, posteriormente, el GÓTICO.

RENACIMIENTO.—Comprende: Finales del siglo XV y siglo XVI.


Características
Históricas: Consolidación de la unidad nacional de España por los Reyes Católicos.
Creación del Imperio español con Carlos I y Felipe II.

6
-Pág.10-
U.D. 5.- LA L I T E R AT U R A

Sociales: La nobleza pierde su carácter belicista, se refina y se convierte en aristocrática.


La burguesía adquiere mayor consideración social.
El pueblo llano se empobrece más.
Culturales: Se retoman los temas y modelos de la cultura clásica greco-latina.
Se dan grandes avances técnicos y científicos.
El descubrimiento de la imprenta potencia la expansión de la cultura.
Los grandes artesanos se convierten en artistas y gozan de mayor libertad y
consideración social.
BARROCO.—Comprende el siglo XVII. (Casi todo este siglo y gran parte del anterior es lo
que se conoce como “Siglo de Oro”, o “Siglos de Oro”).
Características
Históricas: Decadencia del Imperio español.
Continuas guerras en Europa, que empobrecen las arcas castellanas.
Sociales: Las clases bajas se empobrecen aún más.
El gobierno de validos provoca una gran desorganización social, gobiernan “de
cara a la corte y de espaldas al pueblo”.
Culturales: Se buscan nuevas formas de expresión.
Se cultivan todos los géneros.
Se crea la llamada “comedia española”.
Se crean los “corrales de comedia” para representar las obras teatrales.
NEOCLASICISMO.—Siglo XVIII.
Características
Históricas: Final de la dinastía de los Austrias y principio de los Borbones.
Sociales: Intento de la primera reforma agraria.
Se potenció el comercio.
Se propulsó la industria.
Culturales: Se hicieron cambios notables en la organización general de la enseñanza, que
afectaron a todos los niveles.
Se crearon nuevos centros institucionales: La Real Academia de la Lengua, la
Real Academia de la Historia, etc.
ROMANTICISMO.—Comprende desde el inicio del siglo XIX hasta poco más de su pri-
mera mitad.
Características
Históricas: Invasión francesa y guerra de la Independencia.
Período absolutista de Fernando VII.
Reinado de Isabel II hasta su destronamiento (1868).

7
-Pág.11-
L E N G U A Y L I T E R AT U R A

Sociales: Pobreza generalizada, especialmente entre las clases bajas.


Escaso desarrollo industrial.
La agricultura tampoco desarrolló nuevas técnicas de trabajo.
Exilio de los opositores al régimen absolutista de Fernando VII.
Culturales: Valoración de lo subjetivo: libertad, originalidad, etc.
Construcción de edificios de carácter público: teatros, bibliotecas, parques,
mercados, plazas, etc.
La pintura y la música adquieren gran importancia.
REALISMO.—Comprende la segunda mitad del siglo XIX.
Características
Históricas: Destronamiento de Isabel II.
Gobierno Provisional.
Restauración de la monarquía con el rey Alfonso XII.
Pérdida de las últimas colonias de Ultramar.
Sociales: La agricultura no mejoró.
Comenzó un cierto desarrollo industrial.
Movimientos migratorios desde el campo a las ciudades,
Surgen las primeras asociaciones de carácter obrero.
Culturales: Se funda la Institución Libre de Enseñanza.
Los autores reflejan en sus obras la realidad de lo que acontece a su alrededor.
LITERATURA ACTUAL.—Con esta denominación nos referiremos a todo el siglo XX.
Características
Históricas: Reinado de Alfonso XIII.
Dictadura del general Primo de Rivera.
Segunda República.
Guerra Civil (1936-39).
Dictadura del general Franco.
Proclamación del rey Juan Carlos I.
Instauración de la democracia y aprobación de la Constitución de 1978.
Sociales: En el primer cuarto de siglo se da una cierta industrialización del país, que se
ve potenciada con la construcción de nuevas vías ferroviarias y carreteras.
Las clases bajas (el proletariado) se ven cada vez más impotentes de alcanzar
cierto bienestar social por el excesivo enriquecimiento de la burguesía, lo que
provoca graves descontentos.
La lamentable guerra “civil” con sus muertos, y la posterior posguerra con sus
exilios y emigraciones masivas de trabajadores a otros países, sume al país en

8
-Pág.12-
U.D. 5.- LA L I T E R AT U R A

un estado de máxima pobreza, de la que no saldrá hasta la década de los años


“60”, en los que se inicia un importante y definitivo desarrollo económico.
En la actualidad, con la democracia consolidada, la sociedad española partici-
pa en los foros internacionales más importantes.
Culturales: Son muchas e importantes, pero nos vamos a centrar en definir los diversos
movimientos literarios que han configurado la literatura de este siglo XX.
MODERNISMO: Corriente literaria que se caracteriza por su objetivo principal: “Alcanzar
la belleza”, a través de:
– La sonoridad de las palabras.
– La musicalidad de los versos.
– Los temas de sus composiciones: Exóticos, sensuales y la defensa de lo hispano e his-
panoamericano como contraposición a otras nacionalidades e intereses imperialistas.
LA GENERACIÓN DEL “98”: Movimiento literario, formado por un conjunto de escrito-
res: poetas, novelistas, dramaturgos, ensayistas, etc., que comparten su preocupación por Espa-
ña, que acaba de perder las últimas colonias de Ultramar (1898).
Ante todo quieren que la nación salga de la atonía en la que se encuentra inmersa, por ello:
– Exaltan los valores de la “España eterna y espontánea”.
– Rechazan la política del momento.
– Se preguntan por el sentido de la vida, la existencia del hombre, la vida, la muerte, la
religión, etc.
LA GENERACIÓN DEL “27”: Grupo de jóvenes poetas que se dan a conocer con poemas,
publicaciones, conferencias, etc., en la conmemoración del tercer centenario de la muerte del
poeta cordobés don Luis de Góngora y Argote.
Este grupo, admirador del poeta Juan Ramón Jiménez, consigue revitalizar la poesía espa-
ñola de tal forma, que se puede hablar de un segundo siglo de Oro (en la poesía) .
LITERATURA DE POSGUERRA Y AÑOS SIGUIENTES
Hasta los años “60”:
Se inicia con una fuerte censura que provoca cierta desorientación en los escritores, no saben
hasta dónde pueden expresarse.
Al irse suavizando, algunos escritores consiguen sortear la citada censura y tratan temas de
denuncia social: injusticias, falta de libertad, de compromiso ético, etc.
Posteriormente:
Se desarrolla una literatura de tipo experimental, preocupándose por encontrar nuevas for-
mas de expresión, nuevos lenguajes, etc.
En el último cuarto de siglo:
Se abandona parcialmente la literatura experimental y se vuelve a mostrar interés por las for-
mas y temas de siempre. En este sentido, los escritores hispanoamericanos están haciendo una
extraordinaria contribución a la literatura española.

9
-Pág.13-
L E N G U A Y L I T E R AT U R A

2.2. OBRAS Y AUTORES DE ESPECIAL RELEVANCIA


EN CADA PERÍODO

2.2.1. EDAD MEDIA


A) ALTA EDAD MEDIA

➢ El surgimiento de las lenguas romances


En el siglo X fragua de modo definitivo la fragmentación del latín vulgar hablado en
época visigótica, formándose los dialectos romances galaico – portugués, astur – leonés, cas-
tellano, navarro – aragonés y catalán, además del mozárabe del área musulmana. En todo caso,
las diferencias entre ellos eran mínimas y se mantenía, por tanto, una cierta unidad lingüísti-
ca, excepción hecha del vascuence.
Las primeras manifestaciones escritas en lengua romance aparecen a finales del siglo X.
Se trata de las Glosas Emilianenses (San Millán de la Cogolla) y las Glosas Silenses (Santo
Domingo de Silos).
El paso del tiempo y la fragmentación política hicieron posible el paulatino alejamiento
fonético, léxico y sintáctico de los distintos dialectos que se transformaron en lenguas diferen-
tes (gallego, castellano, catalán).
El castellano fue el que experimentó una mayor expansión debido al crecimiento de Cas-
tilla y su posterior hegemonía. En su avance fue absorbiendo al mozárabe y relegando a espa-
cios cada vez más reducidos al galaico – portugués y al dialecto astur – leonés.

➢ El renacimiento cultural a partir del siglo XII


CASTILLA Y LEÓN
Hasta mediados del siglo XII predominan casi de forma absoluta las composiciones en latín;
pero poco a poco se va haciendo sitio la lengua romance como instrumento literario.
Entre las primeras obras en lenguas romances se citan:
— Cantar de Mío Cid, primera obra épica en castellano, escrita a mediados del siglo XII,
narra las hazañas de Rodrigo Díaz de Vivar, el Cid, desde su destierro de Castilla hasta
la venganza de la afrenta que, en Corpes, sufrieron sus hijas de manos de los infantes de
Carrión. Se divide en tres partes o cantares: del destierro, de las bodas, de la afrenta de
Corpes.
— Auto de los Reyes Magos, composición dramática redactada en castellano hacia 1200.
— Cantigas d’amigo y Cantigas de escarnio, escritas en galaico – portugués a finales del
siglo XII.

10
-Pág.14-
U.D. 5.- LA L I T E R AT U R A

CORONA ARAGONESA
El siglo XII supuso el inicio del desarrollo literario como consecuencia de un importantí-
simo influjo provenzal, tanto así que sus primeras manifestaciones se redactan en lengua
provenzal, dando paso posteriormente al catalán como lengua literaria. En estos momentos,
destacan las composiciones líricas de autores como BERENGUER DE PALOL.
Los relatos históricos, al igual que en los reinos occidentales, tuvieron un importante des-
arrollo, siendo dignos de mención:
— Gesta comitum barcinonensium.
— De rebus Hispaniae.
— Libre dels feyts, impulsado por JAIME I y por JAIME SARROCA, obispo de Huesca.

➢ La Escuela de Traductores de Toledo


La conquista de Toledo y de las tierras al sur del Sistema Central supuso, desde el punto de
vista de la cultura, un momento de eclosión. El aluvión de obras y personajes cultos de la
parte de la España musulmana recién incorporada se acrecienta con la llegada, desde más al sur,
de muchos otros, desplazados por el avance de la intransigente ortodoxia religiosa que los almo-
rávides, primero, y, más tarde, los almohades, representaban. Además, estos hechos llamarán a
España a doctos estudiosos europeos, ansiosos de beber directamente en las nuevas fuentes.
PEDRO EL VENERABLE, abad de Cluny, y RAIMUNDO, arzobispo de Toledo, a mediados del
siglo XII, reunieron en la histórica capital a cuatro hombres (ROBERTO DE CHESTER, HERNÁN DE
CARINTIA, el mozárabe PEDRO DE TOLEDO y el musulmán MOHAMED DE TOLEDO) para traducir
el Corán y otras obras sobre MAHOMA que se compilaron en el Corpus Mahometium. Sin
hacerlo expresamente, se había fundado la Escuela de Traductores de Toledo.

B) BAJA EDAD MEDIA


➢ Siglo XIII
Desde el punto de vista de la literatura, el siglo XIII contempla la superación de la tradi-
cional influencia oriental, recibida a través de musulmanes y judíos, por la imposición de
corrientes europeas. Las manifestaciones literarias y artísticas iniciaron un proceso de trans-
formación secularizadora y humanista.
El llamado “mester de clerecía”, con su sempiterna cuaderna vía, protagoniza la lírica y
hace del castellano una lengua plenamente literaria. La figura de GONZALO DE BERCEO, primer
autor en castellano de nombre conocido, y obras como la suya de Los Milagros de Nuestra
Señora y otras como el Libro de Aleixandre y el Poema de Fernán González, representan esta
fórmula literaria.
ALFONSO X EL SABIO, ganó su apelativo como consecuencia de su intensa labor enciclopé-
dica, lírica y jurídica. Impulsó la Escuela de Traductores de Toledo, dirigió sus obras y fue
autor de otras muchas, relativas a diversidad de temas, entre las que destacan:

11
-Pág.15-
L E N G U A Y L I T E R AT U R A

— Tablas astronómicas alfonsíes.


— Libros del saber de Astronomía.
— Estoria de España.
— General Estoria.
Fue autor, además, de una rica producción lírica, encabezada por las Cantigas, escritas en
gallego, y patrocinó la recopilación jurídica del Código de las Siete Partidas.
ALFONSO X dio el impulso definitivo al castellano, que se convierte en la lengua oficial de
la corte. Los textos administrativos y jurídicos se redactan en la lengua nacida siglos atrás en el
norte de Castilla.
En la Corona aragonesa, en el siglo XIII, la conquista de Valencia y Mallorca supondrá la
llegada de influencias musulmanas. En este siglo, además de ARNAU DE VILANOVA, la figura de
RAIMOND LLULL representa la culminación del catalán medieval. Escribió 250 libros, casi
todos en catalán, destacando entre ellos, fundamentalmente:
— Libre de meravelles.
— Arbre de Sciència.
— Libre de l’orde de cavalleria.
— Libre de contemplació.
— Libre del gentil e los tres savis.

➢ Siglo XIV
El crecimiento de las clases burguesas potenció el proceso de secularización y el huma-
nismo. Tiempo, además, de cambios políticos, económicos, demográficos y sociales, la litera-
tura y el arte serán su fiel reflejo. La Iglesia pierde el monopolio de la cultura y ésta desciende
en la pirámide social, haciéndose cortesana y popular.
En este marco se desarrolla la actividad literaria, con autores y obras como:
— El infante DON JUAN MANUEL, creador del Conde Lucanor, el Libro de los Estados y el
Libro del Caballero y el Escudero.
— JUAN RUIZ, más conocido como ARCIPRESTE DE HITA, autor del popular Libro del Buen
Amor.
— El canciller DON PEDRO LÓPEZ DE AYALA, con su Rimado de Palacio.
— DON SEM TOB, judío que dedicó a PEDRO I EL CRUEL sus Proverbios morales.
— El Cantar de Rodrigo.
— El Poema de Alfonso Onceno.

12
-Pág.16-
U.D. 5.- LA L I T E R AT U R A

➢ Siglo XV
El paulatino triunfo del humanismo clasicista se iniciará con las influencias humanistas
europeas llegadas desde la corte del Papa cismático PEDRO DE LUNA en Aviñón. La filosofía, la
historiografía y la narrativa preludian el Renacimiento, que alcanzó su plenitud desde el rei-
nado de los REYES CATÓLICOS.
Entre los autores y obras literarias del siglo destacan:
— El MARQUÉS DE SANTILLANA, ÍÑIGO LÓPEZ DE MENDOZA, autor del Prohemio al Con-
destable de Portugal, las Serranillas, la Comedieta de Ponza, sonetos y otros poemas.
— JUAN DE MENA, con su Laberinto de Fortuna.
— JORGE MANRIQUE, autor de la elegía Coplas a la muerte de su padre.
— Los romances, relatos muy populares con diversidad de temas, que fueron siendo reu-
nidos en colecciones.
— ENRIQUE DE VILLENA, autor, en prosa, del Arte de Trovar y Los Doce trabajos de Hér-
cules.
— Los poetas aragoneses ANDREU FEBRER, JORDI DE SANT JORDI y, sobre todo, AUSIAS
MARCH.
— JOHANOT MARTORELL y MARTÍ JOHAN DE GALBA, autores de la novela de caballería
Tirant Lo Blanch, de trascendentalísima influencia posterior.

2.2.2. RENACIMIENTO

A) ÉPOCA DE LOS REYES CATÓLICOS

El desarrollo de la literatura en lengua castellana sustenta sus bases sobre importantes apor-
taciones culturales fruto del espíritu humanista. Efectivamente, uno de los principales huma-
nistas de la época, ELIO ANTONIO DE NEBRIJA (cuyo verdadero nombre era ANTONIO MARTÍNEZ
DE CALA) fue el autor o participó en las más importantes aportaciones intelectuales para unir la
tradición latina con la lengua castellana y puso los pilares para convertir ésta en vehículo cul-
tural de primer orden. NEBRIJA publica en 1492 su Gramática o Arte de la lengua castellana,
primera obra de este tipo dedicada a una lengua distinta del latín y surgida de él. Impulsado por
razones políticas, como él mismo reconoce en el prólogo, esta obra permite la consolidación de
los aspectos morfológicos y sintácticos del castellano. Su labor lingüística en este sentido se
completa con las Reglas de Ortografía castellana (1517) y con el Vocabulario español – latín,
latín – español.

Obra trascendental fue también la Biblia Políglota (1517), primer gran fruto intelectual de la
Universidad de Alcalá de Henares, iniciativa del CARDENAL CISNEROS y en la que colaboraron,

13
-Pág.17-
L E N G U A Y L I T E R AT U R A

entre otros, ANTONIO DE NEBRIJA, DEMETRIO EL CRETENSE, HERNÁN NÚÑEZ, PABLO CORONEL,
ALFONSO DE ZAMORA y ALFONSO DE ALCALÁ. En ella, el Antiguo Testamento se presenta en
hebreo y caldeo además de en griego y latín, lenguas en las que se redacta el Nuevo Testamento.

POESÍA
En la época de los REYES CATÓLICOS la poesía alcanza un importante desarrollo y perfec-
ción. El tema religioso es predominante, destacando las obras de FRAY ÍÑIGO DE MENDOZA, FRAY
AMBROSIO MONTESINO y JUAN DE PADILLA; pero también los temas profanos encuentran su
ámbito en las obras de GARCI SÁNCHEZ DE BADAJOZ y RODRIGO DE COTA.

TEATRO
Después de un largo paréntesis, desde el Auto de los Reyes Magos (siglo XII), sin ningún
texto dramático conservado, es ahora cuando se inicia el despegue de este género, que alcanza-
rá en los dos siglos siguientes un importante desarrollo.

Gómez Manrique (1412-1490)


Es el primer autor teatral de nombre conocido en la historia de la literatura española,
autor de:
— Representación del nacimiento de Nuestro Señor
— Coplas fechas para Semana Santa

Juan del Encina (1469-1529)


Se le ha llamado “padre del teatro español”
— Églogas de Navidad
— Églogas de la Pasión
— Égloga de Antruejo o Carnestolendas
— Égloga de Fileno, Zambrano y Cardonio
— Cristino y Febea

Lucas Fernández (1474-1542)


— Auto de la Pasión

La Celestina
Mención aparte merece una de las obras cumbre de la literatura española, precursora del
género narrativo en prosa, de la novela. La Tragicomedia de Calisto y Melibea, conocida
como La Celestina, apareció publicada por primera vez en Burgos en 1499. Con casi

14
-Pág.18-
U.D. 5.- LA L I T E R AT U R A

completa seguridad, el primer acto de la misma es obra anterior, que circulaba por los
ambientes universitarios salmantinos, recogida por el autor FERNANDO DE ROJAS quien
completó los actos restantes (16 inicialmente y hasta 22 en la edición de 1522).
El fiel retrato de los caracteres, el fondo moralizante y los trazos cómicos confieren a esta
obra romántica, que alcanzó gran popularidad, una importancia capital en la historia de la
literatura española.

B) ÉPOCA DE CARLOS I Y FELIPE II

En el siglo XVI se inicia un ciclo, que continuará en la centuria siguiente, que con el térmi-
no Siglo de Oro, marca la etapa más brillante, prolífica y rica de la historia de la literatura espa-
ñola. Todos los géneros son cultivados, surgen, incluso, subgéneros específicos y la producción
se multiplica y populariza gracias a la introducción de la imprenta.

POESÍA
Garcilaso de la Vega (1501-1536)
Caballero, guerrero y poeta, bebió en las fuentes italianas adaptando sus formas poéticas al
sentir hispano, introduciendo en nuestra poesía el soneto, entre otras estrofas y junto a otras
técnicas y recursos. Entre sus obras figuran 38 sonetos, tres églogas y otras composiciones.

Juan Boscán Almogáver (¿-1542)


Escribió numerosos poemas imitando las formas italianas.

Fernando de Herrera (1534-1597)


Principal representante de la escuela lírica sevillana, cultivó los temas amorosos y patrióti-
cos, destacando entre sus obras Por la victoria de Lepanto y A don Juan de Austria.

Fray Luis de León (1527-1591)


Aun cuando él mismo no concedía excesiva importancia a su obra poética, debemos consi-
derarlo, aunque su producción sea de sólo 23 poemas, entre los más importantes poetas de
la historia de la literatura española. Además, destacó como prosista en obras de contenido
didáctico religioso como La perfecta casada, Los nombres de Cristo y Exposición del
Libro de Job.

Alonso de Ercilla y Zúñiga (1533-1594)


Es el principal representante de la poesía épica, autor de La Araucana, en la que describe
la conquista de Chile.

15

-Pág.19-
L E N G U A Y L I T E R AT U R A

LA NARRATIVA
La novela inicia su despegue como manifestación principal del género narrativo. Junto a la
novela o libros de caballerías, cuyo pilar indiscutible es el Amadís de Gaula (1508), se des-
arrolla la novela pastoril (JORGE DE MONTEMAYOR, GASPAR GIL POLO) o la novela morisca
(Historia del abencerraje y de la hermosa Jarifa) y aparece, como subgénero típicamente espa-
ñol, la novela picaresca. La obra anónima (tal vez del jerónimo FRAY JUAN DE ORTEGA) Vida
del Lazarillo de Tormes y de sus fortunas y adversidades marca el inicio de este tipo de narra-
ciones que reflejan y satirizan la sociedad de la España del XVI. Finalizando el siglo se publi-
cará la segunda novela picaresca en importancia, el Guzmán de Alfarache (1599) de MATEO
ALEMÁN.

ASCÉTICA Y MÍSTICA
Fruto de la omnipresencia de la religión en la vida española, debemos diferenciar en la pro-
ducción literaria lo que podríamos catalogar de género específico, el que tiene en la manifesta-
ción del “ejercicio” para alcanzar a Dios y del saber revelado por la oración y la meditación, su
razón de ser. Obras en prosa y en verso cargadas de unción religiosa, de encendida profesión de
fe, marcan este género característico de la literatura española, que cuenta como principales
representantes con FRAY LUIS DE GRANADA (1504-1588) (Introducción al símbolo de la fe),
SANTA TERESA DE JESÚS (1515-1582) (Camino de perfección, El Castillo interior o Las Mora-
das), SAN JUAN DE LA CRUZ (1542-1591) (Subida al Monte Carmelo, Noche oscura del alma,
Llama de amor viva, Cántico espiritual) y SAN IGNACIO DE LOYOLA (1491-1556) (Ejercicios
espirituales).

TEATRO
Aun cuando los momentos más brillantes del teatro español del Siglo de Oro estaban por lle-
gar en la centuria siguiente, en el siglo XVI aparecen los primeros grandes autores, entre los que
podemos citar a GIL VICENTE, TORRES NAHARRO, JUAN DE LA CUEVA (Los siete infantes de
Lara, Bernardo del Carpio) y LOPE DE RUEDA.

2.2.3. BARROCO

El Siglo de Oro de las artes y las letras hispanas, iniciado en el siglo anterior, continúa
durante buena parte del siglo XVII en el que se inscribe la labor creativa de alguno de los más
grandes genios de nuestra literatura. Sírvanos la siguiente relación sistematizada de autores y
obras para dar una visión lo más completa posible de esta etapa.

16
-Pág.20-
U.D. 5.- LA L I T E R AT U R A

POESÍA
El siglo XVII está presidido por el enfrentamiento entre dos concepciones diametralmente
opuestas de lo que debe ser la estructura y el lenguaje poéticos. Hablamos del culteranismo y del
conceptismo. El primero, representado por GÓNGORA, prefiere las descripciones ampulosas, car-
gadas de referencias mitológicas, metáforas y otros recursos lingüísticos. Por su parte, el con-
ceptismo, cuyo más encarnizado defensor y enemigo de GÓNGORA, fue QUEVEDO, opta por la
expresión sobria y austera del concepto huyendo de las complicaciones expresivas, si bien, oca-
sionalmente, cae en la privación de los entramados básicos que hagan inteligible el contenido.

NARRATIVA
El XVII es el siglo del triunfo definitivo de la narrativa que contará con el monumento más
importante de la Historia de la Literatura española, El Quijote, y con un subgénero específica-
mente hispano cual es la novela picaresca.

TEATRO
El género dramático vivirá la aparición de lo que se ha venido en llamar “el teatro nacio-
nal”, un teatro popular y con fondo moralizador que cierra la divergencia experimentada entre
el teatro culto y el de, digamos, consumo. Algunos de sus autores siguen ocupando la cumbre
del género.

AUTORES Y OBRAS
Luis de Góngora y Argote (1561-1627)
Principal representante del culteranismo, fue capellán de FELIPE III. A pesar de su ordena-
ción sacerdotal, no hizo del religioso tema de su poesía, inspirándose en la belleza para impreg-
nar de la misma su obra poética. Sus principales obras son:
— Fábula de Polifemo y Galatea (1613).
— Soledades (1613).
— Panegírico al duque de Lerma (1617).
— Otros poemas: Amarrado al duro banco, Servía en Orán al rey, No son todo ruiseñores,
Ande yo caliente.

Francisco de Quevedo y Villegas (1580-1645)


Su carácter y su obra estuvieron muy influidos por los avatares de su vida, que osciló entre
el favor público de su actividad política al amparo del DUQUE DE OSUNA y la desgracia y la cár-
cel. Acérrimo enemigo literario e, incluso, personal de GÓNGORA, su carácter agrio inspiró pági-
nas brillantes cargadas de sarcasmo y humorismo.

17
-Pág.21-
L E N G U A Y L I T E R AT U R A

Prosa
— La vida del Buscón llamado Pablos (1603).
— Los sueños (1605-1622).
— Política de Dios, gobierno de Cristo, tiranía de Satanás (1626).
— Marco Bruto (1646).
Poesía.—Su obra fue publicada tras su muerte en dos volúmenes:
— Parnaso español (1648).
— Las tres musas (1670).

Miguel de Cervantes Saavedra (1547-1616)


Nacido en Alcalá de Henares (Madrid), discurrió su
vida primero en la milicia, al servicio del CARDENAL
ACQUAVIVA, participando en la Batalla de Lepanto
(1571) en la que sufrió las heridas que, inutilizándole el
brazo izquierdo, le dieron su sobrenombre de “EL
MANCO DE LEPANTO”. Sufrió prisión en Árgel entre 1575
y 1580 y tras ser liberado volvió a España donde su vida
se debatió entre la cárcel y las penurias económicas. Su
producción literaria, llena de ingenio, con estilo sencillo Miguel de Cervantes Saavedra
a la par que brillante, se volcó inicialmente en la poesía
y el teatro, géneros en los que no alcanzó el éxito que su estilo merecía. Lo contrario le sucedió
con su obra narrativa que le ha dado su fama y le ha encumbrado a la cabeza de la producción
literaria hispana de todos los tiempos. Sus principales obras son:
Poesía
— Viaje del Parnaso (1614).

Teatro
— El Trato de Argel (1580-1587).
— El gallardo español (1615).
— Los baños de Argel (1615).

Novela
— La Galatea (desde 1585).
— Las Novelas Ejemplares (1613), recopilación de novelas cortas escritas entre 1590 y
1612: Rinconete y Cortadillo, La gitanilla, El amante liberal, La española inglesa, El
Licenciado Vidriera, La fuerza de la sangre, El celoso extremeño, La ilustre fregona, El
coloquio de los perros...
— El Ingenioso Hidalgo Don Quijote de la Mancha (1605/1615), obra cumbre de la lite-
ratura española, fue terminada por CERVANTES, en su primera parte, en el verano de 1604

18
-Pág.22-
U.D. 5.- LA L I T E R AT U R A

y publicada en 1605. El éxito obtenido alimentó la aparición de una supuesta segunda


parte, publicada apócrifamente por ALONSO FERNÁNDEZ DE AVELLANEDA. Este hecho
movió a CERVANTES a escribir la auténtica segunda parte que fue publicada en 1615. La
crítica sarcástica a las novelas de caballería que, según el propio autor, motivó su redac-
ción, superó las expectativas iniciales desbordando los límites de su propia pretensión
hasta convertir a la obra en el paradigma de la dualidad de la actitud humana ante la
vida, representada, respectivamente, por sus dos protagonistas. DON QUIJOTE, encarna la
imagen del idealista y soñador; SANCHO PANZA, la del pragmático y realista. La novela
de novelas que es El Quijote, no es sólo la más importante obra de la literatura españo-
la sino una de las más brillantes de la literatura universal.
— Los Trabajos de Persiles y Segismunda (1617), publicada después de su muerte.

Lope Félix de Vega Carpio (1562-1635)


Prolífico autor en verso y prosa, conocido fundamentalmente
por su producción dramática, si bien brilló igualmente en la lírica
y la narrativa. Su obra fue siendo producida a la par que se des-
arrollaba una vida llena de siempre difíciles amoríos que fueron
musas de su pluma. Al final de su vida sus obras teatrales llagaban
a 1.500 y las del resto de géneros se medían por centenares. Entre
tan abrumadora producción cabe destacar:
Lope de Vega
Poesía
— Arte nuevo de hacer comedias (1609).
— Dragontea (1602).
— Rimas (1602).
— Rimas sacras (1614).

Narrativa
— El peregrino en su patria (1604).
— La Arcadia (1598).
— La Dorotea (1632).

Teatro.—(Es difícil precisar la fecha de sus obras. Casi todas después de 1600)
— Fuenteovejuna.
— El caballero de Olmedo.
— Peribáñez y El Comendador de Ocaña.
— La dama boba.
— El perro del Hortelano.
— El mejor alcalde, el rey.

19
-Pág.23-
L E N G U A Y L I T E R AT U R A

Pedro Calderón de la Barca (1600-1681)


La juventud del autor madrileño estuvo marcada por la milicia mientras que su madurez y
vejez lo estuvo por su ordenación sacerdotal. Gozó de fama y del favor de la Corte que ampa-
ró su producción teatral y le convirtió en capellán de Palacio. Después de LOPE DE VEGA es el
dramaturgo español más importante. Entre sus obras podemos citar:
— La dama duende (1629).
— Casa con dos puertas mala es de guardar (1632).
— El médico de su honra (1635).
— El gran teatro del mundo (1636).
— El mágico prodigioso (1637).
— El Alcalde de Zalamea (1640).
— La Vida es sueño (1640).

Fray Gabriel Téllez, Tirso de Molina (1579-1648)


Dramaturgo que deberíamos situar tras Lope y Calderón. Entre sus obras destacan:
— Don Gil de las calzas verdes (1615).
— El burlador de Sevilla y convidado de piedra (1630).
— El condenado por desconfiado (1635).

Juan Ruiz de Alarcón y Mendoza (1581-1639)


Autor teatral que completa el cuarteto de los grandes dramaturgos del Siglo de Oro.
— La verdad sospechosa.
— Las paredes oyen.

Mateo Alemán (1547-1613)


Pasa por ser, con su Guzmán de Alfarache (1599-1604), el iniciador de la novela picaresca.

Baltasar Gracián (1601-1658)


Forma parte, junto con Quevedo, del grupo de autores conceptistas. Su producción literaria
se centra en el género de la didáctica, siendo sus principales obras:
— Agudeza y arte del ingenio (1642).
— El Criticón (1651,1653,1655).

Guillén de Castro (1569-1631).


Dramaturgo cuya principal obra es:
— Las mocedades del Cid (1618).

20
-Pág.24-
U.D. 5.- LA L I T E R AT U R A

2.2.4. ÉPOCA NEOCLÁSICA (SIGLO XVIII)


Tras una transición con una lamentable pérdida de calidad en el estilo, reducción de la pro-
ducción y vacuidad en los contenidos, que se inicia al final de las grandes creaciones barrocas
(hacia 1680), se inicia una paulatina recuperación con posterioridad a la Guerra de Sucesión (en
torno a 1725). Las influencias extranjeras, principalmente la francesa, sirvieron de base a esta
reacción, distinguiéndose a lo largo del siglo tres etapas literarias diferenciadas:
— Etapa de oposición al Barroco.—Marcada por el ensayo y la sátira como expresión de
la superioridad de la crítica didáctica sobre la creación narrativa y dramática.
— Neoclásico.—Predominio de la estética característica del clasicismo francés a la que se
añaden influencias italianas. La razón predomina sobre la expresión de los sentimientos.
— Prerromanticismo.—Marcado por una reacción sentimental por influencia inglesa.

La literatura no alcanzó, en cualquier caso, un nivel importante; pero la elaboración lin-


güística, la reflexión sobre la propia Lengua, al amparo de los afanes ilustrados, permitió dar al
español un perfil moderno. En tal empresa jugó un papel fundamental la Real Academia Espa-
ñola, fundada en 1713 bajo la dirección de JUAN MANUEL FERNÁNDEZ PACHECO, y que produ-
jo, como frutos inmediatos, el Diccionario de Autoridades, la Ortografía y la Gramática.
Los principales autores y obras de la literatura dieciochesca, divididos por etapas, son los
siguientes:

A) ETAPA DE LUCHA CONTRA EL BARROCO

Fray Benito Feijoo y Montenegro (1676-1764)


Preocupado más por la finalidad didáctica que por la belleza de la expresión, el PADRE FEI-
JOO cultivó fundamentalmente el ensayo. Sus principales obras son:

— Teatro crítico universal (1726-1740).


— Cartas eruditas y curiosas (1742-1760).

Fray Francisco de Isla (1703-1781)


Su afán didáctico queda reflejado en su novela:
— Historia del famoso predicador fray Gerundio de Campazas, alias Zotes.

Diego de Torres Villarroel (1693-1770)


Personaje curioso y aventurero que alternó actividades de lo más variopinto a lo largo de su
vida. Son famosos sus cuadros costumbristas de Madrid y su autobiografía.

21
-Pág.25-
L E N G U A Y L I T E R AT U R A

B) NEOCLASICISMO

José de Cadalso (1741-1782)


Poeta y narrador, sus principales obras son:
— Noches lúgubres.
— Cartas marruecas.

Juan Meléndez Valdés (1754-1817)


Quizás el más importante poeta del siglo fue reconocido en su tiempo como el renovador
de la lírica hispana. Cultivó las odas, sonetos y romances.

Gaspar Melchor de Jovellanos (1744-1810)


Además de su labor política y de sus estudios de carácter económico (Informe sobre la ley
agraria), realizó estudios de carácter histórico (Memoria del Castillo de Bellver) y obras de
creación poética y dramática.

Leandro Fernández de Moratín (1760-1828)


Es el escritor más importante del siglo, superando con creces la obra de su padre NICOLÁS
FERNÁNDEZ DE MORATÍN. Poeta, crítico y dramaturgo, escribió, entre otras obras:
— Sátira contra los vicios introducidos en la poesía castellana.
— La derrota de los pedantes (1789).
— La comedia nueva o el café (1792).
— El sí de las niñas (1806).

Tomás de Iriarte (1750-1791)


Autor de unas 60 fábulas recogidas en sus Fábulas Literarias (entre las que figura, por
ejemplo, El burro flautista).

Félix María Samaniego (1745-1801)


Autor de las Fábulas morales (El pacto de los montes, La lechera).

Ramón de la Cruz (1731-1794)


Cultivador del sainete (pieza teatral breve de carácter costumbrista y diálogos ingeniosos).
Entre ellos destacan:
— El Prado por la noche (1763).
— Manolo, tragedia para reír o sainete para llorar (1769).
— El rastro por la mañana (1770).

22
-Pág.26-
U.D. 5.- LA L I T E R AT U R A

C) PRERROMANTICISMO

Manuel José de Quintana (1772-1857)


Entre sus obras destacan:
— A Guzmán el Bueno.
— Al combate de Trafalgar.

Alberto Lista (1775-1848).


Fue maestro de ESPRONCEDA. Entre sus obras figuran:
— La muerte de Jesús.
— Himno del desgraciado.

2.2.5. SIGLO XIX


Durante el siglo XIX la literatura española pasa por tres fases:
— Continuación de las corrientes del XVIII y prerromanticismo (1800-1830).
— Romanticismo (1830-1850).
— Realismo (1850-1898).
En todo caso, al igual que en la pintura, los estilos o movimientos literarios que definen el
siglo son, específicamente, el romanticismo y el realismo.

A) ROMANTICISMO

El movimiento literario romántico se inicia en España por iniciativa de los exiliados libera-
les que regresan, cargados de influencias europeas, tras la muerte de FERNANDO VII.

TEATRO
Ángel de Saavedra, duque de Rivas (1794-1865)
Se trata del autor angular del romanticismo español cuyo triun-
fo definitivo se debe al estreno en 1835 de su drama Don Álva-
ro o la fuerza del sino, que escribió en Francia durante su
exilio. El éxito logrado con su obra le llevó al primer plano de
la vida pública, ocupando diversos cargos políticos y diplomá-
ticos y siendo elegido Presidente de la Real Academia Espa-
ñola en 1862.

Duque de Rivas

23
-Pág.27-
L E N G U A Y L I T E R AT U R A

Antonio García Gutiérrez (1813-1884)


Autor del segundo gran éxito, cronológicamente hablando, del teatro romántico español, su
drama El trovador, estrenado en 1836.

Juan Eugenio Hartzenbuch (1806-1880)


El dramaturgo madrileño fue miembro de la Real Academia Española y director de la Biblio-
teca Nacional. Entre sus obras figuran Los amantes de Teruel y La jura de Santa Gadea.

José Zorrilla (1817-1893)


Autor muy popular, principalmente como poeta, su obra más conocida, y de enorme tras-
cendencia, es el drama Don Juan Tenorio (1844). Entre sus obras figuran, además, Traidor,
inconfeso y mártir, El zapatero y el rey y El puñal del godo.

POESÍA
José de Espronceda (1802-1842)
Sus obras más significativas son El estudiante de Salamanca, El diablo mundo, Canto a
Teresa y Canción del Pirata.

Gustavo Adolfo Bécquer (1836-1870)


Paradigma del individuo romántico, se trata de un hom-
bre tímido, retraído y soñador, cuya corta vida fructificó
en una producción literaria breve pero cualitativamente
enorme. Sus “Rimas”, un conjunto de setenta y seis poe-
mas breves, publicadas bajo el título de El libro de los
gorriones un año después de su muerte, se caracterizan
por su musicalidad y su intenso sentimiento. Gustavo Adolfo Bécquer
Además de su obra poética, tienen gran trascendencia en la historia de la literatura españo-
la sus leyendas en prosa. Publicadas inicialmente entre 1861 y 1863 en diversos periódi-
cos, se caracterizan por el arte con el que BÉCQUER sabe crear ambientes fantásticos de
poesía y ensueño. Del conjunto, podemos destacar: El rayo de Luna, El monte de las áni-
mas, Maese Pérez el Organista, Los ojos verdes y El Miserere.

Otros poetas
Además de los citados, cabe mencionar a JUAN AROLAS (A una bella), NICOMEDES PASTOR
DÍAZ (Al acueducto de Segovia), GERTRUDIS GÓMEZ DE AVELLANEDA (A él), CAROLINA CORO-
NADO (El amor de los amores) y ROSALÍA DE CASTRO (En las orillas del Sar).

24
-Pág.28-
U.D. 5.- LA L I T E R AT U R A

PROSA

Mariano José de Larra (1809-1837)


A pesar de su corta vida, se convirtió en el prosista más importante del siglo XIX, habiendo
dejado una trascendental huella en la literatura española. Especialmente agudo y sarcástico,
fue un crítico implacable de su época en artículos publicados bajo diversos seudónimos
(FÍGARO, ANDRÉS NIPORESAS).

Otros prosistas
Cabe citar, además, a los novelistas ENRIQUE GIL Y CARRASCO, FRANCISCO NAVARRO VILLOS-
LADA y al propio ANTONIO CÁNOVAS DEL CASTILLO, junto a los ENSAYISTAS JAIME BALMES y
JUAN DONOSO CORTÉS.

B) REALISMO

Conviviendo durante algunos años con el romanticismo, el costumbrismo servirá de antece-


dente a la nueva corriente literaria que, en los últimos años del siglo se, digamos, radicalizará
en el llamado naturalismo.

ESCRITORES COSTUMBRISTAS
Entre ellos cabe citar a RAMÓN DE MESONERO ROMANOS (1803-1882), autor de El curioso
parlante y Escenas matritenses, y a ESTÉBANEZ CALDERÓN (1799-1867).

NOVELA

Pedro Antonio de Alarcón (1833-1891)


Costumbrista y romántico, dedicó parte de su producción a los libros de viajes. En su madu-
rez derivó al realismo con obras como El escándalo o El niño de la bola y, fundamental-
mente, El sombrero de tres picos.

José María de Pereda (1833-1906)


El autor cántabro logró su mejor inspiración en su tierra, tanto en la costa, asunto esencial
de su novela Sotileza; como en el montañoso interior, ambiente de Peñas Arriba.

25

-Pág.29-
L E N G U A Y L I T E R AT U R A

Benito Pérez Galdós (1843-1920)


El autor canario es, sin duda, el narrador más prolífi-
co del XIX. Escribió más de cincuenta novelas de la
importancia y calidad de La fontana de oro, su opera
prima, Marianela, Fortunata y Jacinta y Misericor-
dia. Además de éstas, entre 1873 y 1912 escribe los 46
relatos que constituyen los Episodios Nacionales,
agrupados en cinco series que repasan de forma nove-
lada los acontecimientos de la Historia de España
Benito Pérez Galdós
desde 1807 (Trafalgar) a la Restauración (Cánovas).

Otros novelistas
Cabe citar también a CECILIA BÖHL DE FABER (FERNÁN CABALLERO), autora de cuadros cos-
tumbristas y de novelas realistas como La Gaviota y Lágrimas; JUAN VALERA (Pepita Jimé-
nez), EMILIA PARDO BAZÁN (Los Pazos de Ulloa), LEOPOLDO ALAS “CLARÍN” (La
Regenta), ARMANDO PALACIO VALDÉS (La hermana San Sulpicio), VICENTE BLASCO IBÁÑEZ
(Cañas y barro).

ENSAYO
El más importante ensayista de esta etapa y una de las figuras más importantes de la cultu-
ra española del XIX es MARCELINO MENÉNDEZ PELAYO (1816-1912).

POESÍA
La poesía realista carece de la sinceridad y lirismo de la romántica. Entre sus cultivadores
se encuentran RAMÓN DE CAMPOAMOR, GASPAR NÚÑEZ DE ARCE, VENTURA RUIZ AGUILERA,
VICENTE WENCESLAO QUEROL y EMILIO FERRARI.

TEATRO
Los principales dramaturgos realistas son VENTURA DE LA VEGA, ADELARDO LÓPEZ DE AYALA
y MANUEL TAMAYO Y BAUS.

Literaturas peninsulares
No podemos olvidar el resurgimiento, con una renovada fuerza impulsada por el romanticis-
mo, de las literaturas en catalán y gallego. Efectivamente, en Cataluña, donde su lengua apenas

26
-Pág.30-
U.D. 5.- LA L I T E R AT U R A

se había impreso en lenguaje literario desde el Renacimiento, ésta cobra lozanía. La poesía cuen-
ta con uno de los más importantes autores en catalán de todos los tiempos, JACINTO VERDAGUER
(La Atlántida), y con JOAN MARAGALL (Nausica). La narrativa está representada por NARCISO
OLLER y el teatro por ÁNGEL GUIMERÁ (Tierra Baja) y SANTIAGO RUSIÑOL (El Místico).
Por su parte, la lengua gallega también renace como soporte de expresión literaria, contan-
do con autores como ROSALÍA DE CASTRO, MANUEL CURROS ENRÍQUEZ (A virxe de cristal),
EDUARDO PONDAL (Queixumes dos piños), VALENTÍN LAMAS CARVAJAL o FRANCISCO AÑÓN.

2.2.6. SIGLO XX

A) MODERNISMO

Entre los autores modernistas destacan el poeta MANUEL MACHADO y los dramaturgos
EDUARDO MARQUINA (La ermita, la fuente y el río), JACINTO GRAU (El conde Alarcos) y
FRANCISCO VILLAESPESA.

Juan Ramón Jiménez (1881-1958)


Autor inicialmente modernista, preocupado por los aspectos estéticos de la lengua y reco-
nocido como maestro por la posterior Generación del 27, se alejó de las exuberancias
modernistas en pos de un mayor intimismo y una sencillez formal elegante y sutil. Obtuvo
el Premio Nobel de Literatura en 1956. Entre toda su producción, destacamos:
La soledad sonora (1911).
Platero y yo (1917).
Diario de un poeta recién casado (1917).
Animal de fondo (1949).

B) GENERACIÓN 98

José Martínez Ruiz, “Azorín” (1873-1967)


Su estilo es sencillo y natural, con un lenguaje rico y riguroso y temas populares. Fue él
quien dio nombre a la Generación del 98.
Antonio Azorín (1903), novela autobiográfica.
Castilla (1912), ensayo.
Al margen de los clásicos (1915), crítica literaria.

27
-Pág.31-
L E N G U A Y L I T E R AT U R A

Miguel de Unamuno (1864-1936)


Prolífico autor que cultivó todos los géneros literarios y la Filosofía. Fue rector de la Uni-
versidad de Salamanca.
Vida de Don Quijote y Sancho (1905), El sentimiento trágico de la vida (1913),
La agonía del cristianismo (1925), obras filosóficas.
Niebla (1914), La tía Tula (1933), San Manuel Bueno, mártir (1933), novelas.
El Cristo de Velázquez (1920), poesía.
Raquel encadenada (1921), drama.

Pío Baroja (1872-1957)


Es el novelista más importante de la generación.
Camino de perfección (1902).
Zalacaín el aventurero (1909).
El árbol de la ciencia (1911).
La busca (1904), La mala hierba (1904) y Aurora roja (1905), que forman la trilogía
La lucha por la vida.

Ramón María del Valle-Inclán (1866-1936)


Periodista bohemio, ingenioso y mordaz, novelista, poeta y dramaturgo.
Los cruzados de la causa (1908), El resplandor de la hoguera (1909), Gerifaltes de
antaño (1909), novelas que forman la trilogía La guerra carlista.
Cuento de abril (1910), La marquesa Rosalinda (1913), dramas.
Divinas palabras (1920), tragedia.
Luces de bohemia (1920)
Tirano Banderas (1926), novela que pasa por ser su obra maestra.
El ruedo ibérico, serie de novelas de contenido histórico escrita desde 1927.

Antonio Machado (1875-1939)


Es posiblemente el poeta más importante del siglo XX y uno de los más importantes de la
historia de la literatura española. Sus primeros poemas, recogidos en Soledades (1903) son
modernistas; pero pronto define un estilo personal, ya mostrado en su colección de poemas
Soledades, galerías y otros poemas (1917). Castilla se convierte en el tema de inspiración
machadiano a partir de su estancia en Soria y se plasma en los poemas recogidos en Cam-
pos de Castilla (1912). Su preocupación social y sus reflexiones filosóficas se manifiestan
en la colección Nuevas Canciones (1917). El conjunto de su producción se publicó, además,
en sucesivas ediciones de Obras completas (1917, 1928, 1933).

28
-Pág.32-
U.D. 5.- LA L I T E R AT U R A

Jacinto Benavente (1866-1954)


Dramaturgo madrileño que obtuvo el Premio Nobel de Literatura en 1922.
Los intereses creados (1907).
La malquerida (1913).
El nido ajeno, Rosas de otoño, de su primera época.
La infanzona, Lo increíble, Y amargaba, Su amante esposa, producciones posteriores a
1939, comedias de ingenio agudo y burlón.

C) NOVECENTISMO

José Ortega y Gasset (1883-1955)


Filósofo profundo y trascendental cuyo pensamiento se popularizó entorno a su expresión
“yo soy yo y mi circunstancia”. Fue catedrático de metafísica de la Universidad de Madrid
y fundador de la Revista de Occidente en 1923 y del diario El Sol.
Meditaciones del Quijote (1914).
España invertebrada (1921).
El tema de nuestro tiempo (1923).
La rebelión de las masas (1930).
Ideas y creencias (1940).
Meditaciones sobre Europa (1957)
El espectador (1916-1935), serie de ocho tomos que recogen sus artículos en periódicos y
revistas.

Otros autores
EUGENIO D’ORS (La bien plantada), RAMÓN PÉREZ DE AYALA (Troteras y danzaderas),
GABRIEL MIRÓ, RAMÓN GÓMEZ DE LA SERNA, CONCHA ESPINA, WENCESLAO FERNÁNDEZ FLÓ-
REZ (El bosque animado), JULIO CAMBA, CARLOS ARNICHES (La señorita de Trevélez),
SERAFÍN y JOAQUÍN ÁLVAREZ QUINTERO, PEDRO MUÑOZ SECA…

D) GENERACIÓN DEL 27

Federico García Lorca (1898-1936)


Poeta y dramaturgo de trascendencia mundial, cuya fama se acrecentó por su ejecución
durante la Guerra Civil.
Romancero gitano (1928), colección de poemas influidos por la obra de Juan Ramón Jiménez.

29
-Pág.33-
L E N G U A Y L I T E R AT U R A

Llanto por la muerte de Ignacio Sánchez Mejías (1935), quizás su obra poética más impor-
tante.
Poeta en Nueva York (1929-1930), poemas surrealistas.
Bodas de sangre (1933), Yerma (1934), La casa de Bernarda Alba (1936), tragedias de
ambiente rural que constituyen sus tres mejores obras dramáticas.
Doña Rosita la soltera o el lenguaje de las flores (1935), comedia.
Mariana Pineda (1927), drama histórico.

Rafael Alberti (1902-1999)


Su producción literaria se ha centrado fundamentalmente en la poesía si bien ha cultivado
también el teatro.
Marinero en tierra (1924).
El alba del alhelí (1926).
Cal y canto (1929).
Entre el clavel y la espada (1941).
Retornos de los vivo lejano (1952).
Canciones de Altair (1988).
El adefesio (1944), obra dramática.

Vicente Aleixandre (1898-1984)


Su producción poética ha tenido una gran influencia en autores posteriores. Obtuvo el Pre-
mio Nobel de Literatura en 1977.
Espadas como labios (1932)
Sombra de paraíso (1944).
Poemas de la consumacióní (1958).
Diálogos del conocimiento (1974).
En gran noche, publicada en 1991, tras su muerte.

Otros autores
Destacan también JORGE GUILLÉN (Cántico), PEDRO SALINAS (La voz a ti debida), GERARDO
DIEGO (Versos humanos), MIGUEL HERNÁNDEZ (El rayo que no cesa), LUIS CERNUDA, DÁMA-
SO ALONSO…

E) POETAS DE LA GENERACIÓN DEL 36

Destacan LUIS ROSALES, CARMEN CONDE, LEOPOLDO PANERO y GERMÁN BLEIBERG.

30
-Pág.34-
U.D. 5.- LA L I T E R AT U R A

F) POETAS POSTERIORES

GABRIEL CELAYA, BLAS DE OTERO, JOSÉ HIERRO, CARLOS BOUSOÑO, LEOPOLDO MARÍA
PANERO, JOSÉ MARÍA CABALLERO BONALD, JOSÉ GIL DE BIEDMA, FÉLIX DE AZÚA, PERE GIM-
FERRER, GLORIA FUERTES...

G) NOVELA POSTERIOR AL 36
Camilo José Cela (1916)
El más importante narrador contemporáneo, galardonado con
el Premio Nobel de Literatura en 1989.
Viaje a la Alcarria (1948).
La familia de Pascual Duarte (1942).
La colmena (1951).
San Camilo, 1936 (1969).
Camilo José Cela Cristo versus Arizona (1988).

Miguel Delibes (1920)


Uno de los más prolíficos autores contemporáneos.
La sombra del ciprés es alargada (1947).
Las ratas (1962).
Cinco horas con Mario (1967).

Otros
CARMEN LAFORET (Nada, 1944), RAFAEL SÁNCHEZ FERLOSIO (El Jarama), JOSÉ MARÍA
GIRONELLA (Los cipreses creen en Dios), RAMÓN J. SENDER (Réquiem por un campesino
español, 1962), JUAN GOYTISOLO (Paisajes después de la batalla, 1982), ANA MARÍA
MATUTE, RAFAEL SÁNCHEZ MAZAS, ELENA QUIROGA, IGNACIO ALDECOA, ALFONSO GROSSO,
JUAN MARSÉ, ARTURO PÉREZ-REVERTE, CARMEN MARTÍN GAITE…

H) TEATRO POSTERIOR AL 36

Antonio Buero Vallejo (1916)


El estreno de Historia de una escalera en 1949, supuso una auténtica revelación que dio nue-
vos impulsos al teatro.
Las cartas boca abajo (1957).
El concierto de San Ovidio (1962).
El tragaluz (1967).
El sueño de la razón (1970).
Diálogo secreto (1984).

31
-Pág.35-
L E N G U A Y L I T E R AT U R A

Otros
ENRIQUE JARDIEL PONCELA (Los ladrones somos gente honrada), VÍCTOR RUIZ IRIARTE
(El landó de seis caballos), MIGUEL MIHURA (Tres sombreros de copa), ALEJANDRO CASO-
NA (La dama del alba), JOSÉ MARÍA PEMÁN (El divino impaciente), JUAN IGNACIO LUCA DE
TENA, JOAQUÍN CALVO SOTELO, ALFONSO PASO, ALFONSO SASTRE, CARLOS MUÑIZ, CARLOS
LLOPIS…

I) ENSAYISTAS CONTEMPORÁNEOS
JAVIER ZUBIRI, PEDRO LAÍN ENTRALGO, JOSÉ LUIS ARANGUREN, JULIÁN MARÍAS, GUILLERMO
DÍAZ-PLAJA, FRANCISCO AYALA, ANTONIO VALLEJO NÁJERA…

3. LOS GÉNEROS LITERARIOS


Los géneros literarios son los grupos en que clasificamos las obras literarias considerando
sus elementos predominantes y características.

3.1. LA ÉPICA (argumento, narrador, espacio, tiempo, estructura, tema...).


OBRAS DESTACADAS DEL GÉNERO ÉPICO. EVOLUCIÓN
DEL GÉNERO ÉPICO

La épica (del griego epos, significa relato, narración), es un género narrativo en el que el
autor narra los hechos de un personaje, de un pueblo entero (en este caso se denomina epope-
ya), etc.
En todo relato épico, ya sea en verso o en prosa, debemos destacar:
El narrador: Es quien cuenta la historia.
Espacio: Es el marco o lugar donde ocurre los hechos.
Tiempo: Sitúa los hechos temporalmente.
Tema: El contenido esencial de la historia narrada.
Estructura: Generalmente suelen tener: Presentación, desarrollo y desenlace.

OBRAS DESTACADAS DEL GÉNERO ÉPICO.


Los cantares de gesta: El Cantar de Mío Cid.
Romances: El romance del Prisionero.
Cuentos y Leyendas, etc.

32
-Pág.36-
U.D. 5.- LA L I T E R AT U R A

EVOLUCIÓN DEL GÉNERO ÉPICO.


Tradicionalmente el género épico se entendía el relacionado con el relato de alguna hazaña
o gesta heroica de un personaje o de una comunidad, pero con el correr del tiempo ha pasado
a denominar cualquier narración referida a acciones de las personas normales, sin que tengan
que ser héroes necesariamente. Ej.: El relato de un partido de fútbol.

3.2. LA DRAMÁTICA (texto y espectáculo, estructura, personajes,


recursos, etc.). OBRAS DESTACADAS DEL GÉNERO DRAMÁTI-
CO
La dramática (del griego drama, significa acción) es el género literario en el que los perso-
najes ofrecen una acción dialogada y representable ante un público.
El texto dramático (su finalidad es ser representado como espectáculo ante un público) debe
comprender:
Un conflicto: Entre personajes.
Conjunto de acciones alrededor de una principal: Diversos hechos que complementan a
una acción central.
Una caracterización de los personajes: Que les haga parecerse lo más posible a los que
el autor pretende.

ESTRUCTURA.
Las obras se dividen en actos, y éstos a su vez están formados por escenas.
En toda obra dramática, independientemente de los actos que la compongan, siempre se
diferenciarán: La exposición, el nudo o trama y el desenlace o resolución de la obra.
Dependiendo del tipo de conflicto que presente el texto, las obras dramáticas las clasifica-
mos en: Tragedias, dramas y comedias.

OBRAS DESTACADAS DEL GÉNERO DRAMÁTICO.


Algunas obras importantes de nuestro teatro son:
De Lope de Vega: Fuenteovejuna.
De Calderón: La vida es sueño.
De Buero Vallejo: Historia de una escalera, etc.

EVOLUCIÓN DEL GÉNERO DRAMÁTICO.


El género dramático, nunca ha dejado de cultivarse según los cánones tradicionales, pero
han ido surgiendo espectáculos en los que se han incorporado otras actividades complementa-
rias a la mera representación teatral. Ej.: Zarzuelas, comedias musicales, etc.

33
-Pág.37-
L E N G U A Y L I T E R AT U R A

3.3. LA LÍRICA (verso y prosa, temas, recursos retóricos). OBRAS DESTACA-


DAS DEL GÉNERO LÍRICO. EVOLUCIÓN DEL GÉNERO LÍRICO
La lírica (del instrumento musical “lira”, con el que se solía acompañar el canto poético) es
el género literario en el que el autor expresa sus sentimientos: dolor, alegría, tristeza, melanco-
lía, etc. Es un género muy propio del verso, pero también se da en la prosa.
Según los temas y la forma que utilice el autor, podemos hacer una clasificación del género
lírico:
La oda: Es un canto de alabanza a la vida, a la belleza, al amor...
La canción: Composición lírica para ser cantada.
La elegía: Es un poema de lamento, de dolor por la pérdida de un ser querido, puede ser
individual o colectiva.
La sátira: Composición poética que ridiculiza los defectos o vicios ajenos.
En la lírica, el poeta hace uso de los recursos retóricos (recursos literarios) para conseguir
que su composición resulte lo más bella posible.
OBRAS DESTACADAS DEL GÉNERO LÍRICO.
Algunas obras representativas de este género son:
De Fray Luis de León: Oda a Salinas.
De Miguel Hernández: Elegía a Ramón Sijé.
De Pablo Neruda: Veinte poemas de amor y una canción desesperada, etc.
EVOLUCIÓN DEL GÉNERO LÍRICO.
En la actualidad conviven las formas y temas de la poesía tradicional con un tipo de poesía
“más libre”, especialmente en cuanto a la métrica, parece que hay una cierta predilección por el
uso del verso libre.

3.4. EL ENSAYO Y OTROS GÉNEROS LITERARIOS

El ensayo abarca todo, se caracteriza por la forma de tratar los temas: el ensayista trata de
conseguir una obra que pueda ser seguida perfectamente por cualquier persona, incluso por per-
sonas que no tengan grandes conocimientos sobre el tema tratado; ahí está su principal éxito.
El ensayo, ante todo, invita a reflexionar, a buscar nuevos puntos de vista, no es un tratado
estructurado técnicamente, ni tiene pretensiones científicas, únicamente persigue que el lector
piense por su cuenta sobre el tema planteado.
Otros géneros literarios: El “guión cinematográfico”, el “guión radiofónico”, los “artículos
de prensa”, etc. son textos que se escriben con características literarias.

34
-Pág.38-
U.D. 5.- LA L I T E R AT U R A

3.5. TEMAS Y MITOS LITERARIOS

Entendemos por mito, algo fabuloso y legendario; puede ser una persona o cosa a la que se
le empieza a atribuir determinadas cualidades que la distinguen notoriamente de las demás, con
lo que llegan a convertirse en un ser superior a sus semejantes.
Esto ocurre con algunos temas o personajes de la literatura cuando son tratados por diversos
autores de épocas diferentes, cada uno va añadiéndole algún adorno más, con lo que se convier-
ten en personajes de leyenda, que en muchas ocasiones llegan a desfigurar totalmente la realidad.
Otras veces, es el mismo público, el que empieza a valorar a un personaje y lo convierte en
centro de atención preferente en cualquier acontecimiento que pueda estar relacionado con él.
En nuestra literatura tenemos bastantes ejemplos de mitos. Ej.: El Cid es un personaje que
forma parte de nuestra mitología literaria.

RESUMEN

— La literatura es el arte que se expresa a través de la palabra.


— Las obras literarias son producciones lingüísticas.
— Cada literatura aparece en el seno de una sociedad determinada.
— Los períodos literarios son coincidentes, en general, con los períodos de las otras artes.
— Entendemos por generación a un grupo de escritores con características literarias simi-
lares.
— Los géneros literarios son: La épica, la lírica y la dramática.

35

-Pág.39-
L E N G U A Y L I T E R AT U R A

EJERCICIOS DE AUTOCOMPROBACIÓN

11. La primera comunicación literaria fue de forma:


A. Oral B. Escrita C. AyB D. Ninguna de las anteriores

12. La función principal del lenguaje literario es:


A. Conativa B. Estética C. Metalingüística D. Social

13. El periodo literario que comprende desde el siglo X hasta el XV, se denomina:
A. Edad Media B. Renacimiento C. Clasicismo D. Romanticismo

14. La Celestina corresponde al período:


A. Romanticismo B. Renacimiento C. Barroco D. Generación del 98

15. La corriente literaria denominada: “Culteranismo”, se produjo en:


A. La Edad Media B. El Barroco C. El Neoclasicismo D. Generación del 98

16. La Generación del 27, estaba formada por un grupo de:


A. Novelistas B. Dramaturgos C. Prosistas D. Poetas

17. La corriente literaria denominada “Conceptismo”, se produjo en:


A. La Edad Media B. El Renacimiento C. El Barroco D. Siglo XIX

18. Larra escribió sus Artículos en el siglo:


A. XV B. XVI C. XIX D. XX

19. La “oda” es una composición que pertenece al género:


A. Épico B. Dramático C. Lírico D. Didáctico

10. La elegía es una composición de tipo:


A. Festivo B. Alegre C. Triste D. Legal

RESPUESTAS A LOS EJERCICIOS


11. A 12. B 13. A 14. B 15. B

16. D 17. C 18. C 19. C 10. C

36
-Pág.40-
portada TROPA 19/3/07 19:51 Página 1

FUERZAS ARMADAS
PROFESIONALES
CURSO DE APOYO
A LA PREPARACIÓN
DE LAS PRUEBAS DE ACCESO
A UNA RELACIÓN DE SERVICIOS
DE CARÁCTER PERMANENTE

MATEMÁTICAS
1ª parte
Unidades didácticas 1 y 2

DIGEREM

MINISTERIO
DE DEFENSA
FUERZAS ARMADAS SUBDIRECCIîN GENERAL
DE TROPA Y MARINERIA
PROFESIONAL
PROFESIONALES
CURSO DE APOYO
Ó
A LA PREPARACION
DE LAS PRUEBAS DE ACCESO
A UNA RELACIÓN DE SERVICIOS
DE CARÁCTER PERMANENTE

MATEMÁTICAS
1ª parte
Unidades didácticas 1 y 2

-Pág.1-
La Ley 8/2006 de Tropa y Marinería, en su artículo 16,1, establece que “la formación
en las Fuerzas Armadas garantizará que los militares profesionales de tropa y
marinería puedan adquirir, actualizar o ampliar sus conocimientos para un mayor
desarrollo personal y profesional”. En cumplimiento de este mandato, el Ministerio
de Defensa edita el presente material didáctico para facilitar a los militares
profesionales de tropa y marinería, alumnos de los cursos de formación
presencial que se imparten a través de la Dirección General de Reclutamiento y
Enseñanza Militar, los apoyos necesarios para preparación de dichos cursos, que
permitirán, siempre que superen las pruebas correspondientes, la obtención de la
titulación de graduado en Educación Secundaria, acreditación para el acceso a
los ciclos formativos de la Formación Profesional de grado medio o de grado
superior, acceso a las Escalas de Suboficiales, Tropa Permanente, Guardia Civil
y Policía Nacional.

CATÁLOGO GENERAL DE PUBLICACIONES


http://www.060.es

Edita:

© Autor y editor
NIPO: 076-10-204-9 NIPO: 076-10-205-4 (edición en línea)
Depósito Legal: M-32363-2009
Diseño y programación: cimapress
Tirada: 1300 ejemplares
Fecha de edición: septiembre, 2010

Prohibida la reproducción total o parcial de esta obra, por cualquier medio sin autorización escrita del editor
MATEMÁTICAS
1ª parte

SUMARIO

Unidad didáctica Pág.

1. NÚMEROS Y OPERACIONES: SIGNIFICADOS 5


ESTRATÉGIAS Y SIMBOLIZACIÓN

2. MEDIDA, ESTIMACIÓN Y CÁLCULO DE MAGNITUDES 29

-Pág.3-
U. D. 1 . - N Ú M E R O S Y O P E R AC I O N E S

ÍNDICE
Pag.
OBJETIVOS . . . . . . . . . . . . . . . . . . . . . . . . . . . . . . . . . . . . . . . . . . . . . . . . . . . . . . . . . . . 2
INTRODUCCIÓN . . . . . . . . . . . . . . . . . . . . . . . . . . . . . . . . . . . . . . . . . . . . . . . . . . . . . . 3
MAPA CONCEPTUAL . . . . . . . . . . . . . . . . . . . . . . . . . . . . . . . . . . . . . . . . . . . . . . . . . . 4
DESARROLLO DE CONTENIDOS . . . . . . . . . . . . . . . . . . . . . . . . . . . . . . . . . . . . . . . 5
1. LOS NÚMEROS NATURALES . . . . . . . . . . . . . . . . . . . . . . . . . . . . . . . . . . . . . 5
1.1. CONCEPTO DE NÚMERO NATURAL . . . . . . . . . . . . . . . . . . . . . . . . . . . . . . . 5
1.2. OPERACIONES CON NÚMEROS NATURALES . . . . . . . . . . . . . . . . . . . . . . . . 5
2. LOS NÚMEROS ENTEROS . . . . . . . . . . . . . . . . . . . . . . . . . . . . . . . . . . . . . . . . 8
2.1. REPRESENTACIÓN Y ORDENACIÓN . . . . . . . . . . . . . . . . . . . . . . . . . . . . . . . 8
2.2. OPERACIONES CON NÚMEROS ENTEROS . . . . . . . . . . . . . . . . . . . . . . . . . . 8
3. LOS NÚMEROS DECIMALES . . . . . . . . . . . . . . . . . . . . . . . . . . . . . . . . . . . . 10
3.1. CONCEPTO DE NÚMERO DECIMAL . . . . . . . . . . . . . . . . . . . . . . . . . . . . . . . 10
3.2. OPERACIONES CON NÚMEROS DECIMALES . . . . . . . . . . . . . . . . . . . . . . . 11
4. LOS NÚMEROS FRACCIONARIOS . . . . . . . . . . . . . . . . . . . . . . . . . . . . . . . 12
4.1. CONCEPTO DE NÚMERO FRACCIONARIO . . . . . . . . . . . . . . . . . . . . . . . . . 12
4.2. FRACCIONES EQUIVALENTES . . . . . . . . . . . . . . . . . . . . . . . . . . . . . . . . . . . 12
4.3. REDUCCIÓN DE FRACCIONES A COMÚN DENOMINADOR . . . . . . . . . . . . 13
4.4. COMPARACIÓN DE FRACCIONES . . . . . . . . . . . . . . . . . . . . . . . . . . . . . . . . 13
4.5. OPERACIONES CON FRACCIONES . . . . . . . . . . . . . . . . . . . . . . . . . . . . . . . . 13
4.6. ORDEN Y REPRESENTACIÓN DE LOS NÚMEROS EN LA RECTA. . . . . . . . 14
4.7. APROXIMACIÓN Y ESTIMACIÓN DE CANTIDADES . . . . . . . . . . . . . . . . . . 15
5. MAGNITUDES PROPORCIONALES. . . . . . . . . . . . . . . . . . . . . . . . . . . . . . . 15
5.1. MAGNITUDES DIRECTAMENTE PROPORCIONALES . . . . . . . . . . . . . . . . . 15
5.2. MAGNITUDES INVERSAMENTE PROPORCIONALES . . . . . . . . . . . . . . . . . 16
5.3. REGLA DE TRES SIMPLE DIRECTA . . . . . . . . . . . . . . . . . . . . . . . . . . . . . . . 16
5.4. REGLA DE TRES INVERSA . . . . . . . . . . . . . . . . . . . . . . . . . . . . . . . . . . . . . . 16
5.5. PORCENTAJES . . . . . . . . . . . . . . . . . . . . . . . . . . . . . . . . . . . . . . . . . . . . . . . . 17
6. ALGORITMOS BÁSICOS E INSTRUMENTOS DE CÁLCULO . . . . . . . . 17
6.1. OPERACIONES ARITMÉTICAS . . . . . . . . . . . . . . . . . . . . . . . . . . . . . . . . . . . 17
6.2. CÁLCULO . . . . . . . . . . . . . . . . . . . . . . . . . . . . . . . . . . . . . . . . . . . . . . . . . . . . 19
6.3. REGLAS DE USO DE LA CALCULADORA . . . . . . . . . . . . . . . . . . . . . . . . . . 19
RESUMEN . . . . . . . . . . . . . . . . . . . . . . . . . . . . . . . . . . . . . . . . . . . . . . . . . . . . . . . . . . . 21
EJERCICIOS DE AUTOCOMPROBACIÓN . . . . . . . . . . . . . . . . . . . . . . . . . . . . . . . 22
RESPUESTAS A LOS EJERCICIOS . . . . . . . . . . . . . . . . . . . . . . . . . . . . . . . . . . . . . 23

-Pág.5-
M AT E M Á T I C A S

OBJETIVOS
Al finalizar esta Unidad Didáctica, el alumno será capaz de:

• Distinguir los diferentes tipos de números: naturales, enteros, decimales y


fraccionarios.

• Dominar las diferentes operaciones que se pueden realizar con los números:
adición, sustracción, multiplicación, división, etc.

• Utilizar los diferentes tipos de números para contar, medir, ordenar,


ordenar,codificar y expresar cantidades y sus aproximaciones.

• Aplicar la proporcionalidad junto con sus expresiones más usuales.

• Manejar correctamente una calculadora.

-Pág.6-
U. D. 1 . - N Ú M E R O S Y O P E R AC I O N E S

INTRODUCCIÓN
U n a de las cosas que más trabajo ha costado inventar a los hombres es el modo
y forma de escribir los números para que las operaciones aritméticas resulten
sencillas.

Desde la antigüedad se han empleado diferentes medios para contar, es decir, para
evaluar cantidades, a través de instrumentos, símbolos u otros procedimientos.

Hoy en día utilizamos los números para todos los campos de nuestra vida, así con
ellos, podemos contar, medir, ordenar, codificar, expresar cantidades, entre otras
muchas cosas.

Para poder realizar todo esto, nos encontramos con los diferentes tipos de números.
En primer lugar, tenemos los números naturales, que van desde el cero al infinito y
son todos positivos. Al intentar operar con ellos surgen los números enteros, que
pueden ser positivos y negativos. Y, como no siempre operamos con números
enteros, es decir, con la unidad, nos hace falta otro tipo de números denominados
fraccionarios, que es el resultado de dividir una unidad en un número de partes
iguales. Asimismo, y como expresión de esas fracciones, tenemos los números
decimales, que nos permiten escribir cómodamente aproximaciones de fracciones y,
de este modo, facilitan las operaciones aritméticas básicas (suma, resta,
multiplicación y división) al tratarlas como una extensión de las operaciones con
números enteros.

-Pág.7-
M AT E M Á T I C A S

M A PA C O N C E P T UA L

NÚMEROS NATURALES

NÚMEROS ENTEROS

NÚMEROS DECIMALES

NÚMEROS FRACCIONARIOS

-Pág.8-
U. D. 1 . - N Ú M E R O S Y O P E R AC I O N E S

1. LOS NÚMEROS NATURALES

1.1. CONCEPTO DE NÚMERO NATURAL


Los números naturales son el conjunto de todos los números que normalmente
utilizamos para contar objetos (del cero al infinito) y se representan por la letra N.

Este conjunto se caracteriza por ser ilimitado, empieza en el cero, le siguen el uno, dos,
tres, cuatro, etc., cada elemento tiene un siguiente y no existe un último elemento.

1.2. OPERACIONES CON NÚMEROS NATURALES

1.2.1. SUMA Y RESTA DE NÚMEROS NATURALES

Suma

La suma de dos números naturales a y b es un número natural c.

a+b=c ejemplo: 2 + 7 = 9

Los números a y b se denominan sumandos y c es la suma de ellos.

Propiedades de la suma

Asociativa.—La forma en que se asocian dos o más sumandos, no varia el resultado final.

(a + b) + c = a + (b + c) = d ejemplo: (4 + 5) + 10 = 19 ó, lo que es lo mismo


(a + b) + c = a + (b + c) = d ejemplo : 4 + (5 + 10) = 19 ó, lo que es lo mismo

Conmutativa.—El orden de los sumandos no altera el resultado.

a+b+c=d lo que es igual que c+a+b=d

ejemplo: 3 + 6 + 7 = 16 ó 7 + 3 + 6 = 16

TAMadrid
5
-Pág.9-
M AT E M Á T I C A S

Resta

Dados dos números naturales a y b tal que a * b (a es mayor o igual a b) se denomina


resta o diferencia entre ellos al número c tal que sumado a b da por resultado el número a.
Es decir:

a–b=c ‰ c+b=a ejemplo: 15 – 5 = 10 ‰ 10 + 5 = 15

La resta no tiene propiedades asociativa ni conmutativa.

1.2.2. PRODUCTO Y DIVISIÓN DE NÚMEROS NATURALES

P roducto

Dados dos números a llamado multiplicando y b llamado multiplicador, forman un


tercero c, llamado producto.

a×b=c ejemplo: 4 × 8 = 32

Los múltiplos de un número se obtienen multiplicando ese número por los números
naturales. Por ejemplo, los múltiplos de 4 serán: 4, 8, 12, 16, 20, 24, etc.

Propiedades del producto

Conmutativa.—Si a y b son dos números naturales cualesquiera, siempre se verifica que:

a×b=b×a ejemplo: 8 × 5 = 5 × 8 = 40

Asociativa.—Si a, b y c son tres números naturales cualesquiera, se verifica:

(a × b) × c = a × (b × c) ejemplo: (3 × 5) × 2 = 3 × (5 × 2) = 30

Existencia del Elemento Neutro.—Existe un único número, el uno, cuyo producto por un
número natural cualquiera, da como resultado el mismo número natural.

a×1=a ejemplo: 365 × 1 = 365

Distributiva.—Siendo a, b y c tres números naturales cualesquiera, se cumple:

a × (b + c) = a × b + a × c ejemplo: 4 × (8 + 3) = 4 × 8 + 4 × 3 = 44

6 TAMadrid

-Pág.10-
U. D. 1 . - N Ú M E R O S Y O P E R AC I O N E S

División

Un número es divisor de otro cuando la división del segundo por el primero es exacta.
a / b = c. Ej.: Si tengo 8 trozos en una tarta y los quiero repartir entre 4 personas, será:
8 : 4 = 2 trozos por persona. Ahora bien, si los mismos 8 trozos los quiero repartir entre
3 personas, no nos da un número natural exacto, lo que quiere decir que 8 no es divisible
por 3.
A continuación vamos a ver los criterios de divisibilidad más utilizados. Un criterio de
divisibilidad es una regla que permite reconocer, sin efectuar la división, si un número es o
no divisible por otro número dado.

Criterio de divisibilidad por Enunciado Ejemplo


1.002 Un número es divisible por 2 si la 2406 es divisible por 2 porque
cifra de sus unidades es cero o un 6 es par.
número par.
1.003 Un número es divisible por 3 si la 7824 es divisible por 3 porque
suma de sus cifras es divisible por 3. 7+8+2+4=21, que es divisible
por 3.
1.005 Un número es divisible por 5 si la 490 es divisible por 5 porque
cifra de sus unidades es 0 ó 5. termina en 0.
20985 es divisible por 5 porque
termina en 5.
1.009 Un número es divisible por 9 si la 57834 es divisible por 9 porque
suma de sus cifras es divisible por 9. 5+7+8+3+4=27, que es
divisible por 9.

1.010 Un número es divisible por 10 si la 2430 es divisible por 10 porque


cifra de sus unidades es cero. termina en 0.
1.011 Un número es divisible por 11 si lo es 907280 es divisible por 11
la diferencia entre la suma de las porque:
cifras que ocupan lugar par y las que (9+7+8)–(0+2+0)=22, que es
ocupan lugar impar en el número divisible por 11.
dado (si esta resta no se puede hacer
porque resulta negativa, se calcula la
diferencia de la suma de las cifras que
ocupan lugar impar y las que ocupan
lugar par). Si la resta da cero también
es divisible por 11, ya que el cero es
divisible por todos los números.
1.100 Un número es divisible por 100 si las 5600 es divisible por 100
cifras de sus decenas y unidades son porque termina en 00.
ceros.
1000 Un número es divisible por 1000 si 2040000 es divisible por 1000
las cifras de sus centenas, decenas y porque termina en 000.
unidades son ceros.

TAMadrid
7
-Pág.11-
M AT E M Á T I C A S

2. LOS NÚMEROS ENTEROS

2.1. REPRESENTACIÓN Y ORDENACIÓN


El conjunto de los números enteros, que representaremos por la letra Z, abarca a todos
los números naturales (números enteros positivos) y a los números naturales precedidos del
signo (–), (números enteros negativos). Diremos que N es un subconjunto de Z.
Se llama valor absoluto de un número entero al número natural que resulta al suprimir
su signo. Ejemplo: El valor absoluto de –5 = 5 y se representa: ⱍ–5ⱍ= 5.
Para ordenar dos números enteros tenemos que seguir una serie de criterios:
– Dados dos números enteros y positivos, es menor el que tiene un valor absoluto
inferior.
– Dados dos números enteros y negativos, es menor el que tiene mayor valor absoluto.
– Dado un número entero positivo y un número entero negativo es mayor siempre el
positivo.
– El cero siempre es mayor que cualquier número negativo y menor que cualquier
positivo.

2.2. OPERACIONES CON NÚMEROS ENTEROS

2.2.1. SUMA Y PROPIEDADES

Para sumar dos números enteros de igual signo, se suman sus valores absolutos. El
resultado es otro número de igual signo que los sumandos.
Ejemplos: (+5) + (+9) = +14 (–3) + (–7) = –10

Para sumar dos números enteros de distinto signo, se restan sus valores absolutos.
El resultado es otro número cuyo signo es igual al de mayor valor absoluto.
Ejemplos: (–5) + (+1) = –4 (+5) + (–1) = +4

La suma de números enteros tiene las siguientes propiedades:

Conmutativa.—Si cambiamos el orden de los sumandos el resultado no varía.

8 TAMadrid

-Pág.12-
U. D. 1 . - N Ú M E R O S Y O P E R AC I O N E S

Asociativa.—Cuando realizamos la suma de tres números podemos efectuarla de dos


maneras distintas, según el orden en que empleemos los sumandos, lo cual señalaremos
mediante corchetes. Es similar a la de los naturales.
Elemento Neutro.—Al sumar el 0 a cualquier número entero, obtendremos como
resultado el mismo número. El cero es, pues, el elemento neutro de la suma.
Elemento Opuesto.—Si + a es un número entero, – a es su elemento opuesto.

2.2.2. RESTA

Para restar dos números enteros se suma al minuendo el opuesto del sustraendo.
Ejemplos: (–3) – (–5) = (–3) + (+5) = +2 (+8) – (–3)= (+8)+ (+3) = +11

2.2.3. MULTIPLICACIÓN Y PROPIEDADES

Para multiplicar dos números enteros de igual signo, se multiplican sus valores
absolutos. Su resultado es otro número entero y positivo.
Ejemplos: (+12) • (+2) = +24 (–8) • (–5) = +40

Para multiplicar dos números enteros de diferente signo, se multiplican sus valores
absolutos. El resultado es otro número entero y negativo.
Ejemplo: (–10) • (+5) = –50

La multiplicación de números enteros tiene las siguientes propiedades:


Conmutativa.—El orden de los factores no altera el valor del producto.
Asociativa.—Para todo a, b, c D Z, se cumple: (a × b) × c = a × (b × c).
Elemento Neutro.—El número entero (+1) es el elemento neutro en el producto de
números enteros, ya que al multiplicar cualquier número por él, nos da como resultado el
mismo número.
Distributiva.—La multiplicación es distributiva con respecto a la suma. Para todo a, b,
c D Z se cumple que: a × (b + c) = a × b + a × c ó (a + b) × c = a × c + b × c.

2.2.4. DIVISIÓN

Para dividir dos números enteros de igual signo, se dividen sus valores absolutos.
El resultado es otro número entero y positivo.
Ejemplo: (+10) : (+2) = +5 (–10) : (–2) = +5

TAMadrid
9
-Pág.13-
M AT E M Á T I C A S

Para dividir dos números enteros de distinto signo, se dividen sus valores absolutos.
El resultado es otro número entero y negativo.
Ejemplo: (+10) : (–2) = –5 (–10) : (+2) = –5

2.2.5. POTENCIA DE UN NÚMERO ENTERO

Las potencias de exponente natural de un número entero es otro número entero, cuyo
valor absoluto es la potencia del valor absoluto y cuyo signo es el que se deduce de las
siguientes reglas:

1. Las potencias de exponente par son siempre positivas.

2. Las potencias de exponente impar tienen el mismo signo que el número base.
Ejemplos: (+2) = +8
3 3
2. (–4) = –64

2.2.6. OPERACIONES COMBINADAS

Podemos realizar conjuntamente varias operaciones, sumas, restas, productos,


divisiones y potencias. Esto es lo que denominamos operaciones combinadas. Para poder
efectuar éstas operaciones se debe seguir siempre un orden:

1.o Primero se efectúan los paréntesis y a continuación los corchetes.

2.o Se calculan los productos y las divisiones en el orden en que aparecen.

3.o Por último se efectúan las sumas y restas en el orden en que aparecen.
Ejemplo: 14 – [7 + (4 × 3) – (4 × 2 – 6)] + (4 + 6 – 5 × 3) + 3 – (5 – 8 : 2) =
= 14 – [19 – 2] + (–5) + 3 – (1) = 14 – 17 – 5 + 3 – 1 = –6

3. LOS NÚMEROS DECIMALES

3.1. CONCEPTO DE NÚMERO DECIMAL


Los números decimales son aquellos que nos permiten escribir aproximaciones de
fracciones, facilitándonos efectuar las operaciones básicas.
Una fracción decimal es aquella cuyo denominador es la unidad seguida de ceros.
Ejemplos: 8/10 = 0,8 52/10000 = 0,0052

10 TAMadrid

-Pág.14-
U. D. 1 . - N Ú M E R O S Y O P E R AC I O N E S

Cualquier fracción decimal se puede escribir como número decimal de la siguiente


forma:
– Se escribe el numerador y se separan con una coma tantas cifras a la derecha, como
ceros acompañan al denominador, en caso necesario se agregan ceros a la izquierda.
– Las cifras situadas a la izquierda de la coma decimal forman la parte entera de la
división, las de la derecha, la parte decimal.
– Ejemplos: 6/10 = 6 : 10 = 0,6 Se lee seis décimas.
– Ejemplos: 825/100 = 825 : 100 = 8,25 Se lee ocho enteros y veinticinco centésimas.

Si se desea transformar en fracción un número decimal, el numerador se formará


con las cifras, suprimidos los ceros de la izquierda del número decimal, y el denominador
será la unidad acompañada de tantos ceros como cifras tiene la parte decimal.
Ejemplos: 0,008 = 8/1000 9,58 = 958/100

3.2. OPERACIONES CON NÚMEROS DECIMALES

3.2.1. SUMA DE NÚMEROS DECIMALES


Los números decimales se suman como si fuesen números naturales, teniendo en cuenta
que: a) se escribe un número debajo del otro de manera que coincidan las unidades del
mismo orden y la coma decimal; b) en el resultado se pone la coma debajo de las comas de
los sumandos.
Ejemplo: 25,693
+
20,010
25,703
3.2.2. RESTA DE NÚMEROS DECIMALES
Se efectúa el mismo procedimiento que en la suma pero aplicando la operación de restar:
Ejemplo: 18,993

26,521
12,472
3.2.3. MULTIPLICACIÓN DE UN NÚMERO DECIMAL
Los números decimales se multiplican igual que los números naturales, tanto si
multiplicamos por un número natural, como si es por otro decimal. Después se corre la
coma tantos lugares como decimales tenga el número.
Ejemplos: 0,95 × 5 = 4,75 1,25 × 2,15 = 2,68

TAMadrid
11
-Pág.15-
M AT E M Á T I C A S

3.2.4. DIVISIÓN DE NÚMEROS DECIMALES


Para dividir se opera igual que con los números naturales y en el resultado se separan
con una coma, empezando por la derecha, tantas cifras decimales como tenga el número
decimal.
Ejemplo: 99,853 : 37 = 2,698
Para el caso de que la división fuese entre dos números decimales, se multiplica el
dividendo y el divisor por 10, 100, 1000, etc. se ponen tantos ceros como cifras decimales
tenga el número, de modo que el divisor se transforme en un número natural, y después se
realiza la división.
Ejemplo: 632,58 : 25,46 = 63258 : 2546 = 24,84

4. LOS NÚMEROS FRACCIONARIOS


4.1. CONCEPTO DE NÚMERO FRACCIONARIO
Se denomina número fraccionario o fracción al cociente indicado de dos números
llamados numerador y denominador. Son números que expresan una o varias partes de la
unidad dividida en partes iguales.

La figura superior representa al Planeta Tierra, del cual las 3/5 partes están cubiertas de
agua. Así se expresan las fracciones: en el numerador (3) representa las partes cubiertas de
agua y en el denominador el total de partes iguales.

4.2. FRACCIONES EQUIVALENTES


Dos fracciones son equivalentes cuando una de ellas resulta de multiplicar o dividir los
dos términos (numerador y denominador) de la otra por un mismo número.
Ejemplo: 3/5 es equivalente a 6/10 ya que hemos multiplicado por 2 el numerador y
el denominador de la primera fracción. Su representación fráfica sería:

3/5
6/10

12 TAMadrid

-Pág.16-
U. D. 1 . - N Ú M E R O S Y O P E R AC I O N E S

4.3. REDUCCIÓN DE FRACCIONES A COMÚN DENOMINADOR


Para reducir fracciones a común denominador:
– Se realiza el producto de los denominadores y se pone como denominador.
– Se multiplica cada numerador por todos los denominadores, menos por el suyo.
Ejemplo: Sean las fracciones 3/4, 1/3 y 5/6
Ejemplo: 3/4 = (3 × 3 × 6) / (4 × 3 × 6) = 54/72 las tres tienen el mismo denominador
Ejemplo: 1/3 = (1 × 6 × 4) / (3 × 6 × 4) = 24/72 las tres tienen el mismo denominador
Ejemplo: 5/6 = (5 × 3 × 4) / (6 × 3 × 4) = 60/72las tres tienen el mismo denominador

4.4. COMPARACIÓN DE FRACCIONES


4.4.1. CON EL MISMO DENOMINADOR
Es mayor la que tiene el mayor numerador, por ejemplo: 1/5 y 3/5, es mayor 3/5, lo que
puede verse gráficamente.

4.4.2. CON EL MISMO NUMERADOR


Cuando dos fracciones tienen el mismo numerador, es mayor la que tiene menor
denominador.
Ejemplo: Es mayor 5/8 que 5/12.

4.4.3. CON NUMERADORES Y DENOMINADORES DISTINTOS


Para realizar esta comparación es necesario calcular el común denominador.
Ejemplo: ¿Cuál de las siguientes fracciones es mayor, 2/7 ó 3/5?
Ejemplo: 2/7 = (2 × 5) / (5 × 7) = 10/35 3/5 = (3 × 7) / (5 × 7) = 21/35
Ejemplo: El resultado correcto es la fracción 3/5.

4.5. OPERACIONES CON FRACCIONES


4.5.1. SUMA Y RESTA DE FRACCIONES

Si los denominadores son iguales, se suman o restan los numeradores y se pone el


mismo denominador.
Ejemplo: 8/9 + 3/9 = 11/9 8/9 – 3/9 = 5/9

TAMadrid
13
-Pág.17-
M AT E M Á T I C A S

Ahora bien, si los denominadores son distintos, hay que reducirlos a común
denominador, y a continuación, sumar o restar las fracciones obtenidas.
Ejemplos: Sumar 2/3 + 3/4
2/3 = (2 × 4) / (3 × 4) = 8/12 3/4 = (3 × 3) / (4 × 3) = 9/12
(8/12) + (9/12) = 17/12
Ejemplos: Restar 6/7 – 4/5
6/7 = (6 × 5) / (7 × 5) = 30/35 4/5 = (4 × 7) / (7 × 5) = 28/35
(30/35) – (28/35) = 2/35

4.5.2. MULTIPLICACIÓN DE FRACCIONES


a) Para multiplicar una fracción por un número entero se multiplica el numerador por
el número entero se pone el mismo denominador.
Ejemplo: 4/5 × 7 = 28/5
b) Para multiplicar dos fracciones, se multiplican los numeradores y su producto es
el numerador; el denominador es el producto de los denominadores.
Ejemplo: 5/10 × 6/7 = 30/70

4.5.3. DIVISIÓN DE FRACCIONES


Para dividir dos fracciones, se multiplica la fracción dividendo por la inversa de la
fracción divisor.
Ejemplo: 3/5 : 4/7 = 3/5 × 7/4 = 21/20

4.6. ORDEN Y REPRESENTACIÓN DE LOS NÚMEROS


EN LA RECTA
Los números se pueden representar gráficamente, mediante los puntos de una recta. En
una recta horizontal (eje de abcisas), se toma un punto cualquiera que se señala como cero,
y a éste punto lo llamamos origen. Desde el origen hacia la derecha y a distancias iguales
se señalan los enteros positivos, y desde el origen para la izquierda y a las misma distancia
que los anteriores, se señalan los enteros negativos. Si dividimos esas distancias en partes
iguales, representaremos los números fraccionarios, tanto positivos como negativos.
–8/3 –8/5 –2/3 1/2 5/4 17/7

–4 –3 –2 –1 0 1 2 3 4

De cualquier par de números representados gráficamente, siempre es mayor aquel que


esté más a la derecha y menor aquel que está más a la izquierda.

14 TAMadrid

-Pág.18-
U. D. 1 . - N Ú M E R O S Y O P E R AC I O N E S

4.7. APROXIMACIÓN Y ESTIMACIÓN DE CANTIDADES


4.7.1. APROXIMACIÓN DECIMAL DE UN COCIENTE INEXACTO

Podemos decir que un cociente es aproximado con un error menor que una cifra decimal
dada, cuando su diferencia del verdadero valor es menor que la unidad decimal del orden
expresado.

La aproximación puede ser por defecto y por exceso.


Ejemplo: Si dividimos 39,68 : 25,33 = 1,56652 con un error menor de una centésima
(e < 0,01). Tenemos que la aproximación del cociente por defecto seria: 1,56, y la
aproximación por exceso seria: 1,57.

5. MAGNITUDES PROPORCIONALES

5.1. MAGNITUDES DIRECTAMENTE PROPORCIONALES


Para que dos magnitudes sean directamente proporcionales se tiene que cumplir:

a) Que a una cantidad determinada de la primera, le corresponda una cantidad


determinada de la segunda.

b) Y que al multiplicar o dividir una de ellas por un número cualquiera, la otra queda
multiplicada por ese mismo número.
Ejemplo: Es directamente proporcional los kilos de fruta con su costo, o sea a mayor
números de kilos, costará más dinero que a menor número de kilos. El gasto de
combustible con la velocidad, etc.

5.1.1. CONSTANTE DE PROPORCIONALIDAD

La proporción que se forma entre dos magnitudes directamente proporcionales se


le llama constante de proporcionalidad.
Ejemplo: Si un automóvil recorre en 1 hora 80 Km., en 2 horas 160 Km., y dado que
el tiempo empleado y el espacio recorrido son magnitudes directamente proporcionales,
tendremos que su constante de proporcionalidad, K, será:
K = Espacio/Tiempo = 80/1= 160/2 = 80

TAMadrid
15
-Pág.19-
M AT E M Á T I C A S

5.2. MAGNITUDES INVERSAMENTE PROPORCIONALES


Para que dos magnitudes sean inversamente proporcionales se tiene que cumplir:

a) Que a una cantidad determinada de la primera le corresponda una cantidad


determinada de la segunda.

b) Y que al multiplicar o dividir una de ellas por un número, la otra queda dividida o
multiplicada por el mismo número.

Ejemplo: Es inversamente proporcional, por ejemplo: el número de niños con el


tiempo que tardan en comerse una tarta, o sea, que a mayor número de niños implica
menor tiempo en comerse una tarta. Otro ejemplo es, el número de obreros y el tiempo
necesario para construir una casa, etc.

5.3. REGLA DE TRES SIMPLE DIRECTA


Sirve, para dadas dos cantidades correspondientes a magnitudes directamente
proporcionales, calcular la cantidad de una de esas magnitudes correspondiente a una
cantidad dada de la otra magnitud. Veamos ésto mejor con un ejemplo:

Si 5 Kg. de manzanas cuestan 4,25 €. ¿cuánto costarán 11 Kg.? Se expresa:


Si 15 Kg. —— cuestan —— 4,25 €.
11 Kg. —— cuestan —— 42x €.

y aplicando la regla de tres directa: x = (11 × 4,25) / 5 = 9,35 €.

5.4. REGLA DE TRES INVERSA


La regla de tres inversa sirve, para que dadas dos cantidades correspondientes a
magnitudes inversamente proporcionales, calcular la cantidad de una de estas
magnitudes, que corresponde a una cantidad dada de la otra magnitud. Veamos esto
mejor con un ejemplo:

Si 4 niños tardan 30 minutos en comerse una tarta. ¿cuántos minutos tardarán en


comerse la misma tarta 7 niños? Se expresa:
Si 4 niños —— tardan –—— 30 minutos
7 niños —— tardarán —— 4x minutos

y aplicando la regla de tres inversa: x = (4 × 30) / 7 = 17,14 minutos.

16 TAMadrid

-Pág.20-
U. D. 1 . - N Ú M E R O S Y O P E R AC I O N E S

5.5. PORCENTAJES
Los porcentajes, o tantos por ciento, es una aplicación de la proporcionalidad. En
dos magnitudes directamente proporcionales, a la cantidad de una de ellas que corresponde
a 100 unidades de la otra, se le llama tanto por ciento.
Con los porcentajes se nos pueden presentar dos tipos de problemas, que los vamos a
ver resolviendo los dos ejemplos siguientes:
Ejemplo 1: Calcular el 9% de 75500. Se efectúa multiplicando la cantidad por el
cociente que resulta de dividir el porcentaje por 100. Así tenemos:
X = 75500 × (9/100) = 6795
Ejemplo 2: En 1999 el alquiler de una casa costaba 500 €. En el año 2000 subió el
precio 30 . más. ¿Cuánto es el tanto por ciento de aumento? El resultado se calcula
estableciendo una regla de tres directa.
Si de 500 €. —— subiór —— 30 €.
De 100 €. —— subirá —— 0x Ps.
x = (100 × 30) / 500 = 6%.

6. ALGORITMOS BÁSICOS E INSTRUMENTO


DE CÁLCULO
6.1. OPERACIONES ARITMÉTICAS
Son todas aquellas operaciones que tienen por objeto obtener un número llamado
resultado, mediante dos o más números llamados datos.
Las principales operaciones son las que ya hemos visto en temas anteriores: suma, resta,
multiplicación, división, potenciación y radicación. Esta última operación la veremos a
continuación.

6.1.1. RAÍZ CUADRADA


La raíz cuadrada de un número es otro número que elevado al cuadrado da el
primero.
Ejemplo: La raíz cuadrada de 49 es 7, porque 7 elevado al cuadrado da 49.

La raíz cuadrada se representa por medio del signo 3 , llamado signo radical, en la

siguiente forma: 364 = 8. La cantidad que figura al lado del signo radical se llama radicando
y el resultado de la operación raíz. En el ejemplo anterior, el radicando sería 64 y la raíz 8.

TAMadrid
17
-Pág.21-
M AT E M Á T I C A S

La raíz cuadrada es una operación inversa a la del cuadrado de un número, ya que sucede
lo siguiente: Si el cuadrado de 5 es 25, la raíz cuadrada de 25 es 5.

Raíz cuadrada de un número menor que 100.—Para hallar la raíz cuadrada de un número
menor que 100, basta buscar entre las cifras significativas aquella que elevada al cuadrado dé,
exactamente, el número citado. Y si no exactamente, del modo más aproximado posible.
Ejemplo: La raíz cuadrada de 81 es exactamente 9. Y la raíz cuadrada de 38 diremos
que es 6 y pico, pero no exactamente, porque el cuadrado de 6 es 36 y todavía faltan dos
unidades para llegar a 38. En este caso decimos que hay un resto de 2 unidades.

Raíz cuadrada de un número mayor que 100.—Vamos a extraer por ejemplo, la raíz
cuadrada de 70423.

Escrito el número se cubre con el signo radical, y a la derecha se trazan dos líneas, una
vertical y otra horizontal, como si se tratara de hacer una división. En el lugar reservado
para el divisor es donde van apareciendo las cifras de la raíz.
7 04 23 265
4 47 • 7 = 329 (7 es grande)
3 04
2 76 46 • 6 = 276 (6 es bueno)
2 28 23 525 • 5 = 2625
2 26 25
7 01 98... (resto)

Se divide el número dado en grupos de dos cifras a partir de la derecha (el grupo de la
izquierda puede resultar de una cifra). Se extrae de memoria la raíz cuadrada del primer
grupo de la izquierda, de 7. El resultado obtenido, 2, se anota como la primera cifra de la
raíz. Se eleva 2 al cuadrado, y el resultado 4 se escribe de bajo del 7. Se resta y queda 3.
Se baja el grupo siguiente 04, que con el 3 forma 304. De este número separamos la
cifra de la derecha 4, dejando a la izquierda 30. Vayamos ahora a la raíz. Lo que allí se
hubiera escrito se multiplica por 2, y el resultado 4 se escribe debajo mismo de la raíz. El
número 30 que habíamos apartado a la izquierda se divide por ese 4. El cociente será
probablemente, la nueva cifra de la raíz.
Para probar si esa cifra es buena se escribe a la derecha del 4, y el número formado 47
se multiplica por la misma cifra 7. Si el producto obtenido puede restarse de 304, la cifra es
buena. Si no se puede es grande. En este caso es grande, habrá que ensayar con otra más
pequeña, el 6.
Para ello se repite el 4 debajo del ensayo anterior. Se escribe el 6 a su derecha y se
multiplica por 6. El producto obtenido 276 ya puede restarse de 304, luego el 6 es bueno, y
sin esperar más lo llevamos a la raíz, al lado del 2. Ya tenemos en la raíz 26.

18 TAMadrid

-Pág.22-
U. D. 1 . - N Ú M E R O S Y O P E R AC I O N E S

El 276 se escribe debajo de 304, se resta y queda 28. Se baja el grupo siguiente 23, que
con el 28 forma 2823. De este número se separa la cifra de la derecha, dejando a la izquierda
282.
Volvemos a la raíz. Lo que allí hubiera, 26 se multiplica por 2, el resultado obtenido 52
se escribe debajo de los ensayos anteriores.
Se divide el número 282, que habíamos dejado a la izquierda, por ese 52. El cociente 5
será probablemente la nueva cifra de la raíz.
Para probarla, se escribe a la derecha de 52, y el número formado 525 se multiplica por
5. Como el producto obtenido puede restarse de 2823, la cifra es buena.
Llevaremos pues el 5 a la derecha de la raíz, de 26, formando así la raíz completa: 265.
El producto obtenido en el último ensayo 2625 se escribe debajo de 2823. Se resta, y la
diferencia 198, que es la última, será el resto de la operación.
Nota: Si alguna de las divisiones diera “cero” en el cociente, se escribe “cero” en la raíz y se baja el grupo
siguiente. Después se separa la cifra de la derecha y se continúa como antes.

6.2. CÁLCULO
El cálculo es el hecho de efectuar la operación u operaciones indicadas. Podemos
distinguir entre dos tipos de cálculo.

Cálculo numérico.—Es el que se realiza con números. Por ejemplo, la raíz cuadrada
efectuada en el apartado anterior.

Cálculo literal.—Es aquel en el que se sustituyen los números por letras del abecedario,
tiene la gran ventaja de que sus resultados reflejan formas generales.

6.3. REGLAS DE USO DE LA CALCULADORA


Hoy en día, las calculadoras son el instrumento más utilizado para realizar operaciones
matemáticas. En el mercado existen infinidad de modelos, desde las más básicas, que sólo
realizan operaciones de suma, resta, multiplicación y división, hasta las más sofisticadas,
las cuales son como un ordenador en pequeño, ya que permiten realizar pequeños
programas informáticos.

Sin embargo, quizás la calculadora más usual sea la llamada calculadora científica como
la que muestra la figura siguiente. Esta tiene además de las operaciones básicas, ciertas
operaciones más complejas, como puede ser la radicación, potenciación, hallar logaritmos,
puede operar directamente con fracciones, puede hallar las funciones trigonométricas más

TAMadrid
19
-Pág.23-
M AT E M Á T I C A S

importantes como el seno, coseno y tangente de un ángulo, puede realizar operaciones


estadísticas, etc.

De todos modos, y debido a la gran cantidad de modelos


existentes, no podemos establecer unas reglas de uso, ya que
cada fabricante establece las suyas propias.

En líneas generales, su utilización es bastante sencilla, ya


que en la mayoría se le introducen los datos de forma similar
a como nosotros los expresamos en el papel. Veamos unas
operaciones de ejemplo, según la calculadora de la
fotografia.
Ejemplo 1: Multiplicar 678 por 987. Primero
encendemos la calculadora, a continuación introducimos
con el teclado numérico la cifra 678. Pulsamos la tecla del
símbolo × y seguidamente introducimos la otra cifra que
queramos multiplicar 987. Pulsamos la tecla del =, y
obtenemos el resultado, 669186.
Ejemplo 2: Realizar la raíz cuadrada del número 852,96. Introducimos la cifra con el
teclado numérico, teniendo en cuenta , que en vez de la coma ponemos un punto.
Seguidamente pulsamos la tecla del signo radical, y directamente obtenemos el
resultado, 29.205479 = 29,205479.

Normalmente, las calculadoras tienen ocho cifras o dígitos, y además casi todas las
teclas tienen más de una función, que se cambia en la tecla INV.

Pero el elemento de cálculo más importante que existe hoy día es el ordenador, ya que
a través de los distintos programas puede realizar cálculos complejos, que sin él serian
prácticamente imposibles.

20 TAMadrid

-Pág.24-
U. D. 1 . - N Ú M E R O S Y O P E R AC I O N E S

RESUMEN

Recuerda que:

1. Los números podemos clasificarlos en Naturales, Enteros, Decimales y


Fraccionarios.

2. Los números naturales van del cero al infinito y los utilizamos habitualmente para
contar. Con ellos podemos realizar todo tipo de operaciones básicas: suma, resta,
multiplicación, división; cada una de las cuales tiene unas propiedades.

3. Los números enteros abarcan todos los números naturales precedidos de los signos
(+) o (–), según sean positivos o negativos. Podemos operar con ellos igual que con
los números naturales.

4. Los números decimales son los que nos permiten escribir aproximaciones de frac-
ciones, facilitándonos efectuar las operaciones básicas, que no eran posible sólo con
los números naturales.

5. Los números fraccionarios son los que expresan una o varias partes de la unidad
dividida en partes iguales. Las fracciones las podemos comparar para ver si son
equivalentes. Con ellas podemos realizar todo tipo de operaciones.

6. Las aplicaciones de las magnitudes proporcionales son: regla de tres simple directa,
regla de tres inversa y el cálculo de los tantos por ciento.

7. La raíz cuadrada es una operación inversa al cuadrado de un número.

8. Es muy conveniente leerse las instrucciones de la calculadora antes del manejo de


la misma ya que no todas las calculadoras operan de la misma forma.

TAMadrid
21
-Pág.25-
M AT E M Á T I C A S

EJERCICIOS DE AUTOCOMPROBACIÓN

1. ¿Cuál de las siguientes expresiones representa la propiedad asociativa de la suma de los


números naturales?
A. (a + b) + c = d o a + (b + c) = d B. (a + b + c) = d o (a + b + c) = d
C. a + b + c = d o b+c+a=d D. (a + b + c) = d – a o a – b = c – d

2. Indica cuál de los siguientes números es divisible por 3.


A. 47.321 B. 52.468 C. 64.851 D. 97.483

3. ¿Cuál es el resultado de la siguiente operación: (–3) + (–6) + 5?


A. 5 B. (–5) C. (–4) D. 4

4. Representa la expresión decimal de 548 / 100000.


A. 0,0548 B. 0,00548 C. 0,000548 D. 0,0000548

5. Un tren esta compuesto por 5 vagones, que miden cada uno 12,6 m. Halla la longitud total de
los vagones del tren.
A. 64,2 m B. 63 m C. 78 m D. 82,5 m

6. Indica cuales de las siguientes fracciones son equivalentes:


A. 4/3 y 40/30 B. 24/12 y 35/42 C. 29/32 y 34/85 D. 16/48 y 31/93

7. Reduce a común denominador las siguientes fracciones: 3/5, 4/6 y 7/9


A. 162/270, 180/270, 210/270 B. 152/280, 198/280, 194/390
C. 248/320, 324/320, 128/320 D. 15/25, 26/25, 32/25

8. Halla el resultado de la siguiente operación: 8/5 – 4/3


A. 6/15 B. 4/15 C. 2/15 D. 5/15

9. Un automóvil tarda 4 horas en ir de Madrid a Cáceres a 70 Km/h. ¿Cuánto tiempo tardará otro
que va a 120 Km/h.?
A. 7 horas B. 3,33 horas C. 2,33 horas D. 4 horas

10. Calcula el precio de un video sabiendo que después de hacernos un descuento del 15% hemos
pagado por él 510 .
A. 525 €. B. 600 €. C. 615 €. D. 510 €.

22 TAMadrid

-Pág.26-
U. D. 1 . - N Ú M E R O S Y O P E R AC I O N E S

RESPUESTAS A LOS EJERCICIOS

1. A

2. C
3. C
4. B
5. B
6. A
7. A
8. B
9. C
10. B

TAMadrid
23
-Pág.27-
-Pág.28-
U. D. 2 . - M E D I D A , E S T I M AC I Ó N Y C Á L C U L O D E M AG N I T U D E S

ÍNDICE
Pag.
OBJETIVOS . . . . . . . . . . . . . . . . . . . . . . . . . . . . . . . . . . . . . . . . . . . . . . . . . . . . . . . . . . . 2

INTRODUCCIÓN . . . . . . . . . . . . . . . . . . . . . . . . . . . . . . . . . . . . . . . . . . . . . . . . . . . . . . 3

MAPA CONCEPTUAL . . . . . . . . . . . . . . . . . . . . . . . . . . . . . . . . . . . . . . . . . . . . . . . . . . 4

DESARROLLO DE CONTENIDOS . . . . . . . . . . . . . . . . . . . . . . . . . . . . . . . . . . . . . . . 5

1. MEDICIÓN DE MAGNITUDES . . . . . . . . . . . . . . . . . . . . . . . . . . . . . . . . . . . . 5

2. MEDIDAS APROXIMADAS Y CÁLCULO DE ERRORES . . . . . . . . . . . . . 11

3. PRINCIPALES FÓRMULAS DE FIGURAS PLANAS


Y CUERPOS GEOMÉTRICOS . . . . . . . . . . . . . . . . . . . . . . . . . . . . . . . . . . . . 13

4. RAZONES TRIGONOMÉTRICAS . . . . . . . . . . . . . . . . . . . . . . . . . . . . . . . . . 16

5. INSTRUMENTOS DE MEDIDA . . . . . . . . . . . . . . . . . . . . . . . . . . . . . . . . . . . 22

RESUMEN . . . . . . . . . . . . . . . . . . . . . . . . . . . . . . . . . . . . . . . . . . . . . . . . . . . . . . . . . . . 24

EJERCICIOS DE AUTOCOMPROBACIÓN . . . . . . . . . . . . . . . . . . . . . . . . . . . . . . . 25

RESPUESTAS A LOS EJERCICIOS . . . . . . . . . . . . . . . . . . . . . . . . . . . . . . . . . . . . . 26

-Pág.29-
M AT E M Á T I C A S

OBJETIVOS
Al finalizar esta Unidad Didáctica, el alumno será capaz de:

• Utilizar las principales magnitudes de medida del Sistema Métrico Decimal


así como sus múltiplos y submúltiplos, para longitudes, áreas, volúmenes y
masas.

• Distinguir las diferentes medidas de tiempo.

• Dominar las distintas medidas de ángulos y el sistema sexagesimal de me-


dida.

• Calcular los errores cometidos en una medición.

• Aplicar las principales razones trigonométricas mas elementales.

• Manejar los instrumentos de medida más frecuentes.

-Pág.30-
U. D. 2 . - M E D I D A , E S T I M AC I Ó N Y C Á L C U L O D E M AG N I T U D E S

INTRODUCCIÓN
L as unidades de medida que conocemos hoy en día, no han existido siempre
sino que antiguamente, cada país, o incluso cada zona de un mismo país,
empleaban diferentes unidades de medida. Esto ocasionaba con frecuencia que las
relaciones comerciales y científicas se vieran dificultadas por la diversidad existen-
te de unidades de medida.

Si además, a esto le sumamos, que los múltiplos y submúltiplos de esas unidades de


medida no se construían tomando como base el número 10, la dificultad de inter-
cambio se complicaba aún más.

Todos estos inconvenientes se solucionarían unificando las unidades y sus múltiplos


y submúltiplos para todos los países. Esto ocurrió en Francia a finales del siglo
XVIII, dónde se implantó un sistema de medidas que tenia como base el número 10.

A este sistema se le denominó “Sistema Métrico Decimal” y poco a poco se fue


implantándose en todos los países civilizados.

En España se introdujo a comienzos del siglo XIX, y en la actualidad es de obliga-


do cumplimiento en la práctica totalidad de los países.

El Sistema Métrico Decimal ha ido evolucionando a lo largo de los años con los nue-
vos descubrimientos científicos y técnicos.

Actualmente se utiliza una versión de éste denominada “Sistema Internacional”, que


se designa mediante las letras SI.

Este sistema tiene seis magnitudes fundamentales, dos magnitudes suplementarias y


veintisiete magnitudes derivadas. Cada una de estas magnitudes tiene su símbolo de
representación y su unidad.

En la definición de los múltiplos y submúltiplos se utiliza como base el número 10,


exceptuando las unidades de ángulos y de tiempo.

-Pág.31-
M AT E M Á T I C A S

M A PA C O N C E P T UA L
MAGNITUDES UNIDADES

LONGITUD METRO

SUPERFICIE METRO CUADRADO

VOLUMEN METRO CÚBICO

PESO GRAMO

TIEMPO SEGUNDO

ÁNGULO GRADO SEXAGESIMAL

CAPACIDAD LITRO

-Pág.32-
U. D. 2 . - M E D I D A , E S T I M AC I Ó N Y C Á L C U L O D E M AG N I T U D E S

1. MEDICIÓN DE MAGNITUDES
A continuación vamos a estudiar las magnitudes fundamentales del “Sistema Inter-
nacional” que son cinco: Longitud, superficie, volumen, capacidad y peso. Así como
sus múltiplos y submúltiplos, ya que son necesarios para definir de manera más clara las
cantidades de cada magnitud.

Además de las magnitudes fundamentales, vamos a estudiar otras no decimales, o sea


las que no tienen como base el 10, que son muy importantes como son: el tiempo y los
ángulos.

Antes de empezar, podemos decir que una magnitud es todo ente abstracto en el que
se pueden establecer la igualdad y la suma. Cantidad es un valor determinado de una
magnitud.

1.1. MEDIDAS DE LONGITUD


La unidad de longitud es el metro (m). Su definición ha cambiado con el paso del tiem-
po, empezó definiéndose como: Metro es la diezmillonésima parte de la longitud del
cuadrante del meridiano terrestre.

Esta longitud se construyó en una barra de platino e iridio, en la que se marcaron dos
rayas muy finas que a 0o C, distaban entre sí exactamente un metro. Esta barra se conserva
en el Museo de Pesas y Medidas de Paris.

Posteriormente, se ha perfeccionado esta definición gracias a los nuevos desarrollos


científicos, cuya definición actual es muy compleja y por tal la omitimos.

1.1.1. MÚLTIPLOS Y SUBMÚLTIPLOS


Los principales múltiplos son:

Denominación Equivalencia Símbolo


Decámetro 00010 m dam
Hectómetro 00100 m hm
Kilómetro 01000 m km
Miriámetro 10000 m mam

TAMadrid
5
-Pág.33-
M AT E M Á T I C A S

Y los principales submúltiplos son:

Denominación Equivalencia Símbolo


Decímetro 000,1 m dm
Centímetro 00,01 m cm
Milímetro 0,001 m mm

1.1.2. UNIDADES ASTRONÓMICAS

Para el estudio de la Astronomía, es necesario emplear unidades excesivamente grandes,


que no se emplean en la vida ordinaria.

Para el caso de unidades grandes, se emplean múltiplos especiales como son los siguientes:

Denominación Equivalencia Símbolo


Megametro 0000001000000 m Mm
Gigametro 0001000000000 m Gm
Terametro 1000000000000 m Tm

Ejemplo: El diámetro del Sol es 1,39 Gm, la longitud de la trayectoria de la Tierra


alrededor del Sol es 0,94 Tm.

1.2. MEDIDAS DE SUPERFICIE

La unidad de medida de superficie es el metro cuadrado (m2), que representa la super-


ficie de un cuadrado de un metro de lado.

1.2.1. MÚLTIPLOS Y SUBMÚLTIPLOS

Los principales múltiplos son los que refleja la siguiente tabla:

Denominación Equivalencia Símbolo


Decámetro cuadrado 000000100 m2 dam2
Hectómetro cuadrado 000010000 m2 hm2
Kilómetro cuadrado 001000000 m2 km2
Miriámetro cuadrado 100000000 m2 mam2

6 TAMadrid

-Pág.34-
U. D. 2 . - M E D I D A , E S T I M AC I Ó N Y C Á L C U L O D E M AG N I T U D E S

Y los principales submúltiplos son:

Denominación Equivalencia Símbolo


Decímetro cuadrado 00000,01 m2 dm2
Centímetro cuadrado 000,0001 m2 cm2
Milímetro cuadrado 0,000001 m2 mm2

Ejemplos: La superficie de un folio DIN A4 es 62580 mm2, la superficie en planta del


Monasterio del Escorial es de 3,74 hm2. Etc.

1.3. MEDIDAS DE VOLUMEN


La unidad de volumen es el metro cúbico (m3), que representa el volumen de un cubo
de un metro de lado.

1.3.1. MÚLTIPLOS Y SUBMÚLTIPLOS

Los principales múltiplos del metro cúbico son:

Denominación Equivalencia Símbolo


Decámetro cúbico 0000000001000 m3 dam3
Hectómetro cúbico 0000001000000 m3 hm3
Kilómetro cúbico 0001000000000 m3 km3
Miriámetro cúbico 1000000000000 m3 mam3

Y los principales submúltiplos son los representados en la siguiente tabla:

Denominación Equivalencia Símbolo


Decímetro cúbico 0000000,001 m3 dm3
Centímetro cúbico 0000,000001 m3 cm3
Milímetro cúbico 0,000000001 m3 mm3

Ejemplos: El volumen de la pirámide de Gizeh en Egipto es de 2 hm3, y el volumen


de un balón de fútbol es de 6 dm3.

1.4. MEDIDAS DE CAPACIDAD


La unidad de medida de la capacidad es el litro, que tiene la siguiente equivalencia:
1 litro = 1 decímetro cúbico = 0,001 m3

TAMadrid
7
-Pág.35-
M AT E M Á T I C A S

1.4.1. MÚLTIPLOS Y SUBMÚLTIPLOS

Los principales múltiplos del litro son los que aparecen en la siguiente tabla:

Denominación Equivalencia Símbolo


Decalitro 00010 l dal
Hectolitro 00100 l hl
Kilolitro 01000 l kl
Mirialitro 10000 l mal

Y los principales submúltiplos del litro son:

Denominación Equivalencia Símbolo


Decilitro 000,1 l dl
Centilitro 00,01 l cl
Mililitro 0,001 l ml

Ejemplos: La capacidad de una botella pequeña de cerveza es de 33 cl, la capacidad


de una piscina olímpica es aproximadamente de 1000 kl.

1.5. MEDIDAS DE PESO


La unidad de medida de peso es el gramo. Un gramo es la milésima parte del peso
patrón internacional, que consiste en un cilindro de platino e iridio de 39 mm de diá-
metro por 39 mm de altura aproximadamente. Este se encuentra en el Museo de Pesas y
Medidas de París.

1.5.1. MÚLTIPLOS Y SUBMÚLTIPLOS

Los principales múltiplos del gramo son los siguientes:

Denominación Equivalencia Símbolo


Decagramo 0000010 g dag
Hectogramo 0000100 g hg
Kilogramo 0001000 g kg
Miriagramo 0010000 g mag
Quintal métrico 0100000 g q
Tonelada métrica 1000000 g t

8 TAMadrid

-Pág.36-
U. D. 2 . - M E D I D A , E S T I M AC I Ó N Y C Á L C U L O D E M AG N I T U D E S

Y los principales submúltiplos del gramo son:

Denominación Equivalencia Símbolo


Decigramo 000,1 g dg
Centigramo 00,01 g cg
Miligramo 0,001 g mg

1.6. LA MEDIDA DEL TIEMPO


La medida del tiempo no utiliza el Sistema Métrico Decimal, ya que sus múltiplos y
submúltiplos no se forman mediante potencias de diez.

La unidad de tiempo es el segundo (s), cuya definición más exacta es: Un segundo es
la duración de 9192631770 periodos de radiación correspondiente a la transición entre
dos niveles hiperfinos del estado fundamental del átomo de cesio 133.

1.6.1. MÚLTIPLOS DEL SEGUNDO

Los principales múltiplos del segundo son:


Minuto.—Equivale a sesenta segundos y se simboliza por min.
Hora.—Equivale a sesenta minutos y se simboliza por h.
Día.—Equivale a veinticuatro horas y se simboliza por d. Representa el tiempo que
tarda la Tierra en dar una vuelta alrededor de su eje.
Se representan: 1 d = 24 h = 1440 min. = 86400 s

Para realizar mediciones de carácter más extenso y a nivel civil podemos considerar los
siguientes múltiplos:
Semana.—Equivale a 7 días
Mes.—Tiene una duración media de treinta días. Los meses de Enero, Marzo, Mayo,
Julio, Agosto, Octubre y Diciembre tienen 31 días. Los meses de Abril, Junio, Septiembre
y Noviembre tienen 30 días. El mes de Febrero es un poco extraña su duración, ya que nor-
malmente tiene 28 días, excepto los años bisiestos, en los que tiene 29 días. Los años bisies-
tos son los múltiplos de cuatro, exceptuando los que terminando en dos ceros y suprimiendo
éstos no formen un número divisible por cuatro.
Año solar.—Es el tiempo que tarda la Tierra en dar una vuelta alrededor del Sol. Su dura-
ción real es: 365 días, 5 horas, 48 minutos y 47 segundos.

TAMadrid
9
-Pág.37-
M AT E M Á T I C A S

Año civil.—Equivale a 365 días, excepto los años bisiestos, que tienen 366 días.
Bienio.—Equivale a 2 años.
Trienio.—Equivale a 3 años.
Lustro.—Equivale a 5 años.
Decenio o Década.—Equivale a 10 años.
Siglo.—Equivale a 100 años
Milenio.—Equivale a 1000 años

1.6.2. OPERACIONES CON UNIDADES DE TIEMPO

Vamos a aprender a operar con las unidades de tiempo, a través de los siguientes ejem-
plos:
Ejemplo 1: Calcular los segundos que tienen 3 horas y 20 min.: Primero, pasamos los
20 min. a segundos, sabiendo que 1 min. = 60 s. Aplicamos una regla de tres directa.
x = (20 × 60) / 1 = 1200 s

Ahora, tenemos que pasar las 3 h a segundos, sabiendo que 1 h = 60 min. = 3600 s: Con
lo cuál aplicamos otra regla de tres simple directa:
1 h ——— 3600 s x = (3 x 3600) / 1 = 10800 s
3 h ——— 36x0 s x = (3 × 3600) / 1 = 10800 s

por lo tanto el resultado final se obtendrá de sumar los dos anteriores, o sea:
3 h y 20 min = 10800 s + 1200 = 12000 s

Ejemplo 2: Pasar 172800 s a días. En principio, pasaremos los segundos a minutos,


simplemente dividiendo por 60, o bien a través de regla de tres. Después, transformare-
mos los minutos en horas, volviendo a dividir por 60. Y para finalizar, pasaremos las
horas a días, simplemente dividiendo por 24.
172800 / 60 = 2880 min. 2880 / 60 = 48 h 48 / 24 = 2 días

1.7. LA MEDIDA DE LOS ÁNGULOS


Los ángulos se miden igual que los arcos, ya que los ángulos se miden por los arcos que
abarcan. En este tipo de magnitudes se consideran dos tipos de medidas: centesimales y
sexagesimales.

10 TAMadrid

-Pág.38-
U. D. 2 . - M E D I D A , E S T I M AC I Ó N Y C Á L C U L O D E M AG N I T U D E S

1.7.1. MEDIDAS CENTESIMALES


En las medidas centesimales, la unidad fundamental es el grado centesimal (g). Este se
obtiene al dividir una circunferencia en cuatrocientas partes iguales, cada una de esas par-
tes es el grado centesimal.
Los submúltiplos del grado centesimal son:
Minuto centesimal.—Se obtiene al dividir un grado centesimal en cien partes iguales, se
representa por m, y su equivalencia es: 1 g = 100 m.
Segundo centesimal.—Se obtiene al dividir un minuto centesimal en cien partes iguales,
se representa por s, y su equivalencia es: 1 g = 100 m = 10000 s.
Ejemplo: 37 g 22 m 4 s = 372204 s

1.7.2. MEDIDAS SEXAGESIMALES

Son más utilizadas que las anteriores. Su unidad fundamental es el grado sexagesimal
(o). Este se obtiene al dividir una circunferencia en 360 partes iguales, cada una de esas
partes es el grado sexagesimal.

Los submúltiplos del grado sexagesimal son:


Minuto sexagesimal.—Se obtiene al dividir un grado sexagesimal en 60 partes iguales,
se representa por ('), y su equivalencia es: 1o = 60'.
Segundo sexagesimal.—Se obtiene al dividir un minuto sexagesimal en 60 partes igua-
les, se representa por ("), y su equivalencia es: 1o = 60' = 3600".
Ejemplo: 32o 42' 36" = 1962,6'

2. MEDIDAS APROXIMADAS Y CÁLCULO DE ERRORES


2.1. MEDIDAS IMPRECISAS
La mayor parte de las medidas se expresan mediante un número, y éste nunca refleja el
valor exacto de la cantidad que queremos medir, sino que es una medida aproximada.

Esto ocurre principalmente, por dos motivos: La propia imprecisión que tiene implícita
el aparato utilizado en realizar esa medida y la mayor o menor destreza de la persona que
lo maneja, lo que impide conseguir medidas exactas.

TAMadrid
11
-Pág.39-
M AT E M Á T I C A S

Sin embargo, si podemos dar un intervalo exacto, en el cuál se encuentre la medida


exacta a llevar a cabo. Por ejemplo, si yo quiero medir la longitud de la base del rectángu-
lo de la figura con una regla graduada podría decir exactamente que la longitud de la base
se encuentra entre 89 y 90 mm.

2.2. CÁLCULO DE ERRORES


2.2.1. ERROR ABSOLUTO

El error absoluto de una medida es la diferencia entre el valor aproximado y el valor


exacto. Se representa por e.

Si M' es la medida aproximada, y M es la medida exacta A e = M' – M; el error abso-


luto puede ser:
Por exceso: Cuando el resultado es positivo, o sea, cuando M' > M.
Por defecto: Cuando el resultado es negativo, o sea, cuando M' < M.
Ejemplo: ¿Qué error absoluto se comete al tomar 3,14 como /?
e = M' – M = 3,14 – / = 3,14 – 3,14159... = –0,00159

2.2.2. ERROR RELATIVO

El error relativo de una medida es el cociente que resulta de dividir el error absoluto
entre el valor exacto. Se representa por ¡.

Este error da una idea más clara de la aproximación de una medida que el error absoluto,
ya que por ejemplo, si conocemos que error absoluto cometido en una medición es de 1cm,
tiene muy distinto significado, si ese error se ha cometido en una distancia de 1 m, o en 1 km.
¡=e//
Ejemplo: Calcular el error relativo al tomar / con un valor de 3,14. Primeramente cal-
culamos el error absoluto (este ya lo tenemos calculado del ejemplo anterior
e= –0,00159), y a continuación lo hallamos como sigue:
¡ = e / / = - 0,00159 / / = –0,000506112

12 TAMadrid

-Pág.40-
U. D. 2 . - M E D I D A , E S T I M AC I Ó N Y C Á L C U L O D E M AG N I T U D E S

PRINCIPALES FÓRMULAS DE FIGURAS PLANAS


3. Y CUERPOS GEOMÉTRICOS

3.1. ÁREAS DE FIGURAS PLANAS

TAMadrid
13
-Pág.41-
M AT E M Á T I C A S

3.2. CUERPOS GEOMÉTRICOS: ÁREAS

14 TAMadrid

-Pág.42-
U. D. 2 . - M E D I D A , E S T I M AC I Ó N Y C Á L C U L O D E M AG N I T U D E S

3.3. CUERPOS GEOMÉTRICOS: VOLÚMENES

TAMadrid
15
-Pág.43-
M AT E M Á T I C A S

4. RAZONES TRIGONOMÉTRICAS

4.1. TEOREMA DE PITÁGORAS


Antes de comenzar el estudio de la trigonometría es muy conveniente conocer el Teore-
ma de Pitágoras.

El Teorema de Pitágoras dice: El cuadrado de la hipotenusa de un triángulo rectángulo


es igual a la suma de los cuadrados de los catetos:

h h2 = c12 + c22
c2

c1

Un triángulo rectángulo es aquel que tiene un ángulo recto, o sea que uno de sus ángu-
los forme 90o. Y la hipotenusa de un triángulo rectángulo, siempre es el lado opuesto al
ángulo recto y además, siempre es el lado de mayor longitud. Los otros dos lados, son los
que denominamos catetos.

Ejemplo: Calcular el valor del lado de un triángulo rectángulo sabiendo que su otro
lado vale 5 cm, y su hipotenusa 8 cm: Primeramente, vamos a despejar de la fórmula
h2 = c12 + c22, el valor de c1, por ejemplo: c1 = (h 2 –c 22) , y a continuación sustitui-
mos los valores, obteniéndose así el resultado pedido.
c1 = (82 – 52) = 6,2449 cm

4.2. TRIGONOMETRÍA
La trigonometría es la parte de las Matemáticas que tiene por objeto el estudio de las
relaciones existentes entre los lados y los ángulos de un triángulo rectángulo.
Todo triángulo tiene tres lados y tres ángulos, gracias a la trigonometría, podemos cal-
cular cada uno de los seis valores con solamente el conocimiento de dos de ellos, denomi-
nados datos. Esto ocurre, porque existen unas relaciones entre los lados y los ángulos de un
triángulo rectángulo, llamadas razones trigonométricas.

16 TAMadrid

-Pág.44-
U. D. 2 . - M E D I D A , E S T I M AC I Ó N Y C Á L C U L O D E M AG N I T U D E S

4.3. PRINCIPALES RAZONES TRIGONOMÉTRICAS


Dado un triángulo rectángulo cualquiera, se pueden definir en él, seis razones trigono-
métricas denominadas: seno, coseno, tangente, secante, cosecante y cotangente, todas ellas
de un ángulo (nunca pueden ser de un lado) del triángulo rectángulo.
En esta unidad elemental, solamente vamos a ver las principales razones trigonométri-
cas, que son las tres primeras: seno, coseno y tangente de un ángulo. Cada una de ellas se
representa como sigue:

Seno . . . . . . . . . . . . . . . . . .sen
Coseno . . . . . . . . . . . . . . . .cos
Tangente . . . . . . . . . . . . . . . .tg

Para definir mejor, dichas razones trigonométricas, nos vamos a auxiliar del siguiente
triángulo rectángulo:

Cateto opuesto Hipotenusa (h)


(co)
_
Cateto contiguo (cc)

4.3.1. SENO DE UN ÁNGULO


En un triángulo rectángulo se denomina seno de un ángulo _, al cociente que resulta de
dividir el cateto opuesto a dicho ángulo entre la hipotenusa.
sen _ = cateto opuesto / hipotenusa = co / h

4.3.2. COSENO DE UN ÁNGULO


En un triángulo rectángulo se denomina coseno de un ángulo _, al cociente que resulta
de dividir el cateto contiguo a dicho ángulo entre la hipotenusa.
cos _ = cateto contiguo / hipotenusa = cc / h

4.3.3. TANGENTE DE UN ÁNGULO


En un triángulo rectángulo, se denomina tangente de un ángulo _, al cociente que resul-
ta de dividir el cateto opuesto a dicho ángulo entre el cateto contiguo.
tg _ = cateto opuesto / cateto contiguo = co / cc
La tangente de un ángulo _ también resulta de dividir el sen _, entre el cos _:
tg _ = sen _ / cos _ = (co / h) / (cc / h) = co / cc

TAMadrid
17
-Pág.45-
M AT E M Á T I C A S

Ejemplo: Calcular todas las razones trigonométricas principales del ángulo _, supo-
niendo que en el triángulo rectángulo de la figura anterior, el cateto opuesto vale 4 cm
y el cateto contiguo vale 8 cm.
Primeramente, tenemos que calcular el valor de la hipotenusa, por ejemplo por el teore-
ma de Pitágoras: h2 = c12 + c22 A h = (c12 + c22) = (42 + 82) = 8,9442 cm
sen _ = co / h = 4 / 8,9442 = 0, 4472
cos _ = cc / h = 8 / 8,9442 = 0,8944
tg _ = co / cc = 4 / 8 = 0,5

4.4. RAZONES TRIGONOMÉTRICAS DE ANGULOS


COMPLEMENTARIOS
La suma de todos los ángulos de un triángulo cualquiera, tiene que ser igual a 180º;
ahora bien, en el caso de un triángulo rectángulo, como este tiene un ángulo recto por defi-
nición, la suma de los otros dos ángulos tiene que ser igual a 90o, ya que 90o + 90o = 180o.
Se definen dos ángulos complementarios, aquellos que al sumarlos dan 90o. Por ejem-
plo el ángulo complementario de 30o es 60º, ya que: 30o + 60o = 90o.
Por todo esto se cumple que: El seno de un ángulo es igual al coseno de su comple-
mentario, y el coseno de un ángulo es igual al seno de su complementario.
sen _ = cos (90 – _)
cos _ = sen (90 – _)

4.5. LAS RELACIONES FUNDAMENTALES


Las razones trigonométricas de un ángulo, seno, coseno y tangente, no son valores inde-
pendientes entre sí, sino que existen una serie de relaciones, que se cumplen siempre, deno-
minadas “relaciones fundamentales”, éstas son:
1. La suma de los cuadrados del seno y del coseno de cualquier ángulo es igual a uno.
sen2 _ + cos2 _ = 1
2. La tangente de un ángulo _, resulta de dividir el sen _, entre el cos _:
tg _ = sen _ / cos _

4.6. TABLAS TRIGONOMÉTRICAS


Las tablas trigonométricas son un conjunto de tablas en las que figuran expresados los
valores de las principales razones trigonométricas.

18 TAMadrid

-Pág.46-
U. D. 2 . - M E D I D A , E S T I M AC I Ó N Y C Á L C U L O D E M AG N I T U D E S

Existen distintos tipos de tablas, pero en todas ellas figura la razón trigonométrica
correspondiente, los grados y minutos de los diferentes ángulos. Hay que hacer constar que
los valores expresados en las tablas son adimensionales.

En el empleo de las tablas trigonométricas se pueden presentar dos casos:

a.—Si nos dan un ángulo, y nos piden hallar el valor de sus razones trigonométricas. En
este caso, hay que localizar los grados en la tabla correspondiente a la razón trigonométri-
ca pedida, y además en su misma horizontal, buscar la columna correspondiente a los minu-
tos pedidos. Veamos esto mejor con un ejemplo.
Ejemplo: Calcular el valor del sen 30o 10'.(Ver tablas adjuntas). Primeramente, busca-
mos la tabla del seno, y a continuación, buscamos en la columna de la izquierda , en este
caso, 30o y si seguimos la horizontal vemos que en la segunda columna en la parte supe-
rior pone los 10'. Ese valor es el pedido: sen 30o 10' = 0,50252.

b.—Si nos dan el valor de una razón trigonométrica y nos piden determinar el ángulo
que corresponde. Suponemos que la razón se encuentra directamente en las tablas. Prime-
ramente, se localiza el valor dado en la tabla correspondiente, y a continuación buscamos
los grados y minutos que corresponden a ese valor en su fila y columna respectivamente.
Veamos esto mejor con un ejemplo:
Ejemplo: Hallar el valor del ángulo _, sabiendo que la tg _ = 1,72047. (Ver tablas
adjuntas). Vamos a la tabla correspondiente de las tangentes, y en ellas buscamos el
valor 1,72047. Una vez localizado, miramos a su izquierda en la misma fila y observa-
mos que corresponde con 59o, y a continuación, miramos en la parte superior de su
columna y vemos que pone 50'. Por lo tanto: tg _ = 1,72047 A _ = 59o 50'.

El uso de las tablas trigonométricas es cada vez menor, ya que con la generalización del
uso de las calculadoras, todos estos valores se obtienen mucho más rápidamente y con
mayor precisión que mediante el empleo de dichas tablas.

A continuación, adjuntamos un ejemplo de tablas de las principales razones trigonomé-


tricas.

TAMadrid
19
-Pág.47-
20
SENOS (0° - 45° SENOS (45o - 90o)
Grados 0’ 10’ 20’ 30’ 40’ 50’ 60’ Grados 0’ 10’ 20’ 30’ 40’ 50’ 60’
10 0,00000 0,00291 0,00582 0,00873 0,01164 0,01454 0,01745 89 45 0,70711 0,70916 0,71121 0,71325 0,71529 0,71732 0,71934 44
11 0,01745 0,02036 0,02327 0,02618 0,02908 0,03199 0,03490 88 46 0,71934 0,72136 0,72337 0,72537 0,72737 0,72937 0,73135 43
12 0,03490 0,03781 0,04071 0,04362 0,04653 0,04943 0,05234 87 47 0,73135 0,73333 0,73531 0,73728 0,73924 0,74120 0,74314 42
13 0,05234 0,05524 0,05814 0,06105 0,06395 0,06685 0,06976 86 48 0,74314 0,74509 0,74703 0,74896 0,75188 0,75280 0,75471 41
14 0,06976 0,07266 0,07556 0,07846 0,08136 0,08426 0,08716 85 49 0,75471 0,75661 0,75851 0,76041 0,76229 0,76417 0,76604 40
15 0,08716 0,09005 0,09295 0,09585 0,09874 0,10164 0,10453 84 50 0,76604 0,76791 0,76977 0,77162 0,77347 0,77531 0,77715 39
16 0,10453 0,10742 0,11031 0,11320 0,11609 0,11898 0,12187 83 51 0,77715 0,77897 0,78079 0,78261 0,78442 0,78622 0,78801 38
17 0,12187 0,12476 0,12764 0,13053 0,13341 0,13629 0,13917 82 52 0,78801 0,78980 0,79158 0,79335 0,79512 0,79688 0,79864 37
18 0,13917 0,14205 0,14493 0,14781 0,15069 0,15356 0,15643 81 53 0,19864 0,80038 0,80212 0,80386 0,80558 0,80730 0,80902 36
19 0,15643 0,15931 0,16218 0,16505 0,46792 0,17078 0,17365 80 54 0,80902 0,81072 0,81242 0,81412 0,81580 0,81748 0,81915 35
M AT E M Á T I C A S

10 0,17365 0,17651 0,17937 0,18224 0,18509 0,18795 0,19081 79 55 0,81915 0,82082 0,82248 0,82413 0,82577 0,82741 0,82904 34
11 0,19081 0,19366 0,19652 0,19937 0,20222 0,20507 0,20791 78 56 0,82904 0,83066 0,83228 0,83389 0,83549 0,83708 0,83867 33
12 0,20791 0,21076 0,21360 0,21644 0,21928 0,22212 0,22495 77 57 0,83867 0,84025 0,84182 0,84339 0,84495 0,84650 0,84805 32
13 0,22495 0,22778 0,23062 0,23345 0,23627 0,23910 0,24192 76 58 0,84805 0,84959 0,85112 0,85264 0,85416 0,85567 0,85717 31
14 0,24192 0,24474 0,24756 0,25038 0,25320 0,25601 0,52882 75 59 0,85717 0,85866 0,86015 0,86163 0,86310 0,86457 0,86603 30
15 0,25882 0,26163 0,26443 0,26724 0,27004 0,27284 0,27564 74 60 0,86603 0,86748 0,86892 0,87036 0,87178 0,87321 0,87462 29
16 0,27564 0,27843 0,28123 0,28402 0,28680 0,28959 0,29237 73 61 0,87462 0,87603 0,87743 0,87882 0,88020 0,88158 0,88295 28
17 0,29237 0,29515 0,29793 0,30071 0,30348 0,30625 0,30902 72 62 0,88295 0,88431 0,88566 0,88701 0,88835 0,88968 0,89101 27
18 0,30902 0,31178 0,31454 0,31730 0,32006 0,32282 0,32557 71 63 0,89101 0,89232 0,89363 0,89493 0,89623 0,89752 0,89879 26
19 0,32557 0,32832 0,33106 0,33381 0,33655 0,33929 0,34202 70 64 0,89879 0,90007 0,90133 0,90259 0,90383 0,90507 0,90631 25
20 0,34202 0,34475 0,34748 0,35021 0,35293 0,35565 0,35837 69 65 0,90631 0,90753 0,90875 0,90996 0,91116 0,91236 0,91355 24
21 0,35837 0,36108 0,36379 0,36650 0,36921 0,37191 0,37461 68 66 0,91355 0,91472 0,91590 0,91706 0,91822 0,91936 0,92050 23
22 0,37461 0,37730 0,37999 0,38268 0,38537 0,38805 0,39073 67 67 0,92050 0,92164 0,92276 0,92388 0,92499 0,92609 0,92718 22
23 0,39073 0,39341 0,39608 0,39875 0,40141 0,40408 0,40674 66 68 0,92718 0,92827 0,92935 0,93042 0,93148 0,93253 0,93358 21

-Pág.48-
24 0,40674 0,40939 0,41204 0,41469 0,41734 0,41998 0,42262 65 69 0,93358 0,93462 0,93565 0,93667 0,93769 0,93869 0,93969 20
25 0,42262 0,42525 0,42788 0,43051 0,43313 0,43575 0,43837 64 70 0,93969 0,94068 0,94167 0,94264 0,94361 0,94457 0,94552 19

TAMadrid
26 0,43837 0,44098 0,44359 0,44620 0,44880 0,45140 0,45399 63 71 0,94552 0,94646 0,94740 0,94832 0,94924 0,95015 0,95106 18
27 0,45399 0,45658 0,45917 0,46175 0,46433 0,46690 0,46947 62 72 0,95106 0,95195 0,95284 0,95372 0,95459 0,95545 0,95360 17
28 0,46947 0,47204 0,47460 0,47716 0,47971 0,48226 0,48481 61 73 0,95630 0,95715 0,95799 0,95882 0,95964 0,96046 0,96126 16
29 0,48481 0,48735 0,48989 0,49242 0,49495 0,49748 0,50000 60 74 0,96126 0,96206 0,96285 0,96363 0,96440 0,96517 0,96593 15
30 0,50000 0,50252 0,50503 0,50754 0,51004 0,51254 0,51504 59 75 0,96593 0,96667 0,96742 0,96815 0,96887 0,96959 0,97030 14
31 0,51504 0,51753 0,52002 0,52250 0,52498 0,52745 0,52992 58 76 0,97030 0,97100 0,97169 0,97237 0,97304 0,97371 0,97437 13
32 0,52992 0,53238 0,53484 0,53730 0,53975 0,54220 0,54464 57 77 0,97437 0,97502 0,97566 0,97630 0,97692 0,97754 0,97815 12
33 0,54464 0,54708 0,54951 0,55194 0,55436 0,55678 0,55919 56 78 0,97815 0,97875 0,97934 0,97992 0,98050 0,98107 0,98163 11
34 0,55919 0,56160 0,56401 0,56641 0,56880 0,57119 0,57358 55 79 0,98163 0,98218 0,98272 0,98325 0,98378 0,98430 0,98481 10
35 0,57358 0,57596 0,57833 0,58070 0,58307 0,58543 0,58779 54 81 0,98481 0,98531 0,98580 0,98629 0,98676 0,98723 0,98769 19
36 0,58779 0,59014 0,59248 0,59482 0,59716 0,59949 0,60182 53 81 0,98769 0,98814 0,98858 0,98902 0,98944 0,98986 0,99027 18
37 0,60182 0,60414 0,60645 0,60876 0,61107 0,61337 0,61566 52 82 0,99027 0,99067 0,99106 0,99144 0,99182 0,99219 0,99255 17
38 0,61566 0,51795 0,62024 0,62251 0,62479 0,62706 0,62932 51 86 0,99255 0,99290 0,99324 0,99357 0,99390 0,99421 0,99452 16
39 0,62932 0,53158 0,63383 0,63608 0,63832 0,64056 0,64279 50 84 0,99452 0,99482 0,99511 0,99540 0,99567 0,99594 0,99619 15
40 0,64279 0,64501 0,64723 0,64945 0,65166 0,65386 0,65606 49 85 0,99619 0,99644 0,99668 0,99692 0,99714 0,99736 0,99756 14
41 0,65606 0,65825 0,66044 0,66262 0,66480 0,66697 0,66913 48 86 0,99756 0,99776 0,99795 0,99813 0,99831 0,99847 0,99863 13
42 0,66913 0,67129 0,67344 0,67559 0,67773 0,67987 0,68200 47 87 0,99863 0,99878 0,99892 0,99905 0,99917 0,99929 0,99939 12
43 0,68200 0,68412 0,68624 0,68835 0,69046 0,69256 0,69466 46 88 0,99939 0,99949 0,99958 0,99966 0,99973 0,99979 0,99985 11
44 0,69466 0,69675 0,69883 0,70091 0,70298 0,70505 0,70711 45 89 0,99985 0,99989 0,99993 0,99996 0,99998 1,00000 1,00000 10
60’ 50’ 40’ 30’ 20’ 10’ 0’ Grados 60’ 50’ 40’ 30’ 20’ 10’ 0’ Grados
COSENOS (45o - 90o) COSENOS (0o - 45o)
TANGENTES (0o - 45o) TANGENTES (45o - 90o)
Grados 0’ 10’ 20’ 30’ 40’ 50’ 60’ Grados 0’ 10’ 20’ 30’ 40’ 50’ 60’
10 0,00000 0,00291 0,00582 0,00873 0,01164 0,01455 0,01746 89 45 1,00000 1,00583 1,01170 1,01761 1,02355 1,02952 1,03553 44
11 0,07146 0,02036 0,02328 0,02619 0,02910 0,03201 0,03492 88 46 1,03553 1,04158 1,04766 1,05378 1,05994 1,06613 1,07237 43
12 0,03492 0,03783 0,04075 0,04366 0,04658 0,04949 0,05241 87 47 1,07237 1,07864 1,08496 1,09131 1,09770 1,10414 1,11061 42
13 0,05241 0,05533 0,05824 0,06116 0,06408 0,06700 0,06993 86 48 1,11061 1,11713 1,12369 1,13029 1,13694 1,14363 1,15037 41
14 0,06993 0,07285 0,07578 0,07870 0,08163 0,08456 0,08749 85 49 1,15037 1,15715 1,16398 1,17085 1,17777 1,18474 1,19175 40
15 0,08749 0,09042 0,09335 0,09629 0,09923 0,10216 0,10510 84 50 1,19175 1,19882 1,20593 1,21310 1,22031 1,22758 1,23490 39
16 0,10510 0,10805 0,11099 0,11394 0,11688 0,11983 0,12278 83 51 1,23490 1,24227 1,24969 1,25717 1,26471 1,27230 1,27994 38
17 0,12278 0,12574 0,12869 0,13165 0,13461 0,13758 0,14054 82 52 1,27994 1,28764 1,29541 1,30323 1,31110 1,31904 1,32704 37
18 0,14054 0,14351 0,14648 0,14945 0,15243 0,15540 0,15838 81 53 1,32704 1,33511 1,34323 1,35142 1,35968 1,36800 1,37638 36
19 0,15838 0,16137 0,16435 0,16734 0,17033 0,17333 0,17633 80 54 1,37638 1,38484 1,39336 1,40195 1,41061 1,41934 1,42815 35
10 0,17633 0,17933 0,18233 0,18534 0,18835 0,19136 0,19438 79 55 1,42815 1,43703 1,44598 1,45501 1,46411 1,47330 1,48256 34
11 0,19438 0,19740 0,20042 0,20345 0,20648 0,20952 0,21256 78 56 1,48256 1,49190 1,50133 1,51084 1,52043 1,53010 1,53986 33
12 0,21256 0,21560 0,21864 0,22169 0,22475 0,22781 0,23087 77 57 1,53986 1,54972 1,55966 1,56969 1,57981 1,59002 1,60033 32
13 0,23087 0,23393 0,23700 0,24008 0,24316 0,24624 0,24933 76 58 1,60033 1,61074 1,62125 1,63185 1,64256 1,65337 1,66428 31
14 0,24933 0,25242 0,25552 0,25862 0,26172 0,26483 0,26795 75 59 1,66428 1,67530 1,68643 1,69766 1,70901 1,72047 1,73205 30
15 0,26795 0,27107 0,27419 0,27732 0,28046 0,28360 0,28675 74 60 1,73205 1,74375 1,75556 1,76749 1,77955 1,79174 1,80405 29
16 0,28675 0,28990 0,29305 0,29621 0,29938 0,30255 0,30573 73 61 1,80405 1,81649 1,82906 1,84177 1,85462 1,86760 1,88073 28
17 0,30573 0,20891 0,31210 0,31530 0,31850 0,62171 0,32492 72 62 1,88073 1,89400 1,90741 1,92098 1,93470 1,94858 1,96261 27
U. D. 2 . - M E D I D A ,

18 0,32492 0,32814 0,33136 0,33460 0,33783 0,34108 0,34433 71 63 1,96261 1,97681 1,99116 2,00569 2,02039 2,03526 2,05030 26
19 0,34433 0,34758 0,35085 0,35412 0,35740 0,36068 0,36397 70 64 2,05030 2,06553 2,08094 2,09654 2,11233 2,12832 2,14451 25
20 0,36397 0,36727 0,37057 0,37388 0,37720 0,38053 0,38386 69 65 2,14451 2,16090 2,17749 2,19430 2,21132 2,22857 2,24604 24
21 0,38386 0,38721 0,39055 0,39391 0,39727 0,40065 0,40403 68 66 2,24604 2,26374 2,28167 2,29984 2,31826 2,33693 2,35585 23
22 0,40403 0,40741 0,41081 0,41421 0,41763 0,42105 0,42447 67 67 2,35585 2,37504 2,39449 2,41421 2,43422 2,45451 2,47509 22
23 0,42447 0,42737 0,43136 0,43481 0,43828 0,44175 0,44523 66 68 2,47509 2,49597 2,51715 2,53865 2,56046 2,58261 2,60509 21

-Pág.49-
24 0,44523 0,44872 0,45222 0,45573 0,45924 0,46277 0,46631 65 69 2,60509 2,62791 2,65109 2,67462 2,69853 2,72281 2,74748 20
25 0,46631 0,46985 0,47341 0,47698 0,48055 0,48414 0,48773 64 70 2,74748 2,77254 2,79802 2,82391 2,85023 2,87700 2,90421 19

TAMadrid
26 0,48773 0,49134 0,49495 0,49858 0,50222 0,50587 0,50953 63 71 2,90421 2,93189 2,96004 2,98868 3,01783 3,04749 3,07768 18
27 0,50953 0,51319 0,51688 0,52057 0,52427 0,52798 0,53171 62 72 3,07768 3,10842 3,13972 3,17159 3,20406 3,23714 3,27085 17
28 0,53171 0,53545 0,53920 0,54296 0,54673 0,55051 0,55431 61 73 3,27085 3,30521 3,34023 3,37594 3,41236 3,44951 3,48741 16
29 0,55431 0,55812 0,56194 0,56577 0,56962 0,57348 0,57735 60 74 3,48741 3,52609 3,56557 3,60588 3,64705 3,68909 3,73205 15
30 0,57735 0,58124 0,58513 0,58905 0,59297 0,59691 0,60086 59 75 3,73205 3,77595 3,82083 3,86671 3,91364 3,96165 4,01078 14
31 0,60086 0,60483 0,60881 0,61280 0,61681 0,62083 0,52487 58 76 4,01078 4,06107 4,11256 4,16530 4,21933 4,27471 4,33148 13
32 0,52487 0,62892 0,63299 0,63707 0,64117 0,64528 0,64941 57 77 4,33148 4,38969 4,44942 4,51071 4,57363 4,63825 4,70463 12
33 0,54941 0,65355 0,65771 0,66189 0,66608 0,67028 0,67451 56 78 4,70163 4,77286 4,84300 4,91516 4,98940 5,06584 5,14455 11
34 0,67451 0,67875 0,68301 0,68728 0,69157 0,69588 0,70021 55 79 5,14455 5,22566 5,30928 5,39552 5,48451 5,57638 5,67128 10
35 0,70021 0,70455 0,70891 0,71329 0,71769 0,72211 0,72654 54 81 5,67128 5,76937 5,87080 5,97576 6,08444 6,19703 6,31375 19
36 0,72654 0,73100 0,73547 0,73996 0,74447 0,74900 0,75355 53 81 6,31375 6,43484 6,56055 6,69116 6,82694 6,96823 7,11537 18
37 0,75355 0,75812 0,76272 0,76733 0,77196 0,77661 0,78129 52 82 7,11537 7,26873 7,42871 7,59575 7,77035 7,95302 8,14435 17
38 0,78129 0,78598 0,79070 0,79544 0,80020 0,80498 0,80978 51 86 8,14435 8,34496 8,55555 8,77689 9,00983 9,25530 9,51436 16
39 0,80978 0,81461 0,81946 0,82434 0,82923 0,83415 0,83910 50 84 9,51436 9,78817 10,0780 10,3854 10,7119 11,0594 11,4301 15
40 0,83910 0,84407 0,84906 0,85408 0,85912 0,86419 0,86929 49 85 11,4301 11,8262 12,2505 12,7062 13,1969 13,7267 14,3007 14
41 0,86929 0,87441 0,87955 0,88473 0,88992 0,89515 0,90040 48 86 14,3007 14,9244 15,6048 16,3499 17,1693 18,0750 19,0811 13
42 0,90040 0,90569 0,91099 0,91633 0,92170 0,92709 0,93252 47 87 19,0811 20,2056 21,4704 22,9038 24,5418 26,4316 28,6363 12
43 0,93252 0,93797 0,94345 0,94896 0,95451 0,96008 0,96569 46 88 28,6363 31,2416 34,3678 38,1885 42,9641 49,1039 57,2900 11
44 0,96569 0,97133 0,97700 0,98270 0,98843 0,99420 1,00000 45 89 57,2900 68,7501 85,9398 114,589 171,885 343,774 ' 10
60’ 50’ 40’ 30’ 20’ 10’ 0’ Grados 60’ 50’ 40’ 30’ 20’ 10’ 0’ Grados
E S T I M AC I Ó N Y C Á L C U L O D E M AG N I T U D E S

COTANGENTES (45o - 90o) COTANGENTES (0o - 45o)

21
M AT E M Á T I C A S

5. INSTRUMENTOS DE MEDIDA
Los instrumentos de medida son los aparatos que se utilizan para realizar prácticamen-
te las distintas mediciones.

Se pueden clasificar en tres grandes grupos, según la clase de medición que realicen,
estos son:
– Instrumentos de medida propiamente dichos.
– Instrumentos comparadores.
– Instrumentos verificadores.

5.1. INSTRUMENTOS UTILIZADOS EN LA MEDIDA


DE LONGITUDES
Los instrumentos utilizados para medir longitudes, son aquellos que sirven para leer
directamente la dimensión comprendida entre dos caras, generatrices o puntos.
Los más utilizados normalmente son: Flexómetros, reglas graduadas, calibres pie de rey
y micrómetros.

5.1.1. FLEXÓMETROS
Llamados también comúnmente “metros” a secas. Están formados por una cinta de
acero graduada en centímetros y milímetros, que se enrolla dentro de una caja. Se
construyen en longitudes de uno a cinco metros y son muy utilizados y prácticos hasta para
medir contornos curvilíneos.

5.1.2. REGLAS GRADUADAS


Se construyen de hoja de acero principalmente, graduadas en milímetros y medios
milímetros. Su longitud varía entre 10 cm y 500 cm normalmente.
Se utiliza para realizar medidas de menor longitud que las realizadas con los flexóme-
tros y además con mayor precisión.

Regla graduada

22 TAMadrid

-Pág.50-
U. D. 2 . - M E D I D A , E S T I M AC I Ó N Y C Á L C U L O D E M AG N I T U D E S

5.1.3. CALIBRES PIE DE REY


Constan de una regla de acero, cuyas caras y cantos son perfectamente planas y
perpendiculares entre sí, y se encuentra graduada en centímetros y milímetros por un
lado. Por el otro lado puede estar graduada en unidades inglesas, o sea en pulgadas.
Perpendicularmente a esta regla va montado un brazo denominado boca fija. Por último,
lleva una pieza denominada corredera o boca móvil, que se desplaza por rozamiento sobre
la regla.
Esta boca móvil puede inmovilizarse en cualquier punto de la regla, mediante un torni-
llo de presión. El desplazamiento de esta boca se realiza por medio de un pulsador solida-
rio a la corredera.
Para realizar mediciones más precisas que una regla normal, la boca móvil lleva un sis-
tema llamado nonius. Según las diferentes divisiones del nonius, obtenemos mayor o menor
apreciación en el calibre, pudiéndose llegar a apreciaciones del orden de 1/50 = 0,02 mm
de apreciación.
Existen muchos tipos de calibres, el que vemos en la siguiente figura es el llamado cali-
bre ordinario, pero también existen: calibres de interiores, calibres con bocas curvas, cali-
bres con bocas puntiagudas, calibres con tornillo de aproximación, calibres de profundidad,
etc, cada uno de ellos están adaptados para realizar medidas más o menos complejas.

Calibre pie de rey

5.1.4. MICRÓMETROS
También llamados Palmer en honor a su inventor. Consta de un tornillo principal
de paso conocido, que lleva una cabeza graduada. Este tornillo engrana con una tuer-
ca que ni se desplaza ni gira, y esto hace que el tornillo avance o retroceda, siguiendo
el eje de dos tetones, uno solidario al tornillo y otro a la tuerca, por medio de un cuer-
po en forma de “U”, de acero.
Los micrómetros abarcan medidas de 25 en 25 mm, o sea, hay un micrómetro que mide
de 0 a 25 mm, otro que mide de 25 a 50 mm, otro de 50 a 75 mm y etc. Estos son más pre-
cisos que los calibres, pudiendo realizar medidas con una aproximación de 0,001 mm.
Existen diversos tipos de micrómetros, el
representado en la figura es el micrómetro
ordinario o de exteriores, pero también hay
micrómetros para interiores, de profundidad,
de bocas anchas, etc.
Micrómetro de exteriores

TAMadrid
23
-Pág.51-
M AT E M Á T I C A S

5.2. OTROS INSTRUMENTOS DE MEDIDA


Existe en el mercado infinidad de instrumentos de medida que
no vamos a desarrollar en esta unidad elemental, pero si al menos
vamos a mencionarlos, como pueden ser: Aparatos comparadores,
calibres fijos de verificación, escuadras, goniómetros (para el con-
trol de ángulos), microscopios, niveles, máquinas de medir por coor-
denadas, polímetro digital (para medir parámetros eléctricos), etc.

RESUMEN Polímetro digital

— Hoy en día, la medición de magnitudes se basa en el Sistema Internacional (SI), sis-


tema aceptado por todos los países industrializados. La mayoría de estas magnitu-
des toman como base el Sistema Métrico Decimal, o sea, sus unidades son
potencias de 10.
— La unidad de longitud es el metro, de superficie es el metro cuadrado, de volumen
es el metro cúbico, de capacidad es el litro que equivale a un decímetro cúbico, y
de peso es el gramo. Todas estas unidades tienen múltiplos y submúltiplos y ade-
más, utilizan el Sistema Métrico Decimal.
— La unidad de tiempo es el segundo y sus principales múltiplos son: Minuto, hora,
día, semana, año civil, bienio, trienio, lustro, decenio, siglo y milenio. Esta unidad
no utiliza el Sistema Métrico Decimal, pero esta comúnmente aceptada en todo el
mundo.
— Para la medida de los ángulos se utilizan dos tipos de magnitudes, que son: Medi-
das centesimales, cuya unidad es el grado centesimal y sus submúltiplos si son
potencias de 10, y medidas sexagesimales, más usadas que las anteriores, cuya uni-
dad es el grado sexagesimal y sus submúltiplos minuto y segundo sexagesimal, no
se forman con potencias de 10.
— Prácticamente, todas la medidas que podemos realizar llevan implícito un error en
su medición. Los errores pueden ser absolutos y relativos.
— La trigonometría es una parte importante de las Matemáticas, se encarga de estudiar las
relaciones existentes entre los lados y los ángulos de un triángulo rectángulo. Las prin-
cipales razones trigonométricas son: seno, coseno y tangente de un ángulo.
— Los instrumentos de medida más utilizados son los que miden longitudes y entre
ellos podemos destacar: Flexómetros, reglas graduadas, calibre pie de rey y micró-
metros.

24 TAMadrid

-Pág.52-
U. D. 2 . - M E D I D A , E S T I M AC I Ó N Y C Á L C U L O D E M AG N I T U D E S

EJERCICIOS DE AUTOCOMPROBACIÓN

11. ¿Cuántos metros son 4 km, 5 hm y 8 dam?


A. 5480 m B. 458 m C. 0, 4580 m D. 4580 m

12. ¿Cuántos litros tiene un hm3?


A. 1000000 litros B. 1000000000 litros
C. 1000 litros D. 0,0001 litros

13. Una bicicleta pesa 9 kg, 6 hg y 7 dag. ¿Cuántos gramos pesa la bicicleta?
A. 9670 g B. 967 g
C. 96700 g D. 9,67 g

14. Un atleta tardó 17700 segundos en realizar un recorrido de un maratón. Halla las horas y minu-
tos que empleó en dicho recorrido.
A. 4 horas y 5 min B. 4 horas y 55 min C. 17 horas y 55 min D. 2 horas y 80 min

15. ¿Qué expresión de tiempo equivale a 43800 horas?


A. Bienio B. Trienio C. Lustro D. Decenio

16. Pasar 225418" a grados, minutos y segundos.


A. 62,36o B. 36o 62' 58" C. 62o 36'58" D. 58o 36' 55"

17. Un pentágono regular esta inscrito dentro de una circunferencia de 5 m de radio. ¿Qué superficie ocupa?
A. 60,25 m2 B. 51,45 m2 C. 25 m2 D. 59,45 m2

18. Un bidón de gasolina tiene forma de cilindro; su diámetro es de 1 m y su altura es el triple


de su radio. Calcula el volumen en litros que puede almacenar.
A. 1177,5 litros B. 500 litros C. 117,80 litros D. 780,17 litros

19. Si en un triángulo rectángulo el cateto opuesto vale 2 dm y el cateto contiguo al ángulo _ 4 dm,
calcula el valor del ángulo _.
A. 60o B. 30o 50' C. 26o 30’ D. 0,5o

10. Calcula el sen _, sabiendo que la tg _ = 0,782.


A. 0,738 B. 0,615 C. 0,250 D. 1,534

TAMadrid
25
-Pág.53-
M AT E M Á T I C A S

RESPUESTAS A LOS EJERCICIOS

1. D
2. B
3. A
4. B
5. C
6. C
7. D
8. A
9. C
10. B

26 TAMadrid

-Pág.54-
U. D. 2 . - M E D I D A , NOTAS
E S T I M AC I Ó N Y C Á L C U L O D E M AG N I T U D E S

-Pág.55-
U. D. 2 . - M E D I D A , NOTAS
E S T I M AC I Ó N Y C Á L C U L O D E M AG N I T U D E S

-Pág.54-
-Pág.55-
portada TROPA 19/3/07 19:51 Página 1

FUERZAS ARMADAS
PROFESIONALES
CURSO DE APOYO
A LA PREPARACIÓN
DE LAS PRUEBAS DE ACCESO
A UNA RELACIÓN DE SERVICIOS
DE CARÁCTER PERMANENTE

MATEMÁTICAS
2ª parte
Unidades didácticas 3 y 4

DIGEREM

MINISTERIO
DE DEFENSA
FUERZAS ARMADAS SUBDIRECCIîN GENERAL
DE TROPA Y MARINERIA
PROFESIONAL
PROFESIONALES
CURSO DE APOYO
A LA PREPARACIÓN
DE LAS PRUEBAS DE ACCESO
A UNA RELACIÓN DE SERVICIOS
DE CARÁCTER PERMANENTE

MATEMÁTICAS
2ª parte
Unidades didácticas 3 y 4
La Ley 8/2006 de Tropa y Marinería, en su artículo 16,1, establece que “la formación
en las Fuerzas Armadas garantizará que los militares profesionales de tropa y
marinería puedan adquirir, actualizar o ampliar sus conocimientos para un mayor
desarrollo personal y profesional”. En cumplimiento de este mandato, el Ministerio
de Defensa edita el presente material didáctico para facilitar a los militares
profesionales de tropa y marinería, alumnos de los cursos de formación
presencial que se imparten a través de la Dirección General de Reclutamiento y
Enseñanza Militar, los apoyos necesarios para preparación de dichos cursos, que
permitirán, siempre que superen las pruebas correspondientes, la obtención de la
titulación de graduado en Educación Secundaria, acreditación para el acceso a
los ciclos formativos de la Formación Profesional de grado medio o de grado
superior, acceso a las Escalas de Suboficiales, Tropa Permanente, Guardia Civil
y Policía Nacional.

CATÁLOGO GENERAL DE PUBLICACIONES


http://www.060.es

Edita:

© Autor y editor
NIPO: 076-10-204-9 NIPO: 076-10-205-4 (edición en línea)
Depósito Legal: M-32363-2009
Diseño y programación: cimapress
Tirada: 1300 ejemplares
Fecha de edición: septiembre, 2010

Prohibida la reproducción total o parcial de esta obra, por cualquier medio sin autorización escrita del editor
MATEMÁTICAS
2ª parte

SUMARIO

Unidad didáctica Pág.

3. REPRESENTACIÓN Y ORGANIZACIÓN 5
EN EL ESPACIO

4. INTERPRETACIÓN, REPRESENTACIÓN Y 33
TRATAMIENTO DE LA INFORMACIÓN
U . D . 3 . - R E S P R E S E N TA C I Ó N Y O R G A N I Z A C I Ó N E N E L E S PA C I O

ÍNDICE
OBJETIVOS . . . . . . . . . . . . . . . . . . . . . . . . . . . . . . . . . . . . . . . . . . . . . . . . . . . . . . . . . . . . . 2
INTRODUCCIÓN . . . . . . . . . . . . . . . . . . . . . . . . . . . . . . . . . . . . . . . . . . . . . . . . . . . . . . . . . . 3
MAPA CONCEPTUAL. . . . . . . . . . . . . . . . . . . . . . . . . . . . . . . . . . . . . . . . . . . . . . . . . . . . . . 4
DESARROLLO DE CONTENIDOS
1. ELEMENTOS GEOMÉTRICOS EN EL PLANO Y EN EL ESPACIO . . . . . . . 5
2. SISTEMAS DE REFERENCIA . . . . . . . . . . . . . . . . . . . . . . . . . . . . . . . . . . . . . . . . 8
2.1 COORDENADAS CARTESIANAS EN EL PLANO Y EN EL ESPACIO. . . . . . . . . . . 8
2.2 COORDENADAS EN LA SUPERFICIE ESFÉRICA: LONGITUD Y LATITUD . . . . 9
3. FIGURAS Y CUERPOS . . . . . . . . . . . . . . . . . . . . . . . . . . . . . . . . . . . . . . . . . . . . . . 9
3.1 CLASIFICACIÓN DE FIGURAS Y CUERPOS ATENDIENDO A DIVERSOS
CRITERIOS . . . . . . . . . . . . . . . . . . . . . . . . . . . . . . . . . . . . . . . . . . . . . . . . . . . . . . . 9
3.2 ELEMENTOS CARACTERÍSTICOS DE POLÍGONOS Y CÓNICAS . . . . . . . . . . . . 11
3.3 ELEMENTOS CARACTERÍSTICOS DE POLIEDROS Y CUERPOS REDONDOS. 15
3.4 RELACIONES DE INSCRIPCIÓN, DESCOMPOSICIÓN E INSERCIÓN DE
FIGURAS Y CUERPOS . . . . . . . . . . . . . . . . . . . . . . . . . . . . . . . . . . . . . . . . . . . . . . . . . 17
3.5 REGULARIDAD Y SIMETRÍAS EN FIGURAS, CUERPOS Y
CONFIGURACIONES GEOMÉTRICAS. . . . . . . . . . . . . . . . . . . . . . . . . . . . . . . . . . . . 19
3.6 UTILIDAD E IMPORTANCIA DE ALGUNAS FIGURAS Y CUERPOS PARA
PROPÓSITOS CONCRETOS . . . . . . . . . . . . . . . . . . . . . . . . . . . . . . . . . . . . . . . . . . . . . 20
4. FIGURAS SEMEJANTES: LA REPRESENTACIÓN A ESCALA . . . . . . . . . . 20
4.1 REPRESENTACIONES MANEJABLES DE LA REALIDAD:
PLANOS, MAPAS Y MAQUETAS . . . . . . . . . . . . . . . . . . . . . . . . . . . . . . . . . . . . . . . . 20
4.2 CARACTERÍSTICAS DE LAS DOS FORMAS IGUALES:
IGUALDAD DE ÁNGULOS Y PROPORCIONALIDAD DE LONGITUDES . . . . . . 21
4.3 EL TEOREMA DE TALES. . . . . . . . . . . . . . . . . . . . . . . . . . . . . . . . . . . . . . . . . . . . . . . 22
4.4 RELACIÓN ENTRE EL ÁREA Y EL VOLUMEN DE FIGURAS SEMEJANTES . . 22
5. TRANSFORMACIONES ISOMÉTRICAS . . . . . . . . . . . . . . . . . . . . . . . . . . . . . 22
5.1 TRASLACIONES, GIROS Y SIMETRÍAS . . . . . . . . . . . . . . . . . . . . . . . . . . . . . . . . . . 23
5.2 PROPIEDADES QUE SE CONSERVAN CON LAS TRANSFORMACIONES. . . . . . . 23
5.3 COMPOSICIÓN DE TRANSFORMACIONES EN CASOS SENCILLOS . . . . . . . . . 24
RESUMEN . . . . . . . . . . . . . . . . . . . . . . . . . . . . . . . . . . . . . . . . . . . . . . . . . . . . . . . . . . . . 25
EJERCICIOS DE AUTOCOMPROBACIÓN . . . . . . . . . . . . . . . . . . . . . . . . . . . . . . . . . . 26
RESPUESTAS A LOS EJERCICIOS . . . . . . . . . . . . . . . . . . . . . . . . . . . . . . . . . . . . . . . . . 28

-Pág.5-
M AT E M AT I C A S

OBJETIVOS
Al finalizar el estudio de esta Unidad Didáctica, el alumno será capaz de:

• Descubrir e interpretar la idea de lo que es la geometría.

• Distinguir y representar los diferentes cuerpos y figuras en el plano y en el espacio.

• Percibir y analizar los movimientos en el plano y en el espacio.

• Diferenciar los sistemas de referencia.

• Identificar las figuras y cuerpos más importantes y sus relaciones de semejanza.

• Diferenciar traslación, giro y simetría.

-Pág.6-
U . D . 3 . - R E S P R E S E N TA C I Ó N Y O R G A N I Z A C I Ó N E N E L E S PA C I O

INTRODUCCIÓN
L os griegos consideraban la Geometría como la ciencia que enseñaba al hombre a
razonar. Su estudio no debía hacerse solo con fines prácticos, sino para el honor de
la mente humana.

En el siglo XVI Galileo afirmó que era tan importante entender la lengua hablada como
conocer el lenguaje matemático, los caracteres de los triángulos, círculos y otras figuras
geométricas.

En nuestros días la arquitectura y la geometría están muy unidas.

-Pág.7-
M AT E M AT I C A S

M A PA C O N C E P T UA L
PUNTO

ELEMENTOS
GEOMÉTRICOS RECTA

PLANO

POLÍGONOS
FIGURAS
PLANAS

CÓNICAS

GEOMETRÍA

POLIEDROS
CUERPOS
GEOMÉTRICOS CUERPOS
REDONDOS

PROPORCIONALIDADES TEOREMA
DE SEGMENTOS DE TALES

4
-Pág.8-
U . D . 3 . - R E S P R E S E N TA C I Ó N Y O R G A N I Z A C I Ó N E N E L E S PA C I O

1. REPRESENTACIÓN Y ORGANIZACIÓN
EN EL ESPACIO

1. ELEMENTOS GEOMÉTRICOS EN EL PLANO Y EN EL ESPACIO


La Geometría plana estudia las figuras que están contenidas en un plano, y la Geometría del
espacio estudia las figuras que no están contenidas en un plano. El espacio se caracteriza por
tener tres dimensiones.
Los elementos geométricos en el plano y en el espacio son los siguientes:
EL PLANO
Se caracteriza por tener dos dimensiones, es ilimitado y se representa mediante un
paralelogramo y se simboliza con letras griegas.

Plano

Se denomina semiplano a cada una de las partes en que queda dividido un plano por una
cualquiera de sus rectas.

Determinación de un plano:

– Un plano contiene infinitas rectas.


– Por una recta pasan infinitos planos.
– Una recta y un punto exterior a ella determinan un plano.
– Tres puntos no alineados determinan un plano.
– Dos rectas paralelas determinan un plano.
– Dos rectas que se cortan determinan un plano.

Posiciones relativas de dos planos:


– Dos planos pueden ser: coincidentes, si tienen todos los puntos comunes, serán
secantes, si tienen una recta común y serán paralelos sin no tienen ningún punto
común.

5
-Pág.9-
M AT E M AT I C A S

Ángulos en el plano:
- Dos rectas de un plano que se cortan en un punto determinan cuatro regiones en dicho
plano, cada una de estas regiones es un ángulo.
Se recuerda:
Cada una de las semirectas que limitan esta región es un lado del ángulo y el origen
común de las dos semirectas, el vértice del ángulo. Un ángulo es agudo si su amplitud
es menor que la de un recto; y es obtuso si su amplitud es mayor que la de un recto.

Ángulos en el espacio:
– A la región del espacio limitada por dos semiplanos que tiene una recta común se le
denomina ángulo diedro. Cada semiplano es una cara del diedro y la recta común,
su arista.
– A la región del espacio comprendida entre tres o más planos que tienen únicamente
un punto en común se le denomina ángulo poliedro. El punto común es el vértice y
las dos caras consecutivas forman un ángulo diedro, además, los lados comunes a
dos de las caras son las aristas.

EL PUNTO
Se caracteriza porque no tiene dimensiones y se representa por dos pequeños trazos que se
cortan o por un pequeño circulo y se designa por una letra mayúscula.

representaciones del punto


A B

LA RECTA
Tiene solo una dimensión y es ilimitada, por lo que solo representamos parte de ella y se
designa por una letra minúscula.
r representación de una recta

6
-Pág.10-
U . D . 3 . - R E S P R E S E N TA C I Ó N Y O R G A N I Z A C I Ó N E N E L E S PA C I O

Si en una recta se marca un punto, la recta queda dividida en dos partes, a cada una de estas
partes se le denomina semirrecta, ésta no tiene principio ni fin, si no se considera el punto como
origen o como final.
Si en una recta se marcan dos puntos, a la porción de la recta comprendida entre estos dos
puntos se le denomina segmento.

Determinación de rectas:
– Dos puntos determinan una recta.
– Por un punto pasan infinitas rectas.
– Una recta contiene infinitos puntos.

Posiciones relativas de dos rectas en el plano:


– Dos rectas situadas en el mismo plano son coincidentes (si tienen todos los puntos
comunes), son secantes si tienen un solo punto común y son paralelas si no tienen
ningún punto común.

Posiciones relativas de dos rectas en el espacio:


– Dos rectas se cruzan si no tienen ningún punto en común y no existe ningún plano
que las contenga.

Posiciones relativas de recta y plano:


– Una recta ésta contenida en el plano, si todos sus puntos son del plano; es paralela
al plano si no tienen ningún punto en común, y corta o es secante al plano si tienen
un solo punto en común.

Rectas perpendiculares:
– Son aquellas que se cortan formando cuatro ángulos iguales.
– Por un punto de una recta pasa una perpendicular a dicha recta y solo una.
– Por un punto exterior a una recta pasa una perpendicular a dicha recta y solo una.

Cuándo una recta es perpendicular a un plano:


– Una recta es perpendicular a un plano si lo es a todas las rectas del plano que pasan
por su pie.
– Si una recta r es perpendicular a dos rectas que pasan por uno de sus puntos, también
lo es al plano determinado por estas rectas.

7
-Pág.11-
M AT E M AT I C A S

– Dado un plano α y un punto cualquiera P, existe una única recta que pasa por P y es
perpendicular a α.
– Dados una recta r y un punto cualquiera P, existe un único plano que pasa por P y es
perpendicular a r.
Rectas paralelas:
– Dos rectas son paralelas cuando, hallándose situadas en el mismo plano, no tienen
ningún punto común.

Recta y plano paralelos:


– Una recta y un plano son paralelos cuando no tienen ningún punto común.

2. SISTEMAS DE REFERENCIA

2.1. COORDENADAS CARTESIANAS EN EL PLANO


Y EN EL ESPACIO
– Se denomina sistema de Coordenadas Cartesianas en el plano al conjunto formado
por dos rectas perpendiculares entre sí, que sirven para determinar cualquier punto del
plano en el que se consideren.

Sus elementos son:

– Ejes de coordenadas: son las rectas perpendiculares, diferenciándose el eje de


abscisas que es el eje horizontal y se representa por X, y el eje de ordenadas que es
el eje vertical y se representa por Y.

– Origen de coordenadas: es el punto donde se cortan los ejes y se representa por O.


– Tres rectas graduadas, perpendiculares dos a dos y con un punto común, forman un
sistema de Coordenadas Cartesianas en el espacio. Éste sirve para determinar la
posición exacta de puntos en el espacio.

8
-Pág.12-
U . D . 3 . - R E S P R E S E N TA C I Ó N Y O R G A N I Z A C I Ó N E N E L E S PA C I O

2.2. COORDENADAS EN LA SUPERFICIE ESFÉRICA:


LONGITUD Y LATITUD
– La esfera es el volumen que encierra una superficie esférica. Una superficie esférica es
el lugar geométrico de los puntos del espacio que se encuentran a la misma distancia de un
punto fijo, que llamamos centro de la esfera.
– Se denomina Sistema de coordenadas en una superficie terrestre (esfera) al conjunto
formado por dos líneas de referencia, que sirven para determinar la posición de cualquier punto
de la superficie terrestre.
– Las líneas de referencia son: la circunferencia ecuatorial que realiza la función del eje
de abscisas en el sistema de coordenadas cartesianas, y el meridiano (son las circunferencias
máximas que pasan por los polos) que pasa por Greenwich, que actúa de forma equivalente al
eje de ordenadas.
– En la circunferencia ecuatorial, se toma la longitud del punto , que es el ángulo formado
por el meridiano que pasa por un punto y el meridiano de referencia. Las longitudes se miden
de 0 a 180 grados sexagesimales hacia el Este o hacia el Oeste, lo que se simboliza mediante
las letras E y O.
– En el meridiano de referencia se toma la latitud del punto, que es el ángulo formado por
el radio terrestre que pasa por el punto y el plano ecuatorial. Las latitudes se miden de 0 a 90
grados sexagesimales a partir del Ecuador, y pueden ser hacia el Norte o hacia el Sur, lo que se
indica mediante las letras N y S.

3. FIGURAS Y CUERPOS

3.1. CLASIFICACIÓN DE FIGURAS Y CUERPOS ATENDIENDO


A DIVERSOS CRITERIOS
Las figuras planas se clasifican según su contorno en:

Polígonos
Son las superficies planas que se obtienen al unir, por medio de segmentos, n puntos del
plano.

9
-Pág.13-
M AT E M AT I C A S

Según el numero de lados, pueden ser:


Triángulo: 3 lados

Cuadrado: 4 lados

Pentágono: 5 lados

Hexágono: 6 lados

Heptágono: 7 lados

Octógono: 8 lados

Pueden ser:
Convexos: si ninguna recta los puede cortar en más de dos puntos o todos los ángulos
interiores son convexos (<180º)
Cóncavos: cuando alguna recta los puede cortar en más de dos puntos o algún ángulo
interior es cóncavo ( >180º)
Pueden ser:
Regulares: cuando tienen todos sus lados y sus ángulos interiores iguales.
Irregulares: cuando falta alguna condición de regularidad

Cónicas
Son las líneas que resultan de la intersección de una superficie cónica y un plano.
Se clasifican en:
Circunferencia: es el lugar geométrico (conjunto de puntos de un plano que cumplen
una condición y esta puede expresarse geométricamente o algebraicamente) de los
puntos del plano P(x,y), que están a la misma distancia de un punto llamado centro,
C(a,b). La distancia de los puntos de la circunferencia al centro se llama radio de la
circunferencia.

10
-Pág.14-
U . D . 3 . - R E S P R E S E N TA C I Ó N Y O R G A N I Z A C I Ó N E N E L E S PA C I O

Elipse: es el lugar geométrico de los puntos del plano, P(x,y), cuya suma de distancias
a dos puntos fijos, F y F´, llamados focos, es constante e igual al eje mayor de la elipse,
2a.
Hipérbola: es el lugar geométrico de los puntos del plano, P(x.y), cuya diferencia de
distancias a dos puntos llamados focos, F y F´, es constante e igual al eje transverso,2a.
Parábola: es el lugar geométrico de los puntos del plano cuyas distancias a un punto
fijo, llamado foco, y a una recta fija, llamada directriz, son iguales.

Los cuerpos según sus caras, se clasifican en:

Poliedros (caras planas)


Poliedros regulares: son aquellos cuyas caras son polígonos regulares y en cada vértice
concurren el mismo número de caras.
Prismas: son poliedros cuyas bases, paralelas entre sí, son dos polígonos iguales y sus
caras laterales son paralelogramos.
Pirámides: son poliedros que tienen por base un polígono y sus caras laterales son
triángulos que concurren en un vértice.

Cuerpos redondos (caras curvas)


Cilindro: es el cuerpo geométrico que se obtiene al girar un rectángulo alrededor de un
lado.
Cono: es el cuerpo geométrico que se obtiene al girar un triángulo rectángulo alrededor
de un cateto.
Esfera: es el cuerpo geométrico que se obtiene al girar un semicírculo alrededor de su
diámetro.

3.2. ELEMENTOS CARACTERÍSTICOS DE POLÍGONOS Y CÓNICAS


Elementos característicos de polígonos:
A. Vértices (letras mayúsculas)
B. Lados (letras minúsculas)
C. Diagonales
D. Ángulos interiores
E. Ángulos exteriores

11
-Pág.15-
M AT E M AT I C A S

En los polígonos regulares además existen otros elementos:


– Centro (O): punto interior que se halla a igual distancia de todos los vértices y de
todos los lados.
– Apotema (a): es el segmento que une el centro con el punto medio de cada lado. Es
perpendicular al lado correspondiente.
– Radio (r ): es el segmento que une el centro con cualquier vértice.
– Angulo Central: es el ángulo formado por dos radios consecutivos y su valor está en
función del numero n de lados
C = 360º / n
– Angulo interior: como todos los ángulos interiores de un polígono regular son
iguales cada uno de ellos, vale:
I = 180º x (n-2) / n

Elementos característicos de la circunferencia


Una circunferencia es el lugar geométrico de los puntos del plano que se encuentran a una
distancia fija llamada radio, de un punto dado, llamado centro.

Elementos característicos de la elipse


Se llama elipse al lugar geométrico de los puntos tales que la suma de sus distancias a dos
puntos fijos, llamados focos, es una constante.
La línea que une los dos focos se llama eje principal de la elipse y la mediatriz de los
mismos eje secundario.
Se llaman vértices de la elipse a los puntos donde ésta corta a sus ejes.
El punto medio de los dos focos se llama centro de la elipse y la distancia entre ellos se llama
distancia focal.
Generalmente el eje principal se representa por 2a y la distancia focal por 2c. Los valores a
y c se llaman semieje principal y semidistancia focal, respectivamente.

12
-Pág.16-
U . D . 3 . - R E S P R E S E N TA C I Ó N Y O R G A N I Z A C I Ó N E N E L E S PA C I O

B(0,b)

A(–a,0) F(–c,0) F’(c,0) A’(a,0)

B’(0,–b)

Segmento A,A’ se denomina eje principal y su longitud es igual a 2a


Segmento B,B’ se denomina eje secundario y su longitud es igual a 2b
Segmento F, F’ se denomina distancia focal y su longitud es igual a 2c

Relación fundamental de la elipse


Se denomina relación fundamental de la elipse a la ecuación:
a2 = b2 + c2
Donde a es igual a la mitad del eje mayor de la elipse; b es la mitad del eje secundario y c
es la mitad de la distancia focal

Excentricidad de la elipse
Se define excentricidad (e) como el cociente entre la semidistancia focal (c) y el semieje
mayor (a).
c
e = ——
a

Elementos característicos de la hipérbola


Se llama hipérbola al lugar geométrico de los puntos del plano tales que la diferencia de sus
distancias a dos puntos fijos, llamados focos, es una constante (se representa por 2c).

13
-Pág.17-
M AT E M AT I C A S

La recta que une los dos focos se llama eje principal o real de la hipérbola (2a) y la
mediatriz se llama eje secundario o imaginario (2b) de la hipérbola.
El punto donde se cortan ambos ejes (que es, evidentemente, el punto medio de los focos)
se llama centro de la hipérbola.
Los puntos donde la hipérbola corta a los ejes (se verá que únicamente corta al eje real) se
llaman vértices de la hipérbola.
Al igual que en la elipse, se llama distancia focal a la distancia entre los dos focos y a las
distancias desde un punto cualquiera de la hipérbola a ambos focos se les llama radios vectores
del punto.

Relación fundamental de la hipérbola


Se denomina relación fundamental de la hipérbola a la ecuación:
c2 = a2 + b2
Donde a es igual a la mitad del eje mayor de la elipse; b es la mitad del eje secundario y c
es la mitad de la distancia focal.

Excentricidad de la hipérbola
Se define excentricidad (e) como el cociente entre la semidistancia focal (c) y el semieje
mayor (a).
c
e = ——
a

Elementos característicos de la parábola


Se llama parábola al lugar geométrico de los puntos del plano que equidistan de un punto
fijo, llamado foco, y de una recta fija llamada directriz.
2

14
-Pág.18-
U . D . 3 . - R E S P R E S E N TA C I Ó N Y O R G A N I Z A C I Ó N E N E L E S PA C I O

La distancia entre el foco y la directriz de una parábola recibe el nombre de parámetro de


la parábola (suele denotarse por p).
Dada una parábola, se llama eje de la misma la recta que contiene al foco y es perpendicular
a la directriz.
Se llama vértice de la parábola al punto donde ésta corta a su eje.
Para simplificar la parábola, se supondrá que el vértice es el origen de coordenadas y que el
foco se encuentra en el semieje positivo de abscisas.

3.3. ELEMENTOS CARACTERÍSTICOS DE POLIEDROS


Y CUERPOS REDONDOS

Elementos característicos de poliedros:


– Poliedros regulares:
- Caras o polígonos que lo limitan
- Vértices o puntos de corte con las aristas
- Aristas o lados de las caras
Los poliedros regulares son: tetraedro, cubo, octaedro, dodecaedro e icosaedro.
– Los prismas poseen los siguientes elementos:
- Caras laterales
Vértice Base
- Bases
- Aristas laterales Arista
Cara lateral
lateral
- Aristas básicas
- Altura
Arista Base
básica

15
-Pág.19-
M AT E M AT I C A S

Los prismas se clasifican según sean:


– Rectos u oblícuos: según que las aristas laterales sean perpendiculares u oblícuas
a las bases.
– Triangulares, cuadrangulares…: según el numero de lados de sus bases.
– Regulares o irregulares: son regulares cuando son rectos y sus bases son
polígonos regulares, e irregulares cuando falta alguna condición de regularidad.
– Paralelepípedos: son prismas cuyas bases son paralelogramos. Tienen la
propiedad de que sus caras son paralelas dos a dos.
– Los elementos más característicos de las pirámides además de los generales de
los poliedros son:
- Altura (h), o distancia del vértice al plano que contiene a la base.
- Apotema lateral (a),es la altura de las caras laterales.
- Apotema de la base (ab), es la altura de la base.
Las pirámides se clasifican en :
– Rectas y oblícuas según que los ángulos diedros que forman las caras laterales
con las bases sean iguales o no.
– Por el número de lados de la base pueden ser triangulares, cuadrangulares o
tetragonales, pentagonales, etc.
– Regulares o irregulares: son regulares cuando la base es un polígono regular y
es recta, y son irregulares cuando falta alguna de las condiciones de regularidad.

Elementos característicos de los cuerpos redondos


– Elementos característicos del cilindro
- Altura, (h) es el segmento que une el centro de las dos bases. Es perpendicular a
ambas bases.
- Radio de la base, (r), es el radio de cada uno de los círculos que forman sus bases.
- Generatriz (g). Es el segmento que genera el cilindro . su medida coincide con la
altura.

Base Superficie
lateral

Generatriz
Base

16
-Pág.20-
U . D . 3 . - R E S P R E S E N TA C I Ó N Y O R G A N I Z A C I Ó N E N E L E S PA C I O

– Elementos característicos del cono


- Altura (h), es el segmento que
une el vértice y el centro de la
base. Es perpendicular a la
Vértice
base.
- Radio (r), es el radio del Generatriz
h
círculo que forma su base.
Base
- Generatriz (g), es el segmento Superficie
Radio lateral
que genera el cono.
Estos tres elementos forman un
triángulo rectángulo.

– Elementos característicos de la esfera


- Radio, R, es el segmento que
une el centro con un punto Centro
cualquiera de la superficie que
limita la esfera.
- Diámetro, d, es el segmento Radio
que une dos puntos de la
superficie esférica pasando por
el centro.
- Paralelo, es la circunferencia que se obtiene al cortar la esfera por un plano que
no pasa por el centro.
- Meridiano, es la circunferencia que se obtiene de cortar la esfera por un plano que
contiene al centro (plano diametral).

3.4. RELACIONES DE INSCRIPCIÓN, DESCOMPOSICIÓN E INSERCIÓN


DE FIGURAS Y CUERPOS

Para estudiar este tipo de relaciones debemos de saber las fórmulas que permiten determinar
las áreas de figuras planas más usuales:
Área del cuadrado: A = l2
Área del rectángulo: A = a · b
Área del triángulo: A = b · h / 2
Área del trapecio: A = (B + b) · h / 2
Área del círculo: A = π · r2

17
-Pág.21-
M AT E M AT I C A S

Con estas fórmulas podemos determinar el área de una región plana siempre que su forma
coincida con una de ellas.
Pero en otras situaciones, la región plana de la que queremos determinar el área no es ni un
cuadrado, ni un círculo, ni un trapecio. En muchas de estas situaciones, la región dada se puede
descomponer en dos o más regiones cuya forma coincida con la de alguna de las figuras
conocidas. En estos casos utilizamos la técnica de la descomposición.
Podemos considerar por ejemplo una figura que descompuesta en dos trozos es un
rectángulo y un semicírculo, sabiendo de antemano las áreas de cada una de estas figuras, el
área total será la suma de ellas.
Otras veces la descomposición es más compleja, debido a la forma de la figura por lo que
debemos utilizar la técnica de la triangulación.
– Si se trata del cálculo de superficies de sólidos por descomposición:
A veces necesitamos conocer la superficie de cuerpos formados por sólidos cuyas áreas se
pueden determinar mediante una fórmula conocida.
Área total de un prisma rectangular: At = 2 ab + 2 bc + 2 ac
Área total de una pirámide de base cuadrada: At = A base + 4 A cara
Área total de un cilindro: At = 2 π r2 + 2 π r g
Área total de un cono: At = π r2 + π r g
Área de una esfera: At = 4 π r2
Por ejemplo, si se quiere pintar un envase cuya parte inferior es un cubo de 3 dm de arista
y cuya parte superior es un cilindro de 3 dm de diámetro y 2 dm de altura, para calcular la
pintura necesaria, sabiendo que con un litro de pintura se pueden pintar 4 m2. La superficie
exterior del envase:
La superficie completa del cubo es: Acubo = 6 · 32 = 54 dm2
La superficie completa del cilindro es: Acilindro = 2 · π · 1,5 · 2 + 2 · 1,52 ·π = 32,99 dm2
Por tanto, Acubo + Acilindro = 86,99 dm2
Como una de las bases del cilindro está apoyada sobre una cara del cubo, no se puede pintar
y, además, impide que se pinte la misma superficie en la cara del cubo. Es decir, debemos de
restar a la superficie total calculada el doble del área de una base del cilindro:
Abase cilindro = 2 · π · 1,52 = 14,14 dm2
Así pues, la superficie a pintar es: 86,99 -14,14 = 72,85 dm2
Como con un litro de pintura podemos pintar 4m2 o, lo que es lo mismo, 400 dm2, para
pintar 72,85 dm2 harán falta: 72,85/400 = 0,18 litros.

18
-Pág.22-
U . D . 3 . - R E S P R E S E N TA C I Ó N Y O R G A N I Z A C I Ó N E N E L E S PA C I O

3.5. REGULARIDAD Y SIMETRÍAS EN FIGURAS, CUERPOS


Y CONFIGURACIONES GEOMÉTRICAS
– Una simetría de eje respecto de una recta e transforma un punto P del plano en otro punto
P' del plano, tal que: el segmento PP' es perpendicular al eje e y la distancia del punto P al eje
e es igual que la distancia del punto P' al eje e. Los puntos simétricos P y P' se llaman
homólogos. Las simetrías respecto de un eje se llaman también axiales.
– Una simetría respecto del punto O transforma un punto P del plano en otro punto P' del
plano, tal que: los puntos PP' y O están alineados y la distancia del punto P a O es igual que la
distancia del punto P' a O. Los puntos simétricos P y P' se llaman homólogos. Las simetrías
respecto de un punto se llaman también centrales.
– En ángulos la bisectriz es el eje de simetría.
– En el triángulo isósceles la mediana, la altura, la bisectriz y la mediatriz es el eje de
simetría y el triángulo equilátero tiene tres ejes de simetría y no tiene centro de simetría.
– En paralelogramos: (todos los paralelogramos tienen un centro de simetría: el punto de
intersección de sus diagonales.)
El cuadrado tiene cuatro ejes de simetría y un centro de simetría.
El rectángulo tiene dos ejes de simetría y un centro de simetría.
El rombo tiene dos ejes de simetría y un centro de simetría.
El romboide no tiene ejes de simetría y sí un centro de simetría.
– Los polígonos regulares que tienen n lados tienen n ejes de simetría.
Si n es par, los ejes de simetría son las rectas que unen entre sí vértices opuestos o puntos
medios de lados opuestos. El centro del polígono es el centro de simetría.
Si n es impar, los ejes de simetría son las rectas que unen cada vértice con el punto
medio del lado opuesto.
En la circunferencia cada diámetro es un eje de simetría y su centro es el centro de
simetría.

19
-Pág.23-
M AT E M AT I C A S

3.6. UTILIDAD E IMPORTANCIA DE ALGUNAS FIGURAS


Y CUERPOS PARA PROPÓSITOS CONCRETOS
– Teselar el plano consiste en cubrirlo mediante la repetición de una o varias figuras
planas, sean geométricas o no. Al conjunto formado por las figuras se le denomina mosaico.
– Rotar: un giro esta determinado por su centro (el punto alrededor del cual giramos los
puntos y figuras), por su amplitud (dada por una medida angular, y por su sentido, siendo
positivo se el sentido es opuesto al de las manecillas del reloj y negativo en caso contrario. La
distancia desde el centro de giro hasta un punto y hasta su homólogo es la misma.
– Minimizar el área, perímetro, longitud y volumen: dos cuerpos semejantes, con la
misma forma pero distinto tamaño, tienen longitudes proporcionales y además ángulos iguales.
La razón de proporcionalidad de las longitudes es la razón de semejanza. La razón de
proporcionalidad entre áreas es el cuadrado de la razón de semejanza, y entre volúmenes el cubo
de esa razón.

4. FIGURAS SEMEJANTES: LA REPRESENTACIÓN


A ESCALA
4.1. REPRESENTACIONES MANEJABLES DE LA REALIDAD:
PLANOS, MAPAS Y MAQUETAS

Planos:
Son representaciones gráficas en una superficie y mediante procedimientos técnicos, de un
terreno o de cualquier cosa semejante.

Mapas:
Son representaciones geográficas de la tierra. También pueden definirse como parte de esas
representaciones geográficas en una superficie plana.
El mapa es una figura semejante a la realidad, esto significa que el segmento que unen por
ejemplo dos ciudades, o puntos cualquiera del mapa y sus correspondientes en la realidad son
proporcionales.
La proporcionalidad está marcada en el mapa por la escala. La escala es la razón de las
distancias de dos puntos del mapa y sus homólogos en la realidad.

Maquetas:
Las representaciones planas a escala sólo nos dan información sobre las dimensiones y la
forma de una determinada sección o vista del objeto representado. A veces, son necesarias
varias representaciones para poder determinar su forma.

20
-Pág.24-
U . D . 3 . - R E S P R E S E N TA C I Ó N Y O R G A N I Z A C I Ó N E N E L E S PA C I O

En los casos en que se quiere tener una visión global del objeto, se realiza una maqueta.
Una maqueta es un modelo en tamaño reducido de un objeto.
Las dimensiones de la maqueta y las de objeto real son proporcionales.
La constante de proporcionalidad es la escala de la maqueta.

4.2. CARACTERÍSTICAS DE LAS DOS FORMAS IGUALES:


IGUALDAD DE ÁNGULOS Y PROPORCIONALIDAD DE LONGITUDES

Igualdad de ángulos:
– Un ángulo es la porción de plano comprendida entre dos semirectas cuyo extremo es
común.
– Dos ángulos son iguales cuando superpuestos coinciden.
– Dos ángulos son homólogos en dos figuras si se corresponden mediante semejanza.

Proporcionalidad de longitudes:
– Si dos rectas concurrentes son cortadas por varias rectas paralelas de forma tal que
interceptan en una de ellas segmentos iguales, los segmentos interceptados en la otra
también son iguales.
– Dos polígonos son semejantes cuando sus ángulos son respectivamente iguales y sus
lados correspondientes son proporcionales.
– En dos figuras semejantes, se denomina razón de semejanza a la razón constante entre dos
segmentos homólogos o lo que es lo mismo es la razón de proporcionalidad entre los
lados correspondientes de una y otra figura.
– Dos longitudes son homólogas en dos figuras si se corresponden mediante semejanza.
– La razón entre los perímetros de figuras semejantes es la razón de semejanza o
proporcionalidad.
– Si dos figuras son semejantes con razón de semejanza k, el área de una de ellas es igual
al área de la otra multiplicada por k2.

21
-Pág.25-
M AT E M AT I C A S

4.3. EL TEOREMA DE TALES


Si dos rectas son cortadas por varias rectas paralelas, los segmentos de las primeras
comprendidos entre rectas paralelas son proporcionales.
Mediante este Teorema se puede calcular longitudes desconocidas en determinadas
situaciones geométricas.
Toda paralela a un lado de un triángulo, que corta a los otros dos lados, determina un
triángulo pequeño semejante al grande.
Si una recta corta a dos lados de un triángulo y los lados del triángulo pequeño son
proporcionales a los del grande, la recta es paralela al tercer lado.
Si dos triángulos están en posición de Tales, son semejantes.

4.4. RELACIÓN ENTRE EL ÁREA Y EL VOLUMEN DE FIGURAS


SEMEJANTES
– Si dos figuras son semejantes con razón de semejanza k, la razón entre sus superficies
(áreas) es k2.
– Si dos cuerpos son semejantes con razón de semejanza k, la razón entre sus
volúmenes es k3.
– Si dos triángulos tienen dos ángulos iguales, son semejantes.
– Si dos triángulos tienen un ángulo igual y los lados que lo forman son proporcionales,
entonces son semejantes.
– Si dos triángulos tienen los tres lados iguales, son semejantes.
– Dos triángulos rectángulos son semejantes si un ángulo agudo de uno de ellos es igual
a un ángulo agudo del otro.

5. TRANSFORMACIONES ISOMÉTRICAS
Una transformación geométrica es una aplicación tal que a cada punto del plano le hace
corresponder otro único punto del mismo plano.
Movimiento o isometría es una transformación geométrica que conserva la forma y el
tamaño. Distinguimos tres tipos de movimientos: traslación, giro y simetría.

22
-Pág.26-
U . D . 3 . - R E S P R E S E N TA C I Ó N Y O R G A N I Z A C I Ó N E N E L E S PA C I O

5.1. TRASLACIONES, GIROS Y SIMETRÍAS


Se llama traslación de vector libre v, y se denota por Tv, a un movimiento que transforma
un punto P del plano en otro P', de modo que: PP' = v.
– Un vector fijo es un segmento orientado caracterizado por:
• Origen
• Dirección o recta que lo contiene
• Sentido u orientación en la recta.
• Módulo, longitud o medida del segmento correspondiente.
– Un vector libre es el conjunto de vectores fijos que tienen igual módulo, dirección y
sentido.
Se llama giro de centro O y ángulo orientado α (de sentido positivo o negativo) y lo denotamos
por GO, α, a un movimiento que transforma un punto P del plano en otro punto P' de modo que:
D (O, P) = D (O, P')
POP' = α
Se denomina simetría central de centro O a un movimiento que transforma un punto P del
plano en otro punto P' de modo que :
D (O, P) = D (O, P')
Se llama simetría axial de eje e, y la denotamos por Se, al movimiento que transforma un
punto P del plano en otro punto P' de modo que:
D (P, e) = D (P', e)

5.2. PROPIEDADES QUE SE CONSERVAN


CON LAS TRANSFORMACIONES
Las propiedades más importantes de la traslación son:
– El transformado de un segmento es otro segmento de la misma longitud y paralelo a él.
– La transformada de una recta es otra recta paralela.
– El transformado de un ángulo es otro ángulo de la misma medida y lados paralelos.

Las propiedades más importantes del giro son:


– El transformado de un segmento es otro segmento de la misma longitud.
– La transformada de una recta es otra recta
– El transformado de un ángulo es otro ángulo de la misma medida

23
-Pág.27-
M AT E M AT I C A S

Las propiedades más importantes de la simetría axial son:


– El transformado de un segmento es otro segmento de la misma longitud.
– La transformada de una recta es otra recta
– El transformado de un ángulo es otro ángulo de la misma amplitud y sentido contrario.

5.3. COMPOSICIÓN DE TRANSFORMACIONES EN CASOS SENCILLOS

– El producto de dos traslaciones de vectores libres es otra traslación de vector libre.


– El producto de las simetrías de ejes paralelos es una traslación de vector libre. El
módulo o longitud es doble de la distancia entre los ejes, su dirección es la de la recta
perpendicular a ambos ejes y su sentido es la orientación que tenga la recta.
– El producto de las simetrías axiales de ejes concurrentes es un giro cuyo centro es el
punto O donde se cortan los ejes y de amplitud doble que del ángulo formado entre el
primer y segundo ejes y del mismo sentido.
– El producto de una traslación, de vector libre, por un giro, es otro giro de la misma
amplitud y sentido.
– El producto de dos giros de distinto centro, es un giro cuya amplitud es la suma de las
dos amplitudes.

24
-Pág.28-
U . D . 3 . - R E S P R E S E N TA C I Ó N Y O R G A N I Z A C I Ó N E N E L E S PA C I O

RESUMEN
Los elementos geométricos en el plano y en el espacio son: el punto, la recta y el plano.
Los sistemas de referencia se representan por las coordenadas cartesianas tanto en el plano
como en el espacio.
Las figuras planas se clasifican según su contorno en polígonos y cónicas. Los polígonos
tienen los siguientes elementos: lados, vértices, diagonales, ángulos interiores y exteriores, se
clasifican en cóncavos y convexos, según el número de lados en triángulo... y según su forma en
regulares e irregulares, y las cónicas se clasifican en circunferencia, elipse, parábola, e hipérbola.
Los cuerpos geométricos según sus caras se clasifican en: caras planas (poliedros, prismas,
y pirámides) y en caras curvas (cilindro, cono y esfera).
En las transformaciones geométricas hay una serie de movimientos como son las
traslaciones que tienen como característica los vectores libres, las simetrías (axial y central) y
los giros que se caracterizan por tener un centro de giro y un ángulo de giro.

25
-Pág.29-
M AT E M AT I C A S

EJERCICIOS DE AUTOCOMPROBACIÓN

1. Dos planos son secantes cuando:

A. Tienen todos los puntos comunes B. Tienen una recta común

C. No tienen ningún punto común D. Ninguna de las anteriores es correcta

2. Calcular el área de un paralelogramo, sabiendo que sus lados miden 40 cm y 18 cm, y que uno de
sus ángulos es de 45º.

A. 509,12 cm2 B. 509,12 cm C. 500,12 cm2 D. 120,50 cm2

3. Calcular el área de un círculo de 20 mm de diámetro.

A. 3,14 mm2 B. 31,16 mm2 C. 314,16 mm2 D. 34,16 mm2

4. Las áreas de dos polígonos semejantes son 144 cm2 y 64 cm2. Un lado del polígono pequeño mide
10 cm. ¿cuánto mide el lado homólogo en el polígono grande?

A. 20 cm B. 25 cm C. 10 cm D. 15 cm

5. Calcula el área de la esfera de 3 unidades de radio y verifica que es igual al área total del cilindro
con igual radio y altura tambien 3.

A. Es mayor el B. El área del C. Ambas miden D. Ninguna de


área de la esfera cilimdro mide 18π 36π las anteriores

6. Las sombras de dos árboles son, a la misma hora del día, de 1,2 m y de 8,4 m. Medimos el pequeño
y da una altura de 3 m. ¿cuál es la altura del grande?

A. 21 m B. 31 m C. 20 m D. 30 m

7. Indica la fracción de área de un círculo que abarca un sector de 90 grados.

A. 1/2 B. 1/3 C. 1/4 D. 1/8

8. Dos ángulos complementarios se diferencian en 30 grados. ¿podrías hallarlos?

A. 30 grados y 90 grados B. 30 grados y 60 grados

C. 30 grados y 45 grados D. 45 grados y 60 grados

26
-Pág.30-
U . D . 3 . - R E S P R E S E N TA C I Ó N Y O R G A N I Z A C I Ó N E N E L E S PA C I O

9. Dibuje una circunferencia de 4 cm de radio, y con el mismo centro, una elipse cuyo eje mayor sea
el diámetro de la circunferencia y el eje menor el radio, sabiendo que la semidistancia foral, c es la
raíz cuadrada de a2 -b2 y la excentricidad de la circunferencia es 0 y la de la elipse es e = c/a .
compara las excentricidades de ambas cónicas.

A. No tienen comparación B. Son muy amplias

C. La elipse está muy achatada D. ninguna de las anteriores

10. Un depósito de agua tiene forma de prisma pentagonal regular de 4,5 m de altura y 1,5 m de arista
básica. Calcula la capacidad del depósito.

A. 1000 litros B. 15381 litros C. 16381 litros D. 17381 litros

27
-Pág.31-
M AT E M AT I C A S

RESPUESTAS A LOS EJERCICIOS

11. B

12. A

13. C

14. D

15. C

16. A

17. C

18. B

19. C

10. D

28
-Pág.32-
U . D . 4 . - INTERPRETACIÓN, REPRESENTACIÓN Y TRATAMIENTO DE LA INFORMACIÓN

ÍNDICE
OBJETIVOS . . . . . . . . . . . . . . . . . . . . . . . . . . . . . . . . . . . . . . . . . . . . . . . . . . . . . . . . . . . . . .2

INTRODUCCIÓN . . . . . . . . . . . . . . . . . . . . . . . . . . . . . . . . . . . . . . . . . . . . . . . . . . . . . . . . . .3

MAPA CONCEPTUAL . . . . . . . . . . . . . . . . . . . . . . . . . . . . . . . . . . . . . . . . . . . . . . . . . . . . . .4

DESARROLLO DE CONTENIDOS

1. INTERPRETACIÓN, REPRESENTACIÓN Y TRATAMIENTO DE LA


INFORMACIÓN . . . . . . . . . . . . . . . . . . . . . . . . . . . . . . . . . . . . . . . . . . . . . . . . . . . .5
1.1. DEPENDENCIA FUNCIONAL . . . . . . . . . . . . . . . . . . . . . . . . . . . . . . . . . . . . . . . . . .5

2. CARACTERÍSTICAS DE LAS GRÁFICAS . . . . . . . . . . . . . . . . . . . . . . . . . . . . .6


2.1. ASPECTOS GLOBALES . . . . . . . . . . . . . . . . . . . . . . . . . . . . . . . . . . . . . . . . . . . . . . .6
2.2. ASPECTOS LOCALES . . . . . . . . . . . . . . . . . . . . . . . . . . . . . . . . . . . . . . . . . . . . . . . .8
2.3 GRÁFICAS LINEALES . . . . . . . . . . . . . . . . . . . . . . . . . . . . . . . . . . . . . . . . . . . . . . . .8

3. FUNCIONES ELEMENTALES . . . . . . . . . . . . . . . . . . . . . . . . . . . . . . . . . . . . . . . .9
3.1. FENÓMENOS Y GRÁFICAS DE PROPORCIONALIDAD INVERSA,
CUADRÁTICOS, EXPONENCIALES, Y PERIÓDICOS . . . . . . . . . . . . . . . . . . . . . .9
3.2. EXPRESIÓN ALGEBRAICA ASOCIADA A UNA GRÁFICA . . . . . . . . . . . . . . . .13
3.3. INFORMACIÓN SOBRE FENÓMENOS ALEATORIOS . . . . . . . . . . . . . . . . . . . .14
3.4. INFORMACIÓN SOBRE ELEMENTOS ALEATORIOS . . . . . . . . . . . . . . . . . . . . .14

4. OBTENCIÓN DE LA INFORMACIÓN SOBRE FENÓMENOS ALEATORIOS 14


4.1. LAS MUESTRAS Y SU REPRESENTATIVIDAD . . . . . . . . . . . . . . . . . . . . . . . . . .15
4.2. FRECUENCIAS ABSOLUTAS, RELATIVAS Y PORCENTUALES . . . . . . . . . . . .16
4.3 GRÁFICAS ESTADÍSTICAS USUALES . . . . . . . . . . . . . . . . . . . . . . . . . . . . . . . . .18

5. PARÁMETROS ESTADÍSTICOS . . . . . . . . . . . . . . . . . . . . . . . . . . . . . . . . . . . . .20


5.1. LOS PARÁMETROS CENTRALES Y DE DISPERSIÓN . . . . . . . . . . . . . . . . . . . .20
5.2. PARÁMETROS MAS UTILIZADOS EN UNA DISTRIBUCIÓN ESTADÍSTICA
NORMAL . . . . . . . . . . . . . . . . . . . . . . . . . . . . . . . . . . . . . . . . . . . . . . . . . . . . . . . . . .23

6. DEPENDENCIA ALEATORIA ENTRE DOS VARIABLES . . . . . . . . . . . . . . .24

RESUMEN . . . . . . . . . . . . . . . . . . . . . . . . . . . . . . . . . . . . . . . . . . . . . . . . . . . . . . . . . . . . .26

EJERCICIOS DE AUTOCOMPROBACIÓN . . . . . . . . . . . . . . . . . . . . . . . . . . . . . . . . . .27

RESPUESTAS A LOS EJERCICIOS . . . . . . . . . . . . . . . . . . . . . . . . . . . . . . . . . . . . . . . . .28

-Pág.33-
M AT E M Á T I C A S

OBJETIVOS
Al finalizar el estudio de esta Unidad Didáctica, el alumno será capaz de:

CONCEPTOS

• Identificar y determinar los parámetros estadísticos centrales.

• Obtener los parámetros estadísticos de dispersión.

• Interpretrar tablas y gráficas.

• Distinguir los conceptos propios de las funciones: dominio, recorrido,


crecimiento y decrecimiento, continuidad, periodicidad...

• Identificar las propiedades de las funciones elementales.

• Observar las ventajas e inconvenientes de la utilización del lenguaje gráfico.

-Pág.34-
U . D . 4 . - INTERPRETACIÓN, REPRESENTACIÓN Y TRATAMIENTO DE LA INFORMACIÓN

INTRODUCCIÓN
L a información matemática que se obtiene de los fenómenos naturales y sociales
suele venir dada como una relación entre magnitudes, bien de tipo causal o de tipo
estadístico.

El lenguaje es el medio que nos permite comunicarnos con los demás. Los lenguajes más
utilizados son: oral, escrito y gráfico.

El lenguaje gráfico constituye una forma de conocimiento y de transmisión de la


información. La interpretación de las tablas y de los gráficos es rápida y muestran como
unas variables dependen unas de otras.

Por otra parte, estudiaremos un poco de estadística, las primeras noticias de la existencia
de esta parte de las matemáticas datan de 2238 a. C., en China, posteriormente fueron los
egipcios quienes empezaron a hacer trabajos simples estadísticos y en el Imperio
Romano se realizaban censos de bienes y personas. En nuestros días todos los estudios
un poco serios emplean datos estadísticos que nos dan mucha información sobre
determinadas muestras.

-Pág.35-
M AT E M Á T I C A S

M A PA C O N C E P T UA L
FUNCIONES

GRÁFICAS FAMILIAS DE FUNCIONES

FUNCIÓN DE
PROPORCIONALIDAD INVERSA

FUNCIÓN
EXPONENCIAL

FUNCIÓN
CUADRÁTICA

FUNCIÓN
PERIÓDICA

ESTADÍSTICA

POBLACIÓN Y MUESTRAS VARIABLES ESTADÍSTICAS

TABLAS:
MEDIDAS DE CENTRALIZACIÓN
MEDIDAS DE DISPERSIÓN

CORRELACIÓN: PEARSON

REGRESIÓN

4
-Pág.36-
U . D . 4 . - INTERPRETACIÓN, REPRESENTACIÓN Y TRATAMIENTO DE LA INFORMACIÓN

1. INTERPRETACIÓN, REPRESENTACIÓN
Y TRATAMIENTO DE LA INFORMACIÓN

1.1. DEPENDENCIA FUNCIONAL


Debemos recordar varios conceptos:
– La variable dependiente es la que depende de otra que denominamos variable
independiente.
– La variable dependiente se suele designar con la letra y, la variable independiente
con la letra x.
– La relación que existe entre dos variables le llamamos correspondencia.
– En toda correspondencia deben tenerse en cuenta dos conjuntos numéricos.
Conjunto inicial o conjunto en el que toma valores la variable independiente, y
conjunto final o conjunto en el que toma valores la variable dependiente.
– Las correspondencias más importantes son las funciones.
– Una función es una correspondencia en la que a cada valor de la variable
independiente le corresponde un valor y solo un valor de la variable dependiente.
– En toda función interesan los siguientes conjuntos numéricos: dominio de la
función e imagen y recorrido de la función.

1.1.1. FORMAS DE EXPRESAR LA DEPENDENCIA ENTRE VARIABLES:


DESCRIPCIÓN VERBAL, TABLA, GRÁFICA Y FÓRMULA
– La descripción verbal nos proporciona una visión descriptiva y cualitativa de la
relación funcional.
– La tabla de valores de una función está tomada por dos filas o dos columnas. En la
primera fila o columna figuran los valores que toma la variable independiente y en
la segunda fila o columna figuran los correspondientes que toma la variable
dependiente.
– Las gráficas nos dan una visión cualitativa de las funciones sobre unos ejes
coordenados.
– La expresión o fórmula matemática permite calcular los valores de la variable
dependiente para todos los valores de la variable independiente.

5
-Pág.37-
M AT E M Á T I C A S

2. CARACTERÍSTICAS DE LAS GRÁFICAS

2.1. ASPECTOS GLOBALES: CONTINUIDAD, CRECIMIENTO,


VALORES EXTREMOS, PERIODICIDAD, TENDENCIA

– Continuidad:
– Una función es continua si su gráfica puede dibujarse con trazo continuo, es decir, sin
levantar el lápiz del papel.
– Una función es discontinua si su gráfica no puede dibujarse con trazo continuo y, por
tanto, presenta saltos o agujeros en su trazado.

– Crecimiento:
– Una función y = f (x) es creciente en un intervalo si para dos valores cualquiera x1 y x2
del mismo se cumple:
x1< x2 ⇒ f (x1) < f (x2)
– Una función y = f (x) es decreciente en un intervalo si para dos valores cualquiera x1 y
x2 del mismo se cumple:
x1< x2 ⇒ f (x1) > f (x2)
– Una función es constante en un intervalo cuando al aumentar o disminuir la variable
independiente se mantiene fija.

y y

5 5

0 x
0 x
–5 0 5
–5 0 5

–5
–5

6
-Pág.38-
U . D . 4 . - INTERPRETACIÓN, REPRESENTACIÓN Y TRATAMIENTO DE LA INFORMACIÓN

– Valores extremos: máximos y mínimos


– Una función cuya gráfica no presenta cortes tiene un máximo relativo en un punto si a la
izquierda de este punto la función crece y a la derecha de este punto la función decrece.
– La función cuya gráfica no presenta cortes tiene un máximo absoluto en un punto para el
mayor de los valores que puede tomar la función, si este existe.
– Una función cuya gráfica no presenta cortes tiene un mínimo relativo en un punto si a la
izquierda de este punto la función decrece y a la derecha de este punto la función crece.
– La función cuya gráfica no presenta cortes tiene un mínimo absoluto en un punto para el
menor de los valores que puede tomar la función, si este existe.

– Tendencia: simetrías y cortes con los ejes:


– Una función y = f(x) es simétrica respecto al eje de ordenadas si para cualquier valor x
de su dominio se cumple:
- f(x) = f(– x)
estas funciones son conocidas como funciones pares.
– Una función y = f(x) es simétrica respecto al origen de coordenadas si para cualquier
valor x de su dominio se cumple:
- f(x) = – f(–x)
- Estas funciones son conocidas como funciones impares.
– Los puntos de corte con los ejes coordenados se obtienen al resolver los siguientes
sistemas:

7
-Pág.39-
M AT E M Á T I C A S

(x, f(x))

y = x2

y = x3
(–x, f(x)) (x, f(x))

(–x, f(x))

Simetría par Simetría impar

2.2. ASPECTOS LOCALES: TASA DE VARIACIÓN MEDIA


– Llamamos tasa de variación de una función f entre dos valores x1 y x2, x1 < x2, a la
diferencia entre los valores que toma la función:
- tv [ x1 , x2 ] = f (x2 ) – f (x1)
– Llamamos tasa de variación media de una función f entre dos valores x1 y x2, x1 < x2, al
cociente de dividir la tasa de variación por x2 – x1.
- tvm [ x1 , x2 ] = f (x2) – f (x1) / x2 – x1

La tasa de variación media representa la variación de la variable dependiente por cada


unidad de la variable independiente.

2.3. GRÁFICAS LINEALES: SIGNIFICADO EN TÉRMINOS


DE PROPORCIONALIDAD
Una función constante es aquella en la cual el valor de la variable dependiente siempre es el
mismo sea cual sea el valor de la variable independiente.

• Su gráfica es una línea recta paralela al eje de abscisas OX.


• Su expresión matemáticas es: y = b.

8
-Pág.40-
U . D . 4 . - INTERPRETACIÓN, REPRESENTACIÓN Y TRATAMIENTO DE LA INFORMACIÓN

Las funciones lineales o de proporcionalidad directa son funciones cuya gráfica es una línea
recta que pasa por el origen de coordenadas. Su fórmula matemática es: y = mx.

• Si m > 0 la función es creciente, es decir, a medida que se avanza en la horizontal se


produce un aumento en la vertical.
• Si m < 0 la función es decreciente, es decir, a medida que se avanza en la horizontal se
produce una disminución en la vertical.

Las funciones afines son funciones cuya gráfica es una línea recta que no pasa por el origen
de coordenadas, su formula matemática es y = mx + b (“m” es la pendiente de la recta, si “m”
es positivo la función es creciente y si “m” es negativo la función es decreciente).
• El parámetro “b” es la ordenada en el origen. La recta de ecuación y = mx + b corta al eje
de ordenadas OY en el punto (0, b).

Las rectas de las funciones afines y lineales que tienen igual pendiente son rectas paralelas.

3. FUNCIONES ELEMENTALES

3.1. FENÓMENOS Y GRÁFICAS DE PROPORCIONALIDAD


INVERSA, CUADRÁTICOS, EXPONENCIALES Y PERIÓDICOS
– Funciones de proporcionalidad inversa:
- Las funciones cuya expresión es y = k / x se llaman funciones de proporcionalidad inversa.
- Las gráficas de estas funciones son hipérbolas equiláteras cuyas asíntotas son los ejes
coordenados.
- Las propiedades de estas funciones son:
Dominio: es el conjunto formado por todos los números reales excepto x = 0.
Imagen: es el conjunto de todos los números reales excepto el 0.
Simetría: son simétricas respecto al eje de coordenadas.
Crecimiento y decrecimiento: estas funciones son siempre crecientes para k < 0 y siempre
decrecientes para k > 0.

9
-Pág.41-
M AT E M Á T I C A S

Máximos y mínimos: no tienen.


Continuidad: son continuas en todos los valores de su dominio.
Cortes con los ejes: no tienen cortes con los ejes coordenados.

– Funciones cuadráticas:
- Las funciones cuya expresión es un polinomio de grado dos, se llaman funciones
cuadráticas.
Y = ax2 + bx + c
- Las gráficas de estas funciones cuadráticas son parábolas con eje vertical.
- Las propiedades de estas funciones son:

Vértice de la parábola: para la función cuadrática Y = ax2 + bx + c, las coordenadas del


vértice son:
V (–b / 2a, -(b2 – 4ac) / 4a
(

Orientación de la parábola:
En la función cuadrática y = a x2 + b x + c se obtiene:
- Si a es positivo, las ramas de la parábola van hacia arriba.
- Si a es negativo, las ramas de la parábola van hacia abajo.

Simetría: las parábolas son simétricas respecto a la recta x = – b / 2a, que es el eje de
simetría de la gráfica.

Máximos y mínimos: la función cuadrática tiene un mínimo en su vértice si a es positivo,


y tiene un máximo si a es negativo.

Continuidad: la función cuadrática es continua en todos los valores de su dominio


constituido por todos los números reales.

Puntos de corte con los ejes:


Con el eje de ordenadas O Y. El punto (O, c)

Con el eje de abscisas O X. Pueden existir dos puntos, uno o ninguno.

10
-Pág.42-
U . D . 4 . - INTERPRETACIÓN, REPRESENTACIÓN Y TRATAMIENTO DE LA INFORMACIÓN

Ejemplo:

Representa gráficamente la función y = –x2 + 4x – 6

Cálculo de las coordenadas del vértice

⎛ b b 2 – 4 ac ⎞
V ⎜– ,– ⎟ = (2, –2)
⎝ 2a 4a ⎠

Ecuación del eje de simetría: x = -b/2a ⇒ x = 2

Corte con los ejes de coordenadas:

Eje OY: y = –x2 + 4x – 6 si x = 0 se tiene el punto (0, -6)


Eje OX. y = –x2 + 4x – 6 si y = 0 no tiene solución la ecuación por lo tanto no corta con
el eje de las abscisas.

Calculando algunos valores nos permite representar la función:

11
-Pág.43-
M AT E M Á T I C A S

– Funciones exponenciales:
- La función exponencial es una función cuya expresión es: y = ax, siendo la base a un
número real positivo y distinto de uno.
- Las propiedades de estas funciones son:

- Base mayor que uno (a > 1)


- Dominio: es el conjunto de todos los números reales.
- Imagen: es el conjunto de los números reales positivos.
- Crecimiento y decrecimiento: son siempre crecientes.
- Continuidad: son continuas en todos los puntos de su dominio.
- Máximos y mínimos: no tienen.
- Cortes con los ejes: estas funciones cortan al eje O Y en el punto (0, 1) y no tienen
cortes con el eje O X.
- Base comprendida entre 0 y 1 (0 < a < 1)
- Dominio: es el conjunto de todos los números reales.
- Imagen: es el conjunto de los números reales positivos.
- Crecimiento y decrecimiento: son siempre decrecientes.
- Continuidad: son continuas en todos los puntos de su dominio.
- Máximos y mínimos: no tienen.
- Cortes con los ejes: estas funciones cortan al eje O Y en el punto (0, 1) y no tienen
cortes con el eje O X.
– Funciones periódicas:
- Las funciones periódicas son aquellas cuya gráfica se va repitiendo cada vez que la
variable independiente recorre un intervalo determinado.
- A la amplitud o longitud, T, del citado intervalo se le llama período.
- La expresión y = f (x) de una función periódica cumple:
- f(x) = f (x + T) = f (x – T) =f (x + 2T) = f (x – 2T) = …
- En general, f (x) = f (x + kT), k es un número entero.

12
-Pág.44-
U . D . 4 . - INTERPRETACIÓN, REPRESENTACIÓN Y TRATAMIENTO DE LA INFORMACIÓN

Y Y

8 4

3
6
2
4
1
2
–3 –2 –1 0 1 2 3 X

0 1 2 3 4 X

Función exponencial a > 1 Función exponencial 0 < a < 1

Función periódica de período T

3.2. EXPRESIÓN ALGEBRAICA ASOCIADA A UNA GRÁFICA


La gráfica de una función es la representación, en unos ejes de coordenadas, de todos los
puntos de la forma (x, f (x), siendo x del dominio de f.

Ejemplo:
Un banco ofrece cuentas corrientes con un 2,5% de interés si el saldo es inferior a 3000 euros,
5% de interés para saldos entre 3000 euros y 6000 euros, y 7,5% para saldos superiores a 6000
euros. La tabla que genera esta función nos da el interés en función del saldo.
Necesitamos tres fórmulas para definir dicha función.

3000 euros

3000 6000 euros

6000 euros

Las funciones de este tipo se llaman definidas a trozos.

13
-Pág.45-
M AT E M Á T I C A S

Para completar la tabla de valores tendremos que dar a x valores dentro de cada intervalo y
aplicar la fórmula adecuada.

x Intervalo F (x)
1000 x < 3000 25

1600 x < 3000 40

3000 3000 ≤ x ≤ 6000 150

6000 3000 ≤ x ≤ 6000 300

8000 x > 6000 600

15000 x > 6000 1125

3.3. INFORMACIÓN SOBRE FENÓMENOS ALEATORIOS


Según sea o no posible predecir el resultado podemos distinguir dos tipos de experimentos:
Experimentos deterministas son aquellos en los que se puede calcular y determinar
exactamente su resultado.
Experimentos aleatorios cuando no podemos asegurar anticipadamente cuál va a ser su
resultado. Ejemplo: tirar un dado.

OBTENCIÓN DE LA INFORMACIÓN
4. SOBRE FENÓMENOS ALEATORIOS
La estadística es la ciencia que se ocupa de la recogida de datos, su organización y análisis,
así como de las predicciones que, a partir de estos datos, pueden hacerse.
La estadística descriptiva se ocupa de tomar los datos de un conjunto, organizarlos en tablas
o en representaciones gráficas y del cálculo de unos números que nos informen de manera
global del conjunto estudiado.

14
-Pág.46-
U . D . 4 . - INTERPRETACIÓN, REPRESENTACIÓN Y TRATAMIENTO DE LA INFORMACIÓN

La estadística inferencial trata sobre la elaboración de conclusiones para la población,


partiendo de los resultados de una muestra y del grado de fiabilidad de estas conclusiones.

4.1. LAS MUESTRAS Y SU REPRESENTATIVIDAD


– Los conceptos básicos que aparecen en cualquier estudio estadístico son:
- Población: es el conjunto formado por todos los elementos que existen para el estudio
de un determinado fenómeno.
- Individuo u objeto: es cada elemento de una población.
- Muestra: es el subconjunto que tomamos de la población para determinar el estudio del
fenómeno.
- Tamaño de la muestra: es el número de individuos que la componen.
– Variables o caracteres estadísticos
– Cada una de la cualidades o propiedades referidas a los elementos de una población objeto
de estudio estadístico se llama variable o carácter estadístico.
– Las variables estadísticas pueden ser:
– Variables o caracteres cualitativos son aquellos que no se pueden medir y se describen
con palabras. Ejemplo: la raza de un perro…
– Variables o caracteres cuantitativos son aquellos que se pueden expresar y medir con
números.
– Variables o caracteres cuantitativos discretos son aquellos que pueden tomar solamente
un número finito de valores numéricos. Ejemplo: el número de hermanos de cada
alumno de una clase…
– Variables o caracteres cuantitativos continuos son aquellos que pueden tomar cualquier
valor en un intervalo dado. Ejemplo: la estatura de los alumnos de una clase…
– Encuestas y muestreo
– Se llama encuesta a las preguntas que se formulan a un cierto número de individuos de un
colectivo o población.
– Las características más significativas que debe poseer una encuesta son:
– El tema de la encuesta y las variables que intervengan deben ser claras y concretas .
– Las preguntas deben ser precisas para que puedan ser entendidas por los encuestados.
– Las preguntas no deben influir en los encuestados.

15
-Pág.47-
M AT E M Á T I C A S

– Debe de ser contestada con una palabra.


– Las preguntas deben estar ordenadas para que unas respuestas no condicionen a otras.
– Una vez que se ha elaborado una encuesta y la aplicamos a una parte de la población, a la
cual llamamos muestra, cada elemento de la población debe tener la misma oportunidad
de encontrarse en la muestra. Las características de esos elementos deben de reproducir
con la máxima exactitud a los de la población, y la muestra no debe ser muy pequeña
porque se pueden introducir muchos errores.

4.2. FRECUENCIAS ABSOLUTAS, RELATIVAS Y PORCENTUALES


– Frecuencia absoluta, fi, de una cualidad o de un valor xi de la variable estadística es un
número total de veces que aparece esta cualidad o valor.
- Frecuencia absoluta acumulada, Fi,de una cualidad o de un valor xi, es la suma de todas
las frecuencias absolutas correspondientes a los valores anteriores a xi y a la suya
propia.
– Frecuencia relativa o proporción, hi, de una cualidad o de un valor xi, es el cociente que
resulta de dividir su frecuencia absoluta entre el número total, N, de individuos.
- Frecuencia relativa acumulada, Hi, de una cualidad o de un valor xi es la suma de todas
las frecuencias relativas correspondientes a los valores anteriores a xi y a la suya propia.
– Frecuencia porcentual o porcentaje, pi, de una cualidad o de un valor xi es el tanto por
ciento que representa este valor o cualidad respecto del total. Se calcula multiplicando la
frecuencia relativa por 100.
- Frecuencia porcentual acumulada Pi, de una cualidad o de un valor xi es la suma de las
frecuencias porcentuales correspondientes a los valores anteriores a xi y a la suya propia.

Ejemplo

Las puntuaciones obtenidas en un test de razonamiento abstracto por 20 alumnos son las
siguientes:

16, 22, 21, 20, 23, 22, 17, 15, 13, 22, 17, 18, 20, 17, 22, 16, 23, 21, 22, 18

a) Construye la tabla de frecuencias

b) Representa gráficamente la función

16
-Pág.48-
U . D . 4 . - INTERPRETACIÓN, REPRESENTACIÓN Y TRATAMIENTO DE LA INFORMACIÓN

a) Recuento y tabla

Valor de la Frecuencia Frecuencia abso- Frecuencia Frecuencia relativa


variable absoluta luta acumulada relativa acumulada
13 1 1 0,050 0,050
15 1 2 0,050 0,100
16 2 4 0,100 0,200
17 3 7 0,150 0,350
18 2 9 0,100 0,450
20 2 11 0,100 0,550
21 2 13 0,100 0,650
22 5 18 0,250 0,900
23 2 20 0,100 1,000
20 1,000

b) Diagrama de barras

6
FREC. ABSOLUTAS

5
4
3
2
1
0
13 15 16 17 18 20 21 22 23

PUNTUACIONES

17
-Pág.49-
M AT E M Á T I C A S

4.3. GRÁFICAS ESTADÍSTICAS USUALES


– Gráficas para variables estadísticas cualitativas:
- Diagrama de rectángulos: consiste en dibujar un rectángulo por cada una de las
cualidades de la variable, de modo que las bases sean todas iguales apoyadas en el eje,
en el que se indican las cualidades, y la altura de cada rectángulo debe ser proporcional
a la frecuencia de la cualidad representada.
El diagrama muestra la facturación de dos constructoras españolas en el transcurso del
año 1999.
CONSTRUCTORAS ESPAÑOLAS

416000

325000

DRAGADOS GINES Y NAVARRO

- Diagrama de sectores: consiste en dividir un círculo, a veces un semicírculo, en


sectores circulares, cada uno de ellos representa una cualidad variable. La amplitud, en
grados, de cada sector es proporcional a la frecuencia o porcentaje de la cualidad a la
que representa.

- Pictogramas: consiste en realizar dibujos alusivos a la distribución que se desea


representar. Son gráficos poco precisos, aunque fáciles de interpretar a simple vista.

- Cartogramas: consisten en representar sobre un mapa cualquier tipo de datos


relacionados con un área geográfica.

18
-Pág.50-
U . D . 4 . - INTERPRETACIÓN, REPRESENTACIÓN Y TRATAMIENTO DE LA INFORMACIÓN

– Gráficas para variables estadísticas cuantitativas:


- Diagrama de barras o de frecuencias: representan distribuciones de variables discretas
por medio de barras o columnas independientes, situadas encima de la variable
representada. En muchas ocasiones se superponen dos o mas diagramas con el fin de
comparar los datos de diferentes situaciones.
- Diagrama de frecuencias: también se les denomina polígonos de frecuencias. Se
forman uniendo los extremos de las barras o columnas mediante una línea quebrada.
Son muy utilizados, los que representan las frecuencias acumuladas, en el estudio del
crecimiento de determinados fenómenos.
- Histogramas: son análogos a los diagramas de barras o columnas y se utilizan para
representar distribuciones de frecuencias de variables continuas. Consisten en
rectángulos cuyas bases son cada uno de los intervalos y la altura es la frecuencia
absoluta correspondiente a dicho intervalo, siempre que todos los intervalos tengan la
misma amplitud.En caso contrario, las alturas serán tales que las áreas de los
rectángulos sean proporcionales a las correspondientes frecuencias.
– Series temporales y otras gráficas:
- Series temporales: muestran las fluctuaciones de una o varias variables estadísticas con
el paso del tiempo. Son diagramas lineales en los que interesa considerar la altura de la
línea con respecto a la base del dibujo.
- Pirámide de poblaciones: son gráficos de población clasificada por la edad y por su
sexo. La variable edad se toma sobre el eje vertical y las frecuencias o porcentajes de
mujeres (derecha) y hombres (izquierda) se toman sobre el eje horizontal. Es decir, una
pirámide de población consiste en dos histogramas, cuya variable estadística está en el
eje vertical y las frecuencias o porcentajes en el eje horizontal.

19
-Pág.51-
M AT E M Á T I C A S

5. PARÁMETROS ESTADÍSTICOS

5.1. LOS PARÁMETROS CENTRALES Y DE DISPERSIÓN COMO


RESUMEN DE UN CONJUNTO DE DATOS ESTADÍSTICOS

– La información recogida en una tabla o gráfica estadística suele resumirse en unos pocos
valores que nos informan del comportamiento de todos los individuos del colectivo que
se estudia. Estos valores, representativos de todos los de una distribución, se llaman
parámetros o medidas de centralización. Y son los siguientes:
- Media aritmética: de una variable estadística es el cociente que resulta de dividir la
suma de todos los valores por el número total de estos y se representa por –x.
x– = Σ xi / N
– - Ejemplo: el número de libros leídos por los alumnos de una clase durante un trimestre
del curso son los que aparecen en la tabla que se adjunta.

Nº libros leídos 0 1 2 3 4 5

Nº alumnos 25 27 18 8 12 10

- Calcula el número medio de libros leídos en el citado trimestre.

xi fi xifi
0 25 0
1 27 27
2 18 36
3 8 24
4 12 48
5 10 50
Sumas 100 185

x = 185/100 =1,85

20
-Pág.52-
U . D . 4 . - INTERPRETACIÓN, REPRESENTACIÓN Y TRATAMIENTO DE LA INFORMACIÓN

- Moda: la moda de una variable estadística discreta o cualitativa es el valor o cualidad


de la variable con mayor frecuencia absoluta. Se representa por Mo.
Si los resultados de una quiniela son:

ZARAGOZA - ESPAÑOL x
MÉRIDA - BETIS 2
OVIEDO - AT. MADRID 2
CELTA - SALAMANCA 1
ATHLETIC - REALSOCIEDAD x
VALLADOLID - VALENCIA 2
REAL MADRID - DEPORTIVO x
RACING - SPORTING 1
BARCELONA - TENERIFE 1
COMPOSTELA - MALLORCA x
EÍBAR - RAYO VALLECANO 2
LAS PALMAS - HÉRCULES 1
JAÉN - ALAVES x
BADAJOZ - LOGROÑES x
EXTREMADURA - SEVILLA 1

La moda de “los resultados de esta quiniela” es x.

- La mediana: Es el valor que deja a su izquierda un número de datos iguales a los que
deja a su derecha cuando los datos de la serie se encuentran ordenados. Se representa
por Me.
Ejemplo: las notas que obtuvieron los 32 alumnos de una clase en idioma extranjero
fueron:

Notas xi fi Fi

1 2 2
2 2 4
3 3 7
4 5 12<16
5 7 19>16
6 5 24
7 3 27
8 2 29
9 2 31
10 1 32
n = 32

21
-Pág.53-
M AT E M Á T I C A S

– - La mediana es el primer valor cuya frecuencia acumulada Fi, sea mayor que la mitad del
número de individuos: N/2 = 16.
– - En nuestro caso la mediana es Me = 5.
– Los parámetros de dispersión son los valores numéricos que nos informan de las
desviaciones que sufren los datos de una distribución estadística respecto de los
parámetros centrales, en particular respecto de la media aritmética.
– - Varianza: es la media aritmética de los cuadrados de las desviaciones de todos los datos
o marcas de clase respecto de la media. Se representa por σ2.

– - Desviación típica: es la raíz cuadrada positiva de la varianza. Se representa por σ.

– - Ejemplo: calcula las desviaciones típicas de las distribuciones estadísticas relativas a las
notas de dos cursos. Las respectivas notas medias son 5,03 y 5,1.

Distribución estadística del curso con media 5,1


xi fi xi - x (xi - x)2 (xi -x )2. fi
0 4 -5,1 26,01 104,04
1 3 -4,1 16,81 50,43
2 4 -3,1 9,61 38,44
3 1 -2,1 4,41 4,41
4 1 -1,1 1,21 1,21
5 3 -0,1 0,01 0,03
6 3 0,9 0,81 2,43
7 1 1,9 3,61 3,61
8 1 2,9 8,41 8,41
9 3 3,9 15,21 45,63
10 6 4,9 24,01 144,06
Total 30 402,7

22
-Pág.54-
U . D . 4 . - INTERPRETACIÓN, REPRESENTACIÓN Y TRATAMIENTO DE LA INFORMACIÓN

Distribución estadística del curso con media 5,03


x fi fi xi fi xi2
1 1 1 1
2 2 4 8
3 3 9 27
4 4 16 64
5 10 50 250
6 4 24 144
7 2 14 98
8 3 24 192
9 1 9 81
Total 30 865

σ(media clase 5,1) = 3,66


σ(media clase 5,03) = 1.88
la desviación típica es menor en la clase con media mas baja 5,03 que en la clase con media
más alta 5,1.

5.2. LOS PARÁMETROS MAS UTILIZADOS EN UNA DISTRIBUCIÓN


ESTADÍSTICA NORMAL
– La media aritmética y la desviación típica, son los parámetros estadísticos más utilizados.
– En una distribución estadística, el estudio del comportamiento conjunto de estos dos
parámetros nos aporta numerosa información sobre la distribución de frecuencias estudiadas.
Una distribución estadística de tipo normal es aquella que tiene polígonos de frecuencias
muy simétricos respecto de un valor central de la variable estadística. Y para que esto se
cumpla:
- La media aritmética menos la desviación típica y la media aritmética más la
desviación típica debe estar en un 68,26% del total de los individuos o (x– – σ, x– + σ)
esté entre el 68,26% del total de los individuos.
- La media aritmética menos dos veces la desviación típica y la media aritmética
más dos veces la desviación típica debe estar en un 95,44% del total de los
individuos o (x– – 2σ, x– + 2σ) esté entre el 95,44% del total de individuos.
- La media aritmética menos tres veces la desviación típica y la media
aritmética más tres veces la desviación típica debe estar en un 99,73% del total
de los individuos o (x– - 3σ, –x + 3σ) esté entre el 99’73% del total de individuos.

23
-Pág.55-
M AT E M Á T I C A S

f(x)

Campana de Gauss

µ–σ µ µ+σ x

- Para poder comparar dos datos correspondientes a dos distribuciones distintas, hay
que tipificar o normalizar dichos valores, es decir, calcular los valores de
x–x
z=
σ
- Ejemplo: en una prueba de idioma, Juan obtiene una nota de 7 y en el conjunto
de la clase se tiene una media aritmética de 5,5 y una desviación típica de 2. En otra
prueba saca 6,8 siendo las calificaciones de la clase la media 6 y la desviación
típica 1. ¿cuál de las calificaciones es mejor respecto de la clase?
Para poder comparar ambas calificaciones, lo hacemos a través de sus
correspondientes puntuaciones típicas.
Para x = 7, tenemos z7 = 7 - 5,5 /2 = 0,75
Para x = 6,8 tenemos z6,8 = 6,8 - 6 /1 = 0,8
Al ser mayor la segunda puntuación típica, la segunda calificación puede
considerarse mejor que la primera.

6. DEPENDENCIA ALEATORIA ENTRE DOS VARIABLES


– La variable estadística bidimensional es el estudio unidimensional del conjunto de dos
caracteres o variables estadísticas X e Y sobre una misma población.
– Se denominan distribuciones bidimensionales a las tablas estadísticas bidimensionales
formadas por todas las frecuencias absolutas de todos los posibles valores de la variable
estadística bidimesional (X,Y).
– Las tablas bidimensionales simples adoptan esta forma:

24
-Pág.56-
U . D . 4 . - INTERPRETACIÓN, REPRESENTACIÓN Y TRATAMIENTO DE LA INFORMACIÓN

Variable X Variable Y Frecuencia absoluta


x1 y1 f1
x2 y2 f2
… … …
xi yi fi
… … …
xn yn fn
Σ fi = N

– También existen tablas de doble entrada.


– El diagrama de dispersión es la gráfica que se obtiene al representar en unos ejes
coordenados una distribución bidimensional. A este diagrama se le llama también nube de
puntos.
– Según la disposición de la nube de puntos, se puede apreciar, de forma cualitativa el tipo
y el grado de relación o dependencia entre ambas variables. A esa dependencia se le llama
correlación.
– La dependencia o correlación puede ser:
- Dependencia funcional, si la nube de puntos se sitúa en la gráfica de una función.
- Dependencia lineal, si la nube de puntos se sitúa sobre una recta.
- Correlación o dependencia aleatoria, si la nube de punto se sitúa próxima a la gráfica
de una función.
- Independencia o ausencia de correlación.
– El grado de correlación, a su vez puede ser:
- Correlación fuerte, si la nube de puntos se aproxima mucho a una recta o a una curva.
- Correlación débil, si la nube de puntos se aproxima poco a una recta o a una curva.
- Correlación positiva, si, a medida que crece una variable, crece la otra.
- Correlación negativa, si, a medida que crece una variable, la otra decrece.
– La correlación de tipo lineal se mide mediante un coeficiente universalmente aceptado,
llamado coeficiente de correlación lineal de Pearson, cuyo valor puede calcularse con la
expresión:
r

éstas se corresponden con los conceptos siguientes:

25
-Pág.57-
M AT E M Á T I C A S

ε xi yi
σxy es la covarianza o varianza conjunta de las variables x e y . σxy = −−x . y−
Ν
σx · σy son las desviaciones típicas de las variables marginales x e y respectivamente.
– El coeficiente de correlación lineal de Pearson, r, siempre toma valores entre –1 y 1.
Si –1< r < 0, existe correlación lineal negativa y será más fuerte cuanto más se aproxime r a –1.
Si 0 < r < 1, existe correlación lineal positiva y será más fuerte cuanto más se aproxime r a 1.
Si r = 0 no existe correlación lineal o las variables no están correlacionadas linealmente.
Esto no excluye que las variables estadísticas puedan estar relacionadas por una
correlación curvilínea.
El coeficiente de correlación nos permite cuantificar el grado de aproximación de los
puntos de la nube de una recta. La obtención de la ecuación de esta recta es fundamental
en el estudio de las distribuciones bidimensionales, pues permite la predicción de posibles
resultados. A esta recta la llamamos recta de regresión.
– Entre dos variables hay una relación funcional cuando podemos hallar una fórmula que
permite calcular para cada valor de una de ellas el valor exacto de la otra variable.
– Entre dos variables hay una relación estadística cuando no podemos hallar una fórmula
que permite calcular para cada valor de una de ellas el valor exacto de la otra variable.

RESUMEN

Una función es una correspondencia entre dos variables en la que a cada valor de una de
ellas le corresponde uno y solo uno de la otra.
La gráfica de una función se dibuja sobre unos ejes cartesianos. Una función es continua si
se puede dibujar de un solo trazo, al contrario será discontinua. Una función es creciente si al
aumentar la variable independiente aumenta la dependiente. Al punto donde la función cambia
de creciente a decreciente se le denomina máximo y al contrario mínimo.
Las poblaciones las integran las personas u objetos que quieran estudiarse. Las variables
pueden ser cualitativas y cuantitativas. La parte representativa de una población se le denomina
muestra. Los gráficos estadísticos mas habituales son: los diagramas de barras, los histogramas,
los polígonos de frecuencias, los diagramas de sectores o áreas, los pictogramas y los
cartogramas.
Los parámetros centrales estadísticos son: la media aritmética, la moda y la mediana. Los
parámetros de dispersión son: la desviación típica, la varianza.

26
-Pág.58-
U . D . 4 . - INTERPRETACIÓN, REPRESENTACIÓN Y TRATAMIENTO DE LA INFORMACIÓN

EJERCICIOS DE AUTOCOMPROBACIÓN

1. En las bibliotecas de seis poblaciones se han analizado la afluencia de lectores (X expresada en


miles de personas) y el número de libros prestados (Y), obteniéndose los datos de la tabla:

X 0,50 1 1,3 1,7 2 2,5


Y 180 240 250 300 340 400
¿cuál es el número medio de libros prestados en el conjunto de las bibliotecas?
A. 285 B. 386 C. 360 D. 400

2. Calcule el coeficiente de correlación lineal de Pearson en el problema anterior.


A. 1 B. 0,994 C. 1,5 D. 1,994

3. Se conecta una batería en un circuito en el que existe una resistencia variable y un amperímetro. Al
variar la resistencia la intensidad de la corriente varía como muestra la tabla siguiente: (experiencia
para comprobar la ley de Ohm)

R (ohmios) 2 4 6 8 10 12
I (amperios) 6 3 2 1,5 1,2 1

¿de qué tipo de función se trata?


A. función cuadrática B. función exponencial
C. función de proporcionalidad inversa D. función logarítmica

4. Las simetrías de las funciones cuadráticas son respecto:


A. al origen de coordenadas B. respecto al eje de la parábola
C. respecto al eje de la hipérbola D. ninguna de las anteriores

5. Si tenemos una función y = - x2 + 4x - 6. Los cortes con el eje OY estarán en el punto:


A. (0,6) B. (6,0) C. (0,-6) D. No existen cortes

6. ¿y los cortes con el eje OX? Del ejercicio anterior.


A. (0,4) B. (4,0) C. (0,-4) D. no existen cortes

7. Un empresario embotella aceite que obtiene de mezclar dos aceites de distintas calidades, que
compra a 3,50 euros y a 4,50 euros el litro, respectivamente. El coste de cada litro de una mezcla de
300 l del barato y 100 l del caro es la media ponderada ¿qué valor tendría?
A. 3,75 €/l B 4,25 €/l C. 3,80 €/l D. 3,95€/l

27
-Pág.59-
M AT E M Á T I C A S

8. Calcule la mediana a partir de esta tabla de frecuencias:

Edades 12 13 14 15 16 17 18 19 20 Total
Frec. Absoluta 120 130 120 100 90 60 50 40 10 720
A. 15 B. 14 C. 13 D. 16

9. Calcule la moda de los siguientes datos: 3, 8, 9, 2, 1, 6, 7, 3.


A. 9 B. 6 C. 3 D. 7

10. El número de hijos de 10 familias es: 5, 2, 0, 2, 3, 1, 2, 3, 1, 4. Calcule la media.


A. 2,3 B. 3,3 C. 2,1 D. 2,5

RESPUESTAS A LOS EJERCICIOS

11. A

12. B

13. C

14. B

15. C

16. D

17. A

18. B

19. C

10. A

28
-Pág.60-
portada TROPA 19/3/07 19:51 Página 1

FUERZAS ARMADAS
PROFESIONALES
CURSO DE APOYO
A LA PREPARACIÓN
DE LAS PRUEBAS DE ACCESO
A UNA RELACIÓN DE SERVICIOS
DE CARÁCTER PERMANENTE

MATEMÁTICAS
3ª parte
Unidad didáctica 5

DIGEREM

MINISTERIO
DE DEFENSA
FUERZAS ARMADAS SUBDIRECCIîN GENERAL
DE TROPA Y MARINERIA
PROFESIONAL
PROFESIONALES
CURSO DE APOYO
A LA PREPARACIÓN
DE LAS PRUEBAS DE ACCESO
A UNA RELACIÓN DE SERVICIOS
DE CARÁCTER PERMANENTE

MATEMÁTICAS
3ª parte
Unidad didáctica 5

-Pág.1-
La Ley 8/2006 de Tropa y Marinería, en su artículo 16,1, establece que “la formación
en las Fuerzas Armadas garantizará que los militares profesionales de tropa y
marinería puedan adquirir, actualizar o ampliar sus conocimientos para un mayor
desarrollo personal y profesional”. En cumplimiento de este mandato, el Ministerio
de Defensa edita el presente material didáctico para facilitar a los militares
profesionales de tropa y marinería, alumnos de los cursos de formación
presencial que se imparten a través de la Dirección General de Reclutamiento y
Enseñanza Militar, los apoyos necesarios para preparación de dichos cursos, que
permitirán, siempre que superen las pruebas correspondientes, la obtención de la
titulación de graduado en Educación Secundaria, acreditación para el acceso a
los ciclos formativos de la Formación Profesional de grado medio o de grado
superior, acceso a las Escalas de Suboficiales, Tropa Permanente, Guardia Civil
y Policía Nacional.

CATÁLOGO GENERAL DE PUBLICACIONES


http://www.060.es

Edita:

© Autor y editor
NIPO: 076-10-204-9 NIPO: 076-10-205-4 (edición en línea)
Depósito Legal: M-32363-2009
Diseño y programación: cimapress
Tirada: 1300 ejemplares
Fecha de edición: septiembre, 2010

Prohibida la reproducción total o parcial de esta obra, por cualquier medio sin autorización escrita del editor
MATEMÁTICAS
3ª parte

SUMARIO

Unidad didáctica Pág.

5. TRATAMIENTO DEL AZAR 5

-Pág.3-
U . D . 5 . - T R ATA M I E N T O DEL AZAR

ÍNDICE
OBJETIVOS. . . . . . . . . . . . . . . . . . . . . . . . . . . . . . . . . . . . . . . . . . . . . . . . . . . . . . . . . . . . . . .2

INTRODUCCIÓN . . . . . . . . . . . . . . . . . . . . . . . . . . . . . . . . . . . . . . . . . . . . . . . . . . . . . . . . . .3

MAPA CONCEPTUAL . . . . . . . . . . . . . . . . . . . . . . . . . . . . . . . . . . . . . . . . . . . . . . . . . . . . . .4

DESARROLLO DE CONTENIDOS . . . . . . . . . . . . . . . . . . . . . . . . . . . . . . . . . . . . . . . . . . .5

1. FENÓMENOS ALEATORIOS Y TERMINOLOGÍA . . . . . . . . . . . . . . . . . . . . . . . . .5

2. ASIGNACIÓN DE PROBABILIDAD A SUCESOS . . . . . . . . . . . . . . . . . . . . . . . . . .8

3. ASIGNACIÓN DE PROBABILIDADES EN EXPERIMENTOS COMPUESTOS . . . . .14

RESUMEN. . . . . . . . . . . . . . . . . . . . . . . . . . . . . . . . . . . . . . . . . . . . . . . . . . . . . . . . . . . . . . . .22

EJERCICIOS DE AUTOCOMPROBACIÓN . . . . . . . . . . . . . . . . . . . . . . . . . . . . . . . . . . .23

RESPUESTAS A LOS EJERCICIOS . . . . . . . . . . . . . . . . . . . . . . . . . . . . . . . . . . . . . . . . . .24

-Pág.5-
M AT E M Á T I C A S

O B J E T I VO S
Al finalizar el estudio de esta Unidad Didáctica, el alumno será capaz de:

• Distinguir fenómenos deterministas y aleatorios en situaciones cotidianas,


pudiendo describir los resultados de dichos experimentos.

• Confeccionar tablas de frecuencias para representar los sucesos elementales en


fenómenos aleatorios.

• Construir diagramas de árbol para representar y analizar los sucesos elementales


en fenómenos aleatorios.

• Utilizar el vocabulario básico asociado a los fenómenos en los que interviene el


azar.

• Aplicar principios de recuento para asignar probabilidades.

• Usar la regla de Laplace en casos sencillos

• Resolver problemas sencillos de la vida cotidiana donde aparezcan fenómenos


aleatorios, reconociendo y valorando el papel de las matemáticas para interpre-
tar, describir y predecir situaciones de incertidumbre.

• Diferenciar las situaciones correspondientes a sucesos independientes y depen-


dientes.

-Pág.6-
U . D . 5 . - T R ATA M I E N T O DEL AZAR

INTRODUCCIÓN
D esde la más remota antigüedad, el hombre ha sido consciente de la existencia de
fenómenos cuyo desarrollo se puede predecir, tales como la caída de un cuerpo, los
movimientos de los astros, ...

Asimismo, existen situaciones completamente impredecibles que en la mayoría de los


casos están asociadas a experimentos aleatorios o de azar, como los que se plantean en
los juegos de azar, las expectativas meteorológicas a largo plazo, etc.

Como consecuencia de este hecho diferencial, se pueden considerar dos tipos de fenó-
menos: deterministas y aleatorios.

Se retoman conceptos como los de espacio muestral, sucesos, clases de sucesos, leyes,
probabilidad, además de otros asociados a fenómenos y experimentos aleatorios, y se
asigna un valor numérico a los posibles resultados de un experimento compuesto, calcu-
laremos la probabilidad de un suceso y para ello utilizaremos la regla de Laplace.

-Pág.7-
M AT E M Á T I C A S

M A PA C O N C E P T U A L
EXPERIMENTOS

ALEATORIOS DETERMINISTAS

Dan lugar
UNIÓN
FORMAS ESPACIO
DE CONTAR MUESTRAL
INTERSECCIÓN

DIAGRAMA
DE ÁRBOL Formados por

PRINCIPIO SUCESOS
DE LA S. ELEMENTAL
SUMA
S. SEGURO
PRINCIPIO
DE MULTI- Se miden con S. IMPOSIBLE
PLICACIÓN

S. CONTRARIO
PROBABILIDAD

S. COM-
PUESTO
AXIOMAS
PROPIEDADES
Ley de PROBABILIDAD
Laplace P (E) = 1 PROPIEDAD CONDICIONADA
COMPUESTA O
P (S) ≥ 0 DEL PRODUCTO

P (Ø) = 0
– SUCESOS
P (A) = 1 – P (A)
INDEPENDIENTES
A∩B=Ø
P (A ∪ B) = A∩B≠Ø SUCESOS
P (A) + P (B) P (A ∪ B) = DEPENDIENTES
P (A) + P (B) –
P (A ∩ B)

-Pág.8-
U . D . 5 . - T R ATA M I E N T O DEL AZAR

1. FENÓMENOS ALEATORIOS Y TERMINOLOGÍA


Existen dos tipos de fenómenos, como ya recordarán:
Fenómenos deterministas que son aquellos que se desarrollan de tal modo que, por muchas
veces que se repitan, y siempre que se verifiquen partiendo de las mismas condiciones inicia-
les, terminan con el mismo resultado.
— Si se deja caer una piedra a un pozo, aunque se repita mucho el experimento el resulta-
do será el mismo, la piedra caerá al pozo.
Fenómenos aleatorios, son aquellos que al repetirse varias veces, partiendo de las mismas
condiciones iniciales, pueden dar lugar a distintos resultados.

Ejemplo: al lanzar un dado se pueden obtener diferentes resultados.

1.1. IMPREVISIBILIDAD Y REGULARIDADES EN FENÓMENOS


Y EXPERIMENTOS ALEATORIOS
Aunque los resultados de los fenómenos aleatorios no se pueden predecir, sí nos encontra-
mos con distintos grados de posibilidades de:
— Podemos obtener determinados resultados de un mismo fenómeno aleatorio. Así, por
ejemplo:
– Es más probable obtener un número par ⎨2, 4, 6⎬ que el 2 al lanzar un dado.
– Es igual de probable obtener cara o cruz al lanzar una moneda.
— Podemos obtener o no un resultado en un fenómeno aleatorio. Así, por ejemplo:
– Es muy probable o casi seguro que al lanzar un dado se obtenga un número menor
que 6.
– Es poco probable que al extraer una carta de la baraja se obtenga el as de oros.

1.2. POSIBILIDAD DE REALIZACIÓN DE UN SUCESO


— Se denomina suceso a cada uno de los distintos resultados que puede tener un fenóme-
no aleatorio.
— En el fenómeno aleatorio de lanzar un dado, un suceso podría consistir en obtener una
puntuación superior a tres.

5
-Pág.9-
M AT E M Á T I C A S

— Un suceso elemental es todo aquel que no puede descomponerse en otros más senci-
llos o cada uno de los resultados que se pueden obtener al realizar un suceso aleatorio.
— El conjunto de sucesos elementales de un experimento aleatorio es el espacio muestral
y se representa por la letra Ω.
– En el lanzamiento de un dado los seis sucesos elementales posibles son: obtener un
uno, obtener un dos, obtener un tres, obtener un cuatro, obtener un cinco o bien
obtener un seis, y el espacio muestral Ω = ⎨1, 2, 3, 4, 5, 6⎬
— Los sucesos compuestos son los que están formados por dos o más resultados del
experimento. Por ejemplo en el lanzamiento de un dado obtener un número par,
B = ⎨2, 4, 6⎬
— Un suceso imposible es aquel que nunca puede presentarse como resultado de un fenó-
meno aleatorio o el suceso que no ocurre jamás. Se representa por el símbolo ∅.
– En el lanzamiento de un dado un suceso imposible sería obtener una puntuación
igual o mayor que 7.
— Un suceso seguro es aquel que se ha de obtener forzosamente como resultado de un
fenómeno aleatorio o el suceso que ocurre siempre que se realiza el experimento, y
coincide con el espacio muestral Ω.
– Al lanzar un dado obtener una puntuación menor que 7 es un suceso seguro.
— Se denomina suceso contrario de uno dado, al suceso consistente en que no se verifi-

que aquél. Lo representamos como A. La unión entre un suceso cualquiera y el suceso
contrario es el espacio muestral. Y la intersección es el suceso imposible, esto ocurre
con todo par de sucesos contrarios.
– Si obtenemos una puntuación par en el fenómeno aleatorio de tirar un dado, el suce-
so contrario será evidentemente sacar una puntuación impar.
— Un suceso es subsuceso de otro cuando todos los sucesos elementales que figuran en
el primero, figuran también en el segundo.
– En el fenómeno aleatorio de coger una carta de la baraja, el suceso A: extraer un
caballo es un subsuceso del suceso B extraer una figura.
— Una ventaja que ofrecen los sucesos es que éstos pueden operarse y de esta forma obtene-
mos nuevos sucesos. Si consideramos los sucesos del experimento de un dado: A = ⎨salir
un número impar⎬ = ⎨1, 3, 5⎬ y B = ⎨salir un número mayor que dos⎬ = ⎨3, 4, 5, 6⎬.
— Formamos los sucesos C = ⎨obtener un número impar o mayor que dos⎬ y D= ⎨salir un
número impar y mayor que dos⎬. Los sucesos C y D, llamados unión e intersección de
A y de B, respectivamente, están formados por los siguientes sucesos elementales:
Unión de A y B, C = A U B = ⎨1, 3, 4, 5, 6⎬.

6
-Pág.10-
U . D . 5 . - T R ATA M I E N T O DEL AZAR

Intersección de A y B, D = A ∩ B = ⎨3, 5⎬.


— La unión de dos sucesos A y B es un suceso que ocurre cuando lo hacen A o B. Lo repre-
sentamos por A U B.
— La intersección de dos sucesos A y B es un suceso que se realiza cuando lo hacen A y
B. La representamos por A ∩ B.
— La intersección de dos sucesos, a veces, puede ser el suceso imposible, este es el caso
de A = ⎨salir par⎬ y B = ⎨salir impar⎬. En este caso decimos que A y B son incompati-
bles. En el caso contrario decimos que A y B son compatibles.
— Por lo tanto si tomamos dos sucesos A y B de un mismo experimento aleatorio puede
ocurrir que sean:
Sucesos incompatibles si A ∩ B = Ø
Sucesos compatibles si A ∩ B ≠ Ø

Ejemplo:

Lanzamos un dado de hacer quinielas como el del dibujo y anotamos el resultado que apa-
rece en la cara superior.

Describa el espacio muestral. De igual forma, expresa los suce-


sos elementales, el suceso seguro, el suceso imposible y algunos
sucesos compuestos.

➢ El espacio muestral, que contiene todos los resultados


posibles es:
E = {1, X, 2}

➢ Los sucesos que se citan en el enunciado son:


• Sucesos elementales: {1}, {X}, {2}.
• Suceso seguro: {1, X, 2}.
• Suceso imposible: Ø = {salir 3}.
• Sucesos compuestos: A = : {1, X}; B = : {X, 2}.

7
-Pág.11-
M AT E M Á T I C A S

2. ASIGNACIÓN DE PROBABILIDAD A SUCESOS

2.1. FRECUENCIA Y PROBABILIDAD DE UN SUCESO


Si suponemos que el experimento aleatorio de lanzar al aire una moneda se ha realizado 20
veces y los resultados que se han obtenido han sido los que aparecen a continuación:
– 9 monedas cara y 11 monedas cruz.
El número de veces que se ha obtenido el suceso “sacar cara” y el suceso “sacar cruz” reci-
be el nombre de frecuencia absoluta de cada uno de los sucesos y se obtiene de realizar un
recuento de cada uno de ellos.

Estos resultados se recogen en una tabla denominada tabla de frecuencias, de esta forma:

Resultados del experimento de lanzar 20 veces una moneda


Suceso elemental Recuento N° de veces o frecuencia
Sacar cara IIIIIIIIII 9
Sacar cruz IIIIIIIIIII 11

A partir de estos datos podemos hallar:


– La frecuencia del suceso seguro “sacar cara o cruz” sería 9 + 11 = 20
– La frecuencia del suceso imposible “no sacar cara ni cruz” es 0, pues no se ha presenta-
do ninguna vez este resultado.
El número de veces que ocurre un suceso al realizar un experimento aleatorio en un núme-
ro determinado de ocasiones, se llama frecuencia absoluta del suceso.

Hay otra forma de expresar las frecuencias de los sucesos y se obtienen dividiendo cada una
de las frecuencias absolutas de los sucesos entre el número total de pruebas que se han llevado
a cabo. Estas frecuencias reciben el nombre de frecuencias relativas.

Si calculamos las frecuencias relativas del experimento de lanzar 20 veces un dado, cuyos
resultados han sido los que aparecen en la tabla:

8
-Pág.12-
U . D . 5 . - T R ATA M I E N T O DEL AZAR

Resultados del experimento aleatorio de lanzar 20 veces un dado


Suceso elemental Recuentos Frecuencias absolutas Frecuencias relativas
Saco un uno IIII 4 4/20 = 0,2
Saco un dos IIIIII 6 6/20 = 0,3
Saco un tres III 3 3/20 = 0,15
Saco un cuatro II 2 2/20 = 0,1
Saco un cinco IIIII 5 5/20 = 0,25
Saco un seis 0 0 0/20 = 0

Al emplear la palabra “probable” estamos expresando la posibilidad de que ocurra una situa-
ción de la que no estamos seguros. En este caso es necesario medir el grado de certeza de dicha
situación. Esta medida es la probabilidad.
Si decimos que un suceso X es muy probable que suceda es que estamos casi seguros de que
ocurra ese suceso X. En cambio, decimos que es muy improbable que suceda el suceso Y si cre-
emos firmemente que el suceso Y no ocurrirá.
La probabilidad de un suceso A, en matemáticas se mide asignándole unos valores entre 0 y
1, y lo llamamos P(A).
– Cuando P(A) = 0 significa que el suceso no ocurrirá nunca al realizar el experimento ale-
atorio.
– Cuando P(A) = 1 significa que A ocurrirá con absoluta seguridad.
Dado un suceso A asociado a un experimento aleatorio, la probabilidad P(A) es el valor al
que tienden las frecuencias relativas de A al aumentar el número de realizaciones del experi-
mento.
Al considerar un suceso correspondiente a un determinado fenómeno aleatorio, se diferen-
cian dos tipos de sucesos elementales:
— Casos favorables, son los sucesos elementales en los que puede descomponerse el
suceso dado.
— Casos posibles, son sucesos elementales en los que puede descomponerse el suceso
seguro correspondiente.

Ejemplo: en el fenómeno aleatorio de lanzar un dado. Si consideramos el suceso con-


sistente en obtener una puntuación par, se puede obtener un dos, un cuatro, un seis, son
los casos favorables, mientras que los casos posibles son obtener un uno, un dos, un
tres, un cuatro, un cinco, un seis.

9
-Pág.13-
M AT E M Á T I C A S

La probabilidad de un suceso es el cociente que resulta de dividir el número de casos favo-


rables entre el número de casos posibles.
p = n/N

siendo p la probabilidad, n el número de casos favorables y N el número de casos posibles.

Ejemplo: consideremos el suceso consistente en que, al extraer una carta de la baraja


española, se obtenga una figura.
Casos favorables: el n.° total de figuras de la baraja n =12
Casos posibles: la totalidad de cartas N = 40
P = n/N = 12/40 = 3/10

— Propiedades de la probabilidad:
– La probabilidad de un suceso cualquiera está comprendida entre 0 y 1.
– La probabilidad de un suceso imposible es nula.
– La probabilidad de un suceso seguro es la unidad.
– Si A es un subsuceso de B y A ≠ B, la probabilidad del suceso A es menor que la del
suceso B.
– La probabilidad de un suceso más la probabilidad de su contrario es igual a la uni-
dad.
– Si la probabilidad de un suceso es p, la probabilidad de realización de su contrario,
que se representa por q, es: q = 1 – p.
– La probabilidad del suceso unión de dos sucesos incompatibles es igual a la suma
de las probabilidades de cada uno de ellos. pu = pa + pb + pc +... este es el concep-
to de probabilidad total.

Ejemplo: en el fenómeno aleatorio de lanzar un dado, consideremos dos sucesos


incompatibles:
A: obtener una puntuación par
B: obtener un tres

{
casos favorables: 2, 4, 6 (n =3)
Suceso A: Casos posibles: 1, 2, 3, 4, 5, 6 (N = 6)
PA = 3/6

10
-Pág.14-
U . D . 5 . - T R ATA M I E N T O DEL AZAR

{
casos favorables: 3 (n =1)
Suceso B: Casos posibles: 1, 2, 3, 4, 5, 6 (N = 6)
PB = 1/6

Suceso unión:
pu = PA + PB

— Axiomas de la probabilidad
– La probabilidad del suceso seguro o espacio muestral es 1.
– Cualquiera que sea el suceso S, su probabilidad es un número positivo o cero.
– Si dos sucesos A y B son incompatibles, A ∩ B = ∅, la probabilidad del suceso
unión, A ∪ B, es la suma de las probabilidades de los sucesos A y B.

2.2. LEY DE LAPLACE


Si consideramos el lanzamiento de una moneda. Parece lógico que la mitad de las tiradas
tengan como resultado cara y la otra mitad cruz. A esta situación se le denomina situación de
equiprobabilidad.
Diremos que estamos en una situación de equiprobabilidad cuando en un experimento alea-
torio se puede suponer que los diferentes sucesos elementales son igualmente posibles.
En una situación de equiprobabilidad se puede calcular la probabilidad de cualquier suceso
A mediante la llamada Ley de Laplace.
Cuando los sucesos elementales son equiprobables, la probabilidad del suceso A se calcula
como el cociente entre el número de casos favorables al suceso A y el número de casos posi-
bles. Se representa por P(A).
P(A) = Número de resultados favorables de A/Número de resultados posibles
Siempre que vayamos a aplicar la regla de Laplace, debemos tener en cuenta que:
– Sólo puede aplicarse cuando los sucesos elementales del experimento aleatorio son equi-
probables.
– Los casos posibles son los resultados del experimento; es decir, todos los sucesos ele-
mentales que componen el espacio muestral.
– Los casos favorables son todos los sucesos elementales que componen el suceso dado,
del cual queremos calcular su probabilidad.

11
-Pág.15-
M AT E M Á T I C A S

Ejemplo: Disponemos de una bolsa con doce bolas numeradas del 1 al 12. Realizamos
el experimento que consiste en extraer una bola, observarla y anotar su número. Cal-
cula la probabilidad de obtener:
A un número impar.
B un número primo.
El espacio muestral es E = [ 1, 2, 3, 4, 5, 6, 7, 8, 9, 10, 11, 12] y los sucesos cuya pro-
babilidad nos piden:
A = [ obtener n.° impar ] = [ 1, 3, 5, 7, 9, 11] P(A) = 6/12 = 1/2
B = [ obtener n.° primo ] = [ 1, 2, 3, 5, 7, 11] P(B) = 6/12 = 1/2

En resumen:
– La probabilidad de un suceso elemental es:
P (suceso elemental) = 1/ n.º de sucesos elementales E

– En el caso de considerar un suceso cualquiera, su probabilidad esta asociada a la proba-


bilidad de los sucesos elementales que lo forman. Así para el suceso A, se tiene:
P (A) = n.º de sucesos elementales A/n.º sucesos elementales E

En el lanzamiento de un dado, la probabilidad de cada uno de los sucesos elementales es


1/6. Si consideramos el suceso A, obtener n.º par, formado por A = ⎨2, 4, 6⎬, su proba-
bilidad es:
P (A) = P ⎨2, 4, 6⎬ = P(2) + P(4) + P(6) = 1/6 + 1/6 + 1/6 = 1/2

A partir de la última definición, podemos enlazar con la conocida regla de Laplace; que dice
así:
– Cuando los sucesos elementales son equiprobables, la probabilidad de un suceso A,
se calcula como el cociente entre el número de casos favorables al suceso A y el
número de casos posibles.

Ejemplo: Extraemos una carta de la baraja española. Calcula las siguientes probabilidades:
a. La carta extraída sea de espadas.
P(espadas) = 10 /40 = 1/4
b. La carta extraída sea sota o caballo.
P(sota o caballo) = 8/40 = 1/5
c. La carta extraída no sea un as.
P(no as) = 36/40 = 9/10.

12
-Pág.16-
U . D . 5 . - T R ATA M I E N T O DEL AZAR

2.3. EXPERIMENTOS DEPENDIENTES E INDEPENDIENTES


— Dos sucesos son dependientes cuando el resultado de uno de ellos influye en el resul-
tado del otro.

Ejemplo: En el fenómeno aleatorio de extraer cinco cartas de una baraja, considere-


mos los sucesos:
A: que las dos primeras cartas sean oros.
B: que de las cinco cartas, al menos cuatro sean oros.
Estos dos sucesos son dependientes, ya que el resultado del primer suceso influye
notablemente en el éxito o fracaso del segundo suceso.

— Dos sucesos son independientes cuando el resultado de uno de ellos no influye en el


resultado del otro.

Ejemplo: Al lanzar dos veces un dado, consideremos los sucesos:


A: que la puntuación del primer lanzamiento sea par.
B: que la puntuación del segundo lanzamiento sea tres.
Estos dos sucesos son independientes, puesto que el resultado del primer suceso no
influye en el segundo.

— Para dos experimentos A y B que se efectúan consecutivamente se dice que:


– Si A y B son independientes, se cumple:
P (A y B) = P(A ∩ B) = P(A) x P(B)
– Si A y B son dependientes, se cumple:
P (A y B) = P(A ∩ B) = P(A) x P(B supuesto que ha ocurrido A)

Ejemplo: Calcule la probabilidad de que al extraer dos cartas, sucesivamente, de una


baraja española de 40 cartas , sean espadas.
P (E1 ∩ E 2) = P(E1) . P(E2/E1 ) = 10/40 x 9/39 = 0,0577
Si la carta extraída se vuelve a introducir, los sucesos son independientes y la pro-
babilidad buscada es:
P (E1 ∩ E 2 ) = P(E1) . P(E2 ) = 10/40 x 10/40 = 0,0625

13
-Pág.17-
M AT E M Á T I C A S

3. ASIGNACIÓN DE PROBABILIDADES EN EXPERIMENTOS


COMPUESTOS
Llamamos experimentos compuestos a varios formados por experimentos simples que se
efectúan consecutivamente.
Si lanzamos tres veces al aire un dado o bien tres dados simultáneamente, ambas experien-
cias son equivalentes, observamos en cada dado si sale 5 o no sale 5. El diagrama de árbol aso-
ciado a este experimento compuesto es:

En el experimento compuesto anterior las probabilidades de los sucesos (obtener tres cin-
cos), (obtener en los dos primeros 5 y en el último no) y (obtener dos 5 y un no cinco) son:

14
-Pág.18-
U . D . 5 . - T R ATA M I E N T O DEL AZAR

– P (obtener tres cincos) = P (5, 5, 5) = 1/6 x 1/6 x 1/6 = 1/216


– P(obtener en los dos primeros cinco y en el último no) = P (5, 5, no 5) = 1/6 x 1/6 x
x 5/6 = 5/216
– P (obtener dos 5 y un no cinco) = P (5, 5, no 5) + P (5, no 5, 5) + P (no 5, 5, 5) = 1/6 x
x 1/6 x 5/6 + 1/6 x 5/6 x 1/6 + 5/6 x 1/6 x 1/6 = 15/216
La probabilidad de un suceso en un experimento compuesto es igual al producto de las pro-
babilidades de los sucesos que componen este suceso compuesto.

Nota:
Dados dos sucesos compatibles e independientes, la probabilidad de que ambos se verifi-
quen simultáneamente es igual al producto de las probabilidades de cada uno de ellos, a esto se
le denomina probabilidad compuesta.
La diferencia entre la probabilidad total y la probabilidad compuesta es que la primera se
refiere a sucesos incompatibles, siendo suficiente que se verifique uno de ellos, mientras que la
segunda se refiere a sucesos compatibles e independientes, que han de verificarse simultánea-
mente.
Sean A y B dos sucesos compatibles e independientes; nA y nB, el número de sucesos ele-
mentales respectivos; y N, el número total de sucesos posibles.
– Según sabemos, la probabilidad del suceso contrario es q = 1 – p
– En consecuencia, la probabilidad de que no se verifique A es:
q A = 1 – pA

– Y la probabilidad de que no se verifique B es: q B = 1 – pB


– La probabilidad de que no se verifique ni el suceso A ni el suceso B es:
q = qA · qB

– El suceso A ∪ B, consistente en que se verifique al menos uno de los dos, es el contrario.


– La probabilidad de la unión de dos sucesos compatibles e independientes PU ,es igual a:
PU = PA + PB – P A · P B

Un diagrama de árbol es un gráfico que empleamos para representar todas las posibilidades
lógicas de una situación que presenta un número finito de variantes. Los diagramas de árbol
pueden ser de dos tipos:
– Diagrama de árbol regular: En cada columna sale el mismo número de ramas de cada
nudo.
– Diagrama de árbol irregular: Hay columnas en las que el número de ramas salientes
de cada nudo no siempre es el mismo.

15
-P·g.19-
M AT E M Á T I C A S

En resumen, llamamos experimentos compuestos a los formados por varios experi-


mentos que se efectúan consecutivamente. El conjunto de todos los resultados elementales
que tienen lugar en un experimento compuesto, forma el espacio compuesto o espacio pro-
ducto. La probabilidad de un suceso elemental de un espacio compuesto puede calcularse
multiplicando las probabilidades de los sucesos elementales que conforman la experiencia
compuesta.

Ejemplo: Imagínese que la probabilidad de nacer varón es de 0,46. De una familia


con 3 hijos, calcula la probabilidad de que:

a) Los tres sean varones.


b) Ninguno sea varón.
a) P (3 varones) = P (varón) · P (varón) · P (varón) = 0,46 · 0,46 · 0,46 = 0,0973
b) P (ningún varón) = P (3 mujeres) = P (mujer) · P (mujer) · P (mujer) = 0,54 · 0,54 · 0,54 =
= 0,1574 P (mujer)

Las tablas de contingencia representan también todas las posibilidades, de los sucesos que
conforman dichas tablas y sus intersecciones. Para los fenómenos estudiados de ahora en ade-
lante adoptaremos la siguiente notación: una de las características estudiadas se corresponderá
– –
con los sucesos A y no A (A) y la otra se dividirá entre los sucesos B y no B (B). A partir de esta
notación, la tabla de contingencia correspondiente responderá al siguiente esquema:

A NO A TOTALES
B A∩B NO A ∩ B
NO B A ∩ NO B NO A ∩ NO B
TOTALES N

Ejemplo: En tres cursos de un mismo nivel en un instituto hay 65 alumnos entre chicos y
chicas. En una evaluación los aprobados y suspensos fueron los siguientes: de las 40 chicas, 20
aprobaron y 20 suspendieron y de los 25 chicos, 15 aprobaron y 10 suspendieron.

CHICOS CHICAS TOTALES


APROBADOS 15 20 35
SUSPENSOS 10 20 30
TOTALES 25 40 65

16

-Pág.20-
U . D . 5 . - T R ATA M I E N T O DEL AZAR

3.1. PROBABILIDAD CONDICIONADA


En la situación del ejemplo anterior se pueden calcular las siguientes probabilidades:
– La probabilidad de que un estudiante sea chico y este aprobado.
P (chico ∩ aprobado) = 15/65 = 0,23

– La probabilidad de que un chico este aprobado.


P (aprobado/chico) = 15/25 = 15/65 =
25/65

= P (chico ∩ aprobado)/P (chico) = 0,23/0,38 = 0,6

El suceso (aprobado/chico) se denomina suceso condicionado y la probabilidad correspon-


diente probabilidad condicionada.
Se llama probabilidad condicionada del suceso B respecto del (condicionado al) suceso A,
y lo denotamos P (B/A) al cociente:
P (B/A) = P (A ∩ B)/P (A)

La probabilidad compuesta o del producto resulta de despejar la probabilidad condicionada


de la fórmula anterior:
P (A ∩ B) = P (A)·P (B/A)

Ejemplo: Hemos realizado una encuesta a 50 personas sobre el color de los ojos y el color
del pelo. Los resultados obtenidos son los siguientes:
OJOS

CLAROS OSCUROS TOTALES


PELO RUBIO 14 16 30
PELO MORENO 8 12 20
TOTALES 22 28 50
8/50
P (CLAROS/MORENO) = P (CLAROS ∩ MORENO)/P (MORENO) = = 8/20 = 0,4
20/50

8/50
P (MORENO/CLAROS) = P (MORENO ∩ CLAROS)/P (CLAROS) = = 8/22 = 0,36
22/50

17
-Pág.21-
M AT E M Á T I C A S

A continuación veremos algunas técnicas que serán de gran utilidad para contar el número
de resultados posibles de ciertos experimentos aleatorios. (Técnicas de recuento).
— Los diagramas de árbol, que anteriormente hemos definido, ya que si se tienen n ele-
mentos de cierto tipo, y a cada uno de ellos se le asignan m elementos de un segundo
tipo, el número total de pares que podemos formar es n · m.

Ejemplo: El menú del día de un restaurante consta de 4 primeros platos, 5 segundos


platos y 3 postres entre los que puede elegir el cliente. ¿Cuántos menús distintos se
pueden formar?

Este es el diagrama correspondiente al primer plato, para completar todo el diagrama


de árbol tendríamos que hacer lo mismo para el segundo, tercero y cuarto ya que, hay
para elegir 4 primeros platos. Puede seguir completándolo.

Se observa que, elegido un primer plato, hay 5 posibilidades para el segundo y, elegi-
dos el primero y el segundo, hay tres posibilidades para el postre. Por tanto, el número
de menús diferentes es:

4 x 5 x 3 = 60

— Las Variaciones con repetición de n elementos tomados de m en m son los distintos


grupos de m elementos que se pueden formar con los n elementos de modo que:

18
-Pág.22-
U . D . 5 . - T R ATA M I E N T O DEL AZAR

– Aparezcan o no elementos repetidos.


– Un grupo se distinga de otro por los elementos que contiene o por su colocación.
– Se representan por VRn,m

– Se calculan: VRn,m = nm

Ejemplo: Disponemos de una bolsa con 3 bolas de colores diferentes; una roja (R), una
negra (N) y una blanca (B). Sacamos al azar dos de estas bolas, una después de la otra,
con devolución. ¿Cuál es el número de resultados posibles?
Se trata de variaciones con repetición de 3 elementos tomados de 2 en 2 se escribe VR3,2

VR3,2 = 32 = 9

— Las Variaciones ordinarias o sin repetición de n elementos tomados de m en m son los


distintos grupos de m elementos que se pueden formar con los n elementos de modo que:
– No aparezcan elementos repetidos.
– Un grupo se distingue de otro por contener elementos diferentes o los mismos ele-
mentos en orden distinto.
– Se representan por Vn,m.
– Se calculan multiplicando m factores consecutivos decrecientes a partir de n:

Vn,m = n (n – 1) · (n – 2)…(n – m + 1)

Ejemplo: Supongamos que en la experiencia anterior, las dos bolas se extraen una des-
pués de la otra sin devolución. ¿Cuál es ahora el número de resultados posibles?
Se trata de variaciones sin repetición de 3 elementos tomados de 2 en 2. Y se escri-
be V3,2.

V3,2 = 3 · 2 = 6

— Si se observa que m = n , los diferentes resultados sólo se distinguen en el orden de sus


elementos; son las diferentes ordenaciones de los n elementos. A estas ordenaciones se
les llama Permutaciones de n elementos y se representan por Pn.

Pn = n · (n – 1) · (n – 2) · … · 1

19
-Pág.23-
M AT E M Á T I C A S

Ejemplo: Nos preguntamos ¿Cuántos números de 4 cifras pueden formarse con los
dígitos 1, 2, 3, 4 de modo que no se repita ninguna cifra?
P4 = V4,4 = 4 · 3 · 2 · 1 = 24

Nota: Para un número natural n mayor que 1, llamamos factorial de n al producto de los
n primeros números naturales. El factorial de n se representa por n!

n! = n · (n – 1) · (n – 2) · (n – 3) · … · 3 · 2 · 1

Para n = 0 y n = 1, definimos 0! = 1 y 1! = 1.
Expresión factorial del número de variaciones sin repetición:

Vn,m = n !/(n – m) !

Pn = Vn, n = n !

En su calculadora para calcular el factorial de un número por ejemplo 8! Pulsamos 8


SHIFT X ! = 40. 320 o también 8 X !EXE = 40. 320
— Las Combinaciones de n elementos tomados de m en m son los distintos grupos de m
elementos que se pueden formar de modo que:
– No aparezcan elementos repetidos.
– Un grupo se distingue de otro por los elementos que contiene y no por su coloca-
ción.
– Se representan por Cn,m
– Se calculan:

Cn, m = Vn, m/Pm

Ejemplo: Supongamos (del ejemplo anterior) que se extraen dos bolas simultánea-
mente, es decir al mismo tiempo. ¿Cuantos resultados posibles habrá ahora?
C3,2 = V3,2/P2 = 3 · 2/2 = 3
n
El número Cn,m se llama también número combinatorio. Se representa por m ( )

20
-Pág.24-
U . D . 5 . - T R ATA M I E N T O DEL AZAR

ORGANIGRAMA

COMBINATORIA
PROBABILIDAD

Variaciones Variaciones Permutaciones Combinaciones Combinaciones


con repetición ordinarias ordinarias con repetición ordinarias

⎛ n⎞ n!
VR n, r = n r n! Pn = n! C n, r = ⎜ ⎟ =
Vn, r = ⎝ r ⎠ ( n ± r )!⋅r!
( n ± r )!

Vn, r = n( n ± 1) ⋅ ( n ± 2) ⋅ ( n ± r + 1) ⎛ n + r ± 1⎞ ( n + r ± 1)!
CR n, r = ⎜ ⎟=
⎝ r ⎠ r!( n ± 1)!

EJERCICIOS RESUELTOS
1. Tenemos una baraja de 40 cartas y se extraen 4 cartas con devolución en cada extrac-
ción. Calcula el número diferente de obtener 4 cartas.
Para las 4 extracciones se disponen de 40 cartas y podemos repetir la carta extraída con
anterioridad. Son variaciones con repetición.

VR 40,4 = 40 4 = 2560000

2. En una carrera de maratón participan 12 atletas. ¿De cuántas formas pueden llegar los
tres primeros a la meta?
Si designamos a los 12 atletas con las 12 primeras letras del abecedario:
A ,B ,C ,D ,E ,F ,G ,H ,I ,J ,K ,L
Entre otras diferentes formas de llegar a la meta de los corredores nos podemos encontrar:
A, B, C B, C, A C, B, A
A, C, B B, A, C C, A, B

Luego la llegada de tres mismos corredores difiere según el puesto de colocación en la meta.
V12,3 = n ( n − 1) ⋅ ( n − 2)....( n − m + 1) n! 12!
V12,3 = = = 1320
V12,3 = 12(12 − 1) ⋅ (12 − 2) = 1320
( ) (
n – r ! 12 – 3 ! )

21
-Pág.25-
M AT E M Á T I C A S

Si no nos importase el puesto de colocación en la meta de los diferentes corredores,


y la llegada de los mismos corredores se considerase como una única forma sería
una combinación.
3. Dado un conjunto {1, 2, 3, 4} calcular el número permutaciones.
Número de permutaciones: como el conjunto dado consta de una total de cuatro ele-
mentos, es claro que:
n=4
Sustituyendo este valor en la fórmula, resulta:
P4 = 4! = 4 · 3 · 2 · 1 = 24
4. Dados los elementos a, a, b, b, c; calcule el número de permutaciones con repetición.
Se dispone de un total de 5 elementos, de los cuales “a” figura 2 veces, el “b” también
dos veces y el “c” una sola vez:
n=5;α=2;β=2;λ=1
si se sustituye en la fórmula:

5! 120
PR52,2,1 = = = 30
2!⋅2!⋅1! 4

5. A una reunión acuden 30 personas. Se decide constituir comisiones de seis personas


para estudiar un cierto plan. ¿Cuántas comisiones diferentes se pueden formar?

n! 30!
C30,6 = = = 593
593775
.775
(n – r )!⋅r! (30 – 6)!⋅6!

RESUMEN
Un fenómeno o experimento es aleatorio si es imposible predecir el resultado cada vez que
sucede o se realiza, y determinista cuando sí se puede predecir.
– Un suceso elemental es cada uno de los posibles resultados que se pueden obtener en un
experimento aleatorio.
– El conjunto formado por todos los sucesos elementales asociados a un experimento ale-
atorio es su espacio muestral (E).
– Todo subconjunto del espacio muestral asociado a un experimento aleatorio es un suceso.

22
-Pág.26-
U . D . 5 . - T R ATA M I E N T O DEL AZAR

– Un suceso seguro (E) es aquel suceso que se presenta siempre que se realiza un experi-
mento aleatorio.
– Un suceso imposible (∅) es aquel suceso que no se presenta nunca.
– Si dos sucesos se pueden realizar simultáneamente, son compatibles, y si no es posible
que se realicen a la vez, son incompatibles.
– Dos sucesos son contrarios o complementarios, si son incompatibles y siempre que no
se realiza uno de ellos se realiza el otro.
– La frecuencia absoluta de un suceso es el número de veces que se ha presentado.
– La frecuencia relativa de un suceso es el cociente de la frecuencia absoluta entre el
número de pruebas realizadas.
– Los sucesos equiprobables son aquellos que tienen la misma posibilidad de producirse.
– Por la Ley de Laplace, la probabilidad de un suceso es igual al resultado del cociente:
número de casos favorables entre el de casos posibles.

EJERCICIOS DE AUTOCOMPROBACIÓN

1. Indique cuál de los siguientes fenómenos no es aleatorio:

A. La congelación del agua B. Las quinielas de fútbol

C. El juego de la oca D. La lotería de Navidad

2. La siguiente tabla contiene las frecuencias relativas de los sucesos cara y cruz a lo largo de 900 rea-
lizaciones del experimento de lanzar una moneda. Se han anotado los resultados de 100 en 100 tira-
das. ¿Cuál es la probabilidad de los sucesos cara y cruz?

Lanzamientos
Suceso
100 200 300 400 500 600 700 800 900
Cara 0,560 0,535 0,473 0,488 0,490 0,493 0,490 0,496 0,497

Cruz 0,440 0,465 0,527 0,512 0,510 0,507 0,510 0,504 0,503

A. P (Cara) = 0,6 P (Cruz) = 0,4 B. P (Cara) = 0,4 P (Cruz) = 0,6

C. P (Cara) = 0,5 P (Cruz) = 0,5 D. P (Cara) = 1,0 P (Cruz) = 1,0

23
-Pág.27-
M AT E M Á T I C A S

3. Hagamos el experimento siguiente: se lanza un dado y se mira el resultado. Halla la probabilidad


de: A = Sacar un uno; B = Sacar más de cuatro y C = Sacar un número par. Se supone que estamos
en una situación de equiprobabilidad.
A. P (A) = 1/3; P (B) = 1/2 ; P (C) =1/6 B. P (A) = 1/6; P (B) = 1/3; P (C) = 1/2
C. P (A) = 1/3; P (B) = 1/6; P (C) = 1/2 D. P (A) = 1/6; P (B) = 1/2; P (C) = 1/3

4. Se lanzan dos dados, ¿Cuál es la probabilidad del suceso A: obtener puntuaciones distintas en ambos
dados?
A. P(A) = 1/2 B. P(A) = 1 C. P(A) = 1/6 D. P(A) = 5/6

5. ¿Cuántos equipos de baloncesto pueden formarse con 11 jugadores?


A. 462 B. 504 C. 624 D. 405

6. ¿Cuántas palabras distintas, tengan o no sentido, se pueden formar con todas las letras de la palabra
ÁRBOL?
A. 12 B. 25 C. 120 D. 21

7. Extraemos dos cartas de la baraja española. Calcula la probabilidad de que la primera sea un caba-
llo y la segunda un rey, si la primera carta no se devuelve a la baraja después de su extracción.
A. 2/1560 B. 2/195 C. 16/195 D. 39/40

8. En un hotel hay 3 cajas fuertes. En una de ellas hay 6 joyas buenas y 2 falsas; en otra, 5 joyas de
valor y 1 falsa; y en la tercera hay 6 joyas valiosas y 3 falsas. Suponiendo que un ladrón sólo puede
abrir una caja fuerte y llevarse una joya, ¿cuál es la probabilidad de que se lleve bisutería?
A. 0,25 B. 0,50 C. 0,75 D. 0

9. Los conjuntos A = ⎨1, 3 , 5⎬ y B = ⎨2, 4⎬ son sucesos del juego tirar un dado. Hallar A – B:
A. ⎨2, 4⎬ B. ⎨1, 2, 3, 4, 5, 6⎬
C. ⎨1, 3, 5⎬ D. Ninguna de las anteriores es correcta

10. La probabilidad de que un alumno apruebe Lengua Española es 4/5; y la probabilidad de que aprue-
be Matemáticas, 7/10. ¿Cuál es la probabilidad de que apruebe al menos una de las dos asignaturas?
A. P = 47/50 B. P=1 C. P = 1/2 D. P=0

RESPUESTAS A LOS EJERCICIOS

1. A 2. C 3. B 4. D 5. A

6. C 7. B 8. A 9. C 10. A

24
-Pág.28-
portada TROPA 19/3/07 19:51 Página 1

FUERZAS ARMADAS
PROFESIONALES
CURSO DE APOYO
A LA PREPARACIÓN
DE LAS PRUEBAS DE ACCESO
A UNA RELACIÓN DE SERVICIOS
DE CARÁCTER PERMANENTE

TECNOLOGÍA
1ª parte
Unidades didácticas 1 y 2

DIGEREM

MINISTERIO
DE DEFENSA
FUERZAS ARMADAS SUBDIRECCIîN GENERAL
DE TROPA Y MARINERIA
PROFESIONAL
PROFESIONALES
CURSO DE APOYO
A LA PREPARACION
Ó
DE LAS PRUEBAS DE ACCESO
A UNA RELACIÓN DE SERVICIOS
DE CARÁCTER PERMANENTE

TECNOLOGÍA
1ª parte
Unidades didácticas 1 y 2
La Ley 8/2006 de Tropa y Marinería, en su artículo 16,1, establece que “la formación
en las Fuerzas Armadas garantizará que los militares profesionales de tropa y
marinería puedan adquirir, actualizar o ampliar sus conocimientos para un mayor
desarrollo personal y profesional”. En cumplimiento de este mandato, el Ministerio
de Defensa edita el presente material didáctico para facilitar a los militares
profesionales de tropa y marinería, alumnos de los cursos de formación
presencial que se imparten a través de la Dirección General de Reclutamiento y
Enseñanza Militar, los apoyos necesarios para preparación de dichos cursos, que
permitirán, siempre que superen las pruebas correspondientes, la obtención de la
titulación de graduado en Educación Secundaria, acreditación para el acceso a
los ciclos formativos de la Formación Profesional de grado medio o de grado
superior, acceso a las Escalas de Suboficiales, Tropa Permanente, Guardia Civil
y Policía Nacional.

CATÁLOGO GENERAL DE PUBLICACIONES


http://www.060.es

Edita:

© Autor y editor
NIPO: 076-10-204-9 NIPO: 076-10-205-4 (edición en línea)
Depósito Legal: M-32363-2009
Diseño y programación: cimapress
Tirada: 1300 ejemplares
Fecha de edición: septiembre, 2010

Prohibida la reproducción total o parcial de esta obra, por cualquier medio sin autorización escrita del editor
TECNOLOGÍA
1ª parte

SUMARIO

Unidad didáctica Pág.

1. PROCESOS DE RESOLUCIÓN TÉCNICA 5


DE PROBLEMAS

2. EXPLORACIÓN Y COMUNICACIÓN DE IDEAS 37


U. D. 1 - P RO C E S O D E R E S O L U C I Ó N T É C N I C A D E P RO B L E M A S

ÍNDICE
Pag.

OBJETIVOS . . . . . . . . . . . . . . . . . . . . . . . . . . . . . . . . . . . . . . . . . . . . . . . . . . . . . . . . . . . 2

INTRODUCCIÓN . . . . . . . . . . . . . . . . . . . . . . . . . . . . . . . . . . . . . . . . . . . . . . . . . . . . . . 3

MAPA CONCEPTUAL . . . . . . . . . . . . . . . . . . . . . . . . . . . . . . . . . . . . . . . . . . . . . . . . . . 4

DESARROLLO DE CONTENIDOS . . . . . . . . . . . . . . . . . . . . . . . . . . . . . . . . . . . . . . . 5

1. PROBLEMAS Y NECESIDADES HUMANAS . . . . . . . . . . . . . . . . . . . . . . . . . 5


1.1. OBJETOS . . . . . . . . . . . . . . . . . . . . . . . . . . . . . . . . . . . . . . . . . . . . . . . . . . . . 5
1.2. FUENTES DE ENERGÍA E INSTALACIONES . . . . . . . . . . . . . . . . . . . . . . . . 6
1.3. AMBIENTES ARTIFICIALES . . . . . . . . . . . . . . . . . . . . . . . . . . . . . . . . . . . . 13

2. PROYECTO TÉCNICO . . . . . . . . . . . . . . . . . . . . . . . . . . . . . . . . . . . . . . . . . . . 16
2.1. FASES DE UN PROYECTO TÉCNICO . . . . . . . . . . . . . . . . . . . . . . . . . . . . . . 16

3. DISEÑO . . . . . . . . . . . . . . . . . . . . . . . . . . . . . . . . . . . . . . . . . . . . . . . . . . . . . . . . 20
3.1. MÉTODOS Y TIPOS DE DISEÑO . . . . . . . . . . . . . . . . . . . . . . . . . . . . . . . . . . 20
3.2. ANÁLISIS DEL PRODUCTO . . . . . . . . . . . . . . . . . . . . . . . . . . . . . . . . . . . . . 21

RESUMEN . . . . . . . . . . . . . . . . . . . . . . . . . . . . . . . . . . . . . . . . . . . . . . . . . . . . . . . . . . . 24

EJERCICIOS DE AUTOCOMPROBACIÓN . . . . . . . . . . . . . . . . . . . . . . . . . . . . . . . 25

RESPUESTAS A LOS EJERCICIOS . . . . . . . . . . . . . . . . . . . . . . . . . . . . . . . . . . . . . 29

-Pág.5-
T ECNOLOGÍA

OBJETIVOS
Al finalizar el estudio de ésta Unidad Didáctica, el alumno será capaz de:

• Distinguir los ingenios técnicos de cada época.

• Describir las ventajas e inconvenientes que surgen para el ser humano debi-
do al desarrollo tecnológico.

• Describir por qué son importantes los proyectos técnicos.

• Identificar las fases de un proyecto técnico.

• Distinguir los tipos y métodos de diseño de objetos fabricados para el ser


humano.

• Observar las implicaciones de las instalaciones y ambientes en el diseño.

-Pág.6-
U. D. 1 - P RO C E S O D E R E S O L U C I Ó N T É C N I C A D E P RO B L E M A S

INTRODUCCIÓN
U na de las cosas que diferencia al ser humano de los animales es su afán por
investigar soluciones que resuelvan los problemas que le rodean.

La evolución de los ingenios tecnológicos se ha producido basándose precisamente


en ese afán de investigar y en los conocimientos de cada época.

La segunda revolución industrial generó objetos simples e impersonales, que no


indican nada acerca del operario que los fabricaba ni tampoco de las aficiones y gus-
tos de las personas que lo utilizaban.

Actualmente los productos deben ser suficientemente atractivos como para encon-
trar el número suficiente de compradores que amorticen las inversiones realizadas,
con lo cual todo producto debe pasar de antemano por un diseño.

En la actualidad el acceso a la información técnica existente ha originado avances


técnicos vertiginosos. En función de todo esto, el ser humano está continuamente
buscando ideas para llevar a la práctica. El alto coste económico que esto supone
hace necesario el desarrollo de una serie de fases, y procesos que, plasmados sobre
un papel, constituyen un proyecto técnico.

-Pág.7-
T ECNOLOGÍA

M A PA C O N C E P T UA L

SER HUMANO

PROBLEMA NECESIDAD

PROYECTO

PUEDE GENERAR

DISEÑO

NO
SOLUCIÓN

SI

OBJETIVO

-Pág.8-
U. D. 1 - P RO C E S O D E R E S O L U C I Ó N T É C N I C A D E P RO B L E M A S

1. PROBLEMAS Y NECESIDADES HUMANAS

El hombre, ya desde la prehistoria, se ha encontrado con problemas para abastecer sus


necesidades y a la vez ha ido descubriendo nuevas posibilidades.

En su afán de mejorar, de adaptarse a los nuevos tiempos y de hacer su vida mucho más
cómoda, ha ido creando objetos que le ayuden a solucionar lo diversos problemas, descu-
briendo nuevas fuentes de energía que él ha ido adaptando en instalaciones más o menos
complejas que a su vez le han llevado a crear ambientes artificiales.

1.1. OBJETOS

Podemos distinguir cronológicamente una serie de etapas en la historia donde el hom-


bre ha creado objetos que han hecho cambiar sustancialmente su forma de vida.

3.000.000 a de C.
PREHISTORIA
Objetos tallados en piedra
Fuego
Objetos de cobre
Azada, Hoz, Tejido, Telar 4.000 a de C.

3.500 a de C.
PRIMERAS CIVILIZACIONES
(Mesopotamia y Egipto)
Rueda
Torno
Arado, Balanza
Metalurgia del Cobre, Hierro
1.000 a de C.

600 a de C.
GRECIA Y ROMA
Primeras instalaciones:
Acueductos y Termos
Tecnología militar
500

600
ISLAM
Molino de Piedra
Libro impreso
Importación de inventos chinos pero
ellos los extendieron: Papel, pólvora
1.200

TAMadrid
5
-Pág.9-
T ECNOLOGÍA

1.300
EDAD MEDIA
Contabilidad
Gafas
1.400

1.500
RENACIMIENTO
La imprenta
Prensa hidráulica
Telescopio
1.600

1.600
REVOLUCIÓN INDUSTRIAL
Barómetro
Reloj de péndulo
Olla a presión
Pararrayos
Máquina de Vapor
1.800

SIGLO XIX
Pila eléctrica, Dinamo
Motor eléctrico, Teléfono
Bombilla, Motor gasolina
Automóvil, Calculadora

SIGLO XX
Radar, Nylon
Marca paso, Satélite
Láser
Vídeo
Ordenador personal

1.2. FUENTES DE ENERGÍA E INSTALACIONES


El desarrollo de cualquier actividad humana pasa por un continuo consumo de energía.

El descubrimiento de las nuevas fuentes de energía hizo que la calidad de vida aumen-
tase considerablemente, intensificando el aprovechamiento de estas nuevas fuentes de ener-
gía y disminuyendo el uso de la fuerza muscular.

6 TAMadrid

-Pág.10-
U. D. 1 - P RO C E S O D E R E S O L U C I Ó N T É C N I C A D E P RO B L E M A S

Principales fuentes de energía.

Solar
Geotérmica
RENOVABLE Hidráulica
Fuentes Eólica CALOR
de Residuos Sólidos Urbanos Y
Energía ELECTRICIDAD

Nuclear
NO RENOVABLE
Combustibles Fósiles

1.2.1. FUENTES DE ENERGÍA RENOVABLE

Son aquellas que no se agotan en el transcurso del tiempo.

1.2.1.1. Energía Solar

Procede de las radiaciones del Sol.

El Sol nos proporciona dos tipos de energía, la Térmica (calor) y la Fotovoltaica (luz).

TAMadrid
7
-Pág.11-
T ECNOLOGÍA

Energía Térmica.

Esta energía se aprovecha de forma:

Pasiva

Consiste en utilizar el calor que proporciona los rayos del sol aprovechando únicamen-
te las características y dispositivos de los materiales. Ejemplo: edificios con grandes crista-
leras.

Activa

Consiste en aprovechar el calor que produce los rayos del Sol para calentar un líquido
normalmente agua que se encuentra en un depósito y utilizarla. Ejemplo: agua sanitaria
caliente.

Energía Solar Fotovoltaica

Se produce cuando la luz solar incide sobre materiales semiconductores. Ejemplo: el


silicio, que transforma la energía solar en electricidad.

1.2.1.2. Energía Geotérmica

Es la que nos proporciona el calor interno de la Tierra.

8 TAMadrid

-Pág.12-
U. D. 1 - P RO C E S O D E R E S O L U C I Ó N T É C N I C A D E P RO B L E M A S

El procedimiento de obtención de dicha energía es el siguiente:

– Se introduce agua fría a presión a través de una tubería a una profundidad determi-
nada donde se detecta una cavidad natural en la corteza terrestre la temperatura es muy ele-
vada (a 4.000 m. de profundidad la temperatura es de 250ºC).

– El agua se transforma en vapor en esa cavidad y sale por otra tubería al exterior.

– Se aprovecha la fuerza del vapor para mover una turbina que a su vez mueve un gene-
rador que produce electricidad.

1.2.1.3. Energía Hidráulica

Se consigue mediante el aprovechamiento de la velocidad y presión del agua en los ríos.

Dicha energía se consigue en la central hidroeléctrica que transforma la energía hidráu-


lica en eléctrica.

Se obtiene mediante el remanse del agua (presa o embalse), el cual se canaliza median-
te una tubería consiguiendo una velocidad y presión altas que mueven una turbina que a su
vez produce la electricidad mediante un generador.

TAMadrid
9
-Pág.13-
T ECNOLOGÍA

1.2.1.4. Energía Eólica

Se consigue mediante el aprovechamiento de la fuerza del viento, éste hace girar una
gran hélice la cual mueve un generador que produce electricidad.

1.2.1.5. Residuos Sólidos Urbanos (RSU)

La materia orgánica procedente de la basura que generan los seres humanos en sus hoga-
res se somete a procesos de fermentación para obtener un producto llamado "Compost", uti-
lizado como abono en la agricultura y también para obtener bio-gas que se utiliza como
combustible en centrales térmicas.

1.2.2. FUENTES DE ENERGÍA NO RENOVABLES

Son aquellas que tarde o temprano se agotan.

10 TAMadrid

-Pág.14-
U. D. 1 - P RO C E S O D E R E S O L U C I Ó N T É C N I C A D E P RO B L E M A S

1.2.2.1. Energía nuclear

Se obtiene aprovechando la energía o calor que se desprende al romper los núcleos de


los átomos; este calor se aprovecha para calentar agua y obtener vapor, que al hacer girar
una turbina mueve un generador y produce electricidad.

1.2.2.2. Energía de los Combustibles Fósiles

Son las fuentes de energía más utilizadas en la actualidad.

Los principales combustibles fósiles utilizados son:

Petróleo

Es el más utilizado. Su mayor aplicación es, una vez refinado, en motores de combus-
tión interna. Ejemplo transporte público, aviones, ferrocarril, etc.

TAMadrid
11
-Pág.15-
T ECNOLOGÍA

Carbón

A partir de la revolución industrial fue la fuente de energía más utilizada para mover las
máquinas de vapor. Sirve también de combustible en centrales térmicas. Su aplicación, por
su alto coste de extracción, es menor que la del petróleo y la del gas.

Gas Natural

Es un combustible que no contamina el medio ambiente. Su aplicación cada día es más


extensa, por lo que sustituye a los dos anteriores.

En la actualidad se están cambiando las calefacciones de carbón o petróleo por las de


Gas Natural.

12 TAMadrid

-Pág.16-
U. D. 1 - P RO C E S O D E R E S O L U C I Ó N T É C N I C A D E P RO B L E M A S

1.3. AMBIENTES ARTIFICIALES


Toda persona tiene tendencia a buscar la mayor comodidad en el lugar donde se encuen-
tra: hogar, calle, ocupación habitual...

Para ello se idean ambientes artificiales ajenos a los que nos ofrecen las condiciones
climatológicas y a la acción que estemos realizando.

La situación en la que cada persona se encuentra o en la que realiza una actividad de


forma agradable se denomina confort.

Condiciones ambientales

Térmico La actividad física

CONFORT El vestuario

Ergonómico

1.3.1. CONFORT TÉRMICO

Es el ambiente térmico en el cual cada persona se encuentra agradablemente. Los aspec-


tos que influyen en el confort térmico son:

1.3.1.1. Condiciones ambientales

Éstas están definidas por cuatro variables:

• La temperatura del aire.

• La velocidad del aire.

• La humedad (cantidad de vapor de agua que contiene el aire. Su valor máximo es


de 100%. Cuando la humedad es muy elevada, es difícil evaporar el sudor y la sen-
sación térmica es desagradable).

• La temperatura radiante (si nos encontramos cerca de una fuente de calor / frío).

TAMadrid
13
-Pág.17-
T ECNOLOGÍA

1.3.1.2. La actividad física

Cuanto más intenso sea el trabajo que se realice, mayor será la sensación de calor.

El 90% de la energía que consume el cuer-


po humano se transforma en calor y sólo el
10% en trabajo útil.

1.3.1.3. El vestuario

Como es evidente, la ropa utilizada en cada


estación del año no es la misma, por lo que
queda claro que es un factor a tener en
cuenta en el confort.

14 TAMadrid

-Pág.18-
U. D. 1 - P RO C E S O D E R E S O L U C I Ó N T É C N I C A D E P RO B L E M A S

1.3.2. CONFORT ERGONÓMICO

Entendemos por Ergonomía la ciencia que trata de la adaptación del mobiliario y las
máquinas al hombre, para conseguir un ambiente artificial, un mayor confort posicional,
seguridad y rendimiento en el trabajo.

La siguiente tabla expresa los valores de temperatura, humedad y velocidad del aire
según el tipo de trabajo que se desarrolle.

Temperatura Grado de Velocidad del


TIPO DE TRABAJO
Optima (ºC) Humedad Aire (m/sg)
Trabajo físico ligero
18 a 24 40% a 70% 0'1
en posición sentada.
Trabajo medio en
17 a 22 40% a 70% 0'1 a 0'2
posición de pie.

Trabajo duro. 15 a 21 30% a 65% 0'4 a 0'5

Trabajo muy duro. 12 a 18 20% a 60% 1'0 a 1'5

TAMadrid
15
-Pág.19-
T ECNOLOGÍA

2. PROYECTO TÉCNICO

Ante el problema de resolver necesidades humanas, se realiza una búsqueda de solucio-


nes planteando ideas innovadoras.

En la creación de un objeto nuevo o en la modificación o mejora de uno ya existente, se


buscan los procesos necesarios para reducir los problemas que puedan surgir.

Todos estos procesos se plasman en un documento denominado proyecto, con la inten-


ción de que los objetos que se vayan a fabricar sean más eficientes, seguros y fiables.

Antes de llevar a cabo un proyecto, se deben realizar unos pasos previos:

- Aporte de ideas.
- Estudio del mercado (si se adapta al sector de población al que va dirigido).
- Viabilidad (¿existen los recursos necesarios para llevarlo acabo?).
- Rentabilidad (si vamos a obtener un beneficio ).

Todos estos pasos constituyen un anteproyecto

Proyecto técnico es el conjunto de procesos técnicos plasmados en un documento


mediante medios gráficos y que suprimen o eliminan todo tipo de improvisación.

2.1. FASES DE UN PROYECTO TÉCNICO

ESTUDIAR ¿Qué producto? Objetivos Viabilidad

Estudio Mercado Satisfacción del Cliente

INVESTIGAR
Legislación
Información
Proveedores

PROYECTAR Cálculos Técnicos Memoria Planos Presupuesto

16 TAMadrid

-Pág.20-
U. D. 1 - P RO C E S O D E R E S O L U C I Ó N T É C N I C A D E P RO B L E M A S

Técnicas Colegios Profesionales


LEGALIZAR Normas
Legales Licencias

PRESENTAR Imagen

EJECUTAR Prototipos Fabricación en serie

2.1.1. ESTUDIAR

La mayoría de los objetos que utilizamos han sido construidos siguiendo un proceso de
fabricación más o menos complejo.

Estos procesos de fabricación exigen un estudio minucioso en el cual, uno debe hacer-
se la siguiente pregunta.

¿Qué puede satisfacer las necesidades de los consumidores?

Una vez detectada una necesidad en los consumidores se pondrán unos objetivos que
nos marquen lo que queremos conseguir.

Valorar los medios de que disponemos tanto materiales como económicos (viabilidad).

2.1.2. INVESTIGAR

Detectar si realmente interesa el proyecto a los consumidores mediante un estudio de


mercado (encuestas, formularios, test, etc.).

Buscar la información técnica y bibliográfica que afecte a la elaboración del proyecto.

Detectar e investigar sobre los posibles proveedores que puedan suministrar el material
para la ejecución del proyecto.

Estudio de Mercado es el conjunto de datos que nos permite conocer qué grado
de aceptación tiene un producto antes de iniciar su fabricación.

TAMadrid
17
-Pág.21-
T ECNOLOGÍA

2.1.3. PROYECTAR

Mediante el estudio de los datos obtenidos, se desarrolla el proyecto organizándolo en


una serie de documentos básicos que dan respuestas a los objetivos planteados.

2.1.3.1. Pliego de condiciones

Recoge los aspectos que inciden en la ejecución final:

• Características generales.
• Garantías.
• Normativas.

2.1.3.2. Memoria justificativa

Presenta los cálculos técnicos que justifican tanto el modo como las formas de ejecutar
el proyecto.

2.1.3.3. Memoria descriptiva

Refleja los pasos realizados en cada uno de los procesos de producción. También se des-
criben las características de los elementos, cualificaciones profesionales necesarias, etc.

2.1.3.4. Planos

Recoge toda la información gráfica (dibujos, figuras, diagramas....) para poder inter-
pretar el proyecto.

2.1.3.5. Presupuesto

Refleja la cuantía económica que va a suponer la realización del proyecto de forma indi-
vidual y global.

2.1.4. LEGALIZAR

Todo proyecto debe cumplir todas las normas técnicas que le afecten, por lo que debe
ser verificado y aprobado por el Colegio Profesional que le corresponda.

También debe cumplir con la normativa legal vigente, obteniendo las diferentes licen-
cias administrativas necesarias.

18 TAMadrid

-Pág.22-
U. D. 1 - P RO C E S O D E R E S O L U C I Ó N T É C N I C A D E P RO B L E M A S

2.1.5. PRESENTAR

Según el poder económico de la empresa la presentación de los proyectos a gran escala


y con mucho colorido (a bombo y platillo), hace que se fomente el interés por dicho pro-
ducto y se obtengan más beneficios. Ejemplo: Encontrar socios que inviertan su capital para
la puesta en práctica de dicho producto, etc.

2.1.6. EJECUTAR

Se trata de la fabricación física del producto hasta llegar a una producción en serie.
Anteriormente, y dependiendo del producto, se realizan maquetas y/o prototipos para poder
realizar operaciones de rediseño y corregir los pequeños errores que puedan existir.

PLANOS

REDISEÑO PROTOTIPOS

NO CONTROL
CALIDAD

SI

PEQUEÑA FABRI-
CACIÓN

NO
CONTROL

SI

FABRICACIÓN
EN SERIE

TAMadrid
19
-Pág.23-
T ECNOLOGÍA

3. DISEÑO
Debido a la alta competitividad que existe en el mercado, los fabricantes se preocupan
cada día más de presentar sus productos con innovadores diseños, para hacerlos más atrac-
tivos a los consumidores y aumentar su productividad.

El diseño se basa en manejar y combinar aspectos técnicos y comerciales para obtener


un objeto apreciado y deseado por el público.

Diseño es un proceso mediante el cual se desarrolla una idea


definiendo formas y características concretas

A la hora de materializar un diseño de calidad, se deben tener en cuenta los siguientes


aspectos:

• La funcionalidad: Que el objeto sea útil.


• La forma externa: Forma y color más apropiado a la función que tiene que desem-
peñar.
• Materiales: Que tengan la resistencia y el peso más apropiado según su funciona-
lidad.
• Técnicas de construcción: Que sea viable la fabricación del producto y lo más sen-
cilla posible.

3.1. MÉTODOS Y TIPOS DE DISEÑO

El método para diseñar depende del producto y de los usuarios. Existen dos métodos
puros de diseño.

20 TAMadrid

-Pág.24-
U. D. 1 - P RO C E S O D E R E S O L U C I Ó N T É C N I C A D E P RO B L E M A S

Intuitivo: El diseñador se deja llevar por su inspiración creando el objeto; los aspectos
técnicos se adaptan después al diseño.

Racional: Se parte de las características técnicas y el diseñador se adapta a ellas para


definir la forma final del objeto.

A la hora de hacer una propuesta, el diseñador puede optar por seguir los criterios que
estén de moda o hacer una propuesta innovadora que revolucione el mercado.

Así, existen dos tipos de diseño:

Diseño tradicional: En el que el diseñador conoce las tendencias de moda y los gustos
del público y adapta a ellos el producto.

Diseño innovador: En el que el diseñador conoce los gustos del publico e intenta cam-
biarlos con estéticas nuevas.

3.2. ANÁLISIS DEL PRODUCTO

Para analizar un producto, debemos de tener en cuenta los siguientes aspectos:

- Anatómico.
- Funcional.
- Económico.
- Social.

3.2.1. ANATÓMICO

Para el estudio de las relaciones de las personas con los objetos, debemos considerar
diferentes puntos de vista con respecto a las aptitudes y capacidades de sus usuarios.

Antropométrico: Los objetos han de tener las dimensiones adecuadas para poderlos
utilizar con comodidad y seguridad.

Psicológico: Los objetos deben ser adecuados a la edad y deben evitarse las tareas repe-
titivas y monótonas.

Sociológico y cultural: El objeto debe dar a su propietario satisfacciones culturales e


intelectuales.

TAMadrid
21
-Pág.25-
T ECNOLOGÍA

3.2.2. FUNCIONAL

Según la funcionalidad para la que se diseñe un objeto y las sensaciones que se quiere
que éste transmita, la influencia de la forma y el color son de suma importancia.

3.2.2.1. La forma

Se considera que existen tres formas esenciales en el diseño: triángulo, cuadrado y cír-
culo.

Las formas triangulares (pirámides y conos) llaman poderosamente la atención del


observador, por ejemplo en elementos ornamentales y decorativos.

Las formas rectangulares (prismas) se utilizan básicamente para objetos estables y aus-
teros: elementos que se necesitan apilar y almacenar.

Las formas curvas se emplean en objetos que puedan ser utilizados por las personas: las
herramientas manuales, jarras...

3.2.2.2. El color

Al igual que la forma en un objeto, según la función que vaya a desempeñar, también el
color con el que va a ser fabricado tiene gran influencia.

Color Simbolismo Efecto


. . . Blanco . . . . . . . . . . . . . . . Pureza. Paz . . . . . . . . . . . . . . . . Frío . . . . . . . . . .
. . . . Negro . . . . . . . . . . . . . . Lujo. Elegancia . . . . . . . . . . . . . Tristeza . . . . . . . . .
. . . . Rojo . . . . . . . . . . . . . . Peligro. Socorro . . . . . . . . . . . . Agresivo . . . . . . . .
. . . . Verde . . . . . . . . . . . . . Verdad. Esperanza . . . . . . . . . . . . Sedante . . . . . . . . .
. . . Amarillo . . . . . . . . . . . . . . . Mala suerte . . . . . . . . . . . . . . Espiritual . . . . . . . .

Los sistemas internacionales de normalización han adoptado los colores para unificar
los criterios a la hora de seleccionar muchos avisos, mensajes, señales, etc. Ejemplos:

Cables eléctricos
Fases de corriente alterna: Marrón / Negro / Gris.
Fases de corriente continua: Negro / Rojo.
Toma de tierra: Verde y Amarillo.

22 TAMadrid

-Pág.26-
U. D. 1 - P RO C E S O D E R E S O L U C I Ó N T É C N I C A D E P RO B L E M A S

Tuberías
Agua potable: Verde oscuro.
Aguas contra incendios: Rojo.
Aire comprimido: Azul.
Gases o combustibles: Amarillo.

3.2.3. ECONÓMICO

Uno de los aspectos importantes que hay que considerar en el diseño es la producción
masiva de un objeto. El diseño no sólo está centrado en la fabricación artesanal, sino que,
con la aparición de la automatización en las fabricaciones de objetos, este sistema se apli-
ca a las grandes producciones.

La fabricación en serie se emplea para obtener grandes cantidades de un mismo pro-


ducto, abaratar el precio de éste y poder venderlo a precios competitivos.

Un ejemplo de fabricación en serie son los bolígrafos, de lo que existen infinidad de


diseños diferentes e innovadores en el mercado y con una gran cantidad de producción.

3.2.4. SOCIAL

La sociedad moderna tiene cada vez más necesidades. El diseñador debe atender a los
aspectos culturales y sociales que lleva asociado un objeto para su diseño. Esto los pode-
mos observar en esta tabla.

NECESIDAD ESPECIALIDAD
. . . . . . . . . Calles y parques . . . . . . . . . . . . . . . Diseño Urbano . . . . . . . . . . . . . . .
. . . . . . . . . . . . Vivienda . . . . . . . . . . . . . . . . Diseño de Interiores . . . . . . . . . . . . .
. . . . . . . . . . . . . Bienes . . . . . . . . . . . . . . . . . . Diseño Industrial . . . . . . . . . . . . . .
. . . . . . . . . . . . Mensajes . . . . . . . . . . . . . . . . . . Diseño Gráfico . . . . . . . . . . . . . . .
. . . . . . . . . . . . Vestidos . . . . . . . . . . . . . . . . . . . Diseño Textil . . . . . . . . . . . . . . . .

TAMadrid
23
-Pág.27-
T ECNOLOGÍA

RESUMEN

• A lo largo de la historia, el hombre, para evolucionar y satisfacer las necesidades


que le iban surgiendo ha ido creando objetos que han hecho cambiar sustancial-
mente su forma de vida: Objetos de piedra, de hierro, edificaciones, papel, pólvo-
ra, imprenta, electricidad, ordenadores, etc...

• El desarrollo de cualquier actividad humana pasa por un continuo consumo de


energía: Renovable (energía solar, térmica, solar fotovoltaica -geotérmica, hidráu-
lica, eólica, residuos sólidos urbanos-), no renovable ( nuclear, energía de los com-
bustibles fósiles -petróleo, carbón, gas natural-).

• Toda persona tiene tendencia a buscar la mayor comodidad en el lugar donde se


encuentra: Hogar, calle, ocupación habitual, etc..., buscando un confort térmico y
ergonómico.

• En la creación de un objeto nuevo o en la modificación o mejora de uno ya exis-


tente, se buscan los procesos necesarios para reducir los problemas que puedan
surgir; estos procesos se plasman en un documento denominado proyecto, cuyas
fases son: estudiar, investigar proyectar, legalizar, presentar y ejecutar. A conti-
nuación se haría el diseño, que es el proceso mediante el cual se desarrolla una idea
definiendo formas y características concretas. Los métodos de diseño son: intuiti-
vo y racional, y los tipos: tradicional e innovador.

24 TAMadrid

-Pág.28-
U. D. 1 - P RO C E S O D E R E S O L U C I Ó N T É C N I C A D E P RO B L E M A S

EJERCICIOS DE AUTOCOMPROBACIÓN

1. Localiza en la sopa de letras el invento más significativo de cada época.

Q F E R F D Y V B N J K I F C
R T Y U U I O P P L M M M E Q
A R F G H J K L M Ñ C C V C O
S O A C U E D U C T O F G V S
D P D D R T Y U I N G L C F E
D A C F G O H H T G I Z X R R
F V D F A E H A H V O F G T P
G A N F G D B H O G C V F Y M
H N N G H I E M E R T Y U U I
J I N H L V O U D F G H U Y O
K U N I J T F F R F G H U H R
K Q D U U D F G H Y T R F J B
K A N A F G H J U Y T G H U I
D M N D F G H J K L L O I I L
K K N C T E L E S C O P I O O

2. Relaciona:

Forma de Fuente de Tipo de


energía energía fuente
“R= Renovable”
“N= No Renovable”
Energía El viento que se produce entre dos puntos de la Tierra con
hidráulica A diferente presión y temperatura.
Energía de Las precipitaciones de lluvia y de nieve permite embalsar
combustible B el agua.
fósil
Energía C Determinados materiales, que se encuentran en el interior
nuclear de la corteza terrestre.
Energía D Determinados materiales, como el uranio, son capaces de
geotérmica generar grandes cantidades de energía al desintegrarse.
Energía E La actividad interna de la Tierra genera diferentes tempe-
eólica raturas.
Energía solar F La luz solar es capaz de actuar sobre determinados mate-
térmica riales semiconductores.
Energía solar G El calor generado por la actividad del Sol se puede con-
fotovoltaica centrar mediante espejos y actuar sobre un fluido.

TAMadrid
25
-Pág.29-
T ECNOLOGÍA

3. Completa el cuadro siguiente a partir de la pregunta anterior.

Energías renovables Energías no renovables

4. Relaciona.

Fases de
un proyecto Es Lo realiza

Según el poder económico de la empresa la


presentación de los proyectos a gran escala y
Investigar con mucho colorido "bombo y platillo", hace
que se fomente el interés por dicho producto.
Oficina
Todo proyecto debe atenerse y cumplir todas Técnica
las normas técnicas que le afecten, por lo que
Estudiar debe ser verificado y aprobado por el colegio
profesional que le corresponda.
Bien sea por que no existe un producto en el
mercado o por que se puede mejorar el pro-
Legalizar ducto ya existente. Una vez detectada una
necesidad en los consumidores se pondrán Taller de
unos objetivos que nos marquen qué quere- Producción
mos conseguir.
Proyectar Se trata de la fabricación física del producto.
Mediante la recopilación de los datos obteni-
dos se desarrolla el proyecto, organizándolo
en una serie de documentos básicos que dan
Ejecutar respuestas a los objetivos planteados. Organismos
Detectar si realmente va a interesar el proyec- Oficiales
to a los consumidores, mediante un estudio de
mercado (encuestas, formularios, test, etc.).
Presentar

26 TAMadrid

-Pág.30-
U. D. 1 - P RO C E S O D E R E S O L U C I Ó N T É C N I C A D E P RO B L E M A S

5. Completa el cuadro siguiente a partir de la pregunta anterior.

1º. Ordena las fases de un proyecto técnico


2º. Indica a quien le corresponde.

Cuestion 1ª Cuestion 2ª

1º.

2º.

3º.

4º.

5º.

6º.

6. Relaciona los componentes que forman parte de un proyecto con los que se realiza en cada uno
de ellos.

Lo forman Es

Presenta los cálculos técnicos que justifi-


Pliego de condiciones can tanto el modo como las formas de ejecu-
tar el proyecto.

Refleja los pasos que hay que seguir en


Memoria justificativa
cada uno de los procesos de producción.

Recoge los aspectos que inciden en la


Memoria descriptiva ejecución final.

Refleja la cuantía económica que va a


suponer la realización del proyecto de forma
Planos
individual y global.

Recoge toda la información gráfica


Presupuesto (dibujos, figuras, diagramas...) para poder
interpretar el proyecto.

TAMadrid
27
-Pág.31-
T ECNOLOGÍA

7. Cita los documentos de un proyecto técnico que quedan afectados cuando se realiza un cam-
bio de medidas y de materiales al construir una parte del objeto.

8. Indica los aspectos en un miniproyecto de un lápiz:

1. Pliego de condiciones

2. Memoria justificativa

3. Memoria descriptiva

4. Planos

5. Presupuesto

9. Definir diversas características de un bolígrafo. Clasificarlas según se pretenda un artículo de


lujo o un elemento común de escritura.

10. Analiza las sensaciones y posibles aplicaciones que producen los siguientes automóviles según
su color:

- Fórmula 1; rojo.

- Berlina; verde.

- Todoterreno; amarillo.

28 TAMadrid

-Pág.32-
U. D. 1 - P RO C E S O D E R E S O L U C I Ó N T É C N I C A D E P RO B L E M A S

RESPUESTAS A LOS EJERCICIOS

1. Localiza en la sopa de letras el invento más significativo de cada época.

Q F E R F D Y V B N J K I F C
R T Y U U I O P P L M M M E Q
A R F G H J K L M Ñ C C V C O
S O A C U E D U C T O F G V S
D P D D R T Y U I N G L C F E
D A C F G O H H T G I Z X R R
F V D F A E H A H V O F G T P
G A N F G D B H O G C V F Y M
H N N G H I E M E R T Y U U I
J I N H L V O U D F G H U Y O
K U N I J T F F R F G H U H R
K Q D U U D F G H Y T R F J B
K A N A F G H J U Y T G H U I
D M N D F G H J K L L O I I L
K K N C T E L E S C O P I O O

2. Relaciona:

Forma de Fuente de Tipo de


energía energía fuente
“R= Renovable”
“N= No Renovable”
Energía El viento que se produce entre dos puntos de la tierra con
hidráulica A E diferente presión y temperatura. R

Energía de Las precipitaciones de lluvia y de nieve permite embalsar


combustible B A el agua. R
fósil
Energía Determinados materiales, que se encuentran en el interior
C B N
nuclear de la corteza terrestre.
Energía Determinados materiales, como el uranio, son capaces de
D C N
geotérmica generar grandes cantidades de energía al desintegrarse.
Energía La actividad interna de la tierra genera diferentes tempe-
E D raturas. R
eólica
Energía solar La luz solar es capaz de actuar sobre determinados mate-
térmica F G riales semiconductores. R

Energía solar El calor generado por la actividad del sol se puede con-
fotovoltaica G F centrar mediante espejos y actuar sobre un fluido. R

TAMadrid
29
-Pág.33-
T ECNOLOGÍA

3. Completa el cuadro siguiente a partir de la pregunta anterior.

Energías renovables Energías no renovables

Solar térmica y fotovoltaica Nuclear

Eólica De combustible fósil

Hidráulica

Geotérmica

4. Relaciona.

Fases de
un proyecto Es Lo realiza

Según el poder económico de la empresa la


presentación de los proyectos a gran escala y
Investigar con mucho colorido "bombo y platillo", hace
que se fomente el interés por dicho producto.
Oficina
Todo proyecto debe atenerse y cumplir todas Técnica
las normas técnicas que le afecten, por lo que
Estudiar debe ser verificado y aprobado por el colegio
profesional que le corresponda.
Bien sea por que no existe un producto en el
mercado o por que se puede mejorar el pro-
Legalizar ducto ya existente. Una vez detectada una
necesidad en los consumidores se pondrán Taller de
unos objetivos que nos marquen qué quere- Producción
mos conseguir.
Proyectar Se trata de la fabricación física del producto.
Mediante la recopilación de los datos obteni-
dos se desarrolla el proyecto, organizándolo
en una serie de documentos básicos que dan
Ejecutar respuestas a los objetivos planteados. Organismos
Detectar si realmente va a interesar el proyec- Oficiales
to a los consumidores, mediante un estudio de
mercado (encuestas, formularios, test, etc.).
Presentar

30 TAMadrid

-Pág.34-
U. D. 1 - P RO C E S O D E R E S O L U C I Ó N T É C N I C A D E P RO B L E M A S

5. Completa el cuadro siguiente a partir de la pregunta anterior.

1º. Ordena las fases de un proyecto técnico


2º. Indica a quien le corresponde.

Cuestion 1ª Cuestion 2ª

1º. Estudiar Oficina Técnica


2º. Investigar Oficina Técnica
3º. Proyectar Oficina Técnica

4º. Legalizar Organismos Oficiales

5º. Presentar Organismos Oficiales

6º. Ejecutar Taller de Producción

6. Relaciona los componentes que forman parte de un proyecto con los que se realiza en cada uno
de ellos.

Lo forman Es

Presenta los cálculos técnicos que justifi-


Pliego de condiciones can tanto el modo como las formas de ejecu-
tar el proyecto.

Refleja los pasos que hay que seguir en


Memoria justificativa
cada uno de los procesos de producción.

Recoge los aspectos que inciden en la


Memoria descriptiva ejecución final.

Refleja la cuantía económica que va a


suponer la realización del proyecto de forma
Planos
individual y global.

Recoge toda la información gráfica


Presupuesto (dibujos, figuras, diagramas...) para poder
interpretar el proyecto.

TAMadrid
31
-Pág.35-
T ECNOLOGÍA

7. Cita los documentos de un proyecto técnico que quedan afectados cuando se realiza un cam-
bio de medidas y de materiales al construir una parte del objeto.

8. Realiza un miniproyecto técnico de un lápiz:

1. Pliego de condiciones Finalidad y Normativa del objeto a fabricar

2. Memoria justificativa Cálculos del tipo de madera, mina y forma del lápiz

3. Memoria descriptiva Pasos a seguir para fabricarle y categoría del operario

4. Planos Medidas

5. Presupuesto Coste de materiales y fabricación

9. Definir diversas características de un bolígrafo. Clasificarlas según se pretenda un artículo de


lujo o un elemento común de escritura.

Art. Lujo: Forma Material (Acero inoxidable, Baño de plata), marca fabricante prestigioso.
Art. Común: Material (Plástico), precio económico, marca fabricante muy corriente

10. Analiza las sensaciones y posibles aplicaciones que producen los siguientes automóviles según
su color:

- Fórmula 1; rojo. Agresivo Carreras de coches

- Berlina; verde. Verdad, Esperanza Elementos Oficiales (ejército)

- Todoterreno; amarillo. Espiritual Agricultura

32 TAMadrid

-Pág.36-
U. D. 2 - E X P L O R AC I Ó N Y C O M U N I C AC I Ó N D E I D E A S

ÍNDICE
Pag.

OBJETIVOS . . . . . . . . . . . . . . . . . . . . . . . . . . . . . . . . . . . . . . . . . . . . . . . . . . . . . . . . . . . 2

INTRODUCCIÓN . . . . . . . . . . . . . . . . . . . . . . . . . . . . . . . . . . . . . . . . . . . . . . . . . . . . . . 3

MAPA CONCEPTUAL . . . . . . . . . . . . . . . . . . . . . . . . . . . . . . . . . . . . . . . . . . . . . . . . . . 4

DESARROLLO DE CONTENIDOS . . . . . . . . . . . . . . . . . . . . . . . . . . . . . . . . . . . . . . . 5

1. REPRESENTACIÓN DE IDEAS . . . . . . . . . . . . . . . . . . . . . . . . . . . . . . . . . . . . 5
1.1.INSTRUMENTOS Y ÚTILES PARA EL DIBUJO Y DISEÑO . . . . . . . . . . . . . . 5
1.2.OTROS INSTRUMENTOS PARA EL REGISTRO DE IDEAS TÉCNICAS . . . 11

2. FORMAS DE REPRESENTACIÓN GRÁFICA . . . . . . . . . . . . . . . . . . . . . . . 14


2.1. BOCETO . . . . . . . . . . . . . . . . . . . . . . . . . . . . . . . . . . . . . . . . . . . . . . . . . . . . 14
2.2. CROQUIS . . . . . . . . . . . . . . . . . . . . . . . . . . . . . . . . . . . . . . . . . . . . . . . . . . . 14
2.3. DELINEADO . . . . . . . . . . . . . . . . . . . . . . . . . . . . . . . . . . . . . . . . . . . . . . . . 14
2.4. PROYECCIÓN DIÉDRICA . . . . . . . . . . . . . . . . . . . . . . . . . . . . . . . . . . . . . . 15
2.5. PERSPECTIVAS . . . . . . . . . . . . . . . . . . . . . . . . . . . . . . . . . . . . . . . . . . . . . . 16

3. OTRAS FORMAS DE REPRESENTACIÓN GRÁFICAS . . . . . . . . . . . . . . 18


3.1. SÍMBOLOS . . . . . . . . . . . . . . . . . . . . . . . . . . . . . . . . . . . . . . . . . . . . . . . . . . 18
3.2. ESQUEMAS TÉCNICOS . . . . . . . . . . . . . . . . . . . . . . . . . . . . . . . . . . . . . . . . 19
3.3. GRÁFICOS Y DIAGRAMAS . . . . . . . . . . . . . . . . . . . . . . . . . . . . . . . . . . . . . 20
3.4. TABLAS DE DATOS . . . . . . . . . . . . . . . . . . . . . . . . . . . . . . . . . . . . . . . . . . . 21

4. NORMALIZACIÓN . . . . . . . . . . . . . . . . . . . . . . . . . . . . . . . . . . . . . . . . . . . . . . 21
4.1. NORMAS DE REPRESENTACIÓN GRÁFICA . . . . . . . . . . . . . . . . . . . . . . . 22

RESUMEN . . . . . . . . . . . . . . . . . . . . . . . . . . . . . . . . . . . . . . . . . . . . . . . . . . . . . . . . . . . 30

EJERCICIOS DE AUTOCOMPROBACIÓN . . . . . . . . . . . . . . . . . . . . . . . . . . . . . . . 31

RESPUESTAS A LOS EJERCICIOS . . . . . . . . . . . . . . . . . . . . . . . . . . . . . . . . . . . . . 34

-Pág.37-
T ECNOLOGÍA

OBJETIVOS
Al finalizar el estudio de ésta Unidad Didáctica, el alumno será capaz de:

• Definir las diferentes formas de recopilar información técnica de una idea.

• Describir los instrumentos, accesorios más comunes, empleados en la reali-


zación de las prácticas de dibujo técnico.

• Tomar conciencia de la importancia de la normalización en todo lo relacio-


nado con la vida del ser humano.

• Diferenciar los tipos de representación gráfica que existen.

• Distinguir los tipos y métodos de diseño de objetos fabricados para el ser


humano.

-Pág.38-
U. D. 2 - E X P L O R AC I Ó N Y C O M U N I C AC I Ó N D E I D E A S

INTRODUCCIÓN
T oda idea que le surja a un investigador o creador se debe transmitir a los demás
de una forma concreta, y clara para que no pueda llevar a confusiones y per-
mita un entendimiento entre el creador, el fabricante y el consumidor.

Para conseguir todo esto nos valemos de instrumentos de representación gráfica.


Estos deben cumplir unas normas para que un diseño tenga un carácter universal y
pueda ser comercializado en todo el mundo.

-Pág.39-
T ECNOLOGÍA

M A PA C O N C E P T UA L

¿QUÉ? Idea Técnica

Útiles

MEDIANTE Instrumentos

Formas

UNIVERSALIZAR Normalización

OBTENEMOS Representación Gráfica

-Pág.40-
U. D. 2 - E X P L O R AC I Ó N Y C O M U N I C AC I Ó N D E I D E A S

1. PRESENTACIÓN DE IDEAS TÉCNICAS

Toda idea que nos pueda surgir debemos reflejarla en algún medio físico, para poder
compartirla y contrastarla con las de los demás.

Uno de los medios más importante de representación es el dibujo, aunque también exis-
ten otras formas de recoger esta información.

Al dibujo se le puede considerar como uno de los leguajes más extendidos por todo el
mundo.

El dibujo técnico es el idioma de los profesionales

Para llevar a cabo estas representaciones nos valemos de instrumentos y útiles que nos
facilitan el trabajo.

1.1. INSTRUMENTOS Y ÚTILES PARA EL DIBUJO Y DISEÑO

El dibujante no debe conformarse con idear y ser capaz de transmitir, sino que los dibu-
jos deben tener una presentación y calidad buena, para ello debe conocer y manejar perfec-
tamente el material y los útiles necesarios para llevarlo a cabo.

1.1.1. PAPEL

Es el medio más utilizado como soporte de un dibujo.

Se presenta normalmente en rollos o en formatos normali-


zados. El papel tiene diferentes grosores según su "gramaje",
o sea, el peso en gramos por metro cuadrado.

TAMadrid
5
-Pág.41-
T ECNOLOGÍA

El papel más utilizado en el dibujo puede ser de dos clases.

Papel opaco: Este puede ser de diferentes colores, generalmente es blanco, debe tener
cola para evitar que absorba la tinta y se extienda sobre la superficie de manera uniforme.

Papel transparente: También conocido como papel vegetal, se puede utilizar para cal-
car otros dibujos.

Otros tipos de papel:

Cansón: Es un papel especial de dibujo, muy liso y resistente.

Pergamino: Se emplea para dibujos a tinta china.

Papel-tela: Es transparente y se usa para planos que van


a tener una larga duración.

Milimetrado: Es un papel rallado horizontal y ver-


ticalmente por líneas espaciadas entre sí (0'5 ó 1 mm.); se
emplea para bocetos, croquis, gráficos, etc.

1.1.2. LÁPIZ

Es una barrita de grafito (mina) pegada en el interior de un cilindro o polígono regular


de madera.

Podemos diferenciarlos por:

• Por el color de su trazo.

Negro para dibujo y escritura.


Color para dibujo artístico y diseño gráfico.

6 TAMadrid

-Pág.42-
U. D. 2 - E X P L O R AC I Ó N Y C O M U N I C AC I Ó N D E I D E A S

• Por la dureza del grafito.

Según su mayor o menor dureza, el trazo será más o menos oscuro. Se cataloga la
dureza según vemos en la tabla inferior.

Los lapiceros de madera están siendo sustituidos por los portaminas en el caso del dibu-
jo y no tanto en el de diseño.

1.1.3. PORTAMINAS

Es un útil de plástico o metal que lleva en su interior, y sin pegar, la mina de grafito la
cual se puede romper.

TAMadrid
7
-Pág.43-
T ECNOLOGÍA

Ventajas sobre los lapiceros:

• Rapidez en el afilado.
• Mayor durabilidad.
• Más limpio.
• Más preciso.

1.1.4. SACAPUNTAS

Es la mejor herramienta para sacar punta al lápiz. Existen


muchas clases y diseños aunque el más recomendado es el saca-
puntas clásico, como el que aparece en la figura.

1.1.5. GOMA DE BORRAR

Debe ser blanda, flexible y de color claro. La dureza de la


goma será mayor a medida que aumenta la dureza del lápiz o
portaminas con el que se ha realizado la línea.

Se aconseja borrar con suavidad y en un solo sentido para


evitar que el papel pueda arrugarse y tener continuamente la
goma limpia; así evitaremos las manchas sobre el papel.

1.1.6. REGLA MILIMÉTRICA

Utensilio de madera o plástico de forma rectangular con un espesor de 2 a 5 milímetros


y una longitud de 30 a 100 cm.

Suele llevar por uno de sus lados una graduación en milímetros para poder medir y
transportar medidas.

8 TAMadrid

-Pág.44-
U. D. 2 - E X P L O R AC I Ó N Y C O M U N I C AC I Ó N D E I D E A S

Existe un tipo de regla especial, llamada escalímetro, cuya


sección es triangular y tiene seis escalas de reducción para
variar fácilmente el tamaño del dibujo.

1.1.7. JUEGO DE ESCUADRA Y CARTABÓN

Está compuesto por dos piezas (escuadra y cartabón), fabricadas normalmente en plás-
tico aunque también puede ser de madera.

Llevan una graduación en centímetros y milímetros.

El Cartabón tiene forma de triángulo rectángu-


lo escaleno (los tres lados desiguales) y dos de sus
ángulos son de 30º y 60º.

La Escuadra tiene forma de triángulo rectángu-


lo isósceles (dos lados iguales) y dos de sus ángulos
son de 45º.

Se utilizan para obtener una línea perpendicular a otra (líneas paralelas horizontales y
verticales), así como para trazar una línea respecto a otra a un ángulo determinado.

1.1.8. COMPÁS

Es un útil de dibujo que se emplea para trazar arcos y circunfe-


rencias. Está formado por dos brazos articulados de los cuales uno
termina en una aguja y el otro en una mina.

En la parte superior dispone de un mango para sujetarlo y hacer-


lo girar.

Accesorios del compás son la alargadera, bigotera. Estos


accesorios se presentan comercialmente en estuche junto al
compás.

TAMadrid
9
-Pág.45-
T
TEEC
CNNO
OLLO
OGGÍÍA
A

Pasos que hay que seguir para el uso correcto del compás:

1º. Buscar el centro de la circunferencia trazando una línea vertical y otra horizon-
tal para obtener los ejes..

2º. Transportar la longitud del radio sobre uno de los ejes.

3º. Pinchar en la intersección de las líneas conocidas como ejes.

4º. Abrir el compás hasta hacer coincidir la mina con la longitud del radio.

5º. Agarrar el compás por el mango con los dedos índice y pulgar y hacerlo girar en
sentido de los agujas del reloj.

1.1.9. ESTILÓGRAFOS

Los estilógrafos o plumas para delinear son los útiles para el pasado a tinta.

Su forma es semejante a una pluma estilográfica.

Los elementos que componen son:

Capuchón: La parte que de protege la puntera e impide que la


punta se seque cuando el estilógrafo no se utiliza.

Puntera: Elemento que marca la línea y determina el grosor de


está.

Boquilla: Parte exterior que une la puntera con el depósito de


tinta.

Depósito: Recipiente que almacena la tinta, pueden ser rellena-


ble o desechable.

Mango: Tubo que se rosca en la boquilla y que aumenta la lon-


gitud del estilógrafo para facilitar su manejo.

Se presenta comercialmente en estuches compuestos por varios estilógrafos ( de dife-


rentes grosores según las normas de dibujo), adaptador para el compás y bote de tinta.

10 TAMadrid

-Pág.46-
U. D. 2 - E X P L O R AC I Ó N Y C O M U N I C AC I Ó N D E I D E A S

Normas de uso y conservación:

1º. Limpiarlos periódicamente, si no se van a uti-


lizar durante un periodo largo de tiempo,
lavarlos y guardarlos sin tinta.

2º. Para que el grosor del trazo sea exacto se colo-


ca el estilógrafo vertical al papel.

3º. Al trazar se debe presionar suavemente el esti-


lógrafo.

1.1.10. PLANTILLAS

Son accesorios de plástico de formas diversas que


ahorran mucho tiempo en la realización de dibujos y
ayudan a trazar con rapidez y precisión curvas, sím-
bolos, letras, etc.

1.2. OTROS INSTRUMENTOS PARA EL REGISTRO DE IDEAS


TÉCNICAS
La necesidad de conservar la información de forma más fia-
ble y duradera ha potenciado la innovación tecnológica en estos
campos sea cada vez mayor.

1.2.1. TRANSPARENCIAS

Son láminas transparentes de acetato sobre las cuales se


puede dibujar, escribir y proyectar sobre una pantalla con ayuda
de un retroproyector como se ve en la figura.

TAMadrid
11
-Pág.47-
T ECNOLOGÍA

Las transparencias son de larga duración, ya que el material con el que están fabricadas
(acetato) no sufre alteraciones significativas por el paso del tiempo.

1.2.2. FOTOGRAFÍA

Para obtener una fotografía necesitamos una


cámara fotográfica provista en su interior de una
película de plástico recubierta de productos sen-
sibles a la luz.

Mediante un proceso de revelado se obtiene la


fotografía.

La fotografía tiene grandes ventajas sobre otros sistemas de registro:

• Se puede reproducir infinidad de veces.


• Imágenes muy precisas y de gran calidad.
• Gran duración: la película revelada no sufre cambios con el paso del tiempo,
sin embargo, sí le afectan las condiciones ambientales.

Actualmente existen cámaras fotográficas


digitales que almacenan las imágenes en
soportes informáticos (discos flexibles,
CD Rom, etc.). Tienen grandes ventajas sobre
la fotografía tradicional: imágenes digitaliza-
das que no sufren ninguna alteración, no se
precisa proceso de revelado.

1.2.3. CINTA DE VÍDEO

Es una grabación magnética de imágenes que más tarde se puede reproducir.

Cámara de vídeo Graba

Vídeo Reproduce

Este sistema nos permite registrar objetos en movimiento, y sonidos, cosa que con los
sistemas anteriores es completamente imposible.

12 TAMadrid

-Pág.48-
U. D. 2 - E X P L O R AC I Ó N Y C O M U N I C AC I Ó N D E I D E A S

1.2.4. SISTEMAS INFORMÁTICOS

La revolución informática hace que la aplicación de estos sistemas sea cada vez mayor.

Los registros y presentaciones de ideas técnicas se realizan con la ayuda de programas


informáticos específicos.

Por ejemplo existen los siguientes tipos de programas:

• Procesador de textos.
• Diseño gráfico.
• Animación y multimedia.
• Simulación.
• Diseño CAD.

Todo esto, está facilitando que la


información se pueda guardar de una
forma más fiable y que las presentacio-
nes de ideas sean cada vez de mayor
calidad e innovación.

TAMadrid
13
-Pág.49-
T ECNOLOGÍA

FORMAS DE REPRESENTACIÓN GRÁFICA


2. DE OBJETOS

2.1. BOCETO
Es un dibujo realizado sin ayuda de regla ni compás. El boce-
to contiene los rasgos generales del objeto representado.

Es muy usado en el diseño de elementos novedosos.

2.2. CROQUIS
Es un dibujo realizado a mano alzada, es decir, sin ayuda de
compás ni regla, pero en el cual están definidas todas las caracterís-
ticas del objeto así como sus dimensiones.

El croquis está bien realizado cuando en él se encuentran todos


los datos para poder fabricar el objeto o realizar el plano definitivo.

Para realizar un croquis no se debe utilizar un lápiz demasiado


blando pues ensucia el dibujo. Es muy útil utilizar papel milimetra-
do para que el dibujo nos salga proporcionado.

2.3. DELINEADO
Una vez que elaborado el croquis se realiza el delineado en un
pliego de dibujo, utilizando una escala normalizada.

Entendemos por delineado la aplicación de todas las técnicas


correctas en el uso de los materiales o instrumentos de dibujo para
la elaboración de un plano.

Los dibujos delineados pueden realizarse a lápiz o a tinta.

Debido a la extensión de los soportes informáticos y de los pro-


gramas de CAD (Dibujo Asistido por Ordenador), el delineado a
mano cada vez se emplea menos, pasando el croquis directamente a
un soporte CAD.

14 TAMadrid

-Pág.50-
U. D. 2 - E X P L O R AC I Ó N Y C O M U N I C AC I Ó N D E I D E A S

2.4. PROYECCIÓN DIÉDRICA


En la actualidad se emplean varios sistemas de representación según el campo de apli-
cación para el que se destine. En el dibujo industrial o planos de taller se utiliza el sistema
diédrico.

Para representar un cuerpo de tres dimensiones en un plano se emplean proyecciones


diédricas ortogonales sobre los planos del sistema diédrico.

Proyección ortogonal es cuando se proyecta un objeto


de forma paralela sobre el plano de proyección. También
se le conoce con el nombre de proyección paralela.

Los planos del sistema diédrico son: el plano vertical


(PV), el plano horizontal (PH) que corta al anterior de
forma perpendicular, formando la línea de tierra (LT) y se
suele considerar otro tercer plano, perpendicular a los dos
anteriores, denominado plano de perfil (PP).

Las proyecciones diédricas se conocen comúnmente con el nombre de vistas y existen


varias dependiendo del plano donde se proyecten, las principales son:

• Proyección vertical o alzado: Es la vista principal de la pieza, se obtiene mediante


la proyección de está en el plano vertical (PV). Es la más significativa de la pieza.

• Proyección horizontal o planta: Se obtiene proyectando el objeto en el plano hori-


zontal (PH).

• Proyección de perfil o vista lateral: Con el alzado y planta la pieza no queda com-
pletamente definida y por ello debemos realizar esta tercera proyección sobre el
plano de perfil (PP).

Aunque con estas tres vistas la gran mayoría de los objetos quedan perfectamente defi-
nidos, para su construcción existen otras vistas que podemos utilizar en caso de necesidad,
son:

• Proyección inferior o vista inferior.


• Proyección posterior o vista posterior.

TAMadrid
15
-Pág.51-
T ECNOLOGÍA

2.5. PERSPECTIVAS

Además de representar un objeto mediante el sistema diédrico, en vistas, también se


puede representar de forma tridimensional.

Perspectivas son sistemas de dibujo en tres dimensiones o volúmenes.

Los sistemas más comúnmente utilizados son:

• Perspectiva isométrica.
• Perspectiva caballera.
• Perspectiva cónica.

2.5.1. PERSPECTIVA ISOMÉTRICA

En este sistema los cuerpos se proyectan sobre los ejes de coordenadas según las dimen-
siones principales de estos (largo, ancho, alto). Los ejes de coordenadas forman entre sí
ángulos iguales (120º).

16 TAMadrid

-Pág.52-
U. D. 2 - E X P L O R AC I Ó N Y C O M U N I C AC I Ó N D E I D E A S

2.5.2. PERSPECTIVA CABALLERA

Es aquella en la que dos ejes de coordenadas forman entre sí un ángulo de 90º, donde
situaremos la cara frontal del elemento que queremos proyectar. El tercer eje de coordena-
das nos proporciona la oblicuidad del objeto representado (largo del objeto).

2.5.3. PERSPECTIVA CÓNICA

En este sistema el objeto se representa en el plano de dibujo a partir de un punto. Es la


representación que más se parece a la imagen que aprecia el ojo humano o que sale en una
fotografía.

Se utiliza en el paisajismo, obras públicas, arquitectu-


ra y pintura, por la sensación de proporción, profundidad
y volumen que nos transmite

TAMadrid
17
-Pág.53-
T ECNOLOGÍA

3. OTRAS FORMAS DE REPRESENTACIÓN TÉCNICA

Existen formas diferentes de presentar objetos en un plano, fuera de la representación


del dibujo debido a varias causas:

• Objetos que se repiten muchas veces o que son normalizados.


• Representación gráfica de datos
• Objetos normalizados.
Cuando existe gran repetición en los objetos a realizar o estos son elementos normali-
zados, se utilizan símbolos y esquemas.

Cuando queremos representar datos técnicos, se utilizan gráficos, diagramas y tablas de


datos.

3.1. SÍMBOLOS
Son representaciones abreviadas de elementos muy utilizados.

Cada profesión tiene símbolos propios, como podemos ver a continuación:

Mecánicos

18 TAMadrid

-Pág.54-
U. D. 2 - E X P L O R AC I Ó N Y C O M U N I C AC I Ó N D E I D E A S

Eléctricos Electrónicos

3.2. ESQUEMAS TÉCNICOS


Es un modo de representación abreviada de circuitos e instalaciones, mediante la com-
binación de símbolos normalizados.

Ejemplos de esquemas empleados.

TAMadrid
19
-Pág.55-
T ECNOLOGÍA

Mecánico "Sistema Neumático" Eléctrico

Electrónico

3.3. GRÁFICOS O DIAGRAMAS


Los gráficos son formas de representación de resultados numéricos obtenidos en proce-
sos técnicos o investigaciones científicas.

Los diagramas nos permiten visualizar de una forma rápida los resultados obtenidos.

Gráfico de Sectores Gráfico de Áreas Gráfico de Barras

20 TAMadrid

-Pág.56-
U. D. 2 - E X P L O R AC I Ó N Y C O M U N I C AC I Ó N D E I D E A S

3.4. TABLAS DE DATOS


Las tablas de datos son la forma más utilizada para ordenar y clasificar información
cuando existe un número de datos iguales que pertenecen a diferentes entidades (objetos y
personas...).

4. NORMALIZACIÓN
El dibujo es uno de los mejores sistemas para transmitir ideas técnicas, para que un men-
saje tenga carácter universal, los países han creado organismos competentes encargados de
elaborar acuerdos para unificar criterios mediante convenios de normalización.

La Organización Internacional de Normalización ISO, define como normalización:

La actividad que tiene por objeto instaurar un proceso a través del cual
se unifican criterios respecto a determinadas materias y posibilita la utili-
zación de un lenguaje común en un determinado campo de actuación.

Los objetivos principales de la normalización son:

• Unificación.
• Intercambiabilidad.
• Garantía.
• Economía.

TAMadrid
21
-Pág.57-
T ECNOLOGÍA

Los objetivos nos aportan unas ventajas que son:

• De cara al producto:
• Disminución de modelos.
• Disminución de costes de fabricación.
• Facilidad de acceso al mercado.
• Agilidad en los pedidos.
• Aumento de la confianza de los proveedores.

• De cara a los consumidores:


• Marcas y grados de calidad.
• Servicio y garantía.
• Reducción de costes y tiempo de entrega.
• De cara a la administración:
• Disponer de una documentación.
• Facilitar al usuario en cuanto a normas y especificaciones.

4.1. NORMAS DE REPRESENTACIÓN GRÁFICA


Las normas que codifican un dibujo están recogidas por la norma UNE (Una Norma
Española). Esta contiene las reglas para representar un dibujo con todas sus características
(vistas, acotación, líneas....).

Estamentos que rigen las normas Nacionales e Internacionales.

NORMAS NACIONALES NORMAS INTERNACIONALES


DIN Alemania ISO Organización Internacional de Normalización
BS Reino Unido ISO/R Recomendación elaborada por ISO
UNI Italia ISO/TR Informe técnico elaborado por ISO
UNE España CEN Comité Técnico de Normalización Europeo
NF Francia CEI Comisión Electrónica Internacional
JIS Japón ECISS Comité Europeo de Normalización de Acero
ASA USA
NBN Bélgica

22 TAMadrid

-Pág.58-
U. D. 2 - E X P L O R AC I Ó N Y C O M U N I C AC I Ó N D E I D E A S

4.1.1. FORMATOS DE PAPEL NORMALIZADO

El formato origen es un rectángulo cuya superficie es un m2. y mide (841 x 1189 mm.).

Designación Medidas en mm.


A0 841 x 1.189
A1 594 x 841
A2 420 x 594
A3 297 x 420
A4 210 x 297

Los formatos más pequeños son la mitad del anterior y lo formatos más grandes el
doble.

4.1.2. LÍNEAS NORMALIZADAS

Tipo de línea Forma Grosor


Línea gruesa 0,7
Línea fina 0,35
Línea fina a mano alzada 0,35
Gruesa de trazos 0,5
Fina de trazos 0,35
Fina de trazos y puntos 0,35
Gruesa de trazos y puntos 0,7
Fina de trazos y doble punto 0,35

Aplicaciones en el dibujo técnico

Línea gruesa:

• Aristas y contornos visibles de los cuerpos.


• Límites de roscas.
• Símbolos.
• Representación de espesores de tabiques en la construcción.

TAMadrid
23
-Pág.59-
T ECNOLOGÍA

Línea fina:

• Rayado en secciones y cortes.


• Abatimientos. Secciones abatidas.
• Líneas de cotas.
• Líneas auxiliares de cotas.
• Líneas de referencia.
• Aristas ficticias.
• Ejes cortos.
• Signos superficiales.

Línea llena fina a mano alzada.

• Línea de rotura.
• Parte cortada en cualquier material.
• Rayado en secciones y cortes en las piezas de madera.
Línea gruesa y fina a trazos.

• Aristas y contornos ocultos de las piezas.


Línea fina de trazo y punto.

• Ejes de simetría.
• Partes de los cortes que se encuentran delante de lo representado.
• Limitación de detalles dibujados aparte.
• Trayectorias.

Línea gruesa de trazo y punto.

• Características de tratamiento superficial.


Línea fina de trazos y doble punto.

• Contorno de piezas adyacentes.


• Forma primitiva de la pieza.
• Posiciones externas de piezas móviles, como palancas.

24 TAMadrid

-Pág.60-
U. D. 2 - E X P L O R AC I Ó N Y C O M U N I C AC I Ó N D E I D E A S

Grosor de las líneas.

La anchura adoptada para las líneas tiene que guardar relación con el tamaño y el tipo
del dibujo y mantenerla en toda la representación de un dibujo a la misma escala.

En las líneas finas y gruesas, las anchuras más utilizadas son: 0'25, 0'35, 0'5 y 0'7 mm.

4.1.3. ROTULACIÓN

La rotulación formada por letras y números, es el complemento necesario y estético de


todo dibujo técnico. Las cotas, nombres de la pieza, materiales,... son aclaraciones del dibu-
jo que deben rotularse.

Actualmente, apenas se rotula a mano, se utilizan plantillas normalizadas, letras y cifras


adhesivas de todas las medidas y gran variedad de tipos, y sobre todo, cada vez se dibuja y
se rotula más con ordenador.

A pesar de esto, vamos a recordar algunas características de la rotulación a mano:

• Las proporciones de la escritura vienen normalizadas según UNE 1034-75.


• Las alturas de la mayúsculas y minúsculas no serán inferiores a 2'5 mm.
• La gama de alturas de la escritura es la siguiente: 2'5, 3'5, 5, 7, 17, 20 mm.
• La gama de anchura de trazos normalizados es la siguiente: 0'18, 0'25, 0'35, 0'7 mm.

TAMadrid
25
-Pág.61-
T ECNOLOGÍA

La rotulación puede ser inclinada o cursiva, siendo la inclinación de 75º. La primera se


utiliza preferentemente en la industria metalúrgica, y la segunda en la construcción y en cir-
cuitos y esquemas eléctricos y electrónicos.

4.1.4. REPRESENTACIÓN NORMALIZADA

Las normas a seguir al representar un objeto están determinadas por el sistema diédrico.

Una vez obtenidas las proyecciones se abaten los planos para obtener las vistas. La colo-
cación de estas siguen dos sistemas:

4.1.4.1. Sistema Europeo

Es el método empleado en Europa según UNE 1032.

El signo que se coloca para saber que un plano está desa-


rrollado en el Sistema Europeo es el siguiente:

26 TAMadrid

-Pág.62-
U. D. 2 - E X P L O R AC I Ó N Y C O M U N I C AC I Ó N D E I D E A S

4.1.4.2. Sistema Americano

Es el sistema empleado en Estados Unidos y Canadá,


normalizado según DIN 6.

Su signo es el siguiente:

4.1.5. ACOTACIÓN NORMALIZADA

Acotación es el conjunto de líneas, cifras y signos indicados en un dibujo que determi-


nan la forma y dimensiones del objeto representado. Es una operación muy importante, en
la cual no debe olvidarse nada ni se pueden cometer errores.

TAMadrid
27
-Pág.63-
T ECNOLOGÍA

Las reglas de acotación de dibujo están determinadas por las normas UNE 1039-75 y
DIN 406.

4.1.5.1. Elementos empleados en la acotación

Líneas de cota.

Son líneas continuas finas, acabadas en punta de flecha que


se colocan paralelas a la línea cuya longitud indica.

Líneas de referencia.

Son líneas continuas finas perpendiculares a la línea cota y


sobre salen 2 a 3 mm. de ésta.

Números de cota.

Expresan la longitud de la línea indicada. Las unidades que se emplean: milímetros y


grados, minutos y segundos para medidas angulares. Se colocan sobre la medida de cota.

28 TAMadrid

-Pág.64-
U. D. 2 - E X P L O R AC I Ó N Y C O M U N I C AC I Ó N D E I D E A S

4.1.5.2. Formas de acotación

Cota continu a Cota en línea base

Cota por coordenadas Cota radial

Cota angular

TAMadrid
29
-Pág.65-
T ECNOLOGÍA

RESUMEN

• Toda idea que nos pueda surgir debemos reflejarla en algún medio físico, para
poder compartirla y contrastarla con las de los demás.

• El dibujo técnico es el idioma de los profesionales del diseño gráfico. Los instru-
mentos del dibujo técnico son: Papel ( opaco, transparente, cansón, pergamino,
papel-tela, milimetrado), lápiz, portaminas, sacapuntas, goma de borrar, regla mili-
métrica, escuadra y cartabón, compás, estilógrafos, plantillas.

• Otros instrumentos para el registro de ideas técnicas: Transparencias, fotografía,


cinta de vídeo, sistemas informáticos.

• Formas de representación gráfica de objetos: Boceto, croquis, delineado, proyec-


ción diédrica, perspectivas (isométrica, caballera, cónica).

• Otras formas de representación técnica: Símbolos, esquemas técnicos, gráficos o


diagramas, tablas de datos.

• La normalización es la actividad que tiene por objeto instaurar un proceso a través


del cual se unifican criterios respecto a determinadas materias y posibilita la utili-
zación de un lenguaje común en un determinado campo de actuación. Sus objeti-
vos son: Unificación, intercambiabilidad, garantía, economía.

30 TAMadrid

-Pág.66-
U. D. 2 - E X P L O R AC I Ó N Y C O M U N I C AC I Ó N D E I D E A S

EJERCICIOS DE AUTOCOMPROBACIÓN

1. Escribe los términos siguientes junto a la descripción que le corresponda.

plantillas - estilógrafo - lápiz - escuadra - cartabón - papel - por-


taminas - sacapuntas - compas - regla - goma

_____ Es el medio más utilizado como soporte de un dibujo.


_____ Es una barrita de grafito (mina) pegada en el interior de un cilíndro.
_________ Es un útil de plástico o metal que lleva en su interior, y sin pegar, la
mina de grafito.
__________ Es la mejor herramienta para sacar punta al lápiz
____ Debe ser blanda, flexible y de color claro
_____ Utensilio de madera o plástico de forma rectangular con un espesor
de 2 a 5 milímetros y una longitud de 30 a 100 cm.
_______ Tiene forma de triángulo rectángulo escaleno (los tres lados desi-
guales) y dos de sus ángulos son de 30º y 60º.
________ Tiene forma de triángulo rectángulo isósceles (dos lados iguales) y
dos de sus ángulos son de 45º.
______ Es un útil de dibujo que se emplea para trazar arcos y circunferen-
cias.
___________ Para delinear son los útiles para el pasado a tinta.
_________ Son accesorios de plástico de formas diversas que ahorran mucho
tiempo en la realización de dibujos

2. Relaciona las partes del estilógrafo con su utilidad.

A. Capuchón B. Puntera C. Boquilla D. Depósito E. Mango

Elemento Utilidad
Almacena la tinta
Marca la línea
Facilita su manejo
Impide que se seque
Une la puntera con el depósito

TAMadrid
31
-Pág.67-
T ECNOLOGÍA

3. Indica si las siguientes afirmaciones son verdaderas o falsas.


V F
  Boceto es un dibujo realizado con ayuda de la regla y compás y que están definidas las
características que generales del objeto.

  Los diagramas son formas abreviadas de representación de elementos muy utilizados.

  Las siglas UNE significan, Unión de Normas Españolas

  La proyección vertical o alzada es la vista principal del objeto.

  La perspectiva cónica es la representación que más se parece a lo que aprecia el ojo


humano.

"Tacha lo que no corresponda".

4. Indica en la cota el nombre de cada una de sus partes.

1) Línea de cota

2) Línea de referencia 25

3) Número de cota.

5. Une con flechas cada formato normalizado, con sus dimensiones.

Formato • • Dimensión en mm.

A0 • • 297 x 420

A1 • • 594 x 841

A2 • • 841 x 1.189

A3 • • 420 x 594

A4 • • 210 x 297

A5 • • 105 x 148

A6 • • 148 x 210

32 TAMadrid

-Pág.68-
U. D. 2 - E X P L O R AC I Ó N Y C O M U N I C AC I Ó N D E I D E A S

6. Identifica las caras de la perspectivas en las diferentes vistas.

7. Dibuja las diferentes vistas de la pieza ayudándote de los cuadros de la parte inferior.

8. Representa en perspectiva isométrica la siguiente figura.

9. Representa en perspectiva caballera la siguiente figura.

TAMadrid
33
-Pág.69-
T ECNOLOGÍA

RESPUESTAS A LOS EJERCICIOS

1. Escribe los términos siguientes junto a la descripción que le corresponda.

plantillas - estilógrafo - lápiz - escuadra - cartabón - papel

PAPEL Es el medio más utilizado como soporte de un dibujo.


LAPIZ Es una barrita de grafito (mina) pegada en el interior de un cilíndro.
PORTAMINAS Es un útil de plástico o metal que lleva en su interior, y sin pegar, la
mina de grafito.
SACAPUNTAS Es la mejor herramienta para sacar punta al lápiz
GOMA Debe ser blanda, flexible y de color claro
REGLA Utensilio de madera o plástico de forma rectangular con un espesor
de 2 a 5 milímetros y una longitud de 30 a 100 cm.
CARTABON Tiene forma de triángulo rectángulo escaleno (los tres lados desi-
guales) y dos de sus ángulos son de 30º y 60º.
ESCUADRA Tiene forma de triángulo rectángulo isósceles (dos lados iguales) y
dos de sus ángulos son de 45º.
COMPAS Es un útil de dibujo que se emplea para trazar arcos y circunferen-
cias.
ESTILOGRAFO Para delinear son los útiles para el pasado a tinta.
PLANTILLA Son accesorios de plástico de formas diversas que ahorran mucho
tiempo en la realización de dibujos

2. Relaciona las partes del estilógrafo con su utilidad.

A. Capuchón B. Puntera C. Boquilla D. Depósito E. Mango

Elemento Utilidad
D Almacena la tinta
B Marca la línea
E Facilita su manejo
A Impide que se seque
C Une la puntera con el depósito

34 TAMadrid

-Pág.70-
U. D. 2 - E X P L O R AC I Ó N Y C O M U N I C AC I Ó N D E I D E A S

3. Indica si las siguientes afirmaciones son verdaderas o falsas.


V F
 
X Boceto es un dibujo realizado con ayuda de la regla y compás y que están definidas las
características que generales del objeto.

 
X Los diagramas son formas abreviadas de representación de elementos muy utilizados.

 
X Las siglas UNE significan, Unión de Normas Españolas


X  La proyección vertical o alzada es la vista principal del objeto.


X  La perspectiva cónica es la representación que más se parece a lo que aprecia el ojo
humano.

"Tacha lo que no corresponda".

4. Indica en la cota el nombre de cada una de sus partes.

1) Línea de cota 3

2) Línea de referencia 25

3) Número de cota. 1 2

5. Une con flechas cada formato normalizado, con sus dimensiones.

Formato • • Dimensión en mm.

A0 • • 297 x 420

A1 • • 594 x 841

A2 • • 841 x 1.189

A3 • • 420 x 594

A4 • • 210 x 297

A5 • • 105 x 148

A6 • • 148 x 210

TAMadrid
35
-Pág.71-
T ECNOLOGÍA

6. Identifica las caras de las perspectivas en las diferentes vistas.

1 5 2 5 3
4

3 2
6
4 9 6
9 1
7 7
8 8

7. Dibuja las diferentes vistas de la pieza ayudándote de los cuadros de la parte inferior.

8. Representa en perspectiva isométrica la siguiente figura.

9. Representa en perspectiva caballera la siguiente figura.

36 TAMadrid

-Pág.72-
portada TROPA 19/3/07 19:51 Página 1

FUERZAS ARMADAS
PROFESIONALES
CURSO DE APOYO
A LA PREPARACIÓN
DE LAS PRUEBAS DE ACCESO
A UNA RELACIÓN DE SERVICIOS
DE CARÁCTER PERMANENTE

TECNOLOGÍA
2ª parte
Unidades didácticas 3 y 4

DIGEREM

MINISTERIO
DE DEFENSA
FUERZAS ARMADAS SUBDIRECCIîN GENERAL
DE TROPA Y MARINERIA
PROFESIONAL
PROFESIONALES
CURSO DE APOYO
A LA PREPARACIÓN
DE LAS PRUEBAS DE ACCESO
A UNA RELACIÓN DE SERVICIOS
DE CARÁCTER PERMANENTE

TECNOLOGÍA
2ª parte
Unidades didácticas 3 y 4
La Ley 8/2006 de Tropa y Marinería, en su artículo 16,1, establece que “la formación
en las Fuerzas Armadas garantizará que los militares profesionales de tropa y
marinería puedan adquirir, actualizar o ampliar sus conocimientos para un mayor
desarrollo personal y profesional”. En cumplimiento de este mandato, el Ministerio
de Defensa edita el presente material didáctico para facilitar a los militares
profesionales de tropa y marinería, alumnos de los cursos de formación
presencial que se imparten a través de la Dirección General de Reclutamiento y
Enseñanza Militar, los apoyos necesarios para preparación de dichos cursos, que
permitirán, siempre que superen las pruebas correspondientes, la obtención de la
titulación de graduado en Educación Secundaria, acreditación para el acceso a
los ciclos formativos de la Formación Profesional de grado medio o de grado
superior, acceso a las Escalas de Suboficiales, Tropa Permanente, Guardia Civil
y Policía Nacional.

CATÁLOGO GENERAL DE PUBLICACIONES


http://www.060.es

Edita:

© Autor y editor
NIPO: 076-10-204-9 NIPO: 076-10-205-4 (edición en línea)
Depósito Legal: M-32363-2009
Diseño y programación: cimapress
Tirada: 1300 ejemplares
Fecha de edición: septiembre, 2010

Prohibida la reproducción total o parcial de esta obra, por cualquier medio sin autorización escrita del editor
TECNOLOGÍA
2ª parte

SUMARIO

Unidad didáctica Pág.

3. PLANIFICACIÓN Y REALIZACIÓN 5

4. ORGANIZACIÓN Y GESTIÓN 45
U . D . 3 . - P LANIFICACIÓN Y REALIZACIÓN

ÍNDICE
OBJETIVOS . . . . . . . . . . . . . . . . . . . . . . . . . . . . . . . . . . . . . . . . . . . . . . . . . . . . . . . . . . . . . .2

INTRODUCCIÓN . . . . . . . . . . . . . . . . . . . . . . . . . . . . . . . . . . . . . . . . . . . . . . . . . . . . . . . . .3

MAPA CONCEPTUAL . . . . . . . . . . . . . . . . . . . . . . . . . . . . . . . . . . . . . . . . . . . . . . . . . . . . .4

DESARROLLO DE CONTENIDOS . . . . . . . . . . . . . . . . . . . . . . . . . . . . . . . . . . . . . . . . . .5

1. PROCESOS DE TRABAJO . . . . . . . . . . . . . . . . . . . . . . . . . . . . . . . . . . . . . . . . . .5
1.1. ORGANIZACIÓN DE UN PROCESO . . . . . . . . . . . . . . . . . . . . . . . . . . . . . . . . . . . .6

2. DOCUMENTACIÓN DE UN PROCESO . . . . . . . . . . . . . . . . . . . . . . . . . . . . . . .7
2.1. HOJAS DE PROCESO . . . . . . . . . . . . . . . . . . . . . . . . . . . . . . . . . . . . . . . . . . . . . . . .7
2.2. DIAGRAMAS . . . . . . . . . . . . . . . . . . . . . . . . . . . . . . . . . . . . . . . . . . . . . . . . . . . . . . .8

3. HERRAMIENTAS Y SUS CLASES. MÁQUINAS-HERRAMIENTAS.


ÚTILES. PROCEDIMIENTOS DE FABRICACIÓN Y ACABADO . . . . . . . . . .11
3.1. HERRAMIENTAS . . . . . . . . . . . . . . . . . . . . . . . . . . . . . . . . . . . . . . . . . . . . . . . . . . .12
3.2. MÁQUINAS HERRAMIENTAS . . . . . . . . . . . . . . . . . . . . . . . . . . . . . . . . . . . . . . . .18
3.3. ÚTILES . . . . . . . . . . . . . . . . . . . . . . . . . . . . . . . . . . . . . . . . . . . . . . . . . . . . . . . . . . .21
3.4. PROCEDIMIENTOS DE FABRICACIÓN Y ACABADO . . . . . . . . . . . . . . . . . . . .22

4. SEGURIDAD EN PUESTO DE TRABAJO.


NORMAS BÁSICAS DE SEGURIDAD EN EL TALLER . . . . . . . . . . . . . . . . .29

5. TOLERANCIA. CONTROL DE CALIDAD . . . . . . . . . . . . . . . . . . . . . . . . . . . .32


5.1. TOLERANCIA . . . . . . . . . . . . . . . . . . . . . . . . . . . . . . . . . . . . . . . . . . . . . . . . . . . . . .32
5.2. CONTROL DE CALIDAD . . . . . . . . . . . . . . . . . . . . . . . . . . . . . . . . . . . . . . . . . . . .32

RESUMEN . . . . . . . . . . . . . . . . . . . . . . . . . . . . . . . . . . . . . . . . . . . . . . . . . . . . . . . . . . . . .33

EJERCICIOS DE AUTOCOMPROBACIÓN . . . . . . . . . . . . . . . . . . . . . . . . . . . . . . . . .34

RESPUESTAS A LOS EJERCICIOS . . . . . . . . . . . . . . . . . . . . . . . . . . . . . . . . . . . . . . . .36

-Pág.5-
TECNOLOGÍA

OBJETIVOS
Al finalizar el estudio de esta Unidad Didáctica, el alumno será capaz de:

• Comprender la importancia de la planificación del trabajo.

• Elaborar documentos que faciliten la planificación de la realización de un pro-


ducto.

• Valorar los conceptos básicos de tolerancias.

• Elegir los diferentes tipos de máquinas y sus herramientas para su aplicación en


la realización de un producto.

• Aplicar las normas correctas de seguridad en el puesto de trabajo.

• Diferenciar los procedimientos para la obtención de productos de calidad.

-Pág.6-
U . D . 3 . - P LANIFICACIÓN Y REALIZACIÓN

INTRODUCCIÓN
E n la industria moderna, la elaboración de un producto pasa por las siguientes fases:
Planificar “pensar”, plasmar “obtener el producto” y verificar “dar el visto bueno”.

Con la finalidad de obtener un producto de mayor calidad y competitivo, para satisfacer


las necesidades del comprador, se emplea más tiempo en pensar cómo vamos a conse-
guir los mejores resultados con el precio más económico.

El alto coste económico que supone los errores cometidos durante la realización del pro-
ducto, hace muy necesario una buena planificación antes de una realización.

-Pág.7-
TECNOLOGÍA

M A PA C O N C E P T UA L

PLANIFICACIÓN
Y REALIZACIÓN

DAR EL
PENSAR FABRICAR
VISTO BUENO

ORGANIZACIÓN CONTROL
MÁQUINAS
DOCUMENTACIÓN DE CALIDAD

MEDIR
PLANO HERRAMIENTAS

COMPARAR
DIAGRAMA DE SEGURIDAD E
RECORRIDO HIGIENE
VERIFICAR

DIAGRAMA DE
PROCESO

-Pág.8-
U . D . 3 . - P LANIFICACIÓN Y REALIZACIÓN

1. PROCESOS DE TRABAJO
Cuando un cliente realiza un pedido, la primera necesidad de la empresa que elabora dicho
producto es la de planificar un proceso de producción.
Esté proceso de producción requiere el estudio de cada uno de los pasos a seguir, desde el
suministro de materia prima hasta la entrega del producto terminado, al cliente.

Un proceso es un conjunto de pasos seguidos en el cual


el material se va transformando conforme a un plan previsto.

Los procesos pueden ser:


– Procesos Primarios.
– Procesos Secundarios.
Procesos Primarios son los realizados sobre materias primas de origen natural por ejemplo,
el tratamiento del petróleo para la obtención de aceites industriales y combustible.
Procesos Secundarios son tratamientos de materiales que ya han sufrido un proceso prima-
rio, para obtener objetos de mayor complejidad, por ejemplo un tornillo de acero (el acero
ya ha sufrido un proceso primario).

Una “producción” es el conjunto de procesos que llevan a un producto final. Esta produc-
ción estará condicionada por el volumen del pedido y el plazo de ejecución, por ejemplo, no es
igual una producción de vehículos que otra de productos de temporada, como el calzado.
Según estos condicionantes podemos distinguir dos tipos de producción:
– Producción continua.
– Producción flexible.
“Producción continua o en serie” es la ideal para la obtención de grandes cantidades del
mismo producto, durante un largo periodo de tiempo, a un precio competitivo. Cada opera-
rio realiza un único paso u operación dentro del proceso de fabricación de forma repetitiva
y rutinaria, con el objeto de realizarlo en el menor tiempo posible.
Para poner en marcha este tipo de producción se necesita que las empresas tengan una línea
o cadena de fabricación, el operario tiene un puesto fijo y es el producto el que pasa por cada
fase y operario hasta que se acaba, ejemplo de este tipo de producción es:
– Montaje de Vehículos.
– Fabricación de Electrodomésticos.

5
-Pág.9-
TECNOLOGÍA

“Producción flexible o intermitente” es la utilizada para la fabricación de un producto por


un tiempo determinado. Posteriormente se tendrá la posibilidad de realizar cambios en
máquinas y utillajes para permitir la realización de otros productos, son ejemplos de este
tipo de producción:
– Confección de ropa, ya que en verano no se realiza la misma producción que en invierno.
– Manufacturas de productos hortícolas, debido a que en agosto no se cosecha lo mismo
que en abril.

La fabricación es el fruto de sucesivos procesos secundarios.

1.1. ORGANIZACIÓN DE UN PROCESO


La única forma de obtener más cantidad de producto de la mejor calidad y en el menor tiem-
po posible, utilizando el menor número de recursos (materiales, económicos y humanos), es
mediante una organización perfecta de un proceso productivo, siguiendo unas pautas concretas:

HERRAMIENTAS
CONTINUA
ESTUDIO Estudio del Mercado
DEMANDA
INTERMITENTE

CANTIDADES
PROGRAMACIÓN Locales, Máquinas, Personas
PRODUCCIÓN
TIPO DE PROYECTO

STOCKS
MATERIALES Y Gestión de Almacén
SUMINISTROS
MATERIAS PRIMAS
SUMINISTROS “Just in time”

SECUENCIACIÓN
Fabricación
HOJAS DE RUTA RECURSOS, MATERIALES

MÁQUINAS, HERRAMIENTAS

ANÁLISIS DE MUESTRAS
CONTROL Calidad Total
DE CALIDAD
RITMO DE PRODUCCIÓN

6
-Pág.10-
U . D . 3 . - P LANIFICACIÓN Y REALIZACIÓN

2. DOCUMENTACIÓN DE UN PROCESO
La documentación de un proceso consiste en reflejar de forma física (papel) las actividades
que se han planificado “pensado” para coordinar y controlar todos los elementos que van a
intervenir en el proceso de obtención del producto.
Los tipos de documentos utilizados en la planificación de la fabricación de un producto son:
– Hojas de procesos.
– Diagramas.

2.1. HOJAS DE PROCESO


Se utilizan para especificar los detalles de cada una de las operaciones que intervienen en la
fabricación de un producto.
Existen dos tipo de hojas de proceso:
– Hoja de ruta.
– Hoja de proceso.

2.1.1. HOJA DE RUTA


Conocida técnicamente como hoja analítica de
fases. Se utiliza para ver de una forma general todas
las operaciones y máquinas por las que ha de pasar
el producto desde su entrada como material en
bruto, hasta su entrada en almacén como producto
terminado.

7
-Pág.11-
TECNOLOGÍA

2.1.2. HOJA DE PROCESO


Conocida técnicamente como hoja analítica de
operaciones. Es un resumen breve y conciso de los
factores que intervienen en la fabricación del pro-
ducto.
Se especifica herramientas, útiles de trabajo,
tiempo programado, datos técnicos (velocidad de
trabajo, etc.).

2.2. DIAGRAMAS
Conocidos técnicamente como cronogramas. Son esquemas gráficos que facilitan la visión
global de un proceso productivo.
Las acciones que se realizan dentro del proceso se representan mediante unos signos nor-
malizados, según se muestra en la siguiente tabla, donde se puede observar el símbolo, como
afecta al producto y por último la acción concreta que se realiza sobre este.

Símbolo Resultado Acción


Modifica un objeto Operación

Verifica cantidad o calidad Inspección

Stock Almacenaje

No permite la ejecución inmediata de la acción siguiente Espera/Demora

Mueve el objeto de un lugar de trabajo a otro Transporte

Control a píe de máquina Acciones simultáneas

8
-Pág.12-
U . D . 3 . - P LANIFICACIÓN Y REALIZACIÓN

Se utilizan dos tipos de diagramas:


– Diagrama de operaciones.
– Diagrama de recorrido.

2.2.1. DIAGRAMA DE OPERACIONES


Es una representación detallada de tareas concretas (operaciones o verificaciones) sobre el
producto. Este diagrama permite apreciar el orden de las operaciones importantes, a simple
vista, para realizar un objeto, así como los materiales que se necesitan para su fabricación o
montaje.

Ejemplo de diagrama de operaciones

Se efectúa colocando los símbolos de las acciones programadas sobre una línea vertical y
los materiales que se necesiten se indicarán mediante líneas horizontales.

9
-Pág.13-
TECNOLOGÍA

2.2.2. DIAGRAMA DE RECORRIDO


Documento en el cual se anotan y describen todas las acciones, en el mismo orden que se
realizan en el proceso de fabricación y montaje de un producto.

10
-Pág.14-
U . D . 3 . - P LANIFICACIÓN Y REALIZACIÓN

HERRAMIENTAS Y SUS CLASES. MÁQUINAS-


3. HERRAMIENTAS. ÚTILES. PROCEDIMIENTOS
DE FABRICACIÓN Y ACABADO MÁS CORRIENTES
Herramientas para sujetar
Herramientas para cortar
Auxiliares
Herramientas para girar
Herramientas para golpear
Manuales
Lima
Macho
Mecanizado Para roscar
Cojinete
Herramientas Sierra

Broca
Para máquina Cuchillas
Muelas abrasivas

Portatil
Taladradora
Fija

Electroesmeriladora
Máquinas Esmeril
Fijo

Torno paralelo

Mordaza
De sujeción o amarre Bridas
Platos
Útiles

De ángulos
De verificación o control
De formas

Sin arranque de viruta


Fabricación
Procedimientos Con arranque de viruta
de fabricación y
acabado
Acabado Tratamientos superficiales

11
-Pág.15-
TECNOLOGÍA

INTRODUCCIÓN
Antes de entrar a definir cada uno de los conceptos de herramienta, máquina y máquina
herramienta diferenciemos claramente cada uno de estos conceptos:
– Máquina, en general, es un artificio para aprovechar la acción de una fuerza. Sirven para
disminuir los esfuerzos del hombre así como su fatiga.
– Herramienta, instrumento que pone en movimiento la mano del hombre.
– Máquina-herramienta, es la que, por procedimientos mecánicos, hace funcionar a una
herramienta sustituyendo la mano del hombre. La máquina-herramienta es actualmente
una herramienta fundamental en la industria por aportar las siguientes ventajas:
– - Transforma y aumenta las fuerzas humanas haciendo más llevadero el trabajo.
– - Aumenta la velocidad de trabajo. Se aumenta la producción y se disminuye el precio
del producto.

3.1. HERRAMIENTAS
Su funcionamiento se basa en la aplicación sobre ella de un movimiento lineal o circular,
mediante una fuerza, haciendo que se produzca una modificación sobre el elemento que se
emplea.
Las herramientas se pueden clasificar:
– Herramientas manuales:
– - Auxiliares.
– - De mecanizado.
– Herramientas para máquina.

3.1.1. HERRAMIENTAS AUXILIARES


Son las herramientas manuales cuya acción sobre el elemento que se emplean no produce
ningún arranque de material.
Las herramientas auxiliares utilizadas en el taller son:
– Para sujetar.
– Para corte.
– Para girar.
– Para golpear.

12
-Pág.16-
U . D . 3 . - P LANIFICACIÓN Y REALIZACIÓN

3.1.1.1. Para sujetar


Sirven para sujetar chapas, cortar o doblar alambres y montar arandelas elásticas.
Los más utilizados son:

Alicates universales Alicates de boca plana

Alicates de boca redonda Alicates de presión

Alicates para arandelas elásticas Tenazas

3.1.1.2. Para corte


Se emplean para cortar materiales metálicos, su tamaño y forma depende del grosor y tama-
ño de la pieza a cortar.
Las más utilizadas son el cortaalambres y las tijeras de cortar chapa.

Cortaalambres Tijeras de chapa

3.1.1.3. Para girar


Se emplean para apretar o aflojar elementos roscados (tornillo, tuercas, etc.) se pueden cla-
sificar en:
– Llaves.
– Destornilladores.
Llaves, son instrumentos de acero que se emplean para apretar o aflojar las tuercas en los
tornillos. Sus partes principales son: Cuerpo, cabeza y boca.
Las llaves se clasifican esencialmente en: Fijas, ajustables, de uña.

13
-Pág.17-
TECNOLOGÍA

– Llaves fijas, se llaman así por que el tamaño de la boca solo sirve para una medida de
tuerca, se clasifican en:

Plana Estrella

De vaso De tubo

– Llaves ajustables, se denominan así por tener una boca adaptable al tamaño de la tuerca
a apretar o aflojar.
– Es aconsejable en su utilización:
– - Emplear este tipo de llaves SÓLO cuando no se tiene otra apropiada.
– - No emplearla para grandes esfuerzos.
– - Ajustar la boca dejando el mínimo espacio entre ella y la tuerca.
– - Mantener limpio el mecanismo de ajuste.
– Existen dos tipos, de caras planas (vulgarmente llamada llave inglesa) y para tubo (mal
llamada llave grifa) conocida técnicamente con el nombre de llave Stillon. Existe una
variante de está que es la llave de cadena.

Llave inglesa Llave de grifa Llave de cadena

– Llaves de uña, reciben el nombre de llaves de gancho. Se utilizan para tuercas redondas
con muescas.

Destornilladores, son herramientas que se emplean para apretar o aflojar los tornillos y tira-
fondos, ajustándolos en una ranura que tienen estos en la cabeza. Constan de mango, vástago y

14
-Pág.18-
U . D . 3 . - P LANIFICACIÓN Y REALIZACIÓN

punta. Su nombre siempre va acompañado de la forma de la ranura de la cabeza del tornillo para
el que se utiliza. Los más utilizados son:

De punta plana Phillips o de estrella Torx

3.1.1.4. Para golpear


Para golpear se emplean generalmente los martillos, una veces directamente sobre el ele-
mento y otras con la ayuda de distintas herramientas como punzones y barras de acero.
Podemos distinguir las siguientes herramientas para golpear:
– Martillos.
– Mazas.
– Punzones o botadores.
Martillo, herramienta para golpear. Su forma y tamaño son muy variados, de acuerdo con el
trabajo a realizar. Los principales son:

De bola De peña De carpintero

Mazas, son martillos de forma y material adecuados para trabajos especiales como por ejem-
plo el montaje de piezas acabadas, para golpear materiales blandos, etc. Las más utilizadas son:

De plástico De goma

Punzones o botadores, son varillas de acero que terminan en punta. Se utilizan para colocar
y sacar pasadores.

15
-Pág.19-
TECNOLOGÍA

3.1.2. HERRAMIENTAS DE MECANIZADO


Son las herramientas manuales cuya acción sobre el elemento que se emplean produce
arranque de material.
Las herramientas para mecanizado más utilizadas en el taller son:
– Lima
– Sierra
– Para roscar: Macho y cojinete.

3.1.2.1. Limas
Barra de acero templada, de superficie áspera, que se utiliza para rebajar y pulir piezas.
En ella podemos distinguir las siguientes partes:

A la hora de elegir la lima debemos tener en cuenta las siguientes características:


Forma, que tiene a lo largo de su cuerpo.

Cuadrada Rectangular Triangular Redonda

Tamaño, es la longitud del cuerpo expresada en pulgadas. La lima debe ser más larga que la
superficie a limar.
Picado, es la aspereza que tiene la lima en su cuerpo.

Picado simple Picado doble

3.1.2.2. Sierra de mano


Es un utensilio que consta de dos partes:
Arco de Sierra, su función es la de sujetar y tensar
la hoja de sierra.

16
-Pág.20-
U . D . 3 . - P LANIFICACIÓN Y REALIZACIÓN

Hoja de sierra, hoja de acero flexible pro-


vista de dientes triangulares que son los
que realizan el corte.
Las características principales de una hoja de sierra son: Tamaño y disposición de los dientes.

3.1.2.3. Para roscar


Por roscar entendemos la operación que se realiza para obtener una rosca en piezas metáli-
cas, estas pueden ser, una rosca interior (tuerca) o una exterior (tornillo).
Para realizar esta operación de forma manual se utilizan las siguientes herramientas, machos
de roscar (roscas interiores) y cojinetes de roscar (roscas exteriores).
Machos de roscar, son tornillos de acero de gran dureza provistos de ranuras que al girar
cortan el material realizando la rosca en agujeros previamente taladrados.

Para ayudarnos a realizar el trabajo


(girar) existen los bandeadores o
giramachos.

Cojinetes de roscar, anillo de acero de gran dureza provisto de


ranuras que al girar cortan el material.

Para ayudarnos a hacer el trabajo


(girar) existen los portacojinetes o
portaterrajas.

3.1.3. HERRAMIENTAS DE MÁQUINA

3.1.3.1. Broca
Barra redonda de acero de gran dureza, con
surcos espirales y afilada en la punta por un extre-
mo, que al girar corta y penetra en una pieza.

17
-Pág.21-
TECNOLOGÍA

3.1.3.2. Cuchillas
Barra rectangular de material de gran dureza, cuya
punta termina en forma de cuña, afilada según unos
ángulos determinados para cortar el material.

3.1.3.3. Muelas abrasivas


Son granos de abrasivo unidos entre sí por
un pegamento, formando múltiples aristas de
corte. La forma de las muelas es cilíndrica.

3.2. MÁQUINAS HERRAMIENTAS


Como ya definimos máquina herramienta, es la máquina que por procedimientos mecánicos,
hace mover a una herramienta para que arranque material.
Las máquinas herramientas más usadas en el taller mecánico son:
– Taladradora.
– Electroesmeriladora (Esmeril).
– Torno Paralelo.

3.2.1. TALADRADORA
Es la máquina herramienta que hace que la broca arranque material. Para ello la taladrado-
ra dispone de dos movimientos, uno de rotación (la broca gira) y otro de avance (la broca pene-
tra en la pieza).
Es la máquina herramienta más utilizada en el taller para hacer agujeros de pequeña y
mediana medida y de no mucha precisión.
Las taladradoras se pueden clasificar:
– Portátiles.
– Fijas: de sobremesa y de columna.

3.2.1.1. Taladradora portátil


Es una taladradora para uso manual, en sitios o elementos de difícil acceso con otros tipos
de taladradoras.

18
-Pág.22-
U . D . 3 . - P LANIFICACIÓN Y REALIZACIÓN

Las características principales son que el movimiento de avance se realiza mediante el empuje
del operario y el movimiento de rotación puede ser por un motor eléctrico o un motor neumático.

Eléctrica Neumática

3.2.1.2. Taladradoras fijas


Dispositivo de
Son máquinas para trabajos en serie o fijación de la
taladros de mayor diámetro. broca
Los elementos que componen la taladra-
dora son:
Cabezal
– Soporte general o bancada.
– Mesa de trabajo.
– Cabezal.
– Dispositivo de fijación de broca.
Los tipos más importantes de taladrado-
ras fijas son de columna y de sobremesa. Mesa

Taladradora fija de columna, su armazón


principal es una columna redonda, sobre
la cual esta sujeta la mesa de trabajo y
que tiene la particularidad de subir, bajar
y también desplazarse lateralmente para
poder apoyar en la base las piezas de
mucha altura que necesitemos taladrar. Soporte

Taladradora fija de sobremesa, se denomina así por que


es de pequeña altura y se sitúa encima de un banco o
mesa de trabajo.
Se utiliza por ello para piezas de pequeña altura.

19
-Pág.23-
TECNOLOGÍA

3.2.2. ELECTROESMERILADORA
Su nombre más conocido es esmeril. Es una máquina herramienta que mediante una muela
abrasiva gira y produce el desprendimiento de pequeñas partículas de una pieza.
El esmeril consta:
– Carcasas Protectoras.
– Soporte.
– Motor.
– Pie o bastidor.
Carcasas
La función del pie es la de elevar el esmeril a una altura Soporte
correcta de trabajo y sujetarlo al suelo. El motor eléctrico, es
el que realiza el giro, este lleva un eje por cada extremo
donde se sujetan las muelas. Las carcasas de protección, en
cada una de las muelas, retienen las partículas desprendidas
Motor
e impiden que se pueda introducir una por algún sitio que no
sea el indicado para trabajar. El soporte es una pletina regu-
lable, que sirve de apoyo para el elemento que vamos a esme-
rilar y para que el operario no tenga que mantenerlo a pulso.
Las funciones principales del esmeril son: afilar herra- Pie o bastidor
mientas, quitar rebabas y pulir.

3.2.3. TORNO PARALELO


Es la máquina herramienta más utilizada para el mecanizado de piezas de revolución, enten-
diendo por piezas de revolución las cilíndricas, conos, etc.
En el torno, la pieza que se ha de mecanizar, gira alrededor de un eje de rotación y la herra-
mienta avanza contra la pieza.
Las partes principales de un torno son:
Plato de agarre Torre portaherramientas

Contracabezal

Cabezal
Carros

Bancada

20
-Pág.24-
U . D . 3 . - P LANIFICACIÓN Y REALIZACIÓN

Bancada, es el cuerpo o chasis de la máquina, sobre ella están apoyados el resto de los ele-
mentos. En la parte superior de la bancada están talladas unas guías en forma triangular sobre
la cual se apoyan y deslizan los carros.
Cabezal, es el elemento que transmite el movimiento de giro a la pieza desde el motor.
Desde este podemos variar la velocidad de giro de la pieza.
Carros, son los elementos que hacen que la cuchilla pueda avanzar contra la pieza, para
arrancar material, de forma manual (mediante una manivela) o de forma automática.
Los carros principales de un torno son:
– Carro principal o longitudinal, es el que avanza la cuchilla contra la pieza a lo largo de
esta.
– Carro transversal, es el que avanza la cuchilla contra la pieza en su ancho (el diámetro),
está situado encima del carro principal.
– Carro orientable o charriot, es el que nos permite realizar conos en estas máquinas,
situado encima del carro transversal. Su movimiento es siempre manual.
Contracabezal, es un elemento auxiliar del torno que nos sirve para sujetar piezas de mucha
longitud por el lado contrario al cabezal y tener mayor seguridad a la hora de mecanizar, evi-
tando que las piezas puedan salir despedidas.
Se conoce el contracabezal con el nombre de contrapunto. El contrapunto dispone de un
cilindro con un agujero cilíndrico donde se puede adaptar portabrocas y taladrar en el centro de
la pieza.
Torreta portaherramientas, es el elemento en el cual se colocan y fijan las cuchillas apre-
tándolas con unos tornillos. Está situado sobre el carro orientable.
Plato de agarre, es el elemento con el cual sujetamos las piezas a mecanizar. El plato dis-
pone de unas uñas móviles llamadas garras las cuales se abren y cierran contra la pieza al girar
una tuerca que dispone en su parte exterior.

3.3. ÚTILES
Son elementos de ayuda a la hora de trabajar la pieza, podemos encontrarnos los siguientes:
– Útil de sujeción.
– Útil de verificación.

3.3.1. ÚTILES DE SUJECIÓN


Son los elementos que nos ayudan a que las piezas estén sujetas, para que no exista peligro
al trabajarlas.

21
-Pág.25-
TECNOLOGÍA

Según la forma de estas, los útiles para sujetar las piezas son:

Mordaza Brida Plato

3.3.2. ÚTILES DE VERIFICACIÓN


Son útiles que nos ayudan a comprobar si el elemento que se está fabricando cumple los
requisitos del plano (hoja de proceso).
A la hora de comprobar la pieza podemos verificar con los siguientes instrumentos:
De ángulos, son instrumentos fabricados con un ángulo fijo de gran pre-
cisión y en material resistente.
Los útiles de ángulos son escuadras y plantillas de ángulos.

De formas, conocidas como plantillas o galgas nos ayudan a verificar la pieza con la forma
de la plantilla.
Existen varios tipos:

Espesores Radios Roscas

3.4. PROCEDIMIENTOS DE FABRICACIÓN Y ACABADO


Entendemos por procedimientos de fabricación y acabado al conjunto de técnicas y proce-
sos (primarios y/o secundarios) que se emplean para transformar un elemento en un producto
acabado.

22
-Pág.26-
U . D . 3 . - P LANIFICACIÓN Y REALIZACIÓN

3.4.1. FABRICACIÓN
Los procedimientos de fabricación, son el conjunto de procesos que modifican o cambian la
forma física del elemento.
Forjado
Sin arranque de viruta
Estampación
“Conformado”
Laminación

Fabricación Limado
Manual Aserrado
Roscado
Con arranque de viruta
“Mecanizado”
Taladrado
En máquina Esmerilado
Torneado
3.4.1.1. Sin arranque de viruta
Son todas las operaciones que realizamos con un material para obtener una forma concreta,
aprovechando todo el material de partida.
Estas se realizan mediante técnicas de conformado. Las operaciones de conformado más
importantes son:
– Forjado.
– Estampación.
– Laminación.
Forjado, es la técnica usada por los herreros en la antigüedad. Es un pro-
ceso realizado en caliente, que consiste en someter a grandes esfuerzos al
material previamente calentado. El proceso de forjado sigue los siguientes
pasos: Calentamiento Deformación Enfriamiento

Estampación, es la técnica que se usa para la fabricación de monedas.


Mediante un golpe se deforma el material a la forma de la estampa con la que
se golpea.

Laminación, es la técnica empleada para la fabricación de chapas metálicas. Haciendo pasar


el material a través de unos rodillos que se van estrechando y a su vez se alarga a medida que
se disminuye en su grosor.

23
-Pág.27-
TECNOLOGÍA

3.4.1.2. Con arranque de viruta


Manuales
Son todas las operaciones que realizamos con un material para modificar su forma, del trozo
de material inicial que queda transformada en viruta o recortes.
Estas operaciones se pueden realizar de forma manual o mediante una máquina herramienta.
Manual, los procesos manuales de arranque de viruta son aquellos en que los esfuerzos de
corte son realizados mediante una fuerza muscular, los más conocidos son:
– Limado.
– Aserrado.
– Roscado.
Limado, es la operación con la cual rebajamos, pulimos
o retocamos piezas metálicas, arrancando pequeñas porcio-
nes de material en forma de virutas o limaduras.
La herramienta para realizar el lima-
do es la lima. Podemos trabajar tanto
superficies planas como curvas, interio-
res y exteriores, para conseguir las medi-
das correctas y la calidad superficial, paralelismo y escuadra entre las caras.
Las piezas a limar se sujetan en un tornillo de banco, que debemos tener
situado a la altura correcta. Para comprobar que el tornillo esta situado a la
altura correcta adoptar la posición que se muestra en la figura de la derecha.

Aserrado, es la operación de troceado mecánico con


desprendimiento de viruta. La herramienta para realizar esta
operación es la sierra. Por su consistencia débil, la sierra
necesita de un soporte para ser utilizada (arco de sierra).
Al montar la sierra en su arco debemos tener en cuenta las siguientes consideraciones:
– Los dientes de la sierra deben ir hacia delante.
– Tensar fuertemente la hoja por medio del tornillo y palomilla que tiene el arco de sierra.
Para aserrar a mano debemos tener las siguientes normas en cuenta:
– Al empezar un corte asegurarse que esta paralela a la línea de corte.
– No hacer fuerza a hacia abajo en el retroceso de la hoja de sierra.
– No girar bruscamente hacia los lados la hoja de sierra.
– Se debe usar todo el recorrido de la hoja de sierra.

24
-Pág.28-
U . D . 3 . - P LANIFICACIÓN Y REALIZACIÓN

Roscado, es la operación con la cual fabricamos un tornillo o/y tuerca, para ello utilizamos
una terraja o macho de roscar respectivamente.
Roscado con machos
Para construir un agujero roscado debemos seguir los
siguientes pasos.
1.o Taladrar el agujero a la medida correcta, para cal-
cularlo empleamos la siguiente regla:
Ø Taladro = Ø Nominal – Paso de la rosca
1.o Entendemos por: Ø = Diámetro.
Ø Nominal = Ø Exterior del tornillo [por ejemplo M-12 (Métrica 12) tiene un Ø Nomi-
nal de 12 mm.].
Paso = distancia entre los dientes (esta medida es estándar para todos los tornillos
según su Medida como se puede ver en la siguiente tabla).
Ø DE TALADRO
Ø Nominal Paso Ø Taladro
16 1,25 15,75
18 1,25 16,75
10 1,52 18,57
12 1,75 10,25

2.o Avellanar el taladro para facilitar la entrada del


macho. El avellanado consiste en realizar un peque-
ño taladro cónico a la entrada del taladro anterior.
Está operación se realiza con una herramienta lla-
mada avellanador.
3.o Para realizar una rosca se emplea el juego de machos progresivos compuesto por tres
machos como se ve en la figura.
N.o 1 Inicia la rosca

N.o 2 Debastar la rosca

N.o 3 Acaba la rosca

Para diferenciar los machos claramente vienen provistos de unas ranuras en la parte supe-
rior, el n.o 1 una ranura, el n.o 2 dos ranuras y el n.o 3 tres o ninguna ranura.
Pasos a seguir en la utilización de los machos:
A. Colocar el macho n.o 1 en el bandeador o gira mucho.
B. Posicionar la punta del macho en el agujero lo más perpendicular posible a la pieza.
C. Girar una vuelta en sentido de las agujas del reloj y después fi vuelta hacia el lado con-
trario para romper la viruta.
D. Continuar con este movimiento hasta llegar al fondo de la rosca.
Pasar los otros dos machos restantes siguiendo las mismas indicaciones.

25
-Pág.29-
TECNOLOGÍA

Roscado con terraja


Para roscar un tornillo es necesario que la varilla donde
vamos a realizar tenga el mismo Ø Exterior que la misma
medida de rosca que vamos a hacer. Por ejemplo para ros-
car un tornillo de M-6 la varilla tiene que ser de Ø 6 mm.
Los pasos para realizar la rosca son:
A. Colocar la varilla en un tornillo de banco.
B. Colocar la terraja en el porta terrajas apretándola fuertemente.
C. Posicionar la terraja lo más perpendicular a la varilla.
D. Girar en sentido horario 2 vueltas y presionar ligeramente.
Realizar los pasos C y D de los machos de roscar.
En cualquier roscado es conveniente lubricar (añadir un líquido que facilite el corte del
material como aceite, petróleo, etc.).
A máquina
Estos procesos de fabricación son los que se realizan mediante una máquina herramienta, los
esfuerzos musculares son sustituidos por fuerzas mecánicas. Los más conocidos son:
– Taladrado.
– Esmerilado.
– Torneado.
Taladrado, es la operación de hacer agujeros en piezas con desprendimiento de viruta, esta
operación la realizamos en una taladradora y la herramienta a utilizar es la broca.
Para realizar un taladro en una pieza, debemos seguir las
siguientes normas básicas:
1.o Trazar la pieza para sacar el centro del taladro (trazar
consiste en realizar 2 o más líneas que se cruzan en un
punto, el cual es el centro del taladro).
o
2. Granetear para marcar el centro del taladro y ayudar a
la broca a iniciar la operación (Granetear consiste en
golpear con un martillo sobre un granete donde se cru-
zan las líneas trazadas haciendo una pequeña hendidu-
ra sobre la pieza).
o
3. Sujetar fuertemente la pieza sobre con la mordaza, para
evitar accidentes a trabajar.
o
4. Escoger la medida de la broca, colocándola en el porta
brocas de la máquina de taladrar.
5.o Poner la velocidad de giro de la broca (a mayor diámetro de broca menor velocidad de
giro en la taladradora).
6.o Taladrar, mover la broca contra la pieza para realizar el taladro. Tendremos que tener
en cuenta si el agujero es pasante o ciego (no traspasa de lado a lado).
7.o Sacar la broca del agujero sin parar la máquina.
Para eliminar, en parte, el calor desarrollado en el mecanizado se utiliza una disolución lla-
mada taladrina.

26
-Pág.30-
U . D . 3 . - P LANIFICACIÓN Y REALIZACIÓN

Esmerilado, es la operación mecánica que tiene por misión


mecanizar superficies metálicas eliminando imperfecciones,
mediante la abrasión.
Esta operación la realizamos en un electroesmeril y la herra-
mienta es una muela abrasiva.
Está indicada para pequeños volúmenes de material a eliminar.
Para esmerilar debemos llevar una buena protección en los ojos,
para evitar que las chispas que se desprenden puedan accidentarnos.
La operación más importante que realizamos en el esmeril es
el afilado de cuchillas, brocas, etc.
Para la práctica del afilado debemos seguir unas normas básicas:
1.o Ajustar correctamente, tanto en altura como en proximidad, el soporte a la muela.
2.o Poner en marcha el esmeril.
3.o Afilar por la cara frontal de la muela, nunca por los laterales (podría partirse la muela).
4.o Sujetar fuertemente la pieza y desplazar a todo lo ancho de la muela.
5.o Enfriar la pieza sumergiéndola en agua de forma frecuente para evitar que se caliente
excesivamente y pierda alguna cualidad.
Torneado, es la operación mecánica con la cual modificamos la forma de la pieza en formas
de revolución (cilíndricas, cónicas).
Para realizar esta operación utilizamos como máquina el torno paralelo y como herramien-
ta cuchillas afiladas de diferentes formas, según el trabajo a realizar.
Las operaciones básicas que se pueden realizar en un torno son:
Refrentado, consiste en mecanizar las caras perpendiculares al eje de la pieza,
caras planas, para dejar la pieza a la longitud deseada. Se realiza con el carro
transversal.

Cilindrado, consiste en mecanizar la parte paralela al eje, creando formas


cilíndricas (de revolución) lo que hace que disminuya el diámetro de la pieza
hasta la medida deseada. Se realiza con el carro principal o longitudinal.

Torneado cónico, es un cilindrado especial por el cual obtenemos piezas en


forma cónica. Sé realizada con el carro orientable o charriot.

Taladrado, es una operación auxiliar por la cual realizamos un taladro en el


eje de rotación de la pieza. Se realiza colocando la broca en el contra cabe-
zal mediante un porta brocas o un cono y haciendo avanzar la broca contra
el giro de la pieza.

27
-Pág.31-
TECNOLOGÍA

Mandrinado, consiste en realizar un cilindrado en la parte interior de la


pieza, previamente hay que realizar un taladro.

Roscado, es realizar una rosca exterior o interior, mediante una cuchilla de


forma especial.

Cuando vamos a realizar cualquier operación de torneado tenemos que tener en cuenta las
siguientes normas:
1.o Elegir la herramienta adecuada.
2.o Seleccionar la velocidad de giro de la pieza correctamente (a mayor diámetro de la
pieza menor velocidad de giro de esta).
o
3. Colocar las herramientas en la torre portaherramientas a la altura correcta (el filo de la
herramienta tiene que estar a la altura del eje de giro de la pieza) y apretarla fuertemente.
o
4. Sujetar la pieza en el plato estando bien centrada y apretada.
5.o Utilizar sistemas de protección personal adecuados (gafas, gancho extractor de virutas,
etc.).

3.4.2. ACABADO
Entendemos por procedimientos de acabado los procesos que se pueden realizar a los obje-
tos después de un proceso de fabricación, para mejorar sus características o protegerlos de agen-
tes externos. Estos procesos de acabado son tratamientos que se realizan sobre las piezas y
pueden ser:
Pavonado
Químicos Baños
Pintado

Temple
Fabricación Térmicos Revenido
Recocido

Cementación
Termoquímicos
Nitruración

3.4.2.1. Tratamientos Químicos


Son tratamientos superficiales con los cuales protegemos al metal contra las agresiones
externas.
Los más utilizados son:
Pavonado, se realiza provocando un ataque químico de un ácido al metal, creándose una
capa exterior de color negro, que hace de barrera protectora.

28
-Pág.32-
U . D . 3 . - P LANIFICACIÓN Y REALIZACIÓN

Baños, se realizan recubriendo el metal con otro metal, normalmente noble, que no es ata-
cado por los agentes exteriores, los más conocidos son el dorado y plateado.
Pintado, consiste en extender una capa de pintura, con características antioxidantes, sobre el
exterior de la pieza.

3.4.2.2. Tratamientos Térmicos


Son aquellos en cuyo proceso no varían los componentes del elemento al que se aplican,
sino sus propiedades físicas.
Las fases de los tratamientos térmicos son calentamiento a temperatura determinada, mante-
nimiento de esta temperatura por un tiempo determinado y enfriamiento a velocidad adecuada.
Los tratamientos térmicos más conocidos son:
Temple, consiste en dar al material de la pieza mayor dureza y resistencia.
Revenido, es un proceso posterior al temple en el cual eliminamos las tensiones creadas en
el material al sufrir cambios determinados por su calentamiento.
Recocido, es el tratamiento opuesto al temple, con el conseguimos en materiales que se han
endurecido a procesos de fabricación, quitarles esa dureza, haciéndoles dúctiles.

3.4.2.3. Tratamientos Termoquímicos


Estos tratamientos añaden nuevos elementos a la composición del material o cambian la pro-
piedad de los existentes.
La finalidad de todos ellos es la de obtener una capa exterior muy dura y resistente, mien-
tras que el interior de la pieza queda con menor dureza.
Los tratamientos termoquímicos más utilizados son:
Cementación, se añade carbono.
Nitruración, se añade nitrógeno.

SEGURIDAD EN EL TRABAJO.
4. NORMAS BÁSICAS DE SEGURIDAD EN EL TALLER
Riesgos
Seguridad
Seguridad en el trabajo
Normas
Señalización

29
-Pág.33-
TECNOLOGÍA

Todo trabajo está sometido a unos riesgos que pueden provocar daños a los trabajadores y a
su entorno familiar. Los daños se pueden clasificar:
Enfermedad profesional, es la contraída a consecuencia del trabajo realizado y que está pro-
vocada por la acción de los elementos o sustancias que están en el lugar de trabajo, por ejem-
plo la exposición permanente a la radiación en un central nuclear.
Accidente laboral, es toda lesión corporal que el trabajador sufre con ocasión o a conse-
cuencia del trabajo que se ejecuta por cuenta ajena, (Art. 114 de la Ley Gral. de la S.S.) por
ejemplo la caída de un andamio.
Las causas que provocan accidentes laborales pueden ser debidas al trabajador o ajenas a él.
Causas debidas al trabajador:
– Imprudencia en el uso de las herramientas y máquinas.
– Desconocimientos de los peligros.
– Incumplimientos de las normas de seguridad.
– No encontrarse en buenas condiciones físicas o psíquicas.
Causas ajenas al trabajador:
– Deficientes condiciones del lugar de trabajo.
– Máquinas que no cumplen las normas de seguridad y protección.
– Instalaciones sin medios de seguridad.
Para controlar los riesgos y actuar sobre las causas que provocan los accidentes y enferme-
dades profesionales se ha creado una asignatura y departamentos en empresas dedicados a la
seguridad.

La seguridad en el trabajo es el conjunto de técnicas que tienen por objeto


eliminar o reducir los riesgos, de los distintos puestos de trabajo,
para evitar los accidentes laborales.
En cualquier puesto de trabajo, que pueda entrañar riesgos, deben existir las normas de segu-
ridad puestas en un documento y que indique la forma de actuar en ese puesto de trabajo.

Las normas de seguridad son instrucciones y órdenes concretas


que indican al operario como debe actuar en un puesto de trabajo determinado,
para prevenir y evitar accidentes.

En el mundo laboral, se dan situaciones de peligro, en las cuales los trabajadores deben de
recibir una información visual directa y clara.

Las señales de seguridad son una combinación de colores, formas geométricas y


símbolos que logran comunicar de un riesgo de forma simple y rápida.
Es una simbología universal.

30
-Pág.34-
U . D . 3 . - P LANIFICACIÓN Y REALIZACIÓN

Para que las señales de seguridad cumplan con su objetivo deben tener los siguientes requisitos:
• Atracción. • Anticipación. • Claridad.

Señales de Significado Color de seguridad Símbolo


Prohíbe un comportamiento de
Prohibición Rojo
provocar peligro
Obliga a mantener un comportamiento
Obligación Azul
determinado

Advertencia Advierte de un peligro potencial Amarillo

Proporciona una información de


Información Verde
seguridad

Señales de seguridad

Normas básicas de seguridad en el Taller


– Tener siempre el puesto de trabajo en orden y limpio de virutas o de derrames de líquidos.
– Usar un equipo de ropa adecuada. Es conveniente que la ropa esté lo más ceñida posible,
mangas ajustadas y no llevar ningún complemento que sobresalga (corbatas, bufandas, etc.).
– Tener una buena iluminación.
– Cubrir con defensas los elementos que puedan rodar.
– Tener puestas todas las tomas de tierra de los aparatos y máquinas eléctricas.
– Cumplir siempre con las normas de seguridad por muy incómodas que puedan parecer.
– No olvidar las protecciones personales para cada trabajo que realices (gafas, cascos,
máscaras).

Las prisas son siempre causa de accidentes, debido a que no se piensa lo que se hace.

31
-Pág.35-
TECNOLOGÍA

5. TOLERANCIAS. CONTROL DE CALIDAD


5.1. TOLERANCIA
Ante la imposibilidad de obtener una pieza absolutamente exacta, se admite en ella una dife-
rencia mayor o menor de medidas, según la importancia de la pieza a realizar.
Tolerancia, es el margen de error admisible en la fabricación
respecto de la medida nominal y su forma geométrica.
– Medida nominal: Es la medida que nos pide el plano.
– Medida máxima: La mayor medida admisible en la fabricación.
– Medida mínima: La menor medida admisible en la fabricación.

5.2. CONTROL DE CALIDAD


La calidad es el conjunto de características que hacen que un producto cumpla para lo que
fue diseñado, con sus condiciones de uso, duración y seguridad.
Calidad es satisfacer las necesidades del cliente.
Toda empresa en su deseo de ser competitiva, debe de introducir un Departamento de Cali-
dad. La función principal de este departamento es la de velar por la calidad del producto.
Las funciones generales de un Departamento de Calidad son las siguientes:
– Aceptar o rechazar el producto para su distribución.
– Detectar donde se producen defectos y proponer su mejora.
– Asegurar el nivel de Calidad.
Actualmente, ante la implantación de Certificaciones de Calidad en las empresas, la responsabi-
lidad del producto de calidad ya no recae únicamente en el Departamento de Calidad si no en todos
los miembros de la empresa, asumiendo de forma individualizada un autocontrol de su trabajo.
Certificar en calidad consiste en que un organismo competitivo que no conoce como traba-
jas, compruebe y verifique que tenemos puestos en marcha los medios necesarios para obtener
un producto de calidad. Este organismo (en el caso de España la Asociación Española de Nor-
malización AENOR) mediante una auditoria de calidad se encarga de certificar que la empresa
cumple con la norma ISO-9000.
CALIDAD ES COSA DE TODOS

32
-Pág.36-
U . D . 3 . - P LANIFICACIÓN Y REALIZACIÓN

RESUMEN

Cuando emprendemos el proyecto de realizar un objeto o producto, hay que llevar un pro-
ceso para que el producto, al pasar por las distintas secciones de la empresa, tenga asignado el
trabajo a realizar.
Existen unos documentos que facilitan la cumplimentación de estos pasos. Dentro de los
pasos a realizar un producto se rellena la hoja de ruta, y para indicar la fabricación de una pieza
se rellena la hoja de proceso.
También se puede representar gráficamente la ruta que va seguir un producto por la empre-
sa, para que así éste recorra el menor espacio y esté parado el menor tiempo posible.
A la hora de fabricar un producto existen varios procedimientos y con cada uno se emplean
diversas máquinas, en las que se trabaja con diferentes herramientas, que están provistas de
unos útiles para poder sujetar a la herramienta o pieza a trabajar. Cuando estamos realizando el
trabajo en las máquinas existen unos instrumentos de medida así como unas normas para llevar
un control de calidad.
Debemos tener muy en cuenta que a la hora de realizar estas operaciones se debe seguir un
orden para mayor seguridad de la máquina y operario. También existen unas normas generales
que ayudan a tener una mayor seguridad en el puesto de trabajo.

33
-Pág.37-
TECNOLOGÍA

EJERCICIOS DE AUTOCOMPROBACIÓN

1. Explica qué es un juego de machos y cómo se identifica.

2. Describe las principales causas de los accidentes laborales.

3. Realiza un diagrama de operaciones que sirva para construir la silla donde estás sentado.

4. Indica cuales de los siguientes tratamientos superficiales son tratamientos térmicos:

Dorado Templado Cementado

Pavonado Pintado Revenido

5. Indica cuales de las siguientes operaciones se pueden realizar en una torno paralelo:

Taladrado Cilindrado Limado Graneteado

Planeado Roscado Mandrinado Escariado

6. Indica el nombre y las partes principales de las siguientes máquinas-herramientas:

Máquina: Partes:

1. .........................
4
5 2. .........................
1
3 3. .........................

4. .........................

5. .........................

34
-Pág.38-
U . D . 3 . - P LANIFICACIÓN Y REALIZACIÓN

Máquina: Partes:

1. .....................................

2. .....................................
4
3. .....................................
3 4. .....................................

7. Une mediante una línea cada procedimiento de fabricación con el tipo al que pertenece.
Limado

Esmerilado

Laminación
Con arranque de viruta
Taladrado

Roscado

Forjado
Sin arranque de viruta
Torneado

Aserrado

Estampado

8. Indica el significado de los siguientes símbolos:

............. ............. ............. .............

35
-Pág.39-
TECNOLOGÍA

RESPUESTAS A LOS EJERCICIOS


1. Explica qué es un juego de machos y cómo se identifica.
Es un juego de 3 piezas (tornillos) con una ranuras que al girar realizan una rosca en un agujero.
Se identifican por unas ranuras o marcas que tienen en la parte del mango.

2. Describe las principales causas de los accidentes laborales.


Causas debidas al trabajador:
– Imprudencia en el uso de las herramientas y máquinas.
– Desconocimientos de los peligros.
– Incumplimientos de las normas de seguridad.
– No encontrarse en buenas condiciones físicas o psíquicas.
Causas ajenas al trabajador:
– Deficientes condiciones del lugar de trabajo.
– Máquinas que no cumplen las normas de seguridad y protección.
– Instalaciones sin medios de seguridad.

3. Realiza un diagrama de operaciones que sirva para construir la silla donde estás sentado.
Listón madera (patas y travesaños)
Cortar piezas (patas y travesaños)

Agujerear para ensamblaje

Montaje y Barnizado

Secado

Verificar montaje

Asiento
Tapizado

Verificar el tapizado

Almacén

36
-Pág.40-
U . D . 3 . - P LANIFICACIÓN Y REALIZACIÓN

4. Indica cuales de los siguientes tratamientos superficiales son tratamientos térmicos:


Dorado 嘺 Templado Cementado
Pavonado Pintado 嘺 Revenido

5. Indica cuales de las siguientes operaciones se pueden realizar en una torno paralelo:
嘺 Taladrado 嘺 Cilindrado Limado Graneteado
Planeado 嘺 Roscado 嘺 Mandrinado Escariado

6. Indica el nombre y las partes principales de las siguientes máquinas-herramientas:


Máquina: Torno Paralelo Partes:

4 1. Cabezal
5
1 2. Base
3
3. Carros

4. Torre porta-herramientas

5. Contracabezal

Máquina: Taladradora de Columna Partes:

1. Soporte
4
2. Mesa
3 3. Dispositivo fijación broca

4. Cabezal

37
-Pág.41-
TECNOLOGÍA

7. Une mediante una línea cada procedimiento de fabricación con el tipo al que pertenece.

Limado

Esmerilado

Laminación
Con arranque de viruta
Taladrado

Roscado

Forjado
Sin arranque de viruta
Torneado

Aserrado

Estampado

8. Indica el significado de los siguientes símbolos:

Botiquín Prohibido Obligatorio Peligro


fumar utilizar gafas corriente eléctrica

38
-Pág.42-
NOTAS

-Pág.43-
NOTAS

-Pág.44-
U . D . 4 . - O RGANIZACIÓN Y GESTIÓN

ÍNDICE
OBJETIVOS . . . . . . . . . . . . . . . . . . . . . . . . . . . . . . . . . . . . . . . . . . . . . . . . . . . . . . . . . . . . . 2

INTRODUCCIÓN . . . . . . . . . . . . . . . . . . . . . . . . . . . . . . . . . . . . . . . . . . . . . . . . . . . . . . . . . 3

MAPA CONCEPTUAL . . . . . . . . . . . . . . . . . . . . . . . . . . . . . . . . . . . . . . . . . . . . . . . . . . . . . 4

DESARROLLO DE CONTENIDOS . . . . . . . . . . . . . . . . . . . . . . . . . . . . . . . . . . . . . . . . . . 5

1. CONCEPTOS, PRINCIPIOS DE ORGANIZACIÓN Y GESTIÓN


DE UN PROYECTO TÉCNICO . . . . . . . . . . . . . . . . . . . . . . . . . . . . . . . . . . . . . . . 5
1.1. EL PROYECTO TÉCNICO EN EL AULA TALLER . . . . . . . . . . . . . . . . . . . . . . . . 5
1.2. FASE DE CONSTRUCCIÓN . . . . . . . . . . . . . . . . . . . . . . . . . . . . . . . . . . . . . . . . . . . 8

2. CONCEPTOS Y PRINCIPIOS DE ORGANIZACIÓN Y GESTIÓN


DE LA INFORMACIÓN . . . . . . . . . . . . . . . . . . . . . . . . . . . . . . . . . . . . . . . . . . . . . 9
2.1. OFIMÁTICA . . . . . . . . . . . . . . . . . . . . . . . . . . . . . . . . . . . . . . . . . . . . . . . . . . . . . . . . 9

3. DOCUMENTOS EMPLEADOS EN LA ORGANIZACIÓN Y GESTIÓN


DE PROYECTOS TÉCNICOS . . . . . . . . . . . . . . . . . . . . . . . . . . . . . . . . . . . . . . . 11
3.1. PRESUPUESTO . . . . . . . . . . . . . . . . . . . . . . . . . . . . . . . . . . . . . . . . . . . . . . . . . . . . 11
3.2. CORRESPONDENCIA COMERCIAL. . . . . . . . . . . . . . . . . . . . . . . . . . . . . . . . . . . 12
3.3. DOCUMENTOS MERCANTILES . . . . . . . . . . . . . . . . . . . . . . . . . . . . . . . . . . . . . . 15
3.4. DOCUMENTOS DE COBROS Y PAGOS . . . . . . . . . . . . . . . . . . . . . . . . . . . . . . . . 17

RESUMEN . . . . . . . . . . . . . . . . . . . . . . . . . . . . . . . . . . . . . . . . . . . . . . . . . . . . . . . . . . . . 21

EJERCICIOS DE AUTOCOMPROBACIÓN. . . . . . . . . . . . . . . . . . . . . . . . . . . . . . . . . . 22

RESPUESTAS A LOS EJERCICIOS. . . . . . . . . . . . . . . . . . . . . . . . . . . . . . . . . . . . . . . . . 25

-Pág.45-
TECNOLOGÍA

OBJETIVOS
Al finalizar el estudio de esta Unidad Didáctica, el alumno será capaz de:

• Redactar una carta comercial y una instancia.

• Confeccionar una hoja de pedido, albarán y una factura.

• Distinguir los distintos documentos de pago.

• Realizar proyectos sencillos.

• Diferenciar entre maqueta y/o prototipo.

• Conocer los medios de organización y gestión de la información.

-Pág.46-
U . D . 4 . - O RGANIZACIÓN Y GESTIÓN

INTRODUCCIÓN
E n la actualidad se necesita conocer de forma pormenorizada todos los detalles de
elaboración antes de iniciar el trabajo, de modo que garantice el éxito de la empresa
acometida.

El alto coste que implica desarrollar un trabajo hace necesario seguir unos pasos y que
éstos tengan unos recursos, que nos permitan ponerlo en marcha.

Las relaciones comerciales de las empresas se rigen por una serie de reglas escritas o no
escritas. Sobre las reglas escritas debemos decir, que son enteramente obligatorias, en
cuanto a las segundas, son un poco de formalidad, cortesía y ética en los negocios.

Los documentos mercantiles facilitan la realización de estas normas y permiten llevar de


una manera fácil a las empresas sus actividades.

-Pág.47-
TECNOLOGÍA

M A PA C O N C E P T UA L

HOJA DE CÁLCULO

PROCESADOR DE TEXTOS

PROYECTO TÉCNICO
BASE DE DATOS

ORGANIZACIÓN

INFORMACIÓN

GESTIÓN

PRESUPUESTO

PEDIDO

PAGO

-Pág.48-
U . D . 4 . - O RGANIZACIÓN Y GESTIÓN

CONCEPTOS, PRINCIPIOS DE ORGANIZACIÓN


1. Y GESTIÓN DE UN PROYECTO TÉCNICO
La realización de una acampada requiere una preparación previa para determinar el
material que será necesario llevar, el itinerario a seguir, el presupuesto para llevarla a cabo y
una serie de cosas más. Esta preparación solemos plasmarla de una forma sencilla en notas
manuscritas; es lo que denominamos plan o proyecto.
De igual forma, para la realización de los diferentes tipos de productos se necesita una
documentación específica que facilite su elaboración; es lo que se le llamamos proyecto técnico.
Existen múltiples razones que justifican la necesidad de elaboración de los proyectos
técnicos. Entre ellas podemos destacar:
1.a Detalla las ideas relacionadas con el producto antes de su construcción.
2.a Transmite información a otras personas sobre lo que se piensa hacer para realizarlo y
plasmarlo sobre un documento.
3.a Permite prever los recursos necesarios para su realización y desarrollarlo en el menor
tiempo posible.
4.a Posibilita el control de realización y contrastarlo con lo pensado de antemano.
Esto lo podemos ver concretamente tomando como ejemplo el Monasterio del Escorial, que
fundado por Felipe II en 1563 se terminó en 1584, durante los veintiún años que duró su
construcción se fue materializando un proyecto donde las técnicas utilizadas, los materiales
empleados o la organización de la obra, tuvieron que irse adaptando y contrastando a lo largo
de todo el proceso de construcción.

1.1. EL PROYECTO TÉCNICO EN EL AULA-TALLER


En la ejecución de proyectos técnicos en el aula-taller también se parte de un problema.
El estudio inicial del problema, su posible solución y las investigaciones previas se deben
realizar a nivel individual, se elaboran propuestas personalizadas que han de ser explicadas al
grupo de trabajo para que, en reunión se decida sobre cuál o cuáles de las presentadas resuelve
mejor el problema y es más apropiada para diseñarla, construirla y apoyarse documentalmente
en una carpeta proyecto.
Una vez consensuada la propuesta, se ha de proceder a repartir, entre los miembros del
equipo, las tareas que conducen a confeccionar:
– Los primeros bocetos, planos que aportan las ideas sobre el problema.
– Las listas provisionales de los materiales a utilizar.

5
-Pág.49-
TECNOLOGÍA

– Las fuentes de recursos escritos, como la de los materiales.


– El proceso de elaboración del objeto.
– Las hojas de procesos de las partes que componen el objeto.
Podemos ver el esquema de trabajo que se puede seguir en la realización de un proyecto
técnico en el aula-taller:
Problema / Necesidad

Análisis del problema en equipo

Investigaciones previas
Diseños iniciales
Propuesta de solución

Discusión de propuesta en equipo

Propuesta

Reparto de tareas

Soluciones dadas Biblioteca Asesoramiento técnico

Elaboración en equipo

Croquis / Bocetos Materiales Diagramas de trabajo Procesos de trabajo

MAQUETA / PROTOTIPO

Los primeros bocetos y croquis aportan las ideas


generales sobre la solución que se ha tomado en la
reunión en equipo.

6
-Pág.50-
U . D . 4 . - O RGANIZACIÓN Y GESTIÓN

En las listas de materiales es conveniente hacer referencia a la pieza en el dibujo de todo el


conjunto, tipo de material y proceso de trabajo donde se indican las operaciones de construcción
que se han de realizar en esa pieza en concreto.

Pieza n.o Denominación Material Cantidad Observaciones


1 Tapa Plástico 1 Hoja trabajo n.o 1
2 Soporte Nylon 1 Hoja trabajo n.o 2
— — — — —
— — — — —
— — — — —
8 Núcleo Baquelita 1 Hoja trabajo n.o 8

El proyecto técnico en el aula-taller es la recopilación de los conocimientos adquiridos


en las diferentes áreas de conocimiento con el objeto de resolver necesidades.
El objeto de resolver problemas nos hacen plantearnos un proyecto técnico, y su culmina-
ción es el realizar una maqueta o prototipo.

Maqueta
Tienen la misión de proporcionar información en tres dimensiones sobre la apariencia que
tendrá el producto de antemano proyectado. Normalmente se realizan a escala y no simulan
ningún funcionamiento.
Las maquetas se pueden realizar de diferentes modos:
– Sencilla. Solamente se ve el exterior, a tamaño normal o escala del objeto.
– De detalle. En ella se perfeccionan detalles de partes específicas del
producto.
– Completa. Sirve para la venta del producto, así como para la presentación
y simulación ya que contienen todos los detalles del mismo, son las más
empleadas, como por ejemplo, la maqueta de una vivienda.
Los materiales más utilizados en la realización de las maquetas suelen ser:
– Yeso.
– Arcilla.
– Cartón.
– Madera.
– Plástico.

7
-Pág.51-
TECNOLOGÍA

Prototipos
Son productos fabricados antes de la producción en serie, destinados a realizar pruebas y
comprobar su fiabilidad y que sus características sean las proyectadas. Los prototipos tienen la
misión de:
– Detectar deficiencias en el diseño.
– Deficiencias en los cálculos realizados.
– Deficiencias en los materiales de cara a la fabricación.
Normalmente los prototipos se someten a ritmos de trabajo superiores a los que van a
trabajar para ver el comportamiento del producto y ver si cumple las necesidades del comprador.
De esta forma podemos comprobar la duración de los componentes y más o menos el tiempo
medio de averías para predeterminar los costes de cara a su mantenimiento.

1.2. FASE DE CONSTRUCCIÓN


Una vez conocido el “que”, “cómo”, “quién” y “con que” se va a fabricar, se inicia la fase
de realización práctica de lo proyectado. En ella han de surgir correcciones y rediseños de la
idea original:
Documentación

Reparto de tareas Reparto de tareas


Rediseño

Fabricación

Si Necesita No
Objeto terminado
rectificación

Los planos. Son el conjunto de documentos de expresión gráfica para conocer las formas
del objeto. Al plano que representa el objeto en perspectiva deben adjuntarse los del conjunto y
del despiece. Igualmente, si fuese necesario los del montaje y fabricación, utilizando la
normalización técnica necesaria.
La evaluación. Es importante hacer una inspección como final del proyecto que implique
una reflexión sobre la funcionalidad de lo proyectado y fabricado, para tomar conciencia de si
cubre las necesidades de lo proyectado. Se debe tener en cuenta:
– La incidencia social que puede producir la aparición de soluciones a problemas
concretos, y que puede concretarse en la eliminación de unos puestos de trabajo y en la
creación de otros nuevos específicos que requieren un grado de cualificación.

8
-Pág.52-
U . D . 4 . - O RGANIZACIÓN Y GESTIÓN

– El impacto ambiental derivado de la fabricación y posteriormente de su uso (residuos,


modificación del paisaje, agotamiento de las materias primas).
– El análisis de los procesos tecnológicos implicados, que puede aportar información
sobre las posibilidades de mejora en las fases de la realización del proyecto técnico en
el aula-taller.
La carpeta del proyecto. Representa la recopilación de toda la información escrita que
acompaña al objeto fabricado, por lo que su buena estructuración y presentación es
importantísima. Se aconseja hacer una inclusión de índices por secciones, bibliografía, memoria
descriptiva, cálculos, etc. También pueden incluirse otras secciones relacionadas con la seguri-
dad e higiene en el trabajo en el aula-taller.

El proyecto técnico elaborado en el aula-taller estará correctamente realizado


cuando, a través de la documentación generada en su carpeta correspondiente,
cualquier equipo de trabajo ajeno puede realizar
la maqueta o prototipo correspondiente sin verlo previamente

2. CONCEPTOS Y PRINCIPIOS DE ORGANIZACIÓN


Y GESTIÓN DE LA INFORMACIÓN
Por información se entiende todo acontecimiento, hecho o criterio puesto en conocimiento
de un público más o menos numeroso en forma de imágenes, textos, discursos o sonidos.
En los años ochenta se han consolidado nuevas tecnologías para la comunicación (cable,
satélite, teletexto, vídeo doméstico, videodiscos, ordenadores) que ofrecen nuevas posibilidades
de difusión de la información.
Dentro del organigrama operativo de una industria, la llamada “oficina técnica” o “departamento
técnico” constituye el equipo encargado de preparar la documentación y gestionar la información de
todos los aspectos directamente relacionados con la producción y el mantenimiento de las
instalaciones. En la mayoría de los casos es el departamento encargado de diseñar componentes o la
totalidad del producto fabricado, y también de los utillajes y parte de la maquinaria empleada. En la
oficina técnica se diseñan y calculan, generalmente, parte de las modificaciones que el tiempo
aconseja o fuerza a introducir en los procesos e instalaciones de producción. Para ello elabora los
planos necesarios y realiza los proyectos que contemplen las tareas a realizar, bien sea por personal
propio de la empresa o bien con la ayuda de colaboradores externos.

2.1. OFIMÁTICA
También llamada “burótica”, utiliza todas las técnicas informáticas y telemáticas que
facilitan y posibilitan los trabajos de oficina y administración, tanto en la empresa pública como

9
-Pág.53-
TECNOLOGÍA

privada. Los equipos informáticos existentes permiten un tratamiento altamente automatizado


de la información, especialmente la información escrita y el tratamiento sistematizado de
documentos.
Inicialmente, la ofimática permitió una cierta automatización de las tareas individuales en
los puestos de trabajo de las oficinas, dedicadas fundamentalmente a la elaboración de
documentos escritos de distinta índole, gracias a la introducción de las máquinas de escribir
electrónicas, el papel autocopiante y las fotocopiadoras electrostáticas.
Más recientemente, la informatización integral de las oficinas ha propiciado una progresiva
sustitución del papel por el soporte magnético y la transmisión física de datos por correo
electrónico, con la ayuda de aparatos como el modem y el fax. Paralelamente, se tiende a un
más alto grado de interconexión entre los terminales de los distintos puestos de trabajo,
integrándolos en redes que permiten una utilización conjunta y simultánea de recursos, como
las bases de datos y las aplicaciones concretas que emplee la empresa.
En los trabajos de oficina se usa un conjunto de aplicaciones complementarias para un
ordenador personal, cada una de las cuales cubre un área clave en el desarrollo del trabajo. Estas
aplicaciones reciben el nombre de paquetes integrados y se comercializan conjuntamente.

El paquete integrado más utilizado en la oficina actual está compuesto por:


– Procesador de texto.
– Base de datos.
– Hoja de cálculo.

2.1.1. PROCESADOR DE TEXTO

Programa que permite crear archivos de texto (cartas, documentos, informes) y realizar todo
tipo de tratamientos sobre éstos. Funciona como una máquina de escribir, haciendo la pantalla
del ordenador las veces de papel. Con él se consigue mejorar la calidad de la presentación de
un documento por medio de una serie de utilidades incluidas. Se pueden emplear distintos tipos
de letras (fonts), paginar, justificar márgenes, reestructurar textos, etcétera.
Los procesadores de texto poseen casi la misma funcionalidad que los programas de
autoedición. Entre sus prestaciones y características más relevantes se encuentran: visualización
del texto en pantalla tal como se imprimirá (esto se denomina W.Y.S.I.W.Y.G., siglas de la
expresión inglesa What you see is what you get, lo que uno ve es lo que obtiene), corrección
automática de errores tipográficos, incorporación de imágenes, cuadros de texto y tablas,
creación automática de ínidices y referencias cruzadas, elaboración de cartas personalizadas al
combinar el documento con listas de correspondencia, extracción e integración de información
procedente de bases de datos u otros programas, exportación del documento a distintos formatos
para que puedan utilizarlo en otras aplicaciones y conversión al formato H.T.M.L. para

10
-Pág.54-
U . D . 4 . - O RGANIZACIÓN Y GESTIÓN

publicarlo como página WEB. Además, los procesadores de texto incluyen diccionario de
sinónimos y módulos correctores de errores ortográficos, gramaticales y estilísticos.

2.1.2. HOJA DE CÁLCULO


Programa que permite introducir y ordenar datos por filas y columnas, para realizar todo tipo
de operaciones numéricas con ellos.
Las hojas de cálculo adoptan la forma de un estadillo en el que cada celda de la hoja puede
definirse alfabéticamente, adoptar un valor absoluto, realizar un cálculo o determinar un valor
numérico o lógico, en función de los valores que en aquel momento tengan otras casillas de la
misma hoja.
Se emplean para realizar presupuestos, hacer balances y tareas de contabilidad y, en general,
para cualquier tarea que necesite manejar grandes tablas de números de este tipo.

2.1.3. BASE DE DATOS


Programa que permite guardar y ordenar información como si se tratara de las fichas de un
archivador, y manejarlas realizando todo tipo de búsquedas, selecciones y ordenaciones.
Posteriormente, se pueden hacer consultas a ese archivador y obtener información según los
criterios deseados, ya que la información se tiene almacenada de manera que los usuarios
acceden a ella de forma sencilla, gracias a la forma en que está estructurada. Una base de datos
informática es una base de datos convencional en la que la información se guarda utilizando
medios informáticos tales como discos duros, CD-ROM, cintas, etc. El acceso a la información
contenida en este tipo de bases de datos se realiza a través de un ordenador.

DOCUMENTOS EMPLEADOS EN LA ORGANIZACIÓN


3. Y GESTIÓN DE UN PROYECTO TÉCNICO

3.1. PRESUPUESTO
Para elaborar un determinado producto es preciso saber de forma anticipada los medios
materiales y humanos que van a ser necesarios. Las razones que justifican esta necesidad son,
entre otras, las siguientes:
– Hay que conocer los recursos financieros que la empresa necesita utilizar.
– Es preciso negociar con los proveedores para que los materiales necesarios estén listos
en las cantidades y fechas previstas.
– Hay que conocer con anticipación los diferentes costes en los que se va a incurrir para
determinar el coste de obtención del producto.

11
-Pág.55-
TECNOLOGÍA

Para solucionar estas cuestiones se elaboran los presupuestos.


El presupuesto es una previsión monetaria sobre los gastos e ingresos que va a tener una
empresa en un periodo corto de tiempo, normalmente un año, como consecuencia de las
actividades que desarrolla.
El presupuesto detalla el número de recursos necesarios y su coste, así como el número de
productos a obtener y su precio. Describe de forma detallada:
• Las materias primas a emplear, con detalle de cantidades y precio.
• Los materiales que, no formando parte del producto, son necesarios para su fabricación
(por ejemplo, los equipos de protección individual, como unas pantallas protectoras).
• La mano de obra para fabricar el producto, con detalle de especialidad, número de horas
y cualificación.
• Las máquinas y herramientas que se van a utilizar, haciendo referencia a tipo, consumo
y criterio para el cálculo del coste.
• Parte de los gastos generales: consumo eléctrico, personal auxiliar, personal directivo,
etcétera. A falta de su detalle, se puede asignar un porcentaje o una cantidad fija
aproximada, para su amortización.
Antes de fabricar un producto es necesario calcular el coste de todos sus componentes,
veamos a continuación un ejemplo:
Fundación para el Deporte Oficina de Estudios Producto: Portabrocas ISO-30
Presupuesto Industrial Fecha 23/3/00 Cantidad: 1.000 unidades
Concepto ./unidad Cantidad Consumo Coste unitario
Acero F-114 1,25 €/Kg. 1’5 Kg./producto 1.500 Kg. 1,88 €
Broca ∅ 6 3,50 €/Broca 200 productos/broca 5 Brocas 0,018 €
Broca ∅ 14 7,50 €/Broca 200 productos/broca 5 Brocas 0,038 €
Herramienta de torno 7,25 €/cuchilla 200 productos/cuchilla 5 Cuchillas 0,036 €
Herramienta de fresa 6,25 €/Fresa 200 productos/fresa 5 Fresas 0,031 €
Operario tornero 30 €/hora 0,4 horas/producto 500 horas 12 €
Operario fresador 30 €/hora 0,5 horas/producto 600 horas 15 €
Uso máquina torno 20 €/hora 0,4 horas/producto 500 horas 8€
Uso máquina fresa 30 €/hora 0,5 horas/producto 600 horas 15 €
Coste unitario 52,003 €
Asignación gastos generales al producto (40% sobre el coste) 20,801 €
COSTE UNITARIO TOTAL 72,804 €

3.2. CORRESPONDENCIA COMERCIAL


Las personas muchas veces necesitan comunicarse con una empresa mediante un
documento oficial, también éstas por diversas necesidades tienen que formalizar una serie

12
-Pág.56-
U . D . 4 . - O RGANIZACIÓN Y GESTIÓN

de tramites con otra, para todo esto se utilizan una serie de documentos oficiales que iremos
detallando a continuación.

3.2.1. LA CARTA COMERCIAL


Es un medio de comunicación de las empresas en sus actividades con otras personas y
entidades: por ejemplo, para solicitar las condiciones de compra de unas mercancías, efectuar
reclamaciones, etcétera. Para que la transmisión de la información a través de la carta comercial
sea lo más útil posible, el contenido de la misma debe cumplir una serie de requisitos:
– La presentación de la carta debe ser apropiada. Se deben cuidar aspectos como el tamaño
y tipo de papel, el tipo de letra, etcétera.
– El lenguaje utilizado debe ser sencillo, claro y conciso.
– La información que contenga la carta comercial debe ser exacta de forma que no sea
necesario ningún tipo de aclaración posterior.
Las partes que componen un carta comercial son las siguientes:
– El encabezamiento. Incluye los datos de la persona o entidad que escribe la carta (puede
incluir el logotipo), es decir: nombre, dirección, teléfono de quien remite la carta, los
datos del destinatario, la fecha y el lugar desde donde se escribe la carta. Además de
estos datos se pueden incluir dos más: el asunto, que en una frase corta hace alusión al
contenido de la carta, y la referencia, cuando la carta sea de contestación a otra.
– El cuerpo. Es la parte de la carta donde se facilita la información que se quiere transmitir.
– La despedida y firma. En toda relación comercial deben estar presente el respeto y la
cortesía. Esta parte de la carta tiene por objeto reafirmar este aspecto.

La carta comercial es un documento que se compone de encabezamiento, cuerpo,


despedida y cierre, y ha de ser escrita con lenguaje claro y conciso.

13
-Pág.57-
TECNOLOGÍA

3.2.2. LA INSTANCIA
Es un modelo de solicitud de algún tipo de EMILIO GONZÁLEZ PÉREZ, mayor de edad, casado,
demanda, dirigida a personas o departamentos de de profesión camarero, natural de Arcones, con
domicilio en c/ verja,26 de Madrid 28028-Madrid, con
jerarquía superior, o a las administraciones públicas. DNI 1.234.567.

EXPONE:
Se utiliza para diversos propósitos, como la solicitud Que deseando abrir un local comercial dedicado a la
venta de revistas, libros y utensilios de papelería en la
de un puesto de trabajo, petición de certificados, petición c/ Fornillos,45 de esta localidad y estando en posesión de
de permisos para la realización de obras, petición de la Licencia Fiscal y demás documentación requerida por
la legislación vigente, es por lo que,
apertura de nuevas industrias, etcétera.
SOLICITA:
Le sea admitida esta instancia y concedida la licencia de
En algunos organismos oficiales, y dado lo recurrente apertura necesaria para poder emprender a la mayor
brevedad posible las actividades comerciales en el citado
de las peticiones, existen modelos previamente impresos local.
que se rellenan convenientemente. Las instancias constan Madrid, dos de mayo del dos mil.
de los siguientes apartados: EMILIO GONZÁLEZ PÉREZ

– El encabezamiento. Incluye todos los datos del


solicitante, como nombre y apellidos, SR. ALCALDE DEL AYUNTAMIENTO DE MADRID

profesión, estado civil, DNI y demás


circunstancias personales que tengan relación con el objeto de la instancia.
– El cuerpo de la instancia. En él se exponen las razones que fundamentan la petición y
donde se solicita que sea atendida la misma.
– El pie. Incluye dónde y cuándo se solicita la instancia, la firma del solicitante y a quién
va dirigida.

3.2.3. EL INFORME
Para adoptar decisiones, la dirección de una empresa necesita conocer datos como, por
ejemplo, la evolución del producto fabricado en
mercado, innovaciones técnicas que se pueden
aplicar al proceso productivo, etcétera. Los
informes pueden ser, por tanto, técnicos, de
mercado, personales, comerciales, etcétera.
Los informes pueden ser confeccionados por
un especialista de la compañía o por una empresa
externa; en ambos casos realizarán una
investigación del asunto objeto de informe.

Un informe es un documento que incluye los objetivos de la investigación,


y el tipo de estudio realizado, las conclusiones que se han alcanzado así como el tipo de
actuaciones que se recomienda seguir.
La misión principal del informe es la de ayudar a tomar decisiones correctas y concretas.

14
-Pág.58-
U . D . 4 . - O RGANIZACIÓN Y GESTIÓN

3.3. DOCUMENTOS MERCANTILES


Una vez acordadas las condiciones que van a guiar las relaciones comerciales entre dos o
varias empresas, todos los pasos posteriores de la compraventa se documentan mediante una
serie de impresos que aparecen en orden cronológico y que garantizan el buen término de la
misma. Estos documentos son el pedido, el albarán y la factura.

3.3.1. EL PEDIDO
Se puede realizar utilizando diversas vías: teléfono, fax, carta, etcétera. Contiene una
información completa que como mínimo debe incluir:
– Identificación del emisor del pedido y del destinatario, con referencia a nombre o razón
social, domicilio social, NIF, etcétera.
– Número de pedido y fecha de emisión.
– Descripción de las mercancías solicitadas, con referencia a tipo, cantidad y precio.
– Lugar y plazo de entrega.
– Forma de envío.
– Determinación de quién paga los portes.
– Procedimiento de pago.
– Firma autorizada del comprador.
Del pedido se realizarán varios impresos, el
original, para la empresa vendedora, y diversas
copias para el departamento de contabilidad, para el
almacén y para el departamento de compras.

El pedido es la solicitud formal de compra que hace la empresa compradora a


la vendedora y, por tanto, es el documento que inicia una relación comercial.

3.3.2. EL ALBARÁN
Una vez admitido el pedido por la empresa
vendedora, el siguiente paso es el envío de los
productos solicitados.
La empresa receptora comprueba con la hoja de
pedido si los productos entregados coinciden con
los solicitados. Si está de acuerdo, firma en el alba-
rán original (que vuelve a la empresa vendedora) en
el apartado Recibí, y conserva al menos tres copias,

15
-Pág.59-
TECNOLOGÍA

una para almacén, otra para el departamento de contabilidad y otra para el de compras. Los
datos que contiene el albarán son similares a los del pedido.
El albarán es un documento con dos misiones: justificar la salida del almacén
de los productos de la empresa vendedora y acompañarlos
hasta su recepción por el cliente acreditando la entrega de los mismos.

3.3.3. LA FACTURA
Una vez efectuada la entrega de las mercancías, para que la operación finalice se procede al
pago. El documento que lo registra es la factura, que acredita legalmente la existencia de la
compraventa.
Los datos que deberá contener son:
– Nombre y apellidos o denominación social, NIF y
domicilio expedidor y destinatario.
– Número y, en su caso, serie, que será correlativa.
– Lugar y fecha de emisión,
– Descripción de la operación.
– Descuentos (pronto pago y comerciales).
– Otros gastos (transporte, envases, etcétera).
– Base imponible, cuotas del IVA y total.
– Forma de pago.
La factura es un documento que justifica el pago y es la base
de las posibles reclamaciones a que hubiere lugar, según el Código de Comercio y
la ley Reglamento del Impuesto sobre el Valor Añadido (IVA).
A la hora de comprar cualquier producto debemos seguir un proceso el cual está fijado por la
empresa. A continuación vemos reflejado el proceso de compra más utilizado por las empresas:

Selección del proveedor Negociaciones entre El cliente envía El proveedor


que suministrará las partes para fijar hoja de pedido envía la mercancía
los bienes y servicios las condiciones detallando la compra con el albarán

Pago de la factura al contado, El proveedor El cliente da


con cheque u envía la factura el visto bueno
otro instrumento ya fijado para su pago al recibir la mercancía

El cliente evalúa Finalización En caso de pago a plazos usando


el servicio recibido de la relación letra de cambio no finaliza
por el proveedor comercial la relación hasta el vencimiento

16
-Pág.60-
U . D . 4 . - O RGANIZACIÓN Y GESTIÓN

3.4. DOCUMENTOS DE COBROS Y PAGOS


Una operación comercial finaliza (excepto en lo que se refiere a las garantías postventa)
cuando se realiza el pago por los bienes o servicios objeto de la transacción.
Este pago se puede realizar mediante dinero en efectivo o mediante el uso de otros
instrumentos distintos convertibles en dinero; entre éstos nos encontramos con la letra de
cambio, el cheque y la nómina

3.4.1. LETRA DE CAMBIO


Este documento contiene los siguientes datos:
– La denominación de “letra de cambio” inserta en el texto mismo del título.
– El mandato puro y simple de pagar una suma determinada.
– El nombre de la persona que ha de pagar, denominado librado, que debe firmar la letra
(el acepto) para quedar obligado.
– La indicación del vencimiento. La letra puede vencer a una fecha fija, a un plazo contado
desde su emisión, a la vista (a su presentación) o a un plazo contado desde la vista.
– El lugar donde ha de hacerse efectivo el pago.
– El nombre de la persona a quien se ha de hacer el pago, denominado tomador. Puede ser
una tercera persona o el propio librador.
– La fecha y el lugar en que se libra la letra.
– La firma del que emite la letra, denominado librador.

La letra de cambio es un documento formalizado en un papel timbrado


mediante el cual una persona “librador” ordena a otra llamada “librado” pagar una
cantidad de dinero a una tercera persona “tomador”, en el lugar y
plazo de tiempo establecidos en el documento.
La letra de cambio puede transmitirse mediante el endoso, que permite que sea cobrada por
otra persona que no tuvo participación en la operación mercantil que la originó. Así mismo, se

17
-Pág.61-
TECNOLOGÍA

puede garantizar mediante el aval de una persona que hará frente al pago en caso de que el
librado no lo efectúe.

3.4.2. EL CHEQUE
Este documento contiene los siguientes datos:
– La denominación de “cheque” inserta en el texto mismo del título.
– El mandato puro y simple de pagar una suma determinada de dinero.
– El nombre del que debe pagar, llamado librado, que ha de ser un banco.
– El lugar de pago.
– La fecha y el lugar de la emisión del cheque.
– La firma de quien expide el cheque “el librador”.

El cheque, que es pagadero a la vista, puede ser librado para que se pague:
– Al portador. En este caso no es necesaria la identificación del tomador.
– Nominativo. Con la cláusula no a la orden u otra equivalente incluye en el cheque el
nombre del tomador y por tanto se exige su identificación.
– A la orden. Es igual que el anterior pero incluye la cláusula a la orden, que permite su
transmisión por endoso.
Un cheque es un documento mediante el cual una persona “librador”, con dinero
depositado en una cuenta corriente de un banco, ordena que dicho banco “librado”
pague una determinada cantidad a otra persona “tomador”, que porta el cheque.
Cheques especiales
– Cruzado. Viene cruzado por dos barras paralelas sobre el anverso. El banco sólo puede
pagar el cheque al banco designado entre las barras; de no existir esta mención, podrá
pagarlo a un banco o a un cliente de aquél.
– Para abonar en cuenta. El banco no puede pagar este cheque en efectivo, sólo lo puede
hacer mediante su ingreso en una cuenta. Se debe indicar en el anverso del cheque la
leyenda “para abonar en cuenta”.

18
-Pág.62-
U . D . 4 . - O RGANIZACIÓN Y GESTIÓN

– Conformado. Es un cheque en la que el banco (librado) acredita la autenticidad de éste


y la existencia de fondos suficientes, reteniendo la cantidad necesaria para el pago
durante un tiempo determinado.
Debemos tener en cuenta que en la letra de cambio y cheque el librador y el tomador pueden
ser la misma persona.

LIBRADO
Paga
Ordena

LIBRADOR TOMADOR

3.4.3. LA REMUNERACIÓN DEL TRABAJO


El salario es el conjunto de las prestaciones que un empleado recibe por su trabajo. Puede
ser en forma monetaria “dinero” o en “especie”:
El salario monetario. Puede constar de dos partes diferenciadas: una parte fija, que es el salario
base, y un complemento, que suele depender de las características personales del trabajador y que
puede retribuir el rendimiento (productividad), la antigüedad, la formación recibida, etcétera.
El cobro en especie. Es parte del
salario en forma de ventajas económicas
o sociales. Son ejemplos el coche de la
empresa, los planes de pensiones, los
seguros médicos, los viajes, etc.
Este documento contiene los siguien-
tes datos:
– Datos identificativos de la em-
presa: nombre de la empresa y
trabajador, domicilio, categoría
del trabajador.
– Periodo de liquidación.
– Conceptos de cobro, cuyo total es
denominado salario devengado.
– Conceptos de descuento denomi-
nado retenido.
– La base de cotización de la
seguridad social y el IRPF
– Líquido a percibir.
– Firma y sello de la entidad que
emite la nómina.

19
-Pág.63-
TECNOLOGÍA

Las empresas calculan los salarios tomando en consideración aspectos como la competencia
profesional, la fatiga mental o física que acarrea la realización del trabajo, el grado de
responsabilidad asumido, las condiciones de seguridad e higiene en el trabajo, etcétera. La
motivación de los empleados se fomenta estableciendo, además el salario base, las siguientes
formas de retribución:
– Prima de productividad, que consiste en una cantidad variable que retribuye la
productividad del trabajador.
– Prima de antigüedad, que premia la fidelidad del empleado mediante un complemento
variable en función de los años de permanencia en la empresa. El pago de este concepto
puede ser 2 años (bienios), 3 años (trienios) o 5 años (quinquenios).
– Aumentos de sueldo, que se consolidan de forma permanente en el sueldo del trabajador.
– Gratificaciones extraordinarias, que se otorgan a los empleados de forma puntual.
– Participación en beneficios y en el capital de la empresa, para ligar el salario percibido
por el trabajador al buen funcionamiento de la empresa.

20
-Pág.64-
U . D . 4 . - O RGANIZACIÓN Y GESTIÓN

RESUMEN

Antes de empezar la fabricación de un objeto debemos buscar información sobre el


producto que vamos a fabricar, para saber si es posible su construcción y puede llevarse a cabo
de la mejor manera posible.
Una vez contrastados los materiales y demás elementos necesarios para fabricar el objeto en
el aula taller, debemos organizar toda esta información con los medios más utilizados por las
empresas y gestionar, posteriormente, las obligaciones que conlleva el realizar ese objeto, como
presupuestar los que vamos a construir, pedir a los proveedores los materiales necesarios y
pagar a estos como a las personas que nos han ayudado a fabricar el objeto.

21
-Pág.65-
TECNOLOGÍA

EJERCICIOS DE AUTOCOMPROBACIÓN

1. Pensando en realizar un proyecto, lo primero que debemos hacer es el presupuesto de lo que va a


costar el objeto que vamos a fabricar: “Raqueta de PING-PONG”.

Lista de materiales

Pieza n.o Denominación Material Cantidad


1 Pala Contrachapado 1
2 Protector Corcho 2
3 Amortiguador Goma 2
4 Cacha Madera 2

Presupuesto

Oficina de Estudios Producto: Raqueta de Ping-Pong


Presupuesto Industrial
Fecha ..../..../.... Cantidad: 2.000 unidades
Concepto ./unidad Cantidad Consumo Coste unitario
Contrachapado 10 mm. 26 €/m2 m2/producto m2 €
Goma 3 mm. 210 €/m2 m2/producto m2 €
Corcho 5 mm. 23 €/m2 m2/producto m2 €
Listón pino 30 x 15 1,25 €/m. lineal metros metros €
Herramienta de serrar 2,25 €/unidad 250 productos/sierra sierra €
Operario de sierra 20 €/hora 0,2 horas/producto horas €
Operario montador 20 €/hora 0,3 horas/producto horas €
Uso máquina sierra 10 €/hora 0,2 horas/producto horas €
Coste unitario €
Asignación gastos generales al producto (40% sobre el coste) €
COSTE UNITARIO TOTAL €

22
-Pág.66-
U . D . 4 . - O RGANIZACIÓN Y GESTIÓN

2. Redacta una carta comercial para una marca prestigiosa de Manufacturas LINK S.A.
C/ Corresponsal, 45 2.o A
material deportivo ofreciéndole la venta de 2.000 28029 - MADRID
NIF-Q32/12345
unidades del objeto que has realizado el presupuesto
Madrid a 23 de mayo de 1999
“raqueta de ping-pong”.
NIKE S.A.
Marca comercial: NIKE S.A. Pol. Ind. Cobo Calleja
C/ Torno, nave 34
28956 - Fuenlabrada (Madrid)
Pol. Ind. Cobo Calleja
C/ Torno, nave 34
28956 - Fuenlabrada (Madrid)

3. Redacta una instancia al alcalde del ayuntamiento para pedir la licencia de apertura de una taller
para fabricar material deportivo.
Nota: Los datos identificativos de la empresa están en el documento de pedido de la pregunta siguiente. Los
datos de la persona que la realiza pon tus datos personales.

4. Cumplimenta un documento de pedido a un proveedor para que te suministre el material necesario


para fabricar las 2.000 raquetas.

Manufacturas LINK S.A. Pedido N.o: 1234-00


C/ Correponsal,45 2.o A Realizado a:
28029 - MADRID Suministros Industriales JAVI
NIF - Q32/12345 C/ Fresa, 2
FECHA: 12-3-99 28897 - ALCORCÓN (Madrid)
Pago: 30% contado 70% Letra NIF: A-28/567895
Portes: Vendedor
Forma de envío: TDM

Descripción Cantidad Precio unitario

Firma:

23
-Pág.67-
TECNOLOGÍA

5. Genera la factura del material que has recibido para fabricar las 2.000 unidades.
Suministros Industriales JAVI Factura N.o: 1234-00
C/ Fresa, 2 Albarán: 12
28897 - ALCORCÓN (Madrid) Pedido ref.: 1234-00
NIF - A-567895 Manufacturas LINK S.A.
Fecha: C/ Corresponsal, 45 2.o A
Cliente: 1234 28029 - MADRID
NIF: Q32/12345

Descripción Cantidad Precio unitario Importe

IMPORTE TOTAL BRUTO


Descuento comercial (10%)
Descuento por pronto pago (2%)
Gastos de transporte
Base imponible
IVA (16%)
IMPORTE TOTAL
Forma de pago: 30% contado 70% Letra

6. Formaliza los pagos a la empresa que te suministró el material.


30% a la entrega material mediante cheque bancario.

70% letra de cambio a 90 días

24
-Pág.68-
U . D . 4 . - O RGANIZACIÓN Y GESTIÓN

RESPUESTAS A LOS EJERCICIOS


1. PRESUPUESTO
Oficina de Estudios Producto: Raqueta de Ping-Pong
Presupuesto Industrial
Fecha ..../..../.... Cantidad: 2.000 unidades
Concepto ./unidad Cantidad Consumo Coste unitario
1. Contrachapado 10 mm. 26 €/m2 0,039 m2/producto 78 m2 0,234 €
2. Goma 3 mm. 210 €/m2 0,048 m2/producto 96 m2 0.48 €
3. Corcho 5 mm. 23 €/m2 0,048 m2/producto 96 m2 0,144 €
4. Listón pino 30 x 15 1,25 €/m. lineal 0,2 metros 400 metros 0,25 €
5. Herramienta de serrar 2,25 €/unidad 250 productos/sierra 8 sierra 0,009 €
6. Operario de sierra 20 €/hora 0,2 horas/producto 400 horas 4€
7. Operario montador 20 €/hora 0,3 horas/producto 600 horas 6€
8. Uso máquina sierra 10 €/hora 0,2 horas/producto 400 horas 2€
Coste unitario 13,117 €
Asignación gastos generales al producto (40% sobre el coste) 5,246 €
COSTE UNITARIO TOTAL 18,363 €

Concepto 1: Cantidad Sup = 0,15 x 0,26 = 0,039 m2


Consumo Total = 0,039 x 2.000 unidades = 78 m2
Coste €/unidad = 0,039 x 6 €/m2 = 0,234 €

Concepto 2: Cantidad Sup = (0,16 x 0,15) x 2 unidades por pieza = 0,048m2


Consumo Total = 0,048 x 2.000 unidades= 96 m2
Coste €/unidad = 0,048 x 10 €/m2 = 0,48 €

Concepto 3: Cantidad Sup = (0,16 x 0,15) x 2 unidades por pieza = 0,048m2


Consumo Total = 0,048 x 2.000 unidades= 96 m2
Coste €/unidad = 0,048 x 3 €/m2 = 0,144 €

Concepto 4: Cantidad Sup = 0,1 x 2 unidades por pieza = 0,2m2


Consumo Total = 0,2 x 2.000 unidades= 400 m2
Coste €/unidad = 0,2 x 1,25 €/m2 = 0,25 €

Concepto 5: Consumo Total = 2.000 unidades / 250 piezas por sierra = 8 unidades
Coste €/unidad = 2,25 € x 250 piezas = 0,009 €

Concepto 6: Consumo Total = 0,2 horas/producto x 2.000 unidades = 400 horas


Coste €/unidad = 20 €/hora x 0,2 horas/producto = 4 €/unid.

Concepto 7: Consumo Total = 0,3 horas/producto x 2.000 unidades = 600 horas


Coste €/unidad = 20 €./hora x 0,3 horas/producto = 6 €/unid.

Concepto 8: Consumo Total = 0,2 horas/producto x 2.000 unidades = 400 horas


Coste €/unidad = 10 €./hora x 0,2 horas/producto = 2 €/unid.

25
-Pág.69-
TECNOLOGÍA

2. CARTA COMERCIAL
Manufacturas LINK S.A.
C/ Corresponsal, 45 2.o A
28029 - MADRID
NIF-Q32/12345

Madrid a 23 de mayo de 1999


NIKE S.A.
Pol. Ind. Cobo Calleja
C/ Torno, nave 34
28956 - Fuenlabrada (Madrid)
A la atención del Sr. Andrés Calvo

Muy Sres. Míos:


Me pongo en contacto con Vds. para ofertarles una raqueta de ping-pong en
unas condiciones de precio pocas veces vistas.
2.000 unidades al precio de 18,363 €/unidad.

Sin otro particular y en espera de sus gratas noticias se despide atentamente.xxx

DIRECTOR GENERALxxx

Fdo.: Emilio González Pérezxxx

3. INSTANCIA
EMILIO GONZÁLEZ PÉREZ, mayor de edad, casado, de profesión carpintero,
natural de Madrid, con domicilio en c/ verja, 26 de Madrid, 28028 - Madrid, con
DNI 1.231.567

EXPONE:
Que deseando abrir un local comercial dedicado a la fabricación de
material deportivo en la c/ Corresponsal, 45 piso 2.o A de esta localidad
y estando en posesión de la Licencia Fiscal y demás documentación
requerida por la legislación vigente, es por lo que,

SOLICITA:
Le sea admitida esta instancia y concedida la licencia de apertura
necesaria para poder emprender a la mayor brevedad posible las
actividades comerciales en el citado local.

Madrid, dos de mayo de dos mil.


xxx
EMILIO GONZÁLEZ PÉREZxxx

SR. ALCALCE DEL AYUNTAMIENTO DE MADRID

26
-Pág.70-
U . D . 4 . - O RGANIZACIÓN Y GESTIÓN

4. PEDIDO

Manufacturas LINK S.A. Pedido N.o: 1234-00


C/ Correponsal,45 2.o A Realizado a:
28029 - MADRID Suministros Industriales JAVI
NIF - Q32/12345 C/ Fresa, 2
FECHA: 12-3-99 28897 - ALCORCÓN (Madrid)
Plazo de entrega: 15 días NIF: A-28/567895
Pago: 30% contado 70% Letra
Portes: Vendedor
Forma de envío: TDM

Descripción Cantidad Precio unitario


Contrachapado de 10 mm. 78 m2 6 €/m2
Goma adhesiva de 3 mm. 96 m2 10 €/m2
Corcho adhesivo de 5 mm. 96 m2 3 €/m2
Listón madera pino 30 x 15 400 metros lineales 1,25 €/metro lineal

Firma:

5. FACTURA

Suministros Industriales JAVI Factura N.o: 1234-00


C/ Fresa, 2 Albarán: 12
28897 - ALCORCÓN (Madrid) Pedido ref.: 1234-00
NIF - A-567895 Manufacturas LINK S.A.
Fecha: C/ Corresponsal, 45 2.o A
Cliente: 1234 28029 - MADRID
NIF: Q32/12345

Descripción Cantidad Precio unitario Importe


Contrachapado de 10 mm. 78 m2 6 €/m2 468
Goma adhesiva de 3 mm. 96 m2 10 €/m2 960
Corcho adhesivo de 5 mm. 96 m2 3 €/m2 288
Listón madera pino 30 x 15 400 metros lineales 1,25 €/metro lineal 500

IMPORTE TOTAL BRUTO 2216


Descuento comercial (10%) 221,6
Descuento por pronto pago (2%) 39,88
Gastos de transporte —
Base imponible 1954,52
IVA (16%) 312,72
IMPORTE TOTAL 2267,24
Forma de pago: 30% contado 70% Letra

27
-Pág.71-
TECNOLOGÍA

6. PAGO DEL 30% CON CHEQUE

# 680,17 € #
Suministros Industriales JAVI
Seiscientos ochenta con diecisiete euros
Madrid a doce Junio 99

PAGO DEL 70% CON LETRA DE CAMBIO


EUROS 1587,07 €

12 JUNIO 1999 12 SEP. 1999


Suministros Industriales JAVI

mil quinientos ochenta y siete euros con siete céntimos

Manufacturas LINK S.A.


C/ Corresponsal, 45 2.o A
28029 MADRID

Emilio González Pérez


MADRID
MADRID

28
-Pág.72-
portada TROPA 19/3/07 19:51 Página 1

FUERZAS ARMADAS
PROFESIONALES
CURSO DE APOYO
A LA PREPARACIÓN
DE LAS PRUEBAS DE ACCESO
A UNA RELACIÓN DE SERVICIOS
DE CARÁCTER PERMANENTE

TECNOLOGÍA
3ª parte
Unidades didácticas 5 y 6

DIGEREM

MINISTERIO
DE DEFENSA
FUERZAS ARMADAS SUBDIRECCIîN GENERAL
DE TROPA Y MARINERIA
PROFESIONAL
PROFESIONALES
CURSO DE APOYO
A LA PREPARACIÓN
DE LAS PRUEBAS DE ACCESO
A UNA RELACIÓN DE SERVICIOS
DE CARÁCTER PERMANENTE

TECNOLOGÍA
3ª parte
Unidades didácticas 5 y 6

-Pág.1-
La Ley 8/2006 de Tropa y Marinería, en su artículo 16,1, establece que “la formación
en las Fuerzas Armadas garantizará que los militares profesionales de tropa y
marinería puedan adquirir, actualizar o ampliar sus conocimientos para un mayor
desarrollo personal y profesional”. En cumplimiento de este mandato, el Ministerio
de Defensa edita el presente material didáctico para facilitar a los militares
profesionales de tropa y marinería, alumnos de los cursos de formación
presencial que se imparten a través de la Dirección General de Reclutamiento y
Enseñanza Militar, los apoyos necesarios para preparación de dichos cursos, que
permitirán, siempre que superen las pruebas correspondientes, la obtención de la
titulación de graduado en Educación Secundaria, acreditación para el acceso a
los ciclos formativos de la Formación Profesional de grado medio o de grado
superior, acceso a las Escalas de Suboficiales, Tropa Permanente, Guardia Civil
y Policía Nacional.

CATÁLOGO GENERAL DE PUBLICACIONES


http://www.060.es

Edita:

© Autor y editor
NIPO: 076-10-204-9 NIPO: 076-10-205-4 (edición en línea)
Depósito Legal: M-32363-2009
Diseño y programación: cimapress
Tirada: 1300 ejemplares
Fecha de edición: septiembre, 2010

Prohibida la reproducción total o parcial de esta obra, por cualquier medio sin autorización escrita del editor
TECNOLOGÍA
3ª parte

SUMARIO

Unidad didáctica Pág.

5. RECURSOS CIENTÍFICOS Y TÉCNICOS 5

6. TECNOLOGÍA Y SOCIEDAD 73

-Pág.3-
U.D. 5.- RECURSOS CIENTÍFICOS Y TÉCNICOS

ÍNDICE
OBJETIVOS . . . . . . . . . . . . . . . . . . . . . . . . . . . . . . . . . . . . . . . . . . . . . . . . . . . . . . . . . . .2

INTRODUCCIÓN . . . . . . . . . . . . . . . . . . . . . . . . . . . . . . . . . . . . . . . . . . . . . . . . . . . . . . .3

MAPA CONCEPTUAL . . . . . . . . . . . . . . . . . . . . . . . . . . . . . . . . . . . . . . . . . . . . . . . . . . .4

DESARROLLO DE CONTENIDOS . . . . . . . . . . . . . . . . . . . . . . . . . . . . . . . . . . . . . . . . .5

1. ESFUERZOS Y ESTRUCTURAS . . . . . . . . . . . . . . . . . . . . . . . . . . . . . . . . . . . . . .5

2. ELEMENTOS DE UNIÓN . . . . . . . . . . . . . . . . . . . . . . . . . . . . . . . . . . . . . . . . . .12

3. TRANSMISIÓN, TRANSFORMACIÓN DE ESFUERZOS Y MOVIMIENTOS . .16

4. CIRCUITOS Y SUS ELEMENTOS . . . . . . . . . . . . . . . . . . . . . . . . . . . . . . . . . . .24

5. LOS MATERIALES . . . . . . . . . . . . . . . . . . . . . . . . . . . . . . . . . . . . . . . . . . . . . . .33

6. REPERCUSIONES MEDIOAMBIENTALES DE LA EXPLOTACIÓN DE LOS


MATERIALES TÉCNICOS . . . . . . . . . . . . . . . . . . . . . . . . . . . . . . . . . . . . . . . . .49

7. TÉCNICAS DE MEDIDA . . . . . . . . . . . . . . . . . . . . . . . . . . . . . . . . . . . . . . . . . . .53

RESUMEN. . . . . . . . . . . . . . . . . . . . . . . . . . . . . . . . . . . . . . . . . . . . . . . . . . . . . . . . . . . .57

EJERCICIOS DE AUTOCOMPROBACIÓN . . . . . . . . . . . . . . . . . . . . . . . . . . . . . . . . .58

RESPUESTAS A LOS EJERCICIOS . . . . . . . . . . . . . . . . . . . . . . . . . . . . . . . . . . . . . . .62

-Pág.5-
TECNOLOGÍA

O B J E T I VO S
Al finalizar el estudio de esta Unidad Didáctica, el alumno será capaz de:

• Analizar las deformaciones, esfuerzos y tensiones que se pueden producir en los


objetos.

• Identificar los elementos fundamentales de todo soporte.

• Distinguir los sistemas de unión para saber como están construidas las estructu-
ras que nos rodean.

• Clasificar y saber diferenciar entre mecanismos que transforman un movimien-


to circular en lineal o viceversa.

• Diferenciar los distintos tipos de circuitos para el control de mecanismos y sus


elementos.

• Enumerar los diferentes tipos de materiales para la fabricación de objetos y sus


aplicaciones.

• Analizar el comportamiento de los materiales según sus propiedades.

• Reconocer las principales repercusiones medioambientales por la explotación de


los materiales.

• Diferenciar y aplicar los conceptos de magnitud, medida, verificación y tipos de


errores.

• Definir correctamente las unidades de medida.

• Manejar los instrumentos básicos para medir longitudes y ángulos en el aula


taller.

-Pág.6-
U.D. 5.- RECURSOS CIENTÍFICOS Y TÉCNICOS

INTRODUCCIÓN
A un cuando, por regla general, no lo percibimos, nos encontramos rodeados de
estructuras. Algunas son enormes y espectaculares y otras son pequeñas, elementa-
les y cotidianas.

Éstas están diseñadas cuidadosamente para soportar los distintos esfuerzos que se pro-
ducen en ellas interiormente o por las condiciones ambientales y climatológicas que le
rodean. Se han construido con diversos materiales y éstos están unidos mediante dife-
rentes técnicas para hacer más seguro el montaje de la estructura.

En ellas, muchas veces, se originan por mecanismos una serie de movimientos. A través
de los cuales puede controlarse su funcionamiento por diferentes tipos de circuitos según
sea la energía a mandar.

Ahora bien, para hacer el ensamblaje se utilizan diferentes técnicas de medida que per-
mitan verificar su perfecta puesta en marcha.

Esta sociedad está avanzando espectacularmente gracias a grandes adelantos, de los cua-
les no nos damos cuenta y están afectando de una manera muy grave a la naturaleza y a
la corteza terrestre.

-Pág.7-
TECNOLOGÍA

M A PA C O N C E P T UA L
RECURSOS CIENTÍFICOS
Y TÉCNICOS DUREZA, ELASTICIDAD
RESISTENCIA, PLASTICIDAD
PROPIEDADES DE
ESFUERZOS
LOS MATERIALES

ENSAYOS
ESTRUCTURAS

RÍGIDAS Y ARTICULADAS
TIPOS DE
SE ESTRUCTURAS
MONTAN VERTICALES Y HORIZONTALES

UNIONES FIJAS
SISTEMAS
DE UNIÓN
UNIONES
DESMONTABLES
SIRVEN PARA MOVIMIENTO CIRCULAR-LINEAL
TRANSMITIR
MOVIMIENTO TRANSMISIÓN Y
MOVIMIENTO LINEAL-CIRCULAR
TRANSFORMACIÓN

MOVIMIENTO CIRCULAR-CIRCULAR

CIRCUITOS ELÉCTRICOS

EL
MECANISMOS CIRCUITOS ELECTRÓNICOS
CONTROL

CIRCUITOS NEUMÁTICOS

TIPOS Y APLICACIONES
FABRICADOS MATERIALES
INDUSTRIAS DERIVADAS

REPERCUSIONES MEDIOAMBIENTALES
ERRORES DE MEDIDA

VERIFICAR SU TÉCNICAS DE SISTEMA DE UNIDADES


FABRICACIÓN MEDICIÓN
MEDICIÓN

-Pág.8-
U.D. 5.- RECURSOS CIENTÍFICOS Y TÉCNICOS

1. ESFUERZOS Y ESTRUCTURAS
Prácticamente, casi todos los cuerpos están sostenidos por algún tipo de estructura que
soporta cargas y esfuerzos, haciendo que éstos reaccionen ante estas cargas de alguna forma.
Por ello, estamos en contacto permanente con las estructuras que forman parte de nuestra
vida. Desde que se construyó la primera cabaña hasta los días de hoy, las estructuras han ido
evolucionando y con ellas los distintos objetos que forman parte de estas: máquinas, edificios.

1.1. DEFORMACIONES, ESFUERZOS Y TENSIONES


Debido a unas fuerzas de cohesión internas, los átomos de los sólidos se mantienen unidos
o en equilibrio de manera natural. Si los cuerpos se ven sometidos a una fuerza exterior, ésta
modifica el equilibrio, produciendo un deslizamiento en los átomos. Si dicha fuerza persiste y
supera a las internas de cohesión, el resultado es que se produce una deformación.
Las deformaciones de los cuerpos sólidos pueden ser de dos clases: elásticas y plásticas.
Deformaciones elásticas. El desplazamiento de los átomos, debido a fuerzas externas, no
sobrepasa un valor límite característico de cada material: al cesar la fuerza externa, los átomos
vuelven a su estado de equilibrio.
Deformaciones plásticas. El desplazamiento de los átomos sobrepasa el límite de elastici-
dad; al cesar las fuerzas externas los desplazamientos se mantienen permanentes y el cuerpo se
fractura o deforma.

Deformaciones elásticas Deformaciones plásticas

Todos los cuerpos, cuando se someten a la acción de una fuerza exterior, reaccionan con otra
fuerza interior que trata de equilibrarla.
– Las fuerzas externas aplicadas a los cuerpos se denominan esfuerzos.
– Las fuerzas internas que oponen los cuerpos se denominan tensiones.

5
-Pág.9-
TECNOLOGÍA

a) Si tratamos de estirar un muelle aplicándole una fuerza


(Fa), reacciona en contra Fa con otra equivalente en sen-
tido contrario (Fr).

b) Si es a la inversa, al tratar de comprimirlo (Fa) detecta-


mos una fuerza contraria (Fr) a la acción de nuestra
mano.

1.2. TIPOS DE ESFUERZOS


Las estructuras deben estar correctamente medidas y calculadas para los esfuerzos a los que
están o pueden estar sometidas. Estos esfuerzos se deben a fuerzas estáticas, relacionadas con
pesos de las propias estructuras que soportan (tejados, nieve), o fuerzas dinámicas relacionadas
con elementos sometidos a movimiento (circulación de vehículos, terremotos, vientos).
Los esfuerzos aplicados a un elemento estructural o a una estructura pueden ser efectuados
por fuerzas resultantes o por pares de fuerzas.
Las fuerzas resultantes producen cambios dimensionales, de longitud fundamentalmente
(tracción y compresión) o deslizamiento de las caras internas (cortadura o cizallamiento). Los
pares de fuerzas modifican la curvatura (flexión) o determinan un giro (torsión).

1.2.1. TRACCIÓN
Un elemento o cuerpo está sometido a un esfuerzo de
tracción si dos fuerzas iguales y de sentido contrario tien-
den a alargarlo, produciendo un incremento de su longi-
tud y un estrechamiento de su sección en la parte central.
Trabajan con esfuerzo de tracción: el cable de un ascen-
sor, cualquier tirante, las cuerdas de un columpio, etc.

1.2.2. COMPRESIÓN
Se dice que un cuerpo está sometido a un esfuerzo de compre-
sión cuando las fuerzas exteriores tienden a comprimirlo aumen-
tando su sección central.
Trabajan con esfuerzo de compresión: todos los pilares, las
patas de una silla, de una mesa, las ruedas de los automóviles, etc.

6
-Pág.10-
U.D. 5.- RECURSOS CIENTÍFICOS Y TÉCNICOS

1.2.3. CORTADURA O CIZALLAMIENTO


Si se produce un deslizamiento entre dos caras de la
sección de un elemento, debido a la acción que ejercen
sobre él dos fuerzas de sentido contrario que actúan en
planos paralelos muy próximos, se dice que el elemento
está sometido a un esfuerzo de cortadura o cizallamiento.
Trabajan con este tipo de esfuerzos: el pasador de la
cadena de una bicicleta, tornillos y ganchos que sirven
para colgar peso, etc.

1.2.4. FLEXIÓN
Supongamos una viga apo-
yada en sus dos extremos a la
que se le aplica un peso o carga
en el centro. En los dos puntos
de apoyo aparecen, como ten-
siones, dos fuerzas paralelas en
sentido contrario a la fuerza
producida por la carga. Esto
produce una curvatura en la
viga y se dice que está sometida a un esfuerzo de flexión.
Están sometidos a esfuerzos de flexión: las vigas, las baldas de las estanterías, etc.
En realidad, en la flexión se producen en la viga dos esfuerzos: uno de compresión en la
parte superior, en la que sus fibras tienden a acortarse, y otro de tracción en su parte inferior, en
la que sus fibras tienden a estirarse; existe una línea central en la que sus fibras no están some-
tidas ni a compresión ni a tracción, denominada línea neutra.

1.2.5. TORSIÓN
Si un elemento está sometido a dos pares de fuer-
zas, uno en cada extremo de distinto sentido, el ele-
mento está sometido a un esfuerzo de torsión. Los
pares de fuerzas tienden a girarlo en distinto sentido,
con lo que las secciones próximas tenderán a desli-
zarse con un movimiento giratorio.
Están sometidos a torsión: los ejes, los tornillos,
las brocas, etc.

7
-Pág.11-
TECNOLOGÍA

1.2.6. PANDEO
Al someter al esfuerzo de compresión a un elemento de una estructura cuya longitud es muy
grande con respecto a su sección, y con una fuerza ligeramente descentrada con la de reacción
del suelo, se produce el llamado esfuerzo de pandeo. El elemento se comporta como si estuvie-
ra sometido a una flexión originada por la resultante de las tensiones.

Trabajan sometidos a un esfuerzo de pandeo: la pértiga de un sal-


tador, algunos tipos de columnas, etc. En el pandeo influye la longi-
tud del elemento y su sección.

1.3. ELEMENTOS DE SOPORTE


Los soportes o estructuras pueden formarse con una gran variedad de elementos. Citaremos
algunas de las más utilizadas:
El pilar. Es un elemento de las estructuras que se sitúa de forma vertical y que soporta los
esfuerzos paralelos a su eje. Puede ser metálico, de madera, de hormigón armado (hormigón con
barras de acero), etc.
Adoptan la función de pilares las patas de una mesa o silla, los postes que soportan un por-
che, los muros y elementos verticales de la estructura de un edificio, etc.
La viga. Es un elemento que se coloca de manera horizontal en un armazón sostenido por
dos o más puntos, frecuentemente sobre pilares, y que soporta esfuerzos perpendiculares a su
eje. Las vigas pueden ser simples (un solo elemento) o compuestas por varios elementos, en

8
-Pág.12-
U.D. 5.- RECURSOS CIENTÍFICOS Y TÉCNICOS

celosía. Si la viga tiene un extremo empo-


trado y el otro libre se denomina ménsula.
Pueden ser de materiales muy variados,
dependiendo de su función. Son ejemplos
de elementos que actúan como vigas: el
forjado de un edificio, un tablón para atra-
vesar una zanja, etcétera.

El tensor. Actúa tensando o tirando de otros elementos.


Los tensores pueden ser rígidos, como los cables de acero, o
flexibles, como las cuerdas.
Suelen instalarse en grandes mástiles, en estructuras
metálicas de antenas, en letreros de cierto tamaño, en mén-
sulas, etc.

El tirante. Es un elemento muy importante en las estruc-


turas de un armazón hechas con barras ensambladas. Evita
las posibles deformaciones en las formas rectangulares, al
colocarse diagonalmente en las mismas, y da origen a for-
mas triangulares típicas de las estructuras metálicas que les
proporcionan rigidez.
Algunos ejemplos de tirantes están presentes en las grúas
de la construcción, en las cubiertas de naves, en recintos
feriales, etc.

Cada elemento de una estructura está calculado para realizar la función de soportar
parcialmente un esfuerzo o peso, y participa en el global de la misma; si es débil,
puede fallar toda la estructura.

9
-Pág.13-
TECNOLOGÍA

Otros elementos de soportes y estructuras


Las escuadras. Se suelen colocar en las partes angulares de los soportes o estructuras rec-
tangulares para impedir deformaciones o variaciones de los ángulos de las mismas con el peso
o los esfuerzos. Las escuadras actúan como tirantes. Se utilizan en estanterías metálicas, mue-
bles, etc., adoptando formas comerciales variadas.
El arco. Se usa fundamentalmente en arqui-
tectura, y consiste en una curva que cubre un
vacío o vano existente entre dos pilares o puntos
fijos. Están presentes en muchos puentes o acue-
ductos.
A veces, las estructuras se recubren con ele-
mentos muy variados, como paredes verticales,
planchas prefabricadas o láminas de chapa de
acero, a los que se les dan diferentes formas
(automóviles, barcos, aviones o depósitos). A
veces, por motivos económicos o funcionales,
no se recubren estas estructuras, como es el caso
de puentes de ferrocarril, pasarelas o grúas.

1.4. DISPOSICIONES ESTRUCTURALES


Una estructura es un conjunto de elementos organizados y unidos o ensamblados de tal
manera que adquieren una forma constructiva coherente (armadura, carrocería) y que cumple la
función de soportar o transportar cargas, así como absorber los esfuerzos que se producen, sin
deformarse o romperse.
Existen estructuras naturales, como las estructuras cristalinas
de compuestos sólidos o el esqueleto de las personas y los ani-
males; pero son las realizadas por el ser humano, para cubrir sus
necesidades o realizar una función específica, las que se tratarán
con detenimiento.
Dichas estructuras se pueden clasificar atendiendo a diferen-
tes criterios:
Materiales empleados en su construcción. Metales, madera,
plásticos, etcétera. Las estructuras metálicas se utilizan con
mucha frecuencia y se forman con barras y elementos unidos
mediante soldaduras, tornillos o remaches formando triangula-

10
-Pág.14-
U.D. 5.- RECURSOS CIENTÍFICOS Y TÉCNICOS

ciones que les confieren rigidez, además de ser ligeras. De este tipo son las torres de alta ten-
sión, los puentes, etc.
Grado de permanencia en un deter-
minado lugar. En este caso se podría reali-
zar, a su vez, una segunda clasificación en
estructuras: fijas o estáticas y móviles o
dinámicas. El grupo de las fijas compren-
dería aquellas que se refieren directamente
a la construcción, como palacios de depor-
tes, centros comerciales o edificios de
diversas funciones; en el grupo de las
móviles se encontrarían los elementos que
se utilizan para transporte, barcos, grúas,
aviones, puentes-grúa, etc.
Función que desempeñan. Pueden ser
permanentes o temporales (andamios, esce-
narios) y rígidas o articuladas (hamacas,
sillas plegables).
En el diseño de una estructura se busca
combinar elementos ligeros que cumplan
su función con el menor coste posible,
siempre que se ajuste a los criterios de fun-
cionalidad y seguridad.
A la hora de diseñar una estructura debemos tener en cuenta su estabilidad y centro de gra-
vedad.
Se dice que una estructura es estable cuando ésta no se vuelca fácilmente por la acción de
las fuerzas que actúan sobre ella. Las estructuras estables se pueden mover hasta cierto punto
sin volcarse mientras actúan fuerzas sobre ellas, y recuperan la posición inicial al cesar dichas
fuerzas. La estabilidad de una estructura depende de la amplitud de su base de apoyo y de la
situación de su centro de gravedad.
Es evidente que cuanto más grande sea la base de un objeto respecto de su altura, más difí-
cil resultará volcarlo.

El centro de gravedad de un cuerpo es el punto donde confluye la fuerza resultante de la


suma de todas las fuerzas que constituyen el peso del cuerpo o estructura.

11
-Pág.15-
TECNOLOGÍA

Si al moverse un cuerpo su centro


de gravedad sube, al cesar la fuerza que
le hace moverse el cuerpo recuperará
su posición por la acción de su propio
peso; por tanto, será un cuerpo o
estructura estable. En cambio, si el cen-
tro de gravedad desciende, no sólo no
recuperará su posición sino que irá bas-
culando hasta volcar impulsado por su
propio peso; en este caso se trata de un
cuerpo o estructura inestable.
En consecuencia podemos afirmar
que los cuerpos, o estructuras, serán
más estables cuanto más bajo se
encuentre su centro de gravedad res-
pecto a lugar donde están apoyados.
Para determinar aproximadamente
la altura del centro de gravedad, dibu-
jaremos la silueta del objeto en cartón
grueso. A continuación la colgaremos
de un clavo del que también cuelgue
un peso suspendido de un hilo o cuer-
da. Dibujaremos sobre el cartón la
línea que marca el hilo y después col-
garemos la silueta del objeto por otro
punto. El punto de intersección de las
líneas dibujadas marcará la altura del
centro de gravedad.

2. UNIÓN DE ELEMENTOS
La mayoría de los objetos que construimos pueden tener una mayor o menor complejidad y
estar compuestos por un mayor o menor número de elementos.
Una de las tareas fundamentales, cuando se construye un objeto, es unir las piezas o ele-
mentos que lo componen.
La unión de las piezas que integran un objeto puede ser fija o desmontable.

12
-Pág.16-
U.D. 5.- RECURSOS CIENTÍFICOS Y TÉCNICOS

2.1. UNIONES FIJAS


Las uniones fijas son aquéllas en las que no se pueden desmontar las piezas una vez unidas,
si no es destruyendo o deformando el elemento de unión o las propias piezas. Las uniones fijas
se pueden llevar a cabo mediante uniones remachadas, pegadas y soldadas.

2.1.1. UNIONES REMACHADAS


El remachado es una operación que se realiza mediante rema-
ches para conseguir uniones fijas. Los remaches son piezas de
materiales blandos y deformables, cuya parte cilíndrica, llamada
caña o espiga, está rematada en una cabeza que puede tener dife-
rentes formas y tamaños.
La operación del remachado consiste en atravesar con el rema-
che las piezas a unir, que se han de taladrar previamente. El rema-
che, cuyo diámetro ha de ser equivalente al del agujero taladrado, se
introduce completamente en el agujero; de esta forma, la parte de la
espiga que sobresale por el otro extremo queda deformada (aplasta-
da). Esta operación puede realizarse a mano, golpeando con un mar-
tillo, o a máquina, con unas herramientas llamadas remachadoras.
También existen diferentes tipos de remaches así como de
remachadoras, la composición del material de trabajo.

2.1.2. UNIONES PEGADAS


Las uniones pegadas son aquéllas que se realizan utilizando colas y pegamentos. A la hora
de realizar el pegado, primero se tienen que limpiar y desengrasar las superficies a unir y, a con-
tinuación, aplicar la cola o el pegamento tal como indican las instrucciones del producto. Tam-
bién pueden unirse piezas por termo encolado, que consiste en aplicar la cola de finas barritas
de adhesivo termo fusible, que se funde con las llamadas pistolas termo encoladoras; estas pis-
tolas, además, facilitan la aplicación.

13
-Pág.17-
TECNOLOGÍA

La manipulación de colas y pegamentos se ha de realizar con las máximas precauciones


posibles, siguiendo escrupulosamente las instrucciones que el fabricante está obligado a incluir
en estos productos.

2.1.3. UNIONES SOLDADAS


La soldadura es un procedimiento para realizar uniones
fijas entre piezas metálicas en el que las piezas se someten a
la acción del calor. La soldadura se puede realizar con un
metal de aportación que al fundirse se deposita en la junta de
las piezas a unir; o también sin metal de aportación, en cuyo
caso la unión se produce por la fusión de los metales en con-
tacto donde se juntan las dos piezas.
Como metal de aportación se puede emplear el mismo material del que están formadas las
piezas que se van a unir u otro diferente.
Básicamente existen dos tipos de soldadura: la soldadura blanda y la soldadura fuerte.
– La soldadura blanda es aquélla en la que se emplea un material de aportación de bajo
punto de fusión (estaño, estaño-plata) diferente del material de las piezas a unir, para que
éste no se deforme al calentarlo, lo que permite desoldarlas al calentarlas nuevamente. Un
ejemplo de soldaduras blandas son las que se emplean en fontanería, utilizando sopletes
de gas y las que se emplean en electrónica utilizando soldadores eléctricos.

– La soldadura fuerte es aquélla que necesita de altas temperaturas para que se produzca
la fusión de las piezas a soldar o de los materiales de aportación. Según el procedimien-
to utilizado, se distinguen dos tipos de soldadura fuerte: la oxiacetilénica y la soldadura
por arco eléctrico.
En la soldadura oxiacetilénica se utiliza un soplete en que se mezcla oxígeno y acetileno
para producir una llama de altísimo poder calorífico capaz de fundir metales de alto punto de
fusión.

14
-Pág.18-
U.D. 5.- RECURSOS CIENTÍFICOS Y TÉCNICOS

La soldadura por arco es la que utiliza como fuente calor el arco eléctrico que se forma entre
una varilla llamada electrodo, que además cumple las funciones de material de aportación, y las
piezas a soldar.

En este tipo de soldadura debemos tener muy en cuenta la seguridad del operario, por lo que
se ha de disponer de unas protecciones adecuadas para evitar las quemaduras.

2.2. UNIONES DESMONTABLES


Las uniones desmontables son aquéllas
que pueden desmontarse siempre que se
desee sin necesidad de destruir o deteriorar
los elementos que integran el objeto. Son
uniones desmontables las que se llevan a
cabo mediante tornillos, tuercas, abrazade-
ras, etc.

2.2.1. ELEMENTOS ROSCADOS


Los elementos roscados, como tornillos, espárragos, tuercas y tirafondos, son los que más
se utilizan para realizar uniones desmontables.
Los tornillos son cilindros parcial o totalmente roscados
provistos de una cabeza que puede tener diferentes formas.
Los espárragos son cilindros roscados sin cabeza. La unión
de piezas con tornillos y espárragos puede realizarse directa-
mente cuando éstas van provistas de agujeros roscados. Tam-
bién se unen piezas mediante la presión que se ejerce con un
tornillo y una tuerca, o con un espárrago que incorpore una tuerca en cada extremo.

15
-Pág.19-
TECNOLOGÍA

Los tirafondos son tornillos cónicos que se emplean básicamente para unir piezas de madera.

2.2.2. ABRAZADERAS Y BRIDAS


Las abrazaderas y las bridas son elementos muy uti-
lizados en la unión, sujeción y fijación de piezas. Existe
una gran variedad de formas según la función que tengan
que desempeñar.

TRANSMISIÓN, TRANSFORMACIÓN DE ESFUERZOS


3. Y MOVIMIENTOS
Los sistemas de transmisión tienen como objetivo llevar a los
diferentes elementos de una máquina la potencia y el movimien-
to producidos por un elemento motriz (motor) que es el que
genera el movimiento, de manera que la máquina pueda funcio-
nar y cumplir la finalidad para la que fue construida, llamando
elemento conducido, de que recibe el movimiento.
La transmisión de la fuerza y el movimiento producido por
un motor se realiza mediante cadenas cinemáticas, que son sis-
temas de elementos mecánicos convenientemente conectados
para transmitir la fuerza y el movimiento.

16
-Pág.20-
U.D. 5.- RECURSOS CIENTÍFICOS Y TÉCNICOS

Los elementos mecánicos más


empleados para transmitir la fuerza
y el movimiento a través de cadenas
cinemáticas, en las que normalmen-
te el movimiento que se transmite es
circular, son:
– Ruedas de fricción.
– Transmisiones por polea.
– Transmisiones por cadena.
– Engranajes.

Todos estos elementos mecánicos pueden ir montados


sobre los llamados ejes de transmisión o sobre árboles de
transmisión. Los ejes de transmisión son piezas cilíndricas,
generalmente de corta longitud, que sirven de soporte a pole-
as, ruedas de fricción, etc. Pueden ser fijos o moverse en sen-
tido giratorio, y no transmiten fuerza sino únicamente
movimiento. Los árboles de transmisión son piezas cilíndricas
más o menos largas que trabajan a torsión y flexión; son siempre giratorios y transmiten poten-
cia y movimiento. En la práctica se utiliza muchas veces la expresión “ejes de transmisión” para
designar tanto a los árboles de transmisión como a los ejes de transmisión propiamente dichos.
Generalmente, por necesidades de funcionamiento, las máquinas se construyen de forma
que la velocidad de giro o desplazamiento de la parte motriz no coincida con la velocidad de
giro o desplazamiento de la parte conducida.
Podemos ver a continuación los diferentes tipos de transmisión:

TIPOS DE TRANSFORMACIONES

Circular-Circular Circular-Rectilíneo Rectilíneo-Circular

Sistemas Sistemas de Piñón y Leva


de poleas engranajes cremallera
Biela y Piñón y
manivela cremallera
Piñón y Ruedas de Biela y Torno
cadena fricción manivela

Tornillo sin fin y rueda helicoidal Tornillo

17
-Pág.21-
TECNOLOGÍA

3.1. TRANSMISIÓN MEDIANTE RUEDAS DE FRICCIÓN


La transmisión de movimiento mediante ruedas de
fricción se realiza poniendo en contacto dos ruedas,
(la primera que es la motriz que arrastra a la otra (con-
ducida) mediante la fuerza que produce el rozamiento
entre ambas. Para poder transmitir el movimiento, las
ruedas han de estar en contacto ejerciendo una cierta
presión una sobre la otra. Así, al moverse bien una de
ellas arrastrará a la otra.
El sentido de giro de la rueda conducida es contrario al sentido de giro de la rueda motriz.
Por tanto, si queremos mantener el sentido de giro del motor, tendremos que emplear un núme-
ro impar de ruedas de fricción.
Las ruedas de fricción pueden ser
cilíndricas, cónicas o esféricas. Esto
permite transmitir el movimiento no
sólo entre ejes paralelos, sino también
entre ejes que se cortan o se cruzan en
el espacio.

Las ruedas de fricción tienen el gran inconveniente de no poder transmitir grandes poten-
cias, ya que pueden resbalar una sobre otra, con la consiguiente pérdida de velocidad. Otro de
los inconvenientes del uso de las ruedas de fricción es su desgaste, debido a que funcionan por
rozamiento y presión.

3.2. TRANSMISIÓN MEDIANTE POLEAS


Están formados por varias poleas que se transmiten el movimiento unas a otras a través de
la acción de una correa. Se utilizan para transmitir un movimiento de rotación entre ejes para-
lelos y distantes entre sí.

18
-Pág.22-
U.D. 5.- RECURSOS CIENTÍFICOS Y TÉCNICOS

Las correas utilizadas en este mecanismo pueden ser de distinta forma, dependiendo del tipo
de aplicación a la que se destinen. Las formas más utilizadas son la correa plana, la trapezoidal
y la dentada:

El factor más importante a la hora de elegir las poleas para un mecanismo o máquina es su
tamaño relativo.
Dependiendo del tamaño de la polea motriz y de la conducida, se consiguen velocidades de
giro distintas. El eje de la polea menor siempre gira más rápido que el eje de la mayor.
Para conseguir velocidades complicadas, o cuando la distancia entre los ejes que se quieren
conectar es excesivamente grande como para colocar dos poleas, con una sola correa se usan
los trenes de poleas, que consisten en varias poleas de distintos tamaños que giran a la vez y que
sirven de mecanismo intermedio para transmitir el movimiento al elemento final.

Relación de la velocidad (RV) en sistemas de poleas


Para facilitar la comprensión del cálculo de este parámetro se describe un caso concreto: la
polea motriz tiene un diámetro de 50 mm., con una velocidad de 500 RPM., y la polea condu-
cida, de 100 mm. La relación de velocidades se puede calcular aplicando la fórmula general de
la ley de transmisión.

Rueda motriz = Rueda conducida

Debemos tener en cuenta que en cada rueda influyen una serie de factores como son la velo-
cidad a que gira esta y la dimensión. Teniendo en cuenta estos factores la fórmula anterior queda
definitivamente así:
Nm Nm x Dm = Nc x Dc
Dc
50 x 500 = Nc x 100

Dm Nc 50 x 500
Nc = = 250 R.P.M.
100

19
-Pág.23-
TECNOLOGÍA

3.3. TRANSMISIÓN MEDIANTE ENGRANAJES


Están formados por varias ruedas dentadas, cuyos dientes engranan unos con otros. Se uti-
lizan para transmitir movimiento rotatorio entre distintas partes de una máquina.

A diferencia de los sistemas de poleas, en éstos no se necesita ningún elemento de enlace


(correas, cadenas) para transmitir el movimiento, ya que los dientes son los encargados de des-
empeñar esta función. Como en todo mecanismo, existirá un engranaje motriz que genera movi-
miento y otro conducido que lo recibe.
Una de las aplicaciones más interesantes de
este mecanismo es la variación de velocidad de
una parte de la máquina. Si se parte de un siste-
ma formado por dos engranajes, en el que uno de
ellos tiene menor número de dientes que el otro,
el eje conectado al engranaje menor siempre gira-
rá más rápido que el conectado al engranaje con
mayor número de dientes.
Generalmente, para desarrollar una función
dentro de la máquina no se utiliza un solo par de engranajes, sino que se agrupan varios pares
distintos. A este agrupamiento se le llama tren de engranajes. Se puede realizar la siguiente cla-
sificación:
Tren simple. Formado por dos, tres o más ruedas que engranan directamente entre sí, en un
mismo plano. En estos sistemas, se denomina engranaje intermedio o loco a aquél que se colo-
ca entre otros dos engranajes para conseguir que giren en el mismo sentido.

20
-Pág.24-
U.D. 5.- RECURSOS CIENTÍFICOS Y TÉCNICOS

Tren compuesto. Formado, como mínimo, por cua-


tro ruedas dentadas que engranan dos a dos. Los pares de
ruedas intermedias tienen el eje común y giran a la
misma velocidad. Los rozamientos disminuyen la efica-
cia de un sistema de engranajes pero se pueden minimi-
zar mediante lubricación.

Relación de la velocidad (RV) en sistemas de poleas


Para el estudio del parámetro RV, se toma el número de dientes como parámetro caracterís-
tico de este mecanismo.
Como en el caso de los sistemas de poleas, la relación de velocidades se puede calcular apli-
cando la fórmula general de la ley de transmisión. Se analiza a continuación un ejemplo con-
creto de un tren de engranajes simple formado por dos ruedas dentadas, donde el engranaje
motriz tiene 120 dientes y gira a una velocidad de 500 RPM. y el engranaje conducido tiene 40
dientes.

Nm x Zm = Nc x Zc
Nm
500 x 120 = Nc x 40

500 x 120
Nc = = 1.500 r.p.m.
40

Zc
Nc Zm

Estas velocidades se expresan en revoluciones por minuto (r.p.m.).

3.4. TRANSMISIÓN MEDIANTE CADENA Y PIÑONES


Está formado por dos ruedas dentadas (también llamadas piñones) que se transmiten el
movimiento a través de una cadena. Se utilizan para transmitir un movimiento de rotación entre
ejes paralelos y distantes también entre sí.

21
-Pág.25-
TECNOLOGÍA

Símbolo gráfico

Este mecanismo sustituye a los sistemas de poleas cuando se necesita transmitir un esfuer-
zo muy grande y se desea evitar un deslizamiento o rotura de la correa. La cadena está forma-
da por eslabones que se van engranando progresivamente en los dientes de las ruedas en
movimiento, evitando así el deslizamiento. Se utilizan en bicicletas, ciclomotores, automóviles,
puertas de garajes, etc.
En este sistema se calcula la relación de velocidades (RV) como en el caso de los engranajes.

3.5. TRANSMISIÓN MEDIANTE SIN FIN Y RUEDA HELICOIDAL


Este mecanismo está formado por un tornillo sin fin, de uno o varios hilos o entradas de
rosca, y una rueda dentada helicoidal. Se utilizan para transmitir un movimiento giratorio entre
dos ejes que forman un ángulo de 90º.

Rueda motriz

Rueda conducida

El tornillo realiza un movimiento de rotación alrededor de su eje, lo que provoca que su


rosca impulse los dientes de la rueda helicoidal, para que gire.
La característica más importante de este mecanismo es que puede conseguir una relación de
velocidades muy alta. Su funcionamiento es muy silencioso. Las aplicaciones más importantes
son los cuentavueltas, los reductores de velocidad, el accionamiento de grandes válvulas, etc.

22
-Pág.26-
U.D. 5.- RECURSOS CIENTÍFICOS Y TÉCNICOS

Como el tornillo sin fin es el elemento motriz, la expresión para calcular la relación de velo-
cidades (RV) tiene la forma:

Nm x Hr = Nc x Zc

Para mayor sencillez, se supone que el tornillo sin fin solamente tiene un hilo de rosca (Hr),
aunque hay que tener en cuenta que los hay con más de uno.

3.6. TRANSMISIÓN MEDIANTE PIÑÓN Y CREMALLERA


Este mecanismo está formado por una barra dentada (cremallera) que se desplaza longitudi-
nalmente arrastrada por un piñón dentado que gira. Se utiliza para transformar un movimiento
circular en otro rectilíneo o viceversa.

3.7. TRANSMISIÓN MEDIANTE BIELA Y MANIVELA


Es un mecanismo utilizado para realizar la transformación de un movimiento circular en
otro rectilíneo alternativo (vaivén) o viceversa. La disposición de sus elementos es la siguiente:

La biela es una barra que se articula con elementos giratorios entre la manivela y el patín. La
cabeza de la biela describe una circunferencia con movimiento continuo uniforme, mientras que el
pie articulado con el patín se desplaza guiado en un movimiento rectilíneo alternativo de velocidad
variable. Los puntos inicial y final de este movimiento se denominan puntos muertos, y la distan-
cia entre ellos, carrera del patín, que es el doble de la longitud entre los ejes de la manivela.

23
-Pág.27-
TECNOLOGÍA

El elemento motriz puede ser la manivela o el patín: si el patín realiza un movimiento line-
al alternativo, obliga a la manivela a realizar un movimiento circular. En este caso la biela trans-
mite el movimiento a la manivela.
Para ciertas aplicaciones de este mecanismo se sustituye la manivela por un cigüeñal (con-
junto de manivelas montadas sobre un mismo eje).
Las aplicaciones más importantes son: motores de combustión, máquina de vapor, compre-
sores, bombas, sierra alternativa, etc.

4. CIRCUITOS Y SUS ELEMENTOS


La mayoría de las máquinas y aparatos que usamos a diario, como el ordenador, teléfono,
automóvil, etc., disponen en su interior de circuitos eléctricos, electrónicos y neumáticos, que
se ocupan de controlar los diferentes mecanismos o de transformar la energía que reciben para
producir el movimiento, imágenes o sonido.
Esta gran variedad de aparatos, máquinas y sistemas lleva consigo una enorme diversidad
de configuraciones, aunque hay pocos elementos básicos que aparecen en casi todos los circui-
tos. Vamos a identificar estos elementos para conocer su función en cada unos de los distintos
tipos de circuitos (eléctrico, electrónico y neumático).
Tenemos que tener en cuenta que cada uno de los circuitos anteriormente citados, están
montados y unidos entre sí formando instalaciones.
Una instalación es una serie de elementos, generalmente fijos y relacionados entre sí, que
tienen una misión específica que cumplir. Si dicha misión es la obtención final del objeto téc-
nico, la instalación se denomina instalación principal, y si es sólo de ayuda, aunque ésta sea
imprescindible, se llama instalación auxiliar.

INSTALACIONES EN...
VIVIENDAS TALLERES AUTOMÓVIL
Mobiliario Máquinas-herramientas Motor
Ajuar Baños de cromado Encendido
Eléctrica Producción de vapor Carburación
Sanitaria Eléctrica Frenado
Calefacción Depuración de residuos Alumbrado
Etcétera Aire comprimido Refrigeración
Combustibles Etcétera
Etcétera

24
-Pág.28-
U.D. 5.- RECURSOS CIENTÍFICOS Y TÉCNICOS

Los elementos de las instalaciones son de dos tipos:


Elementos pasivos. Tienen la función de transporte, regulación, maniobra, control de las
condiciones de utilización, etcétera, sin transformar energía.
Elementos activos. Tienen un carácter dinámico (mueven los equipos, hacen circular los
fluidos, etcétera). Son transformadores de energía, es decir, toman una forma de energía dispo-
nible en ese punto y momento (eléctrica, mecánica, térmica...) y la transforman en otra utiliza-
ble para el fin que se persigue (mecánica, química, luminosa...).
Los elementos activos de mayor importancia tecnológica, por la extensión de su uso y por
las facilidades de utilización, son los intertransformadores de mecánica y electricidad, es decir,
motores y generadores eléctricos.

Las instalaciones contienen elementos pasivos, como los elementos de regulación


y control, y elementos activos, como los motores y generadores.

4.1. CIRCUITOS ELÉCTRICOS


Los fenómenos eléctricos han causado asom-
bro y extrañeza desde tiempos remotos. Aunque no
ofrecían ninguna utilidad, no pasaban de ser meras
curiosidades. En 1832, Faraday descubrió un modo
sencillo de generar electricidad: bastaba con mover
un conductor en un campo magnético para que
apareciera una corriente eléctrica. Este descubri-
miento permitió el desarrollo y la aplicación gene-
ralizada de la electricidad en todos los ámbitos.
La electricidad constituye una de las fuentes energéticas más extendidas. Las muchas apli-
caciones de la energía eléctrica se deben a su fácil transformación.
Energía térmica. El paso de corriente eléctrica a través de un conductor produce calenta-
miento en el citado conductor.
Energía mecánica. Si una corriente eléctrica pasa por un conductor arrollado que se
encuentra entre imanes, el conductor comienza a girar.
Energía luminosa. Cuando el conductor por el que pasa la corriente eléctrica es largo y muy
fino, y además resiste altas temperaturas, es capaz de producir una luz intensa.
Energía química. La corriente eléctrica puede descomponer ciertas sustancias.

25
-Pág.29-
TECNOLOGÍA

Según la aplicación que se vaya a controlar con la electricidad, existen dos tipos de corrientes:
Corriente continua. En este tipo de corriente, el flujo o circulación de los electrones siem-
pre tiene el mismo sentido. Además, la cantidad de electrones que pasan por un punto determi-
nado del circuito es constante.

Corriente alterna. Los electrones circulan alternativamente en uno y otro sentido, varian-
do la cantidad de ellos al mismo tiempo. Se denomina frecuencia al número de veces por segun-
do en que la corriente cambia de sentido.

4.1.1. COMPONENTES DE LOS CIRCUITOS ELÉCTRICOS


Para la realización de los circuitos eléctricos disponemos de una gran variedad de elemen-
tos o componentes que se diferencian entre sí tanto por su función como por sus características
(tensión de funcionamiento, valor de resistencia, potencia de consumo, tipo de corriente, etc.).
En las siguientes figuras puedes reconocer algunos de los componentes eléctricos más usa-
dos, así como el símbolo utilizado para cada uno en los esquemas de circuito.

26
-Pág.30-
U.D. 5.- RECURSOS CIENTÍFICOS Y TÉCNICOS

Generadores: las pilas, las baterías o las Conductores y elementos de conexión:


dinamos proporcionan energía eléctrica en los conductores transportan con facilidad la
forma de corriente continua. En las bases de corriente. Su espesor depende de la intensidad
enchufe se recibe la energía eléctrica, en de corriente que vaya a pasar por ellos.
forma de corriente alterna, procedente de las
Los elementos de conexión evitan riesgos
centrales de producción eléctrica.
en la manipulación de los circuitos.

Elementos de maniobra y protección: Receptores: transforman la energía eléc-


los interruptores, pulsadores o relés se ocupan trica en movimiento, luz, calor, etc. Entre
del cierre y apertura de los circuitos. ellos se encuentran los motores eléctricos, las
lámparas, las resistencias o los electroimanes.
Los fusibles protegen el circuito contra
subidas de intensidad de corriente que podrí- Según su potencia de uso, necesitan de
an quemar los conductores. más o menos intensidad de corriente para fun-
cionar.

27
-Pág.31-
TECNOLOGÍA

4.2. CIRCUITOS ELECTRÓNICOS


Una de las ramas de la ciencia que mayor impacto ha tenido en el desarrollo de la sociedad
ha sido la electrónica. Gracias a ella se pueden realizar de forma automática procesos muy com-
plejos utilizando pequeños circuitos.
Cualquier circuito electrónico se representa mediante esquemas en los que intervienen los
símbolos de los diferentes componentes. Cada uno de ellos tiene unas características de funcio-
namiento que lo hacen apto para determinadas funciones. A partir de los esquemas, se montan
los circuitos utilizando diferentes herramientas. Una vez montados, debe verificarse su funcio-
namiento mediante aparatos de medida adecuados.

La electrónica es el campo de la técnica que se ocupa de aquellos sistemas


en los que tiene lugar la conducción de electrones a través del vacío de un gas
o de un material semiconductor.

Algunos de estos componentes son conocidos porque también intervienen en los circuitos
eléctricos. Otros son específicos de los circuitos electrónicos.
La mayoría de los circuitos electrónicos están formados por un gran número de componen-
tes. Según sus características, construcción o funcionamiento, se pueden distinguir dos grandes
grupos: pasivos y semiconductores.
Los componentes pasivos. Están constituidos por diversos materiales, como el cobre, el
acero y el carbón. Presentan unas características particulares que los hacen aptos para determi-
nados valores de tensión e intensidad de la corriente. Entre ellos destacan las resistencias y los
condensadores.
Los componentes semiconductores. Están constituidos por materiales como el selenio, el
germanio y el silicio, que se comportan como conductores eléctricos en determinadas condi-
ciones.

4.2.1. COMPONENTES DE LOS CIRCUITOS ELECTRÓNICOS


La electrónica estudia los electrones y los componentes por los que circulan así como las
aplicaciones de los elementos y equipos que forman dichos componentes. Los elementos elec-
trónicos son muy variados, aunque los más utilizados en los circuitos son las resistencias, los
condensadores y las inductancias o bobinas.
Las resistencias ofrecen cierta oposición a la circulación de electrones, dicha dificultad se
mide en ohmios (Ω). Se construyen con materiales muy diversos: cerámica, carbón, etc.
a) Las resistencias fijas tienen un valor óhmico determinado, prácticamente constante. Son
ejemplos las bombillas, los calentadores, etc.

28
-Pág.32-
U.D. 5.- RECURSOS CIENTÍFICOS Y TÉCNICOS

b) Las resistencias variables pueden modificar su valor aumentándolo o disminuyéndolo


por diversas causas:

Los potenciómetros o Las resistencias LDR o Las resistencias NTC y


reostatos son tipos de resis- fotorresistencias varían su PTC o termistores varían su
tencias cuyo valor se puede valor óhmico con el grado de valor con el aumento de la
regular accionando manual- intensidad luminosa disminu- temperatura. En las primeras
mente un cursor. yendo con el aumento de luz. disminuye y en las segundas
aumenta.

Los condensadores son componentes que almacenan cargas


eléctricas al aplicarles una diferencia de potencial entre sus contac-
tos (periodo de carga) y que ceden en el instante en que se los
conecta a un circuito o elemento (periodo de descarga). Su capaci-
dad de almacenamiento de carga eléctrica también puede ser fija o
variable. Los condensadores pueden construirse de papel, aire,
electrolíticos, de poliéster, cerámicos, etc.

29
-Pág.33-
TECNOLOGÍA

Las inductancias o bobinas están formadas por un hilo con-


ductor esmaltado arrollado en hélice. A cada vuelta de hilo se le
llama espira. Cuando circula corriente por el hilo se produce un
campo magnético, resultado del efecto magnético creado por cada
una de las espiras. Para aumentar el campo magnético creado se
introduce en su interior un núcleo de hierro dulce.
Los diodos son componentes electrónicos que dejan pasar la corriente eléctrica en un senti-
do y no en el contrario. Por ello, el diodo puede rectificar, es decir, producir una corriente con-
tinua a partir de una alterna. Los diodos LED se caracterizan por emitir luz cuando se polarizan
de forma correcta. Se utilizan como pilotos de señalización, habiendo sustituido a las bombillas
como indicadores.

Los transistores son componentes semiconductores con tres


terminales formados por dos uniones P-N. Los terminales que
están conectados en los extremos se llaman emisor y colector, y
el que está conectado a la zona central se denomina base. Se dice
que es un elemento amplificador porque controla una intensidad
de corriente varias veces superior a la que lo activa.
Actualmente, la tecnología electrónica permite fabricar cir-
cuitos de dimensiones microscópicas, formados con transistores
y otros elementos, sobre una placa de material semiconductor de
tamaño variable, que no excede de unos pocos milímetros. A esta
placa se la conoce con el nombre de circuito integrado o micro-
chip.
La utilización de este tipo de elementos va desde los circui-
tos electrónicos con amplificadores, filtros y conmutadores hasta
las memorias de ordenadores y microprocesadores.

30
-Pág.34-
U.D. 5.- RECURSOS CIENTÍFICOS Y TÉCNICOS

4.3. CIRCUITOS NEUMÁTICOS


Los términos neumático y neumática provienen de la palabra griega «pneuma», que signifi-
ca aliento o soplo.
En la vida cotidiana podemos encontrar multitud de objetos que, de una forma u otra, depen-
den del aire para su funcionamiento. Así, una trompeta o un silbato producen sonido cuando
pasa el aire a través de ellos, los barcos de vela son impulsados por la fuerza del viento, las rue-
das de las bicicletas llevan cámaras con aire en su interior para amortiguar los golpes que se pro-
ducen al rodar debido a las irregularidades el suelo, las pelotas botan gracias al aire que
contienen en su interior. etc.
Llamamos instalaciones neumáticas al conjunto de máquinas y aparatos que funcionan uti-
lizando como energía la que les proporciona el aire comprimido. Cuando hablamos de neumá-
tica nos estamos refiriendo a aquellas aplicaciones prácticas del aire comprimido que se llevan
a cabo mediante instalaciones neumáticas.

4.3.1. COMPONENTES DE LOS CIRCUITOS NEUMÁTICOS


El aire a presión se produce con relativa facilidad, es limpio, fácil de almacenar y se mani-
pula con escaso riesgo. Para usar el aire comprimido se precisa un circuito compuesto de:
– generador (compresor),
– conductores (tuberías),
– elementos de control de paso de aire (válvulas y distribuidores),
– actuadores que utilizan la energía.

El compresor es una máquina capaz de producir aire comprimido aprovechando fuentes de


energía tan diversas como la fuerza muscular, la electricidad o la energía que proporciona los
motores de combustión interna. En otras palabras, un compresor es una máquina capaz de trans-
formar diferentes energías en energía neumática. En las instalaciones neumáticas se utilizan
compresores para producir y almacenar aire comprimido, esto último con ayuda del depósito.

31
-Pág.35-
TECNOLOGÍA

Básicamente existen dos tipos de compresores:


Los compresores fijos sirven para alimentar circuitos
neumáticos complejos, como los utilizados por industrias y
talleres que emplean el aire comprimido para el acciona-
miento de circuitos neumáticos y para el accionamiento de
máquinas y herramientas neumáticas, como por ejemplo
destornilladores y taladros. Los compresores fijos son
accionados por motores eléctricos.

Los compresores portátiles suministran aire comprimi-


do a los equipos neumáticos móviles: martillos neumáticos,
equipos de pintura, etc. Aunque existen compresores portá-
tiles que se accionan eléctricamente, generalmente son
accionados por motores de explosión, lo que permite utili-
zarlos en lugares adonde no llega el suministro eléctrico.

Las válvulas son los elementos que contro-


lan el paso del aire, regulan y distribuyen el aire
que recibe el cilindro. Existen diferentes tipos
según la misión que vayan a desempeñar; las
principales son las válvulas distribuidoras como
se muestra en esta figura en la cual vemos el
funcionamiento.

Los actuadores o cilindros son


los más representativos de los circui-
tos neumáticos; se encargan de trans-
formar la energía neumática en
energía mecánica. Según el movi-
miento a realizar, éstos pueden ser
lineales o giratorios, según la fuerza a
realizar en el mecanismo, pueden ser
de simple o doble efecto.

32
-Pág.36-
U.D. 5.- RECURSOS CIENTÍFICOS Y TÉCNICOS

5. LOS MATERIALES
Una de las primeras decisiones que hemos de tomar cuando diseñamos y construimos un
producto consiste en elegir el material o materiales con los que realizaremos cada una de sus
piezas.
Para utilizar el más apropiado en cada caso, debemos conocer los distintos tipos de mate-
riales, sus propiedades, características y aplicaciones más usuales. De igual manera, hemos de
saber dónde se adquieren, en qué formas y tamaños se presentan y con qué herramientas y téc-
nicas se trabajan.
En las tiendas de bricolaje, almacenes de plásticos o de madera, mercerías o ferreterías
encontraremos la mayor parte de estos materiales, herramientas y utensilios necesarios para rea-
lizar los objetos proyectados.

5.1. TIPOLOGÍA DE LOS MATERIALES Y SU CLASIFICACIÓN


Se denominan materiales las sustancias de que están hechas las cosas. En tecnología, los
materiales son los elementos de construcción de los objetos técnicos. Así, se dice que un vaso
es de vidrio, una casa es de piedra o unas botas son de cuero.
La elección de los materiales para la fabricación o construcción de un objeto viene determi-
nada por factores de funcionalidad y economía. Algunos de los requisitos que debe cumplir el
material seleccionado son:
1. Debe ser adecuado para los fines a los que se destina.
2. Debe responder a la duración que de él se espera.
3. Cuando el objeto técnico quede inservible, sus materiales se deben poder reciclar o eli-
minar sin riesgos.
4. Debe cumplir las normas de seguridad e higiene vigentes en su fabricación y utilización.
5. Ha de tener un precio asequible (para que sus usuarios lo puedan comprar) y competiti-
vo (para que se pueda vender).
En general, las ventajas de su utilización han de ser superiores a los inconvenientes que
genera.
Todos los productos están hechos con materiales que proceden de la naturaleza. Los mate-
riales se pueden clasificar atendiendo a su origen:

33
-Pág.37-
TECNOLOGÍA

• Materiales de origen mineral. Son los que proceden de la corteza terrestre. Inicialmen-
te es necesario extraer las rocas y los minerales. Después se transforman para obtener el
material que interesa. Dentro de este tipo de materiales están el aluminio, el hierro, el
cobre, el oro, el granito, el mármol.
• Materiales de origen animal. En general, se obtienen de diversas partes del cuerpo de
ciertos animales o son producidos por ellos. Son importantes la lana de oveja, el cuero, la
seda, etc.
• Materiales de origen vegetal. Provienen de los árboles y las plantas. Destacan la made-
ra, el algodón, el caucho, etc.

Origen mineral Origen animal Origen vegetal

5.2. PROPIEDADES FÍSICAS DE LOS MATERIALES


Y SUS APLICACIONES
Son propiedades físicas aquellas que afectan al estado y las circunstancias del material: por
ejemplo, la resistencia mecánica ante los esfuerzos, las cualidades ópticas o el comportamiento
ante la electricidad o el calor. Las propiedades químicas afectan a su composición y de ellas
dependen la resistencia a la corrosión y a la acción de los disolventes, su toxicidad para las per-
sonas o el medio ambiente, etcétera.

PROPIEDADES DE LOS MATERIALES


FÍSICAS QUÍMICAS
Mecánicas Electromagnéticas Térmicas Solubilidad Resistencia Toxicidad
Elasticidad Resistividad Conductividad Agua La luz Personas
Dureza Permeabilidad Dilatación Gasolina La oxidación Animales
Fragilidad Punto de fusión El calor Plantas

34
-Pág.38-
U.D. 5.- RECURSOS CIENTÍFICOS Y TÉCNICOS

5.2.1. PROPIEDADES QUÍMICAS


Los materiales poseen unas propiedades químicas que pueden cambiar mediante reacciones
químicas con los sólidos, líquidos y gases que ellos mismos contienen, o con el aire y el agua y
los posibles contaminantes que éstos arrastren. También, por la acción de organismos vivos.

Los objetos técnicos se encuentran en un medio ambiente con factores físicos (radiación
ultravioleta, temperatura...), químicos (aire, agua, ácidos, disolventes) o biológicos (bacterias,
levaduras, hongos...) que afectan a la composición química de sus materiales y, por tanto, a sus
propiedades.

a) Solubilidad de los materiales. Los materiales sólidos pueden perder su estructura por
disolución en líquidos que, de forma permanente o accidental, se pongan en contacto
con ellos. El líquido más frecuente es el agua.

– Algunos materiales, como el mármol y las calizas, son casi insolubles en agua pura,
pero en las zonas industriales se disuelven en el agua de lluvia de manera alarmante
debido a la cantidad de ácidos disueltos que ésta contiene.

– Los plásticos y los productos de caucho natural son solubles en gasolina y disolven-
tes para pinturas, mientras que el neopreno y las siliconas resisten a estos disolven-
tes.

b) Toxicidad. Si un material es tóxico para sus posibles usuarios o agresivo con el medio
ambiente, se debe buscar un producto alternativo que no resulte peligroso. Por ejemplo,
una pintura que contiene compuestos de plomo puede utilizarse para la imprimación
antioxidante de una estructura de hierro, pero nunca debe utilizarse para colorear un
juguete; de igual modo, un pegamento industrial para revestimientos plásticos no debe
utilizarse como adhesivo doméstico o escolar.

c) Degradación de materiales. Un objetivo de fabricación es conseguir productos dura-


deros. Para ello es preciso que sus materiales sean resistentes a factores como, por ejem-
plo, la oxidación, que, en términos generales, es el proceso químico por el cual muchos
metales pierden electrones de forma natural y forman óxidos o sales (sulfuros, sulfatos,
carbonatos...).

5.2.2. PROPIEDADES TÉRMICAS


Al elegir los materiales para la elaboración de un objeto técnico se han de tener en cuenta
características como la capacidad de transmisión mediante calor y la variación de sus propie-
dades físicas y químicas con la temperatura.

35
-Pág.39-
TECNOLOGÍA

Materiales metálicos Materiales no metálicos

Si el átomo A de un metal se pone a vibrar Para los no metálicos, el enlace largo


como consecuencia del calentamiento, choca retrasa y amortigua la vibración de A de
inmediatamente con los que le rodean y los manera que Z no vibra hasta transcurrido un
hace oscilar. La vibración alcanza rápidamen- tiempo. Para entonces la vibración de A puede
te al átomo Z en la periferia. Todas las oscila- ser tan grande que esa parte de la lámina ya se
ciones son similares y el cuerpo se dilata de haya fundido, descompuesto, y el cuerpo se
un modo uniforme. agrieta o se rompe.

Los metales conducen bien el calor y se dilatan uniformemente, en cambio los


materiales no metálicos conducen mal el calor y se dilatan de forma no uniforme,
lo que ocasiona grietas y/o rupturas.

5.2.3. PROPIEDADES MECÁNICAS


Las propiedades mecánicas de los materiales (resistencia al desgaste, elasticidad, tenacidad,
capacidades de soportar cargas sin deformarse...) no son independientes entre sí, ya que todas
dependen de la estructura del material. En general, los materiales duros son frágiles, mientras
que los deformables tienen un punto de fusión muy bajo y poca resistencia al desgaste, etc.
Cuando esto ocurre, se elaboran los objetos con una asociación de materiales; el hormigón
armado y los nuevos materiales de fibra de carbono asociada con poliéster son los más signifi-
cativos.

36
-Pág.40-
U.D. 5.- RECURSOS CIENTÍFICOS Y TÉCNICOS

A veces, la asociación de propiedades deseada puede lograrse sobre un único material dotan-
do a la superficie de unas propiedades y reservando el interior para las complementarias. Esta
práctica se denomina tratamiento superficial y puede lograrse por modificación local de la
estructura del material mediante tratamientos térmicos (temple, revenido, recocido...) o quími-
cos (cianuración, nitruración, cementación...).
A la hora de elegir un material y decidir cómo utilizarlo hay que tener en cuenta, además,
dos características importantes:
• La fatiga del material. Si los esfuerzos que ha de soportar no son permanentes sino que
cambian constantemente de sentido, su resistencia disminuye mucho. Un ejemplo clari-
ficador de este concepto es la fácil rotura de una varilla de hierro después de haberla
doblado sucesivamente de un lado a otro.
• La isotropía. Un material es isótropo si tiene las mismas propiedades en todas las direc-
ciones y anisótropo si las propiedades varían. El acero es isótropo, mientras que la made-
ra es anisótropa, pues, por ejemplo, su resistencia a la tracción es mucho mayor en la
dirección de la fibra que en sentido perpendicular.

5.2.4. DUREZA
La idea de dureza se entiende de forma diferente según el tipo de material:
• La dureza de los minerales (mármol, yeso, cuarzo, etc.) es la resistencia de un mineral a
ser rayado por otro.
• La dureza de los productos metalúrgicos es la resistencia que opone un material a dejar-
se penetrar por otro.

37
-Pág.41-
TECNOLOGÍA

La dureza de los materiales condiciona muchas cualidades mecánicas. La más significativa


es la resistencia al desgaste. Por esta razón se elaboran con materiales duros los ejes, piñones y
ruedas de mecanismos, bolas de rodamiento, herramientas de corte, etc.
Los metales duros tienen elevado punto de fusión y son bastante resistentes a la oxidación,
pero suelen ser muy frágiles, y por ello se recurre al endurecimiento superficial de los materia-
les más resistentes mediante tratamientos térmicos como el temple o la nitruración.

5.2.5. ELASTICIDAD
No hay ningún material que sea totalmente rígido. Todos se deforman en mayor o menor
proporción ante una fuerza exterior. Puede suceder que el material recupere su forma y dimen-
siones cuando cese la fuerza o bien que quede deformado permanentemente.

Se llama elasticidad a la propiedad que tienen los materiales de recuperar su forma


inicial cuando cesa la fuerza que los deforma.

La propiedad contraria es la plasticidad, o capacidad de los materiales de sufrir deforma-


ciones permanentes al cesar las fuerzas que las producen.
Todos los materiales son elásticos dentro de un rango de fuerzas, y plásticos cuando las fuer-
zas superan dicho valor. Ocurre en ocasiones que el material se rompe antes de que estas defor-
maciones alcancen valores significativos, pero eso no quiere decir que no se produzcan.

5.3. INDUSTRIAS DERIVADAS


El progreso tecnológico de la humanidad ha permitido fabricar desde hachas de piedra a
chips de microprocesadores. La investigación técnica y científica a lo largo de los siglos ha lle-
vado al hombre al descubrimiento y obtención de los materiales que ha ido necesitando.
Curiosamente, estos artículos, tan dispares en cuanto a su elaboración, no están demasiado
lejos de su origen y para unos materiales básicos los que mueven los materiales de todos los
productos que se vayan a fabricar.

5.3.1. LA MADERA
La madera es uno de los primeros materiales usados por el ser humano. Le ha servido para
la construcción de sus viviendas, como combustible y para fabricar diversos útiles. Actualmen-
te se sigue utilizando en la fabricación de mobiliario, embalajes, etc.

38
-Pág.42-
U.D. 5.- RECURSOS CIENTÍFICOS Y TÉCNICOS

La madera se obtiene de los árboles y los arbustos. Tiene por tanto una naturaleza orgánica
con una estructura en fibras, compuestas fundamentalmente de celulosa. Ésta es la causa de que
tenga una duración menor que otros materiales, ya que puede ser destruida por el fuego o ata-
cada por hongos, microorganismos o insectos. Para aumentar la vida útil de la madera, se la
somete a diversos tratamientos químicos.
Un corte transversal al tronco de un árbol permite observar en la sección las siguientes
capas:

Corteza. Corresponde a la Médula. Es la parte central


capa exterior. Suele estar del árbol y está constituida
reseca y tiene una textura por un tejido poroso. El tama-
rugosa. Se encarga de prote- ño de esta zona va disminu-
ger el árbol contra el ataque yendo al envejecer el árbol.
de agentes externos.

Albura. Es la madera de for- Duramen. Es la madera de la


mación más reciente. Se parte interior del tronco. Pre-
encuentra en la región externa senta una coloración más
del tronco, bajo la corteza, y oscura que la del exterior y
su coloración es más clara también son mayores su resis-
que la del duramen. La for- tencia mecánica y su durabili-
man un número variable de dad. Las capas de la zona
los últimos anillos de creci- exterior se convierten en dura-
miento. men con el paso del tiempo.

En general, es la albura y el duramen lo que se aprovecha para obtener lo que conocemos


como madera natural. La corteza sólo se aprovecha en un determinado tipo de árboles, como
por ejemplo en el alcornoque, del que se obtiene el corcho.

La madera es un material de origen vegetal y de naturaleza orgánica.


Se compone fundamentalmente de celulosa.

Propiedades de la madera
Las propiedades de la madera varían según el proceso de fabricación. Posee ventajas respecto
a otros materiales, como, por ejemplo, la facilidad con que se obtiene y se labra y su baja densidad.
También presenta algunos inconvenientes, como la fácil combustibilidad y la poca resisten-
cia a la agresión de agentes externos, tales como hongos e insectos, los cuales pueden llegar a
destruirla.

39
-Pág.43-
TECNOLOGÍA

Las propiedades de la madera están determinadas por su naturaleza fibrilar:


– Dureza. Es la resistencia que presenta a ser rayada. Esta propiedad está estrechamente
relacionada con la densidad. En general, cuanto más densa es una madera, mayor dureza
presenta. El duramen es la parte más dura.
– Resistencia mecánica. Es la resistencia a romperse que presenta al aplicarle una deter-
minada carga. Esta propiedad depende de si el esfuerzo se realiza en la dirección de las
fibras o en sentido opuesto:
a) La resistencia ante un esfuerzo de tracción es mayor si se hace en el sentido de las
fibras que si se realiza en otro sentido.
b) En cambio, la resistencia a la flexión es mayor si la fuerza es aplicada en el sentido
perpendicular a las fibras.
– Conductividad térmica y eléctrica. Es la capacidad para conducir el calor y la electrici-
dad. Las fibras que componen la madera están huecas y en su interior se alojan pequeñas
burbujas de aire. Debido a esto, la madera se comporta como un buen aislante térmico y
eléctrico (siempre que esté seca).
– Retracción higroscópica. La madera tiene la capacidad de absorber el agua. Es una de
las propiedades que más influye en su comportamiento. En la madera recién cortada, la
humedad es mayor en su interior que en el exterior, debido a la alimentación de la planta
y a la menor evaporación en su interior.
– Deformabilidad. Permite modificar su forma al aumentar su grado de humedad y poste-
riormente someterla a presión y calor en un molde dado.

Transformación de la madera. Obtención de tableros


La industria de la madera comprende, en un sentido amplio, la transformación del tronco y
de las ramas del árbol en productos directamente consumibles por el hombre.
– El desenrollado de la madera se efectúa utilizando máquinas de serrar de cuchilla para
obtener hojas delgadas que mediante el uso de tornos y vapor se transforman en la chapa
continua.
– El aserrado de troncos se efectúa mediante sierras que pueden ser de movimiento rectilí-
neo (cinta) o circular (disco). Permite obtener diversos listones y tablas.
– El armado de listones consiste en unirlos mediante adhesivos para conseguir otros de
mayor resistencia mecánica.
– El prensado de la madera se efectúa en una o dos direcciones perpendiculares a las fibras
con prensas o mediante martilleo.

40
-Pág.44-
U.D. 5.- RECURSOS CIENTÍFICOS Y TÉCNICOS

– El triturado consiste en desmenuzar la madera en partículas muy pequeñas. Estas partí-


culas se mezclan con aditivos y se unen mediante adhesivos. Sometidas posteriormente a
altas presiones, se consiguen los tableros aglomerados.

Fabricación de papel
El papel está formado de fibras de celulosa entrelazadas que proceden de la madera. Ini-
cialmente se descorteza y tritura la madera, obteniéndose la pasta mecánica. También se puede
desmenuzar la madera mediante procesos químicos, obteniéndose la pasta química. General-
mente, se mezclan las
dos pastas y se amasa
todo el producto con
un gran volumen de
agua para separar la
celulosa. En el paso
siguiente, se añaden
aditivos químicos,
entre ellos el cloro,
para blanquear la
pasta. Por último, se
extiende en forma de
película delgada, se
prensa y se seca. El
resultado es el papel.

41
-Pág.45-
TECNOLOGÍA

5.3.2. LOS METALES


Los metales pertenecen al grupo de los materiales de origen mineral. Se encuentran en la
corteza terrestre y para su utilización es necesaria una extracción y generalmente una posterior
manipulación.
Los metales pueden presentarse en estado puro o combinados con otros elementos. Se deno-
mina materiales metálicos tanto a los elementos metálicos como a sus mezclas. Poseen un bri-
llo particular y unas propiedades mecánicas, eléctricas y térmicas características.
Las mezclas homogéneas de dos o más metales se llaman aleaciones. Se obtienen fundien-
do los metales en un crisol.

5.3.2.1. Férricos
Los metales férricos son aquellos cuyo componente principal es el hierro. Éste es de color
blanco azulado cuando está pulido y limpio. Al poco tiempo de bruñirlo (pulirlo), el hierro se
oxida, cubriéndose con una capa de óxido, color tierra, que penetra poco a poco hasta el inte-
rior y acaba por afectar a todo el material.
El hierro puro es un metal con una densidad de 7,8 g/cm3. Tiene escasa utilidad tecnológi-
ca, por ser frágil y quebradizo. El hierro deja de ser quebradizo si se combina con pequeñas can-
tidades de carbono, que lo hacen más duro y tenaz. Las llamadas fundiciones presentan un
contenido en carbono que varía entre 2 % y 6,7 %. La fundición es una aleación muy frágil y
poco dúctil y maleable.

42
-Pág.46-
U.D. 5.- RECURSOS CIENTÍFICOS Y TÉCNICOS

Se adapta muy bien a cualquier forma en un molde y es un material muy duro que resiste
muy bien el desgaste, de ahí que se utilice en zapatas de freno, carcasas o envolventes de moto-
res, bancadas de máquinas, bastidores de herramientas, farolas, tapas de alcantarillado, etc.

5.3.2.2. No férricos
Los metales no férricos son aquellos que poseen un componente principal distinto al hierro,
incluso pueden no contener nada de éste. Destacan por su importancia el cobre, el aluminio, el
estaño, el plomo, el latón, el bronce, el cinc, la plata, el oro, etc.
– Cobre. Es un metal de color rojizo. Tiene una alta conductividad eléctrica y térmica,
buena resistencia a la corrosión y una densidad alta (8,9 g/cm3). Es dúctil y maleable, por
lo que se trabaja con facilidad. Se comercializa en forma de alambres e hilos (conducto-
res eléctricos), varillas, chapas y tubos (fontanería).
– Aluminio. Presenta un color gris plateado brillante. Tiene una densidad pequeña (2,7
g/cm3) y una gran resistencia a la corrosión. Es buen conductor eléctrico y térmico, es
blando y se trabaja con facilidad. Se comercializa en forma de chapas (botes de refresco),
perfiles (ventanas), tubos, papel para envolver, etc.

– Estaño. Tiene un color blanco brillante. Es un buen conductor, es resistente a la corrosión


y se trabaja con facilidad al ser relativamente blando y con bajo punto de fusión (232ºC).
Se comercializa en forma de chapas, hojas (para envoltorios de productos), hilos y vari-
llas (para soldadura).
– Plomo. Presenta un color blanco grisáceo. Sus propiedades más importantes son su ele-
vada densidad (11,3 g/cm3) y su relativamente baja conductividad térmica y eléctrica. Es
fácil de trabajar, por ser blando y con bajo punto de fusión (328ºC). Se comercializa en
forma de tubos y elementos para fontanería, bolas de relleno (pesas y perdigones), vari-
llas, etc.

43
-Pág.47-
TECNOLOGÍA

Aleaciones no férricas
Algunas aleaciones no férricas importantes son:
– Latón. Es una aleación de cobre y cinc. Pre-
senta un color amarillento. Sus propiedades
más características son su resistencia a la
corrosión; se puede pulir hasta conseguir un
brillo intenso; se trabaja de forma aceptable.
Se comercializa en forma de chapas, varillas,
hilos, artículos decorativos, tornillería,
herrajes (bisagras, pomos para puertas), etc.
– Bronce. Es una aleación de cobre y estaño,
fundamentalmente, aunque si se añaden
otros elementos aparecen distintos tipos de
bronces (bronce de silicio, bronce de magne-
sio, bronce de fósforo y plomo, etc. Tiene
muchas aplicaciones, dependiendo del tipo:
fabricación de hélices de barco, campanas,
objetos artísticos, etc.
Es duro, posee alta resistencia a la corrosión y tiene excelentes cualidades para la fundi-
ción, lo que ha permitido su aplicación para fabricar objetos y herramientas desde la anti-
güedad.
– Aluminio-cobre. Es la aleación más empleada en la fundición. Contiene entre un 8 % de
cobre (aleación americana) y un 12 % (aleación inglesa); si además contiene magnesio y
silicio, forma los duraluminios, empleados en construcciones aeronáuticas, culatas de
motores, palas de hélices, etc.

5.3.3. LOS PLÁSTICOS


Los materiales plásticos aparecen en el último tercio del siglo XIX, una época muy flore-
ciente de la ciencia. Actualmente, los plásticos ocupan un papel decisivo en nuestra vida.
Se trata de unos materiales que han proporcionado a la sociedad instrumentos adaptados a
cada necesidad, como lo demuestra su presencia en casi todos los campos: comunicaciones,
construcción, mobiliario, agricultura, medicina, etc.
Las materias primas necesarias para la fabricación de plásticos son muy variadas y pueden
ser de origen animal, vegetal o mineral: leche (caseína), madera y paja (celulosa), caucho, petró-
leo, hulla, gas natural, etc. Habitualmente, son compuestos de carbono (C) e hidrógeno (H), y
según los casos, contienen además oxígeno (0), nitrógeno (N), azufre (S) y cloro (C1).

44
-Pág.48-
U.D. 5.- RECURSOS CIENTÍFICOS Y TÉCNICOS

En la actualidad, la mayoría de los plásticos que se comercializan provienen de la destila-


ción (fraccionamiento) del petróleo. La industria de plásticos utiliza el 6 % del petróleo de las
refinerías.

El petróleo y el gas natural son las principales materias primas de los plásticos.

Propiedades de los plásticos


– Transparencia. Dependiendo de la composición y estructura química, los plásticos pueden
ser transparentes, translúcidos u opacos. A menudo se colorean durante su fabricación.
– Conductividad eléctrica. Es muy pequeña, debido al reducido número de electrones
libres e iones móviles. De ahí su utilización en revestimiento de cables eléctricos.
– Combustibilidad. Muchos plásticos arden bien, por tener en su composición carbono e
hidrógeno. El aspecto de la llama y el olor que desprenden al quemarlos permite diferen-
ciarlos.
– Inalterabilidad. Generalmente son muy resistentes a los ataques por agentes químicos
(degradación, corrosión). Por ello son muy adecuados para revestimientos en industrias
químicas, conducciones de fluidos, objetos a la intemperie, etc. Sin embargo, suelen ser
atacados por disolventes orgánicos y ozono.

45
-Pág.49-
TECNOLOGÍA

– Peso específico. Es muy bajo, por lo que son idóneos para piezas y componentes en la
industria del transporte (aviones, barcos, coches, etc.).

5.3.4. LAS FIBRAS TEXTILES


Se puede definir una fibra textil como un material de longitud muy superior a su diámetro,
que puede ser hilado o tejido. El hilo es un arrollamiento o torcida de fibras textiles. Para poder
llegar a formar hilos a partir de las fibras textiles, éstas deben arrollarse o torcerse unas junto a
otras, asociando el final de unas con el principio de otras. Los tejidos están formados por el
entrelazamiento ordenado de estos hilos.

Las fibras textiles pueden obtenerse directamente de la naturaleza o ser fabricadas por el
hombre. Las primeras se denominan fibras naturales y las segundas fibras sintéticas o artifi-
ciales.

a) Fibras naturales. Pueden ser de origen animal, vegetal o mineral. Mediante procesos
mecánicos sencillos se preparan para ser hiladas.

b) Fibras artificiales. Aparecen como consecuencia del avance de la técnica y de la


mayor demanda de fibras por parte de la industria textil, demanda que no pueden cubrir
las fibras naturales.

Las propiedades de las fibras textiles tienen una influencia directa en sus procesos posterio-
res de fabricación de hilos, tejidos y prendas. Las propiedades más importantes que caracteri-
zan una fibra son:

– Resistencia a la tracción. Es la fuerza que es capaz de soportar al ser estirada antes de lle-
gar a romperse.

– Textura. Es el aspecto general que presenta una fibra a los sentidos de la vista y el tacto.

– Elasticidad. Capacidad para recuperarse tras sufrir una deformación.

– Higroscopicidad. Capacidad para absorber líquidos.

– Inflamabilidad. Facilidad para arder al aproximar un foco de calor.

– Afinidad por los tintes. Capacidad de asimilación de colorantes.

5.3.5. LOS MATERIALES CERÁMICOS


Son aquellos que se fabrican con arcillas. Estos materiales se obtienen de la corteza terres-
tre mezclados con otros muchos productos que han de ser eliminados. Las propiedades más

46
-Pág.50-
U.D. 5.- RECURSOS CIENTÍFICOS Y TÉCNICOS

importantes de los materiales cerámicos son la dureza, la fragilidad y ser muy resistentes a
las altas temperaturas y la corrosión. Los materiales cerámicos se clasifican en porosos y
compactos.

Materiales porosos
Son poco permeables al agua. La arcilla se muele hasta un tamaño de grano grueso. Los más
importantes son, el ladrillo y la teja.

Materiales compactos
Son totalmente impermeables al agua y el grano es mucho más fino. Destacan, azulejo, gres
y porcelana.

5.3.6. LOS MATERIALES DE ACABADO


La fabricación de un objeto suele llevar implícito un tratamiento de su superficie exterior.
Existen muchos tratamientos superficiales: recubrimiento con plásticos, materiales textiles,
papeles, baños químicos (galvanizados, cromados, plateados), pinturas, barnices, etc. Este tra-
tamiento puede tener una doble finalidad:

– Adorno. Se consigue un aspecto exterior más agradable. Se utiliza para decorar.

– Protección. Se consigue que ciertos agentes externos, sol, humedad, aire, etcétera, no lle-
guen a degradar el objeto.

En este apartado se tratará de los tipos y aplicaciones de las pinturas y barnices, debido a su
amplia utilización.

Pinturas. El principal componente de las pinturas es la base. Es un compuesto en estado


líquido que solidifica por diversas causas; se extiende como película protectora sobre un
material. Además contienen pigmentos disolventes y plastificantes como complementos de la
base.

Según sea la base, se denominan:

a) Pinturas al agua. Se disuelven en


agua.

b) Pinturas al temple. Introducen colas


vegetales o animales. Se utilizan para
recubrir paredes y techos en interio-
res. No se utilizan sobre madera.

47
-Pág.51-
TECNOLOGÍA

c) Pinturas al óleo. La base es aceite de linaza u otro componente polimerizable por los
agentes externos. Son solubles en aguarrás y disolventes orgánicos. En la construcción
se les llama lacas o esmaltes. Ofrecen acabados resistentes y estables. Se suelen aplicar
sobre maderas.
d) Pinturas plásticas. Son resinas artificiales con agua. Poseen gran resistencia y estabili-
dad. Se pueden aplicar sobre muebles, paredes, techos, ladrillos, etc.
Barnices. Tienen una composición muy parecida a las pinturas. No se les suele añadir pig-
mentos, aunque se comercializan barnices coloreados que imitan distintos tipos de madera
(roble, nogal, caoba).
Destacan los siguientes tipos:
a) Barnices al óleo. Utilizan aceites en su fabri-
cación. Se emplean para recubrimientos,
tanto interiores como exteriores.
b) Barnices al caucho. Utilizan como disolven-
te la nafta y el sulfuro de carbono. Son muy
resistentes al agua, por lo que se emplean
para recubrimientos antihumedad (piscinas,
laboratorios, etc.).
c) Barnices aislantes. Se utilizan para recubrimiento de cables eléctricos en bobinados de
motores y transformadores.

5.4. RIESGOS PARA LA SALUD Y PRECAUCIONES DERIVADOS


DE LOS MATERIALES TÉCNICOS
El fuego es el resultado de una reacción química exotérmica que denominamos combustión
y donde intervienen al menos tres factores: el combustible, el material que se quema, el com-
burente, que permite la combustión (generalmente es el oxígeno del aire), y la fuente de calor,
que inicia el fuego.
Los materiales son afectados por el fuego de varias formas: si es combustible, el material
arderá, contribuyendo a la propagación o mantenimiento del fenómeno; si tiene una temperatu-
ra de fusión o descomposición baja, aunque no arda, perderá sus propiedades mecánicas o quí-
micas; o puede que el fuego no le afecte de forma significativa.
– Combustibles inflamables. Son los que a temperatura
ambiente desprenden vapores en cantidad suficiente como
para que comiencen a arder si se produce una chispa o un
foco muy pequeño de calor.

48
-Pág.52-
U.D. 5.- RECURSOS CIENTÍFICOS Y TÉCNICOS

– Combustibles no inflamables. Son los materiales que pre-


cisan ser calentados previamente para comenzar a arder.
– Combustibles autoextinguibles. Son los materiales que
arden en presencia de fuego, pero que no son capaces de
mantenerlo por sí mismos. Si se retira la causa del fuego, se
apagan.
Clasificación de los fuegos:
La normativa española UNE 20010 establece cinco clases de fuego:
Clase A: fuegos de materiales sólidos comunes, general-
mente de tipo orgánico, tales como maderas, papel o carbón,
que arden dejando brasas.
Clase B: fuegos de líquidos y sólidos de bajo punto de
fusión, tales como ceras o termoplásticos.
Clase C: fuegos de productos gaseosos.
Clase D: fuegos de metales ligeros y productos químicos
muy reactivos.
Clase E: cualquiera de los fuegos anteriores cuando se
encuentren afectando a instalaciones eléctricas o en sus proxi-
midades.

REPERCUSIONES MEDIOAMBIENTALES DE
6. LA EXPLOTACIÓN DE LOS MATERIALES TÉCNICOS
El ser humano es el único ser vivo capaz de transformar rápida, constantemente y de forma
consciente el aspecto de nuestro planeta. Casi todas las modificaciones que el hombre introduce
artificialmente en el medio producen la destrucción de los hábitats. El problema se está agravando
por momentos, y es necesario poner en práctica las medidas oportunas para frenar esta tendencia.
Actualmente, extensas zonas de bosques se talan o queman. Nuestras fábricas, centrales tér-
micas y medios de transporte, entre otros, vierten al medio gases y residuos industriales que
contaminan el aire, los mares y los océanos.
Si no actuamos todos de una forma urgente y eficaz, pronto nos encontraremos en un callejón
sin salida. Si se siguen cortando árboles de forma indiscriminada y sustituyendo zonas verdes por
asfalto, industrias, etc., los problemas medioambientales (lluvia ácida, efecto invernadero, destruc-
ción de la capa de ozono, etc.), que en muchas ocasiones habrás oído nombrar, se incrementarán.

49
-Pág.53-
TECNOLOGÍA

A continuación vamos a estudiar los problemas medioambientales que afectan actualmente a


nuestra sociedad, analizando las causas y consecuencias directas que tienen sobre nuestro entorno.

6.1. LA DEFORESTACIÓN
Es la pérdida de masa forestal. Diariamente
se arrasan grandes extensiones de bosques por
causas tales como los incendios provocados y las
talas abusivas, tanto para el comercio de la made-
ra como para la reconversión de los bosques en
tierras para la agricultura y ganadería.
Los efectos de la deforestación, que se están
haciendo cada vez más patentes, son los siguientes:
– Disminución de la fertilidad del suelo por
pérdida de nutrientes que son necesarios
para el desarrollo de las especies.
– Cambios climáticos. Los bosques favorecen el aumento de la humedad ambiental y las
precipitaciones, de modo que su destrucción origina una mayor sequía.
– Descenso o desaparición de las especies animales y vegetales.
– Erosión y desertización. Cada año miles de hectáreas se van convirtiendo en suelos ári-
dos incultivables.

6.2. LA LLUVIA ÁCIDA


Se dice que el agua de lluvia es ligeramente ácida, pero en algunas zonas puede ser tan ácida
o incluso más que el jugo del limón o el vinagre. A veces, la lluvia ácida proviene de una emi-
sión de contaminantes que puede estar a gran distancia; por ejemplo, la lluvia ácida que cae en
Castellón procede de los residuos de la central térmica de Andorra (Teruel).
Pero, ¿cómo se forma la lluvia ácida? Como ya sabes, los ácidos se forman al unirse los óxi-
dos con el agua. Los óxidos que más contribuyen a la formación de la lluvia ácida son el óxido
de nitrógeno y el dióxido de azufre, que provienen fundamentalmente de los motores de gaso-
lina y diesel, de las centrales térmicas de carbón y petróleo y de las calefacciones de nuestros
hogares.
Cuando estos óxidos llegan a la atmósfera, reaccionan con las moléculas de agua y se trans-
forman en ácido nítrico y sulfúrico. Posteriormente, el agua de lluvia los arrastrará consigo
hacia la superficie, provocando la conocida lluvia ácida.

50
-Pág.54-
U.D. 5.- RECURSOS CIENTÍFICOS Y TÉCNICOS

El principal efecto de este fenómeno es la acidificación de los suelos, lagos, ríos y aguas
subterráneas. Las aguas se van volviendo ácidas y acaban matando a miles de animales y plan-
tas que viven en ellas.

6.3. EL AGUJERO EN LA CAPA DE OZONO


El sol es una estrella situada en el centro del sistema solar, una enorme bola de gases incan-
descentes que alcanzan una temperatura de unos 6000º C. Esta temperatura se produce por la
continua actividad del sol, que mediante reacciones nucleares originadas en su núcleo producen
y emiten gran cantidad de energía en forma de luz y calor, pero de las formas de radiación que
emite el sol, algunas pueden ser perjudiciales para la vida como la radiación ultravioleta (U.V.).
Afortunadamente, muchas de estas radiaciones no llegan a nuestro planeta, ya que en la atmós-
fera se encuentra una fina capa que se encarga de filtrarlas e impedir su paso, la capa de ozono.
El ozono es una forma del oxígeno compuesta por tres átomos del mismo. Se crea cuando
la radiación ultravioleta procedente del sol se encuentra con las moléculas de oxígeno de la
atmósfera, a las que rompe en átomos aislados de oxígeno que, al ser muy inestables y tener un
enorme poder reactivo, se unen con otros y forman las moléculas de ozono.
Hace algunos años, un grupo de científicos descubrieron un agujero en la capa de ozono
situado sobre la Antártida que, con el tiempo, se ha ido extendiendo. Entre las causas principa-
les de esta destrucción se encuentran los CFCs (clorofluorocarbonos) utilizados en sprays, espu-
mas, equipos de refrigeración. etc. Cuando estos gases escapan a la atmósfera, los rayos
ultravioleta rompen sus moléculas y liberan átomos de cloro que reaccionan con el ozono trans-
formándolo.

51
-Pág.55-
TECNOLOGÍA

Si toda la radiación ultravioleta no fuese filtrada y llegase íntegra a la Tierra, tendría efec-
tos destructores en plantas y animales. En el hombre originaría cáncer de piel, cataratas, etc. Las
alteraciones que podría ocasionar en los seres vivos darían lugar a la ruptura de las cadenas tró-
ficas, con las terribles consecuencias que esto supondría.

6.4. EL EFECTO INVERNADERO


Hay ciertos gases, como el dióxido de carbono, que por un lado permiten el paso de energía
del sol y por otro evitan que la radiación que refleja la Tierra escape, lo que hace posible la vida
en nuestro planeta.

Sin embargo, el aumento artificial de estos compuestos está produciendo que la retención
del calor en la Tierra sea cada vez mayor. En otoño de 1996, los científicos han reconocido, por
primera vez, que la temperatura de nuestro planeta ha subido ya medio grado como consecuen-
cia directa de la actividad humana, y se prevé que, de no tomar medidas urgentes, se pueda reca-
lentar más de tres grados. Esta subida, que puede parecer insignificante, originaría cambios
climáticos en todo el planeta, lo que favorecería la desertización y las inundaciones. A su vez
puede desencadenar la fusión de los hielos y, como consecuencia, la subida del nivel del mar.

Entre los gases que más perjudican la atmósfera y producen el llamado efecto invernadero
destacamos el dióxido de carbono (como ya hemos mencionado), que proviene de la quema de
combustibles (carbón, petróleo, etc.) y de los incendios forestales; este fenómeno se agrava con
la deforestación, ya que los árboles se encargan de transformar el dióxido de carbono. Otros
gases que hacen aumen-
tar el efecto invernadero
son el metano, los
CFCs, los óxidos de
nitrógeno, etc.

Una solución a la
emisión de estos gases
sería la utilización de
energías alternativas
como la eólica, solar,
etc.

52
-Pág.56-
U.D. 5.- RECURSOS CIENTÍFICOS Y TÉCNICOS

7. TÉCNICAS DE MEDIDA
La metrología es el tratado sobre las medidas. Deriva de las palabras griegas “metrón”, que
quiere decir medida, y «logos», que significa tratado. Algunos conceptos utilizados en metrolo-
gía son los siguientes:

Magnitudes físicas. Son propiedades de los cuerpos que pueden evaluarse con precisión.
Siempre se ha necesitado conocer las dimensiones y otras características de objetos, fenómenos
o manifestaciones de la naturaleza; así las personas, para mejorar su comunicación con los
demás y en su afán por conocer e investigar, han definido diversas magnitudes, a las cuales se
les asignan unas unidades de medida.

Medida. Medir una magnitud es compararla con otra que se toma como patrón. Si es de la
misma naturaleza, se habla de medición directa, y bien si se mide a través de otra de distinta
naturaleza, se habla de medición indirecta. Una vez que se ha comprobado cuántas veces está
contenida una unidad en la magnitud que estamos midiendo, obtenemos un valor. Este valor es
el resultado de la medida, y al proceso que se sigue para obtenerla lo llamamos medir. Al valor
numérico obtenido siempre se le añade la unidad correspondiente.

Verificación. Verificar es comprobar el cumplimiento de una determinada propiedad, como,


por ejemplo, planitud, ángulos entre caras, nivel, etc. Verificar no es lo mismo que medir, ya
que no se obtiene ningún valor por comparación con la unidad de medida.

Medir es comprobar cuántas veces una magnitud contiene a la unidad tomada como
patrón. Medida es el valor numérico, junto a la sigla de la unidad, obtenido al medir.
Verificar es comprobar, sin obtener valores numéricos.

53
-Pág.57-
TECNOLOGÍA

7.1. LOS ERRORES DE MEDIDA


Cuando en una medición se obtiene un valor distinto al verdadero, se dice que se ha come-
tido un error. Los errores se clasifican en accidentales, que dependen de factores externos o per-
sonales del observador que realiza la medida, o sistemáticos, que generalmente dependen del
uso o funcionamiento del aparato de medida y que se pueden cuantificar.
Error absoluto (Ea). Es la diferencia entre el valor medido (Vm) y el valor real o verdade-
ro (Vr), bien sea por exceso o por defecto. En general, el valor real o verdadero no se suele cono-
cer y en su lugar se utiliza el máximo error que se puede cometer con el aparato, que coincide
con la precisión del mismo.
Error relativo. Es el cociente que existe entre el error absoluto y el valor rea1.
Er = Ea/Vr
Para expresarlo en %, se multiplica la expresión anterior por 100.

7.2. SISTEMA DE UNIDADES PARA MEDIDAS DE LONGITUDES


Y ÁNGULOS
La medición de longitudes nos sirve para determinar la exactitud de las dimensiones de las
piezas, locales, etc, Para medir longitudes se utilizan normalmente dos sistemas de medida:
– El sistema internacional.
– El sistema inglés.
En el sistema internacional, la unidad de longitud es el
1 kilómetro (km) = 103 m
metro, y sus múltiplos y submúltiplos varían en un factor de
10. Este sistema es el más utilizado. 1 hectómetro (hm) = 102 m
1 decámetro (dam) = 10 m
En el sistema inglés, la unidad de medida de longitud es la
1 metro (m)
yarda, que equivale a 0,9144 m. Un submúltiplo de ésta es la
–1
pulgada, que equivale a 25,4 mm. La pulgada, junto con el 1 decímetro (dm) = 10 m
–2
milímetro, es la unidad más utilizada en la industria mecánica 1 centímetro (cm) = 10 m
y se representa por (") en la parte superior derecha de la canti- 1 milímetro (mm) = 10–3 m
dad (2" se lee “dos pulgadas”). Al importar tecnología de paí-
ses de origen anglosajón, las denominaciones de los objetos adquiridos se expresan en estas
unidades.
La medición de ángulos nos sirve para definir la abertura de los mismos. Los ángulos se pue-
den medir en dos sistemas: el centesimal y el sexagesimal; este último es el más frecuentemen-
te utilizado.

54
-Pág.58-
U.D. 5.- RECURSOS CIENTÍFICOS Y TÉCNICOS

– En el sistema centesimal el ángulo recto – En el sistema sexagesimal el ángulo recto


tiene 100º, cada grado 100’ minutos y cada tiene 90º, cada grado 60’ minutos y cada
minuto 100” segundos. minuto 60” segundos.

7.3. MEDICIÓN DE LONGITUDES Y ÁNGULOS CON ÚTILES


BÁSICOS

Medidas que expresan una longitud


Estas medidas se llaman lineales. Los útiles básicos para medir longitudes ordenados de
menor a mayor apreciación son: cinta métrica, metro y reglas graduadas.
La cinta métrica. Esta formada por una cinta de tela
o plástico, especialmente armada para evitar estiramien-
tos, de unos 25 m, generalmente graduada en centíme-
tros. Suele ir enrollada en una caja de forma cilíndrica
con una pequeña manivela para que, una vez desplegada
y realizada la medida, se pueda recogen.
El metro. Los más frecuentes, llamados flexómetros,
consisten en una cinta acerada flexible convenientemen-
te pintada y graduada en milímetros, que se recoge en una
caja enrollándose en su interior sobre sí misma.
La regla graduada. Está formada, generalmente, de barras de plástico, para dibujo, y de
acero, para los talleres. Tienen sección rectangular y están graduadas frecuentemente en dos
escalas: una métrica en milímetros o 0,5 mm y otra en pulgadas. Su longitud es variable.

55
-Pág.59-
TECNOLOGÍA

El calibre o pie de rey


La graduación en milímetros o centímetros de reglas y flexómetros, a veces, no es adecua-
da para medir longitudes muy pequeñas, como diámetros de alambres o espesores de chapas
metálicas. Para ello se utiliza el calibre, cuya apreciación en las medidas es del orden de 0,1 mm
o 0,05 mm.
En el siguiente esquema se detallan las distintas partes del calibre:

Puntas. Se utilizan para


medir diámetros Sonda. Mide profundidades
interiores de tubos y y espesores de distintos
otras piezas. Las puntas materiales; se desplaza
miden siempre la misma siempre con el movimiento
Regla
longitud que las bocas y del cursor deslizante y mide
graduada
la sonda la misma longitud que las
fija
Cursor puntas y la apertura de las
deslizante bocas.

Bocas. Miden diámetros Nonius. Reglilla que se


exteriores, pequeñas desliza a lo largo de la regla
longitudes y espesores de fija. Cada división del
chapas o láminas. nonius es 9/10 de las
La apertura de las bocas divisiones de la regla fija.
coincide siempre con la de Es, por tanto, capaz de
las puntas y la sonda. apreciar medidas de
décimas de milímetro.

Fundamento del calibre y mediciones


Ejemplo de medición con el calibre:
a) El cero del nonius marca siempre milíme-
tros enteros. Primero hay que observar
cuántos milímetros enteros abarca, y si coin-
cide exactamente ese valor en la regla de
milímetros será la medida. En el caso del
gráfico 2 mm.

56
-Pág.60-
U.D. 5.- RECURSOS CIENTÍFICOS Y TÉCNICOS

b) La fracción de milímetro se determina con el resto de divisiones del nonius y se deter-


minará por la única división de éste que coincida exactamente con una de la regla. En
el caso del gráfico 0,2 mm.
La medida será igual a la suma de los valores obtenidos en a y b; en nuestro ejemplo: 2,2 mm.

Medidas que expresan aberturas de los ángulos


Estas medidas se llaman angulares. Los instru-
mentos básicos para medir ángulos son: transpor-
tador, escuadras universales y goniómetros.
El transportador de ángulos. Consiste en un
círculo o semicírculo graduado en grados centesi-
males o sexagesimales. No tienen mucha preci-
sión, pero son de fácil manejo, por lo que se suelen
utilizar frecuentemente.

RESUMEN
Hemos estudiado en esta unidad que hay diferentes tipos de estructuras y que éstas tienen
que estar fabricadas para soportar diversos esfuerzos, para que dichos esfuerzos no puedan deri-
varla; que existen diversos sistemas de unión entre los elementos, elementos que muchas veces
forman un grupo de piezas, llamamos mecanismos, teniendo éstos la función de transmitir y
transformar movimientos, para que los elementos de la estructura se puedan mover.
Los mecanismos están controlados por diversos tipos de circuitos, estando éstos formados
por una serie de elementos, los cuales se fabrican con distintos materiales para hacerlos apro-
piados a la utilidad destinada. Estos materiales derivan en una serie de industrias, las cuales
actualmente están teniendo unas repercusiones ambientales que dañan seriamente la vida de los
seres humanos.
También hemos de decir que para el montaje de circuitos, mecanismos y estructuras se nece-
sita conocer los sistemas de medida y útiles que hay.

57
-Pág.61-
TECNOLOGÍA

EJERCICIOS DE AUTOCOMPROBACIÓN

1.Indica en las siguientes figuras los tipos de esfuerzos que se realizan:

2.Haz corresponder con una flecha los elementos con el tipo de unión que es.
UNIR TUBERÍAS DE COBRE
ABRAZADERA
PEGAMENTO UNIÓN FIJA
REMACHE
TIRAFONDO
SOLDADOR ESTAÑO
UNIR TUBERÍAS CON RACOR DE PRESIÓN
TORNILLO UNIÓN DESMONTABLE
SOLDAR VIGAS
TUERCA
ABRAZADERA DE PLÁSTICO

58
-Pág.62-
U.D. 5.- RECURSOS CIENTÍFICOS Y TÉCNICOS

3.Calcula la velocidad de las poleas conducidas en la siguiente figura.

Polea-1

Polea-2:

4.Rellena los datos que faltan en la siguiente tabla.

IMAGEN NOMBRE SIMBOLO MOVIMIENTO

CIRCULAR A
CIRCULAR

Sin fin y rueda


helicoidal

59
-Pág.63-
TECNOLOGÍA

5.Haz corresponder con una línea según la figura que se muestra el nombre de elemento, tipo de circui-
to a que corresponde y el tipo de elemento en los siguientes componentes de circuitos.

TIPO ELEMENTO NOMBRE CIRCUITO


Válvula

Pasivo
Electrónico

Diodo

Cilindro

Neumático

Activo Resistencia

Pulsador
Eléctrico

Bombilla

60
-Pág.64-
U.D. 5.- RECURSOS CIENTÍFICOS Y TÉCNICOS

6 Indica que propiedad caracteriza a los siguientes productos y el material de que están fabricados:

PRODUCTO MATERIAL PROPIEDAD


Muelle
Tubería
Ventana
Cerco de ventana Aluminio Resistencia
Viga
Cable de conexión

7.Indica las siguientes actividades, que repercusiones medioambientales más nociva es la que producen:

ACTIVIDAD EFECTO
Fabricación de papel
Siderurgia
Envasado de Spray
Calefacción
Chimenea
Vertido residuos al río

8.Indica de las siguientes definiciones a que conceptos nos referimos.

DEFINICIÓN CONCEPTO
Comprobar el cumplimiento de una determinada propiedad,
como, por ejemplo, planitud, nivel, etc.
Propiedades de los cuerpos que pueden evaluarse con preci-
sión
Diferencia entre el valor medido (Vm) y el valor real o ver-
dadero (Vr), bien sea por exceso o por defecto.
Medir una magnitud es compararla con otra que se toma
como patrón
La graduación en milímetros o centímetros de reglas y flexó-
metros, a veces, no es adecuada para medir longitudes muy
pequeñas, como diámetros de alambres o espesores de cha-
pas metálicas.
Los más frecuentes, llamados flexómetros, consisten en una
cinta acerada flexible convenientemente pintada y graduada.
El cociente que existe entre el error absoluto y el valor rea1.

61
-Pág.65-
TECNOLOGÍA

RESPUESTAS A LOS EJERCICIOS

1.Indica en las siguientes figuras los tipos de esfuerzos que se realizan:

TRACCIÓN

CORTADURA

FLEXIÓN

TORSIÓN

PANDEO

COMPRESIÓN

2.Haz corresponder con una flecha los elementos con el tipo de unión que es.
UNIR TUBERÍAS DE COBRE
ABRAZADERA
PEGAMENTO UNIÓN FIJA
REMACHE
TIRAFONDO
SOLDADOR ESTAÑO
UNIR TUBERÍAS CON RACOR DE PRESIÓN
TORNILLO UNIÓN DESMONTABLE
SOLDAR VIGAS
TUERCA
ABRAZADERA DE PLÁSTICO

62
-Pág.66-
U.D. 5.- RECURSOS CIENTÍFICOS Y TÉCNICOS

3.Calcula la velocidad de las poleas conducidas en la siguiente figura.

Polea-1
350 x 100 = Nc x 50
350 x 100
Nc = = 700 R.P.M.
50

Polea-2:
250 x 100 = Nc x 150
250 x 100
Nc = = 166 R.P.M.
150

4.Rellena los datos que faltan en la siguiente tabla.

IMAGEN NOMBRE SIMBOLO MOVIMIENTO

CIRCULAR A
POLEAS
CIRCULAR

CIRCULAR A
ENGRANAJES
CIRCULAR

RUEDA Y CIRCULAR A
CADENA CIRCULAR

Sin fin y rueda CIRCULAR A


helicoidal CIRCULAR

PIÑÓN Y CIRCULAR A
CREMALLERA LINEAL

MANIVELA CIRCULAR A
Y BIELA LINEAL

63
-Pág.67-
TECNOLOGÍA

5.Haz corresponder con una línea según la figura que se muestra el nombre de elemento, tipo de circui-
to a que corresponde y el tipo de elemento en los siguientes componentes de circuitos.

TIPO ELEMENTO NOMBRE CIRCUITO


Válvula

Pasivo
Electrónico

Diodo

Cilindro

Neumático

Activo Resistencia

Pulsador
Eléctrico

Bombilla

64
-Pág.68-
U.D. 5.- RECURSOS CIENTÍFICOS Y TÉCNICOS

6.Indica qué propiedad caracteriza a los siguientes productos y el material de que están fabricados:

PRODUCTO MATERIAL PROPIEDAD


Muelle Acero Mecánica
Tubería Plástico Electromagnética
Ventana Vidrio Resistencia
Cerco de ventana Aluminio Resistencia
Viga Acero Mecánica
Cable de conexión Cobre Térmica

7.Indica las siguientes actividades, qué repercusiones medioambientales más nociva es la que producen:

ACTIVIDAD EFECTO
Fabricación de papel Deforestación
Siderurgia Lluvia ácida
Envasado de Spray Agujero capa de ozono
Calefacción Efecto invernadero
Chimenea Efecto invernadero
Vertido residuos al río Lluvia ácida

8.Indica de las siguientes definiciones a qué conceptos nos referimos.

DEFINICIÓN CONCEPTO
Comprobar el cumplimiento de una determinada propiedad,
como, por ejemplo, planitud, nivel, etc. VERIFICIÓN

Propiedades de los cuerpos que pueden evaluarse con preci-


sión MAGNITUDES FÍSICAS

Diferencia entre el valor medido (Vm) y el valor real o ver-


dadero (Vr), bien sea por exceso o por defecto. ERROR ABSOLUTO

Medir una magnitud es compararla con otra que se toma


como patrón MEDICIÓN

La graduación en milímetros o centímetros de reglas y flexó-


metros, a veces, no es adecuada para medir longitudes muy
pequeñas, como diámetros de alambres o espesores de cha- CALIBRE O PIE DE REY
pas metálicas.
Los más frecuentes, llamados flexómetros, consisten en una
cinta acerada flexible convenientemente pintada y graduada. METRO

El cociente que existe entre el error absoluto y el valor rea1. ERROR RELATIVO

65
-Pág.69-
-Pág.70-
-Pág.71-
-Pág.72-
U.D. 6.- TECNOLOGÍA Y SOCIEDAD

ÍNDICE
OBJETIVOS . . . . . . . . . . . . . . . . . . . . . . . . . . . . . . . . . . . . . . . . . . . . . . . . . . . . . . . . . . . . . . 2

INTRODUCCIÓN . . . . . . . . . . . . . . . . . . . . . . . . . . . . . . . . . . . . . . . . . . . . . . . . . . . . . . . . . 3

MAPA CONCEPTUAL . . . . . . . . . . . . . . . . . . . . . . . . . . . . . . . . . . . . . . . . . . . . . . . . . . . . . 4

DESARROLLO DE CONTENIDOS . . . . . . . . . . . . . . . . . . . . . . . . . . . . . . . . . . . . . . . . . . 5

1. DESARROLLO TECNOLÓGICO . . . . . . . . . . . . . . . . . . . . . . . . . . . . . . . . . . . . . . 5
1.1. EVOLUCIÓN HISTÓRICA DE LOS PRODUCTOS . . . . . . . . . . . . . . . . . . . . . . . . . 5
1.2. NECESIDADES INDIVIDUALES E INTERÉS SOCIAL. CALIDAD DE VIDA . . . . 6
1.3. INFLUENCIAS DEL DESARROLLO TECNOLÓGICO . . . . . . . . . . . . . . . . . . . . . . 6

2. ORGANIZACIÓN DEL TRABAJO . . . . . . . . . . . . . . . . . . . . . . . . . . . . . . . . . . . . . 7


2.1. ORGANIZACIÓN TÁCTICA . . . . . . . . . . . . . . . . . . . . . . . . . . . . . . . . . . . . . . . . . . 8
2.2. ORGANIZACIÓN SOCIAL . . . . . . . . . . . . . . . . . . . . . . . . . . . . . . . . . . . . . . . . . . . 8
2.3. CUALIFICACIÓN Y REMUNERACIÓN . . . . . . . . . . . . . . . . . . . . . . . . . . . . . . . . . 10
2.4. DISCRIMINACIÓN LABORAL . . . . . . . . . . . . . . . . . . . . . . . . . . . . . . . . . . . . . . . . 11

3. EL MERCADO Y LA DISTRIBUCIÓN DE PRODUCTOS . . . . . . . . . . . . . . . . . 11


3.1. TIPOS DE MERCADOS . . . . . . . . . . . . . . . . . . . . . . . . . . . . . . . . . . . . . . . . . . . . . 12
3.2. EL COSTE DEL PRODUCTO . . . . . . . . . . . . . . . . . . . . . . . . . . . . . . . . . . . . . . . . . 13
3.3. FACTORES QUE INTERVIENEN EN EL PRECIO DE VENTA DE UN PRODUCTO . . . 14

RESUMEN . . . . . . . . . . . . . . . . . . . . . . . . . . . . . . . . . . . . . . . . . . . . . . . . . . . . . . . . . . . . . . . 16

EJERCICIOS DE AUTOCOMPROBACIÓN . . . . . . . . . . . . . . . . . . . . . . . . . . . . . . . . . . . 17

RESPUESTAS A LOS EJERCICIOS . . . . . . . . . . . . . . . . . . . . . . . . . . . . . . . . . . . . . . . . . 19

-Pág.73-
TECNOLOGÍA

O B J E T I VO S
Al finalizar el estudio de esta Unidad Didáctica, el alumno será capaz de:

• Analizar cómo influye la evolución de los objetos en la calidad de vida.

• Detectar las ventajas y riesgos de los avances tecnológicos.

• Describir cómo se organiza el trabajo.

• Distinguir los diferentes tipos de mercado.

• Valorar los costes de un producto.

• Analizar cuáles son los factores que intervienen en el precio de venta.

-Pág.74-
U.D. 6.- TECNOLOGÍA Y SOCIEDAD

INTRODUCCIÓN
T oda actividad tecnológica va relacionada al ser humano mediante varios aspectos
que le influyen tanto en su calidad de vida como en la forma de llevar ésta. En el
desarrollo de la tecnología el ser humano ha visto mejorada su vida al no tener que rea-
lizar trabajos tan pesados, así como con la creación de nuevos objetos se ha mejorado su
relación social.

Todo esto también ha traído circunstancias nuevas que tiene que ir solucionando a medi-
da que suceden, como es la adaptación y el aprendizaje de nuevas tecnologías como la
informática.

Lo más negativo que tiene la tecnología y su desarrollo respecto a la sociedad es la


degradación que está sufriendo el planeta, en contaminación, deforestación, etc..

-Pág.75-
TECNOLOGÍA

M A PA C O N C E P T UA L
DESARROLLO NECESIDADES CALIDAD DE
TECNOLÓGICO SOCIALES VIDA

FABRICACIÓN
ARTESANAL

TÁCTICA
TECNOLOGÍA Y SOCIEDAD

PRODUCCIÓN
EN SERIE

CATEGORÍA
ORGANIZACIÓN PROFESIONAL
DEL TRABAJO

DISCRIMINACIÓN
SOCIAL JERARQUIZACIÓN

SALARIO

COMPETENCIA

TIPOS OLIGOPOLIO

MONOPOLIO

EL
MERCADO DE
PRODUCCIÓN DERECHOS
CONSUMIDOR

COSTES PRECIO DEL OFERTA Y


PRODUCTO DEMANDA

DE PUBLICIDAD
COMERCIALIZACIÓN

-Pág.76-
U.D. 6.- TECNOLOGÍA Y SOCIEDAD

1. DESARROLLO TECNOLÓGICO, FORMAS


Y CALIDAD DE VIDA
La actividad tecnológica está continuamente en desarrollo, y se aplica de forma muy espe-
cializada; primero mediante los oficios, que representan el dominio de técnicas y trabajos sobre
ciertos materiales y productos; y más tarde, y debido a ese desarrollo tecnológico, surgen nue-
vas especializaciones agrupadas en dos grandes campos:

A) Energía, materiales y procesos


Un gran número de especialidades tecnológicas se ocupan de la producción de energía, de
las propiedades de los materiales y de los procedimientos para su extracción y elaboración y del
estudio y mejora de los procesos de fabricación en el desarrollo de nuevas instalaciones.

B) El desarrollo de productos
Existen muchas nuevas especialidades en los campos del diseño y elaboración de objetos
novedosos.

1.1. EVOLUCIÓN HISTÓRICA DE LOS PRODUCTOS


Como habrás podido observar, para atender a una misma necesidad existen productos muy
diversos. Este hecho responde a una evolución en el perfeccionamiento de sus funciones para
que mejore en algo la calidad de vida del usuario o consumidor. Uno de los factores que favo-
rece este afán de mejora en los productos es la competencia entre las empresas para aumentar
sus ventas.

Las disponibilidades de energía se han ido adaptando, innovando y descubriendo otras, en


el afán del ser humano de eliminar el esfuerzo físico.

5
-Pág.77-
TECNOLOGÍA

El siguiente esquema muestra esta evolución:

ESFUERZO ENERGÍA
HUMANO EÓLICA ELECTRICIDAD
HIDRÁULICA

ESFUERZO
ANIMALES VAPOR
DOMESTICADOS

1.2. NECESIDADES INDIVIDUALES E INTERÉS SOCIAL.


CALIDAD DE VIDA
El avance tecnológico hace que se desarrollen nuevos productos que habitualmente respon-
den a la demanda de un interés social, descubierto a través de un estudio de mercado.
Pero también la relación puede ser contraria, que la creación de un producto nuevo termine
originando una necesidad en las personas, pasando a ser productos necesarios y, a veces,
imprescindibles.
Un ejemplo claro de este efecto es el ordenador. Hace muchos años, en cualquier oficina, se
trabajaba sin él, y hoy parece imposible realizar un trabajo sin la ayuda del mismo.
Los productos tecnológicos responden a diferentes necesidades y deseos de los seres huma-
nos, y reflejan el estado de la calidad de vida de cada época y sociedad.

1.3. INFLUENCIAS DEL DESARROLLO TECNOLÓGICO


Las repercusiones que tiene el avance tecnológico sobre nuestra forma de vida han sido y
son muy grandes:
En el favorecimiento de las mejoras sociales en el trabajo, haciendo que éstos sean más lle-
vaderos.
En las tareas domésticas, con la incorporación en los hogares de agua corriente, electricidad
y gas, haciendo posible la invención de aparatos que facilitan los trabajos del hogar.
El uso generalizado de la electricidad y la presencia de electrodomésticos en los hogares favo-
recieron a mediados del siglo XX la incorporación de la mujer al mundo laboral, experimentando
un gran avance la lucha por la igualdad de hombres y mujeres en los países industrializados.

6
-Pág.78-
U.D. 6.- TECNOLOGÍA Y SOCIEDAD

Los avances tecnológicos no siempre se han utilizado para una mejora del bienestar social,
sino para la mejora de unos pocos, teniendo esto repercusiones y problemas graves que afectan
a la calidad de vida:
– Avances en tecnología militar, como instrumento de dominio.
– Contaminación ambiental.
– Agotamiento de los recursos naturales.
– Explotación de países subdesarrollados.
– Dependencia tecnológica.
Las diferencias socioeconómicas entre los países de la tierra son notables. Frente a países
con una mortalidad infantil elevada y muy baja esperanza de vida, se encuentran otros con una
mortalidad infantil 25 veces menor y una esperanza de vida de casi el doble. En estos datos
están indicados el consumo de energía y el producto nacional bruto P.N.B., donde se refleja el
potencial de desarrollo tecnológico de ese país.

País Esperanza de vida Tasa de mortalidad PNB per cápita Consumo de


al nacer en años infantil por cada en dólares energía eléctrica
1000 nacidos vivos per cápita en kw/h
Japón 80 4 32380 3749
Etiopia 43 107 100 18
España 78 5 14080 7083
Zambia 43 113 330 560
Datos anuario de EL PAIS 2000.

2. ORGANIZACIÓN DEL TRABAJO


La fabricación de los productos se realiza en empresas especializadas. El tamaño, caracte-
rísticas y organización de cada una de ellas depende de la complejidad del producto a elaborar.
Para la elaboración del producto, la empresa debe contar con una gran variedad de recursos,
tanto humanos como materiales. Para aprovechar estos recursos de forma adecuada, la empre-
sa adopta unas estructuras organizativas.

7
-Pág.79-
TECNOLOGÍA

2.1. ORGANIZACIÓN TÁCTICA DEL TRABAJO


Una vez que se decide que un producto debe ser fabricado para su posterior comercializa-
ción, se plantea un proceso de organización táctica para la fabricación de dicho producto, de
modo que sea fabricado con el menor coste posible.
Dependiendo del tipo de producto a fabricar, existen dos formas de organización:

FABRICACIÓN UNITARIA
Se aplica para productos de gran compleji-
dad o para la cual sólo sea necesario obtener una
unidad. Este tipo de fabricación se realiza por
operarios con una especialización determinada.
También está toda la fabricación artesanal.

FABRICACIÓN EN SERIE
Se aplica este tipo de organización del traba-
jo para obtener una gran cantidad de productos
iguales.

2.2. ORGANIZACIÓN SOCIAL DEL TRABAJO


Toda empresa, según su envergadura, debe disponer de una estructura organizativa, más o
menos compleja, que facilite la formación de grupos homogéneos, agilizando la coordinación y
la unión de esfuerzos para conseguir los objetivos de la empresa, al menor coste posible.
Mediante la agrupación de personas en departamentos y la posterior cadena de mandos, se
habrá formado una estructura organizativa.
Cualquier estructura organizativa debe cumplir las siguientes características:
A) Especificar tareas: todo empleado debe conocer el trabajo que debe desempeñar y su
posición dentro de la empresa.
B) Estructura jerárquica: personas que den órdenes y otras que las ejecuten.
C) Formación continua de los trabajadores.
D) Estabilidad laboral.
E) Seguridad e higiene en el trabajo.
F) Política de ascensos.
G) Sistema de remuneración justo.
H) Reconocimiento al trabajador.

8
-Pág.80-
U.D. 6.- TECNOLOGÍA Y SOCIEDAD

2.2.1. JERARQUIZACIÓN
Las estructuras organizativas se representan gráficamente mediante organigramas.
Las estructuras empresariales más significativas son:

Estructural lineal FERNANDO DOMENECH


DIRECTOR GENERAL

Antonio Suescun Joaquín García Javier Mackguiber


Director Producción Director Comercial Director de Personal

Mecánico Mecánico Mecánico Vendedor Vendedor Administrativo Informático

Estructural funcional FERNANDO DOMENECH


DIRECTOR GENERAL

Antonio Suescun Joaquín García Joaquín García Javier Mackguiber


Director Producción Director Comercial Director Comercial Director de Personal

Mecánicos Informáticos Electricistas Administrativos

Estructural Staff FERNANDO DOMENECH


DIRECTOR GENERAL

Departamento
Asesor Director

Antonio Suescun Joaquín García Javier Mackguiber


Director Producción Director Comercial Director de Personal

Julián Fernández Ángel Deaza


Asesor Producción Asesor Legal

Sección Sección
Ventas Compras

Sección Sección Sección Sección Sección


Máquinas Mantenimiento Eléctrica Máquinas Máquinas

9
-Pág.81-
TECNOLOGÍA

2.3. CUALIFICACIÓN Y REMUNERACIÓN DEL TRABAJO


Uno de los factores que hace competitiva una empresa es la productividad; a mayor pro-
ductividad, menores costes de obtención del producto.
La productividad está relacionada con la mano de obra y depende de dos factores:
– De las habilidades y capacidades de los trabajadores que les permite ser competentes en
su puesto de trabajo.
– De la motivación que tiene un trabajador para realizar su trabajo (un trabajador contento
y motivado, produce más).
Por lo tanto, para incrementar la productividad, es necesario potenciar estos dos aspectos
mediante programas de formación continua y una buena política de salarios.

2.3.1. PROGRAMAS DE FORMACIÓN


El objetivo es dotar al operario de la cualificación necesaria para realizar su trabajo, adap-
tándose a los avances tecnológicos.
Fases de un programa formativo:
a) Detección de las necesidades.
b) Impartición de la formación.
c) Evaluación de los resultados.
Técnicas utilizadas para impartir formación:
a) En el puesto de trabajo.
b) Mediante simuladores.
c) En el aula.

2.3.2. POLÍTICAS DE SALARIOS


Salario es el conjunto de prestaciones que percibe un trabajador por su trabajo. Puede estar
formado por:
– Cobro monetario: es el dinero que percibe.
– Cobro en especie: es la parte del salario en forma de ventajas económicas o sociales
(coche de la empresa, plan de pensiones, servicios médicos, viajes...).

10
-Pág.82-
U.D. 6.- TECNOLOGÍA Y SOCIEDAD

Aspectos que tiene en cuenta la empresa al calcular un salario:


– Competencia profesional.
– Fatiga mental o física.
– Grado de responsabilidad.
– Condiciones de seguridad e higiene.
La motivación del empleado se obtiene estableciendo otros tipos de retribución económica:
– Prima de productividad.
– Prima de antigüedad.
– Aumento de sueldo.
– Gratificaciones extraordinarias.
– Participación en beneficios.

2.4. DISCRIMINACIÓN LABORAL


Pese a ser anticonstitucional, todavía se dan casos de discriminación en el mundo laboral.
Esta puede manifestarse de dos formas diferentes:
– Poner barreras a la promoción de determinados grupos
sociales.
– Remunerar de forma distinta un mismo tipo de trabajo.
Para evitar estos casos, en algunos países se aprueban leyes de
“discriminación positiva” que favorecen la contratación de grupos
sociales determinados mediante exenciones fiscales a las
empresas.

3. EL MERCADO Y LA DISTRIBUCIÓN DE PRODUCTOS


Las condiciones ideales del mercado consisten en que el productor y el comprador se pon-
gan de acuerdo en el precio y la cantidad del producto. Estas circunstancias se dan en muy pocas
ocasiones; la realidad es que cada uno intenta imponer sus criterios.

11
-Pág.83-
TECNOLOGÍA

3.1. TIPOS DE MERCADO


Los tipos de mercado que existen están determinados por el número de productores y com-
pradores que tenga un producto.

COMPETENCIA MONOPOLIO OLIGOPOLIO


PRODUCTORES Muchos Uno Pocos
COMPRADORES Muchos Muchos Muchos

3.1.1. MERCADO DE COMPETENCIA PERFECTA


Éste es el mercado más beneficioso para el consumidor, ya que las empresas, al tener que
competir, deben ajustar más sus precios (productos de consumo diario: fruta, pescado, carne...)

3.1.2. MONOPOLIO
En este tipo de mercado existe un solo fabricante del producto, del que no existen sustitu-
tos, y muchos compradores.
Se genera monopolio cuando:
– Una empresa tiene la patente de un objeto único.
– Se explota un servicio público por concesión estatal (RENFE).
– Grandes instalaciones (Gas Natural).
– El propio Estado comercializa algún bien o servicio (Tabacalera).

3.1.3. OLIGOPOLIO
Es el mercado habitual de los productos de consumo. Existen muchos compradores y pocos
productores, pero con poder suficiente para influir en el mercado (fabricante vehículos).

12
-Pág.84-
U.D. 6.- TECNOLOGÍA Y SOCIEDAD

3.2. EL COSTE DEL PRODUCTO


Una empresa emplea muchos recursos para la fabricación de un producto, lo que le supone un
desembolso económico. Más tarde, con la venta de dicho producto, se producirán unos ingresos.
La diferencia entre los gastos y los ingresos es el beneficio obtenido.

3.2.1. COSTES DE PRODUCCIÓN


En la fabricación del producto, la empresa tiene unos costes que se clasifican en fijos y
variables:
— Costes fijos: Son los que no dependen del número de productos fabricados. Por ejem-
plo, la maquinaria o los edificios. Los más frecuentes son:
– Alquileres. Coste del uso de las instalaciones.
– Seguros. Pago para prevenir pérdidas por incendios, inundaciones, etc.
– Impuestos. Pagos al Estado por la actividad.
– Amortizaciones. De maquinaria por el deterioro en su uso o por el paso del tiempo.

— Costes variables: Son los que aumentan o disminuyen según el número de productos a
fabricar. Los más frecuentes son:
– Materia prima. Material de partida para obtener el producto terminado.
– Mano de obra. Trabajadores que intervienen en la fabricación.
– Materiales. Elementos que no son parte del producto, pero son necesarios para su
fabricación (aceites, disolventes, etc.).
– Energía. Consumo de fuentes energéticas.

3.2.2. COSTES DE COMERCIALIZACIÓN


La comercialización es el proceso que lleva un producto desde que se ha fabricado hasta que
llega a sus destinos de consumo.

Los tres costes más importantes en un proceso de comercialización son:

Almacenamiento – Transporte – Puntos de venta.

– Almacenamiento: Se produce por la diferencia


de tiempo entre la producción y la venta. Las
empresas tienen un nivel de reservas denomina-
do “Stock”, teniendo en cuenta el número de
ventas previsto y si el producto es perecedero.

13
-Pág.85-
TECNOLOGÍA

– Transporte: Son los gastos derivados del traslado que se realiza del producto desde el
almacén hasta los puntos de venta o directamente al comprador.
– Puntos de venta: Es donde se ofrece el producto al consumidor a un precio determinado.

3.3. FACTORES QUE INTERVIENEN EN EL PRECIO DE VENTA


DE UN PRODUCTO
La función última de un producto que fabrica una empresa es que éste tenga un número de
compradores total que sea rentable para la empresa.
Para que un producto sea adquirido por un comprador, tiene que tener grandes prestaciones
que cubran las necesidades de éste, pero sobre todo, un precio asequible.
El precio único de un producto viene dado por los costes que éste tenga en su proceso de
fabricación y puesta a la venta. En este punto se alcanzaría el umbral de la rentabilidad, es decir,
la empresa ni pierde ni gana dinero.

GASTOS = INGRESOS

A parte de los factores propios de los costes de fabricación y comercialización de un pro-


ducto, existen otros que son muy influyentes en el precio de venta de éste:
– Oferta y demanda.
– Valores del consumidor.
– La publicidad.

3.3.1. OFERTA Y DEMANDA


La oferta la comportan vendedores y productores que ofrecen sus productos al mercado.
La demanda la integran los consumidores que solicitan productos para su satisfacción.

OFERTA DEMANDA PRECIO


MUCHA POCA BAJA
POCA MUCHA SUBE
MUCHA MUCHA EQUILIBRIO

14
-Pág.86-
U.D. 6.- TECNOLOGÍA Y SOCIEDAD

3.3.2. VALORES DEL CONSUMIDOR


La educación en consumo y la información que recibe el consumidor hace que al tomar la
decisión de adquirir un producto no se deje llevar por el eslogan “lo más caro lo mejor” si no
que tenga en cuenta otros aspectos.
La Constitución Española protege el uso y consumo de productos a través de la Ley Gene-
ral para la defensa de los Consumidores y Usuarios. En ella se recogen los siguientes derechos:
– A la protección de su salud y seguridad.
– A la protección de sus intereses económicos y sociales.
– A la protección de daños y perjuicios.
– A la representación por asociaciones de Consumidores.
– A la protección ante situaciones de inferioridad.
Como Consumidores, estamos protegidos por la Ley y debemos ejercer este derecho ante
posibles abusos.

3.3.3. LA PUBLICIDAD
Mediante ésta se busca informar y persuadir a los consumidores para que adquieran el pro-
ducto promocionado.
La publicidad ejerce su influencia cuando existe diferencia y confusión al tener que elegir
entre opciones poco diferenciadas.
Mediante la publicidad se intentan conseguir diferentes objetivos hacia el producto.

15
-Pág.87-
TECNOLOGÍA

TIPO DE PUBLICIDAD OBJETIVO QUE SE PRETENDE


De lanzamiento Dar a conocer un nuevo producto
De mantenimiento Recordar la existencia de un producto
De prestigio Realzar la imagen de la empresa o del producto
Genérica Aumentar el número de consumidores
Competitiva Destacar las características del producto que la diferencian
de otros

Todo el trabajo publicitario se concreta en campañas publicitarias, las cuales están formadas
por un conjunto de actividades planificadas alrededor de un tema central que llame la atención
del público y provoque el deseo de comprar el producto anunciado.

RESUMEN
La introducción de las nuevas tecnologías y los procesos productivos están en continua evo-
lución, se ha pasado de una fabricación artesanal a una producción automatizada, con la consi-
guiente mejora en la calidad de vida del ser humano.
En ese afán de mejorar, el ser humano crea nuevas necesidades para mejorar, tanto su vida,
como su ocio, y muchas veces sin tener en cuenta los factores negativos que le aportan, como
son la dependencia a los objetos, la contaminación ambiental, el aislamiento social, etc..
La organización del trabajo sufre cambios, ya que se crean nuevas profesiones, así como la
necesidad del reciclaje de los trabajadores para afrontar los cambios tecnológicos.
El avance en los medios de comunicación hace que la información sea cada vez mayor para
el trabajador, evitándose abusos de poder y discriminaciones.
Este avance en los medios de comunicación es aprovechado por el sector empresarial para
la promoción de sus productos mediante campañas publicitarias.
Debido a la gran oferta de productos, las empresas se ven obligadas a disminuir sus costes
de fabricación y comercialización, para lanzar el producto a un precio más competitivo.

16
-Pág.88-
U.D. 6.- TECNOLOGÍA Y SOCIEDAD

EJERCICIOS DE AUTOCOMPROBACIÓN

1. Une el tipo de publicidad con el objetivo a conseguir:

GENÉRICA DESTACAR LAS CARACTERÍSTICAS DEL PRODUCTO

COMPETITIVA RECORDAR LA EXISTENCIA DE UN PRODUCTO

DE MANTENIMIENTO AUMENTAR EL NÚMERO DE CONSUMIDORES

2. Señala qué ocurre con el precio de un producto según la relación oferta/demanda:

SUBE
MUCHA DEMANDA Y POCA OFERTA EQUILIBRIO
BAJA

SUBE
MUCHA OFERTA Y MUCHA DEMANDA EQUILIBRIO
BAJA

SUBE
MUCHA OFERTA Y POCA DEMANDA EQUILIBRIO
BAJA

3. En cuál de estas tres afirmaciones dirías que una empresa está en el umbral de la rentabilidad:

A) Los costes de producción y comercialización son superiores a los ingresos por la venta del pro-
ducto.

B) Los costes de producción y comercialización son inferiores a los ingresos por la venta del pro-
ducto.

C) Los costes de producción y comercialización son iguales a los ingresos por la venta del producto.

Respuesta:

4. Indica qué tipo de mercado (monopolio, oligopolio o competencia) son los siguientes ejemplos:

EMPRESA TIPO MERCADO


Compañía eléctrica
Supermercado del barrio
Concesionario de coches
Compañía telefonía móvil
Todo a 100
Compañía de suministro de agua

17
-Pág.89-
TECNOLOGÍA

5. Ordenar cronológicamente el tipo de energía aprovechada por el ser humano.


(Electricidad, Eólica e Hidráulica, Esfuerzo humano, Esfuerzo animales, Vapor).

6. Localiza en la sopa de letras cuatro Costes Fijos del coste de producción de un producto.

A M E F I T F U I O L A
S M D O N A U M H K J L
Ñ U O B O L I L L O S H
U S U R A Q G U B H E K
M O K U T U S O L P M C
L R U O L I J E B Y T T
P U E F U L Z A A E E D
I G F J T E M A I K Y P
L E S V E R N E C J F L
D S O T S E U P M I U K
G A H B U S N O O T O H
Z A F I R E E I Ñ A J N

18
-Pág.90-
U.D. 6.- TECNOLOGÍA Y SOCIEDAD

RESPUESTAS A LOS EJERCICIOS

1. Une el tipo de publicidad con el objetivo a conseguir:

GENÉRICA DESTACAR LAS CARACTERÍSTICAS DEL PRODUCTO

COMPETITIVA RECORDAR LA EXISTENCIA DE UN PRODUCTO

DE MANTENIMIENTO AUMENTAR EL NÚMERO DE CONSUMIDORES

2. Señala qué ocurre con el precio de un producto según la relación oferta/demanda:

SUBE
MUCHA DEMANDA Y POCA OFERTA EQUILIBRIO
BAJA

SUBE
MUCHA OFERTA Y MUCHA DEMANDA EQUILIBRIO
BAJA

SUBE
MUCHA OFERTA Y POCA DEMANDA EQUILIBRIO
BAJA

3. En cuál de estas tres afirmaciones dirías que una empresa está en el umbral de la rentabilidad:

A) Los costes de producción y comercialización son superiores a los ingresos por la venta del pro-
ducto.

B) Los costes de producción y comercialización son inferiores a los ingresos por la venta del pro-
ducto.

C) Los costes de producción y comercialización son iguales a los ingresos por la venta del producto.

Respuesta: C

4. Indica qué tipo de mercado (monopolio, oligopolio o competencia) son los siguientes ejemplos:

EMPRESA TIPO MERCADO


Compañía eléctrica Monopolio
Supermercado del barrio Competencia
Concesionario de coches Oligopolio
Compañía telefonía móvil Oligopolio
Todo a 100 Competencia
Compañía de suministro de agua Monopolio

19
-Pág.91-
TECNOLOGÍA

5. Ordenar cronológicamente el tipo de energía aprovechada por el ser humano.


(Electricidad, Eólica e Hidráulica, Esfuerzo humano, Esfuerzo animales, Vapor).

Esfuerzo Esfuerzo Eólica e


Vapor Electricidad
humano animales Hidráulica

6. Localiza en la sopa de letras cuatro Costes Fijos del coste de producción de un producto.

A M E F I T F U I O L A
S M D O N A U M H K J L
Ñ U O B O L I L L O S H
U S U R A Q G U B H E K
M O K U T U S O L P M C
L R U O L I J E B Y T T
P U E F U L Z A A E E D
I G F J T E M A I K Y P
L E S V E R N E C J F L
D S O T S E U P M I U K
G A H B U S N O O T O H
Z A F I R E E I Ñ A J N

20
-Pág.92-

También podría gustarte